You are on page 1of 839

El-Zohry

MRCP Questions Bank


OnExamination 2010
PasTest 2010
Medical Masterclass 2010
MRCP Part 2
(1604 questions)
Endocrinology (143)
Cardiology (130)
Gastroentrology (150)
Nephrology (149)
Rheumatology (104)
Medical Masterclass
Hematology (130)
2010
Infectious disease (150)
Respiratory (160)
Neurology (154) Khalid Yusuf El-Zohry
Psychiatry (42) Sohag Teaching Hospital - Egypt
Dermatology (85) elzohryxp@yahoo.com
Oncology (80) https://www.facebook.com/elzohryxp
Pharmacology (120)
Basic science (7)
Special Thanks to Dr. Ahmed Maher Eliwa
MRCPstudy For his great effort in this material.
ReviseMRCP https://www.facebook.com/ahmedmahereliwa
El-Zohry MRCP Questions Bank (Part 2) – Medical Masterclass 2010

Dedications

To my father,
my mother,
my wife,
my sons:
Abd El-Rahman,
Muhammed,
and Amr

To president Muhammad Mursi

Special Thanks to Dr. Ahmed Maher Eliwa


For his great effort in this material.
https://www.facebook.com/ahmedmahereliwa

Dr. Khalid Yusuf El-Zohry – Sohag Teaching Hospital (01118391123) Page | 2


El-Zohry MRCP Questions Bank (Part 2) – Medical Masterclass 2010

‫ غفر هللا لنا جميعا‬،‫كلما نظرت في وجهه رأيت أبي‬


Dr. Khalid Yusuf El-Zohry – Sohag Teaching Hospital (01118391123) Page | 3
El-Zohry MRCP Questions Bank (Part 2) – Medical Masterclass 2010

Take the first step, and your mind will mobilize


all its forces to your aid.

But

The first essential is that you begin

Once the battle is startled, all that is within


and without you will come to your assistance

Dr. Khalid Yusuf El-Zohry – Sohag Teaching Hospital (01118391123) Page | 4


El-Zohry MRCP Questions Bank (Part 2) – Medical Masterclass 2010

Medical Masterclass 2010


Reference ranges.............................................................................................. 7

Endocrinology (143 Questions)......................................................................... 9

Cardiology (130 Questions) .......................................................................... 101

Gastroenterology (150 Questions) ............................................................... 165

Nephrology (149 Questions) ......................................................................... 251

Rheumatology (104 Questions) .................................................................... 333

Hematology (130 Questions) ........................................................................ 387

Infectious Disease (150 Questions) ............................................................... 451

Respiratory (160 Questions) ......................................................................... 523

Neurology (154 Questions) ........................................................................... 605

Psychiatry (42 Questions) ............................................................................. 677

Dermatology (85 Questions)......................................................................... 697

Oncology (80 Questions) .............................................................................. 735

Pharmacology (120 Questions) ..................................................................... 781

Basic Science (7 Questions) .......................................................................... 835

Dr. Khalid Yusuf El-Zohry – Sohag Teaching Hospital (01118391123) Page | 5


El-Zohry MRCP Questions Bank (Part 2) – Medical Masterclass 2010

Dr. Khalid Yusuf El-Zohry – Sohag Teaching Hospital (01118391123) Page | 6


El-Zohry MRCP Questions Bank (Part 2) – Medical Masterclass 2010

Reference ranges
Reference ranges vary according to individual labs.
All values are for adults unless otherwise stated

Full blood count Ferritin 20-230 ng/ml


Haemoglobin Men: 13.5-18 g/dl Vitamin B12 200-900 ng/l
Women: 11.5-16 g/dl Folate 3.0 nmol/l
Mean cell volume 82-100 fl Reticulocytes 0.5-1.5%
Platelets 150-400 x 109/l
Other biochemistry
White blood cells 4-11 x 109/l|
Calcium 2.1-2.6 mmol/l
Urea and electrolytes Phosphate 0.8-1.4 mmol/l
Sodium 135-145 mmol/l CRP < 10 mg/l
Potassium 3.5 - 5.0 mmol/l Thyroid stimulating hormone (TSH) 0.5-5.5
mu/l
Urea 2.0-7 mmol/l
Free thyroxine (T4) 9-18 pmol/l
Creatinine 55-120 umol/l
Total thyroxine (T4) 70-140 nmol/l
Bicarbonate 22-28 mmol/l
Amylase 70-300 u/l
Liver function tests Uric acid 0.18-0.48 mmol/l
Bilirubin 3-17 umol/l
Arterial blood gases
Alanine transferase (ALT) 3-40 iu/l
pH 7.35 - 7.45
Aspartate transaminase (AST) 3-30 iu/l
pCO2 4.5 - 6.0 kPa
Alkaline phosphatase (ALP) 30-100 umol/l
pO2 10 - 14 kPa
Gamma glutamyl transferase (yGT) 8-60 u/l
Albumin 35-50 g/l Lipids
Total protein 60-80 g/l
Desirable lipid values depend on other risk
factors for cardiovascular disease, below is
Other haematology just a guide:
Erythrocyte sedimentation rate (ESR) Total cholesterol < 5 mmol/l
Men: < (age / 2) mm/hr Triglycerides < 2 mmol/l
Women: < ((age + 10) / 2) mm/hr HDL cholesterol > 1 mmol/l
Prothrombin time (PT) 10-14 secs LDL cholesterol < 3 mmol/l
Activated partial thromboplastin time
(APTT) 25-35 secs

Dr. Khalid Yusuf El-Zohry – Sohag Teaching Hospital (01118391123) Page | 7


El-Zohry MRCP Questions Bank (Part 2) – Medical Masterclass 2010

Answer & Comments

Endocrinology Correct answer: B

(143 Questions) The plasma insulin level is inappropriate for the


ambient glucose concentration and is
(Medical Masterclass – Part 2) consistent with insulin-mediated
hypoglycaemia. The history of weight gain
suggests the possibility of insulinoma, but other
causes of this biochemical picture must be
[ Q: 1 ] MasterClass Part2 excluded before embarking on a search for a
(2010) - Endocrinology pancreatic lesion. Exogenous insulin
administration and sulphonylurea use may both
A 41-year-old woman is referred by her GP with
lead to hypoglycaemia with inappropriate
suspected hypoglycaemic episodes. She has
hyperinsulinaemia. Measurement of plasma C-
previously been fit and well, but has recently
peptide levels helps to distinguish exogenous
gained weight. Her daughter has well-
insulin administration from other causes
controlled type 1 diabetes mellitus. On
because endogenous proinsulin secretion, from
examination she is overweight (BMI 28.0 kg/m2)
which both insulin and C-peptide are derived, is
and her BP is 120/80 mmHg lying and 110/75
suppressed if hypoglycaemia is driven by
mmHg standing. She is admitted to hospital for
exogenous insulin. In contrast, sulphonylureas
a prolonged fast and becomes symptomatic at
stimulate endogenous insulin and C-peptide
18 hours, at which time investigations reveal
secretion from pancreatic β cells and
serum sodium 137 mmol/L (normal range 137-
measurement of plasma or urine sulphonylurea
144), serum potassium 4.8 mmol/L (normal
levels when the patient is hypoglycaemic may
range 3.5-4.9), serum creatinine 70 μmol/L
be necessary to confirm/refute clinical
(normal range 60-110), serum albumin 40 g/L
suspicions. In this case the patient's daughter
(normal range 37-49), serum total bilirubin 10
has type 1 diabetes mellitus, so the patient is
μmol/L (normal range 1-22), serum alanine
likely to have access to insulin, making it
aminotransferase 25 U/L (normal range 5-35),
particularly important to exclude factitious
serum alkaline phosphatase 100 U/L (normal
insulin administration.
range 45-105), plasma glucose 1.9 mmol/L
(normal range 3-6) and plasma insulin 65
pmol/L (normal <21). [ Q: 2 ] MasterClass Part2
(2010) - Endocrinology
Which additional investigation is most likely to
help distinguish between the possible causes of
In which of the following patients would you be
her symptoms?
least concerned about the finding of
A. Plasma Haemoglobin A1c gynaecomastia?
B. Plasma C-peptide A. 64-year-old man with controlled atrial
C. Plasma pancreatic polypeptide fibrillation

D. Plasma sulphonylurea screen B. 42-year-old man with headaches

E. Synacthen test C. 64-year-old man with unexplained


lethargy
D. 24-year-old man presenting to GP with
painful breast enlargement

Dr. Khalid Yusuf El-Zohry – Sohag Teaching Hospital (01118391123) Page | 9


El-Zohry MRCP Questions Bank (Part 2) – Medical Masterclass 2010

E. 38-year-old man being investigated for a What is the most likely diagnosis?
testicular lump.
A. Coeliac disease
B. Amyloidosis
Answer & Comments
C. Addison’s Disease
Correct answer: A
D. Gastroparesis
Gynaecomastia is the presence of palpable E. Pancreatitis due to dyslipidaemia
breast tissue in a male patient. It results from
an increase in the effect of oestrogen relative to Answer & Comments
the effect of androgen acting on the breast
tissue. In overweight males there may appear Correct answer: D
to be extra breast tissue but this is more often
lipomastia (fat deposition). This man has advanced microvascular
complications of diabetes, including
Histological changes of gynaecomastia are
retinopathy, nephropathy, sensory and
found in 40% of autopsy specimens - it is very
autonomic neuropathy (postural blood pressure
common but can indicate underlying pathology.
drop, resting tachycardia) and gustatory
Digoxin, spironolactone, cimetidine and
sweating. In this scenario he is highly likely to
metronidazole can all cause gynaecomastia, as
also have some degree of denervation of his
can the use of recreational steroids.
gut, leading to gastroparesis and constipation.
In answer B, the man may have underlying This leads not only to intractable nausea and
prolactinoma; answer C could have renal or constipation, but also can contribute to erratic
liver impairment, or a tumour producing hCG, glycaemic control due to unpredictable post-
oestrogen or androgens. Answer A is most likely prandial glycaemic excursions.
to be due to the use of digoxin, and unless the
Nuclear medicine can be used to support the
onset was recent or there was any change,
diagnosis with gastric emptying studies.
would not require further investigation.
Management centres around stabilisation of
glycaemic control, but in severe cases can
[ Q: 3 ] MasterClass Part2 include surgery or parenteral nutrition. There is
(2010) - Endocrinology also limited experience with gastric pacing.

A 36-year-old man with a 23-year history of


type 1 diabetes is admitted with intractable
postprandial vomiting and poor glycaemic
control. He has had retinopathy extensively
treated in the past, has established
macroalbuminuria, a creatinine of 220
micromol/l, and has known peripheral sensory
neuropathy.
On examination he is euvolaemic with a resting
tachycardia of 95 bpm, and a 20mmHg systolic
blood pressure drop on standing, and also is
seen to sweat profusely when he starts to eat
his hospital meal.
His HbA1c is 12%.

Dr. Khalid Yusuf El-Zohry – Sohag Teaching Hospital (01118391123) Page | 10


El-Zohry MRCP Questions Bank (Part 2) – Medical Masterclass 2010

[ Q: 4 ] MasterClass Part2 management by his GP. His blood pressure is


(2010) - Endocrinology 139/87, and his annual review tests include the
following: HbA1c 8.6%, cholesterol 5.6 mmol/L
This 25-year-old man has type 1 diabetes of 10 (HDL 1.0, LDL 3.7), triglycerides 3.1 mmol/L. His
years' duration. Recently he has had erratic urinary albumin : creatinine ratio is 4.1, and
glycaemic control, with both hypo- and subsequent timed urine collection reveals an
hyperglycaemia. He shows you a painless lump albumin excretion rate of 250 mg/24 hours.
in clinic (see image). On examination it is
regular, smooth, soft and non-tender. Which of the following is correct with respect to
his further management?
What is the most appropriate intervention?
A. The urinary protein loss will increase
A. To prescribe a different sort of insulin inexorably.
B. To prescribe an antibiotic with gram B. ACE inhibitors and angiotensin II receptor
positive cover antagonists have both been proven to
C. To prescribe an antibiotic with gram improve the renal prognosis.
negative cover C. The main focus of effort should be the
D. To perform a fine needle aspiration preservation of renal function.
biopsy D. There is no benefit in further reduction of
E. To advise more frequent rotation of blood pressure.
injection sites. E. 10 year coronary risk should be estimated
by reference to the Joint Societies
Answer & Comments prediction charts, as published in the
British National Formulary.
Correct answer: E

Answer & Comments


This is lipohypertrophy caused by repeated
insulin injection into the same site. It is Correct answer: B
probably caused by a direct effect of insulin on
the subcutaneous tissues promoting The great importance of microalbuminuria is
lipogenesis. Absorption of insulin from the that it confers an additional risk of
hypertophied site is variable, hence the recent cardiovascular disease above that resulting
erratic glycaemic control. For this and for from other established risk factors, perhaps
cosmetic reasons patients are advised to rotate because it is a surrogate marker of generalised
insulin injection sites. Avoiding further injection endothelial dysfunction. Importantly, this
into the hypertrophied area will usually allow excess risk is not encompassed by the Joint
the tissue to regress. Diabetic patients should Societies charts in the B.N.F., which thus
have injection sites checked for lipohypertrophy underestimate the risk. Hence, it is important
at clinic visits, especially if there is otherwise to intensify control of conventional risk factors,
unexplained deterioration of glycaemic control. as the greatest threat to a patient such as this is
macrovascular disease. Microalbuminuria is
[ Q: 5 ] MasterClass Part2 also important as the most easily measured
early stage of diabetic nephropathy. It is,
(2010) - Endocrinology
moreover, also reversible; the chances of this
A 58-year-old man is seen for the first time in being maximised by scrupulous blood pressure
the diabetes clinic after several years of

Dr. Khalid Yusuf El-Zohry – Sohag Teaching Hospital (01118391123) Page | 11


El-Zohry MRCP Questions Bank (Part 2) – Medical Masterclass 2010

management and blockage of the renin- Hypogonadism can occur in either sex, the
angiotensin system. hypogonadotrophic group including patients
with anorexia nervosa, athletic amenorrhoea,
Both ACE inhibitors and ATII antagonists have
prolactinoma or lesions of the hypothalamus
been shown in large trials to improve the renal
and pituitary. Loss of growth hormone may also
prognosis, and emerging evidence suggests
play a role in causing osteoporosis in patients
their effects are comparable. Trial data suggests
with pituitary tumours.
that lowering blood pressure improves
prognosis even within the normal range, and it Rarer causes of osteoporosis include
may be that targets below the current one of malignancy, e.g. multiple myeloma; chronic
135/80 may be justified hepatitis; intestinal disorders and drugs, e.g.
phenytoin, cyclosporin A and heparin.
[ Q: 6 ] MasterClass Part2
(2010) - Endocrinology [ Q: 7 ] MasterClass Part2
(2010) - Endocrinology
A 35-year-old woman presents with a vertebral
crush fracture. A 37-year-old woman with a 6-month history of
tiredness and lethargy is admitted as an
Which TWO of the following are LEAST useful in emergency complaining of a severe retro-
determining the aetiology of her condition? orbital headache and visual disturbance. On
A. DEXA scan examination she has a left third nerve palsy.
The findings of an urgent MRI scan are shown in
B. Serum gonadotrophin levels
the image.
C. 24-hour urinary cortisol excretion
D. Thyroid function tests
E. Drug history
F. Serum PTH measurement
G. Serum creatinine
H. Body Mass Index
I. Serum calcium
J. Full blood count.

Answer & Comments

Correct answer: GJ

The most common cause of secondary


osteoporosis is exogenous glucocorticoids.
Other causes that must be considered and
pursued in a young woman include primary
hyperparathyroidism, Cushing's syndrome,
hypogonadism, hyperthyroidism, prolactinoma, What is the most likely diagnosis?
diabetes mellitus, acromegaly, pregnancy and
lactation.

Dr. Khalid Yusuf El-Zohry – Sohag Teaching Hospital (01118391123) Page | 12


El-Zohry MRCP Questions Bank (Part 2) – Medical Masterclass 2010

A. Subarachnoid haemorrhage due to a Answer & Comments


posterior communicating artery
Correct answer: E
aneurysm
B. Haemorrhage into a pituitary If endogenous insulin were the cause, whether
macroadenoma
generated by a tumour (insulinoma) or in
C. Lymphocytic hypophysitis with response to stimulation by an oral
suprasellar extension hypoglycaemic agent, then levels of both insulin
and C-peptide would be the same. The only
D. Haemorrhagic hypophysitis
thing that can produce dissociation between
E. Empty sella syndrome insulin and C-peptide levels is administration of
exogenous insulin.
Answer & Comments

Correct answer: B [ Q: 9 ] MasterClass Part2


(2010) - Endocrinology
The clinical presentation and scan findings are A 56-year-old woman is referred to the hospital
typical of pituitary apoplexy caused by an area by her GP for further investigation of weight
of haemorrhage within a pituitary loss and palpitations. She has no past medical
macroadenoma and complicated by a third history of note and is on no regular
nerve palsy. medications. There is no family history of
thyroid disease. On examination her pulse is
[ Q: 8 ] MasterClass Part2 110 bpm in sinus rhythm and she has a fine
(2010) - Endocrinology resting tremor. She has a small goitre with no
discrete palpable nodules and no cervical
A 38-year-old man presents with recurrent lymphadenopathy. There is no evidence of
‘funny turns’. During one of these he is found to dysthyroid eye disease. Investigations reveal
have a blood glucose concentration of 1.6 plasma free T4 55.5 pmol/L (normal range 10-
mmol/l. A clever pre-registration house officer 22), plasma free T3 15.5 pmol/L (normal range
sent a concurrent serum sample to the 5-10), plasma thyroid-stimulating hormone <0.1
biochemistry laboratory: this is found to mU/L (normal range 0.4-5.0) and serum
contain insulin, but C-peptide levels are low. antithyroid peroxidase 15 IU/mL (normal <50).

This means that hypoglycaemia is likely to have Which investigation is most likely to help
been caused by: determine the cause of this woman's
A. endogenous insulin, e.g. produced by an presentation?
insulinoma A. CT scan of neck and upper thorax
B. consumption of an oral hypoglycaemic B. Fine-needle aspiration biopsy
agent
C. Sestamibi scan
C. liver disease
D. Technetium uptake scan
D. starvation
E. Ultrasound neck
E. exogenous insulin, e.g. self-
administration. Answer & Comments

Correct answer: D

Dr. Khalid Yusuf El-Zohry – Sohag Teaching Hospital (01118391123) Page | 13


El-Zohry MRCP Questions Bank (Part 2) – Medical Masterclass 2010

A technetium uptake scan would help in E. Grave's ophthalmopathy is caused by


differentiating between the possible causes of muscle hypertrophy.
thyrotoxicosis: Graves' disease (diffuse
F. The presence of a warm or hot nodule
increased uptake), toxic multinodular goitre
excludes malignancy.
(patchy increased uptake), toxic adenoma
(discrete hot nodule), thyroiditis (absent or G. I131 is a gamma emitter.
reduced uptake). In this case there is no H. Toxic multinodular goitre is the
relevant family history, eye signs or other commonest form of thyrotoxicosis under
features to allow a clinical diagnosis of Graves' 50 year olds.
disease to be made. While the patient is older
than is classical for a new presentation of I. Lid lag and lid retraction are specific for
Graves' disease, this remains a possibility. The Graves' disease.
detection of TSH receptor antibodies in the J. In thyroiditis, transient hyperthyroidism
patient's serum would provide an alternative may be followed by transient
means of diagnosing Graves' disease and in hypothyroidism.
these circumstances a thyroid uptake scan
would not be necessary unless the patient opts Answer & Comments
for radioactive iodine and it is used to plan the
dose. A thyroid ultrasound or CT scan is not Correct answer: CJ
helpful in the routine diagnosis or differential
diagnosis of thyrotoxicosis. In the absence of a Carbimazole usually take 3-4 weeks to render
dominant thyroid nodule there is no indication the patient euthyroid. Before radioiodine
for fine-needle aspiration biopsy to exclude treatment, the patient must be off amiodarone
thyroid carcinoma. Sestamibi scanning is used for several months so this is not a very practical
in the investigation of hyperparathyroidism to treatment. I 131 emits beta particles. The
identify a parathyroid adenoma. ophthalmopathy in Graves' is caused by
infiltration of the muscle cells with
[ Q: 10 ] MasterClass Part2 mononuclear cells and accumulation of
glycosoaminoglycans which promote fluid
(2010) - Endocrinology
retention. Lid lag and retraction are seen in
A 45-year-old female presents with thyrotoxic patients regardless of the underlying
palpitations, tremor and fatigue and her TSH is aetiology.
found to be suppressed.
[ Q: 11 ] MasterClass Part2
Which TWO statements are true?
(2010) - Endocrinology
A. Carbimazole renders most patients
euthyroid in 2 weeks. A 38-year-old woman is referred to the general
medical outpatient clinic because she is obese.
B. I131 is useful treatment for amiodarone-
She weighs 132 kg, with a BMI of 38.3. Tests of
induced thyrotoxicosis.
thyroid and adrenal function are normal. You
C. Fifteen percent of patients with Graves' want to motivate her to lose weight. A 10kg
disease have a close relative with the weight loss would bring all of the medical
same condition. benefits listed below, except two.
D. Toxic multinodular goitre is more
Which TWO of the following are NOT benefits of
common in men than women.
losing 10kg in weight?
A. Reduction of 15 mm Hg systolic BP
Dr. Khalid Yusuf El-Zohry – Sohag Teaching Hospital (01118391123) Page | 14
El-Zohry MRCP Questions Bank (Part 2) – Medical Masterclass 2010

B. Reduction of 20 mm Hg diastolic BP C. The most likely diagnosis is a non-


functioning pituitary adenoma with stalk
C. 30% reduction in fasting glucose
compression.
D. 10% reduction in total cholesterol
D. The first line treatment is bromocriptine,
E. Reduction in sleep apnoea which will reverse her amenorrhoea.
F. Decreased breathlesssness E. She should proceed directly to treatment
G. >20% reduction in total mortality with transphenoidal surgery.

H. >30% reduction in diabetes-related


Answer & Comments
deaths
I. >40% reduction in obesity-related cancer Correct answer: C
deaths
The most likely diagnosis is a non-functioning
J. Increase in HDL of 8%.
macroadenoma with stalk compression. You
would expect a much higher prolactin (certainly
Answer & Comments greater than 5000mU/l) if this were a
Correct answer: AC macroprolactinoma of this size. The prolactin
level might be consistent with a
microprolactinoma, however the adenoma
Based on data from The Scottish Intercollegiate
would be less than 1cm on MRI. Other
Guidelines Network the benefits of a 10 kg
important differentials for a modestly elevated
weight loss would include all of those listed
prolactin, as seen in this case, are physiological
above, except that the reduction in systolic
(pregnancy, lactation), drug induced (dopamine
blood pressure is estimated to be 10 mm Hg
antagonists), primary hypothyroidism and
and the reduction in fasting glucose 50%.
polycystic ovarian syndrome.
Whilst microadenomas are a relatively common
[ Q: 12 ] MasterClass Part2
incidental finding on current, highly sensitive
(2010) - Endocrinology
pituitary MRIs and could still be consistent with
A 36-year-old woman presents with a 10-month elevated prolactin from another cause, such as
history of amenorrhoea and tiredness. She is drugs or PCOS, macroadenomas are not a
taking no medications and has no other clinical common coincidental finding.
features of note. Investigations include a Bromocriptine will rapidly reduce prolactin as it
negative pregnancy test, normal TSH and is a dopamine agonist, and will substitute for
elevated prolactin of 1600 mU/l. MRI scan the dopamine that is missing due to stalk
shows a pituitary macroadenoma measuring compression. It may reverse her amenorrhoea
1.5cm in diameter. if it is only caused by elevated prolactin,
however, she may also have
Which of the following is correct?
hypogonadotrophic hypogonadism secondary
A. The most likely diagnosis is a to her pituitary macroadenoma compromising
macroprolactinoma. gonadotroph function.
B. The most likely diagnosis is a Surgery may be indicated, but she needs a
microprolactinoma. careful endocrine workup, including full
baseline and dynamic testing of her pituitary
axes and formal evaluation of her visual fields.
If her steroid axis is suppressed she will need

Dr. Khalid Yusuf El-Zohry – Sohag Teaching Hospital (01118391123) Page | 15


El-Zohry MRCP Questions Bank (Part 2) – Medical Masterclass 2010

steroid replacement prior to any other unlikely and the caecal lesion may be a
hormone replacement or surgery. A low-normal coincidental finding
TSH does not rule out secondary
hypothyroidism, as thryoid hormone levels may Answer & Comments
be low. A visual field defect would prompt
urgent surgery. Otherwise, the timing of Correct answer: B
surgery requires careful discussion with the
patient and the pituitary surgeon. If her Chromogranin A staining is consistent with a
pituitary function and fields are intact it may be neuroendocrine tumour and the caecal-
preferable to defer surgery, as this may cause appendiceal region is the commonest location
hypopituitarism. In this case, treatment with for a carcinoid primary. These tumours are
bromocriptine to restore menses and indeed slow growing, however the presence of
monitoring visual fields may be preferable, systemic symptoms suggestive of carcinoid
particularly if fertility is desired. syndrome along with elevated urinary 5HIAA,
suggest there are already liver metastases
[ Q: 13 ] MasterClass Part2 causing release of 5HT into the systemic
circulation, and circumventing first-pass
(2010) - Endocrinology
metabolism in the liver.
A 50-year-old woman presents with a two-year Octreotide scanning is positive in up to 85% of
history of diarrhoea, intermittent facial flushing cases, however a negative scan does not rule
and wheeze with minor (3kg) weight loss. out liver metastases. The liver should be
Thyroid function and fasting plasma glucose are imaged by high resolution CT with fine cuts or
normal but 24-hour urinary 5-HIAA levels are by USS. The sensitivity of USS may be increased
elevated. A colonoscopy performed in view of by the use of microbubble contrast medium
her diarrhoea has identified a caecal lesion. (levovist), which is available at some centres.
Biopsy of this lesion showed cells staining for Fasting gut hormones should be measured as
chromogranin A on histology. Octreotide neuroendocrine tumours may co-secrete other
scanning of the liver was negative. hormones such as VIP, which may contribute to
the diarrhoea.
What is the most likely diagnosis?
The prognosis is generally good. Although
A. A neuroendocrine tumour causing
patients are not generally cured, their
carcinoid syndrome. As this is a slow
symptoms can be effectively managed and they
growing tumour, resection of the caecal
are likely to live for many years and may die of
lesion should be curative
unrelated causes. Ongoing management should
B. Carcinoid syndrome. Further imaging of involve echocardiography to screen for
the liver with ultrasound scanning or CT carcinoid heart disease (right-sided valvular
should be performed along with lesions). Symptomatic management may
measurement of fasting gut hormones include somatostatin analogues, hepatic
C. Phaeochromocytoma and further imaging embolisation, hepatic chemo-embolisation and
with MIBG scan should be performed chemotherapy. Octreotide is less likely to be
effective if octreotide scan negative, but other
D. Inflammatory bowel disease analogues such as lanreotide have different
E. Cause is irritable bowel disease/anxiety; affinities for different somatostatin receptor
as the octreotide scan is negative, a subtypes, which may be present on the tumour.
diagnosis of carcinoid syndrome is Patients should be managed at a tertiary
referral centre with a multidisciplinary team

Dr. Khalid Yusuf El-Zohry – Sohag Teaching Hospital (01118391123) Page | 16


El-Zohry MRCP Questions Bank (Part 2) – Medical Masterclass 2010

available (eg: endocrinologist, oncologist, (acromegaly) and would warrant further


interventional radiologist). investigations to confirm it.

[ Q: 14 ] MasterClass Part2 [ Q: 15 ] MasterClass Part2


(2010) - Endocrinology (2010) - Endocrinology
A 33-year-old woman is admitted for the
Which of the following sets of investigations is
not compatible with a non-functioning pituitary investigation of lethargy and weakness. She has
adenoma in a 52-year-old man? a history of depression and admits to drinking
12-14 bottles of strong cider per week for the
A. Thyroid-stimulating hormone (TSH) 1.02 last 2 years. She is taking an antidepressant but
mU/l (0.4 to 4.0 mU/l) and Free T4 of 7.1 cannot remember its name. She was
pmol/l (9.0 to 21.0 pmol/l) investigated recently for headaches by a private
B. Prolactin 1856 mU/l (0 to 450mU/l) physician and MRI of the brain was reported as
normal.
C. Short Synacthen Test: Cortisol (0 min) 56
nmol/l & Cortisol (30 min) 167 nmol/l She is obese and plethoric with a blood
(Abnormal) pressure of 164/98. Her skin is thinned and she
is slow rising from her chair. A dexamethasone
D. IGF-1 raised at greater than 30 % above
suppression test is performed.
the upper limit of age-related reference
range Cortisol 0900 baseline = 887 nmol/l

E. Follicle-stimulating hormone (FSH) 0.3 Cortisol 0900 post 48 hours of dexamethasone


mIU/l (Low), LH 0.5 mIU/l (Low), 0.5mg every 6 hours = 728 nmol/l
Testosterone 5 pg/ml (Low). Cortisol 0900 post 48 hours of dexamethasone
2.0mg every 6 hours = 265 nmol/l.
Answer & Comments
The most likely diagnosis is:
Correct answer: D
A. Glucocorticoid resistance syndrome
Non-functioning pituitary adenomas, as the B. Pseudo-Cushing’s
name suggests, do not secrete any pituitary
hormones. They may present with local C. Cushing’s disease
pressure effects, varying degrees of D. Ectopic adrenocorticotrophic hormone
hypopituitarism, hyperprolactinaemia due to (ACTH) secretion
pituitary stalk compression or they may be
E. Adrenal adenoma.
found incidentally.
Pituitary stalk compression may lead to Answer & Comments
impaired dopamine delivery, hence causing
increased prolactin release due to lack of Correct answer: C
inhibition by dopamine.
In a normal individual, plasma cortisol
All of the above sets of investigations, except suppresses to < 50 nmol/l after 48 hours of
raised IGF-1 levels, are compatible with dexamethasone 0.5 mg 6 hourly(1). When
hypopituitarism associated with a non- differentiating pseudo-Cushing’s from Cushing’s
functioning pituitary adenoma. Such high levels syndrome a cut off of 38 nmol/l at this time
of IGF-1 would, in fact, point towards the point gave 100% specificity for diagnosing
likelihood of a functioning adenoma Cushing’s syndrome 1 and this therefore makes

Dr. Khalid Yusuf El-Zohry – Sohag Teaching Hospital (01118391123) Page | 17


El-Zohry MRCP Questions Bank (Part 2) – Medical Masterclass 2010

pseudo-Cushing’s highly unlikely in this E. Poor (sub-normal) cortisol response to a


scenario. short synacthen test.
Classically pituitary dependent Cushing’s
disease is differentiated from ectopic ACTH Answer & Comments
production and non-ACTH dependent Cushing’s Correct answer: C
by measurement of plasma ACTH levels and
suppression of plasma cortisol by 50% from This patient's clinical picture is highly suggestive
baseline after 48 hours of high dose of a pituitary adenoma. It is difficult on the
dexamethasone. Approximately 10% of subjects given information to ascertain whether he has a
with pituitary dependent Cushing’s may fail to secretory or a non-secretory tumour. High
suppress however, and a few people with prolactin levels may be seen in prolactinomas
ectopic ACTH production also will 2. or any other pituitary or non-pituitary tumours
Nonetheless, these results are highly suggestive where pressure from the tumour mass on the
of pituitary dependent Cushing’s disease. pituitary stalk functionally disconnects the
Glucocorticoid resistance syndrome is rare and inhibitory input from the hypothalamus. Some
is due to a mutation of the glucocorticoid growth hormone secreting adenomas also
receptor. Subjects have high ACTH levels and secrete prolactin.
high cortisol levels because of lack of negative Low levels of certain pituitary hormones occur
feedback via the glucocorticoid receptors. due to local effects of the pituitary adenoma
Women may demonstrate the virilising effects (whether secretory or non-secretory).
of excess androgen secretion under the Secondary hypothyroidism due to lack of
influence of increased ACTH, and may have thyroid-stimulating hormone (TSH),
hypertension due to the action of salt retaining adrenocortical deficiency due to lack of
steroids, but fail to demonstrate the other adrenocorticotrophic hormone (ACTH), low IGF-
features of Cushing’s syndrome such as 1 associated with growth hormone deficiency
thinning of the skin and myopathy1. and low testosterone associated with FSH and
LH deficiency (hypogonadotrophic
[ Q: 16 ] MasterClass Part2 hypogonadism) can all occur together, in
(2010) - Endocrinology different combinations or on their own.

A 45-year-old man is referred with severe


[ Q: 17 ] MasterClass Part2
tiredness, frontal headaches and lack of libido.
He has a bitemporal hemianopia on
(2010) - Endocrinology
confrontation. A previously fit 64-year-old woman is referred
to endocrine clinic for investigation. Her
Which of the following results would not be husband died six months ago and her daughter
consistent with the presence of a pituitary
is concerned that she has gained weight and
adenoma?
become lethargic and depressed. She was
A. Low insulin-like growth factor-1 (IGF-1) diagnosed 1-year-ago with type 2 diabetes. This
levels was well controlled initially on diet, but her
glycaemic control has recently deteriorated and
B. High prolactin levels
she was found by her GP to be hypertensive (BP
C. High follicle-stimulating hormone (FSH) 168/92). As part of your initial investigations
and luteinizing hormone (LH) levels with a you perform an overnight dexamethasone
low testosterone level suppression test, which is positive (plasma
D. Secondary hypothyroidism cortisol fails to suppress to less than 50nmol/l).

Dr. Khalid Yusuf El-Zohry – Sohag Teaching Hospital (01118391123) Page | 18


El-Zohry MRCP Questions Bank (Part 2) – Medical Masterclass 2010

Which of the following statements regarding far from certain. This is a screening test with a
this patient are FALSE? high sensitivity and a low specificity; ie it should
A. The differential diagnosis includes not miss genuine Cushing’s but has a high false
depression positive rate of 20-30%. One difficulty in the
diagnosis of Cushing’s syndrome is that
B. The differential diagnosis includes alcohol symptoms are non-specific and common, in
abuse particular central obesity. Proximal myopathy, if
C. The differential diagnosis includes present, has a higher positive predictive value
Cushing’s syndrome in determining which patients will prove to
have Cushing’s syndrome.
D. The positive overnight dexamethasone
suppression test makes Cushing’s This lady may be at risk of depression or alcohol
syndrome almost certain abuse in view of her recent bereavement,
which along with taking less care about her
E. The differential diagnosis includes
diet, could lead to deterioration in diabetic
consumption of exogenous steroids
control and hypertension. Alcoholism and
F. A low dose dexamethasone test should be depression can cause pseudocushing’s states
performed, aiming to distinguish pituitary presenting with many of the clinical features of
Cushing’s disease from other causes of Cushing’s syndrome and having positive
Cushing’s syndrome screening tests for Cushing’s. Other causes
include obstructive sleep apnoea.
G. A low dose dexamethasone test should
be performed, aiming to establish or 24-hour urinary free cortisol (UFC) is another
exclude a diagnosis of Cushing’s screening test for Cushing’s with relatively high
syndrome sensitivity and low specificity. Two or more
collections should be performed and figures
H. A sleeping midnight cortisol
above the normal range investigated further,
measurement would be helpful in making
however, pseudocushing’s states can also
a diagnosis in this patient
elevate 24-hour UFC as well as causing false-
I. Proximal myopathy, if present, is a more positive overnight dexamethasone suppression
specific sign of Cushing’s syndrome than test results.
central obesity
The low dose dexamethasone suppression test
J. An elevated 24-hour urinary free cortisol is currently considered the gold standard in
measurement does not necessarily establishing a diagnosis of Cushing’s syndrome.
confirm a diagnosis of Cushing’s It does not distinguish between causes of
syndrome Cushing’s syndrome. To distinguish a cause, first
establish whether ACTH-dependent (elevated
Answer & Comments ACTH paired with cortisol) or ACTH-
independent (suppressed ACTH – should ideally
Correct answer: DF
be undetectable).
The overnight dexamethasone suppression test An elevated midnight cortisol is a sensitive and
(measurement of 9am cortisol after specific indicator of Cushing’s syndrome (of any
consumption of dexamethasone 1-2mg at 11pm cause) as diurnal rhythm is lost, however
the previous night) is sometimes used as a patients must be unaware the test is to be
screening test for Cushing’s syndrome. Failure performed, must be sleeping and the blood
to suppress warrants further investigation, sampling completed within 15 minutes of
however, a diagnosis of Cushing’s syndrome is waking them to perform the test, otherwise

Dr. Khalid Yusuf El-Zohry – Sohag Teaching Hospital (01118391123) Page | 19


El-Zohry MRCP Questions Bank (Part 2) – Medical Masterclass 2010

cortisol will be elevated as a normal stress uptake and an autoantibody screen which is
response. positive for thyroid peroxidase (TPO)
autoantibodies.
The differential diagnosis in ACTH-dependent
Cushing’s is a pituitary adenoma (Cushing’s
Which of the following TWO statements are
disease, the most common cause), ectopic
correct?
ACTH and extremely rarely ectopic CRH. Further
investigation may include high dose A. The diagnosis is Graves' disease.
dexamethasone suppression test (in ectopic B. The patient should be given carbimazole
ACTH cortisol classically does not suppress vs as soon as possible.
suppression by at least 50% in pituitary
C. The patient should be given
Cushing’s, but has poor specificity), pituitary
propylthiouracil as soon as possible.
imaging (co-incidental non-functioning tumours
may lead to confusion) and inferior petrosal D. The patient's condition may resolve over
sinus (high sensitivity and specificity for a few weeks without any specific
determining a pituitary source of ACTH but medication.
technically demanding and requires a definite
E. The patient will require life-long
diagnosis of Cushing’s syndrome first as the
medication.
normal pituitary makes ACTH). ACTH-
independent Cushing’s is caused by exogenous F. The patient may be reassured that the
steroids or adrenal lesions and is investigated condition is unlikely to recur.
further by careful drug history and adrenal G. The patient may be suitable for
imaging. radioiodine treatment.
H. The patient may become hypothyroid at
[ Q: 18 ] MasterClass Part2 a later stage in her disease.
(2010) - Endocrinology
I. Beta-blockers are contra-indicated for
A 31-year-old lady presents to her GP with a 1- symptom relief.
month history of insomnia, palpitations and J. A thyroid ultrasound scan would be useful
weight loss. She had initially thought this was in the management of this condition.
due to looking after her 4-month-old baby but
her symptoms continued even when the baby
Answer & Comments
was sleeping through the night. She remembers
having a similar episode after the birth of her Correct answer: DH
first child but never sought help. Her mother is
known to take thyroxine but the patient feels This lady had developed postpartum thyroiditis.
reassured by the fact that her midwife told her The cardinal features include thyrotoxicosis
that her thyroid function tests were normal in associated with decreased tracer uptake – a
the antenatal clinic. feature which helps to distinguish this condition
from Graves’ disease or a toxic multinodular
Her GP does a repeat set of thyroid function
goitre. In the latter conditions there is
tests, as follows: TSH < 0.01 mU/L (NR 0.3-4
increased tracer uptake due to true
mU/L) free T4 36.7 pmol/l (NR 11-25 pmol/l)
hyperthyroidism.
and total T3 3.1nmol/l (1-2.7 nmol/l). The
patient is then referred to the endocrine clinic The underlying pathophysiology is due to
for further advice. Further investigations apoptosis of the follicular epithelial cells, which
include a 99Tc scan which shows a mildly releases the preformed hormone thyroglobulin
enlarged thyroid gland with reduced tracer and some abnormal iodinated materials into

Dr. Khalid Yusuf El-Zohry – Sohag Teaching Hospital (01118391123) Page | 20


El-Zohry MRCP Questions Bank (Part 2) – Medical Masterclass 2010

the circulation. Serum concentrations of T3 and Which of the following tests would you like to
T4 are therefore elevated sufficiently to do next?
produce clinical thyrotoxicosis and suppress A. Semen analysis for sperm count
TSH secretion. As a result of this, all thyroid
function is suppressed and thyroid hormone B. Tests for sense of smell and colour
synthesis ceases. Later on in the disease, when blindness
stores of preformed hormone are depleted and C. Pituitary MRI scan
thyroid function remains suppressed the
D. Karyotype
patient may pass through a hypothyroid stage
before finally becoming euthyroid once more. E. Serum Prolactin levels
Patients present with transient thyrotoxicosis
with low tracer uptake usually within 3 to 6 Answer & Comments
months of delivery and based on the above Correct answer: D
pathophysiology it is clear why they often pass
through a hypothyroid state lasting 2-9 months This boy has hypergonadotrophic
before becoming euthyroid. However, in some hypogonadism. Presence of gynaecomastia and
patients only the hypothyroid state is evident. small testes, points towards the diagnosis of
The incidence of postpartum thyroiditis varies Klinefelter's syndrome. His body habitus may be
geographically, but is thought to occur in about eunuchoid. The most important test at this
10% of women and in more than 30% of those point is to perform a Karyotype analysis to see
with TPO autoantibodies. Recurrences are also whether he has 47XXY of Klinefelter's
common after subsequent pregnancies. This syndrome.
patient may have had a similar undetected Kallmann's syndrome is associated with
episode after the birth of her first child. Most features like absence of sense of smell and
patients have a small goitre and positive tests colour blindness. Hypogonadism in Kallmann's
for TPO antibodies, although the levels are not is of hypogonadotrophic type (hypothalamic in
usually particularly high. origin).
Treatment of a patient like this with antithyroid MRI scan of the pituitary is not indicated in the
agents would have rendered her profoundly given setting.
hypothyroid and is not indicated as the
Hyperprolactinaemia is associated with
synthetic function of the gland is, in fact,
hypogonadotrophic hypogonadism. Semen
already suppressed. Symptoms can be
analysis and sperm count will not help in
controlled with beta-blockade provided there
establishing a diagnosis and is not required at
are no other contraindications. Radioiodine is
this stage.
not indicated and in fact cannot be given safely
to a patient who is in close contact with small
children (under 14). [ Q: 20 ] MasterClass Part2
(2010) - Endocrinology
[ Q: 19 ] MasterClass Part2 As medical registrar on call, you are fast
(2010) - Endocrinology bleeped to the ophthalmology emergency clinic
to see a 44-year-old woman who has collapsed
An 18-year-old man is referred to the Endocrine
while being seen. She was referred urgently by
clinic with bilateral gynaecomastia. He has
her GP after developing a severe frontal
bilateral small testes. His gonadotrophins (FSH
headache with nausea and double vision while
& LH) are high and testosterone level is low.
at work. Her past history was remarkable only

Dr. Khalid Yusuf El-Zohry – Sohag Teaching Hospital (01118391123) Page | 21


El-Zohry MRCP Questions Bank (Part 2) – Medical Masterclass 2010

for migraines, hypothyroidism, and mild this situation the key step is to consider the
asthma. She was taking HRT and thyroxine. diagnosis and administer adequate intravenous
steroid (e.g. 100mg hydrocortisone) to facilitate
Before collapsing the ophthalmologist had
resuscitation before emergency MRI imaging to
established that she had an almost complete
look for pituitary haemorrhage and mass effect.
right third nerve palsy with a left sixth nerve
This will lead to emergency transphenoidal
palsy in addition. The discs were rather pale. On
debulking if necessary. Failure to demonstrate
examination now she is barely conscious with a
pituitary pathology should lead to angiography
pulse rate of 130 bpm and blood pressure of
to look for a berry aneurysm.
70/40 despite 500ml of intravenous colloid.
There is no evidence of haemorrhage and the
abdomen is soft. [ Q: 21 ] MasterClass Part2
A further litre of intravenous colloid fails to (2010) - Endocrinology
improve the situation substantially. The man shown has been referred to the
endocrine clinic by his GP who suspects that he
Which of the following diagnoses is most likely?
has acromegaly (see image).
A. Subarachnoid haemorrhage complicated
by a sympathetic storm
B. Opthalmoplegic migraine complicated by
a vasovagal collapse
C. Cavernous sinus thrombosis with raised
intracranial pressure
D. Pituitary haemorrhage with secondary
hypotensive shock
E. Phaeochromocytoma crisis with midbrain
infarction.

Answer & Comments


Which TWO of the following clinical / laboratory
Correct answer: D
findings would support his working diagnosis:

This woman is hypothyroid and has presumably A. Bitemporal hemianopia


experienced an early menopause, together B. Paradoxical growth hormone rise in
suggestive either of pituitary disease or response to a 75g oral glucose challenge
autoimmune polyglandular disease. The
C. Suppressed insulin-like growth factor 1
presentation with headache, third and sixth
(IGF-1) levels
nerve palsies, and refractory hypotension,
however, tips the balance towards pituitary D. Hypergonadotrophic hypogonadism
disease, such as a macroprolactinoma. Such
E. Proximal myopathy
large tumours are at risk of infarction or
haemorrhage, which rapidly expands them into F. Left homonymous hemianopia
the cavernous sinus and superiorly to the optic G. Normal pituitary MRI scan appearances
chiasm, causing ophthalmoplegia and/or
H. Anosmia
bitemporal hemianopia. It may also
masquerade as subarachnoid haemorrhage. In I. Macrocytic anaemia

Dr. Khalid Yusuf El-Zohry – Sohag Teaching Hospital (01118391123) Page | 22


El-Zohry MRCP Questions Bank (Part 2) – Medical Masterclass 2010

J. Hypohydrosis.

Answer & Comments

Correct answer: AB

Acromegaly is the clinical disorder resulting


from hypersecretion of growth hormone (GH).
Most cases are caused by a pituitary adenoma,
and of these 70-75% are macroadenomas
(>1cm in diameter) and 20-25% are
microadenomas (<1cm in diameter).
Macroadenomas frequently exhibit suprasellar
and / or cavernous sinus extension, and the
former may result in optic chiasm compression
with a bitemporal field defect.
In addition to GH hypersecretion, there may be
loss of other pituitary hormones (typically
gonadotrophins, then adrenocorticotrophic
She had been confined to her house for the
hormone (ACTH) and finally thyroid-stimulating
previous 2 months with increasing lethargy, and
hormone (TSH)) due to compression of the
had been experiencing worsening frontal
remaining normal pituitary tissue.
headaches with occasional double vision over
Biochemical confirmation of the diagnosis is the preceding 3 weeks. She said she had had a
dependent up on the demonstration of a knee replacement 2 years before, suffered mild
paradoxical rise or failure of suppression of GH angina, hypertension and hypothyroidism, and
in response to a glucose challenge, together had had major abdominal surgery some 20
with an elevated IGF-1 level. years before, though she wasn’t entirely clear
why. Her medication consisted of aspirin 75mg
[ Q: 22 ] MasterClass Part2 od, atenolol 50mg od, L-thyroxine 75mcg od,
hydrocortisone 10mg in the morning and 10mg
(2010) - Endocrinology
at lunchtime, and fludrocortisone 100 mcg od.
This 74-year-old woman was seen as an Her appearance is shown below. Blood pressure
emergency GP referral just before Christmas. was 160/95. Urea and electrolytes, full blood
count and glucose were within the normal
laboratory range. Free T4 was 16 pmol/l and
TSH<0.03 mU/l.

Which of the following is most likely?


A. She has untreated haemochromatosis
B. She is non compliant with her therapy
C. She has developed small bowel
malabsorption
D. She has an underlying small cell
carcinoma of the lung

Dr. Khalid Yusuf El-Zohry – Sohag Teaching Hospital (01118391123) Page | 23


El-Zohry MRCP Questions Bank (Part 2) – Medical Masterclass 2010

E. She has hyperplasia of the anterior Which of the followings is NOT raised in adult-
pituitary gland type congenital adrenal hyperplasia (non-
classical CAH)?
Answer & Comments A. 17 hydroxyprogesterone (17-OHP)
Correct answer: E B. Adrenal Corticotrophic Hormone (ACTH)
C. Cortisol
This patient has Nelson’s syndrome, as
D. Testosterone
evidenced by her intense, seasonally-
inappropriate pigmentation, the circumstancial E. Dehydroepiandrostenedione (DHEA)
evidence of hypopituitarism (drug regimen,
thyroid function tests, and history consistent Answer & Comments
with bilateral adrenalectomy in the past, with
recent expansion of pituitary mass), and the Correct answer: C
lack of historical or clinical evidence of
hypoadrenalism. With improvement in Adult-type, non-classical CAH with deficiency of
techniques for tumour localisation and 21-hydroxylase will lead to reduced production
extirpation, bilateral adrenalectomy as a second of cortisol. ACTH will rise through the feedback
line procedure is much rarer than formerly in process in order to increase cortisol production.
the treatment of Cushing’s disease, but when This will cause accumulation of hormones
performed, is associated with a risk of Nelson’s proximal to the block. These precursor
syndrome of between 8 and 45%. Nelson’s hormones will then get shunted down the
tumours can be aggressive and locally invasive, adjacent pathways leading to increased
and prophylactic pituitary radiotherapy after production of 17 OHP, testosterone,
adrenalectomy is favoured by many. androstenedione and DHEA. Consequently,
Monitoring is with ACTH levels and serial there will be hyperandrogenaemia causing
pituitary imaging. Pigmentation arises from the hirsutism, menstrual irregularities and
MSH products of the proteolysis of POMC, infertility. Measurement of 17 OHP levels alone
which also produces ACTH. Normoglycaemia, or preferably in response to a short synacthen
normal electrolytes and a good blood pressure test, is usually used as initial tests in the
suggest that this is not inadequately treated investigations of suspected CAH.
Addison’s disease, while in ectopic ACTH
production from an aggressive small cell [ Q: 24 ] MasterClass Part2
carcinoma, diabetes, hypokalaemia and severe
(2010) - Endocrinology
muscle weakness are characteristic. In this
situation, there is generally only a short term A 65-year-old woman has Paget's disease. She
between diagnosis and death. has increased bone pain and deformity in the
right femur. Blood tests reveal a raised alkaline
[ Q: 23 ] MasterClass Part2 phosphatase, and a radiograph of the femur
shows no evidence of fracture or of
(2010) - Endocrinology
osteosarcoma.
A 19-year-old university student is being
investigated for hirsutism and menstrual Apart from increasing her analgaesia, the
irregularity. She has had quite a few correct treatment is:
investigations. A. Commence an oral non-steroidal anti-
inflammatory drug

Dr. Khalid Yusuf El-Zohry – Sohag Teaching Hospital (01118391123) Page | 24


El-Zohry MRCP Questions Bank (Part 2) – Medical Masterclass 2010

B. Commence subcutaneous injections nmol/24 hours (normal values up to 270


of calcitonin nmol/l)
C. Commence intravenous pamidronate 0900 hrs cortisol 1450 nmol/l (Normal: 200-700
infusions nmol/l)
D. Commence oral alendronate 0900 hrs ACTH 82 ng/l (Normal: up to 80 ng/l)
E. Orthopaedic referral 0900 hrs cortisol suppressed to <50 nmol/l
following a 48 hours high dose dexamethasone
Answer & Comments suppression test

Correct answer: C Corticotrophin-releasing hormone (CRH) test:


Exaggerated Adrenocorticotrophic hormone
(ACTH) and cortisol response.
Paget's disease affects men and women almost
equally, but men tend to be more symptomatic. Which of the following conditions is most likely
The disease is usually not clinically apparent to be present?
until age 50 to 60. The pelvis, femur, spine and
A. Ectopic ACTH production
tibia are most commonly affected. It may be
discovered in asymptomatic patients because of B. Pseudocushings
an elevated serum alkaline phosphatase
C. Cushing's disease
obtained on routine screening.
D. Exogenous steroids
In the past, patients with Paget's disease were
simply observed until symptoms were clear cut. E. Cortisol producing adrenal tumour.
The treatment of choice currently is
intravenous infusion of sodium pamidronate. A Answer & Comments
single dose of 60mg can produce remission for
Correct answer: C
six months or longer in mild or limited Paget's
disease: those with more extensive disease
require multiple doses. This patient presents with weight gain and
hypertension. Further history and examination
Alendronate is approved in the US and
are obviously very important. If one suspects
risendronate and tiludronate are undergoing
Cushing's syndrome, 24 hours urinary free
trials.
cortisol should be checked as a screening tool
There is an increased risk of osteosarcoma and and repeated.
pathological fracture in Paget's disease, hence
As the patient seems to have significantly raised
radiological investigation is needed to exclude
levels of cortisol in urine, further investigations
these pathologies in a patient with increased
should be carried out. Basal cortisol is high and
bone pain.
basal ACTH is high normal. This would point
towards pituitary dependent Cushing's disease
[ Q: 25 ] MasterClass Part2 and away from other causes. But the patient
(2010) - Endocrinology needs further evaluation to confirm this.

A 44-year-old woman presents with weight gain Suppression of cortisol levels to high dose
and hypertension. dexamethasone test and an exaggerated ACTH
and cortisol response to CRH test points to the
Urinary free cortisol: First specimen: diagnosis of pituitary dependent Cushing’s
330nmols/24 hours; second specimen: 376 disease and excludes ectopic ACTH,
pseudocushings and other causes.

Dr. Khalid Yusuf El-Zohry – Sohag Teaching Hospital (01118391123) Page | 25


El-Zohry MRCP Questions Bank (Part 2) – Medical Masterclass 2010

Petrosal venous sampling before and after CRH Patients with primary hyperparathyroidism or
stimulation, showing greater than twice the Ca bronchus (with ectopic PTH production or
levels of ACTH as compared to peripheral bony metastases) will usually have
blood, confirms Cushing's disease and may also hypercalcaemia. This lady's renal function is
help in lateralizing the pituitary adenoma. grossly normal and the low calcium and fairly
low phosphate rule out tertiary
[ Q: 26 ] MasterClass Part2 hyperparathyroidism.

(2010) - Endocrinology Patients with uncomplicated Paget's disease


and osteoporosis usually have normal calcium
A 60-year-old lady of Gujrati origin presents levels and PTH levels.
with a 6-month history of generalized aches
and pains. Her corrected calcium is 1.67 mmol/l High levels of PTH exclude post-thyroidectomy
with albumin of 38g/l. Her creatinine is 88 hypocalcaemia.
micromol/l, alkaline phosphatse 560 U/l, In pseudohypoparathyroidism, PTH resistance
phosphate 0.80 mmol/l and parathyroid leads to high circulating PTH, with high
hormone (PTH) levels are 173 pg/ml (ref; upper phosphate and low calcium, sometimes in
limit of normal: 80pg/ml). association with multiple bony abnormalities. In
pseudopseudohypoparathyroidism, these bony
Which of the following are associated with this abnormalities occur without any derangement
picture? of calcium levels
A. Primary hyperparathyroidism
B. Osteoporosis [ Q: 27 ] MasterClass Part2
C. Pseudopseudohypoparathyroidism (2010) - Endocrinology

D. Previous thyroidectomy A 72-year-old woman is admitted with


headaches, polyuria and polydipsia of recent
E. Carcinoma of the bronchus
onset. She has previously had a mastectomy for
F. Chronic renal failure breast cancer and a CT head scan shows
multiple cerebral metastases.
G. Secondary hyperparathyroidism
Her admission biochemistry results are as
H. Paget's disease
follows: sodium 153 mmol/l, potassium 4.0
I. Tertiary hyperparathyroidism mmol/l, urea 5.0 mmol/l, creatinine 110
J. Osteomalacia. micomol/l, and glucose 5mmol/l. Over the
next 24 hours she has a urinary volume of 4.4
litres and further tests reveal plasma osmolality
Answer & Comments
320mOsm/kg and urinary osmolality:
Correct answer: GJ 254mOsm/kg.

Which one of the following treatments you will


This lady has osteomalacia. Because of chronic
use?
vitamin D deficiency and associated
hypocalcaemia she has developed secondary A. Hypotonic saline
hyperparathyroidism, as evidenced by the
B. Hydrochlorthiazide
markedly increased levels of parathormone
(PTH). C. Desmopressin (DDAVP)
D. Demeclocycline

Dr. Khalid Yusuf El-Zohry – Sohag Teaching Hospital (01118391123) Page | 26


El-Zohry MRCP Questions Bank (Part 2) – Medical Masterclass 2010

E. Water restriction J. Calcitonin.

Answer & Comments Answer & Comments

Correct answer: C Correct answer: AH

This woman appears to have cranial diabetes Chronic hypopituitarism may present with
insipidus secondary to metastatic disease from general fatigue, symptoms of hypogonadism
her breast cancer. Her investigations are not and possibly (but less likely) symptoms of
complete, but out of the above choices, DDAVP hypothyroidism and hypoadrenalism. Physical
(in the form of nasal spray or subcutaneous signs will be those of the primary hormone
injections) appears to be the most suitable deficiency syndromes and / or those relating to
treatment. the presence of a pituitary mass, e.g.
bitemporal hemianopia.
Hypotonic saline (or 5% dextrose) would have
the effect of restoring her serum sodium If hypopituitarism is suspected then
towards normal, but replacing ADH should be investigation should include:
the aim, with the patient allowed to drink
1. Complete biochemical assessment of
according to her thirst.
pituitary function
Hydrochlorthiazide may be used to treat
2. MRI (or CT) scan of pituitary fossa
nephrogenic diabetes insipidus. Demeclocycline
and lithium carbonate actually induce 3. Formal testing of the patient’s visual
nephrogenic diabetes insipidus and may be fields and acuity.
used in certain cases of SIADH. Water
restriction will be very dangerous in the given [ Q: 29 ] MasterClass Part2
clinical scenario for obvious reasons. (2010) - Endocrinology

[ Q: 28 ] MasterClass Part2 Turner's Syndrome is associated with which of


(2010) - Endocrinology the following:
A. Autosomal recessive inheritance
A 48-year-old man has a pituitary tumour: the
B. Elevated luteinizing hormone (LH) and
function of which TWO pituitary hormone
systems is most likely to be lost? follicle-stimulating hormone (FSH) levels

A. Growth hormone (GH) C. 45 XY karyotype

B. Adrenocorticotrophic hormone (ACTH) D. Decreased risk of heart disease

C. Thyrotropin-releasing factor (TRF) E. Hyperthyroidism.

D. Thyroid-stimulating hormone (TSH)


Answer & Comments
E. Antidiuretic hormone (ADH)
Correct answer: B
F. Melatonin
G. Thyroxine Patients with Turner's syndrome classically have
webbed neck, low hairline, widely spaced
H. Luteinizing hormone (LH) and follicle-
nipples and cubitus valgus.
stimulating hormone (FSH)
I. Insulin

Dr. Khalid Yusuf El-Zohry – Sohag Teaching Hospital (01118391123) Page | 27


El-Zohry MRCP Questions Bank (Part 2) – Medical Masterclass 2010

Women with Turner's syndrome are at patients. An electrocardiogram will classically


increased risk of ostoeporosis. They have a show low voltage changes with bradycardia.
three-fold increased risk of heart disease.
Laevo-thyroxine is preferred to triiodothyronine
Turner's syndrome is associated with primary because of its longer half life and greater
autoimmune hypothyroidism. chance of compliance with once a day
treatment.
Karyotyping usually reveals 45X0. If there is
only partial X chromosome deletion then the
clinical features will be attenuated. There may [ Q: 31 ] MasterClass Part2
occasionally be mosaicism with some Y (2010) - Endocrinology
chromosome material - 45 XO/46XY.
A 65-year-old lady is admitted to the coronary
care unit with unstable angina. Three days later
[ Q: 30 ] MasterClass Part2 when she has been discharged you notice that
(2010) - Endocrinology her random blood glucose result on admission
was 10.2 mmol/l.
A 77-year-old lady has been referred by her
general practitioner with an eight-month
What would be the most appropriate course of
history of constipation, lethargy and weight
action?
increase. Thyroid function tests reveal her to be
hypothyroid. A. Take no action as she would have been
stressed because of the admission.
Which of the following statements is correct?
B. Wait for her to be admitted as planned
A. The commonest cause of primary thyroid for a coronary angiogram and take a
failure is Hashimoto's thyroiditis. repeat random glucose then.
B. The most discriminatory clinical feature C. Telephone the lady and if she has
of hypothyroidism is absent ankle jerks. hyperglycaemic symptoms advise the GP
to commence oral hypoglycaemic
C. In primary hypothyroidism cardiac
treatment.
evaluation is unimportant.
D. Telephone the lady and if she has
D. An electrocardiogram will show high
hyperglycaemic symptoms arrange for
voltage changes with bradycardia.
her to have an oral glucose tolerance test.
E. Triiodothyronine therapy rather than
E. Telephone the lady and arrange for her to
laevothyroxine is preferred in older
have an oral glucose tolerance test.
patients.

Answer & Comments


Answer & Comments
Correct answer: E
Correct answer: A

She needs to have an oral glucose tolerance


The commonest cause of primary thyroid
test whether she is symptomatic or not to
failure is Hashimoto's thyroiditis.
confirm the diagnosis. In a person with
The most discriminatory clinical feature for hyperglycaemic symptoms and signs (e.g. thirst,
hypothyroidism are slow-relaxing tendon jerks. polyuria, weight loss) and a venous plasma
In primary hypothyroidism cardiac evaluation is glucose greater than 11.1 mmol/l the diagnosis
important, especially in middle aged and older can be made without further testing. In other
cases when there are no symptoms or signs and
Dr. Khalid Yusuf El-Zohry – Sohag Teaching Hospital (01118391123) Page | 28
El-Zohry MRCP Questions Bank (Part 2) – Medical Masterclass 2010

the glucose is less than 11.1 mmol/l then a 75-g Hypercalcaemia and primary
OGTT should be performed. hyperparathyroidism are recognized side
effects of long-term treatment with lithium in
A fasting glucose is insufficient to confirm or
subjects with chronic affective psychiatric
dispute the diagnosis, as this may be normal
disorders. It has been suggested that lithium
even though a 2-hour glucose is elevated.
alters the sensitivity of the parathyroid cells to
In this scenario the diagnosis should be calcium and, perhaps not surprisingly therefore,
confirmed or refuted in order that her risk all four glands may be affected (hyperplasia).
factors can be treated as appropriate. The first Other studies have however failed to confirm
treatment should be dietary advice. an excess of parathyroid hyperplasia in this
If her elevated blood sugar was noticed during population, suggesting instead that lithium
her admission then her acute management selectively stimulates growth of parathyroid
should have included insulin as shown in the adenomas in susceptible patients, who are best
DIGAMI study. Mortality rates were significantly treated therefore with adenoma excision rather
decreased, at 12 months, in those individuals than total parathyroidectomy.
found to have elevated blood glucose levels A number of the selective serotonin re-uptake
who were commenced on an insulin infusion inhibitors and related antidepressants have
and then subcutaneous insulin continued after been recognized to cause hyponatraemia
discharge. There are further trials ongoing to (possibly through inappropriate anti-diuretic
determine if the improvement in survival is due hormone secretion).
to the improved blood glucose in the first 24
hours or to the insulin over the following 12
[ Q: 33 ] MasterClass Part2
months
(2010) - Endocrinology
[ Q: 32 ] MasterClass Part2 A 36-year-old woman is admitted with lower
(2010) - Endocrinology limb cellulitis, and an astute house officer notes
the patient to have a Cushingoid appearance. A
A 57-year-old woman with a long history of 24-hour urinary free cortisol during the
depressive illness is referred to the endocrine inpatient stay is elevated at 430 nmol (normal
clinic for investigation of hypercalcaemia <270).
(corrected serum calcium level 2.75 mmol/l –
normal range 2.1-2.5), which was discovered Which of the following is true of her further
incidentally. management?
A. A low dose dexamethasone suppression
Which of the following drugs is most likely to be
test will be sufficient to rule out pseudo-
of relevance?
Cushing’s syndrome.
A. Citalopram
B. Low dose dexamethasone supression
B. Haloperidol followed immediately by metyrapone
C. Lithium stimulation testing is highly sensitive and
specific for a diagnosis of Cushing’s
D. Venlafaxine
disease.
E. Amitriptyline C. The most likely underlying pathology in
endogenous Cushing’s syndrome is a
Answer & Comments pituitary macroadenoma.
Correct answer: C

Dr. Khalid Yusuf El-Zohry – Sohag Teaching Hospital (01118391123) Page | 29


El-Zohry MRCP Questions Bank (Part 2) – Medical Masterclass 2010

D. Ketoconazole can be used initially to E. It can be used to diagnose partial


control the disease medically. pituitary ACTH deficiency.
E. A random plasma cortisol is likely to F. If the patient is taking hydrocortisone this
provide useful supporting evidence of should be omitted on the morning of the
hypercortisolism. test.
G. If patient is taking dexamethasone this
Answer & Comments should be continued on the morning of
Correct answer: D the test.
H. A cortisol result of 750 nmol/L during the
Cushing’s syndrome, when not due to test would indicate sufficient adrenal
exogenous steroids, is in around 80% of cases reserve.
due to a pituitary adenoma. Because these tend I. Blood samples are taken for cortisol at 0,
to present early, they are generally 20, 40 and 60 minutes.
microadenomas (<1cm). Although random
cortisol levels are unhelpful, carefully taken 9 J. It should be used with caution in patients
a.m. and midnight levels can provide important known to have asthma.
evidence of the loss of the circadian cortisol
profile characteristic of Cushing’s syndrome. Answer & Comments
Low dose dexamethasone suppression may
Correct answer: EI
lead to false positives in pseudo-Cushing's,
while combined low dose dexamethasone
Administration of tetracosactrin, a synthetic
suppression test (LDDST) and corticotrophin-
releasing hormone (CRH) stimulation is highly ACTH (also known as SYNACTHEN) allows the
sensitive and specific. Medical suppression of acute adrenal response to ACTH to be
steroid biosynthesis can be a useful prelude to measured. The test can be used to diagnose
both primary and secondary adrenal
surgery, with agents such as metyrapone or
insufficiency and assists in the diagnosis of
ketoconazole.
congenital adrenal hyperplasia.

[ Q: 34 ] MasterClass Part2 250 ug of tetracosactrin should be given either


intramuscularly or intravenously at 09.00 hours.
(2010) - Endocrinology
Blood samples for cortisol should be taken
A 27-year-old lady is referred to you with immediately prior to the injection and then 30
weight loss and tiredness. and 60 minutes later. Blood samples for ACTH
should be taken prior to the injection and will
Which TWO of the following statements are help distinguish between primary and
incorrect regarding the short synacthen test? secondary adrenal failure.
A. Tetracosactrin is administered to the A normal short synacthen test does not exclude
patient. partial pituitary ACTH deficiency in patients
B. Synthetic adrenocorticotrophic hormone whose basal ACTH production is sufficient to
(ACTH) is administered to the patient. prevent adrenal atrophy, but in whom the
ACTH response to stress is attenuated.
C. It can be used to diagnose primary
adrenal insufficiency.
D. It can be used to diagnose secondary
adrenal insufficiency.

Dr. Khalid Yusuf El-Zohry – Sohag Teaching Hospital (01118391123) Page | 30


El-Zohry MRCP Questions Bank (Part 2) – Medical Masterclass 2010

[ Q: 35 ] MasterClass Part2 The main significance of microalbuminuria in


(2010) - Endocrinology both type 1 and type 2 diabetes is the increased
risk that microalbuminuria confers on later
You see a 31-year-old gentleman with type 1 developing overt proteinuria and increased
diabetes for his annual review. The clinic nurse mortality. In those patients with
asks you if you would like his urine sending to microalbuminuria, normalisation of blood
the laboratory and if so what would you like it pressure, optimal glycaemic control and
sending for. correcting cardiovascular risk factors is
recommended. Angiotensin-converting enzyme
Which of the following is the most appropriate (ACE) inhibitors reduce albumin excretion in
response? non-hypertensive diabetic patients with
A. The dipstick urine is negative. Request microalbuminuria.
the urine be sent for an MSU. If a patient is shown to have persistent
B. The dipstick urine is positive for protein microalbuminuria they should be on an ACE
and blood. Request the urine be sent for inhibitor unless there are other contra-
microalbuminuria testing. indications.
C. The dipstick is positive for protein. The In the above scenario if a dipstick urine is
patient has a creatinine of 345umol/l. negative there is no indication for the urine to
Request the urine be sent for be sent for an MSU. If the dipstick is positive for
microalbuminuria testing. protein and blood the urine should be sent for
an MSU to exclude an infection, before it is sent
D. The dipstick is negative. The patient has a
for microalbuminuria. If a patient has a creatine
creatinine of 85 umol/l. Request the urine
of 345 umol/l with proteinuria there is no
be sent for microalbuminuria testing.
added benefit of checking a microalbuminuria.
E. The dipstick is positive for protein. There If a patient is known to have microalbuminuria
are three previous results in the notes there is no further benefit from repeating this
showing microalbuminuria. Request the test - they should be being treated already.
urine be sent for microalbuminuria
testing.
[ Q: 36 ] MasterClass Part2
(2010) - Endocrinology
Answer & Comments
A 58-year-old female presents to her GP with
Correct answer: D
symptoms suggestive of bilateral carpal tunnel
syndrome. He suspects that she has acromegaly
Early microalbuminuria is due to an increase in and refers her to the endocrine clinic for further
transglomerular pressure, which is followed by evaluation.
loss of charge on the basement membrane and
thus reduced charge repulsion between the Which of the following findings would support
membrane and the polyanionic albumin his provisional diagnosis?
molecule.
A. Random growth hormone (GH) level of
Microalbuminuria or proteinuria supervenes 6mU/l
when the pore size enlarges. Once persistent
B. GH nadir of 1.0mU/l during an oral
proteinuria has developed, renal function
glucose tolerance test (OGTT)
usually declines gradually but progressively
towards end-stage renal failure. C. Elevated insulin-like growth factor 1 (IGF-
1) level

Dr. Khalid Yusuf El-Zohry – Sohag Teaching Hospital (01118391123) Page | 31


El-Zohry MRCP Questions Bank (Part 2) – Medical Masterclass 2010

D. Left homonymous hemianopia on formal [ Q: 37 ] MasterClass Part2


visual field testing (2010) - Endocrinology
E. Elevated gonadotrophin (FSH, LH) levels.
A 33-year-old man is referred to the endocrine
clinic for routine follow-up having recently
Answer & Comments moved into the region. He has previously
Correct answer: C undergone a total parathyroidectomy for
hyperparathyroidism, arising in the setting of
multiple endocrine neoplasia type 1 (MEN1), a
Whilst GH has some direct actions (e.g.
diagnosis that has been confirmed through
antagonism of insulin), many of its effects are
genetic screening.
mediated through IGF-1, which is produced by
the liver and, accordingly, in acromegaly IGF-1 Even in the absence of symptoms, it would be
levels are typically elevated above the age- and appropriate to check which TWO of the
sex-related reference range. following biochemical parameters?
Unless the GH level is dramatically elevated, the A. Fasting calcitonin
diagnosis of acromegaly should not be made on
the basis of a single random GH measurement - B. Serum prolactin
if sampling coincides with a normal C. Urinary catecholamines
physiological ‘ spike’ in GH secretion a false
D. Urinary 5 hydroxyindoleacetic acid (5-
positive result may ensue. Indeed, even in the
HIAA)
presence of an obviously elevated GH, the
OGTT is still considered the ‘gold standard’ test E. Serum corrected calcium
for diagnosing this disorder – in normal subjects F. Free T3
GH levels should fall to a nadir of <2mU/l (many
actually suppress to <1mU/l). In contrast, in G. Serum ferritin
acromegaly not only do the levels fail to H. Cortisol response to synacthen
suppress adequately, but in many instances a
I. Plasma glucose
paradoxical rise in GH is observed in response
to a glucose challenge. J. Serum thyroid stimulating hormone (TSH).

The majority of cases of acromegaly


(approximately 75%) arise in the setting of a Answer & Comments
pituitary macroadenoma (>1cm in diameter). Correct answer: BE
These tumours may encroach on the optic
chiasm, classically resulting in a bitemporal
The key components of the MEN1 syndrome
visual field defect. In addition, the remaining
are:
normal anterior pituitary tissue is often
compressed – in these circumstances one might 1. Parathyroid hyperplasia
expect the gonadotrophin levels to fall within or 2. Pancreatic tumours (e.g. gastrinoma,
below the reference range for younger females, insulinoma)
which would of course be inappropriate in the
post-menopausal state, where the 3. Pituitary tumours (most commonly
gonadotrophins are typically elevated. prolactinomas).
Medullary thyroid carcinoma (calcitonin) and
phaeochromocytoma (urinary catecholamines)

Dr. Khalid Yusuf El-Zohry – Sohag Teaching Hospital (01118391123) Page | 32


El-Zohry MRCP Questions Bank (Part 2) – Medical Masterclass 2010

are typically associated with the MEN2 Answer & Comments


syndrome.
Correct answer: D
Although Carcinoid tumours occur in a small
number (3-5%) of patients with MEN1, routine Addison’s disease is associated with other
screening with urinary 5-HIAA is not generally
autoimmune conditions in the autoimmune
advocated in asymptomatic individuals. polyglandular syndromes (both type 1 and type
Measurement of a random plasma glucose level 2). It is type 1 diabetes mellitus, another
is not a useful means of screening for the autoimmune condition, that is associated with
presence of an insulinoma. This diagnosis Addison’s disease and not type 2 diabetes.
should be based on Whipple’s triad Presence of macrocytosis in the context of
(biochemical hypoglycaemia with symptoms autoimmunity associated with Addison’s
that are relieved by administration of glucose). disease raises the possibilty of co-existing
Despite the history of prior ‘total’ pernicious anaemia. It is usually a normocytic
parathyroidectomy, it is necessary to normochromic anaemia that may be present in
periodically monitor serum calcium levels to Addison’s disease. There may be neutropenia
ensure adequacy of replacement with vitamin and eosinophilia as well.
D/calcium supplements, and in case the Typically, Addison’s disease, due to lack of
individual harbours an additional ectopically
steroids, is associated with hyponatraemia and
sited gland that could result in a relapse. hyperkalaemia. There may also be metabolic
acidosis. Hypokalaemic alkalosis is a feature of
[ Q: 38 ] MasterClass Part2 steroid excess as in Cushing’s syndrome.
(2010) - Endocrinology As mentioned earlier, there is an association
A 22-year-old medical student is found to have with other autoimmune conditions. Addison’s
Addison's disease. She asks an enormous disease may be associated with autoimmune
number of questions. thyroid disease. At other times, there may be
abnormalities of thyroid function present that
Which of the following of her statements is true may revert back to normal with glucocorticoid
regarding Addison’s disease? therapy.

A. Addison’s disease has a recognised Adrenal antibodies are present in many


association with type 2 diabetes mellitus. patients with Addison’s disease but are not
essential for making a diagnosis. The diagnosis
B. Presence of macrocytosis suggests co-
of Addison’s disease is made on clinical
existing autoimmune haemolytic
grounds, along with typical abnormalities of
anaemia.
biochemistry, cortisol and adenocorticotropic
C. Hypokalaemic alkalosis is a feature of hormone (ACTH) levels, and provocative tests
Addison’s disease. like the Short Syacthen Test.
D. Thyroid function abnormalities may
revert to normal with glucocorticoid [ Q: 39 ] MasterClass Part2
therapy. (2010) - Endocrinology
E. Diagnosis is made on detecting adrenal A 29-year-old woman complains of excessive
antibodies in the blood.
weight gain, despite repeated attempts to diet,
following the birth of her second child 2 years
earlier. Her only regular medication is the

Dr. Khalid Yusuf El-Zohry – Sohag Teaching Hospital (01118391123) Page | 33


El-Zohry MRCP Questions Bank (Part 2) – Medical Masterclass 2010

combined oral contraceptive pill. Her BMI is 38 pituitary adenoma. Weight gain occurs in some
kg/m2. She has mild facial hirsutism, but the patients on the combined oral contraceptive
remainder of the physical examination is pill, but this is usually modest and unlikely to
unremarkable. explain the gross obesity in this case. Although
the patient has a mildly elevated antithyroid
Investigations reveal fasting plasma glucose 4.6
peroxidase titre and a free T4 level in the lower
mmol/L (normal range 3-6), plasma free T4 10.5
part of the reference range, the plasma thyroid-
pmol/L (normal range 10-22), plasma thyroid-
stimulating hormone level, which is the most
stimulating hormone 1.2 mU/L (normal range
sensitive indicator of primary thyroid
0.4-5.0), serum antithyroid peroxidase 100
dysfunction, is unequivocally normal.
IU/mL (normal <50), midnight serum cortisol 75
nmol/L (normal <100), 24-hour urinary free
cortisol 240 nmol/L (normal range 55-250), and [ Q: 40 ] MasterClass Part2
cortisol after overnight dexamethasone (2010) - Endocrinology
suppression test 90 nmol/L (normal <50).
A 70-year-old man presents to A&E drowsy and
What is the most likely cause for her weight irritable. The following results are available:
gain? sodium 159 mmol/l; potassium 4.5 mmol/l;
chloride 105 mmol/l; bicarbonate 29 mmol/l;
A. Cushing's disease
serum urea 15 mmol/l; serum glucose 50
B. Cushing's syndrome mmol/l. Urinary examination is negative for
C. Exogenous oestrogen therapy ketones.

D. Simple obesity The following treatments are advisable EXCEPT:


E. Subclinical hypothyroidism A. Subcutaneous heparin 5000iu twice daily
B. Blood cultures and commencement of
Answer & Comments broad spectrum antibiotics
Correct answer: D C. Regular serum potassium measurements
D. Rapid infusion of 0.45% saline IVI
Although the overnight dexamethasone
suppression test is a sensitive test for the E. Intravenous insulin using a sliding scale.
exclusion of Cushing's syndrome, false-positive
results may occur in patients who are obese, on Answer & Comments
enzyme-inducing medications, suffering with
Correct answer: D
depressive illness, or consuming excessive
alcohol. In addition, the use of exogenous
oestrogen therapy raises cortisol-binding This patient has hyperosmolar nonketotic coma
globulin levels and thus measured serum total with a sodium >150 mmol/l and severe
cortisol, which may also yield a false-positive hyperosmolality ((2x(Na+K)+glucose+urea)).
result. This is often precipitated by intercurrent
infection. These patients are profoundly
In this case, the finding of a normal midnight dehydrated and at risk of venous thrombosis.
cortisol level and normal urinary cortisol They need resuscitation and rehydration.
excretion makes the diagnosis of Cushing's Careful use of 0.45% N/saline would seem
syndrome unlikely. Cushing's disease refers advantageous as long as correction of
exclusively to cases of Cushing's syndrome hypernatraemia does not occur too quickly; use
arising as a consequence of a corticotroph N/saline once sodium less than 150. Insulin

Dr. Khalid Yusuf El-Zohry – Sohag Teaching Hospital (01118391123) Page | 34


El-Zohry MRCP Questions Bank (Part 2) – Medical Masterclass 2010

should be given intravenously using a sliding [ Q: 42 ] MasterClass Part2


scale. (2010) - Endocrinology
A surgical house officer notices that a patient
[ Q: 41 ] MasterClass Part2
admitted for elective cholecystectomy looks as
(2010) - Endocrinology if they might be acromegalic.
A 54-year-old man presents with renal calculi
They ask you what the best way to establish the
and bone pain. His mother was told she had a
diagnosis is. You answer:
high calcium level and his grandmother died of
a perforated duodenal ulcer. A. Oral glucose tolerance test
B. CT scan pituitary fossa
Which of the following findings favours a
diagnosis of Multiple Endocrine Neoplasia Type C. Random growth hormone level
1?
D. Insulin tolerance test
A. Raised serum calcium with a suppressed
E. Lateral skull radiograph.
parathyroid hormone (PTH)
B. Family history consistent with autosomal Answer & Comments
recessive pattern of inheritance
Correct answer: A
C. A prolactin level within the reference
range
Growth hormone (GH) secretion is pulsatile,
D. Medullary thyroid carcinoma hence a single random GH level is not a reliable
E. Hypercalcuria. assessment of GH secretion.
After 75 g of oral glucose suppression of GH to
Answer & Comments <1 mU/l excludes the diagnosis of acromegaly.

Correct answer: E Once the diagnosis is established, imaging of


the pituitary fossa (preferably by MRI) can
distinguish micro- from macro- adenomas.
The association of parathyroid, pancreatic islet
Visual acuity, visual fields and anterior pituitary
and pituitary hyperplasia or neoplasia is called
function also require formal assessment.
MEN1, which is an autosomal dominant
disease. Findings consistent with the diagnosis
include a raised corrected serum calcium with a [ Q: 43 ] MasterClass Part2
normal or elevated PTH measurement, (2010) - Endocrinology
indicating primary hyperparathyroidism.
Hypercalciuria resulting from a high serum A 74-year-old man is readmitted to hospital
calcium level may be found, as may elevated having recently been treated for
prolactin measurements, possibly indicating decompensated congestive cardiac failure. On
prolactinoma. his last admission he had been started on
spironolactone in addition to his loop diuretic
Medullary thyroid carcinoma is associated with and angiotensin converting enzyme inhibitor.
MEN 2. His serum potassium was 7.2 mmol/l.

Which one of the following is correct with


respect to his 12 lead ECG?
A. Only the T waves will be affected

Dr. Khalid Yusuf El-Zohry – Sohag Teaching Hospital (01118391123) Page | 35


El-Zohry MRCP Questions Bank (Part 2) – Medical Masterclass 2010

B. The ECG may show diagnostic changes of micromol/l, corrected calcium 2.80 mmol/l,
hyperkalemia phosphate 1.9 mmol/l. A 24 hour urinary
collection reveals creatinine clearance 18
C. Expect to see an increase in P wave
ml/min.
amplitude
Parathyroid hormone (PTH), performed to
D. Tall T waves only occur in hyperkalaemia
assess the hypercalcaemia, is 65.5 pmol/l
E. The serum potassium cannot be (normal range <7.2 pmol/l).
accurately predicted from the ECG
changes. The most likely diagnosis is:
A. Primary hyperparathyroidism
Answer & Comments
B. Secondary hyperparathyroidism
Correct answer: E
C. Tertiary hyperparathyroidism
D. Pseudohypoparathyroidism
As serum potassium rises the following
alterations to the ECG occur progressively: E. Hypercalcaemia secondary to exogenous
tenting of the T waves, diminution of the P replacement therapy.
wave and lengthening of the PR interval,
broadening of the QRS complex. Answer & Comments
There are no diagnostic changes of
Correct answer: C
hyperkalaemia. All changes are non-specific and
may be seen, for example, in acute myocardial
PTH is raised, calcium is raised and so is
infarction. Tall T waves may occur as a normal
phosphate, whilst creatinine clearance is
variant or in true posterior ischaemia.
deceased. The raised calcium excludes
There is no close correlation between the secondary hyperparathyroidism, since calcium
serum potassium concentration and the is normal and PTH is raised in this condition.
morphology of the ECG. The ECG changes
Pseudohypoparathyroidism may lead to PTH
indicate the effect of potassium on an
and phosphate levels this high, but calcium
individual's heart, but there is wide variability in
should be low. Exogenous replacement therapy
sensitivity to rising serum potassium. Patients
(unless delivered in primary
who are chronically hyperkalaemic will have
hyperparathyroidism) should suppress PTH.
lesser changes on the ECG than those with
acute hyperkalaemia. Primary hyperparathyroidism should lead to
low phosphate levels and rarely leads to PTH
[ Q: 44 ] MasterClass Part2 levels this high (though it has been known). The
situation is complicated by renal failure,
(2010) - Endocrinology
however, and it is possible that decreased renal
A 67-year-old woman is admitted following a phosphate excretion due to renal failure in the
fall. She is known to be hypertensive and is on presence of primary hyperparathyroidism could
treatment with an angiotensin-converting lead to these results, but the most likely
enzyme (ACE) inhibitor. She has long standing explanation is tertiary hyperparathyroidism. In
chronic renal failure that is presumed to be this condition prolonged hypocalcaemia (due to
secondary to hypertension. deficient vitamin D hydroxylation in the kidney
in this case but also occurring in
Routine bloods show: Na+ 136 mmol/l, K+ 5.2
malabsorption), leads to parathyroid over-
mmol/l, Urea 20.1 mmol/l, Creatinine 363
activity to maintain serum calcium

Dr. Khalid Yusuf El-Zohry – Sohag Teaching Hospital (01118391123) Page | 36


El-Zohry MRCP Questions Bank (Part 2) – Medical Masterclass 2010

concentrations. This initially corrects [ Q: 46 ] MasterClass Part2


hypocalcaemia (secondary (2010) - Endocrinology
hyperparathyroidism) then may proceed to
unrestrained parathyroid over-activity A 38-year-old lady is seen in the clinic with a 2-
(autonomous gland function and hyperplasia) year history of secondary amenorrhoea. Her
with hypercalcaemia. PTH levels are typically periods actually did not resume after the birth
high, phosphate levels are high secondary to of her second child. She did not breast feed her
renal failure, and creatinine clearance is baby. She is also complaining of mood swings
markedly reduced (typically < 30ml/min). All and occasional hot flushes. Clinically, she has
these features are present in this patient. vitiligo but nothing else of note. There is family
history of thyroid disorders and type 1 diabetes.
Her blood results are as follows:
[ Q: 45 ] MasterClass Part2
(2010) - Endocrinology  FSH: 34 u/l (3-8 u/L)
 LH:21 u/l (3-10 u/L)
A 20-year-old student presents with
amenorrhoea. She had menarche at age 13  Oestradial: <70 (low)
years and a regular menstrual cycle before  Prolactin: 550 mU/L (normal up to 500)
going to college, where she has taken up
serious running. She runs for about 10 miles What is the most likely diagnosis?
every day and is training for next year’s London
A. Sheehan's syndrome
marathon. Examination is unremarkable,
excepting that she is thin with BMI 19. B. Premature ovarian failure
C. Prolactinoma
The most likely cause of her amenorrhoea is:
D. Carcinoid syndrome
A. pregnancy
B. prolactinoma E. Polycystic ovary syndrome.

C. excessive exercise
Answer & Comments
D. premature ovarian failure
Correct answer: B
E. polycystic ovarian syndrome.
In this case there are quite a few hints towards
Answer & Comments polyglandular autoimmune syndrome. She has
family history of thyroid disorders and type 1
Correct answer: C
diabetes mellitus. She herself has vitiligo.
Premature ovarian failure is the most likely
The ‘female athletic triad’ consists of
diagnosis.
disordered eating, amenorrhoea and
osteoporosis. It is believed that energy Presence of raised gonadotrophins in
imbalance (lots of exercise, low calorie intake) association with low oestradial levels suggests
leads (mechanism uncertain) to suppression of primary ovarian failure. This denotes that the
the activity of the gonadotrophin-releasing patholgy is at the ovarian level (e.g.,
hormone (GnRH) pulse generator, and thereby autoimmune ovarian damage as in this case).
to amenorrhoea. Hence the gonadotrophins are high through the
negative feedback mechanism. Prolactin is only
minimally raised, so is unlikely to have caused
amenorrhoea. Furthermore, gonadotrophins

Dr. Khalid Yusuf El-Zohry – Sohag Teaching Hospital (01118391123) Page | 37


El-Zohry MRCP Questions Bank (Part 2) – Medical Masterclass 2010

are high rather than low, making pituitary Androstenedione is produced from both the
causes unlikely. adrenals and ovaries, while
dehydroepiandrosterone sulfate (DHEAS) is
Polycystic ovary syndrome may be associated
adrenal-specific.
with secondary amenorrhoea but none of the
other features and biochemical results seem Mildly elevated testosterone levels are
related to PCOS. common in benign cases of hirsutism (such as
those due to polycystic ovary syndrome
[ Q: 47 ] MasterClass Part2 (PCOS)), with 5nmol/l suggested as a cut-off for
more urgent investigation of possible
(2010) - Endocrinology
underlying malignancy.
A 25-year-old woman is referred complaining Antiandrogens must never be prescribed alone
of excessive hair growth on her face and where pregnancy is a possibility due to their
stomach. On further questioning she says this teratogenicity, and the slow hair growth cycle
has worsened greatly over the past 2 years, means that at least 3, and preferably 6-12
together with significant weight gain after a months should elapse before a treatment is
change of job. Her menses have always been deemed to have failed.
rather erratic, but this has also become more
marked recently. She takes no medication. On In this case the likely cause of hirsutism is PCOS
examination her BMI is 30 kg/m2 and blood exacerbated by the insulin resistance of obesity.
pressure 140/85 mmHg. She has recently Insulin sensitisation through weight loss would
depilated her face, but has a marked have a salutary effect.
escutcheon and sparse hairs on her nipples.
There is no sign of masculinisation or other
obvious abnormality. The serum testosterone
level is 3 nmol/l (upper limit of normal, 2).

Which of the following is true with respect to


her further management?
A. Androstenedione measurement would
specifically be useful in detecting adrenal
disease.
B. The elevated testosterone is strongly
suggestive of an underlying virilising
tumour.
C. Cyproterone acetate alone would be an
appropriate initial treatment.
D. A further assessment should be made in
4-6 weeks, and treatment changed if
ineffective.
E. Sustained weight loss would be likely to
lessen the hirsutism.

Answer & Comments

Correct answer: E

Dr. Khalid Yusuf El-Zohry – Sohag Teaching Hospital (01118391123) Page | 38


El-Zohry MRCP Questions Bank (Part 2) – Medical Masterclass 2010

[ Q: 48 ] MasterClass Part2 B. Commence anti-resorptive therapy like


(2010) - Endocrinology Alendronate.
C. Advise her to take high calcium diet and
Which of the following statements about exercise regularly.
necrobiosis lipoidica diabeticorum (see image) is
D. Start her on Hormone Replacement
true?
Therapy (HRT).
A. Lesions can occur on legs or arms.
E. Consider antiresorptive therapy only if
B. Lesions usually resolve spontaneously steroids are continued beyond 2 years.
after 4-5 years.
C. Topical steroids are of benefit. Answer & Comments
D. Lesions should usually be biopsied to Correct answer: B
confirm the diagnosis.
E. It is more common in males than in According to the recent guidelines from the
females. Royal College of Physicians, men and women
over the age of 65 yr who are on chronic (more
Answer & Comments than 3 months) steroid therapy should be
started on treatment such as antiresorptive
Correct answer: A drugs to prevent osteoporosis. They should go
onto such therapy at the same time as they
Necrobiosis lipoidica diabeticorum usually commence steroids. A DEXA scan is not
affects the shins but may occur on arms or even required as they need preventive therapy
on the trunk. The lesions are chronic and rarely regardless of the result such a scan might show.
resolve. There is no effective treatment. Topical HRT may have a limited role early in the
or injected steroids are sometimes used but postmenopausal period, but only for a limited
have not been shown to be of any benefit. time and mainly for vasomotor symptoms. At
Females are affected much more than males. the age of 66 yr HRT is not recommended for
These lesions are unrelated to microvascular osteoporosis prevention therapy.
complications. The lesions show an atrophic
centre with dilated capillaries and a slight raised [ Q: 50 ] MasterClass Part2
pinkish rim. Ulceration may occur in the centre.
(2010) - Endocrinology

[ Q: 49 ] MasterClass Part2 A 40-year-old medical secretary is being


investigated for hypertension. Her 24-hour
(2010) - Endocrinology
urinary free cortisol levels were high on two
A 66-year-old lady was diagnosed with occasions and she has significantly raised serum
polymyalgia rheumatica and was commenced cortisol levels with loss of diurnal rhythm.
on long-term steroid therapy. Looking at her
chest radiograph, your house officer thinks that Which of the following features would favour
she may already have osteoporosis. benign adrenal adenoma as the cause of her
Cushing's syndrome over the other causes?
Which one of the following steps would you A. Presence of hypokalaemia
take?
B. Absence of hirsutism
A. Arrange out-patient DEXA scan and treat
only if she has osteoporosis. C. Presence of weight loss

Dr. Khalid Yusuf El-Zohry – Sohag Teaching Hospital (01118391123) Page | 39


El-Zohry MRCP Questions Bank (Part 2) – Medical Masterclass 2010

D. Absence of typical features of Cushing's C. 31-year-old pregnant lady with blood


syndrome glucose of 8.7 mmol/l
E. Normal MRI of the pituitary.. D. 31-year-old pregnant lady with glycosuria
E. 67-year-old male with a blood glucose of
Answer & Comments 9.4 mmol/l 2 hours after a 75g glucose
Correct answer: B load
F. 67-year-old male with lethargy and a
Patients with Cushing's syndrome may have fasting blood glucose of 9.4 mmol/l
different presentations and features depending G. 54-year-old lady with Addison's disease,
upon the cause. Most patients will have polydipsia, polyuria and lethargy
features that are more or less typical of
Cushing’s but those with ectopic ACTH from a H. 92-year-old lady admitted with a
malignant tumour or the ones with malignant myocardial infarction and a blood glucose
tumour of the adrenal itself may present rather of 10.2 mmol/l
quickly and may have weight loss, I. 92-year-old lady on steroids for COPD and
hypertension, hypokalaemia and obvious a random blood glucose of 9.8 mmol/l
tumour and features of its spread and not much
J. 18-year-old man with ketonuria,
in the way of Cushingoid features.
glycosuria and proteinuria.
Patients with Cushing's disease (pituitary
dependent), adrenal adenoma and the ones Answer & Comments
with carcinoid tumour will have the usual
features but those with adrenal adenoma do Correct answer: AF
not have features of hyperandrogenaemia like
hirsutism as benign adrenal tumours produce To confirm a diagnosis of diabetes mellitus, the
cortisol but not the androgens. Absence of patient must have symptoms (polyuria,
hirsutism and virilisation in a patient with other polydipsia or lethargy) and a fasting blood
features of Cushing's syndrome favours adrenal glucose greater than or equal to 7.0 mmol/l or a
adenoma but needs further investigations. glucose of greater than or equal to 11.1mmol/l
Normal MRI scan of the pituitary does not two hours after a 75g glucose tolerance test.
differentiate between different non-pituitary If the patient is asymptomatic, then the
dependent causes. elevated blood glucose should be confirmed on
two separate occasions.
[ Q: 51 ] MasterClass Part2 If the patient's blood glucose values are greater
(2010) - Endocrinology than or equal to 11.1 mmol/l on two occasions,
with or without symptoms, then a glucose
In which TWO of the following situations could tolerance test is not necessary to confirm the
the diagnosis of diabetes be confirmed on the diagnosis.
basis of the information given?
A. 44-year-old male with polydipsia and [ Q: 52 ] MasterClass Part2
fasting blood glucose of 8.1 mmol/l (2010) - Endocrinology
B. 44-year-old male with polyuria and A 34-year-old woman is seen in the infertility
lethargy and random blood glucose of 8.1 clinic and the question of the polycystic ovarian
mmol/l syndrome (PCOS) arises.

Dr. Khalid Yusuf El-Zohry – Sohag Teaching Hospital (01118391123) Page | 40


El-Zohry MRCP Questions Bank (Part 2) – Medical Masterclass 2010

Which one of the following clinical or the Emergency Department by ambulance.


biochemical findings would be expected in this They tell you that they were called because she
condition? was having seizures at home. She has no past
A. Oestrogen deficiency history of seizures but the GP, present at the
scene, told the ambulance crew that he had
B. Raised 17-hydroxyprogesterone levels been investigating her for polyuria and malaise,
C. Anovulatory cycles and that blood results back that day showed a
calcium of 3.4 mmol/l and a high normal PTH
D. Low luteinizing hormone (LH) / follicle-
concentration. They administered 10mg of
stimulating hormone (FSH) ratio
rectal diazepam and started IV fluids at the
E. Markedly elevated testosterone levels. scene, which caused the seizures to stop,
although she had not recovered consciousness.
Answer & Comments On arrival in the Emergency Department she
Correct answer: C has a GCS of 6 and is intubated with minimal
sedation. She has a 1l bag of 5% dextrose
running and capillary glucose is 5.8 mmol/l. The
Typical features of the polycystic ovarian
calcium level is confirmed at 3.6mmol/l and she
syndrome (PCOS) include:
is prescribed intravenous saline and
 polycystic ovaries furosemide, intravenous phenytoin at 15mg/kg,
 and subsequently given 90mg intravenous
hyperandrogenism
pamidronate over 1 hour. She improves initially
 anovulatory cycles. and is extubated.
Patients with PCOS are not deficient in You are called because she has become
oestrogen. The LH:FSH ratio is typically high increasingly drowsy and is suffering repeated
('reversed'), reflecting a lack of progesterone to seizures requiring further IV sedation and
inhibit LH release. intubation.
Testosterone levels are usually only mildly
What is the most important test to perform
raised in PCOS. Very high levels may occur with
next?
androgen-producing tumours of the ovary or
adrenal gland. If suspected, a more complete A. CT brain to exclude intracranial lesion
androgen profile, including
B. Prolactin level
dehydroepiandrosterone sulfate (DHEAS) and
androstenedione, should be checked. C. Parathyroid hormone (PTH) level

Hirsutism and menstrual irregularities, which D. Capillary and plasma glucose


are common manifestations of PCOS, are also E. Repeat plasma calcium.
features of adult onset congenital adrenal
hyperplasia (CAH). 17-hydroxyprogesterone
Answer & Comments
levels are raised in the adult variety of CAH and
are used as a screening test. They are not raised Correct answer: D
in PCOS.
It is highly unusual to have primary
[ Q: 53 ] MasterClass Part2 hyperparathyroidism at such a young age and
(2010) - Endocrinology the diagnosis of MEN 1 should be considered.
This lady stopped fitting at home when IV fluids
A 25-year-old woman is brought to were administered in combination with

Dr. Khalid Yusuf El-Zohry – Sohag Teaching Hospital (01118391123) Page | 41


El-Zohry MRCP Questions Bank (Part 2) – Medical Masterclass 2010

diazepam and it is pointed out in the history which was successfully treated with a short
that she had IV dextrose running when she course of anti-thyroid drugs.
arrived in the department and had her capillary
glucose checked, so the suspicion should be
there that she might have a concurrent
insulinoma. When the IV dextrose is stopped
and replaced with IV saline to rehydrate her
and help with calcium excretion, she
deteriorates again despite all other measures.
This should raise the suspicion of
hypoglycaemia despite the earlier normal
reading. A plasma insulin and c-peptide
concentration should also be performed if the
plasma glucose is low to help with the
confirmation of the diagnosis of insulinoma.
There is no merit in repeating the PTH or
checking the prolactin levels (this used to be
performed in many centres to try and confirm
that a genuine seizure had occurred and to help
diagnose pseudo seizures, though its use has
tailed off because of difficulties with
interpretation, and in this case the prolactin
levels may have been high due to a
prolactinoma related to MEN1).
Repeating the plasma calcium may have some
merit but it is unlikely to have changed
Based on the appearances shown in the image,
precipitously with current treatment. A CT brain
which of the following blood tests is most likely
is likely to be performed, but it is far more
to yield an abnormal result?
important to exclude and treat hypoglycaemia
first, since this is quickly remediable and will A. Oral glucose tolerance test (OGTT) with
prevent permanent brain damage. In fact, one measurement of growth hormone (GH)
could argue that irrespective of the likely levels
diagnosis of MEN1 and possible insulinoma that B. Thyroid stimulating hormone (TSH)
this is the test of choice anyway in anyone who
C. Full blood count
is fitting, even if blood glucose was normal with
the previous seizure. D. Corrected calcium
E. Short synacthen test.
[ Q: 54 ] MasterClass Part2
(2010) - Endocrinology Answer & Comments
A 47-year-old male is referred to the clinic with Correct answer: E
a 6-month history of lethargy and intermittent
bouts of feeling ‘completely washed out’. His The most striking abnormality shown is the
past medical history is unremarkable aside from presence of marked pigmentation within the
a bout of thyrotoxicosis during his mid-20’s, palmer and digital creases. In the clinical
setting, this should raise very strong suspicions

Dr. Khalid Yusuf El-Zohry – Sohag Teaching Hospital (01118391123) Page | 42


El-Zohry MRCP Questions Bank (Part 2) – Medical Masterclass 2010

regarding the possibility of primary adrenal A. The most likely diagnosis is an


insufficiency (Addison’s disease). Although aldosterone producing adenoma (Conn’s
thyroid dysfunction (either relapse of his syndrome).
thyrotoxicosis or subsequent development of
B. The most likely diagnosis is idiopathic
hypothyroidism – both of which would be most
hyperaldosteronism (bilateral adrenal
likely to be of autoimmune origin) should also
glomerulosa hyperplasia).
be considered, the clinical appearances would
mandate that an assessment of adrenal reserve C. The most likely diagnosis is an
be carried out in any case – this is of particular aldosterone-producing adrenal carcinoma
importance if thyroxine replacement is being because of the very high aldosterone
contemplated, given its potential to provoke an levels.
acute adrenal crisis if hypoadrenalism goes D. Liquorice abuse is a possible cause for
unrecognized. this clinical and biochemical picture.
It would also be sensible to check for the E. Primary adrenal hyperplasia is the most
presence of anaemia or hypercalcaemia in any likely diagnosis.
subject with tiredness and lethargy, although
the appearances in figure 1 specifically point to F. You can’t distinguish between primary
primary adrenal insufficiency. There are no adrenal hyperplasia and Conn’s syndrome
pre-operatively but this makes little
features to suggest acromegaly as an
difference since the treatment is the
underlying diagnosis.
same.

[ Q: 55 ] MasterClass Part2 G. Adrenal vein sampling has nothing to


add.
(2010) - Endocrinology
H. The strong family history of hypertension
A 38-year-old lady with a strong family history makes a diagnosis of Conn’s syndrome
of hypertension is referred to your clinic for more likely.
further evaluation having been diagnosed as
suffering from hypertension resistant to I. Dexamethasone suppressible
lifestyle modification one year previously. She hyperaldosteronism has been excluded.
says she is taking no medications. Test results J. Dietary sodium should be restricted for 3
show: days prior to performing this test for
BP 169/96. Pulse 64 regular. Na+ 141 mmol/L, reliable results because of pre-existing
K+ 3.3 mmol/L, urea 6.1 mmol/L, creatinine 70 sodium overload.
mmol/L, HCO3- 33 mmol/L. Plasma renin
activity (PRA) supine overnight: 0.1 mg/L/hr Answer & Comments
(normal 0.15-2.33), supine plasma aldosterone
Correct answer: AF
(PA) 1210 pmol/L (normal <240pmol/L). After
being upright and ambulant for 4 hours: PRA
0.1mg/L/hr (1.31-3.95), PA 810 pmol/L (140- Plasma renin activity (PRA) is suppressed and
560). CT adrenals had been performed and no aldosterone levels increased when at rest in the
abnormality was detected. supine position indicating primary
hyperaldosteronism. This excludes liquorice
Choose TWO correct statements from the abuse which suppresses both renin and
following list: aldosterone secretion. Dietary sodium intake
should be liberal for 1 week prior to this test (>
150 mEq/day). This should suppress renin and

Dr. Khalid Yusuf El-Zohry – Sohag Teaching Hospital (01118391123) Page | 43


El-Zohry MRCP Questions Bank (Part 2) – Medical Masterclass 2010

aldosterone activity in normal individuals and diagnosed acromegaly, which of the following
those with essential hypertension. would you not expect to find?
Aldosterone falls on standing and plasma renin A. Erectile dysfunction
activity remains suppressed. This makes B. Galactorrhoea
bilateral adrenal hyperplasia less likely and
Conn’s more likely for the following reasons: C. Carpal tunnel syndrome

 In Conn’s syndrome, aldosterone D. Arthralgia


release is partly regulated by E. Hypohydrosis.
adrenocorticotropic hormone (ACTH) -
not angiotensin II - and so typically
Answer & Comments
aldosterone levels are at their highest
when ACTH and cortisol are at their Correct answer: E
highest in the morning. As the day
progress aldosterone levels fall and are Acromegaly is the clinical disorder resulting
not reliant on posture. from hypersecretion of growth hormone and
 In idiopathic hyperaldosteronism , may be present for many years before it is
aldosterone secretion retains sensitivity diagnosed. Any of the complications associated
to angiotensin II (in fact sensitivity is with acromegaly may therefore be present at
probably increased), and therefore diagnosis.
increases as angiotensin levels increase
on changing posture [ Q: 57 ] MasterClass Part2
 This lady has primary (2010) - Endocrinology
hyperaldosteronism and Conn’s
(2) A 43-year-old woman is referred to the
syndrome is the most common cause
endocrine clinic with a corrected serum calcium
(approx 65% of cases) and idiopathic
of 1.56mmol/l, which was detected when she
hyperaldosteronism is the second most
had cataract surgery done recently.
common cause (30% cases).
Her previous medical history is non-significant
Aldosterone levels are well in the range for
and so is the family history. Her periods are
Conn’s syndrome, and adrenal carcinoma is
normal and regular. She went through normal
very rare and usually greater than 6cm in size
schooling and went to college and studied
when discovered, meaning that CT would be
beauty therapy. She does admit to spasm in her
likely to have shown the lesion.
hands when she gets anxious, but otherwise
The family history makes essential hypertension has no symptoms of chronic hypocalcaemia.
more likely, though if it appeared due to
She is short (4 feet 10 inches), has horizontal
autosomal dominant inheritance then
ridges on her nails, and Trousseau's sign is
dexamethasone suppressible
positive. There are no other abnormalities.
hyperaldosteronism (also called glucocorticoid
remediable hyperaldosteronism) should be Other blood results show a raised phosphate at
considered. 2.56mmol/l, and very low (undetectable) levels
of parathyroid hormone (PTH). Her 0900 hours
cortisol, thyroid function tests, urea and
[ Q: 56 ] MasterClass Part2
creatinine, and other electrolytes are normal.
(2010) - Endocrinology
Which of the following diagnoses is most likely?
In a 62-year-old gentleman with newly

Dr. Khalid Yusuf El-Zohry – Sohag Teaching Hospital (01118391123) Page | 44


El-Zohry MRCP Questions Bank (Part 2) – Medical Masterclass 2010

A. Pseudohypoparathyroidism Which of the following statements about


corticosteroid induced osteoporosis is correct?
B. Idiopathic (acquired) hypoparathyroidism
A. Corticosteroids increase osteoblastic
C. Pseudopseudohypoparathyroidism
activity.
D. Autoimmune hypoparathyroidism
B. Corticosteroids increase bone mass.
(Polyglandular Autoimmune Syndrome
type 1) C. Corticosteroids reduce intestinal calcium
absorption.
E. Chronic renal failure with secondary
hyperparathyroidism. D. Corticosteroids increase circulating sex
steroid levels.
Answer & Comments E. Corticosteroid induced osteoporosis is
greatest 12 months after starting
Correct answer: B
treatment.

This woman has hypoparathyroidism. Patients


Answer & Comments
with pseudohypoparathyroidism have somatic
features and PTH levels are high because of Correct answer: C
target organ resistance. Patients with
pseudopseudohypoparathyroidism have the The pathogenesis of corticosteroid induced
somatic features without biochemical osteoporosis is multifactorial:
abnormality. Renal failure is excluded by
 Corticosteroids reduce osteoblastic
normal urea and creatinine.
activity, and the resulting
It is difficult to differentiate between idiopathic osteoblast/osteoclast imbalance causes
hypoparathyroidism and the loss of bone.
familial/autoimmune hypoparathyroidism that
occurs as a part of polyglandular autoimmune  Corticosteroids reduce intestinal
syndrome type 1. However, without any family calcium absorption and lower
history of autoimmune disorder and without circulating sex steroid levels.
any personal history of problems such as  Corticosteroid induced bone loss is
mucocutaneous candidiasis, vitiligo or alopecia, fastest in the first 6-12 months of
this woman’s hypocalcaemia is most likely to be therapy
due to idiopathic/acquired hypoparathyroidism.

[ Q: 59 ] MasterClass Part2
[ Q: 58 ] MasterClass Part2 (2010) - Endocrinology
(2010) - Endocrinology
A 44-year-old overweight man is investigated
An 82-year-old woman is admitted having for fatigue. His TSH is 1.2 mU/L, fT4 12.9
sustained a hip fracture. Two years ago she pmol/L, and electrolytes are normal. A short
sustained a wrist fracture. On examination she synacthen test reveals basal cortisol of 550 nM
has bilateral cataracts and reduced visual and 30 minute level of 650 nM. A 75g oral
acuity. In passing she mentions that she has glucose tolerance test shows fasting glucose 6.4
been on prednisolone 5 mg/day for 8 years for mmol/L, and 120 min glucose 9.1 mmol/L.
polymyalgia, but she takes no other medication
and has not seen a doctor for 2 years. On the basis of these results, the patient should
be advised that:

Dr. Khalid Yusuf El-Zohry – Sohag Teaching Hospital (01118391123) Page | 45


El-Zohry MRCP Questions Bank (Part 2) – Medical Masterclass 2010

A. He has “pre diabetes” but is not yet at web and is concerned she may have
risk of the complications of diabetes. hypopituitarism.
B. He has around an 80% risk of progression
Which TWO of the following statements are
to diabetes with no treatment over the
incorrect about the insulin tolerance test?
next 5 years.
A. It is useful in assessing cortisol reserves.
C. No interventions have been proven to
improve prognosis at this stage. B. It is useful in assessing growth hormone
(GH) reserves.
D. Metformin may reduce his chances of
progressing to diabetes. C. Thyroxine should be replaced prior to test
if hypothyroidism is suspected.
E. He has an underactive adrenal gland.
D. It is contraindicated in patients with
ischaemic heart disease.
Answer & Comments
E. It is contraindicated in patients with
Correct answer: D
epilepsy.
F. It is contraindicated in patients with
Impaired glucose tolerance (IGT) is not a benign
asthma.
condition. Although a significant minority of
those with IGT do normalise following medical G. It is contraindicated if 9.00 am cortisol is
attention, there is a rate of progression to less than 100 nmol/l.
diabetes of around 30% over five years.
H. 50 ml of 50% dextrose should be
Furthermore, despite not achieving the
available prior to commencing the test.
threshold criteria for diabetes, those with IGT
are at significantly increased risk of I. 100 mg hydrocortisone should be
macrovascular disease (the risk increases available prior to commencing the test.
throughout the whole range of blood glucose), J. Intravenous insulin is administered for the
and at a much smaller risk of microvascular test.
disease. It is now well established from a series
of large prospective studies that intensive
Answer & Comments
lifestyle modification can significantly reduce
the chances of developing diabetes. This is also Correct answer: CF
true of metformin therapy, albeit to a smaller
degree. Insulin-induced hypoglycaemia is a powerful
Thus, those with IGT should have cardiovascular stimulus to cortisol and GH secretion. This is
risk formally assessed, and should be directed accepted as the gold standard test for the
to an intensive dietary and exercise assessment of cortisol and GH reserves in
programme, with or without metformin. This patients with known or suspected
man’s adrenal function is likely to be normal, hypothalamopituitary dysfunction.
albeit with a stressed basal level of cortisol. Contraindications to this test include ischaemic
heart disease, dysrhythmias, abnormal resting
[ Q: 60 ] MasterClass Part2 ECG, epilepsy and a 9.00 am cortisol < 100
(2010) - Endocrinology nmol/l.
If there is any possibility of adrenal
A 43-year-old lady is referred to you with
insufficiency, thyroxine replacement to correct
headaches of recent onset, amenorrhoea and
hypothyroidism must not be commenced until
general fatigue. She has been searching the

Dr. Khalid Yusuf El-Zohry – Sohag Teaching Hospital (01118391123) Page | 46


El-Zohry MRCP Questions Bank (Part 2) – Medical Masterclass 2010

after glucocorticoid replacement has occurred. The preceding normal menstrual cycle makes
Otherwise there is a significant risk of PCOS less likely, although mildly elevated
precipitating a hypoadrenal crisis. prolactin levels can be found in this condition.
Primary hypothyroidism should always be
[ Q: 61 ] MasterClass Part2 excluded in patients exhibiting
(2010) - Endocrinology hyperprolactinaemia, since TRH also acts as a
trophic stimulus to lactotrophs. However, the
A 28-year-old lady with a previously regular brief clinical description indicates that the
menstrual cycle, is referred by the patient is otherwise well.
gynaecologist with a 10-month history of
amenorrhoea and an elevated prolactin level of In this patient a microprolactinoma must be
1600mU/L (normal range up to 300mU/L). She sought (preferably by MRI scanning), although
is otherwise well, and her past medical history the use of anti-dopaminergic agents, as part of
is notable only for a bipolar disorder, which is the treatment strategy for managing the bipolar
under the continuing care of a psychiatrist. disorder, may well be sufficient to account for
the modest elevation in prolactin noted here.
The main differential diagnosis lies between
which TWO of the following? [ Q: 62 ] MasterClass Part2
A. Macroprolactinoma (2010) - Endocrinology
B. Acromegaly A 55-year-old man with type 2 diabetes has a
C. Drug induced hyperprolactinaemia proteinuria of 1.5 gm/24 hours and his
creatinine is 255 micromol/l. He is hypertensive
D. Microprolactinoma
on triple antihypertensives and his blood
E. Polycystic Ovarian Syndrome (PCOS) pressure is averaging 130/75 mmHg. He is a
smoker and his cholesterol is 6.4 mmol/l, with
F. Primary hypothyroidism
high-density lipoprotein (HDL) 0.9 mmol/l. His
G. Cushing's syndrome anti-diabetic therapy includes metformin and
H. Post pill amenorrhoea gliclazide and his last HbA1c was 6.7 %.

I. Inaccurate reporting of symptoms by the Which TWO of the following ten steps are the
patient most important?
J. Gonadotrophin deficiency. A. Commence insulin therapy
B. Refer for renal dialysis
Answer & Comments
C. Arrange for renal biopsy
Correct answer: CD
D. Anticoagulate with warfarin

The prolactin level is too low for a E. Start a statin (HMG co reductase
macroprolactinoma, and although co-secretion inhibitor)
of growth hormone and prolactin can be seen F. Stop metformin
in acromegaly (mixed sommato-lactotroph
G. Add further antihypertensives
tumour or ‘stalk disconnection’), there are no
other features to suggest this disorder. H. Commence a low protein diet
I. Stop gliclazide
J. Start diuretic therapy.

Dr. Khalid Yusuf El-Zohry – Sohag Teaching Hospital (01118391123) Page | 47


El-Zohry MRCP Questions Bank (Part 2) – Medical Masterclass 2010

Answer & Comments One of her TWO paternal uncles also developed
diabetes in his thirties, and her paternal
Correct answer: EF
grandfather died in his early fifties, and was
also thought to be diabetic.
This man appears to have his hypertension and
A. The likely risk of transmission to each of
diabetes under good control, and well within
her children in future is 25%.
the targets. He has nephropathy with renal
impairment and non-nephrotic range B. This is most likely to be latent
proteinuria. He is a smoker and has significantly autoimmune diabetes of adulthood.
raised cholesterol and reduced HDL, which C. The most likely underlying defect is a
together with his diabetes put him at mutation in the glucokinase gene.
extremely high risk for ischaemic heart disease.
He should stop smoking and attain a healthy D. She has a 30-40% chance of requiring
lifestyle with regular exercise. He will benefit insulin in future.
from seeing a dietician for appropriate advice E. High doses of insulin are likely to be
regarding his lipid profile, nephropathy and required to control glycaemia in due
weight (if overweight). He should also be course.
started on a statin in sufficient dose to lower his
cholesterol at least below 5 mmol/l. He may
Answer & Comments
also obtain benefit from a regular low dose
aspirin (if there are no contraindications). Correct answer: D
Most diabetics with nephropathy do not need a
renal biopsy. Diabetics usually require renal Features in this case that are suggestive of
replacement therapy at relatively lower levels Maturity Onset Diabetes of the Young (MODY)
of creatinine (500-550 micromol/l) as compared include the family history, which is consistent
with non-diabetics. with autosomal dominant inheritance (with
50% chance of transmission to offspring), and
From the given information, there does not the hypersensitivity to sulphonylureas. The
appear to be any indication to stop gliclazide or commonest type of MODY (MODY 3) is due to
to start insulin. He does not need mutation in the HNF1alpha gene, which
anticoagulation with warfarin, but his encodes a transcription factor involved in beta
metformin should be stopped because of the cell development. Other HNFs are implicated in
risk of lactic acidosis in the presence of further types of MODY, and in general the
significant renal impairment. hyperglycaemia in these subtypes is
progressive, with 30-40% requiring insulin, and
[ Q: 63 ] MasterClass Part2 risk of microvascular complications which is
(2010) - Endocrinology commensurate with that in type 1 diabetes,
matched for glycaemia. Glucokinase mutations
A slim, 21-year-old woman is referred with a reset the set point for glucose homeostasis, and
recent diagnosis of diabetes made after a tend to result in mild, non-progressive
routine medical examination at her GP’s hyperglycaemia, with low risk of complications
surgery. A blood glucose test was requested
due to her family history of diabetes; her father
has had diabetes for thirty years, and for the
[ Q: 64 ] MasterClass Part2
last 15 years he has been taking insulin. Despite (2010) - Endocrinology
extensive treatment for retinopathy, he is A 32-year-old man with long-standing type 1
otherwise fairly well. diabetes mellitus presents to his GP with a 3-

Dr. Khalid Yusuf El-Zohry – Sohag Teaching Hospital (01118391123) Page | 48


El-Zohry MRCP Questions Bank (Part 2) – Medical Masterclass 2010

month history of tiredness and lethargy. He also [ Q: 65 ] MasterClass Part2


reports increasingly frequent hypoglycaemic (2010) - Endocrinology
episodes despite having reduced his total daily
insulin dose on three separate occasions during In the investigation of amenorrhoea, luteinizing
the previous 2 weeks. On examination he is slim hormone (LH) and follicle stimulating hormone
(BMI 21.5 kg/m2), his pulse is 68 bpm sinus (FSH) may be reduced in which of the following
rhythm and his BP is 100/65 mmHg. patients?
Investigations reveal serum sodium 136 mmol/L
(normal range 137-144), serum potassium 5.1 A. 18-year-old woman with Turner's
syndrome
mmol/L (normal range 3.5-4.9), serum urea 7.8
mmol/L (normal range 2.5-7), serum creatinine B. 24-year-old woman with polycystic
98 μmol/L (normal range 60-110) and HbA1c ovarian syndrome
6.4% (normal range 3.8-6.4).
C. 24-year-old woman who is 12 weeks
Which of the following investigations is most pregnant
likely to identify the cause for his symptoms? D. 24-year-old woman with excessive
A. Anti-tissue transglutaminase antibodies weight loss

B. Plasma thyroid-stimulating hormone E. 22-year-old woman with adult-onset


congenital adrenal hyperplasia.
C. Serum vitamin B12
D. Serum corrected calcium Answer & Comments
E. Short tetracosactide (Synacthen) test Correct answer: D

Answer & Comments Amenorrhoea is traditionally subdivided into:


Correct answer: E 1. Primary amenorrhoea - lack of menses
by age 16
Type 1 diabetes mellitus may be associated 2. Secondary amenorrhoea - absence of
with other autoimmune disorders, including menstrual periods for 6 months or
Addison's disease, Graves' disease, Hashimoto's more after cyclical menses have been
thyroiditis, primary gonadal failure, coeliac established.
disease, pernicious anaemia, vitiligo, alopecia
and primary hypoparathyroidism (as part of the In practice, there is overlap between the two
type 2 or, less commonly, type 1 polyglandular and a number of conditions can give rise to
endocrinopathy syndromes). Tiredness and either primary or secondary amenorrhoea.
lethargy are common presenting features of Amenorrhoea can also be subdivided into
several of these conditions, but the history of patients with hypogonadotrophin,
recurrent hypoglycaemic episodes despite normogonadotrophin or hypergonadotrophin
adjustments to his insulin regimen, low BMI, levels. Answers B and E are associated with
hypotension and abnormal electrolytes (with normogonadotrophin levels. Answers A and C
low serum sodium and elevated serum are associated with increased gonadotrophin
potassium and urea) make Addison's disease levels, and excessive weight loss can cause
most likely in this case. decreased levels of gonadotrophins. This is
often the case with athletes or patients with
anorexia.

Dr. Khalid Yusuf El-Zohry – Sohag Teaching Hospital (01118391123) Page | 49


El-Zohry MRCP Questions Bank (Part 2) – Medical Masterclass 2010

[ Q: 66 ] MasterClass Part2 fractures. Several studies have shown


(2010) - Endocrinology improvement in surrogate markers of these
diseases, including markers of bone turnover
A 42-year-old man has proven growth hormone and plasma lipids. Wellbeing scores and
deficiency post pituitary surgery for pituitary exercise tolerance have also been shown to
adenoma. He is hyperlipidaemic with a family improve, hence the concentration on
history of ischaemic heart disease. He symptomatic patients. Although there is a
complains that he has lost his energy and theoretical concern about promoting tumour
complains that he finds he is unable to work out growth, the dose of GH is titrated against IGF-1
in the gym as he used to. Other pituitary axes to try to prevent over-replacement. So far there
are either normal or adequately replaced. He is is no evidence to suggest an increased risk of
interested in a trial of growth hormone therapy. new tumour formation or recurrence of a
previously treated pituitary tumour.
Which of the following statements are true?
A. He should be commenced on growth [ Q: 67 ] MasterClass Part2
hormone as there is evidence that it will
(2010) - Endocrinology
reduce his risk of cardiovascular mortality
and morbidity. A 25-year-old man presents to the endocrine
clinic complaining that he and his wife have
B. He should not commence on growth
been trying to conceive for almost 2 years
hormone as it may adversely affect his
without success. His wife has a 2-year-old
lipid profile.
daughter by a previous partner and so the
C. Treatment with growth hormone may patient is concerned that there may be
improve his lipid profile and exercise ’something wrong with him‘. On further
tolerance. questioning he admits that he has only ever
D. Growth hormone treatment is shaved twice a week, has poor libido and had a
contraindicated due to the risk of re- left orchidopexy as a small child. On
growth of his pituitary tumour. examination he is a tall gentleman with very
little facial hair, subtle bilateral gynaecomastia
E. Growth hormone therapy is important to
and small firm testis, approximately 5mls in
reduce his risk of osteoporotic fracture.
volume. Baseline tests reveal a testosterone of
10.1nmol/l, LH 17 IU/l and FSH 23 IU/l.
Answer & Comments
The single most important test in confirming the
Correct answer: C
diagnosis is:
A. A semen analysis
Growth hormone deficiency is associated with
loss of lean body mass and increase in fat mass, B. An MRI pituitary
hyperlipidaemia, increased risk of osteoporosis
C. Measurement of plasma estradiol levels
and cardiovascular morbidity and mortality.
D. A karyotype analysis
However treatment with GH in adults is
controversial and many clinicians only consider E. A testicular biopsy
it in patients who are symptomatic in addition
to biochemically GH deficient. This is because Answer & Comments
GH is relatively expensive and as yet there is no
Correct answer: D
hard end-point data on reduction of
cardiovascular adverse events or osteoporotic

Dr. Khalid Yusuf El-Zohry – Sohag Teaching Hospital (01118391123) Page | 50


El-Zohry MRCP Questions Bank (Part 2) – Medical Masterclass 2010

This gentleman was suffering from Klinefelter’s It seems that normal complements of germ
syndrome, which not uncommonly presents in cells are present in these patients in early foetal
adult life rather than at adolescence. This life but during late gestation and early infancy
syndrome is characterised by dysgenesis of the there is a dramatic loss of spermatogonia. In
seminiferous tubules and is a common cause of adult patients, spermatogenesis may rarely be
primary hypogonadism and male infertility. present but most fertile patients have proved
Karyotype analysis of unselected newborns has to have sex chromosome mosaicism (46, XY/47
estimated an incidence of 1 per 800 to 1000 XXY) and often lack the features which
males, the most common human chromosomal distinguish them from typical patients with
abnormality. Klinefelter’s syndrome. The technique of
intracytoplasmic sperm injection (ICSI) has been
Patients are phenotypically male with testes of
used with some success in this group of
usually < 3cm in length and azoospermia.
patients – however, there is also an increased
Gynaecomastia is also common. Karyotype
risk of trisomy 21 in children of patients with
analysis typically demonstrates the 47, XXY
Klinefelter’s syndrome.
karyotype. Patients show poor to normal
virilization at puberty and tend to have a tall The mainstay of treatment is testosterone
stature due to disproportionately long legs. replacement. Patients diagnosed pre-pubertally
may be given small doses of testosterone
Testosterone levels vary but are usually
intramuscularly (e.g. 50mg im monthly) to
decreased and gonadotrophins are raised –
initiate puberty and avoid the psychological and
particularly FSH. Plasma estradiol levels are
physical consequences of hypogonadism. The
usually normal or elevated, similarly SHBG.
dose may be gradually increased (100mg im,
Undescended testes are 3 times as common as monthly) once bone age has advanced to allow
in normal boys. Prepubertal studies indicate full growth potential to be realised. Once the
that children with the 47, XXY Karyotype have final height has been reached an adult
lower birth weights, smaller mean head replacement dose of testosterone, e.g. 250mg
circumferences, a slightly increased risk of im 3 weekly. Alternatively testosterone patches
congenital anomalies, height percentiles that or the newer gels or buccal preparations may
increase with age, a lower verbal IQ than be used in adults.
normal boys and poor motor and muscle tone.
Associated abnormalities include an increased [ Q: 68 ] MasterClass Part2
frequency of diabetes mellitus; 19% of patients (2010) - Endocrinology
have been reported to have impaired glucose
tolerance and 8% to be diabetic. 47 XXY A 44-year-old lady with hypertension is seen in
patients with gynaecomastia also have an your clinic. You suspect she may have primary
increased predisposition to cancer of the hyperaldosteronism.
breast. In addition, 20-50% of boys 8 years or
older with primary mediastinal germ cell Which of the following would point away from
tumours have Klinefelter’s syndrome. this diagnosis?

25% of adults with Klinefelter’s have A. A low renin level


osteoporosis, although this is uncommon in B. Hypokalaemia
patients on testosterone replacement. Chronic
C. Polyuria
lung disease and varicose veins are also thought
to be more common. D. Acidosis
E. A good response to spironolactone.

Dr. Khalid Yusuf El-Zohry – Sohag Teaching Hospital (01118391123) Page | 51


El-Zohry MRCP Questions Bank (Part 2) – Medical Masterclass 2010

Answer & Comments [ Q: 70 ] MasterClass Part2


Correct answer: D (2010) - Endocrinology
A 27-year-old woman is referred to endocrine
Primary hyperaldosteronism is an important clinic complaining of gradually increasing facial
cause of hypertenison in the young to middle hirsutism and acne over the last three years,
aged. The majority of cases are caused by associated with weight gain (an increase in her
benign aldosterone producing adrenal dress size from 10 to 14). After reading an
adenomas - Conn's syndrome. article in “Bella” magazine, she thinks that she
Most appear during the investigation of may have polycystic ovarian syndrome (PCOS)
hypertension or unexplained hypokalaemia. If and she is worried that this means she will be
potassium depletion is severe polydipsia, infertile. She tells you that her periods were
polyuria, paraesthesiae and alkalosis may occur. erratic when they started aged 15, she was on
Headaches are common. Retinopathy is mild the pill and had regular withdrawal bleeds from
and retinal haemorrhages are rarely seen. aged 19 to 24 years. She stopped taking oral
contraception 3 years ago and has had irregular
periods with a cycle length of 33-38 days since.
[ Q: 69 ] MasterClass Part2 On examination her current BMI is 29 and her
(2010) - Endocrinology blood pressure is 140/85. She has no obvious
facial hirsutism, but has recently had
In secondary hyperlipidaemia, which of electrolysis and she has moderate hirsutism on
following conditions or drugs typically causes her chest and abdomen with no other signs of
hypertriglyceridaemia rather than virilization.
hyperlipidaemia?
A. Hypothyroidism The following statements are correct:

B. Oral contraceptive pill A. Based on the history a diagnosis of PCOS


is unlikely.
C. Anorexia nervosa
B. A diagnosis of congenital adrenal
D. Obstructive jaundice hyperplasia can be excluded due to the
E. Corticosteroids. late onset and lack of virilisation.
C. The most likely diagnosis based on the
Answer & Comments history is an androgen secreting tumour.

Correct answer: B D. The first line tablet treatment should be


cyproterone acetate.
Hypercholesterolaemia can be caused by E. The first line tablet treatment should be
hypothyroidism, obstructive jaundice, the use spironolactone.
of corticosteroids and anorexia nervosa.
F. The first line tablet treatment should be
Hypertriglyceridaemia can be caused by
dianette.
diabetes, the use of oral contraceptive pill,
excess alcohol and the use of thiazide diuretics. G. A managed programme of reduced
A mixed dyslipidaemia can be caused by dietary intake and increased activity to
diabetes, obesity, renal failure, nephrotic promote weight loss is likely to improve
syndrome, glycogen storage diseases or her symptoms.
paraproteinaemia.

Dr. Khalid Yusuf El-Zohry – Sohag Teaching Hospital (01118391123) Page | 52


El-Zohry MRCP Questions Bank (Part 2) – Medical Masterclass 2010

H. The likely diagnosis is PCOS, but she and hopefully also provide cardiovascular risk
should be reassured that this is an reduction.
entirely benign condition.
Metformin, another treatment which reduces
I. She should have a progesterone insulin resistance, is often used in this condition
measurement on day 21 of her cycle to in women desiring fertility and increases the
determine whether she is ovulating. frequency of ovulatory cycles.
J. Her condition is likely to be associated Cyproterone acetate and sprionolactone, have
with insulin resistance and measures to anti-androgenic actions and may help hirsutism,
improve this are therapeutically useful. but they are teratogenic and should only be
used in women of childbearing age if they are
Answer & Comments combined with a reliable contraceptive.

Correct answer: GJ Dianette is a combination of cyproterone


acetate and ethinyloestradiol, is effective as an
OCP and reduces hirsutism in PCOS, but is not a
The most common causes of hirsutism, by far,
good treatment if she is keen to have children.
are idiopathic, racial/familial and PCOS. The
She stopped the OCP three years ago, knows
likely diagnosis based on the history is PCOS.
her cycle length and is worried about infertility,
The typical history is a background of
so she may be trying to get pregnant.
oligomenorrhoea, usually dating from the
menarche, with variable degrees of hirsutism, Day 21 progesterone will detect ovulation in a
which worsens in association with weight gain. patient with a 28-day cycle, but will be too early
in this woman with a 33-38 day cycle. The
Although congenital adrenal hyperplasia is
timing needs to be 7 days before the next
much less likely, given the history, partial
period is due, so she should have serial
enzyme defects can present in adult life as
measurements between days 26 and 31 to
hirsutism without virilization and most
catch the progesterone peak.
endocrinologists would exclude the diagnosis
by measurement of 17-hydroxyprogesterone
(elevated in CAH). An androgen secreting [ Q: 71 ] MasterClass Part2
tumour is unlikley given the long history. (2010) - Endocrinology
Testosterone should be measured and a level of
A 62-year-old man is found to have glycosuria
<5 mmol/l would be reassuring.
on a routine urine test. His blood glucose is
The management of PCOS depends largely found to be 21 mmol/l and a diagnosis of
upon what the patient’s treatment goals are maturity onset diabetes mellitus is made.
(weight loss, reduction of hirsuitism/acne or
fertility). However, if the patient is overweight, He is very concerned about the implications for
treatment should always include a managed his driving licence: which one of the following
weight-loss programme. PCOS is not entirely statements is true?
benign as it can be associated with sub-fertility, A. Tablet-treated diabetic patients are
as well as features of the metabolic syndrome allowed up to a 3-year licence.
including markedly increased risk of type 2
diabetes, dyslipidaemia and ischemic heart B. Insulin-treated diabetic patients are
disease. Weight loss and exercise will improve allowed up to a 3-year licence.
the underlying insulin resistance and C. Diet-controlled diabetic patients are
hyperinsulinaemia. This will improve symptoms allowed up to a 5-year licence.

Dr. Khalid Yusuf El-Zohry – Sohag Teaching Hospital (01118391123) Page | 53


El-Zohry MRCP Questions Bank (Part 2) – Medical Masterclass 2010

D. Insulin-treated gestational diabetes cortisol 510 nmol/L (normal range 200-700) and
patients are not allowed to drive urine osmolality 420 mosmol/kg (normal range
350-1000).
E. Diabetic patients with eyesight
complications have separate rules to non-
Which of the following drugs is most likely to
diabetic patients.
cause this biochemical picture?
A. Chlorpromazine
Answer & Comments
B. Lithium carbonate
Correct answer: B
C. Olanzapine
Patients managed by diet alone need not notify D. Sodium valproate
DVLA unless they develop relevant disabilities
E. Venlafaxine
(such as eye problems) or they need drug
treatment for their diabetes.
Answer & Comments
Patients managed by oral hypoglycaemic agents
can retain their licence up to the age of 70 Correct answer: E
years unless they develop relevant disabilites or
they need insulin treatment for their diabetes. In the presence of normal renal, adrenal and
thyroid function and in a patient who is
Patients managed on insulin must demonstrate
clinically euvolaemic, this biochemical profile is
satisfactory control, recognise warning
consistent with a diagnosis of syndrome of
symptoms of hypoglycaemia and meet required
inappropriate antidiuretic hormone (SIADH).
visual standards. They will be given a one, two
Venlafaxine is a well-recognised cause of this
or three-year licence.
disorder.
Patients with insulin-treated gestational
diabetes must notify the DVLA but may retain
[ Q: 73 ] MasterClass Part2
their licence if they have good control.
(2010) - Endocrinology
Patients with diabetes and eyesight
complications have the same rules as non- A 19-year-old Italian student is referred by her
diabetic drivers. GP because of cosmetically distressing
hirsutism, acne and irregular menses. She is
otherwise symptomatically well. Menarche
[ Q: 72 ] MasterClass Part2
began at age 8, and symptoms developed
(2010) - Endocrinology progressively thereafter.
A 45-year-old woman with a long-standing On examination she is slim, and has moderate
history of schizoaffective disorder is found on to severe hirsutism affecting face, chest and
routine investigation to be hyponatraemic. lower abdomen, and pustular acne. There is
Physical examination is unremarkable. also some temporal hair recession, and she 5’0”
Investigations show serum sodium 122 mmol/L (mother 5’6” and father 6’02”). There is
(normal range 137-144), serum potassium 4.5 moderate cliteromegaly.
mmol/L (normal range 3.5-4.9), serum urea 2.2
Untimed blood tests reveal the following:
mmol/L (normal range 2.5-7), serum creatinine
55 μmol/L (normal range 60-110), plasma Na 140 mmol/l, K 4.8 mmol/l, urea 3.6 mmol/l,
osmolality 262 mosmol/kg (normal range 278- Cr 75 micromol/l. Normal liver function panel
305), plasma thyroid-stimulating hormone 1.8 and full blood count. LH 4.5 iU/l FSH 2.3 iU/l
mU/L (normal range 0.4-5.0), 9 a.m. serum Estradiol 129 pmol/l Testosterone 5.1 nmol/l

Dr. Khalid Yusuf El-Zohry – Sohag Teaching Hospital (01118391123) Page | 54


El-Zohry MRCP Questions Bank (Part 2) – Medical Masterclass 2010

DHEA-S 16 micromol/l (0-7) Androstenedione likely show features consistent with PCOS,
15 nmol/l (3-12) found in most women with untreated CAH.
Treatment is with glucocorticoid replacement
Which 2 of the following investigations or
to reduce ACTH drive to adrenal androgen
treatments would you choose next?
production. Where hirsutism is difficult to
A. Karyotype control despite glucocorticoid, adjunctive
B. Trial of Dianette oral contraceptive antiandrogen treatment is sometimes used (but
only if effective contraceptive measures are in
C. Diagnostic Laparoscopy
place, in view of the teratogenicity of anti-
D. Short synacthen test (with measurements androgens).
of cortisol & 17 hydroxyprogesterone)
E. Luteal phase 9a.m. 17 [ Q: 74 ] MasterClass Part2
hydroxyprogesterone (2010) - Endocrinology
F. Transvaginal ultrasound A 36-year-old man presents to his GP
G. 24-hour urinary free cortisol complaining of numbness and tingling in both
hands, which is particularly troublesome at
H. Ovarian vein sampling
night. He had been fit and well until 2 years
I. Trial of Finasteride alone earlier when he had been diagnosed with
J. Oral glucose tolerance test hypertension. On examination he has coarse
facial features, prognathism and evidence of
bilateral carpal tunnel syndrome.
Answer & Comments

Correct answer: DG Which of the following would confirm a


diagnosis of acromegaly?

The key clinical points in this case are the early A. Blunted growth hormone response to
puberty, long history of symptoms, and the insulin-induced hypoglycaemia
presence of some virilising features with a high B. Detectable midnight growth hormone
testosterone. In addition, the elevation of
DHEA-S, which is produced by the adrenals, C. Exaggerated rise in growth hormone
suggests an adrenal origin of the androgens. All levels following glucagon stimulation
of this makes nonclassical CAH the most likely D. 09.00h growth hormone >20 mU/L
diagnosis (it is also said to be more common in
E. Paradoxical rise in growth hormone levels
the Italian population than in the UK). A serious
during oral glucose tolerance test
differential diagnosis would be a virilising
adrenal tumour. No specific features of
Cushing’s syndrome are described, but should Answer & Comments
be carefully sought, and UFC and adrenal Correct answer: E
imaging would be appropriate. A follicular
phase 9 a.m. 17OHP is a reasonable screening
The oral glucose tolerance test remains the gold
test for CAH, but synacthen testing with
standard for diagnosing acromegaly, with
measurement of both 17OHP and cortisol is
affected subjects exhibiting failure of
more sensitive, and gives additional
suppression of growth hormone levels in
information about cortisol reserve.
response to a glucose challenge; indeed in
Investigation of the ovaries is not indicated on many cases a paradoxical rise is observed.
the basis of the given information, but would

Dr. Khalid Yusuf El-Zohry – Sohag Teaching Hospital (01118391123) Page | 55


El-Zohry MRCP Questions Bank (Part 2) – Medical Masterclass 2010

In most acromegalics, insulin-like growth factor C. Periorbital oedema


(IGF)-1 levels are also elevated, but this finding
D. Lid retraction
should not be used as the sole criterion for
diagnosis. E. Visible nodular goitre.

In normal physiology, growth hormone is


secreted in a pulsatile manner and random Answer & Comments
measurements should not be used to diagnose Correct answer: C
growth hormone deficiency or excess. The
insulin tolerance test and glucagon stimulation
Both lid lag and lid retraction reflect enhanced
test are used to assess growth hormone reserve
sensitivity to circulating catecholamines, and
in subjects with suspected deficiency
may therefore be found in most thyrotoxic
states. The following eye signs are specific to
[ Q: 75 ] MasterClass Part2 Graves’ disease:
(2010) - Endocrinology  proptosis
A medical student attached to the endocrine  ophthalmoplegia
firm is asked to comment on the eye signs
 chemosis
exhibited by the patient shown below. She
specifically comments that the patient has  periorbital oedema
Graves’ dysthyroid eye disease on the basis of This patient exhibits neither ophthalmoplegia
the clinical findings listed below. nor chemosis. The goitre in Graves’ disease is
typically smooth rather than nodular.

[ Q: 76 ] MasterClass Part2
(2010) - Endocrinology
A 20-year-old woman was incidentally
diagnosed with pseudohypoparathyroidism.
The image shows one of the features of
pseudohypoparathyroidism.

Which of her observations is true?


A. Ophthalmoplegia
B. Chemosis

Dr. Khalid Yusuf El-Zohry – Sohag Teaching Hospital (01118391123) Page | 56


El-Zohry MRCP Questions Bank (Part 2) – Medical Masterclass 2010

Features of chronic hypocalcaemia


include:
 lethargy
 malaise
 paraesthesia
 cramps
 dystrophic nails
 alopecia
 cataracts
 papilloedema
 rarely movement
disorders (basal ganglia
calcification): tetany, fits, etc.
Clinically, Trousseau's sign and Chvostek's sign
Which TWO of the following ten are NOT
may be demonstrable and a blood gas may
features?
show underlying alkalosis.
A. Basal ganglia calcification
Infusion of PTH does not result in the normal
B. Osteoporosis response of an increase in the urinary cAMP,
C. High parathyroid hormone (PTH) levels demonstrating target organ resistance to the
effects of PTH. This is the basis of the
D. Short stature Ellsworth–Howard test.
E. Low intelligence Osteoporosis is not a disorder of calcium
F. Short 4th and 5th metacarpals metabolism and is not a feature of
pseudohypoparathyroidism.
G. Cataracts
H. Failure to increase urinary cAMP with
[ Q: 77 ] MasterClass Part2
infused PTH
(2010) - Endocrinology
I. Tetany
A very anxious 38-year-old diabetic woman
J. Papilloedema.
comes to clinic. She has searched on the
internet and presents a long list of her
Answer & Comments symptoms that she thinks are caused by
Correct answer: BE autonomic neuropathy.

Which of the following symptoms are NOT


Pseudohypoparathyroidism is rare and results caused by diabetic autonomic neuropathy?
from target organ resistance to the actions of
A. Postural hypotension
PTH, resulting in chronic hypocalcaemia and
raised levels of PTH. They have typical B. Peripheral oedema
phenotype with round face, short stature and
C. Constipation
short 4th and 5th metacarpals (see image).
Intelligence is within normal range. D. Diarrhoea

Dr. Khalid Yusuf El-Zohry – Sohag Teaching Hospital (01118391123) Page | 57


El-Zohry MRCP Questions Bank (Part 2) – Medical Masterclass 2010

E. Urinary incontinence The most likely underlying diagnosis is:


F. Headache A. Cushing’s disease
G. Vomiting B. Conn’s syndrome
H. Gustatory sweating C. Phaeochromocytoma
I. Bradycardia D. Multiple Endocrine Neoplasia (M.E.N.)
type 1
Answer & Comments E. Nephrocalcinosis secondary to primary
hyperparathyroidism.
Correct answer: FI

Answer & Comments


Autonomic neuropathy may manifest in many
different ways. Cardiovascular autonomic Correct answer: C
changes include tachycardia, absence of sinus
arrhythmia and postural hypotension.
The image reveals a large left suprarenal mass.
Delayed gastric emptying occurs and may give The appearances are typical of a
rise to vomiting. Both diarrhoea and phaeochromocytoma which, unlike most other
constipation can result. adrenal tumours, demonstrates a distinctive
‘bright white’ signal on T2-weighted MRI.
The bladder may become atonic and
intermittent self catheterization may be needed In Cushing’s disease (corticotroph pituitary
to prevent overflow incontinence and further adenoma) bilateral adrenal hyperplasia may be
problems. seen on adrenal imaging.
Phaeochromocytoma is one component of the
[ Q: 78 ] MasterClass Part2 M.E.N. type 2 syndrome (medullary thyroid
(2010) - Endocrinology carcinoma, phaeochromocytoma, parathyroid
hyperplasia) and not the M.E.N. type 1
The MRI appearances on T2-weighted images syndrome (parathyroid hyperplasia, pancreatic
from a 46-year-old female with refractory tumours, pituitary tumours).
hypertension are shown (see image).

[ Q: 79 ] MasterClass Part2
(2010) - Endocrinology
A 42-year-old woman presents with weight
gain, lethargy and hypertension.

Concerning investigation for Cushing’s


syndrome, which of the following statements is
true?
A. The overnight dexamethasone
suppression test is highly specific for the
diagnosis of Cushing’s syndrome.
B. High dose dexamethasone suppression
test is a useful screening test for
Cushing’s syndrome.

Dr. Khalid Yusuf El-Zohry – Sohag Teaching Hospital (01118391123) Page | 58


El-Zohry MRCP Questions Bank (Part 2) – Medical Masterclass 2010

C. Suppression of cortisol in a high dose (LDDST). 1mg dexamethasone is taken at


dexamethasone suppression test 23.00h and then cortisone is measured at 9am
excludes Cushing’s syndrome. the next morning.
D. The overnight dexamethasone Failure to suppress (cut off of < 50 or
suppression test is a useful outpatient increasingly < 35 nmol/l with sensitive cortisol
screening test for Cushing’s syndrome. assays) indicates possible Cushing’s syndrome.
The lower the cut-off the higher the false
E. She need not discontinue her oral
positive rate.
contraceptive medication prior to
investigation. This is a sensitive test (98% using a cut off of 35
nmol/l) but not specific. Failure of suppression
Answer & Comments can occur rarely in normal individuals (2%) in
depression (10-50% or systemic ill health (10-
Correct answer: D 20%).
The overnight dexamethasone suppression test
Cushing’s syndrome describes glucocorticoid
remains a good screening test especially if
excess of any cause (including ectopic
combined with 24-hour urinary-free cortisol.
adrenocorticotrophic hormone (ACTH), ACTH-
producing pituitary adenoma, glucocorticoid- Low dose dexamethasone suppression testing
producing tumour, exogenous steroids). (0.5mg 6 hourly for 48 hours) has similar
Cushing’s disease describes glucocorticoid sensitivity to the overnight test but increased
excess due to an ACTH producing pituitary specificity. However, it requires inpatient
adenoma. admission and has the additional potential side
effects of worsening diabetes and precipitating
Dexamethasone suppression tests rely on the
or worsening psychological disturbance. It
principal that normal cortisol production is
should be used if there is a high index of
suppressed by exogenous glucocorticoid
suspicion of Cushing’s syndrome.
(dexamethasone) whereas it will fail to
suppress in Cushing’s syndrome. High dose dexamethasone test (2mg 6-hourly
for 48 hours) is not a screening test for
The interpretation of these test will be difficult
Cushing’s syndrome but is used to distinguish
in the presence of enzyme-inducing drugs as
Cushing’s disease from other causes of
they may rapidly metabolise dexamethasone or
Cushing’s syndrome. Pituitary adenomas
oestrogens (e.g. pregnancy, HRT, combined
secreting ACTH are generally not entirely
OCP) as they may induce cortisol binding
autonomous and may be suppressed by
protein and artefactually increase total cortisol
sufficient exogenous steroid. If the final cortisol
levels. Exogenous oestrogens should therefore
measurement after 48 hours of dexamethasone
be discontinued for 6 weeks prior to
is less than 50% of the basal value then
investigation.
suppression is said to have occurred.
24-hour urinary-free cortisol collection is a Suppression is usually seen in Cushing’s disease
complementary investigation but should not but not in ectopic ACTH production or adrenal
occur during dexamethasone suppression tests tumours. However, the test is neither very
specific nor sensitive. Suppression occurs in
Overnight dexamethasone suppression test is a
75% of patients with Cushing’s disease, 10-25%
useful initial screening test in a patient with a
of patients with ectopic ACTH and 0-6% of
low clinical suspicion of Cushing’s syndrome, if
patients with adrenal tumours.
you have a high index of suspicion go straight to
low dose dexamethasone suppression test

Dr. Khalid Yusuf El-Zohry – Sohag Teaching Hospital (01118391123) Page | 59


El-Zohry MRCP Questions Bank (Part 2) – Medical Masterclass 2010

[ Q: 80 ] MasterClass Part2 her in clinic, you note that she has never had a
(2010) - Endocrinology menstrual bleed and appears prepubertal. She
is 5’0” tall (parents 5’6” and 5’11”) and is
A 65-year-old man is admitted having tripped clinically hypothyroid. There are no other
over a mat. Unfortunately he sustained a severe abnormalities apart from some keloid scars on
femoral neck fracture. He suffers with her back. Further investigation reveals LH 34 U/l
emphysema and takes prednisolone 20 mg FSH 53 U/l Estradiol 50 pmol/l
once daily together with bronchodilators but no
other treatment. In theatre, a bone biopsy is What is the next key investigation?
taken at the time a hemiarthroplasty is A. Karyotype
performed. Histopathology reports the bone as
being severely osteoporotic. B. Insulin tolerance testing with growth
hormone and cortisol measurements
Which one of the following investigations would C. Pregnancy test
NOT be appropriate?
D. Pituitary MRI
A. Erythrocyte sedimentation rate (ESR)
E. Abdominal ultrasonography
B. Protein electrophoresis
C. Estradiol Answer & Comments
D. Calcium and alkaline phosphatase Correct answer: A
E. Thyroid function tests.
This is essentially a case of short stature with
Answer & Comments primary amenorrhoea and biochemistry
suggestive of ovarian failure. In this setting a
Correct answer: C karyotype is essential to look for Turner’s
syndrome, with 45,X karyotype. There is a
It is important to identify any contributory spectrum of clinical severity of the syndrome,
causes of osteoporosis which may require but in this case the keloid scars and primary
treatment. Although hip fractures are more hypothyroidism (much more common in
common in women (1 in 3 women will have had Turner’s) are additional clues. A differential
a hip fracture by the age of 90), hip fractures diagnosis in this case might be autoimmune
occur in men who also have osteoporosis. hypothyroidism with autoimmune ovarian
It would be appropriate to check estradiol in failure developing peri-puberty. Although
amenorrrhoeic premenopausal women with hypothyroidism per se may lead to growth
low trauma fractures, since premature retardation and other abnormalities, in this
menopause (<45years) is a risk factor for case the free thyroxine is only relatively mildly
osteoporosis. depressed, and so a second underlying
pathology is deemed more likely.

[ Q: 81 ] MasterClass Part2 After a diagnosis of Turner's syndrome is made,


abdominal ultrasonography becomes important
(2010) - Endocrinology
to look for renal abnormalities and ovaries.
An 18-year-old girl is found to have the
following thyroid function tests when
investigated for fatigue by her GP: TSH 32 mU/l
(0.4-4) FT4 7.5 pmol/l (9-22) Antithyroid
microsomal antibody titre 1:3,000 Reviewing

Dr. Khalid Yusuf El-Zohry – Sohag Teaching Hospital (01118391123) Page | 60


El-Zohry MRCP Questions Bank (Part 2) – Medical Masterclass 2010

[ Q: 82 ] MasterClass Part2 typically an MRI scan or a bone scan, but CT


(2010) - Endocrinology may also be useful.

A 54-year-old man with type 2 diabetes mellitus


[ Q: 83 ] MasterClass Part2
is admitted to hospital with an infected second
toe on the right foot. Despite 3 weeks of high- (2010) - Endocrinology
dose oral antibiotics from his GP, the toe has A 36-year-old schoolteacher with a
shown no sign of improvement. On longstanding history of headaches and non-
examination he is pyrexial (temperature 37.7°C) specific variable muscular pains gives a 1-year
but not systemically unwell. The toe is swollen history of marked polyuria and thirst. On
and a small deep ulcer is noted at the tip, with questioning she describes a continually dry
spreading cellulitis over the adjacent foot. mouth and passing large volumes of clear urine.
She passes urine 2-3 times a night and always
Which of the following investigations would it
has a glass of water on her bedside table.
be most appropriate to organise immediately at
Examination is unremarkable.
this stage?
She is booked for a water deprivation test and
A. Bone scan
attends after a light breakfast.
B. CT scan of foot
Baseline bloods reveal: Glucose 7.8 mmol/l,
C. X-ray of foot Na+ 134 mmol/l, K+ 3.5 mmol/l, Urea 3.0
D. MRI scan of foot mmol/l, Creatinine 60 micromol/l, Ca2+ 2.70
mmol/l, Albumin 38 g/l. The water deprivation
E. White cell scan test results are as follows:

Answer & Comments 2 4 6


Time: 0hrs 8hrs
hrs hrs hrs
Correct answer: C
Urine osmolality
270 270 310 380 420
(mOsm/kg)
It is quite likely that this man has underlying
osteomyelitis and that the tiny ulcer over the Plasma
tip of his right second toe extends to the bone. Osmolality 281 281 285 288 290
There are several imaging studies that would (mOsm/kg)
help in identifying osteomyelitis, but the most
readily available and appropriate in the first You are called to review the results to
instance is a plain X-ray. Although plain films determine what should be done next.
may not show bony changes in the early stages
of osteomyelitis, there is often some evidence What should you do?
of soft tissue reaction. This man has had A. Diagnose syndrome of inappropriate
symptoms and signs for a sufficient duration of antidiuretic hormone secretion (SIADH)
time (>2 weeks) for X-ray changes of and stop the test
osteomyelitis to be evident (haziness and loss
B. See that the results are suggestive of
of density of the affected bone, periosteal
psychogenic polydipsia but inconclusive
thickening and/or elevation and focal
and continue the dehydration phase for
osteopenia, or the typical lytic changes).
another hour to see if plasma osmolality
However, if plain X-rays are negative and there
rises before taking other measures
is sufficient clinical concern of an underlying
infection, then further imaging is required,

Dr. Khalid Yusuf El-Zohry – Sohag Teaching Hospital (01118391123) Page | 61


El-Zohry MRCP Questions Bank (Part 2) – Medical Masterclass 2010

C. See that nephrogenic diabetes insipidus is concentrate their urine (urine osmolality:
most likely and give 2 mcg of IM plasma osmolality < 2.0) and who has cranial DI,
desmopressin before collecting urine should concentrate their urine normally in
samples for another 4 hours response to desmopressin. In nephrogenic
diabetes insipidus the urine fails to concentrate
D. See that cranial diabetes insipidus is most
normally in response to desmopressin.
likely and give 2 mcg of IM desmopressin
before collecting urine samples for A normal individual would concentrate their
another 4 hours urine throughout the test and there should be a
fall off in urine volume. Plasma osmolality rises
E. Diagnose diabetes mellitus as the likely
but remains below 295 mOsm/kg. Urine
cause for the polyuria and stop the test.
osmolality: plasma osmolality ratio should rise
above 2.0 during the test. In psychogenic
Answer & Comments
polydipsia, continual water loading can
Correct answer: B “washout” the renal concentrating mechanism,
but urine osmolality: plasma osmolality rises to
> 2.0 by the end of the test provided adequate
Syndrome of inappropriate antidiuretic
dehydration is achieved. Since, however, many
hormone secretion is characterised by low
of these subjects are fluid overloaded at the
plasma osmolality, low plasma sodium and
start of the test (as in this case) 8 hours may
relatively high urine osmolality. These basic
not be enough to achieve adequate
criteria are not met in this instance.
dehydration. If at the end of 8 hours urine
The plasma glucose sample is a post-prandial output has not tailed off and/or urine
sample and therefore a result of 7.8 mmol/l is osmolality: plasma osmolality is < 2.0, but
not diagnostic of diabetes mellitus, so this is not plasma osmolality is < 295, then water
a consideration in this case. deprivation should be continued for another
Both cranial and diabetes insipidus (DI) are hour before administration of desmopressin. If
characterised by an inability of the kidneys to results are still equivocal then a hypertonic
concentrate the urine. In cranial diabetes saline infusion test may be considered.
insipidus this is because the
hypothalamus/pituitary has lost the ability to [ Q: 84 ] MasterClass Part2
produce vasopressin. In nephrogenic DI there is (2010) - Endocrinology
no response to the action of vasopressin in the
distal renal tubule. In both types of DI plasma A 32-year-old man with long-standing type 1
osmolality is usually raised and the urine is not diabetes mellitus presents to his GP with a 3-
appropriately concentrated (urine osmolality: month history of tiredness and lethargy. He also
plasma osmolality <2.0). It would be very reports increasingly frequent hypoglycaemic
unusual for plasma osmolality not to rise above episodes despite having reduced his total daily
295 mOsm/kg during the course of a water insulin dose on three separate occasions during
deprivation test in DI (with the lack of an the previous 2 weeks. On examination he is slim
appropriate urine concentrating response). (BMI 21.5 kg/m2), his pulse is 68 bpm sinus
rhythm and his BP is 100/65 mmHg.
Cranial DI is differentiated from nephrogenic DI
Investigations reveal serum sodium 136 mmol/L
by the response to 2mg IM desmopressin at the
(normal range 137-144), serum potassium 5.1
end of the test (when the individual is allowed
mmol/L (normal range 3.5-4.9), serum urea 7.8
to drink free fluids). An individual who has
mmol/L (normal range 2.5-7), serum creatinine
developed plasma osmolality >295mOsm/kg
and who has failed to appropriately

Dr. Khalid Yusuf El-Zohry – Sohag Teaching Hospital (01118391123) Page | 62


El-Zohry MRCP Questions Bank (Part 2) – Medical Masterclass 2010

98 μmol/L (normal range 60-110) and HbA1c


6.4% (normal range 3.8-6.4).

Which of the following investigations is most


likely to identify the cause for his symptoms?
A. Serum anti-tissue transglutaminase
antibodies
B. Plasma thyroid-stimulating hormone
C. Serum vitamin B12
D. Serum corrected calcium
E. Short tetracosactide (Synacthen) test
Which of the following is true regarding
Answer & Comments maculopathy?

Correct answer: E A. Maculopathy causes loss of peripheral


vision
Type 1 diabetes mellitus may be associated B. Maculopathy requires routine referral for
with other autoimmune disorders, including an ophthalmology opinion
Addison's disease, Graves' disease, Hashimoto's
C. Maculopathy is unaffected by poor
thyroiditis, primary gonadal failure, coeliac
glycaemic control
disease, pernicious anaemia, vitiligo, alopecia
and primary hypoparathyroidism (as part of the D. Circinate exudates may indicate
type 2 or, less commonly, type 1 polyglandular maculopathy
endocrinopathy syndromes). Tiredness and E. Maculopathy is unaffected by
lethargy are common presenting features of hypertension.
several of these conditions, but the history of
recurrent hypoglycaemic episodes despite
Answer & Comments
adjustments to his insulin regimen, low BMI,
hypotension and abnormal electrolytes (with Correct answer: D
low serum sodium and elevated serum
potassium and urea) make Addison's disease Maculopathy is the commonest threat to vision
most likely in this case. in type 2 diabetes. It typically produces loss of
central vision and if discovered or suspected
[ Q: 85 ] MasterClass Part2 requires urgent referral to an ophthalmologist.
(2010) - Endocrinology The 3-year risk of severe visual loss in
maculopathy is reduced by over 50% with
A 62-year-old gentleman with type 2 diabetes photocoagulation. Macular grid laser therapy is
for 17 years is found to have decreased visual usually used. Both glycaemia and hypertension
acuity at his annual review. should be as well controlled as possible.
Circinate exudates occur around areas of
microvascular leakage and should therefore
lead to the suspicion of macular ischaemia.

Dr. Khalid Yusuf El-Zohry – Sohag Teaching Hospital (01118391123) Page | 63


El-Zohry MRCP Questions Bank (Part 2) – Medical Masterclass 2010

[ Q: 86 ] MasterClass Part2 Research continues into the use of insulin orally


(2010) - Endocrinology and intranasally.
Repaglinide and Nateglinide stimulate insulin
A 58-year-old professor of Greek has recently
release from the beta-cells.
been diagnosed as having diabetes. In clinic he
questions you about the various treatments. Rosiglitazone and Pioglitazone reduce
peripheral insulin resistance - but are not
Which of the following statements about licensed for use in combination with insulin.
medication for diabetes are true?
A. Gliclazide is a long acting sulphonylurea [ Q: 87 ] MasterClass Part2
B. Gliclazide is associated with weight gain (2010) - Endocrinology
C. Metformin is associated with weight gain A 79-year-old frail woman has a toxic adenoma
D. Repaglinide is a short acting of thyroid and has failed to respond to
sulphonylurea radioactive iodine, which she had 8 months
ago. She remains on 20 mg of carbimazole a day
E. Repaglinide decreases peripheral insulin and her latest thyroid function test showed a
resistance TSH of 0.3 mU/l (0.4-4.0 mU/l), free T4 of 22
F. Rosiglitazone may be used in combination pmol/l (10-20 pmol/l) and free T3 of 6.3 pmol/l
with insulin (3-5.5pmol/l).

G. Metformin increases gluconeogenesis What would be the most appropriate line of


H. All insulin preparations are U-100 action?
formulation A. Leave her on carbimazole indefinitely
I. Zinc is added to insulin preparations to B. Block and replace (use high dose
delay the absorption of insulin carbimazole to block and replace with
J. Human insulatard is available as an oral thyroxine)
preparation. C. Change carbimazole to propylthiouracil
D. Refer her for a second dose of radioactive
Answer & Comments
iodine
Correct answer: BI E. Refer her to surgeons for thyroidectomy.

Most insulins are U-100 formulation. There are Answer & Comments
100 international units of insulin per millilitre of
fluid. However U-500 insulin (500 international Correct answer: D
units of insulin per millilitre of fluid) is available
for those patients requiring large doses of Long-term carbimazole is an option, but dose
insulin (>400 units per day) as the injectable requirement is high and she may run into side
volume of fluid would be too great. effects. Giving another dose of radioactive
iodine is simpler, quite likely to be successful
Metformin decreases gluconeogenesis and
and may be curative. Surgery is also an option,
increases peripheral utilization of glucose.
but in a frail patient it would be standard
Insulin is degraded by gastrointestinal enzymes practice to try at least one further dose of
and therefore must be given by injection. radioactive iodine before considering this.
Propylthiouracil has no advantage over

Dr. Khalid Yusuf El-Zohry – Sohag Teaching Hospital (01118391123) Page | 64


El-Zohry MRCP Questions Bank (Part 2) – Medical Masterclass 2010

carbimazole. Block and replace therapy is not thyrotoxicosis are often absent. The diagnosis
used in this clinical situation. may be detected incidentally on investigation of
tiredness or as part of screening in patients
[ Q: 88 ] MasterClass Part2 with atrial fibrillation or congestive cardiac
failure.
(2010) - Endocrinology
Non-thyroidal illness (sick euthyroid syndrome)
A 72-year-old woman is referred to the manifests most commonly with low TSH,
Endocrine Clinic by her GP after she is low/normal free T4 and low free T3 levels.
discovered to have abnormal thyroid function Hashimoto's thyroiditis is classically associated
tests while under investigation for tiredness. with hypothyroidism and only rarely with
She reports no other symptoms of thyroid hyperthyroidism, which may occur in the
dysfunction, but had been admitted to hospital earliest stages of the condition (so-called
4 months earlier with a diagnosis of congestive ‘Hashitoxicosis'). Although Graves' disease is
cardiac failure and atrial fibrillation. She has no the commonest cause of thyrotoxicosis overall,
family history of thyroid disease. Her it is typically seen in a younger age group, with
medication includes digoxin 125μg/day, goitre, eye signs and a positive family history.
furosemide 80 mg/day and warfarin 3 mg/day. Subacute thyroiditis may be associated with
On examination her pulse is 80 bpm atrial preceding pain in the neck or, if painless,
fibrillation and her BP 135/85 mmHg. She has a typically occurs in a younger age group,
small goitre, no cervical lymphadenopathy and particularly when occurring in the postpartum
there is no evidence of dysthyroid eye disease. period or when triggered by certain drugs.
Investigations reveal plasma free T4 17.5 pmol/L
(normal range 10-22), plasma free T3 9.5 pmol/L
(normal range 5-10), plasma thyroid-stimulating [ Q: 89 ] MasterClass Part2
hormone <0.1 mU/L (normal range 0.4-5.0) and (2010) - Endocrinology
serum antithyroid peroxidase 35 IU/mL (normal
A 26-year-old female has recently moved into
<50).
the area and is referred by her new GP for
routine endocrine follow-up of her primary
What is the most likely cause for her abnormal
hypothyroidism and Addison’s disease, which
thyroid function tests?
were diagnosed approximately 5 years ago. She
A. Graves' disease is currently taking thyroxine (100
B. Hashimoto's thyroiditis micrograms/day), hydrocortisone (10/5/5
mg/day) and fludrocortisone (100
C. Non-thyroidal illness (sick euthyroid
micrograms/day).
syndrome)
D. Subacute thyroiditis It would be appropriate to perform the
following blood tests even if she is clinically
E. Toxic multinodular goitre euthyroid, euadrenal and otherwise
asymptomatic:
Answer & Comments
A. Full blood count/vitamin B12
Correct answer: E B. Calcitonin
C. Random cortisol
This is a typical presentation of toxic
multinodular goitre, which is the commonest D. Adrenal antibodies
cause of hyperthyroidism in the elderly, in E. Prolactin
whom classical clinical symptoms and signs of

Dr. Khalid Yusuf El-Zohry – Sohag Teaching Hospital (01118391123) Page | 65


El-Zohry MRCP Questions Bank (Part 2) – Medical Masterclass 2010

F. Luteinising hormone (LH) and follicle of the other options listed above are relevant to
stimulating hormone (FSH) screening for individuals affected by either the
type 1 or type 2 multiple endocrine neoplasia
G. Fasting gut hormone profile
(MEN) syndromes, and not the polyglandular
H. Plasma catecholamines syndromes.
I. Thyroglobulin
J. Short synacthen test. [ Q: 90 ] MasterClass Part2
(2010) - Endocrinology
Answer & Comments A 16-year-old boy presents with short stature.
Correct answer: AF He had a normal birth and normal growth and
development until about the age of 11 years,
when he started to fall behind his peers. He is
The co-occurrence of primary hypothyroidism
otherwise well. General examination is normal,
and primary hypoadrenalism (Addison’s
except that he is pre-pubertal (testes 3ml,
disease) raises the possibility of either the type
Tanner stage G1, P1) with height below the 3rd
1 or type 2 autoimmune polyglandular
centile. His bone age is 4 years behind his
syndromes.
chronological age. His mid-parental height is on
In this case, with an onset in young adulthood, the 50th centile.
and with no documented history of
hypoparathyroidism, the type 2 syndrome is The likely diagnosis is:
more likely. Affected individuals typically A. Growth hormone deficiency
exhibit primary adrenal insufficiency and
thyroid dysfunction (hypo- or hyperthyroidism), B. Kleinfelter's syndrome
but may also develop type 1 diabetes mellitus C. Kallman's syndrome
and primary gonadal failure. Non-endocrine
D. Constitutional delayed puberty
manifestations of the condition include
myasthenia gravis, pernicious anaemia, vitiligo E. Coeliac disease
and alopecia. Accordingly, it would be
appropriate to check a full blood count and Answer & Comments
vitamin B12 level and to look for early signs of
primary gonadal failure (persistent elevation of Correct answer: D
the gonadotrophin (LH and FSH) levels) even in
the presence of regular menses. In these Constitutional delay is common in boys and by
circumstances, the patient needs to be aware far the commonest cause of short stature with
that fertility may be a problem if they choose to delayed pubertal development. In contrast
delay starting a family. constitutional delay of puberty is relatively
uncommon in girls and an underlying cause
There is little clinically to be gained from serially
should be aggressively investigated.
checking for the presence of adrenal antibodies
in those with established primary adrenal A diagnosis of constitutional delayed puberty
failure, nor is there a need to repeat the requires there to be no pubertal development
synacthen test as recovery of adrenal function by the age of 14 in boys (13 in girls) and for the
is unlikely. Random cortisol levels are not bone age to be at least 3 years less than the
helpful when assessing the adequacy of chronological age (showing that there is still
glucocorticoid replacement – a hydrocortisone potential for growth).
day profile would be more appropriate. Many

Dr. Khalid Yusuf El-Zohry – Sohag Teaching Hospital (01118391123) Page | 66


El-Zohry MRCP Questions Bank (Part 2) – Medical Masterclass 2010

The condition may not require treatment other IU/l. All other baseline anterior pituitary
than reassurance that puberty and increased hormones were within the reference range.
growth will eventually occur along with follow-
up to monitor growth. However, if the Which of the following statements is correct?
condition is causing distress, puberty can be A. The patient has premature ovarian failure
accelerated by treatment with testosterone
B. The patient should be counselled that she
which primes the hypothalamo-pituitary-
will not be able to conceive without egg
gonadal axis.
donation
Coeliac disease and other chronic illnesses can
C. There is a pathological process affecting
also cause delayed puberty, but this would be a
the anterior pituitary gland
less common cause and you might expect there
to be other associated symptoms or signs. D. An MRI of the brain is likely to reveal
abnormalities of the olfactory bulbs or
Growth hormone deficiency in childhood is
sulci
much rarer than constitutional delayed puberty
and presents with failure of linear growth at E. Oestrogen replacement therapy is not
any age, rather than the characteristic normal indicated
growth followed by falling behind as peers
undergo their pubertal growth spurt described Answer & Comments
above.
Correct answer: D
Kleinfelter’s syndrome (47XXY) is associated
with primary hypogonadism (i.e. testicular
This patient had Kallmann’s syndrome. This is
failure) whilst Kallman’s syndrome is
the most common form of isolated
characterized by secondary
hypogonadotrophic hypogonadism with
(hypogonadotrophic) hypogonadism with
delayed puberty in which anosmia resulting
anosmia. Both conditions are generally
from agenesis of the olfactory lobes or sulci is
associated with tall rather than short stature.
associated with LHRH deficiency.

[ Q: 91 ] MasterClass Part2 The condition is four times more common in


boys than in girls. In some cases the LHRH
(2010) - Endocrinology
deficiency may be partial (the fertile eunuch
An 18-year-old girl presented to the endocrine syndrome) and rarely affected men who had a
clinic accompanied by her mother. The family delayed puberty may recover spontaneously,
were concerned that the girl was excessively experience an increase in testicular size and
tall compared to other members of her family enter full puberty. Undescended testes and
and that she had still not had her first period, gynaecomastia are common in boys, as with
although she had developed pubic hair at the other forms of hypogonadotrophic
age of 13 years. She had a cleft palate repaired hypogonadism.
as an infant. On examination, she was 180cm Associated defects that may be variably present
with a weight of 67kg. Her mother was 150cm include cleft lip, cleft palate, imperfect facial
and her father 175cm. Pubic hair was stage 3 fusion, seizure disorders, short metacarpals,
but breast and nipple development were pes cavus, neurosensory hearing loss, cerebellar
prepubertal (stage 1). Baseline tests showed ataxia and nystagmus, oculomotor
that plasma estradiol levels were undetectable abnormalities and unilateral or rarely bilateral
with baseline LH levels of 1.9 IU/l and FSH 3.2 renal aplasia.

Dr. Khalid Yusuf El-Zohry – Sohag Teaching Hospital (01118391123) Page | 67


El-Zohry MRCP Questions Bank (Part 2) – Medical Masterclass 2010

In a review of MRI findings of patients with [ Q: 92 ] MasterClass Part2


Kallmann’s syndrome, 56% had absent or (2010) - Endocrinology
abnormal olfactory sulci bilaterally; in 17% the
agenesis was unilateral. The syndrome can be A 54-year-old woman is diagnosed as having
transmitted as an X-linked, autosomal acromegaly.
dominant or autosomal recessive trait. The
molecular genetics of the X-linked form have She asks you what treatment the
been well established. The KAL 1 gene is endocrinologist, who she is seeing next week, is
located at the Xp22.3 locus and encodes a 680 likely to recommend in the first instance. You
amino acid glycoprotein, named anosmin-1. answer:
This glycoprotein is thought to play a part in the A. Radiotherapy
guidance of LHRH neurons to the medial basal
B. Surgery
hypothalamus. A wide variety of deletions and
mutations of the KAL gene have been C. Octreotide (or other somatostatin
described. analogue)

Management includes sex steroid replacement D. Bromocriptine (or other dopamine


therapy to induce and maintain primary and agonist)
secondary sexual characteristics and maintain E. A growth hormone receptor antagonist.
bone and muscle mass. Premenopausal women
a combined oral contraceptive pill may be used
Answer & Comments
or alternatively oestrogen patches or gels with
concomitant cyclic progesterone therapy in Correct answer: B
women with an intact uterus to prevent
unopposed endometrial proliferation. The first line treatment is usually surgery, with
For males, who do not wish to be fertile, transsphenoidal adenectomy curing 80% of
intramuscular testosterone (sustanon) 250mg patients with microadenomas and 50% of those
three times weekly is effective. Alternatively, with macroadenomas.
testosterone patches, gels or buccal Somatostatin analogues are effective
preparations can be used. symptomatically in 60% of cases and produce a
Women who wish to become pregnant respond significant reduction in tumour size in 30%, but
well to gonadotrophin or pulsatile GnRH they can only be given as injections, are
therapy. The latter is less likely to result in expensive, and have a number of side effects.
multiple pregnancy. Dopamine agonists are of very limited efficacy,
Men who wish to be fertile should have their although they can be useful if the tumour co-
testosterone replacement stopped, since secretes prolactin.
testosterone therapy may suppress
spermatogenesis. These patients also respond [ Q: 93 ] MasterClass Part2
well to pulsatile GnRH therapy. However, (2010) - Endocrinology
adequate spermatogenesis may take up to two
years to achieve and frozen aliquots should be A 63-year-old woman with a history of
stored for further use. Alternatively, patients hypertension treated with furosemide 40
may respond to β hCG (500-2000 IU) given mg/day and amlodipine 10 mg/day is referred
intramuscularly or subcutaneously three times by her GP to the Endocrine Clinic for further
a week; FSH 75IU may be added to improve investigation of hypercalcaemia. On
sperm quantity after six months, if necessary. examination her pulse is 76 bpm in sinus

Dr. Khalid Yusuf El-Zohry – Sohag Teaching Hospital (01118391123) Page | 68


El-Zohry MRCP Questions Bank (Part 2) – Medical Masterclass 2010

rhythm and BP is 145/85 mmHg. Auscultation further elevate the serum calcium level. Whilst
of the chest is unremarkable. She has mild bisphosphonates may be used to help lower
peripheral oedema. Investigations show serum serum calcium levels in the short term, their
sodium 137 mmol/L (normal range 137-144), use in the longer term should be reserved for
serum potassium 3.6 mmol/L (normal range those in whom surgery is not possible. In
3.5-4.9), serum urea 6.8 mmol/L (normal range addition, in this case the patient has only mild
2.5-7), serum creatinine 105 μmol/L (normal osteopenia at a single site, and this is likely to
range 60-110), serum corrected calcium 2.85 respond to correction of the
mmol/L (normal range 2.2-2.6), serum hyperparathyroidism. Although long-term
phosphate 0.72 mmol/L (normal range 0.8-1.4), surveillance in clinically asymptomatic subjects
plasma parathyroid hormone 7.6 pmol/L has previously been favoured by some
(normal range 0.9-5.40) and 24-hour urinary clinicians, this approach is no longer
calcium 8.3 mmol/L (normal range 2.5-7.5). recommended in patients who are suitable for
Dual energy X-ray absorptiometry (DEXA) scan surgery, especially when there is evidence of
shows total hip T-score of -0.96 and lumbar associated complications (eg nephrolithiasis,
spine T-score of -1.65. osteopenia/
osteoporosis, hypertension). Low-calcium diets
What is the most appropriate management?
are not appropriate, and indeed may
A. Change furosemide to exacerbate associated bone disease.
bendroflumethiazide
B. Commence bisphosphonate [ Q: 94 ] MasterClass Part2
C. Observation with repeat serum calcium in (2010) - Endocrinology
3 months
A 29-year-old woman has recently been
D. Parathyroidectomy diagnosed as having polycystic ovarian
E. Start low-calcium diet syndrome (PCOS). She is oligomenorrhoeic,
menstruating only two to three times per year,
and you have confirmed that the cycles are
Answer & Comments
anovulatory. Her BMI is 34. She wishes to start
Correct answer: D a family and asks for advice about the most
appropriate treatment.
This patient has primary hyperparathyroidism,
Which of the following statements is true?
with the classical biochemical pattern of
elevated serum calcium, low serum phosphate A. Glitazones are now well established as an
and an inappropriately elevated parathyroid effective therapy in PCOS.
hormone level. She also has evidence of B. Metformin will give her, on average 5-6
osteopenia in the lumbar spine on DEXA extra ovulatory cycles per year.
scanning. In these circumstances, and in the
absence of any family history of inherited C. Dietary and lifestyle changes that aimed
endocrinopathy, it is likely that she has a at producing weight loss and increasing
solitary parathyroid adenoma. In many centres activity levels can lead to ovulation in
preoperative localisation using ultrasound more than 50% of women.
and/or 99mTc-sestamibi facilitates minimally D. The effects of metformin on ovulation
invasive selective adenomectomy. are due to its effect in producing weight
Bendroflumethiazide acts to reduce renal loss.
calcium excretion and is likely therefore to

Dr. Khalid Yusuf El-Zohry – Sohag Teaching Hospital (01118391123) Page | 69


El-Zohry MRCP Questions Bank (Part 2) – Medical Masterclass 2010

E. Both glitazones and metformin improve ultrasound is helpful in distinguishing between


hirsutism in PCOS. solid, cystic or mixed nodules. It does not
differentiate between malignant or benign
Answer & Comments lesions. A FNA biopsy can help distinguish
between benign and malignant, but does not
Correct answer: C always do so. A surgical opinion may be
needed.
Weight loss and lifestyle change can produce
A radioisotope scan is the least useful of the
ovulation in 58% of those who participate in a
above investigations. It can identify cold or hot
managed programme.
nodules but this does not necessarily correlate
There is no good evidence to support the use of with the presence of a malignant or benign
glitazones as a treatment of PCOS. lesion.
Metformin is widely used and many
publications have supported its use in fertility [ Q: 96 ] MasterClass Part2
treatment (particularly in combination with (2010) - Endocrinology
clomiphene), menstrual regulation and
management of hirsutism. However, there have A 19-year-old man presents to his GP
been few adequately powered randomised complaining of erectile dysfunction. His past
trials, and these have failed to support these medical history is unremarkable, he is on no
claims. regular medication, and he denies excessive
alcohol consumption. On examination he
appears anxious and embarrassed when asked
[ Q: 95 ] MasterClass Part2 to undress. He has bilateral gynaecomastia and
(2010) - Endocrinology only sparse pubic hair. Testicular volumes are 5
mL bilaterally, with no identifiable masses.
A 45-year-old lady is referred to you by her GP
Investigations show plasma luteinising hormone
and found to have one prominent nodule in a
45 U/L (normal range 1-10), plasma follicle-
nodular thyroid gland.
stimulating hormone 56 U/L (normal range 1-7),
Which of the following investigations would be serum testosterone 3.1 nmol/L (normal range
of lowest priority? 9-35), plasma prolactin 555 mU/L (normal
<360), plasma free T4 14.5 pmol/L (normal
A. Free T4 and free T3 levels
range 10-22) and plasma thyroid-stimulating
B. Ultrasound scan of thyroid hormone 1.5 mU/L (normal range 0.4-5.0).
C. Radioisotope scan of thyroid
What is the most likely cause for his
D. Serum thyroid stimulating hormone (TSH) hypogonadism?
level
A. Haemochromatosis
E. Fine needle aspiration biopsy.
B. Kallmann's syndrome
C. Klinefelter's syndrome
Answer & Comments
D. Non-functioning pituitary adenoma
Correct answer: C
E. Prolactinoma
Thyroid nodules and goitres are common. FT4,
FT3 and TSH levels should be checked to Answer & Comments
exclude overt thyroid dysfunction. An
Correct answer: C

Dr. Khalid Yusuf El-Zohry – Sohag Teaching Hospital (01118391123) Page | 70


El-Zohry MRCP Questions Bank (Part 2) – Medical Masterclass 2010

This man presents with the classical features of B. Radiograph right foot
Klinefelter's syndrome (karyotype 47,XXY). Due
C. Fundoscopy through dilated pupils
to the abnormal gonadal development,
testosterone levels are low with consequent D. Ultrasound scan of renal tract
elevation of gonadotrophins. Patients are E. 24-hour urinary protein estimation.
usually azoospermic and infertile. Kallmann's
syndrome is an inherited form of
Answer & Comments
hypogonadotrophic hypogonadism, typically
associated with anosmia. Although the Correct answer: C
prolactin level is slightly elevated, it is likely that
this is a reflection of the patient's anxiety, This patient has poorly controlled diabetes. He
rather than a consequence of a true has not had his annual screening for at least 2
microprolactinoma or non-functioning pituitary years. He appears to have diabetic nephropathy
adenoma with so-called ‘stalk disconnection and may well have nephrotic syndrome.
syndrome'. Moreover, both of the latter would Furthermore, he has signs of peripheral
predispose to hypogonadotrophic rather than neuropathy and possibly Charcot foot.
hypergonadotrophic hypogonadism. Given the
It is quite likely that he has diabetic retinopathy
absence of any significant past medical history,
and the preferred next step must be to exclude
it is extremely unlikely that excessive iron
sight threatening retinopathy like maculopathy
deposition is the cause of hypogonadism in a
or proliferative retinopathy. Blurred vision may
man of this age.
be due to poor glycaemic control, but it is vital
to perform fundoscopy and exclude any
[ Q: 97 ] MasterClass Part2 retinopathy that requires urgent
(2010) - Endocrinology ophthalmological attention.
A 55-year-old man with type 2 diabetes The patient has dipstick positive proteinuria,
mellitus, who has been lost to follow-up for hence estimation of urinary albumin:creatinine
over 2 years, is admitted with poorly controlled ratio will be appropriate to quantitate the
diabetes. Apart from polyuria, polydipsia and proteinuria, and ultrasound of the renal tract to
tiredness, he is also complaining about blurred see the size of kidneys and to exclude
vision, especially in right eye. He has lost weight obstructive uropathy is also appropriate - but
but has dependent oedema of his lower legs. neither of these take immediate preference
He has signs of peripheral neuropathy and over checking his fundi.
there's clinical evidence of right Charcot's Radiograph of his foot and possibly some other
foot.His urine dipstix test reveals 4+ protein and investigations will also be needed to assess the
4+glucose. problem and extent of his foot disease, but
His laboratory results are as follows: Hb. 11 urgent fundoscopy can lead to preservation of
gm/dl, MCV 84 fl, WBC 9.0x109/l, Platelets his sight from imminent threats.
323x109/l, ESR 18 mm/hour, Sodium
138,mmol/l, Potassium 4.7,mmol/l, Urea [ Q: 98 ] MasterClass Part2
11.8mmol/l, Creatinine 198 micromol/l,
(2010) - Endocrinology
Albumin 32 g/l, HbA1c 11.9%.
A 54-year-old gentleman is followed up in the
What do you think the preferred next step endocrine clinic following his bilateral
should be? adrenalectomy in 1976 for Cushing's disease.
A. Urinary albumin:creatinine ratio He is on hydrocortisone 20 mg in the morning

Dr. Khalid Yusuf El-Zohry – Sohag Teaching Hospital (01118391123) Page | 71


El-Zohry MRCP Questions Bank (Part 2) – Medical Masterclass 2010

and 10 mg at teatime, and 200 micrograms of has phoned to say the patient may have a
fludrocortisone a day. His most recent cortisol thyroid crisis.
day profile is satisfactory. His sodium was 149
mmol/l (136-150 mmol/l) and potassium 3.2 After reviewing the patient which of the
mmol/l (3.6-5.0 mmol/l). His blood pressure, on following should you not prescribe?
two occasions, was greater than 170 systolic A. Carbimazole
and 100 diastolic.
B. Aspirin
What action will you take? C. Potassium iodide
A. Reduce hydrocortisone dose to half D. Propranolol
B. Start antihypertensive treatment E. Dexamethsaone.
C. Reduce fludrocortisone to 100
micrograms a day Answer & Comments
D. Start at a small dose of spironolactone Correct answer: B
and gradually increase
E. Advise a low salt diet and review in clinic. Thyroid crisis is a serious and potentially fatal
disorder and needs emergency treatment.
Answer & Comments Antithyroid drugs should be given immediately
and then 6-8 hourly. Carbimazole or
Correct answer: C propylthiouracil can be used. After an
antithyroid drugs has been given iodide should
It is quite obvious from his drug history, then be given to prevent release of any stored
biochemical profile and blood pressure that he thyroid hormone.
is being over treated with fludrocortisone. His
Propranolol and dexamethasone will help
recent cortisol day profile reflected satisfactory
decrease the conversion to T4 to T3.
replacement with glucocorticoids.
Propranolol also blocks the peripheral actions
Treating him with antihypertensives may help of T3. Although it would act as an antipyretic
but it does not make sense, as the cause agent, aspirin should be avoided as it can
appears to be remediable, and if not corrected, displace thyroid hormones from thyroid binding
may lead to adverse events. If his blood globulin.
pressure remained high despite readjusting his
fludrocortisone doses to required levels, he [ Q: 100 ] MasterClass Part2
may then need further investigations and
(2010) - Endocrinology
treatment with antihypertensives.
Spironolactone is anti-aldosterone; its A 21-year-old woman with a family history of
simultaneous use with a mineralocorticoid like acute intermittent porphyria is admitted
fludrocortisone does not make any sense. acutely unwell. She is highly agitated,
complaining of severe abdominal pain, and has
[ Q: 99 ] MasterClass Part2 a sinus tachycardia of 120 bpm, blood pressure
of 180/95, and serum sodium of 124 mM. She
(2010) - Endocrinology
has no past medical history of note, and her
A consultant colleague seeks an on-call review only medication is a combined oral
of one of her patients because the biochemist contraceptive pill. Otherwise examination and
on call investigation fail to reveal any further
abnormality. The next day addition of Ehrlich’s
Dr. Khalid Yusuf El-Zohry – Sohag Teaching Hospital (01118391123) Page | 72
El-Zohry MRCP Questions Bank (Part 2) – Medical Masterclass 2010

aldehyde to a urine specimen results in a pink and 6 months of weight gain, hirsutism and
colour which is not extracted by butanol. male pattern alopecia.

Select one true statement from the following Which one of the following five investigations is
with regard to her management: unnecessary?
A. Temazepam would be an appropriate oral A. 17-OH progesterone
sedative to relieve the acute anxiety
B. 24 hours urinary free cortisol
overnight.
C. Serum testosterone
B. Care must be taken to continue giving the
oral contraceptive to avoid exacerbating D. MRI scan of the adrenals
the situation. E. Gonadotrophin-releasing hormone test
C. Prior to discharge the patient should be (GnRH test).
advised to avoid sunlight as much as
possible during future episodes. Answer & Comments
D. The mainstay of specific management is Correct answer: E
intravenous haem arginate and a high
protein diet.
The above presentation is compatible with
E. In future the patient may be reassured androgen excess. There may be several causes.
that, although her children may inherit Important differentials should include:
the gene, the chances of developing the  polycystic ovarian syndrome (PCOS)
disease are small.
 androgen-producing adrenal and
ovarian tumours
Answer & Comments
 congenital adrenal hyperplasia (CAH)
Correct answer: A
 Cushing's syndrome.
This woman is almost certain to have acute Estimation of 17-OH progesterone is relevant
intermittent porphyria (AIP), presenting with an regarding CAH but the levels may be raised in
acute attack perhaps provoked/exacerbated by certain other adrenal conditions. Short
the contraceptive pill. Extreme care must be synacthen test, involving estimation of 17-OH
exercised in prescribing for such patients, progesterone alongside cortisol in response to
including the avoidance of exogenous sex synacthen, is the standard test for adult type
steroids. However, temazepam is believed to be CAH.
safe. Specific management includes a high
Menstrual irregularities, hirsutism with
carbohydrate diet and intravenous haem.
virilisation and weight gain are features that
Photosensitivity is not a feature of AIP, and
may be present in Cushing's syndrome, hence
unlike many metabolic diseases, it is inherited
estimation of 24 hours urinary free cortisol on
in an autosomal dominant fashion.
at least two occasions is a good screening test.
Estimation of serum testosterone, other
[ Q: 101 ] MasterClass Part2
androgens, sex hormone binding globulin
(2010) - Endocrinology (SHBG) and free androgen index (FAI), will be
A 28-year-old woman is seen in outpatients important in all of the above conditions as
with a 10-month history of oligomenorrhoea hyperandrogenism may be present in all of
them.

Dr. Khalid Yusuf El-Zohry – Sohag Teaching Hospital (01118391123) Page | 73


El-Zohry MRCP Questions Bank (Part 2) – Medical Masterclass 2010

Imaging of adrenals and ovaries, such as CT or [ Q: 103 ] MasterClass Part2


MRI scans, is relevant in the above setting but (2010) - Endocrinology
obviously there will be some further indications
on basic examination and investigations to You see a 32-year-old man in clinic after several
suggest the possibility of adrenal tumour, episodes of severe paroxysmal hypertension,
hyperplasia or ovarian tumour. associated with diaphoresis and headache. On
the last occasion he had been admitted
Estimation of gonadotrophins (follicle-
overnight until the blood pressure settled. A 24-
stimulating hormone (FSH) and luteinizing
hour urine collection revealed VMA excretion of
hormone (LH)) along with androgens and
10 times the upper limit of normal on that
estrogen, is important in the given presentation
occasion, while plasma catecholamines were
but there is no indication to perform a
similarly elevated. An emergency CT scan
gonadotrphin-releasing hormone test. This test
showed a 3cm left adrenal mass. He now
is done as a part of the combined triple test in
attends for further advice.
suspected hypopituitarism and in the
investigation of delayed puberty. Which of the following is true?
A. Arteriography is advisable to corroborate
[ Q: 102 ] MasterClass Part2 the CT finding prior to surgery.
(2010) - Endocrinology
B. No alpha blocker should be given until
A 56-year-old gentleman is referred to you from beta blockade is established.
his GP.
C. There is a 60% chance that the tumour is
malignant.
Which of the following is not one of the
presenting features of type 2 diabetes? D. Labetol would be an appropriate agent to
use in a further hypertensive crisis.
A. Weight loss
E. Perioperative diuresis is likely to be
B. Weight gain
necessary in the event of resection.
C. Ketonuria
D. Polyuria Answer & Comments
E. Peripheral neuropathy. Correct answer: D

Answer & Comments This man has a phaeochromocytoma, roughly


10% of which are malignant. Alpha blockade
Correct answer: C
must be established prior to beta blockade (to
relieve the intense peripheral vasoconstriction -
Weight gain is one of the risk factors for type 2
prior to negative inotropic interventions on the
diabetes and therefore may be one of the
heart). Labetalol, which has around 1:5 alpha:
presenting features. If a patient has elevated
beta antagonism, is a satisfactory agent to use
blood sugars over a prolonged period of time
in the emergency setting, although a titrating
they may present with weight loss. Ketonuria is
dose of phenoxybenzamine, an irreversible
a feature of type 1 diabetes. Both polyuria and
inhibitor, is more common as preparation for
polydipsia are consequences of elevated blood
surgery. Arteriography may precipitate a
sugars. Tiredness, blurred vision, and infections
hypertensive crisis and should be avoided.
are also common presenting features. If
Because high tonic levels of catecholamines
diagnosis of diabetes is delayed the patient may
deplete intravascular volume, fluid repletion is
present with complications
Dr. Khalid Yusuf El-Zohry – Sohag Teaching Hospital (01118391123) Page | 74
El-Zohry MRCP Questions Bank (Part 2) – Medical Masterclass 2010

the mainstay of early post resection confirmed by biochemical testing. Her blood
management. pressure is 106/65 mmHg, her pulse rate is 136
per minute and her temperature is 39ºC. Her
[ Q: 104 ] MasterClass Part2 TSH reading is <0.03 mU/L (0.4-4.0 mU/L) and
Free T4 is 134 pmol/L (9.0-20 pmol/L).
(2010) - Endocrinology
A 36-year-old woman, married with two Which of the following would you AVOID in the
children, is overweight (BMI 28) and rather management of this patient?
hairy. A friend suggests that ‘she must have a A. Potassium iodide
problem with her glands’, and she consults you
B. Beta blockers
about this. She has no other symptoms.
C. Dexametasone
The most likely cause of her hirsutism is:
D. Antithyroid drugs
A. polycystic ovarian syndrome
E. Aspirin.
B. ovarian androgen-secreting tumour
C. adrenal tumour Answer & Comments

D. Idiopathic Correct answer: E


E. Cushing’s syndrome.
Thyroid storm (or ‘Thyroid Crisis’) is a
Answer & Comments potentially fatal medical emergency and
prompt treatment is required. Apart from
Correct answer: D supportive treatment, antithyroid drugs are
commenced immediately and continued every
The commonest causes of hirsutism are 6 to 8 hours. Sodium iodide or saturated
idiopathic (no cause known), familial / racial, solution of potassium iodide should be added
and the polycystic ovarian syndrome (PCOS). to treatment at least 1 hour after the first dose
of antithyroid drugs. This will prevent further
The PCOS usually presents with gradual onset
release of thyroid hormones.
of hirsutism and weight gain on a background
of long-standing oligomenorrhoea, typically Beta blockers like propranolol (one to two
dating back to puberty. milligrams intravenously followed by 40 to 80
mg orally three to four times a day) will help in
Virilizing tumours (ovarian or adrenal) are much
blocking the peripheral effects of T3 and will
less common, but should be suspected if there
also reduce the peripheral conversion of T4 to
is a sudden onset of symptoms, when
T3. Dexamethasone is similarly helpful in that it
examination should look specifically for signs of
may reduce the peripheral conversion of T4 to
virilization (male pattern muscle development,
T3.
clitoromegally).
Antipyretics are used and other cooling
methods may be employed, but aspirin is to be
[ Q: 105 ] MasterClass Part2
avoided as it may make thyrotoxicosis worse by
(2010) - Endocrinology displacing thyroid hormones from the thyroid-
A 21-year-old woman with previously binding globulins.
unrecognised thyrotoxicosis becomes unwell
following appendicectomy. A diagnosis of
thyroid storm is reached clinically and

Dr. Khalid Yusuf El-Zohry – Sohag Teaching Hospital (01118391123) Page | 75


El-Zohry MRCP Questions Bank (Part 2) – Medical Masterclass 2010

[ Q: 106 ] MasterClass Part2 The stress of DKA itself can lead to a raised
(2010) - Endocrinology white blood cell count so this is difficult to
interpret in DKA. Infection is cited as the most
A 31-year-old man with type 1 diabetesis common precipitant of DKA but there is a high
admitted to the Emergency Department with a likelihood that its presence is overestimated for
24-hour history of central abdominal pain, this reason.
nausea and vomiting. His normal weight is 76kg.
Amylase is raised in DKA per se (?salivary in
His plasma glucose is 29.3mmol/l. Arterial blood
origin), although pancreatitis may cause DKA.
gas analysis reveals pH 7.1, pCO2 2.8 kPa, pO2
Amylase cannot be used as a reliable indicator
25.1 kPa, HCO3- 12 mmol/l, and base excess –
of the presence or absence of pancreatitis in
16. Other laboratory tests show Hb 16.1 g/dl,
this circumstance.
wbc 19.3 x 109/l, plts 380 x 109/l, Na+ 127
mmol/l, K+ 6.4 mmol/l, Urea 15.2 mmol/l, Average fluid loss at presentation with DKA is in
creatinine 124 micromol/l, amylase 380 iu/l the region of 100ml/kg (just over 7L in this
(normal<50). The urine ketostix reaction is +++. case).
Bicarbonate therapy has not been shown to be
Choose the TWO correct statements from the
of benefit and is probably harmful in DKA when
following list:
the pH is > 6.9. No trials have been conducted
A. The raised white blood cell count makes on subjects presenting with pH lower than this.
infection the most likely precipitant of
Potassium and phosphate are both depleted in
diabetic ketoacidosis (DKA) in this case.
DKA. Plasma K+ concentrations are raised
B. The amylase suggests pancreatitis as the because this intracellular cation is displaced
likely precipitant of DKA in this case. from the intracellular compartment in acidotic
C. His fluid deficit is likely to be around 7 L. conditions. This actually increases its availability
to be lost in urine or vomit. Most of the
D. Bicarbonate therapy is indicated. potassium and phosphate are lost in the urine
E. Whole body potassium levels are as a result of the profound diuresis. In this case
increased. the degree of renal impairment noted is likely
to be due to dehydration and it is unlikely that
F. Phosphate concentrations are likely to be
this man has the hyperphosphataemia of renal
increased because of renal impairment.
failure.
G. The principal role of insulin therapy is to
DKA is primarily the result of unrestrained
increase glucose uptake into peripheral
lipolysis and gluconeogenesis, augmented by
tissues and liver.
the raised levels of counter-regulatory
H. The principal role of insulin therapy is to hormones (principally glucagon and
inhibit lipolysis and gluconeogenesis. catecholamines) found in this condition. It is
I. The low sodium concentration is entirely lipolysis and unrestrained gluconeogenesis that
due to renal losses. lead to ketosis, and this is the process that
needs to be reversed in order to improve
J. The principal ketone body in plasma and acidosis. Remember, it is the acidosis that kills
urine in DKA is acetoacetate. patients, not a high blood glucose reading.
The low sodium concentration is partly due to
Answer & Comments
electrolyte losses but is also partly artefactual.
Correct answer: CH Hyperglycaemia is restricted to the extracellular
space, so water moves from the intracellular to

Dr. Khalid Yusuf El-Zohry – Sohag Teaching Hospital (01118391123) Page | 76


El-Zohry MRCP Questions Bank (Part 2) – Medical Masterclass 2010

the extracellular compartment initially in DKA, H. The normal potassium makes a diagnosis
diluting plasma sodium. During the of primary hyperaldosteronism very
accompanying osmotic diuresis, water is unlikely.
generally lost in excess of sodium until
I. The likely diagnosis is bilateral renal artery
eventually the loss of water is similar for both
stenosis.
extra and intracellular compartments.
Therefore, in DKA, which is usually of relatively J. The likely diagnosis is
brief duration (<24 hours), plasma sodium phaeochromocytoma.
concentrations may be artificially lowered.
Answer & Comments
3-hydroxybutyrate concentrations in plasma are
usually two to three times those of Correct answer: CE
acetoacetate, but in acidotic states this ratio is
increased further.
In primary hyperaldosteronism, primary
elevation of aldosterone results in suppression
[ Q: 107 ] MasterClass Part2 of plasma renin, by feedback mechanisms, and
(2010) - Endocrinology thus an elevated aldosterone/renin ratio (ARR),
which is a good screening investigation for
A 43-year-old woman presents with primary hyperaldosteronism. In general an ARR
hypertension, poorly controlled on atenolol, >2000 pmol/l per mcg/l.h is almost certainly
perindopril and doxazosin at adequate doses due to primary hyperaldosteronism, >1000
with good compliance. She is found to have an pmol/l per mcg/l.h indicates possible primary
elevated aldosterone/renin ratio (ARR) of hyperaldosteronism, and <800 pmol/l per
greater than 2000 pmol/l per mcg/l.h, normal mcg/l.h excludes primary hyperaldosteronism,
creatinine and a serum potassium of 3.7 but with important caveats.
mmol/l.
The test is affected by drugs and in particular is
Which TWO of the following statements are completely uninterpretable in a patient on
true? beta-blockers. This is because renin secretion is
regulated by the sympathetic nervous system
A. The above investigations suggest a
and blockade of beta adrenoreceptors results in
diagnosis of primary hyperaldosteronism.
direct suppression of plasma renin activity, thus
B. The above investigations exclude a elevation of the ARR. Aldosterone levels may be
diagnosis of primary hyperaldosteronism. relatively low, but the ratio is elevated.
C. Primary hyperaldosteronism can be For formal investigation of suspected primary
neither confirmed nor excluded. hyperaldosteronism, it is recommended that
D. The aldosterone/renin ratio cannot be interfering drugs should be stopped for the
interpreted as she is on doxazosin. following durations prior to testing:
 Spironolactone/oestrogens: 6weeks
E. The aldosterone/renin ratio cannot be
interpreted as she is on atenolol.  Diuretics: 4 weeks
F. The aldosterone/renin ratio cannot be  ACE inhibitors: 2 weeks
interpreted as the patient was not
 NSAIDS: 2 weeks
recumbent.
 Calcium channel blockers: 1 week
G. The aldosterone/renin ratio cannot be
interpreted as the patient was not  Sympathomimetics: 1 week
sodium restricted.

Dr. Khalid Yusuf El-Zohry – Sohag Teaching Hospital (01118391123) Page | 77


El-Zohry MRCP Questions Bank (Part 2) – Medical Masterclass 2010

 Beta-blockers: 1 week received. The patient concerned is a 52-year-


old woman who has recently been discharged
Alpha-blockers (e.g. doxazosin) do not interfere
from hospital, where she had been admitted
and can be continued if necessary to control
with new atrial fibrillation. In view of her atrial
blood pressure.
fibrillation, she was started on warfarin with
Plasma aldosterone and plasma renin activity heparin cover for the first week. This treatment
can then be measured, having made sure the was then completed at her home with the help
patient is salt loaded (liberal dietary sodium of the local home anticoagulation service. As
intake or supplements, eg NaCl tablets 2g po requested on the discharge letter, the GP had
tds on the day before measurement), as sent thyroid function tests when he saw the
elevation of aldosterone is a normal patient the day after discharge, which was a
physiological response in a sodium depleted Friday. The results are as follows:
patient (renin would also be high).
TSH 0.9 mU/l (0.4-4) fT4 31 pmol/l (9.5-22) fT3
Aldosterone/renin ratio is an outpatient 11 pmol/l (4.5- 7.0) (performed by lab in view of
screening procedure and does not require fT4 result)
postural studies.
What is the most appropriate next step?
Postural studies are a great source of confusion
to patients and doctors alike and are often A. Ask the GP to complete a pituitary
badly performed. They are not necessary for function screen and refer to the
the diagnosis of primary hyperaldosteronism endocrinology clinic
but can be useful in differential diagnosis
B. Request anti-TSH Receptor antibodies
between Conn’s Syndrome (a single
aldosterone secreting adenoma, where the C. Reassure the GP that the thyroid function
normal postural increase in renin and is likely to be normal
aldosterone is lost) and idiopathic D. Explain this is an example of sick
hyperaldosteronism/bilateral adrenal euthyroidism
hyperplasia (where the response may be
E. Request a thyroid technetium uptake
preserved or exaggerated). This differentiation
scan
is not highly sensitive or specific and many
centres have abandoned postural studies in
favour of high resolution CT/MRI combined Answer & Comments
with adrenal vein sampling in the differential Correct answer: C
diagnosis of primary hyperaldosteronism. The
potassium in the low-end of the normal range
The key in this case is to appreciate that the
in a patient on an ACE inhibitor (which elevates
patient was taking heparin when the blood was
potassium) is entirely consistent with primary
drawn. Heparin releases lipases from
hyperaldosteronism. There are no clinical
endothelial surfaces, and if there is a delay
pointers to suggest phaeochromocytoma or
before the sample is analysed (e.g. taken before
bilateral renal artery stenosis and the creatinine
the weekend in this case) the lipases in the
is normal on an ACE inhibitor.
sample may act on triglyceride to release free
fatty acids. When these rise high enough, they
[ Q: 108 ] MasterClass Part2 displace free thyroid hormones from their
(2010) - Endocrinology binding proteins, and give rise to artefactually
high fT3 and fT4, as in this case. This is a fairly
You are called by a GP who is concerned about
common but underpublicised problem, and
some thyroid function results he has just
may lead to unnecessary further investigation.

Dr. Khalid Yusuf El-Zohry – Sohag Teaching Hospital (01118391123) Page | 78


El-Zohry MRCP Questions Bank (Part 2) – Medical Masterclass 2010

[ Q: 109 ] MasterClass Part2 additional therapy with other antihypertensives


(2010) - Endocrinology may also be needed.
Carbenoxolone has mineralocorticoid activity
A 58-year-old woman has mild hypertension
and is to be avoided in any type of
(150/95) and hypokalaemia (serum potassium
hyperaldosteronism.
3.1 mmol/l). She is found to have bilateral
adrenal hyperplasia. Glucocorticoids are useful in patients with
Glucocorticoid remediable aldosteronism. It is
What is the most appropriate treatment? an autosomal dominant disorder where the
A. Oral potassium supplements synthesis of mineralocorticoids is also
adrenocorticotropic horomone (ACTH)
B. Bilateral adrenalectomy dependent. Therapy with glucocorticoid will
C. Spironolactone reduce the ACTH levels through a negative
feed-back mechanism and hence there will be
D. Carbenoxolone
less ACTH-dependent mineralocorticoid
E. Glucocorticoids. production.

Answer & Comments [ Q: 110 ] MasterClass Part2


Correct answer: C (2010) - Endocrinology
A 50-year-old hypertensive type 2 diabetic, who
The most common cause of primary has been lost to follow-up for several years, is
hyperaldosteronism is a benign aldosterone- re-referred to clinic by his optician whom he
producing adenoma of the adrenal gland had consulted as his glasses ‘no longer worked’.
(Conn's syndrome). Other less common causes Fundoscopy reveals the appearances shown
of primary hyperaldosteronism include: (see image).
 Idiopathic hyperaldosteronism,
associated with bilateral adrenal
hyperplasia
 Adrenal Carcinoma
 Glucocrticoid remediable aldosteronism
(GRA)
Treatment with spironolactone is helpful in
reducing the mineralocorticoid levels, hence
controlling blood pressure and restoring
normokalaemia. But definitive and long-term
treatments differ for each aetiology.
Surgical removal of the adrenal gland
(adrenalectomy) for Conn's adenoma and also Which of the following statements are true?
for the adrenal carcinoma.
A. Routine ophthalmology opinion should
Long-term treatment with spironolactone or be sought as there are no signs of
amiloride is the treatment of choice in proliferative or pre-proliferative
idiopathic hyperaldosteronism associated with retinopathy.
bilateral adrenal hyperplasia. Sometimes,

Dr. Khalid Yusuf El-Zohry – Sohag Teaching Hospital (01118391123) Page | 79


El-Zohry MRCP Questions Bank (Part 2) – Medical Masterclass 2010

B. The appearances are consistent with with type 1 diabetes where proliferative
hypertensive retinopathy. retinopathy is more common.
C. He is likely to have macular oedema and
requires urgent ophthalmological [ Q: 111 ] MasterClass Part2
evaluation. (2010) - Endocrinology
D. The condition shown is the commonest A 38-year-old sales representative is referred to
sight-threatening complication of type 2 your endocrine clinic. He tells you he has a
diabetes. lifelong history of compulsive water drinking.
E. The appearances shown are seen more He suffered from nocturnal enuresis as a child
commonly in type 1 diabetes. and the diagnosis of psychogenic polydipsia was
agreed by a number of doctors at that time who
F. The appearances shown are consistent
instructed the family on strict behavioural
with pre-proliferative retinopathy.
measures to combat the problem. He had not
G. The appearances shown are consistent seen a doctor since his early teenage years but
with proliferative retinopathy. had become worried that his psychological
H. Commencement of insulin therapy is condition was worsening when he found
contraindicated. himself drinking water from a roadside drain on
a warm day when on a business trip abroad. He
I. Commencement of an ACE-inhibitor is attends reluctantly since his GP is requesting
contraindicated. one final medical opinion prior to referral to the
J. The progression of this condition is psychiatrists. He attends for the water
unaffected by glycaemic or hypertensive deprivation test you arrange with 4l of fluid
control. concealed in his bags and becomes agitated
when they are removed.
Answer & Comments Baseline bloods from clinic: TSH 1.21iu/l,
glucose 5.4 mmol/l, Na+ 138 mmol/l, K+ 3.5
Correct answer: CD
mmol/l, Urea 7.1 mmol/l, Creatinine 110
mmol/l, Ca2+ 2.78 mmol/l, Albumin 45 g/l
The appearances are characteristic of diabetic
maculopathy with circinate exudates around Water deprivation test:
the macula. This generally indicates macula (2mg IM desmopressin given at 8 hours and
oedema, the likely cause of his deterioration in allowed to drink from then)
vision, which can only be visualized directly by
binocular indirect ophthalmoscopy. In addition Time (hrs) 0 2 4 6 8 10
background changes (microaneurysms, dot and
Urine
blot haemorrhages) are seen. He requires osmolarity 205 205 220 240 265 588
urgent treatment by an ophthalmologist as well (mOsm/kg)
as attention to control of hyperglycaemia and
Plasma
hypertension, as both of these are implicated in
Osmolarity 285 290 298 305 310 293
promoting this condition. An ACE inhibitor (mOsm/kg)
should be considered as these may have further
advantages over other anti-hypertensives. What should you do?
This is the commonest sight-threatening A. Water restrict the patient and suggest to
complication of type 2 diabetes, in contrast the GP that they proceed with the
psychiatry referral

Dr. Khalid Yusuf El-Zohry – Sohag Teaching Hospital (01118391123) Page | 80


El-Zohry MRCP Questions Bank (Part 2) – Medical Masterclass 2010

B. Investigate hypercalcaemia as a possible E. should have her white blood count (WBC)
cause of polyuria checked if she develops a sore throat.
C. Prescribe oral or nasal desmopressin
Answer & Comments
D. Prescribe a thiazide diuretic and mild
sodium restriction Correct answer: E
E. Prescribe oral or nasal desmopressin and
arrange an outpatient MRI scan of the Neutropaenia and agranulocytosis are rare but
pituitary. serious side-effects of carbimazole and the
subject of a cerebrospinal meningitis (CSM)
warning, so patients must be told (written
Answer & Comments
information) to report any infection and the
Correct answer: E WBC should be checked. However, the
development of a rash or pruritus can be
This gentleman fails to concentrate his urine treated with an antihistamine with continuation
despite a rise in plasma osmolality above 295 of the treatment. If problems arise with
mOsm/kg and this is diagnostic of diabetes carbimazole, propylthiouracil may be tried as
insipidus (DI). Following administration of sensitivity is not necessarily found to both
desmopressin he concentrates his urine, the drugs. Beta-blockers can be used with
urine osmolality rising by more than 50% and carbimazole and the patient usually becomes
producing a urine osmolality: plasma osmolality euthyroid four to eight weeks after starting
ratio of > 2.0, demonstrating a response of the carbimazole.
kidneys to exogenous desmopressin. This
means that he must have cranial DI, the [ Q: 113 ] MasterClass Part2
treatment for which is oral or intranasal (2010) - Endocrinology
DDAVP. It would therefore be wise to perform
an MRI of the pituitary to look for a structural A 26-year-old man is brought to the Emergency
pituitary lesion that might be the cause. Department with a 12-hour history of nausea,
vomiting and drowsiness. On examination he
The treatment of nephrogenic DI is often
has a reduced Glasgow Coma Scale score of 9
achieved by the use of thiazide diuretics and
(E3, V2, M4). He is apyrexial, with a pulse rate
sodium restriction.
of 120 bpm in sinus rhythm and BP 95/55
mmHg. His heart sounds are normal and his
[ Q: 112 ] MasterClass Part2 chest is clear to auscultation. There are no focal
(2010) - Endocrinology neurological signs.

An 83-year-old woman on carbimazole for Which of the following investigations should


thyrotoxicosis: take immediate priority?

A. usually takes three to six months to A. Arterial blood gases


become euthyroid B. Blood cultures
B. should stop treatment at once if she C. Fingerprick glucose
develops a rash
D. Serum cortisol
C. cannot be given propylthiouracil if she
E. Urine toxicology screen
has a sensitivity reaction to carbimazole
D. should have her propranolol discontinued

Dr. Khalid Yusuf El-Zohry – Sohag Teaching Hospital (01118391123) Page | 81


El-Zohry MRCP Questions Bank (Part 2) – Medical Masterclass 2010

Answer & Comments Long distance running (and other exercise if


excessive), weight loss and psychological stress
Correct answer: C
can all suppress GnRH and therefore lead to
amenorrhoea.
In any patient with a reduced conscious level
Prolactin also suppresses GnRH levels and
the first investigation should be a fingerprick
glucose measurement to exclude accordingly a prolactinoma may also cause
hyperglycaemia or hypoglycaemia: this patient amenorrhoea.
could be a new presentation of diabetic Hypothyroidism would be expected to lead to
ketoacidosis or Addisonian crisis. Remember menorrhagia rather than amenorrhoea.
that it is also important for a sample to be sent
to the laboratory for confirmation of the result.
[ Q: 115 ] MasterClass Part2
The other listed investigations are all potentially
(2010) - Endocrinology
important, and depending on the initial
assessment of the patient it is likely that they (1) A 45-year-old taxi driver is seen in the
would also be requested at an early stage. If outpatient clinic with recently diagnosed
adrenal insufficiency (Addison's disease) is moderate to severe hypertension. His serum
suspected (pigmentation, hypotension, low potassium levels has been persistently low (2.5-
serum sodium and elevated serum potassium), 3.2 mmol/l). His plasma renin activity (PRA) is
then blood should be drawn and sent to the suppressed, and a concomitant aldosterone
laboratory for cortisol and ACTH measurement, level is significantly raised.
and parenteral corticosteroids administered
immediately (eg hydrocortisone 50-100 mg Which TWO of the following conditions are
stat) and continued thereafter until adrenal most likely to be present?
insufficiency has been excluded. A. Phaeochromocytoma
B. Renal artery stenosis
[ Q: 114 ] MasterClass Part2
C. Coarctation of aorta
(2010) - Endocrinology
D. Idiopathic hyperaldosteronism (bilateral
A 34-year-old lady is referred to you with a two-
adrenal zona glomerulosa hyperplasia)
year history of amenorrhoea.
E. Essential hypertension
Which of the following is least likely to be
F. Diuretic treatment
responsible for her symptoms.
G. Carcinoid syndrome
A. Long distance running
H. Conn's syndrome
B. Severe weight loss
I. Excessive liquorice ingestion
C. Prolactinoma
J. Cushing's syndrome.
D. Hypothyroidism
E. Premature ovarian failure. Answer & Comments

Correct answer: DH
Answer & Comments

Correct answer: D Moderate to severe hypertension in a relatively


young patient in association with persistent
hypokalaemia raises several possibilities. It may

Dr. Khalid Yusuf El-Zohry – Sohag Teaching Hospital (01118391123) Page | 82


El-Zohry MRCP Questions Bank (Part 2) – Medical Masterclass 2010

still represent essential hypertension in C. Lithium


association with another cause for
D. Haloperidol
hypokalaemia such as diuretic therapy.
However, one must also consider rarer E. Venlafaxine
possibilities such as pheochromocytoma,
primary hyperaldosteronism and Cushing's Answer & Comments
syndrome.
Correct answer: C
In this case, suppressed plasma renin activity
(PRA) in conjunction with significantly raised
Hypercalcaemia and hyperparathyroidism are
levels of aldosterone demonstrate that the
recognised side effects of long-term treatment
normal feedback loop has broken down, and
with lithium in patients with chronic affective
point to primary hyperaldosteronism as the
psychiatric disorders.
most likely explanation. The findings are
compatible with all different causes of primary
hyperaldosteronism including Conn's syndrome [ Q: 117 ] MasterClass Part2
(adenoma) (~65% of the cases) and idiopathic (2010) - Endocrinology
hyperaldosteronism (bilateral adrenal zona
A 66-year-old man with recently diagnosed type
glomerulosa hyperplasia) (~30%).
2 diabetes is referred by his GP for retinal
In renal artery stenosis, PRA will also be high screening. Digital retinal photography is
with high aldosterone levels, and in habitual performed, and the appearances shown (see
excessive liquorice ingestion, both PRA and image) are noted.
aldosterone levels will be low or fairly low.

[ Q: 116 ] MasterClass Part2


(2010) - Endocrinology
A 57-year-old woman with a long history of
depressive illness is referred to the Endocrine
Clinic for further investigation of polydipsia and
polyuria. Her past medical history includes
primary hypothyroidism for which she is on
long-term thyroxine replacement. On
examination she is mildly overweight (BMI 27.5
kg/m2) and her BP is 140/85 mmHg.
These findings are most likely to be attributable
Investigations show fasting plasma glucose 5.6
to:
mmol/L (normal range 3-6), serum corrected
calcium 2.8 mmol/L (normal range 2.2-2.6), A. previously unrecognized hypertension
serum phosphate 0.7 mmol/L (normal range B. poor glycaemic control
0.8-1.4) and plasma thyroid-stimulating
hormone 2.2 mU/L (normal range 0.4-5.0). C. age-related macular degeneration
D. hyperlipidaemia
Which of the following drugs is most likely to be
of relevance to her presentation? E. previous laser therapy.

A. Amitriptyline
Answer & Comments
B. Citalopram
Correct answer: A

Dr. Khalid Yusuf El-Zohry – Sohag Teaching Hospital (01118391123) Page | 83


El-Zohry MRCP Questions Bank (Part 2) – Medical Masterclass 2010

The presence of flame and blot haemorrhages, high triglycerides are characteristic of type 2
cotton wool spots and blurring of the optic disc diabetes. Autoimmune hypothyroidism is most
margins are typical of the retinal changes that commonly associated with type 1 rather than
are seen in advanced hypertensive retinopathy. type 2 diabetes, and islet cell antibodies are
Whilst some of these findings are also observed typical of the former.
in diabetic eye disease (e.g. dot and blot
haemorrhages, cotton wool spots), the absence [ Q: 119 ] MasterClass Part2
of other features (e.g. hard exudates, venous
(2010) - Endocrinology
beading) should alert the clinician to other
possible diagnoses.
Which of the following would not be on your list
of differential diagnoses for the cause of a
[ Q: 118 ] MasterClass Part2 corrected calcium level of 3.2 mmol/l in a 61-
(2010) - Endocrinology year-old lady?

A 31-year-old woman presents to her GP with A. Tuberculosis


recurrent vaginal candidiasis. Her past medical B. Osteoporosis
history includes hypothyroidism for which she is
C. Berylliosis
on long-term thyroxine replacement. On
examination she is mildly overweight (BMI 28 D. Addison's disease
kg/m2) and her BP is 140/85 mmHg. E. Paget's disease
Investigations reveal fasting plasma glucose
12.2 mmol/L (normal range 3-6) and HbA1c 9.8% F. Primary hyperparathyroidism
(normal range 3.8-6.4). G. Secondary hyperparathyroidism

Which of the following would favour a diagnosis H. Tertiary hyperparathyroidism


of type 2 rather than type 1 diabetes mellitus? I. Bendrofluazide medication
A. Autoimmune aetiology of hypothyroidism J. Lithium medication.
B. Detection of islet cell autoantibodies
Answer & Comments
C. Elevated low-density lipoprotein
cholesterol level Correct answer: BG
D. Normal urinary albumin/creatinine ratio
Primary and tertiary hyperparathyroidism will
E. Presence of dot and blot haemorrhages
cause hypercalcaemia whereas secondary
on fundoscopy
hyperparathyroidism is caused by overactivity
of the parathyroid glands to compensate for
Answer & Comments long standing hypocalcaemia and does not
Correct answer: E usually result in elevated calcium levels.
Any granulomatous disease may cause
Subjects with type 2 diabetes mellitus may hypercalcaemia - including sarcoidosis,
exhibit microvascular and macrovascular berylliosis and tuberculosis.
complications at presentation, reflecting the
fact that their diabetes has often been present
for some time before being diagnosed. This is
not typically the case in type 1 diabetes. Low
high-density lipoprotein cholesterol levels and

Dr. Khalid Yusuf El-Zohry – Sohag Teaching Hospital (01118391123) Page | 84


El-Zohry MRCP Questions Bank (Part 2) – Medical Masterclass 2010

[ Q: 120 ] MasterClass Part2 indicating an effect on the pituitary specific


(2010) - Endocrinology type 2 deiodinase enzyme), which may, or may
not, be associated with an increase in T4 levels
A 56-year-old gentleman is referred by his GP at the expense of T3 (through inhibition of the
for advice with regards abnormal thyroid peripheral type 1 deiodinase enzyme).
function tests, which were discovered when he
attended for routine follow-up in the cardiology
[ Q: 121 ] MasterClass Part2
clinic. The patient is currently asymptomatic,
but was started on amiodarone recently,
(2010) - Endocrinology
following a hospital admission with A 43-year-old man with type 2 diabetes is
'troublesome atrial fibrillation': referred by his GP with deteriorating glycaemic
control on dietary therapy. On examination he
Free T4 15.2 pmol/L (9-20)
has no evidence of diabetic complications and is
TSH 9.8 mU/L (0.4-4) normotensive. You commence him on
metformin. He is a mini-cab driver and is
You should advise the GP: worried that this will affect his job.

A. to discontinue the amiodarone Which of the following pieces of information


immediately would you give him?
B. that the patient is developing A. That he does not need to inform the
hypothyroidism and requires thyroxine DVLA as he will be treated with tablets
replacement rather than insulin.
C. that the patient's thyroid dysfunction is B. That he does not need to inform the
the likely underlying cause for his DVLA as metformin, unlike oral
'troublesome atrial fibrillation' hypoglycaemics which promote insulin
D. to simply repeat the thyroid function secretion, is not associated with
tests in six to eight weeks providing the hypoglycaemia.
patient remains well C. That he should already have informed the
E. to commence carbimazole to minimise DVLA when he was treated with diet
the risk of further cardiac upset due to alone and is likely to be issued with a
uncontrolled thyroid dysfunction. restricted licence for 1, 2 or 3 years now
that he is treated with tablets.
Answer & Comments D. That he should inform the DVLA that he is
a diabetic treated with tablets, this
Correct answer: D
probably will not affect his licence but he
will be advised to re-contact them if his
The anti-arrhythmic agent amiodarone has a condition or treatment changes
high organic iodine content (approximately 1/3
by weight) and bears structural similarities to E. That you will have to inform the DVLA
both T4 and T3. Standard maintenance doses of and it is unlikely that he will be able to
amiodarone result in a massive expansion of continue his job as a mini-cab driver
the iodide pool and can influence thyroid
physiology in a variety of ways, not all of which Answer & Comments
require intervention! For example, it is not
Correct answer: D
uncommon to observe a rise in TSH levels
during the early phases of treatment (possibly

Dr. Khalid Yusuf El-Zohry – Sohag Teaching Hospital (01118391123) Page | 85


El-Zohry MRCP Questions Bank (Part 2) – Medical Masterclass 2010

Diabetes and Driving [ Q: 122 ] MasterClass Part2


Informing the DVLA (2010) - Endocrinology
Patients with diabetes treated by insulin or A GP rings you about a 55-year-old lady with
tablets must by law inform the DVLA (Driver metastatic carcinoma of the breast who has
and Vehicle Licensing Agency) as soon as become acutely confused.
possible. If someone is diagnosed with diabetes
and treated with diet alone, they do not have to What advice would you give?
inform the DVLA, but if they already have A. Start the patient on high dose steroids
diabetes treated with diet alone they must
B. Start the patient on diazepam
inform them if they are applying for a licence.
C. Ask the GP to exclude hypercalcaemia
Ordinary vehicle licences
D. Start the patient on antibiotics
Patients treated with tablets will receive a
letter from the DVLA explaining that they must E. Offer to send out a Community Macmillan
renotify the agency if their condition changes Nurse.
(e.g. if they develop frequent hypos,
complications of their diabetes or are started Answer & Comments
on insulin). They will normally retain an ‘until
70’ licence. Patients treated with insulin will be Correct answer: C
issued with a restricted licence for 1, 2 or 3
years. Hypercalcaemia can present as acute confusion
and can only be diagnosed by a blood test. It
Vision and Driving
can be treated with intravenous fluids and
As well as adequate visual acuity patients must bisphosphonates.
also have no significant field defect and have a
It is important to exclude infection as a cause of
field of vision covering at lease 120 degrees.
confusion. If she is very agitated an
Group 2 vehicle licences antipsychotic such as haloperidol would be
Patients with diabetes treated with diet or more appropriate than diazepam.
tablets are normally allowed to hold licences for
large goods vehicles (LGV) and passenger [ Q: 123 ] MasterClass Part2
carrying vehicles (PCV), which includes (2010) - Endocrinology
minibuses (formally known as HGV and PSV
licences). They must be in good general health. A 44-year-old man presents to his GP
People treated with insulin are not allowed to complaining of erectile dysfunction. He had
hold these licences. A recent amendment been fit and well until the age of 39 years when
allows insulin-treated diabetics to drive he had been diagnosed with type 2 diabetes
medium sized (3.5-7.5 tonnes) vehicles mellitus at an insurance medical. He is a non-
provided they meet strict criteria for self smoker who consumes no alcohol. On
monitoring (at least twice daily) and have had examination he is of normal height but mildly
no hypoglycaemic attacks requiring assistance overweight (BMI 28.5 kg/m2), with central
in the last 12 months. adiposity. He appears hypogonadal and has
bilateral gynaecomastia. Examination of the
NB The law does not bar insulin-users from
external genitalia reveals bilateral testicular
driving taxis (provided there are less than nine
volumes of 10-15 mL. Visual fields are full to
seats); however many local taxi licensing
confrontation. Investigations reveal serum
authorities do apply blanket restrictions.
sodium 139 mmol/L (normal range 137-144),

Dr. Khalid Yusuf El-Zohry – Sohag Teaching Hospital (01118391123) Page | 86


El-Zohry MRCP Questions Bank (Part 2) – Medical Masterclass 2010

serum potassium 4.0 mmol/L (normal range [ Q: 124 ] MasterClass Part2


3.5-4.9), serum creatinine 80 μmol/L (normal (2010) - Endocrinology
range 60-110), serum albumin 42 g/L (normal
range 37-49), serum total bilirubin 20 μmol/L A 42-year-old woman was referred by her GP
(normal range 1-22), serum alanine on account of thyrotoxicosis picked up during
aminotransferase 150 U/L (normal range 5-35), assessment of a sore neck. Further history
serum alkaline phosphatase 165 U/L (normal revealed she had been feeling out of sorts for
range 45-105), HbA1c 8.0% (normal range 3.8- the previous week with aching joints and
6.4), plasma luteinising hormone 22 U/L muscles, which had now been replaced by
(normal range 1-10), plasma follicle stimulating discomfort in the neck. On examination there
hormone 31 U/L (normal range 1-7) and serum was a moderate goitre, slightly more prominent
testosterone 3.1 nmol/L (normal range 9-35). on the right, which was mildly tender to
palpation. There was lid lag and eyelid
Which investigation is most likely to help retraction, and she was clinically hyperthyroid.
elucidate the cause for his erectile dysfunction? Free T4 was 29 pmol/l (9-20), free T3 11 pmol/l
A. Autonomic function testing (3.0-7.5), TSH <0.03 mU/l (0.4-4). The thyroid
scintigram is shown (see image).
B. Karyotype analysis
C. Plasma prolactin
D. Serum oestradiol
E. Serum transferrin saturation

Answer & Comments

Correct answer: E

This man has type 2 diabetes mellitus, primary


hypogonadism and deranged liver function
tests, a combination that raises the possibility
of haemochromatosis. Genetic testing for the
common mutation in the HFE gene is
increasingly available, but initial screening with
Which of the following is true?
measurement of serum transferrin saturation
will identify most cases. Although central A. After 6 months of a blocking dose of
obesity per se may be associated with hepatic carbimazole and thyroxine, there is about
steatosis and borderline low/normal a 50% chance of cure.
testosterone levels, the gonadotrophins are
B. Medical therapy, radioiodine and subtotal
usually within the normal range, reflecting the
thyroidectomy should be discussed as
fact that these subjects typically have normal
options with the patient.
free testosterone levels. However,
measurements of total testosterone may be C. She is most likely to have Riedel’s
borderline low due to a reduction in circulating thyroiditis
sex hormone-binding globulin levels. D. Oral prednisolone would be likely to
relieve symptoms rapidly.

Dr. Khalid Yusuf El-Zohry – Sohag Teaching Hospital (01118391123) Page | 87


El-Zohry MRCP Questions Bank (Part 2) – Medical Masterclass 2010

E. Anti thyroid peroxisomal antibodies are F. Licquorice addiction


likely to be present in high titre.
G. Addison’s disease
H. Chronic diarrhoea
Answer & Comments
I. Hyporeninaemic hypoaldosteronism
Correct answer: D
J. Gitelman’s syndrome.
The presentation with symptoms suggestive of
a viral infection, thyroid pain and tenderness, Answer & Comments
thyrotoxicosis with free T4 and T3 in
Correct answer: GI
proportion, and the silent scintigram all point
strongly to de Quervain’s thyroiditis. This
destructive thyroiditis is most commonly Addison’s disease is associated with
associated with viral infection, and has a hyperkalaemia. Hyporeninaemic
hypoaldosteronism is a common cause of
biphasic pattern, with 2-3 months of
hyperkalaemia out of proportion to reduction in
thyrotoxicosis typically followed by a similar
glomerular filtration rate in patients with
period of hypothyroidism before
interstitial renal disase.
reestablishment of euthyroidism. Treatment is
with simple analgesia or steroids if necessary, In an 18-year-old woman the most likely causes
with symptoms settling rapidly on oral of hypokalaemia would be surreptitious
prednisolone. The condition is only around one vomiting (characterized by finding a very low
fifth as common as Graves’ disease. The ESR is urinary chloride concentration), diuretic abuse
generally very high. Occasionally the early (look for the presence of diuretics in any urine
phase of Hashimoto’s thyroiditis can be sample with a high potassium concentration)
associated with thyroid discomfort and and Gitelman’s syndrome (caused by a defect in
hyperthyroidism, but more commonly it is the thiazide-sensitive NaCl co-transporter in the
insidious, silent and progressive with high distal tubule and the commonest genetic cause
autoantibody titres, unlike de Quervain’s of hypokalaemia).
thyroiditis, where any immunological
perturbation is transitory and mild.
[ Q: 126 ] MasterClass Part2
(2010) - Endocrinology
[ Q: 125 ] MasterClass Part2
A 36-year-old woman presents with a 10-month
(2010) - Endocrinology
history of amenorrhoea and tiredness. She is
An 18-year-old woman presents with taking no medications and has no other clinical
generalized weakness and tiredness. Her serum features of note. Investigations include a
potassium is found to be 2.2 mmol/l. negative pregnancy test, normal TSH and
elevated prolactin of 1600 mU/l. MRI scan
Which TWO of the following diagnoses do NOT shows a pituitary macroadenoma measuring
need to be considered? 1.5cm in diameter.
A. Diuretic abuse
Which of the following is correct?
B. Conn’s syndrome
A. The most likely diagnosis is a
C. Laxative abuse macroprolactinoma.
D. Surreptitious vomiting B. The most likely diagnosis is a
E. Cushing’s syndrome microprolactinoma.

Dr. Khalid Yusuf El-Zohry – Sohag Teaching Hospital (01118391123) Page | 88


El-Zohry MRCP Questions Bank (Part 2) – Medical Masterclass 2010

C. The most likely diagnosis is a non- steroid replacement prior to any other
functioning pituitary adenoma with stalk hormone replacement or surgery. A low-normal
compression. TSH does not rule out secondary
hypothyroidism, as thryoid hormone levels may
D. The first line treatment is bromocriptine,
be low. A visual field defect would prompt
which will reverse her amenorrhoea.
urgent surgery. Otherwise, the timing of
E. She should proceed directly to treatment surgery requires careful discussion with the
with transphenoidal surgery. patient and the pituitary surgeon. If her
pituitary function and fields are intact it may be
Answer & Comments preferable to defer surgery, as this may cause
hypopituitarism. In this case, treatment with
Correct answer: C
bromocriptine to restore menses and
monitoring visual fields may be preferable,
The most likely diagnosis is a non-functioning particularly if fertility is desired.
macroadenoma with stalk compression. You
would expect a much higher prolactin (certainly
[ Q: 127 ] MasterClass Part2
greater than 5000mU/l) if this were a
macroprolactinoma of this size. The prolactin (2010) - Endocrinology
level might be consistent with a A 29-year-old woman presents with a history of
microprolactinoma, however the adenoma oligomenorrhoea and headaches. Initial
would be less than 1cm on MRI. Other investigations reveal a plasma prolactin of 2600
important differentials for a modestly elevated mU/L.
prolactin, as seen in this case, are physiological
(pregnancy, lactation), drug induced (dopamine Which of the following statements are untrue?
antagonists), primary hypothyroidism and
A. The differential diagnosis includes
polycystic ovarian syndrome.
microprolactinoma.
Whilst microadenomas are a relatively common
B. The differential diagnosis includes
incidental finding on current, highly sensitive
polycystic ovarian syndrome (PCOS).
pituitary MRIs and could still be consistent with
elevated prolactin from another cause, such as C. The differential diagnosis includes
drugs or PCOS, macroadenomas are not a primary hyperthyroidism.
common coincidental finding. D. The differential diagnosis includes non-
Bromocriptine will rapidly reduce prolactin as it functioning pituitary adenoma.
is a dopamine agonist, and will substitute for E. Anti-migraine medication that she has
the dopamine that is missing due to stalk been taking may account for her
compression. It may reverse her amenorrhoea hyperprolactinaemia.
if it is only caused by elevated prolactin,
however, she may also have F. The most likely diagnosis is
hypogonadotrophic hypogonadism secondary macroprolactinoma.
to her pituitary macroadenoma compromising G. It is important to take a psychiatric
gonadotroph function. history in this case.
Surgery may be indicated, but she needs a H. In the case of prolactinoma, medical
careful endocrine workup, including full therapy would generally be seen as the
baseline and dynamic testing of her pituitary treatment of choice.
axes and formal evaluation of her visual fields.
If her steroid axis is suppressed she will need

Dr. Khalid Yusuf El-Zohry – Sohag Teaching Hospital (01118391123) Page | 89


El-Zohry MRCP Questions Bank (Part 2) – Medical Masterclass 2010

I. In the case of a non-functioning pituitary Psychiatric history is important in evaluation of


adenoma, surgery would generally be the hyperprolactinaemia both to exclude
treatment of choice. phenothiazines as a cause (many patients will
not volunteer this information) and because
J. Evaluation of this patient should include
dopamine agonists (e.g. bromocriptine,
plasma or urinary beta-hCG
cabergoline) which are the first line treatment
measurement.
of prolactinoma may cause psychiatric
disturbance, particularly in susceptible
Answer & Comments individuals.
Correct answer: CF Any patient presenting with oligomenorrhoea
or secondary amenorrhoea should have beta-
An elevated prolactin concentration above hCG measured to exclude pregnancy.
5000 mU/L usually indicates a prolactinoma,
Headaches are common and they may be
whilst lesser levels are associated with a
unrelated to the hyperprolactinaemia, but raise
number of conditions in addition to
the concern of large non-functioning tumour
prolactinoma, including: pituitary stalk
causing raised intra-cranial pressure and stalk
compression (most commonly by a non-
compression. Pituitary MRI should be
functioning pituitary adenoma), PCOS,
performed along with formal visual field
dopamine antagonist drugs (including
assessment if a macroadenoma is identified.
phenothiazines and metoclopramide, which is
part of certain anti-migraine treatments),
hepatic or renal failure, stress, pregnancy and [ Q: 128 ] MasterClass Part2
lactation. (2010) - Endocrinology
Hyperprolactinaemia is associated with primary You see a 32-year-old man in clinic after several
hypothyroidism rather than hyperthyroidism episodes of severe paroxysmal hypertension,
(secondarily elevated TRH stimulates prolactin associated with diaphoresis and headache. On
release) the last occasion he had been admitted
Macroprolactinoma is not the most likely overnight until the blood pressure settled. A 24-
diagnosis as the degree of plasma prolactin hour urine collection revealed VMA excretion of
elevation is normally roughly commensurate 10 times the upper limit of normal on that
with the size of the prolactinoma. The prolactin occasion, while plasma catecholamines were
here is only modestly elevated, thus more similarly elevated. An emergency CT scan
compatible with a small microprolactinoma. If a showed a 3cm left adrenal mass. He now
large pituitary tumour were visualized in this attends for further advice.
case it would be more likely to be a non-
Which of the following is true?
functioning adenoma causing stalk
compression. A macroprolactinoma could, A. Arteriography is advisable to corroborate
theoretically, present late in its evolution with a the CT finding prior to surgery.
modestly elevated prolactin if auto-infarction B. No alpha blocker should be given until
had occurred. This would be unlikely in a beta blockade is established.
woman, as they usually present early due to
oligo/amenorrhoea. Prolactinomas tend to C. There is a 60% chance that the tumour is
present later in men, where symptoms of malignant.
prolactin elevation are more subtle and less D. Labetol would be an appropriate agent to
specific. use in a further hypertensive crisis.

Dr. Khalid Yusuf El-Zohry – Sohag Teaching Hospital (01118391123) Page | 90


El-Zohry MRCP Questions Bank (Part 2) – Medical Masterclass 2010

E. Perioperative diuresis is likely to be Answer & Comments


necessary in the event of resection.
Correct answer: CH

Answer & Comments


The causes of hyponatraemia can be classified
Correct answer: D depending on the fluid status of the patient:
hypovolaemic, euvolaemic or hypervolaemic.
This man has a phaeochromocytoma, roughly Within each of these groups the causes can be
10% of which are malignant. Alpha blockade subdivided further.
must be established prior to beta blockade (to Gliclazide has not been shown to cause
relieve the intense peripheral vasoconstriction - hyponatraemia. Chlorpropamide, one of the
prior to negative inotropic interventions on the longer acting sulphonylureas, can do so by
heart). Labetalol, which has around 1:5 alpha: enhancing the action of antidiuretic hormone.
beta antagonism, is a satisfactory agent to use
Hyperparathyroidism causes hypercalcaemia
in the emergency setting, although a titrating
but does not cause hyponatraemia.
dose of phenoxybenzamine, an irreversible
inhibitor, is more common as preparation for
surgery. Arteriography may precipitate a [ Q: 130 ] MasterClass Part2
hypertensive crisis and should be avoided. (2010) - Endocrinology
Because high tonic levels of catecholamines
deplete intravascular volume, fluid repletion is A 42-year-old gentleman with type 1 diabetes
the mainstay of early post resection has read about a new type of insulin.
management.
Which of the following is not true concerning
insulin glargine?
[ Q: 129 ] MasterClass Part2
A. Insulin glargine can be used as a basal
(2010) - Endocrinology insulin.
A 68-year-old man is admitted with confusion B. Insulin glargine is less painful to inject
and a serum sodium concentration of 117 than human insulatard.
mmol/l.
C. Insulin glargine is a clear insulin.
Which TWO of the following would be UNLIKELY D. Insulin glargine is licensed for both type 1
to cause hyponatraemia? and type 2 diabetes.
A. Bendrofluazide medication E. Insulin glargine is thought to cause fewer
B. Carbamazepine medication severe hypoglycaemic episodes than
isophane insulin.
C. Gliclazide medication
D. Subdural haematoma Answer & Comments
E. Small cell lung cancer Correct answer: B
F. Chronic liver failure
G. Congestive cardiac failure Insulin glargine has been available in the UK
since September 2002. It is produced by
H. Primary hyperparathyrodism recombinant DNA technology. Two arginine
I. Addison's disease residues have been added to the beta chain and
a glycine substituted for asparagine in the alpha
J. Hypothyroidism.

Dr. Khalid Yusuf El-Zohry – Sohag Teaching Hospital (01118391123) Page | 91


El-Zohry MRCP Questions Bank (Part 2) – Medical Masterclass 2010

chain. This alters the isoelectric point from D. Measurement of sex hormone binding
being acidic to a near neutral pH. Therefore globulin (SHBG) would be useful.
glargine remains in solution in the acidic
E. Full pituitary function testing should be
conditions of the vial or cartridge and
conducted.
precipitates in the neutral pH of subcutaneous
tissues. It is therefore a clear insulin. These
Answer & Comments
changes mean delayed absorption, a late onset
of action and prolonged duration of action. Correct answer: D
Given as a single daily dose glargine produces a
flat pharmacokinetic profile. Because of the
Simplistically, LH stimulates testosterone
acidity glargine can be more painful to inject
production and follicle-stimulating hormone
than Insulatard. There is evidence showing that
(FSH) spermatogenesis, so B is wrong. This man
there is a reduced risk of hypoglycaemia with
has a normal testicular volume (20-30ml) and
glargine than with traditional basal insulins
therefore is highly unlikely to be
hypogonadotrophic. Penile length is also
[ Q: 131 ] MasterClass Part2 normal (mean stretched length of flaccid
(2010) - Endocrinology erectile tissue 12.4±2.7cm in Caucasian males,
14.6cm African/African Caribbean descent, 10.6
A 32-year-old man is referred to your clinic by cm Asian males) and he shaves daily with
his general practitioner who has diagnosed him normal pubic hair all making hypogonadism
as having hypogonadotrophic hypogonadism. unlikely.
He went to see his GP because of difficulty
maintaining erections and concerns regarding He has no evidence of pituitary insufficiency
the size of his penis, which he regards as small. and his low testosterone concentration is likely
He went through puberty later than his peers at to be a reflection of lowered SHBG
school, though thinks he started by age 16. He concentrations due to his obesity (insulin
has attained a normal adult height of 1.78 m, downregulates SHBG expression and secretion
and weighs 108 kg; BMI is therefore 34 kg/m2. and thus the hyperinsulinaemia of obesity leads
He shaves most days. Examination reveals scant to lower SHBG levels). Measurement of his
axillary and body hair, normal pubic hair, SHBG would confirm low levels and imply that
stretched penile length of 10.4cm and testicular free testosterone concentrations are normal
volume of 25ml (orchidometer assessment). (some people use a ‘free androgen index’ for
Testosterone (9am) is 8nmol/L (normal 10-35 the calculated free testosterone though there is
nmol/L), LH 4IU/L (1.3-13) and FSH 3.6 IU/L debate about its utility in this situation). His
(0.9-15). Thyroid function tests performed at sexual difficulties are likely to have other origins
GP surgery are reported as normal and and counselling may prove the best
prolactin was 40 mmol/L (10-110). intervention here.

Which of the following statements is correct? [ Q: 132 ] MasterClass Part2


A. He has hypogonadotrophic (2010) - Endocrinology
hypogonadism.
A 28-year-old woman with gestational diabetes
B. Luteinizing hormone (LH) stimulates mellitus has required insulin therapy in the
spermatogenesis. form of a basal bolus regimen from 28 weeks'
C. MRI pituitary is indicated. gestation. She is commenced on a sliding-scale
intravenous insulin infusion during labour and
the delivery proceeds uneventfully.

Dr. Khalid Yusuf El-Zohry – Sohag Teaching Hospital (01118391123) Page | 92


El-Zohry MRCP Questions Bank (Part 2) – Medical Masterclass 2010

Following delivery she should be advised to: A. Squamous cell carcinoma of lung
A. Restart her basal bolus subcutaneous B. Oat cell carcinoma of lung
insulin regimen
C. Uterine fibroma
B. Start gliclazide
D. Bronchial carcinoid
C. Start metformin
E. Pancreatic carcinoma
D. Start rosiglitazone
E. Stop all treatment Answer & Comments

Correct answer: B
Answer & Comments

Correct answer: E This woman is quite likely to have an underlying


malignancy. She has hyponatraemia. Her
calculated plasma osmolality is around 255
Gestational diabetes mellitus (GDM) is a
mosmol/kg. Urine osmolality is
transient phenomenon. Dietary measures are
disproportionately high at 590. Urinary sodium
tried in the first instance, but if blood glucose
is in excess of 50 mmol/l. Her picture is
levels remain above target then insulin therapy
compatible with SIADH. It could be due to
is the mainstay of treatment during the
ectopic production of ADH from a tumour. Oat
remainder of the pregnancy. Most patients with
cell carcinoma of lung is associated with ectopic
GDM are able to discontinue all therapy
ADH production.
following delivery of the placenta. However,
women with a history of GDM are at higher risk Squamous cell carcinoma of the lung may be
of developing type 2 diabetes mellitus in later associated with ectopic production of PTHrP
life. Accordingly they should be encouraged to with consequent hypercalcaemia. Pancreatic
follow a healthy diet and lifestyle. Repeating carcinoma and bronchial carcinoids may be
the oral glucose tolerance test at 6-8 weeks associated with ectopic production of ACTH and
after delivery can help to identify those at uterine fibroids may produce erythrpoietin.
higher risk.
[ Q: 134 ] MasterClass Part2
[ Q: 133 ] MasterClass Part2 (2010) - Endocrinology
(2010) - Endocrinology
A 50-year-old woman presents with a two-year
These are the blood results of a 62-year-old history of diarrhoea, intermittent facial flushing
lady with severe weight loss, cachexia and and wheeze with minor (3kg) weight loss.
general debility. Thyroid function and fasting plasma glucose are
normal but 24-hour urinary 5-HIAA levels are
Sodium: 121 mmol/l (136-150 mmol/l),
elevated. A colonoscopy performed in view of
Potassium: 3.6 mmol/l (3.6-5.0 mmol/l), Urea:
her diarrhoea has identified a caecal lesion.
3.2 mmol/l (2.5-7.5 mmol/l), Creatinine: 64
Biopsy of this lesion showed cells staining for
micromol/l (50-120 micromol/l), Glucose: 5.6
chromogranin A on histology. Octreotide
mmol/l, Urine Osmolality: 590 mosmol/Kg,
scanning of the liver was negative.
Urinary Sodium: 56 mmol/l, TSH: 3.4 mmol/l
(0.4-4.0 mmol/l)
What is the most likely diagnosis?
Which of the following diagnoses is most A. A neuroendocrine tumour causing
compatible with above picture? carcinoid syndrome. As this is a slow

Dr. Khalid Yusuf El-Zohry – Sohag Teaching Hospital (01118391123) Page | 93


El-Zohry MRCP Questions Bank (Part 2) – Medical Masterclass 2010

growing tumour, resection of the caecal are likely to live for many years and may die of
lesion should be curative unrelated causes. Ongoing management should
involve echocardiography to screen for
B. Carcinoid syndrome. Further imaging of
carcinoid heart disease (right-sided valvular
the liver with ultrasound scanning or CT
lesions). Symptomatic management may
should be performed along with
include somatostatin analogues, hepatic
measurement of fasting gut hormones
embolisation, hepatic chemo-embolisation and
C. Phaeochromocytoma and further imaging chemotherapy. Octreotide is less likely to be
with MIBG scan should be performed effective if octreotide scan negative, but other
D. Inflammatory bowel disease analogues such as lanreotide have different
affinities for different somatostatin receptor
E. Cause is irritable bowel disease/anxiety; subtypes, which may be present on the tumour.
as the octreotide scan is negative, a
Patients should be managed at a tertiary
diagnosis of carcinoid syndrome is referral centre with a multidisciplinary team
unlikely and the caecal lesion may be a available (eg: endocrinologist, oncologist,
coincidental finding interventional radiologist).

Answer & Comments


[ Q: 135 ] MasterClass Part2
Correct answer: B (2010) - Endocrinology
A 15-year-old girl is admitted for
Chromogranin A staining is consistent with a
parathyroidectomy. Her mother had had the
neuroendocrine tumour and the caecal-
same operation 16 years previously though it
appendiceal region is the commonest location
had had to be repeated 6 months later because
for a carcinoid primary. These tumours are
of recurrent hypercalcaemia. Investigations
indeed slow growing, however the presence of
showed:
systemic symptoms suggestive of carcinoid
syndrome along with elevated urinary 5HIAA, Calcium: 2.88 mmol/l
suggest there are already liver metastases Phosphate: 0.81 mmol/l
causing release of 5HT into the systemic
circulation, and circumventing first-pass Creatinine: 70 micromol/l
metabolism in the liver. Albumin: 42g/l
Octreotide scanning is positive in up to 85% of PTH: 36 ng/l (10-65)
cases, however a negative scan does not rule
Urinary calcium excretion: 2 mmol/24 hrs
out liver metastases. The liver should be
(normal <7.5 mmol/24 hrs)
imaged by high resolution CT with fine cuts or
by USS. The sensitivity of USS may be increased Urine creatinine concentration 6 mmol/l
by the use of microbubble contrast medium
(levovist), which is available at some centres. What is the most likely diagnosis?
Fasting gut hormones should be measured as A. Primary hyperparathyroidism
neuroendocrine tumours may co-secrete other
B. Multiple Endocrine Neoplasia Type 1
hormones such as VIP, which may contribute to
the diarrhoea. C. Multiple Endocrine Neoplasia Type 2a
The prognosis is generally good. Although D. Multiple Endocrine Neoplasia Type 2b
patients are not generally cured, their E. Familial benign hypocalciuric
symptoms can be effectively managed and they hypercalcaemia.

Dr. Khalid Yusuf El-Zohry – Sohag Teaching Hospital (01118391123) Page | 94


El-Zohry MRCP Questions Bank (Part 2) – Medical Masterclass 2010

Answer & Comments [ Q: 136 ] MasterClass Part2


Correct answer: E (2010) - Endocrinology
A 27-year-old woman is referred for a medical
Calcium excretion is low whereas it is usually opinion because of hirsutism.
increased in primary hyperparathyroidism. A
useful tool for assessing this is the calcium Which of the following features from her history
clearance/creatinine clearance ratio: would be of most concern?

{[urine Ca++]X[plasma creat]} A. History of hirsutism dating back six


months
{[plasma Ca++]X[urine creat]}
B. Family history of hirsutism
(NB: correct plasma creatinine concentrations
to mmol/l from micromol/l) C. Family history of infertility

A value > 0.01 is suggestive of D. The use of oral corticosteroids to control


hyperparathyroidism. A value less than this asthma
implies inappropriately low calcium excretion E. Weight gain over preceding six months.
(corrected for plasma calcium and renal
function).
Answer & Comments
Certainly the presentation of hypercalcaemia in
such a young individual should prompt Correct answer: A
investigation into the possibility of MEN 1 or
MEN 2a if hyperparathyroidism is confirmed. A Hirsutism can occasionally be a sign of serious
family history would also be important. MEN 2b underlying pathology. If the history is as short
is not associated with hyperparathyroidism and as six months, this would increase the
so need not be considered here. possibility that the woman may have an
underlying virilizing adrenal or ovarian tumour.
In this case the family history (particularly of
failed parathyroidectomy), high calcium, normal
parathyroid hormone (PTH) and low calcium [ Q: 137 ] MasterClass Part2
excretion suggest the diagnosis of familial (2010) - Endocrinology
benign hypocalciuric hypercalcaemia (FBHH).
A 36-year-old woman, married with two
This is a disorder of the calcium sensing
children, is overweight (BMI 28) and rather
receptor and is inherited in an autosomal
hairy. A friend suggests that ‘she must have a
dominant manner. Individuals are
problem with her glands’, and she consults you
asymptomatic, calcium concentrations are
about this. She has no other symptoms.
usually less than 3 mmol/l and PTH is usually in
the normal range (though inappropriately high
The most likely cause of her hirsutism is:
for the prevailing calcium concentrations).
Urine calcium excretion is reduced. The A. polycystic ovarian syndrome
homozygous form presents in the neonatal B. ovarian androgen-secreting tumour
period with severe hypercalcaemia and requires
C. adrenal tumour
urgent total parathyroidectomy to prevent
death from rib fractures amongst other D. idiopathic
complications.
E. Cushing’s syndrome.

Dr. Khalid Yusuf El-Zohry – Sohag Teaching Hospital (01118391123) Page | 95


El-Zohry MRCP Questions Bank (Part 2) – Medical Masterclass 2010

Answer & Comments A. Cerebral salt wasting syndrome


secondary to subarachnoid haemorrhage
Correct answer: D
B. Cranial diabetes insipidus secondary to
The commonest causes of hirsutism are meningitis
idiopathic (no cause known), familial / racial, C. Cranial diabetes insipidus secondary to
and the polycystic ovarian syndrome (PCOS). ruptured intracranial aneurysm
The PCOS usually presents with gradual onset D. Pituitary apoplexy with acute onset
of hirsutism and weight gain on a background anterior pituitary insufficiency
of long-standing oligomenorrhoea, typically
E. Syndrome of inappropriate antidiuretic
dating back to puberty.
hormone (SIADH) secondary to opiates
Virilizing tumours (ovarian or adrenal) are much
less common, but should be suspected if there Answer & Comments
is a sudden onset of symptoms, when
examination should look specifically for signs of Correct answer: A
virilization (male pattern muscle development,
clitoromegally). A normal CT scan does not exclude
subarachnoid haemorrhage (SAH) and there is
[ Q: 138 ] MasterClass Part2 no indication that a lumbar puncture has been
performed so SAH should still be considered.
(2010) - Endocrinology
Diabetes insipidus is a possible consequence of
A 36-year-old woman on the general medical a ruptured intra cranial aneurysm or infective
ward has become increasingly drowsy. She is a involvement of the pituitary and would cause
schoolteacher, 5 months pregnant, and was polyuria; however cranial DI would lead to
admitted 3 days previously with a sudden onset increased free water clearance and an increase
of a left parietal headache. CT scan of the brain in plasma osmolality rather than the low
was reported as normal and she had been plasma osmolality and sodium seen here. SIADH
admitted and given paracetamol and is a cause of hyponatraemia with increased
dihydrocodeine for analgesia. Because she urine sodium concentration > 20-30 mmol/l,
became incontinent of urine, a urinary catheter but this lady has a large diuresis and is clinically
was inserted. The fluid charts show a urinary hypovolaemic thus excluding SIADH.
output of 2L over the past 8 hours, during The differential therefore lies between cerebral
which time she has drunk only 200mL, and she salt wasting syndrome secondary to SAH or
has not received any intravenous fluids. pituitary apoplexy. Clinically this is unlike
She is markedly drowsy, with GCS 9, but there pituitary apoplexy; there are no eye signs and
are no localising neurological signs. Routine the distance between the onset of headache
observations show temperature 37.3°C, pulse and clinical demise is longer than you might
110/min and blood pressure 106/63 mmHg. Her expect. A normal CT scan is also reassuring in
mucous membranes are dry, and the JVP not this regard (though it does not exclude the
visible. Investigations show Hb 12.1 g/l, WBC diagnosis) and this level of diuresis is unlikely
15.0 x106/l, Plts 280 x109/l, Na+ 126 mmol/l, with cortisol deficiency secondary to anterior
K+ 4.9 mmol/l, urea 11.3 mmol/l, creatinine 90 pituitary failure. Cranial DI can complicate
micromol/l, glucose 5.2 mmol/l. The urinary pituitary apoplexy but is a later event and this is
sodium concentration is 60 mmol/l. not a picture of DI as explained earlier. This
leaves the diagnosis of cerebral salt wasting
What is the most likely diagnosis? syndrome. This is a recognized complication of

Dr. Khalid Yusuf El-Zohry – Sohag Teaching Hospital (01118391123) Page | 96


El-Zohry MRCP Questions Bank (Part 2) – Medical Masterclass 2010

subarachnoid haemorrhage and of If there are no hyperglycaemic symptoms then


neurosurgery as well as other intracranial there have to be two glucose results within the
disease, and is characterized by the diagnostic range on separate days.
development of excessive natriuresis and
A fasting plasma glucose greater than or equal
hyponatraemic dehydration usually over a week
to 7 mmol/l or a 2-hour post 75 glucose load
or so following the intracranial insult. The exact
plasma glucose greater than or equal to 11.1
cause is unknown though hypotheses include
mmol/l would meet diagnostic criteria.
sympathetic nervous system activation and
excessive dopamine release, the release of The diagnostic criteria are the same in
natriuretic factors including atrial natriuretic pregnancy. In children the standard 75-g OGTT
peptide and brain natriuretic peptide have also does not apply. Instead 1.75g/kg of glucose is
been proposed. Treatment consists of given.
management of the underlying condition and These glucose values were agreed by the World
rigorous rehydration with normal saline to Health Authority in 1997 as the levels required
replace the salt deficit. Distinguishing this to diagnose diabetes.
syndrome from SIADH is therefore imperative
since the treatments are markedly different.
[ Q: 140 ] MasterClass Part2
(2010) - Endocrinology
[ Q: 139 ] MasterClass Part2
(2010) - Endocrinology A 28-year-old woman is diagnosed as having
Graves’ disease. She has prominent eyes.
In which of the following patients can diabetes
be diagnosed from the information given? A medical student asks you to define what is
meant by the term ‘lid lag’. You reply:
A. A 46-year-old gentleman with polydipsia,
polyuria and a glucose of 12.1 mmol/l. A. When the patient is looking straight
ahead, you can see a bit of the sclera all
B. A 33-year-old lady with a fasting plasma around the iris.
glucose of 9 mmol/l.
B. When the patient tries to close her eyes,
C. A 33-year-old lady with headaches and a the lids only touch each other with
random plasma glucose of 14 mmol/l. difficulty.
D. A 62-year-old lady with polydipsia, C. When the patient follows your finger,
polyuria and a glucose of 10.4 mmol/l. moving upwards from below, the upper
E. A lady who is 32 weeks pregnant and has eyelid has difficulty in moving out of the
a random glucose of 14.1 mmol/l. way.
D. When looking at the patient from the
Answer & Comments side, you see that the eyes are proptosed.

Correct answer: A E. When the patient follows your finger,


moving downwards from above, the
In people with symptoms of hyperglycaemia sclera can temporarily be seen above the
iris.
(e.g. thirst, polyuria, weight loss) diabetes can
be diagnosed with a random glucose greater
than 11.1 mmol/l. Headaches are not Answer & Comments
hyperglycaemic symptoms.
Correct answer: E

Dr. Khalid Yusuf El-Zohry – Sohag Teaching Hospital (01118391123) Page | 97


El-Zohry MRCP Questions Bank (Part 2) – Medical Masterclass 2010

Option A is the definition of lid retraction. hyperparathyroidism will be compatible with


"D" and hypervitaminosis D may be associated
Other ocular features of Graves’ disease include
with a biochemical profile given in "E".
proptosis, ophthalmoplegia, periorbital oedema
and chemosis.
[ Q: 142 ] MasterClass Part2
[ Q: 141 ] MasterClass Part2 (2010) - Endocrinology
(2010) - Endocrinology (2) A 43-year-old woman is referred to the
endocrine clinic with a corrected serum calcium
A 39-year-old woman is seen in the clinic,
of 1.56mmol/l, which was detected when she
complaining of generalised aches and pains.
had cataract surgery done recently.
She also complains of difficulty in climbing
stairs. She has proximal weakness in her limbs Her previous medical history is non-significant
on clinical examination. Her pelvis X-ray reveals and so is the family history. Her periods are
the presence of Looser's zones. normal and regular. She went through normal
schooling and went to college and studied
Which of the following sets of test results is beauty therapy. She does admit to spasm in her
likely to be hers? hands when she gets anxious, but otherwise
A. High calcium, low phosphate, normal has no symptoms of chronic hypocalcaemia.
alkaline phosphatase and a high PTH. She is short (4 feet 10 inches), has horizontal
B. Low calcium, low phosphate, high alkaline ridges on her nails, and Trousseau's sign is
phosphatase and a high PTH. positive. There are no other abnormalities.

C. Normal calcium, normal phosphatase, Other blood results show a raised phosphate at
high alkaline phosphatse and a normal 2.56mmol/l, and very low (undetectable) levels
PTH. of parathyroid hormone (PTH). Her 0900 hours
cortisol, thyroid function tests, urea and
D. Low calcium, high phosphate, normal
creatinine, and other electrolytes are normal.
alkaline phosphatse and a high PTH.
E. High calcium, high phosphate, normal Which of the following diagnoses is most likely?
alkaline phosphatse and a low PTH. A. Pseudohypoparathyroidism
B. Idiopathic (acquired) hypoparathyroidism
Answer & Comments
C. Pseudopseudohypoparathyroidism
Correct answer: B
D. Autoimmune hypoparathyroidism
(Polyglandular Autoimmune Syndrome
This lady has typical symptoms, signs and X-ray
type 1)
findings of osteomalacia due to vitamin D
deficiency. Lack of vitamin D leads to low or low E. Chronic renal failure with secondary
to normal calcium, low or low to normal hyperparathyroidism.
phosphate and typically, high levels of alkaline
phosphatase. Secondary hyperparathyroidism Answer & Comments
occurs in response to hypocalcaemia. Primary
Correct answer: B
hyperparathyroidism will have a profile like the
one given in option "A." Paget’s disease may be
compatible with option "C" with raised alkaline This woman has hypoparathyroidism. Patients
phosphatase levels. Chronic renal failure with with pseudohypoparathyroidism have somatic
hypocalcaemia and secondary features and PTH levels are high because of

Dr. Khalid Yusuf El-Zohry – Sohag Teaching Hospital (01118391123) Page | 98


El-Zohry MRCP Questions Bank (Part 2) – Medical Masterclass 2010

target organ resistance. Patients with


pseudopseudohypoparathyroidism have the
somatic features without biochemical
abnormality. Renal failure is excluded by
normal urea and creatinine.
It is difficult to differentiate between idiopathic
hypoparathyroidism and the
familial/autoimmune hypoparathyroidism that
occurs as a part of polyglandular autoimmune
syndrome type 1. However, without any family
history of autoimmune disorder and without
any personal history of problems such as
mucocutaneous candidiasis, vitiligo or alopecia,
this woman’s hypocalcaemia is most likely to be
due to idiopathic/acquired hypoparathyroidism.

[ Q: 143 ] MasterClass Part2


(2010) - Endocrinology
A 27-year-old man with type 1 diabetes has
retinal photographs performed as part of his
annual review.

Which one of the following findings would


specifically suggest the existence of pre-
proliferative diabetic retinopathy?
A. Hard exudates
B. Microaneurysms
C. Intraretinal microvascular abnormalities
D. Lipaemia retinalis
E. Retinal detachment

Answer & Comments

Correct answer: C

The classical features of pre-proliferative


retinopathy include:
 venous beading/looping
 multiple haemorrhages
 multiple cotton wool spots
 Intraretinal microvascular abnormalities

Dr. Khalid Yusuf El-Zohry – Sohag Teaching Hospital (01118391123) Page | 99


El-Zohry MRCP Questions Bank (Part 2) – Medical Masterclass 2010

[ Q: 2 ] MasterClass Part2

Cardiology (2010) - Cardiology


A 68 yr old man presents with breathlessness.
On examination he has an early diastolic
(130 Questions) murmur at the left sternal edge.

(Medical Masterclass – Part 2) Which of the following features would suggest


that his aortic incompetence is clinically
significant?
A. Narrow pulse pressure
[ Q: 1 ] MasterClass Part2
(2010) - Cardiology B. Normal cardio-thoracic ratio on chest
radiograph
A 55-year-old man presents with an acute ST-
C. Long diastolic murmur
segment elevation myocardial infarction. He
receives treatment with thrombolysis and D. Graham-Steele murmur
aspirin. Four hours later he has ventricular E. Dislpaced, hyperdynamic apex beat.
fibrillation; sinus rhythm is restored with a
single 200-joule shock.
Answer & Comments
Which of the following is the appropriate next Correct answer: E
treatment?
A. Amiodarone Severe aortic incompetence is suggested by the
B. Beta-blocker presence of one or more of the following
clinical features:
C. Dobutamine
1. Short early diastolic murmur (diastolic
D. Implantable cardioverter defibrillator pressure in the left ventricle rapidly
(ICD) approaches that in the aorta)
E. Temporary wire 2. Wide pulse-pressure
3. Displaced, hyperdynamic apex beat -
Answer & Comments
implying significant volume overload.
Correct answer: A The Austin-Flint murmur may be heard in aortic
incompetence (fluttering of the anterior mitral
Ventricular fibrillation is not uncommon within valve leaflet due to turbulance from the
the first 24 hours following myocardial regurgitant jet). Whilst a normal heart size on
infarction, and is treated with DC-shock. Beta- chest radiograph does not preclude a diagnosis
blockers confer early mortality and morbidity of severe aortic incompetence (e.g. acute), it is
benefit following acute myocardial infarction, far more commonly enlarged.
and hence should be prescribed routinely as
long as no contra-indications exist. Dobutamine
is indicated for patients with cardiogenic shock.

Dr. Khalid Yusuf El-Zohry – Sohag Teaching Hospital (01118391123) Page | 101
El-Zohry MRCP Questions Bank (Part 2) – Medical Masterclass 2010

[ Q: 3 ] MasterClass Part2 ‘routine’ biochemical screen is normal,


(2010) - Cardiology excepting for potassium 3.3 mmol/l.

A 57-year-old lady is being pre-assessed for a The TWO most likely causes of his hypertension
gynaecological procedure and is found to have are:
a murmur. A. Renal hypertension
B. Hypothyroidism
C. Renovascular hypertension
D. Cushing’s syndrome
E. Primary hyperaldosteronism (Conn’s
syndrome)
F. Acromegaly
G. Essential hypertension
H. Isolated clinic (‘white coat’) hypertension
I. Phaeochromocytoma
J. Coarctation of the aorta.
What does her echo image show?
A. Aortic regurgitation Answer & Comments
B. Aortic stenosis
Correct answer: GH
C. Mitral regurgitation
D. Mitral stenosis All of the conditions listed, excepting
hypothyroidism, might explain hypertension,
E. Infective endocarditis.
but all other than essential hypertension and
‘white coat’ hypertension are rare (together
Answer & Comments accounting for less than 5% of cases).
Correct answer: C Although a secondary cause of hypertension is
very unlikely it would be important to look for
The image demonstrates a posteriorly directed clues in history and examination that might
jet of mitral regurgitation in a lady with anterior suggest renovascular disease (ischaemic heart
leaflet prolapse. disease, transient ischmaemic attack (TIA) /
stroke, peripheral vascular disease), renal
disease (previous nephritis, results of urine
[ Q: 4 ] MasterClass Part2
testing for e.g. insurance / employment
(2010) - Cardiology medicals).
A 48-year-old man is found to have a blood Episodes of palpitations, sweating or headache
pressure of 176/112 mmHg when he attends may suggest phaeochromocytoma, but a less
his general practitioner for a ‘new patient exotic cause such as anxiety would be a much
check-up’. He takes occasional anxiolytics for more likely explanation. The serum potassium
anxiety, but his past medical history is concentration is just below the lower limit of
otherwise unremarkable. Physical examination normal, but primary aldosteronism (Conn’s
is normal, excepting for obesity (BMI 32). A syndrome) remains exceedingly unlikely.

Dr. Khalid Yusuf El-Zohry – Sohag Teaching Hospital (01118391123) Page | 102
El-Zohry MRCP Questions Bank (Part 2) – Medical Masterclass 2010

In the case of an obese man it is also important elevation. This is seen in leads V1-V5, indicating
to note that the blood pressure reading may be that the infarct is anterior.
falsely elevated as a result of inadequate blood
The rhythm strip shows a number of ectopics,
pressure cuff size, and it would be important to
of which there is a compensatory pause,
ensure that readings were taken with
indicating that they are ventricular in origin.
appropriate equipment.

[ Q: 6 ] MasterClass Part2
[ Q: 5 ] MasterClass Part2
(2010) - Cardiology
(2010) - Cardiology
A 35-year-old man presents with a history of
A 62-year-old man presents with chest pain.
collapse and transient loss of consciousness for
a few seconds
after running
upstairs. He has
a 10-year
history of chest
pains on
exertion and
presyncope. On
examination he
has an ejection
systolic murmur
at the left
sternal edge with a normal aortic closure sound
What TWO features does his ECG show (see and no radiation to the carotids. His ECG
image)? demonstrated left ventricular hypertrophy with
A. Posterior myocardial infarction inferolateral ST segment/T wave changes. His
echocardiogram is shown (see image).
B. Atrial fibrillation
C. Anterior myocardial infarction
D. Right axis deviation
E. Unstable angina
F. Ventricular ectopic beats
G. Inferior myocardial infarction
H. Atrial ectopic beats
I. Left axis deviation
J. Left bundle branch block. What is the most likely cause of his collapse?
A. Bicuspid aortic valve
Answer & Comments
B. Atrioseptal defect
Correct answer: CF C. Acute myocardial infarction
D. Hypertrophic obstructive
This ECG shows the classical appearances of
cardiomyopathy
acute myocardial infarction with ST segment

Dr. Khalid Yusuf El-Zohry – Sohag Teaching Hospital (01118391123) Page | 103
El-Zohry MRCP Questions Bank (Part 2) – Medical Masterclass 2010

E. Coarctation of the aorta. E. Thrombolysis should be avoided because


she may require a temporary pacing wire.
Answer & Comments
Answer & Comments
Correct answer: D
Correct answer: C
20-30% of patients with hypertrophic
cardiomyopathy have dynamic left ventricular The top priority is to achieve myocardial
outflow tract obstruction caused by systolic reperfusion. The presence of chest pain and ST
contact between the anterior mitral valve segment elevation, despite Q waves, on ECG
leaflet and the interventricular septum. indicate that thrombolysis is needed
immediately.
Clinically the presence of left ventricular
hypertrophy and an ejection systolic murmur at
the left sternal edge with a normal aortic [ Q: 8 ] MasterClass Part2
closure and no radiation to the neck should (2010) - Cardiology
suggest the diagnosis, particularly if associated
with mitral regurgitation. The murmur is A 28-year-old woman is sent to the Medical
characteristically increased by manoeuvres that Assessment Unit with pain in the left side of her
reduce afterload or preload. chest. The pain is sharp, exacerbated by
twisting, taking a deep breath, or coughing. She
The M-mode echocardiogram demonstrates a is tender over the left 5th and 6th ribs, but
parasternal long-axis view of the left ventricular examination is otherwise entirely normal. Her
with asymmetric septal hypertrophy and oxygen saturation (breathing air) by pulse
contact between the mitral leaflet and septum oximetry is 99%. She says that the pain came on
in systole. shortly after she had to haul her misbehaving
three-year-old daughter out of a shop.
[ Q: 7 ] MasterClass Part2
What is the most likely cause of her chest pain?
(2010) - Cardiology
A. Pulmonary embolism
A 70 year old woman presents with 8 hours of
chest pain. Her pulse rate is 40/minute and B. Pneumothorax
blood pressure 105/85. The ECG shows C. Musculoskeletal
complete heart block, ST segment elevation and
D. Pneumonia
Q waves in leads II, III and AVF.
E. Pericarditis.
Which of the following statements is correct?
A. Atropine should be given immediately. Answer & Comments
B. An isoprenaline infusion should be set up Correct answer: C
immediately.
C. Thrombolysis should be given The history and physical findings mean that
immediately. musculoskeletal pain is much the most likely
cause for her presentation, but pneumothorax
D. Thrombolysis should be avoided because
and pulmonary embolism would be reasonable
she has completed her myocardial
considerations, although both would seem
infarction.
unlikely.

Dr. Khalid Yusuf El-Zohry – Sohag Teaching Hospital (01118391123) Page | 104
El-Zohry MRCP Questions Bank (Part 2) – Medical Masterclass 2010

Pneumothorax is best excluded by chest dissection, and in type B dissection if there is


radiography. evidence of ongoing dissection with organ or
limb ischaemia.
If blood D-dimer concentration is not elevated,
then the likelihood of pulmonary embolism is
negligible in a patient with low pre-test [ Q: 10 ] MasterClass Part2
probability of this diagnosis. Hence, if D-dimers (2010) - Cardiology
are not elevated in this patient, she can be
reassured and discharged (with analgesic A 52-year-old man is on an ECG monitor
medication if required) without recourse to following an acute myocardial infarct the
investigation such as ventilation / perfusion previous day. Whilst you are on the ward round
scanning or CT pulmonary angiography. he suddenly loses consciousness and the nurse
cannot find a pulse. The ECG monitor shows a
broad complex tachycardia.
[ Q: 9 ] MasterClass Part2
(2010) - Cardiology After checking it is safe to approach, your first
action should be to:
A 32-year-old athlete presents with severe
interscapular pain after training. He is of slim A. start chest compressions.
build and tall (210 cm). He has a sinus B. defibrillate at 360 joules.
tachycardia and his BP is 180/100 mmHg with
C. give a precordial thump.
no deficit between his right and left arm. There
are no murmurs and all his peripheral pulses D. perform synchronised cardioversion at
are palpable. The rest of his physical 200 joules.
examination is normal. A CXR shows widened
E. give lignocaine (lidocaine) 100 mg IV.
mediastinum and a CT confirms an aortic
dissection distal to the left subclavian artery
Answer & Comments
that does not involve the aortic arch.
Correct answer: C
Which of the following is the most appropriate
intervention?
Pulseless ventricular tachycardia is the likeliest
A. Intravenous calcium antagonist diagnosis. This should be treated exactly the
B. Intravenous labetalol same as ventricular fibrillation. If the arrest is
monitored, a precordial thump is indicated as it
C. Urgent cardiothoracic surgical referral
may terminate the VT. If this is unsuccessful,
D. Transoesophageal echocardiography the patient should be defibrillated a soon as
possible.
E. Oral angiotensin-converting enzyme
inhibitor
[ Q: 11 ] MasterClass Part2
Answer & Comments (2010) - Cardiology
Correct answer: B A 50-year-old woman presents with shortness
of breath and palpitations. Diagnostic cardiac
This man has a type B dissection and so the catheterization is performed.
main treatment is to control the BP. A beta-
blocker would be the first-choice agent as this
reduces the shear wall stress of the aorta.
Cardiothoracic referral is indicated in type A
Dr. Khalid Yusuf El-Zohry – Sohag Teaching Hospital (01118391123) Page | 105
El-Zohry MRCP Questions Bank (Part 2) – Medical Masterclass 2010

[ Q: 12 ] MasterClass
Part2 (2010) -
Cardiology
A 25-year-old woman is referred
because during a routine
antenatal visit at 32 weeks of
pregnancy the obstetrician hears
a systolic murmur. She has no
cardiac symptoms.

Which are the TWO most likely


causes of her murmur?
A. Aortic stenosis
From the information given in the table, what is
the most likely complete diagnosis? B. Ventricular septal defect

A. ASD C. Aortic regurgitation with flow murmur

B. ASD and aortic stenosis D. Atrial septal defect

C. Fallots tetralogy E. Triscuspid regurgitation

D. Sub-valvular aortic membrane F. Peripartum cardiomyopathy

E. VSD and aortic stenosis. G. Hypertrophic obstructive


cardiomyopathy
Answer & Comments H. Aortic sclerosis

Correct answer: B I. Innocent systolic murmur


J. Mitral valve prolapse.
The question asks for the complete diagnosis.
The pressure data demonstrates a significant Answer & Comments
difference in pressure from LV to aorta (45
mmHg). This means that there is at least Correct answer: IJ
moderate aortic stenosis (from the data it is not
possible to determine whether this is occurring Given that the woman is well, it is almost
at the level of the valve or from a sub-valvular certain that the murmur has an innocent cause,
membrane). The right heart pressures (RA, RV, being due to the hyperdynamic circulation of
PA) are slightly elevated, whereas the wedge pregnancy. It is also possible that the
pressure (effectively left atrial pressure) is obstetrician has heard a venous hum or a
normal. The aortic saturation is respectable at mammary soufflé. Mitral valve prolapse would
95% and therefore excludes the diagnosis of be much the commonest of the ‘cardiac lesions’
Fallots. There is a step-up in saturation at the listed in a 25-year-old woman.
level of the RA, thereby confirming the
The presence of symptoms should raise the
presence of an ASD. Although there is a minor
suspicion of significant pathology, but it is
increase from RA to RV, this is not significant
important to remember that some degree of
and relates to the mixing of blood in the RA and
weakness, exertional dyspnoea, dizziness and
the exact point of sampling of RA blood.
peripheral oedema are common during
pregnancy. It is therefore very important to
Dr. Khalid Yusuf El-Zohry – Sohag Teaching Hospital (01118391123) Page | 106
El-Zohry MRCP Questions Bank (Part 2) – Medical Masterclass 2010

gauge the severity of any symptoms in relation care unit with central chest pain,
to the stage of pregnancy, and a great deal of breathlessness and a normal ECG. Cardiac
unnecessary anxiety can be generated by enzymes are normal. He has an exercise
doctors who fail to do so. tolerance test which demonstrates no ECG
changes but he gets chest pain at peak exercise.
The investigation of choice to exclude a
Angiography of his right coronary artery is
significant cardiac lesion is echocardiography.
normal. The angiogram of his left coronary
artery is shown (see image).
[ Q: 13 ] MasterClass Part2
(2010) - Cardiology
A 78-year-old woman is admitted with a
massive pulmonary embolism. Her 12 lead ECG
shows a sinus tachycardia at 124 beats per
minute with an S1 Q3 T3 pattern.

Which of the following statements is correct?


A. Most patients with a pulmonary
embolism have an S1 Q3 T3 pattern on
their ECG.
B. Most patients with a pulmonary
embolism have right bundle branch block.
C. Most patients with pulmonary embolism
have atrial fibrillation.
D. Most patients with pulmonary embolism
have atrial fibrillation with right bundle Which of the following would be the most
branch block. appropriate management?
E. The most frequent ECG abnormality is T A. Increased anti-anginal medication
wave inversion in the anterior leads.
B. Coronary artery bypass grafting
C. Percutaneous transluminal angionplasty
Answer & Comments
(PTCA) and stent deployment
Correct answer: E
D. Off-pump coronary artery bypass grafting
E. PTCA alone.
The most frequent ECG abnormality is T wave
inversion in the anterior leads especially V1 to
V4. These are probably produced by reciprocal Answer & Comments
changes due to compression of the right
Correct answer: C
coronary artery by the right ventricle as a result
of pressure overload.
This man's coronary angiogram demonstrates a
severe stenosis of the proximal left anterior
[ Q: 14 ] MasterClass Part2 descending artery. This is certainly amenable to
(2010) - Cardiology PTCA. Where possible if the anatomy allows,
stent deployment offers less
A 45-year-old man presents to the coronary
restenosis/occlusion rates than PTCA alone.

Dr. Khalid Yusuf El-Zohry – Sohag Teaching Hospital (01118391123) Page | 107
El-Zohry MRCP Questions Bank (Part 2) – Medical Masterclass 2010

Coronary artery bypass grafting for single vessel [ Q: 16 ] MasterClass Part2


coronary artery disease is generally only (2010) - Cardiology
considered if PTCA is not possible or has failed.
A 63-year-old man with Marfan's syndrome
Off-pump coronary artery bypass grafting of the
presents with chest pain and is found to have
left anterior descending artery is an evolving
an acute aortic dissection.
concept utilizing equipment that holds the
beating heart steadily enough to suture a graft Which TWO of the following cardiac conditions
into position. are associated with Marfan's?
A. Atrial myxoma
[ Q: 15 ] MasterClass Part2
B. Mitral stenosis
(2010) - Cardiology
C. Pulmonary regurgitation
A 58-year-old woman is admitted acutely with
pulmonary oedema and hypotension. She had a D. Ventricular tachycardia
mitral valve replacement four years previously E. Atrial septal defect
for mitral regurgitation. Clinical examination
F. Ventricular septal defect
reveals a mitral diastolic murmur and
pulmonary oedema. Her biochemistry and full G. Cardiac amyloid
blood count is normal. Her INR is 1.8.
H. Sarcoidosis
Echocardiography demonstrates thrombus
around the valve. I. Mitral valve prolapse
J. Aortic stenosis.
What is the best treatment?
A. Intravenous heparin Answer & Comments
B. Thrombolysis
Correct answer: CI
C. Urgent surgical intervention
D. Increased dose of warfarin Cardiac complications of Marfan’s are relatively
common. Usually they are related to aortic
E. Low molecular weight heparin.
involvement. However, Marfan’s is associated
with the following:
Answer & Comments
 Mitral valve prolapse and regurgitation
Correct answer: C  Left ventricular dilatation and cardiac
failure
Mechanical mitral valves are more prone to
 Pulmonary artery dilatation
thrombosis than aortic valves if the INR drops
below the therapeutic level. This is a  Regurgitation of the pulmonary valve.
medical/surgical emergency that can be rapidly
fatal without immediate surgical intervention. [ Q: 17 ] MasterClass Part2
Thrombus in this context cannot be treated
(2010) - Cardiology
with anticoagulants/fibrinolytics.
Anticoagulation is clearly an important A 30-year-old male with a heart murmur and
consideration when considering what type of breathlessness on effort is referred for
valve prosthesis should be considered for an transthoracic echocardiogram (see image).
individual.

Dr. Khalid Yusuf El-Zohry – Sohag Teaching Hospital (01118391123) Page | 108
El-Zohry MRCP Questions Bank (Part 2) – Medical Masterclass 2010

septal hypertrophy
and a resting outflow
tract gradient of
55mmHg.
The ECG may be
normal in 25% cases
and morphological
expression of the
disease may not be
completed until the
end of puberty.
Twenty-five per cent
of all cases occur in
those over 75.
Sudden death occurs
in 1% cases and those
considered high risk should be referred for an
Which of the following is true of his conditio?
implantable defibrillator.
A. Sudden death is very common.
Exercise testing is extremely useful at
B. Exercise testing is contraindicated. identifying those patients with dynamic outflow
C. Atenolol may help his symptoms. tract gradients who would benefit from more
aggressive therapy.
D. Frusemide will help his shortness of
breath.
[ Q: 18 ] MasterClass Part2
E. The ECG is always abnormal.
(2010) - Cardiology
F. It is very rare in elderly patients over 75
years of age. Which of the following is not associated with
G. The condition can be confidently cocaine abuse?
diagnosed in patients over 10 years pf A. Hypotension
age.
B. Contraction band necrosis
H. Ischaemia may occur with a normal
C. Dilated cardiomyopathy
coronary arteriogram.
D. Acceleration of atherosclerosis
I. Endocarditis prophylaxis is not required.
E. Aortic dissection.
J. Prophylactic amioderone is mandatory to
prevent sudden death.
Answer & Comments

Answer & Comments Correct answer: A

Correct answer: CH
Cocaine is associated with both acute and long-
term cardiovascular disorders. Most commonly
The above patient has hypertrophic
it is associated with myocardial ischaemia
cardiomyopathy with marked asymmetrical
secondary to coronary spasm. Coronary spasm
usually responds to nitrates, calcium

Dr. Khalid Yusuf El-Zohry – Sohag Teaching Hospital (01118391123) Page | 109
El-Zohry MRCP Questions Bank (Part 2) – Medical Masterclass 2010

antagonists or alpha blockers. Beta-blockers Acute myocardial infarction rarely presents


should be used with caution. with syncope with no ECG changes and ongoing
hypoxia would not be explained with a 'normal'
[ Q: 19 ] MasterClass Part2 chest radiograph.

(2010) - Cardiology Chronic obstructive pulmonary disease (COPD)


is also unlikely given history of syncope and
A 59-year-old man with moderate chronic little evidence of severe airways obstruction.
obstructive lung disease is admitted breathless
following an episode of syncope while
shopping. There is no previous history of [ Q: 20 ] MasterClass Part2
sycope. Past history includes multiple stab (2010) - Cardiology
ligations for bilateral varicose veins four weeks
A 59-year-old man is admitted breathless
previously and open cholecystectomy 10 years
following an episode of syncope while
before. On examination he is breathless at rest,
shopping. There is no previous history of
apyrexial, pulse 104, BP 110/60. Heart sounds
syncope. Past history includes multiple stab
normal. Chest examination reveals a few
ligations for bilateral varicose veins 4 weeks
scattered wheezes. Neurological examination
previously, mild asthma and borderline
was normal ECG sinus tachycardia, inverted T
hypertension (not on treatment). He has
waves in V1-V3. Chest X-ray normal. Oxygen
received 500 mL of volume expansion in the
saturations 92% on 40% oxygen. PEFR
Emergency Department.
290L/min.
On examination he is breathless at rest,
What is the most likely diagnosis? apyrexial, pulse 104 bpm (regular) and BP 90/50
mmHg. His heart sounds are normal. Chest
A. Acute pulmonary oedema
examination reveals a few scattered wheezes.
B. Acute exacerbation of chronic obstructive ECG shows sinus tachycardia with inverted T
pulmonary disease waves in leads V1-V3. CXR is normal. Oxygen
C. Pulmonary embolism saturation is 92% on 40% oxygen. Peak
expiratory flow rate is 300 L/min.
D. Acute myocardial infarction
E. Pneumocystis. What investigation would be most helpful in
directing his immediate treatment?
Answer & Comments A. CT pulmonary angiogram

Correct answer: C B. Serum D-dimer


C. Transoesophageal echocardiography
The correct answer is pulmonary
D. Serum Troponin
thromboembolism. This gentleman presents
with breathlessness, which is the commonest E. Ultrasound of leg veins (bilateral)
symptom associated with pulmonary embolism,
and syncope, which is a less common but often Answer & Comments
poorly recognised presenting symptom. He is
Correct answer: A
hypoxic with oxygen saturations of only 92% on
40% oxygen and has non-specific but
recognised ECG changes. His recent varicose This clinical presentation is highly suggestive of
vein surgery also gives him an additional risk life-threatening pulmonary embolism. He
factor. remains hypotensive (he normally tends to be

Dr. Khalid Yusuf El-Zohry – Sohag Teaching Hospital (01118391123) Page | 110
El-Zohry MRCP Questions Bank (Part 2) – Medical Masterclass 2010

hypertensive) and tachycardic. The next step of Bruce protocol exercise test (bicycle or
must be to confirm the presence of proximal treadmill), off anti-anginal medication for 48
pulmonary embolism and to consider hours. During the test they should remain
thrombolysis (surgery 4 weeks previously for symptom free, with no evidence of
varicose vein stab ligation is not a hypotension, ventricular tachycardia or
contraindication). CT pulmonary angiography significant ST segment shift (i.e. greater than
would be the most desirable investigation. 2mm horizontal or down-sloping ST segment
Whilst transthoracic echocardiography would depression).
be of value in demonstrating a dilated right
Coronary angiography is not required but, if it
heart, this is not specific for pulmonary embolic
has been undertaken, re-licensing will not be
disease and can be seen in other instances, for
permitted if left ventricular ejection fraction is <
example severe pneumonia.
40%, or if there is a significant, untreated left
Transoesophageal echocardiography would main stem stenosis equal to or > 50%, and/or
carry substantial risk in this haemodynamically proximal left anterior descending artery
compromised man. An elevated serum troponin stenosis equal to or greater than 75%.
(right ventricular ischaemia) in the context of
pulmonary embolism indicates a worse [ Q: 22 ] MasterClass Part2
prognosis and some would suggest that this
(2010) - Cardiology
would support administration of thrombolytic
therapy. A patient with primary pulmonary hypertension
is found to have a mean right atrial pressure of
[ Q: 21 ] MasterClass Part2 20 mmHg and a cardiac output of 1.5 L per min.
The mean pulmonary artery pressure is 80
(2010) - Cardiology
mmHg. There is no vasoactive response with
intravenous prostacyclin. He is already on
An HGV (group 2 entitlement) driver should not
warfarin.
drive post myocardial infarction (MI) until which
of the following conditions has been met?
Which is the most appropriate first-line
A. Four weeks post infarction. treatment:
B. Angiography has been undertaken. A. Ace-inhibitor
C. Completion of 3 stages of Bruce protocol B. Oral calcium antagonist
exercise test 6 weeks post-MI.
C. Heart-Lung transplantation
D. Completion of 3 stages of Modified Bruce
D. Prostacyclin treatment
protocol exercise test 4 weeks post-MI.
E. None of the above.
E. Six months free of symptoms.

Answer & Comments


Answer & Comments
Correct answer: D
Correct answer: C

This patient has severe pulmonary hypertension


Following MI, group 2 entitlement holders are
with poor prognosis untreated. Calcium
disqualified from driving for at least 6 weeks.
antagonists should only be given to those who
Re-licensing may then be permitted if the
show reactivity to vasodilator challenge and
following exercise test requirements are met:
have adequate cardiac output (>2.0 L/min).
the driver should be able to complete 3 stages

Dr. Khalid Yusuf El-Zohry – Sohag Teaching Hospital (01118391123) Page | 111
El-Zohry MRCP Questions Bank (Part 2) – Medical Masterclass 2010

Even those patients with severe disease who  Female sex is a powerful predictor of
fail to show vasoreactivity on provocation the risk of torsade de pointes in
testing appear to benefit from prostacyclin patients with congenital and acquired
treatment so this is offered to all such patients long QT intervals.
initially. Only those who do not benefit from  Sotalol can prolong the QT interval and
prostacyclin or continue to deteriorate are
may cause torsade de pointe.
considered for transplantation.
 The risk of torsade de pointe is NOT a
linear function of the QT interval.
[ Q: 23 ] MasterClass Part2
(2010) - Cardiology
[ Q: 24 ] MasterClass Part2
A 28-year-old woman is referred by her general (2010) - Cardiology
practitioner with a history of recurrent syncopal
episodes dating back to her early teens. She is A 48-year-old woman is referred for a
not on any regular medication. A 12-lead cardiological opinion because of palpitations.
Electrocardiogram taken in his surgery shortly There are no other associated symptoms, but
after her last blackout shows polymorphic she does have a family history of sudden
ventricular tachycardia. cardiac death. Physical examination is normal,
as is her ECG and echocardiogram. A 24 hour
Which one of the following statements is ECG demonstrates frequent ventricular ectopic
correct? activity.
A. It is unlikely that she has a congenital
Which of the following is the likely diagnosis?
long QT interval.
A. Brugada syndrome
B. Hypokalaemia is not a risk factor for
polymorphic ventricular tacycardia. B. Long QT syndrome

C. Female sex is not a risk factor for C. Hypertrophic obstructive cardiomyopathy


polymorphic ventricular tachycardia. D. Arrhythmogenic right ventricular
D. Sotalol may cause torsade de pointes. dysplasia

E. The risk of torsade de pointe is a linear E. None of the above.


function of the QT interval.
Answer & Comments
Answer & Comments
Correct answer: E
Correct answer: D
Palpitations are a very common presenting
Although torsade de pointe can occur in many feature and reassurance is generally all that is
settings it is usually seen in patients with one of needed in the context of a normal ECG and
the congenital long QT syndromes or in echo. In this woman the most concerning
association with drug therapy. Given that she is feature is the history of sudden cardiac death in
not on medication a congenital long QT the family - more information regarding this is
syndrome needs to be considered. Some required to assess her risk. Conditions listed A
further points to be considered are: to D will generally have abnormal ECGs, and in
the absence of a specific family history of them
 Hypokalaemia increases the risk of
it is likely that she will need no further
torsade de pointe.
investigation.

Dr. Khalid Yusuf El-Zohry – Sohag Teaching Hospital (01118391123) Page | 112
El-Zohry MRCP Questions Bank (Part 2) – Medical Masterclass 2010

[ Q: 25 ] MasterClass Part2 ventricular outflow tract and generally carries


(2010) - Cardiology an excellent prognosis. It is best treated with
radiofrequency ablation. This is in contrast to
A 45-year-old man is referred by his general ischaemic ventricular tachycardia which carries
practitioner with palpitations. He has no other a very poor prognosis unless treated
associated symptons and specifically he is not appropriately. In the context of impaired left
presyncopal. Holter monitoring has ventricular function this invariably means with
demonstrated short non-sutained runs of a an implantable cardioverter defibrillator (ICD).
monomorphic broad complex tachycardia.

Which TWO of the following are the most likely [ Q: 26 ] MasterClass Part2
arrhythmias? (2010) - Cardiology
A. Sinus tachycardia A 45-year-old lady is seen in cardiac outpatients
B. Atrial fibrillation with intermittent rate with a blood pressure of 160/80. Consistently it
associated bundle branch block has been greater than 145>85. However
ambulatory monitoring demonstrated a mean
C. Right ventricular outflow tract BP of 130/75.
tachycardia
Which of the following are inappropriate
D. Atrial flutter with one to one conduction
indications for ambulatory blood pressure
E. Wolff-Parkinson-White syndrome monitoring?
F. Ischaemic ventricular tachycardia A. Deciding diagnosis in borderline
G. Atrioventricular nodal reentry hypertension
tachycardia B. Making a diagnosis of left ventricular
H. Ventricular Fibrillation failure

I. Torsades de pointes C. Identification of nocturnal hypertension

J. Atrioventricular reentry tachycardia. D. Diagnosing Phaeochromocytoma


E. Hypertensive patients resistant to
Answer & Comments treatment

Correct answer: CF F. Hypertension of pregnancy


G. Diagnosis of hypotension
Whenever you approach a patient with a broad
H. Monitoring hypertensive tretament
complex tachycardia it is always safest to
presume that they have ventricular tachycardia I. Elderly patients
until proven otherwise, and indeed this is more J. Exclusion of white coat hypertension.
likely than one of the other possibilities above,
which may produce similar traces on a Holter
Answer & Comments
monitor.
Correct answer: BD
Patients are not always syncopal with
ventricular tachycardia.
A diagnosis of "white coat hypertension" was
Right ventricular outflow tract tachycardia
made in this lady and thus she did not require
usually presents with just palpitations. It occurs
antihypertensive treatment. This is probably
as a result of a triggered focus in the right
the strongest indication for ambulatory BP

Dr. Khalid Yusuf El-Zohry – Sohag Teaching Hospital (01118391123) Page | 113
El-Zohry MRCP Questions Bank (Part 2) – Medical Masterclass 2010

monitoring but the indications are broad. A I. It should not be used in patients with very
diagnosis of left ventricular failure will not be poor left ventriuclar function.
made on BP alone. Equally a
J. A normal image indicates risk of of
phaeochromocytoma may have transient rises
coronary events of 2-3% per year.
in BP but as most ambulatory monitors will only
measure once or twice an hour it is unlikely to
Answer & Comments
be helpful in this situation
Correct answer: DF
[ Q: 27 ] MasterClass Part2
(2010) - Cardiology Radionucleotide myocardial perfusion imaging
is very useful in assessing the degree of
A 68-year-old man with a past history of coronary obstruction in patients with suspected
myocardial infarction, from which he made a coronary artery disease. It provides diagnostic
good recovery, now presents with chest pain and prognostic data. Patients can be stressed
that might be due to cardiac ischaemia, but the using conventional treadmill or pharmacological
history is not typical. He would not be able to stress with agents such as adenosine or
perform a Bruce protocol treadmill test because dobutamine. Adenosine should be avoided in
of severe osteoarthritis of his knees and it is asthmatics. A normal image indicates risk of
decided to perform radionucleotide myocardial coronary events of <1% per year.
perfusion imaging.
The test can also be helpful in targeting
Which TWO of the following statements about intervention following angiography and in the
such imaging are correct? detection of hibernating myocardium in
patients with poor left ventricular function.
A. It should be avoided in asthmatic
patients.
[ Q: 28 ] MasterClass Part2
B. It should be avoided in patients with
aortic stenosis. (2010) - Cardiology

C. It is useful in determining the functional A 60-year-old gentleman is being investigated


severity of coronary lesions but does not for chest pain and undergoes exercise tolerance
predict prognosis. testing.

D. It can be used to risk stratify patients Which feature is associated with a worse
undergoing surgical procedures. prognosis?
E. It is not helpful in patients unable to A. Increased metabolic rate
physically exercise to maximal capacity.
B. Ventricular tachycardia
F. It is readily possible to differentiate
C. Increase in blood pressure with exercise
between reversible ischaemia and
infarcted areas of myocardium. D. Rapid resolution of heart rate in recovery
G. It is of no value following coronary E. Absence of symptoms during exercise.
angiography.
H. False positives are seen more commonly Answer & Comments
in middle aged women as compared to Correct answer: B
conventional exercise testing

Dr. Khalid Yusuf El-Zohry – Sohag Teaching Hospital (01118391123) Page | 114
El-Zohry MRCP Questions Bank (Part 2) – Medical Masterclass 2010

Exercise testing has long been an established therapy with primary angioplasty (n=2606
method for identifying patients with underlying patients) demonstrated that primary
coronary disease. Apart from changes in the ST angioplasty was superior to thrombolytic
segments other features have been associated therapy in terms of in-hospital mortality, non-
with underlying disease, including: fatal reinfarction, long-term survival and a
 lower incidence of stroke. The largest trial
ventricular arrhythmias
(PAMI-1) showed greatest benefit in high risk
 inadequate blood pressure response patients (cardiogenic shock, elderly patients,
 inadequate heart rate response anterior wall myocardial infarctions). The under
use of primary angioplasty in the UK is largely
 angina due to lack of resources, although most regard
 poor MET response. it as the optimal treatment strategy.
GUSTO-1 demonstrated that Streptokinase was
[ Q: 29 ] MasterClass Part2 inferior to tPA when used as an accelerated 90-
(2010) - Cardiology minute bolus infusion along with iv heparin.

A 60-year-old man presents to the Emergency An analysis of pooled data from 28 trials
Department with a 1.5 hour history of chest showed an average 28% mortality reduction
pain, a heart rate of 68 / min, blood pressure with use of beta blockers in acute myocardial
140/85 mmHg, and no signs of cardiac failure. infarction. The ISIS-1 trial first used intravenous
An ECG shows ST elevation in leads V1-V3. atenolol (5mg) in AMI with a favourable
mortality benefit, but this treatment is widely
Which of the following statements is correct? underused in current practice.

A. Treatment with a thrombolytic is


preferable to primary coronary [ Q: 30 ] MasterClass Part2
angioplasty. (2010) - Cardiology
B. Streptokinase is the thrombolytic agent A 20-year-old female student presents with
of choice. central chest pain after four days of a ‘flu-like
C. There is randomised controlled trial illness.
evidence supporting use of warfarin
The most likely diagnosis is:
therapy.
A. Acute viral pericarditis
D. There is randomised controlled trial
evidence supporting use of intravenous B. Gastro-oesophageal reflux
beta blocker therapy. C. Acute myocardial infarction
E. Intravenous heparin is mandatory D. Systemic lupus erythematosus
whatever treatment strategy is used.
E. Pulmonary embolism.
Answer & Comments
Answer & Comments
Correct answer: D
Correct answer: A
In the early 1990s, controversy still existed
about the use of angioplasty for the treatment The combination of a young patient and a ‘flu-
of AMI. A meta-analysis in 1997 of 10 like illness make acute viral pericarditis the
randomized trials comparing thrombolytic most likely diagnosis in this case. The chest pain

Dr. Khalid Yusuf El-Zohry – Sohag Teaching Hospital (01118391123) Page | 115
El-Zohry MRCP Questions Bank (Part 2) – Medical Masterclass 2010

of pericarditis can be indistinguishable from [ Q: 32 ] MasterClass Part2


that of myocardial infarction, excepting that (2010) - Cardiology
sitting forward often eases it.
(2) A 37-year-old woman is seen in outpatients
The key physical sign to elicit would be a
with frequent episodes of palpitations. She tells
pericardial rub, and the key initial investigation
you that she has had them fortwo years, and
would be the ECG, looking for widespread ST
that she gets chest discomfort during the
segment elevation, concave upwards.
attacks. Her ECG is shown (see image).

What is the diagnosis?


[ Q: 31 ] MasterClass Part2 A. Anxiety-producing sinus tachycardia
(2010) - Cardiology B. Wolf-Parkinson-White syndrome
A 32-year-old woman who is 22 weeks' C. Lown-Ganong-Levine syndrome
pregnant is admitted to the maternity ward and
found to be hypertensive with BP 180/100 D. Ventricular tachycardia
mmHg. This is her first pregnancy and it is E. Left bundle branch block.
otherwise uncomplicated. She has no other
relevant medical history and her urinalysis is Answer & Comments
normal, as is renal and liver function.
Correct answer: B
Which of the following should not be used to
control her blood pressure? Wolf-Parkinson-White is the syndrome of pre-
A. Hydralazine excitation secondary to antegrade conduction
down an accessory pathway from atrium to
B. Methyldopa
ventricle. This is characterized on the 12-lead
C. Labetalol surface ECG by a short PR interval, a delta wave
D. Ramipril and broad complexes (as a result of the delta
wave). The accessory pathway can be in any
E. Amlodipine location around the atrioventricular valves, i.e.
mitral or tricuspid valves. Using various
Answer & Comments algorithms it is possible to accurately locate the
position of the accessory pathway.
Correct answer: D
In this patient's case there is a dominantly
upright QRS deflection in the right praecordial

Dr. Khalid Yusuf El-Zohry – Sohag Teaching Hospital (01118391123) Page | 116
El-Zohry MRCP Questions Bank (Part 2) – Medical Masterclass 2010

leads, resulting in tall R waves in leads V1 and I. Recurrent ipsilateral DVT can be confused
V2. This is the picture of Type A WPW, where clinically with the post-thrombotic
pre-excitation is usually of the left ventricle. syndrome
Lown-Ganon-Levine syndrome differs from J. Magnetic resonance venography may
Wolf-Parkinson-White syndrome in that it has a prove useful for diagnosis in future
short PR interval without a delta wave. This is
thought to be due to accessory conduction Answer & Comments
tissue from the atria to the atrioventricular
node. Whilst the complexes are broad, this is Correct answer: CE
clearly sinus rhythm and not ventriuclar
tachcardia. The increased risk of thrombosis persists for
several months after surgery. About half of
[ Q: 33 ] MasterClass Part2 patients with a first spontaneous thrombosis
are found to have an abnormality in the
(2010) - Cardiology
coagulation system.
You suspect that a 61-year-old woman with a
painful swollen leg, three weeks after femoral [ Q: 34 ] MasterClass Part2
hernia repair, has deep vein thrombosis.
(2010) - Cardiology
Which TWO of the following statements are A 57-year-old man presents with dyspnoea and
NOT correct? orthopnea. He has an early diastolic murmur.
A. Virchow's pathophysiological triad is still See image for an illustration of a dilated aorta.
valid today
B. Patients with early oedema are most
likely to have residual thrombosis
C. Risk for thrombosis after surgery returns
to normal within 2 weeks provided the
patient has regained full mobility
D. After a first spontaneous DVT the annual
likelihood of recurrence is 5 -15%
E. Abnormalities in the coagulation system
can be detected in around one quarter of
patients with a first spontaneous
thrombosis
F. Low molecular weight heparin is effective
and safe in both medical and surgical
patients
G. The gold standard for diagnosis of DVT Which of the following does not cause an
remains contrast venography aortopathy?

H. Ultrasonography is less sensitive and less A. Marfan's syndrome


specific than venograph B. Ehlers Danlos syndrome
C. Syphilis infection

Dr. Khalid Yusuf El-Zohry – Sohag Teaching Hospital (01118391123) Page | 117
El-Zohry MRCP Questions Bank (Part 2) – Medical Masterclass 2010

D. Atherosclerosis peaks at 6-8 hours. Values return to normal 20-


36 hours.
E. Duchenne's muscular dystrophy.
CK starts to rise 4-6 hours with the CK-MB
Answer & Comments fraction 3-4 hours.

Correct answer: E Troponin I and T start to rise 4-6 hours and peak
12 hours after myocardial infarction. A new
form, Troponin L, may be detected earlier.
The image is an aortogram demonstrating a
dilated ascending thoracic aorta. This most Myoglobin and CK are raised in both cardiac
commonly arises from the process of cystic muscle and skeletal muscle damage (striated
medial degeneration from conditions such as muscle).
Ehlers Danlo’s syndrome or Marfan’s syndrome.
These aneurysms often involve the aortic root. [ Q: 36 ] MasterClass Part2
Dilatation of the root causes aortic (2010) - Cardiology
regurgitation. The treatment of choice is usually
aortic root and valve replacement. A 60-year-old man is given streptokinase for an
anterior myocardial infarction (MI) six hours
Atherosclerosis is a common cause whilst
after the onset of pain. Ninety minutes post-
syphilitic infection is becoming considerably
thrombolysis he continues to have ischaemic
less common.
sounding chest pain with persistent anterior ST
Duchenne’s muscular dystrophy is associated elevation.
with a cardiomyopathy rather than a vasculitic
process. Which of the following is the most appropriate
treatment in this situation:
[ Q: 35 ] MasterClass Part2 A. Start intravenous heparin
(2010) - Cardiology B. Give further thrombolysis with
streptokinase
A 65-year-old man presented with severe chest
pain. A diagnosis of acute myocardial infarction C. Give further thrombolysis with tPA
was made.
D. Emergency angiography and target PTCA
or bypass surgery
Which serum marker would be elevated first?
E. Start tirofiban
A. Troponin I
B. Troponin T
Answer & Comments
C. Creatine kinase (CK)
Correct answer: D
D. CK-MB fraction
E. Myoglobin The best way to manage patients with failed
thrombolysis is with rescue percutaneous
Answer & Comments transluminal coronary angioplasty (PTCA) in
patients still within 12 hours of presentation
Correct answer: E (REACT and other studies).

Myoglobin is found in striated muscle. It begins


to rise 2 hours after myocardial infarction and

Dr. Khalid Yusuf El-Zohry – Sohag Teaching Hospital (01118391123) Page | 118
El-Zohry MRCP Questions Bank (Part 2) – Medical Masterclass 2010

[ Q: 37 ] MasterClass Part2 Patients with significant left stem main disease


(2010) - Cardiology (>50% occlusion) or significant three vessel
disease with depressed LV function have
A 76-year-old woman is admitted to the increase survival if they undergo CABG,
coronary care unit with unstable angina. Her 12 although patients older than 75 yrs were not
lead electrocardiogram shows ST depression of included in the trials that demonstrated such
more than 2mm in the anterior leads. Clinical benefit.
examination shows evidence of acute heart
failure. Despite medical treatment she has
[ Q: 38 ] MasterClass Part2
chest pain two hours after admission.
(2010) - Cardiology
Which of the following is correct?
An 83-year-old lady is admitted having been
A. Morphine should not be given to relieve found on the floor by her home help. She had
pain if sublingual GTN has been fallen 6 hours prior to admission and thought
ineffective. she may have blacked out, but was not able to
give any more details.
B. Verapamil should be given if she is unable
to tolerate a beta blocker.
Which TWO of the following are correct?
C. Patients with impaired LV function
A. Each year in the community 2/3 of people
(EF<50%) should not undergo coronary
over 65 years have at least one fall/year.
artery bypass grafting (CABG).
B. Age-related changes affecting vision,
D. Cardiac catheterisation is used to assess
vestibular system and proprioception
patients for CABG only.
have no overall effect on balance.
E. Patients with significant left main stem
C. Among older people that fall serious
disease have increased survival with
injury is more likely in those with
CABG.
cognitive impairment.
D. A prolonged lie time (time on the floor) is
Answer & Comments
not associated with significant morbidity.
Correct answer: E
E. In older patients presenting to A and E
with recurrent falls, carotid
Morphine may be necessary to relieve anginal hypersensitivity is found in less than 5%
pain - age is not a contraindication, although of cases.
aliquots of smaller doses e.g. 2.5mg should be
used to achieve pain control. F. Orthosatic postural hypotension can be
diagnosed if there is a fall in systolic
Calcium channel blockers should not be given blood pressure of 10mmHg.
to patients in pulmonary oedema or with
impaired LV function. G. If there is no history of joint pain,
physical examination of the neck is
Impaired LV function (EF<50%) is not a unnecessary.
contraindication to CABG.
H. There is no evidence that balance training
Cardiac catheterisation informs the choice reduces the incidence of falls in older
between medical treatment, CABG and people.
percutaneous transluminal coronary
angioplasty. I. The Get Up and Go test is useful for
checking gait abnormalities.

Dr. Khalid Yusuf El-Zohry – Sohag Teaching Hospital (01118391123) Page | 119
El-Zohry MRCP Questions Bank (Part 2) – Medical Masterclass 2010

J. There is no evidence that reviewing [ Q: 39 ] MasterClass Part2


medications in older patients prevents (2010) - Cardiology
falls.
You suggest that a 48-year-old woman wears
Answer & Comments compression stockings after a deep vein
thrombosis to prevent the development of post
Correct answer: CI thrombotic syndrome in her leg. She finds them
uncomfortable and does not want to wear
A 1/3 of people aged 65 years and above have them.
at least one fall/year.
You explain that the percentage reduction of
Age-related changes afffecting vision, post thrombotic syndrome in patients treated
proprioception, vestibular input, muscle with compression stockings after a deep vein
strength and joint flexibilty can all compromise thrombosis (DVT) is:
balance.
A. nil
In older patients that fall, serious injury is more
likely in women and in those with cognitive B. about 10%
impairment, a low body mass index, or with C. about 50%
two or more chronic conditions e.g. asthma,
D. about 80%
cancer and diabetes mellitus.
E. about 100%.
A prolonged lie time is associated with
bronchopneumonia, pressure sores,
hypothermia, dehydration or rhabdomyolysis. Answer & Comments

About 1/3 of patients 50 years and over Correct answer: C


presenting to A+E after a fall have evidence of
carotid hypersensitivity. Grade 2, or Grade 3 for active patients,
Orthostatic postural hypotension can be compression stockings should be recommended
diagnosed if lying and standing blood pressures as soon as the swelling is settling in patients
show a sustained postural drop: systolic BP by with a DVT. About half of the patients not using
at least 20mmHg, or diastolic BP by at least stockings develop post thrombotic syndrome
10mmHg, on standing for at least two minutes. including:
 aching and itching,
Examination should include movement of neck
to see if this causes symptoms such as dizziness  swelling and, eventually,
or syncope.  leg ulceration.
There is evidence from seven randomized
controlled trials that balance exercise can
[ Q: 40 ] MasterClass Part2
reduce the number of falls in older people.
(2010) - Cardiology
The Get Up and Go test is very useful for
assessing mobility and checking for gait A 43-year-old male with myotonic dystrophy is
abnormalities. admitted with ventricular tachycardia. This
requires DC cardioversion as he is hypotensive.
Careful review of the type and number of Ultimately he has an implantable cardioverter
medications is an essential element to defibrillator to protect him.
preventing further falls. There is evidence that
reducing medications reduces falls.

Dr. Khalid Yusuf El-Zohry – Sohag Teaching Hospital (01118391123) Page | 120
El-Zohry MRCP Questions Bank (Part 2) – Medical Masterclass 2010

Which of the following is NOT a recognized pacemaker gives to a sensed beat (I=inhibit;
cardiac complication of myotonic dystrophy? T=trigger; D=both). The fourth, usually R (rate
A. Mitral valve prolapse responsive) is for more fancy technologies!

B. Pulmonary stenosis In this case of complete heart block, in order to


maintain AV synchrony, a dual chamber
C. Atrial flutter pacemaker is required (DDD). Assuming the
D. Ischaemic heart disease atrial (p wave) rate is normal the function will
be generally sensing the p wave and then
E. Diastolic heart failure.
pacing the ventricle

Answer & Comments


[ Q: 42 ] MasterClass Part2
Correct answer: B (2010) - Cardiology
A 66-year-old man with a history of
Endomyocardial biopsies frequently
hypertension undergoes coronary artery bypass
demonstrate non-specific changes such as
grafting for triple vessel disease. Twelve hours
fibrofatty infiltration, hypertrophy and
later, shortly after extubation on the ICU, his
myocarditis. All of these may lead to all of the
blood pressure falls to 70/50 mmHg. He is cool
above apart, from pulmonary stenosis.
peripherally, his urine output stops, and
increasing inotropic support does not lead to
[ Q: 41 ] MasterClass Part2 significant improvement. You are called to
(2010) - Cardiology assess him. His ECG does not show any features
to suggest acute myocardial infarction.
A 60-year old female presents with recurrent
syncope; ECG reveals complete heart block. She What is the next most appropriate
is scheduled for a pacemaker. investigation?

Which of the following pacemakers would be A. Urgent cardiac catheterization


most appropriate? B. Re-exploration of surgery
A. AOO C. Transoesophageal echocardiogram
B. DDD D. Ventilation perfusion scan
C. Implantable cardioverter defibrillator E. Serum potassium measurement.
(dual chamber)
D. VVI Answer & Comments
E. VVIR. Correct answer: C

Answer & Comments The most likely explanation of this gentleman’s


Correct answer: B deterioration is cardiac tamponade. Although
trans-thoracic echocardiogram would be of help
it does not exclude the diagnosis. Pericardial
Pacemaker types are identified by a 3 or 4 letter
effusions, particularly in the early post-
code. The first letter = chamber paced
operative period, can be localized and difficult
(A=atrium; V=ventricle; D=both or dual). The
to identify by trans-thoracic echocardiography.
second letter represents which chamber is
An effusion may only occur posteriorly and
sensed. The third is what response the

Dr. Khalid Yusuf El-Zohry – Sohag Teaching Hospital (01118391123) Page | 121
El-Zohry MRCP Questions Bank (Part 2) – Medical Masterclass 2010

therefore the best way to visualize these would I. Paradoxical splitting of the second heart
be through transoesophageal echo. sound
J. Kussmaul's sign
[ Q: 43 ] MasterClass Part2
(2010) - Cardiology Answer & Comments
A 35-year-old man is referred for a Correct answer: CE
transthoracic echocardiogram (see image) after
a murmur is heard by his GP. This patient has mitral valve prolapse (MVP) of

the posterior leaflet with mitral regurgitation.


Which of the following physical signs would be Typical findings include a mid systolic click
expected. followed by a late apical systolic murmur. When
the mitral regurgitation becomes severe the
A. An immediate diastolic murmur
murmur can become pansystolic with a third
B. A machinery murmur heart sound and a displaced hyperdynamic
C. An ejection click apex. The first heart sound is usually soft. Atrial
fibrillation may also occur.
D. A loud first heart sound
E. A late systolic murmur [ Q: 44 ] MasterClass Part2
F. A fourth heart sound (2010) - Cardiology
G. Pulsus alternans A 50-year-old woman is seen in cardiology clinic
H. A wide pulse pressure with progressive breathlessness and ankle

Dr. Khalid Yusuf El-Zohry – Sohag Teaching Hospital (01118391123) Page | 122
El-Zohry MRCP Questions Bank (Part 2) – Medical Masterclass 2010

swelling. Auscultation of her heart reveals a Pressure (mmHg) Oxygen saturation (%)
fixed split second heart sound.
IVC
Which of the following is true?
RA a 16; v 38 68
A. The most likely diagnosis is an ostium
primum atrial septal defect. RV 81/17 67

B. The most likely diagnosis is a sinus PA 78/52 63


venosus defect.
LV 123/6 97
C. The most likely diagnosis is mitral
Aorta 128/70 98
stenosis.
D. She is likely to have pulmonary vascular A pulmonary angiogram showed no evidence of
disease. thromboembolic disease.

E. Surgery is indicated in the majority of Which of the following is true concerning her
cases. condition:
A. The condition is more common in males.
Answer & Comments
B. Presentation is usually in the first decade
Correct answer: D of life.
C. An ejection systolic murmur is commonly
The most likely diagnosis is an ostium
heart.
secundum atrial septal defect (ASD). Eighty-five
per cent of ASDs are of the secundum type, D. A pan systolic murmur is commonly
11% of the primum type and 4% sinus venosus heard.
defects involving the inferior vena cava (IVC) or E. Pulmonary oedema is a common feature.
superior vena cava (SVC). Clinical signs are
rarely present. When people present in middle F. The ECG often shows left axis deviation.
age with symptoms this usually relates to the G. Echo is usually unhelpful in the diagnosis.
development of progressive pulmonary vascular
H. Warfarin is not recommended.
disease, pulmonary arterial hypertrophy and a
change in the direction of flow across the shunt I. Calcium antagonists are contraindicated.
(left to right changes to right to left). J. Atrial septostomy may provide temporary
Many ASDs are now being closed using improvement.
percutaneous closure devices.
Answer & Comments

Correct answer: DJ
[ Q: 45 ] MasterClass Part2
(2010) - Cardiology This lady has primary pulmonary hypertension
with no evidence of right to left shunting. The
A 40-year-old female presents with a 6-month high v wave pressure in the right atrium is due
history of progressive breathlessness. Cardiac to tricuspid regurgitation. The condition is more
catheter data are as follows: common in females and usually presents in the
fourth decade of life. Physical signs include
parasternal heave, loud pulmonary second
sound, pansystolic murmur (due to tricuspid

Dr. Khalid Yusuf El-Zohry – Sohag Teaching Hospital (01118391123) Page | 123
El-Zohry MRCP Questions Bank (Part 2) – Medical Masterclass 2010

regurgitation) and evidence of right heart consequence of cerebral hyoperfusion. A CT


failure. ECG typically shows right axis deviation head scan is also mandatory in this age group to
and evidence of right ventricular hypertrophy. exclude a space-occupying lesion. Subsequent
investigations will be dictated by the exact
Warfarin is recommended to all patients and
nature of the event – history from a witness is
high dose calcium antagonists only to those
extremely valuable.
with a significant drop in pulmonary pressures
with vasodilator testing. Patients with evidence
of vasoactivity may also respond to [ Q: 47 ] MasterClass Part2
prostacycline or nitric oxide therapy. Lung or (2010) - Cardiology
heart-lung transplantation is offered to those
who fail to respond to medical treatment. A 67-year-old man presents with a 6-week
history of malaise, low-grade pyrexia and rigors
6 months after having a prosthetic aortic valve
[ Q: 46 ] MasterClass Part2 replacement. His C-reactive protein is 150
(2010) - Cardiology mg/dL (normal <5 mg/dL) and his creatinine
150 micromol/L (normal <120 micromol/L). A
A 71-year male has a witnessed tonic-clonic
transoesophageal echocardiogram confirms
seizure whilst standing at a bar, enjoying a pint.
vegetation on the aortic prosthesis.
He has had a prior anterior myocardial
infarction 2-years ago. Examination is normal.
Which of the following organisms is most likely
ECG shows sinus rhythm with anterior Q waves. to be responsible?
Which TWO of the following are the most A. Staphylococcus aureus
appropriate initial investigations?
B. Streptococcus viridans
A. Alcohol provocation test
C. Staphylococcus epidermidis
B. Coronary angiography
D. Escherichia coli
C. Carotid Dopplers
E. Candida albicans
D. CT head
E. Echocardiogram Answer & Comments

F. Electrophysiological study Correct answer: C


G. Exercise tolerance test
Although streptococci are the most common
H. Tilt test cause of native valve endocarditis,
I. 24-hour tape postoperatively staphylococci are most likely
and the history is suggestive of Staphylococcus
J. Urinary cathecholamine levels.
epidermidis. Staphylococcus aureus is more
likely to present in the early postoperative
Answer & Comments period and follows a more fulminant course.
Correct answer: DI
[ Q: 48 ] MasterClass Part2
A 24-hour tape is important to help to identify (2010) - Cardiology
any brady or tachycardia (e.g. ventricular
tachycardia) that might account for an A 60-year old man is admitted with an acute
arrhythmic cause of the seizure, as a inferior myocardial infarction. He is
thrombolysed with streptokinase, and treated

Dr. Khalid Yusuf El-Zohry – Sohag Teaching Hospital (01118391123) Page | 124
El-Zohry MRCP Questions Bank (Part 2) – Medical Masterclass 2010

with aspirin and clopidogrel. Two hours later D. Implantable cardioverter defibrillator and
you are asked to see him as his BP has beta-blocker
decreased from 110/70 mmHg to 70/40 mmHg,
E. Percutaneous angioplasty to the LAD.
although he is pain-free. Examination reveals a
clear chest, normal heart sounds and a raised
JVP. His ECG shows complete resolution of the Answer & Comments
ST-segment elevation. Correct answer: D

What is the appropriate course of action?


This man has ischaemic cardiomyopathy, but no
A. Administer intravenous diuretic evidence of reversible ischaemia on functional
B. Administer 250 mL 0.9% saline (thallium) assessment. He has had symptomatic
intravenously VT and therefore is at high risk of sudden death.
Current evidence suggests that these patients
C. Administer inotropes
gain prognostic benefit from implantation of an
D. Wait for 30 minutes to see if his BP picks ICD. Beta-blockers have also been shown to
up independently improve prognosis (and
symptoms) in patients with impaired left
E. Administer hydrocortisone
ventricular function.

Answer & Comments


[ Q: 50 ] MasterClass Part2
Correct answer: B (2010) - Cardiology

This is likely to be a right ventricular infarct. A 35-year-old man is admitted with chest pain.
Fluid should be administered, and an His ECG and cardiac enzymes are normal.
echocardiogram organised. Because a murmur is heard he has a
transesophageal echocardiography (TOE) (see
image).
[ Q: 49 ] MasterClass Part2
(2010) - Cardiology
A 70-year old man with prior anterior
myocardial infarction presents with syncope
and ventricular tachycardia (VT). Angiography
reveals occluded LAD and poor left ventricular
function. Thallium (nuclear) imaging reveals a
fixed anterior defect with no evidence of
reversible ischaemia.

What would be the optimal therapeutic


strategy?
A. Amiodarone
B. CABG and amiodarone
C. Implantable cardioverter defibrillator
(ICD)

Dr. Khalid Yusuf El-Zohry – Sohag Teaching Hospital (01118391123) Page | 125
El-Zohry MRCP Questions Bank (Part 2) – Medical Masterclass 2010

Which of the following is true concerning the involvement of the mesenteric arteries.
disorder shown? AoA is ascending aorta, AoD is Pregnancy is a rare but recognized cause. In
descending aorta. young patients atherosclerotic disease is rarely
A. Medical treatment is the best long term the cause.
option.
B. The most likely cause of the murmur is
[ Q: 51 ] MasterClass Part2
mitral regurgitation. (2010) - Cardiology
C. Methyldopa is contraindicated in the A 28-year-old woman is admitted with left
initial management. sided pleuritic chest pain that began suddenly
six hours ago. She is very anxious, with a
D. The diagnosis can usually be made as
respiratory rate of 18/min and a pulse rate of
easily with transthoracic echo.
90/min.
E. CT scan of the chest is usually superior to
TOE in making the diagnosis. The following would reassure you that she has
not had a pulmonary embolus:
F. Pregnancy is protective from condition.
A. Entirely normal physical examination
G. Atherosclerosis is the most likely
(aside from RR 18/min and PR 90/min)
underlying cause of the condition in this
case. B. None of the other answers is correct
H. Untreated overall prognosis is good. C. Oxygen saturation of 99% on air
I. Paraplegia is a recognized complication. D. Slight tenderness elicited by pressing on
the left side of the chest
J. Persistent abdominal pain may be a
worrying feature. E. Her admission that she is anxious because
a friend of hers had a pulmonary embolus
Answer & Comments recently.

Correct answer: IJ Answer & Comments

The TOE images demonstrate a Type A aortic Correct answer: B


dissection with an intimal flap seen in the
ascending and descending aorta. The common causes of pleuritic chest pain are
pulmonary embolism, pneumonia and
The most likely cause for his murmur is aortic
musculoskeletal pain. The safe and correct
regurgitation or co-existent bicuspid aortic
management is to assume pulmonary embolism
valve. Untreated surgically, mortality is high so
unless the history or examination findings
early referral to a cardiothoracic unit is crucial.
enable another positive diagnosis to be made,
Type B dissections involving the descending and this woman has had a pulmonary embolus
aorta only are usually treated medically in the until proven otherwise.
first instance as the complications of surgery
A history of local trauma or definite localised
are high. TOE, MRI and CT chest have similar
tenderness may enable a confident diagnosis of
sensitivities and specificities in making the
musculoskeletal pain to be made, but slight
diagnosis. Methyldopa may be used in the
tenderness is not uncommon in pulmonary
initial management to control blood pressure.
embolism, when examination can be normal, as
When the descending aorta is involved
can oxygen saturation.
persistent abdominal pain may indicate

Dr. Khalid Yusuf El-Zohry – Sohag Teaching Hospital (01118391123) Page | 126
El-Zohry MRCP Questions Bank (Part 2) – Medical Masterclass 2010

[ Q: 52 ] MasterClass Part2 [ Q: 53 ] MasterClass Part2


(2010) - Cardiology (2010) - Cardiology
A 75-year-old woman presents with A 59-year-old man has an inferior myocardial
palpitations. infarct and is treated with streptokinase.
Twenty-four hours later he suddenly becomes
Her ECG shows:

hypotensive.
What is the diagnosis?
Which of the following diagnoses does NOT
A. Atrial fibrillation
need to be considered?
B. Atrial fibrillation with acute inferior
A. Mitral regurgitation
myocardial infarction
B. Pulmonary embolus
C. Atrial flutter
C. Ventricular septal defect
D. Atrial flutter with acute inferior
myocardial infarction D. Right ventricular infarction

E. Atrioventricular re-entrant tachycardia. E. Hypotensive effect of streptokinase.

Answer & Comments Answer & Comments

Correct answer: C Correct answer: E

The ECG shows atrial flutter with 4/1 block. Streptokinase can cause hypotension, but not
There are no features to suggest myocardial after a delay of 24 hours.
infarction. Diagnoses that require serious consideration
are:
 Arrhythmia
 Re-infarction (of left or right ventricle)

Dr. Khalid Yusuf El-Zohry – Sohag Teaching Hospital (01118391123) Page | 127
El-Zohry MRCP Questions Bank (Part 2) – Medical Masterclass 2010

 Acute mitral regurgitation (due to many would commence treatment with an


dysfunction or rupture of papillary ACEI, and at some stage increase its titration
muscle) and add a low-dose beta-blocker.
 Ventricular septal defect

[ Q: 55 ] MasterClass Part2
Cardiac rupture with tamponade.
(2010) - Cardiology
Pulmonary embolism cannot be excluded, but is
much less likely than any of the other three A 79-year-old man is admitted via the
possible diagnoses listed. Emergency Department with an unwitnessed
blackout. He has had several in the past 6
months, and usually feels well within minutes
[ Q: 54 ] MasterClass Part2
of the event. Clinical examination is
(2010) - Cardiology unremarkable, and his 12-lead ECG is normal.
A 50-year-old man without any prior history is
What is the most appropriate investigation?
referred to the outpatient department for
assessment of increasing exertional dyspnoea. A. CT head
Examination reveals no abnormalities: his BP is
B. EEG
120/60 mmHg and his pulse 70 bpm in sinus
rhythm. His ECG shows left bundle-branch block C. Echocardiogram
and echocardiography confirms moderate D. 24 hr ECG (Holter) monitor
impairment of left ventricular function. His GP
E. Tilt table test
has already started him on oral furosemide and
he is now virtually asymptomatic.
Answer & Comments
What would be the most appropriate treatment
Correct answer: E
to add next?
A. Angiotensin-converting enzyme inhibitor
With blackouts having such a rapid recovery
B. Angiotensin receptor blocker time, there is likely to be a cardiological
aetiology and therefore CT head and EEG will
C. Aspirin
have low yield. 24 hr ECG (Holter) monitors
D. Beta-blocker have a low pick up rate. With a normal ECG, the
E. Spironolactone heart structure, and therefore an echo is likely
to be normal. The best investigation is a tilt-
table test.
Answer & Comments

Correct answer: A [ Q: 56 ] MasterClass Part2


(2010) - Cardiology
Although this man is now asymptomatic, he has
been rendered such by diuretic therapy. A 35-year-old woman is admitted with severe
Prognostically he will benefit from using both a pulmonary oedema 2 months after her second
long-term angiotensin-converting enzyme pregnancy. A diagnosis of peripartum
inhibitor (ACEI) and a beta-blocker. Most cardiomyopathy is made.
guidelines recommend the addition and then
increased titration of an ACEI as first-line Which of the following statements about
therapy, followed by a beta-blocker. A recent peripartum cardiomyopathy is false?
trial, CBIS III, supports this approach. In practice A. It may occur at any stage of pregnancy.

Dr. Khalid Yusuf El-Zohry – Sohag Teaching Hospital (01118391123) Page | 128
El-Zohry MRCP Questions Bank (Part 2) – Medical Masterclass 2010

B. Maternal mortality is approximately 20%. Answer & Comments


C. If cardiac function returns to normal Correct answer: D
there is an increased risk of recurrence in
subsequent pregnancies. Clinically this man has an infected MVR, with
D. Recovery of left ventricular function may severe paravalvular leak. Following surgery, the
take up to a year. commonest infecting organisms (up to around 9
months) are coagulase negative staphylococci.
E. Fifty per cent make a full recovery.
Antibiotics alone will not cure the infection; the
valve must be replaced again. In this case this
Answer & Comments should be with a metallic valve since a
Correct answer: A bioprosthetic valve would be likely to need
replacing after 10-15 years due to degeneration
thereby subjecting him to a high-risk third
Heart disease is the highest cause of maternal
operation. Bioprosthetic and metallic valves
death in the UK. Peripartum cardiomyopathy is
have similar risk for subsequent endocarditis.
defined as cardiac failure occurring in the last
month of pregnancy or within 5 months of
delivery. It is rare and thought to represent a [ Q: 58 ] MasterClass Part2
form of myocarditis. Adverse risk factors (2010) - Cardiology
include being older than thirty, giving birth to
twins and multiparity. A 68-year-old woman is referred to the
outpatient clinic with a three-month history of
dyspnoea and significant peripheral oedema.
[ Q: 57 ] MasterClass Part2 Clinical findings are solely of the oedema.
(2010) - Cardiology Echocardiography demonstrates thickened
myocardium with impaired relaxation and a
A 50-year old man presents 3 months post
bright speckled appearance.
mitral valve replacement (metallic) (MVR) with
increasing shortness of breath, fever and
What is the likely diagnosis?
weight loss. Clinically he is in pulmonary
oedema. Transoesophageal echo (TOE) A. Hypertrophic cardiomyopathy
confirms severe paravalvular mitral B. Cor pulmonale
regurgitation. The first blood culture is positive
C. Sarcoidosis
for Staph epidermidis.
D. Amyloidosis
Which is the optimal therapeutic approach?
E. Secondary neoplastic myocardial
A. Intravenous antibiotics for 4 weeks and deposition.
then repeat TOE
B. Oral antibiotics and angiotensin- Answer & Comments
converting enzyme (ACE) -inhibitor
Correct answer: D
C. Re-do mitral valve replacement -
bioprosthetic
The commonest cause of death in amyloidosis
D. Re-do mitral valve replacement - metallic is secondary to cardiac involvement. It may
present with the insidious onset of vague
E. Withold antibiotics and repeat 3 sets of
symptoms, such as lethargy. Later peripheral
blood cultures.
oedema becomes a prominent feature.

Dr. Khalid Yusuf El-Zohry – Sohag Teaching Hospital (01118391123) Page | 129
El-Zohry MRCP Questions Bank (Part 2) – Medical Masterclass 2010

Amyloid depositis in the heart produce Answer & Comments


generalized thickening of the myocardium (as
Correct answer: C
opposed to asymmetrical septal hypertrophy
commonly seen in hypertrophic
cardiomyopathy) and diastolic dysfunction. This Two distinct categories of cardiovascular
produces a "stiffened" appearance to relaxation mechanoreceptors exist: high-pressure and
in the diastolic phase. low-pressure.

A search for non-cardiac amyloid deposits is High-pressure receptors are located in the left
usually the most efficient way to coinfirm the ventricle, carotid sinus, aortic arch and renal
diagnosis histologically. Cardiac biopsy may be juxtaglomerular apparatus. These receptors
useful. The treatment of cardiac amyloid is respond to decreases in arterial pressure,
supportive and generally it carries a very poor peripheral vascular resistance or renal
prognosis. perfusion by stimulating reflexes that result in
the activation of the sympathetic and the renin-
angiotensin-aldosterone systems and the non-
[ Q: 59 ] MasterClass Part2 osmotic release of vasopressin. The major net
(2010) - Cardiology effects are vasoconstriction and the retention
of sodium and water.
Which of the following statements is correct
regarding the neurohormonal response to Low-pressure receptors are primarily found
stimulation of cardiovascular within the atria. An increase in atrial stretch
mechanoreceptors? results in enhanced secretion of ANP with
subsequent vasodilatation and both natriuresis
A. non-osmotic release of vasopressin and diuresis. Endothelin-1 is a potent
occurs as a result of increased atrial vasoconstrictor. The interactions between
stretch these systems are relatively complex in both
B. a reduction in blood pressure leads to health and diseased states. For example ANP,
stimulation of high-pressure angiotensin II and the renal sympathetic nerves
mechanoreceptors in the aortic arch and are able to modulate the renal juxtaglomerular
subsequent inhibition of renal release of renin.
sympathetic activity
C. a reduction in peripheral vascular [ Q: 60 ] MasterClass Part2
resistance is sensed by the high-pressure (2010) - Cardiology
mechanoreceptors and results in
stimulation of the renin-angiotensin- A 32-year-old woman has been referred to you
aldosterone system by her doctor, after complaining of syncope and
breathlessness. Her sister died suddenly in her
D. atrial natriuretic peptide (ANP) is 20’s. Clinically she has loud pulmonary second
released in response to stimulation of heart sound.
receptors in the carotid sinus and results
in natriuresis and diuresis What is the most likely diagnosis?
E. endothelin-1 is a potent vasodilator, A. Aortic stenosis
released in response to elevated blood
B. Mitral stenosis
pressure
C. Tricuspid regurgitation
D. Primary pulmonary hypertension

Dr. Khalid Yusuf El-Zohry – Sohag Teaching Hospital (01118391123) Page | 130
El-Zohry MRCP Questions Bank (Part 2) – Medical Masterclass 2010

E. Aortic regurgitation. admitted as an emergency with severe


pulmonary oedema. He is sitting with his legs
Answer & Comments over the side of the casualty trolley and
gasping. He has smoked 20 cigarettes per day
Correct answer: D for many years.

Given the lack of signs and family history of What is the first treatment he should receive?
sudden death the most likely diagnosis is A. Frusemide 40 mg intravenously
pulmonary hypertension. Further investigations
B. High flow oxygen via reservoir bag
would include transthoracic echocardiography
and left and right cardiac catheterisation. C. 35% oxygen
D. Diamorphine 2.5 mg intravenously, with
[ Q: 61 ] MasterClass Part2 anti-emetic
(2010) - Cardiology E. Isosorbide dinitrate by intravenous
infusion at dose titrated against blood
In a patient with resistant hypertension, which pressure.
of the following would increase the degree of
clinical suspicion for significant renovascular
Answer & Comments
disease as aetiology?
A. Arterial bruits Correct answer: B

B. Co-morbidity with diabetes


Unless it is definitely known that a patient has a
C. Concentric left ventricular hypertrophy tendency to retain carbon dioxide, all patients
D. Grade III hypertensive change on with severe breathlessness should be given high
fundoscopy flow oxygen via a reservoir bag once it has been
established that their airway is clear.
E. Palpable kidney.
After the patient has been started on high flow
oxygen, give frusemide 40-80 mg IV and
Answer & Comments
diamorphine 2.5-5 mg IV. If matters do not
Correct answer: A improve consider isosorbide dinitrate 2-10
mg/hr IV. If matters worsen, then call the ICU
The highest degree of clinical suspicion of sooner rather than later (assuming that the
renovascular disease will occur in patients with man’s condition prior to this acute presentation
arterial bruits, discrepancy in renal size of >1.5 means that this is appropriate).
cm (on ultrasound) and co-morbid vascular Concurrently try to establish a cause for the
disease. If a patient has none of these then acute deterioration: has he had a myocardial
renovascular disease is highly unlikely, infarction? If so, would he benefit from
Evidence of end-organ damage is not helpful in thrombolysis?
differentiating aetiology.
[ Q: 63 ] MasterClass Part2
[ Q: 62 ] MasterClass Part2 (2010) - Cardiology
(2010) - Cardiology A 58-year-old woman presents to casualty with
A 78-year-old man with known ischaemic heart back pain and has a blood pressure of 240/120.
disease and congestive cardiac failure is A CT scan of her chest and abdomen is

Dr. Khalid Yusuf El-Zohry – Sohag Teaching Hospital (01118391123) Page | 131
El-Zohry MRCP Questions Bank (Part 2) – Medical Masterclass 2010

organised and demonstrates an adrenal mass [ Q: 64 ] MasterClass Part2


with type B aortic dissection, involving the left (2010) - Cardiology
renal artery.
A 30-year-old woman is referred for assessment
Which of the following treatments are LEAST of her hypertension. Multiple readings have
appropriate? demonstrated a BP of over 160/98 mmHg. She
A. Methyl dopa has been intermittently depressed since the
death of her sister 1 year ago. During this
B. Endovascular aortic stenting period she has gained 15 kg in weight. She also
C. Ramipril reports menstrual irregularity, tiredness and
weakness, which have all been attributed to her
D. Surgical repair
depression. On examination she has
E. Intravenous GTN generalised obesity. There are multiple stretch
F. Sodium nitroprusside marks on her abdomen. Her ankles are
moderately swollen and she has bruises on her
G. Atenolol legs.
H. Amlodipine
Which one of the following conditions best
I. Thiazides describes the aetiology for her high BP?
J. Phenoxybenzamine. A. Severe depression
B. Polycystic ovarian syndrome
Answer & Comments
C. Diabetes mellitus
Correct answer: CG
D. Essential hypertension

A type B aortic dissection includes all aortic E. Cushing's syndrome


dissections not involving the ascending aorta.
Their downward course variably involves the Answer & Comments
splanchnic and renal arteries. Most cases are
managed conservatively without the need for Correct answer: E
surgical repair or endovascular stenting,
although these treatments may be needed. The High BP, depression, fatigue, weakness, weight
priority is blood pressure control. All the above gain, menstrual irregularity and bruising can all
blood pressure lowering drugs are useful apart be the result of Cushing's syndrome. Polycystic
from angiotensin-converting enzyme (ACE) - ovarian syndrome can result in insulin
inhibitors which should be avoided (one of the resistance, weight gain and menstrual
renal arteries is involved and there is an irregularity but does not cause hypertension.
appreciable risk of coexistent renal artery Patients with severe depression often lose
atherosclerosis in patients with aortic weight.
atherosclerosis). The adrenal mass and marked
hypertension in a relatively young woman raise
the possibility of a phaeochromocytoma.
Unopposed betablockade (that is without alpha
[ Q: 65 ] MasterClass Part2
blockade) should be avoided in such patients. (2010) - Cardiology
A 27-year-old female presented with a 6-month
history of breathlessness. Her cardiac catheter
data shows the following:

Dr. Khalid Yusuf El-Zohry – Sohag Teaching Hospital (01118391123) Page | 132
El-Zohry MRCP Questions Bank (Part 2) – Medical Masterclass 2010

Pressure (mmHg) Oxygen saturation (%) Prognosis is very poor and pregnancy results in
maternal death. Closure of the shunt at this
IVC 19 51 stage does not change the prognosis.
RA 22 52
[ Q: 66 ] MasterClass Part2
RV 120/15 55
(2010) - Cardiology
PA 121/70 56
A 68-year-old woman presents with 6 weeks of
LV 112/11 77 increasing breathlessness. She is in atrial
fibrillation at rate of 120/min and has signs of
Aorta 108/67 78
mild/moderate cardiac failure. She gives no
history of previous arrhythmia or to suggest
Which TWO statements are true of her
ischaemic heart disease, but has had mild
condition?
hypertension for at least 5 years. There are no
A. The patient will appear pale. cardiac murmurs. There are no ischaemic
B. Haemopysis is a frequent symptom. changes on the ECG. The chest radioagraph
shows a normal sized heart and mild pulmonary
C. The presence of clubbing should lead to oedema. She is not thyrotoxic.
an alternative diagnosis.
D. A fixed split second sound will be heard. Select the best of these management options:

E. Pulmonary oedema is a common finding. A. Perform DC cardioversion within next 24


hours.
F. The condition usually presents in later
life. B. Give sotalol with aim of achieving
‘chemical cardioversion’.
G. Embolic complications may occur.
C. Give diuretic and digoxin; perform DC
H. Pregnancy is safe. cardioversion within next 24 hours.
I. Prognosis is good with medical treatment. D. Give diuretic and digoxin; initiate
J. Surgical correction of the cardiac lesion is anticoagulation; perform cardioversion in
recommended. 6 weeks.
E. Give diuretic; start full-dose intravenous
Answer & Comments heparin; perform DC cardioversion within
next 24 hours.
Correct answer: BG

Answer & Comments


This patient has Eisenmenger's syndrome. An
uncorrected right to left shunt has resulted in Correct answer: D
systemic pulmonary pressures with subsequent
shunt reversal as evidenced by desaturation in In a patient with a probable first episode of AF
left ventricular blood. From these data one an attempt should be made to restore sinus
cannot determine the level of the shunt. rhythm. However, if AF has been present for
The clinical features of this condition are of >48 hrs – as seems very likely in this case – it is
right heart failure, systemic oxygen necessary to anticoagulate before
desaturation (causing central cyanosis), and cardioversion, unless it can be documented by
polycythaemia which can lead to embolic trans-oesophageal echocardiography (TOE) that
complications. Clubbing is common. the left atrium is free of thrombus. If this were

Dr. Khalid Yusuf El-Zohry – Sohag Teaching Hospital (01118391123) Page | 133
El-Zohry MRCP Questions Bank (Part 2) – Medical Masterclass 2010

the case, then cardioversion within the next 24 The above image shows a patent foramen ovale
hours would be the preferred management which occurs in 10% of the population and is
option, with warfarinisation for a month found in 40% of patients with a past history of
afterwards (which is a high risk period for stroke.
thromboembolism).
If these defects are found in patients with a
past history of a cerebrovascular accident, a
[ Q: 67 ] MasterClass Part2 right to left shunt must be looked for by
(2010) - Cardiology injecting agitated saline and asking the patient
to perform a Valsalva manoeuvre. If a
A 40-year-old man has a transoesophageal echo significant right to left shunt is demonstrated,
for investigation of a soft systolic heart murmur then either long-term anticoagulation or
after inadequate images are obtained from percutaneous closure is recommended, as
transthoracic imaging. He has no past history of there is a risk of paradoxical embolus.
note and is asymptomatic. Apart from the
feature shown (see image), his study was The results of ongoing studies should help
normal. determine whether percutaneous closure is
superior to anticoagulation
alone in this context. When
seen as an incidental finding in
patients without a history of
stroke, no treatment is
required.

[ Q: 68 ]
MasterClass Part2
(2010) -
Cardiology
A 50-year-old man is being
investigated for carcinoma of
the colon. He presents to
hospital with a 1-week history of becoming
What is the most appropriate management
progressively unwell. He reports intermittent
A. Urgent referral for surgical closure sweats and a loss of appetite. On examination
B. Addition of aspirin he appears pale, has low-grade pyrexia and is
tachycardic. He has a non-tender rash on his
C. Reassurance and regular follow-up with feet. His heart sounds are normal, but there is
echo an aortic systolic murmur and a faint aortic
D. Reassurance and no follow-up diastolic murmur. His lung fields are clear. His
abdomen is mildly tender but there is no
E. Urgent referral for percutaneous closure
evidence of peritonism. There is no
hepatomegaly, but the tip of his spleen is
Answer & Comments palpable.
Correct answer: D
Which one of the following investigations will
be most helpful in providing a diagnosis for his
subacute deterioration?

Dr. Khalid Yusuf El-Zohry – Sohag Teaching Hospital (01118391123) Page | 134
El-Zohry MRCP Questions Bank (Part 2) – Medical Masterclass 2010

A. Chest radiograph J. Warfarin


B. Abdominal ultrasound
Answer & Comments
C. Biopsy of the skin lesion
Correct answer: DJ
D. Colonoscopy
E. Blood cultures
Beta-blockers have prognostic and symptomatic
benefit in heart failure. In the UK bisoprolol,
Answer & Comments carvedilol and nebivolol are licensed for this
use. Digoxin can improve symptoms in cases of
Correct answer: E
severe heart failure, but should not be used at
the expense of a beta-blocker. This patient has
The most likely diagnosis in this man is infective permanent atrial fibrillation (AF) and as such
endocarditis. He demonstrates multiple non- there is no benefit in trying to restore sinus
specific features of insidious-onset infective
rhythm, either chemically or electrically.
endocarditis (low-grade pyrexia, sweats, heart
Impaired left ventricular function and AF carries
murmur of aortic insufficiency, mild
a significant risk of thromboembolic
splenomegaly and rash). Colonic malignancy is a
complications and hence anticoagulation with
recognised risk for endocarditis. warfarin should be recommended unless
contraindicated.
[ Q: 69 ] MasterClass Part2
(2010) - Cardiology [ Q: 70 ] MasterClass Part2
A patient who is otherwise well and with no (2010) - Cardiology
significant past medical history has permanent
A 78-year-old man is admitted with fast atrial
atrial fibrillation. His resting heart rate is 100 fibrillation and chest pain. He is given digoxin in
bpm and systolic BP 120 mmHg; there is no the Emergency Department, which reduces his
evidence of fluid retention. Echocardiography
rate to 90/min. His pain settles and his troponin
demonstrates systolic dysfunction. He is already
I is negative. He has a previous history of
receiving appropriate doses of furosemide and myocardial infarction and heart failure.
an angiotensin-converting enzyme inhibitor. His
serum creatinine is normal. Which of the following would be the optimal
management subsequently?
Which TWO of the following would be the most
appropriate drugs to add? A. Continue digoxin and add aspirin

A. Amiodarone B. DC cardiovert immediately

B. Aspirin C. Start amiodarone, aspirin and cardiovert


after 6 weeks
C. Atenolol
D. Continue digoxin and warfarinise
D. Bisoprolol
E. Start amiodarone, warfarin and
E. Candesartan cardiovert after 6 weeks.
F. Digoxin
G. Diltiazem Answer & Comments

H. Enoxaparin Correct answer: E


I. Sotalol

Dr. Khalid Yusuf El-Zohry – Sohag Teaching Hospital (01118391123) Page | 135
El-Zohry MRCP Questions Bank (Part 2) – Medical Masterclass 2010

A number of recent studies have suggested that Answer & Comments


restoration of sinus rhythm is not necessarily
Correct answer: D
better than just controlling heart rate.
However, this patient has impaired left
ventricular function and would benefit from Aspirin, Enoxaparin, Clopidogrel and beta-
restoration of sinus rhythm. This is best blockers have all been shown to reduce
achieved with amiodarone. If this fails, DC morbidity in patients presenting with acute
cardioversion would be appropriate. Aspirin is coronary syndromes. The abnormal ECG and
insufficient in this situation to prevent elevated troponin confirm increased risk and
thromboembolism, especially if DC hence aggressive medical treatment. Glyceryl
cardioversion is being considered. trinitrate may offer pain relief for patients with
on-going angina but has no effect on long-term
mortality.
[ Q: 71 ] MasterClass Part2
(2010) - Cardiology
[ Q: 72 ] MasterClass Part2
(2) A 45-year-old man with no significant past (2010) - Cardiology
medical history has been experiencing
increasingly frequent episodes of exertional This is an angiogram from a 65-year-old man
chest pain over the past 4 weeks. He comes with limiting angina. He currently takes aspirin.
to the Emergency Department on the
recommendation of NHS Direct after
experiencing an episode of pain at rest the
previous night. On arrival he is pain free and
looks well. A 12-lead ECG reveals 1mm planar
ST segment depression in leads II, III and AVF.
Troponin-I measured 12 hours from the onset
of his symptoms is moderately elevated at
0.8mg/l.

Which of the following drugs or


drug combinations is most likely to improve his
long-term morbidity and / or mortality?
A. Aspirin 300mg
B. Aspirin 300mg + Enoxaparin 1mg / kg bd
C. Aspirin 300mg + Enoxaparin 1mg / kg bd
+ Clopidogrel 300mg
D. Aspirin 300mg + Enoxaparin 1mg / kg bd
+ Clopidogrel 300mg + Atenolol 25mg The most appropriate treatment option is:
E. Aspirin 300mg + Enoxaparin 1mg / kg bd + A. long acting oral nitrate
Clopidogrel 300mg + Atenolol 25mg +
Intravenous infusion of glyceryl trinitrate B. percutaneous coronary angioplasty with
(GTN). stenting
C. coronary artery bypass grafting
D. oral beta blocker

Dr. Khalid Yusuf El-Zohry – Sohag Teaching Hospital (01118391123) Page | 136
El-Zohry MRCP Questions Bank (Part 2) – Medical Masterclass 2010

E. intravenous GIIb/IIa receptor antagonist. Answer & Comments

Correct answer: C
Answer & Comments

Correct answer: C In some patients with aortic stenosis the peak


LV to Aortic gradient can be misleading because
The angiogram demonstrates significant left of reduced cardiac output due to impaired left
main stem (LMS) and left anterior descending ventricular function. In such cases the
artery (LAD) stenoses. Coronary artery bypass calculated aortic valve area is more useful as a
grafting has been shown to improve survival in guide to severity.
patients with left main stem stenosis (> 60%), In all cases of aortic stenosis the development
triple vessel disease and impaired left of syncope signifies a poor prognosis and
ventricular function and triple vessel disease urgent surgery is usually needed to replace the
with proximal left anterior descending artery valve. Balloon valvotomy has been used
disease, when compared to medical therapy. successfully in adolescent non-calcified valvular
Angioplasty is exceedingly high risk and stenosis, but is rarely used in adults and not in
therefore is only contemplated when patients heavily calcified valves.
are deemed as non-operative candidates.

[ Q: 74 ] MasterClass Part2
[ Q: 73 ] MasterClass Part2 (2010) - Cardiology
(2010) - Cardiology
A 75-year-old man had a pacemaker implanted
An 85-year-old lady presents to the A&E 18 months earlier. He presents with pain over
department with an episode of syncope. An his pacemaker. This is what you find (see
ECG shows a left bundle branch block pattern image).
and her echocardiogram reveals a heavily
calcified aortic valve with restricted movement.

Which of the following statements is correct?


A. A peak aortic valve gradient of <50mmHg
indicates that the aortic stenosis will not
require surgical treatment.
B. A peak aortic valve gradient of >50mmHg
confirms that the aortic stenosis is the
cause but her old age means that medical
therapy is the safest option.
C. Syncope carries a poor prognosis in aortic
stenosis and she requires urgent
assessment for aortic valve replacement. What is the appropriate management?
D. Ballon valvotomy may avoid the need for A. Intravenous antibiotics and closure of the
surgey and should be the initial approach. wound
E. Her left bundle branch block on ECG B. Reassurance
suggests that transient bradycardia is
C. Replace pacemaker generator and treat
more likely to be the cause of her
with antibiotics
syncope and should first be investigated.

Dr. Khalid Yusuf El-Zohry – Sohag Teaching Hospital (01118391123) Page | 137
El-Zohry MRCP Questions Bank (Part 2) – Medical Masterclass 2010

D. Sterile dressings and wait for healing via Answer & Comments
secondary intention
Correct answer: AE
E. None of the above.
A history of syncope that occurs only on
Answer & Comments exertion is suggestive of outflow tract
obstruction, when it results from reflex
Correct answer: E
bradycardia and vasodilatation. Aortic stenosis
is therefore one of the diagnoses to consider,
The generator has eroded through the chest but it would be most unusual for hypertrophic
wall skin. This denotes infection and therefore
obstructive cardiomyopathy to present at this
requires extraction of the whole pacemaker age.
system, i.e. the generator and leads. It is
inevitable that the leads will be infected as well. Arrhythmias can occur at any time, but they
Replacement of the generator alone will be may happen only on exercise if they are
inadequate. sensitive to increased sympathetic outflow or
ischaemia, and coronary artery disease would
Pacing system infection is rare (<1%) but a very
clearly be a likely diagnosis in a man of 78 years
serious complication. Endocarditis associated with these symptoms.
with infected pacing systems carries a mortality
of a third. This system will need explanting The key physical signs to establish a diagnosis of
using specialized lead extraction tools in an aortic stenosis would be a slow rising carotid
expert centre. The procedure carries with it a upstroke, an undisplaced thrusting apex, and an
mortality of up to 5%. ejection systolic murmur radiating to the
carotids, perhaps with a palpable thrill.

[ Q: 75 ] MasterClass Part2
(2010) - Cardiology [ Q: 76 ] MasterClass Part2
(2010) - Cardiology
A 78-year-old man presents with a 3-month
history of syncopal episodes and chest pain, A 79-year-old woman is admitted via A&E
both occurring only on exertion. having been found on her floor at home. Her
neighbours had not seen her for at least 48
Select the TWO most likely diagnoses from the hours. Her core temperature was 30ºC.
list below:
Her ECG traces might show?
A. Aortic stenosis
A. Atrioventricular block
B. Epilepsy
B. Bradycardia
C. Orthostatic hypotension
C. ST segment changes
D. Sick sinus syndrome
D. Ventricular fibrillation
E. Coronary artery disease
E. All of the above.
F. Intermittent third degree heart block
G. Supraventricular tachycardia Answer & Comments
H. Atrial fibrillation Correct answer: E
I. Vasovagal syncope
J. Hypertrophic obstructive cardiomyopathy.

Dr. Khalid Yusuf El-Zohry – Sohag Teaching Hospital (01118391123) Page | 138
El-Zohry MRCP Questions Bank (Part 2) – Medical Masterclass 2010

The 'classical' feature of the ECG in diagnosis is hard to sustain with a normal
hypothermia is the 'J' wave, (although this is physical examination.
not pathognomic of hypothermia). As the
Complete heart block causes syncope, but there
cooling becomes more profound, the QRS
is rarely any relation of this to exercise and it
complex broadens. There may be ST elevation
does not cause pain.
or depression and T wave inversion, which
worsens with acidaemia. Second degree and
third degree atrioventricular block may evolve. [ Q: 78 ] MasterClass Part2
Ventricular fibrillation becomes more common (2010) - Cardiology
with lower core temperatures.
(1) A 77-year old man presents with 5 hours of
chest pain at rest. He has a past history
[ Q: 77 ] MasterClass Part2 of stenting to his left anterior descending artery
(2010) - Cardiology 4 years previously. The ECG shows inferior T-
wave inversion, with ventricular ectopics. His
A 66-year-old man presents with a three-month
troponin T is elevated at 0.4. He is already
history of syncopal episodes and chest pain on
taking aspirin.
exertion. There are no abnormal physical
findings on examination. Which TWO of the following would be
considered appropriate intial therapeutic
The most likely diagnosis is: interventions?
A. Aortic stenosis A. Amiodarone
B. Hypertrophic obstructive cardiomyopathy B. Change aspirin to clopidogrel
C. Orthostatic hypotension
C. Coronary artery bypass grafting
D. Ischaemic heart disease D. Digoxin
E. Complete heart block. E. Flecainide
F. GIIb/IIIa receptor blocker
Answer & Comments
G. Percutaneous coronary intervention
Correct answer: D
H. Dipyridamole (modified release)
Aortic stenosis would certainly need to be I. Prophylactic dose of low molecular weight
considered in a man of this age with this heparin
history, but without any signs it is hard to
J. Thrombolysis.
sustain this diagnosis. The signs to look for
would include a slow rising pulse, narrow pulse
pressure, forceful apex, and aortic systolic Answer & Comments
murmur radiating to the neck (perhaps with Correct answer: FG
carotid thrill).
Hypertrophic cardiomyopathy would be an This patient has presented with a non-ST-
uncommon diagnosis in a man of 66 years but segment elevation myocardial infarction
would require serious consideration in a (NSTEMI). This group of patients has a high risk
younger patient not otherwise at risk of of further adverse cardiac events. The mainstay
coronary disease. However, once again this of initial treatment is aggressive anti-platetlet
therapy: aspirin with the addition of

Dr. Khalid Yusuf El-Zohry – Sohag Teaching Hospital (01118391123) Page | 139
El-Zohry MRCP Questions Bank (Part 2) – Medical Masterclass 2010

clopidogrel, and consideration of an infusion of Pressure(mmHg) Saturations (%)


GIIb/IIa receptor blocker (blocks the platelet
receptor). Therapeutic dose of low molecular RA 15 67
weight heparin should be commenced. Early
RV 40/7 67
angiography and percutaneous coronary
intervention should also be considered. PA 43/16 68
Thrombolysis has not been shown to benefit
PCWP (mean) 26
patients with NSTEMI.
110/8 96
[ Q: 79 ] MasterClass Part2 Ao 107/68 96
(2010) - Cardiology
What is the diagnosis?
A previously fit young woman presents with
pleuritic chest pain 48 hours after a A. Aortic stenosis
transatlantic aeroplane flight. Her chest B. Ventricular septal defect (VSD)
radiograph is clear.
C. Aortic Regurgitation
The correct course of action is: D. Mitral stenosis
A. Order a lung ventilation / perfusion scan. E. Pulmonary stenosis.
Give low molecular weight heparin if this
is positive
Answer & Comments
B. Reassure. Give non-steroidal anti-
inflammatory agent for pain control Correct answer: D

C. Give oral amoxicillin and clarithromycin


There is no significant pressure gradient across
D. Give low molecular weight heparin. Order the aortic or pulmonary valves thereby
a lung ventilation / perfusion scan excluding aortic or pulmonary stenosis. There is
E. Order a chest radiograph in expiration to no change in saturation from RA to RV, thereby
exclude pneumothorax. excluding the diagnosis of a VSD with left to
right shunting. We can, however, see the
increase in mean capillary wedge pressure
Answer & Comments
(providing a reflection of left atrial pressure)
Correct answer: D which is greater than left ventricular diastolic
pressure (8 mmHg) in keeping with mitral
This woman has had a pulmonary embolus until stenosis. There is also an associated
proven otherwise. It is correct to start mild/moderate elevation in pulmonary
treatment immediately when the index of pressures.
suspicion is very high and the risks of treatment
(anticoagulation) are low, as in this case. [ Q: 81 ] MasterClass Part2
(2010) - Cardiology
[ Q: 80 ] MasterClass Part2
A 65-year-old man presents with increasing
(2010) - Cardiology breathlessness and swelling of his legs. These
The following results were found at cardiac symptoms have occurred over a period of 2
catheterization in a 45-year-old man who has months. He has otherwise been previously well,
been complaining of increasing breathlessness. apart from a 2-year history of diarrhoea which

Dr. Khalid Yusuf El-Zohry – Sohag Teaching Hospital (01118391123) Page | 140
El-Zohry MRCP Questions Bank (Part 2) – Medical Masterclass 2010

his gastroenterologists have been investigating. Which of the following would NOT support
On echo, he is found to have severe tricuspid considering him for an implantable cardioverter
regurgitation and a diagnosis of carcinoid defibrillator?
syndrome is made. A. Ejection fraction on echocardiography of
40%
Which of the following statements about
carcinoid syndrome is NOT true? B. Four beats of ventricular tachycardia on
CCU monitor
A. Common sites of carcinoid tumours are
the appendix and terminal ileum. C. Left bundle branch block
B. After the tricuspid valve, the mitral valve D. Previous treatment with amiodarone
is most commonly affected.
E. Atrial fibrillation.
C. Patients with cardiac carcinoid ususally
die from valvular disease rather than Answer & Comments
carcinomatosis.
Correct answer: A
D. Patients with cardiac involvement usually
have more advanced disease.
Ejection fraction is probably the most important
E. Patients with cardiac involvement usually stratification for considering patients for an ICD
have higher levels of 5-HIAA. on a background of coronary artery disease.
Most studies (e.g. MADIT) required patients to
Answer & Comments have an ejection fraction <30 or 35%. Non-
sustained VT is also an important predictor of
Correct answer: B
risk. Left bundle branch block (LBBB) on its own
is not a specific indicator for an ICD, but
Carcinoid heart disease is rare and usually patients with very broad complexes are at
affects the right-sided valves (i.e. tricuspid and greater risk. These patients may also be suitable
pulmonary valves). Cardiac involvement is for cardiac resynchronisation pacing.
associated with more advanced disease and
carries with it a poorer prognosis. It is
[ Q: 83 ] MasterClass Part2
associated with higher circulating levels of 5-
HIAA. Most patients present with right-sided (2010) - Cardiology
heart failure. Treatment of cardiac carcinoid A 60-year-old woman develops hypotension
involves management of the right-sided valve and a new systolic murmur 36 hours after being
failure, pharmacotherapy to reduce secretion of successfully thrombolysed for an anterior
tumour products and surgical intervention for myocardial infarction.
the valvular pathology
Which of the following statements is correct?
[ Q: 82 ] MasterClass Part2 A. Acute mitral incompetence due to
(2010) - Cardiology rupture of the posterior papillary muscle
is the most likely diagnosis.
A 65-year-old male presents to the Emergency
Department with palpitations but no syncope. B. Acute mitral incompetence due to
He has a previous history of anterior myocardial rupture of the anterior papillary muscle is
infarction and a hospital admission with the most likely diagnosis..
pulmonary oedema. C. A basal ventricular septal defect (VSD) is
the most likely diagnosis.

Dr. Khalid Yusuf El-Zohry – Sohag Teaching Hospital (01118391123) Page | 141
El-Zohry MRCP Questions Bank (Part 2) – Medical Masterclass 2010

D. An apical ventricular septal defect is the D. Exercise testing should not be performed
most likely diagnosis. in patients with a previous history of
ventricular arrhythmias
E. The systolic murmur is likely to be due to
mitral valve prolapse. E. There is a higher false-positive rate in
males compared with females
Answer & Comments
Answer & Comments
Correct answer: D
Correct answer: C
Anterior myocardial infarction is typically
associated with apical VSDs whilst inferior A reduction in haemodynamic performance
myocardial infarctions are more commonly with exercise may represent significant
associated with basal VSD or posterior papillary coronary artery disease and should be
muscle rupture. After confirmation of diagnosis investigated further. Generally, a heart rate
by echocardiography or right heart catheter, >85% of that predicted is accepted as a target
which reveals a step up in oxygenation at to achieve. The ST-segment changes associated
ventricular level, urgent referral to a surgical with digoxin use make the exercise ECG very
centre is required, the outlook without surgical difficult to interpret, a matter further
repair being extremely poor. complicated by this patient's atrial fibrillation.
Anterior myocardial infarction associated with
apical VSD carries a better surgical outlook than [ Q: 85 ] MasterClass Part2
inferior myocardial infarction associated with (2010) - Cardiology
basal VSD.
A 74-year-old woman with a past history
of breast cancer presents with general malaise
[ Q: 84 ] MasterClass Part2 and severe peripheral oedema that has
(2010) - Cardiology developed gradually over six months. Her blood
pressure is 90/70 mmHg and her JVP is
A 48-year woman is referred to the rapid access
considerably raised. Echocardiography reveals a
chest pain clinic by her GP with chest tightness.
thickened, calcified pericardium.
This occurs on exertion but also occasionally
after meals. Her only past medical history is
Which TWO of the following conditions are the
hypertension and permanent atrial fibrillation.
most likely causes of her underlying condition?
Her only medication is digoxin and warfarin.
A. Aortic stenosis
Which of the following regarding exercise tests B. Coronary artery disease
is true?
C. Tuberculosis
A. A fall in BP with increasing exercise is a
non-significant finding D. Chronic renal failure

B. Providing that a heart rate >75% of that E. Recurrence of breast cancer


predicted is achieved and no ECG changes F. Viral pericarditis
are documented, ischaemic heart disease
G. Previous radiotherapy for breast cancer
can be confidently excluded
C. Digoxin therapy makes interpretation of H. Infective endocarditis
exercise tests difficult I. Previous chemotherapy for breast cancer

Dr. Khalid Yusuf El-Zohry – Sohag Teaching Hospital (01118391123) Page | 142
El-Zohry MRCP Questions Bank (Part 2) – Medical Masterclass 2010

J. Cardiac amyloidosis AV node ablation should not be considered in a


young patient with paroxysmal symptoms.
Answer & Comments Generally this is reserved for patients in chronic
atrial fibrillation resistant to drug treatment.
Correct answer: CG
An atrial defibrillator may be appropriate but
certainly not an ICD used to treat malignant
This woman has constrictive pericarditis, which
ventricular arrhythmias.
causes impaired filling of the ventricles and
reduced ventricular function. The commonest Pulmonary vein isolation for atrial fibrillation is
causes are tuberculosis, mediastinal irradiation an evolving technique and may be appropriate
and previous cardiac surgery. Imaging such as for this patient.
echocardiography, CT and MRI will usually
demonstrate thickened and calcified [ Q: 87 ] MasterClass Part2
pericardium. (2010) - Cardiology
Five days after total hip replacement a 70-year-
[ Q: 86 ] MasterClass Part2
old man with known chronic obstructive
(2010) - Cardiology airways disease (COPD) becomes short of
A 37-year-old man presents with recurrent breath with a cough. He has left-sided pleuritic
episode of atrial fibrillation. He is having chest pain and a low-grade temperature. His
approximately three episodes each week, CXR is consistent with long-standing COPD, but
lasting for up to 5 hours. He has previously no other abnormalities are seen. His arterial
been treated with flecainide, sotalol and blood gases on air demonstrate pH 7.34, Pco2
amiodarone. 4.0 kPa and Po2 10.2 kPa. After 2 hours of
oxygen therapy (Fio2 30%) his Po2 has risen to
Which of the following is appropriate in his 11.0 kPa, but the other parameters are
mangement? unchanged. By this stage his FBC results show
A. Consider digoxin haemoglobin 14 g/dL, white cell count 4 x 109/L
and platelets 600 x 109/L. Biochemistry shows
B. Consider AV node ablation and sodium 131 mmol/L, potassium 5.2 mmol/L,
pacemaker implantation urea 14 mmol/L and creatinine 170 mmol/L.
C. Consider him for an implantable
cardioverter defibrillator (ICD) What is the most likely cause for his clinical
picture?
D. Consider radiofrequency ablation to
A. Infective exacerbation of COPD
isolate his pulmonary veins
B. Left-sided pneumonia
E. None of the above.
C. Acute pulmonary embolus
Answer & Comments D. Small pneumothorax
Correct answer: D E. Panic attack

He is spending a significant proportion of his Answer & Comments


time in atrial fibrillation (high atrial fibrillation
Correct answer: C
burden). However, it is paroxysmal and
therefore digoxin is inappropriate and may
increase the number of episodes.

Dr. Khalid Yusuf El-Zohry – Sohag Teaching Hospital (01118391123) Page | 143
El-Zohry MRCP Questions Bank (Part 2) – Medical Masterclass 2010

This patient has hypoxaemia with low normal  syncope


Pco2. The two main differentials are pulmonary  ventricular arrhythmias on 24-hour
embolus and chest infection. However, there is
tape.
nothing in his results to suggest a chest
infection. The CXR does not demonstrate
cosolidation and/or other changes of a chest [ Q: 89 ] MasterClass Part2
infection, and furthermore there is no increase (2010) - Cardiology
in the white blood cell count.
A 55-year-old man with a history of severe
asthma and ischaemic heart disease is brought
[ Q: 88 ] MasterClass Part2 to the Emergency Department complaining of
(2010) - Cardiology palpitations and syncope. On examination he
has a weak, regular pulse with a rate of 180 /
A 32-year-old gentleman is currently being
minute and blood pressure 110/70 mmHg. A 12
investigated for hypertrophic cardiomyopathy
lead ECG reveals a broad complex tachycardia.
(HCM).
Which of the following statements is correct?
Which of the following statements are
associated with sudden cardiac death and A. Absence of capture or fusion beats on a
HCM? long rhythm strip is strongly suggestive of
supraventricular tachycardia (SVT) with
A. Family history of sudden cardiac death
aberrant conduction.
(SCD)
B. SVT is more likely than ventricular
B. Chest pain
tachycardia (VT) if the patient gives a
C. Breathlessness history of recent myocardial infarction.
D. NSVT on 24-hour tape C. Intravenous adenosine should be used to
E. Age>40 distinguish SVT from VT.

F. Mutation in myosin-binding protein C D. QRS width greater than 0.2 seconds is


usually indicative of VT.
G. Increased VO2 max
E. SVT with aberrant conduction is unlikely if
H. Male sex a recent ECG showed no evidence of
I. LV wall thickness of 15mm bundle branch block.

J. T wave inversion on ECG.


Answer & Comments

Answer & Comments Correct answer: D

Correct answer: AD
Although the presence of capture and fusion
beats is pathognomonic of VT, they are rarely
Hypertrophic cardiomyopathy is associated
seen, and their absence cannot therefore be
with sudden cardiac death. The following
relied upon to rule out the diagnosis. Broad
features are particularly associated with an
complex tachycardia in the first week after a
adverse prognosis (the presence of > 1 of these
myocardial infarction is proven to be ventricular
features increases an individual’s risk):
in origin in over 90% of cases. Adenosine may
 LV septal thickness >30mm induce bronchospasm, and therefore should be

avoided in patients with prior history of
family history of sudden cardiac death

Dr. Khalid Yusuf El-Zohry – Sohag Teaching Hospital (01118391123) Page | 144
El-Zohry MRCP Questions Bank (Part 2) – Medical Masterclass 2010

asthma. The absence of bundle branch block on [ Q: 91 ] MasterClass Part2


a recent ECG is an unreliable indicator of the (2010) - Cardiology
source of the tachycardia, since bundle branch
block is frequently rate-related, and often A 34-year-old male known to have AIDS
resolves on termination of the dysrhythmia. presents with dyspnoea and cardiomegaly. His
echo demonstrates a dilated poorly functioning
left ventricle.
[ Q: 90 ] MasterClass Part2
(2010) - Cardiology Which of the following are not common cardiac
manifestations of HIV/AIDS?
A 78-year-old man with advanced cancer
presents with painless pitting oedema of both A. Aortic dissection
legs. B. Pericardial effusion
Which of the following diagnoses is least likely C. Endocarditis
to apply?
D. Myocarditis
A. Cardiac failure
E. Pulmonary hypertension.
B. Venous thromboembolism
C. Lymphoedema Answer & Comments

D. Hypocalcaemia Correct answer: A


E. Hypoalbuminaemia.
Cardiac complications of HIV infection are
common with myocardial, pericardial and
Answer & Comments
endocardial involvement. This is a result of not
Correct answer: D only direct involvement as a result of HIV
infection, but also the cardiotoxicity of the
Cardiac failure and hypoalbuminaemia are drugs used in the management of HIV-infected
common causes of this condition in terminally patients.
ill patients. Venous thromboembolism is Aortic dissection is not a common recognized
thought to be present in up to 50% of complication.
terminally ill patients and usually presents with
insidious painless swelling that may be
[ Q: 92 ] MasterClass Part2
associated with episodes of breathlessness.
Lymphoedema, a protein rich exudate, is (2010) - Cardiology
indistinguishable from other causes of oedema A 48-year-old man with Marfan syndrome
in the early stages. In time (weeks to months) presents acutely with a Type A aortic dissection.
the protein becomes organised and fibrosed
and clinically becomes as 'non-pitting'. Which TWO of the following cardiac conditions
Disorders of calcium homeostasis are not are also associated with Marfan's?
commonly linked with oedema in this (or any A. Hypertrophic cardiomyopathy
other) context.
B. Myocardial infarction
C. Amyloidosis
D. Sarcoidosis
E. Ventricular septal defect

Dr. Khalid Yusuf El-Zohry – Sohag Teaching Hospital (01118391123) Page | 145
El-Zohry MRCP Questions Bank (Part 2) – Medical Masterclass 2010

F. Patent ductus arteriosus D. Intravenous beta-blockers


G. Ventricular tachycardia E. Inotropic support
H. Mitral valve prolapse F. Urgent echocardiography
I. Wolff-Parkinson-White syndrome G. Swann–Ganz pulmonary artery catheter
J. Restrictive cardiomyopathy. H. Intravenous diamorphine
I. Transfer to cardiothoracic surgical centre
Answer & Comments
J. Consider positive pressure ventilation.
Correct answer: BH
Answer & Comments
Cardiovascular complications of Marfan
Correct answer: AD
syndrome include the following: aortic
dilatation and dissection (which may cause
myocardial infarction), mitral valve prolapse Increasing revascularisation both with coronary
and regurgitation, left ventricular dilatation, artery bypass grafting (CABG) and percutaneous
pulmonary arterial dilatation. Aortic coronary intervention (PCI) has seen reductions
regurgitation is common in the context of a in the number of deaths following acute
dilated aortic root. myocardial infarction. However, shock remains
the leading cause of death in hospital.
Beta blockers and ACE inhibitors should be
considered in any patient with Marfan's and a It is clear that this patient has most likely
dilated aortic root, but surgical intervention is developed a post MI ventricular septal defect or
the definitive treatment when the aortic root is acute mitral regurgitation. In the absence of
greater than 5 cms in diameter, when the risk new ECG changes, thrombolysis is clearly
of dissection is significant. contraindicated. The use of intravenous beta-
blockers in the shocked patient is also relatively
Marfan patients should generally be screened
contraindicated. The mainstay of treatment is
for cardiovascular complications on an annual
therefore supportive and surgical intervention,
basis.
if appropriate.

[ Q: 93 ] MasterClass Part2 [ Q: 94 ] MasterClass Part2


(2010) - Cardiology (2010) - Cardiology
A 56-year-old man develops cardiogenic shock
A patient presents with a suspected aortic
8 hours after admission with an anterior
dissection.
myocardial infarction. He was thrombolysed
with tPA on admission with rapid resolution of Which of the following investigations is least
the ST elevation. He has a loud pansystolic helpful in confirming/refuting the diagnosis?
murmur which is new.
A. Aortography
Which of the following are inappropriate in the B. CT scan
management of this patient?
C. Transthoracic echocardiogram
A. Repeat thrombolysis
D. Magnetic resonance imaging
B. Coronary angiography
E. Transoesophageal echocardiogram.
C. Intra aortic balloon pump

Dr. Khalid Yusuf El-Zohry – Sohag Teaching Hospital (01118391123) Page | 146
El-Zohry MRCP Questions Bank (Part 2) – Medical Masterclass 2010

Answer & Comments that the pacemaker is switched on and that the
leads are still connected. The presence of
Correct answer: C
pacemaker spikes on the ECG confirms that
these are okay. It is likely that the tip of the
The choice of investigation will depend upon pacing wire has migrated and that the
local availability and expertise in interpretation threshold has increased. This may be overcome
of the results. Magnetic resonance imaging and by increasing the voltage. If this is successful,
CT scanning are excellent methods for imaging the pacemaker wire will need to be
the aorta, but require transfer of the patient to repositioned. If inceasing the voltage is
the radiology department. Transoesophageal unsuccessful, external pacing should be
echocardiography offers some advantages in substituted.
particularly unstable patients, since it can be
performed in the anaesthetic room, or on ITU.
Whilst a Transthoracic echocardiogram may
[ Q: 96 ] MasterClass Part2
provide valuable information, such as detection (2010) - Cardiology
of pericardial effusion or involvement of the A 42-year-old man visits his GP, complaining of
aortic valve (regurgitation), it cannot exclude a recent onset of shortness of breath and
the diagnosis with any degree of certainty. dizziness upon exertion. His GP arranges for an
echocardiogram, which showed hypertrophic
[ Q: 95 ] MasterClass Part2 obstructive cardiomyopathy (HOCM) with
(2010) - Cardiology thickening of the septal wall and a left
ventricular outflow gradient of 86mmHg. The
A 72-year-old man on the coronary care unit man is referred to a cardiology outpatient clinic,
has had a temporary transvenous pacing wire where he volunteers that his father died
inserted. He suddenly becomes symptomatic suddenly of a heart attack aged 38 years of age
with pre-syncope and a palpable pulse of 28 and his younger brother died aged 17 years of
bpm. The ECG monitor shows pacing spikes that age during a rugby scrum more than 20 years
are not related to the QRS complexes. ago. He is not sure of the cause of death for his
younger brother but does not think a post-
Which of the following actions is appropriate?
mortem was ever carried out. He has two
A. Check that the pacemaker is switched on teenaged sons, both of whom are well, but his
14-year-old son fainted after scoring a goal for
B. Check the pacemaker leads are
the school team last week.
connected
C. Arrange for an urgent permanent Which of the following statements are correct?
pacemaker
A. The majority of patients with HOCM are
D. Start chest compressions symptomatic throughout life.
E. Increase the pacing voltage to a B. The overall mortality in patients with
maximum. HOCM is 50% per year.
C. All first degree family members should
Answer & Comments undergo screening every 6 months.
Correct answer: E D. All patients should receive advice on
avoiding dehydration and strenous
If a patient with a temporary pacemaker exercise.
becomes symptomatic it is important to check

Dr. Khalid Yusuf El-Zohry – Sohag Teaching Hospital (01118391123) Page | 147
El-Zohry MRCP Questions Bank (Part 2) – Medical Masterclass 2010

E. Patients with HOCM do not need B. Trans-oesophageal echocardiography


prophylaxis against infective endocarditis.
C. Cardiac MRI
D. 24-hour tape
Answer & Comments
E. Coronary angiography.
Correct answer: D

Answer & Comments


 The majority of patients with HOCM are
ASYMPTOMATIC. Correct answer: B
 The overall mortality in patients with
HOCM is <1% per year. The result of the catheterisation demonstrates
a significant gradient across the mitral valve
 All first degree family members should
(PAW = pulmonary artery wedge, reflecting left
undergo periodic screening with
atrial pressure)indicating mitral stenosis. The
echocardigraphy every 5 years for this
next investigation would be trans thoracic
autosomal dominant disorder since
echocardiography to assess the suitability of
hypertrophy may not occur until 6th or
the mitral valve for balloon valvuloplasty.
7th decade of life. Annual screening is
General contra-indications to percutaneous
recommended for adolescents 12 to 18
balloon valvuloplasty are: significant mitral
years of age.
regurgitation, calcification of mitral valver
 All patients should be advised to avoid leaflets, left atrial thrombus. All of these can be
dehydration and strenuous exercise. assessed by trans oesophageal echo.
 All patients with HOCM should be
advised to use prophylaxis against [ Q: 98 ] MasterClass Part2
infective endocarditis. (2010) - Cardiology
A 32-year-old woman with a history of
[ Q: 97 ] MasterClass Part2 congenital heart disease is planning to start a
(2010) - Cardiology family. She has no other significant medical
history and is currently symptomatically stable
The following measurements were made from a
on a small dose of loop diuretic.
cardiac catheterisation of a 28-year-old
gentleman with palpitations and
Which one of the following conditions is
breathlessness:
associated with an unacceptably high risk in
RA 6mmHg
pregnancy?
A. Corrected tetralogy of Fallot
RV 22/6
B. Perimembranous ventricular septal
PA 30/10
defect
PAW 28 C. Secundum atrial septal defect
LV 128/10 D. Coarctation of the aorta
E. Pulmonary hypertension of any cause
What is the most appropriate next
investigation?
Answer & Comments
A. Exercise treadmill testing
Correct answer: E

Dr. Khalid Yusuf El-Zohry – Sohag Teaching Hospital (01118391123) Page | 148
El-Zohry MRCP Questions Bank (Part 2) – Medical Masterclass 2010

Pulmonary hypertension of any cause is G. Echocardiography shows left ventricular


associated with a 50% maternal mortality. dilatation.
Other high-risk cardiac conditions are severe
H. Medical treatment is recommended in
mitral, pulmonary or aortic stenosis and
this case.
Marfan's syndrome with a dilated aortic root.
Intracardiac shunts without pulmonary I. Arrhythmias are rare with this condition.
hypertension are relatively safe but require J. In children, this condition may be
closer follow-up. Operated tetralogy of Fallot associated with skeletal abnormalities.
without residual pulmonary stenosis is lower
risk; coarctation is associated with dissection
Answer & Comments
and is intermediate risk.
Correct answer: CJ
[ Q: 99 ] MasterClass Part2
(2010) - Cardiology This patient has a secundum atrial septal defect
as shown by a step-up in oxygen saturations at
A 40-year-old woman was referred for atrial level. She also has pulmonary
investigation of shortness of breath on effort hypertension. Characteristic findings on
over the preceding 1 year. Data from her auscultation include a wide fixed-split second
cardiac catheterisation iare as follows: sound, ejection systolic murmur and a mid
diastolic murmur. The murmurs are caused by
Pressure (mmHg) Oxygen saturation (%)
increased flow across the pulmonary and
SVC 15 64 tricuspid valves respectively. The ECG often
shows a right bundle branch block pattern and
IVC 13 65 the chest radiograph shows right ventricular
RA 13 82 enlargement and prominent pulmonary
arteries. Surgical or percutaneous closure is
RV 51/7 85 recommended in patients with symptoms, right
PA 53/31 83
ventricular dilatation, or a pulmonary: systemic
flow ratio > 1.5. The Holt-Oram syndrome is the
LV 134/3 99 association of an atrial septal defect with bony
abnormalities of the extremities.
Aorta 135/82 99

Which of the following are true concerning her [ Q: 100 ] MasterClass Part2
diagnosis? (2010) - Cardiology
A. This condition rarely presents in A 55-year-old woman has a 2-year history of
adulthood. exertional chest pain and has a positive exercise
B. Paradoxical splitting of the second sound test with significant ST segement depression.
is common. She then has a coronary angiogram which
demonstrates normal coronary arteries. She is
C. A rumbling mid diastolic murmur may
told she has Cardiac Syndrome X.
occur.
D. A systolic murmur is rare. Which one of the following statements about
Syndrome X is NOT correct?
E. The ECG often shows left bundle branch
block. A. Patients have typical symptoms of
occlusive coronary artery disease.
F. The chest radiograph is usually normal.

Dr. Khalid Yusuf El-Zohry – Sohag Teaching Hospital (01118391123) Page | 149
El-Zohry MRCP Questions Bank (Part 2) – Medical Masterclass 2010

B. Myocardial perfusion imaging is usually In intravenous drug abusers Staph aureus


normal. accounts for 50% of cases of infective
endocarditis, with other causes being
C. Life expectancy is normal.
enterococci (10%), Strep viridans (5%), gram
D. Beta blockers, calcium antagonists and negative bacteria (5%), other bacteria (5%),
nitrates have all been shown to improve fungi (5%) and polymicrobial infection (5%).
symptoms. Coagulase-negative staphylococci account for
E. It is caused by coronary microvascular less than 5%.
dysfunction. By contrast, in patients who do not abuse drugs
intravenously (and do not have prosthetic
Answer & Comments valves) the frequency of infection is as follows:
Strep viridans 35%, enterococci 25%, Staph
Correct answer: B
aureus 23%, culture negative 5-10%, other
identified organisms 5-10%.
Cardiac Sydrome X is characterised by typical
exertional angina, ST changes on ECG, and
[ Q: 102 ] MasterClass Part2
normal coronary arteries on angiography. It is
associated with microvascular abnormalities (2010) - Cardiology
such as diabetes and hypertension and thought A 42-year-old man reviewed in outpatients has
to be due to microvascular dysfunction. Life a 6-month history of increasing shortness of
expectancy is normal. breath on exertion and feelings of
About 50% of patients with syndrome X will lightheadedness when digging in his garden. His
have abnormal myocardial perfusion imaging. general practitioner organised an open access
echocardiogram which showed a septal
thickness of 26mm and a left ventricular
[ Q: 101 ] MasterClass Part2
outflow gradient of 85mmHg.
(2010) - Cardiology
Which of the following is NOT a risk factor for
A 28-year-old woman who abuses heroin
sudden death in patients with hypertrophic
intravenously presents with a high fever. She
cardiomyopathy?
has aortic incompetence.
A. Unexplained syncope
Which of the following organisms is most likely
B. Sudden death from HOCM in 2 or more
to be the cause of her infective endocarditis?
first degree relatives <40years of age
A. Staphylococcus aureus
C. Family history of sudden death
B. Streptococcus viridans
D. left ventricular wall thickness of >30mm
C. An Enterococcus
E. Hypertension.
D. Staphylococcus albus
E. A gram-negative bacillus Answer & Comments

Correct answer: E
Answer & Comments

Correct answer: A Major risk factors for hypertrophic


cardiomyopathy are:
 Cardiac arrest (ventricular fibrillation)

Dr. Khalid Yusuf El-Zohry – Sohag Teaching Hospital (01118391123) Page | 150
El-Zohry MRCP Questions Bank (Part 2) – Medical Masterclass 2010

 Spontaneous sustained ventricular has combined alpha and beta-adrenoceptor


tachycardia blocking activity and causes peripheral
 arteriolar vasodilatation without reflex
Family history of sudden death
tachycardia.
 Minor risk factors
Intravenous Nitroprusside causes tachycardia
 Unexplained syncope and would normally be combined with a beta-
 Left ventricualr wall thickness>30mm blocking agent. The other agents should not be
used in the acute setting, when very careful
 Abnormal blood pressure on titration of blood pressure is needed. They
exercise(failure to rise from baseline by
could be used later for more chronic control of
25mmHg) blood pressure after the emergency has been
 Non sustained Ventricular tachycardia managed.
 Left ventricular outflow obstruction

[ Q: 104 ] MasterClass Part2
Microvascular obstruction
(2010) - Cardiology
 High risk genetic defect.
A 32-year-old woman is referred to the
cardiology outpatient clinic after complaining of
[ Q: 103 ] MasterClass Part2
syncope and breathlessness. Her sister died
(2010) - Cardiology suddenly in her twenties. Examination is
A 70-year-old man presents with severe normal, excepting that she has a loud second
anterior chest pain which radiates to his back. heart sound. An ECG shows changes compatible
Chest radiography reveals a widened with right ventricular hypertrophy and strain.
mediastinum, and an acute type B thoracic
What would be the most appropriate
aortic dissection is confirmed on angiography.
investigation to consider next?
Which of the following is the most appropriate A. CT pulmonary angiography
treatment?
B. Echocardiography
A. Oral Nifedipine
C. Holter ambulatory monitoring
B. Intravenous Labetalol
D. Right and left heart catheterisation
C. Intravenous Sodium nitroprusside
E. Ventilation-perfusion scan
D. Oral Bendroflumethiazide
E. Oral Lisinopril Answer & Comments

Correct answer: B
Answer & Comments

Correct answer: B Given the history, lack of signs (except


accentuated P2) and family history of sudden
Type A aortic dissections require surgical death, the most likely diagnosis is pulmonary
treatment. Type B dissections are usually hypertension. Echocardiography is the key
managed medically with careful blood pressure initial investigation to evaluate cardiac
control. In the acute phase, control of blood structure and provide a non-invasive estimate
pressure with an intravenous infusion of of right heart pressures. If confirmed, other
Labetalol would be the ideal option. Labetalol investigations directed towards establishing
aetiology will be warranted.
Dr. Khalid Yusuf El-Zohry – Sohag Teaching Hospital (01118391123) Page | 151
El-Zohry MRCP Questions Bank (Part 2) – Medical Masterclass 2010

[ Q: 105 ] MasterClass Part2 Transoesophageal echocardiography is superior


(2010) - Cardiology to transthoracic echocardiography for the
diagnosis of which of the following?
A 45-year-old man with dilated cardiomyopathy
A. Left ventricular apical thrombus
is being considered for cardiac transplantation.
B. Left atrial appendage thrombus
Which of the following is generally deemed to
C. Left ventricular non-compaction
be a contraindication to cardiac
transplantation? D. Hypertrophic cardiomyopathy
A. Any previous cancer E. Ventricular septal defect
B. Pulmonary artery wedge pressure
<20mmHg Answer & Comments

C. Creatinine clearance <50 ml min-1 Correct answer: B

D. Previous alcoholism
For transoesophageal echocardiography (TOE)
E. Hypertension. the transducer is positioned against the left
atrium, and hence this technique is superior for
Answer & Comments visualisation of the left atrium, interatrial
septum, pulmonary veins and the aortic arch.
Correct answer: C
The low signal-to-noise ratio also makes TOE
superior for prosthetic valve assessment. The
Patients with heart failure are considered for left ventricular apex lies in the far field and is
cardiac transplantation when significant foreshortened with TOE. Transthoracic
symptoms persist despite maximal medical echocardiography is therefore superior for left
therapy. Due to the shortage of donors clinical ventricular apical disease, especially if used in
guidelines have been established highlighting conjunction with intravenous contrast agents.
patients most suitable for transplantation.
Significant impairment of renal function is
generally considered to be a contraindication, [ Q: 107 ] MasterClass Part2
assuming this is not reversible. Patients with (2010) - Cardiology
prior history of cancer may be considered if
You find a middle aged man on a path in a park.
there is no evidence recurrence (> 5 years on
He has no pulse and is not breathing.
from diagnosis of cancer).
Which is the appropriate next step:
[ Q: 106 ] MasterClass Part2 A. Give two rescue breaths and initiate CPR
(2010) - Cardiology at a ratio of 15 compressions to 2 breaths

An 84-year-old woman is admitted with a left B. Give two rescue breaths and initiate CPR
hemiplegia and expressive dysphasia. Her at a ratio of 30 compressions to 2 breaths
family tells you that she has a previous cardiac C. Give a precordial thump
history but they are unsure what this is.
Transthoracic echocardiography is normal and D. Give two rescue breaths and go for help.
so it is recommended that you request E. Go to call 999 (emergency services)
transoesophageal echocardiography. immediately.

Dr. Khalid Yusuf El-Zohry – Sohag Teaching Hospital (01118391123) Page | 152
El-Zohry MRCP Questions Bank (Part 2) – Medical Masterclass 2010

Answer & Comments However, DC cardioversion is not likely to lead


to permanent restoration of sinus rhythm in a
Correct answer: E
patient who has had previous episodes of AF,
hence in this case an attempt at ‘chemical
The guidelines on basic life support identify the cardioversion’ is appropriate.
importance of early access to defibrillation in
cardiac arrest. They therefore suggest that no Class III agents – potassium channel blockers
CPR is commenced until a call for emergency that prolong myocyte repolarisation – are most
services has been made and the potential for appropriate: sotalol or amiodarone.
early defibrillation is made possible. Once CPR Digoxin can be used for rate control but does
begins the ratio of compressions to ventilations not promote return of sinus rhythm, indeed it’s
is 30 to 2. A precordial thump is not indicated in use may make this more unlikely.
the unwitnessed collapse.
Class I agents – e.g. quinidine, procaineamide
and disopyramide, which prolong the action
[ Q: 108 ] MasterClass Part2 potential – have been used to try to restore
(2010) - Cardiology sinus rhythm, but NOT in patients with
ischaemic heart disease (such as this man). Use
(1) A 70-year-old man, who is known to have of these agents in atrial fibrillation has been
ischaemic heart disease and has had short-lived superseded by that of sotalol and amiodarone.
episodes of atrial fibrillation in the past,
presents with 48 hours of fatigue and
breathlessness. He is not very ill, but his pulse is [ Q: 109 ] MasterClass Part2
150 / min in atrial fibrillation. (2010) - Cardiology

Which TWO drugs would be most appropriate A 68-year-old man, previously fit and well,
to achieve ‘chemical cardioversion’? presents with anterior myocardial infarction
(MI) for which he receives appropriate
A. Digoxin thrombolysis, antiplatelet therapy and
B. Quinidine diamorphine. Over the next 4 hours he
complains of increasing dyspnoea. His pulse
C. Procainamide
rate is 110 bpm (sinus), BP 160/90 mmHg and
D. Disopyramide arterial saturation is 92% on 60% inspired
E. Sotalol oxygen.

F. Atenolol What is the most appropriate intravenous


pharmacological treatment?
G. Propanolol
A. Atenolol
H. Verapamil
B. Dobutamine
I. Amiodarone
C. Enalapril
J. Diltiazem
D. Furosemide
Answer & Comments E. Nitrate
Correct answer: EI
Answer & Comments
Restoration of sinus rhythm can be achieved Correct answer: E
pharmacologically or by DC cardioversion.

Dr. Khalid Yusuf El-Zohry – Sohag Teaching Hospital (01118391123) Page | 153
El-Zohry MRCP Questions Bank (Part 2) – Medical Masterclass 2010

This man has developed acute pulmonary Which of the following treatments will improve
oedema secondary to acute myocardial his prognosis long term?
infarction. He has an elevated BP and as such A. Atenolol
the optimal initial therapy is intravenous
vasodilator. Intravenous nitrate is the most B. Carvedilol
attractive option listed, to be followed by an C. Pindolol
oral angiotensin-converting enzyme (ACE)
D. Digoxin
inhibitor (studies of early intravenous ACE
inhibition have not demonstrated early E. Dobutamine
benefit). Subsequent small aliquots of loop
F. Milrinone
diuretic may be required.
G. Frusemide

[ Q: 110 ] MasterClass Part2 H. Bendrofluazide


(2010) - Cardiology I. Spironolactone

This is a M-Mode trace from a transthoracic J. Amlodipine.


echocardiogram (see image) taken on a 60-
year-old man presenting with shortness of Answer & Comments
breath for 6 months. A subsequent angiogram
showed normal coronary arteries. He is in sinus Correct answer: BI
rhythm.
The M-mode trace
demonstrates a dilated
cardiomyopathy with
significantly reduced
ejection fraction. There is
also a small pericardial
effusion seen below the
posterior wall of the left
ventricle.
Carvedilol, bisoprolol,
bucindolol and metoprolol
are the only beta-blockers
thus far which improve
long-term prognosis.
Inotropes long term are
associated with a worse
prognosis.
Digoxin has little impact on
mortality but may improve
quality of life and reduce
hospital readmission rates
in end stage disease.
Amlodipine has no impact
on mortality.

Dr. Khalid Yusuf El-Zohry – Sohag Teaching Hospital (01118391123) Page | 154
El-Zohry MRCP Questions Bank (Part 2) – Medical Masterclass 2010

[ Q: 111 ] MasterClass Part2 Patients with significant left stem main disease
(2010) - Cardiology (>50% occlusion) or significant three vessel
disease with depressed LV function have
A 76-year-old woman is admitted to the increase survival if they undergo CABG,
coronary care unit with unstable angina. Her 12 although patients older than 75 yrs were not
lead electrocardiogram shows ST depression of included in the trials that demonstrated such
more than 2mm in the anterior leads. Clinical benefit.
examination shows evidence of acute heart
failure. Despite medical treatment she has
[ Q: 112 ] MasterClass Part2
chest pain two hours after admission.
(2010) - Cardiology
Which of the following is correct?
A 37-year-old man presents with recurrent
A. Morphine should not be given to relieve episode of atrial fibrillation. He is having
pain if sublingual GTN has been approximately three episodes each week,
ineffective. lasting for up to 5 hours. He has previously
been treated with flecainide, sotalol and
B. Verapamil should be given if she is unable
amiodarone.
to tolerate a beta blocker.
C. Patients with impaired LV function Which of the following is appropriate in his
(EF<50%) should not undergo coronary management?
artery bypass grafting (CABG).
A. Consider digoxin
D. Cardiac catheterisation is used to assess
B. Consider AV node ablation and
patients for CABG only.
pacemaker implantation
E. Patients with significant left main stem
C. Consider him for an implantable
disease have increased survival with
cardioverter defibrillator (ICD)
CABG.
D. Consider radiofrequency ablation to
isolate his pulmonary veins
Answer & Comments
E. None of the above.
Correct answer: E

Answer & Comments


Morphine may be necessary to relieve anginal
pain - age is not a contraindication, although Correct answer: D
aliquots of smaller doses e.g. 2.5mg should be
used to achieve pain control. He is spending a significant proportion of his
Calcium channel blockers should not be given time in atrial fibrillation (high atrial fibrillation
to patients in pulmonary oedema or with burden). However, it is paroxysmal and
impaired LV function. therefore digoxin is inappropriate and may
increase the number of episodes.
Impaired LV function (EF<50%) is not a
contraindication to CABG. AV node ablation should not be considered in a
young patient with paroxysmal symptoms.
Cardiac catheterisation informs the choice
Generally this is reserved for patients in chronic
between medical treatment, CABG and
atrial fibrillation resistant to drug treatment.
percutaneous transluminal coronary
angioplasty.

Dr. Khalid Yusuf El-Zohry – Sohag Teaching Hospital (01118391123) Page | 155
El-Zohry MRCP Questions Bank (Part 2) – Medical Masterclass 2010

An atrial defibrillator may be appropriate but heart failure. Treatment of cardiac carcinoid
certainly not an ICD used to treat malignant involves management of the right-sided valve
ventricular arrhythmias. failure, pharmacotherapy to reduce secretion of
tumour products and surgical intervention for
Pulmonary vein isolation for atrial fibrillation is
the valvular pathology.
an evolving technique and may be appropriate
for this patient.
[ Q: 114 ] MasterClass Part2
[ Q: 113 ] MasterClass Part2 (2010) - Cardiology
(2010) - Cardiology A 57-year-old diabetic man with stable angina
wishes to use Sildenafil (Viagra) to treat
A 65-year-old man presents with increasing
impotency.
breathlessness and swelling of his legs. These
symptoms have occurred over a period of 2
Concomitant use of which TWO of the following
months. He has otherwise been previously well,
cardiac medications is contraindicated?
apart from a 2-year history of diarrhoea which
his gastroenterologists have been investigating. A. Aspirin
On echo, he is found to have severe tricuspid B. Clopidogrel
regurgitation and a diagnosis of carcinoid
C. Atenolol
syndrome is made.
D. Amlodipine
Which of the following statements about
E. Digoxin
carcinoid syndrome is NOT true?
A. Common sites of carcinoid tumours are F. Isosorbide mononitrate
the appendix and terminal ileum. G. Lisinopril
B. After the tricuspid valve, the mitral valve H. Nicorandil
is most commonly affected.
I. Simvastatin
C. Patients with cardiac carcinoid ususally
J. Warfarin.
die from valvular disease rather than
carcinomatosis.
Answer & Comments
D. Patients with cardiac involvement usually
have more advanced disease. Correct answer: FH

E. Patients with cardiac involvement usually


Sildenafil enhances Nitric Oxide (NO) mediated
have higher levels of 5-HIAA.
smooth muscle relaxation by blocking the
degradation of cGMP. Although it is relatively
Answer & Comments
selective to the corpus cavernosum it does still
Correct answer: B have some systemic effect. Both Isosorbide
Mononitrate and Nicorandil release NO and
Carcinoid heart disease is rare and usually their action is potentiated by Sildenafil, which
affects the right-sided valves (i.e. tricuspid and can lead to profound hypotension.
pulmonary valves). Cardiac involvement is The other commonly used cardiac medications
associated with more advanced disease and listed have no specific interaction with
carries with it a poorer prognosis. It is Sildenafil.
associated with higher circulating levels of 5-
HIAA. Most patients present with right-sided
Dr. Khalid Yusuf El-Zohry – Sohag Teaching Hospital (01118391123) Page | 156
El-Zohry MRCP Questions Bank (Part 2) – Medical Masterclass 2010

[ Q: 115 ] MasterClass Part2 activated simultaneously. This can cause reflux


(2010) - Cardiology of jugular venous blood and a sensation of
pounding in the throat.
A 19-year-old man presents to the Emergency
Department with a fast pounding in his throat
[ Q: 116 ] MasterClass Part2
and presyncope. He is found to be tachycardic
and his ECG is as shown.
(2010) - Cardiology
A 79-year old female with
known aortic stenosis
(aortic valve gradient 60
mmHg 2 years previously),
presents with uncontrolled
atrial fibrillation (140
bpm). She is dyspnoiec on
exertion and on
examination has an
elevated jugular venous
pressure and basal
crepitations.

Which TWO of the


following are indicated as
part of the initial
What is the most likely arrhythmia? treatment regimen?

A. Atrial fibrillation (AF) A. Angiotensin-converting-enzyme inhibitors

B. Ventricular tachycardia (VT) B. Amiodarone

C. Atrial tachycardia C. Beta-blockers

D. Atrio-ventricular nodal re-entry D. Digoxin


tachycardia (AVNRT) E. Flecainide
E. Atrial flutter. F. Intravenous diuretics
G. Intravenous nitrates
Answer & Comments
H. Valvuloplasty
Correct answer: D
I. Verapamil

The ECG shows a regular narrow complex J. Warfarin.


tachycardia at a rate of 170/min. This excludes
VT (broad complex) and AF (irregular). Although Answer & Comments
atrial flutter and atrial tachycardia are possible
Correct answer: BF
diagnoses there are no visible p waves, which
would be expected.
Negative inotropic drugs and arterial
The most likely diagnosis by far is AVNRT, which
vasodilators should be avoided if possible in
is caused by a re-entry circuit formed within the
patients with significant aortic stenosis. The
AV node causing atria and ventricles to be
definitive treatment for symptomatic aortic

Dr. Khalid Yusuf El-Zohry – Sohag Teaching Hospital (01118391123) Page | 157
El-Zohry MRCP Questions Bank (Part 2) – Medical Masterclass 2010

stenosis is valve replacement. In adults, E. None of the above.


valvuloplasty has produced very disappointing
results. If patients develop atrial fibrillation, Answer & Comments
attempts should be made to restore sinus
rhythm and thereby maintain the atrial Correct answer: E
component to ventricular filling. Amiodarone
has a much lesser negative inotropic effect than The generator has eroded through the chest
class I agents. Diuretics are required in patients wall skin. This denotes infection and therefore
with evidence of fluid overload, but caution requires extraction of the whole pacemaker
must be exercised to avoid hypovolaemia and system, i.e. the generator and leads. It is
thereby reduce left ventricular diastolic filling inevitable that the leads will be infected as well.
pressures (and therefore cardiac output). Replacement of the generator alone will be
Anticoagulation would be required as inadequate.
prophylaxis in view of the atrial fibrillation, but
Pacing system infection is rare (<1%) but a very
one should aim to obtain immediate
serious complication. Endocarditis associated
therapeutic anticoagulation with heparin (or
with infected pacing systems carries a mortality
LMWH).
of a third. This system will need explanting
using specialized lead extraction tools in an
[ Q: 117 ] MasterClass Part2 expert centre. The procedure carries with it a
(2010) - Cardiology mortality of up to 5%.

A 75-year-old man had a pacemaker implanted


18 months earlier. He presents with pain over [ Q: 118 ] MasterClass Part2
his pacemaker. This is what you find (see (2010) - Cardiology
image).
A 55-year-old woman has a 2-year history of
exertional chest pain and has a positive exercise
test with significant ST segement depression.
She then has a coronary angiogram which
demonstrates normal coronary arteries. She is
told she has Cardiac Syndrome X.

Which one of the following statements about


Syndrome X is NOT correct?
A. Patients have typical symptoms of
occlusive coronary artery disease.
B. Myocardial perfusion imaging is usually
What is the appropriate management? normal.

A. Intravenous antibiotics and closure of the C. Life expectancy is normal.


wound D. Beta blockers, calcium antagonists and
B. Reassurance nitrates have all been shown to improve
symptoms.
C. Replace pacemaker generator and treat
with antibiotics E. It is caused by coronary microvascular
dysfunction.
D. Sterile dressings and wait for healing via
secondary intention

Dr. Khalid Yusuf El-Zohry – Sohag Teaching Hospital (01118391123) Page | 158
El-Zohry MRCP Questions Bank (Part 2) – Medical Masterclass 2010

Answer & Comments Whenever you approach a patient with a broad


complex tachycardia it is always safest to
Correct answer: B
presume that they have ventricular tachycardia
until proven otherwise, and indeed this is more
Cardiac Sydrome X is characterised by typical likely than one of the other possibilities above,
exertional angina, ST changes on ECG, and which may produce similar traces on a Holter
normal coronary arteries on angiography. It is monitor.
associated with microvascular abnormalities
such as diabetes and hypertension and thought Patients are not always syncopal with
to be due to microvascular dysfunction. Life ventricular tachycardia.
expectancy is normal. Right ventricular outflow tract tachycardia
About 50% of patients with syndrome X will usually presents with just palpitations. It occurs
have abnormal myocardial perfusion imaging. as a result of a triggered focus in the right
ventricular outflow tract and generally carries
an excellent prognosis. It is best treated with
[ Q: 119 ] MasterClass Part2 radiofrequency ablation. This is in contrast to
(2010) - Cardiology ischaemic ventricular tachycardia which carries
a very poor prognosis unless treated
A 45-year-old man is referred by his general
appropriately. In the context of impaired left
practitioner with palpitations. He has no other
ventricular function this invariably means with
associated symptons and specifically he is not
an implantable cardioverter defibrillator (ICD).
presyncopal. Holter monitoring has
demonstrated short non-sutained runs of a
monomorphic broad complex tachycardia. [ Q: 120 ] MasterClass Part2
(2010) - Cardiology
Which TWO of the following are the most likely
arrhythmias? You suspect that a 61-year-old woman with a
painful swollen leg, three weeks after femoral
A. Sinus tachycardia
hernia repair, has deep vein thrombosis.
B. Atrial fibrillation with intermittent rate
associated bundle branch block Which TWO of the following statements are
NOT correct?
C. Right ventricular outflow tract
tachycardia A. Virchow's pathophysiological triad is still
valid today
D. Atrial flutter with one to one conduction
B. Patients with early oedema are most
E. Wolff-Parkinson-White syndrome
likely to have residual thrombosis
F. Ischaemic ventricular tachycardia
C. Risk for thrombosis after surgery returns
G. Atrioventricular nodal reentry to normal within 2 weeks provided the
tachycardia patient has regained full mobility
H. Ventricular Fibrillation D. After a first spontaneous DVT the annual
likelihood of recurrence is 5 -15%
I. Torsades de pointes
E. Abnormalities in the coagulation system
J. Atrioventricular reentry tachycardia.
can be detected in around one quarter of
patients with a first spontaneous
Answer & Comments
thrombosis
Correct answer: CF

Dr. Khalid Yusuf El-Zohry – Sohag Teaching Hospital (01118391123) Page | 159
El-Zohry MRCP Questions Bank (Part 2) – Medical Masterclass 2010

F. Low molecular weight heparin is effective C. Give diuretic and digoxin; perform DC
and safe in both medical and surgical cardioversion within next 24 hours.
patients
D. Give diuretic and digoxin; initiate
G. The gold standard for diagnosis of DVT anticoagulation; perform cardioversion in
remains contrast venography 6 weeks.
H. Ultrasonography is less sensitive and less E. Give diuretic; start full-dose intravenous
specific than venograph heparin; perform DC cardioversion within
next 24 hours.
I. Recurrent ipsilateral DVT can be confused
clinically with the post-thrombotic
syndrome Answer & Comments

J. Magnetic resonance venography may Correct answer: D


prove useful for diagnosis in future
In a patient with a probable first episode of AF
Answer & Comments an attempt should be made to restore sinus
rhythm. However, if AF has been present for
Correct answer: CE
>48 hrs – as seems very likely in this case – it is
necessary to anticoagulate before
The increased risk of thrombosis persists for cardioversion, unless it can be documented by
several months after surgery. About half of trans-oesophageal echocardiography (TOE) that
patients with a first spontaneous thrombosis the left atrium is free of thrombus. If this were
are found to have an abnormality in the the case, then cardioversion within the next 24
coagulation system. hours would be the preferred management
option, with warfarinisation for a month
[ Q: 121 ] MasterClass Part2 afterwards (which is a high risk period for
(2010) - Cardiology thromboembolism).

A 68-year-old woman presents with 6 weeks of


[ Q: 122 ] MasterClass Part2
increasing breathlessness. She is in atrial
fibrillation at rate of 120/min and has signs of
(2010) - Cardiology
mild/moderate cardiac failure. She gives no You find a middle aged man on a path in a park.
history of previous arrhythmia or to suggest He has no pulse and is not breathing.
ischaemic heart disease, but has had mild
hypertension for at least 5 years. There are no Which is the appropriate next step:
cardiac murmurs. There are no ischaemic
A. Give two rescue breaths and initiate CPR
changes on the ECG. The chest radioagraph
at a ratio of 15 compressions to 2 breaths
shows a normal sized heart and mild pulmonary
oedema. She is not thyrotoxic. B. Give two rescue breaths and initiate CPR
at a ratio of 30 compressions to 2 breaths
Select the best of these management options:
C. Give a precordial thump
A. Perform DC cardioversion within next 24
D. Give two rescue breaths and go for help.
hours.
E. Go to call 999 (emergency services)
B. Give sotalol with aim of achieving
immediately.
‘chemical cardioversion’.

Dr. Khalid Yusuf El-Zohry – Sohag Teaching Hospital (01118391123) Page | 160
El-Zohry MRCP Questions Bank (Part 2) – Medical Masterclass 2010

Answer & Comments [ Q: 124 ] MasterClass Part2


Correct answer: E (2010) - Cardiology
A 32-year-old woman with a history of
The guidelines on basic life support identify the congenital heart disease is planning to start a
importance of early access to defibrillation in family. She has no other significant medical
cardiac arrest. They therefore suggest that no history and is currently symptomatically stable
CPR is commenced until a call for emergency on a small dose of loop diuretic.
services has been made and the potential for
early defibrillation is made possible. Once CPR Which one of the following conditions is
begins the ratio of compressions to ventilations associated with an unacceptably high risk in
is 30 to 2. A precordial thump is not indicated in pregnancy?
the unwitnessed collapse. A. Corrected tetralogy of Fallot
B. Perimembranous ventricular septal
[ Q: 123 ] MasterClass Part2 defect
(2010) - Cardiology
C. Secundum atrial septal defect
A 20-year-old female student presents with
D. Coarctation of the aorta
central chest pain after four days of a ‘flu-like
illness. E. Pulmonary hypertension of any cause

The most likely diagnosis is: Answer & Comments


A. Acute viral pericarditis
Correct answer: E
B. Gastro-oesophageal reflux
C. Acute myocardial infarction Pulmonary hypertension of any cause is
associated with a 50% maternal mortality.
D. Systemic lupus erythematosus Other high-risk cardiac conditions are severe
E. Pulmonary embolism. mitral, pulmonary or aortic stenosis and
Marfan's syndrome with a dilated aortic root.
Answer & Comments Intracardiac shunts without pulmonary
hypertension are relatively safe but require
Correct answer: A closer follow-up. Operated tetralogy of Fallot
without residual pulmonary stenosis is lower
The combination of a young patient and a ‘flu- risk; coarctation is associated with dissection
like illness make acute viral pericarditis the and is intermediate risk.
most likely diagnosis in this case. The chest pain
of pericarditis can be indistinguishable from [ Q: 125 ] MasterClass Part2
that of myocardial infarction, excepting that
(2010) - Cardiology
sitting forward often eases it.
The key physical sign to elicit would be a An 85-year-old lady presents to the A&E
pericardial rub, and the key initial investigation department with an episode of syncope. An
would be the ECG, looking for widespread ST ECG shows a left bundle branch block pattern
segment elevation, concave upwards. and her echocardiogram reveals a heavily
calcified aortic valve with restricted movement.

Which of the following statements is correct?

Dr. Khalid Yusuf El-Zohry – Sohag Teaching Hospital (01118391123) Page | 161
El-Zohry MRCP Questions Bank (Part 2) – Medical Masterclass 2010

A. A peak aortic valve gradient of <50mmHg B. Pulmonary artery wedge pressure


indicates that the aortic stenosis will not <20mmHg
require surgical treatment.
C. Creatinine clearance <50 ml min-1
B. A peak aortic valve gradient of >50mmHg
D. Previous alcoholism
confirms that the aortic stenosis is the
cause but her old age means that medical E. Hypertension.
therapy is the safest option.
Answer & Comments
C. Syncope carries a poor prognosis in aortic
stenosis and she requires urgent Correct answer: C
assessment for aortic valve replacement.
D. Ballon valvotomy may avoid the need for Patients with heart failure are considered for
surgey and should be the initial approach. cardiac transplantation when significant
symptoms persist despite maximal medical
E. Her left bundle branch block on ECG
therapy. Due to the shortage of donors clinical
suggests that transient bradycardia is
guidelines have been established highlighting
more likely to be the cause of her
patients most suitable for transplantation.
syncope and should first be investigated.
Significant impairment of renal function is
generally considered to be a contraindication,
Answer & Comments assuming this is not reversible. Patients with
Correct answer: C prior history of cancer may be considered if
there is no evidence recurrence (> 5 years on
from diagnosis of cancer).
In some patients with aortic stenosis the peak
LV to Aortic gradient can be misleading because
of reduced cardiac output due to impaired left [ Q: 127 ] MasterClass Part2
ventricular function. In such cases the (2010) - Cardiology
calculated aortic valve area is more useful as a
guide to severity. A 70 year old woman presents with 8 hours of
chest pain. Her pulse rate is 40/minute and
In all cases of aortic stenosis the development blood pressure 105/85. The ECG shows
of syncope signifies a poor prognosis and complete heart block, ST segment elevation and
urgent surgery is usually needed to replace the Q waves in leads II, III and AVF.
valve. Balloon valvotomy has been used
successfully in adolescent non-calcified valvular Which of the following statements is correct?
stenosis, but is rarely used in adults and not in
A. Atropine should be given immediately.
heavily calcified valves.
B. An isoprenaline infusion should be set up
immediately.
[ Q: 126 ] MasterClass Part2
(2010) - Cardiology C. Thrombolysis should be given
immediately.
A 45-year-old man with dilated cardiomyopathy
D. Thrombolysis should be avoided because
is being considered for cardiac transplantation.
she has completed her myocardial
Which of the following is generally deemed to infarction.
be a contraindication to cardiac E. Thrombolysis should be avoided because
transplantation? she may require a temporary pacing wire.
A. Any previous cancer
Dr. Khalid Yusuf El-Zohry – Sohag Teaching Hospital (01118391123) Page | 162
El-Zohry MRCP Questions Bank (Part 2) – Medical Masterclass 2010

Answer & Comments [ Q: 129 ] MasterClass Part2


Correct answer: C (2010) - Cardiology
A 79-year-old woman is admitted via A&E
The top priority is to achieve myocardial having been found on her floor at home. Her
reperfusion. The presence of chest pain and ST neighbours had not seen her for at least 48
segment elevation, despite Q waves, on ECG hours. Her core temperature was 30ºC.
indicate that thrombolysis is needed
immediately. Her ECG traces might show?
A. Atrioventricular block
[ Q: 128 ] MasterClass Part2 B. Bradycardia
(2010) - Cardiology
C. ST segment changes
An asymptomatic 45-year-old patient has an
D. Ventricular fibrillation
echocardiogram as part of a screening
programme. A suspected left atrial myxoma is E. All of the above.
identified.
Answer & Comments
What is the optimal initial treatment strategy?
Correct answer: E
A. Aspirin and repeat echo in 1-2 months
B. Chemotherapy The 'classical' feature of the ECG in
C. Immediate surgical removal hypothermia is the 'J' wave, (although this is
not pathognomic of hypothermia). As the
D. Repeat echo in 1-2 months
cooling becomes more profound, the QRS
E. Warfarinisation. complex broadens. There may be ST elevation
or depression and T wave inversion, which
Answer & Comments worsens with acidaemia. Second degree and
third degree atrioventricular block may evolve.
Correct answer: C Ventricular fibrillation becomes more common
with lower core temperatures.
Atrial myxomas are the commonest form of
cardiac tumour. They far more commonly arise [ Q: 130 ] MasterClass Part2
in the left atrium, most frequently attached to
(2010) - Cardiology
the inter-atrial septum. Systemic embolic
complications are common and can be A 59-year-old man is admitted breathless
devastating. Hence, unless severe co-morbidity following an episode of syncope while
precludes anaesthesis, immediate surgical shopping. There is no previous history of
removal is advocated. syncope. Past history includes multiple stab
Patients may present with dyspnoea, evidence ligations for bilateral varicose veins 4 weeks
of systemic embolisation or constitutional upset previously, mild asthma and borderline
with fever and elevated inflammatory markers. hypertension (not on treatment). He has
received 500 mL of volume expansion in the
Emergency Department.
On examination he is breathless at rest,
apyrexial, pulse 104 bpm (regular) and BP 90/50
mmHg. His heart sounds are normal. Chest

Dr. Khalid Yusuf El-Zohry – Sohag Teaching Hospital (01118391123) Page | 163
El-Zohry MRCP Questions Bank (Part 2) – Medical Masterclass 2010

examination reveals a few scattered wheezes.


ECG shows sinus tachycardia with inverted T
waves in leads V1-V3. CXR is normal. Oxygen
saturation is 92% on 40% oxygen. Peak
expiratory flow rate is 300 L/min.

What investigation would be most helpful in


directing his immediate treatment?
A. CT pulmonary angiogram
B. Serum D-dimer
C. Transoesophageal echocardiography
D. Serum Troponin
E. Ultrasound of leg veins (bilateral)

Answer & Comments

Correct answer: A

This clinical presentation is highly suggestive of


life-threatening pulmonary embolism. He
remains hypotensive (he normally tends to be
hypertensive) and tachycardic. The next step
must be to confirm the presence of proximal
pulmonary embolism and to consider
thrombolysis (surgery 4 weeks previously for
varicose vein stab ligation is not a
contraindication). CT pulmonary angiography
would be the most desirable investigation.
Whilst transthoracic echocardiography would
be of value in demonstrating a dilated right
heart, this is not specific for pulmonary embolic
disease and can be seen in other instances, for
example severe pneumonia.
Transoesophageal echocardiography would
carry substantial risk in this haemodynamically
compromised man. An elevated serum troponin
(right ventricular ischaemia) in the context of
pulmonary embolism indicates a worse
prognosis and some would suggest that this
would support administration of thrombolytic
therapy.

Dr. Khalid Yusuf El-Zohry – Sohag Teaching Hospital (01118391123) Page | 164
El-Zohry MRCP Questions Bank (Part 2) – Medical Masterclass 2010

5. Drug-associated diarrhoea (e.g.


Gastroenterology nelfinavir)
6. Idiopathic diarrhoea (HIV enteropathy).
(150 Questions)
CMV colitis is uncommon in patients who are
(Medical Masterclass – Part 2) not severely immunocompromized. GI tract
involvement may occur alone or in the setting
of disseminated disease. CMV has three major
patterns of infection: primary infection (in the
[ Q: 1 ] MasterClass Part2 immunocompetent host causing few or no
symptoms; after initial infection, latent viral
(2010) - Gastroenterology
infection occurs), secondary reactivation
A 34-year-old HIV positive man, poorly (patients seropositive with latent infection who
compliant with treatment, presents to the reactivate because of immunodeficiency) and
gastroenterologists with diarrhoea. superinfection. Within the colon, ulcerative
Sigmoidoscopy revealed mucosal inflammation changes may be seen and watery diarrhoea can
with focal haemorrhage, oedematous folds and develop. As ulcers increase in depth, erosion
polypoid lesions. Inclusion bodies were visible into blood vessels may cause profuse bloody
on examination of a biopsy. diarrhoea. Inflammatory polyps do develop,
which, rarely, obstruct the colon. Severe
What is the most likely cause of his symptoms inflammation and vasculitis sometimes leads to
and signs? ischaemia and transmural necrosis, with
A. Cytomegalovirus (CMV) colitis subsequent perforation and peritonitis.
Classically findings on microscopy include giant
B. Cryptosporidiosis cells with large ovoid or pleomorphic nuclei
C. Amoebiasis containing basophilic inclusions (owl's eyes).
D. Disseminated mycobacterium avium
intracellulare (MAI) [ Q: 2 ] MasterClass Part2
E. Ulcerative colitis. (2010) - Gastroenterology
A 27-year-old mother of three presents in the
Answer & Comments outpatient clinic with a 2-year history of
alternating constipation and diarrhoea
Correct answer: A
associated with abdominal bloating.

The broad differential diagnosis for HIV Which of the following statements are true?
associated diarrhoea includes:
A. Nocturnal diarrhoea is highly suggestive
1. Bacterial infections (Salmonella, of irritable bowel syndrome.
Shigella, Campylobacter)
B. A past history of sexual abuse may be
2. Parasitic infections (Cryptosporidium, relevant.
Isospora, Giardia, Microsporidia,
C. Recent weight gain is a sinister feature.
Entamoeba histolytica)
D. Barium follow-through is the
3. Mycobacterial infections
investigation of choice.
(Mycobacterium avium complex (MAC),
Mycobacterium tuberculosis) E. Allergy testing should be performed in
order to exclude dietary precipitants.
4. Viral infections (Cytomegalovirus)

Dr. Khalid Yusuf El-Zohry – Sohag Teaching Hospital (01118391123) Page | 165
El-Zohry MRCP Questions Bank (Part 2) – Medical Masterclass 2010

F. Full colonoscopy is mandatory in order to woman of this age, GI investigation should take
exclude Crohn’s disease. place.
G. Microcytic anaemia can be safely Complementary therapies may help lower
attributed to her history of menorrhagia. stress levels in patients and can bring about
improvement in symptoms. There is little
H. Complimentary therapies should be
evidence to support the effect of currently
discouraged as they are likely to interfere
available pharmacological treatments much
with standard therapy.
beyond a placebo effect. While the concept of
I. Systemic yeast infection should be systemic fungal infection has gained popularity
excluded. with some alternative practitioners, there is
J. Biofeedback techniques may be beneficial little scientific evidence to support this.
in symptom control. Biofeedback techniques have been
demonstrated to improve patient symptoms,
Answer & Comments but are time-consuming to teach and not
available in all centres.
Correct answer: BJ

[ Q: 3 ] MasterClass Part2
Nocturnal diarrhoea is not a feature of IBS and
should raise suspicion of inflammatory bowel (2010) - Gastroenterology
disease. A 78-year-old woman is admitted on take
Some studies have identified an increased because she is unable to cope at home. She
incidence of past sexual abuse in patients with says that she feels ‘a bit weak’, but admits to no
IBS as compared to controls. other symptoms. On examination she looks as
though she has lost a lot of weight and is
Documented, unexplained weight loss is a
jaundiced, but there are no other abnormal
cause for concern, but weight gain is rarely
physical signs.
significant.
Barium follow-through involves considerable The most likely diagnosis is:
radiation exposure to a woman of childbearing A. Gall stones
age and should be reserved for those
individuals who have objective evidence or B. Carcinoma of the stomach with hilar
histories highly suggestive of small bowel lymph nodes
Crohn's disease. C. Chronic pancreatitis
While some patients will have food D. Carcinoma of the pancreas
intolerances, these are not classical allergies
E. Cholangiocarcinoma.
and will not be identified by skin-testing.
Elimination diets may be useful, but are time-
consuming and may be disruptive to the Answer & Comments
patient's lifestyle. Correct answer: D
In many patients, flexible sigmoidoscopy and
random rectal biopsies may be sufficient to Pancreatic carcinoma classically causes painless
exclude significant colonnic disease. jaundice, but the absence of pain does not
Microcytic anaemia cannot be ignored, and exclude the diagnosis of gallstones. The other
while gynaecological causes are more likely in a diagnoses listed can all cause jaundice but are

Dr. Khalid Yusuf El-Zohry – Sohag Teaching Hospital (01118391123) Page | 166
El-Zohry MRCP Questions Bank (Part 2) – Medical Masterclass 2010

much less likely to be the explanation in this The blood count shows a normocytic anaemia,
case. consistent with the anaemia of chronic
disorders and not suggestive of iron deficiency.
If pancreatic cancer is the diagnosis, then liver
blood tests are likely to reveal elevation of Acute loss of blood does not lead to any
alkaline phosphatase (ALP) and gamma- immediate change in the full blood count:
glutamyl transpeptidase (GGT). Elevation of haemodilution takes some time to occur, hence
aspartate transaminase (AST) and alanine the full blood count can never be used to
transaminase (ALT) will be less marked, and decide whether or not someone has suffered a
they may be normal. The key investigation is significant acute haemorrhage. Physical
ultrasonography, which is likely to reveal examination will tell you this, the two most
dilatation of intra- and extra-hepatic bile ducts. reliable signs of intravascular volume depletion
Endoscopic retrograde being postural hypotension (lying and sitting)
cholangiopancreatography (ERCP) or CT and a reduced jugular venous pressure.
scanning should confirm the diagnosis.
[ Q: 5 ] MasterClass Part2
[ Q: 4 ] MasterClass Part2 (2010) - Gastroenterology
(2010) - Gastroenterology
A 48 year-old woman is admitted as an
A 76-year-old man with a long history of emergency with a history of epigastirc pain
rheumatoid arthritis is admitted after a radiating through to her back and associated
haematemesis. His full blood count is as with vomiting. A diagnosis of acute pancreatitis
follows: Hb 10.8 g/dl, MCV 86 fl, WBC 7.4 x is made when her serum amylase is found to be
109/l, platelets 243 x 109/l. more than 3 times the upper limit of
normal. She is agitated and hypotensive
Which of the following statements is true? despite an initial fluid challenge, and a decision
A. The normal platelet count makes it is made by the admitting team to transfer her
unlikely that he has had a significant to the intensive care unit.
gastrointestinal bleed.
Which of the following would support a
B. The normal blood count is reassuring: he diagnosis of severe acute pancreatitis?
is unlikely to have had a significant
A. Albumen <42g/l
gastrointestinal bleed.
B. Calcium < 2.5mmol/l
C. The blood count does not tell you
whether or not he has had a significant C. CRP >210mg/l
gastrointestinal bleed.
D. Lactate dehydrogenase > 200 units/l
D. The blood count suggests iron deficiency
E. PaO2 < 90mmHg (on air)
anaemia.
E. The high platelet count is consistent with Answer & Comments
recent gastrointestinal haemorrhage, but
could also reflect activity of his arthritis. Correct answer: C

Answer & Comments The standard method of assessing severity of


acute pancreatitis is by using the
Correct answer: C Glasgow/Ranson criteria. Three or more of the
following criteria, based on initial admission

Dr. Khalid Yusuf El-Zohry – Sohag Teaching Hospital (01118391123) Page | 167
El-Zohry MRCP Questions Bank (Part 2) – Medical Masterclass 2010

score and subsequent repeat tests over 48 Answer & Comments


hours, constitutes severe disease
Correct answer: D
 Age >55 years
 White blood cell count >15 x10 9/l This man has chronic treatment-resistant
colitis. He might be best served by colectomy,
 Glucose>10 mmol/l
which because he almost certainly has
 Urea>16 mmol/l ulcerative colitis, should be curative. The
 PaO2 <60 mmHg depressed serum albumin and haemoglobin
level also suggest that he should be
 Calcium <2 mmol/l investigated for the possibility that he may have
 Albumen <32 g/l developed a complication of long-standing
severe ulcerative colitis, such as Primary
 Lactate dehydrogenase >600 units/l
Sclerosing Cholangitis, or colorectal cancer. The
 Aspartate/alanine aminotransferase other treatments are inappropriate.
>100 units
Human CD31 is an adhesion molecule
A CRP>210mg/L in first 4 days of attack expressed on platelets, endothelial cells,
indicates severe disease, and an APACHE score leukocytes and their bone marrow precursors.
of 9 or more also constitutes a severe attack. Although many biological agents have and
(See Medical Masterclass, Gastroenterology continue to be considered in the treatment of
and Hepatology Module, Section 2.4.1) colitis, to date CD31 has not been targeted.

[ Q: 6 ] MasterClass Part2 [ Q: 7 ] MasterClass Part2


(2010) - Gastroenterology (2010) - Gastroenterology
A 30-year-old man with known pan-colitis A 78-year-old woman presents with progressive
(histologically ulcerative colitis with rectal dysphagia to solids, anaemia and 10kg weight
involvement and normal terminal ileum) is seen loss.
in clinic following a recent admission for
intravenous steroids. Despite continuing on Which of the following statements are true?
azathioprine, Asacol and low dose oral steroid A. The patient is unlikely to be able to
treatment, he continues to have his bowels perform a barium swallow.
open 10 times per day and is losing weight. His
serum albumin is 20 g/l and haemoglobin 9 B. The patient is most likely to have a
g/dl. Previously he has developed side effects foreign body in the oesophagus.
on ciclosporin. C. The anaemia is unlikely to be related to
the oesophageal diagnosis.
Appropriate advice would be to recommend:
D. Histology is not required to make a
A. Trial of live yoghurt with probiotics diagnosis.
B. Addition of asacol foam enemas E. Surgery is the treatment of choice in the
C. Prescription of anti-CD31 monoclonal majority of patients.
antibody therapy F. Dilatation at initial endoscopy may be
D. Referral to lower GI surgeon to discuss required.
colectomy
E. Implementation of strict elemental diet

Dr. Khalid Yusuf El-Zohry – Sohag Teaching Hospital (01118391123) Page | 168
El-Zohry MRCP Questions Bank (Part 2) – Medical Masterclass 2010

G. Endoscopic perforation rate is higher in [ Q: 8 ] MasterClass Part2


malignant strictures than in benign peptic (2010) - Gastroenterology
strictures.
A 56-year-old man is admitted with melaena,
H. There is no role for localised
tachycardia and hypotension.
radiotherapy.
I. A misplaced wall stent can easily be Which of the following are true?
removed and repositioned. A. Consider an urgent abdominal CT with
J. Overall 5-year survival rate is contrast if he has previously had an
approximately 20%. abdominal aortic aneurysm repair.
B. Intravenous omeprazole is indicated.
Answer & Comments
C. If the ulcer base is clean, there is a 20%
Correct answer: FG risk of re-bleeding.
D. A normal haemaglobin level signifies a
Barium swallow should be the investigation of minor bleed.
choice and should be arranged as a matter of
E. The overall mortality rate in the UK for
urgency in this scenario. Perforation rate is
upper gastrointestinal bleeding is about
higher with malignant strictures than with
5%.
benign strictures, and the swallow can guide
the endoscopist to the level at which the
tumour is likely to be found. Many patients will Answer & Comments
be anaemic due to slow but constant blood loss Correct answer: A
from the friable surface of the tumour.
Endoscopic biopsies are essential to confirm the
A suspected aorto-enteric fistula requires
diagnosis and differentiate between squamous
urgent investigation. If endoscopy reveals a
and adenocarcinomas.
clean ulcer base, the re-bleed rate is less than
Less than a third of patients are amenable to 5%. Stigmata of recent haemorrhage on
surgery due to advanced disease or comorbidity endoscopy (fresh clot, visible vessel or active
in this elderly group of patients. In patients with bleeding) are indications for intravenous
advanced dysphagia, it may be necessary to omeprazole. These findings are also indications
dilate at the initial biopsy though this carries a for endoscopic therapy. The overall mortality
risk of perforation. Other palliative measures rate for upper gastrointestinal bleeding in the
include the placement of expanding UK is 14%.
oesophageal wall-stents, laser
photocoagulation or radiotherapy (either
[ Q: 9 ] MasterClass Part2
external beam, or implantation of radioactive
wires into the tumour). Once sited, wall stents
(2010) - Gastroenterology
cannot be removed. Tumour ingrowth may A 35-year-old woman presents with abdominal
require further stents to be placed within pain and vomiting. She has been unwell for
existing ones. about six weeks with anorexia, nausea and
The diagnosis of oesophageal carcinoma has a weight loss. Examination reveals a tender
universally poop prognosis with median survival palpable mass in the right iliac fossa.
of less than a year, and 5-year survival rates less
The TWO most likely diagnoses are:
than 5%.
A. Psoas abscess

Dr. Khalid Yusuf El-Zohry – Sohag Teaching Hospital (01118391123) Page | 169
El-Zohry MRCP Questions Bank (Part 2) – Medical Masterclass 2010

B. Tuberculosis B. There is no clinical benefit to correcting


under nutrition in older patients.
C. Abdominal secondaries
C. Malnutrition develops when there is
D. Crohn’s disease
reduced nutritional intake only.
E. Campylobacter
D. Malnutrition does not increase length of
F. Caecal carcinoma stay in hospital.
G. Hypernephroma E. Malnutrition has no effect on
H. Appendix mass pyschological well-being.

I. Ovarian carcinoma F. Nutritional support is needed when


patients have lost 25% of their body
J. Lymphoma. weight.
G. Malnutrition impairs the cell mediated
Answer & Comments
immune response.
Correct answer: DH
H. BMI can distinguish fat mass from lean
mass.
The presentation with gastrointestinal
I. BMI may be misleading if there is muscle
symptoms occurs on a background suggesting a
wasting due to motor neurone disease.
systemic component to the illness, hence the
most likely diagnoses are an inflammatory mass J. The majority of hospital trusts ensure all
associated with Crohn’s disease or an appendix patients are weighed at least once-
abscess. In an older patient caecal carcinoma weekly.
would be a more likely explanation.
Aside from assessment of cardiovascular status Answer & Comments
(is resuscitation required?) and for the presence Correct answer: GI
of peritonism (which would suggest a
perforated viscus and the need for laparotomy),
Malnutrition is poorly recognised in hospital.
examine carefully for signs that would support
a diagnosis of Crohn’s disease. These include There is clinical benefit to correcting under
clubbing, aphthous ulceration, perianal skin nutrition in older patients.
tags / ulceration / fistulae and (less commonly)
Malnutrition may develop when intake is
seronegative arthritis, sacroiliitis, iritis and skin
reduced, there is increased nutrient
rashes (erythema nodosum, pyoderma
requirement or altered ability to utilise or
gangrenosum).
absorb nutrients.
Malnutrition increases length of stay in hospital
[ Q: 10 ] MasterClass Part2
and adversely affects psychological well-being
(2010) - Gastroenterology (depression anxiety etc).
An 87-year-old lady is admitted after a fall. Nutritional support is required when a patient
Physical examination shows her to be cachexic has lost 5 to 10% of their body weight and
with a body mass index (BMI) of less than 18. further weight loss is likely.

Which of the following statements are true? The cell-mediated immune response is impaired
in malnutrition.
A. Malnutrition is readily recognised in
hospital.

Dr. Khalid Yusuf El-Zohry – Sohag Teaching Hospital (01118391123) Page | 170
El-Zohry MRCP Questions Bank (Part 2) – Medical Masterclass 2010

BMI does not distinguish between fat mass and I. Globus, a functional GI disorder, may also
lean mass. present in this way.
BMI may be misleading if there is muscle J. Although oesophageal cancer is a possible
wasting due to motor neurone disease. diagnosis, this is unlikely if the patient is a
native of the United Kingdom, as the
In 2001 less than 50 % of trusts weighed their
incidence of this cancer, which is rare in
patients on a weekly basis.
any case, is falling.

[ Q: 11 ] MasterClass Part2
Answer & Comments
(2010) - Gastroenterology
Correct answer: AF
A middle-aged man presents with progressive
difficulty in swallowing. There is no preceding
The most common causes of dysphagia, which
history of heartburn. He has lost weight, and is
is a serious symptom that should prompt a
worried about having cancer.
rapid diagnostic and therapeutic response, are
Which of the following statements are true? peptic strictures, carcinoma of the oesophagus
or cardia, and oesophageal dysmotility.
A. You share his worry about cancer, Neurological disease may also cause difficulty in
although you maintain that the symptoms swallowing, and consequent weight loss, often
may be due to non-malignant disease. associated with choking, coughing, and
B. Barrett’s oesophagus and regurgitation. Globus, which describes the
adenocarcinoma are particularly unlikely sensation of a lump in the throat, is not
given the lack of preceding associated with weight loss.
gastroesophageal reflux. In middle-aged men, Barrett’s oesophagus and
C. To expedite management, a one-stop adenocarcinoma seem to be increasing in
clinic offering immediate endoscopy as incidence, and the conditions are often not
the next step, and subsequently preceded by a history of heartburn. This makes
appropriate initial treatment, would be it harder to rationally select patients for
ideal. screening programmes.

D. Neurological disorders such as motor Endoscopy performed before barium swallow


neurone disease may also cause difficulty or some other non-invasive means of
with swallowing, although they are not determining the oesophageal anatomy can be
associated with weight loss. disastrous, as a friable tumour may be
perforated on endoscopy. Although CT or MRI
E. As the patient does not volunteer
scans provide important information on the
heartburn or odynophagia, peptic
potential spread of disease beyond the
stricture is unlikely.
oesophagus, or involvement of the oesophagus
F. Oesophageal dysmotility syndromes can by thoracic disease, they are not first-line
cause dysphagia, and present exactly like investigations. Barium swallow, endoscopy, and
this. oesophageal manometry are the most helpful
G. A high resolution CT scan of the thorax is initial investigations, as they will provide a
now probably the best initial diagnosis.
investigational modality. Dysmotility syndromes are amenable to
H. Sadly, at this stage the outlook is endoscopic therapy, and some early neoplasms
hopeless. can be treated successfully, or palliated

Dr. Khalid Yusuf El-Zohry – Sohag Teaching Hospital (01118391123) Page | 171
El-Zohry MRCP Questions Bank (Part 2) – Medical Masterclass 2010

remarkably by a combination of surgery, I. The most likely diagnosis is diarrhoea


endoscopic therapy, and supportive care. predominant IBS (IBS-D), and the advent
of new treatments for this condition will
Unfortunately, while squamous cell carcinoma
probably modify our approach to patients
of the oesophagus is rare in the UK,
such as this
adenocarcinoma of the oesophagus, associated
with Barrett’s oesophagus, seems to be J. The clinical scenario strongly suggests
increasing in incidence among men in the dumping syndrome and intestinal hurry,
Western world making diabetes mellitus a strong
possibility
[ Q: 12 ] MasterClass Part2
(2010) - Gastroenterology Answer & Comments

A 22-year-old patient who is otherwise in good Correct answer: CD


health complains that certain foods cause her
to suffer severe abdominal pain and develop A patient complaining of abdominal pain and
diarrhoea shortly after eating. She is convinced altered bowel habit could certainly have a form
that she has a yeast infestation. of irritable bowel syndrome (IBS). However, the
diagnosis cannot be established without some
Which of the following could be recommended consideration of alternative explanations, as
initially? exclusion of organic pathology remains a
A. A short course of treatment with diagnostic criterion. Although there is currently
fluconazole, which should eliminate a great deal of interest in the use of specific 5-
candidal infection, if it were present, is a HT3 receptor antagonists in the treatment of
reasonable first step. IBS with diarrhoea, the recent experience of
toxicity associated with Alosetron (which was
B. The symptoms are typical of irritable withdrawn from use) suggests that physicians
bowel syndrome, and should be treated should be cautious about introducing related
symptomatically, without further agents as the first line of therapy. Amitriptyline,
investigation. used at a low dose, offers some analgesic effect
C. A careful dietary history would be helpful in IBS. Nonetheless, it seems unsuitable as an
initial treatment without a clear diagnosis, and
D. The symptoms are compatible with
would have no effect on the diarrhoea.
lactose intolerance
The patient’s symptoms are not typical of
E. Coeliac disease is highly likely particularly
coeliac disease, where the diarrhoea is not
if the patient has a family history
usually related so closely to eating, or to
F. A therapeutic trial of treatment with giardiasis, where cramping abdominal pains,
metronidazole, for presumed giardiasis is wind, and diarrhoea are usually unrelated to
a reasonable first step eating. Abdominal pain, faintness, nausea, and
G. Amitriptyline, used at a relatively low possibly diarrhoea may occur in the dumping
dose, particularly at night, may be helpful syndrome, however, in this context it seems a
for the abdominal pain distant possibility. Abdominal pain is a typical
feature of peptic ulceration and Helicobacter
H. Peptic ulcer disease or Helicobacter pylori associated intestinal disease, and
pylori associated gastritis are unlikely depending on the demographics of one’s
because they do not cause diarrhoea practice, it may be a frequent cause of
symptoms such as this. Peptic ulceration

Dr. Khalid Yusuf El-Zohry – Sohag Teaching Hospital (01118391123) Page | 172
El-Zohry MRCP Questions Bank (Part 2) – Medical Masterclass 2010

associated with the Zollinger-Ellison syndrome J. Heavy or persistent bleeding from biopsy
may certainly cause diarrhoea, and the or polypectomy sites occurs in around 1
diagnosis cannot be ruled out entirely, but it is out of 1000 procedures.
a very rare disease. As the patient reports that
eating makes her unwell, a careful dietary Answer & Comments
history is indicated. Her symptoms are
consistent with lactose intolerance, and Correct answer: CJ
suspicion would be increased if the yeast she
was wary of was that found in cheese. A Colonoscopy is a very common and safe
positive lactose intolerance test (e.g. breath procedure. Risks are associated with any
test or duodenal biopsy for lactase activity) procedure and in this case include the
would clinch the diagnosis. following:
1. The perforation rate is ~ 1 in 500,
[ Q: 13 ] MasterClass Part2 resulting in leakage of bowel contents
(2010) - Gastroenterology into the abdomen. The risk is higher
when polyps are removed – about 1 in
A 62-year-old woman is being investigated for 100, depending on the size of the
anaemia and change in bowel habit. You are the polyp. If conservative management fails
doctor asked to consent her for colonoscopy. surgery can occasionally be necessary

Which TWO statements best reflect information 2. Following polypectomy a few develop
about the procedure that the patient may have severe abdominal pain and sepsis 12
concerns about? hours to 5 days after the procedure.
Diathermy injury may cause this, if the
A. The risk of death is 1 in 500.
polyp had been difficult to remove and
B. The commonest side effect is profuse whilst most settle within 48 hours,
bleeding. bowel rest and antibiotics are needed
C. Bowel perforation, requiring an in a few
operation, occurs in around 2 out of 3. Bleeding from the bowel occurs in 1 in
every 1000 procedures. 50 people, following biopsy and/or
D. The procedure is usually performed removal of polyps. This usually settles
without sedation. without further treatment but rarely
transfusion or surgery may be needed.
E. Bowel preparation is not required with For delayed bleeding, another
modern colonoscopes. colonoscopy may be necessary to halt
F. Colonoscopy is suitable for all patients bleeding at the polypectomy sites
without exception. 4. Bloating and mild pain and discomfort
G. If a polyp is found in the lower rectum is common
the procedure will be terminated. 5. Bacteraemia is rare
H. Colonoscopy is only performed when a 6. Side effects of sedation particularly in
diagnosis of cancer is suspected. the elderly, those with co-existent
I. It will be safe for this patient to go home cardiorespiratory illness or liver disease
unaccompanied following the procedure. must not be over looked, and
appropriate monitoring during and

Dr. Khalid Yusuf El-Zohry – Sohag Teaching Hospital (01118391123) Page | 173
El-Zohry MRCP Questions Bank (Part 2) – Medical Masterclass 2010

after the procedure must always take CA19-9 is neither highly sensitive nor specific,
place. and cytology brushings are positive in only 50-
70% of cases. Cross-sectional imaging is
[ Q: 14 ] MasterClass Part2 required to stage the disease, identify
metastases and plan management. Ascending
(2010) - Gastroenterology
cholangitis is common and should be treated
A 54-year-old man with a long history of appropriately. Metal stents, while remaining
ulcerative colitis and primary sclerosing patent for longer than plastic stents, are
cholangitis presents with obstructive jaundice, difficult to remove and should only be placed
weight loss, fever and abdominal pain. once surgery has been excluded. Where
Endoscopic retrograde drainage cannot be obtained at ERCP,
cholangiopancreatography (ERCP) reveals a percutaneous transhepatic cholangiography
tight stricture of the common bile duct. (PTC) may be necessary.
Cholangiocarcinoma is an absolute
Which are the TWO most appropriate
contraindication to transplantation as outcome
responses?
is universally poor. Two-year survival is under
A. CA19-9 is highly specific and sensitive for 20%.
cholangiocarcinoma.
B. Cytology brushings at ERCP have a 95% [ Q: 15 ] MasterClass Part2
pick-up rate. (2010) - Gastroenterology
C. Drainage of the biliary system may be
A 52-year-old Asian woman is admitted
achieved percutaneously.
comatose with acute liver failure. Auto-
D. There is no need for cross sectional antibodies, viral serology, ferritin, copper and
imaging as the diagnosis is clear from the alpha-1-antitrypsin are all negative or normal.
ERCP.
Which of the following statements concerning
E. He should be commenced on broad-
hepatic drug toxicity is true?
spectrum antibiotics.
A. Acute liver failure can rarely be ascribed
F. Cholangiocarcinomas never metastasise. to anti-tuberculous therapy.
G. Patients with cholangiocarcinoma should B. A history of herbal remedies may well be
be referred urgently for consideration of
relevent.
liver transplant.
C. A course of clavulanic acid-amoxicillin
H. A metal stent should be inserted while completed 2 weeks ago can safely be
awaiting definitive surgery. excluded as the cause.
I. There is no place for radical surgery in the D. Prognosis is better than acute liver failure
management of cholangiocarcinoma. secondary to viral hepatitis.
J. Five-year survival rate for E. Liver transplantation is contraindicated.
cholangiocarcinoma is approximately
30%.
Answer & Comments

Answer & Comments Correct answer: B

Correct answer: CE
Always consider drug toxicity in the presence of
jaundice or abnormal liver biochemistry. This
Dr. Khalid Yusuf El-Zohry – Sohag Teaching Hospital (01118391123) Page | 174
El-Zohry MRCP Questions Bank (Part 2) – Medical Masterclass 2010

often means obtaining a detailed drug history Reflux oesophagitis is very common and easily
from the GP and asking family members about treated with simple postural measures and acid
herbal remedies and over-the-counter suppression.
medication. Acute liver failure has frequently
Most patients respond to treatment and there
been reported following anti-tuberculous
is no need to repeat endoscopy in every case.
therapy. Although there is usually a very clear
Severe disease, presence of oesophageal
temporal association between taking the drug
ulceration and follow up of Barrett’s
and the onset of hepatotoxicity, halothane and
oesophagitis are reasonable indications for
co-amoxiclav may present up to three weeks
repeat endoscopy.
after taking the drug. Prognosis is worse than
with liver failure secondary to acute viral There is no convincing evidence for routine
hepatitis. Helicobacter eradication in this clinical situation
and the lowest dose of acid suppression should
While the mainstay of treatment is supportive,
be given when symptoms have settled, if any is
the need for transplanataion should be borne in
required at all.
mind, and early discussions should take place
with a transplant unit.
[ Q: 17 ] MasterClass Part2
[ Q: 16 ] MasterClass Part2 (2010) - Gastroenterology
(2010) - Gastroenterology A 45-year-old man with a past history of
alcohol-related chronic liver disease presents
A 65-year-old man complains of epigastric pain
to the Emergency Department following a
and difficulty swallowing. At endoscopy Grade I
400ml fresh haematemesis. On examination he
reflux oesophagitis is seen.
is jaundiced with palmar erythema and marked
ascites. Pulse is 120 beats per minute and blood
Which one of the following statements is NOT
pressure 100/70 mmHg.
true?
A. Acid suppression with a proton pump In addition to fluid resuscitation, which of the
inhibitor should relieve symptoms. following treatments is most likely to be
beneficial in his initial management, while
B. Repeat endoscopy is required to ascertain
awaiting upper GI endoscopy?
response to treatment.
A. Ranitidine 50mg intravenously
C. Helicobacter eradication is not needed
unless there is associated duodenitis. B. Omeprazole 40mg intravenous bolus
D. Raising the head of the bed and avoiding C. Tranexamic acid 1g intravenously
eating/drinking 3 hours prior to going to D. Terlipressin 2mg intravenous bolus
bed can aid symptoms.
E. Propranolol 40mg orally.
E. The lowest dose of proton pump inhibitor
should be used in the long term to treat
Answer & Comments
his symptoms.
Correct answer: D
Answer & Comments
It is likely that this patient is bleeding from
Correct answer: B
oesophageal varices, resulting from alcohol-
induced portal hypertension.

Dr. Khalid Yusuf El-Zohry – Sohag Teaching Hospital (01118391123) Page | 175
El-Zohry MRCP Questions Bank (Part 2) – Medical Masterclass 2010

Terlipressin reduces the likelihood of continued F. Antibody titres against tTG decline as the
bleeding by reducing portal pressure, and may disease is controlled.
be helpful either prior to endoscopy, or as an
G. Dapsone inhibits activity of the tTG
adjunct to endoscopic therapy.
enzyme.
Intravenous omeprazole has been shown to
H. Antibodies to tTG are probably a false-
reduce the likelihood of peptic ulcer rebleeding
positive as the patient has Crohn’s
after endoscopic therapy, but there is no
disease.
evidence to support its use as an empirical
therapy prior to endoscopy or in the I. Antibodies to tTG are likely to be
management of variceal haemorrhage. suppressed initially, and will appear in the
chronic phase of the illness.
Tranexamic acid is an antifibrinolytic and has
been shown to provide a slight reduction in J. Antibodies to tTG are irrelevant as this is a
mortality following peptic ulcer haemorrhage in classical presentation of glucagonoma.
one meta-analysis, while intravenous ranitidine
has no impact on outcome of upper GI bleeding Answer & Comments
from any source.
Correct answer: CF
Propranolol is helpful in the primary and
secondary prevention of variceal haemorrhage
tTG antibodies are a reliable marker for active
but has no role in the acute setting.
coeliac disease, and modern enzyme-linked
immunosorbent assay (ELISA) tests have
[ Q: 18 ] MasterClass Part2 sensitivities and specifities approaching 100%.
(2010) - Gastroenterology tTG may play a pathogenic role in coeliac
disease, although so far no inherited mutations
In a patient with a bullous rash and altered associated with the disease have been
bowel habit, which of the following statements discovered.
regarding tissue transglutaminase (tTG) are Dermatitis herpetiformis, a bullous rash that is
relevant? associated with coeliac disease, may be caused
A. Coeliac disease is associated with by antibody reaction against a form of
mutations in the promoter region of the transglutaminase expressed in the skin.
tTG gene. Dapsone is effective against dermatitis
herpetiformis, but not against coeliac disease.
B. Overexpression of the tTG gene in
epithelial cells leads to activation of Gluten peptides are modified by tTG, creating
gluten peptides. immunodominant epitopes that stimulate
intestinal epithelial T cell activation and
C. IgA antibodies react with a form of this
proliferation.
enzyme in the skin, causing dermatitis
herpetiformis.
[ Q: 19 ] MasterClass Part2
D. An ELISA test for IgA antibodies against
tTG is easier to perform than (2010) - Gastroenterology
immunofluorescence, although it has a You admit a 45-year-old man who had a liver
lower sensitivity and specificity. transplant just over 3 months ago for primary
E. Antibodies to tTG are a marker for coeliac sclerosing cholangitis. He complains of fever,
disease, although the protein probably abdominal pain and diarrhoea, which has come
has no pathogenic role in the disease. on over the last week. He has a platelet count

Dr. Khalid Yusuf El-Zohry – Sohag Teaching Hospital (01118391123) Page | 176
El-Zohry MRCP Questions Bank (Part 2) – Medical Masterclass 2010

of 60 x 109/L and alanine transaminase (ALT) of A. Barrett’s oesophagus is found in over 5%


300 U/L with a normal bilirubin. He is taking of the asymptomatic population.
tacrolimus and prednisolone for
B. Yearly gastroscopy and multiple biopsies
immunosuppression, and tells you that he
reduce mortality in patients with long
recently stopped taking valganciclovir.
segment Barrett’s oesophagus.
What is the most likely diagnosis? C. The severity of reflux symptoms correlate
with the degree of inflammation and
A. Acute rejection of liver transplant
presence of Barrett’s oesophagus.
B. Chronic rejection of liver transplant
D. Short segment Barrett’s oesophagus does
C. Donor-acquired toxoplasmosis not have an increased risk of malignancy.
D. Donor-acquired cytomegalovirus E. Barrett’s oesophagus carries a 40-fold
E. Recurrence of primary sclerosing increased risk of squamous cell
cholangitis carcinoma.

Answer & Comments Answer & Comments

Correct answer: D Correct answer: A

Cytomegalovirus (CMV) infection is an No treatment has been shown to reduce the


important consideration in any organ transplant risk of malignancy or mortality rate in patients
recipient, particularly where the recipient has with Barrett’s oesophagus, which carries a 40-
never had infection but the donor is a carrier, fold increased risk of adenocarcinoma. The
since the recipient is then at risk of primary majority of patients die from unrelated causes.
infection. For this reason close attention is
given to prophylaxis, particularly in the first 3 [ Q: 21 ] MasterClass Part2
months after transplantation. When (2010) - Gastroenterology
prophylaxis is stopped there is still a chance of
infection, which in the case of liver transplant A 70-year-old woman presents with malaise
recipients will often present as fever, and loose stool, and a mild iron-deficiency
abdominal pain, diarrhoea (colitis), anaemia is noted. She has had vague limb pains
breathlessness (pneumonitis) and hepatitis. for a number of years. A colonoscopy is normal.
Other features of the illness can include A gastroscopy is macroscopically normal but a
haematological abnormalities, retinitis and duodenal biopsy taken shows blunting of villi
oesophagitis. Diagnosis is by quantifying CMV and inflammatory cells in the lamina propria.
viraemia in the blood. Treatment is with
intravenous ganciclovir and reduction of Which of the following are true of coeliac
immunosuppression if possible. disease in older people?
A. It is more common in Asian countries.
[ Q: 20 ] MasterClass Part2 B. Less than 5% are diagnosed after the age
(2010) - Gastroenterology of 60.

A 62-year-old man has a gastroscopy because of C. The toxicity of oats protein is greater than
reflux symptoms. wheat or rye.

Which of the following statements are true?

Dr. Khalid Yusuf El-Zohry – Sohag Teaching Hospital (01118391123) Page | 177
El-Zohry MRCP Questions Bank (Part 2) – Medical Masterclass 2010

D. Minor gastrointestinal and constitutional  diarrhoea


symptoms are more common than overt  aphthous ulcers
malabsorption.
 bone pain (due to vitamin D
E. Bone pains are due to poor absorption of
malabsorption and osteomalacia).
vitamin E.
Gate ataxia and peripheral neuropathy are the
F. Dementia is the most commonly
most common neurological problems.
associated neurological condition.
Dermatitis herpetiformis tends to affect the
G. Dermatitis herpetiformis is an associated extensor surfaces of the arms, knees and
disease which has a predilection for the buttocks. Folate and iron deficiencies may lead
trunk. to macrocytosis or a dimorphic blood film.
Howell-Jolly bodies characteristic of
H. Polyvalent pneumococcal vaccine may be
hyposplenism may also be seen in the film.
required.
Such patients require pneumococcal
I. Anti-gliadin antibodies are a more specific vaccination.
indicator of coeliac disease than anti-
Antigliadin antibodies are a sensitive but not
endomysial antibodies.
specific serological test for screening.
J. There is an increased risk of colonic Antireticulin and antiendomysial antibodies are
carcinoma. more specific. None are 100% sensitive
however and if suspicious, intestinal biopsy
Answer & Comments should still be arranged. Treatment is a gluten-
free diet, for symptomatic relief but also to
Correct answer: DH
reduce the associated risk of small bowel
lymphoma and adenocarcinoma, and squamous
Coeliac disease predominantly affects cell carcinoma of the pharynx and oesophagus.
Europeans and those with European heritage,
with prevalence rates up to 1 in 500 in Ireland.
[ Q: 22 ] MasterClass Part2
It affects all ages with a mean age of
presentation in the 40s and at least 20% (2010) - Gastroenterology
diagnosed after age 60. Genetic predisposition A 50-year-old man presents with flushing worse
(most commonly DQ2) and environmental after drinking alcohol and diarrhoea.
factors lead to cell-mediated damage of the Examination reveals a thin individual with
small intestine and the characteristic hepatomegaly.
histological features. Classic symptoms of the
condition are steatorrhoea, osteomalacia and The most likely diagnosis is:
anaemia. However, more subtle and vague
A. glucagonoma
symptoms are more common in older people
and the diagnosis may not be considered. B. mastocytosis
Gluten in wheat, rye and barley has an C. hyperthyroidism
established causal relationship. The toxicity of
oats is unclear. D. carcinoid syndrome

The wide range of symptoms include: E. phaeochromocytoma.

 lethargy
Answer & Comments
 anorexia
Correct answer: D
 weight loss

Dr. Khalid Yusuf El-Zohry – Sohag Teaching Hospital (01118391123) Page | 178
El-Zohry MRCP Questions Bank (Part 2) – Medical Masterclass 2010

Carcinoid syndrome is usually associated with included negative endomysial antibodies,


functioning malignant tumours that produce normal hydrogen breath test and normal
serotonin and arise from enteroendocrine cells barium follow through.
in the ileum. It may occur with comparable
tumours elsewhere in the GI tract, pancreas, Which method is the most specific and simple
gonads, or bronchi and rarely, highly malignant way of diagnosing chronic giardiasis?
tumours such as oat cell carcinomas of the lung, A. Urine culture
pancreatic islet cell carcinomas, and medullary
B. Serology
carcinomas of the thyroid gland can be
responsible. Intestinal tumours do not usually C. Histology from the second part of the
produce the syndrome unless metastatic spread duodenum
to the liver has occurred, because the metabolic
D. Colonoscopic appearances
products released by the tumour are rapidly
degraded by blood and liver enzymes in the E. Small bowel enteroscopy.
portal circulation (e.g. hepatic monoamine
oxidase and serotonin). Answer & Comments
Hepatic secondaries release metabolic products Correct answer: C
directly into the systemic circulation via the
hepatic veins.
Giardia lamblia is a very common intestinal
Primary pulmonary and ovarian carcinoid parasite and a frequent cause of diarrhoeal
products bypass the portal route, as do rare illness throughout the world. Although water
intestinal carcinoids with only intra-abdominal remains the most common mode of
spread that drain directly into the systemic transmission, there has been an increase in the
circulation or the lymphatics. A common and number of person-to-person cases, especially
early sign is uncomfortable flushing, typically of related to children, as well as an increase in
the head and neck, often precipitated by food-borne cases. Chronic diarrhoea and
emotion or eating hot food and drinks or malabsorption are seen with persistent
following alcohol consumption. Notable skin infection and since treatment is simple and
colour changes may occur, ranging from pallor effective, diagnosis by duodenal biopsy is
or erythema/cyanosis. Abdominal cramps with important. New antigen detection tests for
recurrent diarrhoea often occur. Right-sided stool may ultimately replace histology in some
endocardial fibrosis, leading to pulmonary situations. The greatest clinical experience in
stenosis and tricuspid regurgitation may also treatment is with the nitroimidazole drugs, i.e.,
develop, whilst left heart lesions, reported with metronidazole, tinidazole, and ornidazole,
bronchial carcinoids, are rare because serotonin which are highly effective. A 5- to 7-day course
is destroyed during passage through the lung. A of metronidazole can be expected to cure over
few patients have asthmatic wheezing, and 90% of individuals, and a single dose of
some have decreased libido and impotence; tinidazole or ornidazole will cure a similar
pellagra develops rarely. number.

[ Q: 23 ] MasterClass Part2 [ Q: 24 ] MasterClass Part2


(2010) - Gastroenterology (2010) - Gastroenterology
(2) A 30-year-old woman is seen with weight A 28-year-old man who is 1 year post stem cell
loss and diarrhoea. Three years ago she spent 6 transplant for acute leukaemia presents with
months in Egypt. Previous investigations have increasing confusion, jaundice and abdominal

Dr. Khalid Yusuf El-Zohry – Sohag Teaching Hospital (01118391123) Page | 179
El-Zohry MRCP Questions Bank (Part 2) – Medical Masterclass 2010

pain. Laboratory results include ALT 4300IU/l, A. She should stop her azathioprine.
AST 3500IU/l, GGT 870IU/l, and also reveal
B. She should stop her methotrexate.
signifcant impairment of hepatic synthetic
function. Imaging of the liver by C. She should avoid prednisolone if the
ultrasonography, including Doppler studies of disease becomes active.
the portal and hepatic veins, is unremarkable. D. Daily folic acid is recommended.

What is the most likely cause for liver failure in E. She should stop her mesalazine.
this case? F. Flexible sigmoidoscopy is contraindicated.
A. Graft versus host disease G. Premature births are less common.
B. Hepatic vein thrombosis (Budd Chiari H. Smoking has an adverse effect on the
syndrome) baby but not Crohn's disease.
C. Primary sclerosing cholangitis I. She should avoid breast feeding if taking
D. Herpes simplex hepatitis prednisolone.

E. Portal vein thrombosis J. She should have a caesarian section at


term.
Answer & Comments
Answer & Comments
Correct answer: D
Correct answer: BD
Levels of ALT or AST as high as this generally
indicate the presence of marked hepatocellular There is a slight increase in preterm and low
injury from medications (e.g. paracetamol) or birth weight of babies born to mothers with
toxins (e.g. herbal remedies or mushroom inflammatory bowel disease.
toxicity), viruses, or ischaemia. In an Standard doses (up to 3g/day) of 5-
immunocompromised patient such as this, one aminosalicylic acid drugs appear safe in
must consider unusual causes of hepatitis. In pregnancy.
particular cytomegalovirus, Epstein–Barr virus,
and herpes simplex viruses (HSV) 1, 2, and 6 There appears to be no increased risk to
have all been known to cause severe acute mother and baby of continuing azathioprine
hepatitis. Imaging in such patients must during pregnancy, although studies in
specifically include Doppler studies of the portal inflammatory bowel disease are small. If the
and hepatic veins in order not to miss portal patient has been well for some years, the
vein thrombosis or a Budd–Chiari syndrome physician may discuss withdrawal of the drug
(hepatic vein thrombosis). before conception.
Steroids used for exacerbations of disease are
[ Q: 25 ] MasterClass Part2 also probably safe.
(2010) - Gastroenterology Prednisolone is largely metabolised by the
placenta and whilst it is present in breast milk,
A 26-year-old woman with Crohn’s disease is 8 the concentration is low.
weeks pregnant. She asks several questions
about her medications and the effect of the The risks to mother and baby of active
disease on her unborn child. inflammatory bowel disease during pregnancy
outweigh the risks of these treatments.
Which of the following are true?

Dr. Khalid Yusuf El-Zohry – Sohag Teaching Hospital (01118391123) Page | 180
El-Zohry MRCP Questions Bank (Part 2) – Medical Masterclass 2010

Methotrexate is teratogenic and should not be [ Q: 27 ] MasterClass Part2


used in pregnancy. (2010) - Gastroenterology
Smoking is associated with Crohn's disease and
An 83-year-old woman presents with
continuing smoking adversely affects prognosis.
a 3-month history of 15kg weight loss,
progressive jaundice, pale stools and dark
[ Q: 26 ] MasterClass Part2 urine. Abdominal CT scan reveals an ill-defined
(2010) - Gastroenterology mass within the pancreatic head and dilated
distal pancreatic duct.
You are looking after a 35-year-old woman with
a 10-year history of alcohol abuse (>50 Which of the following statements are correct?
units/week). She has been given a diagnosis of
A. A history of alcohol abuse may be
alcoholic hepatitis based on her history, and
relevant.
investigations show a bilirubin of 250
micromol/L, alanine transaminase (ALT) of 120 B. The gallbladder can be palpated in the
U/L and INR of 2. Her creatinine has increased majority of individuals with this condition
over the last 2 days from 90 to 250 micromol/L. (Courvoisier's sign).
C. The tumour marker CA-125 is elevated in
In managing her further which single
most cases.
investigation would be most helpful?
D. Thrombophlebitis migrans is an
A. Urine osmolality
associated complication.
B. Urine microscopy
E. Any ascites present would be expected to
C. Serum antineutrophil cytoplasmic have a high serum/ascites albumin
antibody gradient (SAAG).
D. Creatinine clearance F. A percutaneous, transhepatic drain is the
E. Spot urine sodium preferred method of facilitating biliary
decompression.
Answer & Comments G. Pancreaticoduodenectomy (Whipple’s
procedure) offers the possibility of a
Correct answer: E
surgical cure in the majority of patients.
H. Surgery should only be attempted for
Although not diagnostic, a low spot urinary
potentially curative procedures.
sodium (<5 mmol/L) is in keeping with a
diagnosis of hepatorenal syndrome (HRS), I. Tumours of the ampulla of Vater have a
assuming that the patient does not have poorer prognosis than those of the
intravascular volume depletion. HRS is defined pancreas.
as an acute, functional and progressive
J. One-year survival rate is just 40%.
reduction in renal blood flow and glomerular
filtration rate (GFR) secondary to intense renal
cortical vasoconstriction in the setting of Answer & Comments
decompensated liver disease. Other aetiologies Correct answer: AD
for renal failure in liver disease must be
excluded. HRS can be classified as type 1, with a
Risk factors for adenocarcinoma of the
rapidly progressive decline in GFR (<2 weeks),
pancreas include age, male sex, smoking,
or type 2, which is not as rapidly progressive (>2
alcohol, chronic pancreatitis and diabetes. A
weeks).
Dr. Khalid Yusuf El-Zohry – Sohag Teaching Hospital (01118391123) Page | 181
El-Zohry MRCP Questions Bank (Part 2) – Medical Masterclass 2010

palpable gall bladder is suggestive of pancreatic Common dose-related symptoms include


cancer but is only rarely observed. CA19-9 may anorexia, nausea, dyspepsia and diarrhoea,
be elevated. Associated ascites has a high- whilst haematological complications include
protein content (ie a low SAAG). ERCP is the impairment of folate absorption and Heinz-
preferred method of decompression, PTC being body haemolytic anaemia. Hypospermia may
reserved for those cases that fail ERCP. occur but is reversible upon drug withdrawal.
Whipple's offers a surgical cure in less than 20% Hypersensitivity reactions are rarer but include
of cases, but palliative surgery may be required fever, rash, bone marrow suppression,
for duodenal obstruction and biliary infiltrative lung disease, a lupus-like syndrome,
decompression. Ampullary tumours have a pancreatitis, interstitial nephritis and hepatic
better prognosis than pancreatic cancer - the toxicity.
latter having a mortality of over 90% within a
Aside from urinary tract infection, the other
year of diagnosis
conditions listed are all possible causes of
sterile pyuria, but none except interstitial
[ Q: 28 ] MasterClass Part2 nephritis due to sulphasalazine are suggested
(2010) - Gastroenterology by this clinical scenario.

At a routine follow-up, a 65-year-old man with


long standing ulcerative colitis is noted to have [ Q: 29 ] MasterClass Part2
a sterile pyuria with eosinophils and (2010) - Gastroenterology
dysmorphic red cells present on urine
An 82 year-old woman is admitted as an
microscopy. He denies any urinary symptoms
emergency with a history of a fall. She lives
but his creatinine is mildly elevated.
alone, is normally housebound, and has a poor
dietary intake. She has been complaining of
What is the most likely diagnosis?
pain in her left groin for the last 6 months,
A. Urinary tract infection which her general practitioner has attributed to
B. TB of the urinary tract osteoarthritis. She is unable to stand up by
herself, and physical examination shows
C. Interstitial nephritis secondary to
evidence of a proximal myopathy. A pelvic x-
sulphasalazine
ray, performed to exclude a fractured neck of
D. Polycystic kidney disease femur (which she does not have), shows an
abnormality in her right pubic rami.
E. Brucellosis.
Which of the following are most likely to be her
Answer & Comments laboratory results?
Correct answer: C A. Low calcium/low phosphate/high alkaline
phosphatase
Sulphasalazine is metabolised by colonic flora, B. Low calcium/high phosphate/low alkaline
releasing sulphapyridine, an absorbable phosphatase
antibiotic, and 5-aminosalicylic acid (5-ASA), the
C. High calcium/low phosphate/high alkaline
active ingredient. The sulphapyridine acts only
phosphatase
to carry the 5-ASA to the colon and, when
released by bacterial metabolism, is absorbed D. High calcium/low phosphate/low alkaline
and is responsible for dose-related side effects. phosphatase
E. Normal calcium/normal
phosphate/normal alkaline phosphatase
Dr. Khalid Yusuf El-Zohry – Sohag Teaching Hospital (01118391123) Page | 182
El-Zohry MRCP Questions Bank (Part 2) – Medical Masterclass 2010

Answer & Comments G. Mumps infection

Correct answer: A H. Pancreas divisum


I. Traumatic pancreatitis
The presentation and findings would fit with a
diagnosis of osteomalacia due to Vitamin D J. Pancreatic tumour
deficiency. Typical biochemical findings in this
condition would be low or normal calcium/low Answer & Comments
phosphate/high alkaline phosphatase/high PTH.
Correct answer: CD
Vitamin D is obtained both by dietary intake
and from the action of the sun on skin Acute pancreatitis is an inflammatory process
photoactivating 7-dehydrocholesterol. Older affecting the pancreas with variable
people, especially those who are housebound, involvement of other regional tissues or remote
are vunerable to developing osteomalacia due organ systems. There are many causes of acute
to low sun exposure. Patients may present with pancreatitis but gallstones (30–50%) and
bone pain, proximal myopathy, rickets alcohol (10–40%) account for the majority of
(especially in younger patients), and cases. Less common causes include: (i) drugs
radiographs of bones may show Looser zones. (e.g. azathioprine, sulphasalazine, furosemide),
(See Medical Masterclass, Gastroenterology (ii) toxins, (iii) trauma (blunt trauma to the
and Hepatology, Section 2.14.1, Table 42). abdomen as well as iatrogenic trauma e.g.
postoperative and ERCP), (iv)
[ Q: 30 ] MasterClass Part2 hypertriglyceridemia, (v) hypercalcaemia, (vi)
infections (eg mumps, Coxsackie,
(2010) - Gastroenterology
cytomegalovirus), (vii) congenital anomalies
You are called to review a 67-year-old man (pancreas divisum, choledochocele), (viii)
admitted 4 days ago with central abdominal ampullary or pancreatic tumors, (ix) vascular
pain. A diagnosis of acute pancreatitis was abnormalities (atherosclerotic emboli,
made clinically and biochemically, and the hypoperfusion, vasculitis), (x) hereditary
surgeons have managed him conservatively. A pancreatitis (mutations in trypsinogen gene),
few small gallbladder stones were noted on and (xi) idiopathic causes (10–25% of patients).
ultrasonography, but the common bile duct was In this man alcohol can be excluded since he is a
undilated and there were no intraduct stones. A strict Muslim. Of the other causes gallstone
CT scan has been requested. He is a strict pancreatitis remains the most likely. Although
Muslim whose only other past medical history gallstone pancreatitis can occur at any age, it is
is of ulcerative colitis, for which he is on most common between the ages of 50 and 70
treatment. years and, the diagnosis of gallstone-induced
pancreatitis can often be missed. Ultrasound is
The TWO most likely causes of his pancreatitis
only 70% to 80% sensitive in detecting
are:
gallstones during the acute phase of
A. Alcohol-induced pancreatitis pancreatitis. His history of ulcerative colitis is
B. Hereditary pancreatitis important since azathioprine has become a
common treatment for this chronic condition
C. Drug-induced pancreatitis and pancreatitis is a well recognised and
D. Gallstone pancreatitis important complication of azathioprine. Other
side effects of azathioprine include dose-
E. Idiopathic pancreatitis
related bone-marrow depression (leucopaenia,
F. Hyperparathyroidism thrombocytopenia, anaemia), cholestatic liver

Dr. Khalid Yusuf El-Zohry – Sohag Teaching Hospital (01118391123) Page | 183
El-Zohry MRCP Questions Bank (Part 2) – Medical Masterclass 2010

damage and rarely veno-occlusive liver disease. Comorbidity – None (0), Other (1), Cardiac
Azathioprine is also associated with gastro- failure / ischaemic heart disease (2), renal or
intestinal disturbances, reversible alopecia, liver failure (3).
rashes, muscle and joint pains, fever, rigors,
The total score predicts mortality as follows:
pneumonitis, meningitis, arrhythmias, renal
Score 0, 0.2%; score 2, 5%; score 4, 24%; score
dysfunction and hypotension, some or all of
6, 49%.
which may represent hypersensitivity reactions.
The other causes of pancreatitis whilst [ Q: 32 ] MasterClass Part2
recognised are rare and unlikely. Idiopathic
(2010) - Gastroenterology
pancreatitis is a diagnosis of exclusion.
A 66-year-old woman is referred urgently with
[ Q: 31 ] MasterClass Part2 painless jaundice and weight loss. Bilirubin is
212 mmol/L, alanine transaminase (ALT) 60 U/L,
(2010) - Gastroenterology
alkaline phosphatase (ALP) 605 U/L, albumin 34
A 54-year-old man is admitted following a g/L and prothrombin time 17 seconds.
haematemesis. You telephone the Ultrasound scanning of the abdomen shows a
gastroenterologist on call to request an urgent grossly dilated biliary tree and a dilated
endoscopy. He asks you what the patient’s pancreatic duct, but no mass is seen.
Rockall Score is.
What is the next most appropriate step in her
This is calculated on the basis of: management?
A. co-morbidity, age, peripheral perfusion, A. Endoscopic retrograde
systolic BP cholangiopancreatography (ERCP)
B. co-morbidity, age, pulse, systolic BP B. Abdominal CT scan
C. age, pulse, mean BP C. Pancreatic endoscopic ultrasound
D. co-morbidity, age, peripheral perfusion, D. Measure serum CA19-9
pulse, systolic BP
E. Laparoscopy
E. previous history GI bleed, age, pulse,
mean BP. Answer & Comments

Correct answer: B
Answer & Comments

Correct answer: B This scenario is highly suspicious of cancer in


the pancreatic head as there is obstructive
The Rockall score is often used to assess jaundice and ultrasound demonstrates the
severity of gastrointestinal haemorrhage and / ‘double duct' sign. The absence of a mass on
or to triage patients for emergency endoscopy. ultrasound does not exclude this diagnosis and
therefore a CT scan is required to further image
Four parameters are used to calculate the
the pancreatic head and simultaneously stage
Rockall score:
the disease. ERCP would demonstrate the
Age (yrs) - <60 (score 0), 60-79 (1), >80 (2) stricture but gives no staging information, and
Systolic BP (mmHg) and Pulse (/min) – SBP>100 endoscopic ultrasound may not pick up liver
with P<100 (0), SBP>100 with P>100 (1), metastases. CA19-9 is not specific for
SBP<100 (2). pancreatic cancer and is often raised in
obstructive jaundice of any aetiology.

Dr. Khalid Yusuf El-Zohry – Sohag Teaching Hospital (01118391123) Page | 184
El-Zohry MRCP Questions Bank (Part 2) – Medical Masterclass 2010

Laparoscopy is too invasive at this stage B. bacterial overgrowth in the small bowel
without further imaging.
C. Hypolactasia
D. infection with H pylori
[ Q: 33 ] MasterClass Part2
(2010) - Gastroenterology E. malabsorption due to small intestinal
disease.
At upper GI endoscopy antral gastritis is noted
in a patient with duodenal ulceration. However, Answer & Comments
a urease test for Helicobacter pylori is negative.
Correct answer: B
The patient should be:
Bacterial fermentation of an oral dose of the
A. investigated for other causes of duodenal
carbohydrate lactulose releases hydrogen. If
ulceration with a fasting gastrin level
there is an increase in bacteria in the upper
B. managed with a proton pump inhibitor small bowel, or if there is rapid intestinal
(PPI) alone transit, then an early peak in hydrogen can be
C. treated with eradication therapy detected in expired air.
nevertheless An oral dose of conjugated bile acid, often
D. offered surgery as a definitive cure glycine-glycocholate, with radiolabelled carbon
in the amino portion, can be given as an
E. have repeat endoscopy in 3 months. alternative to lactulose. Bacterial action
releases radiolabelled glycine, detected
Answer & Comments following metabolism as labelled carbon
dioxide in the breath.
Correct answer: C
What functional tests would you use to detect
Helicobacter pylori is associated with 95% of hypolactasia, H pylori infection or
duodenal ulcers and 80% of gastric ulcers. Tests malabsorption due to small intestinal disease?
for H pylori include serology, histological
biopsy, urease testing, urea breath testing and
[ Q: 35 ] MasterClass Part2
faecal antigen assays. False negative urease
testing can occur if patients have been treated
(2010) - Gastroenterology
with antibiotics, bismuth or proton pump A 34-year-old woman, born in Hong Kong, is
inhibitors in the prior 1-2 weeks. Thus in this referred to clinic because at a hospital
case where a H pylori- associated gastritis and employment medical she is found to have
duodenal ulcer is most likely, eradication should hepatitis B. Her serology is as follows: HbsAg
be prescribed regardless of the urease test positive, HBe Ab positive. Liver function tests,
full blood count and clotting profile are all
[ Q: 34 ] MasterClass Part2 normal.
(2010) - Gastroenterology Her partner has the following serology: HBs Ab
positive, HBc Ab positive. There is no family
A 58-year-old man presents with diarrhoea and
history of liver disease.
weight loss. Amongst many investigations he
has a lactulose breath test. Which of the following statements is correct?
The reason for the test is to detect: A. Pregnancy is contraindicated.

A. diarrhoea due to laxative consumption

Dr. Khalid Yusuf El-Zohry – Sohag Teaching Hospital (01118391123) Page | 185
El-Zohry MRCP Questions Bank (Part 2) – Medical Masterclass 2010

B. She cannot perform exposure prone A. Mallory Weiss tear


procedures without her viral load being
B. Oesophageal varices
measured.
C. Oesophageal malignancy
C. Her risk of developing hepatocellular
carcinoma is high. D. Gastric varices

D. She needs interferon treatment. E. Gastric malignancy

E. She is at high risk of infecting her partner F. Peptic ulcer


sexually with hepatitis B. G. Portal hypertensive gastropathy
H. Aorto-enteric fistula
Answer & Comments
I. Reflux oesophagitis
Correct answer: B
J. Duodenal malignancy.
She is likely to be a ‘healthy carrier’ of hepatitis
B. Her risk of clearing hepatitis B spontaneously Answer & Comments
is 1% a year, which is not increased by
Correct answer: BF
interferon. Interferon is used to treat eAg
positive individuals with high serum
The causes of haematemesis in patients with
transaminases and in this setting causes
liver cirrhosis are as follows: bleeding
seroconvertion to eAb, a fall in viral load, and
oesophageal varices (60%), bleeding peptic
normalisation of liver biochemistry in about
ulcer (20%), portal hypertensive gastropathy
30% cases.
(5%), bleeding gastric varices (5%), other causes
This young woman, with a normal liver and eAb / undiagnosed (10%).
(which normally corresponds to a low viral load
Significant gastrointestinal haemorrhage from
of <105 copies/ml), is at low risk of
oesophageal varices can present with melaena
hepatocellular carcinoma. Her partner is
in the absence of haematemesis. Variceal
immune to hepatitis B from previous infection.
haemorrhage can be the presenting feature of
Until relatively recently, if an individual was eAb previously undiagnosed liver disease
positive, it was possible to perform invasive
exposure prone medical procedures (i.e. non-
[ Q: 37 ] MasterClass Part2
laparoscopic invasive surgery). This is now only
(2010) - Gastroenterology
possible if a viral load is consistently less than
103 copies/ml. A 45-year-old man with chronic alcoholic liver
Any child she has in the UK will be given disease was admitted earlier in the day
hepatitis B immunoglobulins and vaccinated at following a large haematemesis. He was treated
birth to prevent transmission of hepatitis B. with intravenous terlipressin, and urgent
endoscopy was arranged after initial fluid
resuscitation and correction of a mild
[ Q: 36 ] MasterClass Part2 coagulopathy with fresh frozen plasma.
(2010) - Gastroenterology Endoscopy revealed bleeding oesophageal
A 48-year-old man with alcoholic cirrhosis of varices, which were injected with sclerosant
the liver is admitted with haematemesis and with apparently good effect.
melaena. He has been stable on the ward for the past 6
hours, but you are called to see him in the early
The TWO most likely sources of bleeding are: hours of the following morning because he has

Dr. Khalid Yusuf El-Zohry – Sohag Teaching Hospital (01118391123) Page | 186
El-Zohry MRCP Questions Bank (Part 2) – Medical Masterclass 2010

had a further 500ml haematemesis, and his [ Q: 38 ] MasterClass Part2


blood pressure has dropped from 130/90 (2010) - Gastroenterology
mmHg to 90/50 mmHg.
A 45-year-old man with no previous medical
Which of the following measures would be most history is found to have oesophageal varices
appropriate in addition to fluid resuscitation? when endoscoped to investigate dyspepsia.
A. Urgent repeat endoscopy and
Which of the following are true:
sclerotherapy
A. Variceal sclerotherapy is indicated.
B. Change terlipressin to octreotide
B. Variceal banding is indicated.
C. Administer 10mg vitamin K intravenously
C. Beta-blockers do not affect the long term
D. Insert Sengstaken tube, inflate gastric
outcome.
balloon and apply traction
D. If admitted with variceal bleeding,
E. Urgent surgical intervention.
terlipressin may reduce bleeding.

Answer & Comments E. Variceal bleeding has a 10% mortality


rate.
Correct answer: D
Answer & Comments
Balloon tamponade is the most effective
treatment for control of variceal bleeding if Correct answer: D
endoscopic therapy has failed. Even though the
varices were bleeding from within the Beta-blockers are of overall benefit in primary
oesophagus, inflation of the gastric balloon with prophylaxis, i.e. in reducing the chance of a first
application of traction usually stops the bleed. Variceal bleeding has a mortality rate of
bleeding by compressing the vessels as they about 30%, and terlipressin may reduce
cross the gastrooesophageal junction. Rarely, bleeding when administered acutely.
inflation of the oesophageal balloon may be
required. [ Q: 39 ] MasterClass Part2
It is unlikely that repeat endoscopy would (2010) - Gastroenterology
prevent on-going bleeding if he has already
A 48-year-old man with type 1 diabetes mellitus
received successful sclerotherapy, unless there
of 8 years' duration attends outpatients with a
was doubt about the source of bleeding
history of opening his bowels five to six times
previously. Octreotide is less effective than
terlipressin in controlling portal pressure, and per day over the last 6 months. There is no
therefore switching therapy is unlikely to be blood or mucus in the stool and he has not had
any abdominal pain. His HbA1c is 7.0%. FBC,
helpful.
urea and electrolytes, liver function tests,
Although the effects of the fresh frozen plasma thyroid function tests, abdominal ultrasound
administered earlier will probably have worn and colonoscopy with random biopsies are all
off, vitamin K will probably not help as the normal.
coagulopathy is probably due to synthetic
dysfunction rather than vitamin K deficiency. Which TWO of the following tests are most
likely to lead to a diagnosis?
Surgical transection of the oesophagus is rarely
indicated, and has largely been superseded by A. Autonomic function tests
radiological shunt insertion (e.g. TIPSS).
B. Helicobacter serology

Dr. Khalid Yusuf El-Zohry – Sohag Teaching Hospital (01118391123) Page | 187
El-Zohry MRCP Questions Bank (Part 2) – Medical Masterclass 2010

C. Stool for microscopy, culture and In her management plan, which statement is
sensitivity LEAST appropriate?
D. Abdominal CT scan A. Rehydrate orally or intravenously
depending on clinical and biochemical
E. Small bowel enema
assessment.
F. Stool for faecal elastase
B. Nurse her in isolation.
G. Lactose hydrogen breath test
C. X-ray her abdomen and measure
H. Small bowel manometry inflammatory markers such as ESR/CRP.
I. Glycocholate hydrogen breath test D. Send stool on multiple occasions for
J. Rigid sigmoidoscopy and rectal deep culture and sensitivity, including C.
rectal biopsy difficile toxin.
E. Request a colonoscopy as an inpatient.
Answer & Comments
Answer & Comments
Correct answer: FI
Correct answer: E
The three most likely associations with type 1
diabetes mellitus that cause diarrhoea are
Not all episodes of infectious diarrhoea need
coeliac disease (up to 5%, so check coeliac
treating with antibiotics, and often, resistance
serology), exocrine pancreatic insufficiency
to certain antibiotics (e.g. ciprofloxacin) leads to
(over 30% in some studies) and small bowel
sub-optimal treatment. Any patient who has
bacterial overgrowth (SBBO). SBBO commonly
been in hospital recently is at risk of C. difficile
occurs when there is autonomic dysfunction,
infection - especially if they may have had
but tests of the autonomic nervous system may
antibiotics. This needs to be actively tested for
be normal, and SBBO may not be present in
and treated appropriately (metronidazole as
those with dysfunction. This patient also has
first-line treatment, then oral vancomycin).
good metabolic control of his diabetes as
Hence, it is imperative in managing patients
judged by HbA1c of 7.0%. It will be
with diarrhoea to ensure adequate and suitable
appropriate to send stool for microscopy,
stool samples are sent early in the illness.
culture and sensitivity, but after a history of 6
Colonic imaging is usually not indicated in
months this is likely to be normal. CT scanning
infective colitis (other than a simple X-ray to
or small bowel imaging will have a low yield.
exclude toxic megacolon) and is only necessary
Lactose hydrogen breath testing may be falsely
if symptoms persist and stool cultures are
positive due to SBBO. Further colonic biopsies
negative
are unlikely to add anything.

[ Q: 41 ] MasterClass Part2
[ Q: 40 ] MasterClass Part2
(2010) - Gastroenterology
(2010) - Gastroenterology
A 33-year-old injecting drug user with known
A 40-year-old female nursing home resident
chronic hepatitis B presents to casualty with
recently in hospital with Salmonella
jaundice, pruritus and confusion. Liver function
gastroenteritis is readmitted with further
tests suggest a hepatitic picture. Ultrasound
diarrhoea.
examination of the liver and biliary system is
unremarkable.

Dr. Khalid Yusuf El-Zohry – Sohag Teaching Hospital (01118391123) Page | 188
El-Zohry MRCP Questions Bank (Part 2) – Medical Masterclass 2010

Which of the following is the most likely A. Admit the patient from clinic for further
explanation? investigations
A. Acute hepatitis C B. Arrange a CT abdomen and chest to look
B. Development of hepatocellular for metastatic cancer
carcinoma C. Arrange an appointment in the combined
C. Biliary stones oncology-surgery clinic

D. Superinfection with hepatitis D virus D. Presribe a high dose of a proton pump


inhibitor (PPI)
E. Acute hepatitis E
E. Book the patient for percutaneous
endoscopic gastrostomy (PEG) insertion.
Answer & Comments

Correct answer: D Answer & Comments

Correct answer: A
Hepatitis D is a RNA virus, structurally unrelated
to hepatitis A, B, or C, that causes an infection
requiring the assistance of hepatitis B virus This man has absolute dysphagia and needs to
particles in order to replicate. Disease be in hospital. He may have a malignancy, a
manifestations include a self-limited acute peptic stricture or a food bolus. Neurological or
infection, acute fulminant liver failure and end muscle disorders are less likely. Most would
stage liver disease from chronic infection. Since arrange early cautious upper GI endoscopy or
hepatitis D virus is transmitted by blood and failing that a gastrograffin swallow (risk of
blood products, the risk factors for infection are aspirating barium). If a malignant lesion is likely
similar to hepatitis B virus and infection may then further investigations are needed to
occur at the same time as hepatitis B or determine the best treatment strategy. Options
include stenting, chemo-radiotherapy, PEG
represent super-infection; the latter scenario
feeding (inserted either endoscopically or
should be suspected in a patient with chronic
hepatitis B whose condition suddenly surgically) or palliation. Investigations such as
deteriorates. Diagnosis is usually serological CT, PET and endoscopic ultrasound allow
(IgM antibodies indicate acute infection). staging of malignant disease and a multi-
Clinical manifestations of acute hepatitis C are disciplinary approach to treatment is probably
extremely rare (usually within 2 months of best.
exposure to HCV).
Acute hepatitis E is most common in the
[ Q: 43 ] MasterClass Part2
developing world (tropical climates, inadequate (2010) - Gastroenterology
sanitation, and poor personal hygiene). A 32-year-old woman is referred to the
gastroenterology clinic with lethargy and mild
[ Q: 42 ] MasterClass Part2 elevation in her serum alanine
(2010) - Gastroenterology aminotransferase (ALT 85 iu/L; normal <40).
Other investigations include: aspartate
A 70-year-old man with dysphagia is seen in aminotransferase 80iu/L, GGT 60 iu/L (normal
clinic. He is unable to swallow any solids or <40). The bilirubin, alkaline phosphatase,
fluids. albumin and prothrombin time are normal. Her
full blood count is normal, with MCV 85fL
What is the best course of action?
(normal).

Dr. Khalid Yusuf El-Zohry – Sohag Teaching Hospital (01118391123) Page | 189
El-Zohry MRCP Questions Bank (Part 2) – Medical Masterclass 2010

What is the most likely diagnosis? The most likely diagnosis is:
A. Chronic hepatitis C A. Multiple metastasis
B. Alcoholic hepatitis B. Focal nodular hyperplasia
C. Primary biliary cirrhosis C. Adenomas
D. Haemochromatosis D. Haemangiomas
E. Wilson’s disease E. Liver abscesses

Answer & Comments Answer & Comments

Correct answer: A Correct answer: D

Chronic hepatitis C is a very common cause of Liver imaging must always be interpreted in the
minor elevations in serum transaminases. Other context of the clinical information, particularly
liver function tests can be entirely normal and the presence or absence of relevant symptoms,
assessment of viral status by PCR along with the presence or absence of liver function test
histological assessment may be needed, abnormalities, and the presence or absence of
particularly if treatment is being considered. cirrhosis. Liver masses with typical imaging
Isolated elevations in AST (aspartate features of simple cyst or haemangioma in
aminotransferase) can occur with muscle patients not known to have, or not suspected of
disease and the AST:ALT ratio can be useful in having, a malignancy may be classified as
diagnosing alcoholic liver disease, because benign.
more than two-thirds of patients will have a
Haemangiomas, the commonest of the focal
ratio greater than 2. Menstruating women are
benign liver lesions, arise from the endothelial
generally protected against haemochromatosis.
cells that line the blood vessels and consist of
Wilson’s disease although rare must always be
multiple large vascular channels lined by a
considered in a young person with chronic
single layer of endothelial cells and supported
hepatitis. However, cirrhosis is often present by
by collagenous walls. They are more common in
the age of 31, co-existent haemolysis (elevated
women and occur at all ages, but most
bilirubin) is common and transaminases are
frequently in the third, fourth and fifth decades
often higher.
of life. Most are seen incidentally and are
usually less than 1 cm in diameter. They are
[ Q: 44 ] MasterClass Part2 generally asymptomatic, although if large (>5
(2010) - Gastroenterology cm) may cause symptoms/signs such as pain,
nausea or enlargement of the liver; very rarely
A 35-year-old woman has an ultrasound scan they can rupture, causing severe pain and
for investigation of epigastic pain. Her liver bleeding into the abdomen, or become
function tests are all normal, and by the time of infected. On ultrasound imaging hepatic
her scan her symptoms have abated on haemangiomas are hyperechoic in 60-70% of
ranitidine. The report reads as follows: ‘There cases, hypoechoic in 20% of cases, and have a
are multiple small hyperechoic lesions in both mixed pattern with discrete margins in 20%;
lobes of the liver. No other focal lesions. No they are multiple in 10% of cases.
features to suggest cirrhosis or portal
hypertension. Intrahepatic and extrahepatic
biliary tree normal. Pancreas, kidneys, spleen
normal.'

Dr. Khalid Yusuf El-Zohry – Sohag Teaching Hospital (01118391123) Page | 190
El-Zohry MRCP Questions Bank (Part 2) – Medical Masterclass 2010

[ Q: 45 ] MasterClass Part2 In relation to Sengstaken–Blakemore tube


(2010) - Gastroenterology placement, which of the following statements is
incorrect?
A 23-year-old woman is referred to clinic
A. Inflation of the oesophageal balloon is
because her General Practitioner thinks she has
likely to be required if bleeding is
developed acute hepatitis B.
originating above the cardia.
Which of the following investigations done B. Formal airway protection with the
when she became jaundiced is consistent with insertion of a cuffed endotracheal tube
this diagnosis? with appropriate anaesthesia is
A. HB core antibody positive and surface recommended if the patient has grade II
antibody positive encephalopathy.

B. HbsAb positive and core antibody C. If the oesophageal balloon is inflated,


negative pressure should be released for up to 5
minutes every 30 minutes.
C. HbsAg and HBeAg positive
D. Aspiration ports should be left to drain
D. An alkaline phosphatase of 800 IU/l and freely and aspirated regularly.
an ALT of 50 IU/l
E. Gastric aspiration port may be used for
E. HBs Ag positive and e antibody positive. oral medication.

Answer & Comments Answer & Comments


Correct answer: C Correct answer: A

At the time of clinical acute hepatitis B with Since oesophageal varices usually originate in
onset of jaundice, there is active viral the stomach, tamponade of the varices at the
replication with elevated HBV DNA levels and gastrooesophageal junction by inflation of the
presence of e antigen and surface antigen. High gastric balloon is usually effective in controlling
serum transaminase levels are common haemorrhage. Inflation of the oesophageal
although a cholestatic phase with high alkaline balloon is rarely required and risks necrosis of
phosphatase levels can occur following the the oesophageal wall, necessitating regular
transaminitis in 5%. Subsequently e antibodies reduction of the pressure. Regular aspiration of
develop followed by surface antibodies in the the appropriate ports prevents build-up of
majority of individuals who clear the virus. secretions; the gastric aspirate port may be
Evidence of previous infection is indicated by used as a nasogastric tube for delivery of
the presence of antibodies to core protein and essential oral medication, such as lactulose for
surface protein. Those individuals who have the treatment of encephalopathy.
been immunised will have antibodies to the
surface protein only. Aspiration pneumonia is a major cause of death
and significant morbidity in these patients; the
risk is particularly high in confused or
[ Q: 46 ] MasterClass Part2 encephalopathic patients who should be
(2010) - Gastroenterology intubated for airway protection prior to
insertion of the tube.
A 47-year-old man has re-bled on the ward
following emergency endoscopic treatment of
his oesophageal varices.

Dr. Khalid Yusuf El-Zohry – Sohag Teaching Hospital (01118391123) Page | 191
El-Zohry MRCP Questions Bank (Part 2) – Medical Masterclass 2010

[ Q: 47 ] MasterClass Part2 patients will not require a proton-pump


(2010) - Gastroenterology inhibitor.
Although milk has a short-term alkali effect, its
A 37-year-old building labourer presents with
fat content and presence in the stomach will
heartburn and acid regurgitation.
stimulate gastrin and acid production. Patients
Which of the following statements are true? should be advised to eat or drink nothing for
three or four hours before retiring. Belts and
A. Oesophagogastroduodenoscopy (OGD) is corsets increase intra-abdominal pressure and
required in order to diagnose gastro- will exacerbate symptoms, so they should be
oesophageal reflux disease (GORD). discouraged.
B. First-line management should be a Not only does nicotine relax the lower
proton-pump inhibitor. oesophageal sphincter, but also smokers
C. He should be encouraged to drink milk at produce less saliva, one of the clearing
bedtime. mechanisms for transient lower oesophageal
sphincter relaxations.
D. It is essential to eradicate Helicobacter
pylori for optimal symptom control. H. pylori has no role in GORD, and indeed some
studies suggest eradication may worsen
E. The patient should be encouraged to stop
symptoms.
smoking.
Oesophageal pH monitoring is not sufficiently
F. A surgical corset may improve posture
available for routine diagnosis, but it may be
and hence symptoms.
useful in confirming the diagnosis in apparently
G. Oesophageal pH monitoring may be resistant cases.
useful in managing drug-resistant cases.
The debate over Barrett’s surveillance
H. All patients with Barrett’s oesophagus continues, though none would argue for it in
should be entered into a surveillance short-segment disease or the elderly.
programme.
While in patients with oesophageal cancer
I. There is an increased incidence of there is an increased history of reflux
oesophageal cancer in patients with a symptoms, the reverse is not true. Any
long history of gastro-oesophageal reflux association has yet to be proven.
disease.
While surgical (or endoscopic) fundoplication
J. Surgical fundoplication optimises may be beneficial in a small number of drug-
symptom control and is the treatment of resistant cases, there is significant morbidity
choice in the majority of cases. (gas bloat, flatulence etc) and mortality.
Symptoms also frequently recur over time
Answer & Comments requiring a return to medical therapy.

Correct answer: EG
[ Q: 48 ] MasterClass Part2
GORD is a clinical diagnosis based on history.
(2010) - Gastroenterology
Only a minority of patients with GORD will have A 50-year-old man had an abdomino-perineal
endoscopic oesophagitis, and a normal OGD resection for a Dukes A rectal carcinoma 9 years
does not exclude the diagnosis. First-line ago. His brother died from metastatic colorectal
management should be lifestyle advice, cancer. Interval colonoscopy at 5 years post
followed by simple antacids. A proportion of

Dr. Khalid Yusuf El-Zohry – Sohag Teaching Hospital (01118391123) Page | 192
El-Zohry MRCP Questions Bank (Part 2) – Medical Masterclass 2010

surgery was normal. He now presents to clinic Colonoscopy may not be beneficial for patients
with an iron deficiency, anaemia and jaundice. with clean colons and life-expectancy of less
than 15 years, since they are very unlikely to
In his investigations, which statement is LEAST develop a new colorectal tumour during this
appropriate? period. The role of carcinoembryonic antigen
A. Further lower GI investigation with (CEA) monitoring is uncertain. There is still no
flexible sigmoidoscopy should be evidence that the lead time provided by CEA
performed as a matter of urgency monitoring confers any survival benefit.

B. Abdominal ultrasound is an appropriate Meta-analysis shows that for people with at


first line investigation for his jaundice. least one affected first-degree relative (parent,
child or sibling), the risk of having a diagnosis of
C. Post-operative screening by colonoscopy
colorectal cancer is more than double that for
at 5-year intervals is reasonable practice.
the general population. People with two first-
D. Measurement of CEA is of limited value in degree relatives with colorectal cancer, or one
monitoring colorectal cancer patients. first-degree relative whose colorectal cancer is
diagnosed before the age of 45, have a lifetime
E. Family screening for the man's children is
risk of death from colorectal cancer of one in six
appropriate.
or one in 10 respectively. The BSG/ACPGBI
guidelines suggest that people who meet these
Answer & Comments
criteria should be referred for colonoscopy at
Correct answer: A 35-40 years of age, or as soon thereafter as the
risk is recognised. Stated more generally,
screening in relatives should commence at an
Up to 50% of colorectal cancer patients will
age 10 years younger than the age at which the
develop a recurrence of their tumour and most
affected index case was diagnosed with colon
of these will die from their disease. Eighty per-
cancer.
cent of recurrences after resection of colorectal
cancer occur within the first two years after Polyps can be seen and removed during
surgery, but even an intensive investigation endoscopic examination of the bowel; 90% of
follow up programme will fail to detect adenomas can be detected by colonoscopy and
approximately 50% of asymptomatic about 70% by flexible sigmoidoscopy, which
recurrences. typically only examines the colon distal to the
splenic flexure. However, in an individual at
Patients who did not undergo complete
high risk of synchronous, metachronous, or
colonoscopy before surgery should be offered
recurrent cancer (and where iron deficiency
colonoscopy within six months of discharge.
suggests the possibility of gastrointestinal
This can be regarded as completion of the initial
bleeding) a full colonoscopy, rather than a
diagnostic work-up to identify individuals who
limited examination, is indicated.
have polyps or a tumour elsewhere in the
colon. Colonoscopy may then be offered at five-
yearly intervals to check for new polyps or [ Q: 49 ] MasterClass Part2
tumours. It should not normally be carried out (2010) - Gastroenterology
more frequently when the patient has a ‘clean’
(polyp-free) colon, but patients with five or A 23-year-old woman presents with a 6-week
more adenomas should be offered more history of bloody diarrhoea opening her bowels
frequent checks. ten times daily, and three times at night. She is
tachycardic and has abdominal pain. She has no

Dr. Khalid Yusuf El-Zohry – Sohag Teaching Hospital (01118391123) Page | 193
El-Zohry MRCP Questions Bank (Part 2) – Medical Masterclass 2010

other previous medical history. She is a non- [ Q: 50 ] MasterClass Part2


smoker. (2010) - Gastroenterology
Which one of the following statements is true? You are reviewing an 80-year-old man who
A. Stool culture is not necessary as the presents to clinic with a 2-week history of
diarrhoea has been present for 6 weeks. jaundice, not associated with any pain. He feels
lethargic but is otherwise well. Initial hepatitis
B. Sigmoidoscopy is mandatory. serology is negative, ultrasound shows no duct
C. Histology reliably differentiates infection dilatation, and liver function tests are
from inflammatory bowel disease. cholestatic in nature. His medications have not
changed over the last 2 years, apart from a
D. Plain abdominal radiograph should not
recent course of an antibiotic for cellulitis.
be performed as she is of child-bearing
age. Which medication is most likely to explain his
E. Colectomy is the treatment of choice. jaundice?
A. Atorvastatin
Answer & Comments
B. Aspirin
Correct answer: B C. Co-amoxiclav
D. Amiodarone
The likelihood is that this lady has inflammatory
bowel disease. Bloody diarrhoea suggests E. Diclofenac
colonic involvement and the fact that she is a
non-smoker would make ulcerative colitis more Answer & Comments
likely. It is essential to take stool cultures, as
this may be a primary infective colitis. Correct answer: C
Alternatively infection may co-present with a
flare-up or initial presentation of ulcerative Virtually all drugs can cause liver injury. In this
colitis and needs treating in its own right. case the most recently introduced medication is
most likely to be the cause of liver injury,
Endoscopic examination of the colon is
assuming other investigations for chronic liver
mandatory. It allows visualisation of the colonic
disease are negative. The prescription drugs to
mucosal and biopsy for histological
be particularly aware of when assessing a
examination. It may determine the extent of
patient with an undiagnosed liver disorder
the disease. Although histology may be
include Augmentin (co-amoxiclav), diclofenac,
diagnostic, it is sometimes very difficult to
isoniazid, erythromycin, sodium valproate,
exclude infection as a cause.
amiodarone and phenytoin.
Bowel frequency of thirteen times daily with
tachycardia and abdominal pain would indicate
[ Q: 51 ] MasterClass Part2
severe colitis. Such a patient should be
admitted for intravenous steroid therapy.
(2010) - Gastroenterology
Approximately 70% of patients will respond to
In a patient who is jaundiced and HBsAg and
steroids and thus avoid colectomy.
HBeAg positive with evidence of cirrhosis
A plain abdominal radiograph is essential to (ascites) and an albumin of 28g/L and a
rule out the complication of toxic megacolon, prothrombin time of 21 sec, the best treatment
although clinical judgement may be used if would be:
pregnancy is confirmed.

Dr. Khalid Yusuf El-Zohry – Sohag Teaching Hospital (01118391123) Page | 194
El-Zohry MRCP Questions Bank (Part 2) – Medical Masterclass 2010

A. Ribavirin and alpha-interferon A. His symptoms are consistent with a


Crohn’s stricture.
B. Ribavirin
B. Chronic pancreatitis is very unlikely in
C. Lamivudine
view of the normal CT.
D. Beta-interferon
C. Infliximab should be prescribed as soon
E. Alpha-interferon. as possible.
D. Regular monitoring of his electrolytes is
Answer & Comments
indicated in view of a risk of refeeding
Correct answer: C syndrome.
E. He should be informed of a risk of a
Ribavirin has no effect on hepatitis B stoma if surgery is indicated.
replication, but used in combination with
interferon is more effective than ribavirin alone Answer & Comments
in eradicating chronic infection with hepatitis C.
Correct answer: C
Interferon was until recently first line treatment
for chronic hepatitis B infection associated with
This gentleman had two strictures at operation
elevated serum transaminases. However, there
(right hemicolectomy) which were due to
is often a flare in ALT (alamine
Crohn’s disease.The best treatment for such a
aminotransferase) on starting treatment, which
patient is surgical and not medical. There is no
in the presence of end stage liver disease
clear evidence to suggest that post-operative
(elevated PT, bilirubin and ascites here) can
medical therapy reduces the risk of recurrent
lead to liver failure so, in these circumstances, it
Crohn’s disease necessitating surgery, although
is contra-indicated.
typically Pentasa and/or immunomodulators
Lamivudine suppresses HBV replication and is such as Azathioprine are prescribed. However,
safe to use in decompensated end-stage abstinence from smoking should be encouraged
cirrhosis. as the rate of recurrence is reduced by
approximately 50% in non-smokers.
[ Q: 52 ] MasterClass Part2 In view of his recent near-total starvation and
(2010) - Gastroenterology significant weight loss, refeeding syndrome is a
possibility; careful attention to his electrolyte
A 30-year-old man presents with progressive
levels, gradual re-introduction of feeding, and
central abdominal pain and vomiting associated
replenishment of B vitamins is required. All
with significant weight loss (five stone in 3
patients undergoing colonic surgery need to be
months).He gives a history of binge drinking
aware of the possibility of stoma formation,
and depression, and smokes twenty cigarettes
even if, as in this case, it is unlikely.
per day. Because eating provokes abdominal
pain and vomiting, he has eaten virtually
nothing for a month. CT scanning of his [ Q: 53 ] MasterClass Part2
abdomen showed a normal pancreas but (2010) - Gastroenterology
dilated loops of small bowel with a possible
As part of a routine health check, a young
terminal ileal stricture. His albumin level was 20
women is found to have the following liver
and C-reactive protein level was 50.
function tests: Bilirubin 37 µmol/L (0-18), AST
Which statement is NOT true? 14 IU/L (5-40), ALT 26 IU/L (5-45), alkaline
phosphatase 125 IU/L, Albumin 40 g/L (36-45),

Dr. Khalid Yusuf El-Zohry – Sohag Teaching Hospital (01118391123) Page | 195
El-Zohry MRCP Questions Bank (Part 2) – Medical Masterclass 2010

Hb 13.7 g/dL (11.5-15.5), reticulocyte count Answer & Comments


1.2%.
Correct answer: C
What is the most likely diagnosis?
Azathioprine and 6 mercaptopurine have been
A. Chronic alcoholic liver disease
demonstrated to be effective in closing fistulae
B. Autoimmune haemolytic anaemia in trials as well as a meta-analysis, and these
C. Common bile duct stone should be considered standard therapies in
patients in whom fistulae are troublesome
D. Gilbert's syndrome (Pearson, May et al, 1995). Asymptomatic
E. Primary biliary cirrhosis. perianal fistulae in the context of Crohn's
disease, with or without minor discharge, do
Answer & Comments not necessarily require specific treatment.
If there is large volume discharge or recurrent
Correct answer: D
sepsis, then investigation in the form of pelvic
MRI scan, endoanal ultrasound and
An isolated hyperbilirubinaemia is invariably examination under anaesthesia (EUA) is
due to Gilbert's syndrome (4% of the warranted. EUA also allows the laying open of
population), which should be regarded as a fistulae and the placement of Seton sutures
variation of normal. The most important through the fistulous tracts. The Setons prevent
condition in adulthood that could give similar the fistulous tract from closing down, and thus
biochemical results would be haemolytic prevent the formation of abscess cavities along
anaemia. A reticulocyte count (raised) and the tracts. This is particularly important to
serum haptoglobin (mops up the haem. prevent destruction of the anal sphincters, and
therefore low) would differentiate. timely conservative surgical management is of
key importance. Aggressive surgical
[ Q: 54 ] MasterClass Part2 management of fistulising Crohn's disease is
(2010) - Gastroenterology usually inappropriate. Defunctioning ileostomy,
with diversion of the faecal stream, may allow
A 28-year-old woman has fistulizing Crohn's resolution of perineal sepsis, but there is a high
disease. incidence of relapse when bowel continuity is
restored.
Which one of the following statements is true?
In clinical trials infliximab has been shown to be
A. Fistulogram is indicated in peri-anal
effective in closing fistulae in 46% when given
disease to assess communication with the
at Weeks 0, 2 and 6 at a dose of 5 mg/kg
bowel.
(Present, Rutgeerts et al, 1999). However, if
B. Medical or surgical therapy is always possible, it should be used in concert with an
indicted. immunosuppressant treatment such as
C. TPN with bowel rest is a recognised azathioprine or methotrextate, both to reduce
treatment. the likelihood of provoking an immune reaction
to infliximab (which is a chimeric antibody
D. Peri-anal sepsis requires aggressive derived from both human and mouse
surgical intervention. antibodies) and to hold disease in remission
E. Repeated courses of infliximab are safe induced by infliximab. Repeated infusions of
and effective for severe disease. infliximab can be used, but efficacy tends to

Dr. Khalid Yusuf El-Zohry – Sohag Teaching Hospital (01118391123) Page | 196
El-Zohry MRCP Questions Bank (Part 2) – Medical Masterclass 2010

diminish over time and the long term biological [ Q: 56 ] MasterClass Part2
consequences of this approach are uncertain. (2010) - Gastroenterology
TPN may be indicated particularly for high
A 50-year-old woman who is generally well
fistulae in the post-surgical setting. Active
apart from having troublesome osteoarthritis of
Crohn's disease must be treated, the patient
the knees complains of profuse watery
often kept nil by mouth and proton pump
diarrhoea that has steadily worsened over the
inhibitors and octreotide used to help reduce
last 3 months.
volume of fistula output.
Fistulogram is rarely helpful in delineating Select the most appropriate response from the
fistulous tracts. MRI and EUA are generally list below:
much better. A. A full clinical evaluation, followed by
blood tests and colonoscopy are probably
[ Q: 55 ] MasterClass Part2 required
(2010) - Gastroenterology B. It is likely that this is an adverse event
related to medications she might be
A 55-year-old man is still anaemic, has some
taking, such as NSAIDs
loose stools and has failed to regain weight
several months after starting a gluten-free diet C. Watery diarrhoea with hypokalaemia,
for coeliac disease. caused by a vasoactive intestinal peptide
(VIP)-secreting tumour, is the likely
Which of the following is the commonest cause diagnosis
of failure to respond to treatment?
D. The progressive nature of the symptoms
A. Pancreatic insufficiency suggests a malignant process
B. Dietary non-compliance E. Microscopic colitis is the most likely
C. Small bowel adenocarcinoma diagnosis

D. Enteropathy associated B cell lymphoma


Answer & Comments
of the small intestine
E. Ulcerative jejunitis. Correct answer: A

Answer & Comments Chronic diarrhoea may be due to a large


number of different causes, hence a full
Correct answer: B evaluation of patients presenting in this way is
mandatory. This patient may well have used
Poor dietary compliance accounts for most NSAIDs that are associated with the
cases where there is a failure to improve on development of microscopic or lymphocytic
treatment. Ulcerative jejunitis, small bowel colitis, which is a typical cause of watery
adenocarcinoma and an enteropathy- diarrhoea in middle-aged and elderly women.
associated T cell lymphoma are recognised but However, without a full history, examination
rare complications and there is an association
and supportive investigations, this possibility is
with pancreatic sufficiency.
not necessarily highly or most likely. VIPoma is
exceedingly rare, and there is no evidence in
favour of a malignant process in this patient.

Dr. Khalid Yusuf El-Zohry – Sohag Teaching Hospital (01118391123) Page | 197
El-Zohry MRCP Questions Bank (Part 2) – Medical Masterclass 2010

[ Q: 57 ] MasterClass Part2 [ Q: 58 ] MasterClass Part2


(2010) - Gastroenterology (2010) - Gastroenterology
An 81-year-old man was admitted with a 3- A patient with progressive bulbar palsy is
week history of diarrhoea following a course of referred for placement of a percutaneous
antibiotics 4 weeks previously for a chest enterogastrostomy tube for feeding.
infection. His GP had treated him with
Imodium, but this had produced little Which of the following is correct?
improvement. On examination he was tender A. There is a significant risk of bleeding,
all over his abdomen, but without guarding. infection, and pain following the
procedure, and a substantial risk of
Which one of the following investigations would
procedure-related mortality.
be most useful in excluding a diagnosis of
pseudomembranous colitis? B. The procedure requires a general
anaesthetic and is most efficiently
A. Abdominal radiograph
performed by means of a small
B. Barium enema laparotomy.
C. Positive stool sample for C. difficile toxin C. The procedure is ideally performed under
D. Flexible sigmoidoscopy radiological guidance, which obviates the
need for endoscopy and sedation.
E. Abdominal ultrasound.
D. The main advantage for this patient
would be the reduced risk of aspiration of
Answer & Comments
gastric contents.
Correct answer: D E. In view of the likelihood that the patient
has a neurological condition such as
The diagnosis of pseudomembranous colitis motor-neurone disease, blood gas
requires direct visualisation of measurement should be obtained before
pseudomembranes on bowel mucosa or on the procedure is undertaken.
microscopic examination of a biopsy sample.
90% of cases can be detected by flexible Answer & Comments
sigmoidoscopy, although in mild cases
pseudomembranes may not be evident Correct answer: A
macroscopically. The classic sigmoidoscopic
appearance of 2 to 10 mm raised yellow PEG tube placement is often requested from
nodules are pathognomonic. the gastroenterology and endoscopy service
An abdominal radiograph may be normal in because endoscopic placement of the tube
pseudomembranous colitis, although in severe obviates the need for general anaesthesia and
cases it may show a complicating toxic surgery, and because the durability of
megacolon. Barium enema and abdominal endoscopically-placed tubes is greater than that
ultrasound are of no use in confirming or of radiologically-placed tubes. However, there
is a significant risk of complications, including
excluding the diagnosis of pseudomembranous
colitis, and a positive stool culture for C. difficile life-threatening bleeding and infection. This
toxin would make (rather than exclude) the should be explained to the patient and their
diagnosis. carers as part of the informed consent process.
Although bypassing the need to swallow may
reduce aspiration caused by disordered

Dr. Khalid Yusuf El-Zohry – Sohag Teaching Hospital (01118391123) Page | 198
El-Zohry MRCP Questions Bank (Part 2) – Medical Masterclass 2010

swallowing, a PEG tube does not prevent Answer & Comments


regurgitation of gastric contents, or their
Correct answer: CH
aspiration into the larynx, trachea, and
bronchial tree. It is quite correct to be wary of
respiratory difficulties occurring in patients with Paracentesis is an effective treatment for tense
neurological disorders who undergo endoscopy. ascites. There is no limit to the amount of fluid
However, normal blood gas measurements that can be drained providing that albumin is
could provide false reassurance – the critical infused at the same time as this prevents the
measurement is the patient’s ventilatory development of renal failure. Albumin is more
reserve, best determined by measuring their effective than crystalloids in preventing renal
vital capacity. impairment following paracentesis. Ultrasound
guidance is usually only needed after repeated
paracentesis when the fluid can become
[ Q: 59 ] MasterClass Part2 loculated. Prompt removal of the drain
(2010) - Gastroenterology prevents secondary peritonitis and the
A 45-year-old man with known cryptogenic procedure can be safely performed as a day
cirrhosis has developed tense ascites despite case. Both hepatocellular carcinoma and portal
diuretics and you are asked to perform a vein thrombosis should be excluded by
paracentesis. ultrasound in a cirrhotic who develops
worsening ascites.
Which TWO of the following statements are
true? [ Q: 60 ] MasterClass Part2
A. The risk of large bowel perforation is 1%. (2010) - Gastroenterology
B. No more than 10 litres of fluid should be A 45-year-old man with a diagnosis of cirrhosis
drained per session. due to hepatitis C and alcohol is being referred
C. 6-8g of albumin should be infused for for liver transplantation.
every 1 litre of fluid drained.
What is the correct advice to give him?
D. Saline is an alternative replacement fluid
A. The 1-year survival following
to albumin.
transplantation is >85%
E. Ultrasound guidance is usually needed.
B. The outcome of transplantation for
F. Leakage from the site of the paracentesis chronic liver disease is greater than that
is uncommon. for acute liver failure.
G. Repeat paracentesis is not as good as a C. Any liver graft will need to be HLA
Levine shunt in controlling tense ascites. matched.
H. The drain should generally be removed D. The overall transplant waiting list
within 6 hours of insertion. mortality is less than 5%.
I. The patient requires overnight stay in E. The commonest indication for liver
hospital. transplantation is alcoholic liver disease.
J. Portal vein thrombosis is not a recognised F. If screening identifies a hepatocellular
cause. carcinoma < 3cm transplantation can still
go ahead.
G. Hepatitis C rarely recurs in the liver graft.

Dr. Khalid Yusuf El-Zohry – Sohag Teaching Hospital (01118391123) Page | 199
El-Zohry MRCP Questions Bank (Part 2) – Medical Masterclass 2010

H. Ribavirin and Interferon is effective for B. Pleural aspiration


hepatitis C post transplant recurrence.
C. Echocardiography
I. Immunosuppression can be stopped 5
D. MRI Spine
years after transplantation.
E. Urinalyis
J. Previous alcohol intake is a
contraindication to liver transplantation.
Answer & Comments

Answer & Comments Correct answer: C

Correct answer: AF
This gentleman has bacterial endocarditis
secondary to translocation of Streptococcus
Livers are matched on the basis of size and ABO
bovis from his GI tract. The most life-
group alone. Up to 10% of individuals die while
threatening feature is potential for valve failure
waiting transplantation. Hepatitis C is now the
and therefore the most important investigation
commonest indication for transplantation
is echocardiography, which in this case
whilst about 15-20% of transplants are
demonstrated severe aortic valve endocarditis
performed for alcoholic cirrhosis and a period
with regurgitation. Colonoscopy confirmed a
of at least 6 months abstinence is required
malignancy whilst his weakness reflected
before assessment for transplantation. Life-long
discitis secondary to septic embolism. Pleural
immunosuppression is needed although often
aspiration was not necessary and urinalysis is of
less than that is needed in renal/cardiac
secondary value in his management (looking for
transplantation. Hepatitis C re-infection is
haematuria secondary to immune complex
universal and current antiviral therapy has a
nephritis). He was commenced on antibiotics
less than 20% chance of achieving viral
(benzylpenicillin and gentamicin) and
clearance post-transplantation.
underwent emergency aortic valve
replacement. In due course near the end of his
[ Q: 61 ] MasterClass Part2 antibiotic course he will need a transverse
(2010) - Gastroenterology colectomy. His prognosis is potentially very
good.
A 50-year-old man presented with diarrhoea,
fevers and weight loss, associated with
progressive weakness of his legs and [ Q: 62 ] MasterClass Part2
breathlessness on exertion. Initial examination (2010) - Gastroenterology
in clinic was unremarkable. Blood tests
An 84-year-old lady is referred to the medical
however revealed a CRP of 100mg/l with a
admission unit by her general practitioner with
haemoglobin of 10g/dl but a normal MCV.
an upper gastrointestinal haemorrhage. She
Chest X-ray and upper GI endoscopy was
was recently started on aspirin for a transient
normal but CT of his thorax and abdomen
ischaemic attack. Her haemoglobin on
demonstrated a transverse colon cancer
admission is 9.4g/dl.
associated with pleural and pericardial
effusions. In view of his illness he was admitted Which of the following is correct?
and blood cultures grew Streptococcus bovis.
A. Mortality from peptic ulcer haemorrhage
Which is the next most important investigation is still high and lies between 25-30%.
in this man’s management? B. Approximately 50% of patients will stop
A. Colonoscopy bleeding spontaneously.

Dr. Khalid Yusuf El-Zohry – Sohag Teaching Hospital (01118391123) Page | 200
El-Zohry MRCP Questions Bank (Part 2) – Medical Masterclass 2010

C. The finding of a clean ulcer base at E. A history of working as a nursery nurse


endoscopy is associated with a 50% risk makes hydatid disease more likely.
of rebleed.
F. Haemangiomas are an incidental finding
D. Age is not a major risk factor for in 1-2% of the population.
uncontrolled bleeding.
G. A history of chronic hepatitis C raises the
E. The presence of a non-bleeding vessel is possibilty of hepatoma even in the
associated with a 50% risk of rebleed. absence of cirrhosis.
H. Cholangiocarcinoma occurs in up to 15%
Answer & Comments of patients with primary sclerosing
Correct answer: E cholangitis.
I. A normal alpha-fetoprotein (AFP) excludes
Mortality from a peptic ulcer haemorrhage lies hepatocellular carcinoma.
between 2 and 6%. J. No further imaging is required.
80% of patients will stop bleeding
spontaneously. Answer & Comments
A clean ulcer base is not associated with further Correct answer: BH
bleeding.
People 65 years and over have a higher risk of The commonest significant focal liver lesion is
uncontrolled bleeding. the benign adenoma - found almost exclusively
in woman on the oral contraceptive pill.
The presence of a non-bleeding visible vessel on
endoscopy is associated with a 50% risk of While hepatoma usually arises in a cirrhotic
rebleeding. liver (of whatever aetiology), chronic hepatitis B
infection may give rise to hepatoma in the non-
[ Q: 63 ] MasterClass Part2 cirrhotic liver. A small but significant minority of
hepatomas are not associated with an elevated
(2010) - Gastroenterology
AFP.
A 38-year-old woman is found to have a solitary Abscess and metastases (especially from
focal lesion of the liver on ultrasound while breast) are other possibilities to be considered
being investigated for vague right upper in a woman of this age.
quadrant pains.
Hydatid disease is more common in agricultural
Which TWO statements are true? workers.

A. A history of oral contraceptive use is Haemangiomas are an incidental finding in 20%


irrelevant of the population.

B. A history of chronic hepatitis B rasies the Although ultrasound is a useful screening tool,
possibilty of hepatoma even in the and may be used to monitor lesion progression,
absence of cirrhosis. further imaging is usually recommended to
better assess the lesion.
C. The lesion is unlikely to represent a
benign adenoma.
D. A history of treated breast cancer four
years previously is irrelevant.

Dr. Khalid Yusuf El-Zohry – Sohag Teaching Hospital (01118391123) Page | 201
El-Zohry MRCP Questions Bank (Part 2) – Medical Masterclass 2010

[ Q: 64 ] MasterClass Part2 unless there are genuine reasons to believe


(2010) - Gastroenterology that the diagnosis possible.
Alpha-1-antitrypsin deficiency, rather than
A 62-year-old woman is referred to the
excess, leads to disease.
outpatient clinic for investigation of abnormal
liver function tests (LFTs). There is no upper age-limit to liver biopsy,
though the risk benefit ratio and likelihood of
Which of the following statements are true? influencing management should be weighed up
A. An ALT/AST ratio greater than one is in all patients.
suggestive of alcoholic liver disease. Patients with pre-core mutant hepatitis B will
B. Copper and caeruloplasmin should be not have detectable HBeAg but are still viraemic
measured in order to exclude Wilson's and hence infectious.
disease. Fatty liver is believed to be the commonest
C. An alpha-1-antitrypsin level 1.5 times the reason for abnormal LFT's in this country, and a
upper limit of normal may explain the significant proportion of people with fatty liver
abnormal LFT's. have type II diabetes.

D. Liver biopsy is contraindicated in patients Antimitichondrial antibodies are highly


over the age of 60 years. indicative of primary biliary cirrhosis, as
opposed to anti-nuclear antibodies, which may
E. Low albumin levels are likely to reflect be raised in autoimmune liver disease but are
malnutrition. highly non-specific.
F. A negative HBeAg excludes active Alpha-fetoprotein may be raised in hepatocyte
infective hepatitis B. regeneration in the absence of HCC, and by
G. An elevated HbA1c may be significant to contrast may be negative in the presence of
the diagnosis. hepatoma.

H. A positive anti-mitochondrial antibody is


highly specific for primary biliary cirrhosis. [ Q: 65 ] MasterClass Part2
I. A positive anti-nuclear antibody is highly
(2010) - Gastroenterology
specific for auto-immune hepatitis. A 24-year-old computer programmer is
J. An elevated alpha-feto protein is admitted with a 3-week history of malaise and
pathognomonic for hepatocellular bloody diarrhoea.
carcinoma.
Which of the following statements are true?

Answer & Comments A. Normal ESR and CRP effectively exclude


inflammatory bowel disease.
Correct answer: GH
B. Elevated alkaline phosphatase levels are
likely to be secondary to drug treatment.
An AST/ALT ratio is suggestive (though not
pathognomonic) of alcoholic liver disease. C. Full colonoscopy should be performed in
order to determine the extent of disease
It would be exceptionally rare for Wilson's
before treatment is commenced.
disease to present for the first time in someone
of this age, and tests for this condition should D. First line treatment should be a single
not be performed in patients over the age of 50 agent, oral 5-ASA derivative.

Dr. Khalid Yusuf El-Zohry – Sohag Teaching Hospital (01118391123) Page | 202
El-Zohry MRCP Questions Bank (Part 2) – Medical Masterclass 2010

E. The patient should be encouraged to Statistically, patients with ulcerative colitis have
follow a wheat-free diet. the same life expectancy as the general
population - any increase in deaths from the
F. Chances of a further relapse are
disease itself is balanced out by the fact that
approximately 40%.
patients with UC are less likely to be smokers.
G. Relapse may occur when the patient
stops smoking.
[ Q: 66 ] MasterClass Part2
H. There is a 50% chance of his twin sister (2010) - Gastroenterology
developing the condition.
A 74-year-old man with a past history of
I. Lifetime chance of colectomy is
myocardial infarction and intermittent
approximately 30%.
claudication is admitted with severe central
J. The patient’s overall life expectancy will abdominal pain. He looks unwell, with a
be reduced. tachycardia, but the abdomen is soft, without
peritonism. His WBC is elevated at 15.8x109/l
Answer & Comments and he is acidotic (pH 7.22).

Correct answer: GI The most likely diagnosis is:


A. Posterior perforation of peptic ulcer
While ESR and CRP are often utilised to monitor
ulcerative colitis, they may be normal in acute B. Appendicitis
disease, especially in distal proctitis (which may C. Myocardial infarction
present with severe symptoms).
D. Pancreatitis
Elevated alkaline phosphatase should always
E. Mesenteric ischaemia.
raise the suspicion of coexistent primary
sclerosing cholangitis. Colonoscopy should be
utilised at some point in order to determine the Answer & Comments
proximal extent of the disease. The initial Correct answer: E
diagnosis however can be made at
sigmoidoscopy, and full investigations should
Mesenteric infarction manifests as an acute
not delay the institution of therapy. In patients
abdomen in the later stages, but earlier on
with active disease, first line treatment would
there is often severe abdominal pain with little
be either topical therapies (for limited disease)
in the way of abdominal signs.
or systemic steroids.
In a patient with abdominal pain but few signs,
While there is limited evidence for dairy-free
an elevated WBC count and the presence of
diets in acute colitis, there is no association
acidosis should always raise the possibility of
with wheat-intolerance.
mesenteric ischaemia / bowel infarction,
Seventy-five per cent of patients will have a especially if the patient is an arteriopath, as in
further relapse at some point in time. this case.
There is a recognised risk of relapse when
patients stop smoking. While there is an
increased incidence of IBD in close family
members, the mechanism of inheritance is
poorly understood, and is much less than 50%
in siblings.

Dr. Khalid Yusuf El-Zohry – Sohag Teaching Hospital (01118391123) Page | 203
El-Zohry MRCP Questions Bank (Part 2) – Medical Masterclass 2010

[ Q: 67 ] MasterClass Part2 She is currently undergoing IVF treatment. Last


(2010) - Gastroenterology year she suffered a Colles’ fracture of the right
arm, and radiographs at the time commented
A 70-year-old man is admitted for investigation on the presence of osteopenia. She is known to
of anorexia and weight loss. He is 1.7m tall, was be IgA deficient.
100kg 6 months earlier and now weighs 85kg.
Which one of the following statements is
Which of the following statements is true? correct?
A. His body mass index is 29.4. A. The most likely diagnosis is Crohn’s
B. Serum albumin correlates well with the disease.
severity of malnutrition. B. Her anti-endomysial antibody test is
C. Malnutrition affects up to 20% of hospital highly likely to be positive.
in-patients. C. The clinical picture is consistent with
D. He is underweight (according to the lactase deficiency.
World Health Organization classification). D. Gastroscopy with duodenal biopsy is
E. Parenteral nutrition should be indicated.
commenced. E. A barium enema is likely to produce
helpful information.
Answer & Comments
Answer & Comments
Correct answer: A
Correct answer: D
Malnutrition affects up to half of hospital
patients. The clinical picture fits best with coeliac disease
Body mass index is calculated from and duodenal biopsies are the gold standard for
weight/height squared. The World Health diagnosis. Anti-endomysial antibodies are
Organization classifies a BMI of <18.5 as sensitive and specific, but miss the disease in
underweight, 18.5-24.9 as normal, 25-30 as about 5% of the population who are IgA
overweight and >30 as obese. deficient. A barium enema is unlikely to be
helpful given the non-bloody nature of the
Low albumin levels are more commonly due to diarrhoea; a small bowel meal may be more
inflammation, malignancy, fluid overload, and helpful to eliminate the differential diagnosis of
renal or gastrointestinal losses than Crohn’s disease.
malnutrition.
He has lost 15% of his body weight, so [ Q: 69 ] MasterClass Part2
nutritional support should be instituted.
(2010) - Gastroenterology
However, it is likely that enteral feeding is more
appropriate as it likely to be more safe, simple A 40-year-old woman with crohn’s disease who
and cheap has recently been treated with chimeric mouse-
human antibody Infliximab (which binds to and
[ Q: 68 ] MasterClass Part2 blocks the activity of tumour necrosis factor-α
(TNFα)), has recurrent disease, and it is
(2010) - Gastroenterology
apparent that the clinical response to Infliximab
A 34-year-old women presents with a 2-year infusions is waning.
history of intermittent non-bloody diarrhoea.

Dr. Khalid Yusuf El-Zohry – Sohag Teaching Hospital (01118391123) Page | 204
El-Zohry MRCP Questions Bank (Part 2) – Medical Masterclass 2010

Which of the following statements is MOST with the antibody had developed the rare and
correct? highly aggressive central nervous system
A. The declining response to treatment may disorder, progressive multifocal
be associated with antibodies against the leukoencephalopathy (PML), which is
associated with JC virus infection.
Infliximab (human anti-chimera
antibodies or HACA), and a period of
immunosuppression may suppress this [ Q: 70 ] MasterClass Part2
untoward reaction (2010) - Gastroenterology
B. The declining response to treatment is A 20-year-old man complains of intermittent
unlikely to be associated with antibodies epigastric pain for 6 months. He has lost 5kg in
against the Infliximab (human anti- weight.
chimera antibodies or HACA), as the
patient is not reported to have had an Which of the following statements are true?
anaphylactic reaction
A. Gastroscopy is the investigation of
C. The declining response to treatment may choice.
be associated with antibodies against the
B. 50% of 20-year-olds are infected with
Infliximab (human anti-chimera
Helicobacter pylori in the UK.
antibodies or HACA), and the patient may
now be effectively treated with a C. A 7-day course of a proton pump inhibitor
different antibody against TNFα, such as plus amoxycillin would be appropriate.
Adalimumab D. A 13C urea breath test may be falsely
negative if the patient is also taking non-
Answer & Comments steroidal anti-inflammatory drugs
(NSAIDs).
Correct answer: C
E. H pylori serology is only effective in
An emerging problem with the use of confirming eradication success if
“biologicals” (usually recombinant proteins) is performed at least 4 weeks after
the emergence of neutralizing antibodies that treatment.
block the efficacy of treatment, without
necessarily causing anaphylaxis or other Answer & Comments
infusion reactions. Structurally different protein
Correct answer: A
therapeutics usually remain effective, and
Adalimumab has been shown to be effective in
patients who no longer respond to Infliximab Gastroscopy would be appropriate given the
infusions due to the development of HACA. presence of the alarm symptom of weight loss.

The soluble TNF receptor Etanercept also blocks Twenty per cent of 20-year-olds and 50% of 50-
the activity of TNFα; however, for unknown year-olds are H pylori positive and the lifetime
reasons, it is not as effective as the antibodies risk of duodenal ulcer is up to 10% in these
in treating crohn’s disease. individuals. H pylori produces a urease which
release 13C from orally administered 13C urea
Natalizumab, the antibody against α4β7 which can be measured in exhaled breath
integrins, which was potentially effective in 13CO2.
some patients with crohn’s disease, has been
withdrawn from clinical use following the Proton pump inhibitors reduce H pylori load
discovery that a number of patients treated and may result in false negative tests.

Dr. Khalid Yusuf El-Zohry – Sohag Teaching Hospital (01118391123) Page | 205
El-Zohry MRCP Questions Bank (Part 2) – Medical Masterclass 2010

Suitable H pylori eradication regimens include Campylobacter, and Shigella as well as


proton pump inhibitor based triple therapies, Clostridium difficile. However, in the context of
usually including amoxycillin (or metronidazole) severe disease, it is not always possible to
and clarithromycin. H pylori antibody levels withhold steroids until negative cultures have
take many months to fall, so ensuring been obtained, and steroid therapy does not
eradication success is more appropriate with a appear to adversely affect outcome even when
urea breath test which can be done 4 weeks stool cultures come out positive. Indeed,
after treatment. infective colitis can trigger an episode of
inflammatory bowel disease.
[ Q: 71 ] MasterClass Part2 While Clostridium difficile may produce the
(2010) - Gastroenterology characteristic pseudomembranes on
macroscopic appearance, it does not always do
A 43-year-old man presents with an acute so. The histopathologist can usually
colitic illness and is passing between five and 10 differentiate idiopathic IBD from C. diff but the
bloody stools per day. gold standard remains detection of C. diff toxin
on stool assay.
Which one of the following statements is true?
Differentiating CMV colitis from ulcerative
A. Steroids are contraindicated until
colitis in which CMV inclusion bodies occur as
negative stool culture results have been
“bystanders” is virtually impossible. For
obtained.
patients with pre-existing IBD therapeutic
B. Approximately 30% of cases of acute immunosuppression may increase the risk of
severe ulcerative colitis (UC) occur in CMV infection. Where CMV inclusion bodies are
newly diagnosed cases. found or special stains demonstrate the
C. Clostridium difficile always produces presence of CMV antigen in the gut mucosa, it
characteristic endoscopic and histological is usually considered prudent to give
changes. intravenous ganciclovir. However, in patients
with known ulcerative colitis it is also necessary
D. The finding of cytomegalovirus (CMV) to treat concomitantly with intravenous
inclusion bodies in biopsy samples is steroids.
conclusive evidence for a pathogenic role
and mandates intravenous ganciclovir Amoebic dysentery usually occurs in people
instead of intravenous corticosteroid. living in or recently returned from the tropics.
However, asymptomatic individuals can
E. Amoebic dysentery can be safely continue to excrete cysts for many years, and
discounted as a possible cause five years this is the case for a small proportion of people
after last travel in the tropics living in Europe. Diagnosis is usually by
microscopy of a hot stool to identify amoebae
Answer & Comments which have phagocytosed red blood cells. These
may also be seen on biopsy specimens from the
Correct answer: B
colon. Treatment is metronidazole 800 mg
three times a day for 5-10 days, followed by
Of patients experiencing their first attack of UC, diloxanide.
20% fulfil the criteria for severe disease. At first
presentation or at the time of a subsequent
flare of acute colitis it is always important to
check stool cultures for standard bacterial
pathogens including Salmonella,

Dr. Khalid Yusuf El-Zohry – Sohag Teaching Hospital (01118391123) Page | 206
El-Zohry MRCP Questions Bank (Part 2) – Medical Masterclass 2010

[ Q: 72 ] MasterClass Part2 Late rebleeding is then reduced by the addition


(2010) - Gastroenterology of a beta-blocker (more effective than nitrates)
or serial endoscopic treatment to eradicate the
A 50-year-old alcoholic presents with his first varices. Repeated sclerotherapy is associated
variceal haemorrhage. with more oesophageal ulceration than
banding.
Which TWO of the following statements are
true? About 20% individuals will be intolerant of
betablockers.
A. Acute use of vasopressin may stop
bleeding and reduce the risk of early A proton pump inhibitor reduces the risk of
rebleeding. bleeding from banding ulcers.

B. Endoscopic management has no place in Bacteraemia and encephalopathy are common


management following the use of complications of variceal haemorrhage and
vasopressin. prophylactic antibiotics and
lactulose/phosphate enemas are indicated.
C. Beta-blockers are well tolerated by such
patients.
[ Q: 73 ] MasterClass Part2
D. Nitrates are as effective as beta-blockers
(2010) - Gastroenterology
in stopping late rebleeding.
E. Encephalopathy is rare following a A 35-year-old woman, diagnosed 18 months
variceal bleed. previously with coeliac disease (biopsy-proven),
now presents to outpatients with unresolved
F. Antibiotics are not indicated. diarrhoea. Her bowels are open up to six times
G. A proton pump inhibitor should be per day. There is no blood in the stools and she
prescribed after endoscopic therapy. does not have abdominal pain.

H. Sclerotherapy is associated with less What are the TWO most useful investigations or
complications than oesophageal banding. maneouvres to perform to investigate her
I. Any gastric varices seen at endoscopy diarrhoea?
should be injected with adrenaline. A. Lactose hydrogen breath test
J. Mechanical ventilation to secure the B. Coeliac antibodies
airway prior to endoscopy is
C. Duodenal biopsy
contraindicated.
D. Colonoscopy
Answer & Comments E. Request review by dietitian
Correct answer: AG F. Ultrasound of the abdomen
G. Glucose hydrogen breath test
Variceal haemorrhage may be the first
H. Faecal elastase-1 level
presentation of alcoholic cirrhosis. Oesophageal
varices are more common than gastric varices. I. Thyroid function tests
Initial pharmacological treatment with
J. Small bowel meal
vasopressin or somatostatin analogues is
indicated as this reduces the risk of early
rebleeding. A definitive endoscopic procedure Answer & Comments
is then indicated (ligation or sclerotherapy). Correct answer: EH

Dr. Khalid Yusuf El-Zohry – Sohag Teaching Hospital (01118391123) Page | 207
El-Zohry MRCP Questions Bank (Part 2) – Medical Masterclass 2010

The most common cause of ongoing Answer & Comments


gastrointestinal symptoms in patients with
Correct answer: A
coeliac disease is continued gluten exposure.
Dietitians are best equipped to assess this and
therefore should be involved in assessment. All veins will be constricted in this man, making
their cannulation difficult. Never attempt
Up to 30% of patients with coeliac disease and subclavian or internal jugular venous
unresolved diarrhoea have associated exocrine cannulation in someone who is obviously
pancreatic insufficiency. The faecal elastase-1 hypovolaemic: the risks are high. Approaches to
test is cheap, non-invasive and simple to use in the subclavian vein may be complicated by
an outpatient setting. pneumothorax or haemothorax, both of which
Positive coeliac antibodies may suggest could easily be fatal in this clinical situation.
continued gluten exposure but are not Some approaches to the internal jugular vein
diagnostic. Repeat duodenal biopsy may still can also be complicated by pneumothorax, and
show villous atrophy even in compliant all can be complicated by puncture of the
patients. Colonoscopy in a patient under 45 carotid artery.
years is unlikely to demonstrate a significant
The femoral vein lies medial to the artery in the
cause for diarrhoea. Although thyroid disease is groin: remember the acronym ‘NAVY’ – Nerve,
associated with coeliac disease and should be
Artery, Vein, Y-fronts. Inadvertent puncture of
screened for, only 1-2% of patients develop the femoral artery is much less likely to have
new thyroid dysfunction over a 12-month dire consequences than the complications of
period. central vein cannulation listed above.

[ Q: 74 ] MasterClass Part2 [ Q: 75 ] MasterClass Part2


(2010) - Gastroenterology (2010) - Gastroenterology
A 48-year-old man who had coronary artery A 21-year-old student develops abdominal pain,
bypass grafts 4 weeks ago is admitted with fever and diarrhoea 12 hours after eating a
melaena. He is pale and shocked, with cold chicken sandwich that had been in his fridge for
hands, P110 / min and systolic BP 90 mmHg. a week.
You cannot obtain intravenous access in his
arms. The most likely cause is:

To obtain vascular access you should: A. Campylobacter

A. insert a femoral venous catheter B. Norwalk virus

B. insert an internal jugular venous catheter C. Salmonella

C. insert a subclavain venous catheter D. E coli O157

D. insert an internal jugular or subclavian E. Giardia.


venous catheter (depending on your
experience and confidence with the two Answer & Comments
techniques)
Correct answer: C
E. place one of his arms in a bowl of warm
water to dilate the veins so that you can Campylobacter is the commonest cause of
obtain peripheral venous access bacterial gastroenteritis in the UK, but its
incubation period is three to five days. By

Dr. Khalid Yusuf El-Zohry – Sohag Teaching Hospital (01118391123) Page | 208
El-Zohry MRCP Questions Bank (Part 2) – Medical Masterclass 2010

contrast, the incubation period of salmonella> subsequently develop liver failure. The
is eight to 48 hours. Treatment of salmonella condition is steroid responsive and patients are
gastroenteritis is symptomatic. If the patient is often maintained on 7.5-10mg prednisolone or
very ill, then ciprofloxacin is the antibiotic of azathioprine monotherapy. High IgA levels are
choice, but antibiotics prolong the duration of seen in alcoholic liver disease. A high IgG level
stool carriage and should not be given i.e. 30-40g/l is very suggestive of autoimmune
routinely. hepatitis in the clinical setting described here.

[ Q: 76 ] MasterClass Part2 [ Q: 77 ] MasterClass Part2


(2010) - Gastroenterology (2010) - Gastroenterology
A 23-year-old man, previously fit and well, A 76-year-old woman was admitted with poor
presented with jaundice. His mother has mobility. Clinical examination showed a body
systemic lupus erythematosus (SLE). mass index of < 18. Her husband had died 8
Investigations show: Bilirubin 175micromol/l, months previously and she had been anorexic
ALP normal, ALT 1228 IU/l, albumin 39g/l, since.
Prothrombin time normal, smooth muscle
antibody positive 1 in 320. A liver biopsy shows Her body temperature was noted by nursing
evidence of a hepatitis with perivenular staff to be low, confirmed by rectal
collapse and a predominance of portal tract temperature.
plasma cells. A. Protein energy malnutrition (PEM) is rare
in older hospitalised patients
Which one of the following best describes this
condition? B. A body mass index of < 18 in older
patients is not associated with increased
A. It is not steroid responsive.
mortality
B. Progression to liver transplantation is
C. There is no association between PEM and
common.
hypothermia
C. It is characterised by high IgA levels.
D. There is no association between PEM and
D. Hepatic fibrosis is often present at heat intolerance
presentation in adults.
E. PEM results in smaller diameter muscle
E. An ERCP is necessary to confirm the fibres as well as loss of muscle fibres.
diagnosis.
Answer & Comments
Answer & Comments
Correct answer: E
Correct answer: D
PEM is common in older patients affecting up
Autoimmune liver disease presents as an acute to 40%.
hepatitis and jaundice may be a feature at
Severe PEM with BMI <18 is associated with
presentation. Many patients have histological
increased mortality in older patients.
evidence of fibrosis and even cirrhosis at
presentation despite being relatively There is an association with heat intolerance
asymptomatic. The prognosis with long-term and hypothermia in patients with PEM.
immunosuppression is excellent even in the PEM does result in loss of muscle fibres and
presence of cirrhosis and few patients smaller diameter muscle fibres.

Dr. Khalid Yusuf El-Zohry – Sohag Teaching Hospital (01118391123) Page | 209
El-Zohry MRCP Questions Bank (Part 2) – Medical Masterclass 2010

[ Q: 78 ] MasterClass Part2 [ Q: 79 ] MasterClass Part2


(2010) - Gastroenterology (2010) - Gastroenterology
A 54-year-old Caucasian man is referred with A previously fit 35-year-old man presents with
mildly abnormal liver function tests (ALT 74 U/l pyrexia and tachycardia, severe bloody
[normal range, 5-45], AST 63 U/l [5-40]). He has diarrhoea, and mucus per rectum. His
been complaining of tiredness and aches and symptoms have been present for three weeks.
pains in his knees, hence his GP also tested his
full blood count (FBC), ferritin and glucose, The most likely diagnosis is:
which show: Hb 15.5g/dl [12.5-15.5], ferritin A. ischaemic colitis
3452 micrograms/litre [12-200], fasting glucose
B. bacterial colitis
13.2 mmol/l [<11].
C. ulcerative colitis
Which TWO tests are most useful in establishing
a diagnosis? D. protozoal colitis

A. Autoimmune profile E. non-steroidal anti-inflammatory drug


(NSAID) induced colitis.
B. Skeletal survey
C. HbA1C Answer & Comments
D. Liver biopsy Correct answer: C
E. HFE gene test
Ulcerative colitis is most likely. An infectious
F. Hepatitis A IgM serology
colitis is less likely if the symptoms have been
G. Duodenal biopsies present for more than one week, but patients
H. Haemoglobin electrophoresis with acute severe colitis should be treated with
oral ciprofloxacin to cover bacterial colitis until
I. Radiograph of knees
the diagnosis is clear. Ischaemic colitis would be
J. Serum uric acid. very unusual at this age and as he was
previously fit significant NSAID consumption is
Answer & Comments not likely.

Correct answer: DE
[ Q: 80 ] MasterClass Part2
(2010) - Gastroenterology
The massively raised ferritin, diabetes and
arthralgia (chondrocalcinosis) strongly suggest An 82-year-old lady is admitted from her
that this man has genetic haemochromatosis. residential home with an ischaemic stroke.
Liver biopsy with estimation of hepatic iron Initial examination showed her to be
stores is the diagnostic test. Over 90% of underweight. Her swallowing was impaired and
affected individuals are homozygous for the she was put on IV fluids for 2 weeks before
HFE Cy282 mutation, and if found screening feeding was introduced via a nasogastric tube.
should be offered to relatives. Subsequent examination revealed further
Coeliac disease may present with mildly weight loss and a body index of less than 18.
abnormal liver function tests, but is associated
Which of the following statements is correct?
with iron (and folate) deficiency.

Dr. Khalid Yusuf El-Zohry – Sohag Teaching Hospital (01118391123) Page | 210
El-Zohry MRCP Questions Bank (Part 2) – Medical Masterclass 2010

A. Protein energy malnutrition (PEM) does Which test might be helpful in determining the
not increase mortality in patients with cause of his pancreatic disease?
stroke. A. Magnetic resonance
B. The rate of weight loss is independent of cholangiopancreatography (MRCP)
the catabolic state of the patient.
B. Serum lipid profile
C. Lean body mass in older people is usually C. Endoscopic ultrasound
increased with age.
D. Serum amylase
D. Appetite is reduced with age.
E. Pancreatic biopsy
E. Stroke volume is increased in patients
with PEM.
Answer & Comments

Answer & Comments Correct answer: B

Correct answer: D
Chronic pancreatitis is most commonly caused
by alcohol, but this man consumes only small
PEM increases morbidity and mortality in older quantities. MRCP may help to exclude
patients but especially those with congestive obstructive causes or congenital abnormalities,
cardiac failure and stroke. but ERCP and CT scan did not reveal either of
The rate of weight loss is increased in patients these. Endoscopic ultrasound is unlikely to
who are ill with an increased catabolic state. demonstrate the cause of pancreatitis at this
stage. Pancreatic biopsy will show evidence of
Lean body mass in older people is usually
chronic pancreatitis but is unlikely to reveal the
decreased with age.
aetiology. It is possible that his pancreatitis may
Appetite is reduced with age Heart rate and be caused by hyperlipidaemia.
stroke volume are decreased with PEM causing
an increased circulatory time.
[ Q: 82 ] MasterClass Part2
(2010) - Gastroenterology
[ Q: 81 ] MasterClass Part2
(2010) - Gastroenterology A patient consults you in clinic, and requests
the new virtual colonoscopy, which his brother
A 45-year-old man presents with chronic in America has just had. He understands that
epigastric pain radiating through to the back the test is a replacement for standard
and steatorrhoea. He consumes 4-5 units of colonoscopy, and uses radiological techniques
alcohol per week and has no family history of and computer algorithms to reconstruct an
pancreatic disease. He has no other significant image of the interior of the colon.
past medical history. CT scanning of the
abdomen reveals a dilated pancreatic duct with Regarding this technique, the following
associated calcification of the gland. statements are correct:
Subsequent endoscopic retrograde A. The technique is greatly superior to
cholangiopancreatography (ERCP) shows similar conventional colonoscopy in its ability to
findings with no dominant structure. He is detect mucosal lesions
started on Creon (pancreatin) 3 tablets tds and
there is improvement in his steatorrhoea. B. A very small risk of perforating the colon
remains, despite the lack of intra-luminal
instrumentation

Dr. Khalid Yusuf El-Zohry – Sohag Teaching Hospital (01118391123) Page | 211
El-Zohry MRCP Questions Bank (Part 2) – Medical Masterclass 2010

C. The main insuperable problem is that the introduces a small risk of colonic perforation.
presence of faecal material introduces Intravenous contrast is not always used and
diagnostic uncertainty, because the reactions are rare nonetheless. Eventually it is
Hounsfield number of faeces is close to conceivable that MRI could be adapted for
that of normal tissue virtual colonoscopy, although this is still not a
reality.
D. Hypersensitivity to iodine is a major
difficulty because of the necessity of A number of studies comparing virtual
administering intravenous contrast agent colonoscopy with conventional colonoscopy
demonstrate that the efficacy of the techniques
E. A major difference with barium enema is
is comparable, although the inability to take
the lack of intra-luminal contrast
mucosal biopsies and perform polypectomy is
F. The main disadvantage of virtual limiting, particularly as virtual colonoscopy has
colonoscopy is the inability to obtain a greater tendency to detect small mucosal
mucosal biopsies and to perform lesions, whose histological character and
polypectomy clinical significance remain undetermined.
G. The technique has gained greater Although an exciting advance radiologically,
acceptance because modern techniques extensive studies along with training of more
allow it to be performed with a very low personnel are needed before this technique will
dose of radiation to the subject replace barium enemas.

H. There is no radiation dose as magnetic


resonance imaging is used [ Q: 83 ] MasterClass Part2
(2010) - Gastroenterology
I. In addition to not having to have
instrumentation of the bowel, the main A 57-year-old man with a past history of heavy
advantage to patients is that they do not alcohol intake presents to the acute medical
have to use powerful laxatives to take after vomiting approximately 500ml fresh
evacuate the bowel beforehand blood. Examination reveals blood pressure
J. Widespread use of this technique has 110/60 mmHg, pulse 90/min. He is jaundiced
meant that barium enemas are no longer and has widespread spider naevi. Abdominal
indicated in routine clinical practice. examination reveals tense ascites.

Which of the following treatments is most likely


Answer & Comments to be beneficial in his initial management?
Correct answer: BF A. Propranolol 40mg orally
B. Terlipressin 2mg intravenous bolus
Virtual colonoscopy involves the use of high-
resolution CT scanning, with rapid, modern C. Omeprazole 40mg intravenous bolus
scanning, and computer-assisted analysis D. Tranexamic acid 1g intravenously
providing the operator with 2-dimensional and
E. Octreotide 50mg per hour by intravenous
3-dimensional reconstructions of the intestine’s
infusion.
lumen. Patients have extensive bowel cleansing
and also ingest a small amount of barium,
which infiltrates any residual faecal matter so Answer & Comments
that the faecal matter is relatively radio-opaque Correct answer: B
and distinguishable from tissue. The bowel is
also insufflated with air. This procedure

Dr. Khalid Yusuf El-Zohry – Sohag Teaching Hospital (01118391123) Page | 212
El-Zohry MRCP Questions Bank (Part 2) – Medical Masterclass 2010

Given the clinical evidence of chronic liver Answer & Comments


disease and portal hypertension, this man
Correct answer: E
should be considered to have had variceal
bleeding until proven otherwise. Early
diagnostic and therapeutic endoscopy should Liver functon tests correlate poorly with
be undertaken as soon as possible following histology in PBC – the disease may progress
resuscitation. Terlipressin, 2mg stat followed by insidiously with normal or near-normal LFTs.
2mg 6-hourly, has been shown to be as Classically IgM levels are elevated. Liver
effective as balloon tamponade in controlling transplantation has a good prognosis (90–95%
variceal haemorrhage, and although survival) in this condition, though many
somatostatin has been shown to be effective in patients are too old for consideration at the
establishing haemostasis there are no time of presentation. No benefit has been
consistent data to support the use of its demonstrated in clinical trials of steroids,
synthetic analogue Octreotide. azathioprine or methotrexate. Osteoporosis is a
common complication, possibly due to vitamin
Propranolol is beneficial in secondary D malabsorption and/or premature ovarian
prevention, but has no role in the acute setting.
failure. All patients with PBC should be
Intravenous omeprazole and tranexamic acid screened for the condition.
may be beneficial in the context of peptic ulcer
bleeding, following endoscopic confirmation,
but have no place in the management of [ Q: 85 ] MasterClass Part2
variceal bleeding or undiagnosed upper GI (2010) - Gastroenterology
haemorrhage.
A 26-year-old woman, who had undergone a
panproctocolectomy with ileoanal anastomosis
[ Q: 84 ] MasterClass Part2 and pouch formation for ulcerative colitis,
(2010) - Gastroenterology presents with a 2-week history of diarrhoea and
rectal bleeding.
A 74-year-old woman presents with a history of
pruritus and lethargy. Which of the following is correct?

On investigation she is found to have a mildly A. The differential diagnosis includes pouch
elevated alkaline phosphatase and a positive inflammation (pouchitis), recurrent
anti-mitochondrial antibody. ulcerative colitis, and anorectal cancer
among the most likely conditions
A. The level of the alkaline phosphatase
correlates well with the extent of hepatic B. A clinical diagnosis of pouchitis could be
fibrosis made, and if the condition were severe,
treatment with oral steroids would be the
B. IgA levels are likely to be elevated
first choice
C. Liver transplantation has a poor outcome
C. A clinical diagnosis of pouchitis could be
for this condition
made, and if the condition were severe,
D. Azathioprine has been shown to be an treatment with rectal steroid enemas
effective treatment would be the first choice
E. The patient should undergo bone mineral D. A clinical diagnosis of pouchitis could be
densitometry. made, and if the condition were severe,
treatment with oral antibiotics would be
the first choice

Dr. Khalid Yusuf El-Zohry – Sohag Teaching Hospital (01118391123) Page | 213
El-Zohry MRCP Questions Bank (Part 2) – Medical Masterclass 2010

E. A clinical diagnosis of pouchitis could be E. Ulcerative colitis


made, and a further endoscopic
F. Caecal carcinoma
examination should be avoided until the
attack has abated, to minimize the risk of G. Irritable bowel syndrome
iatrogenic perforation of the pouch H. Giardiasis
I. Tropical sprue
Answer & Comments
J. Laxative abuse.
Correct answer: D
Answer & Comments
Inflammation of the ileoanal pouch is a
frequent and recognized complication of this Correct answer: BE
surgery when the indication is ulcerative colitis,
occurring in 30-60% of cases. It may be more While any of the conditions in the list can
frequent and severe if there is concomitant present as diarrhoea, the presence of blood in
primary sclerosing cholangitis (PSC), and seems the stool and arthralgia is suggestive of
to be rare when the indication for surgery is inflammatory bowel disease. Bowel cancer can
familial adenomatous polyposis coli. also present with diarrhoea and rectal bleeding
Ulcerative colitis does not recur if the rectal but is much more rare in this age group.
mucosa has been removed, although a cuff of
rectal mucosa remaining after surgery may well [ Q: 87 ] MasterClass Part2
become inflamed. This is rare, as is anorectal (2010) - Gastroenterology
carcinoma in the rectal stump. Oral and rectal
steroids are ineffective in treating pouchitis, A 35-year-old female teacher is referred for
while there is usually a rapid response to oral investigation of abnormal liver tests, which
antibiotic therapy such as Ciprofloxacin or comprise 2-3 fold elevations in the
Metronidazole. The diagnosis may be made aminotransferase. There is no history of liver
clinically although sigmoidoscopy and biopsy disease, and a screen for viral hepatitis is
are probably worthwhile, particular for the first negative. In the trawl of autoantibody tests, a
episode, for recurrent or frequent episodes, or positive anti-tissue transglutaminase antibody
where first-line therapy fails and the diagnosis test is highlighted.
may be in doubt.
Which of the following statements is TRUE?
A. Coeliac disease is recognised as a cause
[ Q: 86 ] MasterClass Part2
of abnormal liver tests, and may be
(2010) - Gastroenterology associated with inflammatory infiltration
A 24-year-old man presents with a 6-month of the hepatic portal tracts, and
history of watery diarrhoea with blood, weight progressive liver disease
loss and arthralgia. B. There is an association of coeliac disease
and Primary Biliary cirrhosis (PBC)
The TWO most likely diagnoses would be:
C. There is an association of coeliac disease
A. Bacterial overgrowth
and primary sclerosing cholangitis (PSC)
B. Crohn's disease
D. The most likely diagnosis is autoimmune
C. Pancreatic carcinoma hepatitis, which is frequently associated
with a raised serum immunoglobulin
D. Antibiotic-induced diarrhoea

Dr. Khalid Yusuf El-Zohry – Sohag Teaching Hospital (01118391123) Page | 214
El-Zohry MRCP Questions Bank (Part 2) – Medical Masterclass 2010

level, and a variety of autoantibodies D. Fistulae require surgical intervention in


including anti-tissue transglutaminase Crohn’s disease.
E. The presence of a positive tissue E. Dietary restriction an important part of
transglutaminase antibody is highly the treatment
suggestive of a diagnosis of coeliac
disease in this context Answer & Comments

Correct answer: A
Answer & Comments

Correct answer: E The relative risk of Crohn’s disease is three-fold


greater in smokers. A similar association exists
A positive tissue transglutaminase antibody is for ulcerative colitis and non-smoking such that
highly suggestive of a diagnosis of coeliac nicotine patches have been used in mild to
disease in any context. It is well recognised that moderately active disease.
liver chemistry may be abnormal in coeliac
Infectious colitis can present with identical
disease, and pathologically this may be
symptoms and macroscopic appearance as
associated with inflammatory cell infiltrates.
idiopathic inflammatory bowel disease.
There are also recognised associations between
coeliac disease and PBC and PSC, although Whilst surgery may have an important role,
there are some discrepancies between different anti-tumour necrosis factor antibody is licensed
studies. There is no satisfactory explanation for for use in both refractory and fistulating
a link between these conditions. Crohn’s disease.

Autoimmune hepatitis typically is associated Malnutrition is a predictor of poor outcome in


with a raised serum immunoglobulin level, and many conditions, including Crohn’s disease.
a variety of autoantibodies against hepatic Even if an elemental diet is not the chosen
antigens, as well as a positive antinuclear therapeutic modality, nutritional support
antibody and frequently positive smooth should form an important part of management.
muscle antibody.
[ Q: 89 ] MasterClass Part2
[ Q: 88 ] MasterClass Part2 (2010) - Gastroenterology
(2010) - Gastroenterology A 42-year-old man with long-standing ulcerative
A 38-year-old smoker presents with 4 month's colitis presents with fatigue, pruritus and an
diarrhoea, abdominal pain and weight loss. She elevated alkaline phosphatase. Endoscopic
has noticed uncomfortable red marks on her retrograde cholangiopancreatography (ERCP)
legs. shows irregular stricturing and beading of the
intra and extra-hepatic bile ducts.
Which of the following statements is true?
Which of the following statements are correct?
A. Crohn’s disease is more likely than
ulcerative colitis. A. The presence of pANCA antibodies are
diagnostic of primary sclerosing
B. Nicotine patches may be of therapeutic
cholangitis (PSC).
benefit.
B. Primary sclerosing cholangitis is more
C. A stool culture is not necessary in the
common in women than men.
absence of recent foreign travel.
C. Liver biopsy may be normal.

Dr. Khalid Yusuf El-Zohry – Sohag Teaching Hospital (01118391123) Page | 215
El-Zohry MRCP Questions Bank (Part 2) – Medical Masterclass 2010

D. Methotrexate has been shown to [ Q: 90 ] MasterClass Part2


improve outcome. (2010) - Gastroenterology
E. The use of ursodeoxycholic acid will
A 36-year-old ex-intravenous drug user is HCV-
significantly improve outcome.
RNA positive with persistently elevated
F. The presence of a cholangiocarcinoma transaminases, and has evidence of moderate
should prompt urgent referral for liver to severe inflammation on liver biopsy.
transplantation.
Which TWO of the following drugs may be used
G. Fever and jaundice should be treated
in her treatment?
with metronidazole.
A. Amantadine
H. Biliary brushings at ERCP have a low pick-
up for cholangiocarcinoma. B. Lobucavir

I. Dominant, non-malignant strictures may C. Lamivudine


benefit from stenting. D. Beta-interferon
J. E. Ribavirin
F. Anti TNF-alpha antibodies
Answer & Comments
G. Gancyclovir
Correct answer: CI
H. Alpha-interferon

Antibodies to pANCA are present in 60-80% of I. Famcyclovir


individuals with PSC but are not specific. The J. Acyclovir.
condition is more common in men (70:30 ratio).
Liver biopsy may be normal, and ERCP is the
Answer & Comments
definitive test. Neither steroids, azathioprine,
methotrexate nor cyclosporin have been shown Correct answer: EH
to benefit outcome. Urso may improve liver
function tests LFT's) and pruritus but does not Combination therapy with alpha-interferon
alter prognosis, although early data suggest a (which may be pegylated in order to produce
reduction in the risk of neoplasia. Cholangitis smoother drug level profiles and reduce side
should be treated with an antibiotic with good effects) and ribavirin is now considered
biliary penetration such as ciprofloxacin. standard therapy of chronic hepatitis C
Dominant strictures should be brushed, which infection.
while not infallible has a sensitivity for
cholangiocarcinoma of about 70%. Dominant The development of new anti-virals and new
strictures can be stented, but patients with combinations of existing anti-virals offer further
end-stage liver disease should be considered hope for the future, but currently have no place
for transplantation. Cholangiocarcinoma is outside of clinical trials.
however an absolute contraindication to
transplantation, as they do universally badl [ Q: 91 ] MasterClass Part2
(2010) - Gastroenterology
A 60-year-old woman with primary biliary
cirrhosis and a history of previous variceal
haemorrhage some 10 years ago, is admitted
on the medical take with confusion, drowsiness

Dr. Khalid Yusuf El-Zohry – Sohag Teaching Hospital (01118391123) Page | 216
El-Zohry MRCP Questions Bank (Part 2) – Medical Masterclass 2010

and fever. A diagnosis of urinary tract infection A. Diclofenac


is made, she is treated with antibiotics, and her
B. Lisinopril
encephalopathy resolves.
C. Warfarin
She returns for follow up in the outpatient
clinic, where she tells you that she is better, but D. Digoxin
that she is continuing to have difficulty E. Hormone replacement therapy
concentrating at work, and that she is not
F. Lanzoprazole
sleeping properly at night.
G. Amoxicillin
Which one of the following should you
consider? H. Ciprofloxacin

A. Investigation for recurrent urinary tract I. Paracetamol


infection J. Temazepam
B. MRI head to exclude a space-occupying
lesion Answer & Comments

C. Serum ammonium determination Correct answer: AI


D. Referral to a liver transplant centre for
review/assessment Nonsteroidal anti-inflammatory agents are a
common cause of abnormal liver function tests.
E. Neuropsychological assessment
Cholestatic liver function tests can occur as an
idiosyncratic reaction to chlorpromazine, the
Answer & Comments dextropropoxyphene component of co-
Correct answer: D proxamol, and fluxcloxacillin.
Although any drug can cause abnormal liver
This patient has advanced liver disease. She is function tests this has only rarely been reported
reporting typical symptoms of chronic hepatic with the other drugs listed, eg. warfarin,
encephalopathy, with reduced concentration lanzoprazole.
span and disturbance of the sleep-wake cycle, Do not forget that inadvertent paracetamol
and therefore should be considered for liver poisoning is not uncommon.
transplantation. Early referral is advised, so the
transplantation timing can be optimised. With respect to drug induced liver disease,
some drugs cause direct toxicity and the injury
is generally predictable, dose-related, and
[ Q: 92 ] MasterClass Part2 characteristic for the drug. Other drugs produce
(2010) - Gastroenterology damage in an apparently idiosyncratic way - the
injury is usually unpredictable and not dose-
A 66-year-old woman is admitted with an acute
related. Most acute cases can be divided into
hepatitis (ALT 670 IU/l). She is taking many
hepatocellular, cholestatic (with or without
prescribed medicines, some of which have
inflammation), and miscellaneous reactions
recently been changed. Of note she has had a
(e.g. mixed forms of hepatic dysfunction, or
painful right hip following a fall seven days ago.
granulomatous reactions). Some drugs can
In the previous month she has taken the produce chronic damage, including chronic
following drugs, which TWO are most likely to hepatitis and tumours.
be the cause of her abnormal liver Mild jaundice may also occur as a result of
biochemistry? drug-induced haemolysis (unconjugated

Dr. Khalid Yusuf El-Zohry – Sohag Teaching Hospital (01118391123) Page | 217
El-Zohry MRCP Questions Bank (Part 2) – Medical Masterclass 2010

hyperbilirubinaemia), without true hepatic referral to a specialist centre for consideration


damage or abnormal liver function tests. of transplantation are arterial pH<7.3, INR>3,
encephalopathy, creatinine >200 micromol/l or
[ Q: 93 ] MasterClass Part2 hypoglycaemia.

(2010) - Gastroenterology
[ Q: 94 ] MasterClass Part2
A 47-year-old man presents with a 1-day history
(2010) - Gastroenterology
of increasing confusion, drowsiness and
jaundice. He has been suffering from An 81-year-old lady is admitted with a 2-week
depression for the last 6months, but has no history of diarrhoea. Six weeks prior to
other significant past medical history. admission she had been treated with a course
of antibiotics. Clinical examination reveals
The TWO most likely causes of his acute liver dehydration. Her full blood count shows a
failure are: neutrophilia and C-reactive protein is elevated
A. Hepatitis A at 120 mg/l (normal range <6 mg/l). A stool
sample is positive for Clostiridium difficile.
B. Autoimmune chronic active hepatitis
C. Primary biliary cirrhosis Which one of the following statements is
correct?
D. Hepatitis B
A. C. difficile infection is always newly
E. Budd Chiari syndrome
acquired.
F. Paracetamol overdose B. The antibiotics given 6 weeks ago is not
G. Hepatitis C responsible for her present diarrhoea.
H. Leptospirosis C. Older people are at higher risk of
developing C. difficile infection than
I. Drug induced (not paracetamol)
younger people.
J. Wilson’s disease.
D. Older people have similar rates of relapse
after treatment as younger people.
Answer & Comments
E. C. difficile does not disrupt intercellular
Correct answer: FI tight junctions in bowel mucosa.

The three commonest causes of acute liver Answer & Comments


failure are paracetamol overdose, non-A non-B
non-C viral hepatitis (cryptogenic) and drug- Correct answer: C
induced. Has he taken an overdose, and has he
been exposed to any new drug in the last two C. difficile diarrhoea may be newly acquired or
weeks, particularly a non-steroidal anti- may be due to endogenous overgrowth
inflammatory agent? Is he at risk of exposure to following disruption of the normal flora, usually
hepatitis B? by antibiotic therapy.
In cases of paracetamol overdose remember Symptoms may develop as early as one day
that N-acetylcysteine improves prognosis, even after exposure to antibiotics, although typically
in patients who present more than 16 hr they develop 5 to 10 days after initiation of
afterwards. On day 2 (24-48 hours) after a antibiotic therapy. They can also arise as late as
paracetamol overdose the indications for 10 weeks after cessation of antibiotic
treatment.
Dr. Khalid Yusuf El-Zohry – Sohag Teaching Hospital (01118391123) Page | 218
El-Zohry MRCP Questions Bank (Part 2) – Medical Masterclass 2010

Older patients are at higher risk of developing bilirubin is an indicator of poor prognosis, but
C. difficile diarrhoea and having recurrent generally the level of alkaline phosphatase does
disease than younger people. not correlate with the extent of liver disease.
Only the smallest bile ducts are affected - best
C. difficile Toxin A causes disruption of the
seen on a liver biopsy. The antibody is directed
intercellular tight junctions causing fluid
against the E2 epitope of the inner
secretion.
mitochondrial membrane. Pruritus may
respond to nalTRExone. The role of UDCA is
[ Q: 95 ] MasterClass Part2 controversial, but its use is generally restricted
(2010) - Gastroenterology to symptomatic patients. PBC is associated with
a statistically increased incidence of a number
A 64-year-old woman is referred to the
of malignancies and autoimmune conditions.
outpatient clinic having been found to have a
moderately elevated alkaline phosphatase on a
background of pruritus. Subsequent [ Q: 96 ] MasterClass Part2
investigations reveal a positive anti- (2010) - Gastroenterology
mitochondrial antibody (AMA).
A 69-year-old man has histologically proven
Which of the following statements are correct? cirrhosis secondary to a combination of chronic
HCV infection and chronic alcohol abuse. He has
A. The condition is equally common in men
a history of oesophageal varices, which were
and women.
banded six months previously. He has an
B. IgG levels are often elevated. albumin of 28g/l and his prothrombin time is
prolonged by four seconds. He consults his GP
C. Alkaline phosphatase levels act as a
with minor ailments.
reliable prognostic marker.
D. Bile duct damage is best identified at Which of the following drugs should NOT be
MRCP. prescribed?
E. The AMA is directed against the E2 A. Senna
epitope of the cell membrane. B. Isosorbide mononitrate
F. Patients are at high risk of osteoporosis. C. Quinine sulphate
G. Pruritus may respond to naloxone. D. Trimethoprim
H. Ursodeoxycholic acid improves survival E. Naproxen
and should be commenced in all patients.
F. Paracetamol
I. It is associated with a reduced risk of
gynaecological malignancies. G. Sodium docusate

J. The condition is associated with CREST H. Magnesium trisilicate


syndrome. I. Temazepam
J. Chlopheniramine
Answer & Comments

Correct answer: FJ Answer & Comments

Correct answer: EI
Ninety percent of cases of PBC occur in women.
Levels of IgM are classiclly elevated. Rising

Dr. Khalid Yusuf El-Zohry – Sohag Teaching Hospital (01118391123) Page | 219
El-Zohry MRCP Questions Bank (Part 2) – Medical Masterclass 2010

Non-steroidal anti-inflammatory drugs may Wilson’s disease is an autosomal recessive


precipitate oesophageal haemorrhage and/or disease in which excess copper is deposited in
hepatorenal failure. Benzodiazepines may the liver, basal ganglia, cornea and other
accumulate and precipitate hepatic organs. It is due to a reduced production of the
encephalopathy. Paracetamol at standard copper carrying enzyme, ferroxidase. Most, but
therapeutic dosages may be used in patients not all, patients present between the ages of 5
with end-stage liver disease and 30.
Five to ten per cent of normal individuals have a
[ Q: 97 ] MasterClass Part2 low serum caeruloplasmin. Most patients with
(2010) - Gastroenterology Wilson’s disease have low serum
caeruloplasmin, low serum copper and high
A 32-year-old man is referred from a neurology urinary copper concentrations.
clinic having been found to have a
parkinsonian-like syndrome and mildly A slit lamp examination is helpful in detecting
abnormal liver function tests. He has a Kayser-Fleishcer rings, which can occasionally
psychiatric history and a reliable witness denies be found in other cholestatic liver diseases.
the previous use of alcohol use or psychotropic Haemolysis is a recognized feature and family
medication. screening should be performed as copper
chelation with penicillamine results in an
Which TWO of the following statements are excellent prognosis in the absence of
true? irreversible damage.
A. The condition is due to an excess of the
copper carrying enzyme, ferroxidase. [ Q: 98 ] MasterClass Part2
B. Requesting a slit lamp examination of his (2010) - Gastroenterology
eyes may be helpful.
Two weeks after returning from a beach holiday
C. A low caeruloplasmin level is diagnostic. in the Eastern Mediterranean a 28-year-old
D. The patient is too old for a diagnosis of woman presents with jaundice, anorexia and
Wilson’s disease. nausea. She has an ALT of 1507IU/L, and no
biliary dilatation on ultrasound.
E. A haemolytic anaemia suggests chronic
autoimmune hepatitis. Which of the following comments are correct?
F. Treatment of this condition is ineffective. A. The presence of HBsAb and absence of
G. Screening of first degree family relatives HBcAb implies acute hepatitis B infection.
is appropriate. B. Irrespective of the history, she should be
H. Regular venesection is indicated. referred to an alcohol counsellor.

I. Systemic steroids are indicated. C. Patients with acute hepatitis A should be


admitted directly to a Liver Unit.
J. Kayser-Fleischer rings are a diagnostic
finding. D. She should be 'signed-off' from her job at
a sandwich bar for the next three
months.
Answer & Comments
E. She should commence treatment with
Correct answer: BG
alfa-interferon as soon as possible.

Dr. Khalid Yusuf El-Zohry – Sohag Teaching Hospital (01118391123) Page | 220
El-Zohry MRCP Questions Bank (Part 2) – Medical Masterclass 2010

F. HAV IgG should be requested to confirm A. He is clinically obese.


the diagnosis of acute hepatitis A.
B. Obesity has a genetic component.
G. The cholestatic phase of hepatitis A may
C. He is at increased risk of developing
last for several months.
diabetes.
H. She should be advised to be vaccinated
D. He most likely has an underlying
before her return trip next year in order
endocrine disorder.
to prevent a repeat occurrence.
E. He is at increased risk of developing
I. She will not be able to donate blood in the
cerebrovascular disease.
future.
F. He should be put on a strict 1000kcal/day
J. Mortality from acute hepatitis A is less
diet.
than 1 in 2000.
G. Hypothyroidism should be excluded.
Answer & Comments H. Calorie controlled diet should be
combined with long-term low-intensity
Correct answer: GJ
exercise.

HBsAb+ve, HBcAb-ve implies hepatitis B I. His abnormal LFTs may be due to fatty
vaccination. The findings in this woman are liver.
consistent with acute hepatitis A and the J. Gastroplication should be reserved for
diagnosis should be confirmed with a positive extreme cases where other methods have
HAV IgM. Patients do not routinely need failed.
admission, unless there is evidence of liver
failure (rising PT, encephalopathy). Answer & Comments
Occasionally, transplantation is required though
mortality is <0.04%. The majority of patients Correct answer: DF
experience a variable cholestatic phase (as
transaminases fall) before complete resolution This man has a BMI of 34 with a WHR>1; by
and subsequent immunity. Faecal shedding of definition obese. Patients are at increased risk
the virus has normally ceased before clinical of a wide range of illnesses including diabetes
disease is apparent. Scrupulous hand hygiene is and cardiovascular disease. The condition
essential to prevent transmission of the virus usually occurs due to a simple imbalance of
while shedding. Blood donations are screened energy input and expenditure and rarely has an
for.previous exposure to hepatitis B and C but underlying primary endocrine disorder.
not A. Management should aim for gradual weight
loss with a 600kcal/day deficit and gentle long-
[ Q: 99 ] MasterClass Part2 term exercise program. Orlistat may be
beneficial if added in to these treatment
(2010) - Gastroenterology
modalities. Surgery remains controversial and
A 44-year-old man with a family history of should be reserved for severe and resistant
diabetes is referred by his GP because of mildly cases
elevated LFT's. He weighs 108Kg, and is 1.78m
tall. His waist measures 168cm and his hips
140cm.

Which of the following statements are FALSE?

Dr. Khalid Yusuf El-Zohry – Sohag Teaching Hospital (01118391123) Page | 221
El-Zohry MRCP Questions Bank (Part 2) – Medical Masterclass 2010

[ Q: 100 ] MasterClass Part2 The image shows the second and third part of
(2010) - Gastroenterology the duodenum, which is smooth with an
absence of well-defined ridges and the normally
In the case of a patient presenting with iron velvety appearance imparted by thousands of
deficiency anaemia and the accompanying villi. The red colour of the mucosa is typical and
endoscopic appearances of a part of the small does not signify fresh or recent haemorrhage.
intestine (see image). Ulceration usually produces a superficial slough
that is typically green or yellow-brown, and
there is no evidence of ulceration in this
picture. The terminal ileum may appear very
similar to this image, but usually bile-stained
fluid is present, and in any case there is no
ulceration present.

[ Q: 101 ] MasterClass Part2


(2010) - Gastroenterology
A 19-year-old male student returns from a
three-week tour of South East Asia and
complains of a two-week ongoing history of
diarrhoea.

Which of the following statements are TRUE?

Which of the following statements are true: A. The most likely pathogen is Yersinia
Enterocolitica
A. The photograph shows an ulcerated
terminal ileum with evidence of lymphoid B. Salmonella usually presents with bloody
nodular hyperplasia. diarrhoea

B. The prominent mucosal folds and C. Infective diarrhoea can be differentiated


atrophic villi are consistent with a from IBD at sigmoidoscopy
diagnosis of coeliac disease. D. The incubation period of shigella is 10-14
C. Shallow ulcers with punctate fresh days
haemorrhages are the likely cause of the E. Entamoeba histolytica infection may lead
anaemia. The appearances are typical of to liver abscesses in 1-3% of cases
NSAID induced enteropathy
F. The development of Reiter's syndrome
D. Fresh haemorrhage from beyond the makes campylobacter infection unlikely
field of view indicates a bleeding point in
G. Yersinia infection may cause terminal
the distal small intestine. These findings
ileitis
are highly suggestive of a bleeding ulcer
within a Meckel’s diverticulum. H. Antibiotics are indicated for all culture
positive infections
E. The appearances are entirely normal.
I. It is safe for him to return to his holiday
Answer & Comments job in a local cafe

Correct answer: B J. There is no role for prophylactic


antibiotics on future trips

Dr. Khalid Yusuf El-Zohry – Sohag Teaching Hospital (01118391123) Page | 222
El-Zohry MRCP Questions Bank (Part 2) – Medical Masterclass 2010

Answer & Comments


Which of the following statements is true?
Correct answer: EG
A. This is more typical of ulcerative colitis
than Campylobacter infection.
The commonest cause of travellers diarrhoea is
enterotoxigenic E.Coli . Unlike some of the B. Relapse after antibiotic treatment of
other causes of bacillary dysentery, salmonella Clostridium difficile infection is rare.
rarely causes bloody diarrhoea. Sigmoidoscopic C. If this patient is known to have ulcerative
and histological examinations may be colitis, no stool culture is necessary.
indistinguishable in gastrointestinal infections
from idiopathic IBD. The two conditions may D. The presence of erythema nodosum
also coexist. The incubation period of shigella is means that the diagnosis is ulcerative
2-3 days. Any of the bacterial enteritides may colitis.
result in a number of extrahepatic E. Enterohaemorrhagic E. coli is associated
manifestations including Reiter's syndrome and with haemorrhagic colitis and haemolytic
Guillain-Barre syndrome. uraemic syndrome.
Most infections are self-limiting, and in the
absence of septicaemia, antibiotics may simply Answer & Comments
prolong stool carriage. Negative cultures are
Correct answer: E
required for some occupations (including food-
handlers) before they can return to work. Some
groups would advocate the use of prophylactic Sigmoidoscopic appearance of infectious colitis
ciprofloxacin or doxicycline as prophylaxis for and chronic idiopathic inflammatory bowel
high-risk areas. disease may be indistinguishable. The mucosal
appearance of Clostridium difficile infection is
also variable, although the adherent yellow-
[ Q: 102 ] MasterClass Part2 white plaques, or ‘pseudomembrane’, is
(2010) - Gastroenterology characteristic.
An elderly patient is admitted with 5 days Relapse after treatment of Clostridium difficile
diarrhoea. Sigmoidoscopy appearance is shown is common, occurring in 30% of cases.
(see image).
Erythema nodosum may occur with
Campylobacter infection.

[ Q: 103 ] MasterClass
Part2 (2010) -
Gastroenterology
A 35-year-old woman with a body mass
index of 35 kg/m2 presents to her
family doctor with mild right upper
quadrant pain. A subsequent
abdominal ultrasound shows an echo
bright liver consistent with fatty liver.
She is found to be diabetic and

Dr. Khalid Yusuf El-Zohry – Sohag Teaching Hospital (01118391123) Page | 223
El-Zohry MRCP Questions Bank (Part 2) – Medical Masterclass 2010

hepatitis C positive; genotype 3, with a high A. Pain


viral load.
B. Oesophageal tear requiring stent
placement
Which of the following statements are TRUE:
C. Aspiration pneumonia
A. Hepatitis C genotype 3 is associated with
insulin resistance and hepatic steatosis D. Oesophageal rupture necessitating
oesophagectomy
B. Pegylated inteferon and Ribavarin would
be inappropriate for this patient E. Bleeding
C. A liver biopsy would be contraindicated in
this patient in view of her weight Answer & Comments

D. Weight loss would not aid the response Correct answer: D


to treatment
E. The risk of hepatocellular carcinoma is Therapeutic endoscopy carries significant risks
not increased in this patient and patients must be informed appropriately
before consenting to any procedure. The
principal significant complications of
Answer & Comments
oesophageal dilatation, apart from mild
Correct answer: A pain/discomfort are perforation, pulmonary
aspiration, and bleeding. A UK audit reported
Right upper quadrant pain is a known an overall perforation rate of 2.6% with a
association with fatty liver disease, as is type 2 mortality of 1%. Perforation was less common
diabetes. With chronic hepatitis C, there is an following dilatation of benign strictures (1.1%
association with insulin resistance and weight with a mortality of 0.5%) than following
loss and insulin sensitising drugs may improve dilatation and/or intubation of malignant
response to anti-viral therapy, which is clearly strictures (6.4% with a mortality of 2.3%).
appropriate in this case. A liver biopsy would be Elderly patients appeared at greater risk. The
necessary to determine the stage of hepatitis C. risk of perforation in achalasia is reported to be
The risk of hepatocellular carcinoma is higher, at up to 7% (mostly 3–4%) with a
increased here both by the diabetes and the mortality of <1%.
chronic hepatitis C. To diagnose perforation requires a high index of
suspicion and typical symptoms include chest
[ Q: 104 ] MasterClass Part2 pain, and dysphagia. Signs include pyrexia,
(2010) - Gastroenterology tachycardia, hypotension, tachypnoea.
Subcutaneous emphysema may be present.
An 83-year-old woman is admitted from A&E Undiagnosed systemic sepsis rapidly develops
unable to swallow solids and with increasing and death often occurs with 48 hours. Chest x-
difficulty for liquids. She is otherwise well, and ray may show pleural air / fluid levels and
blood tests are normal. Barium swallow and mediastinal emphysema. A water-soluble
upper GI endoscopy reveals a tight, smooth contrast swallow can confirm diagnosis.
stricture at 34cm and brushings and biopsies
Conservative management may be appropriate
show no malignant cells. Oesophageal
for small perforations without systemic upset
dilatation is planned.
and small-contained thoracic leaks. Treatment
In the consent discussed with the patient, which requires nil by mouth, antibiotics and
is least likely to occur? intravenous hydration. Failure of conservative

Dr. Khalid Yusuf El-Zohry – Sohag Teaching Hospital (01118391123) Page | 224
El-Zohry MRCP Questions Bank (Part 2) – Medical Masterclass 2010

management will need surgery. If operated on those with a risk of 1 in 12 or greater (which
within 24 hours mortality is 5-10% and if includes those with one first degree and one
delayed more than 48 hours mortality is more second-degree relative or one first-degree
than 50%. relative under 45 years of age).
HNPCC is an autosomal dominant condition
[ Q: 105 ] MasterClass Part2 with a lifetime risk of colon cancer of 1 in 2.
(2010) - Gastroenterology Barium enema misses 10-20% of cancers and
A 30-year-old asks for advice regarding his polyps over 1 cm in size, does not offer the
family history of colorectal cancer, which opportunity of removing pathology identified
affected his father and brother. and is not suitable for surveillance because of
radiation exposure.
Which of the following are true? Faecal occult blood is not detected in 30-50% of
A. Hereditary non-polyposis colon cancer significant polyps and cancers.
(HNPCC) gives a lifetime risk of colon Colonoscopic perforation occurs in about 1 in
cancer of 1:6. 1000 examinations and post-polypectomy
B. Barium enema is the investigation of bleeding may occur up to 14 days later.
choice. Male patients should have upper and lower
C. Colonoscopic surveillance should be gastrointestinal investigations if anaemic,
offered irrespective of response to iron.
D. Three negative faecal occult blood tests
exclude a diagnosis of colon cancer. [ Q: 106 ] MasterClass Part2
E. Colonoscopy causes perforation in 1:10
(2010) - Gastroenterology
000 cases. A 55-year-old diabetic man 8-days post liver
F. Cancers in HNPCC predominantly affect transplantation has persistent hyperkalaemia
the left colon. requiring treatment. His medications include
tacrolimus and insulin. His creatinine is normal,
G. A carcinoembryonic antigen (CEA) level
his pH 7.40, but his bicarbonate 12 and base
should be measured.
deficit -10. Urine pH was 5 and serum lactate
H. He should have colonoscopy if his iron 0.5. Haptoglobins were normal. Direct Coombs
deficiency recurs after 4 months iron test negative. Liver function tests: Bilirubin 28,
supplements. ALP 350, ALT 58.
I. Endoscopic polypectomy may be followed
The MOST likely cause of his hyperkalaemia is:
by bleeding for up to 48 hours.
A. Post transplant haemolysis
J. His lifetime risk of colon cancer is about
1:6. B. Acute rejection
C. Renal tubular acidosis
Answer & Comments D. Enoxaparin use
Correct answer: CJ E. Renal failure

There is a lifetime risk of 1 in 6 to an individual Answer & Comments


with two first-degree relatives with colon
cancer. Colonoscopic surveillance is offered to Correct answer: C

Dr. Khalid Yusuf El-Zohry – Sohag Teaching Hospital (01118391123) Page | 225
El-Zohry MRCP Questions Bank (Part 2) – Medical Masterclass 2010

Hyperkalaemia can be a problem post associated with NSAID use and unlike classical
transplant. There are many causes to consider. ulcerative colitis is rarely associated with toxic
In this case the notable features are a megacolon. Although cholesytramine and
hyperkalaemia associated with a metabolic sulphasalazine have been used effectively as
acidosis but without evidence of renal failure, treatment, budesonide is the only drug shown
lactic acidosis or haemolysis. The patient was to be effective in a controlled clinical trial.
notably diabetic. Although enoxaparin can Collagenous colitis overlaps with this condition
rarely be associated with hyperkalaemia, the in that the clinical symptoms are similar but
presence of risk factors, a metabolic acidosis, microscopically there is a layer of collagen
and an inappropriately low urine pH is more in beneath the submucosa rather than
keeping with a distal renal tubular acidosis. lymphocytic infiltration.
Tacrolimus has been well described to cause
this. [ Q: 108 ] MasterClass Part2
(2010) - Gastroenterology
[ Q: 107 ] MasterClass Part2
On medical take you admit a 90-year-old
(2010) - Gastroenterology
woman with falls. No precipitating cause is
A 70-year-old lady presents with watery found but she is noted to be constipated on PR
diarrhoea. Flexible sigmoidoscopy is reported as examination, and this is felt to be contributory
normal but histology shows a lymphocytic to her admission. She is on multiple
infiltration. medications.

Which of the following best describes optimum Which TWO medications are most likely to be
management and complications? contributing to her constipation?
A. The patient should be warned about an A. Paracetamol
increased risk of colorectal carcinoma.
B. Senna
B. Non-steroidal anti-inflammatory drugs
C. Codeine phosphate
(NSAIDs) are unlikely to have caused her
symptoms and can safely be continued. D. Isosorbide mononitrate

C. Toxic megacolon is a common E. Aspirin


complication. F. Nifedipine
D. Budesonide is effective treatment. G. Atenolol
E. Collagenous colitis has a similar H. Topical eyedrops
microscopic appearance.
I. Ranitidine

Answer & Comments J. Quinine sulphate

Correct answer: D
Answer & Comments

All patients who have diarrhoea and undergo Correct answer: CF


flexible sigmoidoscopy or colonoscopy should
have colon biopsies taken to exclude Falls are a common reason for admission in the
microscopic colitis, which is characterised by a elderly. Many causes for falls can be found, and
normal appearance to the mucosa but a two often-overlooked gastroenterological
lymphocytic infiltration histologically. It is causes include occult GI bleeding and

Dr. Khalid Yusuf El-Zohry – Sohag Teaching Hospital (01118391123) Page | 226
El-Zohry MRCP Questions Bank (Part 2) – Medical Masterclass 2010

constipation. Hence PR examination is an D. It is an adaptive response to chronic


essential part of the clinical examination. reflux and is resistant to ulceration.
Constipation can be considered primary (e.g.
E. It may be precipitated by chronic non-
slow transit syndromes, irritable bowel
steroidal anti-inflammatory (NSAID)
syndrome) or secondary. Secondary causes
ingestion.
include metabolic, gastrointestinal, neurological
and psychiatric.
Answer & Comments
As with many things in the elderly the
complications of polypharmacy are common. Correct answer: B
With regards to the association between drugs
and constipation different classes of medication Barrett’s oesophagus describes a replacement
are more commonly associated. Thus analgesics of the normal oesophageal squamous
(opioid and NSAID) cause constipation as do epithelium by a metaplastic columnar
many anti-depressants. Cardiac medications, in epithelium as a response to chronic reflux. It is
particular ACE-inhibitors and calcium channel relatively common, with a prevalence of around
blockers, are known to cause constipation. 1% in autopsy studies and around 10% in
patients undergoing gastroscopy for reflux
Other well recognised associations exist for
symptoms. The intestinal type of metaplasia
bisphosphonates and proton pump inhibitors.
has premalignant potential and
In this case the two most likely drugs
adenocarcinoma develops in approximately 1%
contributing to constipation are codeine
per year. Proton pump inhibitors may relieve
phosphate and nifedipine. Paracetamol and
symptoms but do not effect cure.
aspirin are not usually a cause of constipation,
senna is a laxative and the others are not noted
for their association with constipation. [ Q: 110 ] MasterClass Part2
(2010) - Gastroenterology
[ Q: 109 ] MasterClass Part2 A 67-year-old man presents with anaemia Hb
(2010) - Gastroenterology 8.3g/dl (12.5-16.5) and ferritin 4
micrograms/litre (12-200). He denies any upper
A 45-year-old man with a long history of reflux
or lower gastrointestinal symptoms. His urine
symptoms is referred for open access
dipstick is negative. He takes low-dose aspirin
endoscopy. The endoscopist notes a red velvety
as he has read in the newspapers that it is good
lining in the distal 10cm of the oesophagus.
for the circulation.
Biopsies from this area are reported as showing
columnar epithelium with intestinal metaplasia.
Which are the best TWO tests in terms of
further investigation?
Which one of the following statements is true?
A. Faecal occult bloods x 3
A. This is a rare condition occurring in
approximately 0.1% of patients B. Colonoscopy
endoscoped for reflux symptoms.
C. Gastroscopy
B. There is approximately a 1% risk per
D. Small bowel enteroscopy
annum of the patient developing
oesophageal carcinoma. E. Small bowel meal

C. Proton pump inhibitors may cure the F. Gastroscopy and duodenal biopsies
condition over time. G. Intrinsic factor antibodies

Dr. Khalid Yusuf El-Zohry – Sohag Teaching Hospital (01118391123) Page | 227
El-Zohry MRCP Questions Bank (Part 2) – Medical Masterclass 2010

H. Bone marrow examination The major side effects of alpha interferon are
neuropsychiatric effects such as depression.
I. Abdominal CT scan
Untreated depression is likely to worsen during
J. Labelled red cell scan. treatment and should be treated before
commencing therapy for hepatitis C.
Answer & Comments Hepatitis B core antibody indicates previous
Correct answer: BF exposure to hepatitis B infection and previous
intravenous drug users (commonest cause of
HCV acquisition in the UK) are likely to have
All men and post menopausal women with iron
been exposed to hepatitis B in the past.
deficiency anaemia should have both their
upper and lower gastrointestinal (GI) tract Cirrhosis is not a contra-indication to therapy
examined. Upper GI tract pathology is seen in providing liver function is preserved (e.g. no
about 50% and lower GI tract pathology in ascites/encephalopathy) and although patients
around 20%. No cause is found in about 25%. with hepatitis C genotype 1 respond less well to
Up to 10% may harbour colorectal neoplasia. treatment than genotype 2 and 3 this is not a
Important dual pathology is seen in about 5% contra-indication to treatment.
hence the need for bi-directional endoscopy.
Factors that may predispose to a higher risk of
Coeliac disease is seen in 2-5% so duodenal
adverse events in addition to major psychiatric
biopsies should always be taken. Small bowel
disorders include:
meal (and enteroscopy) may be required if iron
deficiency anaemia is recurrent despite iron  cardiovascular disease (significant
replacement therapy though its yield is low. arrhythmias
 congestive heart failure
[ Q: 111 ] MasterClass Part2  uncontrolled hypertension or ischaemic
(2010) - Gastroenterology heart disease)
A 38-year-man is referred to clinic having been  active autoimmune diseases
found to be hepatitis C antibody positive and  poorly controlled epilepsy
hepatitis C RNA positive.
 diabetic retinopathy (interferon can
Which of the following is a contraindication to exacerbate diabetic retinopathy)
treating him with combination therapy with  thyroid disease (relative
interferon alpha and ribavirin?
contraindication, as interferon can
A. Previous intravenous drug misuse cause an autoimmune thyroiditis).
B. The presence of Hepatitis B core antibody Other factors increasing the risk of adverse
events include myelosuppression (e.g.
C. Cirrhosis
thrombocytopaenia and neutropaenia).
D. Untreated depression Therapy should not be instituted if the platelet
E. Hepatitis C genotype 1. count is less than 80x109/l or the neutrophil
count is less than 1.0x109/l. Renal failure and
anaemia increase the risk of adverse effects
Answer & Comments
from ribavirin. Finally, ribavirin is teratogenic,
Correct answer: D hence patients (and their partners) must use
adequate contraception.

Dr. Khalid Yusuf El-Zohry – Sohag Teaching Hospital (01118391123) Page | 228
El-Zohry MRCP Questions Bank (Part 2) – Medical Masterclass 2010

Absolute contra-indications to therapy with longer felt to be a good screening tool although
interferon and ribavirin are decompensated it is still a useful diagnostic test. Transplantation
liver disease, active alcohol abuse, pregnancy or is curative for single tumours less than 5cm.
lack of appropriate contraception and expected Small tumours can be ablated with ethanol or
non-compliance. radiofrequency ablation but the risk of new
tumours remains in the cirrhotic liver.
[ Q: 112 ] MasterClass Part2 Chemoembolisation is effective in increasing
life expectancy but is not curative. Interval
(2010) - Gastroenterology
tumours do occur and ultrasound will
A 56-year-old man with hepatitis B cirrhosis is underestimate size and tumour multicentricity
offered 6-monthly ultrasound screening to but the radiation risk of CT and cost precludes
detect hepatocellular carcinoma (HCC). this as a screening tool.

What TWO pieces of advice are correct?


[ Q: 113 ] MasterClass Part2
A. The risk of hepatocellular carcinoma is 1- (2010) - Gastroenterology
3%/year.
A 56-year-old man is admitted with massive
B. Chemoembolisation of the tumour is
painless haematemesis. He gives a twenty-year
curative.
history of alcohol abuse. He is jaundiced with
C. Liver transplantation can cure single multiple spider naevi, hypotensive, confused
tumours less than 10cm. and has demonstrable shifting dullness.
D. Small tumours <3cm can be ablated with
Which of the following are true?
ethanol.
A. A subclavian line should be inserted as
E. An elevated AFP is diagnostic of HCC.
soon as possible
F. The risk of HCC is less in men than B. The patient should be commenced
women.
immediately on chlordiazepoxide or
G. Tumours occur less often in hepatitis B chlormethiazole to cover alcohol
cirrhosis than in alcoholic cirrhosis. withdrawal.
H. Most tumours are hypovascular. C. Thrombocytopaenia is highly suggestive
of portal hypertension.
I. Six-monthly CT scans are an alternative
screening tool. D. Urgent therapeutic paracentesis should
be considered.
J. Screening has been shown to be effective
in controlled clinical trials. E. Bacterial sensitivities on blood and ascitic
cultures should be awaited before
Answer & Comments commencing antibiotics.
F. There is no role for medical therapies.
Correct answer: AD
G. Transjugular intrahepatic portosystemic
There are no controlled clinical trials of stent shunt (TIPSS) should only be
screening for hepatocellular carcinoma. considered if the patient is a transplant
However most clinicians screen cirrhotic candidate.
patients. Those with viral hepatitis and H. Statistically, in this patient, peptic ulcer
haemochromatosis are most at risk. AFP levels disease is more likely than varices
also rise with viral replication and so it is no

Dr. Khalid Yusuf El-Zohry – Sohag Teaching Hospital (01118391123) Page | 229
El-Zohry MRCP Questions Bank (Part 2) – Medical Masterclass 2010

I. If oesophageal varices are present, intervention for bleeding oesophageal varices,


injection sclerotherapy is the intervention although sclerotherapy is an accepted
of choice. alternative where local experience of banding
does not exist. Beta-blockade has been shown
J. Unless contraindicated, the patient should
to be effective in secondary prophylaxis of
be treated with a beta-blocker after they
variceal haemorrhage.
have recovered from this episode.

Answer & Comments [ Q: 114 ] MasterClass Part2


(2010) - Gastroenterology
Correct answer: CJ
A 65-year-old man, retired publican, known to
Fluid resuscitation can be carried out via the diabetic team for long-standing diabetes
multiple large bore peripheral lines. and the cardiologists for long-standing
paroxysmal AF, was admitted with
Subclavian line placement can be hazardous in haematemesis. An endoscopy revealed a gastric
patients with coagulopathy, and if central varix whilst abdominal imaging showed a
venous monitoring is required, the internal cirrhotic looking liver with a suspicious lesion in
jugular vein is the location of choice. the right lobe; splenomegaly and ascites were
Not all patients will experience alcohol also demonstrated. His ferritin was >1000
withdrawal, and benzodiazepines should be ng/ml and transferrin saturation 65%. Alpha-
used with great caution in patients who are fetoprotein was 990 ng/ml.
encephalopathic.
What is the working diagnosis?
Sequestration of platelets in a spleen enlarged
as a result of portal hypertension is the usual A. Cryptogenic cirrhosis secondary to non-
cause of thrombocytopaenia in this scenario. alcoholic steatohepatitis

Therapeutic paracentesis should only be B. Haemochromatosis with hepatocellular


considered if ascites is tense - however, carcinoma
diagnostic paracentesis should be carried out, C. Alcoholic liver disease
and antibiotics prescribed on the basis of an
D. Alpha-1-antitrypsin deficiency
elevated ascitic WCC, not awaiting cultures (in
this scenario, prophylactic antibiotics should be E. Hepatitis C cirrhosis with hepatocellular
prescribed in view of the GI bleed). carcinoma

A number of studies have shown that


somatostatin analogues are as effective in Answer & Comments
controlling acute variceal haemorrhage as Correct answer: B
interventional endoscopy, and drugs should be
commenced while waiting for endoscopy.
This man has haemachromatosis. He is cirrhotic
While TIPSS was initially considered 'a bridge to with a ferritin >1000 ng/mL and a transferrin
transplantation', it may be used in patients with saturation >65%. This indicates he has iron
uncontrollable portal hypertensive overload and haemochromatosis is the
haemorrhage, though deterioration in abnormal accumulation of iron in parenchymal
encephalopathy may result. organs, leading to organ toxicity. Genetic
In patients with cirrhosis, at least 60% of GI haemachromatosis is the most common
haemorrhage will be due to varices. inherited liver disease in Caucasians and the
Oesophageal banding is now the accepted most common autosomal recessive genetic

Dr. Khalid Yusuf El-Zohry – Sohag Teaching Hospital (01118391123) Page | 230
El-Zohry MRCP Questions Bank (Part 2) – Medical Masterclass 2010

disorder. The organs involved are the liver, A. Ultrasound scan of the abdomen is
heart, pancreas, pituitary, joints, and skin, with indicated.
the clinical features of haemochromatosis
B. If ascites is drained, it should be done in
including cirrhosis of the liver, diabetes,
small volumes (0.5-1L) at intervals over
hypermelanotic pigmentation of the skin, and
several days.
heart failure.
C. Spontaneous bacterial peritonitis affects
Primary hepatocellular carcinoma, complicating
up to 5% of patients with ascites.
cirrhosis, is responsible for about one-third of
deaths in affected homozygotes. Since D. Percutaneous liver biopsy should be done
haemochromatosis is a relatively easily treated under ultrasound guidance in the
disorder if diagnosed, this is a form of presence of tense ascites.
preventable cancer. Despite advances in the E. Hepatorenal syndrome is associated with
diagnosis of haemachromatosis (principally a salt-losing nephropathy.
genetic screening for the C282Y mutation
responsible for most, but not all, forms of F. Liver biopsy is contraindicated until the
disease), treatment remains simple, jaundice has improved and clotting is
inexpensive, and safe. normal.

Patients have weekly therapeutic phlebotomy G. Percutaneous liver biopsy has a mortality
rate of 1%.
of 500 mL of whole blood (equivalent to
approximately 200-250 mg of iron). Some H. Refractory ascites is an indication for liver
patients can tolerate twice-weekly phlebotomy. transplantation.
Therapeutic phlebotomy is performed until I. If the patient has a flap, a low protein diet
iron-limited erythropoiesis develops (taken as should be instituted.
failure of the haemoglobin level to recover
J. Nasogastric tube feeding is
before the next phlebotomy; aiming for a
contraindicated in the presence of
transferrin saturation less than 50% and serum
oesophageal varices.
ferritin levels less than 50 g/mL, preferably 20
ng/mL). Most patients then require
maintenance phlebotomy - 1 unit of blood Answer & Comments
every 2 - 3 months. Therapeutic phlebotomy Correct answer: AH
may improve or even cure some of the
manifestations and complications of the
Large volume paracentesis appears safe and
disease, such as fatigue, elevated liver enzymes,
effective when volume expansion is
hepatomegaly, abdominal pain, arthralgia, and
simultaneously given with 5-10g salt-poor
hyperpigmentation. Other complications
albumin per litre drained. Spontaneous
usually show little or no change after
bacterial peritonitis occurs in about 20% of
phlebotomy
patients with ascites. Transjugular biopsy may
be indicated in acute liver disease in which a
[ Q: 115 ] MasterClass Part2 coagulopathy precludes percutaneous biopsy.
(2010) - Gastroenterology Ascites is likely to increase the risk of
percutaneous biopsy, and prior drainage or a
A 35-year-old man is admitted with jaundice transjugular approach are alternatives. Liver
and ascites. biopsy has a mortality rate of 0.01%.
Malnutrition is common in chronic liver disease
Which of the following are true?
and there is no evidence that protein restriction

Dr. Khalid Yusuf El-Zohry – Sohag Teaching Hospital (01118391123) Page | 231
El-Zohry MRCP Questions Bank (Part 2) – Medical Masterclass 2010

is of benefit in encephalopathic patients. [ Q: 117 ] MasterClass Part2


Provided they have not recently bled, (2010) - Gastroenterology
nasogastric feeding does appear safe in patients
with varices. An 87-year-old lady is admitted with left iliac
fossa pain, tenderness and fever.
[ Q: 116 ] MasterClass Part2 Which of the following are true?
(2010) - Gastroenterology
A. Colonoscopy is likely to be helpful.
A 38-year-old woman weighs 103 kg and is
B. Ultrasound or computerised tomography
1.65m tall (BMI=37.8).
(CT) is indicated.
Which of the following statements is true? C. Diverticulosis affects 10% of the
population over 50 years of age.
A. She should aim to consume a diet that
produces a daily negative energy balance D. Bleeding is an unusual complication of
of about 500kcal. diverticular disease.
B. She should be encouraged to continue to E. Colonoscopy is unhelpful in patients
smoke in order to supress her appetite. admitted with bright red rectal bleeding.
C. Orlistat (pancreatic lipase inhibitor) is the F. Diverticulosis is associated with an
easiest way for her to lose weight. increased risk of colorectal neoplasia.
D. There is no restriction on alcohol G. Recurrent urinary infections raise the
consumption so long as it is in the form of possibility of a complication of
wines or spirits. diverticular disease.
E. Her life expectancy is normal. H. A diverticular stricture is readily
distinguished from a malignant stricture.
Answer & Comments I. Following a diverticular abscess, the
Correct answer: A affected diverticulae fibrose and further
problems are uncommon.

Up to 20% of the UK population are clinically J. Uncomplicated extensive diverticulosis


obese (i.e. BMI>30). This has serious may explain weight loss.
consequences both on mortality and morbidity.
Management is based around a negative energy Answer & Comments
balance as a consequence of long-term low-
Correct answer: BG
intensity exercise and diet (500–600 kcal/day
defecit).
Ultrasound and CT are useful in detecting bowel
Crash and fad diets are unlikely to be sustained.
thickening, abscess formation and even fistulae.
All alcohol should be minimised as it contains a
significant number of calories though is devoid As diverticulitis is associated with multiple
of nutrition. Drug treatments should only be micro-perforations, colonoscopy is not advised.
considered where behaviour modification has Diverticular disease is the commonest cause of
failed, and are only likely to work alongside colonic bleeding which may be detected and
exercise and diet. endoscopically treated at colonoscopy.

Dr. Khalid Yusuf El-Zohry – Sohag Teaching Hospital (01118391123) Page | 232
El-Zohry MRCP Questions Bank (Part 2) – Medical Masterclass 2010

Recurrent urinary infections, pneumaturia or [ Q: 119 ] MasterClass Part2


faeces in the urine are features of a colo-vesical (2010) - Gastroenterology
fistula.
A 40-year-old woman presents to the
Diverticulosis affects 50% of the population
outpatient clinic with intermittent epigastric
over 50. Most patients have few if any
discomfort. FBC, urea and electrolytes are
symptoms, but if complications occur (abscess,
normal. Liver function tests reveal alkaline
stricture or bleeding), recurrence is common
phosphatase (ALP) 600 U/L, γ-
and surgery should be considered.
glutamyltransferase (GGT) 768 U/L, bilirubin 13
μmol/L, alanine transaminase (ALT) 26 U/L and
[ Q: 118 ] MasterClass Part2 albumin 39 g/L. Transabdominal ultrasound
(2010) - Gastroenterology shows normal liver architecture, but there are
several small stones in the gallbladder and the
A 42-year-old man presents with abdominal
common bile duct measures 10 mm in
discomfort, bloating and a change in bowel diameter. The pancreas and the main
habit over the last 8 months. He may pass one pancreatic duct are normal.
formed stool or up to six loose stools per day.
The discomfort tends to occur when he has the What would be the most appropriate next step?
diarrhoea and is eased following defaecation.
A. Endoscopic retrograde
Which of the following statements is true? cholangiopancreatography and bile duct
trawl
A. He fulfills the Rome II criteria for the
diagnosis of irritable bowel syndrome B. CT scan
(IBS). C. CA19-9
B. No further investigation is needed. D. Magnetic resonance
C. Coeliac disease is unlikely in the absence cholangiopancreatography (MRCP)
of weight loss. E. Refer for laparoscopic cholecystectomy
D. His father had colon cancer aged 40, and bile duct exploration
giving the patient a lifetime risk of 1:40.
Answer & Comments
E. He is very likely to improve with a high
fibre diet. Correct answer: D

Answer & Comments As this woman is not acutely ill, there is time to
perform further investigations of the biliary
Correct answer: A
tree without moving directly to invasive
procedures. MRCP has a higher sensitivity than
Investigation should be tailored to the CT for detecting distal bile duct stones, which
individual. Genetic inheritance has a role in are the most likely cause of this woman's
coeliac disease, colon cancer (in which the presentation. CA19-9 is useless in this situation.
lifetime risk is 1:10 if a first degree family If her MRCP is negative, then laparoscopic
member presents before the age of 45) and
removal of the gallbladder would be
inflammatory bowel disease, all of which can
appropriate.
occasionally present with similar symptoms.
About half of IBS patients worsen with a high
fibre diet.

Dr. Khalid Yusuf El-Zohry – Sohag Teaching Hospital (01118391123) Page | 233
El-Zohry MRCP Questions Bank (Part 2) – Medical Masterclass 2010

[ Q: 120 ] MasterClass Part2 disease, and those with advanced dementia


(2010) - Gastroenterology have a particularly poor prognosis. The re-
feeding syndrome describes a range of
A 72-year-old man has a percutaneous metabolic disturbances (commonly affecting
endoscopic gastrostomy (PEG) inserted for phosphate, potassium and magnesium) which
swallowing difficulties due to his advanced occur during the re-feeding of a starved patient.
Parkinson’s disease. He becomes confused and Clinical manifestations are many and varied and
has a tachycardia 48 hours after commencing include neurological, cardiac, respiratory,
his PEG feed. haematological and immunological features.
Wernicke’s encephalopathy may occur without
Which of the following statements are true? all the classical features of confusion, ataxia and
A. Aspiration pneumonia is a possible cause. ophthalmoplegia.

B. Serum phosphate should be measured.


[ Q: 121 ] MasterClass Part2
C. The absence of cranial nerve signs
excludes Wernicke’s encephalopathy.
(2010) - Gastroenterology

D. Air under the diaphragm on an erect A 30-year-old woman with long-standing


chest radiograph is an indication for medically-treated Crohn’s disease presents to
laparotomy. clinic with increasing abdominal pain associated
with intermittent vomiting and bloating. Her
E. Anticoagulation is contraindicated.
inflammatory markers are normal.
F. PEG feeding avoids the risks of aspiration
in patients with swallowing difficulties. Which is the most likely cause of her symptoms?

G. 30-day mortality rate after PEG insertion A. Exacerbation of colonic colitis


is less than 5%. B. Stricture formation
H. PEG insertion is contraindicated in C. Colo-vesical fistula
Crohn’s disease.
D. Adhesions
I. Antibiotic prophylaxis prior to PEG
E. Colorectal cancer
insertion is of no proven benefit.
J. PEG feeding improves outcome in patients
Answer & Comments
with advanced dementia.
Correct answer: B
Answer & Comments
The most common symptoms of Crohn's
Correct answer: AB
disease are abdominal pain, often in the lower
right quadrant, and diarrhoea. Rectal bleeding,
PEG site infection and aspiration pneumonia weight loss and fever may also occur. Bleeding
are common complications. Aspiration may may be serious and persistent, leading to
occur during endoscopy but may occur later anaemia. Children with Crohn's disease may
due to aspiration of feed in a recumbent suffer delayed development and stunted
patient. Air under the diaphragm may occur growth. Long standing inflammation can result
even after an uneventful PEG placement. in stricture formation which may present as
Reported 30-day mortality rates after PEG intermittent bowel obstruction. Markers of
insertion vary between 10-30%. It is likely that inflammation such as CRP or ESR are not
many die as a consequence of their underlying

Dr. Khalid Yusuf El-Zohry – Sohag Teaching Hospital (01118391123) Page | 234
El-Zohry MRCP Questions Bank (Part 2) – Medical Masterclass 2010

necessarily significantly raised if the obstruction end-stage liver disease providing liver
is due to fibrosis. biochemistry is monitored. Although
chlordiazepoxide may accumulate causing
Management of Crohn’s disease involves a
encephalopathy, alcoholic withdrawal must be
combination of nutritional, medical and surgical
treated/prevented in a cirrhotic who is actively
treatment. Stricture formation, in particular, is
drinking on admission. Doses must be titrated
usually treated surgically, although endoscopic
on a day-to-day basis.
treatments are possible. Patients with small
bowel strictures are also advised to eat a low
fibre diet, in order to reduce the risk of food [ Q: 123 ] MasterClass Part2
bolus obstruction. (2010) - Gastroenterology
A 46-year-old man presents with jaundice.
[ Q: 122 ] MasterClass Part2 Dipstick testing of his urine reveals the
(2010) - Gastroenterology presence of bilirubin but no urobilinogen.
A 50-year-old man with alcoholic cirrhosis is
This means that:
admitted with diuretic resistant ascites. Initial
investigations show Na 127 mmol/l, creatinine A. jaundice is likely to be pre-hepatic
90micromol/l, bilirubin 110 micromol/l, B. bile must be flowing freely into the gut
prothrombin time 18 seconds, albumin 27g/l.
C. jaundice cannot be obstructive
Which one of the following drugs should be D. renal function is normal
avoided?
E. there must be complete obstruction to
A. Flucloxacillin bile flow.
B. Non-steroidal anti-inflammatory drugs
(NSAIDs) Answer & Comments
C. Paracetamol Correct answer: E
D. Rifampicin
When haemoglobin is broken down the
E. Chlordiazeopoxide.
porphyrin ring is converted into biliverdin and
thence to bilirubin.
Answer & Comments
Unconjugated bilirubin is relatively insoluble
Correct answer: B and is transported in the blood as a complex
with albumin. It is not excreted in the urine.
Once diuretic resistant ascites has occurred,
Hepatocytes take up unconjugated bilirubin and
blood flow to the kidney is reduced and it is
conjugate it to form the soluble diglucuronide,
easy to precipitate hepatorenal failure. NSAIDs
which is excreted into the bile.
do this by further reducing renal blood flow and
should be avoided. Further metabolism by gut bacteria forms the
soluble colourless compound urobilinogen.
Paracetamol is safe in a dose not exceeding 4
Some of this enters the blood stream and is
grams/day.
excreted in the urine. Urobilinogen remaining in
Rifampicin can cause cholestasis when used the gut is converted to the brown pigment,
alone. However it is a useful drug for treating urobilin, and is excreted.
pruritus in biliary diseases when cholestyramine
does not work and can be used in patients with

Dr. Khalid Yusuf El-Zohry – Sohag Teaching Hospital (01118391123) Page | 235
El-Zohry MRCP Questions Bank (Part 2) – Medical Masterclass 2010

If a patient with jaundice has no urobilinogen in salts, bisphosphonates, tetracyclines), radiation


their urine, then they must have complete (e.g. for bronchial carcinoma) and candidiasis.
obstruction to bile flow.
[ Q: 125 ] MasterClass Part2
[ Q: 124 ] MasterClass Part2 (2010) - Gastroenterology
(2010) - Gastroenterology
You are asked to review a 45-year-old man
A 64-year-old woman presents with a six-month recovering from a road traffic accident, just off
history of progressive dysphagia for solids. She intensive care. Although able to eat and drink
has a long-standing history of rheumatoid small amounts he remains generally weak and
arthritis and retrosternal burning pain, regularly has lost 20% of his pre-morbid weight. His
using over-the-counter preparations for bowels are working normally. The orthopaedic
indigestion. team ask your advice regarding his nutrition.

The TWO most likely diagnoses are: Do you suggest:


A. Pharyngeal pouch A. Regular monitoring of his weights only
taking action if he fails to put weight on
B. Duodenal ulceration
B. Insertion of a tunnelled central venous
C. Acute gastritis
line for parenteral nutrition
D. Scleroderma
C. Booking him for percutaneous
E. Benign oesophageal stricture gastrostomy (PEG) at the earliest
F. Gastric carcinoma available opportunity

G. Motor neurone disease D. Oral dietary supplements

H. Aneurysm of the thoracic aorta E. Insertion of a naso-gastric tube and


commencement of supplemental enteral
I. Pseudobulbar palsy nutrition as soon as possible
J. Oesophageal carcinoma.
Answer & Comments
Answer & Comments
Correct answer: E
Correct answer: EJ
Malnutrition is common in hospital and impairs
The history of dysphagia for solids suggests a recovery from illness. This gentleman is
mechanical (rather than neurological) cause, malnourished and recovering from major
and the relatively long history indicates that a trauma and some form of nutrition is necessary.
benign diagnosis is most likely. However, He has evidence that his GI tract is working and
carcinoma of the oesophagus cannot be therefore short-term enteral nutrition is
excluded and barium swallow followed by appropriate and whilst supplements may be
endoscopy is required to rule out malignancy. sufficient it is more likely that intensive
nutrition via a naso-gastric tube is going to be
Gastro-oesophageal reflux disease is the
needed to meet his increased nutritional
likeliest cause of benign stricture in this case,
demands. It is inappropriate to simply monitor
but other benign causes include drugs (non-
him and, in view of a functioning GI tract, not
steroidal anti-inflammatory agents, potassium
appropriate to choose parenteral nutrition. The

Dr. Khalid Yusuf El-Zohry – Sohag Teaching Hospital (01118391123) Page | 236
El-Zohry MRCP Questions Bank (Part 2) – Medical Masterclass 2010

nutrition is needed short-term and therefore a [ Q: 127 ] MasterClass Part2


PEG is not required. (2010) - Gastroenterology
Vomiting of ‘coffee-grounds’ in an 18-year-old
[ Q: 126 ] MasterClass Part2
man who overindulges in drinking alcohol
(2010) - Gastroenterology raises.
An 18-year-old man admits to having taken 12 g
Which TWO of the following diagnostic and
of paracetamol 1 day prior to presentation in
therapeutic options?
the Emergency Department with abdominal
pain. A. This may represent a Mallory-Weiss tear
following bouts of retching, therefore
Which of the following factors is most observe and discharge home if there is no
important in determining treatment? haemodynamic compromise.
A. His paracetamol level B. The patient should be warned not to
B. His serum bilirubin drink coffee as a means of 'sobering up'.

C. His serum alanine transaminase C. Emergency endoscopy as soon as possible


is preferable to establish a diagnosis and
D. His history initiate appropriate therapy.
E. His serum creatinine D. Peptic ulceration is unlikely as a cause of
upper GI haemorrhage.
Answer & Comments
E. If there is no melaena, upper GI
Correct answer: D haemorrhage is unlikely.
F. Variceal haemorrhage due to liver
The history is most important in determining cirrhosis is likely, and as this may cause
the risk of hepatotoxicity: a dose of catastrophic bleeding, the patient should
paracetamol of as little as 75 mg/kg in someone be closely monitored, treated with
at risk can be fatal. Risk factors include: octreotide or vasopressin, and central
 regular ethanol consumption in excess venous access established immediately.
of 21 units/week in males and 14 G. Alcohol-induced gastritis is unlikely, as it
units/week in females; does not cause haematemesis.
 regular use of enzyme-inducing drugs H. On demographic grounds alone, this
(carbamazepine, phenytoin, patient is at high risk of rebleeding and an
phenobarbital, rifampacin); adverse outcome.
 conditions causing glutathione I. It would be prudent to transfuse the
depletion (malnutrition, HIV, eating patient even if the haemoglobin level is
disorders, cystic fibrosis). normal, as it takes a while for
In all scenarios you are strongly advised to treat haemodilution to occur.
with N-acetylcysteine if in doubt. J. If the patient is actively bleeding, consider
inserting a Sengstaken-Blakemore tube.

Answer & Comments

Correct answer: AC

Dr. Khalid Yusuf El-Zohry – Sohag Teaching Hospital (01118391123) Page | 237
El-Zohry MRCP Questions Bank (Part 2) – Medical Masterclass 2010

Vomiting of ‘coffee-grounds’ usually signifies she visited family. Two weeks later she presents
upper GI haemorrhage, as stomach acid acting jaundiced and confused to casualty with an INR
on blood creates a flaky brown residue of 2, alanine transaminase (ALT) of 2,000 U/L
reminiscent of the residue left after coffee is and bilirubin of 200 mmol/L.
brewed. In a young person with a history
particularly of recent alcohol binge-drinking, Which viral infection is most likely to have
gastritis or a Mallory-Weiss tear are likely caused this illness?
diagnoses, and it is less likely that the patient A. Hepatitis A
would have developed cirrhosis and
B. Hepatitis B
oesophageal varices. Thus, measures such as to
reduce mesenteric blood flow, or balloon C. Hepatitis C
tamponade, would not be considered without
D. Hepatitis D
further supportive evidence. In any case
oesophageal variceal haemorrhage usually E. Hepatitis E
comprises vomiting of large volume, unaltered
blood. Answer & Comments
Nonetheless, in most series of upper GI Correct answer: E
haemorrhage, peptic ulceration remains the
single most common cause of bleeding, and
Infective hepatitis can be caused by a variety of
should always be considered. Early endoscopy
different viruses such as hepatitis A, B, C, D and
followed by appropriate therapy allows rapid
E. The incubation period following exposure to
diagnosis and helps to separate those
hepatitis E ranges from 3 to 8 weeks, with a
individuals requiring further treatment from the
mean of 40 days. Hepatitis E is usually a self-
rest, allowing more efficient use of hospital
limiting infection followed by complete
resources. However, where endoscopy is not
recovery; prolonged viraemia or faecal
immediately available, or the service is
shedding is unusual and there is no chronic
oversubscribed, it is reasonable to treat
infection. A fulminant form of hepatitis E can
patients expectantly on the basis of sound
develop, most frequently in pregnancy and with
clinical assessment of the risks and likely
a mortality rate of 20% in the third trimester.
diagnoses.
The condition should therefore be suspected in
The mortality and morbidity associated with outbreaks of water-borne hepatitis occurring in
upper GI haemorrhage is strongly linked to developing countries, especially if the disease is
increased age, and the presence of significant more severe in pregnant women, or if hepatitis
co-morbidity, and young patients tend to do A has been excluded. A correct diagnosis can
better. only be made by testing for the presence of
specific viral antigens and/or antiviral
Transfusion is potentially risky, and should only
antibodies (with or without polymerase chain
be used where there is clear evidence of acute
reaction for active viral replication).
severe blood loss, manifest, for example, by
hypotension and shock.
[ Q: 129 ] MasterClass Part2
[ Q: 128 ] MasterClass Part2 (2010) - Gastroenterology
(2010) - Gastroenterology A 47-year-old woman is referred with iron
deficient anaemia. She has no gastrointestinal
A 25-year-old heavily pregnant woman returns
symptoms.
to the UK from a holiday in Bangladesh where

Dr. Khalid Yusuf El-Zohry – Sohag Teaching Hospital (01118391123) Page | 238
El-Zohry MRCP Questions Bank (Part 2) – Medical Masterclass 2010

Which of the following are true? Barium studies may be normal and as the
histological changes are patchy, four biopsies
A. Coeliac disease is very unlikely in the
are advised.
absence of symptoms.
Endomysial antibody levels usually fall within 6
B. Raised serum anti-gliadin antibody levels
weeks of dietary treatment and non-
are 95% sensitive for the diagnosis of
compliance is the commonest reason for an
coeliac disease.
absence of clinical or serological response.
C. The prevalence of coeliac disease in the
Osteoporosis is a common complicating factor
UK approaches 1 in 200.
and improves following institution of the diet.
D. Howell Jolly bodies in the blood film
Ulcerative jejunitis is associated with an
suggest haemolysis.
increased risk of enteropathy associated T cell
E. Antibody profiles take at least 1 year on a lymphoma.
strict gluten-free diet to become
negative.
[ Q: 130 ] MasterClass Part2
F. Osteoporosis is a recognized complication (2010) - Gastroenterology
of coeliac disease.
(1) A 35-year-old woman, otherwise fit and
G. Coeliac disease patients with ulcerative
well, presents with isolated transaminitis. She
jejunitis tend to be at lower risk of
denies excess alcohol use and ultrasound of her
malignancy.
liver reveals only fatty change. Her standard
H. Barium follow-through invariably shows liver screen is negative.
increased flocculation of barium and
coarser mucosal folds. Which statement best reflects this clinical
scenario?
I. A single biopsy is adequate for the
diagnosis of coeliac disease provided it is A. She most likely has Non A-E Hepatitis and
obtained distal to the duodenal bulb. should be screened for Hepatitis G.

J. Pancreatic insufficiency is the commonest B. Silent alcohol abuse should be assumed.


cause of failure to improve on a gluten- C. Non-alcoholic fatty liver disease is most
free diet. likely.
D. Wilson’s disease needs to be diagnosed
Answer & Comments
by liver biopsy.
Correct answer: CF E. She should be advised that she can’t be a
blood donor.
Anaemia is a common presentation of coeliac
disease. Answer & Comments
Raised anti-endomysial antibodies have a 95%
Correct answer: C
sensitivity for the diagnosis of coeliac disease,
which affects up to 1 in 200 of the UK
population. Non-alcoholic fatty liver disease (NAFLD) is a
spectrum of diseases that includes simple
Howell Jolly bodies are a feature of steatosis, steatohepatitis, advanced fibrosis and
hyposplenism which is associated with coeliac cirrhosis. The diagnosis should be suspected in
disease. patients with persistently elevated alanine
aminotransferase (ALT) values with:

Dr. Khalid Yusuf El-Zohry – Sohag Teaching Hospital (01118391123) Page | 239
El-Zohry MRCP Questions Bank (Part 2) – Medical Masterclass 2010

 negative screening for viral hepatitis Answer & Comments


 autoimmune hepatitis and metabolic Correct answer: B
liver disease
 no high risk alcohol or medication use In patients with cirrhosis and portal
 hypertension, gastrointestinal haemorrhage is a
fatty infiltration on ultrasound.
major complication and cause of death.
NAFLD is associated with the metabolic Oesophageal varices are present in
syndrome (obesity, insulin resistance, approximately 50% of patients with cirrhosis.
hyperlipidaemia and hypertension) and is Among patients with varices, the risk of
therefore often seen in obese patients, who gastrointestinal haemorrhage ranges from 30%
may have impaired glucose intolerance. to 50% and half of these patients die within 6
Treatment and prognosis is unclear, but weeks after bleeding.
attention to modifiable risk factors is important Normal care includes (in addition to
including weight loss, diabetic management and resuscitation) airway management, routine
treatment of hyperlipidaemia. antibiotics, early endoscopy along with
pharmacotherapy with either vasopressin or
[ Q: 131 ] MasterClass Part2 octreotide (or related compounds). Glypressin
(2010) - Gastroenterology is now the widely used drug. Vasopressin and
glypressin reduce portal pressure by splanchnic
A 45-year-old woman known to have alcoholic arteriolar vasoconstriction, whereas
liver disease presents in a hypotensive, somatostatin and octreotide lower portal
tachycardiac and confused state. She shows pressure by decreasing hepatic blood flow.
evidence of haematemesis and melaena.
Temporary tamponade can be achieved with a
Emergency endoscopy shows fresh blood in the
Sengstaken-Blackmore tube. It should be
oesophagus with bleeding varices.
considered as a salvage procedure. Tamponade
Sclerotherapy and banding are attempted but
is 90% successful at stopping haemorrhage.
the patient remains unstable.
Unfortunately 50% patients re-bleed within 24
Which of the following statements is NOT true? hours of removal of tamponade. Prednisolone
has a role, after careful consideration, in
A. Intubation should be considered if a safe patients with alcoholic hepatitis but not in the
airway cannot be maintained. acute management of variceal haemorrhage.
B. Oral prednisolone should be prescribed
for acute alcoholic hepatitis. [ Q: 132 ] MasterClass Part2
C. Intravenous antibiotics (e.g. Cefotaxime) (2010) - Gastroenterology
should routinely be prescribed to patients
A previously healthy patient of 24 years of age
with variceal haemorrhage.
presents to hospital complaining of 12 hours of
D. If further bleeding occurs a Sengstaken epigastric discomfort and vomiting. Blood tests
tube may be required. reveal an Alanine Transaminase of 8,400
E. Glypressin (a synthetic analogue of (normal range <45), Bilirubin 24 (normal <15)
vasopressin) should be prescribed to and INR 2.8.
reduce splanchnic and portal blood flow.
What is the most likely diagnosis?
A. Gilbert's syndrome
B. Acute Hepatitis A
Dr. Khalid Yusuf El-Zohry – Sohag Teaching Hospital (01118391123) Page | 240
El-Zohry MRCP Questions Bank (Part 2) – Medical Masterclass 2010

C. Epstein Barr Virus (EBV) Hepatitis E. Positive culture (>10 6 organisms per ml)
of a small bowel aspirate is diagnostic of
D. Alcoholic Hepatitis
the syndrome of bacterial overgrowth.
E. Undisclosed paracetamol overdose.
Answer & Comments
Answer & Comments
Correct answer: D
Correct answer: E
Although it is not understood why, a surprising
The marked elevation of the INR in the context 10% of patients develop diarrhoea after a
of the raised ALT and relatively normal bilirubin cholecystectomy.
indicated that this patient is likely to be
Nocturnal diarrhoea suggests an organic cause.
developing fulminant hepatic failure.
Idiosyncratic drug reactions (including It is rare to identify an infective cause for
recreational drugs such as 'ecstasy') and diarrhoea when the symptoms have lasted for
paracetamol overdose are the commonest more than 4 weeks.
cause for this presentation.
Antiendomysial antibodies (to tissue
Fulminant hepatic failure rarely occurs with transglutaminase) are the single most reliable
Hepatitis A and EBV hepatitis, although the antibody test for coeliac disease, but there are
short history and the relatively normal bilirubin false negatives.
make this very unlikely in this case.
Although a positive culture of a small bowel
Alcoholic hepatitis may produce an elevated aspirate is almost diagnostic, this can occur in
INR, usually as a result of underlying chronic healthy individuals.
liver disease or cholestasis, but the ALT is rarely
more than 200. [ Q: 134 ] MasterClass Part2
Gilbert's syndrome causes isolated elevation of (2010) - Gastroenterology
bilirubin and is usually asymptomatic.
A 60-year-old man with known ulcerative colitis
and diverticular disease comes to clinic
[ Q: 133 ] MasterClass Part2 complaining of passing faeces per urethra.
(2010) - Gastroenterology Cystoscopy confirms a fistula between his
bladder and bowel.
A sprightly 79-year-old woman is referred to
your clinic with a 2-month history of diarrhoea.
Which treatment is most likely to be effective?
Which of the following statements is correct? A. total parenteral nutrition (TPN)
A. An infective cause is likely. B. steroids
B. Negative anti endomysial antibodies rule C. antibiotics
out coeliac disease.
D. surgery
C. Nocturnal diarrhoea suggests irritable
E. elemental diet.
bowel syndrome.
D. Her cholecystectomy 3 months earlier Answer & Comments
may be the cause.
Correct answer: D

Dr. Khalid Yusuf El-Zohry – Sohag Teaching Hospital (01118391123) Page | 241
El-Zohry MRCP Questions Bank (Part 2) – Medical Masterclass 2010

These cases are difficult to manage. Ulcerative Answer & Comments


colitis is less likely to cause enterovesical
Correct answer: AF
fistulae than is Crohn’s disease, so the fistula in
this case is probably due to diverticular disease.
Malignancy and ischaemia need also to be Three or more Glasgow/Ranson criteria signify
borne in mind. Treatment may be medical, severe disease.
surgical or a combination, but definitive surgery Ultrasound scan should be performed on
is usually favoured where possible. Diagnosis is admission in all patients (looking for gallstones
best made in liaison with the urologists as well and biliary dilatation) and CT in those with
as gastrointestinal surgeons and often involves severe disease at between day 3 and 10 to look
cystoscopy, CT scanning as well as careful lower for secondary complications and pancreatic
GI investigations. necrosis. Early ERCP may improve prognosis in
those with biliary obstruction and cholangitis.
[ Q: 135 ] MasterClass Part2 A degree of ileus affecting the upper GI tract is
(2010) - Gastroenterology common and nasojejunal tube feeding appears
to be safer and associated with fewer
A 56-year-old woman presents with several
complications than parenteral nutrition.
hours of abdominal pain and an amylase of
more than three times the upper limit of Thirty per cent have recurrent attacks,
normal. On admission she has a glucose of hereditary pancreatitis is well described
11mmol/l and a urea of 17mmol/l. (autosomal dominant), and mortality in patients
with a Glasgow/Ranson score of 3-5 is 10%.
Which of the following statements are true?
A. According to the Glasgow/Ranson [ Q: 136 ] MasterClass Part2
criteria, she has severe disease. (2010) - Gastroenterology
B. The presence of two or more
A 78 year-old man is admitted as an emergency
Glasgow/Ranson criteria signify severe
by his General Practitioner with a history of
disease.
recurrent falls and generalised weakness. His
C. A CT scan should be performed wife died 2 years ago and he volunteers that he
immediately. never learnt to cook himself. Since her death he
has survived on frozen meals and biscuits. A
D. Early endoscopic retrograde
full physical examination is unremarkable,
cholangiography (ERCP) is
excepting that he is very thin, with a BMI of less
contraindicated if cholangitis is present.
than 16, and he has perifollicular haemorrhages
E. She should have bowel rest, be nil by on both shins.
mouth and have parenteral nutrition.
Which of the following vitamins is he most likely
F. PaO2 should be monitored.
to be deficient in?
G. Ileus is uncommon.
A. Vitamin A
H. Pancreatitis recurs in less than 10% of
B. Vitamin C
cases.
C. Vitamin D
I. The mortality rate for patients in this
group is between 1-2%. D. Vitamin E
J. Hereditary pancreatitis is poorly E. Vitamin K
described.

Dr. Khalid Yusuf El-Zohry – Sohag Teaching Hospital (01118391123) Page | 242
El-Zohry MRCP Questions Bank (Part 2) – Medical Masterclass 2010

Answer & Comments If he fails to respond to a second course of oral


metronidazole the second line agent would be
Correct answer: B
oral (not intravenous) vancomycin.

Vitamin C is necessary for hydroxylation of


proline to hydroxyproline, which is required in
[ Q: 138 ] MasterClass Part2
the synthesis of collagen. Deficiency still occurs (2010) - Gastroenterology
in the UK, usually in older people who do not A 54-year-old restaurant manager presents
eat vegetables. Signs include perifollicular with chronic abdominal pain, epigastric in origin
haemorrhages, corkscrew hair, bleeding gums and radiating through to the back, precipitated
(if teeth are present), poor wound healing and by alcohol. Plain abdominal radiograph shows
spontaneous bruising. linear calcification in the area. He also gives a
history of weight loss and offensive floating
[ Q: 137 ] MasterClass Part2 diarrhoea.
(2010) - Gastroenterology
Which of the following statements are true?
An 83-year-old man is about to be discharged
A. His condition is likely to be hereditary.
home, having spent 3 weeks in hospital being
treated for C. difficile diarrhoea associated with B. His condition affects at least 1% of the
pseudomembranous colitis. Unfortunately, the population in the UK.
nursing staff report that he has had a C. Erythema ab igne is an unrelated clinical
recurrence of profuse, malodorous, greenish finding.
diarrhoea. He has already had one course of
oral metronidazole therapy. D. Serum amylase is likely to be markedly
elevated.
Which of the following is the most appropriate E. His pancreolauryl test is likely to be
treatment? abnormal.
A. Intravenous metronidazole F. He should be screened for diabetes.
B. Oral metronidazole G. While pancreatic enzyme supplements
C. Intravenous teicoplanin may help the diarrhoea, they are unlikely
to have any effect on his pain.
D. Intravenous vancomycin
H. Opiate analgesia should be escalated
E. Oral ciprofloxacin
until pain control is achieved.

Answer & Comments I. Moderation of alcohol intake will have no


effect at this late stage
Correct answer: B
J. He is at no increased risk of pancreatic
cancer.
Approximately 20% of patients with
pseudomembranous colitis relapse, usually 2
Answer & Comments
weeks to 2 months after treatment. This is
related to the persistence of C.difficile spores Correct answer: EF
that are not killed by antibiotic therapy, rather
than to metronidazole resistance, hence Chronic alcohol abuse is by far the commonest
recurrence of disease should be treated with a cause of chronic pancreatitis in this country. It
second course of oral metronidazole. is a surprisingly rare condition, with an

Dr. Khalid Yusuf El-Zohry – Sohag Teaching Hospital (01118391123) Page | 243
El-Zohry MRCP Questions Bank (Part 2) – Medical Masterclass 2010

estimated incidence of <4 per 100 000. Pain J. Iron and folate deficiency, but not B12
control can be difficult. Patients often apply deficiency, occur in coeliac disease.
heat pads (hence erythema ab igne) and opiate
abuse and dependence can complicate the Answer & Comments
issue. Pain specialists and techniques such as
coeliac blocks may be useful. Amylase is normal Correct answer: CF
or mildly elevated in chronic pancreatitis (unlike
acute pancreatitis). Exocrine function can be Up to 10% of menstruating woman may
assessed by pancreolauryl test, and enzyme develop anaemia and isolated anti-gliadin IgG
supplements may help both malabsorption and antibodies are present in 10% of the
pain. Mild diabetes is common and should be population. Anti-endomysial antibody is the
treated. Alcoholic abstinence should be strongly most sensitive (80-100%) and specific (98-
encouraged. There is an increased incidence of 100%) antibody in the diagnosis of coeliac
pancreatic cancer in this population. disease. It may be negative in IgA deficiency,
which affects up to 4% of patients. A blood film
[ Q: 139 ] MasterClass Part2 may show Howell Jolly bodies, a feature of
hyposplenism which may occur in coeliac
(2010) - Gastroenterology
disease. Iron, folate and B12 deficiency may
A 26-year-old woman is found to be anaemic occur. Obesity may affect up to a quarter of
and has a raised anti-gliadin IgG antibody titre. newly diagnosed patients. Small bowel
radiology is not a sensitive test for coeliac
Which of the following statements are true? disease, which affects between 5-20% of first-
A. A blood film has no role in contributing to degree relatives.
the diagnosis.
B. The diagnosis is unlikely to be anything
[ Q: 140 ] MasterClass Part2
other than coeliac disease. (2010) - Gastroenterology
C. The presence of Howell Jolly bodies A 78-year-old woman admitted 1 week earlier
would support a diagnosis of coeliac with a stroke develops diarrhoea.
disease.
Which of the following statements are true?
D. Endomysial antibodies are less sensitive
than anti-gliadin antibodies in the A. Campylobacter is the most likely cause.
diagnosis of coeliac disease. B. Clostridium difficile rarely relapses after a
E. IgA deficiency is rare in coeliac disease. full course (7 days) of metronidazole.

F. Histological changes may take over a year C. If an abdominal radiograph shows a toxic
to resolve completely on a gluten-free megacolon, ulcerative colitis is the most
diet. likely diagnosis.

G. Obesity virtually excludes a diagnosis of D. There is no indication for sigmoidoscopy


coeliac disease. and biopsy.

H. Small bowel radiology is the appropriate E. Cephalosporins are the commonest


next investigation. antibiotic associated with Clostridium
difficile infection.
I. There is no increased risk of coeliac
disease in family members.

Dr. Khalid Yusuf El-Zohry – Sohag Teaching Hospital (01118391123) Page | 244
El-Zohry MRCP Questions Bank (Part 2) – Medical Masterclass 2010

Answer & Comments dehydration. However, they should be


monitored closely as an out-patient. The acute
Correct answer: E
hepatitic phase is often followed by a
prolonged cholestatic phase, and while
A relapse rate of up to 30% occurs after relapsing disease is rare, patients should be
treatment of C. difficile, which (along with followed until LFTs normalize.
Campylobacter) can cause a toxic megacolon.
The characteristic pseudomembrane seen on
sigmoidoscopy and histological appearance are
[ Q: 142 ] MasterClass Part2
important diagnostic clues. (2010) - Gastroenterology
A 17-year-old girl with diarrhoea has evidence
[ Q: 141 ] MasterClass Part2 of villous atrophy on her duodenal biopsies.
(2010) - Gastroenterology
Which TWO of the following are NOT associated
A 39-year-old presents with bilirubin 75 with villous atrophy?
micromol/l, ALT 1850 IU/l, alkaline phosphatase
A. Coeliac disease
150 IU/l. She has recently returned from a
beach holiday in West Africa where she B. Whipples disease
suffered from gastroenteritis. C. Chronic pancreatitis

Which of the following statements is correct? D. Lymphoma

A. A positive HAVIgG antibody suggests E. Carcinoid syndrome


acute hepatitis A infection. F. Starvation
B. Acute hepatitis A has a mortality of 2%. G. Tropical sprue
C. A live-attenuated virus vaccine is H. Giardiasis
available to protect travellers to high-risk
areas for hepatitis A. I. Hypogammaglobulinaemia

D. Pooled human immunoglobulin can be J. Bacterial overgrowth.


given to close contacts within 2 weeks of
exposure. Answer & Comments

E. Hospital admission is mandatory. Correct answer: CE

Answer & Comments The classic cause of duodenal villous atrophy is


coeliac disease, in which case it is due to
Correct answer: D
intolerance of gluten, a group of proteins found
in wheat, barley and rye. However, there are
Acute hepatitis A is diagnosed on the basis of a many other causes of villous atrophy, as listed
positive IgM antibody (IgG is a marker of above.
previous exposure, which may well have been
asymptomatic). Hepatitis A has a mortality of
less than 0.05%. The vaccine is a formalin-
[ Q: 143 ] MasterClass Part2
inactivated vaccine. Pooled immunoglobulin (2010) - Gastroenterology
prophylaxis is available if less than two weeks An 84-year-old woman presents to the
from exposure. Hospital admission is rarely Emergency Department with confusion and
required unless patient has vomiting and/or epigastric discomfort. She is not hypoxic. Liver

Dr. Khalid Yusuf El-Zohry – Sohag Teaching Hospital (01118391123) Page | 245
El-Zohry MRCP Questions Bank (Part 2) – Medical Masterclass 2010

function tests are abnormal: bilirubin 120 disorder marked by inflammation of the
micromol/l, alkaline phosphatase 750 IU/l and gallbladder. It is usually caused by the passage
AST 110 IU/l. Her CRP is raised at 120 mg/l. An of a stone from the gallbladder into the cystic
ultrasound shows gallstones in the gallbladder duct (connects the gallbladder to the common
and a mildly dilated common bile duct, but no bile duct). In 5-10% of cases, cholecystitis
intraduct stones. develops in the absence of gallstones
(acalculous cholecystitis).
The most likely diagnosis is:
A. Carcinoma of pancreas [ Q: 144 ] MasterClass Part2
B. Cholecystitis (2010) - Gastroenterology
C. Choledocholithiasis
Which of the following is true in the
D. Ampullary carcinoma investigation of a 30-year-old man with a 2-
E. Primary sclerosing cholangitis. month history of diarrhoea and weight loss?
A. Stool examination is most likely to
Answer & Comments identify a cause

Correct answer: C B. Hydrogen breath testing for small-bowel


bacterial overgrowth is a first-line
investigation
This lady has cholangitis which complicates
common bile duct (CBD) stones but not biliary C. IgA anti-endomysial antibody
malignancy, unless a biliary stent has previously determination is mandatory
been inserted and has blocked. D. A normal C-reactive protein (CRP)
Primary sclerosing cholangitis can be excludes inflammatory bowel disease
complicated by cholangitis but this is unusual in E. The absence of a travel history precludes
a patient who has not had previous an infectious aetiology
instrumentation of the biliary tree (i.e. ERCP).
Do not forget that elderly patients rarely Answer & Comments
complain of pain and that common bile duct
stones may be present several years after a Correct answer: C
cholecystectomy and present in this way. About
75% of CBD stones can be delineated by In the investigation of diarrhoea, history is very
ultrasound although detection will be limited important. Organic disease is more likely if the
when bile duct dilatation is absent. Other useful diarrhoea is of less than 3 months duration and
non-invasive imaging modalities include CT and nocturnal. It is unusual to find an infective
MRI of the biliary tree. Cholelithiasis represents cause if symptoms have persisted for over 4
the presence of gallstones within the months, but untrue to suggest that the absence
gallbladder; choledocholithiasis is the presence of overseas travel precludes an infectious
of gallstones within the common bile (10% of aetiology. Coeliac disease is extremely common
patients with gallstones have with >0.2% sero-prevalence in Western
choledocholithiasis, which is sometimes called populations but can present in subtle ways.
common duct stones). Mild diarrhoea associated with a mild anaemia
is an important presentation in adults and
Cholangitis refers to inflammation of the
serology (for IgA anti-endomysial antibody
common bile duct, usually as a result of
against tissue transglutaminase) has a >95%
secondary infection, while cholecystitis is a

Dr. Khalid Yusuf El-Zohry – Sohag Teaching Hospital (01118391123) Page | 246
El-Zohry MRCP Questions Bank (Part 2) – Medical Masterclass 2010

accuracy in diagnosis. Florid ulcerative colitis Answer & Comments


can be present without evidence of an on-going
Correct answer: CI
acute phase response as measured by CRP.
Small-bowel bacterial overgrowth would be
unusual in a young person in the absence of a Both men and post-menopausal women are
structural or functional intestinal disorder (i.e. recommended to have both upper and lower
blind loop from surgery or diabetes mellitis) but gastrointestinal investigations for iron deficient
would need to be considered in the elderly. anaemia as dual pathology occurs in 5-10% of
cases. Gastritis of varying severity is a common
endoscopic finding and would not readily be
[ Q: 145 ] MasterClass Part2 accepted as a cause for anaemia without
(2010) - Gastroenterology further investigation. Large colorectal polyps
A 56-year-old man presents with iron deficient may bleed sufficiently to cause anaemia.
anaemia. Coeliac disease commonly presents with iron
deficient anaemia without overt
Which of the following are true? gastrointestinal symptoms and the blood film
may show Howell Jolly bodies and target cells,
A. Gastritis identified at gastroscopy is an
features of hyposplenism. Basophilic stippling
acceptable cause and avoids the need for
classically occurs in lead poisoning. In an
further investigation.
achlorhydric stomach, there is impaired
B. Coeliac disease is an unlikely cause in the conversion of ferric iron to the better-absorbed
absence of other symptoms. ferrous form, and iron deficiency may occur.
C. Investigations of both upper and lower Although haemoglobin level may improve by
gut are recommended. over 0.5g/dl per week, 3-4 months iron
supplements are needed to replenish iron
D. Ferrous sulphate can be stopped when stores.
the haemaglobin is within normal range.
E. Three negative faecal occult blood tests [ Q: 146 ] MasterClass Part2
exclude a gastrointestinal cause. (2010) - Gastroenterology
F. In the absence of symptoms, a sustained
A 61-year-old lady presents with a 4-year
response to a three-month course of iron
history of hypertension and an incidental
supplements means that gastrointestinal
finding of massive hepatomegaly. There is a
investigation is unnecessary
family history of renal failure. A CT scan
G. Basophilic stippling on the blood film confirms multiple large thin walled hepatic
suggests haemolysis. cysts and multiple cysts in the kidney.
H. Howell Jolly bodies on the blood film
Which of the TWO pieces of information have
suggests portal hypertension and liver
been correctly given to the patient?
disease.
A. It is an autosomal recessive condition.
I. Iron deficiency may occur in association
with pernicious anaemia. B. There is a risk of subarachnoid
haemorrhage.
J. Colorectal polyps do not cause iron
deficient anaemia. C. She is likely to develop liver failure with
jaundice and coagulopathy.
D. Liver transplantation is contraindicated.

Dr. Khalid Yusuf El-Zohry – Sohag Teaching Hospital (01118391123) Page | 247
El-Zohry MRCP Questions Bank (Part 2) – Medical Masterclass 2010

E. Abdominal discomfort from the cysts can colonoscopy and biopsies taken from colon and
be treated surgically. terminal ileum are all normal.
F. Aspiration of liver cysts successfully
Which of the following statements are true?
relieves abdominal discomfort.
A. Facial flushing and a systolic murmur
G. The condition precludes renal
suggest the need to measure urinary
transplantation.
hydroxyindole-acetic acid.
H. Cirrhosis occurs in the non cystic liver.
B. The symptoms are consistent with
I. Angiotensin-converting enzyme (ACE)- irritable bowel syndrome.
inhibitors should not be used to treat the
C. The commonest site of origin of a
hypertension.
gastrointestinal carcinoid is the stomach.
J. Pancreatic cysts do not occur.
D. A serum k+ level of 2.4 is a typical finding
in a patient with a VIPoma.
Answer & Comments
E. Previous duodenal ulcer disease is
Correct answer: BE unlikely to be of relevance.
F. Carcinoid syndrome has a poor prognosis
The patient is likely to have adult polycystic as the tumour grows rapidly.
kidney disease. Cysts can affect the liver and
G. An eating disorder can be discounted.
pancreas. About 50% of individuals with
multiple large liver cysts will have this condition H. Tryptophan is responsible for the
(i.e. also polycystic kidneys). Patients are at risk symptoms in carcinoid syndome.
of subarachnoid haemorrhage and it is
I. Diarrhoea due to a neuroendocrine
important to control hypertension. No
tumour usually reduces with fasting.
treatment is needed for the liver cysts unless
they cause severe discomfort or bleeding has J. Raised levels of gastrin in a fasted patient
occurred into a large cyst causing pain. Cysts are always due to a gastrinoma.
reaccumulate after aspiration but surgical
fenestration of large cysts is successful in Answer & Comments
selected cases. Liver failure in adult polycystic
Correct answer: AD
liver disease is very uncommon but portal
hypertension due to sinusoidal compression is
well described. Although successful liver-kidney The commonest site of origin of intestinal
transplantation has been reported it is rarely carcinoids is the terminal ileum, although they
indicated. However in cases needing renal may arise from the foregut, midgut or hindgut.
transplantation it is often necessary to remove The classical symptoms of the carcinoid
the diseased kidneys to make room for the syndrome are flushing, wheeze and diarrhoea.
transplant kidney.
Fibrosis of tricuspid and pulmonary valves may
occur.
[ Q: 147 ] MasterClass Part2
Metastatic carcinoids take up tryptophan and
(2010) - Gastroenterology
metabolize it to 5-hydroxytryptamine which
A 44-year-old woman presents with 3 months appears to have a role in the pathogenesis of
vague abdominal pain, diarrhoea and weight the diarrhoea and bronchoconstriction.
loss. Duodenal biopsies, small bowel studies,

Dr. Khalid Yusuf El-Zohry – Sohag Teaching Hospital (01118391123) Page | 248
El-Zohry MRCP Questions Bank (Part 2) – Medical Masterclass 2010

Many carcinoids are slow growing with a good of variceal haemorrhage in the long term, but
prognosis. has no role in the acute setting.
Raised gastrin levels may occur in achlorhydria Intravenous proton pump inhibitors may be
due to pernicious anaemia or proton pump beneficial in patients with actively bleeding
inhibition. peptic ulceration, but have no role in variceal
bleeding. Intravenous ranitidine has never been
The diarrhoea of endocrine tumours is
shown to affect outcome following GI bleeding
secretory and tends not to decrease with
of any type, while eradication of helicobacter
fasting.
will only be important in the short term if he is
Weight loss cannot be ascribed to irritable shown at endoscopy to have peptic ulceration
bowel syndrome and diarrhoea may be due to as a cause for his bleeding.
laxatives in eating disorders.

[ Q: 149 ] MasterClass Part2


[ Q: 148 ] MasterClass Part2 (2010) - Gastroenterology
(2010) - Gastroenterology
A 70-year-old man presents to casualty with
A 45-year-old man with a past history of chronic lower abdominal pain and fever. He is not
alcohol abuse presents is brought to A&E after jaundiced and blood analysis revealed only a
vomiting one litre of fresh red blood. Blood raised white cell count, with normal liver
pressure on arrival is 90/40 with pulse 110 function tests. Previous upper GI endoscopy is
/min. He is noted to be jaundiced, with multiple reported as normal and urinalysis and MSU
spider naevi and to have marked ascites. were normal.

Which of the following drugs is most likely to be What is the most likely diagnosis?
beneficial in his initial treatment?
A. Cholecystitis
A. Intravenous ranitidine
B. Diverticulitis
B. Oral propranolol
C. Gastritis
C. Intravenous terlipressin
D. Carcinoma of the bowel
D. Intravenous proton pump inhibitor
E. Pyelonephritis.
E. Helicobacter eradication therapy.
Answer & Comments
Answer & Comments
Correct answer: B
Correct answer: C
Diverticula (usually acquired) are herniations of
This patient has evidence of chronic liver the mucosa and submucosa or the entire wall
disease with portal hypertension, so that a thickness through the muscle layer of any part
diagnosis of bleeing oesophageal varices should of the gastrointestinal tract (sigmoid being the
be strongly considered. most commonly affected segment [>95%]
Intravenous terlipressin has been shown to although disease can involve the descending,
reduce variceal pressure following bleeding, ascending, and transverse colon as well as the
which may influence prognosis. Propranolol is jejunum, ileum, and duodenum). Acute
beneficial in primary and secondary prevention diverticulitis results from the inspissation of
faecal material in the neck of the diverticulum

Dr. Khalid Yusuf El-Zohry – Sohag Teaching Hospital (01118391123) Page | 249
El-Zohry MRCP Questions Bank (Part 2) – Medical Masterclass 2010

and resultant bacterial replication. Thereafter HCC is an important cause of liver morbidity
an abscess or peridiverticular inflammation and mortality, with increasing treatment
occurs following rupture of a microscopic options available (resection, ablation,
mucosal abscess into the mesentery. The transplantation). Chronic hepatitis B,C, alcoholic
infection may progress, fistulise, obstruct, or liver disease, autoimmune hepatitis and
spontaneously resolve. Lower GI bleeding from haemochromatosis are all known causes of
diverticulosis results from rupture of the small HCC. Patients with cirrhosis are therefore,
blood vessels that are stretched while coursing advised to be screened every 6 months by US
over the dome of the diverticula. and measurement of their alpha-fetoprotein
level.
[ Q: 150 ] MasterClass Part2 Colorectal cancer, coeliac disease, irritable
(2010) - Gastroenterology bowel syndrome and ulcerative colitis are not
known to be associated with HCC. Conversely,
A 65-year-old man presents with evidence of there is emerging but clear evidence that
abdominal ultrasound of a well circumscribed diabetes mellitus, probably as a result of its
lesion which on a triphasic CT scan is judged to association with non alcoholic fatty liver
be a primary hepatocellular carcinoma (HCC). disease, is a cause of chronic liver disease and
HCC in its own right.
Which of the following are TRUE?
A. Chronic hepatitis B is not a known cause
of HCC
B. Chronic hepatitis C is not a known cause
of HCC
C. Alcoholic liver disease is not a known
cause of HCC
D. Diabetes mellitus is a known cause of
HCC
E. Genetic haemochromatosis is not a
known cause of HCC
F. Autoimmune hepatitis is not a known
cause of HCC
G. Colorectal rectal cancer is a known cause
of HCC
H. Coeliac disease is not a known cause of
HCC
I. Irritable bowel syndrome is a known cause
of HCC
J. Ulcerative colitis is a known cause of HCC

Answer & Comments

Correct answer: DH

Dr. Khalid Yusuf El-Zohry – Sohag Teaching Hospital (01118391123) Page | 250
El-Zohry MRCP Questions Bank (Part 2) – Medical Masterclass 2010

radiation and is a sensitive method for

Nephrology detecting renal artery stenosis.


At reduced GFR (eGFR<30, CKD4) there is a risk
that exposure to gadolinium in the
(149 Questions) MRA contrast agent might precipitate
nephrogenic fibrosing sclerosis, and CT
(Medical Masterclass – Part 2) angiography would be preferred if renal
function was significantly impaired.

[ Q: 2 ] MasterClass Part2
[ Q: 1 ] MasterClass Part2
(2010) - Nephrology
(2010) - Nephrology
(1) A 24-year-old man presents with a history
A 74-year-old man who is a smoker and has of frank haematuria 3 months previously. He
peripheral vascular disease is referred to the now has blood pressure of 148/87 mmHg and
nephrology clinic following an episode of urinary dipstick analysis reveals blood + as the
sudden-onset pulmonary oedema, despite only abnormality. Laboratory tests reveal serum
having good left ventricular function on sodium 140 mmol/l, potassium 3.9 mmol/l,
echocardiography. He has slightly impaired urea 11.0 mmol/l and creatinine 147
renal function (creatinine 120 μmo/L, eGFR 51, micromoles/l.
CKD3A) and an ultrasound has demonstrated
kidneys measuring 9.1 and 10.9 cm in length. Which of the following is the most likely
The referral letter raises the question as to diagnosis?
whether he may have renovascular disease
A. Focal segmental glomerulosclerosis
causing ‘flash' pulmonary oedema. On
examination he has poor pedal pulses and a left B. IgA nephropathy
carotid bruit. C. Membranous nephropathy
What is the most appropriate investigation to D. Minimal change disease
pursue the possibility that he has renovascular E. Kawasaki’s disease
disease?
A. CT abdomen and pelvis Answer & Comments
B. Captopril renography Correct answer: B
C. Intravenous urography
D. Magnetic resonance angiography IgA nephropathy is the commonest
glomerulonephritis worldwide and can present
E. Doppler ultrasonography with frank haematuria. Hypertension and renal
impairment are well recognised features.
Answer & Comments

Correct answer: D [ Q: 3 ] MasterClass Part2


(2010) - Nephrology
The clinical suspicion of renovascular disease is
A patient’s abdominal radiograph is reported to
high in this case. Magnetic resonance
show nephrocalcinosis.
angiography (MRA) avoids exposure to ionising

Dr. Khalid Yusuf El-Zohry – Sohag Teaching Hospital (01118391123) Page | 251
El-Zohry MRCP Questions Bank (Part 2) – Medical Masterclass 2010

Which TWO of the following diagnoses is NOT D. Proteinuria of 1.2g per 24 hours
causely associated with this finding?
E. Knowledge that the creatinine had been
A. Primary hyperparathyroidism 110 micromoles per litre 18 months
B. Medullary cystic disease earlier.

C. Idiopathic hypercalciuria
Answer & Comments
D. Hypervitaminosis D
Correct answer: A
E. Cortical necrosis
F. Primary hyperoxaluria Diabetic nephropathy is a microvascular
complication and rarely occurs without other
G. Sarcoidosis
evidence of microvascular disease, such as
H. Milk alkali syndrome retinopathy. It is often associated with
I. Chronic hyperuricaemic nephropathy suboptimal glycaemic control and proteinuria.
Renal size is preserved but following the
J. Type I (distal) renal tubular acidosis
development of microalbuminuria the disease
may progress with alarming rapidity
Answer & Comments

Correct answer: BI [ Q: 4 ] MasterClass Part2


(2010) - Nephrology
Nephrocalcinosis is the term used to describe
A 60-year-old man presents acutely with a
diffuse calcification within the kidney. This can
recent history of oedema, proteinuria of 10 g
arise as a result of hypercalcaemia or
per 24 hours and serum albumin of 20 g/L.
hypercalciuria (primary hyperparathyroidism;
idiopathic hypercalciuria; hypervitaminosis D; Which of the following diagnoses is most
sarcoidosis; milk alkali syndrome). In consistent with the presentation?
hyperoxaluria deposition of insoluble calcium
oxalate is favoured, even if the calcium level is A. Amyloidosis
normal. In renal tubular acidosis, failure of B. Sarcoidosis
urinary acidification and a reduction in urinary
C. Wegener's granulomatosis
citrate excretion both favour calcification.
Nephrocalcinosis is seen in renal cortical D. Acute interstitial nephritis
necrosis and medullary sponge kidney, but is E. Hepatorenal syndrome
not caused by chronic hyperuricaemic
nephropathy or medullary cystic disease.
Answer & Comments
4. In a patient with diabetes mellitus and a Correct answer: A
serum creatinine of 210 micromoles per litre,
which one of the following findings would lead
Renal amyloidosis typically presents with
you to suspect a diagnosis other than diabetic
nephrotic syndrome and accounts for a
nephropathy?
significant proportion of all nephrotic
A. Normal fundoscopy presentations in the elderly. None of the other
B. HbA1C of 8.1% conditions typically present with nephrotic
syndrome, although they are all important
C. Normal-sized kidneys on ultrasound causes of renal disease.
examination

Dr. Khalid Yusuf El-Zohry – Sohag Teaching Hospital (01118391123) Page | 252
El-Zohry MRCP Questions Bank (Part 2) – Medical Masterclass 2010

[ Q: 5 ] MasterClass Part2 being told that he had urine infections as a


(2010) - Nephrology child.

A 46-year-old man presents with a 33-year Which TWO of the following statements do you
history of lithium carbonate therapy for bipolar agree with?
affective disorder. He is polyuric (11 litres) and A. The glomerular filtration rate is in the
has 2 grams / 24 hours of proteinuria. His GFR is region of 120 ml/min.
47-mls/min/1.73 m2. BP is raised at 150/100
mm Hg. He has a renal biopsy. B. The patient has the nephrotic syndrome.
C. A micturating cystogram should be
What would be typical findings? performed and is likely to demonstrate
A. IgA glomerulonephritis vesicoureteric reflux.

B. Crescentic glomerulonephritis D. The risk of developing endstage renal


failure is low (less than 5% over 20 years).
C. Renal Vasculitis
E. The patient should be given rotating
D. Focal glomelurosclerosis and interstitial
chemoprophylaxis to prevent urinary
nephritis
tract infection.
E. Granulomata.
F. Renal biopsy is indicated, and is most
likely to show membranous nephropathy.
Answer & Comments
G. A thiazide diuretic is a particularly
Correct answer: D attractive antihypertensive agent in this
setting.
Lithium toxicity is much more common than H. The patient should be advised to drink at
appreciated by most psychiatrists. It is rarely least 2 litres of water a day.
detected early enough for reversibility. It is
typically a chronic interstitial nephritis but a I. The likely diagnosis is reflux nephropathy.
secondary FSGS lesion due to hyperfiltration of J. An angiotensin-converting enzyme (ACE)
remnant nephrons can be seen in 30% of cases. inhibitor is likely to reduce the rate of
Options include running lithium at very low protein excretion.
levels, or a switch to valproate or
carbemazepine. Answer & Comments

Correct answer: IJ
[ Q: 6 ] MasterClass Part2
(2010) - Nephrology
The history is typical of reflux nephropathy (also
A 35-year-old man is referred with heavy called chronic pyelonephritis) with an adult
proteinuria on dipstick testing, hypertension presentation characterised by hypertension,
(150/100 mmHg) and noticeable peripheral renal impairment, and scarred kidneys. Since
oedema. He weighs 70kg. Investigations show this patient is male, hypertensive, and has
plasma creatinine 178 micromoles/l, serum substantial proteinuria and renal impairment
albumin 38 g/l, urine protein excretion 2.5 g/24 the risk of progression is high. His current
hours and renal ultrasound shows bilaterally estimated GFR is approximately 45 ml/min
scarred kidneys of length 8.5 and 9.1 cm. On based on age, sex, weight and creatinine; a GFR
specific questioning the patient remembers of 120 ml/min is normal at this age and is
clearly incorrect given the elevated creatinine.

Dr. Khalid Yusuf El-Zohry – Sohag Teaching Hospital (01118391123) Page | 253
El-Zohry MRCP Questions Bank (Part 2) – Medical Masterclass 2010

ACE inhibition will usually decrease protein F. ACE Inhibitors should be avoided as she
excretion by 25%-50%. Although the patient has has significant renal impairment
oedema and proteinuria he is not nephrotic
G. Primary amyloidosis is the most likely
since the plasma albumin is normal.
diagnosis
Reflux nephropathy is due to vesicoureteric
H. Membranous nephropathy is a possible
reflux (VUR) and infections in infancy; usually
finding on renal biopsy
the VUR itself resolves so a micturating
cystogram is likely to be normal in this patient. I. Loop diuretics should be avoided
There is no evidence that chemoprophylaxis or J. Renal angiography is the most useful next
a maintaining a high fluid intake in adults with diagnostic test
established reflux nephropathy improves their
prognosis.
Answer & Comments

[ Q: 7 ] MasterClass Part2 Correct answer: DH

(2010) - Nephrology
This woman has presented with the nephrotic
A 56-year-old woman is seen in the renal syndrome with evidence of impaired excretory
outpatient department with a history of function in the context of chronic inflammatory
increasing leg oedema in association with new disease. Secondary AA amyloidosis is the most
onset proteinuria. She has a longstanding likely diagnosis here, although membranous
history of seronegative arthritis controlled with nephropathy would also be a contender. Renal
simple analgesia alone, as well as crohn's biopsy is the best diagnostic procedure.
disease held in remission with prednisolone and
Differentiation between Primary (AL) and
azathioprine. Investigations demonstrate
Secondary (AA) amyloidosis can be difficult in
proteinuria at a level of 10g in 24hrs, a low
around 10% of patients, but the absence of a
serum albumin (18g/dl), normal
paraprotein and the long history of chronic
immunoglobulins, Bence Jones negative, and a
inflammatory disease are most supportive of
negative autoantibody screen. Her ESR and CRP
AA amyloid. One would expect an elevated SAA
are both elevated and have been so for at least
level and a normal lambda/kappa ratio on a
10 years. Excretory renal function was reduced
free light chain assay.
with an estimated GFR of 36ml/1.73m2/min.

Which of the following statements are most [ Q: 8 ] MasterClass Part2


likely to be TRUE? (2010) - Nephrology
A. The renal lesion is likely to be interstitial
You wish to calculate a patient's creatinine
nephritis secondary to analgesics
clearance. He is 30 years old, and weighs 70 kg.
B. She is likely to require conversion from His plasma creatinine is 100 micromol/l. A 24-
azathioprine to cyclophosphamide hour urine collection has a volume of 1.2 litres,
C. Minimal change disease is the most likely and a creatinine concentration of 12 mmol/l.
finding on renal biopsy
Which TWO of the following statements are
D. Amyloidosis is a likely finding on renal correct?
biopsy
A. The urine collection is almost certainly
E. Renal biopsy should be avoided as the incomplete.
result will not influence management

Dr. Khalid Yusuf El-Zohry – Sohag Teaching Hospital (01118391123) Page | 254
El-Zohry MRCP Questions Bank (Part 2) – Medical Masterclass 2010

B. The calculated creatinine clearance is 100 hypertension and proteinuria at 34 weeks. For
ml/min. this reason, she underwent induction of labour
at 37 weeks and gave birth to a healthy baby
C. The calculated creatinine clearance is 120
boy. She was well before and after the delivery.
ml/min.
At her first booking appointment at 8 weeks she
D. The calculated creatinine clearance is 50 was noted to have a blood pressure of 156/91
ml/min. and again at 10 weeks this was 162/93.
E. The calculated creatinine clearance is However, this fell to a normal level over the
outside the normal range for a man of next few weeks and was only noted to be high
this age. again at 34 weeks.

F. Urine has almost certainly been collected Which of the following statements are true?
for more than 24 hours.
A. Systemic vascular resistance typically
G. The calculated creatinine production rate rises during pregnancy
is normal for a man of this age.
B. Pre-eclampsia usually presents in early
H. The height of the patient is required to pregnancy
interpret the data.
C. Haematuria in this lady is consistent with
I. Calculated clearance is >20% higher than a diagnosis of IgA nephropathy
predicted on the basis of age, sex, weight
D. Blood pressure typically rises in early
and plasma creatinine.
pregnancy but falls in mid pregnancy
J. The patient probably eats large amounts
E. Proteinuria is a normal finding in
of meat.
pregnant women

Answer & Comments F. This lady is unlikely to have had pre-


eclampsia during her pregnancy
Correct answer: BG
G. Cardiac output falls during pregnancy

The correct calculation is creatinine clearance = H. Renal vascular resistance normally rises
(1.2 x 12 x 1000 x 1000)/(100 x 24 x 60) = 100 during pregnancy
ml/min. This is normal for a man of this age and I. The haematuria is not likely to indicate an
actually agrees rather closely with that underlying glomerulonephritis
predicted using the Cockroft-Gault formula.
J. It is likely that this lady had higher than
This patient's measured creatinine production
average blood pressure before her
is normal, so the values do not suggest the
pregnancy
collection is incomplete or lasted for more than
24 hours.
Answer & Comments

[ Q: 9 ] MasterClass Part2 Correct answer: CJ


(2010) - Nephrology
During pregnancy, systemic vascular resistance
A 30-year-old woman is noted to have
falls due to the effects of prostaglandins
haematuria on a routine health check. Her
produced by the placenta. This lowers systemic
plasma creatinine is 112 micromoles/L. 6
blood pressure, despite a rise in cardiac output.
months previously she had a successful
Blood pressure falls to a nadir in the second
pregnancy that was complicated by
trimester and slowly rises towards full term. If

Dr. Khalid Yusuf El-Zohry – Sohag Teaching Hospital (01118391123) Page | 255
El-Zohry MRCP Questions Bank (Part 2) – Medical Masterclass 2010

blood pressure is relatively high in early [ Q: 11 ] MasterClass Part2


pregnancy, this usually suggests that there was (2010) - Nephrology
pre-existing chronic hypertension. Commonly in
this situation, blood pressure will fall in mid A 28-year-old man was referred with
pregnancy, but rise again in the third trimester. microscopic haematuria and hypertension
Chronic hypertension before pregnancy (158/96). Initial investigations demonstrated
increases the risk of superimposed pre- normal-sized kidneys, a serum creatinine
eclampsia during the pregnancy. Proteinuria is reading of 148 umol/L and a urinary protein
not normal in pregnancy and is a feature of pre- excretion reading of 3g/24hr. A renal biopsy
eclampsia. Haematuria and hypertension are established a diagnosis of IgA nephropathy. He
both features that can occur with IgA was reviewed in outpatients after the biopsy
nephropathy. and would like to know a little about his
condition.
[ Q: 10 ] MasterClass Part2 Select the TWO statements which you agree
(2010) - Nephrology most strongly with.
A 62-year-old man presents with swollen A. There is virtually no risk of end-stage
ankles. Dipstick analysis of his urine reveals renal failure requiring dialysis in this
protein +++, but no other abnormality. He is condition.
noted to have long-standing gout, hypertension B. Blood pressure control should aim to
and hypercholesterolaemia. His plasma bring average readings below 140/90 mm
creatinine is 105micromoles/l and albumin Hg.
22g/l.
C. Anaemia is a common early problem.
Which of the following medications would be D. ACE Inhibitors should be avoided in
most likely to account for his swollen ankles and
patients with renal impairment.
proteinuria?
E. Immunosuppressive therapy is the main
A. Allopurinol
hope of maintaining stable renal function.
B. Frusemide
F. A bone marrow biopsy would be helpful
C. Enalapril to exclude an IgA myeloma.
D. Ibuprofen G. General fatigue is likely to be explained
by this diagnosis.
E. Prednisolone
H. Viagra is contraindicated.
Answer & Comments I. Proteinuria is a poor prognositic marker
Correct answer: D for renal survival.
J. Blood Pressure control is the most
Non-steroidal anti-inflammatory drugs can important management aim in this
cause nephrotic syndrome with microscopical condition.
appearances of minimal change disease. They
can also lower glomerular filtration rate by Answer & Comments
vascular effects and cause an interstitial
Correct answer: IJ
nephritis.

Dr. Khalid Yusuf El-Zohry – Sohag Teaching Hospital (01118391123) Page | 256
El-Zohry MRCP Questions Bank (Part 2) – Medical Masterclass 2010

IgA nephropathy is a common cause of Answer & Comments


glomerular disease. The key three poor
Correct answer: D
prognostic factors for renal survival are
significant proteinuria, hypertension and initial
impaired renal function. Rates of progression The history is consistent with an ANCA-
are fairly variable but around 30% of patients associated vasculitis. Bronchoscopy and lavage
will eventually require renal replacement would be very hazardous. A renal biopsy may
therapy. This man has evidence of significant be appropriate, especially if ANCA are not
renal impairment at presentation but his detected in the blood, but light microscopy and
glomular filtration rate (GFR) is unlikely to be immunofluorescence are more likely to be
low enough to explain fatigue or cause renal useful than the electron microscopic
anaemia. Angiotensin- converting enzyme (ACE) appearance. A genetic test for Alport's
inhibitors are the preferred agent. The target syndrome is not appropriate here.
blood pressure should be stringent, 125/75 or
lower, for a patient with heavy proteinuria. [ Q: 13 ] MasterClass Part2
Immunosuppressive therapy may have a role in (2010) - Nephrology
a subset of patients with IgA disease
(particularly those with rapidly declining A 50-year-old man attends the Accident and
function and crescentic changes on biopsy) Emergency department and reports that on
however this is not backed by robust data from passing urine he also passed a small lump. He
randomised trials. retrieved this and has brought it along in a jam
jar of water. He is otherwise well apart from
severe osteoarthritis of the left hip. His serum
[ Q: 12 ] MasterClass Part2
creatinine is 160 micromoles/l and his albumin
(2010) - Nephrology is 40 g/l. A frozen section is performed by a
A 54-year-old man presents to the Emergency helpful pathologist who reports that the lump is
Department with acute renal failure and a renal papilla.
haemoptysis. He has had arthralgia for the last
Which of the following could account for this in
2 months, but in the past has been completely
his case?
well. He had an insurance medical 8 months
previously at which his BP was normal and A. Membranous nephropathy
urinalysis was negative. B. Non-steroidal anti-inflammatory drug use
for his osteoarthritis
Which single test listed below is most likely to
be helpful in establishing a precise diagnosis? C. Nephrotic syndrome
A. Bronchoscopy and lavage D. HIV nephropathy
B. Identifying a mutation in a gene encoding E. End stage renal disease.
one of the α chains of type IV collagen
C. CT scan of chest and abdomen Answer & Comments

D. Assay for circulating antineutrophil Correct answer: B


cytoplasmic antibodies (ANCA)
E. Examination of a renal biopsy by electron Analgesics, including non-steroidal anti-
microscopy inflammatory drugs can cause papillary
necrosis. This is likely to be caused by
vasoconstriction of blood vessels within the

Dr. Khalid Yusuf El-Zohry – Sohag Teaching Hospital (01118391123) Page | 257
El-Zohry MRCP Questions Bank (Part 2) – Medical Masterclass 2010

kidney. Prostaglandins cause tonic Cortical blindness has been reported but
vasodilatation of renal blood vessels. Non- cyclosporin does not cause disturbed colour
steroidal anti-inflammatory drugs block vision. Cyclosporin may be useful in the
prostaglandin synthesis and so block this treatment of proteinuria. Although cyclosporin
vasodilation, resulting in relative can cause thrombocytopenia (and haemolysis)
vasoconstriction. it rarely causes myelosuppression. It can cause
rashes, but not pigmentation. It may produce
[ Q: 14 ] MasterClass Part2 hyperkalaemia and hypomagnesaemia.

(2010) - Nephrology
[ Q: 15 ] MasterClass Part2
From the following list, select TWO unwanted (2010) - Nephrology
effects that commonly develop in patients
treated with cyclosporin. The image shows an electronmicrograph of part
of a glomerulus. This renal biopsy was from a
A. Angioedema patient with microscopic haematuria and
B. Disturbed colour vision proteinuria (1 g/24 hours). The plasma
creatinine was in the normal range. ‘A’ marks
C. Increased proteinuria
the capillary lumen.
D. Microscopic haematuria
E. Hypertrichosis
F. Leukopenia
G. Hypermagnesaemia
H. Hypertension
I. Skin pigmentation
J. Postural hypotension.

Answer & Comments

Correct answer: EH

Cyclosporin has a long list of side effects.


Select one of the following statements:
Common problems include:
 A. The glomerular basement membrane
gingival hypertrophy
appears normal.
 hypertrichosis
B. The podocytes show extensive foot
 tremor process fusion and effacement.
 burning paraesthesiae C. The arrow indicates subendothelial
 hypertension deposits which are abnormal.

 decline in renal function D. These appearances effectively rule out


systemic lupus erythematosus (SLE),
 confusion
subacute bacterial endocarditis or mixed
 myalgias essential cryoglobulinaemia as causes of
 the nephritis.
weakness.

Dr. Khalid Yusuf El-Zohry – Sohag Teaching Hospital (01118391123) Page | 258
El-Zohry MRCP Questions Bank (Part 2) – Medical Masterclass 2010

E. The most likely diagnosis is post E. His pain may result from urinary tract
streptococcal glomerulonephritis. stone disease

Answer & Comments Answer & Comments

Correct answer: C Correct answer: D

The arrowhead indicates deposits that are Obesity can result in renal impairment due to a
located under the endothelium - i.e. focal segmental glomerulosclerosis. Obesity is
subendothelial, resulting in an abnormally associated with type 2 diabetes mellitus that
widened, irregular, glomerular basement can result in nephropathy. Non-steroidal anti-
membrane. The podocyte foot processes are inflammatory drugs are contraindicated in
essentially intact. patients with renal impairment as they can
further reduce renal perfusion. Normally there
In poststreptococcal nephritis the deposits are
is prostaglandin-induced renal vasodilation,
subepithelial, in contrast to those seen here.
which is inhibited by non-steroidal anti-
Subendothelial deposits reflect deposition of inflammatory drugs. Non-steroidal anti-
circulating immune complexes, and are seen in inflammatory drugs can also cause an
SLE nephritis (usually accompanied by deposits interstitial nephritis and a minimal change
in other sites) with less common causes being nephropathy. Severe colicky loin pain is a
mixed essential cryoglobulinemia, subacute common presentation of renal stone disease.
bacterial endocarditis (SBE), and primary type I
mesangiocapillary glomerulonephritis.
[ Q: 17 ] MasterClass Part2
(2010) - Nephrology
[ Q: 16 ] MasterClass Part2
(2010) - Nephrology A 36-year-old woman with autosomal dominant
polycystic kidney disease is approaching end-
A 58-year-old man presents with severe right stage renal failure. Her husband, aged 40, and
loin pain. He is obese with a body mass index of her sister, aged 44, have both expressed
42 and his blood pressure is 186/93 when interest in donating a kidney. The patient is
measured with an appropriately large cuff. On blood group O. She and her husband do not yet
urine dipstick analysis he has haematuria ++ have any children.
and proteinuria +. His plasma creatinine is
213micromol/L. Which of the following statements are correct?
A. If her sister is blood group A she can
Which of the following responses is NOT
probably be ruled out as a donor.
correct?
B. An advantage of a transplant from a
A. Renal impairment can arise as a result of
sibling is that immunosuppression can
obesity
usually be discontinued after 2 years.
B. He could have diabetic nephropathy
C. If a renal ultrasound shows no renal cysts
C. He could have renal impairment as a in the patient's sister it is reasonable to
result of hypertensive renal damage assume she does not have ADPKD.
D. The analgesic of choice in his case is a D. The probability that her sister is HLA
non-steroidal anti-inflammatory drug identical is 1 in 8.

Dr. Khalid Yusuf El-Zohry – Sohag Teaching Hospital (01118391123) Page | 259
El-Zohry MRCP Questions Bank (Part 2) – Medical Masterclass 2010

E. If the patient wants to have children she transplants between identical twins
should do this before being transplanted immunosuppression is necessary indefinitely.
(because of the side effects of the
Living unrelated transplants, for example
immunosuppressive drugs).
between husband and wife, are ethically and
F. In the United Kingdom, kidney transplants legally acceptable in the UK. Donating a kidney
between husbands and their wives are may increase the lifetime risk of hypertension,
not legally permitted. but there is no evidence for an increase in end
stage renal disease.
G. The patient should be advised that over a
ten year period her mortality if she is If the patient would like to become pregnant
treated by dialysis or by transplantation then the probability of a good outcome for
will be similar. mother and baby is better with a functioning
renal transplant than with severe renal
H. Without any further information
insufficiency.
concerning the sister the probability that
she has ADPKD is 50%.
[ Q: 18 ] MasterClass Part2
I. The total body irradiation that is necessary
before renal transplantation is likely to (2010) - Nephrology
decrease her fertility. A 37-year-old man, a type I diabetic for six
J. The potential donors need to appreciate years, presents with a rising creatinine over two
that if a healthy 40 year old donates a years. There is mild background diabetic
kidney they have an approximately 4 fold retinopathy and no neuropathy. Proteinuria
increase in lifetime risk of end stage renal was also noted two years ago, and is now 12
disease. grams / day and he is nephrotic. He has no
microscopic haematuria. BP is mildly elevated
Answer & Comments at 145/95 mm Hg.

Correct answer: AH This is not typical of diabetic nephropathy


because:
If her sister is blood group A then she is "ABO A. Onset of renal problems too soon after
incompatible" and unsuitable for diagnosis of type I DM
transplantation because the recipient will have
B. Patients under 40 do not get renal
anti-A antibodies. The A2 subgroup expresses
diabetic complications
reduced amount of A antigens and can, on
occasions, be used to transplant into O C. Significant hypertension is invariable in
individuals. diabetic nephropathy

The chance that a sibling has ADPKD is 50%, D. The absence of microscopic haematuria
since it is an autosomal dominant trait. It is safe excludes a glomerular lesion
to assume that a person over the age of 30 with E. Nephrotic syndrome is not due to diabetic
no renal cysts does not have a PKD1 (polycystin) nephropathy.
mutation. Given the severity of this patient's
renal disease the family is very unlikely to have
Answer & Comments
a PKD2 mutation. Siblings have a one in four
chance of being identical for both HLA Correct answer: A
haplotypes. With the rare exception of

Dr. Khalid Yusuf El-Zohry – Sohag Teaching Hospital (01118391123) Page | 260
El-Zohry MRCP Questions Bank (Part 2) – Medical Masterclass 2010

Typically it is 10 - 20 years after the diagnosis of Which TWO of the following statements are
type I DM that patients first experience micro- correct?
albuminuria, then heavier proteinuria, and A. Pancreatitis is the underlying problem.
renal functional decline (as hyperfitration and
raised GFR finishes). Thus this is too soon, and B. Enterolysis should not be attempted.
also the proteinuria did not precede the renal C. Sclerosing encapsulating peritonitis is the
impairment. Hypertension is seen in about 90% correct diagnosis.
of patients with diabetic nephropathy
D. Immunosuppression is contra-indicated.
(remember elevated BP in this setting is
anything > 130/80 mm Hg). Few patients with E. Restarting peritoneal dialysis is
diabetic nephropathy have microscopic important.
haematuria, but some do). About one third of
F. Total parenteral nutrition (TPN) is likely to
diabetic nephropathy patients go through a
be needed for a long time.
nephrotic phase. This patient requires a renal
biopsy (and had membranous nephropathy G. Transplantation cannot now be an
which responded to immunosuppression option.
H. This problem is inevitable in patients on
[ Q: 19 ] MasterClass Part2 peritoneal dialysis for >5 years.
(2010) - Nephrology I. Eosinophilic inflammation is characteristic
on peritoneal biopsy.
A 42-year-old lady with chronic renal failure
who had been on peritoneal dialysis for 9 years J. The large bowel is never affected.
switched to haemodialysis after a severe
episode of peritonitis. Three months later she Answer & Comments
presented with small bowel obstruction. At CT
and laparatomy these appearances were found. Correct answer: CF

This much feared complication of long-term


peritoneal dialysis (PD) - sclerosing
encapsulating peritonitis (SCP) - happens to
about 20% of patients on PD for eight or more
years, but although peritoneal membrane
thickening happens with increased time on PD
and with peritonitis episodes, it is not inevitable
that SCP will ensue, even after 15 or more
years. It is probably unwise though, and often
impractical, to keep patients on PD for this long.
Recurrent infection, possibly beta-blockers, and
more certainly glucose degradation products
and advanced glycation end products, from the
hypertonic glucose production, are responsible.
Peritoneal thickening and calcification can be
seen on the CT scan. At laparotomy there was a
thick inflammatory pannus completely
enveloping the small and parts of the large
bowel. This can be enterolysed, albeit with

Dr. Khalid Yusuf El-Zohry – Sohag Teaching Hospital (01118391123) Page | 261
El-Zohry MRCP Questions Bank (Part 2) – Medical Masterclass 2010

difficulty. TPN is often needed for months. Which TWO of the following statements are
Immunosuppression in several small series is true?
beneficial, though patients can be too ill / septic A. An urgent percutaneous renal transplant
to respond by this stage. If they can recover to biopsy should be carried out.
a degree, then transplantation may be an
option. B. The clinical picture could be due to acute
rejection.

[ Q: 20 ] MasterClass Part2 C. The clinical picture is that of haemolytic


(2010) - Nephrology uraemic syndrome.
D. The thrombocytopenia is due purely to
A 28-year-old woman with biopsy-proven IgA
rapamycin toxicity.
nephropathy and a stable serum creatinine of
~130 μmol/L informs you that she has done a E. The rapamycin should be stopped
pregnancy test and that it is positive. immediately.

She is taking the following medications. F. The clinical picture could be due to CMV
disease.
Which is most important to stop and substitute
G. Platelets should be administered to the
with a safer alternative?
patient.
A. Folic acid
H. The patients clotting profile is likely to be
B. Lisinopril deranged.
C. Ranitidine I. Ciclosporin should be stopped
D. Aspirin immediately.

E. Nifedipine J. Tacrolimus should be substituted for


ciclosporin.

Answer & Comments


Answer & Comments
Correct answer: B
Correct answer: CI

Angiotensin-converting enzyme inhibitors are


contraindicated in pregnancy, especially in the Calcineurin inhibitors such as ciclosporin and
second and third trimester due to increased tacrolimus are associated with haemolytic
fetal malformations. uraemic syndrome (HUS). This is usually seen
on a local level in the transplanted kidney with
intraglomerular thrombi but occasional patients
[ Q: 21 ] MasterClass Part2 have systemic manifestations such as those in
(2010) - Nephrology this patient.
A 40-year-old renal transplant recipient of 4 A transplant biopsy would be contraindicated in
months presents with an acute deterioation of view of the low platelet count. It would be
renal function with the creatinine rising from extremely unusual for acute rejection to
132 micromol/l to 180. His haemoglobin and present in this manner.
platelets have fallen from 13 to 9.8g/dl and 242
Rapamycin can cause a low platelet count but is
to 23 respectively. His immunosuppresive
not associated with HUS unless in combination
regime consists of prednisolone, ciclosporin and
with a calcineurin inhibitor. As both rapamycin
rapamycin.
and ciclosporin are metabolised by the same

Dr. Khalid Yusuf El-Zohry – Sohag Teaching Hospital (01118391123) Page | 262
El-Zohry MRCP Questions Bank (Part 2) – Medical Masterclass 2010

cytochrome P450 system, the level of both [ Q: 23 ] MasterClass Part2


drugs will be elevated if given within four hours (2010) - Nephrology
of each other. The combination of rapamycin
and ciclosporin is reported to be assocatied In a patient presenting with the nephrotic
with severe HUS in a number of cases. This is syndrome, which one of the following features
due to the fact that not only is there is not a recognised association?
competition for cytochrome P450, but in
addition, animal studies have shown that A. Diabetes mellitus
rapamycin also increases the tissue B. Myeloma
concentration of ciclosporin irrespective of
C. Cigarette smoking
blood levels.
D. HIV infection
In HUS the clotting profile is usually normal and
platelet infusion are contraindicated. E. Systemic lupus erythematosus.

[ Q: 22 ] MasterClass Part2 Answer & Comments


(2010) - Nephrology Correct answer: C
A 58-year-old white man presents to hospital
with renal dysfunction, earache and coughing. Diabetic nephropathy can cause heavy
He has an episode of fresh haemoptysis and a proteinuria, sufficient to cause the nephrotic
chest radiograph shows bilateral diffuse syndrome.
alveolar shadowing. Myeloma can cause AL amyloidosis which is
associated with proteinuria that can be severe
Which of the following tests is likely to be enough to cause the nephrotic syndrome.
diagnostic of the cause of his bilateral diffuse
alveolar shadowing? HIV infection has been associated with a form
of focal segmental glomeruosclerosis which is a
A. High-resolution CT scan
cause of the nephrotic syndrome.
B. Bronchoscopy
Systemic lupus erythematosus can cause
C. Echocardiogram various patterns of glomerulonephritis,
including a membranous nephropathy which is
D. Spirometry
a cause of the nephrotic syndrome.
E. Transfer coefficient (Kco)

[ Q: 24 ] MasterClass Part2
Answer & Comments
(2010) - Nephrology
Correct answer: E
A patient chronically infected with Hepatitis C is
found to have renal disease.
This history is suggestive of pulmonary
haemorrhage in the context of antineutrophil Which TWO processes commonly produce renal
cytoplasmic antibody-associated vasculitis. The disease in this setting?
Kco will be elevated in acute pulmonary
A. Membranoproliferative
haemorrhage.
glomerulonephritis
B. Focal necrotising glomerulonephritis
C. Mixed essential cryoglobulinaemia

Dr. Khalid Yusuf El-Zohry – Sohag Teaching Hospital (01118391123) Page | 263
El-Zohry MRCP Questions Bank (Part 2) – Medical Masterclass 2010

D. Focal segmental glomerulosclerosis What is the diagnosis?


E. AA amyloidosis A. Hypokalaemia.
F. AL amyloidosis B. Hypercalcaemia.
G. Mesangial IgA deposition C. Hypothermia.
H. Interstitial nephritis D. Hypocalcaemia.
I. Acute tubular necrosis E. Hyperkalaemia.
J. Haemolytic uraemic syndrome.
Answer & Comments
Answer & Comments Correct answer: E
Correct answer: AC
This tracing shows the classic appearances of
severe hyperkalaemia. There is peaking of the T
The commonest renal pathology seen in
waves, reduction in P wave voltage and
patients with chronic hepatitis C infection is
broadening of the QRS complex. The rate is
membranoproliferative glomerulonephritis,
slowed and an ominous sine wave pattern is
usually associated with mixed
apparent which usually preceeds asystole.
cryoglobulinaemia (in which the cryoprecipitate
Urgent treatment is required.
commonly contains polyclonal IgG anti-HCV
antibodies and virus). Membranous
nephropathy is seen less frequently. [ Q: 26 ] MasterClass Part2
(2010) - Nephrology
You are asked to review a 56-year-old man who
[ Q: 25 ] MasterClass Part2 has presented to the Emergency Department
(2010) - Nephrology complaining of generalised swelling for the past
2 months. On examination he has marked
The following ECG was obtained from a 46- peripheral oedema. Urine dipstick testing
year-old man. reveals proteinuria 4+ with no other
abnormalities. Blood tests show a normal

Dr. Khalid Yusuf El-Zohry – Sohag Teaching Hospital (01118391123) Page | 264
El-Zohry MRCP Questions Bank (Part 2) – Medical Masterclass 2010

serum urea and creatinine, although his E. Mitral valve prolapse.


albumin is low at 30 g/dL (normal range 35-40).
Answer & Comments
Which of the following management strategies
is most appropriate? Correct answer: B
A. Initiation of steroid therapy for presumed
glomerulonephritis, with subsequent Adult polycystic kidney disease (APKD) is the
review by renal services as an outpatient most common inherited renal disease leading
to end-stage renal failure, accounting for 5-10%
B. Urgent admission for initiation of further
of those receiving renal replacement therapy.
investigations
There are two known loci: PKD1 on
C. Referral to renal services as an outpatient
chromosome 16 accounts for 85% of cases and
D. Initiation of diuretic treatment, with PKD2 on chromosome 4 for 10%.
subsequent review by general
Presentation is with abdominal pain,
practitioner
haematuria, hypertension, urinary tract
E. Initiation of high protein diet, with infection, incidental discovery of an abdominal
subsequent review by renal services as an mass, or as a result of family screening (or
outpatient serendipitous imaging of the kidneys, e.g. by
ultrasound examination ordered for another
Answer & Comments purpose).

Correct answer: C
[ Q: 28 ] MasterClass Part2
(2010) - Nephrology
This patient has nephrotic syndrome. A renal
biopsy is indicated to establish a diagnosis. This During a routine medical check, a 27-year-old
should be done soon, but does not require man who is taking no medication is found to
immediate admission. A loop diuretic (eg have a blood pressure of 180/97 and is found to
furosemide) will alleviate the peripheral have a low serum potassium. His urine contains
oedema in the interim. There is no benefit to be only a trace of protein on dipstick analysis.
obtained from a high protein diet.
Which of the following diagnoses is likely?
[ Q: 27 ] MasterClass Part2 A. Minimal change nephropathy
(2010) - Nephrology B. Primary hyperaldosteronism
An anxious 52-year-old woman has adult C. Mesangiocapillary glomerulonephritis
polycystic kidney disease. She shows you a long
D. Primary hyperparathyroidism
list of things that she says are associated with
this condition. E. Pseudohypoaldosteronism.

Which of the following is NOT a recognized Answer & Comments


association?
Correct answer: B
A. Subarachnoid haemorrhage
B. Cerebellar cysts Aldosterone enhances distal nephron sodium
C. Liver cysts reabsorption and potassium secretion in the
collecting tubules. The result is that if
D. Colonic diverticuli
Dr. Khalid Yusuf El-Zohry – Sohag Teaching Hospital (01118391123) Page | 265
El-Zohry MRCP Questions Bank (Part 2) – Medical Masterclass 2010

aldosterone levels are raised, potassium hypercalcaemia (FHH) due to a mutation in the
depletion is commonly seen. High levels of calcium receptor.
aldosterone can also cause sodium retention
Urinary calcium excretion is increased in
and hypertension. A high blood pressure can
patients with hyperparathyroidism in the
itself cause damage to the glomeruli and a trace
absence of vitamin D deficiency but is low in
of proteinuria is a common finding in
patients with FHH, as is the case here. The
hypertensive patients.
urinary calcium to creatinine ratio is useful in
identifying the 2-5% of patients who have FHH,
[ Q: 29 ] MasterClass Part2 who might otherwise be thought to have
(2010) - Nephrology parathyroid adenomas. Here this diagnosis is
also supported by the probable family history.
A 40-year-old woman who has attended a well-
woman clinic is found to have serum calcium Parathyroid surgery should be avoided in FHH,
2.85 mmol/l and phosphate 0.65 mmol/l. as the hyperparathyroidism will recur, and also
Further investigations show that her intact tends to be well tolerated and stable over time.
parathyroid hormone (PTH) level is 9.0 pmol/l
(normal range 1.1 to 6.8), and the 24-hour [ Q: 30 ] MasterClass Part2
urinary calcium excretion is 0.9 mmol/l (low). (2010) - Nephrology
Plasma creatinine and alkaline phosphatase are
both in the normal range. She says she thinks A 73-year-old lady on peritoneal dialysis
her father was found to have a high blood presents profoundly unwell with abdominal
calcium level. pain.

Which of the following is correct? Which of the following statements are true?

A. She should be screened for a mutation in A. A low haemoglobin clearly indicates a


the multiple endocrine neoplasia - 1 gastrotintestinal bleed.
(MEN-1) gene. B. Acute pancreatitis due to calcium
B. The most likely diagnosis is primary phosphate deposition is the likely
hyperparathyroidism. diagnosis.

C. Parathyroid surgery will be necessary in C. An urgent CRP is the priority.


the future, but is not indicated now. D. The presence of glucose in the peritoneal
D. She probably has an abnormality in the dialysate fluid is a poor prognostic sign.
calcium-sensing receptor. E. Analysis of the peritoneal dialysate fluid
E. Treatment with oral phosphate may be helpful.
supplements should suppress her PTH F. Urgent upper endoscopy is mandatory.
level.
G. A surgical opinion should be sought.

Answer & Comments H. The likely cause is dialysis-induced


constipation.
Correct answer: D
I. Staphylococci in the peritonal dialysate
fluid suggest bowel perforation.
The patient has an inappropriately high level of
PTH, but the reduced urine calcium excretion J. It can safely be assumed that the cause is
suggests that she has familial hypocalcuric dialysis-related peritonitis requiring only
antibiotics.

Dr. Khalid Yusuf El-Zohry – Sohag Teaching Hospital (01118391123) Page | 266
El-Zohry MRCP Questions Bank (Part 2) – Medical Masterclass 2010

Answer & Comments E. Ibuprofen

Correct answer: EG
Answer & Comments

Abdominal pain in a patient on peritoneal Correct answer: B


dialysis can arise from peritoneal dialysis
related peritonitis. This is not usually a severe Trimethoprim interferes with the tubular
disease if patients present early, but it can be. secretion of creatinine and so will increase
Nevertheless, all the other causes of abdominal plasma creatinine in the absence of a change in
pain are also possible in patients on peritoneal GFR. Cimetidine also has this effect, but
dialysis and a surgical opinion should be sought ranitidine does not. It is important to reduce
as well as nephrological input. As with any the dose of allopurinol in renal impairment, and
other cause of abdominal pain, a full metformin is relatively contraindicated. NSAIDs
assessment is necessary and appropriate typically reduce GFR due to altered renal
investigations should be undertaken - it cannot haemodynamics.
be assumed that a CRP or an upper endoscopy
will be useful.
[ Q: 32 ] MasterClass Part2
The peritoneal dialysate may show the (2010) - Nephrology
presence of white cells or organisms, indicating
infection. Usually staphylococci suggest that the A 23-year-old presents to hospital in labour.
peritonitis has arisen from skin organisms, She has had standard antenatal care and no
often resulting from poor patient technique. complications have been detected. The day
Most peritoneal dialysis fluid contains glucose before presentation, she had been seen by a
as the osmotic agent, so finding glucose is of no midwife and all was well. The baby is delivered
diagnostic value. A low haemoglobin may result in the Accident and Emergency Department,
from many causes, including end stage renal but complications arise and she undergoes
disease that is inadequately treated with massive haemorrhage due to placental
erythropoietin. abruption and has a cardiac arrest.
Resuscitation attempts take 24 minutes and she
is transferred to the intensive care unit. A week
[ Q: 31 ] MasterClass Part2
later she is well and recovered, but passing no
(2010) - Nephrology urine. She has passed virtually no urine since
A man aged 50 years has stable chronic renal the delivery.
impairment with a creatinine of 200 μmol/L. At
Which TWO of the following statements are
a routine clinic visit his creatinine is
correct?
unexpectedly 280 μmol/L. He has recently
started on new medication. A. Renal recovery is certain.
B. A renal biopsy must be performed
Which of the following drugs causes an increase
urgently without waiting for other
in serum creatinine in the absence of a change
in glomerular filtration rate (GFR)? investigations.

A. Ranitidine C. A DTPA scan may show diagnostically


helpful changes.
B. Trimethoprim
D. Dialysis is best avoided.
C. Allopurinol
E. Acute renal cortical necrosis is a likely
D. Metformin diagnosis.

Dr. Khalid Yusuf El-Zohry – Sohag Teaching Hospital (01118391123) Page | 267
El-Zohry MRCP Questions Bank (Part 2) – Medical Masterclass 2010

F. The likely cause of her renal impariment is C. Diabetic nephropathy


systemic lupus erythematosus.
D. IgA nephropathy
G. She must not breastfeed.
E. Reflux nephropathy
H. Diabetes mellitus is the probable cause.
I. acute cortical necrosis usually recovers Answer & Comments
over a few weeks. Correct answer: A
J. acute tubular necrosis does not occur
during childbirth. Five cysts on ultrasound at this age satisfy the
diagnostic criteria for polycystic kidney disease,
Answer & Comments and his mother's death would be consistent
with a subarachnoid haemorrhage. Therefore it
Correct answer: CE
is likely that he has autosomal dominant
polycystic kidney disease.
Massive haemorrhage during pregnancy or
childbirth is a well-recognised cause of renal
[ Q: 34 ] MasterClass Part2
cortical necrosis. Unlike acute tubular necrosis,
recovery does not usually occur. There are (2010) - Nephrology
characteristic changes seen on perfusion scans A 22-year-old woman presents with recurrent
with loss of cortical perfusion. Changes are symptoms of urinary frequency and dysuria
usually visible on contrast enhanced CT scans. A associated with cloudy urine. These settle
renal biopsy does show specific cortical rapidly with courses of antibiotics.
changes, but is not always undertaken if the
diagnosis is clear. In a haemodynamically stable Which one of the following would NOT be a
patient, dialysis will be necessary. recognised strategy for trying to prevent
recurrence?
[ Q: 33 ] MasterClass Part2 A. Long-term prophylaxis with trimethoprim
(2010) - Nephrology 100 mg at night

A 28-year-old man is found to have dipstick B. Regular drinking of cranberry juice


haematuria at a medical performed for life C. Practice of double micturition
insurance purposes. His BP is 118/75 mmHg.
D. Advice to void the bladder before and
There is no proteinuria and his plasma
after sexual activity
creatinine is normal. There is no other
significant history, except that his mother had E. Use a shower rather than a bath.
urinary tract infections and died of a stroke
aged 35 years, and his father has maturity onset Answer & Comments
diabetes mellitus and became deaf at the age of
52 years. A renal ultrasound shows three cysts Correct answer: E
in his left kidney and two cysts in his right
kidney. Cranberry juice has antiseptic properties and
has been shown in a controlled trial to be
What is the most likely diagnosis? effective at reducing risk of urinary tract
A. Adult polycystic kidney disease infection.

B. Alport's syndrome The practice of double micturition endeavours


to make sure that the bladder is completely

Dr. Khalid Yusuf El-Zohry – Sohag Teaching Hospital (01118391123) Page | 268
El-Zohry MRCP Questions Bank (Part 2) – Medical Masterclass 2010

empty after voiding, making it more difficult for I. A bone biopsy would most likely show
infection to take hold. The woman should be adynamic bone (low bone turnover).
advised to empty her bladder, wait for a further
J. Ectopic calcification is not likely in this
60 seconds on the toilet and then try to void
scenario.
again. Some find that pressing on their
suprapubic region will help them to express
Answer & Comments
more urine.
Correct answer: HI
[ Q: 35 ] MasterClass Part2
(2010) - Nephrology This is most likely to be excessive calcium intake
in the form of the elemental calcium in the
A 24-year-old white woman with end stage phosphate binders, in the context of low
renal disease (ESRD) secondary to IgA turnover bone disease, leading to over-
nephropathy started dialysis 3 months suppression of parathyroid gland activity,
previously with plasma calcium of 2.61 mmol/l, hypercalcaemia and ectopic calcification.
phosphate of 2.2 mmol/l and parathyroid Renagel would be a valid alternative (as would
hormone (PTH) of 89 pg/ml. She was aluminium). Vitamin D analogues are not useful
commenced on oral phosphate binders in this context. The calcium will fall promptly if
(calcium carbonate) and vitamin D. Three the vitamin D and calcium salts are stopped, so
months later the following blood results were with an asymptomatic patient no further action
obtained, with the patient in good general is needed, though the plasma calcium should be
health with no symptoms: calcium 3.02 mmol/l rechecked in a few days to ensure that it is
(corrected for plasma albumin), phosphate 0.9 falling.
mmol/l, intact PTH 4 pg/ml.
Sarcoidosis is a very rare diagnosis in a dialysis
Which TWO of the following statements are patient and is much commoner in black
correct? patients.
A. A surgical parathyroidectomy is needed
urgently. [ Q: 36 ] MasterClass Part2
B. Sarcoidosis is the only reasonable
(2010) - Nephrology
explanation.
The following are NOT used for the estimation
C. A bone biopsy would show high-turnover of renal function:
bone disease.
A. Serum creatinine
D. Calcitonin must be given.
B. Serum cystatin C
E. Palmidronate must be given.
C. Creatinine clearance
F. This is most likely not an iatrogenic
D. Ethylenediaminetetraacetic acid (EDTA)
problem.
clearance
G. Discontinuing calcium carbonate and
E. Urinary albumin/creatinine ratio.
substituting calcium acetate is all that is
needed.
Answer & Comments
H. Sevelamer (Renagel) would be
appropriate here. Correct answer: E

Dr. Khalid Yusuf El-Zohry – Sohag Teaching Hospital (01118391123) Page | 269
El-Zohry MRCP Questions Bank (Part 2) – Medical Masterclass 2010

Serum creatinine is the most commonly used [ Q: 38 ] MasterClass Part2


screening test for renal function. Cystatin C is (2010) - Nephrology
currently being evaluated and may prove to be
more accurate in mild renal impairment. A 28-year-old woman was referred for the
Creatinine clearance is used as a surrogate test investigation of hypertension. Routine
for measuring the GFR. The clearance of the examination demonstrated that she had absent
radioisotope EDTA however is the most pulses in the left arm. She was a non-smoker.
accurate marker of GFR in routine clinical An arch aortogram was performed (see image).
practice. The albumin / creatinine ratio is a
method of quantifying urinary protein losses
and not an estimate of renal function.

[ Q: 37 ] MasterClass Part2
(2010) - Nephrology
A 19-year-old woman, who is normally well,
attends her local emergency department
complaining of a burning sensation on passing
urine. She also mentions that the urine looks
cloudy and has a foul smell.

Which of the following are true?


A. Urinary tract infection with a fungus is
the likely explanation
B. Urinary tract stones are probably present
C. An urgent potassium level should be
obtained
D. Urinary tract infection with a
streptococcal species is a common cause
of this clinical presentation
E. An underlying anatomical abnormality is
not usually present in women of this age
with this clinical presentation

Answer & Comments

Correct answer: E
Which of the following statements is correct?
This clinical presentation is common and is
A. She has coarctation of the aorta which is
typical of lower urinary tract infection.
the likely cause of her hypertension.
Common organisms include E. coli, Proteus and
Klebsiella species. Fungal urinary tract infection B. The selective picture shows a severe left
is very rare and in younger women with urinary subclavian stenosis most probably due to
tract infection, the presence of stones or an atherosclerosis
anatomical abnormality would be unusual.

Dr. Khalid Yusuf El-Zohry – Sohag Teaching Hospital (01118391123) Page | 270
El-Zohry MRCP Questions Bank (Part 2) – Medical Masterclass 2010

C. The study is normal and she most likely Select the most appropriate statement.
has essential hypertension
A. Phosphate restriction in the diet should
D. The selective picture shows a severe left be reinforced, a change of phosphate
subclavian stenosis most probably due to binder may be necessary and alfacalcidol
Takayasu's arteritis should be reduced or stopped.
E. She has hypertension due to renovascular B. The dialysate calcium should be lowered
disease. to return the calcium to the normal
range.
Answer & Comments C. The calcium acetate dose should be
reduced to return the calcium to the
Correct answer: D
normal range.

Hypertension in the young always needs D. The patient has autonomous


investigation. The physical findings in this hyperparathyroidism.
woman were highly suggestive of a large vessel E. These values are satisfactory for a
disease process. haemodialysis patient.
The selective angio picture does not show
coarctation but does reveal critical stenosis of Answer & Comments
the left subclavian artery, which is most likely to
Correct answer: A
be due to Takayasu's arteritis in this case.
Atherosclerosis would be unusual in a young
non-smoker. The calcium x phosphate product is high putting
the patient at risk of metastatic calcification.
Hypertension is probably driven by renal
Lowering the phosphate (rather than the
ischaemia due to either direct renal vessel
calcium) is the most important measure, as the
involvement or indirectly by aortic narrowing
elevated phosphate stimulates parathyroid
(not seen on these pictures). Raised erythrocyte
proliferation and PTH secretion. This is best
sedimentation rate (ESR) is typical of the active
done by reinforcing dietary restriction and
inflammatory phase of this illness; in Japan
ensuring that the patient is recieving adequate
there have been reports of linkage with
dialysis. The vitamin D analogue will be
tuberculosis - steroids and other
increasing absorption of calcium and phosphate
immunosuppressants can be effective.
so this should be reduced or stopped; ideally
the patient will remain on a small dose as this
[ Q: 39 ] MasterClass Part2 helps to stop parthyroid proliferation.
(2010) - Nephrology If these changes do not return the calcium and
A 30-year-old man, a haemodilaysis patient phosphate to satisfactory values, the patient
with diabetic nephropathy, has the following should be changed to a phosphate binder which
blood results: calcium 2.9 mmol/l, phosphate does not contain calcium. Aluminium based
2.5 mmol/l. Alkaline phosphatase and albumen compounds are best avoided because of the
are in the normal range and PTH is at the lower risk of toxicity so sevelamer would be an
end of the normal laboratory range. The appropriate (but expensive) choice.
patient's current medication includes
alfacalcidol each day and calcium acetate as a
phosphate binder before each meal.

Dr. Khalid Yusuf El-Zohry – Sohag Teaching Hospital (01118391123) Page | 271
El-Zohry MRCP Questions Bank (Part 2) – Medical Masterclass 2010

[ Q: 40 ] MasterClass Part2 [ Q: 41 ] MasterClass Part2


(2010) - Nephrology (2010) - Nephrology
A 32-year-old man presents to his GP with a A 70-year-old man has been referred by his GP
non-specific history of malaise. He has no with plasma creatinine of 500 micromol/l. He
history of diarrhoea. Initial investigations feels reasonably well, but his appetite has been
identify a significant anaemia with diminished for the last few days. On
fragmentation on the blood film and low examination you find a mass arising from the
platelets. A clotting screen is normal. His pelvis, which you think is consistent with a
biochemistry identifies significant renal distended bladder. On direct questioning he
dysfunction with a serum creatinine of describes a decrease in his urine flow rate over
260umol/l. BP was recorded at 170/100. the last few years, and a more recent feeling of
incomplete bladder emptying. However, he
What is the most likely diagnosis? says that he thinks he has been passing normal
A. Thrombotic thrombocytopenic purpura amounts of urine.
(TTP)
Which of the following statements do you agree
B. Haemolytic uraemic syndrome with?
C. Malignant hypertension A. If the mass is a distended bladder, the
D. Disseminated intravascular coagulation most likely underlying diagnosis is an
infiltrating carcinoma of the bladder
E. Systemic vasculitis.
B. An intravenous urogram is indicated to
determine if there is obstruction
Answer & Comments
anywhere in the urinary tract
Correct answer: B
C. The fact that he reports passing normal
volumes of urine makes obstructive
This is a classic presentation of non-diarrhoeal uropathy unlikely to be the cause of his
haemolytic uraemic syndrome (D- HUS) or renal impairment
atypical HUS. This may be sporadic or familial.
D. His decreased appetite suggests that he
In sporadic cases association with HIV,
may be uraemic
malignancy, systemic lupus erythematous (SLE)
and some drugs (e.g. cyclosporin) has been E. If he has benign prostatic hypertrophy he
reported. In some familial cases mutations have should have a trans urethral resection
been detected in Factor H which regulates within the next few days
alternative pathway activation of compliment.
F. The first treatment priority is likely to be
TTP produces a similar picture but usually also
insertion of a large bore venous catheter
has evidence of neurological involvement.
for haemodialysis
There is also evidence that the mechanism
underlying the disease is different with reduced G. If the renal pelvis is dilated on both sides
activity of a von Willebrand factor cleaving his creatinine is unlikely to recover to
protease ADAMTS13. Treatment of sporadic normal following intervention
HUS is with plasma exchange and fresh frozen H. He should have an ultrasound scan to
plasma. The renal outlook is relatively poor and confirm that the mass is a distended
mortality around 10%. bladder, and then a suprapubic catheter
should be inserted

Dr. Khalid Yusuf El-Zohry – Sohag Teaching Hospital (01118391123) Page | 272
El-Zohry MRCP Questions Bank (Part 2) – Medical Masterclass 2010

I. If bladder outflow is obstructed he is likely Answer & Comments


to need intravenous fluids once the
Correct answer: C
obstruction is relieved
J. His renal failure is almost certainly chronic Nephrotic syndrome predisposes to renal vein
and he will almost certainly need long-
thrombosis, which typically presents with flank
term dialysis pain, haematuria and a rise in creatinine.

Answer & Comments


[ Q: 43 ] MasterClass Part2
Correct answer: DI (2010) - Nephrology
A 23-year-old man presents with a rash on his
The key investigation is ultrasound of the renal
legs. Stick testing of his urine reveals
tract. If there is good preservation of the renal
proteinuria + and haematuria +++.
cortex the outlook for recovery of renal
function is good. IVU will give poor images in What is the most likely diagnosis?
the presence of renal failure and the contrast
may contribute to renal injury; it is A. Henoch Schonlein purpura
contraindicated. B. Mixed essential cryoglobulinaemia
A history of passing normal volumes of urine C. Minimal change nephropathy
does not rule out obstructive uropathy.
D. IgA nephropathy
Following catheterisation of a chronically
obstructed bladder polyuria is commonly E. Membranous glomerulonephritis.
problematic, and close monitoring with
intravenous fluid replacement is usually Answer & Comments
necessary.
Correct answer: A

[ Q: 42 ] MasterClass Part2
Mixed essential cryoglobulinaemia will often
(2010) - Nephrology present with palpable purpura on the legs and
A 47-year-old man with known nephrotic nephritis, but is an uncommon disease of older
syndrome due to focal segmental patients. If the man did not have a rash, then
glomerulosclerosis attends hospital with acute IgA nephropathy would the most probable
flank pain, proteinuria 3+ and haematuria 3+ on cause of his urinary findings.
urinalysis, and a rise in creatinine from 115 to
167 μmol/L. He is taking prednisolone, [ Q: 44 ] MasterClass Part2
ciclosporin, furosemide and ramipril. (2010) - Nephrology
What is the most likely underlying cause of his You are providing a seminar to patients
presentation? approaching the need for renal replacement. A
A. Acute interstitial nephritis group of patients contemplating starting
continuous ambulatory peritoneal dialysis
B. Spontaneous bacterial peritonitis (CAPD) ask you about medical contraindications
C. Renal vein thrombosis to this modality.

D. Ciclosporin nephrotoxicity Which of the following is not a relative


E. Renal artery stenosis contraindication to CAPD treatment?

Dr. Khalid Yusuf El-Zohry – Sohag Teaching Hospital (01118391123) Page | 273
El-Zohry MRCP Questions Bank (Part 2) – Medical Masterclass 2010

A. Previous perforated diverticular disease peripheral oedema. Blood pressure is 135/85


and sigmoid colectomy mm Hg. HbA1C is 9.5%.
B. Bilateral inguinal herniae
In order to preserve native renal function, which
C. Diabetes mellitus of the following do you consider most
important?
D. Severe chronic obstructive pulmonary
disease A. Strict restriction of dietary phosphate
E. Arthritis mutilans. B. Commence a loop diuretic
C. Commence an HMG CoA reductase
Answer & Comments inhibitor
Correct answer: C D. Commence an angiotensin converting
enzyme inhibitor
Previous pelvic surgery reduces the likelihood E. Greatly improved diabetic control aiming
of successful peritoneal dialysis, adhesions for HbA1c < 7.2 %.
often making catheter placement difficult and
reducing the peritoneal surface available for
Answer & Comments
dialysis.
Correct answer: D
Inguinal herniae rapidly fill with peritoneal
dialysate causing patient discomfort and
inefficient dialysis. There is clear evidence that ACE inhibition in
diabetics delays progression of renal failure
Peritoneal dialysis is a good treatment for even if they are normotensive. A loop diuretic
patients with diabetes mellitus although the will improve the peripheral oedema.
glucose load in the dialysate will necessitate a Cholesterol lowering is appropriate given this
change in insulin dosage, which can be patient's vascular risk factors, but there is little
administered intraperitoneally. evidence that this alters the progression of
Splinting of the diaphragm by a large volume of renal disease
intraperitoneal fluid often exacerbates chronic
obstructive pulmonary disease (COPD), and this [ Q: 46 ] MasterClass Part2
should be considered a relative, but not
(2010) - Nephrology
absolute contraindication.
Effective peritoneal dialysis requires a degree of A 55-year-old man with long-standing type II
manual dexterity and would not be easy for a diabetes and intermittent claudication has
patient with arthritis mutilans. recently started on continuous ambulatory
peritoneal dialysis (CAPD) for end stage renal
failure (ESRF) which is presumed to be due to
[ Q: 45 ] MasterClass Part2 diabetic nephropathy. He is blood group A, his
(2010) - Nephrology wife is blood group O, and they have two
children. He is interested in the possibility of
A 40-year-old man with a 25-year history of
transplantation.
diabetes treated with insulin is found to have
2g proteinuria per 24 hours, haemoglobin 10.5 Which of the following is true?
g/dl, albumen 30g/l, calcium 2.0 mmol/l,
phosphate 1.8 mmol/l, creatine 250 µmol/l, A. Risk benefit analysis shows that diabetics
cholesterol 7 mmol/l. He has significant with end stage renal failure are better
treated with dialysis than transplantation.

Dr. Khalid Yusuf El-Zohry – Sohag Teaching Hospital (01118391123) Page | 274
El-Zohry MRCP Questions Bank (Part 2) – Medical Masterclass 2010

B. In the UK, wife to husband transplants developed 'trash feet' and acute disturbance of
are not permitted under current renal function.
legislation.
Which of the following statements are valid?
C. An exercise ECG should be performed to
determine if he needs coronary A. A renal biopsy is mandatory.
angiography prior to transplantation. B. A renal angiogram would be contra-
D. Recent evidence suggests that this indicated.
patient would benefit from combined C. Peripheral blood eosinophilia would be
kidney and pancreas transplantation. surprising in this context.
E. If a transplant is performed the CAPD D. Both renal arteries must be diseased.
catheter would usually be left in situ at
E. Anti-coagulation is important to preserve
the time of the operation.
renal function.

Answer & Comments F. Statin therapy is desirable.

Correct answer: E G. Irreversible renal failure is very likely.


H. Acute haemorrhage into aorto-renal
Although diabetics with end stage renal disease complex atherosclerotic plaque is likely as
have a relatively poor survival rate following the precipitant.
transplantation, their survival on dialysis is I. Primary vasculitis would explain the
dismal; risk benefit analysis shows that this myocardial infarction and the renal
group may benefit most from transplantation failure.
compared to dialysis.
J. Steroids would be important therapy in
Wife-to-husband transplants (and vice-versa) this context.
are legal in the UK and the results are generally
excellent.
Answer & Comments
An exercise ECG is not sufficient evaluation
Correct answer: FH
here, especially as the intermittent claudication
may limit the test.
It is likely there is generalized atherosclerosis
This patient would need coronary angiography,
and the aorta is affected by ulcerated plaque.
and iliac angiography, prior to transplantation.
The thrombolytic agent, together with any anti-
Pancreatic transplantation is desirable in type I
platelet drugs administered, make
diabetics. The PD catheter is usually left in situ -
haemorrhage into this complex atherosclerotic
in the event that the graft does not work for a
lesion more likely, and acute thrombo-
period the patient can then be dialysed easily.
embolism will then ensue, distally to the feet
The catheter is usually removed 3 months after
and into the mesenteric/ renal arterial tree.
a successful transplant.
Eosinophilia is sometimes seen, as is
consumption of complement, raised
[ Q: 47 ] MasterClass Part2
inflammatory markers, and acute renal
(2010) - Nephrology dysfunction.
A 72-year-old man who presented with an Further anti-coagulation can be disastrous.
acute myocardial infarction 7 days previously,
and who had received tPA as thrombolysis,

Dr. Khalid Yusuf El-Zohry – Sohag Teaching Hospital (01118391123) Page | 275
El-Zohry MRCP Questions Bank (Part 2) – Medical Masterclass 2010

Aggressive statin therapy is warranted for the tubule and the fall in angiotensin II reduces
heart and the kidneys. sodium reabsorption in the proximal tubule.
Conversely, the fall in aldosterone reduces
[ Q: 48 ] MasterClass Part2 distal tubular potassium secretion, so
potassium levels can rise.
(2010) - Nephrology
A 53-year-old man is found to have a blood [ Q: 49 ] MasterClass Part2
pressure of 185/95 mmHg at a routine medical
(2010) - Nephrology
check. His general practitioner checks his
electrolytes and renal function, which are An obese 52-year-old Asian man with a 17-year
normal, and then starts him on an angiotensin- history of type 2 diabetes presents to the
converting enzyme inhibitor. A week later, the outpatient clinic with a serum creatinine of 231
general practitioner repeats these blood tests micromol/L and proteinuria.
and arranges to see the patient again.
Which of the following would be true of this
Which one of the following is NOT a possible patient?
direct consequence of starting an angiotensin-
A. The absence of retinopathy excludes
converting enzyme (ACE) inhibitor ?
diabetic nephropathy.
A. A rise in plasma potassium
B. The presence of nephrotic syndrome in
B. A rise in plasma creatinine this man excludes diabetic nephropathy.
C. A rise in sodium reabsorption by the renal C. He may benefit from combined ACE
tubules inhibitor(ACEI) / Angiotensin receptor
blocker(ARB) therapy.
D. A fall in glomerular filtration rate
D. Statins would improve his risk of
E. A fall in aldosterone production.
cardiovascular death irrespective of his
serum cholesterol.
Answer & Comments
E. Aspirin would be contraindicated because
Correct answer: C of the risk of bleeding.
F. Given his obesity, metformin would be
Angiotensin II has a number of actions on the
the oral agent of choice.
kidney. It is a vasoconstrictor, but its effect on
the efferent arterioles is greater than that on G. Should he require dialysis in the future,
the afferent arterioles, resulting in an increase peritoneal dialysis would be
in glomerular filtration rate. It also promotes contraindicated because of the glucose
sodium reabsorption in the proximal tubule by load in continuous ambulatory peritoneal
an action on sodium/hydrogen exchange at this dialysis (CAPD) fluid.
site. Angiotensin II stimulates aldosterone H. Kidney pancreas transplantation should
release from the adrenal gland. Aldosterone be considered if he develops endstage
promotes distal tubular sodium reabsorption renal failure.
and potassium secretion.
I. His sulphonylurea requirements are likely
ACE inhibitors block the effects of angiotensin II to rise as his renal failure progresses.
by inhibiting its formation. Therefore,
glomerular filtration rate often falls slightly and J. The absence of neuropathy excludes
aldosterone levels fall. The fall in aldosterone diabetic nephropathy.
reduces sodium reabsorption in the distal
Dr. Khalid Yusuf El-Zohry – Sohag Teaching Hospital (01118391123) Page | 276
El-Zohry MRCP Questions Bank (Part 2) – Medical Masterclass 2010

Answer & Comments Which of the following statements is true?

Correct answer: CD A. He should be commenced on alfacalcidol


0.5 mcg od, to correct deficiency of active
vitamin D and suppress parathyroid
There is no relationship between diabetic
hormone (PTH).
nephropathy and peripheral neuropathy.
B. An appropriate treatment would be
In type 2 diabetes more than 50% of patients
calcium acetate 1g bd to be taken with
with nephropathy have no retinopathy.
meals.
The urine protein losses in type 2 diabetes may
C. Aluminium hydroxide should be the first
be result in a nephrotic syndrome.
choice if a phosphate binder is
There is growing evidence that combined prescribed.
ACEI/ARB therapy may offer significant benefits
D. Parathyroidectomy should be considered,
over that of the individual agents on their own.
especially if he is keen on having a renal
Recent studies have also shown that statins
transplant in the future.
improve cardiovascular outcomes in high risk
patients including diabetics irrespective of E. Sevelamer (Renagel) should definitely be
serum cholesterol. There is no contraindication used here rather than other phosphate
to the use of aspirin, however. Impaired renal binders.
function is a contraindication to the use of
metformin as it places the patients at increased Answer & Comments
risk of lactic acidosis. Peritoneal dialysis is not
contraindicated in diabetes and in fact is often Correct answer: B
the treatment of choice given their
cardiovascular comorbidity. Hydroxylated vitamin D does suppress PTH
expression, and alfacalcidol should be
Kidney pancreas transplantation is not carried
considered for prophylaxis against renal bone
out in type 2 diabetes as it is a disease
disease and progressive hyperparathyroidism.
predominantly of insulin resistance.
However, the patient's phosphate level is
Often oral agents are needed in reduced already elevated, and vitamin D
dosages as renal failure progresses and patients supplementation will tend to increase this
previously requiring oral agents can frequently further. Even if the phosphate level were not
be maintained on diet alone. elevated this would be an inappropriately high
starting dose.
[ Q: 50 ] MasterClass Part2 Parathyroidectomy is not indicated here - the
(2010) - Nephrology patient only has a mildly elevated level of PTH
which is likely to improve with better
A 45-year-old man with polycystic kidney
phosphate control. There would also be a risk
disease and plasma creatinine 300µmol/l is
of adynamic bone disease.
found to have potassium 5.1 mmol/l, calcium
2.25 mmol/l, albumin 40 g/l, phosphate 1.65 Aluminium-containing phosphate binders carry
mmol/l, and PTH 20 pmol/l (NR 1.1-6.8). He has the risk of aluminium accumulation and CNS
already seen a specialist dietician for advice effects. Sevelamer is only clearly indicated if
concerning potassium and phosphate intake. calcium-containing binders cannot be used
He is currently taking an angiotensin-converting because the calcium level (or calcium-
enzyme (ACE)-inhibitor for hypertension and a phosphate product) are undesirably high.
proton pump inhibitor for reflux oesophagitis. Calcium acetate is a very reasonable choice

Dr. Khalid Yusuf El-Zohry – Sohag Teaching Hospital (01118391123) Page | 277
El-Zohry MRCP Questions Bank (Part 2) – Medical Masterclass 2010

here. It should be taken with (or just before) high ESR are expected manifestations of
meals and may offer advantages over calcium multiple myeloma (a common cause of renal
carbonate, especially in patients with reduced impairment) and retroperitoneal fibrosis (much
gastric acidity. less common, but the most likely of the
remaining diagnoses given this scenario).
[ Q: 51 ] MasterClass Part2
(2010) - Nephrology [ Q: 52 ] MasterClass Part2
(2010) - Nephrology
(2) A 70-year-old man visits his GP with general
tiredness, some weight loss and backache, A 63-year-old man who has received a renal
which he has noticed for 4 months. Physical transplant for polycystic kidney disease 5 years
examination is unremarkable. Laboratory previously presents with significantly worsening
investigations show Hb 9.5 g/dl (normochromic, renal function. He is immunosuppressed with
normocytic), creatinine 250 µmol/l and ESR is ciclosporin and mycophenolate mofetil. His
90 mm/hr. Urinalysis is negative for blood. creatinine has risen from 120 to 300 μmol/L
(normal range 90-115). Recently he attended
What are the TWO most likely underlying his GP, who treated him with a course of
diagnoses? clarithromycin for a cough.
A. Chronic renal failure due to reflux
What is the most likely cause for his
nephropathy
deterioration in renal function?
B. Rapidly progressive glomerulonephritis
A. Acute allergic interstitial nephritis
related to anti-glomerular basement
membrane antibodies B. Ciclosporin toxicity
C. Analgesic nephropathy C. Dehydration
D. Polycystic kidney disease D. Pulmonary-renal syndrome associated
with circulating antibodies to glomerular
E. Membranous nephropathy with an
basement membrane
underlying malignancy
E. Pyelonephritis
F. Obstructive uropathy related to benign
prostatic hypertrophy
Answer & Comments
G. Systemic vasculitis
Correct answer: B
H. Obstructive uropathy due to
retroperitoneal fibrosis
[ Q: 53 ] MasterClass Part2
I. Multiple myeloma
(2010) - Nephrology
J. Mesangiocapillary glomerulonephritis.
A 37-year-old woman who has previously had a
Answer & Comments renal biopsy showing IgA nephropathy is
reviewed in the outpatient department. Urine
Correct answer: HI dipstick analysis shows 3+ protein and 1+ blood.
BP is recorded at 142/90. She is currently taking
This degree of anaemia and the substantially no medication. Recent blood tests show a
elevated ESR are not likely to be secondary serum creatinine of 172 micromol/l. One year
consequences of the renal impairment. Weight previously it was 148.
loss, normochromic anaemia, backache and

Dr. Khalid Yusuf El-Zohry – Sohag Teaching Hospital (01118391123) Page | 278
El-Zohry MRCP Questions Bank (Part 2) – Medical Masterclass 2010

Select TWO of the following statements that lower. At this GFR she may well have early
you agree with: metabolic evidence of renal bone disease and
A. An ACE Inhibitor should be avoided as she hyperparathyroidism. She will need adequate
has renal impairment information about her condition and frequent
follow up will be required to ensure optimal
B. Only a small minority of patients with this therapy (perhaps every three to four months),
condition progress to end stage kidney and provide appropriate support and education
failure so she can be firmly reassured if her renal function continues to deteriorate.
C. Her blood pressure does not require
treatment at present. It should be [ Q: 54 ] MasterClass Part2
repeated at monthly intervals (2010) - Nephrology
D. Cyclophosphamide 2mg/kg and
A 27-year-old woman with known chronic renal
prednisolone 1mg/kg should be started
disease due to reflux nephropathy (creatinine
E. She should see a dietician for advice on a 200 micromol/l) is referred to the clinic for
low carbohydrate diet advice as she has unexpectedly become
F. Calcium, phosphate and PTH should all be pregnant.
measured as she may have secondary
Which of the following statements is correct?
hyperparathyroidism
A. A termination of pregnancy should be
G. If she develops renal failure,
strongly advised.
transplantation will be ruled out because
of the risk of recurrence in the transplant B. She has about a 90% chance of a
successful fetal outcome.
H. She should be booked for a fistula
formation in preparation for dialysis C. An ACE inhibitor will be the best drug to
treat any hypertension.
I. BP should be reduced to 125/75 or lower
D. Asymptomatic urinary infections should
J. Annual follow up in the renal clinic is
be ignored.
appropriate
E. There are unlikely to be any maternal
Answer & Comments complications.

Correct answer: FI Answer & Comments

Although IgA nephropathy only progresses to Correct answer: B


end stage renal failure in about 10% of cases,
this patient is in a high-risk category. This is Pregnancy in patients with significant renal
because she has high blood pressure (especially impairment is associated with increased
for a woman of her age), significant proteinuria, maternal and fetal complications. With this
renal impairment and deteriorating renal degree of impairment, most will develop
function. Her GFR is 30-40 mls/min/1.73m2, hypertension during pregnancy with an
based on the MDRD formula. The proven associated increase risk of pre-eclampsia (30%),
intervention is blood pressure control, pre-term delivery and potential deterioration in
preferentially using an angiotensin- converting renal function (20%). The maternal renal
enzyme (ACE) inhibitor although it is likely she outlook is often worse in patients with reflux
will require more than one agent. The target nephropathy and urinary infection should be
blood pressure should be stringent, 125/75 or screened for and treated. Despite this the fetal

Dr. Khalid Yusuf El-Zohry – Sohag Teaching Hospital (01118391123) Page | 279
El-Zohry MRCP Questions Bank (Part 2) – Medical Masterclass 2010

outcome is usually good with less than 10% situation as there is a small risk of uncontrolled
fetal loss. haemorrhage which could lead to
nephrectomy. However, renal patholgy in this
ACE-inhibitors are contraindicated.
clinical setting could be very useful and may
Unplanned doesn't mean unwanted, so our goal justify the extra risk. Small dense kidneys
is to inform and then support the mother's indicate chronic renal failure, a biopsy is
decision. difficult, risky and is very unlikely to provide
diagnostic information. Likewise
[ Q: 55 ] MasterClass Part2 hydronephrosis on ultrasound supports a
(2010) - Nephrology diagnosis of obstructive nephropathy which
needs radiological or urological intervention,
A 46-year-old man presents with unexplained not a biopsy.
dialysis-dependent renal failure. You decide a
renal biopsy could be useful.
[ Q: 56 ] MasterClass Part2
Which of the following would be an absolute (2010) - Nephrology
contraindication to carrying out this
An 18-year-old woman had macroscopic
investigation?
haematuria at the time that she had a sore
A. A positive cANCA throat. Following this she is found to have
B. An Hb of 9g/dl persistent microscopic haematuria, 0.1g/24-
hour proteinuria and a normal plasma
C. Ultrasound demonstrates two small 6 cm creatinine. Blood pressure is 115/64. A renal
dense kidneys with no corticomedullary biopsy shows mesangial deposition of IgA on
differentiation immunofluorescence. The tubules and
D. A serum bicarbonate of 15 interstitium are normal.

E. A serum urea of 56 mmol/l Which of the following statements is true?


F. A calcium of 2.02 and Phospate 2.3 A. An angiotensin-converting enzyme (ACE)
G. An abnormal clotting screen inhibitor should be given since it is known
to reduce the risk of progressive renal
H. A low C4
failure in this setting.
I. Ultrasound demonstrates a single 12 cm
B. There is approximately a 30% chance of
right kidney
end stage renal failure over the next 5
J. Ultrasound demonstrates bilateral years.
hydronephrosis.
C. It is important that the patient's siblings
are screened by testing their urine for
Answer & Comments blood.
Correct answer: CJ D. The prognosis is relatively good
compared to other patients with IgA
Unexplained renal failure is a good indication nephropathy.
for renal biopsy and often influences therapy. E. Circulating levels of C3 are likely to be
Although abnormal clotting increases the risk of reduced.
bleeding this can be corrected prior to biopsy,
so would not absolutely rule out a biopsy.
Likewise, a single kidney represents a high risk

Dr. Khalid Yusuf El-Zohry – Sohag Teaching Hospital (01118391123) Page | 280
El-Zohry MRCP Questions Bank (Part 2) – Medical Masterclass 2010

Answer & Comments I. Primary hyperventilation

Correct answer: D J. Opioid overdose

Episodes of macroscopic haematuria, normal Answer & Comments


renal function, female sex, normal blood
Correct answer: CF
pressure, minimal proteinuria and normal
tubulointerstitial morphology all place this
patient in a good prognostic category. This patient has severe metabolic acidosis with
partial compensation through hyperventilation.
Although there is probably a genetic The anion gap is increased. The acidosis is more
component to IgA nephropathy it is very marked than expected for this level of renal
unusual for relatives to be affected. dysfunction. Important possible explanations
include diabetic ketoacidosis and ethylene
[ Q: 57 ] MasterClass Part2 glycol intoxication. Another possibility might be
(2010) - Nephrology a salicylate overdose.
Bulimia and laxative abuse are characterised by
A 36-year-old woman is sent to the Emergency
alkalosis. If the patient has severe chronic
Department because she is disorientated and
obstructive pulmonary disease, this might result
drowsy. Blood tests give the following results:
in hypoxia, CO2 retention and respiratory
Na+ 133 mmol/L (normal range 135-145), K+ 4.6
acidosis, but not the picture seen here.
mmol/L (normal range 3.8-5.2), urea 28 mmol/L
Acetazolamide can rarely cause metabolic
(normal range 4-7), creatinine 470 μmol/L
acidosis, but this is much less likely.
(normal range 90-115), chloride 97 mmol/L
(normal range 95-105) and bicarbonate 16
mmol/L (normal range 22-28). An arterial blood [ Q: 58 ] MasterClass Part2
sample gives the following results: pH 7.1 (2010) - Nephrology
(normal range 7.36-7.44), Po2 92 mmHg
A 27-year-old primigravida with no past medical
(normal range 80-95), Pco2 19 mmHg (normal
range 36-44) and bicarbonate 7 mEq/L (normal history is found to have glycosuria on urinalysis
range 22-26). at 24-weeks gestation.

Which TWO of the following diagnoses do you In this setting which TWO of the following
statements certainly apply?
think are most likely in this patient?
A. Glycosuria indicates the presence of
A. Laxative abuse
intrinsic renal disease.
B. Previous bowel surgery with ureteric
B. Methyldopa would be an appropriate
diversion
drug to use to treat hypertension at this
C. Ethylene glycol intoxication gestational stage.
D. Multiple myeloma with type 2 renal C. Glycosuria indicates the presence of
tubular acidosis gestational diabetes.
E. Bulimia nervosa D. Her plasma creatinine is likely to be
F. Diabetic ketoacidosis higher than before her pregnancy.

G. Severe chronic obstructive pulmonary E. Unless therapy is started, glycosuria is


disease likely to lead to a sub-optimal obstetric
outcome.
H. Use of acetazolamide for glaucoma
Dr. Khalid Yusuf El-Zohry – Sohag Teaching Hospital (01118391123) Page | 281
El-Zohry MRCP Questions Bank (Part 2) – Medical Masterclass 2010

F. Glycosuria indicates the presence of a milky slurry. Plain films show this appearance
urinary tract infection. (see image).
G. She is likely to have microvascular
complications of diabetes mellitus.
H. Glycosuria is a false positive result arising
from the presence of beta hydroxy
butyrate in her urine.
I. Finding her blood pressure to be
consistently higher than at booking
should give cause for concern.
J. Her HbA1C will be abnormal.

Answer & Comments


What is the most likely underlying cause of this
Correct answer: BI problem?
A. Poor control of calcium and phosphate
In the mid-trimester of pregnancy GFR normally
rises, resulting in a fall in plasma creatinine B. Primary hyperparathyroidism
level. The elevated GFR may lead to the
C. Osteolytic metastasis
capacity of the tubules to reabsorb filtered
glucose being exceeded and consequent D. Myeloma
glycosuria. E. Septic arthritis
Blood pressure normally falls in the mid-
trimester so an elevation is a cause for concern. Answer & Comments
Drugs regarded as safe for the treatment of
Correct answer: A
hypertension in pregnancy include methyldopa,
amlodipine / nifedipine and hydralazine.
This is an example of tumoural calcinosis.
Glycosuria may predispose to urinary infection,
Excess calcium and phosphate (which are at
but this would be indicated by the presence of
best only metastable in plasma) precipitate in
leukocytes and nitrites on urinalysis, not
either crystalline or amorphous forms in the
glycosuria. Ketones do not cross-react with the
skin, joints, blood vessels and other places. This
glucose test strip on urinalysis. Glycosuria may,
ectopic calcification can cause local effects as
of course, occur because of intrinsic renal
here, including pain (though often these are
disease, gestational or conventional diabetes
painless) and deformity. Treatment is by
mellitus, and in the latter case associated
engendering a negative calcium and phosphate
abnormalities such as microvascular
balance, e.g. increased dialysis clearance,
complications, an elevated HbA1C and a poor
stopping positive calcium balance, and recourse
obstetric outcome may occur, but none of
to renal transplantation.
these are certain in this setting.

[ Q: 60 ] MasterClass Part2
[ Q: 59 ] MasterClass Part2
(2010) - Nephrology
(2010) - Nephrology
A 68-year-old man receiving regular
A patient on dialysis presents with a large
haemodialysis for chronic renal failure of
swollen tender shoulder. Aspiration yields a

Dr. Khalid Yusuf El-Zohry – Sohag Teaching Hospital (01118391123) Page | 282
El-Zohry MRCP Questions Bank (Part 2) – Medical Masterclass 2010

unknown cause has the following blood test commonly used. Aluminium hydroxide is also
results: sodium 133 mmol/l, potassium 5.1 effective, but is no longer used (excepting in
mmol/l, calcium 1.95 mmol/l and phosphate rare circumstances) because of the adverse
1.98 mmol/l. consequences of aluminium intoxication.

Which TWO of the following medications are


most appropriate to correct these
[ Q: 61 ] MasterClass Part2
abnormalities? (2010) - Nephrology
A. Sodium pamidronate An 82-year-old man presents with shortness of
breath. He has a past medical history of end-
B. Calcium resonium
stage renal failure caused by adult polycystic
C. 1-alpha hydroxycholecalciferol kidney disease, for which he received
D. Prednisolone haemodialysis for 12 years before receiving a
renal transplant 24 years ago. His medication is
E. Ergocalciferol ciclosporin, azathioprine, prednisolone,
F. Erythopoietin pravastatin, enalapril and frusemide. His
creatinine, when checked at a routine clinic visit
G. Calcium carbonate
14 days before admission, was
H. Aluminium hydroxide 287micromoles/l.
I. Alendronate sodium
Which TWO of the following are the most likely
J. Sodium bicarbonate. causes of his shortness of breath?
A. Asthma
Answer & Comments
B. Pulmonary fibrosis
Correct answer: CG
C. Pneumocystis

The major abnormalities are hypocalcaemia and D. Pulmonary embolism


hyperphosphataemia secondary to chronic E. Bacterial chest infection
renal failure.
F. CMV pneumonitis
In chronic renal failure there is failure of the
G. Invasive aspergillosis
renal 1-alpha hydroxylase enzyme, preventing
vitamin D from being converted to 1-alpha H. Constrictive pericarditis
hydroxycholecalciferol, an intermediary on the I. Pulmonary oedema
way to the main active metabolite, 1,25-
dihydroxycholecalciferol. Hypercalcaemia can J. Ciclosporin pneumotoxicity
therefore be corrected with 1-alpha
hydroxycholecalciferol, and this should be done Answer & Comments
to prevent / retard the development of
Correct answer: EI
secondary hyperparathyroidism, which has
long-term adverse consequences, particularly
on bone. This man has a renal transplant but the function
of the transplanted kidney is not good, such
In chronic renal failure there is phosphate that he could easily develop sodium and water
retention. This can be ameliorated by the retention leading to pulmonary oedema and
consumption of phosphate binders before / at shortness of breath. As an immunosuppressed
the same time as food. Calcium carbonate is

Dr. Khalid Yusuf El-Zohry – Sohag Teaching Hospital (01118391123) Page | 283
El-Zohry MRCP Questions Bank (Part 2) – Medical Masterclass 2010

and elderly man he will be vulnerable to [ Q: 63 ] MasterClass Part2


bacterial chest infections. (2010) - Nephrology
A 35-year-old man has nephrotic syndrome and
[ Q: 62 ] MasterClass Part2
undergoes a renal biopsy. The image shown is
(2010) - Nephrology from one of the analyses of the material
A 36-year-old renal transplant receipient of 18 obtained.
months is admitted with a history of a rising
serum creatinine, pyrexia, weight loss, anaemia,
abnormal liver enzymes and cervical
lymphadenopathy. His immediate post
transplant period was complicated by an
episode of cellular rejection and one of vascular
rejection treated with methylprednisolone and
anti thymoglobulin (ATG) respectively.

Which is the most likely diagnosis?


A. Cytomegalovirus (CMV) disease
B. Chronic transplant rejection
C. Tuberculosis
D. Lymphoma
E. Infectious endocarditis

Answer & Comments

Correct answer: D
Which TWO of the following statements do you
agree with?
The risk of post transplant lymphoma - or more
A. The letter A indicates the lumen of a
correctly post transplant lymphoproliferative
disease (PTLD), which is the most likely proximal convoluted tubule.
diagnosis in this case - is related to the degree B. The letter B indicates an area of foot
of previous immunosuppression, and this man process fusion.
was exposed to baseline immunosuppressive
C. The letter C indicates a mitochondrion
drugs and also potent anti-rejection therapy in
which shows abnormal cristae.
the form of ATG.
D. The features are typical of membranous
PTLD is often driven by Epstein–Barr virus and
glomerulonephritis.
can be treated by graded reduction in
immunosuppression. E. The letter C is over the nucleus of a
mesangial cell.
It would be unusual for CMV disease to present
at this time: it usually occurs much earlier in the F. Based on the features of the biopsy,
post transplant period. Chronic rejection does treatment with steroids is unlikely to be
not cause systemic symptoms such as those of benefit.
described here. G. A striking feature is numerous
subepithelial deposits.

Dr. Khalid Yusuf El-Zohry – Sohag Teaching Hospital (01118391123) Page | 284
El-Zohry MRCP Questions Bank (Part 2) – Medical Masterclass 2010

H. The image shows a scanning electron E. Calcific aortic stenosis


micrograph.
F. Pulmonary stenosis
I. The basement membrane shows areas of
G. Bacterial endocarditis
reduplication.
H. Hypocalcaemia
J. Material for immunofluorescence and
light microscopy is likely to have been I. Functional iron deficiency
fixed and processed in the same way. J. Hepatitis A.

Answer & Comments Answer & Comments


Correct answer: BE Correct answer: FJ

The image shown is a transmission electron Haemodialysis patients suffer the complications
micrograph of part of a glomerulus. A is over a of chronic renal failure as well as those of the
capillary lumen, which would be filled with dialysis procedure.
blood in vivo. The letter B is located over the
Hair loss and osteoporosis occur because of
urinary space, with an arrowhead indicating an
area where the foot processes of the epithelial exposure to heparin.
cells are fused together and effaced onto the Carpel tunnel syndrome can occur due to the
basement membrane. These appearances are accumulation of B2-microglobulin resulting in
consistent with minimal change nephrotic dialysis-related amyloid.
syndrome, and as long as the light microscopy
Parathyroid adenomas occur in tertiary
and immunofluorescence findings do not show
hyperparathyroidism as a consequence of
other features, a response to steroids would be
prolonged and uncontrolled secondary
expected.
hyperparathyroidism.
Ectopic calcification occurs in chronic renal
[ Q: 64 ] MasterClass Part2
failure as a result of an elevated calcium:
(2010) - Nephrology phosphate product and this can cause aortic
A 72-year-old man developed end stage renal stenosis.
failure due to IgA nephropathy 5 years ago and Endocarditis is seen in these patients, especially
has been on regular thrice-weekly those with central venous catheters as their
haemodialysis since that time. His general dialysis access, because of recurrent
health has gradually declined and he has bacteriaemias.
suffered many complications of long-term
Hypocalcaemia can occur because of reduced
dialysis.
levels of active vitamin D.
Which TWO of the following are NOT Functional iron deficiency is also a feature of
recognised complications of patients with chronic renal failure where despite adequate
chronic renal failure on haemodialysis? iron stores, iron cannot be mobilised to the
A. Hair loss marrow in sufficient quantities.

B. Osteoporosis
C. Carpel tunnel syndrome
D. Parathyroid adenoma

Dr. Khalid Yusuf El-Zohry – Sohag Teaching Hospital (01118391123) Page | 285
El-Zohry MRCP Questions Bank (Part 2) – Medical Masterclass 2010

[ Q: 65 ] MasterClass Part2 Answer & Comments


(2010) - Nephrology Correct answer: BC
A 60-year-old man presents with non-specific
malaise, weight loss, backache and renal The patient has bilateral upper tract
impairment. The patient is convinced he has obstruction for which the most likely causes are
cancer. C-reactive protein is 30 mg/l (NR 0 - 10) retroperitoneal fibrosis, lymphoma or
and creatinine 420 µmol/l. He weighs 70 kg. malignant infiltration. A CT scan will define the
Urinalysis is negative for protein and blood. site of the obstruction and narrow the list of
Renal ultrasound shows bilateral pelvicalyceal probable causes. Nephrostomies will be
dilatation with good preservation of renal appropriate in the short term. If the diagnosis is
cortical thickness, the prostate is slightly retroperitoneal fibrosis, moderate dose oral
enlarged and the bladder empties completely steroids will be indicated for the systemic
on micturition. symptoms. Although surgical ureterolysis may
be performed, ureteric stents are often a very
Which TWO of the following statements are satisfactory treatment. Once obstruction is
true? relieved the patient is likely to be polyuric, and
A. The patient's GFR is likely to be ~60 the prognosis for recovery of renal function is
ml/min based on age, plasma creatinine good given preserved cortical thickness.
and weight. Bladder catheterisation is not indicated.

B. Computerised tomography would be a


useful investigation to perform next. [ Q: 66 ] MasterClass Part2
(2010) - Nephrology
C. Once obstruction is relieved he may be
polyuric for several days. (2) A 52-year-old man is admitted for the
investigation of nephrotic range proteinuria, in
D. Following treatment, the chance of the
the course of which he undergoes a renal
creatinine returning to the normal range
biopsy. You are called to see him the evening
is less than 5%.
after the biopsy. He is pale, sweaty and in pain,
E. A urethral catheter should be placed to with pulse 110/min and BP 100/70 mmHg. He
monitor output accurately. has passed urine since the biopsy, which is not
F. Initial treatment should be with three obviously blood stained. Examination reveals
daily doses of 500 mg some tenderness in the abdomen on the side of
methylprednisolone. the biopsy but little else.

G. Percutaneous nephrostomies would be a Select TWO of the following courses of action


reasonable long-term treatment for his that you consider MOST reasonable:
obstructive uropathy.
A. Prescribe simple analgaesia and arrange
H. The patient can be reassured that the to review again in 2 hours.
most likely diagnosis is probably
B. Discuss with the on call radiologist with a
retroperitoneal fibrosis and that he does
view to CT or angiography.
not have any form of cancer.
C. Ask the urology team to review the
I. If the diagnosis is retroperitoneal fibrosis,
patient with a view to immediate
surgical intervention will be necessary.
nephrectomy.
J. The first priority is to arrange
D. Insert a central venous catheter.
haemodialysis for this patient.

Dr. Khalid Yusuf El-Zohry – Sohag Teaching Hospital (01118391123) Page | 286
El-Zohry MRCP Questions Bank (Part 2) – Medical Masterclass 2010

E. Insert a urinary catheter. dialysis BP was 160-170/70-80 and after dialysis


was 200-230/80-90 on doxazosin 4 mg b.d. and
F. Prescribe a NSAID (eg ibuprofen) for
amlodipine 10 mg. These drugs were omitted
analagesia and arrange to review again in
on dialysis days.
2 hours.
G. Book an ultrasound scan for the next day Which TWO of the following statements about
to look for the possibility of a perinephric the BP or its consequences are true?
haematoma. A. There is no point treating this BP as there
H. Start a treatment dose of heparin. is no evidence to support BP reduction in
dialysis patients
I. Ensure adequate IV access, infuse fluid,
and request 4 units of blood urgently. B. An angiotensin-converting enzyme
inhibitor or angiotensin II receptor
J. Start oral prednisolone, 60 mg daily.
antagonist may be effective.

Answer & Comments C. Selective increase of dialysate sodium


using sodium profiling will be effective in
Correct answer: BI causing a BP reduction.
D. A phaeochromocytoma is likely.
This man has features highly suggestive of a
post renal biopsy haemorrhage. Complications E. Beta-blockade may be effective.
following renal biopsy are relatively F. This is likely to be white-coat
uncommon, but significant post biopsy bleeding hypertension.
will occur in as many as 1 in 25 patients.
G. Left ventricular hypertrophy is unlikely.
Haematuria may not always be present and
H. Afternoon or evening dialysis slots would
bleeding may also arise from other structures
significantly reduce BP levels.
or vessels near the kidney. All bleeds with
haemodynamic upset need urgent attention I. Omitting BP drugs on dialysis day is
and management. A peripheral large bore essential.
cannula is better, safer and quicker than a
J. Salt restriction is not likely to reduce BP.
central line. Blood should be rapidly available as
a group and saved blood should have been
posted prior to the biopsy. Answer & Comments

The purpose of subsequent investigation is to Correct answer: BE


establish where the bleeding site is and if there
is evidence of active bleeding. A contrast CT is Avoiding hypotensive medication on dialysis
probably the best initial investigation. This can days is a practise that is too prevalent, dating
be followed by an angiogram with therapeutic from the widespread use of potent short-acting
embolisation if appropriate. This is usually BP drugs which, together with dialysis,
quicker than surgical intervention and will conspired to cause significant falls in BP during
usually preserve renal function dialysis (intra-dialytic hypotension) and
problems of cardiovascular stability. However,
[ Q: 67 ] MasterClass Part2 the price for a nice short ‘stable’ dialysis is
hypertension, both intra- and inter-dialytic. In
(2010) - Nephrology
this case it is likely that both the renin-
A 72-year-old man was severely hypertensive angiotensin and the sympathetic nervous
on haemodialysis (morning sessions). His pre- systems are being activated by dialysis -

Dr. Khalid Yusuf El-Zohry – Sohag Teaching Hospital (01118391123) Page | 287
El-Zohry MRCP Questions Bank (Part 2) – Medical Masterclass 2010

perhaps the patient is at or under his dry The rash shown is consistent with a
weight? Thus blockade of the renin angiotensin leucocytoclastic vasculitis, and the history
aldosterone system and sympathetic nervous would be entirely consistent with ANCA
system are appropriate strategies. (antineutrophil cytoplasmic antibodies) positive
systemic vasculitis. A skin biopsy will not yield a
[ Q: 68 ] MasterClass Part2 specific diagnosis, whereas a test for ANCA is
likely to be positive.
(2010) - Nephrology
In view of the preserved renal size, active urine
A 73-year-old man presents with a three month sediment and rash, it is likely that there is an
history of feeling non-specifically unwell, has acute (i.e. reversible) component to the renal
lost 6 kg in weight and has recently noticed the failure and a biopsy should be performed. This
rash shown in the enclosed image. is most likely to show a pauci-immune, focal
segmental glomerulonephritis, but it is
important to exclude other conditions.

[ Q: 69 ] MasterClass Part2
(2010) - Nephrology
A patient is found to have renal impairment. A
native renal biopsy is performed.
Immunofluorescence staining of the biopsy
reveals linear deposition of IgG along the
The creatinine is 340 µmol/l. Urinalysis shows basement membrane.
blood and protein. Renal ultrasound shows
kidneys of 10.8 and 11.2 cm with increased In which TWO conditions is this pattern
cortical echogenicity. commonly observed?
A. Membranous nephropathy
Which of the following statements is correct?
B. Berger's nephropathy
A. Renal biopsy should be performed and is
likely to show a focal necrotising C. Alport's syndrome
glomerulonephritis D. Goodpasture's syndrome
B. The renal impairment is likely to be E. Wegener's granulomatosis
chronic (i.e. irreversible)
F. AL amyloidosis
C. Skin biopsy is likely to yield a specific
G. Sarcoidosis
diagnosis
H. Diabetes mellitus
D. Renal biopsy should be performed and is
most likely to show membranous I. Henoch–Schönlein purpura
glomerulonephritis J. Haemolytic uraemic syndrome.
E. Laboratory blood tests are unlikely to
yield a specific diagnosis. Answer & Comments

Correct answer: DH
Answer & Comments

Correct answer: A

Dr. Khalid Yusuf El-Zohry – Sohag Teaching Hospital (01118391123) Page | 288
El-Zohry MRCP Questions Bank (Part 2) – Medical Masterclass 2010

The classical pattern of immunofluorescence Answer & Comments


seen in membranous nephropathy is granular
Correct answer: BE
deposition of IgG and C'3. Berger described
mesangial deposition of IgA, which is also seen
on renal biopsy in patients affected by Henoch– Whatever the cause of urinary stones,
Schönlein purpura. Goodpasture described the increasing fluid intake always reduces the
association of pulmonary haemorrhage and concentration of stone-forming substances in
acute renal failure, now recognised to be the urine and is always the most important
caused by linear deposition of IgG antibodies modification to diet. Patients should be
binding to the glomerular basement instructed to drink enough to obtain a urinary
membrane. volume of over 3 litres / day, and those who
continue to be troubled by stones may need to
Alport's syndrome, Wegener's granulomatosis, drink even more.
sarcoidosis and haemolytic uraemic syndrome
are not normally associated with Hypercalciuria, usually idiopathic, is found in
immunoglobulin deposition in the glomerulus, 65% of patients with urinary stones and is
although transplantation of a normal kidney explained by intestinal hyperabsorption of
into a patient with Alport's syndrome can result calcium. However, reducing dietary calcium
in generation of antibodies against glomerular intake is NOT the answer, since this leads to
basement membrane. In AL amyloid, light increased intestinal absorption of oxalate and
chains are deposited in a nodular pattern. can thereby increase the risk of stone
formation. Avoidance of non-staple foods high
in oxalate (e.g. rhubarb, spinach) is sensible.
[ Q: 70 ] MasterClass Part2 There is also a (lesser) role for reduction in
(2010) - Nephrology intake of salt and refined sugar.
A 28-year-old man has had several attacks of
renal colic. He passes stones consisting mainly [ Q: 71 ] MasterClass Part2
of calcium oxalate. His 24-hour urinary calcium (2010) - Nephrology
excretion is twice the upper limit of normal.
A 56-year-old man with known chronic renal
Which TWO alterations to his diet are most failure is admitted with general malaise to the
likely to be helpful in reducing the risk of stone Emergency Department. He complains of
recurrence? itching and nausea. His BP is 175/110 mmHg
and plasma creatinine 952 μmol/L (normal
A. Reduced salt intake
range 60-115)
B. Increased fluid intake
Which TWO of the following would be the most
C. Avoidance of alcohol
important reasons for initiating urgent
D. Avoidance of refined sugar haemodialysis in this man?
E. Avoidance of foods high in oxalate A. Hyperchloridaemia
F. Instigation of a calcium-free diet B. A high plasma hydrogen ion
G. Avoidance of dairy products concentration

H. Increased salt intake C. Itch

I. Reduced fluid intake D. Hyperkalaemia

J. Avoidance of dietary meat. E. Headache

Dr. Khalid Yusuf El-Zohry – Sohag Teaching Hospital (01118391123) Page | 289
El-Zohry MRCP Questions Bank (Part 2) – Medical Masterclass 2010

F. Poor appetite J. Cinnarizine


G. Skin pigmentation
Answer & Comments
H. Anaemia
Correct answer: AC
I. A diastolic BP above 98 mmHg
J. A low magnesium level
Non-steroidal anti-inflammatory agents and
antibiotics (particularly penicillins) are the
Answer & Comments commonest causes of acute interstitial
nephritis. Other drugs that have been
Correct answer: BD
incriminated include allopurinol, cimetidine,
frusemide, omeprazole, rifampicin and thiazide
The key reasons for initiating urgent renal diuretics (and many others have been
replacment therapy when renal function is associated in occasional case reports).
severely compromised are hyperkalaemia,
metabolic acidosis, pulmonary oedema and Aside from supportive care (e.g. dialysis if
severe uraemic syndrome (such as a severe required), treatment involves withdrawal of the
confusional state or uraemic pericarditis). precipitating agent and moderate doses of oral
Relatively milder symptoms, such as poor steroids (e.g. prednisolone 40 mg daily). The
appetite, lethargy and nausea, may prompt the efficacy of the latter has not been
elective initiation of renal replacement therapy demonstrated in controlled trials.
in patients with chronic renal impairment, but
they do not require urgent action. [ Q: 73 ] MasterClass Part2
(2010) - Nephrology
[ Q: 72 ] MasterClass Part2
The image shows the fundus of a patient who
(2010) - Nephrology had a grand mal seizure 2 hours previously,
A 68-year-old woman with a complex past resulting in admission to the Emergency
medical history presents with unexplained Department (see image). The blood pressure is
acute renal failure. Renal biopsy reveals acute high following the seizure (200/130 mmHg),
interstitial nephritis. and blood tests reveal severe renal failure, with
creatinine 670 µmol/l and urea 28 mmol/l. The
Which TWO of her many medications are the kidneys are of normal size and not obstructed
most likely cause? on ultrasonography.
A. Amoxicillin
B. Paracetamol
C. Ibuprofen
D. Aspirin
E. Digoxin
F. Enalapril
G. Temazepam
H. Frusemide
I. Co-proxamol

Dr. Khalid Yusuf El-Zohry – Sohag Teaching Hospital (01118391123) Page | 290
El-Zohry MRCP Questions Bank (Part 2) – Medical Masterclass 2010

Select the most appropriate statement. C. Renal biopsy


A. The likely cause of the seizure and renal D. Renal angiogram
failure is an overdose of aspirin
E. Renal venogram
B. The seizure was most probably due to
F. Bone marrow aspirate and trephine
insulin-induced hypoglycaemia
G. A serum amyloid P (SAP) scan
C. The blood film is likely to show evidence
of microangiopathic haemolysis H. Isotope renography

D. The renal failure is likely to be acute, and I. Skin biopsy


due to rhabdomyolysis J. Rectal biopsy.
E. The seizure was probably due to uraemic
encephalopathy Answer & Comments

Correct answer: CF
Answer & Comments

Correct answer: C This is a typical presentation of systemic


amyloidosis. It is likely that the patient has the
The fundus shows papilloedema and AL (immunocyte-related) form of the condition,
haemorrhages. Taken together with the blood based on the presence of the paraprotein and
pressure the most likely cause of the renal the low CRP.
failure is malignant phase hypertension, which A renal biopsy is required to establish the cause
is usually associated with microangiopathy. of the nephrotic syndrome: amyloidosis is likely,
This degree of biochemical disturbance would but other primary glomerular disease cannot be
be unlikely to give rise uraemic encephalopathy confidently excluded without histology.
and seizures without another factor. A bone marrow aspirate and trephine are
Two hours following a seizure is too early for needed because some degree of plasma cell
rhabdomyolysis to induce this degree of acute dyscrasia is likely, and if there is frank myeloma
renal failure. treatment is indicated. Even if there is no frank
myeloma, some treatment, e.g. prednisolone
and melphalan, may be warranted since
[ Q: 74 ] MasterClass Part2
untreated amyloid tends to progress rapidly,
(2010) - Nephrology and AL- amyloid can lead to rapid cardiac and
A 73-year-old lady presented with a 6-month renal decompensation.
history of malaise, anorexia, progressive leg A SAP scan is a research rather than diagnostic
swelling and easy bruising. Plasma creatinine tool, being particularly useful in regression
was 97 umol/l. 24 hours urinary protein was 12 studies. Only one UK centre can perform these
grams. Renal ultrasound showed two 10 cms scans currently (National Amyloid Centre, Royal
echogenic kidneys. CRP was 5 mg/l (NR <5). She Free Hospital, London). The best treatment for
had a 7 g/l IgG lambda paraprotein on serum primary amyloidosis is not known: give the
electrophoresis, but no Bence Jones protein. centre a call if you have a case; they will be able
to educate you, and also give you information
Which further investigations are necessary ? about trials into which the patient might be
A. CT scan chest recruited.
B. CT scan abdomen

Dr. Khalid Yusuf El-Zohry – Sohag Teaching Hospital (01118391123) Page | 291
El-Zohry MRCP Questions Bank (Part 2) – Medical Masterclass 2010

[ Q: 75 ] MasterClass Part2 E. Antibodies to erythropoietin


(2010) - Nephrology F. Myeloma
A 36-year-old man presents with a 2-day history G. Gross obesity
of fresh haematuria. H. Uraemia
Which TWO of the following are likely causes? I. Myelodysplasia
A. Congestive cardiac failure J. Hypocalcaemia.
B. Bronchopulmonary aspergillosis
Answer & Comments
C. Nephrogenic diabetes insipidus
Correct answer: DJ
D. Polycystic kidney disease
E. Hypocalciuria For erythropoietin therapy to work adequate
F. IgA nephropathy iron stores are needed. Aluminium intoxication
impairs the body's ability to utilize iron.
G. Testicular feminisation syndrome
Inflammation and uraemia damp both the
H. Minimal change nephropathy endogenous production of erythropoietin and
I. Acute asthma treated with beta agonists the response to exogenous erythropoietin.

J. Hyperkalaemia. Severe hypertension is a contraindication to


erythropoietin therapy but does not affect the
response if doses continue to be administered.
Answer & Comments
The response to erythropoietin is impaired if
Correct answer: DF the marrow is abnormal and can be blocked by
anti-erythropoietin antibodies, leading to pure
IgA nephropathy can cause fresh haematuria, red cell aplasia. Absorption of subcutaneous
especially with an acute upper respiratory tract doses may be impaired by gross obesity. The
infection. Polycystic kidney disease can cause action of erythropoietin is not influenced by
haematuria if there is bleeding, especially hypocalcaemia.
related to infection or stone formation. Minimal
change nephropathy causes proteinuria, but [ Q: 77 ] MasterClass Part2
not frank haematuria. (2010) - Nephrology
A 64-year-old woman is referred to the renal
[ Q: 76 ] MasterClass Part2
outpatients for investigation of abnormal renal
(2010) - Nephrology function, detected after she consulted the GP
following an episode of macroscopic
In a patient approaching end-stage renal failure haematuria. She has a 20-year history of low
which TWO of the following are unlikely to back pain treated with a combination of
cause hyporesponsiveness to subcutaneous painkillers. An ultrasound identifies two
recombinant human erythropoietin? smallish irregular shaped kidneys.
A. Aluminium toxicity
Which of the following is the most likely
B. Intercurrent infection diagnosis?
C. Iron deficiency A. IgA nephropathy
D. Severe hypertension B. Myeloma kidney

Dr. Khalid Yusuf El-Zohry – Sohag Teaching Hospital (01118391123) Page | 292
El-Zohry MRCP Questions Bank (Part 2) – Medical Masterclass 2010

C. Analgesic nephropathy Select one of the following answers:


D. Acute interstitial nephritis A. High-dose oral or intravenous steroids
are the most effective treatment.
E. Membranous nephropathy.
B. Angiotensin-converting enzyme (ACE)
Answer & Comments inhibitors should be avoided in this
setting.
Correct answer: C
C. The most likely diagnosis is haemolytic–
uraemic syndrome.
The combination of chronic analgesic use,
haematuria and irregular shrunken kidneys all D. Autoantibodies to glomerular basement
suggest analgesic nephropathy. This is a specific membrane are likely to be present in the
form of chronic interstitial nephritis more serum.
common in women and usually related to in E. Renal biopsy is likely to show arteriolar
excess of 5 years of mixed analgesic use. changes, fibrin thrombi and fibrinoid
Urothelial malignancy occurs in around 10% necrosis.
and all should be investigated for this, as should
anyone over 40 with isolated haematuria. The
Answer & Comments
most useful diagnostic pointers are the history
and imaging of the kidneys using either Correct answer: E
ultrasound or CT.
The cutaneous appearances and renal
[ Q: 78 ] MasterClass Part2 presentation are typical of scleroderma with a
(2010) - Nephrology renal crisis. In patients who develop a renal
crisis this often occurs early in the course of the
A 30-year-old woman presents with arthralgia disease. There is usually relatively minor
and tiredness. She is found to be hypertensive proteinuria, hypertension and evidence of
(160/100 mmHg). The creatinine is 300 microangiopathy. Typically the renal biopsy will
micromol/l, there is 0.9 g proteinuria per 24 show arteriolar changes and fibrinoid necrosis,
hours and the kidneys are normal size on and the best current treatment is with ACE-
ultrasound. Full blood count shows a slightly inhibitors.
reduced platelet count, mild anaemia and
evidence of microangiopathy. The patient's
[ Q: 79 ] MasterClass Part2
hand is shown (see image).
(2010) - Nephrology
A 36-year-old man presents with microscopic
haematuria and hypertension. Ultrasound scan
shows several cysts in the kidneys, and two
solid lesions, the larger of which is 5 cm
diameter.
The man is not in contact with his family, but he
knows that his father died undergoing surgery
for a brain tumour aged 30 year, also that one
of his father's two sisters had some kind of
kidney problem and was deaf in one ear.

Dr. Khalid Yusuf El-Zohry – Sohag Teaching Hospital (01118391123) Page | 293
El-Zohry MRCP Questions Bank (Part 2) – Medical Masterclass 2010

What is the most likely diagnosis? D. Rheumatoid arthritis


A. Autosomal dominant polycystic kidney E. Multiple myeloma
disease
B. Autosomal recessive polycystic kidney Answer & Comments
disease
Correct answer: D
C. Tuberous sclerosis
D. von Hippel Lindau disease Monoclonal gammopathy of undetermined
significance and multiple myeloma are
E. Nephronophthisis
associated with paraproteinaemia and could
give rise to AL (primary) amyloid. Chronic
Answer & Comments rheumatoid arthritis is an important cause of
AA amyloid because of the associated
Correct answer: D
persistent inflammatory response.

von Hippel Lindau disease is transmitted as an


autosomal dominant condition. Affected [ Q: 81 ] MasterClass Part2
individuals may have any of the following: (2010) - Nephrology
 Renal cysts A 31-year-old woman presents to her GP with
 Clear cell renal cell carcinoma (CCRCC) marked leg oedema. Urine dipstick analysis
shows 4+ Protein. Blood tests identify normal
 Retinal angiomas renal excretory function (creatinine 68
 Central nervous system micromol/l) and marked hypoalbuminaemia
haemangioblastoma (albumin 19 g/L). A renal biopsy is reported as
normal, other than foot process fusion on the
 Phaeochromocytoma.
electron microscopy.
Another manifestation is endolymphatic sac
tumours, causing deafness. Which of the following agents would NOT be
indicated as part of her initial therapy?
Solid lesions in the kidney that are below 3 cm
in diameter can be monitored, but above this A. Ciclosporin
size they need removal. B. Furosemide
C. Simvastatin
[ Q: 80 ] MasterClass Part2
D. Warfarin
(2010) - Nephrology
E. Prednisolone.
A patient is referred to the renal team with
nephrotic syndrome and her renal biopsy shows
Answer & Comments
amyloid A (AA) amyloid deposits.
Correct answer: A
Which of the following underlying diagnoses is
most likely to have caused this?
The diagnosis is minimal change
A. Monoclonal gammopathy of glomerulonephritis, also known as minimal
undetermined significance change nephropathy, minimal change disease,
B. Systemic lupus erythematosus lipoid nephrosis and idiopathic nephrotic
syndrome. Standard initial therapy is with
C. Sarcoidosis corticosteroids, typically prednisolone at a dose
Dr. Khalid Yusuf El-Zohry – Sohag Teaching Hospital (01118391123) Page | 294
El-Zohry MRCP Questions Bank (Part 2) – Medical Masterclass 2010

of about 1mg/kg/day, which introduces H. Glomerular filtration rate is typically 30%


remission in around 80 % of patients. of normal by the age of 10 years.
Supportive therapies are also given, diuretics to
I. Administration of recombinant human
clear oedema will clearly be appropriate in this
erythropoietin reduces the number of
case, and some nephrologists would prescribe
sickling crises by approximately 50%.
warfarin to reduce risk of thromboembolism
and a statin to reduce hypercholesterolaemia, J. Renal medullary carcinoma is a relatively
although many would elect to see whether or indolent tumour in patients with sickle
not the patient went into a rapid remission that cell disease.
would render these agents unnecessary.
Ciclosporin is useful as a steroid-sparing agent Answer & Comments
in patients who frequently relapse but would
Correct answer: DG
not be used as initial treatment.

In sickle cell disease, sickling is a particular


[ Q: 82 ] MasterClass Part2
problem in the medullary vasa recta due to
(2010) - Nephrology hypertonicity and hypoxia.
You see a 12-year-old with sickle cell (SC) Reducing medullary hypertonicity may reduce
disease who describes a recent episode of renal damage.
macroscopic haematuria. The parents are
Diminished concentrating ability is an early and
worried that the child has serious kidney
universal finding in sickle cell disease.
disease.
Survival in patients with sickle cell nephropathy
Concerning sickle cell disease, select the TWO is similar to non-diabetic dialysis patients.
statements which you agree with: Interestingly early in the course of sickle cell
A. Renal involvement is more likely in SC disease GFR is actually increased, and only falls
disease than in Hb S homozygotes. below normal by the age of about 30.

B. Hypertension is common in patients with Hypertension is significantly less common in


sickle cell disease. sickle cell disease than in normal black
individuals.
C. Renal transplantation is contraindicated
in patients with sickle cell disease and Renal medullary carcinoma is a highly
end stage renal failure. aggressive tumour which occurs in patients
with sickle trait or sickle cell disease.
D. Macroscopic haematuria is usually due to
papillary necrosis.
[ Q: 83 ] MasterClass Part2
E. Survival of patients with sickle cell
(2010) - Nephrology
nephropathy on dialysis programmes is
worse than patients with diabetes of the An 82-year-old lady presents with renal
same age. impairment and unexplained low back pain. On
F. Red cells in the vasa recta of the renal examination she has substantial bilateral pitting
oedema to above her knees. Dipstick urinalysis
medulla are protected from sickling by
the hypertonic environment. shows no haematuria, but proteinuria + + +. No
casts are seen on urinary microscopy. Her
G. Loss of urinary concentrating ability is serum albumin is 12 and serum creatinine 250.
common in patients with sickle cell
disease.

Dr. Khalid Yusuf El-Zohry – Sohag Teaching Hospital (01118391123) Page | 295
El-Zohry MRCP Questions Bank (Part 2) – Medical Masterclass 2010

Which TWO causes could most readily account vasoconstrictive drive, such as occurs with
for all her problems? dehydration.
A. Systemic sclerosis
B. Degenerative lumbar vertebral changes
[ Q: 84 ] MasterClass Part2
treated with a non-steroidal anti- (2010) - Nephrology
inflammatory drug (NSAID) You are called to see a woman who is in her
C. Acromegaly thirtieth week of pregnancy and has
proteinuria.
D. Myeloma
E. A post-streptococcal diffuse proliferative Which of the following findings would make you
glomerulonephritis suspect that the renal protein leak is not due to
pre-eclampsia?
F. Renal tubular acidosis
A. Hypertension
G. Adult polycystic kidney disease
B. Proteinuria at the pregnancy booking visit
H. Renal stones
C. Oedema
I. Rhinovirus infection
D. High serum urate
J. Emphysema.
E. It is her first pregnancy
Answer & Comments
Answer & Comments
Correct answer: BD
Correct answer: B
How can back pain, renal impairment and
nephrotic syndrome best be tied together? Pre-eclampsia is commoner in first pregnancies,
and is characterised by oedema, proteinuria,
Renal impairment in the elderly should always
hypertension and an elevated serum urate.
raise a suspicion of myeloma and this could
Proteinuria at the booking visit indicates
certainly explain these problems. Myeloma can
underlying renal disease that was present
affect the kidney in a number of ways: most
before the pregnancy.
commonly by causing hypercalcaemia or cast
nephropathy, but also through amyloid
deposition or light chain deposition disease. [ Q: 85 ] MasterClass Part2
NSAIDs can cause nephrotic syndrome with a
(2010) - Nephrology
minimal change nephropathy appearance on A 28-year-old woman received a kidney
electron microscopy. In addition, they can also transplant from her brother. Six weeks post
cause an interstitial nephritis which could transplantation she is admitted with epigastric
account for the raised creatinine. In general, pain, vomiting and fever. Routine testing
prostaglandins cause a tonic renal vasodilation, demonstrates markedly abnormal liver blood
so inhibition of prostaglandin synthesis by non- tests, with a hepatocellular pattern. An
steroidal anti-inflammatory drugs causes a oesophago-gastro-duodenoscopy (OGD) is
relative vasoconstriction with a fall in renal performed and the gastric biopsy is shown (see
blood flow and glomerular filtration rate. This image).
can be especially marked in the elderly or under
circumstances when there is an increased

Dr. Khalid Yusuf El-Zohry – Sohag Teaching Hospital (01118391123) Page | 296
El-Zohry MRCP Questions Bank (Part 2) – Medical Masterclass 2010

[ Q: 86 ] MasterClass Part2
(2010) - Nephrology

Which TWO of the following drugs cannot be


effective in treating an anephric patient on
dialysis who is experiencing recurrent attacks of
gout?
A. Rasburicase
B. Allopurinol
C. Ibuprofen
D. Benzbromarone

Which of the following medications had been E. Probenicid


given at the time of transplantation to try to F. Colchicine
prevent this complication?
G. Indomethacin
A. Aspirin
H. Prednisolone
B. Ranitidine
I. Paracetamol
C. Fluconazole
J. Intramuscular ACTH
D. Cotrimoxazole
E. Valganciclovir Answer & Comments

Correct answer: DE
Answer & Comments

Correct answer: E Benzbromarone and probenicid are uricosuric


agents which can only influence the tendency
The gastric biopsy shows a number of very to gout in patients who are making urine.
abnormal large cells. These cells contain both Rasburicase is licensed for the treatment of
nuclear and cytoplasmic inclusion bodies. They hyperuricaemia in the context of malignant
are characteristic of cytomegalovirus infection . disease and would succesfully convert uric acid
The donor in this case was CMV positive and to allantoin in an anephric patient, although
the recipient was CMV negative. This is a very this product would still have to be eliminated.
high risk combination for CMV disease and Allopurinol is a xanthine oxidase inhibitor,
antiviral prophylaxis was given in the form of effective in lowering urate levels in the absence
valganciclovir. of renal function. Exogenous and endogenous
steroids, colchicine and non-steroidals all
The diagnosis in this case was further confirmed
diminish inflammation and pain in severe gout,
by polymerase chain reaction (PCR) analysis of
and paracetamol may be helpful as an
blood to detect CMV DNA. She responded well
analgesic. Non-steroidals, although
to intravenous ganciclovir.
contraindicated in patients with renal
impairment because of their deleterious effects
on renal haemodynamics, can be safely used
once the patient becomes dialysis dependent.

Dr. Khalid Yusuf El-Zohry – Sohag Teaching Hospital (01118391123) Page | 297
El-Zohry MRCP Questions Bank (Part 2) – Medical Masterclass 2010

[ Q: 87 ] MasterClass Part2 E. hyperkalaemia.


(2010) - Nephrology
Answer & Comments
A 76-year-old man presents with general
malaise and is found to have creatinine 256 Correct answer: D
μmol/L (normal range 60-115), plasma calcium
2.9 mmol/L (normal range 2.15-2.55), Rhabdomyolysis results from muscle injury and
haemoglobin 9.5 g/dL (normal range 12-17) and the release of myoglobin from muscle cells.
erythrocyte sedimentation rate (ESR) 80 Myoglobin is toxic to the renal tubules and
mm/hour (normal range <20). oliguria or even anuria can occur.
Myoglobinuria usually occurs. Muscle cells also
Which of the following is the most likely release creatinine kinase and potassium when
diagnosis? they are injured. Red cell casts are not a feature
A. Secondary hyperparathyroidism of rhabdomyolysis and would suggest a
glomerular disease.
B. Myeloma
C. Scleroderma
[ Q: 89 ] MasterClass Part2
D. Systemic lupus erythematosus (2010) - Nephrology
E. Paget's disease
A 28-year-old man is seen in the acute receiving
unit after he had collapsed at home. His friends
Answer & Comments indicate that he had been unrousable and on
Correct answer: B the floor for at least 24 hours after trying
heroin for the first time. He is now fully awake
but has a painful hip and swollen calf. He has
The combination of renal impairment,
passed no urine in the last 12 hours.
hypercalcaemia, anaemia and a raised ESR is
highly suggestive of myeloma. Systemic lupus In the Emergency Department he receives 4
erythematosus can cause a raised ESR, anaemia litres of 0.9% saline over 6 hours, but only 12
and renal impairment, but does not typically mls of urine are recorded on the chart following
cause hypercalcaemia and is very much less a urinary catheter insertion. Urine dipstick
likely. Secondary hyperparathyroidism occurs in analysis shows 4+ blood, with a trace of protein.
response to a low calcium level. His BP is 170/100. His pulse oximeter reads 85%
on air, with a respiratory rate of 38/min. Chest
[ Q: 88 ] MasterClass Part2 examination reveals widespread crackles. Blood
(2010) - Nephrology results are still awaited but arterial blood
gases demonstrate a significant metabolic
A 19-year-old army recruit presents with acute acidosis and type 1 respiratory failure, with
renal failure. potassium measured at 7.8mmol/l.

Which of the following features is not consistent Which of the following actions are MOST
with a diagnosis of rhabdomyolysis? appropriate?
A. elevated plasma creatinine kinase level A. Calcium resonium 15g TDS should be
B. anuria started

C. elevated plasma creatinine B. An urgent ultrasound should be arranged


to rule out obstruction
D. blood and red cell casts in the urine

Dr. Khalid Yusuf El-Zohry – Sohag Teaching Hospital (01118391123) Page | 298
El-Zohry MRCP Questions Bank (Part 2) – Medical Masterclass 2010

C. Cyclophosphamide should be started at [ Q: 90 ] MasterClass Part2


2mg/kg oral daily (2010) - Nephrology
D. An infusion of sodium bicarbonate should
A 47-year-old woman with a 20-year history of
be administered to generate an alkaline
rheumatoid arthritis with severe joint
diuresis
destruction presents with swelling of her legs
E. Calcium gluconate 10% should be that has got progressively worse over about 6
administered intravenously, slowly with months. Investigation reveals a serum
ECG monitoring creatinine of 565 micromoles/litre and a 24-
F. He should receive a further fluid hour urine protein excretion of 6 grams.
challenge since it is likely that he has lost
Which TWO of the following are the most likely
a lot of intravascular volume into his leg
diagnoses?
G. A low dose dopamine infusion should be
A. Polycystic kidney disease
started
B. AL amyloidosis
H. N-Acetyl cysteine should be given 600mg
orally twice daily C. Rheumatoid pericardial disease

I. A Naloxone infusion should be D. AA amyloidosis


commenced E. Haemolytic uraemic syndrome
J. Urgent dialysis is required and he should F. Membranous glomerulonephritis
be referred for this immediately dialysis.
G. Renal artery stenosis

Answer & Comments H. Takayasu’s disease

Correct answer: EJ I. Rheumatoid vasculitis


J. Focal segmental proliferative
This is a not an uncommon presentation of glomerulonephritis.
acute renal failure. He is most likely to have
developed muscle necrosis following prolonged Answer & Comments
immobilisation associated with opiate
ingestion. The acute renal failure is likely to be Correct answer: DF
due to rhabdomyolysis. His clinical situation is
very serious, with life- threatening There area number of possible causes of renal
hyperkalaemia and respiratory failure. The disease in rheumatoid arthritis. Chronic
latter is likely due to fluid overload and inflammation can result in AA amyloidosis. Gold
pulmonary oedema. Although renal recovery is and pencillamine, which are used to treat
the norm for this condition, this is not going to rheumatoid arthritis, can cause a membranous
occur in the next couple of hours, which are nephropathy. Non-steroidal anti-inflammatory
likely to seal his fate. drugs can cause a fall in glomerular filtration
rate, an intestitial nephritis and a minimal
He needs basic ABC measures with high flow
change nephropathy. Patients with rheumatoid
oxygen and monitoring. The adverse effects of
arthritis can develop aggressive vasculitis
hyperkalaemia are best averted with IV calcium
resulting in a rapidly progressive
gluconate with the response assessed on the
glomerulonephritis.
ECG. He should be dialysed as soon as this can
be arranged in order to remove potassium and In this case, the heavy proteinuria and renal
fluid. impairment would fit with either a

Dr. Khalid Yusuf El-Zohry – Sohag Teaching Hospital (01118391123) Page | 299
El-Zohry MRCP Questions Bank (Part 2) – Medical Masterclass 2010

membranous nephropathy or AA amyloid, [ Q: 92 ] MasterClass Part2


which can both cause the nephrotic syndrome (2010) - Nephrology
and renal impairment.
A 30-year-old man with no antecedent illness
AL amyloidosis is associated with myeloma and
presents with severely raised BP (150/110 mm
other conditions in which there is an excess
Hg), frothy urine, peripheral oedema and
level of immunoglobulin.
lethargy. There is +++ blood and ++++ protein
on urinalysis. 24-hour protein loss is 6.6 grams.
[ Q: 91 ] MasterClass Part2 Plasma albumin is 28 g/L. Plasma C3 is 0.10
(2010) - Nephrology (low). Plasma creatinine is 145 umol/l.

A 56-year-old man with osteoarthritis of the left Which of the following renal lesions is most
hip was admitted for elective left total hip likely to be found on renal biopsy?
replacement. Pre-operatively, his plasma
A. IgA nephropathy
creatinine was 130 micromol/l. Two days post-
operatively, his plasma creatinine was 210 B. Lupus
micromol/l. C. Focal sclerosing glomerulosclerosis
Which of the following findings could not D. Post infectious glomerulonephritis
account for this change?
E. Mesangiocapillary glomerulonephritis.
A. Ascending urinary tract infection
secondary to peri-operative bladder Answer & Comments
catheterisation
Correct answer: E
B. Use of non-steroidal anti-inflammatory
drugs for analgesia
This man presents with features of nepthrotic
C. Peri-operative hypotension due to syndrome (oedema, heavy proteinuria and
haemorrhage hypoalbuminaemia) combined with features of
D. Erroneous excess administration of low nephritic syndrome (hypertension, reduced GFR
dose oral prednisolone for two days and haematuria). This combination can be seen
in IgA nephropathy but complement levels are
E. Aminoglycoside antibiotic therapy. usually normal in this condition.
Lupus is unusual in young men. FSGS is typically
Answer & Comments
nephrotic syndrome, and commoner in older
Correct answer: D people. Post infectious Gn is possible but the
infection should be clinically apparent, or in the
Post-operative declines in renal function are history.
common and can be triggered by many factors MCGN presenting in a young man with
including hypotension, sepsis and drug effects, nephrotic syndrome and hypertension and
especially those of non-steroidal anti- hypocomplementaemia would fit best.
inflammatory drugs and aminoglycosides.
However, low dose prednisolone is not a cause
[ Q: 93 ] MasterClass Part2
of renal impairment.
(2010) - Nephrology
A 24-year-old woman presents with acute onset
severe loin pain. A renal ultrasound

Dr. Khalid Yusuf El-Zohry – Sohag Teaching Hospital (01118391123) Page | 300
El-Zohry MRCP Questions Bank (Part 2) – Medical Masterclass 2010

demonstrates unilateral hydronephrosis and been described with the classic form involving
multiple renal calculi in both kidneys. A plain the SCL3A1 gene on chromosome 2. The stones
abdominal X-ray film (KUB) is reported as are radiolucent and are best detected and
normal. A subsequent IVP confirms a ureteric followed with ultrasound.
obstruction with a small calculus. Her pain
Renal failure is very rare with correct
settles with conservative treatment and she
management but a transplant would cure the
passes a small calculus 2 days later. A repeat
renal disorder. Initial management aims to
ultrasound confirms resolution of the
reduce the cystine concentration in the urine
hydronephrosis. Stone analysis is positive for
and to change the solubility product by raising
cystine.
urine pH. If these simple measures fail, then
free cystine can be reduced by drugs that bind
Which of the following statements do you
to cyteine forming a soluble complex. These
AGREE with?
include penicillamine, tiopronin and captopril.
A. She has cystinuria, which is an autosomal
dominant condition
[ Q: 94 ] MasterClass Part2
B. Captopril may be of value in her (2010) - Nephrology
subsequent management
A 31-year-old woman presents with seizures.
C. Her parents, who are unaffected, will
On examination she has a facial rash. There is
definitely have a normal urine cystine
blood and protein in her urine on dipstick
analysis
analysis. Serum creatinine is 575 μmol/L
D. The KUB report must be an error as the (normal range 60-115). On immunological
ultrasound showed multiple stones testing she has antibodies to double-stranded
E. If she developed kidney failure a renal DNA.
transplant would cure her condition
Which of the following conditions is a well-
F. Fluid restriction is an important element recognised renal feature of systemic lupus
of management erythematosus?
G. Her children have a 50% chance of A. Nephrocalcinosis
developing the condition
B. Membranous nephropathy
H. Her children have a 24% chance of
C. IgA nephropathy
developing the condition
D. Amyloidosis
I. Stones are best monitored by follow up
plain X-rays E. Renal cell cancer

J. She is most likely to have an X-linked


disorder Answer & Comments

Correct answer: B
Answer & Comments

Correct answer: BE Systemic lupus erythematosus can affect the


kidney in a number of different ways. Typical
patterns of renal involvement include focal or
She has cystinuria, which is an autosomal
diffuse proliferative, membranous,
recessive disorder caused by a gene defect in a
mesangiocapillary (membranoproliferative) and
tubular transporter protein involved in amino
crescentic glomerulonephritides. Interstitial
acid transport. A number of mutations have

Dr. Khalid Yusuf El-Zohry – Sohag Teaching Hospital (01118391123) Page | 301
El-Zohry MRCP Questions Bank (Part 2) – Medical Masterclass 2010

damage, tubular defects and ultimately Renal sarcoidosis is unusual. It is typically an


glomerular fibrosis and sclerosis can also occur. intense tubulo-interstitial nephritis; rarely
typical sarcoid granulomata can be seen in the
[ Q: 95 ] MasterClass Part2 biopsy. Microscopic nephrocalcinosis is very
typical.
(2010) - Nephrology
Dehydration is very typical in these cases due to
A 67-year-old man presents with confusion, the effects of hypercalcaemia on renal water
hypotension, and renal impairment (plasma handling, and also the fact that involvement of
creatinine 198 umol/l). He has 0.6 grams the renal medulla leads to salt wasting.
urinary protein loss per day. He is Rehydration and steroids would be expected to
hypercalcaemic with plasma calcium 3.04 reduce creatinine substantially, but this may
mmol/l. Parathyroid hormone (PTH) is 5 nmol/l not normalize as the various processes have
(NR 10 - 65 nmol/l). A chest radiograph is probably been going on for months and years.
normal. Renal ultrasound shows echogenic Hypercalcaemia is often insidious.
normal sized kidneys. A renal biopsy is
performed which shows tubulo-interstitial The PTH is appropriately suppressed by the
lymphocytic infiltration, nephrocalcinosis but hypercalcaemia (if it were high, or normal, it
no casts. would make primary hyperparathyroidism a
likely diagnosis).
Which TWO of the following are correct ? Sarcoidosis can certainly affect the kidney alone
A. Focal necrotising lesions will be seen in (but if you look hard enough, e.g. chest CT, one
glomeruli can often find some enlarged lymph nodes).
Gallium scan positivity, and serum angiotensin-
B. The tubulo-interstitial compartment will
converting enzyme (SACE) elevation are both
be normal
nice to see - but I am afraid that neither test is
C. Nephrocalcinosis is likely infallible either way round (i.e. false positive
D. Calcitonin is the treatment of choice for and false negative). It is the clinical picture as a
his hypercalcaemia whole that determines the diagnosis best.

E. The normal chest radiograph excludes


sarcoidosis [ Q: 96 ] MasterClass Part2
(2010) - Nephrology
F. The serum angiotensin converting
enzyme level must be raised for a A 52-year-old man presents with acute colicky
diagnosis of sarcoidosis to be secure pain that radiates from his left loin to his left
groin and is associated with nausea and
G. Renal gallium scan positivity makes a
vomiting. A plain abdominal radiograph is
diagnosis of sarcoidosis secure
unremarkable, but ultrasound examination
H. The response to steroids is likely to be demonstrates pelvi-calyceal dilatation and the
disappointing presence of several masses that cast acoustic
I. The low PTH is surprising given the renal shadows in the left renal pelvis.
impairment
The most likely diagnosis is:
J. Sarcoidosis can affect the kidney alone.
A. Adult polycystic kidney disease

Answer & Comments B. Papillary necrosis


C. Cystine renal stones
Correct answer: CJ

Dr. Khalid Yusuf El-Zohry – Sohag Teaching Hospital (01118391123) Page | 302
El-Zohry MRCP Questions Bank (Part 2) – Medical Masterclass 2010

D. Uric acid renal stones H. Cushing's disease


E. Calcium oxalate renal stones. I. Known hypertension treated with
bumetanide
Answer & Comments J. Activation of the renin-angiotensin
Correct answer: D system.

Urinary stones made of calcium oxalate or of Answer & Comments


cystine are radio-opaque and would be visible Correct answer: CG
on a plain radiograph: by contrast, those made
of uric acid are radiolucent.
Thiazide diuretics and loop diuretics such as
Adult polycystic kidney disease does not cause bumetanide promote renal potassium loss and
renal colic, excepting very rarely if severe so can cause hypokalaemia. Aldosterone
haematuria leads to clot colic, and the enhances sodium reabsorption and potassium
ultrasound findings are of cysts, which do not secretion in the distal nephron. Therefore,
cast acoustic shadows. conditions such as Conn's syndrome or
Cushing's disease or steroid therapy in which
Patients with uric acid stones always excrete
there is excess mineralocorticoid activity are
acid urine. Once the acute problem is dealt
associated with hypokalaemia. In Addison's
with, if necessary by urological intervention, it
disease there is a deficiency of aldosterone, so
will be important to advise this man to drink
there is hyperkalaemia. Spironolactone has a
enough to ensure a urinary volume of at least 2
similar effect because it antagonises the action
litres / day and to prescribe sufficient alkali
of aldosterone. Activation of the renin
(sodium or potassium bicarbonate) to achieve
angiotensin axis promotes aldosterone
urinary pH>6.2.
production. This can happen in renal artery
stenosis or accelerated phase hypertension.
[ Q: 97 ] MasterClass Part2
(2010) - Nephrology [ Q: 98 ] MasterClass Part2
A 69-year-old man with hypertension is found (2010) - Nephrology
to be hypokalaemic.
A 42-year-old woman presents with a plasma
Which TWO of the following could not account creatinine of 240micromoles/l and 3g of
for these findings? proteinuria per 24 hours. She weighs 200 kg. A
renal biopsy is not possible because of her size.
A. Known hypertension treated with a
thiazide diuretic What is the most likely diagnosis?
B. Renal artery stenosis A. minimal change nephropathy
C. Addison's disease B. focal segmental glomerulosclerosis
D. Steroid therapy C. membranous nephropathy
E. Accelerated phase hypertension D. Bartter's syndrome
F. Conn's syndrome E. myeloma.
G. Known hypertension treated with
spironolactone

Dr. Khalid Yusuf El-Zohry – Sohag Teaching Hospital (01118391123) Page | 303
El-Zohry MRCP Questions Bank (Part 2) – Medical Masterclass 2010

Answer & Comments around 20%. Lymphoma is greatly increased,


especially Epstein-Barr virus driven tumours.
Correct answer: B
Early rises in plasma creatinine can be due to
acute tubular necrosis (ATN), calcineurin
There is a recognised association between toxicity, infection or rejection.
severe obesity and focal segmental
glomerulosclerosis.
[ Q: 100 ] MasterClass Part2
(2010) - Nephrology
[ Q: 99 ] MasterClass Part2
(2010) - Nephrology A 68-year-old woman with a 15-year history of
active rheumatoid arthritis presents because of
You are talking to a 42-year-old man with end- worsening oedema over the last 6 weeks.
stage renal failure and his wife regarding the Recent treatment has included diclofenac,
possibility of renal transplantation. penicillamine and gold. Cardiovascular
examination is unremarkable. She is found to
Which one of the following statements
have proteinuria, quantitated at 4.1g per 24
regarding renal transplantation is true?
hours, and her serum albumin is 28 g/l. Changes
A. it is not ethical to allow donation except are found at light microscopy on her renal
for donor-recipient pairs who are proven biopsy.
relations
Which TWO of the following statements are
B. acute rejection rates of > 50% are typical
correct?
C. the rate of lymphoma is increased
A. The biopsy would be expected to show
compared to dialysis patients
Kimmelstiel-Wilson lesions.
D. a rise in plasma creatinine in the first 2
B. Treatment with non-steroidal anti-
weeks is invariably due to rejection
inflammatory drugs may have caused
E. delayed graft function beyond four weeks membranous nephropathy.
will always result in graft loss.
C. Linear staining of the basement
membrane with antibodies to IgG would
Answer & Comments be expected to be present.
Correct answer: C D. A cellular tubulo-interstitial infiltrate is
likely.
Because of the increasing mismatch between E. Staining of the biopsy with antibodies
cadaveric donors and needy recipients, many against C'3 would reveal a granular
approaches have been devised towards pattern.
expanding the donor pool. These include non-
heart beating donation, and increasing the use F. Penicillamine therapy may have provoked
of live-related and live-unrelated donors. In the a mesangiocapillary glomerulonephritis.
situation of living donation, there is typically G. Gold therapy may have induced a
less good tissue-type matching than in membranous nephropathy.
cadaveric donation, but any disadvantage is
H. Congo red positivity on the renal biopsy
heavily outweighed by the excellent quality of
excludes a diagnosis of renal amyloidosis.
the donated kidneys (by design). With modern
anti-rejection drugs (rapamycin, mycophenylate I. In the absence of amyloid deposition the
mofetil, tacrolimus), acute rejection rates are patient's renal function would be

Dr. Khalid Yusuf El-Zohry – Sohag Teaching Hospital (01118391123) Page | 304
El-Zohry MRCP Questions Bank (Part 2) – Medical Masterclass 2010

expected to deteriorate progressively to A. Type 2 (proximal) renal tubular acidosis is


end-stage. often seen in conjunction with Fanconi
syndrome
J. The silver stain would be expected to
reveal evidence of mesangial B. In type 1 (distal) renal tubular acidosis,
proliferation. plasma potassium levels tend to be low
C. Larger doses of oral sodium bicarbonate
Answer & Comments are normally required in the treatment of
Correct answer: EG type 1 (distal) renal tubular acidosis than
type 2 (proximal) renal tubular acidosis

This patient has nephrotic syndrome and in this D. Type 4 renal tubular acidosis is associated
clinical setting the most likely diagnoses are with hyperkalaemia
membranous nephropathy and renal amyloid. E. Mineralocorticoid treatment is often
Membranous nephropathy can be precipitated helpful in type 4 renal tubular acidosis
by drug treatment with penicillamine and gold.
Histologically, it is characterised by granular F. Type 2 (proximal) renal tubular acidosis is
immune complex deposition in the basement associated with rickets and osteomalasia
membrane, indicated by granular staining with G. Type 1 (distal) renal tubular acidosis is a
antibodies against IgG and C'3. The deposits cause of nephrocalcinosis
lead to a characteristic "spiky" appearance of
H. Renin levels are low in type 4 renal
the basement membrane on staining with
tubular acidosis
silver. Without treatment roughly one-third of
patients with idiopathic membranous I. Lithium treatment often leads to type 4
nephropathy progress to end-stage renal renal tubular acidosis
failure. In drug-induced cases there is often J. Diabetic nephropathy commonly leads to
improvement following withdrawal of the type 4 renal tubular acidosis
precipitating agent.
Amyloid deposition leads to Congo red Answer & Comments
positivity and apple green birefringence.
Correct answer: CI
Kimmelstiel-Wilson lesions are found in diabetic
nephropathy. Linear staining of the basement
membrane with IgG is seen in anti-GBM disease Type 1 (distal) renal tubular acidosis results
and in some diabetic kidneys. Non-steroidal from impaired urinary acidification whereas
anti-inflammatory drug treatment sometimes type 2 (proximal) renal tubular acidosis is
leads to nephrotic syndrome, but the classical caused by a failure of bicarbonate reabsorption;
histological appearances are those of minimal both are associated with hypokalaemia. Type 2
change disease. (proximal) renal tubular acidosis, may occur in
isolation but is often associated with the
Fanconi syndrome and can be induced by heavy
[ Q: 101 ] MasterClass Part2
metals, acetazolamide or sulphonamides. Type
(2010) - Nephrology 1 (distal) renal tubular acidosis can be induced
by treatment with lithium or amphotericin.
In patients with renal tubular acidosis, which of Larger doses of oral sodium bicarbonate are
the following statements are FALSE? normally required in the treatment of type 2
(proximal) renal tubular acidosis than type 1
(distal) renal tubular acidosis. Type 4 renal

Dr. Khalid Yusuf El-Zohry – Sohag Teaching Hospital (01118391123) Page | 305
El-Zohry MRCP Questions Bank (Part 2) – Medical Masterclass 2010

tubular acidosis arises as a result of G. If your diagnosis is correct, plasma


hyporeninaemic hypoaldosteronism, which exchange should be offered as soon as
commonly occurs as a result of diabetic possible.
nephropathy, gouty nephropathy, urinary tract
H. The patient may well need dialysis within
obstruction or treatment with non-steroidal
24 hours.
anti-inflammatory drugs or potassium sparing
diuretics. It is associated with hyperkalaemia I. The presence of antibodies to double
and can be effectively treated with stranded DNA would support your
mineralocorticoids. diagnosis.
J. Typically in post-streptococcal
[ Q: 102 ] MasterClass Part2 glomerulonephritis circulating
(2010) - Nephrology complement levels are normal.

A 25-year-old man has been feeling unwell for a Answer & Comments
few days and has noticed his urine is darker
than usual. His GP finds that he is hypertensive Correct answer: CH
(BP 160/100), his urine is positive for blood and
protein, and his creatinine is 940 µmol/l. The The patient certainly has renal failure. Whether
patient remembers having a sore throat about a this is acute or chronic, he is very likely to need
week before becoming ill. You consider post- dialysis within 24 hours. Indeed, emergency
streptococcal glomerulonephritis to be a likely dialysis may be indicated depending on
diagnosis. knowledge of the potassium, pH, the clinical
status of the patient and whether there is a
Select the TWO answers that you agree with. pericardial rub or a flap. Sore throats are
A. A renal biopsy showing a paucimmune common, and it may be that the patient has
focal segmental glomerulonephritis with chronic renal failure, and the sore throat is a
crescent formation would support your coincidence. Post-streptococcal nephritis
diagnosis. generally resolves spontaneously and the
likelihood of requiring long term renal
B. If your diagnosis is correct the patient has
replacement therapy if this is the underlying
an approximately 85% likelihood of
diagnosis is probably about 1%. Post-
needing longterm renal replacement
streptococcal nephritis has a diffuse
therapy.
proliferative appearance on light microscopy,
C. The patient could have chronic renal and is associated with the presence of
failure with small kidneys on renal antibodies to streptococcal antigens and
ultrasound. hypocomplementaemia, both of which are
D. Red cell casts in the urine would make important in making the diagnosis.
post-infectious nephritis unlikely.
E. It would be reasonable to offer the
[ Q: 103 ] MasterClass Part2
patient an outpatient appointment within (2010) - Nephrology
1 week and ask the GP to check the
A 72-year-old man who presented with an
creatinine in 24 hours.
acute myocardial infarction 7 days previously,
F. Serological tests are generally unhelpful and who had received tPA as thrombolysis,
in diagnosing post-streptococcal developed 'trash feet' and acute disturbance of
nephritis. renal function.

Dr. Khalid Yusuf El-Zohry – Sohag Teaching Hospital (01118391123) Page | 306
El-Zohry MRCP Questions Bank (Part 2) – Medical Masterclass 2010

Which of the following statements are valid? [ Q: 104 ] MasterClass Part2


A. A renal biopsy is mandatory. (2010) - Nephrology
B. A renal angiogram would be contra- A 30-year-old man, a haemodilaysis patient
indicated. with diabetic nephropathy, has the following
C. Peripheral blood eosinophilia would be blood results: calcium 2.9 mmol/l, phosphate
surprising in this context. 2.5 mmol/l. Alkaline phosphatase and albumen
are in the normal range and PTH is at the lower
D. Both renal arteries must be diseased. end of the normal laboratory range. The
E. Anti-coagulation is important to preserve patient's current medication includes
renal function. alfacalcidol each day and calcium acetate as a
phosphate binder before each meal.
F. Statin therapy is desirable.
G. Irreversible renal failure is very likely. Select the most appropriate statement.

H. Acute haemorrhage into aorto-renal A. Phosphate restriction in the diet should


complex atherosclerotic plaque is likely as be reinforced, a change of phosphate
the precipitant. binder may be necessary and alfacalcidol
should be reduced or stopped.
I. Primary vasculitis would explain the
myocardial infarction and the renal B. The dialysate calcium should be lowered
failure. to return the calcium to the normal
range.
J. Steroids would be important therapy in
this context. C. The calcium acetate dose should be
reduced to return the calcium to the
Answer & Comments normal range.
D. The patient has autonomous
Correct answer: FH
hyperparathyroidism.

It is likely there is generalized atherosclerosis E. These values are satisfactory for a


and the aorta is affected by ulcerated plaque. haemodialysis patient.

The thrombolytic agent, together with any anti-


Answer & Comments
platelet drugs administered, make
haemorrhage into this complex atherosclerotic Correct answer: A
lesion more likely, and acute thrombo-
embolism will then ensue, distally to the feet The calcium x phosphate product is high putting
and into the mesenteric/ renal arterial tree. the patient at risk of metastatic calcification.
Eosinophilia is sometimes seen, as is Lowering the phosphate (rather than the
consumption of complement, raised calcium) is the most important measure, as the
inflammatory markers, and acute renal elevated phosphate stimulates parathyroid
dysfunction. proliferation and PTH secretion. This is best
done by reinforcing dietary restriction and
Further anti-coagulation can be disastrous.
ensuring that the patient is recieving adequate
Aggressive statin therapy is warranted for the dialysis. The vitamin D analogue will be
heart and the kidneys increasing absorption of calcium and phosphate
so this should be reduced or stopped; ideally

Dr. Khalid Yusuf El-Zohry – Sohag Teaching Hospital (01118391123) Page | 307
El-Zohry MRCP Questions Bank (Part 2) – Medical Masterclass 2010

the patient will remain on a small dose as this [ Q: 106 ] MasterClass Part2
helps to stop parthyroid proliferation. (2010) - Nephrology
If these changes do not return the calcium and
phosphate to satisfactory values, the patient Which of the following statements concerning
should be changed to a phosphate binder which patients with Anderson–Fabry disease is
does not contain calcium. Aluminium based untrue?
compounds are best avoided because of the A. Angiokeratomas particularly affect the
risk of toxicity so sevelamer would be an face
appropriate (but expensive) choice.
B. It is inherited in an X linked fashion

[ Q: 105 ] MasterClass Part2 C. Females can be affected


(2010) - Nephrology D. The defect is in lysosomal alpha-
galactosidase A
A 19-year-old woman, who is normally well,
attends her local emergency department E. Painful peripheral neuropathy is a
complaining of a burning sensation on passing common feature.
urine. She also mentions that the urine looks
cloudy and has a foul smell. Answer & Comments

Which of the following are true? Correct answer: A

A. Urinary tract infection with a fungus is


The incidence is estimated at 1:40 000. The
the likely explanation
angiokeratomas typically affect the ‘bathing
B. Urinary tract stones are probably present trunk’ area, rather than the face. Although
C. An urgent potassium level should be regarded as X linked recessive, it is now clear
obtained that female carriers can be severely affected.

D. Urinary tract infection with a


streptococcal species is a common cause [ Q: 107 ] MasterClass Part2
of this clinical presentation (2010) - Nephrology
E. An underlying anatomical abnormality is
The following are NOT used for the estimation
not usually present in women of this age
of renal function:
with this clinical presentation
A. Serum creatinine
Answer & Comments B. Serum cystatin C

Correct answer: E C. Creatinine clearance


D. Ethylenediaminetetraacetic acid (EDTA)
This clinical presentation is common and is clearance
typical of lower urinary tract infection.
E. Urinary albumin/creatinine ratio.
Common organisms include E. coli, Proteus and
Klebsiella species. Fungal urinary tract infection
is very rare and in younger women with urinary Answer & Comments
tract infection, the presence of stones or an Correct answer: E
anatomical abnormality would be unusual.

Dr. Khalid Yusuf El-Zohry – Sohag Teaching Hospital (01118391123) Page | 308
El-Zohry MRCP Questions Bank (Part 2) – Medical Masterclass 2010

Serum creatinine is the most commonly used The most likely cause is an acute interstitial
screening test for renal function. Cystatin C is nephritis. Common causes of this are antibiotics
currently being evaluated and may prove to be (particularly penicillins, cephalosporins,
more accurate in mild renal impairment. sulphonamides and rifampicin), NSAIDs, proton
Creatinine clearance is used as a surrogate test pump inhibitors, and allopurinol.
for measuring the GFR. The clearance of the
radioisotope EDTA however is the most [ Q: 109 ] MasterClass Part2
accurate marker of GFR in routine clinical
(2010) - Nephrology
practice. The albumin / creatinine ratio is a
method of quantifying urinary protein losses A 27-year-old primigravida with no past medical
and not an estimate of renal function. history is found to have glycosuria on urinalysis
at 24-weeks gestation.
[ Q: 108 ] MasterClass Part2
In this setting which TWO of the following
(2010) - Nephrology statements certainly apply?
A 68-year-old man presents with a red itchy A. Glycosuria indicates the presence of
rash over his trunk and limbs that has been intrinsic renal disease.
present for 5 days. His plasma creatinine is 327
B. Methyldopa would be an appropriate
μmol/L (normal range 60-115), but had been
drug to use to treat hypertension at this
only 98 μmol/L a month ago. He brought a large
gestational stage.
bag of drugs with him.
C. Glycosuria indicates the presence of
Which TWO of the following drugs are the most gestational diabetes.
likely causes of his acute renal failure?
D. Her plasma creatinine is likely to be
A. Thyroxine higher than before her pregnancy.
B. Prednisolone E. Unless therapy is started, glycosuria is
C. Atenolol likely to lead to a sub-optimal obstetric
outcome.
D. Allopurinol
F. Glycosuria indicates the presence of a
E. Simvastatin
urinary tract infection.
F. Doxazosin
G. She is likely to have microvascular
G. Digoxin complications of diabetes mellitus.
H. Amlodipine H. Glycosuria is a false positive result arising
from the presence of beta hydroxy
I. Amoxicillin
butyrate in her urine.
J. Aspirin
I. Finding her blood pressure to be
consistently higher than at booking
Answer & Comments should give cause for concern.
Correct answer: DI J. Her HbA1C will be abnormal.

He has a rash and recent-onset renal Answer & Comments


impairment.
Correct answer: BI

Dr. Khalid Yusuf El-Zohry – Sohag Teaching Hospital (01118391123) Page | 309
El-Zohry MRCP Questions Bank (Part 2) – Medical Masterclass 2010

In the mid-trimester of pregnancy GFR normally Allopurinol, if tolerated, will reduce urate
rises, resulting in a fall in plasma creatinine production, and increased dialysis will improve
level. The elevated GFR may lead to the urate clearance, both reducing the serum urate
capacity of the tubules to reabsorb filtered level and thus the predisposition to attacks.
glucose being exceeded and consequent
Prednisolone and colchicine both have anti-
glycosuria.
inflammatory properties, beneficial in the
Blood pressure normally falls in the mid- treatment of acute attacks.
trimester so an elevation is a cause for concern.
Drugs regarded as safe for the treatment of [ Q: 111 ] MasterClass Part2
hypertension in pregnancy include methyldopa,
(2010) - Nephrology
amlodipine / nifedipine and hydralazine.
Glycosuria may predispose to urinary infection, You are called to see a woman who is in her
but this would be indicated by the presence of thirtieth week of pregnancy and has
leukocytes and nitrites on urinalysis, not proteinuria.
glycosuria. Ketones do not cross-react with the
Which of the following findings would make you
glucose test strip on urinalysis. Glycosuria may,
suspect that the renal protein leak is not due to
of course, occur because of intrinsic renal
pre-eclampsia?
disease, gestational or conventional diabetes
mellitus, and in the latter case associated A. Hypertension
abnormalities such as microvascular B. Proteinuria at the pregnancy booking visit
complications, an elevated HbA1C and a poor
obstetric outcome may occur, but none of C. Oedema
these are certain in this setting. D. High serum urate
E. It is her first pregnancy
[ Q: 110 ] MasterClass Part2
(2010) - Nephrology Answer & Comments
A haemodialysis patient is troubled by gout. Correct answer: B

Which of the following treatments is unlikely to


Pre-eclampsia is commoner in first pregnancies,
prove effective?
and is characterised by oedema, proteinuria,
A. Allopurinol hypertension and an elevated serum urate.
B. Prednisolone Proteinuria at the booking visit indicates
underlying renal disease that was present
C. Colchicine
before the pregnancy.
D. Probenecid
E. Increased dialysis. [ Q: 112 ] MasterClass Part2
(2010) - Nephrology
Answer & Comments
A 68-year-old lady presents with a 4-day history
Correct answer: D of anuria. She has a past history of intermittant
claudication and is a long-term smoker.
Probenecid is a uricosuric agent, likely to
Which of the following statements are correct?
produce little benefit in a patient with severe
renal impairment necessitating haemodialysis.

Dr. Khalid Yusuf El-Zohry – Sohag Teaching Hospital (01118391123) Page | 310
El-Zohry MRCP Questions Bank (Part 2) – Medical Masterclass 2010

A. If kidneys are of normal size renal artery hepatitis serology was negative and a
occlusion can be excluded. coagulation screen was normal. Autoantibody
screening including anti-neutrophil cytoplasmic
B. If there is evidence of renal asymmetry
antibodies (ANCA) was normal. An MRCP was
and first investigation should be a biospy
unremarkable. Urinalysis consistently
of the smaller kidney.
demonstrated 4+ proteinuria on several
C. An IVU would be the investigation of samples but was negative for blood. A spot
choice. protein:creatinine ratio was markedly elevated
D. One should consider urgent at 755 mg/mmol (NR 0- 20). An examination
arteriography and revascularisation. demonstrated considerable volume overload
with an elevated JVP, signs of ascites and
E. Weakly positive pANCA serology would substantial peripheral oedema. Her blood
establish a diagnosis of microscopic
pressure was low at 80/54.
polyangitis.
Which TWO of the following statements would
Answer & Comments represent the most appropriate next course of
action?
Correct answer: D
A. Start oral prednisolone 0.5mg/kg and oral
cyclophosphamide 2mg/kg
Absolute anuria would be a rare presentation of
a rapidly progressive glomerulonephritis. A B. Start dopamine 4ug/kg/min
weakly positive pANCA is a relatively common C. Liver biopsy
finding and in many instances is a false positive
result. One would need histology to confirm the D. 24-hour urine collection for protein
diagnosis in these circumstances. Renal artery estimation
occlusion can present in this way and the E. Start oral prednisolone 0.5mg/kg
presence of two normal size kidneys does to
F. Echocardiogram
exclude the diagnosis. An IVU is not helpful in
these instances, as non-opacification of the G. Renal biopsy
kidneys would occur in the presence of poor
H. ERCP
renal function irrespective of the cause. When
there is renal asymmetry the larger of the two I. Start N-Acetyl Cysteine infusion.
kidneys should be biopsied as the diagnostic J.
yield is likely to be greater. Arteriography and
revascularisation should be considered as this
Answer & Comments
may well salvage the situation is carried out
early enough. Correct answer: FG

[ Q: 113 ] MasterClass Part2 This patient has nephrotic syndrome (significant


(2010) - Nephrology proteinuria with hypoalbuminaemia) in the
context of a liver disease of unknown aetiology.
A 42-year-old woman was originally referred to The differential is fairly wide but the three
the Gastroenterology team with a history of commonest single renal diagnoses in this age
significant weight loss, diarrhoea with abnormal group would be minimal change disease,
liver enzyme tests and a low serum albumin. An membranous nephropathy and amyloidosis.
abdominal CT scan showed that she had She is volume overloaded but hypotensive.
hepatomegaly and mild splenomegaly. Viral Excluding a significant pericardial effusion will

Dr. Khalid Yusuf El-Zohry – Sohag Teaching Hospital (01118391123) Page | 311
El-Zohry MRCP Questions Bank (Part 2) – Medical Masterclass 2010

be important prior to volume control with polyuric when his obstruction is relieved. In
diuretics and salt restriction. The most fruitful some cases there is obstruction of the ureters
diagnostic procedure is a renal biopsy. as well as bladder outflow (eg with infiltrating
Immunosuppressive treatment is inappropriate bladder carcinoma), in which case
until a clear diagnosis supporting its use is nephrostomies may be needed if an upper tract
established. obstruction is not relieved with an indwelling
catheter.
The renal biopsy in this case confirmed
amyloidosis and the ECHO was consistent with
associated cardiac involvement. A subsequent [ Q: 115 ] MasterClass Part2
bone marrow examination demonstrated an (2010) - Nephrology
excess of plasma cells confirming this as a case
of primary AL amyloid associated with A 23-year-old African woman presents with
myeloma. The prognosis is generally poor - seizures, hypertension, a rash, a raised ESR, a
especially in those with cardiac involvement, normal CRP and a creatinine of 373
and despite chemotherapy this patient micromoles/l.
developed renal failure and died 4 months after
What is the most likely diagnosis?
diagnosis.
A. Myeloma
[ Q: 114 ] MasterClass Part2 B. Hashimoto’s disease
(2010) - Nephrology C. Systemic lupus erythematosus
A 70-year-old man with recent fatigue is found D. Staphylococcal septicaemia
to have a serum creatinine of 800 μmol/L and
E. Sickle cell disease.
potassium 6.2 mmol/L. Renal ultrasound shows
urine in the bladder following micturition
(estimated volume 600 mL) and bilateral Answer & Comments
pelvicalyceal dilatation. His BP is 110/80 mmHg, Correct answer: C
there is no peripheral oedema, his JVP is not
elevated and his chest is clear on auscultation.
Systemic lupus erythematosus is a common
condition and the prevalence is higher in
Which of the following do you think is the most
appropriate initial treatment? women than men and is higher in black people
than white. Typical features include skin rashes,
A. Haemodialysis neurological or psychiatric abnormalities and
B. Bilateral antegrade nephrostomies renal disease. It is often found that although
the ESR is elevated, the CRP is normal.
C. Transurethral prostatectomy
D. Urethral catheterisation [ Q: 116 ] MasterClass Part2
E. 0.9% sodium chloride 500 mL IV fast, then (2010) - Nephrology
reassess
A 45-year-old man is found to have a positive
urine dipstick for blood at a routine medical
Answer & Comments
examination. He has no significant past medical
Correct answer: D history, has no symptoms, and is fit and well.

What are the TWO most likely diagnoses?


He should be catheterised and then given
intravenous fluids. He may well become A. Renal carcinoma

Dr. Khalid Yusuf El-Zohry – Sohag Teaching Hospital (01118391123) Page | 312
El-Zohry MRCP Questions Bank (Part 2) – Medical Masterclass 2010

B. Alport’s syndrome
C. Urinary stones
D. Adult polycystic kidney disease
E. Bladder cancer
F. Minimal change glomerulonephritis
G. Loin pain haematuria syndrome
H. Diabetes mellitus
I. IgA nephropathy
Which one of the following statements is
J. Thin membrane nephropathy. correct?
A. The appearances would be consistent
Answer & Comments with cholesterol embolisation.
Correct answer: IJ B. The appearances are secondary to
proteinuria.
Everyone has some red blood cells in their C. A portion of a normal glomerulus is seen
urine. Various studies have shown that 2.5-13% in the biopsy.
of men have positive urine dipstick tests for
D. The procedure was probably complicated
blood, the vast majority of whom do not have
by haemorrhage.
serious urinary tract or renal pathology.
E. There is evidence of interstitial fibrosis.
The younger the patient, the more likely that
haematuria is glomerular in origin, with IgA
nephropathy and thin membrane nephropathy Answer & Comments
the two commonest causes. Urinary tract
Correct answer: D
tumours become more likely in older patients.
20-30% of patients with IgA nephropathy Percutaneous renal biopsy aims to obtain
progress to end-stage renal failure in 20 years. samples of the renal cortex. Samples of renal
There is no effective specific treatment, medulla are not only less likely to yield
although some advocate the use of dietary fish diagnostic information, but also carry a
oil supplements. Management involves the use substantially increased risk of haemmorhage,
of angiotensin-converting enzyme (ACE) particularly if the renal pelvis is breached by the
inhibitors, which retard the rate of progression biopsy needle - as has clearly occurred in this
of all types of proteinuric renal disease. case since the sample includes transitional
epithelium.
[ Q: 117 ] MasterClass Part2
(2010) - Nephrology [ Q: 118 ] MasterClass Part2
The enclosed image shows a portion of a
(2010) - Nephrology
percutaneous renal biopsy taken from a 48- A 60-year-old man who weighs 70kg attends
year-old woman with nephrotic syndrome. This outpatients because of shortness of breath. He
includes medulla and transitional cell looks pale and he has a blood pressure of
epithelium. 142/81mmHg, an elevated JVP and mild
oedema, but no abnormalities of the

Dr. Khalid Yusuf El-Zohry – Sohag Teaching Hospital (01118391123) Page | 313
El-Zohry MRCP Questions Bank (Part 2) – Medical Masterclass 2010

respiratory system are found. Blood tests reveal Answer & Comments
haemoglobin of 8.1g/dl; creatinine of 200
Correct answer: IJ
micromoles per litre; calcium of 3.1 mmol/l;
albumin of 28 g/l. Urinalysis shows 1+
proteinuria. He is treated with a diuretic In steady state, the Cockcroft and Gault formula
pending the results of further blood tests can be used to relate serum creatinine, age, sex
including haematinics. He returns a week later and weight to creatine clearance. In this case,
and his creatinine is found to be 800 at presentation creatinine clearance is 1.23 x
micromoles per litre. (140-60) x 70/200 = 35mls/min. His creatinine
has risen steeply over the week between visits
Which TWO of the following statements are so he will not be in steady state at the second
correct? visit and although his creatinine has gone up
four-fold, his clearance will have fallen by a
A. His creatinine clearance at initial
factor greater than 4.
presentation was less than 20mls/min.
Myelomas that make only IgD or IgE or light
B. A diagnosis of myeloma kidney is
chains can affect the kidney. Normally there will
excluded by normal serum
be immuneparesis in these circumstances, but
immunoglobulin levels.
normal serum immunoglobulin levels do not
C. His creatinine clearance at the second exclude the diagnosis of myeloma. Bence Jones
visit is one-quarter of what it had been at proteins (free light chains) are not detected on
initial presentation. routine urinalysis and do not explain the 1+
D. Bence Jones proteins are detectable on proteinuria in this case.
urinalysis. Primary hyperparathyroidism could explain the
E. Primary hyperparathyroidism would renal impairment and hypercalcaemia, but not
explain his presentation. the anaemia. At his initial level of renal function
anaemia due to erythropoietin deficiency is
F. Symptomatic anaemia is to be expected
extremely unlikely, but an excess of plasma
given his renal impairment.
cells in the bone marrow may lead to anaemia
G. His calcium level can be explained by and in this setting the diseased marrow will not
increased bone turnover resulting from respond normally to erythropoietin.
secondary hyperparathyroidism. Secondary hyperparathyroidism is a feedback
H. A bone scan is an important investigation response to hypocalcaemia and as such will
in the assessment of a patient with tend to move the calcium level towards normal,
myeloma and renal impairment. but not overshoot.
I. An elevated alkaline phosphatase would Osteoblastic activity is reduced so bone scans
suggest a diagnosis of are usually negative in patients with myeloma.
hyperparathyroidism, whereas a normal Bone involvement is more usually assessed by a
alkaline phosphatase level would be more skeletal survey.
likely in myeloma.
J. If the underlying diagnosis is myeloma, [ Q: 119 ] MasterClass Part2
hyporesponsiveness to erythropoietin is (2010) - Nephrology
often seen.
A 68-year-old woman with a complex past
medical history presents with unexplained

Dr. Khalid Yusuf El-Zohry – Sohag Teaching Hospital (01118391123) Page | 314
El-Zohry MRCP Questions Bank (Part 2) – Medical Masterclass 2010

acute renal failure. Renal biopsy reveals acute Which one of the following statements is true
interstitial nephritis. about peritoneal dialysis?
A. it is inferior to haemodialysis
Which TWO of her many medications are the
most likely cause? B. it is associated with long-term changes in
peritoneal membrane structure and
A. Amoxicillin
function
B. Paracetamol
C. infection rates are typically more than 1
C. Ibuprofen episode of peritonitis per 6 months of
D. Aspirin treatment

E. Digoxin D. it is contra-indicated in diabetic patients

F. Enalapril E. it means patients are unable to go to


work because of dialysis exchanges.
G. Temazepam
H. Frusemide Answer & Comments
I. Co-proxamol Correct answer: B
J. Cinnarizine
Peritoneal dialysis (PD) was the bedrock of
Answer & Comments dialysis in the UK in the 1970s, and has become
so in other countries with inadequate health
Correct answer: AC resources. In wealthier countries it has been
slower to become established and accounts for
Non-steroidal anti-inflammatory agents and a far smaller proportion of treated patients.
antibiotics (particularly penicillins) are the
Actuarial survival is the same, or better, for PD
commonest causes of acute interstitial
in the early stages of dialysis, but once residual
nephritis. Other drugs that have been
renal function is lost, PD increasingly struggles
incriminated include allopurinol, cimetidine,
to dialyse many patients well.
frusemide, omeprazole, rifampicin and thiazide
diuretics (and many others have been Some motivated patients can work with co-
associated in occasional case reports). operation from employers. Travel on PD is also
much easier.
Aside from supportive care (e.g. dialysis if
required), treatment involves withdrawal of the Diabetic patients can fare well, though the
precipitating agent and moderate doses of oral glucose load from PD changes insulin
steroids (e.g. prednisolone 40 mg daily). The requirements.
efficacy of the latter has not been Peritonitis rates of one infection per 18 - 24
demonstrated in controlled trials.
patient months are now the norm with modern
systems.
[ Q: 120 ] MasterClass Part2
Automated peritoneal dialysis (shorter duration
(2010) - Nephrology high volume overnight exchanges while the
A 60-year-old man has end-stage renal failure patient is asleep) is more convenient for many
and is about to start renal replacement therapy. patients, and can extend the useful life of PD in
He has read a booklet about peritoneal dialysis, 'fast transporters', i.e. patients in whom the
but seems to have got himself muddled. osmotic gradient collapses quickly.

Dr. Khalid Yusuf El-Zohry – Sohag Teaching Hospital (01118391123) Page | 315
El-Zohry MRCP Questions Bank (Part 2) – Medical Masterclass 2010

[ Q: 121 ] MasterClass Part2 A. 0%


(2010) - Nephrology B. 25%
C. 33.3%
Which of the following would you regard as the
clearest contraindication to choosing D. 50%
continuous ambulatory peritoneal dialysis as E. 75%
treatment modality for a patient with endstage
renal failure?
Answer & Comments
A. Presence of large (22 cm) bilateral
polycystic kidneys Correct answer: B

B. Known impairment of left ventricular


Assuming he has autosomal dominant
function
polycystic kidney disease, each child has a 50%
C. Severe visual impairment risk of being affected, so the risk of both them
D. Previous succesful repair of an inguinal being affected is 25%.
herinia
E. Presence of a defunctioning colostomy [ Q: 123 ] MasterClass Part2
and mucus fistula. (2010) - Nephrology
A 67-year-old man is referred to the acute
Answer & Comments medical team. He has complained of feeling
Correct answer: E unwell for 2 weeks, with fevers, anorexia and a
purple rash on his legs. Blood tests performed
by his GP have shown a rise in creatinine from
Impaired left ventricular function is a more
127 to 254 μmol/L (normal <115 μmol/L),
significant concern for haemodialysis than
together with an elevated erythrocyte
continuous ambulatory peritoneal dialysis
sedimentation rate of 72. He has a past medical
(CAPD). Visual impairment raises some
history of ischaemic heart disease, peptic
challenges for CAPD, but these can be
ulceration and diverticulitis, and had also
overcome in most instances. Moderate size
undergone diagnostic coronary angiography 1
polycystic kidneys do not usually present a
month previously.
problem for CAPD.
What is the most likely diagnosis?
[ Q: 122 ] MasterClass Part2 A. Henoch-Schönlein purpura
(2010) - Nephrology
B. Churg-Strauss syndrome
A 38-year-old man with a family history of
C. Cholesterol emboli
autosomal dominant polycystic kidney disease
is referred with renal impairment and D. Multiple myeloma
hypertension. An ultrasound scan of the E. Contrast nephropathy
abdomen confirms bilateral large cystic kidneys.
He has a question concerning the risk to his
Answer & Comments
children.
Correct answer: C
What do you think the chance is that both of my
TWO children will have polycystic kidney
disease?'

Dr. Khalid Yusuf El-Zohry – Sohag Teaching Hospital (01118391123) Page | 316
El-Zohry MRCP Questions Bank (Part 2) – Medical Masterclass 2010

The history is typical of cholesterol antibodies against the ANCA antigens


embolisation following instrumentation in a myeloperoxidase and proteinase 3.
man with established arterial disease. As in this
case, it can occur after an interval of days to [ Q: 125 ] MasterClass Part2
weeks. The differential diagnosis is wide and
(2010) - Nephrology
would include antineutrophil cytoplasmic
antibody-associated vasculitis and an acute A 58-year-old man is reasonably fit and well,
interstitial nephritis. but has known chronic renal failure due to
reflux nephropathy. His appetite is good, but he
[ Q: 124 ] MasterClass Part2 feels tired. His blood pressure is 120/72 mmHg
on an ACE inhibitor.
(2010) - Nephrology
Investigations reveal plasma creatinine 320
A 53-year-old man presents having felt unwell
µmol/L (the value one year ago having been
for several weeks with general malaise and
280 µmol/L), potassium 5.6 mmol/L, urea 19.0
fatigue. He has had recurrent sinusitis for over a
mmol/L, phosphate 1.8 mmol/L, calcium 2.25
year and occasionally noticed a rash on his
mmol/L, haemoglobin 10.1 g/dL, ferritin 120
chest. Routine blood tests show that he has a
µg/L (normal). A 24 hour urinary collection
serum creatinine of 180micromoles/l.
contains 1.8g of protein.
Which of the following is the most likely
What treatment would you recommend?
diagnosis?
A. Commence dialysis within the next few
A. Chronic renal failure
days.
B. Systemic vasculitis
B. Start oral alfacalcidol at 0.25 mcg per day.
C. Minimal change nephropathy
C. Start recombinant human erythropoietin.
D. Urinary tract infection
D. Stop or reduce the dose of the ACE
E. Focal segmental glomerulosclerosis. inhibitor.
E. Advise dietary protein restriction.
Answer & Comments

Correct answer: B Answer & Comments

Correct answer: C
In any patient with a history of chronic
progressive worsening of symptoms and renal
There is no immediate indication for dialysis,
impairment, it is important to consider systemic
and a good appetite is very reassuring in this
vasculitis as a possible cause. In this case, the
context. He is likely to have significant
rash and sinusitis are further pointers. Sinusitis
hyperparathyroidism, due to
can arise from involvement of the nasal tract
hyperphosphataemia and reduced active
and sinuses in the Wegener's pattern of the
vitamin D. However, commencing alfacalcidol
disease. Often a patient with systemic vasculitis
will further increase his phosphate level, which
will have a long history of indolent disease, but
is undesirable. A more appropriate course is to
will then present late with severe aggressive
ensure that dietary phosphate (and potassium)
disease. Useful immunological tests include
intake is restricted, and to prescribe a
assays for anti-neutrophil cytoplasmic
phosphate binder (such as calcium acetate).
antibodies (ANCA) and direct assays for

Dr. Khalid Yusuf El-Zohry – Sohag Teaching Hospital (01118391123) Page | 317
El-Zohry MRCP Questions Bank (Part 2) – Medical Masterclass 2010

Blood pressure control, preferably with an ACE G. In polycystic kidney disease, the
(angiotensin converting enzyme) inhibitor or phenotype is more severe in the ~10% of
ARB (angiotensin receptor blocker) is the most families who have PKD2 mutations.
important measure in delaying progression of
H. In polycystic kidney disease, a single non-
renal disease. A reading of 120/72 is
sense mutation in the PKD1 gene
satisfactory and not too low. Restriction of
accounts for about 90% of cases.
dietary protein intake has not proved to be an
effective or practical strategy in humans to I. In the UK, autosomal dominant polycystic
prevent progression of renal disease. kidney disease accounts for more cases of
end stage renal failure than diabetic
This patient would almost certainly benefit
nephropathy
from rhEPO to correct his anaemia. The ferritin
above 100 µmol/L makes iron deficiency J. First trimester DNA analysis is generally
unlikely. Other causes of anaemia (eg B12 recommended for pregnancies where one
deficiency, haemolysis, myeloma) should be parent has autosomal dominant
considered, but at this level of renal function he polycystic kidney disease
will have relative EPO deficiency.
Answer & Comments
[ Q: 126 ] MasterClass Part2 Correct answer: AD
(2010) - Nephrology
A 26-year-old woman has recently become ADPKD does show marked age related
pregnant. She lost contact with her father as a penetrance; for example, most individuals do
child, but has been told that more than one not show any clinical manifestations before age
person in his family had kidney disease. She twenty. PKD1 mutations account for about 90%
asks you about kidney diseases that run in of cases, and each family generally has its own
families. mutation. This is one of the reasons why
genetic screening is usually not practical in
Which of the following statements do you agree families with ADPKD. In addition, most couples
with? feel that they would not wish to terminate a
pregnancy on the grounds of carrying an ADPKD
A. Autosomal dominant polycystic renal
mutation. Although ADPKD is not an
disease usually shows marked age-
uncommon cause of end stage renal failure, it is
dependent penetrance.
much less common than diabetic nephropathy.
B. Alport's syndrome is autosomal dominant
Reflux nephropathy shows a very strong familial
in most cases.
predispostion, which is why it is recommended
C. Anderson-Fabry disease is due to that children of affected patients are screened
glucocerebrosidase deficiency. soon after birth. Anderson-Fabry disease is due
D. Severe nephrotic syndrome may be due to alpha galactosidase deficiency. von Hippel
to mutations in the podocin or nephrin Lindau disease is autosomal dominant. Alport's
genes. syndrome is usually X linked.

E. Von Hippel Lindau disease is an X-linked


condition giving rise to renal cysts, renal
carcinoma and phaeochromocytoma.
F. There is little, if any, familial
predisposition to reflux nephropathy.

Dr. Khalid Yusuf El-Zohry – Sohag Teaching Hospital (01118391123) Page | 318
El-Zohry MRCP Questions Bank (Part 2) – Medical Masterclass 2010

[ Q: 127 ] MasterClass Part2 [ Q: 128 ] MasterClass Part2


(2010) - Nephrology (2010) - Nephrology
A 32-year-old professional body builder is A pregnant 35-year-old presents with right
referred to the renal clinic for investigation of upper quadrant pain at 28 weeks gestation. On
an elevated serum creatinine (121 umol/l). examination, her blood pressure is 180/105.
She has proteinuria, a creatinine of
Which of the following would reassure you that 120micromoles/litre and abnormal liver
this patients glomerular filtration rate (GFR) is function tests. Her blood film shows a
likely to be normal? microangiopathic haemolytic anaemia and her
A. A normal IVP platelet count is 30. A few weeks previously her
blood pressure was 110/80 and her platelet
B. An reduced creatinine clearance
count was 120.
calculated from a 24-hour urine collection
C. A normal MRA study of the renal arteries Which TWO of the following statements are
true?
D. A normal ultrasound examination
A. As a pregnant woman her blood pressure
E. A normal calculated creatinine clearance
would be expected to be higher than
using the Cockcroft Gault formula
before the pregnancy.
F. Normal autoantibody screen
B. The low platelets suggest that she has
G. A normal urinary dipstick systemic lupus erythematosus.
H. A normal isotope GFR result C. She certainly has essential hypertension.
I. A dietary history of a high protein intake D. The development of proteinuria suggests
pre-eclampsia.
J. An Hb of 18.2g/dl.
E. Blood pressure is usually higher in the
Answer & Comments first half than the second half of
pregnancy.
Correct answer: EH
F. The problem is unlikely to recurr in a
subsequent pregnancy.
Serum creatinine is a useful marker of excretory
renal function but has significant limitations. In G. Alpha methyl dopa cannot safely be used
many patients with significant renal disease, to lower her blood pressure.
the creatinine is still within the 'normal' range. H. Grand mal seizures may occur if the
In a few patients, particularly well-built condition persists.
muscular young men, the creatinine may be
I. She requires urgent dialysis.
elevated but the GFR normal. Isotope methods
based on the elimination of labelled EDTA are J. Acute liver failure is inevitable.
very useful and probably the 'gold standard' of
clinical practice. The Cockcroft Gault formula Answer & Comments
offers a fairly accurate method of adjusting for
weight, age and sex and gives about an 85% Correct answer: DH
correlation with 'gold standard' methods. 24-
hour urine collections are often performed Blood pressure falls during the first half of
incorrectly and unreliable. pregnancy and then rises again during the
second half of pregnancy to levels approaching

Dr. Khalid Yusuf El-Zohry – Sohag Teaching Hospital (01118391123) Page | 319
El-Zohry MRCP Questions Bank (Part 2) – Medical Masterclass 2010

those before the pregnancy. Generally, blood E. Plasma exchange is not warranted as he is
pressure is a little higher in the second half than not dialysis-dependent.
the first half of pregnancy. These changes
F. If the patient requires dialysis he will not
probably reflect changes in peripheral
recover renal function.
resistance.
G. The patient must be ventilated without
Pregnancy induced hypertension can be severe
delay.
and can be associated with endothelial and
organ dysfunction. The clinical consequences of H. A renal biopsy is warranted.
such dysfunction can be varied and certainly I. Raised CRP indicates infection as the
include proteinuria, renal impairment, underlying aetiology.
abnormal liver function, microangiopathic
J. The patient must be managed in an HDU
haemolytic anaemia, platelet dysfunction and
or ITU setting.
seizures. The priority is to reduce the blood
pressure to prevent these changes. Aspirin is of
prophylactic benefit in subsequent pregnancies, Answer & Comments
which carry a significant risk of recurrence.
Correct answer: HJ
Alpha methyl dopa is a standard well tested
drug used during pregnancy. Labetolol is also The diagnosis could certainly be Goodpasture's
used. Other drugs, such as calcium channel - but detection of circulating anti-GBM
blockers, have been used, but have a less well- antibodies and a renal biopsy (or much more
established track record of safety during rarely ante-mortem lung biopsy) to see the
pregnancy linear IgG immunofluoresence are required to
establish this diagnosis. Alternative diagnoses
[ Q: 129 ] MasterClass Part2 are microscopic polyangiitis, Wegener's, or a
(2010) - Nephrology number of rarer causes of 'pulmonary renal
syndrome'.
A 21-year-old man presents with a short history
A negative ANCA makes systemic vasculitis
of haemoptysis, breathlessness and malaise. On
unlikely, but does not exclude it in a convincing
investigation he has patchy shadowing in both
clinical situation (such as this one).
lung fields on a chest radiograph, a pO2 on
room air of 6.7 KPa, renal impairment Patients with anti-GBM disease rarely recover
(creatinine 480 umol/l), raised CRP (54 mg/l, NR renal function once dialysis dependent, but
< 5), oliguria (urine output 400 mls/day). ANCA-positive patients frequently do.
A renal biopsy is an important investigative and
Which of the following TWO statements are
diagnostic procedure in these cases, providing
true?
diagnostic and prognostic information.
A. The diagnosis is Goodpasture's syndrome. However, it may not be possible to safely
B. Treatment must not commence until an biopsy this man in his current predicament, and
anti-neutrophil cytoplasmic antibodies his treatment is a medical emergency and
(ANCA) is back. should not be delayed whilst waiting for
serological results or for a biopsy to be done.
C. A negative ANCA excludes systemic
vasculitis. Raised CRP is a feature of both infection (which
certainly could be playing a role) as well as
D. Pulsed methylprednisolone is an essential vasculitis.
therapy.

Dr. Khalid Yusuf El-Zohry – Sohag Teaching Hospital (01118391123) Page | 320
El-Zohry MRCP Questions Bank (Part 2) – Medical Masterclass 2010

The MEPEX trial has compared pulsed D. If she has polycystic kidneys she should
methylprednisolone with plasma exchange in receive rotating chemoprophylaxis to
this setting - both are valuable, but most prevent urinary tract infection.
nephrologists would recommend plasma
E. Her father should probably be screened
exchange in this dramatic clinical setting, since
for the presence of intra cerebral
it is the fastest way of removing antibodies that
aneurysms with magnetic resonance
are believed (and in anti-GBM disease proven)
angiography (MRA).
to be pathogenic. This would be given in
conjunction with treatments designed to F. Women with polycystic kidney disease
reduce further antibody production, typically lose GFR at approximately twice the rate
cyclophosphamide and steroids (typically that men do.
prednisolone 60 mg / day or equivalent). G. The most likely genetic defect is a
This patient is suffering from a life-threatening complete deletion of PKD1.
condition, with pulmonary bleeding being a H. Oral contraceptives can be prescribed to
terrifying complication for patient and medical / patients with polycystic kidney disease.
nursing staff alike. He needs to be very closely
monitored indeed and hence must be cared for I. Angiotensin-converting enzyme (ACE)
on an HDU / ITU. If possible ventilation should inhibition reduces the rate of GFR loss to
a comparable extent in polycystic kidney
be avoided as barotrauma provokes more
disease and diabetic nephropathy.
pulmonary capillary rupture and bleeding, but
obviously such patients have to be ventilated J. Ultrasound is more sensitive than CT for
sometimes (and can recover thereafter). the detection of small renal cysts.

[ Q: 130 ] MasterClass Part2 Answer & Comments


(2010) - Nephrology Correct answer: EH
A 20-year-old woman is referred because her
father has recently been found to have Four to ten percent of patients with autosomal
advanced chronic kidney disease (creatinine dominant polycystic kidney disease (ADPKD)
450 micromol/l) with ultrasound scan showing will have cerebral aneurysms and the risk of
enlarged kidneys with multiple cysts. Her subarachnoid haemorrhage is higher in those
paternal grandfather died of a ‘brain with a family history. Current advice is to screen
haemorrhage’ at the age of 35. The woman has ADPKD patients with a family history of SAH.
two children aged three and one. Since the paternal grandfather probably died of
SAH, it would be appropriate for the father to
Which of the following are correct? be screened with MRA. Eighty-five percent of
A. Genetic testing for a defect in polycystin ADPKD is due to mutations in PKD1. Each family
(PKD1) is likely to be the most useful test with ADPKD tends to have a different mutation
in this setting. in PKD1, and it is a large gene making
identification of the mutation difficult. Linkage
B. If renal ultrasound shows no cysts, she analysis can be undertaken in large pedigrees,
should be reassured that she has not but screening is based on the identification of
inherited the genetic defect. renal cysts. However, these may not develop
C. Her children should have renal ultrasound until the third decade in some cases, so there is
scans performed promptly. a 4% risk of ADPKD in this patient if her scan
shows no cysts at the age of 20.

Dr. Khalid Yusuf El-Zohry – Sohag Teaching Hospital (01118391123) Page | 321
El-Zohry MRCP Questions Bank (Part 2) – Medical Masterclass 2010

Although CT scan is more sensitive than there is excess mineralocorticoid activity are
ultrasound in detecting small cysts, early associated with hypokalaemia. In Addison's
diagnosis of ADPKD is not generally an issue disease there is a deficiency of aldosterone, so
since the main intervention is simply to control there is hyperkalaemia. Spironolactone has a
blood pressure. Interestingly antihypertensive similar effect because it antagonises the action
treatment and ACE-inhibition seems to have of aldosterone. Activation of the renin
substantially less effect on the rate of loss of angiotensin axis promotes aldosterone
GFR than in other chronic kidney diseases. production. This can happen in renal artery
stenosis or accelerated phase hypertension.
[ Q: 131 ] MasterClass Part2
(2010) - Nephrology [ Q: 132 ] MasterClass Part2
(2010) - Nephrology
A 69-year-old man with hypertension is found
to be hypokalaemic. A 32-year-old man presents to his GP with a
non-specific history of malaise. He has no
Which TWO of the following could not account history of diarrhoea. Initial investigations
for these findings? identify a significant anaemia with
A. Known hypertension treated with a fragmentation on the blood film and low
thiazide diuretic platelets. A clotting screen is normal. His
biochemistry identifies significant renal
B. Renal artery stenosis
dysfunction with a serum creatinine of
C. Addison's disease 260umol/l. BP was recorded at 170/100.
D. Steroid therapy
What is the most likely diagnosis?
E. Accelerated phase hypertension
A. Thrombotic thrombocytopenic purpura
F. Conn's syndrome (TTP)
G. Known hypertension treated with B. Haemolytic uraemic syndrome
spironolactone
C. Malignant hypertension
H. Cushing's disease
D. Disseminated intravascular coagulation
I. Known hypertension treated with
E. Systemic vasculitis.
bumetanide
J. Activation of the renin-angiotensin Answer & Comments
system.
Correct answer: B
Answer & Comments
This is a classic presentation of non-diarrhoeal
Correct answer: CG haemolytic uraemic syndrome (D- HUS) or
atypical HUS. This may be sporadic or familial.
Thiazide diuretics and loop diuretics such as In sporadic cases association with HIV,
bumetanide promote renal potassium loss and malignancy, systemic lupus erythematous (SLE)
so can cause hypokalaemia. Aldosterone and some drugs (e.g. cyclosporin) has been
enhances sodium reabsorption and potassium reported. In some familial cases mutations have
secretion in the distal nephron. Therefore, been detected in Factor H which regulates
conditions such as Conn's syndrome or alternative pathway activation of compliment.
Cushing's disease or steroid therapy in which TTP produces a similar picture but usually also

Dr. Khalid Yusuf El-Zohry – Sohag Teaching Hospital (01118391123) Page | 322
El-Zohry MRCP Questions Bank (Part 2) – Medical Masterclass 2010

has evidence of neurological involvement. Close monitoring is essential. Asymptomatic


There is also evidence that the mechanism bacteriuria should be treated, as there is a 30%
underlying the disease is different with reduced risk of pyelonephritis. Screening advice should
activity of a von Willebrand factor cleaving be offered as this condition is recognised to be
protease ADAMTS13. Treatment of sporadic hereditary with an autosomal dominant
HUS is with plasma exchange and fresh frozen pattern.
plasma. The renal outlook is relatively poor and
mortality around 10%. [ Q: 134 ] MasterClass Part2
(2010) - Nephrology
[ Q: 133 ] MasterClass Part2
A 37-year-old woman suffers from recurrent
(2010) - Nephrology
Klebsiella urinary tract infections. She now
A 32-year-old woman with a past history of presents to the Emergency Department
recurrent urinary tract infections and known complaining of severe right-sided flank pain. A
reflux nephropathy was seen in an antenatal spiral CT scan reveals a stone in the right ureter.
clinic at 28 weeks gestation. She has a BP of
150/92 and dipstick urine analysis is positive for What is the most likely chemical composition of
protein. She asks how this might affect her the stone?
pregnancy. A. Struvite (magnesium ammonium
phosphate)
Which of the following is correct?
B. Potassium citrate
A. Treatment of aymptomatic bacteriuria is
not required. C. Cystine

B. Vesicoureteric reflux is an inherited D. Calcium acetate


disease (autosomal recessive). E. Uric acid
C. Deterioration of renal function during
pregnancy is extremely unlikely. Answer & Comments
D. Standard pregnancy monitoring will be Correct answer: A
sufficient.
E. Vesicoureteric reflux is an inherited Struvite (magnesium ammonium phosphate)
disease (autosomal dominant). stones occur in patients with chronic infections
with bacteria expressing urease. They account
Answer & Comments for about 10% of all instances of stones. The
most common type of urinary stone is calcium
Correct answer: E oxalate.

Vesicoureteric reflux can lead to renal scarring [ Q: 135 ] MasterClass Part2


and damage and is often termed reflux
(2010) - Nephrology
nephropathy. Renal deterioration is usually
associated with hypertension and proteinuria. A 38-year-old man presented with a short
The importance of recurrent infections in history of increasing lethargy and 3 days of
progression of disease is uncertain. In irregular palpitations. Investigations revealed:
pregnancy there can be acceleration of renal
serum sodium 142 mmol/L (137-144) serum
decline especially in those with pre existing
potassium 7.9 mmol/L (3.5-5.0) serum
impairment and risk factors for progression.

Dr. Khalid Yusuf El-Zohry – Sohag Teaching Hospital (01118391123) Page | 323
El-Zohry MRCP Questions Bank (Part 2) – Medical Masterclass 2010

bicarbonate 10.0 mmol/L (20-28) serum recognition of the alpha 3 component of


creatinine 550 µmol/L (60-110) type IV collagen (COL4a3)

What is the best immediate therapy?


Answer & Comments
A. Nebulised salbutamol
Correct answer: C
B. Intravenous calcium gluconate
C. Intravenous sodium bicarbonate 85-90% of cases of Alport’s syndrome show X-
linked dominant inheritance. In these patients
D. Intravenous insulin and dextrose
mutations occur in the COL4a5 gene coding for
E. Oral calcium resonium the alpha 5 chain of type IV collagen. This
results in abnormal glomerular based
Answer & Comments membrane structure leading to exclusion of the
alpha 3 chain which contains the Goodpasture’s
Correct answer: B
disease antigen. Following transplantation of a
normal kidney the immune system of the
This man has dangerous hyperkalaemia patient is exposed to this antigen for the first
resulting from acute renal failure. Although time and immune recognition may occur.
there is no mention of ECG changes, the history Abnormal collagen in the cochlea, commonly
of palpitations should raise the possibility of causes sensorineural hearing loss and abnormal
arrhythmias. As such, 10mL 10% calcium collagen in the eye can lead to lenticonus and
gluconate i.v. will immediately reduce occasionally spontaneous lens rupture. The
myocardial excitability. This should prevent any outlook for renal function in affected males is
dangerous cardiac arrhythmias. It does not poor with most developing end stage renal
reduce plasma potassium levels. All the other failure in their late teens/early twenties.
agents could be used to lower potassium whilst
arrangements for urgent dialysis are made.
[ Q: 137 ] MasterClass Part2
(2010) - Nephrology
[ Q: 136 ] MasterClass Part2
(2010) - Nephrology A 46-year-old man presents with unexplained
dialysis-dependent renal failure. You decide a
A 16-year-old male patient is found to have renal biopsy could be useful.
Alport’s syndrome inherited from his mother.
Which of the following would be an absolute
Which of the following statements is FALSE? contraindication to carrying out this
A. The mutation lies in the alpha 5 chain of investigation?
type IV collagen (COL4a5) A. A positive cANCA
B. Lenticonus is a recognised feature B. An Hb of 9g/dl
C. Conductive hearing loss arises as a result C. Ultrasound demonstrates two small 6 cm
of collagen abnormalities dense kidneys with no corticomedullary
D. Progression to end stage renal failure is differentiation
likely before the age of 30 D. A serum bicarbonate of 15
E. Following successful renal transplantation E. A serum urea of 56 mmol/l
anti-glomerular basement membrane
F. A calcium of 2.02 and Phospate 2.3
disease may occur as a result of immune

Dr. Khalid Yusuf El-Zohry – Sohag Teaching Hospital (01118391123) Page | 324
El-Zohry MRCP Questions Bank (Part 2) – Medical Masterclass 2010

G. An abnormal clotting screen A. This patient is at risk of thrombosis and


embolism.
H. A low C4
B. The condition is too severe to attempt
I. Ultrasound demonstrates a single 12 cm
definitive treatment.
right kidney
C. The best treatment is high-dose
J. Ultrasound demonstrates bilateral
prednisolone.
hydronephrosis.
D. The pleural effusion is likely to be
Answer & Comments malignant in aetiology.

Correct answer: CJ E. Renal functional decline is inevitable.


F. Spontaneous remission is possible.
Unexplained renal failure is a good indication G. Angiotensin converting enzyme (ACE)
for renal biopsy and often influences therapy. inhibitors or angiotensin 2 receptor
Although abnormal clotting increases the risk of blockers (ARB) alone will abolish
bleeding this can be corrected prior to biopsy, proteinuria.
so would not absolutely rule out a biopsy. H. BP needs to be in the range 130/80 -
Likewise, a single kidney represents a high risk 140/90.
situation as there is a small risk of uncontrolled
haemorrhage which could lead to I. A low protein diet is essential.
nephrectomy. However, renal patholgy in this J. Prednisolone and methotrexate are
clinical setting could be very useful and may proven therapies.
justify the extra risk. Small dense kidneys
indicate chronic renal failure, a biopsy is Answer & Comments
difficult, risky and is very unlikely to provide
diagnostic information. Likewise Correct answer: AF
hydronephrosis on ultrasound supports a
diagnosis of obstructive nephropathy which Patients with the nephrotic syndrome,
needs radiological or urological intervention, especially in children, are at increased risk of
not a biopsy. DVT, PE and arterial thrombosis. Anti-
coagulation (Warfarin) is recommended if
[ Q: 138 ] MasterClass Part2 patients are very oedematous with restricted
mobility, and some (but not all) nephrologists
(2010) - Nephrology
would anticoagulate routinely if plasma
A 42-year-old man presents with a short history albumin is < 25 g/l for a sustained period.
of ankle-swelling and breathlessness. He is Although ACE and ARB, separately or together
hypertensive with gross peripheral oedema. are valuable anti-proteinuric interventions, they
Investigations show normal renal function, 24 will only reduce protein loss by 50% at most in
grams per day urinary protein loss, plasma this setting.
albumin of 12 g/l, and a large right pleural
effusion. A renal biopsy shows membranous Even membranous nephropathy (MN) of this
nephropathy. severity can remit spontaneously, although this
is uncommon. Far from being too severe for
Which TWO of the following statements is treatment, this situation cries out for
true ? intervention as the nephrotic syndome of this
severity has an appreciable mortality and

Dr. Khalid Yusuf El-Zohry – Sohag Teaching Hospital (01118391123) Page | 325
El-Zohry MRCP Questions Bank (Part 2) – Medical Masterclass 2010

morbidity. The best treatment would in fact be 20% of patients. All the complications of the
either prednisolone and chlorambucil or nephrotic syndrome are seen including renal
prednisolone and cyclophosphamide (Claudio vein thrombosis. The renal biopsy shows a
Ponticelli regimens : Ponticelli et al, JASN 9:444- thickened glomerular basement membrane on
450, 1998). light microscopy, granular IgG and C3 on
immunostaining and subepithelial deposits on
These patients are often malnourished and
electron microscopy
anorexic, so a low protein diet may do more
harm than good. BP needs to be as low as
possible, certainly < 125/75 (from the MDRD [ Q: 140 ] MasterClass Part2
trial). Renal functional decline is not inevitable, (2010) - Nephrology
though likely.
A 73-year-old lady presented with a 6-month
Malignancy is not associated with more than a history of malaise, anorexia, progressive leg
few percent of cases of MN. swelling and easy bruising. Plasma creatinine
was 97 umol/l. 24 hours urinary protein was 12
[ Q: 139 ] MasterClass Part2 grams. Renal ultrasound showed two 10 cms
(2010) - Nephrology echogenic kidneys. CRP was 5 mg/l (NR <5). She
had a 7 g/l IgG lambda paraprotein on serum
A 28-year-old man presents with the nephrotic electrophoresis, but no Bence Jones protein.
syndrome. Renal biopsy reveals membranous
nephropathy. Which further investigations are necessary ?
A. CT scan chest
Which one of the following statements is true
regarding this condition? B. CT scan abdomen
A. prognosis for renal survival is universally C. Renal biopsy
poor
D. Renal angiogram
B. it is the commonest cause of nephrotic
E. Renal venogram
syndrome in children
F. Bone marrow aspirate and trephine
C. immunostaining on the renal biopsy is
usually negative G. A serum amyloid P (SAP) scan

D. most patients present with the nephrotic H. Isotope renography


syndrome I. Skin biopsy
E. renal vein thrombosis is a very rare J. Rectal biopsy.
complication.
Answer & Comments
Answer & Comments
Correct answer: CF
Correct answer: D
This is a typical presentation of systemic
Membranous nephropathy is the commonest amyloidosis. It is likely that the patient has the
cause of the nephrotic syndrome in adults, AL (immunocyte-related) form of the condition,
whereas most children have minimal change based on the presence of the paraprotein and
disease. It has a very variable outcome with the low CRP.
spontanous remission occuring in up to 1/3 of
patients. Renal failure can develop in about

Dr. Khalid Yusuf El-Zohry – Sohag Teaching Hospital (01118391123) Page | 326
El-Zohry MRCP Questions Bank (Part 2) – Medical Masterclass 2010

A renal biopsy is required to establish the cause What is the correct diagnosis?
of the nephrotic syndrome: amyloidosis is likely,
A. Hyperkalaemia
but other primary glomerular disease cannot be
confidently excluded without histology. B. Ventricular tachycardia

A bone marrow aspirate and trephine are C. Acute pericarditis


needed because some degree of plasma cell D. Acute anterolateral myocardial infarction
dyscrasia is likely, and if there is frank myeloma
E. Digoxin toxicity.
treatment is indicated. Even if there is no frank
myeloma, some treatment, e.g. prednisolone
and melphalan, may be warranted since Answer & Comments
untreated amyloid tends to progress rapidly, Correct answer: A
and AL- amyloid can lead to rapid cardiac and
renal decompensation.
The presence of tall, peaked T waves, flattened
A SAP scan is a research rather than diagnostic P waves, prolonged PR interval and wide QRS
tool, being particularly useful in regression complexes are pathognomonic of
studies. Only one UK centre can perform these hyperkalaemia. Give IV calcium immediately
scans currently (National Amyloid Centre, Royal (10mls, 10% calcium gluconate) and call the
Free Hospital, London). The best treatment for renal team.
primary amyloidosis is not known: give the
centre a call if you have a case; they will be able
[ Q: 142 ] MasterClass Part2
to educate you, and also give you information
about trials into which the patient might be (2010) - Nephrology
recruited.
Which of the following options do you think
would be the most appropriate in supporting a
diagnosis of reflux nephropathy in a 40-year-old
man with plasma creatinine 540
micromole/litre, 2.5g proteinuria per 24 hours
[ Q: 141 ] MasterClass Part2
and a childhood history of repeated urinary
(2010) - Nephrology tract infections?
This patient presented acutely unwell with the A. Micturating cystogram
following ECG (see image).
B. Renal ultrasound
C. Intravenous urography

Dr. Khalid Yusuf El-Zohry – Sohag Teaching Hospital (01118391123) Page | 327
El-Zohry MRCP Questions Bank (Part 2) – Medical Masterclass 2010

D. Computerized tomography with E. She has impaired function of her adrenal


intravenous contrast cortices.
E. Isotopic imaging with 99mTc-DTPA. F. She has Alport's sydrome.
G. She has upper urinary tract infection.
Answer & Comments
H. She has hyperthyroidism.
Correct answer: B
I. She has cutaneous systemic lupus
erythematosus treated with chloroquine.
In the majority of patients with reflux
nephropathy, ureteric reflux does not persist J. She has severe hypertension.
beyond childhood so a micturating cystogram is
not usually helpful. Answer & Comments

Ultrasound is a good method of detecting renal Correct answer: BG


scars, although it is less sensitive than
intravenous urography or DMSA scanning in Repeatedly positive urine cultures can arise
detecting small scars in those with normal renal with repeated new infections or from
function. resurgence of an untreated or partially treated
Intravenous urography is often relatively infection. In the latter case, this usually arises
unhelpful in patients with this degree of renal because the infection is deep-seated and
impairment due to poor concentration of difficult to eradicate. This can occur with deep
contrast. infection of renal tissue in upper urinary tract
infection or with deep infection of the prostate
DMSA, which is taken up by tubular cells after
in chronic prostatitis. In these cases, a
filtration by the glomerulus, can be used to
prolonged course of appropriate antibiotics,
detect scars (as opposed to DTPA which is
often for around 6 weeks, may be necessary to
filtered by the glomerulus and not taken up by
eradicate the infection. The presence of stones,
tubular cells
foreign bodies such as catheters or anatomical
abnormalities of the urinary tract can also make
[ Q: 143 ] MasterClass Part2 infection difficult to clear resulting in
(2010) - Nephrology repeatedly positive urine cultures.

A 24-year-old woman presents with recurrent


urinary tract infection over several months. Her [ Q: 144 ] MasterClass Part2
general practitioner has diagnosed the (2010) - Nephrology
infections by urine culture, which has been
A 22-year-old woman presents with recurrent
positive with each episode. Each episode has
symptoms of urinary frequency and dysuria
been treated with appropriate antibiotics for
associated with cloudy urine. These settle
several days.
rapidly with courses of antibiotics.
Which TWO of the following could readily
Which one of the following would NOT be a
explain this situation?
recognised strategy for trying to prevent
A. She has Bartter's syndrome. recurrence?
B. She has polycystic kidney disease. A. Long-term prophylaxis with trimethoprim
100 mg at night
C. She has cranial diabetes insipidus.
B. Regular drinking of cranberry juice
D. She has upper airways infection.

Dr. Khalid Yusuf El-Zohry – Sohag Teaching Hospital (01118391123) Page | 328
El-Zohry MRCP Questions Bank (Part 2) – Medical Masterclass 2010

C. Practice of double micturition A. The renal lesion is likely to be interstitial


nephritis secondary to analgesics
D. Advice to void the bladder before and
after sexual activity B. She is likely to require conversion from
azathioprine to cyclophosphamide
E. Use a shower rather than a bath.
C. Minimal change disease is the most likely
Answer & Comments finding on renal biopsy

Correct answer: E D. Amyloidosis is a likely finding on renal


biopsy

Cranberry juice has antiseptic properties and E. Renal biopsy should be avoided as the
has been shown in a controlled trial to be result will not influence management
effective at reducing risk of urinary tract F. ACE Inhibitors should be avoided as she
infection. has significant renal impairment
The practice of double micturition endeavours G. Primary amyloidosis is the most likely
to make sure that the bladder is completely diagnosis
empty after voiding, making it more difficult for
infection to take hold. The woman should be H. Membranous nephropathy is a possible
advised to empty her bladder, wait for a further finding on renal biopsy
60 seconds on the toilet and then try to void I. Loop diuretics should be avoided
again. Some find that pressing on their
J. Renal angiography is the most useful next
suprapubic region will help them to express
diagnostic test
more urine.

Answer & Comments


[ Q: 145 ] MasterClass Part2
(2010) - Nephrology Correct answer: DH

A 56-year-old woman is seen in the renal This woman has presented with the nephrotic
outpatient department with a history of syndrome with evidence of impaired excretory
increasing leg oedema in association with new function in the context of chronic inflammatory
onset proteinuria. She has a longstanding disease. Secondary AA amyloidosis is the most
history of seronegative arthritis controlled with likely diagnosis here, although membranous
simple analgesia alone, as well as crohn's nephropathy would also be a contender. Renal
disease held in remission with prednisolone and biopsy is the best diagnostic procedure.
azathioprine. Investigations demonstrate
proteinuria at a level of 10g in 24hrs, a low Differentiation between Primary (AL) and
serum albumin (18g/dl), normal Secondary (AA) amyloidosis can be difficult in
immunoglobulins, Bence Jones negative, and a around 10% of patients, but the absence of a
negative autoantibody screen. Her ESR and CRP paraprotein and the long history of chronic
are both elevated and have been so for at least inflammatory disease are most supportive of
10 years. Excretory renal function was reduced AA amyloid. One would expect an elevated SAA
with an estimated GFR of 36ml/1.73m2/min. level and a normal lambda/kappa ratio on a
free light chain assay.
Which of the following statements are most
likely to be TRUE?

Dr. Khalid Yusuf El-Zohry – Sohag Teaching Hospital (01118391123) Page | 329
El-Zohry MRCP Questions Bank (Part 2) – Medical Masterclass 2010

[ Q: 146 ] MasterClass Part2 Hepatitis B is associated with three main renal


(2010) - Nephrology conditions: Membranous nephropathy,
Polyarteritis Nodosa (PAN) and
A 43-year-old man is initially seen Membranoproliferative Glomerulonephritis.
after an acute illness characterised by fever, PAN is rare but most commonly seen 3-4
jaundice and abnormal liver enzymes. months after the acute infection. There is
Investigations support a diagnosis of acute usually ongoing active viral replication. The
Hepatitis B infection. He represents 4 months clinical picture is one of systemic upset,
later with acute abdominal pain, hypertension, hypertension, renal dysfunction, abdominal
numbness in his left leg and right hand and pain and neurological abnormalities - most
general malaise. Neurological examination is commonly mononeuritis multiplex. The ANCA is
consistent with a mononeuritis multiplex. Blood usually negative. PAN is a vasculitis that
tests confirm renal impairment (creatinine involves medium sized blood vessels and is best
180umol/l). His autoantibody screen is negative diagnosed on mesenteric angiography, where
as is his ANCA. Hepatitis B surface and e antigen the small associated aneurysms may be seen
are both positive and HBV DNA copies are very particularly in the renal circulation.
high.
Treatment can be difficult and overall mortality
Which of the following statements are most is high. Antiviral therapy is essential and early
likely to be TRUE? reduction in viral load and development of anti
HB e antibodies is associated with a good
A. He has developed acute membranous prognosis. Adjunctive treatment with steroids
nephropathy and plasma exchange may have a role.
B. Renal biopsy is the most valuable initial However, the aggressive immunosuppressive
investigation treatment used in other vasculitides is
contraindicated because it is associated with
C. Mesenteric angiography would be useful
failure to clear the virus and chronic active
D. Immunosuppression with prednisolone infection.
and cyclophosphamide is appropriate
E. Plasma exchange is contraindicated [ Q: 147 ] MasterClass Part2
because of his infection risk (2010) - Nephrology
F. Dialysis should not be offered as he
(1) A 62-year-old man presents with swollen
represents a risk to staff and other
ankles. Dipstick analysis of his urine reveals
patients
protein +++, but no other abnormality. He is
G. Prednisolone alone is ideal therapy noted to have long-standing gout, hypertension
and hypercholesterolaemia. His plasma
H. Screening for Hep C and HIV would be of
creatinine is 105micromoles/l and albumin
no value
22g/l.
I. He represents a low infection risk
Which of the following medications would be
J. Antiviral therapy will be a key element in
most likely to account for his swollen ankles and
his management
proteinuria?

Answer & Comments A. Allopurinol


B. Frusemide
Correct answer: CJ
C. Enalapril

Dr. Khalid Yusuf El-Zohry – Sohag Teaching Hospital (01118391123) Page | 330
El-Zohry MRCP Questions Bank (Part 2) – Medical Masterclass 2010

D. Ibuprofen Answer & Comments


E. Prednisolone Correct answer: DI

Answer & Comments ANCA-associated systemic vasculitis is a


relatively common cause of focal segmental
Correct answer: D
glomerulonephritis. Typically there is minimal
immunoglobulin deposition (hence ‘pauci-
Non-steroidal anti-inflammatory drugs can immune’).
cause nephrotic syndrome with microscopical
appearances of minimal change disease. They Linear deposition of IgG suggests
can also lower glomerular filtration rate by antiglomerular basement membrane antibody
vascular effects and cause an interstitial (anti-GBM) disease; rarely patients have
nephritis. features of both ANCA and anti-GBM disease
but this is certainly not an expected finding and
will alter management. Complement levels are
[ Q: 148 ] MasterClass Part2
not expected to be reduced. Neutrophil
(2010) - Nephrology leucocytosis, elevated platelet count,
A 64-year-old man presents with a 3-month normochromic normocytic anaemia and high
history of malaise and urinalysis is strongly CRP are all common features. The creatinine
positive for protein and blood. The patient has can certainly be normal.
a vasculitic skin rash. A test for anti-neutrophil
cytoplasmic antibodies (ANCA) is positive. [ Q: 149 ] MasterClass Part2
(2010) - Nephrology
Which TWO of the following findings would
raise doubts about a diagnosis of ANCA-
In a patient with hypertension, which one of the
associated systemic vasculitis?
following features is consistent with a clinical
A. Haemoglobin 10 g/dl diagnosis of Liddle's syndrome (a mutation
affecting the sodium channels in the distal
B. Neutrophil leukocytosis
tubules)?
C. Focal necrotising glomerulonephritis on
A. raised catecholamine levels
the renal biopsy
B. high aldosterone levels
D. Significantly reduced circulating levels of
C3 and C4 C. skin nodules
E. Hypergammaglobulinaemia D. high renin levels
F. Elevated C-reactive protein (CRP) E. hypokalaemia.
G. Absence of significant immunoglobulin
deposition in the renal biopsy (assessed Answer & Comments
by immunofluorescence) Correct answer: E
H. Elevated platelet count
I. Linear deposition of IgG on the glomerular Liddle's syndrome is caused by a mutation in
basement membrane in the renal biopsy the sodium channel (ENaC) in the distal
(assessed by immunofluorescence) nephron. The mutation keeps the channel
open, which has a similar effect to a raised
J. Normal plasma creatinine. aldosterone level. However, the aldosterone

Dr. Khalid Yusuf El-Zohry – Sohag Teaching Hospital (01118391123) Page | 331
El-Zohry MRCP Questions Bank (Part 2) – Medical Masterclass 2010

levels are not raised and may be lowered as a


result of feedback from the sodium and water
retention. There is increased sodium
reabsorption and potassium loss. The condition
is sometimes called pseudohyperaldosteronism.
The hypertension is caused by sodium
retention. This is a difficult question, but if you
know that sodium handling is abnormal and
there is hypertension, it is likely that this is
because of sodium retention. In the distal
nephron sodium reabsorption is mechanistically
linked to potassium secretion, so increased
sodium retention will lead to increased
potassium secretion and hypokalaemia.

Dr. Khalid Yusuf El-Zohry – Sohag Teaching Hospital (01118391123) Page | 332
El-Zohry MRCP Questions Bank (Part 2) – Medical Masterclass 2010

postdysenteric or postvenereal illness. The

Rheumatology characteristic rashes of keratoderma


blennorrhagicum and circinate balanitis may be
present.
(104 Questions)
Psoriatic spondyloarthropathy is characterised
(Medical Masterclass – Part 2) by psoriatic plaques. The skin involvement may
be subtle and should be searched for carefully.
The cleft of the buttock, scalp hairline, and
penis are site often involved but may be missed
[ Q: 1 ] MasterClass Part2 easily if thorough examination of these areas
was not actively conducted. Psoriatic nail
(2010) - Rheumatology
changes include onycholysis, yellow nails and
A 32-year-old man presents with a 4-month nail pitting.
history of back pain. The pain is worse in the
Inflammatory bowel disease consists of
morning and after sitting watching TV. Plain
ulcerative colitis and Crohn's disease.
radiograph of the spine/pelvis shows evidence
of sacroiliitis. A diagnosis of ankylosing spondylitis may be
made when specific features of Reiter's
Each of the following clinical features might be syndrome, psoriasis, or inflammatory bowel
identified on clinical examination of this patient disease are absent.
EXCEPT?
Sacroiliitis is also encountered in tuberculosis,
A. Keratoderma blenorhegica sarcoidosis and brucellosis.
B. Rheumatoid nodule Rheumatoid arthritis is not associated with
C. Onycholysis lumber or sacroiliac joint disease. However
cervical spondylitis and atlanto-axial
D. Uveitis
subluxation is not an uncommon feature of RA.
E. Urethritis.
[ Q: 2 ] MasterClass Part2
Answer & Comments (2010) - Rheumatology
Correct answer: B A 73-year-old man presents with a 24-hour
history of a painful swollen left knee. He has
The long (> 1 hr) early morning and rest had minor pain and stiffness in both knees for
stiffness are highly suggestive of an underlying many years and now feels generally unwell. His
inflammatory condition. The pelvic radiograph temperature is 37.4°C.
confirms the presence of sacroiliitis. Bilateral
sacroiliitis are typical for ankylosing spondylitis. What is the most likely diagnosis?
The frequency of asymmetric sacroiliitis may be A. Gout
higher in other spondyloarthropathies, e.g.,
reactive arthritis, Reiter’s syndrome, spondylitis B. Pseudogout
associated with psoriasis, or inflammatory C. Septic arthritis
bowel disease.
D. Reactive arthritis
Reiter's syndrome is characterised by a triad of
E. Rheumatoid arthritis.
arthritis, urethritis, and conjunctivitis. Reiter's
syndrome develops in the setting of

Dr. Khalid Yusuf El-Zohry – Sohag Teaching Hospital (01118391123) Page | 333
El-Zohry MRCP Questions Bank (Part 2) – Medical Masterclass 2010

Answer & Comments E. Takayasu's disease

Correct answer: B F. Churg Strauss syndrome


G. Behcet's disease
It would be important to exclude septic arthritis
H. Aortic dissection
by microscopy and culture of fluid aspirated
from the knee, but this is not the commonest I. Systemic Lupus Erythematosus
cause of an acute monoarthritis. J. Rheumatoid vasculitis.
The differential diagnosis is influenced by age.
Crystal arthritis is the most common cause in Answer & Comments
elderly people, whereas reactive arthritis tends
Correct answer: DF
to occur in sexually active young adults.
Calcium pyrophosphate deposition disease,
The combination of asthma, multisystem
often known as pseudogout, can often present
disease and eosinophilia will suggest Churg
with striking fever and systemic illness.
Strauss vasculitis to most
Treatment is with nonsteroidal anti-
rheumatologists/immunologists. However,
inflammatory drugs (NSAIDs) or intra-articular
asthma is common and the occurrence of livedo
steroid injection, once sepsis has been
changes in the skin and renal impairment
excluded.
shortly after vascular intervention or
anticoagulant/thrombolytic treatment is highly
[ Q: 3 ] MasterClass Part2 suggestive of cholesterol emboli. The
(2010) - Rheumatology cholesterol emboli syndrome can be
accompanied by ‘immunological’ changes in the
A 63-year-old man is admitted with crushing blood such as eosinophilia, low complement
central chest pain. He has previously been concentrations and low titre autoantibodies.
reasonably well but has had troublesome The diagnosis is confirmed histologically, with
asthma for many years and has recently been ‘cholesterol clefts’ being seen in affected
found to have hypertension. An ECG shows an vessels.
acute anterior myocardial infarction, which is
treated promptly with thrombolysis.
[ Q: 4 ] MasterClass Part2
Two days later he is noted to have marked
(2010) - Rheumatology
livedo reticularis over his legs. Investigations
show his creatinine has risen from 115 umol/l A 78-year-old man presents with a 2-day history
on admission to 280 umol/l. His blood count is of severe headache localising to his right
normal apart from an eosinophilia at 3x10*9/l. temple. He also feels tired and lethargic. He
His CRP is elevated at 40mg/l. Dipstick testing of denies any eye problems and pain on chewing.
his urine is positive for blood and protein. The biopsy of his right temporal artery is shown
(see image).
Which TWO of the following diagnoses seem
most likely?
A. Goodpasture's syndrome
B. Subacute bacterial endocarditis
C. Giant cell arteritis
D. Cholesterol emboli syndrome

Dr. Khalid Yusuf El-Zohry – Sohag Teaching Hospital (01118391123) Page | 334
El-Zohry MRCP Questions Bank (Part 2) – Medical Masterclass 2010

Which TWO of the following would be


consistent with a diagnosis of fibromyalgia?
A. Swollen joints
B. Raised creatine kinase (CK)
C. Raised thyroid-stimulating hormone (TSH)
D. Raised elevated sedimentation rate (ESR)
E. Irritable bowel syndrome
F. Unrefreshing sleep
G. Anaemia
Which of the following statements fit best?
H. Impaired renal function
A. This is a normal temporal artery biopsy.
I. Proximal myopathy
B. This man should be treated with 60 mg
prednisolone. J. Photosensitive rash.

C. There is no need to consider bone


Answer & Comments
protection.
D. 50% of patients with this condition Correct answer: EF
develop permanent visual loss.
Fibromyalgia is a diagnosis of exclusion. All
E. An ESR of less than 40mm/hr excludes
blood tests should be normal. Depression,
Giant Cell Arteritis (GCA).
irritable bowel syndrome, unexplained
headache and unexplained chest pain are
Answer & Comments common in these patients.
Correct answer: B
[ Q: 6 ] MasterClass Part2
This biopsy shows thickening and lymphocytic (2010) - Rheumatology
infiltration highly suggestive of GCA. He should
A 26-year-old man with recurrent chest
indeed be treated with high dose steroid
infections is diagnosed as having common
initially. It is essential to consider bone
variable immunodeficiency (CVID).
protection as he is likely to be on steroids for up
to 2 years. (He may also need gastric
From the following, select the TWO most
protection.) 15-20% of patients develop
common infectious agents responsible for
permanent visual loss. The ESR is only elevated
disease in this condition.
in 80% of cases, so if the history is good, the
diagnosis should still be considered. A. S pneumoniae
B. H influenzae
[ Q: 5 ] MasterClass Part2 C. E coli
(2010) - Rheumatology
D. M tuberculosis
A 35-year-old woman attends clinic complaining E. Enteroviruses
of widespread pain and fatigue. A history,
examination and blood tests are performed. F. P Carinii
G. Aspergillosis

Dr. Khalid Yusuf El-Zohry – Sohag Teaching Hospital (01118391123) Page | 335
El-Zohry MRCP Questions Bank (Part 2) – Medical Masterclass 2010

H. Nocardia Answer & Comments


I. T gondii Correct answer: AC
J. L monocytogenes.
Fluid aspirated from an osteoarthritic joint is
Answer & Comments like normal synovial fluid: typically clear, viscous
and virtually acellular. Inflammatory fluid is
Correct answer: AB turbid (reflecting a high cell count) and loses
viscosity because of release of matrix degrading
Ninety-five per cent of patients with CVID enzymes in the synovial fluid.
present with recurrent sinopulmonary
infections, occurring as a result of encapsulated [ Q: 8 ] MasterClass Part2
bacterial infection, most of which are due to S
(2010) - Rheumatology
pneumoniae and H influenzae.
CVID patients are also predisposed to chronic A 67-year-old woman is referred to a specialist
enteroviral infection, but fortunately this rheumatology clinic. She has poorly controlled
complication is rare. rheumatoid arthritis, and for the last fifteen
years has been treated with various disease-
modifying drugs including sulphasalazine and
[ Q: 7 ] MasterClass Part2 methotrexate.
(2010) - Rheumatology
Blocking the biological function of which one of
A 55-year-old woman presents with a 3-month the following molecules will provide significant
history of a painful swollen knee. Examination anti-inflammatory effect in rheumatoid
shows restricted, painful movement and a arthritis?
moderate sized effusion. Synovial fluid is
aspirated for diagnostic purposes. A. Interleukin-10 (IL-10)
B. Interleukin-4 (IL-4)
Which TWO of the following findings in the
synovial fluid would be most compatible with a C. Interleukin-1 (IL-1)
final diagnosis of osteoarthritis? D. Soluble TNF receptors
A. Macroscopically clear fluid E. Transforming growth factor-beta (TGF-
B. Needle shaped crystals on microspcopy beta)

C. Highly viscous fluid


Answer & Comments
D. High numbers of neutrophils on
microscopy Correct answer: C

E. Blood stained fluid


Advances in research have identified specific
F. Thin, watery fluid cells and cell products (primarily cytokines) that
G. High numbers of macrophages on may play distinct roles in RA activity. It has been
microscopy well documented that in RA, proinflammatory
cytokines mediate synovial proliferation and
H. Rhomboid-shaped crystals on microscopy articular tissue destruction. For these reasons,
I. Macroscopically turbid cytokines have become a target for therapy.
Cytokine-based therapies include a) blocking of
J. Gram positive cocci on gram stain.
proinflammatory cytokines (IL1, IL2, IL6, IL8,

Dr. Khalid Yusuf El-Zohry – Sohag Teaching Hospital (01118391123) Page | 336
El-Zohry MRCP Questions Bank (Part 2) – Medical Masterclass 2010

TNFa, G-CSF, GM-CSF) or, b) augmentation of Which TWO of the following diagnoses are most
anti-inflammatory cytokines (IL4, IL10, IL11, compatible with the clinical picture?
IL13, TGFb, soluble TNF receptors, soluble IL2 A. Sjogren’s syndrome
receptors).
B. Goodpasture’s disease
Two of the major cytokines thought to incite
inflammation in RA patients are TNF and IL-1. C. Lupus
This has resulted in the introduction of two D. Henoch schonlein purpura
inhibitors of TNF, the soluble TNF receptor
E. Cryoglobulinaemic vasculitis
construct Etanercept and the anti-TNF
monoclonal antibody (mAb) Infliximab. More F. Polyarteritis nodosa
recently anti-IL-1 Kineret has been licensed for
G. Anti-phospholipid syndrome
use in active RA. On the other hand IL10, IL-4
and TGF-beta may be considered a natural H. Wegener's granulomatosis
‘dampeners’ of the immune response and can I. Microscopic polyangiitis
be used as anti-cytokines. Therefore
J. Takayasu’s arteritis.
augmentation of theses cytokines, rather than
blocking them, hold promise for new biologic
therapy in RA. Answer & Comments

Correct answer: HI
[ Q: 9 ] MasterClass Part2
(2010) - Rheumatology This patient has presented with the pulmonary-
renal syndrome, which is most commonly
A 75-year-old retired farmer has been seen in
caused by the small vessel vasculitides
the outpatient clinic. He has been generally
(Wegener’s granulomatosis and microscopic
unwell for the last month with fever and weight
polyangiitis (MP)) although Goodpasture’s
loss, examination reveals this rash (see image).
disease, lupus and cryoglobulinaemic vasculitis
should also be included in the differential
diagnosis. ANCA positivity in this patient group
has a positive predictive value >90% for
diagnosing small vessel vasculitis. Although
specificity to MPO is more commonly
associated with microscopic polyangiitis, 25% of
patients with Wegner’s will also have a positive
MPO. It is therefore not possible to
differentiate these two diseases, based on
ANCA specificity, nor is it important as
While in the clinic he starts to cough up blood immunosuppressive treatment is indicated
and becomes acutely breathless. He is admitted regardless. A diagnosis of lupus is highly
and initial investigations reveal a serum unlikely with a negative ANA and >90% of
creatinine of 170 micromol/l, a positive patients with cryoglobulinaemic vasculitis will
myeloperoxidase (MPO) ELISA, negative have a low C4 as a result of classical pathway
proteinase 3 (PR3) ELISA, negative GBM ELISA, a activation.
negative ANA and normal complement levels.
Red cell casts are seen on microscopy of his
urine.

Dr. Khalid Yusuf El-Zohry – Sohag Teaching Hospital (01118391123) Page | 337
El-Zohry MRCP Questions Bank (Part 2) – Medical Masterclass 2010

[ Q: 10 ] MasterClass Part2 What investigation(s) would you consider that


(2010) - Rheumatology would most likely provide a definitive diagnosis?
A. Repeat standard pelvic radiograph and
A 70-year-old woman presents with
check a chemical profile.
inflammatory joint pain, fatigue and a dry
mouth. She is found to have marked B. Bone scan or Magnetic Resonance
hypergammaglobulinaemia and a raised Imaging (MRI).
erythrocyte sedimentation rate with a normal C. Dexa (dual energy radiograph
C-reactive protein. absorptiometry) scan.

What is the most likely diagnosis? D. Computerised Tomogram (CT) scan.

A. Rheumatoid arthritis E. Myeloma screen.

B. Pyrophosphate arthropathy
Answer & Comments
C. Primary Sjögren's syndrome
Correct answer: B
D. Systemic lupus erythematosus
E. Fibromyalgia Diagnosis of hip fractures may not always be
evident on a plain radiograph. An
Answer & Comments anteroposterior view obtained with the hip
internally rotated 15 to 20 degrees will provide
Correct answer: C
an optimal image and may reveal a fracture not
evident on on the standard ap view. If there is a
Sicca symptoms, a raised erythrocyte high index of suspicion of a femoral fracture
sedimentation rate but normal C-reactive technetium-99 m bone scanning or magnetic
protein (CRP) and hypergammaglobulinaemia resonance imaging is appropiate and will show
are classic features of primary Sjögren's a femoral fracture not evident on a plain
syndrome. This syndrome tends to begin in the radiograph.
fifth and sixth decades compared with lupus,
which typically begins between the second and
fourth decades. This patient could have
[ Q: 12 ] MasterClass Part2
rheumatoid arthritis but this is unlikely since (2010) - Rheumatology
the CRP is normal. Blood tests are usually
A 27-year-old woman has a 6-year history of
normal in patients with primary fibromyalgia.
systemic lupus erythematosus (SLE) treated
with azathioprine, hydroxychloroquine and
[ Q: 11 ] MasterClass Part2 prednisolone. She presents with a 2-week
(2010) - Rheumatology history of slowly worsening severe pain and
restriction of the right hip.
An 80-year-old woman is admitted having
tripped over a mat. Physical examination is Which TWO of the following diagnoses seem
apparently unremarkable. She is unable to most likely?
weight bear and the Orthopaedic team have
A. Gout
discharged her with a normal pelvic radiograph.
She has been transferred to a medical ward as B. Flare of systemic lupus erythematosus
she is still not able to walk. (SLE)
C. Secondary osteoarthritis

Dr. Khalid Yusuf El-Zohry – Sohag Teaching Hospital (01118391123) Page | 338
El-Zohry MRCP Questions Bank (Part 2) – Medical Masterclass 2010

D. Perthe's disease C. Difficulty turning over in bed


E. Septic arthritis D. Difficulty hanging out washing
F. Secondary fibromyalgia E. Difficulty writing
G. Irritable hip F. Difficulty swallowing
H. Osteoporotic fracture of femoral neck G. Difficulty lifting head off pillow
I. Avascular necrosis of the femoral head H. Difficulty breathing
J. Slipped upper femoral epiphyisis. I. Difficulty getting out of chair
J. Aifficulty dressing.
Answer & Comments

Correct answer: EI Answer & Comments

Correct answer: AE
The most likely diagnosis is avascular necrosis,
the risk factors for this being corticosteroids
The myopathy in dermatomyositis and
and SLE itself (particularly in patients with
polymyositis is classically a proximal myopathy.
cardiolipin antibodies). Sepsis is less likely but
Patients may also have truncal weakness and
possible and must be excluded. The risk of
problems with the muscles involved with
sepsis is increased by immunosuppression
swallowing and breathing.
(which may also modify the presentation of
sepsis- less fever, more insidious onset, bloods
may be normal). [ Q: 14 ] MasterClass Part2
(2010) - Rheumatology
This patient is at increased risk of osteoporosis
but a hip fracture would be a very rare event at A 75-year-old woman presents to the
this age, and would usually be associated with Emergency Department with a hot swollen left
sudden onset of pain rather than insidious knee and difficulty in weight-bearing. She has
onset. mild heart failure and is on treatment.
Osteoarthritis due to synovitis is almost never
Which one of the following tests will be most
seen in SLE which produces a non-erosive
useful in determining how best to manage her
arthritis.
condition?
A. Aspiration, microscopy and culture of
[ Q: 13 ] MasterClass Part2
synovial fluid
(2010) - Rheumatology
B. Plain radiograph of knee
A 19-year-old girl develops muscle weakness, a
C. Full blood count
heliotrope rash and Gottron's papules. The
creatine kinase (CK) is raised. The muscle biopsy D. C-reactive protein
confirms an inflammatory myositis. A diagnosis
E. Serum uric acid
of dermatomyositis is made.

Which of the following symptoms are Answer & Comments


inconsistent with this diagnosis? Correct answer: A
A. Difficulty gripping
B. Difficulty climbing stairs

Dr. Khalid Yusuf El-Zohry – Sohag Teaching Hospital (01118391123) Page | 339
El-Zohry MRCP Questions Bank (Part 2) – Medical Masterclass 2010

In a patient presenting with a monoarthritis, The most probable diagnosis is:


the most important diagnosis to
A. Ankylosing spondylitis (AS)
consider/exclude is a septic arthritis. This is best
diagnosed by microscopy and culture of the B. Alcaptonuria
synovial fluid. Radiography will be helpful in C. Osteoporosis
determining if the patient has pre-existing
D. Spondylolisthesis
osteoarthritis or chondrocalcinosis. A raised
white cell count might suggest infection. A E. Diffuse idiopathic skeletal hyperostosis
raised C-reactive protein would indicate (DISH)
inflammation or infection. This patient has mild
heart failure and so may be taking Answer & Comments
bendroflumethiazide or loop diuretics, which
may cause hyperuricaemia and gout and hence Correct answer: E
a monoarthritis.
Radiographs of the lumbar spine showed only
[ Q: 15 ] MasterClass Part2 minimal degenerative disc disease, but large,
coarse osteophytes that bridged the lower
(2010) - Rheumatology
vertebral bodies. These findings are classic for
A 70-year-old diabetic had problems with DISH. Criteria for DISH require that new bone
decreased mobility of his back. The problem formation bridge four contiguous vertebral
had progressively worsened over the last seven bodies in the absence of significant
years. His lumbar spine was remarkable for loss degenerative disk disease, and in the absence
of the normal lordosis and decreased range of of inflammatory sacroiliac or facet changes.
motion in all planes. He had no pain on There is increased prevalence of DISH in
percussion over the spine or sacroiliac joints. diabetes mellitus and obesity.
He was otherwise in good health, and the The lack of sacroiliac tenderness in this man,
erythrocyte sedimentation rate (ESR) was with longstanding symptoms, normal
normal. A radiograph of the lumbar spine is sedimentation rate and advanced age of onset,
shown. rendered AS an unlikely diagnosis.
Alcaptonuria is a hereditary disease in which
homogentisc acid oxidase activity is missing.
Homogenitisc acid accumulates and cause
pigmentation of cartilage and other connective
tissues (ochronosis). It usually starts in the
second or third decade. The vertebral bodies of
the lumbar spine show degeneration of the
intervertebral discs, with narrowing of the
space and dense calcification of remaining disc
material. There is variable fusion of vertebral
bodies, but little osteophyte formation and
minimal calcification of intervertebral
ligaments.
Osteoporosis is associated with reduced bone
density and wedge fractures of the vertebrae.

Dr. Khalid Yusuf El-Zohry – Sohag Teaching Hospital (01118391123) Page | 340
El-Zohry MRCP Questions Bank (Part 2) – Medical Masterclass 2010

Spodylolisthesis is forward movement of one [ Q: 17 ] MasterClass Part2


vertebra in relation to an adjacent vertebra. (2010) - Rheumatology
A 30-year-old woman presents with a 3-month
[ Q: 16 ] MasterClass Part2
history of recurrent episodes of urticaria. There
(2010) - Rheumatology are no clues in the history to suggest a possible
A 30-year-old woman presents with several trigger. During each episode she is tired but
attacks of pain and swelling affecting the otherwise well and clinical examination is
metacarpophalangeal and proximal unremarkable.
interphalangeal joints over the past few
What is the most likely diagnosis?
months, with morning stiffness and fatigue.
A. C1 inhibitor deficiency
What are the TWO most likely diagnoses?
B. Urticarial vasculitis
A. Ankylosing spondylitis
C. Food allergy
B. Pseudogout
D. Systemic mastocytosis
C. Systemic lupus erythematosus (SLE)
E. Chronic idiopathic urticaria.
D. Primary generalized osteoarthritis
E. Reactive arthritis Answer & Comments
F. Gout Correct answer: E
G. Rheumatoid arthritis
The cardinal presenting feature of C1 inhibitor
H. Psoriatic arthropathy
deficiency is angioedema.
I. Systemic sclerosis
Urticarial vasculitis is characterized by painful
J. Rheumatic fever. wheals often lasting more than 24 hours and
accompanied by systemic symptoms.
Answer & Comments In the absence of any clues in the history, this
Correct answer: CG patient's urticaria is unlikely to be due to food
allergy. Mastocytosis is very rare; the urticaria
associated with mastocytosis is invariably
The most likely cause of these symptoms is
accompanied by Darier's sign and GI
rheumatoid arthritis, but SLE requires careful
involvement.
consideration.
Rheumatoid arthritis can present as an acute
[ Q: 18 ] MasterClass Part2
monoarthritis, an acute polyarthritis, a
subacute insidious polyarthritis, or with a (2010) - Rheumatology
polymyalgic presentation (particularly in the A 34-year-old woman presents to your clinic
elderly, when it needs to be differentiated from complaining of cold hands, particularly in the
polymyalgia rheumatica). winter months. On examination she has cold
Remember that acute synovitis in early dusky hands, splinter aemorrhages and a
rheumatoid arthritis is associated with very petechial rash. Investigations are as follows: Hb
little fixed joint deformity and no extra-articular 10.9 g/dL; Wbc 4.2 x 109/L; Platelets 407 x
features. This contrasts with the deforming 109/L. Antinuclear antibodies present (1/160).
arthritis that is characteristic of chronic disease.

Dr. Khalid Yusuf El-Zohry – Sohag Teaching Hospital (01118391123) Page | 341
El-Zohry MRCP Questions Bank (Part 2) – Medical Masterclass 2010

DNA antibodies not detected. Complement C3


0.79 (NR 0.75-1.25), C4 <0.04 (NR 0.14-0.6).

Which of the following investigations are not


indicated?
A. Liver function tests
B. Skin biopsy
C. Cryoglobulins
D. Urinanalysis
E. Hepatitis C antibodies
F. Extractable nuclear antibodies (ENA)
G. Endomysial antibodies
H. Thyroid antibodies
I. Rheumatoid factor
J. Serum creatinine.

Answer & Comments


Which of the following statements is true about
Correct answer: GH this boy's illness?
A. The urine abnormality and fever is most
This woman’s symptoms are suggestive of
probably related to a recent urinary tract
cryoglobulinaemia and the low C4 would be
infection
compatible with complement consumption due
to mixed or polyclonal cryoglobulins. B. In such cases blood cultures are often
Cryglobulins are proteins that precipitate at positive for Pseudomonas aerogenosa
temperatures lower than 37ºC. This results in C. Renal biopsy typically shows mesangial
sludging of the circulation in the extremities, IgA deposition
and sometimes vasculitis, especially of skin and
kidneys. The commonest causes are hepatitis C D. Bilateral small kidneys are a constant
and Sjörgren’s syndrome. feature on ultrasound of the kidneys
E. Untreated, up to 50% will develop chronic
[ Q: 19 ] MasterClass Part2 renal failure.
(2010) - Rheumatology
Answer & Comments
A 13-year-old boy was hospitalised because of a
purpuric rash involving the legs (see image) Correct answer: C
associated with abdominal pain and fever
(38°C). The patient also complained of Henoch-Schonlein Purpura (HSP) is recognized
arthralgia involving the knees and ankles. as a systemic small vessel vasculitis involving
Urinalysis showed proteinuria with microscopic mainly the blood vessels of the skin, GI tract,
haematuria. A biopsy of the purpuric lesion the kidneys and the joints.
revealed leucocytoclastic vasculitis in the small HSP affects mainly children between the ages of
vessels. three and 10 years. Males are affected more
Dr. Khalid Yusuf El-Zohry – Sohag Teaching Hospital (01118391123) Page | 342
El-Zohry MRCP Questions Bank (Part 2) – Medical Masterclass 2010

often than females (1.5:1) and in about two Intramuscular epinephrine is the key treatment
thirds of children an upper respiratory tract in anaphylaxis. Intravenous epinephrine may
infection precedes the onset of HSP by one to occasionally be used with extreme caution in
three weeks. patients with cardiovascular collapse, but it is
more likely to cause ventricular arrhythmias
Kidney involvement in HSP is secondary to
when administered by this route.
vasculitis which is not associated with previous
Antihistamines and corticosteroids (without
or concurrent urinary tract infection. HSP
epinephrine) are inadequate immediate
nephritis becomes clinically manifest in only 20-
treatments for anaphylaxis, but may be
30%. It usually presents as macroscopic
administered in milder allergic reactions and
haematuria and proteinuria lasting from a
also to prevent late deterioration in the event
matter of days to several weeks. The kidneys
of anaphylaxis.
are usually of normal size. Of those patients
with renal involvement, as many as 10% may
develop chronic renal failure and end-stage [ Q: 21 ] MasterClass Part2
renal disease. However, fewer than 1% of all (2010) - Rheumatology
patients with HSP suffer this poor prognosis.
A 25-year-old African man with known HIV
infection presents with fever, cervical
[ Q: 20 ] MasterClass Part2 lymphadenopathy, splenomegaly and renal
(2010) - Rheumatology impairment. Significant findings on laboratory
testing include a strongly positive anti-
A 23-year-old woman is brought by ambulance
neutrophil cytoplasmic antibody (ANCA) of
to the Emergency Department having collapsed
proteinase-3 specificity (PR3-ANCA).
in a restaurant while eating a curry. Her friends
have given a history of previous allergic
The positive predictive value of his PR3-ANCA
reactions to nuts. On admission she is flushed,
for a diagnosis of small vessel vasculitis is
breathless and wheezy, has a pulse rate of 140 approximately:
bpm and a BP of 84/40 mmHg.
A. <10%
What is the most appropriate first line of B. 10-20%
treatment?
C. 20-40%
A. High-flow oxygen, intravenous
hydrocortisone and chlorpheniramine D. 40-60%

B. High-flow oxygen and intravenous E. >60%.


epinephrine 500 μg
Answer & Comments
C. 24% oxygen and intramuscular
epinephrine 500 μg Correct answer: A
D. 1 litre intravenous saline and intravenous
chlorpheniramine False-positive ANCA antibodies are a well-
recognised pitfall in this situation. A variety of
E. High-flow oxygen and intramuscular
infections are known to be associated with
epinephrine 500 μg
ANCA including bacterial endocarditis and a
range of tropical infections including
Answer & Comments Amoebiasis and Malaria. It underlines the need
Correct answer: E to obtain histological evidence of small vessel

Dr. Khalid Yusuf El-Zohry – Sohag Teaching Hospital (01118391123) Page | 343
El-Zohry MRCP Questions Bank (Part 2) – Medical Masterclass 2010

vasculitis before proceeding to enzyme (ACE) inhibitors. Insect stings, and


immunosuppressive therapy. foods are other predisposing factors

[ Q: 22 ] MasterClass Part2 [ Q: 23 ] MasterClass Part2


(2010) - Rheumatology (2010) - Rheumatology
A 20-year-old woman presented with sudden (2) A 29-year-old man presents with palpable
onset of swelling of the lips and tongue. She purpura and large joint arthritis. He is
also had abdominal pain and vomiting. Her systemically well. Dipstick testing shows blood
mother confirmed that her daughter had similar and protein in the urine. Blood tests
attacks over the years and even as a child. A reveal that his serum creatinine is elevated at
brother and older sister have the same 200 micromol/l, and liver blood tests show an
disorder. ALT of 120 iu/l (normal <40). Immunological
investigations show a positive test for
Which of the following statements about this rheumatoid factor at 1/256 (normal <1/32),
disease is accurate? negative ANA and low C4 at 0.02g/l (normal
A. It has sex-linked inheritance. 0.2-0.6).

B. Animal allergen is often identified in the Which TWO of the following diagnostic tests are
house most likely to lead to a definitive diagnosis?
C. Serum C4 levels are often low. A. Antineutrophil cytoplasmic antibody
D. Antinuclear antibodies (ANA) is often B. Antiglomerular basement membrane
positive. antibody
E. Raised IgE helps differentiate it from C. Serum immunoglobulins and
other immune disorders. electrophoresis
D. Antistreptolysin O
Answer & Comments
E. Echocardiogram
Correct answer: C
F. Cryoglobulins

Hereditary angioneurotic oedema is an G. Blood cultures


autosomal dominantly inherited condition H. Ultrasound of the kidneys and renal tract
caused by a deficiency of C1 esterase inhibitor.
This results in intermittent episodes of I. Hepatitis C serology
spontaneous complement activation. Clinically J. Hepatitis B serology.
the patient suffers oedema of the skin and
mucosal surfaces. Fatalities may occur if the Answer & Comments
airway is compromised. C4 levels are typically
low during an attack; they may be normal in Correct answer: FI
between attacks.
Acquired angioedema (AAE) is angioedema The clinical picture is highly suggestive of
glomerulonephritis associated with systemic
associated with allergic reactions, which is
often associated with urticaria. Approximately vasculitis or infection. All the listed
94% of cases of angioedema are drug-induced. investigations may be indicated in this clinical
Most are patients taking angiotensin-converting situation. However, combination of high titre
rheumatoid factor, negative ANA, very low C4

Dr. Khalid Yusuf El-Zohry – Sohag Teaching Hospital (01118391123) Page | 344
El-Zohry MRCP Questions Bank (Part 2) – Medical Masterclass 2010

and abnormal LFTs raises very strong suspicions [ Q: 25 ] MasterClass Part2


of a mixed cryoglobulinaemic vasculitis (2010) - Rheumatology
associated with hepatitis C infection. Further
questioning revealed a brief history of A 19-year-old woman collapses in a restaurant,
intravenous drug use 6 years previously and shortly after beginning her meal.
investigation confirmed the clinical diagnosis.
Subsequent renal biopsy showed aggressive Which of the following statements is true?
glomerulonephritis with changes suggestive of A. Anaphylaxis is not the cause if neither
cryoglobulinaemia. stridor nor wheeze is audible.
B. If pallor and hypotension are present:
[ Q: 24 ] MasterClass Part2 anaphylaxis is the likely diagnosis.
(2010) - Rheumatology C. If she has facial swelling and wheeze:
A 56-year-old man presents with a 2-month anaphylaxis is likely even if there is no
history of progressive mid-thoracic spinal pain hypotension.
that disturbs his sleep. A CXR is normal. His D. A history of chronic urticaria implies that
erythrocyte sedimentation rate is 60 mm/hour she is allergic to something.
(normal <10 mm).
E. Scrombrotoxin poisoning is a possibility
Which of the following investigations is most and will not respond to intramuscular
likely to clarify the cause of his pain? epinephrine.

A. Isotope bone scan


Answer & Comments
B. MRI scan of the thoracic spine
Correct answer: C
C. CT scan of the thoracic spine
D. Myeloma screen Anaphylaxis is caused by histamine release in
E. Plain radiograph of the thoracic spine response to contact with an allergen. In
sensitized people, specific IgE attached to mast
cells binds to ingested or inhaled allergen,
Answer & Comments
causing the mast cell to degranulate, releasing
Correct answer: B histamine. If sufficient histamine is released,
anaphylaxis occurs. The symptoms of
The clinical picture suggests malignancy or anaphylaxis are facial and laryngeal oedema,
chronic infection, although sometimes wheeze and hypotension. Uriticaria and
osteoarthritis may produce severe pain. flushing may also occur. Not all features are
Osteoporotic crush fractures usually cause pain present in every case. Acute urticaria in this
of sudden onset. An isotope bone scan may situation would be suggestive of anaphylaxis
reveal the site of the painful lesion but is but chronic urticaria is usually non-allergic in
unlikely to reveal the cause, and may be normal nature.
in myeloma. Plain radiographs and CT scans will Pallor and hypotension are features of a simple
show evidence of advanced disease but may be faint. Patients have a slow weak pulse. In
normal in early malignancy and infection. MRI situation B this is the most likely diagnosis
scanning is the most sensitive technique, and unless additional features of anaphylaxis are
will also give information about associated soft present.
tissues.

Dr. Khalid Yusuf El-Zohry – Sohag Teaching Hospital (01118391123) Page | 345
El-Zohry MRCP Questions Bank (Part 2) – Medical Masterclass 2010

Scrombrotoxin poisoning occurs when fishes  progressive neurological deficit


such as mackerel or tuna are spoiled, when  past history of malignancy or
they may produce large amounts of histamine.
immunosuppression
Unlike anaphylaxis, where symptoms are
usually fairly immediate, signs of scrombrotoxin  recent trauma.
poisoning occur at least an hour after ingestion
of the suspect food. Symptoms would be [ Q: 27 ] MasterClass Part2
expected to respond to epinephrine. (2010) - Rheumatology
A 25-year-old man presents with a 3-week
[ Q: 26 ] MasterClass Part2
history of haemoptysis and testicular pain. The
(2010) - Rheumatology only significant findings on examination are
(2) A 34-year-old man presents with severe low testicular tenderness and left-sided episcleritis.
back pain, which has forced him to stop work as The results of initial investigations are as
a bus driver. He has had back pain on and off follows: serum CRP 124 g/l (nr <8), urine
for many years, on occasion with right-sided sediment: red cell casts, serum creatinine 78
sciatica. The pain used to be helped by rest, but micromol/L, serum C3 124 mg/dl (nr 65-185),
is now present more or less all the time and is serum C4 32 mg/dl (nr 15-50), antinuclear
stopping him from sleeping properly. antibody negative, chest radiograph: bilateral
nodular shadowing.
The most likely diagnosis is:
Which of the following diagnoses is most likely
A. mechanical back pain
to account for his clinical presentation?
B. ankylosing spondylitis
A. Systemic lupus erythematosus (SLE)
C. myeloma
B. Small cell lung carcinoma
D. osteoporosis
C. Mixed cryoglobulinaemia
E. osteoarthritis.
D. Small vessel vasculitis of the Wegener's
microscopic polyangiitis spectrum
Answer & Comments
E. Giant cell arteritis.
Correct answer: A
Answer & Comments
The history is typical of a patient with
Correct answer: D
mechanical back pain, best treated by
encouraging mobilization, simple analgaesia, a
graded rehabilitative exercise programme and This patient has an inflammatory multi-system
treatment of depression (if present). disorder of relatively short duration affecting
his lungs, kidneys, eyes and testes. The
‘Red flag’ symptoms, requiring urgent
constellation of nodular lung shadows,
investigation to exclude sinister pathology,
glomerulonephritis (as evidenced by red cell
include:
casts in the urine), episcleritis, testicular
 age >55 or <18 years tenderness and an elevated CRP is highly
 suggestive of an ANCA+ necrotising small vessel
progressive pain
vasculitis. Systemic immune-complex diseases
 night pain such as SLE and mixed cryoglobulinaemia are
 systemic symptoms

Dr. Khalid Yusuf El-Zohry – Sohag Teaching Hospital (01118391123) Page | 346
El-Zohry MRCP Questions Bank (Part 2) – Medical Masterclass 2010

excluded by the negative ANA and normal Synovial fluid analysis shows abundant calcium
complement profile, respectively. pyrophosphate dihydrate (CPPD) crystals.
This patient needs assessment of his ANCA
Which of the following tests is NOT appropriate
status and tissue biopsy-either kidneys or lungs,
for further assessment of this patient illness?
preferably kidneys.
A. Creatinine kinase

[ Q: 28 ] MasterClass Part2 B. Serum calcium


(2010) - Rheumatology C. Thyroid function test

A 40-year-old woman suddenly develops severe D. Serum ferritin level


Raynauds that is troublesome in the summer as E. Hb AIc.
well as the winter. She starts to develop
swelling of the fingers and feels tired. She also
Answer & Comments
develops reflux oesophagitis and has difficulty
swallowing. She has noticed that she has Correct answer: A
become more breathless. On examination, she
has skin thickening affecting her hands, face Pseudogout describes acute attacks of CPPD
and trunk. crystal-induced synovitis which clinically
resembles urate gout. However, the majority of
Which of the following is most likely to be
individuals with CPPD crystal deposition never
positive?
experience such episodes.
A. Ds DNA antibody
Chondrocalcinosis is the term used to describe
B. Ro antibody the calcium-containing deposits that are found
C. Anticentromere antibody in cartilage and which are usually visible on
joint radiographs. A variety of metabolic and
D. Jo-1 antibody endocrine disorders are associated with CPPD
E. Scl-70 antibody. crystal deposition, which include:
 diabetes mellitus,
Answer & Comments  hemochromatosis,
Correct answer: E  Wilson's disease,
 hypothyroidism,
This patient is developing diffuse systemic
sclerosis. Anticentromere pattern is associated  hyperparathyroidism,
with limited systemic sclerosis. Ro antibody is  hypomagnesemia,
common in patients with primary Sjogren's
 hypophosphatasia.
syndrome and systemic lupus erythematosus
(SLE). Jo-1 may be positive in patients with
polymyositis. [ Q: 30 ] MasterClass Part2
(2010) - Rheumatology
[ Q: 29 ] MasterClass Part2
A 30-year-old man has a swollen wrist, low back
(2010) - Rheumatology pain with early morning stiffness lasting 2 hours
A 68-year old man presented with sudden and mouth ulcers. He does not have a rash.
severe pain and swelling in the left knee.
What is the most likely diagnosis?

Dr. Khalid Yusuf El-Zohry – Sohag Teaching Hospital (01118391123) Page | 347
El-Zohry MRCP Questions Bank (Part 2) – Medical Masterclass 2010

A. Rheumatoid arthritis (RA) Amongst the array of mediators released by


mast cells during anaphylaxis, tryptase has
B. Psoriatic arthritis
proven to be a reliable marker of mast cell
C. Ankylosing spondylitis degranulation on account of its stability,
D. Gout relatively long half-life and ease of
measurement. Consequently, an elevated
E. systemic lupus erythematosus (SLE)
tryptase in the correct clinical context is a
useful surrogate marker of an
Answer & Comments anaphylactic/anaphylactoid reaction.
Correct answer: C
[ Q: 32 ] MasterClass Part2
The history is suggestive of a seronegative (2010) - Rheumatology
spondyloarthropathy.
A 52-year-old accountant, with a 6-year history
RA is classically a peripheral, symmetrical of Raynaud’s phenomenon presents to her G.P.
polyarthritis. with a 4-month history of worsening dysphagia.
Gout can affect the wrist but is unusual in a
young person.
Patients with SLE can experience mouth ulcers
but SLE is uncommon in young males.

[ Q: 31 ] MasterClass Part2
(2010) - Rheumatology
An 18-year-old student with known asthma and
peanut allergy collapses following a meal in the
hospital canteen. Strenuous attempts at
resuscitation are tragically unsuccessful. It is
thought that the most likely cause of his
collapse was an anaphylactic reaction to
‘hidden' nuts contained in the meal.

Which of the following mast cell-derived


mediators is it most useful to measure to
confirm the clinical suspicion of anaphylaxis?
A. Mast cell - derived growth factor
B. Chymase
C. Interleukin-6 Incubation of her serum on Hep 2 cells reveals
the following immunofluorescent pattern (see
D. Tryptase image).
E. Transforming growth factor-β A. Primary Raynauds disease
B. systemic lupus erythematosus (SLE)
Answer & Comments
C. Limited systemic sclerosis
Correct answer: D
D. Rheumatoid arthritis

Dr. Khalid Yusuf El-Zohry – Sohag Teaching Hospital (01118391123) Page | 348
El-Zohry MRCP Questions Bank (Part 2) – Medical Masterclass 2010

E. Sjogren's disease. particular secondary tumours or myeloma -


would need to be considered.
Answer & Comments It is most likely that the cause of osteoporosis
Correct answer: C would be postmenopausal in this woman, but
many medical factors influence the likelihood of
this condition.
Raynaud’s phenomenon is common, occurring
in 4-15% of the general population and in the The following tests should be considered in a
majority of cases is not associated with patient presenting with an osteoporotic
connective tissue disease. However, the fracture (although not all need to be performed
detection of an anti-centromere nuclear in all cases): full blood count, renal / liver /
staining pattern has a high specificity for limited bone chemistry, immunoglobulins and serum /
systemic sclerosis, especially in the context of urine electrophoresis (to exclude myeloma),
dysphagia which is a feature of the disease. thyroid function, testosterone (in men),
investigations for Cushing's syndrome.

[ Q: 33 ] MasterClass Part2
(2010) - Rheumatology [ Q: 34 ] MasterClass Part2
(2010) - Rheumatology
A 60-year-old woman develops acute, severe
low thoracic back pain. A radiograph shows a A 36-year old woman is referred with a 1-year
vertebral crush fracture. history of muscle pain, tiredness and sleep
disturbance. She denies fever, weight loss and
Which TWO of the following reduce an arthralgia. Examination reveals tenderness over
individual's risk for developing osteoporosis? her occiput, trapezius and lumbar area. Her
A. Early menarche blood results show a normal ESR, CRP, FBC, a
weakly positive ANA 1:80 and normal
B. Early menopause complement.
C. Smoking
Which is the most likely diagnosis?
D. High alcohol intake
A. Polymyositis
E. Prolonged treatment with corticosteroids
B. System lupus erythematous (SLE)
F. Rheumatoid arthritis
C. Sjogren’s syndrome
G. Crohn's disease
D. Polymyalgia rheumatica
H. Asthma
E. Fibromyalgia
I. Frequent walks
J. Obesity. Answer & Comments

Correct answer: E
Answer & Comments

Correct answer: AI The lack of constitutional symptoms, normal


inflammatory markers and normal examination,
The most likely cause of a vertebral crush factor apart from evidence of tender points, make an
in a woman of this age is obviously inflammatory rheumatological disease unlikely.
osteoporosis, but local vertebral pathology - in The presence of tender points, history of
muscle pain and sleep disturbance are

Dr. Khalid Yusuf El-Zohry – Sohag Teaching Hospital (01118391123) Page | 349
El-Zohry MRCP Questions Bank (Part 2) – Medical Masterclass 2010

suggestive of fibromyalgia – a non- This patient is not able to stand up from a chair
inflammatory pain disorder. unaided. This indicates proximal muscle
weakness. The skin rash and the raised levels of
[ Q: 35 ] MasterClass Part2 muscle enzyme point towards an inflammatory
myopathy, namely dermatomyosytis (DM).
(2010) - Rheumatology
DM is easily recognized and diagnosed by a
This patient was trying to stand up. On characteristic rash:
examination he has periorbital oedema with a
 a heliotrope rash (blue-purple
faint purple hue. The serum creatinine kinase
(CK) levels were 10-times the upper limit of discoloration) on the upper eyelids,
normal. with oedema
 a flat red rash on
the face and upper trunk.
Erythema of the knuckles
accompanied by a raised,
violaceous scaly eruption
(Gottron's sign) is also
characteristic, and may
precede or follow muscle
weakness. Raynaud's
phenomenon with
characteristic color
change (pallor, cyanosis
and rubor) on exposure to
cold is encountered in
patient with DM, more
often when there is
overlap with other
connective tissue disease such as scleroderma
Which of the following manifestations is least or mixed connective tissue disease.
likely to be associated with this disorder?
Inflammation of the skeletal muscles of the
A. blue and white color change in the fingers oropharynx and upper oesophagus lead to
on exposure to cold dysphagia especially for liquids in the initial
B. difficulty in swallowing liquids stages of the disease which progresses to
difficulty in swallowing solids as the disease
C. double vision on looking to the sides
progresses.
D. radial pulse of 40 beats per minute
Electrocardiographic evidence of conduction
E. persistent cough, haemoptysis and defects and arrhythmias occur frequently in
weight loss of 12 kg in the last three childhood and adult DM, although overt clinical
months symptoms are uncommon.
There is an increased incidence of underlying
Answer & Comments malignancy in adult DM, ranging from 5% to
Correct answer: C 15%. Carcinoma of the bronchus, ovaries and
breast are the most common.

Dr. Khalid Yusuf El-Zohry – Sohag Teaching Hospital (01118391123) Page | 350
El-Zohry MRCP Questions Bank (Part 2) – Medical Masterclass 2010

Ocular muscles remain normal, even in Sjorgren's syndrome. The positive ANA and high
advanced, untreated cases, and if these globulins suggest Sjorgren's but could also be
muscles are affected, the diagnosis of associated with chronic infection, such as
inflammatory myopathy should be in doubt. hepatitis C.
Since they precipitate at low temperatures,
[ Q: 36 ] MasterClass Part2 cryoglobulins should always be transported to
(2010) - Rheumatology the lab at 37°C. Failure to do this will result in a
false negative result as the cryos will precipitate
A 34-year-old woman presents to your clinic and be removed with the clot.
complaining of cold hands, particularly in the
winter months. On examination, she has cold
dusky hands and a petechial rash. Investigations [ Q: 37 ] MasterClass Part2
are as follows: Hb 10.9 g/dL; Wbc 4.2 109/L; (2010) - Rheumatology
Platelets 407 x 109/L; urea and electrolytes -
A 69-year-old man with a 10-year history of
normal; liver function tests - normal; albumin
rheumatoid arthritis, controlled with diclofenac
36 g/L; globulin 90 g/L; protein electrophoresis -
and sulphasalazine has reported recurrent
polyclonal increase in gammaglobulins;
heart burn and dyspepsia. His doctor asked him
antinuclear antibodies present (1/160, speckled
to stop the dicofenac sodium and replaces it
pattern); complement - C3 0.79 (NR 0.75-1.25),
with celecoxib (one of the COX 2 inhibitors).
C4 <0.04 (NR 0.14-0.6).
Which of the following is true about COX 2
Which of the following statements is true?
selective agents?
A. Active systemic lupus erythematous (SLE)
A. They eliminate the chance of non-
is unlikely if DNA antibodies are present.
steroidal anti-inflammatory drugs
B. A blood sample sent to the lab on ice may (NSAID)-associated dyspepsia.
show cryoglobulins.
B. They are now regarded as the primary
C. Sjorgren’s syndrome is unlikely if treatment for rheumatoid arthritis.
rheumatoid factor is present.
C. They are more effective than ordinary
D. Sjorgren’s syndrome is likely if Ro and La NSAIDs.
extractable nuclear antigens are present.
D. Because of their potent antiplatelet
E. Hepatitis C is unlikely in this case. effect the addition of low dose aspirin for
ischaemic heart disease prophylaxis is not
Answer & Comments necessary.
E. They can cause acute renal failure when
Correct answer: D
used in patients with impaired renal
function.
The symptoms and signs and low C4 are
suggestive of cryoglobulinaemia.
Answer & Comments
Sludging of proteins at reduced temperatures
(for example hands on a cold day) can cause Correct answer: E
ischaemia and sometimes vasculitis, particularly
of skin or kidneys. NSAIDs inhibit prostaglandin synthesis by
inhibiting the enzyme cyclooxygenase (COX).
Cryoglobulinaemia is commonly associated with
The existence of two different forms of COX
hepatitis C or connective tissue disease, such as
isoenzymes (COX-1 and COX-2) has now been
Dr. Khalid Yusuf El-Zohry – Sohag Teaching Hospital (01118391123) Page | 351
El-Zohry MRCP Questions Bank (Part 2) – Medical Masterclass 2010

recognised. COX-1 has a constitutive Which of the following treatment regimens


'housekeeping' role and COX-2 upregulation is offer her the best chance of having a successful
associated with inflammation. The COX-2- pregnancy?
selective NSAIDs are not more effective than A. Steroids alone
ordinary NSAIDs, they are not the primary
treatment for rheumatoid arthritis, and they do B. Steroids combined with low-dose aspirin
not eliminate the chance of NSAID-associated C. Low-dose aspirin alone
dyspepsia. Unlike aspirin and most traditional
D. Low-dose aspirin combined with low
NSAIDs, they have no antiplatelet effect, so
molecular weight heparin
they are not cardioprotective.
E. Intravenous immunoglobulin.
In clinical trials COX-2 selective NSAIDS reduced
the risk of developing significant upper
gastrointestinal complications in the first six Answer & Comments
months. However, the risk of GI bleeding from Correct answer: D
nonselective NSAIDs is low - an annual
incidence of 1% to 3% - and the COX-2-selective
Of the above treatment options, low-dose
NSAIDs are costly. Therefore, it may be best to
aspirin combined with low molecular weight
reserve these drugs for patients at high-risk
heparin offers her the best chance of having a
patients to include those over 65 years, those
successful pregnancy.
already using medications known to increase
the likelihood of upper gastrointestinal adverse
events, those with serious co-morbidity, and [ Q: 39 ] MasterClass Part2
those requiring prolonged use of maximum (2010) - Rheumatology
recommended doses of standard NSAIDs.
You are called to A&E to review a 28-year-old
All NSAIDs, including the COX-2 selective woman with pleuritic chest pain.
agents, can cause renal toxicity, such as acute
renal failure, nephrotic syndrome, and acute Which of the following statements concerning
interstitial nephritis. Acute renal failure is a this patient are true?
concern particularly in patients with pre-
A. Low white count is consistent with
existing renal dysfunction. NSAIDs should be
systemic lupus erythematosus (SLE)
avoided also in patients with cirrhosis and
ascites or congestive heart failure. Periodic B. Low white count is not consistent with
monitoring of renal function is recommended, tuberculosis.
particularly in elderly patients. C. The presence of anticardiolipin antibodies
IgG 17 GPLU/ml (NR <14), IgM 21
[ Q: 38 ] MasterClass Part2 MPLU/ml (NR<10) suggests that she may
(2010) - Rheumatology have a pulmonary embolus.
D. The presence of antinuclear antibodies at
A 25-year-old woman with a history of 3-second
a titre of 1/80 suggests that she may have
trimester fetal losses is planning a fourth
a connective tissue disease.
pregnancy. She has evidence of the primary
anti-phospholipid syndrome (strongly positive E. The presence of antinuclear cytoplasmic
cardiolipin antibody, positive lupus antibodies (cANCA) at a titre of 1 in 20
anticoagulant but no evidence of lupus). suggests that Wegener's granulomatosis
is likely.

Dr. Khalid Yusuf El-Zohry – Sohag Teaching Hospital (01118391123) Page | 352
El-Zohry MRCP Questions Bank (Part 2) – Medical Masterclass 2010

Answer & Comments consequence of the destruction of synovial


joints, ligaments, and bone. Atlantoaxial
Correct answer: A
subluxation (AAS) is the most common. Patients
may experience weakness, decreased
Active SLE and tuberculosis are both associated endurance, gait difficulty, paresthesias of the
with leukopenia. Low titre antinuclear hands, and loss of fine dexterity. Multiple
antibodies (in the absence of DNA or ENA), neurological signs may be elicited on physical
antineutrophil cytoplasmic antibodies (ANCA) examination, including diffuse hyperreflexia,
and anticardiolipin antibodies are non-specific lower extremity spasticity, a spastic gait, and
and are commonly found in the presence of Babinski's sign.
infection.
Although plain radiography of the cervical spine
is regarded as the initial imaging assessment
[ Q: 40 ] MasterClass Part2 tool for neck pain in patient with RA, those with
(2010) - Rheumatology symptoms or signs of cord compression should
undergo immediate MRI and be sent for
A 66-year-old woman has a 15-year history of
surgical consultation. MRI is considered the
deforming rheumatoid arthritis (RA). She is
most sensitive imaging modality for diagnosing
maintained on D-Penicillamine. Two weeks ago
spinal cord compression, evaluating its extent,
she noticed increased difficulty in climbing
and assessing soft tissues (panus) as well as
stairs and three days before admission she was
bone destruction.
unable to comb her hair or feed herself.
Neurological assessment is difficult D-Penicillamine causes various neuromuscular
but reveals generalised grade 3/5 weakness, disorders including myathenia gravis.
and the Babiniski sign is positive bilaterally. There is no clinical evidence for primary muscle
disease or peripheral neuropathy in this patien,
Which one of the following tests is most likely to
hence EMG or NCS are not necessary.
reveal the diagnosis?
Isotope bone scan may identify increased
A. Plain radiograph of the cervical spine
activity in the cervical region, but this is not
B. Electromyography (EMG) specific and the investigation has little role in
C. Nerve conduction study (NCS) investigation of suspected spinal cord
compression.
D. Isotope bone scan
E. Magnetic resonance imaging (MRI) of the [ Q: 41 ] MasterClass Part2
cervical spine
(2010) - Rheumatology

Answer & Comments A 70-year-old woman with a history of blood


transfusion in the early 1980s presents with a
Correct answer: E 10-month history of malaise and is noted to
have impaired renal function. Her urine
The recent limb weakness and the presence of sediment reveals red cell casts. The results of
pyramidal signs in the legs in patients with RA immunological investigations are as follows:
are highly suggestive of spinal cord lesion at the serum IgG 6.5 g/L (normal range 6-13), IgA 1.5
cervical or thoracic region. g/L (normal range 0.8-4.0), IgM 5.7 g/L (normal
RA primarily affects the cervical spine; affliction range 0.4-2.0), serum electrophoresis shows
of the thoracic or lumbar spine is rare. The faint band in gamma region, complement C3
anatomical abnormalities occur as a 1.02 g/L (normal range 0.75-1.65), complement

Dr. Khalid Yusuf El-Zohry – Sohag Teaching Hospital (01118391123) Page | 353
El-Zohry MRCP Questions Bank (Part 2) – Medical Masterclass 2010

C4 <0.02 g/L (normal range 0.20-0.65) and Answer & Comments


rheumatoid factor 894 IU/L (normal range <40).
Correct answer: D
Which of the following investigations is likely to
be most important in making a definitive Hydroxychloroquine can exacerbate psoriasis.
diagnosis? Sulphasalazine tends to only improve joint
symptoms and not improve the psoriasis. Gold
A. Antineutrophil cytoplasmic antibodies
and penicillamine are not commonly used to
B. Antinuclear antibodies treat this condition.
C. Antiglomerular basement membrane
antibodies [ Q: 43 ] MasterClass Part2
D. Antimyeloperoxidase antibodies (2010) - Rheumatology
E. Cryoglobulins A 45-year-old woman developed Raynaud's 2
years ago. She now complains of breathlessness
Answer & Comments and skin tightness affecting her fingers. A high-
resolution CT scan shows evidence of
Correct answer: E pulmonary fibrosis.

The combination of a markedly low C4 (with Which of the following tests is most likely to be
normal C3), elevated rheumatoid factor, positive?
elevated serum IgM on a background of active A. Anticentromere antibody
urinary sediment and a history of blood
transfusion is highly suggestive of hepatitis C- B. Anti-double-stranded DNA antibody
associated cryoglobulinaemic vasculitis. Of the C. Anti-Ro antibody
investigations listed, cryoglobulins are the
D. Anti-Scl-70 antibody
single most important test in establishing a
definitive diagnosis in this patient. E. Anti-Jo-1 antibody

[ Q: 42 ] MasterClass Part2 Answer & Comments


(2010) - Rheumatology Correct answer: D
A patient with psoriatic arthritis has active
This patient has clinical symptoms suggestive of
joints and troublesome plaque psoriasis.
diffuse cutaneous systemic sclerosis. Pulmonary
fibrosis and anti-Scl-70 are more common in
Which of the following will improve both the
patients with diffuse disease. Anticentromere
joint and skin problems?
antibody is associated with limited cutaneous
A. Sulphasalazine systemic sclerosis. Anti-double-stranded DNA
B. Hydroxychloroquine antibody is associated with systemic lupus
erythematosus. Anti-Ro antibody is associated
C. Gold
with lupus and primary Sjögren's syndrome.
D. Methotrexate Anti-Jo-1 is associated with polymyositis,
particularly in patients with inflammatory lung
E. Penicillamine.
disease.

Dr. Khalid Yusuf El-Zohry – Sohag Teaching Hospital (01118391123) Page | 354
El-Zohry MRCP Questions Bank (Part 2) – Medical Masterclass 2010

[ Q: 44 ] MasterClass Part2 Answer & Comments


(2010) - Rheumatology Correct answer: BE
A 58-year-old woman is referred with pain and
stiffness in her hands and knees. She has a few Swelling of the distal interphalangeal joints
patches of psoriasis on her arms. Her hands are really only occurs in nodal osteoarthritis and
shown in the picture. one of the forms of psoriatic arthritis, which is

usually easily distinguished by nail involvement


What are the TWO most likely diagnoses?
(not present in this case).
A. Gout
Involvement of the base of the thumb is also
B. Nodal osteoarthritis pathognomonic of nodal osteoarthritis, giving
C. Pseudogout the thumb base a characteristically square
appearance.
D. Systemic sclerosis
E. Psoriatic arthritis [ Q: 45 ] MasterClass Part2
F. Systemic lupus erythematosus (2010) - Rheumatology
G. Ankylosing spondylitis A 35-year-old man is referred to you for
H. Rheumatoid arthritis investigation of recurrent infection. He has had
frequent respiratory tract infections for the
I. Reactive arthritis
past 5 years, requiring 4-5 courses of antibiotics
J. SAPHO syndrome. each winter. A month previously he was
admitted with pneumococcal pneumonia. Two
months prior to that he had sinus surgery.

Dr. Khalid Yusuf El-Zohry – Sohag Teaching Hospital (01118391123) Page | 355
El-Zohry MRCP Questions Bank (Part 2) – Medical Masterclass 2010

However, this did not improve his recurrent fever one month after receiving a BCG vaccine.
sinusitis. A lymph node biopsy reveals acid-fast bacilli.

Which of the following is not in the differential Which 2 of the following components of the
diagnosis? immune system is it most important to assess?
A. Antibody deficiency A. T-cells
B. HIV infection B. B-cells
C. Bronchiectasis secondary to recurrent C. C3
infection
D. Neutrophils
D. Complement C6 deficiency
E. Platlets
E. Smoking 5 cigarettes per day.
F. IL-10
G. IFN-gamma
Answer & Comments
H. Mannan-binding-protein
Correct answer: D
I. C4
Antibody deficiency is typically associated with
J. IL-2.
respiratory tract infections. Ask about diarrhoea
and bacterial skin infections which are also
common. Take a careful drug history and bear Answer & Comments
in mind the possibility of lymphoproliferative Correct answer: AG
disease. HIV infection, although primarily
associated with CD4 loss, also results in
Disseminated BCG infection is uncommon and
antibody dysfunction, resulting in recurrent
suggests an underlying immune deficiency. The
respiratory tract infections in some patients.
immune response to mycobacterial infection is
Ask about features of cellular immune
characterised by granulomas composed of T-
deficiency (oral candida, herpes simplex and
cells and macrophages, whose development
zoster, warts). Ask about risk factors.
critically involves interferon gamma, IL-12 and
Recurrent bacterial chest infections, whatever TNF. Patients with inherited deficiencies of IL-
their cause, will eventually result in 12 and IFN-gamma are predisposed to
bronchiectasis, hence the importance of early disseminated BCG infection and tuberculosis is
diagnosis and treatment. Terminal complement a recognised risk in patients treated with anti-
deficiencies (C5-9) are extremely rare. Patients TNF therapy.
are well but have increased susceptibility to
neisserial infection.
[ Q: 47 ] MasterClass Part2
Smoking causes ciliary paralysis. The resultant (2010) - Rheumatology
mucociliary dysfunction is a common (and
reversible) cause of recurrent respiratory tract A 65-year-old man is admitted as an emergency
infection. with a very hot, swollen left knee. On
examination he is unwell and pyrexial. He has
marked loss of range of movement, secondary
[ Q: 46 ] MasterClass Part2 to pain. You have aspirated his knee and have
(2010) - Rheumatology sent the purulent-looking fluid for microscopy.
An infant presents with widespread
Which of the following is true?
lymphadenopathy, hepatosplenomegaly and
Dr. Khalid Yusuf El-Zohry – Sohag Teaching Hospital (01118391123) Page | 356
El-Zohry MRCP Questions Bank (Part 2) – Medical Masterclass 2010

A. He is likely to have acquired this infection In the absence of further clues in the history or
after an arthroscopy or arthrocentesis. examination, which single blood test is the most
important?
B. Seventy to eighty percent of cases will
have an accompanying bacteraemia. A. HIV antibody test
C. Results of the culture should be awaited B. Pneumococcal antibodies
before commencing antibiotics. C. Immunoglobulin levels
D. Antibiotics should cover beta-haemolytic D. Liver function tests
streptococcus and staphylococcus
infections. E. IgG subclass levels.

E. A plain x-ray of the knee will confirm the


Answer & Comments
diagnosis.
Correct answer: C
Answer & Comments
Hypogammaglobulinaemia is associated with
Correct answer: D
recurrent bacterial infections, most commonly
of the respiratory tract. Delay of several years
This is likely to be septic arthritis. Obviously, prior to diagnosis is usual, with associated
examination of the synovial fluid will help to morbidity. Patients with low immunoglobulin
exclude differentials such as gout and levels and recurrent infections should be
pseudogout. treated with immunoglobulin replacement.
The most likely organisms are beta-haemolytic More minor antibody defects, such as IgG
streptococci (20%) and staphlococci (70%). subclass or specific antibody (to pneumococcus)
Antibiotics should be started empirically to defects can often be treated with appropriate
cover these if the clinical suspicion is high. vaccinations and/ or prophylactic antibiotics.
Ideally these should be intravenous for 2 weeks
and then oral for 4 weeks. Arthroscopy is a risk [ Q: 49 ] MasterClass Part2
factor for septic arthritis, but is rare. Fifty
(2010) - Rheumatology
percent of cases will have an associated
bacteraemia. Early x-rays are almost always A 16-year-old boy is admitted to the A&E
normal. department after collapsing following a wasp
sting. His blood pressure is recorded at 90/50
[ Q: 48 ] MasterClass Part2 mmHg and examination reveals a widespread
urticarial rash. His blood pressure and rash
(2010) - Rheumatology
respond to 0.5mg i.m. adrenaline, 100mg
A 28-year-old man is has recently been hydrocortisone i.v. and 10 mg chlorpheniramine
discharged from hospital after treatment for i.v.
pneumococcal pneumonia. He has had
repeated courses of antibiotics for sinus, ear After being seen in clinic TWO weeks later,
and lower respiratory tract infections, and had which TWO of the following tests would be
sinus surgery the previous year. He is a life long most clinically useful?
non-smoker and is not on medication. His blood A. IgE
count prior to discharge was normal.
B. Wasp venom radioallergensobent test
(RAST)
C. Serum tryptase

Dr. Khalid Yusuf El-Zohry – Sohag Teaching Hospital (01118391123) Page | 357
El-Zohry MRCP Questions Bank (Part 2) – Medical Masterclass 2010

D. Antinuclear antibodies (ANA) Answer & Comments


E. Thyroid-stimulating hormone (TSH) Correct answer: D
F. Wasp venom skin prick test
Uncontrolled activation of the classical
G. Urinary catecholamines
complement pathway leading to a marked
H. Liver function test (LFT) reduction in serum C4 levels is a hallmark of C1
I. Eosinophil count inhibitor deficiency. In the absence of a family
history, it is possible that this represents a new
J. Lymphocyte subsets mutation. Alternatively, the possibility of
acquired C1 inhibitor deficiency due to
Answer & Comments lymphoproliferative disease should be
considered.
Correct answer: BF

This patient has suffered an anaphylactic [ Q: 51 ] MasterClass Part2


reaction to bee venom. A rise in serum tryptase (2010) - Rheumatology
(if taken within 6 hours) can help in identifying
A 77-year-old man presents with persistent
cases of anaphylaxis where there is diagnostic
head ache and progressive deafness. On
uncertainty. However, in this patient the
examination the patient has frontal bossing of
classical features and temporal relationship to a
the forehead and conductive deafness, more
bee sting establish the diagnosis clinically. He
severe in the right ear. His serum alkaline
should be tested for venom specific IgE by both
phosphatase is significantly raised at 870 u/L.
skin prick testing and RAST; if either are positive
his anaphylactic risk for a further wasp sting can Which of the following statements is most
be dramatically lowered by venom accurate about this disease?
immunotherapy
A. It usually affects a single bone.

[ Q: 50 ] MasterClass Part2 B. The skull is the most commonly affected


bone.
(2010) - Rheumatology
C. Bone pain is the most common
A 50-year-old woman presents to casualty with presenting feature.
a 2-year history of recurrent angioedema. Her
clinical history fails to reveal an underlying D. Hearing loss is often due to involvement
trigger. Her serum complement profile is as of the middle ear ossicles leading to
follows: serum C3 1.2 g/l (ref range 0.75-1.65), conductive deafness.
serum C4 <0.02 g/l (ref range 0.2-0.6). E. Bone pain is typically increased with rest
and on weight bearing.
The most likely diagnosis is:
A. food allergy Answer & Comments
B. drug allergy Correct answer: E
C. Idiopathic angioedema
D. C1 inhibitor deficiency Paget's disease of bone is a focal disorder of
bone remodelling characterized by an increase
E. venom allergy. in the number and size of osteoclasts in
affected skeletal sites while the rest of the

Dr. Khalid Yusuf El-Zohry – Sohag Teaching Hospital (01118391123) Page | 358
El-Zohry MRCP Questions Bank (Part 2) – Medical Masterclass 2010

skeleton is spared. Paget’s disease most [ Q: 53 ] MasterClass Part2


commonly involves the axial skeleton, the pelvis (2010) - Rheumatology
being the most common, but it can affect any
area. In the majority of patients, the disease A 72-year-old man suffers a stroke causing left
affects at least two bones, but in one third of sided weakness. He has very little voluntary
patients only one bone is affected. In the skull, movement of the left arm and needs assistance
the 8th nerve can be compressed, resulting in to transfer. Whilst on the rehabilitation ward he
hearing loss. This is one of the more common complains of significant pain around his left
complaints, being present in 37% of shoulder.
respondents in a recent survey of 2000 patients
with Paget's disease . Other causes of hearing Which of the following statements is correct?
loss include pagetic involvement of the middle A. Poor handling and positioning of the left
ear ossicles, which dampens the motion of arm by staff are unlikely to have
these ossicles. contributed to the pain
Unlike osteoarthritis, pagetic bone pain usually B. Shoulder pain causes distress but does
increases with rest, on weight bearing, when not affect outcome
the limbs are warmed, and at night. An
C. A radiograph of the shoulder is not
estimated 70% of patients who have Paget’s
necessary
disease have no symptoms. The diagnosis is
typically found incidentally on radiographs and D. Treatment with simple analgesics is a
laboratory investigations. sensible initial approach
E. Intra-articular steroids are likely to
[ Q: 52 ] MasterClass Part2 exacerbate the problem.
(2010) - Rheumatology
Answer & Comments
A young patient is attending the rheumatology
monitoring clinic. She develops leucopaenia. Correct answer: D

Which of the following is unlikely to cause this


Shoulder pain is a common complication of
abnormality?
stroke. The aetiology is thought to be
A. Azathioprine multifactorial and includes poor handling of the
affected arm causing trauma to the shoulder
B. Methotrexate
structures, glenohumeral subluxation,
C. Systemic lupus erythematosus (SLE) abnormal tone (spasticity and flaccidity) and
D. Cyclosporin A pain due to the stroke itself (central post-stroke
pain). It is associated with prolonged hospital
E. Etanercept.
stay and poor recovery of arm function. A
radiograph should be performed to exclude
Answer & Comments fracture or dislocation. Treatment usually starts
with simple analgesics or non-steroidals.
Correct answer: D
Physiotherapists may treat subluxation with
strapping.If the pain persists intra-articular
Patients taking azathioprine, methotrexate and
steroid injections and TENS may help.
etanercept require regular monitoring of the
full blood count to exclude marrow
suppression. Patients with SLE often have a low [ Q: 54 ] MasterClass Part2
white cell count.
Dr. Khalid Yusuf El-Zohry – Sohag Teaching Hospital (01118391123) Page | 359
El-Zohry MRCP Questions Bank (Part 2) – Medical Masterclass 2010

(2010) - Rheumatology they may produce large amounts of histamine.


Unlike anaphylaxis, where symptoms are
A 19-year-old woman collapses in a restaurant, usually fairly immediate, signs of scrombrotoxin
shortly after beginning her meal. poisoning occur at least an hour after ingestion
of the suspect food. Symptoms would be
Which of the following statements is true?
expected to respond to epinephrine.
A. Anaphylaxis is not the cause if neither
stridor nor wheeze is audible.
[ Q: 55 ] MasterClass Part2
B. If pallor and hypotension are present: (2010) - Rheumatology
anaphylaxis is the likely diagnosis.
A 78-year-old woman found it much easier to
C. If she has facial swelling and wheeze: manage after the occupational therapist
anaphylaxis is likely even if there is no provides several aids and appliances for use in
hypotension. the kitchen.
D. A history of chronic urticaria implies that
she is allergic to something.
E. Scrombrotoxin poisoning is a possibility
and will not respond to intramuscular
epinephrine.

Answer & Comments

Correct answer: C

Anaphylaxis is caused by histamine release in


response to contact with an allergen. In
sensitized people, specific IgE attached to mast
cells binds to ingested or inhaled allergen,
causing the mast cell to degranulate, releasing From their appearance, the problem affecting
histamine. If sufficient histamine is released, her hands (see image) is:
anaphylaxis occurs. The symptoms of
A. rheumatoid arthritis
anaphylaxis are facial and laryngeal oedema,
wheeze and hypotension. Uriticaria and B. nodal osteoarthritis
flushing may also occur. Not all features are C. psoriatic arthropathy
present in every case. Acute urticaria in this
situation would be suggestive of anaphylaxis D. tophaceous gout
but chronic urticaria is usually non-allergic in E. Charcot joints.
nature.
Pallor and hypotension are features of a simple Answer & Comments
faint. Patients have a slow weak pulse. In
Correct answer: C
situation B this is the most likely diagnosis
unless additional features of anaphylaxis are
present. This is the classic appearance of arthritis
mutilans in psoriatic arthropathy, with a
Scrombrotoxin poisoning occurs when fishes deforming destructive arthritis of the small
such as mackerel or tuna are spoiled, when joints. Osteolysis results in shortening of some

Dr. Khalid Yusuf El-Zohry – Sohag Teaching Hospital (01118391123) Page | 360
El-Zohry MRCP Questions Bank (Part 2) – Medical Masterclass 2010

of the digits. The pattern is not that of (2010) - Rheumatology


rheumatoid arthritis and the wrists appear to
be spared. There is too much destruction for A 68-year-old woman complained of
osteoarthritis. The appearance of gout may be progressive pain in her left hip after slipping on
similar, though the tophi would be easier to see ice and sustaining a sub-capital fracture of the
in life (as opposed to a photo). Charcot joints left femoral neck hip. Following recovery from a
(neuropathic arthropathy) usually affect the hemiarthroplasty, bone mineral density (BMD)
foot and ankle. measurement of the opposite femur confirmed
the diagnosis of osteoporosis with a T score of -
2.67. She did not wish to take oestrogen
[ Q: 56 ] MasterClass Part2 replacement therapy and was given
(2010) - Rheumatology alendronate 70 mg weekly.
A 56-year-old man presents with recurrent
Which of the following statements is true?
attacks of polyarticular gout despite treatment
with allopurinol 300 mg daily for the last year. A. Normally the peak bone mass in a woman
He is adamant that his compliance with his drug is achieved just before the menopause.
treatment regimen is good. B. Plain radiographs are very sensitive in
identifying minor reduction in bone
Which of the following factors is most likely to
mineral density and hence a very useful
explain his poor response to allopurinol?
screening test.
A. Concurrent treatment with colchicine
C. The T score compares a patient’s BMD
B. Fast metabolism of allopurinol with the mean value for young, healthy
C. High alcohol intake adults of the same sex.

D. Poor urate clearance via the kidney D. A high serum phosphate level may be
suggestive of chronic alcoholism.
E. High dietary purine intake
E. Bisphosphonates such as alendronate act
by stimulating bone formation.
Answer & Comments

Correct answer: C Answer & Comments

Correct answer: C
Persistently high alcohol consumption is a
common cause of poor response to allopurinol,
although the underlying mechanism of this is Bone mass increases during childhood and
unclear. B, D and E are plausible answers, but adolescence and peaks between the ages of 20
are less important in practice. Most adults will and 30 years. Peak bone mass is influenced by
respond to allopurinol 300 mg daily, although a age, sex, genetic factors, hormonal status,
small proportion will require 600 or even 900 exercise, and calcium intake. As women in
mg daily. The aim of treatment should be to general have a lower peak bone mass than
suppress the serum urate level to the lower end men, it is understandable why osteoporosis is
of the normal range or just below. predominantly seem in women.
Plain radiographs are not sensitive enough to
diagnose osteoporosis. Results of bone mineral
density (BMD) tests are typically reported as T
[ Q: 57 ] MasterClass Part2 scores and Z scores.

Dr. Khalid Yusuf El-Zohry – Sohag Teaching Hospital (01118391123) Page | 361
El-Zohry MRCP Questions Bank (Part 2) – Medical Masterclass 2010

 The T score compares a patient's BMD lytic lesions seen in multiple myeloma.
with the mean value for young, healthy Osteomalacia is associated with Looser's zones.
adults of the same sex.
 The Z score compares a patient's BMD [ Q: 59 ] MasterClass Part2
with the mean value for persons of the (2010) - Rheumatology
same age and sex.
A 33-year-old woman presents with a 6-month
Both scores are expressed in terms of standard history of Raynaud's phenomenon affecting her
deviations from the mean. The World Health hands and feet. She is previously well and takes
Organization has defined osteoporosis as a no medication.
BMD of at least 2.5 standard deviations (SD)
below the mean value in young normal adults Which TWO of the following clinical features or
(i.e. T<=-2.5). investigation results are the strongest
Chronic alcoholism is another important cause predictors that she will develop a connective
of low serum phosphate level, in which case the tissue disease in the future?
liver enzymes will be elevated. All A. Age >25 years
bisphosphonates act similarly on bone in
B. Abnormal nail-fold capillary microscopy
binding permanently to mineralized bone
surfaces and inhibiting osteoclastic activity. C. Elevated erythrocyte sedimentation rate
Thus, they inhibit bone resorption and less D. History of recurrent miscarriage
bone is degraded during the remodelling cycle.
They do not stimulate bone formation. E. Family history of Raynaud's
F. Anaemia
[ Q: 58 ] MasterClass Part2 G. Strongly positive antinuclear antibody
(2010) - Rheumatology H. History of chilblains
A 68-year-old man has lost weight. His alkaline I. Raised platelet count
phosphatase is raised at 290 U/L (normal range
J. Dry eyes and dry mouth
35-120). Plain radiographs show sclerotic
lesions of bone.
Answer & Comments
What is the likely diagnosis?
Correct answer: BG
A. Stomach cancer
B. Prostate cancer These factors are strongly predictive of a future
connective tissue disease (CTD), particularly
C. Multiple myeloma
abnormal nail-fold capillaries. The likelihood of
D. Lung cancer developing a CTD also increases with age of
E. Osteomalacia onset of Raynaud's, with a particularly high risk
in those aged over 35 years. All the other
features apart from a family history (which
Answer & Comments
suggests primary Raynaud's) are associated
Correct answer: B with CTD, but have not been shown to have the
same predictive value as B and G.
Cancer of the prostate is typically associated
with sclerotic bone lesions in contrast to the [ Q: 60 ] MasterClass Part2

Dr. Khalid Yusuf El-Zohry – Sohag Teaching Hospital (01118391123) Page | 362
El-Zohry MRCP Questions Bank (Part 2) – Medical Masterclass 2010

(2010) - Rheumatology 1.02 g/l (NR 0.75-1.65), C4 <0.02 g/l (0.20-0.65),


CRP <5mg/l (NR <5), creatinine 145 (NR 50-
A 22-year-old man has chronic diarrhoea and 140).
has had several episodes of sinusitis and
pneumonia. A diagnosis of common variable Which is the most likely diagnosis?
immunodeficiency (CVID) is made.
A. Systemic lupus erythematosus (SLE)
Which TWO of the following are recognized B. Rheumatoid arthritis
long-term complications of this disorder?
C. Mixed cryoglobulinaemia
A. Type II diabetes
D. Primary complement deficiency
B. Bronchiectasis
E. C1 inhibitor deficiency.
C. Hyperparathyroidism
D. Atheroma Answer & Comments

E. Epilepsy Correct answer: C


F. Enteropathy
This lady has a hypocomplementaemic
G. Cardiomyopathy
glomerulonephritis (note active urine sediment)
H. Chronic renal failure and a purpuric rash accompanied by a positive
I. Optic atrophy rheumatoid factor and an isolated positive ANA.

J. Sacroileitis. The combination of renal and skin involvement


in this setting with a markedly low C4 suggests
mixed cryoglobulinaemia. Her serology does
Answer & Comments
not support SLE.
Correct answer: BF The key investigations which would help
confirm the diagnosis would be a warm clotted
CVID patients suffer from recurrent pulmonary sample for cryoglobulins and a renal biopsy.
infections, and resultant repeated structural
lung damage can progress to bronchiectasis.
[ Q: 62 ] MasterClass Part2
Gastrointestinal symptoms are a common (2010) - Rheumatology
problem in CVID with a significant proportion
being attributed to a celiac like enteropathy. A 69-year-old man with rheumatoid arthritis
presents with increased breathlessness and
nose bleeds.
[ Q: 61 ] MasterClass Part2
(2010) - Rheumatology Which of the following treatments for
rheumatoid arthritis are unlikely to cause bone
A 40-year-old woman presents with a 6-month
marrow suppression?
history of a purpuric rash on her legs, non-
specific joint pains and vague ill health. Initial A. Gold
investigations reveal a positive rheumatoid B. Azathioprine
factor. The GP makes a tentative diagnosis of
rheumatoid arthritis and refers her to hospital. C. Non-steroidal anti-inflammatory drugs
Results of further investigations are as follows: (NSAIDs)
Urinalysis: red cell casts, protein+, ANA 1/80, D. Methotrexate
anti-DNA negative, anti-ENA negative, serum C3

Dr. Khalid Yusuf El-Zohry – Sohag Teaching Hospital (01118391123) Page | 363
El-Zohry MRCP Questions Bank (Part 2) – Medical Masterclass 2010

E. Sulphasalazine corneal ulceration, and other less common


entities such as choroiditis, retinal vasculitis,
F. Infliximab
episcleral nodules, retinal detachments, and
G. Leflunomide macular edema.
H. Cyclophosphamide Keratoconjunctivitis sicca, or dry eye syndrome,
I. Penicillamine is the most common ocular manifestation of RA
and has a reported prevalence of 15 to 25 %.
J. Hydroxychloroquine
The patient reports a gritty sensation in the
eyes .
Answer & Comments
Scleritis and episcleritis as causes of recurrent
Correct answer: CF red eyes are distinguished on the basis of
anatomy and appearance. Symptoms may be
Many of the treatments for rheumatoid similar, but the pain in scleritis is more evident
arthritis cause bone marrow suppression. and severe. Tenderness to palpation of the
NSAIDs may be used early in the disease and globe can help differentiate the two. After
cause anaemia via gastrointestinal ulceration. asking the patient to look down with eyelids
Infliximab has been associated with multiple closed, the physician gently presses the globe.
sclerosis-like demyelination, the development Patients with scleritis have tenderness on
of tuberculosis and worsening heart failure. palpation, while those with episcleritis do not..
Unlike scleritis, patients with episcleritis do not
complain of blurred vision or photophobia. The
[ Q: 63 ] MasterClass Part2
importance of correctly diagnosing and
(2010) - Rheumatology distinguishing between scleritis and episcleritis
A 70-year-old female has a 17-year history of is based on the potential ocular and systemic
rheumatoid arthritis. She presents with complications associated with scleritis. Studies
recurrent attacks of red congested eyes with a have shown that patients with RA-associated
sensation of grittiness. scleritis have more widespread systemic
disease and a higher mortality rate than those
The most likely cause of her red eyes is likely to episcleritis.
be:
A. scleritis [ Q: 64 ] MasterClass Part2
B. episcleritis
(2010) - Rheumatology

C. keratitis A 20-year-old man with common variable


antibody deficiency presents to the Emergency
D. keratoconjunctivitis sicca
Department with a 3-day history of cough
E. choroiditis. productive of green sputum. His temperature is
37.5ºC, p 84/min, respiratory rate 12/min,
Answer & Comments and breath sounds are vesicular. Chest
radiograph is unremarkable.
Correct answer: D
Which TWO of the following actions do you
Approximately 25 % of patients with recommend immediately?
rheumatoid arthritis (RA) will have ocular A. Prescribe a 14 day course of antibiotics
manifestations. keratoconjunctivitis sicca,
B. Prescribe nebulised salbutamol
scleritis, episcleritis, keratitis, peripheral

Dr. Khalid Yusuf El-Zohry – Sohag Teaching Hospital (01118391123) Page | 364
El-Zohry MRCP Questions Bank (Part 2) – Medical Masterclass 2010

C. Give an infusion of intravenous micromol/L, serum C3 124 mg/dl (nr 65-185),


immunoglobulin serum C4 32 mg/dl (nr 15-50), antinuclear
antibody negative, chest radiograph: bilateral
D. Take a sputum sample to look for acid
nodular shadowing.
fast bacilli
E. Prescribe a 5-day course of antibiotics Which of the following diagnoses is most likely
to account for his clinical presentation?
F. Check his serum immunoglobulin levels
A. Systemic lupus erythematosus (SLE)
G. Choose an antibiotic regimen suitable for
possible pseudomonas infection B. Small cell lung carcinoma
H. Order a high resolution CT scan C. Mixed cryoglobulinaemia
I. Take a sputum sample for culture D. Small vessel vasculitis of the Wegener's
microscopic polyangiitis spectrum
J. Prescribe a 28 day course of antibiotics.
E. Giant cell arteritis.
Answer & Comments
Answer & Comments
Correct answer: AI
Correct answer: D
Antibody deficient patients need prompt
treatment of presumed bacterial infection. This patient has an inflammatory multi-system
Treatment should be continued for slightly disorder of relatively short duration affecting
longer than normal; 14 days for an his lungs, kidneys, eyes and testes. The
uncomplicated chest infection would be constellation of nodular lung shadows,
appropriate. Most infections are caused by glomerulonephritis (as evidenced by red cell
common organisms such as haemophilus or casts in the urine), episcleritis, testicular
pneumococcus. Psuedomonas is unusual and tenderness and an elevated CRP is highly
mycobacterial disease rare in common variable suggestive of an ANCA+ necrotising small vessel
immunodeficiency. Cultures are invaluable if vasculitis. Systemic immune-complex diseases
there is a poor response to treatment and for such as SLE and mixed cryoglobulinaemia are
guiding future antibiotic choices. For infections excluded by the negative ANA and normal
causing fever, routine antibody replacement complement profile, respectively.
should be deferred for 24-48 hours until there This patient needs assessment of his ANCA
is a clear response to treatment, as adverse status and tissue biopsy-either kidneys or lungs,
reactions are much more common in the preferably kidneys
presence of fever.

[ Q: 66 ] MasterClass Part2
[ Q: 65 ] MasterClass Part2
(2010) - Rheumatology
(2010) - Rheumatology
A 65-year-old woman with a mitral valve
A 25-year-old man presents with a 3-week replacement presents to the Emergency
history of haemoptysis and testicular pain. The Department with pyrexia and fainting. She is
only significant findings on examination are unwell, hypotensive, anaemic and pyrexial. She
testicular tenderness and left-sided episcleritis. has a vague history of suffering from a reaction
The results of initial investigations are as to penicillin in her childhood. After taking blood
follows: serum CRP 124 g/l (nr <8), urine cultures she is started on broad-spectrum
sediment: red cell casts, serum creatinine 78 antibiotics. Cardiac valvular vegetations are
Dr. Khalid Yusuf El-Zohry – Sohag Teaching Hospital (01118391123) Page | 365
El-Zohry MRCP Questions Bank (Part 2) – Medical Masterclass 2010

seen on echocardiography and her blood grows B. Rheumatoid arthritis


methicillin-sensitive Staphylococci – the
C. Pseudogout
microbiologist suggests naficillin as the most
appropriate antibiotic, but is concerned that D. Gout
she may have allergy to beta lactam-based E. Septic arthritis
antibiotics.
F. Osteosarcoma
Which of the following is most appropriate to G. Osteoarthritis
investigate her history of possible penicillin
allergy? H. Reactive arthritis

A. serum tryptase I. Torn medial meniscus

B. skin prick test to penicillin J. Spontaneous haemarthrosis.

C. serum penicillin specific IgE


Answer & Comments
D. patch test to penicillin
Correct answer: DH
E. serum IgE
Reactive arthritis, consequent upon either
Answer & Comments genital chlamydia trachomatis or enteric
Correct answer: B infection, is the most likely diagnosis. He is a
little younger than many patients presenting
with gout, but this is not a rare diagnosis in this
A history of penicillin allergy is relatively
age group, particularly if alcohol intake is high.
common in hospital patients, and is in most not
The important diagnosis to exclude is septic
due to type I hypersensitivity reactions. A
arthritis, but this is a rare cause of
diagnosis of penicillin allergy crucially requires a
monoarthritis in this age group. Aspiration of
detailed history of the drug reaction, and can
synovial fluid is the key diagnostic tool to
be confirmed by a positive skin prick test to the
differentiate between these diagnoses.
major and minor determinants of penicillin.
Skin prick testing is carried out if there is a
clinical need for penicillin treatment e.g. [ Q: 68 ] MasterClass Part2
treatment of infective endocarditis. A patient is (2010) - Rheumatology
unlikely to develop anaphylaxis with a negative
A 65-year-old woman with primary Sjögren's
penicillin skin prick test. The detection of
syndrome, which was diagnosed 1 year
penicillin specific IgE in the serum is unreliable.
previously, is noted to have a persistently
elevated erythrocyte sedimentation rate (ESR)
[ Q: 67 ] MasterClass Part2 of 90-100 mm/hour and a normal C-reactive
(2010) - Rheumatology protein (CRP). At a routine outpatient visit it is
noted that her sicca symptoms are still
A 36-year-old man presents with a 2-day history
troublesome, but there are no other
of severe pain in the left knee. He has recently
abnormalities on clinical examination. The
returned from a holiday in Spain. Examination
following investigations are performed:
shows low grade fever (37.5ºC) and marked
haemoglobin 12.5 g/dL (normal range 12-16),
synovitis of the left knee with a tense effusion.
white cell count 6.4 x 109/L (normal range 4-
Which are the TWO most likely diagnoses? 11), platelet count 320 x 109/L (normal range
150-400), ESR 98 mm/hour, CRP <6 mg/dL
A. Lyme disease

Dr. Khalid Yusuf El-Zohry – Sohag Teaching Hospital (01118391123) Page | 366
El-Zohry MRCP Questions Bank (Part 2) – Medical Masterclass 2010

(normal <6), serum IgG 42 g/L (normal range 6- B. Anti-double stranded DNA (anti ds-DNA)
13), IgA 8.4 g/L (normal range 0.8-4.0) and IgM
C. Anti-Sm antibodies
3.6 g/L (normal range 0.4-2.0), and serum
electrophoresis shows polyclonal D. Anti-histone antibodies
hypergammaglobulinaemia. E. Anti-Ro/SSA antibodies.

Her persistently elevated ESR is best explained


by which of the following? Answer & Comments

A. Development of lymphoma Correct answer: A

B. Poorly controlled sicca symptoms


Almost all patients with SLE have a positive ANA
C. Positive antinuclear antibody test result. The ANA test is sensitive but not
D. Positive antibodies to Ro and La antigens specific for SLE. A negative result argues
strongly against a diagnosis of active SLE, but
E. Polyclonal hypergammaglobulinaemia
does not exclude the possibility of other
autoimmune diseases.
Answer & Comments
Antibodies to Sm antigen are highly specific for
Correct answer: E a diagnosis of SLE (>99%). However, only about
25% of patients with SLE have anti-Sm
Together with fibrinogen, serum antibodies. Anti-DNA antibodies are diagnostic
immunoglobulins are the major driving force of SLE (specificity >99%). However, only 60% of
causing an elevation in erythrocyte patients with SLE will have these antibodies.
sedimentation rate (ESR). Any disorder Therefore, absence of anti-DNA or anti-Sm
associated with a persistently elevated antibodies should not exclude SLE as a
polyclonal hypergammaglobulinaemia, such as diagnosis. Anti-Ro/SS-A antibodies are found in
primary Sjögren's syndrome, is likely to be 30% of patients with SLE. Anti-histone
linked with a persistently elevated ESR. Note antibodies are identified in small proportion of
that her C-reactive protein (CRP) is normal, SLE patients. They are more often seen with
which reflects the fact that CRP production is drug-induced lupus.
not influenced by changes in serum
immunoglobulin levels. [ Q: 70 ] MasterClass Part2
(2010) - Rheumatology
[ Q: 69 ] MasterClass Part2
A 60-year-old-man complained of pain in both
(2010) - Rheumatology wrist evolving over 8 weeks. He noted swelling
A 28-year-old woman presented with fatigue around that area but denied stiffness. On
and extreme tiredness. Physical examination examination there was swelling and tenderness
revealed facial skin rash and tenderness across just proximal to the wrist joints without
the small joints of the hands. She was limitation of movement. There was also
concerned that she might have systemic lupus prominent finger clubbing. Radiographs
erythematosus (SLE). revealed periosteal reaction over the lower end
of the radius and ulnar.
Which of the following tests when NEGATIVE
will virtually exclude the diagnosis of SLE? Each of the following disorders could be the
cause behind this patient complain EXCEPT?
A. Antinuclear antibody (ANA)
A. Mesothelioma

Dr. Khalid Yusuf El-Zohry – Sohag Teaching Hospital (01118391123) Page | 367
El-Zohry MRCP Questions Bank (Part 2) – Medical Masterclass 2010

B. Bronchiectesis [ Q: 71 ] MasterClass Part2


C. Diabetes mellitus (2010) - Rheumatology
D. Crohn's disease A 56-year-old patient presented with 6-month
E. Whipple's disease. history of excessive tiredness, stiffness and pain
across the upper and lower limbs. On
examination he has proximal muscle weakness.
Answer & Comments
The creatinine kinase was 10 times the upper
Correct answer: C limits of normal.

Hypertrophic osteoarthropathy (HOA) is a


syndrome characterized by proliferative
changes in the skin and skeleton. Proliferative
periostitis of the radius and fibula and digital
clubbing are commonly seen. Two forms of the
syndrome are seen, a rare idiopathic from
called pachydermoperiostitis (3-5%) (familial
autosomal dominant), and more common
secondary form (95-97%).
Secondary HOA was initially described in
association with chronic suppurative infection The skin lesions shown in the image are most
and malignancy of the lung and pleura. likely to be:
Therefore it used to be called hypertrophic
pulmonary osteoarthropathy (HPOA). A. garrods pads

Pleural causes include pleural fibroma and B. tendon xanthoma


mesothelioma. C. Gottron's papules
Pulmonary causes include bronchogenic D. rheumatoid nodules
carcinoma, pulmonary tuberculosis; pulmonary
E. gouty tophi.
abscesses, bronchiectasis emphysema; and
Pneumocystis carinii infection in patients with
AIDS, Hodgkin’s disease, metastases, or cystic Answer & Comments
fibrosis. Correct answer: C
Cyanotic heart disease with a right-to-left shunt
is the only cardiac cause described. Gottron's papules, violaceous papules overlying
Abdominal causes include liver cirrhosis, the dorsal interphalangeal or
ulcerative colitis, Crohn disease, Whipple metacarpophalangeal areas, elbow or knee
disease, and biliary atresia. joints, occur in approximately 70% of patients
with dermatomyositis.
In long standing insulin dependent diabetes
mellitus, contracture of the flexor tendons of Garrods fatty pads (knuckle pad syndrome) is a
the fingers with tight, waxy skin appearance relatively common and harmless condition
leads to diabetic cheirarthropathy. There is no characterized by discrete fibromatous skin pads
pain and minor functional impairment. over the dorsum of the interphalangeal joints.

Diabetes mellitus is not associated with HOA.

Dr. Khalid Yusuf El-Zohry – Sohag Teaching Hospital (01118391123) Page | 368
El-Zohry MRCP Questions Bank (Part 2) – Medical Masterclass 2010

[ Q: 72 ] MasterClass Part2 E. Papillomavirus


(2010) - Rheumatology F. Chlamydia trachomatis
A 36-year-old woman presents with G. Mycobacterium tuberculosis
deteriorating nocturnal paraesthesia affecting H. Mycoplasma hominis
both hands. It improves during the morning.
I. Pneumococcus
Which of the following tests is least likely to be J. Candida albicans
helpful in establishing the cause?
A. Thyroid function test Answer & Comments
B. Rheumatoid factor Correct answer: CI
C. Tinels test
D. MRI scan hands [ Q: 74 ] MasterClass Part2
E. Urinary hCG. (2010) - Rheumatology
An 8-year-old has been admitted for a third
Answer & Comments time with haemolytic-uraemic syndrome and is
found to have a low C3.
Correct answer: D
Which of the following is the most likely
Carpal tunnel syndrome classically presents diagnosis?
with nocturnal paraesthesia in the palms of the
A. Lupus
hands which improves by shaking / using the
hands. It is caused by compression of the B. Gram-negative septicaemia
median nerve within the carpal tunnel in the C. Factor H deficiency
wrist. MRI hands would not be indicated.
D. Factor I deficiency
Causes of Carpal tunnel syndrome include
E. C3 deficiency.
hypothyroidism, synovitis including rheumatoid
arthritis and pregnancy. Tinel’s test is a
provocation test to reproduce symptoms by Answer & Comments
percussion over the median nerve.
Correct answer: C

[ Q: 73 ] MasterClass Part2 Factor H deficiency accounts for 5-10% of all


(2010) - Rheumatology cases of haemolytic-uraemic syndrome.
Typically it has a higher mortality and over half
A 47-year-old woman had a splenectomy 10
of patients will suffer relapses. Over-activation
years previously.
of the alternative complement pathway results
She is at increased risk of severe infection with in a low serum C3.
which of the following organisms: Although able to cause a low C3, lupus, factor I
A. Aspergillus nigrans and C3 deficiency are not associated with
recurrent haemolytic-uraemic syndrome.
B. Epstein-Barr virus
C. Falciparum malaria
D. Mycobacterium avium intracellular

Dr. Khalid Yusuf El-Zohry – Sohag Teaching Hospital (01118391123) Page | 369
El-Zohry MRCP Questions Bank (Part 2) – Medical Masterclass 2010

[ Q: 75 ] MasterClass Part2 C. She has been re-infected with a different


(2010) - Rheumatology strain of TB.
D. She has underlying IFNg receptor
A 50-year-old man gives an 8-month history of
deficiency causing increased susceptibility
episodic, painful soft tissue swellings involving
to mycobacteria.
his hands, eyes and lips. There is no temporal
relationship to food. E. She has been poorly adherent to therapy
and her TB has recurred.
Which of the following tests is the most useful?
A. Skin prick tests to various food allergens Answer & Comments
B. Complement Correct answer: E
C. Full blood count (FBC)
Poor adherence is a common cause of
D. Glucose tolerance test
treatment failure or early relapse; directly
E. Urinary 5-hydroxyindoleacetic acid. observed therapy (DOT) may improve
adherence if this is the case. TB usually
Answer & Comments responds well to conventional treatment even if
the patient is co-infected with HIV. In this
Correct answer: B situation, recurrent TB is most often caused by
poor adherence (early) or reinfection (late).
The most likely diagnosis in this patient is
Underlying IFNg receptor/ IL12 deficiency is
chronic idiopathic angioedema, However, the
extremely rare and is associated with
rarer C1-inh deficiency has to be considered
disseminated disease, usually with poorly
and would be excluded by a normal C4 level.
pathogenic environmental mycobacteria..
The lack of any temporal relationship of his Environmental mycobacteria may occasionally
symptoms to the ingestion of any food excludes grow as contaminants in culture, but it is most
an allergic cause, which only rarely presents as unusual for MTB to do so.
chronic angioedema.
[ Q: 77 ] MasterClass Part2
[ Q: 76 ] MasterClass Part2 (2010) - Rheumatology
(2010) - Rheumatology
The A&E SHO rings for your advice concerning a
A 19-year-old woman presents with fever and 57-year-old man who attended following a dog
cough. Sputum samples are negative on bite which required suturing. He has had a
microscopy for acid fast bacilli, but six weeks tetanus booster and is taking penicillin 250mg
later M. tuberculosis is grown. She completed a bd. Twenty four years previously he had a
course of chemotherapy for pulmonary TB two splenectomy following a road traffic accident.
months previously. Her chest radiograph is He has remained well since.
unchanged from one taken at this time.
Which of the following statements are true?
Which of the following is the most likely
A. Large granular lymphocytes may be
explanation for these findings?
visible on blood film.
A. She has HIV co-infection causing
B. He should avoid killed vaccines.
increased susceptibility to mycobacteria.
B. The organism isolated is a contaminant.

Dr. Khalid Yusuf El-Zohry – Sohag Teaching Hospital (01118391123) Page | 370
El-Zohry MRCP Questions Bank (Part 2) – Medical Masterclass 2010

C. He is at risk of pneumocysitis pneumonia Although 5% of the general population have


and appropriate long-term prophylaxis Raynaud's phenomenon, only a minority go on
should be given. to develop systemic connective tissue disease.
A positive ANA is the single best predictor of
D. Penicillin may be stopped after 10 days.
existing or future progression to connective
E. He should not have an annual ‘flu tissue disease in this situation.
vaccination.
F. He is at increased risk of overwhelming [ Q: 79 ] MasterClass Part2
capnocytophagia canimorsus infection (2010) - Rheumatology
following the dog bite. Appropriate
antibiotic cover should be given. A 38-year-old woman consults her dentist
because of dental caries. He notes that she has
G. He is at increased risk of babesiosis
a dry mouth and refers her to you with a
following the dog bite. Appropriate
possible diagnosis of Sjörgren’s syndrome.
antibiotic cover should be given.
H. Howell Jolly bodies may be visible on Which of the following does not support this
blood film. diagnosis?

I. He is not at increased risk of A. Dry eyes


overwhelming pneumococcal infection. B. Parotid swelling
J. He is at increased risk of coeliac disease. C. Fatigue
D. Arthralgia
Answer & Comments
E. Pleural effusion
Correct answer: FH

Answer & Comments


[ Q: 78 ] MasterClass Part2
Correct answer: E
(2010) - Rheumatology
A 30-year-old female nurse presents with a 3- Sjörgren’s syndrome results from an
month history of Raynaud's phenomenon. inflammatory lymphocyte infiltrate of salivary
Clinical examination reveals cold hands but no glands. The most common symptoms are dry
other evidence of connective tissue disease. eyes and mouth, with fatigue. Many people do
not volunteer these symptoms unless asked, for
Which of the following tests is most helpful in example by the dentist, aware that lack of saliva
determining future progression to systemic
predisposes to caries.
connective tissue disease?
Arthralgia is common in primary Sjörgren’s.
A. Positive anti-mitochondrial antibody
Secondary Sjörgren’s is associated with a
B. Positive anti-gastric parietal cell antibody variety of connective tissue diseases, which
may themselves be a cause of arthralgia.
C. Positive smooth muscle antibody
Tricyclic antidepressants are an unrelated cause
D. Positive anti-nuclear antibody
of dry mouth.
E. Positive rheumatoid factor

Answer & Comments

Correct answer: D
Dr. Khalid Yusuf El-Zohry – Sohag Teaching Hospital (01118391123) Page | 371
El-Zohry MRCP Questions Bank (Part 2) – Medical Masterclass 2010

[ Q: 80 ] MasterClass Part2 The clinical picture is highly suggestive of


(2010) - Rheumatology polymyalgia rheumatica. This will usually
respond dramatically to small doses of
A 79-year-old man presents feeling very unwell prednisolone, which will be required for an
with recent and sudden onset of severe pain average duration of 12-18 months before the
and stiffness around his shoulders and hips. He disease goes into remission. The major side
was previously well and admits to no other effect of these low doses of steroid is
symptoms, in particular no headache or visual osteoporosis, and prophylaxis will usually be
disturbance. Examination shows painful, required. This decision may be informed by
restricted shoulders and hips, but no myopathy. bone densitometry, if available.
He has no lymph nodes or abdominal masses.
Initial investigations show a normal blood
[ Q: 81 ] MasterClass Part2
count, biochemical profile, thyroid function,
prostate specific antigen, MSU and chest
(2010) - Rheumatology
radiograph. His ESR is 85mm and CRP is A 31-year-old man with known selective IgA
110mg/l (normal<10). deficiency and abdominal pain is tested for anti-
endomysial antibodies (EMA). His results are as
Which TWO of the following statements give
follows: IgA endomysial antibody negative, IgG
the best advice regarding his treatment?
endomysial antibody positive.
A. He should be started on Prednisolone
60mg daily reduced over 12-18 months. Which of the following statements is true?

B. He should be treated with a Cox II A. Coeliac disease can be confidently


inhibitor. excluded on the basis of these results.

C. He should be started on Prednisolone B. His IgA deficiency is irrelevant to his


15mg daily for 2 weeks only negative IgA EMA status.

D. He should be treated with alfacalcidol C. The negative IgA EMA is of no diagnostic


2ug daily. value in the presence of IgA deficiency.

E. He should be treated with low dose D. Referral for small bowel biopsy is not
methotrexate. warranted on the strength of these
results.
F. He should be started on Prednislone
15mg daily, tailed off over 12-18 months. E. For diagnostic purposes, IgA and IgG EMA
are equally important
G. He should receive 3x1g doses of i.v.
methylprednisolone.
Answer & Comments
H. He should receive prophylactic treatment
for osteoporosis. Correct answer: C

I. He should receive simple analgesia.


Since there is a recognized association between
J. He should receive Prednisolone 60mg coeliac disease and IgA deficiency, it is
daily for 1 month. important to recognize that IgA EMA may be
spuriously negative in patients with coeliac
Answer & Comments disease and total IgA deficiency, as in this case.
For this reason, testing for coeliac disease
Correct answer: FH
antibodies always includes measurement of
both IgA and IgG antibody isotypes or

Dr. Khalid Yusuf El-Zohry – Sohag Teaching Hospital (01118391123) Page | 372
El-Zohry MRCP Questions Bank (Part 2) – Medical Masterclass 2010

measurement of serum IgA. Such a testing on synovial fluid analysis (negatively


strategy will alert both the laboratory and the birefringent ones are found in gout).
requesting clinician of 'false-negative' results as
The knee is the most common site affected by
in the case here. In keeping with the mucosal
this disorder. Other locations include:
route of antigen entry, IgA endomysial
antibodies are more important than IgG  the triangular fibrocartilage of the wrist
antibodies as a diagnostic marker of coeliac  sacroiliac joints
disease.
 symphysis pubis
 the glenoid of the shoulder
[ Q: 82 ] MasterClass Part2
(2010) - Rheumatology  the labrum of the hip
 the elbow
A 75-year-old woman presents with acute pain
and swelling in the right knee. On examination  the ankle
the right knee is hot and tender with moderate
 acromioclavicular joint.
synovial fluid collection. Radiograph of the right
knee shows calcification of the meniscus. CPPD is associated with hemochromatosis,
Synovial fluid was sterile on culture and primary hyperparathyroidism, hypothyroidism,
rhomboid shaped crystals were identified on and hypomagnesemia. Other findings include
microscopy of the fluid. joint space narrowing, large subchondral cysts
and many intra-articular bodies from
Which of the following statements about this subchondral bone fragmentation.
condition is NOT true?
A. Narrowing of the joint space is often [ Q: 83 ] MasterClass Part2
present (2010) - Rheumatology
B. The symphysis pubis is one of many other
A 20-year-old woman presented with sudden
joints that can be affected by this
onset of swelling of the lips and tongue. She
disorder
also had abdominal pain and vomiting. Her
C. Serum calcium levels may be elevated in mother confirmed that her daughter had similar
some patients attacks over the years and even as a child. A
D. Negatively birefringent crystals may be brother and older sister have the same
identified on synovial fluid analysis disorder.

E. There may be large subchondral cysts Which of the following statements about this
disease is accurate?
Answer & Comments A. It has sex-linked inheritance.
Correct answer: D B. Animal allergen is often identified in the
house
This patient presents with acute monoarthritis C. Serum C4 levels are often low.
of the right knee. The clinical picture and the
radiological findings are suggestive of D. Antinuclear antibodies (ANA) is often
pseudogout and calcium pyrophosphate positive.
deposition (CPPD), which is characterised by E. Raised IgE helps differentiate it from
the presence of positively birefringent crystals other immune disorders.

Dr. Khalid Yusuf El-Zohry – Sohag Teaching Hospital (01118391123) Page | 373
El-Zohry MRCP Questions Bank (Part 2) – Medical Masterclass 2010

Answer & Comments consequence of uncontrolled complement


classical pathway activation. At this age, the
Correct answer: C
most likely diagnosis is acquired C1 inhibitor
deficiency associated with lymphoproliferative
Hereditary angioneurotic oedema is an disease, or more rarely due to autoantibodies
autosomal dominantly inherited condition to C1 inhibitor.
caused by a deficiency of C1 esterase inhibitor.
This results in intermittent episodes of
spontaneous complement activation. Clinically
[ Q: 85 ] MasterClass Part2
the patient suffers oedema of the skin and (2010) - Rheumatology
mucosal surfaces. Fatalities may occur if the A 15-year-old boy presented with arthralgia,
airway is compromised. C4 levels are typically skin rash and haematuria. Renal biopsy showed
low during an attack; they may be normal in focal necrotising glomerulonephritis with
between attacks. diffuse mesangial IgA deposits.
Acquired angioedema (AAE) is angioedema
associated with allergic reactions, which is What is the most likely diagnosis?
often associated with urticaria. Approximately A. Systemic lupus erythematosus (SLE)
94% of cases of angioedema are drug-induced.
B. Henoch–Scholein purpura
Most are patients taking angiotensin-converting
enzyme (ACE) inhibitors. Insect stings, and C. Juvenile rheumatoid arthritis
foods are other predisposing factors. D. Post-streptococcal glomerulonephritis
E. Goodpasture's syndrome.
[ Q: 84 ] MasterClass Part2
(2010) - Rheumatology
Answer & Comments
A 65-year-old woman presents with a 3-month
Correct answer: B
history of recurrent attacks of facial
angioedema.
Henoch–Schönlein purpura (HSP) is
Which of the following test results would favour characterised by the tissue deposition of IgA-
a diagnosis of C1 inhibitor deficiency? containing immune complexes. The
pathogenesis of this disorder may be similar to
A. Low complement C3 levels and a normal
that of IgA nephropathy, which is associated
C4
with identical histologic findings in the kidney.
B. Low complement C4 levels and a normal
HSP occurs more often in children than in
C3
adults, and many cases follow an upper
C. Hypergammaglobulinaemia respiratory tract infection, suggesting that the
D. Hypogammaglobulinaemia precipitating antigen may be infectious The
clinical manifestations include a classic tetrad
E. Positive rheumatoid factor that can occur in any order and at any time over
a period of several days to several weeks: rash,
Answer & Comments arthralgias, abdominal pain, and renal disease
typically purpuric (with normal clotting studies)
Correct answer: B
and distributed symmetrically over the lower
legs and arms.
The hallmark of all forms of C1 inhibitor
deficiency is a reduced C4 with normal C3 - a

Dr. Khalid Yusuf El-Zohry – Sohag Teaching Hospital (01118391123) Page | 374
El-Zohry MRCP Questions Bank (Part 2) – Medical Masterclass 2010

[ Q: 86 ] MasterClass Part2 after induction of anaesthesia for


(2010) - Rheumatology cholecystectomy.

A 60-year-old accountant complains of Which of the following blood test results would
recurrent attacks of exquisite pain and swelling suggest that her reaction was associated with
in the left big toe. mast cell degranulation?
A. Elevated plasma tryptase
Which of the following conditions is NOT likely
to be associated with this disorder? B. Hypernatraemia

A. Chronic alcoholism C. Hypokalaemia

B. Obesity D. Hypocomplementaemia

C. Rheumatoid arthritis E. Hypergammaglobulinaemia.

D. Diabetes mellitus
Answer & Comments
E. Diuretic therapy.
Correct answer: A
Answer & Comments
The constellation of acute symptoms in this
Correct answer: C case is highly suggestive of a systemic allergic
reaction, either anaphylaxis (if IgE mediated) or
Acute gout is intensely inflammatory, and is an anaphylactoid reaction (if non-IgE
therefore characterised by severe pain, mediated). Both of these reactions are due to
redness, swelling and disability. At least 80% of extensive mast cell degranulation leading to
initial attacks involve a single joint, typically in release of large amounts of tryptase in to the
the lower extremity, most often at the base of circulation. Elevated tryptase levels in the
the great toe (first metatarsophalangeal joint, context of this lady's reaction are highly
known as podagra), or in the knee. Trauma, suggestive of an anaphylactic/anaphylactoid
surgery, starvation, alcohol ingestion, dietary reaction.
overindulgence, and ingestion of drugs -
This lady will need to be investigated during
diuretics (cyclosporin and low dose aspirin)
convalescence in an allergy clinic to determine
affecting serum urate concentrations may all
the cause of her reaction.
promote gouty attacks. Similarly chronic
disorders such as diabetes mellitus, obesity,
hyperparathyroidism and hypothyroidism are [ Q: 88 ] MasterClass Part2
associated with increased incidence of acute (2010) - Rheumatology
gouty attacks. Rheumatoid arthritis is not
A 68-year-old woman with longstanding
associated with increased incidence of
congestive cardiac failure (ejection fraction
hyperuricaemia or gout.
20%) presents with a hot, swollen right knee.
The following results are obtained: FBC normal,
[ Q: 87 ] MasterClass Part2 Urea 11 mmol/l, Creatinine 196 umol/l.
(2010) - Rheumatology Synovial fluid: many monosodium urate crystals
seen on microscopy, culture sterile.
A 30-year-old woman develops a systemic
reaction characterized by hypotension, What is the best treatment for her acute
bronchospasm and widespread urticaria soon arthritis?

Dr. Khalid Yusuf El-Zohry – Sohag Teaching Hospital (01118391123) Page | 375
El-Zohry MRCP Questions Bank (Part 2) – Medical Masterclass 2010

A. Allopurinol E. Hydroxychloroquine may improve her


arthralgia.
B. Colchicine 0.5mg every 2-4 hours
C. Indomethacin 50mg tds
Answer & Comments
D. Co-codamol 30/500 every 6 hours
Correct answer: E
E. Intra-articular corticosteroid
This history is strongly suggestive of systemic
Answer & Comments lupus erythematosus (SLE) with secondary
antiphospholipid syndrome. Foetal loss is
Correct answer: E
common due to placental infarction and
insufficiency. She is at risk of further thrombosis
This patient has acute gout, probably diuretic- and should have lifelong anticoagulation. If she
induced. Intra-articular corticosteroids are safe is planning further pregnancies she will need
and highly efficacious in this situation, once
specialist referral for intensive monitoring.
sepsis is excluded.
Active lupus is associated with high ESR and
Allopurinol has no role in the acute treatment
DNA antibodies, with reduced complement
of gout. Colchicine at these doses is very poorly (especially C4) levels. High CRP suggests the
tolerated due to GI toxicity. Non-steroidals are possibility of infection.
very likely to precipitate a severe deterioration
in renal function and may also exacerbate heart Hydroxychloroquine is effective for joint and
failure. cutaneous manifestations of SLE.

[ Q: 89 ] MasterClass Part2 [ Q: 90 ] MasterClass Part2


(2010) - Rheumatology (2010) - Rheumatology

A 29-year-old woman has a 3-year history of A 38-year-old man presents with 4 months of
arthralgia and Raynaud's phenomenon. A year pain, stiffness and swelling of the small joints of
ago she had a miscarriage at 29 weeks, his hands and feet. He has a past history of mild
complicated by a deep vein thrombosis. psoriasis. His GP has found his erythrocyte
Investigations show: Hb 11.2 Wcc 4.3 Platelets sedimentation rate to be elevated at 65
145, ANA positive 1/160, DNA negative, ENA Ro mm/hour.
positive, Anticardiolipin antibodies present at
Which of the following clinical features would
moderate titre, lupus anticoagulant present.
be least likely to suggest a diagnosis of psoriatic
Which statement is true? arthritis?

A. She has primary antiphospholipid A. Involvement of the distal interphalangeal


syndrome. joints

B. She should be anticoagulated for a B. Subcutaneous nodules


further 6 months. C. Dactylitis
C. She has active lupus. D. Nail involvement
D. A high C reaction protein (CRP) and E. Asymmetrical arthritis
erythrocyte sedimentation rate (ESR)
would be consistent with active lupus.

Dr. Khalid Yusuf El-Zohry – Sohag Teaching Hospital (01118391123) Page | 376
El-Zohry MRCP Questions Bank (Part 2) – Medical Masterclass 2010

Answer & Comments [ Q: 92 ] MasterClass Part2


Correct answer: B (2010) - Rheumatology
A 30-year-old teacher presents with a 6-month
Subcutaneous nodule formation is very strongly history of swelling and pain involving the distal
suggestive of rheumatoid factor-positive interphalangeal joints of her hands. The ESR is
rheumatoid arthritis. All the other features 65 mm in the first hour.
occur in psoriatic arthritis. Nail involvement is a
feature of skin psoriasis, but is also strongly What is the most likely diagnosis?
associated with distal interphalangeal joint A. Generalised osteoarthritis
involvement in psoriatic arthritis.
B. Rheumatoid arthritis

[ Q: 91 ] MasterClass Part2 C. Psoriatic arthritis


(2010) - Rheumatology D. Systemic lupus erythematosus (SLE)

A 45-year-old woman developed Raynaud's 2 E. Gout.


years ago. She now complains of breathlessness
and skin tightness affecting her fingers. A high- Answer & Comments
resolution CT scan shows evidence of
Correct answer: C
pulmonary fibrosis.

Which of the following tests is most likely to be Arthritis with predominant involvement of the
positive? distal interphalangeal joint occurs most often in
A. Anticentromere antibody generalised osteoarthritis and psoriatic
arthritis. The fact that this patient is relatively
B. Anti-double-stranded DNA antibody young and has a raised ESR indicates an
C. Anti-Ro antibody underlying inflammatory disease is the most
likely cause of her symptoms. Therefore
D. Anti-Scl-70 antibody
psoriatic arthritis is the most likely diagnosis in
E. Anti-Jo-1 antibody this case.
Rheumatoid arthritis and SLE are known to
Answer & Comments affect the proximal interphalangeal (PIPs) and
Correct answer: D the metacarpophalangeal (MCPs) joints.
Chronic gouty arthritis might involve the DIPs,
This patient has clinical symptoms suggestive of but more often it involves the MCPs and PIPs in
diffuse cutaneous systemic sclerosis. Pulmonary asymmetrical fashion with or without tophus
fibrosis and anti-Scl-70 are more common in formation.
patients with diffuse disease. Anticentromere Examination of the skin and nail for psoriasis is
antibody is associated with limited cutaneous very important in confirming the diagnosis.
systemic sclerosis. Anti-double-stranded DNA
antibody is associated with systemic lupus Hairlines in the scalp, the naval and the palms
erythematosus. Anti-Ro antibody is associated are areas often involved in psoriasis but easily
with lupus and primary Sjögren's syndrome. missed.
Anti-Jo-1 is associated with polymyositis,
particularly in patients with inflammatory lung
disease.

Dr. Khalid Yusuf El-Zohry – Sohag Teaching Hospital (01118391123) Page | 377
El-Zohry MRCP Questions Bank (Part 2) – Medical Masterclass 2010

[ Q: 93 ] MasterClass Part2 trauma and her only complaint is pain in her


(2010) - Rheumatology left knee. Examination of the knee shows that it
is painful to touch but there are no signs of
A 57-year-old woman is admitted to the inflammation. An abdominal radiograph
Emergency Department at 05.00h with orofacial incidentally shows a left hip fracture (see
angio-oedema, having eaten a prawn curry the image).
previous evening. She has a past history of
hypertension and hypothyroidism, and has
recently been treated for a chest infection. Her
regular medications include
bendroflumethiazide, lisinopril and
levothyroxine. She has a history of penicillin
allergy.
On admission she is comfortable at rest and has
no urticaria. Observations are as follows:
temperature 37°C, pulse 80 bpm, respiratory
rate 16/minute and BP 170/100 mmHg.

What is the most likely diagnosis?


A. Angiotensin-converting enzyme inhibitor-
induced angio-oedema
B. Anaphylaxis due to prawns Why is she having knee pain?
C. Penicillin allergy A. Pain in the knee is referred to the hip by
D. Idiopathic anaphylaxis the superior gluteal nerve.

E. Diuretic allergy B. Pain in the knee is referred to the hip by


the inferior gluteal nerve.
Answer & Comments C. Pain in the knee is referred to the hip by
the femoral, sciatic and obturator nerves.
Correct answer: A
D. Pain in the knee is referred to the hip by
the musculocutaneous nerve.
Angio-oedema may be related to use of an
angiotensin-converting enzyme inhibitor and E. Pain in the knee is an incidental finding.
may occur even after apparent tolerance of the
drug for many months. In this case the drug Answer & Comments
should be stopped or substituted.
Correct answer: C
Allergic angio-oedema is usually obvious by its
fast onset, typically within a few minutes of
The knee pain was referred from the hip to the
contact with the allergen.
knee. Branches of the femoral, sciatic and
obturator nerves all give twigs to both joints.
[ Q: 94 ] MasterClass Part2 The geniculate branch of the obturator is the
(2010) - Rheumatology main conveyer of pain referred from hip to
knee. Referred pain from the hip to the knee is
An 82-year-old woman is admitted with a
well known to occur in osteoarthritis of the hip
history of poor mobility. There is no history of

Dr. Khalid Yusuf El-Zohry – Sohag Teaching Hospital (01118391123) Page | 378
El-Zohry MRCP Questions Bank (Part 2) – Medical Masterclass 2010

but that it can occur in hip fractures maybe less serpiginous band of low signal surrounds an
well appreciated. area of normal signal.
Some older patients with severe osteoporosis
The most likely diagnosis is:
may sustain a hip fracture simply by turning
with no history of trauma. A. SLE-associated acute inflammatory
arthritis of the right hip

[ Q: 95 ] MasterClass Part2 B. Septic arthritis of the right hip


(2010) - Rheumatology C. Right psoas abscess

A 32-year-old woman with systemic lupus D. Avascular necrosis of the right hip
erythematosus (SLE) and antiphosphlipid E. Thrombosis of the right ilio-femoral vein.
syndrome presented with a three week history
of increasing pain in the right groin and anterior
Answer & Comments
thigh associated with excessive fatigue and
lethargy. She was limping and right hip Correct answer: D
movement was limited with reduced flexion
and internal rotation. Although afebrile with a All of the above options are possible causes for
temperature of 37°C she was given intravenous this patient's complaint. However, the magnetic
antibiotics and kept on her regular maintenance resonance image is almost diagnostic of
dose of prednisolone at 7.5 mg/day. Magnetic avascular necrosis (AVN). Corticosteroids such
resonance image (T1 weighted axial) of the hips as prednisolone are commonly used to treat
(see image) shows the entire femoral head is diseases in which there is inflammation, such as
reduced in signal on the right, indicating diffuse systemic lupus erythematosus. Both steroid
oedema. therapy and SLE could predispose to avascular
necrosis.
If septic arthritis is suspected, immediate joint
aspiration must be performed in order to
obtain cultures to determine the infectious
agent. Underlying bone changes are not
typically present. Inflammatory arthritis in
general is associated with synovial tissue
thickening but again lacks significant bone
changes. Ilio-femoral vein thrombosis and
psoas abscess are not known to be associated
with structural damage of the femoral head. In
differentiating AVN from non-AVN disease of
the femoral head, MR imaging has been
reported to have a specifity of 98% and
sensitivity of 97%.
Mnemonic to remember causes of AVN (taken
from Dahnert's Radiology Review Manual),
"PLASTIC RAGS":
It also shows subchondral collapse with
P - Pancreatitis, Pregnancy
flattening of the right femoral head. A
L - Legg-Perthes Disease, Lupus

Dr. Khalid Yusuf El-Zohry – Sohag Teaching Hospital (01118391123) Page | 379
El-Zohry MRCP Questions Bank (Part 2) – Medical Masterclass 2010

A - Alcoholism, Atherosclerosis Answer & Comments


S - Steroids Correct answer: EG
T - Trauma
This patient has developed carpal tunnel
I - Idiopathic (Spontaneous osteonecrosis of
syndrome due to pressure on the median
knee, Legg-Calve-Perthes Disease, Freiberg
nerve. Carpal tunnel syndrome (CTS) is more
Disease), Infection
common in patients with hypothyroidism,
C - Caisson Disease*, Collagen Vascular Disease diabetes mellitus (DM), rheumatoid arthritis
R - Rheumatoid arthritis, Radiation tx (RA), osteoarthritis and those taking the oral
contraceptive pill. Mechanical factors can also
A - Amyloid exacerbate CTS, particularly those that increase
G - Gaucher's Disease flexion of the wrist e.g. driving and typing.
S - Sickle Cell Disease
[ Q: 97 ] MasterClass Part2
*Caisson Disease: decompression sickness in
divers - ask about occupation! (2010) - Rheumatology

Corticosteroid use and alcohol are the most A 20-year-old woman is rejected as a blood
common associations in the UK. donor on account of Fe deficiency anaemia.

Which one of the following immunological


[ Q: 96 ] MasterClass Part2 investigations is most likely to be helpful in
(2010) - Rheumatology explaining her Fe deficiency?

A 30-year-old woman has started to develop A. Anti-nuclear antibody


paraesthesia affecting the thumb, index and B. Anti-neutrophil cytoplasmic antibody
middle fingers of the right hand. This is worse in
the morning and is relieved by shaking the C. Anti-smooth muscle antibody
hand. D. Anti-glutamic acid decarboxylase
antibody
Which of the following will NOT exacerbate her
symptoms? E. Anti-endomysial antibody.

A. Hypothyroidism
Answer & Comments
B. Diabetes mellitus
Correct answer: E
C. Rheumatoid arthritis
D. Driving a car Coeliac disease is a common cause of iron
E. Frusemide deficiency in both general and hospital practice
and should be excluded in this patient. IgA
F. Oral contraceptive pill endomysial antibodies are highly specific
G. Hydrocortisone markers of coeliac disease (specificity 95-100%,
sensitivity 70-90%, positive predictive value 95-
H. Typing
100%). IgA endomysial antibodies are directed
I. Wrist held in flexion against tissue transglutaminase, the recently
J. Osteoarthritis. described autoantigen in coeliac disease.

Dr. Khalid Yusuf El-Zohry – Sohag Teaching Hospital (01118391123) Page | 380
El-Zohry MRCP Questions Bank (Part 2) – Medical Masterclass 2010

[ Q: 98 ] MasterClass Part2 E. Anti-PM-Scl


(2010) - Rheumatology
A 35-year-old nurse developed Raynaud's
phenomenon six months ago (see image).

The antinuclear antibody (ANA) test was Answer & Comments


positive at 1:1000 with a speckled staining
pattern. She came to the out-patient Correct answer: A
department complaining of excessive tiredness
and lethargy. The creatine phosphokinase (CPK)
The picture shows puffy hands, and fingers with
was three times higher than the upper normal
swan neck and boutonniere deformities. The
limits, and the hand joints radiographs showed
fact that she had gross deformities of the hands
no erosions.
without joint damage or erosions, might
Which one of the following tests would you suggest that she has Jaccoud's arthropathy,
request to establish the diagnosis? which is more often seen in patients with
connective tissue disease such as systemic
A. Anti-U1 RNP
lupus erythematosus (SLE). This patients also
B. Anti-ds DNA exhibits features probably suggestive of an
overlap between SLE, myositis and scleroderma
C. Anti-Scl 70
which constitute the diagnosis of mixed
D. Anti-centromere antibodies connective tissue disease (MCTD).

Dr. Khalid Yusuf El-Zohry – Sohag Teaching Hospital (01118391123) Page | 381
El-Zohry MRCP Questions Bank (Part 2) – Medical Masterclass 2010

MCTD is an overlap syndrome characterised by Aspiration of the knee with microscopy to look
combinations of clinical features of SLE, for pus cells in the case of septic arthritis and
systemic scleroderma and polymyositis. The crystals in the case of gout or pseudogout is the
presenting symptoms of MCTD are most often: most useful investigation. Blood cultures may
 yield an organism. The radiographs are really as
Raynaud's phenomenon
a baseline and are unlikely to show acute
 puffy hands changes. Uric acid levels may not be elevated in
 arthralgias an acute episode of gout. A raised CRP is non-
specific, but can be used to monitor the
 myalgias
effectiveness of treatment.
 fatigue.
The various features of the connective tissue [ Q: 100 ] MasterClass Part2
disorders making up MCTD develop over (2010) - Rheumatology
months and years. A defining feature of MCTD
is the presence of antibodies against the U1 The parents of a 10-year-old asthmatic boy with
ribonucleoprotein (U1 RNP) complex, and peanut allergy are concerned about the risk of
hence the presence of high titre anti-U1 RNP future anaphylaxis if he were to inadvertently
will confirm the clinical diagnosis of MCTD. ingest peanuts.
Although the other tests might also be positive,
Which of the following features is the single
they will not be helpful in establishing the
most important predictor of anaphylaxis in this
underlying cause of the whole clinical spectrum.
situation?
A. Level of peanut-specific IgE in his serum
[ Q: 99 ] MasterClass Part2
(2010) - Rheumatology B. Strength of positive skin test response to
peanut
A 64-year-old man presents to A&E with a 2-day
C. Poorly controlled asthma
history of increasing pain and swelling of his left
knee. He denies a history of trauma. On D. Previous steroid therapy
examination, the knee is hot, red, swollen and E. Family history of nut allergy.
extremely tender.

Which of the following investigations is most Answer & Comments


important? Correct answer: C
A. Plain radiograph of the knee
B. Blood cultures Poorly controlled asthma is an important risk
factor for fatal anaphylaxis in this situation.
C. C-reactive protein (CRP) Children such as this should have their asthma
D. Joint aspiration well controlled and in addition, have ready
access to self-injectable adrenaline.
E. Plasma uric acid level.

Answer & Comments [ Q: 101 ] MasterClass Part2


(2010) - Rheumatology
Correct answer: D
A 55-year-old woman presents with a 3-month
history of a painful swollen knee. Examination
shows restricted, painful movement and a

Dr. Khalid Yusuf El-Zohry – Sohag Teaching Hospital (01118391123) Page | 382
El-Zohry MRCP Questions Bank (Part 2) – Medical Masterclass 2010

moderate sized effusion. Synovial fluid is B. Rheumatoid nodule


aspirated for diagnostic purposes.
C. Onycholysis
Which TWO of the following findings in the D. Uveitis
synovial fluid would be most compatible with a
E. Urethritis.
final diagnosis of osteoarthritis?
A. Macroscopically clear fluid Answer & Comments
B. Needle shaped crystals on microspcopy
Correct answer: B
C. Highly viscous fluid
D. High numbers of neutrophils on The long (> 1 hr) early morning and rest
microscopy stiffness are highly suggestive of an underlying
inflammatory condition. The pelvic radiograph
E. Blood stained fluid
confirms the presence of sacroiliitis. Bilateral
F. Thin, watery fluid sacroiliitis are typical for ankylosing spondylitis.
The frequency of asymmetric sacroiliitis may be
G. High numbers of macrophages on
higher in other spondyloarthropathies, e.g.,
microscopy
reactive arthritis, Reiter’s syndrome, spondylitis
H. Rhomboid-shaped crystals on microscopy associated with psoriasis, or inflammatory
I. Macroscopically turbid bowel disease.

J. Gram positive cocci on gram stain. Reiter's syndrome is characterised by a triad of


arthritis, urethritis, and conjunctivitis. Reiter's
syndrome develops in the setting of
Answer & Comments
postdysenteric or postvenereal illness. The
Correct answer: AC characteristic rashes of keratoderma
blennorrhagicum and circinate balanitis may be
Fluid aspirated from an osteoarthritic joint is present.
like normal synovial fluid: typically clear, viscous Psoriatic spondyloarthropathy is characterised
and virtually acellular. Inflammatory fluid is by psoriatic plaques. The skin involvement may
turbid (reflecting a high cell count) and loses be subtle and should be searched for carefully.
viscosity because of release of matrix degrading The cleft of the buttock, scalp hairline, and
enzymes in the synovial fluid. penis are site often involved but may be missed
easily if thorough examination of these areas
[ Q: 102 ] MasterClass Part2 was not actively conducted. Psoriatic nail
(2010) - Rheumatology changes include onycholysis, yellow nails and
nail pitting.
A 32-year-old man presents with a 4-month
Inflammatory bowel disease consists of
history of back pain. The pain is worse in the
ulcerative colitis and Crohn's disease.
morning and after sitting watching TV. Plain
radiograph of the spine/pelvis shows evidence A diagnosis of ankylosing spondylitis may be
of sacroiliitis. made when specific features of Reiter's
syndrome, psoriasis, or inflammatory bowel
Each of the following clinical features might be disease are absent.
identified on clinical examination of this patient
Sacroiliitis is also encountered in tuberculosis,
EXCEPT?
sarcoidosis and brucellosis.
A. Keratoderma blenorhegica

Dr. Khalid Yusuf El-Zohry – Sohag Teaching Hospital (01118391123) Page | 383
El-Zohry MRCP Questions Bank (Part 2) – Medical Masterclass 2010

Rheumatoid arthritis is not associated with I. Reactive arthritis


lumber or sacroiliac joint disease. However
J. SAPHO syndrome.
cervical spondylitis and atlanto-axial
subluxation is not an uncommon feature of RA.
Answer & Comments

[ Q: 103 ] MasterClass Part2 Correct answer: BE


(2010) - Rheumatology
Swelling of the distal interphalangeal joints
A 58-year-old woman is referred with pain and really only occurs in nodal osteoarthritis and
stiffness in her hands and knees. She has a few one of the forms of psoriatic arthritis, which is
patches of psoriasis on her arms. Her hands are usually easily distinguished by nail involvement
shown in the picture. (not present in this case).

Involvement of the base of the thumb is also


What are the TWO most likely diagnoses?
pathognomonic of nodal osteoarthritis, giving
A. Gout the thumb base a characteristically square
B. Nodal osteoarthritis appearance.

C. Pseudogout
[ Q: 104 ] MasterClass Part2
D. Systemic sclerosis
(2010) - Rheumatology
E. Psoriatic arthritis
A 32-year old woman with systemic lupus
F. Systemic lupus erythematosus erythematous (SLE) is seen in clinic. She had a
G. Ankylosing spondylitis flare of her disease 3 months ago, with
arthralgia and rash, but has been asymptomatic
H. Rheumatoid arthritis
Dr. Khalid Yusuf El-Zohry – Sohag Teaching Hospital (01118391123) Page | 384
El-Zohry MRCP Questions Bank (Part 2) – Medical Masterclass 2010

since a short course of moderate dose


prednisolone. She currently takes
hydroxychloroquine 200 mg bd and
prednisolone 8mg od. Clinical examination is
unremarkable. Her ESR is 60 mm/h and her CRP
is 8 mg/dl.

Which TWO of the following investigations


could explain her discordant inflammatory
markers?
A. Serum immunoglobulins
B. Chest radiograph
C. Plasma viscosity
D. Urinalysis
E. Complement levels
F. Plasma Lipids
G. Plasma urea
H. Serum ANA
I. Serum Anti-ds DNA antibodies
J. Serum ANCA.

Answer & Comments

Correct answer: AF

The C-reactive protein is a more sensitive


marker of inflammation and infection than the
ESR. The normal CRP in this lady fits with the
clinical picture of disease inactivity. Therefore,
the raised ESR must be secondary to a cause
other than inflammation or infection. Such
causes include hyperlipidaemia, anaemia and
hypergammaglobulinaemia, all of which
frequently occur in SLE.

Dr. Khalid Yusuf El-Zohry – Sohag Teaching Hospital (01118391123) Page | 385
El-Zohry MRCP Questions Bank (Part 2) – Medical Masterclass 2010

anaemia in CLL and advances the staging of the

Hematology disease.

[ Q: 2 ] MasterClass Part2
(130 Questions) (2010) - Hematology
(Medical Masterclass – Part 2) A 70-year-old man presents to casualty with a
massive bleed into his left thigh. He has no past
medical history, and had uncomplicated surgery
for a hernia 3 years previously. Examination is
[ Q: 1 ] MasterClass Part2 normal, except for a large thigh haematoma.
(2010) - Hematology Investigations reveal an isolated prolonged
activated partial thromboplastin time (APTT).
A 70-year-old woman is admitted with anaemia
and fatigue. Her full blood count reveals Hb What is the diagnosis?
6.7g/dl, MCV 101fl, WCC 76 x 10^9/l (90% A. Haemophilia A
lymphs), platelets 105 x 10^9/l, and Retics 14%.
She is slightly jaundiced and has splenomegaly. B. Disseminated intravascular coagulopathy
The lab think she probably has chronic (DIC)
lymphatic leukaemia. She is on no medication. C. Intra-abdominal malignancy

Which are the TWO most likely causes of the D. Acquired factor VIII inhibitor
anaemia? E. Trauma.
A. B12 deficiency
Answer & Comments
B. Bleeding
C. Folate deficiency Correct answer: D

D. Autoimmune haemolysis
Although rare, an acquired factor VIII inhibitor
E. Acquired G6PD deficiency should always be considered in a patient who
F. Iron deficiency presents in this way, especially if they have no
past medical or family history. These patients
G. Acute transformation may be very difficult to manage and should be
H. Marrow infiltration with leukaemia referred to a specialist centre as soon as
possible. An underlying cause should always be
I. Hyperviscosity
sought (e.g. malignancy, inflammatory disease).
J. Myelodysplasia. Treatment involves 'bypassing' the inhibitor to
achieve haemostasis and immunosuppression
Answer & Comments to get rid of the inhibitor.

Correct answer: DH
[ Q: 3 ] MasterClass Part2
The MCV is raised due to reticulocytosis - this is
(2010) - Hematology
a feature of autoimmune haemolysis. A You see a patient in clinic who has anaemia,
reticulocytosis does not occur in haematinic hypercalcaemia and osteolytic bone lesions
deficiency. Marrow infiltration also causes with pain. Kidney function appears to be
normal at this stage but he has an IgG kappa

Dr. Khalid Yusuf El-Zohry – Sohag Teaching Hospital (01118391123) Page | 387
El-Zohry MRCP Questions Bank (Part 2) – Medical Masterclass 2010

paraprotein of 50g/l. Bone marrow shows an Asparaginase is used in the treatment of acute
infiltrate of 50% plasma cells. lymphoblastic leukaemia.

Which of the following treatments are NOT


[ Q: 4 ] MasterClass Part2
used at some point in time in the course of this
illness? (2010) - Hematology
A. Paracetamol A 24-year-old woman presents with purpura
(see image). Investigation reveals haemoglobin
B. Melphalan
12.1 g/dL (normal range 11.5-16), mean
C. Prednisolone corpuscular volume 81 fL (normal range 80-
D. Cyclophosphamide 100), platelets 147 x 109/L (normal range 150-
400), white cell count 10 x 109/L (normal range
E. Thalidomide 4-11), prothrombin time 14 seconds (normal
F. Morphine Sulphate Tablets range 12-16), activated partial thromboplastin
time 32 seconds (normal range 30-46) and
G. Anagrelide
fibrinogen 4.1 g/L (normal range 2-4). She had
H. Radiotherapy similar appearances of purpura on her other leg
I. Pamidronate and on the flexor aspects of her forearms.

J. Asparaginase.

Answer & Comments

Correct answer: GJ

This patient has myeloma. This is a relapsing


and remitting disease without any known
curative therapy. Some patients are elderly and
die of an unrelated cause but most patients like
this will die of their myeloma; median survival is
about 2 to 3 years. Treatment is aimed at
getting the disease to a state where by it
(temporarily) stops causing end organ (kidney,
bone marrow, bone) damage. Initial
management includes intravenous combination
chemotherapy generally for younger (under 65)
patients. Older patients are generally treated
with oral chemotherapy. All drug treatments
are tried at some point in the course of the
illness and are given on and off until the tumour What is the most likely diagnosis?
is essentially unresponsive to all drugs.
A. Henoch-Schönlein purpura
Radiotherapy, analgesic drugs and
bisphosphonates are important in the B. Acute leukaemia
management of myeloma. C. Factitious purpura
Anagrelide is used to control platelets in D. Idiopathic thrombocytopenia
essential thrombocythaemia.
E. Thrombotic thrombocytopenia

Dr. Khalid Yusuf El-Zohry – Sohag Teaching Hospital (01118391123) Page | 388
El-Zohry MRCP Questions Bank (Part 2) – Medical Masterclass 2010

Answer & Comments

Correct answer: C A. Protein C deficiency


B. Protein S deficiency
The distribution on the forearms and thighs,
C. Antithrombin III deficiency
together with normal investigations, are typical
of factitious purpura, an uncommon condition D. Factor VIII deficiency
of uncertain (presumed psychological) E. Lupus anticoagulant
aetiology.
Answer & Comments
[ Q: 5 ] MasterClass Part2
Correct answer: D
(2010) - Hematology
A 57-year-old man is found to have Haemostasis and thrombosis is a finely
polycythaemia vera, with a haemoglobin of 20 regulated process that depends on the level
g/dL (normal range 12-16.5) and a haematocrit and configuration of prothrombotic proteins
of 0.48 (normal range 0.38-0.50). He is and natural anticoagulants. They can either be
asymptomatic. inherited or acquired. The inherited
thrombophilias are:
Which is the correct treatment?
 Activated Protein C (APC) resistance
A. No immediate intervention; organise due to an abnormal Factor V i.e. Factor
regular review V Leiden
B. Start oral hydroxycarbamide therapy  Protein C deficiency
C. Organise Venesection  Protein S deficiency
D. Start aspirin  Antithrombin III deficiency.
E. Start subcutaneous interferon injections Inherited hyperhomocysteinaemia is also a
recognised condition associated with aterial
Answer & Comments and venous thrombosis. The lupus
anticoagulant is an acquired anticoagulant in
Correct answer: C
vitro, and prolongs phospholipid dependent
tests but predisposes to thrombosis in patients.
Venesection should aim to get the haematocrit In up to 50% of cases of venous thrombosis it is
below 0.45 in primary polycythaemia. possible to demonstrate an inherited
abnormality of coagulation.
[ Q: 6 ] MasterClass Part2 Factor VIII defiency is an X-linked predisposition
(2010) - Hematology to bleeding called Haemophilia A.
A 38-year-old man has had two deep venous
thromboses in the absence of any recognised [ Q: 7 ] MasterClass Part2
precipitating events. You ask the haematology (2010) - Hematology
laboratory to screen for causes of a
thrombophilic state. A 35-year-old woman who is known to have
chronic idiopathic thrombocytopenia purpura
Which one of the following conditions will they (ITP) attends the haematology day care unit
NOT look for? because she has noted fresh rectal bleeding.

Dr. Khalid Yusuf El-Zohry – Sohag Teaching Hospital (01118391123) Page | 389
El-Zohry MRCP Questions Bank (Part 2) – Medical Masterclass 2010

She is pale and has clearly lost a significant I. Macrocytic


amount of blood. Her platelet count on arrival
J. Thrombocythaemic.
is 5 x 109/l.

Aside from blood transfusion, which of the Answer & Comments


following is the first treatment that should be
Correct answer: EF
given?
A. IV methylprednisolone The normal ranges are as follows: Hb 12-16 g/dl
B. Platelet transfusion (women), 13-17 g/dl (men); MCV 82-102 fl,
MCH 27-32 pg, WBC 4-11 x 10^9/l, platelets
C. IV immunoglobulins (IVIG)
130-400 x 10^9/l.
D. Tranexamic acid
This patient has a normochromic normocytic
E. Vincristine. anaemia. A common cause for this would be
the anaemia of chronic disease, e.g. secondary
Answer & Comments to rheumatoid arthritis or polymyalgia
rheumatica, but it would be wrong to jump
Correct answer: B immediately to this diagnosis. A blood film
should be examined because an MCV and MCH
Platelet transfusions are generally not indicated that fall within the normal range could result
in patients with ITP because the transfused from two populations of cells – one microcytic
platelets are destroyed by the patient's and one macrocytic – which would be described
antibodies. However, they are indicated when as a dimorphic picture. This might result from,
there is significant acute bleeding. Other for example, combined iron and folate
treatments will take longer to work, but may be deficiency
administered simultaneously.
[ Q: 9 ] MasterClass Part2
[ Q: 8 ] MasterClass Part2 (2010) - Hematology
(2010) - Hematology
A 15-year-old Greek Cypriot boy has the
A 61-year-old woman presents with tiredness. following blood results: haemoglobin (Hb) 8.2
Her full blood count is as follows: Hb 9.1 g/dl, g/dL, white blood cell count 11 x 109/L,
MCV 84 fl, MCH 28 pg, WBC 10.2 x 10^9/l, platelets 405 x 109/L, ferritin 3891 ug/L, iron 42
platelets 387 x 10^9/l. umol/L, transferrin saturation 70%, Hb A2= 6.1.
His liver biopsy is shown (see image).
Which TWO descriptive terms are correct?
A. Hypochromic
B. Leucopenic
C. Megaloblastic
D. Microcytic
E. Normochromic
F. Normocytic
G. Leucocytosis
H. Thrombocytopenic

Dr. Khalid Yusuf El-Zohry – Sohag Teaching Hospital (01118391123) Page | 390
El-Zohry MRCP Questions Bank (Part 2) – Medical Masterclass 2010

Which one of the treatments listed would be E. Protein C


best?
F. Factor XIII
A. Oral desferrioxamine treatment
G. Factor V
B. Oral ascorbic acid
H. Antithrombin
C. Daily subcutaneous desferrioxamine
I. Factor XII
infusion
J. Factor VII.
D. Bolus desferrioxamine therapy
E. Periodic transfusion therapy. Answer & Comments

Correct answer: EH
Answer & Comments

Correct answer: C Haemostasis is a finely balanced act between


clotting and excessive bleeding due to the
The boy has got beta thalassemia major and is presence of natural anticoagulant and
transfusion-dependent. His serum ferritin and procoagulant proteins. The natural
iron are high, with a low total iron binding anticoagulants are Protein C, Protein S and
capacity (TIBC) which is a measure of Antithrombin while the rest listed serve to
transferring saturation. initiate and/or stabilise blood clots in normal
His liver biopsy shows excessive iron in the liver circumstances. The boy has homozygous
with some degree of fibrosis. protein C deficiency which manifests as
extensive skin necrosis
Further transfusion is contraindicated, as is
ascorbic acid which can increase iron
absorption from stomach. He requires chelation
[ Q: 11 ] MasterClass Part2
therapy with subcutaneous desferrioxamine (2010) - Hematology
given preferably daily at a dose of 20-40 mg/kg A 73-year-old woman is admitted with Hb 7.5,
over 8-12 hours. It is not active orally and its MCV 99fL, neuts 1.1, platelets 68, reticulocytes
effectiveness depends on its ability to bind to 1%. Her neutrophils appear hypogranular.
iron over time rather than as a bolus. Poikilocytosis and anisocytosis are present. She
is only on atenolol for hypertension. She has
[ Q: 10 ] MasterClass Part2 had no previous illnesses.
(2010) - Hematology
What features do NOT support a diagnosis of
A 6-month old boy presented with skin necrosis myelodysplasia?
and leg swelling. There is a strong family histof
A. Low B12 level
recurrent venous thromboembolism.
B. Normal B12 level
Which of the following blood coagulation
C. Ring sideroblasts in the marrow
proteins inhibit blood coagulation?
D. Monocytosis
A. Fibrinogen
E. Raised ferritin
B. Factor IX
F. Cytogenetic abnormalities on marrow
C. Prothrombin
aspiration
D. Factor VIII
G. 5% blasts in the marrow

Dr. Khalid Yusuf El-Zohry – Sohag Teaching Hospital (01118391123) Page | 391
El-Zohry MRCP Questions Bank (Part 2) – Medical Masterclass 2010

H. Normal folate levels I. Low molecular weight heparin is given


intra-muscularly.
I. Normal retic count
J. She should be advised not to become
J. Negative Coombs' test.
pregnant again.

Answer & Comments


Answer & Comments
Correct answer: AE
Correct answer: AE

B12 and folate levels must be normal to


This patient needs to start LMW heparin.
establish the diagnosis. Blasts can be a feature
Warfarin is teratogenic and should not be given
of evolving myelodysplasia. The retic count is
in the first trimester. Because of bleeding
variable.
during the time of labour warfarin should not
be given in the third trimester. Although
[ Q: 12 ] MasterClass Part2 theoretically warfarin can be given in the
(2010) - Hematology second trimester generally speaking it is not
used and pregnant patients get daily LMW
A 32-year-old woman who is 9 weeks pregnant
heparin throughout pregnancy to treat DVT.
in her first pregnancy develops a painful
swollen calf. Ultrasound scan confirms the Ninety per cent of DVT's in pregnancy occur on
presence of extensive deep vein thrombosis the left.
extending into femoral and illiac vessels. This is Anticoagulation should continue until at least 6
her first clot and there is no other obvious risk weeks post partum in this patient.
factor.
The APTT is used to monitor unfrationated
Which of the following statements regarding heparin and the PT to monitor warfarin.
this lady are correct? Warfarin tablets: 5mg = pink, 3mg = blue, 1mg =
A. The patient should be commenced brown.
immediately on a therapeutic dose of low Penicillin will affect warfarin control but does
molecular weight heparin. not preclude its use.
B. Warfarin should be started and heparin Pregnancy should not be avoided in the future
can be stopped when the INR is greater but prophylactic anticoagulation should be
than 2.0. offered during the next pregnancy.
C. In pregnant women the majority of DVT's
occur in the right leg. [ Q: 13 ] MasterClass Part2
D. This patient requires 3 months (2010) - Hematology
anticoagulation.
A 25-year-old man was shot in the thigh at a
E. 5mg warfarin tablets are pink. local restaurant and was moribund when
F. 1mg warfarin tablets are blue. brought in to A&E. He received 20 units of
blood to maintain normovolaemia. His
G. She needs her APTT monitored whilst
coagulation parameters were PT 23 seconds,
taking warfarin to titrate dose.
APTT 56 seconds and thrmobin time 25
H. Warfarin is contraindicated in patients seconds. His fibrinogen level was 0.6g/L.
taking Penicillin.
Which of the following results can be predicted?

Dr. Khalid Yusuf El-Zohry – Sohag Teaching Hospital (01118391123) Page | 392
El-Zohry MRCP Questions Bank (Part 2) – Medical Masterclass 2010

A. Low platelets B. Thrombotic thrombocytopenic purpura


(TTP)
B. Low antithrombin levels
C. Disseminated intravascular coagulation
C. High protein C level
(DIC)
D. Raised haemoglobin
D. Heparin-induced thrombocytopenia and
E. Elevated factor VIII levels thrombosis (HITT)
F. Shortened euglobulin clot lysis time E. Post-transfusion purpura (PTP).
G. Normal bleeding time of 6 mins
Answer & Comments
H. Normal factor II levels
I. Low D-Dimer levels Correct answer: D

J. Normal Ristocetin Cofactor levels (RICOF).


HITT occurs with low molecular weight (LMW)
heparin and can be fatal. It occurs about 7 days
Answer & Comments after commencing heparin and is an immune-
Correct answer: AB mediated phenomenon. Platelet aggregation
results in thromboembolic events. Stop all
heparin and get advice about an alternative
Disseminated intravascular coagulation results
anticoagulant (hirudin or danaparoid).
from excess thrombin generation arising from
excessive tissue thromboplastin activation of ITP could occur postop but this would be
coagulation. This occurs in presence of reduced unusual.
fibrinolysis, leading to bleeding diathesis. There
There is little to suggest TTP on the film.
are low levels of platelets (and hence a
prolonged bleeding time), protein C/S, The normal fibrinogen is against DIC.
antithrombin, factor VIII, factor VII and other Post-transfusion purpura is unusual but can
coagulation factors due to ongoing occur after any transfusion - another immune
consumption and reduced synthesis. PAI type 1 phenomenon, but less likely than HITT in this
levels, D-Dimers are increased. situation.

[ Q: 14 ] MasterClass Part2 [ Q: 15 ] MasterClass Part2


(2010) - Hematology (2010) - Hematology
A 60-year-old man had a hip replacement. A 63-year-old man was admitted with fever and
Preoperatively all blood tests were normal. He dyspnoea. He had been previously well.
had Clexane in the perioperative period. Seven Investigation revealed haemoglobin was 9.2
days after the operation, he had a large g/dL (normal range 12-16.5), mean corpuscular
pulmonary embolus. INR 1.2, APPT 39 secs, volume 113 fL (normal range 80-100), white cell
Fibrinogen 4.5 g/l, Hb 12.4 g/dl, platelets 22, count 14 x 109/L (normal range 4-11) and
WCC 23 (neuts 21). Blood film showed a left platelets 23 x 109/L (normal range 150-400). His
shift and some anisocytosis. blood film is shown in the image above.

What is the most likely diagnosis?


A. Idiopathic thrombocytopenic purpura
(ITP)

Dr. Khalid Yusuf El-Zohry – Sohag Teaching Hospital (01118391123) Page | 393
El-Zohry MRCP Questions Bank (Part 2) – Medical Masterclass 2010

What is the most likely diagnosis?


A. Renal failure due to
hyperviscosity
B. Cardiac ischaemia due to
anaemia
C. Fludarabine induced haemolysis
D. Relapse of Waldenström’s with
haemolytic transfusion reaction
E. Transformation to acute
leukaemia.

Answer & Comments


What is the most likely diagnosis?
A. Acute myeloid leukaemia Correct answer: D

B. Chronic myeloid leukaemia


The plasma viscosity is only slightly raised and is
C. Myeloma unlikely to produce renal failure at this level.
D. Chronic lymphatic leukaemia Fludarabine can cause Coombs’ positive
E. Mycoplasma pneumonia haemolysis, but not usually 5 years after
treatment.
Answer & Comments There is no evidence for acute myeloid
leukaemia (AML).
Correct answer: E
Most Waldenström’s and myeloma patients
relapse with a rise in their paraprotein; this
Note the clumping of red cells on the film
man's IgM had risen to 29 g/l when tested and
caused by cold agglutinins, which are a feature
the anaemia was a feature of active disease. He
of Mycoplasma pneumonia
developed chest pain due to an acute
haemolytic transfusion reaction, and this
[ Q: 16 ] MasterClass Part2 caused the Coombs’ test to become positive.
(2010) - Hematology He had developed a Jka red cell antibody, which
had not previously been detected. He required
A 76-year-old man had been treated with
dialysis but his renal function improved and he
fludarabine for Waldenström’s
was able to discontinue this. He went on to
macroglobulinaemia 5 years ago. His IgM band
further chemotherapy.
had fallen from 36 g/l to a plateau of 5 g/l. He
was stable for 5 years but suddenly presented
with Hb 6.7 gm/dl, platelets 79, creatinine 130 [ Q: 17 ] MasterClass Part2
micromol/l, plasma viscosity 2.5 mPa/sec. (2010) - Hematology
During blood transfusion, he developed a
A 31-year-old woman is found to be a carrier
tachycardia and chest pain and was transferred
for the factor V Leiden gene mutation. She has
to CCU. The next day results showed Hb 7 g/dl,
had no previous thromboembolic events and
Coombs’ test positive, creatinine 377
has had two normal pregnancies. She uses the
micromol/l.
coil for contraception. Her sister is also a carrier

Dr. Khalid Yusuf El-Zohry – Sohag Teaching Hospital (01118391123) Page | 394
El-Zohry MRCP Questions Bank (Part 2) – Medical Masterclass 2010

and has been on Warfarin for a single deep vein haemoglobin is 20.3g/dl and haematocrit is
thrombosis (DVT). Her mother is homozygous 61%.
for the same mutation and is on life-long
Warfarin following a serious pulmonary Which TWO of the following statements are
embolism (PE). correct?
A. He should be commenced on warfarin
Which of the following treatments is correct?
with a target INR of 2.5.
A. Offered life-long Warfarin
B. He requires a red cell mass in order to
B. Given Heparin in future pregnancies confirm the diagnosis.
C. Put on aspirin C. In untreated primary proliferative
polycythaemia rubra vera (PRV) there is
D. Observed with no therapy
an increased risk of thrombosis.
E. Told to forget about the problem.
D. Untreated secondary polycythaemia is
associated with a 15% risk of stroke over
Answer & Comments 1 year.
Correct answer: D E. PRV is confirmed with a red cell mass
greater than or equal to 200% of
In the absence of a personal thromboembolic predicted.
history, Warfarin is not indicated - she may
F. Because of the risk of leukaemic
develop bleeding on it. However, given the
transformation drugs such as
strong family history, one should be concerned
hydroxyurea and busulphan should be
that additional thrombophilic problems are
not be used in patients with PRV.
present and a full thrombophilia screen should
be performed. If the rest of the screen is G. Patients with PRV have a 50% chance of
negative, one should still be cautious because developing acute myeloid leukaenia over
of the family history - there may be the following 5 years.
thrombophilic tendencies we are yet to H. Venesection is required in patients with
discover. PRV aiming to reduce the haematocrit to
Pregnancy does not need heparin cover in this 35%.
woman. The presence of Factor V Leiden itself I. Patients with PRV are banned from driving
does not prevent the use of the oral because of the risks of stroke.
contraceptive or HRT, but given the strong
J. Median survival for patients with PRV is 6
family history, the woman should be counselled
years.
about the increased risk of thromboembolism
versus the potential benefits and her decision
documented for future reference. Answer & Comments

Correct answer: BC
[ Q: 18 ] MasterClass Part2
(2010) - Hematology PRV is confirmed by a red cell mass of greater
than 125% of predicted. Treatment is aimed at
A 72-year-old man has had a full blood count
reducing the haematocrit to less than 45% by
for investigation of 'dizzy spells'. He is a life-long
either venesection or drugs (such as
smoker and drinks three pints of beer a day.
hydroxyurea or busulphan). Risk of
Otherwise he reports being fit and well. His
transformation to acute leukaemia is very low.

Dr. Khalid Yusuf El-Zohry – Sohag Teaching Hospital (01118391123) Page | 395
El-Zohry MRCP Questions Bank (Part 2) – Medical Masterclass 2010

Secondary polycythaemia is not associated [ Q: 20 ] MasterClass Part2


directly with an increased risk of stroke. (2010) - Hematology
Patients are not banned from driving. Consult
DVLA for advice regarding driving. A 76-year-old man has had refractory anaemia
for 3 years. He has required blood transfusion
The median survival for PRV is 10-15 years.
every 6 weeks. His white cell count (WCC)
normally runs at 3 x 109/l, platelets at 88 x 109/l
[ Q: 19 ] MasterClass Part2 and mean corpuscular volume (MCV) 110 fl. He
(2010) - Hematology is admitted as an emergency with a chest
infection and his full blood count reveals Hb
10.2 g/dl, MCV 116 fl, platelets 22 x 109/l, WCC
77 x 109/l (neuts 1.3 x 109/l). There is no
obvious blood loss.

What is the most likely cause?


A. Iron deficiency
B. Folate deficiency
C. Occult gastrointestinal blood loss

Identify the most obvious feature on this blood D. Transformation to acute leukaemia
film (see image), taken from a patient E. Mycoplasma pneumonia
presenting with bone pain.
A. Red cell clumping Answer & Comments
B. Platelet clumping Correct answer: D
C. Rouleaux
The MCV is often raised as a feature of
D. Hypochromia
myelodysplasia and has not changed
E. Cold agglutinins. significantly here. The sudden collapse in
platelets and rise in WCC signifies progression
Answer & Comments of the disease. The neutrophil count is low, so
the other white cells are probably leukaemic.
Correct answer: C Although a pneumonia could in exceptional
circumstances cause a WCC of 77 x 109/l, the
Multiple myeloma is a malignant proliferation neutrophil count would be much higher. About
of clonal plasma cells in the bone marrow. 40% of patient with myelodysplasia will
Presentation is often non-specific with bone transform into leukaemia.
pain, symptoms of anaemia or symptoms due
to renal failure. The ESR is most often raised
[ Q: 21 ] MasterClass Part2
and a characteristic finding on the blood film is
rouleaux (coin-like stacking up of red cells). This
(2010) - Hematology
should be distinguished from auto-agglutination
A 78-year-old man is receiving treatment for a
where the red cells are matted together( 'akin
deep venous thrombosis. After five days the
to clumped rice'). In most cases of
pharmacist reminds you to check a full blood
myelomatosis, treatment is with chemotherapy
count: why?
and supportive care.

Dr. Khalid Yusuf El-Zohry – Sohag Teaching Hospital (01118391123) Page | 396
El-Zohry MRCP Questions Bank (Part 2) – Medical Masterclass 2010

A. Because of the possibility of Relative polycythaemia occurs mainly in


gastrointestinal haemorrhage overweight middle aged men. This may be
exacerbated by diuretics. True polycythaemia is
B. Because of the possibility of haemolytic
often associated with changes in platelts and
anaemia
WCC. There is no suggestion of lung
C. Because of the possibility of leucocytosis abnormalities here.
D. Because of the possibility of thrombosis
E. Because of the possibility of [ Q: 23 ] MasterClass Part2
thrombocytopenia. (2010) - Hematology
A 22-year-old man has splenomegaly.
Answer & Comments
Which of the following would NOT cause
Correct answer: E
splenomegaly in this patient?

Tinzaparin, as well as unfractionated heparin, A. Non Hodgkin's lymphoma (NHL)


can cause thrombocytopenia which usually B. Chronic myeloid leukaemia (CML)
occurs between days 5 and 7 of treatment.
C. Beta thalassaemia major
Many doctors routinely check the platelet count
on day 5 of treatment to detect this and stop D. Sickle cell disease (HbSS)
further administration if appropriate. E. Acute lymphoblastic leukaemia (ALL)
See also tinzaparin's Summary of Product F. Falciparum malaria
characteristics.
G. Hereditary spherocytosis

[ Q: 22 ] MasterClass Part2 H. Haemophilia A complicated by hepatitis C


infection
(2010) - Hematology
I. Idiopathic thrombocytopenia purpura
A 58 year old man is found to have a Hb of 19
(ITP)
g/dL, haematocrit of 51%, platelets of 298 x
109/L and WCC 12 x 109/L. He is a non-smoker, J. Essential thrombocythaemia (ET).
drinks 10 units of alcohol per week, and is on
medication for hypertension. He is otherwise Answer & Comments
healthy, with no specific symptoms.
Correct answer: DI
The most likely diagnosis is:
The myeloproliferative disorders (although rare
A. Chronic lung disease
in this age group) may all cause splenomegaly.
B. Primary polycythaemia More common in a man of this age is
C. Relative polycythaemia leukaemia, which also may cause splenomegaly.

D. Myelofibrosis Thalassaemia major, unlike beta thalassaemia


trait, causes splenomegaly.
E. Occult kidney tumour
Sickle cell disease causes splenomegaly in
children but by the time individuals are adults
Answer & Comments
the spleen has infarcted and is very small.
Correct answer: C Consequently penicillin prophylaxis and
vaccinations (pneumococcus, meningococcus

Dr. Khalid Yusuf El-Zohry – Sohag Teaching Hospital (01118391123) Page | 397
El-Zohry MRCP Questions Bank (Part 2) – Medical Masterclass 2010

and HIB) are recommended to reduce the risk [ Q: 25 ] MasterClass Part2


of infection with encapsulated organisms. ET, (2010) - Hematology
being a myeloproliferative disorder, is
associated with splenomegaly but ITP is not. (1) A 35-year-old woman is on warfarin for
recurrent DVTs and PEs. Her target INR is 3.5. In
clinic, although well, her INR is found to be 6.
[ Q: 24 ] MasterClass Part2
(2010) - Hematology What is the correct management?
A 19-year old man had a history of sore throat A. Stop the warfarin and restart when the
and lethargy for two weeks. Nodes were felt on INR is 3.5.
both sides of his neck. His fauces looked mildly
B. Stop the warfarin and restart when the
inflamed. His spleen was just palpable, and he
INR is < 5.
was slightly jaundiced. Blood tests revealed: Hb
7.5 g/dl, MCV 107 fl, Neuts 2.1, Lymphs 6.7, C. Stop the warfarin and give 2.5 mg of
Platelets 122. Bilirubin 32, AST 72, GGT 68. Vitamin K.
D. Stop the warfarin and give 5 mg of
The most likely diagnosis is:
Vitamin K.
A. Acute myeloid leukaemia
E. Continue warfarin at a lower dose.
B. Folate deficiency
C. Infectious mononucleosis with Answer & Comments
haemolysis Correct answer: B
D. Chronic lymphocytic leukaemia
E. Hepatitis A. This patient is well with no additional risk
factors for bleeding so does not need the
warfarin reversed. The best approach would be
Answer & Comments
to stop the warfarin, retest and restart as soon
Correct answer: C as the INR is dropping. If you wait until the INR
is at the target range you run the risk of under-
The patient is anaemic with a raised MCV due anticoagulating her.
to reticulocytosis. Haemolysis is a recognized
(but uncommon) complication of infectious [ Q: 26 ] MasterClass Part2
mononucleosis that can require steroid (2010) - Hematology
therapy. A mild hepatitic picture and
splenomegaly is quite common. You are about to start a patient with myeloma
on thalidomide. When starting a new drug you
There is little to suggest leukaemia here, and
must discuss the common side effects and
neither folate deficiency nor hepatitis A could
major risks of taking the treatment before
explain all the features seen. The blood film
gaining consent.
showed polychromasia (due to the presence of
reticulocytes), reactive lymphocytes and One of the following is NOT a common side
Monospot was positive to establish the effect or major risk with taking this drug?
diagnosis.
A. Is teratogenic
B. Causes rash
C. Causes peripheral neuropathy

Dr. Khalid Yusuf El-Zohry – Sohag Teaching Hospital (01118391123) Page | 398
El-Zohry MRCP Questions Bank (Part 2) – Medical Masterclass 2010

D. Causes diarrhoea C. She has developed acute lymphoblastic


leukaemia.
E. Causes somnolence
D. She has developed primary
Answer & Comments thrombocythemia.

Correct answer: D E. She has developed an infection and


should respond to antibiotics.

Thalidomide has been found to be an effective


agent in the management of multiple myeloma. Answer & Comments
Thalidomide is teratogenic and it is vital Correct answer: A
patients are warned not to get pregnant or
allow the drug to be taken by anybody else.
It is not unusual nowadays with the advent of
Thalidomide has a number of common side
routine automated blood counts to pick up a
effects including somnolence, rash, peripheral
patient with chronic myeloid leukaemia and
neuropathy and constipation. Diarrhoea is not a
none of the classical signs of large
common side effect of thalidomide treatment.
hepatosplenomegaly. The picture shows an
elevated white count with no increase in blast
[ Q: 27 ] MasterClass Part2 cells and there are myelocytes and
(2010) - Hematology metamyelocytes suggestive of chronic phase
disease. A bone marrow or peripheral blood
A previously well 57-year-old lady with only a cytogenetics should be carried out looking for
past history of allergy went to her GP the Philadelphia chromosome.
complaining of lethargy and recurrent ‘flu-like’
symptoms. Her complete blood count was Hb
13.1, wbc 279 x 10^9/l and platelets 567 x [ Q: 28 ] MasterClass Part2
10^9/l. There is no organomegaly. A blood film (2010) - Hematology
shows myelocytes, metamyelocytes,
(1) A 29-year-old Asian woman is 16 weeks
eosinophils and basophils.
pregnant. There is no family history of
haemoglobinopathy. She eats meat twice a
week and has a balanced diet. She has had no
gastrointestinal problems. Hb 9.9 gm/dl, MCV
68 fl, iron levels borderline low.

What are the TWO most important steps to


take next?
A. Commence oral iron
B. Commence oral folate
C. Perform HbA2 and HbF

Which of the following statements is true? D. Perform a Coomb's test

A. She has developed Chronic myeloid E. Arrange a blood transfusion


leukaemia. F. Suggest she eats meat more frequently
B. She has developed a leukaemoid G. Commence oral vitamin supplements
response to her bracelet.
H. Commence intramuscular iron

Dr. Khalid Yusuf El-Zohry – Sohag Teaching Hospital (01118391123) Page | 399
El-Zohry MRCP Questions Bank (Part 2) – Medical Masterclass 2010

I. Organise a chorionic villous biopsy


J. Test her partner for any inherited
haemoglobinopathy.

Answer & Comments

Correct answer: CJ

Although iron and folate deficiency are


common in pregnancy and require therapy,
there may be something else happening here.
Beta thalassaemia carriers have a raised HbA2
and HbF. If they reproduce with another carrier
they could give birth to a severely affected
homozygous child - that is why it is important to
test the partner early to alert obstetricians and
paediatricians about the possibility.

[ Q: 29 ] MasterClass Part2
(2010) - Hematology
A 76-year-old woman had been unwell for 2
weeks. Headaches, limb aches and fatigue. Hb
10.2 g/dl, platelets 382, WCC 13.2 (neuts 9.1).
Her erythrocyte sedimentation rate (ESR) tube
is the one on the right.

Which are the TWO most likely diagnoses?


A. Iron deficiency anaemia
B. Breast cancer
C. Rheumatoid arthritis
D. Acute myeloid leukaemia
E. Polycythaemia
F. Polymyalgia rheumatica
G. Systemic lupus erythematosis
H. Myeloma
I. Diabetes mellitus
J. Diverticular disease.

Dr. Khalid Yusuf El-Zohry – Sohag Teaching Hospital (01118391123) Page | 400
El-Zohry MRCP Questions Bank (Part 2) – Medical Masterclass 2010

Answer & Comments The platelet count is raised. Myeloproliferative


disorders such as CML, ET and myelofibrosis
Correct answer: FH
may all cause a raised platelet count. Platelet
count may also rise in response to infection or
The ESR tube on the right shows a very rapid inflammation. Therefore all causes of trauma
sedimentation rate of >120 mm/hr. This is a (including surgery) or inflammation such as
feature of polymyalgia or myeloma. The protein infections or inflammatory disease (e.g. Crohn’s
excess of myeloma neutralizes the negative disease) may cause a raised platelet count.
charge on the erythrocytes and allows them to Secondary causes of a raised platelet count are
get close together and settle rapidly. Anaemia far more common than primary causes
and cancer also raise the ESR but usually in the
order of 40 to 90 mm/hr (e.g. tube on the left).
Plasma viscosity is more reliable than the ESR as
[ Q: 31 ] MasterClass Part2
there are no confounding influences from the (2010) - Hematology
haemoglobin concentration A 30-year-old woman in her second pregnancy
develops DVT and suspected PE at 25 weeks.
[ Q: 30 ] MasterClass Part2 There is no personal or family history of note.
(2010) - Hematology
What advice do you think the haematologist
The laboratory call to inform you that a 43- would give?
year-old lady on a surgical ward has a platelet
A. Delay anticoagulation until the
count of 790 x10^9/l.
thrombophilia tests are through
Given only this information, which TWO B. Commence warfarin immediately
conditions would NOT form part of the
C. Recommend termination
differential diagnosis at this stage?
D. Give low molecular weight heparin
A. Chronic myeloid leukaemia (CML)
throughout pregnancy
B. Cholecystitis
E. Continue LMW heparin throughout
C. 2 days post hemicolectomy pregnancy and use warfarin postpartum
D. Acute lymphoblastic leukaemia (ALL)
Answer & Comments
E. Essential thrombocythemia (ET)
Correct answer: E
F. Recently admitted following a road
accident
Thromboembolism remains the commonest
G. Crohn’s disease
cause of maternal death and requires
H. Acute pancreatitis therapeutic heparinisation in the same way as
non-pregnant patients are treated. A
I. Myelofibrosis
thrombophilia test adds very little at this point
J. Thrombotic thrombocytopenia purpura and should not delay therapy. LMW heparin
(TTP) should be given throughout pregnancy and
converted to warfarin postpartum (the most
Answer & Comments dangerous time for further thromboembolic
events). Warfarin is rarely used in pregnancy
Correct answer: DJ
because of the risks to the foetus.

Dr. Khalid Yusuf El-Zohry – Sohag Teaching Hospital (01118391123) Page | 401
El-Zohry MRCP Questions Bank (Part 2) – Medical Masterclass 2010

[ Q: 32 ] MasterClass Part2 [ Q: 33 ] MasterClass Part2


(2010) - Hematology (2010) - Hematology
A 32-year-old man with homozygous sickle cell A 65-year-old man is found to have a
anaemia complains of pain on adbuction of his haemoglobin of 21g/dl at preclerking before a
shoulder. His haemoglobin is 9.6 g/dL, white knee replacement operation. The orthopaedic
blood cells (WBC) 11 x 109/L and platelets 453 x surgeon refers the man to the haematology
109/L. outpatient clinic and suggests a possible
diagnosis of polycythaemia rubra vera (PRV).

Which of the following is not a feature of this?


A. Raised haematocrit
B. Raised red cell mass
C. Raised urate
D. Hypercellular bone marrow
E. Raised erythropoietin levels.

Answer & Comments

Correct answer: E

Patients with PRV have over production of red


What is the cause of the radiographic changes blood cells with in the bone marrow and as a
shown? result haemoglobin, red cell mass and
haematocrit are all raised. Bone marrow
A. osteoporosis
examination reveals a hypercellular marrow.
B. extramedullary haemopoiesis Remember in apparent polycythaemia there is
C. osteomalacia a reduced plasma volume and raised
haematocrit but there is a normal red cell mass.
D. avascular necrosis of long bones As a result of the increased turnover of cells,
E. bone pain crisis. urate rises and these patients not uncommonly
report symptoms of gout. Erythropoietin levels
Answer & Comments are LOW in PRV as the raised haemoglobin
negatively feeds back on erythropoietin
Correct answer: D production.

Avascular necrosis of the long bones is a [ Q: 34 ] MasterClass Part2


recognised complication of sickle cell disease. It
(2010) - Hematology
is due to repeated vaso-occlusion in the small
blood vessels of the long bones and is A 35-year-old woman presents to her doctor
commoner in the hips than in the upper limbs. with a painful and swollen left leg. Ultrasound
Treatment is usually osteotomy or replacement examination confirms a suspected deep venous
(with varying degrees of success). Hydroxyurea thrombosis. A prolonged APTT is noted and
has not been shown to reduce the incidence of anticardiolipin antibodies are detected.
this complication.

Dr. Khalid Yusuf El-Zohry – Sohag Teaching Hospital (01118391123) Page | 402
El-Zohry MRCP Questions Bank (Part 2) – Medical Masterclass 2010

Which of the following is correct? Answer & Comments


A. She has primary antiphospholipid Correct answer: A
syndrome.
B. She requires life-long anticoagulation. Thrombocytopenia in pregnancy should always
be investigated, irrespective of the mode of
C. She requires a repeat cardiolipin
delivery anticipated.
antibodies after 6 weeks.
D. She should be treated with life-long
[ Q: 36 ] MasterClass Part2
aspirin.
(2010) - Hematology
E. Her prolonged APTT will correct with the
addition of normal plasma. A 26-year-old woman, who feels completely
well, comes to the Accident and Emergency
Answer & Comments department because she has developed spots.
Examination reveals widespread petechiae but
Correct answer: C nothing else abnormal.

The history of venous thrombosis and the The most likely diagnosis is:
detection of cardiolipin antibodies is consistent A. Meningococcal septicaemia
with the anti-phospholipid syndrome. However,
B. Idiopathic thrombocytopenic purpura
anticardiolipin antibodies are not specific and,
therefore, have to be repeated after a 6-week C. Acute lymphatic leukaemia
interval. See Medical Masterclass, D. Systemic lupus erythematosus
Rheumatology and Clinical Immunology
module, Clinical Presentation 1.11, for further E. Haemolytic uraemic syndrome.
discussion.
Answer & Comments
[ Q: 35 ] MasterClass Part2 Correct answer: B
(2010) - Hematology
Patients with meningococcal septicaemia do
A 28-year-old woman presents in her first
not feel well. Both systemic lupus
pregnancy with a platelet count of 65 x 109/l.
erythematosus and acute lymphatic leukaemia
This count is confirmed and a full blood count
can present with thrombocytopenic petechiae,
two years before was normal.
but are less likely diagnoses. Haemolytic
When is it correct to investigate a low platelet uraemic syndrome does not present in this way.
count in the first pregnancy?
A. It should always be investigated [ Q: 37 ] MasterClass Part2
(2010) - Hematology
B. If the platelet count is less than 100
C. If the platelet count is less then 50 A 47-year-old decorator has hereditary
spherocytosis. His Hb is 11 g/dl, MCV 89 fl,
D. Only if the LFTs are also abnormal bilirubin 23 mmol/l. He copes with his job but
E. Only if an epidural is planned asks if splenectomy may help.

Which are the most appropriate responses?

Dr. Khalid Yusuf El-Zohry – Sohag Teaching Hospital (01118391123) Page | 403
El-Zohry MRCP Questions Bank (Part 2) – Medical Masterclass 2010

A. Splenectomy is without serious The most likely diagnosis is:


consequences.
A. Pulmonary embolism
B. Folic acid replacement can usually
B. Myocardial infarction
obviate the need for splenectomy.
C. Pneumococcal pneumonia
C. Splenectomy is indicated for symptomatic
anaemia. D. Pneumothorax

D. Splenectomy should always be followed E. Acute chest syndrome.


by B12 replacement.
Answer & Comments
E. Splenectomy does not require vaccination
in adults. Correct answer: E
F. Splenectomy should be routinely
performed at the age of 7 years. This is a typical presentation of acute chest
syndrome, which is one of the commonest
G. Splenectomy is indicated for
causes of death in adults with sickle cell
splenomegaly in spherocytosis
disease. Pulmonary embolism and pneumonia
H. Splenectomy is indicated for cannot be excluded, but are less likely
uncompensated haemolysis. diagnoses
I. Splenectomy always improves the Hb Key aspects of management are:
level.
1. Give high flow oxygen via reservoir bag
J. Splenectomy reduces the number of
2. Rapid intravenous infusion of 1 litre of
circulating spherocytes.
0.9% saline

Answer & Comments 3. Start intravenous antibiotics, e.g.


ampicillin 500 mg qds
Correct answer: CH
4. Intravenous opioid for pain, e.g.
diamorphine 5 mg, with antiemetic
Splenectomy is not without hazard. It is
reserved for symptomatic anaemia which 5. Prophylaxis against venous
affects quality of life. Vaccination and lifelong thromboembolism, e.g. enoxaparin 20
penicillin are required in adults. Splenectomy mg SC od
removes the source of red cell destruction and Call for specialist advice if the patient
spherocytes may rise in number on the blood deteriorates or does not improve rapidly.
film. Splenectomy is performed in older rather Exchange transfusion may be indicated if the
than young children, but is not routine. patient becomes hypoxic

[ Q: 38 ] MasterClass Part2 [ Q: 39 ] MasterClass Part2


(2010) - Hematology (2010) - Hematology
A tall thin 18-year-old man with sickle cell A 75-year-old lady was admitted via A&E with
disease presents to the Accident and the following blood counts: Hb 4g/dl, white
Emergency department with a 36-hour history blood count 11 x 109/l, mean corpuscular
of cough, fever, breathlessness and pleuritic volume 109 fl, and platelets 540. The
chest pain. reticulocyte count is 6%.

Dr. Khalid Yusuf El-Zohry – Sohag Teaching Hospital (01118391123) Page | 404
El-Zohry MRCP Questions Bank (Part 2) – Medical Masterclass 2010

Which of the following are not useful for relapsed disease, presented with 8 cm
investigations in extravascular haemolytic splenomegaly and bone marrow involvement.
anaemia? Despite this she had a normal full blood count
A. Bilirubin estimation Hb 13.4g/dl, Wbc 8.0 x 109/l and platelets of
347 x 109/l and no B symptoms. A splenectomy
B. Haemoglobin electrophoresis was carried out with good benefit and she
C. G6PD spot test remained in good partial remission for 18
months.
D. Haemosiderinuria
At a routine clinic follow-up she has the
E. Absolute reticulocyte count
following blood count: Hb 8.4g/dl, mcv 124fl,
F. Vitamin B12 estimation Wbc 11x109/l and platelets 287 x 109/l, retics
12%. Coombs test was negative. Examination
G. Serum haptoglobins
was unremarkable apart from a tinge of
H. Osmotic fragility test jaundice. Her blood film is as shown (see
I. Sickle solubility test image).

J. Blood film evaluation.

Answer & Comments

Correct answer: DG

In extravascular haemolysis, destruction of the


damaged red cell occurs in the
reticuloendothelial system (RE) with formation
of haem and globin. The haem is broken down
into iron which is bound to transferrin for
recirculation and the globin is reutilized.
Conditions which cause extravascular
haemolysis include:
 sickle cell What complication has occurred?

 hereditary spherocytosis A. Disease progression

 thalassaemia. B. accessory spleen

Haemosiderinuria is a by-product of C. Auto immune haemolytic anaemia


‘intravascular’ haemolysis and serum D. Myelodysplasia
haptoglobins are low due to the haptoglobin–
E. GI blood loss.
haemoglobin complex being cleared by the RE
system in intravascular haemolysis.
Answer & Comments

[ Q: 40 ] MasterClass Part2 Correct answer: C


(2010) - Hematology
This lady has developed Coombs negative
A 54-year-old woman diagnosed with low grade autoimmune haemolytic anaemia with
B cell non Hodgkin's lymphoma, previously hyperbilirubinaemia and reticulocytosis. The
treated with chemotherapy and radiotherapy blood film shows spherocytosis. This can occur

Dr. Khalid Yusuf El-Zohry – Sohag Teaching Hospital (01118391123) Page | 405
El-Zohry MRCP Questions Bank (Part 2) – Medical Masterclass 2010

at any stage of the disease ( quiescent phase or swelling in the upper body due to poor venous
active progression). The presentation may return. It is usual to confirm the diagnosis with
precede, occur simultaneously with, or follow a linogram before removing the line in case
the diagnosis of lymphoma. It is also more there is another cause for the swelling eg. nodal
common with the use of Fludarabine, a purine masses.
analogue recently used in such conditions.
Heparin is usually used to control the clot,
Treatment is with steroids.
however this needs to be discussed with senior
Treatment related myelodysplasia requires staff because of the relative dangers of
exclusion, but the anaemia with reticulocytosis anticoagulation with falling platelet counts.
is against this option. Some haematologists would discontinue the
heparin once the platelet count fell to 10. The
[ Q: 41 ] MasterClass Part2 patient is unfortunately left with the problem of
a clot and no easy access for
(2010) - Hematology
blood/chemotherapy.
During chemotherapy for acute myeloid
leukaemia, a 60-year-old man develops [ Q: 42 ] MasterClass Part2
headaches, fullness in the neck and swelling in
(2010) - Hematology
his right arm. He has a Hickman line on his right
side. A 72-year-old man presented to casualty with
painless bilateral testicular swelling. His blood
What TWO courses of action would you take? counts showed haemoglobin 12.5 g/dL, white
A. Check the thrombophilia screen blood cells 4.1 x 109/L and platelets 173 x 109/L.
Tumour markers human chorionic
B. Discuss with the radiologist to get some
gonadotropin (HCG) and alpha foetprotein were
imaging
normal. He had a paraprotein in his serum at a
C. Remove the Hickman line immediately low level of 3.4 g/L, no lytic lesions on
radiographs and no Bence Jones proteinuria.
D. Check clotting profile
His beta 2 microglobulin and lactate
E. Heparinise the patient immediately dehydrogenase (LDH) were both moderately
F. Look for the lupus anticoagulant elevated. Bilateral orchidectomy was carried
out. The histology is shown (see image).
G. Tilt the man head-down
H. Following discussions with supervisor,
commence heparin
I. Commence a warfarin-loading dose
J. Reassure the patient that these things
sometimes happen

Answer & Comments

Correct answer: BH

Indwelling intravenous catheters are foreign A bone marrow biopsy was normal.
bodies, which frequently cause thrombosis,
even with thrombocytopenia. This causes What is the diagnosis?

Dr. Khalid Yusuf El-Zohry – Sohag Teaching Hospital (01118391123) Page | 406
El-Zohry MRCP Questions Bank (Part 2) – Medical Masterclass 2010

A. Seminoma of the testes Which of the following statements is correct?


B. Extramedullary plasmacytoma of the A. Sideroblastic anaemia is a likely diagnosis.
testes
B. Felty’s syndrome is a likely diagnosis.
C. Acute lymphoblastic leukaemia
C. Acute blood loss is likely.
D. Gonococcal orchitis
D. Anaemia of chronic disorders is the most
E. Multiple Myeloma. likely diagnosis.
E. A normal ferritin excludes iron deficiency
Answer & Comments as a cause of her anaemia.
Correct answer: B
Answer & Comments
The image shows an extensive and abnormal Correct answer: D
infiltrate of plasma cells with eccentric 'clock
face' nuclei and a paranuclear clear area
Ferritin is an acute phase reactant, hence a
representing the Golgi zone. Binucleate forms
normal value does not exclude iron deficiency
are readily seen.
anaemia. A low ferritin would be a useful result,
The absence of lytic lesions and a normal bone strongly supporting the diagnosis of iron
marrow biopsy excludes myelomatosis and deficiency.
confirms extramedullary plasmacytoma of the
Anaemia of chronic disorders is a feature
testes. The low- level paraprotein is likely to be
ofrheumatoid arthritis, most marked in the
produced by the neoplastic cells. Seminomas
acute phase of the illness. Felty’s syndrome is
usually affect younger people and the tumour
very uncommon.
markers are likely to be elevated. The absence
of other inflammatory cells and gonococci on The differential diagnosis of her anaemia is
staining refutes the diagnosis. wide. Appropriate tests would include: B12,
folate, iron, TIBC, ferritin; inflammatory
The testes can be involved in plasma cell
markers (CRP); thyroid function; rheumatoid
tumours but the nasopharynx or head and neck
factor; immunoglobulins (?myeloma); liver,
area are far commoner. Acute lymphoblastic
renal and bone function tests; chest radiograph.
leukaemias can also involve the testes and carry
a bad prognosis. The median survival of
extramedullary plasmacytoma (EMP) ranges [ Q: 44 ] MasterClass Part2
from four to 10 years. (2010) - Hematology
A 67-year-old man on Warfarin for Atrial
[ Q: 43 ] MasterClass Part2 Fibrillation presented with a right upper arm
(2010) - Hematology DVT confirmed on venography, despite being
on Warfarin with an INR of 7.5. He denies
A 55-year-old woman presents with tiredness.
external blood loss. His right upper limb is
She had been given a diagnosis of rheumatoid
massively swollen.
arthritis at the age of 35 years, but has been fit
and well for many years, without any joint Which TWO explanations are most likely?
problems. Her full blood count is as follows: Hb
A. Lack of compliance with Warfarin
9.3 g/dl, MVC 85 fl, MCH 28 pg, white blood cell
count normal, platelet count normal. B. Stroke with monoparesis
C. Henoch scholein purpura

Dr. Khalid Yusuf El-Zohry – Sohag Teaching Hospital (01118391123) Page | 407
El-Zohry MRCP Questions Bank (Part 2) – Medical Masterclass 2010

D. Severe megaloblastic anaemia Hodgkin’s disease (HD) and non-Hodgkin’s


lymphoma can both present with isolated
E. Cardiac emboli to limb
cervical lymphadenopathy, though this is more
F. Disseminated carcinomatosis commonly seen in HD in this age group. The
G. Acute lymphoblastic leukaemia node(s) are classically painless and non-tender.
The patient is often otherwise well, but may
H. Homozygous protein C deficiency
have ‘B’ symptoms (weight loss, fevers and
I. Rethromboses with background acquired night sweats).
thombophilia
J. Aspirin therapy. [ Q: 46 ] MasterClass Part2
(2010) - Hematology
Answer & Comments
A 57-year-old lady presented with
Correct answer: FI pancytopenia, Hb 9.6g/dl, Wbc 2.0 x 10^9/l and
plts of 56 x 10^9/l. A bone marrow biopsy
Despite what looks like excessive showed widespread granulomas with no other
anticoagulation, thrombosis is rare. One should features of haemopoietic malignancy.
look for sinister causes such as a metastatic
Which of the following diseases do not cause
malignant process and other causes of acquired
bone marrow granulomas?
thrombophilia such as antiphospholipid
syndrome. Homozygous protein C deficiency is A. Sarcoidosis
not compatible with life or will have presented B. Foreign bodies
much earlier on in life. In fact, this patient was
eventually diagnosed with carcinoma of the C. Tuberculosis
pancreas. D. Hodgkin's lymphoma
E. Myelofibrosis
[ Q: 45 ] MasterClass Part2
F. Cytomegalovirus infection
(2010) - Hematology
G. Osteopetrosis
A 24-year-old woman presents to A&E with a
H. AIDS
history of a painless swelling in her neck. She is
otherwise well. I. Brucellosis
J. Ehrlichiosis.
What is the most likely diagnosis from the
following list?
Answer & Comments
A. Acute leukaemia
Correct answer: EG
B. Infectious mononucleosis
C. Non-Hodgkin’s lymphoma
Granulomas can be found in bone marrow as a
D. HIV seroconversion illness manifestation of systemic disease such as
sarcoidosis or tuberculosis, or non-specifically.
E. Hodgkin’s disease.
In many cases the cause is not found.

Answer & Comments Ehrlichia is an infection caused by a protozoa in


immunocompromised patients with
Correct answer: E characteristic inclusion bodies in macrophages.

Dr. Khalid Yusuf El-Zohry – Sohag Teaching Hospital (01118391123) Page | 408
El-Zohry MRCP Questions Bank (Part 2) – Medical Masterclass 2010

It is commonly associated with granuloma prolong the APPT and this will not be corrected
formation. Foreign bodies in the marrow can by the addition of normal plasma (unlike factor
incite granuloma formation with epitheloid cell deficiencies). The PT is not affected
and multinucleate giant cells with fibrous tissue
arranged in a concentric fashion. [ Q: 48 ] MasterClass Part2
Myelofibrosis and osteopetrosis cause marrow (2010) - Hematology
fibrosis or replacement by sclerotic bone which
leads to a leucoerythroblastic blood film. A 78-year-old man with lung cancer presents
with a large deep venous thrombosis.

[ Q: 47 ] MasterClass Part2 Which one of the following mechanisms is NOT


(2010) - Hematology recognized as contributory to the thrombotic
tendency seen in cancer patients?
A young woman is being investigated for
recurrent fetal loss. The clinical suspicion is of A. Activation of the coagulation cascade
antiphospholipid antibody syndrome. B. Hyperfibrinolysis

Which of the following DO NOT support this C. Tumor-endothelium interaction


diagnosis? D. Thrombocytopenia
A. Previous DVT E. Decreased activity of natural
B. Previous arterial thrombosis anticoagulants.

C. A prolonged APPT which corrects on


Answer & Comments
mixing with normal plasma
D. A prolonged APPT which does not correct Correct answer: D
on mixing with normal plasma
Thrombosis is a well recognised complication of
E. Thrombocytopenia
cancer patients (venous > arterial). Up to 15%
F. Trolonged PT of patients with cancer will suffer a thrombotic
G. Migraines event during the course of the illness due to
acquired and inherited risk factors.
H. Anaemia Disemmination (DIC) with bleeding/thrombosis
I. Recurrent skin ulcers can be fatal in haematological (such as acute
promyelocytic leukaemia) and non-
J. Joint pathology.
haematological cases.

Answer & Comments Mechanisms include:


 activation of coagulation pathway by
Correct answer: CF
tissue factor in the extrinsic pathway
 excessive fibrinolysis seen as raised D-
Antiphospholipid antibody syndrome (APAS)
represents a spectrum of abnormalities which is dimers
multi-system in nature. It may be idiopathic or  damage to endothelium by cytotoxics
associated with a number of underlying causes, and the tumor itself via various
including inflammatory arthritis, infarctions and adhesion molecules.
medications. Classically the presence of a lupus
anticoagulant (antiphospholipid antibody) will

Dr. Khalid Yusuf El-Zohry – Sohag Teaching Hospital (01118391123) Page | 409
El-Zohry MRCP Questions Bank (Part 2) – Medical Masterclass 2010

Treatment is by low molecular weight heparin, approximately three-fold (i.e. to about 60%)
warfarin or unfractionated heparin to the and should be given with 3 days of tranexamic
desired therapeutic ratio. acid, which acts to prevent clot break down.
Obviously avoid all NSAIDs and ask the patient
Thrombocytopenia causes excessive bleeding
to contact the dentist and the dentist to contact
and is not known to cause thrombosis. A raised
you if she bleeds
platelet count may be a reactive process in
marrow metastatic disease.
[ Q: 50 ] MasterClass Part2
[ Q: 49 ] MasterClass Part2 (2010) - Hematology
(2010) - Hematology You are given a full blood count report by the
secretary in the gastroenterology department.
You are contacted for advice by the hospital
It is from a 67-year-old woman that was seen in
dentist. He wants to extract a tooth from a 44-
last week’s clinic. It shows Hb 8.5 g/dl, MCV 122
year-old woman with type I von Willebrand’s
fl, platelets 98 x 109/l, and neutrophils 1.2 x
disease who is otherwise completely healthy.
109/l.
Her baseline results are as follows: platelets
201 x 109/l (normal), Factor VIII is 18%, RAg
Your next TWO steps should be to:
18% and RiCof 18%.
A. organise a Schilling test
What specific advice regarding haemostasis
B. phone the GP to ask him/her to check
would you recommend?
B12 and folate levels
A. No specific therapy is required.
C. get a reticulocyte count
B. Avoid non-steroidal anti-inflammatory
D. phone the consultant gastroenterologist
drugs (NSAIDs), but no other action is
required. E. check to see what the blood film showed

C. Avoid NSAIDs and give tranexamic acid 1g F. look in Medical Masterclass


tds for 3 days post extraction. G. phone the consultant haematologist
D. Avoid NSAIDs, give tranexamic acid 1g tds H. write to the patient and ask them to
for 3 days post extraction, and give commence oral folate
Desmopressin diacetate arginine
vasopressin (DDAVP) 0.3mcg/kg 1 hour I. make a note to check her Coomb's test at
pre-extraction. the next clinic

E. Avoid NSAIDs, give tranexamic acid 1g tds J. leave the problem untill the next clinic.
for 3 days post-extraction, and give 1500
units of plasma derived factor VIII one Answer & Comments
hour pre-extraction.
Correct answer: BE

Answer & Comments


Although it is important to seek help from
Correct answer: D senior colleagues and Medical Masterclass, you
should be able to make a decision on this as a
This woman is at increased risk of bleeding. medical SHO. Don't leave this sort of problem
Ideally we would like her factor VIII level to be to a later date or to somebody else - if the
above 30% at the time of dental extraction. patient is lost to follow up they could become
DDAVP will cause the factor VIII level to rise quite ill. The blood film will show if there is

Dr. Khalid Yusuf El-Zohry – Sohag Teaching Hospital (01118391123) Page | 410
El-Zohry MRCP Questions Bank (Part 2) – Medical Masterclass 2010

polychromasia (immune haemolysis) or A. Her hepatosplenomegaly indicates


features of megaloblastic anaemia. The film will alcoholic liver disease.
give you more information than just a
B. Serum haptoglobins are increased.
reticulocyte count (is this high in megaloblastic
anaemia?). The B12 and folate levels should be C. Antibody screen and identification for
checked quickly because restoring these compatible units may be helpful.
haematinics can make people feel a lot better D. The urine contains metabolites of Coca
very quickly. Cola.
E. Hepatitis serology is likely to be positive.
[ Q: 51 ] MasterClass Part2
(2010) - Hematology Answer & Comments
A 75-year-old woman presents to the accident Correct answer: C
and emergency department with the following
full blood count results (also see image):
Cold haemagluttinin disease (CHAD) is usually
idiopathic in aetiology, although a recent
history of mycoplasma infection should be
elicited and serology done for atypical
infections.
Lymphomas can cause hepatosplenomegaly.
Lymphoproliferative conditions can cause CHAD
and a bone marrow biopsy is required.
Treatment with high doses of steroids can be
instituted but transfusion using a blood warmer
+/- alkylating agents are usually preffered. The
antibody screen shows a positive Coombs' test
with anti-I/i specificity usually in idiopathic
cases. There is evidence of intravascular
haemolysis, namely:
 haemoglobinaemia
 haemoglbinuria, causing the colour
change in the urine
 low serum haptoglobins
 haemosiderinuria, if prolonged
Haemaglobin (Hb) 4.8 g/dL, white blood cells  iron deficiency ultimately can result.
(WBC) 11.0 x 109/L, platelets 206 x 109/L. She is
noticed to be jaundiced and comments that her [ Q: 52 ] MasterClass Part2
urine has been 'Coca Cola' coloured. She denies
(2010) - Hematology
any recent history of chest symptoms or
excessive alcohol intake. A 32-year-old pregnant woman attends A & E as
she has noticed bruises on her arms. Her
Which of the following statements is true? platelet count is found to be 24 x 109/l.

Dr. Khalid Yusuf El-Zohry – Sohag Teaching Hospital (01118391123) Page | 411
El-Zohry MRCP Questions Bank (Part 2) – Medical Masterclass 2010

Which of the following are NOT well-recognised


causes of thrombocytopenia in pregnancy?
A. ITP
B. TTP
C. Methydopa
D. Antepartum haemorrhage
E. DIC
F. Iron deficiency anaemia (IDA)
G. Eclampsia
H. Heparin
Which are the most important next TWO steps?
I. HELLP syndrome
A. Remove the Hickman line
J. HUS
B. Commence heparin
Answer & Comments C. Obtain three separate blood cultures
Correct answer: DF D. Speak to the cardiac surgeons
E. Perform a Coomb's test
Any cause of thrombocytopenia, which can
F. Perform urinalysis
occur outside of pregnancy, can also present at
this time. In addition there are a number of G. Commence tazocillin, gentamycin and
syndromes that are particular to pregnancy teicoplanin
(eclampsia, HELLP syndrome), which must be
H. Perform ultrasound for splenomegaly
immediately recognised and treated. Pregnancy
is a well-known association with TTP. I. Check liver function tests
Antepartum haemorrhage may result from J. Commence aspirin.
thrombocytopenia but is unlikely to cause this.
IDA is usually associated with a raised platelet
Answer & Comments
count.
Correct answer: CG
[ Q: 53 ] MasterClass Part2
(2010) - Hematology Endocarditis can be a problem with Hickman
lines - in fact, this man had a mitral valve
A 75-year-old man had high grade non- prolapse prior to therapy. It is vital to establish
Hodgkin’s lymphoma. He received the organism by blood culture. Then commence
chemotherapy through a Hickman line. On his broad spectrum antibiotics, including gram
third course he developed persisting positive cover (associated with Hickman lines) -
pancytopenia and a fever. The echocardiogram speak to the microbiologist!! All other actions
from the left ventricle is shown (see image). can wait. Anticoagulation is not indicated in the
absence of embolus. It may become necessary
to remove the Hickman line, but this may be
the only means of venous access in some
patients - a tough decision (involve the patient

Dr. Khalid Yusuf El-Zohry – Sohag Teaching Hospital (01118391123) Page | 412
El-Zohry MRCP Questions Bank (Part 2) – Medical Masterclass 2010

[ Q: 54 ] MasterClass Part2
(2010) - Hematology
A 30-year-old Asian woman is 12 weeks
pregnant. She complains of increasing
tiredness. The results of her blood count are as
follows: Hb 9.6 g/dl, MCV 63.2 fl, MCH 22.5 pg,
white blood cell count normal, platelet count
normal.

What is the most likely cause of her anaemia?


A. Lead poisoning The single most likely diagnosis is:
B. Sideroblastic anaemia A. Acute lymphoid leukaemia (ALL)
C. Iron deficiency anaemia B. Chronic lymphocytic leukaemia (CLL)
D. Thalassaemia trait C. Chronic myeloid leukaemia (CML)
E. Anaemia of chronic disease. D. Acute myeloid leukaemia (AML)
E. Myelofibrosis.
Answer & Comments

Correct answer: D Answer & Comments

Correct answer: C
Although anaemia could be multifactorial, with
a contribution from iron deficiency, the very The image shows one large chromosome 9 and
low MCV, particularly when combined with only one small chromosome 22 - this results from
modest anaemia, strongly suggests the the Philadelphia translocation between 9 and
diagnosis of thalassaemia in an Asian woman. 22. This is typical of CML.
The most likely cause would be beta The Philadelphia chromosome can occur in AML
thalassaemia minor (the heterozygote form). and ALL but these diseases usually present with
The patient may be aware of the diagnosis. thrombocytopenia rather than thrombocytosis.
Haemoglobin electrophoresis would be
expected to show HbA2 > 3.5% in most cases.
[ Q: 56 ] MasterClass Part2
(2010) - Hematology
[ Q: 55 ] MasterClass Part2
(2010) - Hematology (2) A 22-year-old woman has had heavy
periods ever since she can remember, certainly
A 45-year-old man is admitted with blurred for many years, and is now iron deficient.
vision. His WCC is 132, Hb 113 g/l, Platelets 722.
His marrow goes for karyotypic analysis (see What TWO conditions must be considered?
image).
A. Haemophilia
B. Coeliac disease
C. Disseminated intravascular coagulation
(DIC)
D. Chronic liver disease

Dr. Khalid Yusuf El-Zohry – Sohag Teaching Hospital (01118391123) Page | 413
El-Zohry MRCP Questions Bank (Part 2) – Medical Masterclass 2010

E. Idiopathic thrombocytopenic purpura fibrinogen of 0.5g/l. The blood film is shown


(ITP) (see image).
F. Hypothyroidism
G. Cushing’s syndrome
H. Von Willebrand’s disease
I. Chronic renal failure
J. Hyperthyroidism.

Answer & Comments

Correct answer: FH

Von Willebrand’s disease occurs in 1% of the


population and could present in this way.
Acquired bleeding problems such as DIC, liver
disease, ITP or uraemia are excluded by the
length of the history.
Von Willebrand’s factor (VWF) is the carrier for What is the most likely diagnosis?
factor VIII and in Von Willebrand’s disease the
A. Acute lymphoblastic leukaemia
activated prothrombin time (APTT) may be
prolonged due to a low factor VIII level. Specific B. Acute myeloid leukaemia FAb type M5
assays include those for VWF and ristocetin C. Acute myeloid leukaemia FAb type M3
cofactor.
D. Chronic myeloid leukaemia
Treatment in this case might include oral iron
(to correct iron deficiency) and the use of E. Haemophilia A.
DDAVP (which promotes release of VWF)
intranasally at the time of the menses to reduce Answer & Comments
blood loss. Operative procedures can be
Correct answer: C
covered with intravenous DDAVP.
Do not forget to take a family history: others The differential diagnosis of the blood count
may be affected (autosomal dominant). above is very broad. However, in the context of
deranged clotting tests (suggestive of a
[ Q: 57 ] MasterClass Part2 disseminated intravascular coagulation (DIC))
(2010) - Hematology AML M3 is the most likely of the options given.
In addition to chemotherapy, patients with AML
A 20-year-old man presents to casualty with a M3 should be treated aggressively with
short history of malaise. Over the last day he platelet/ clotting factor support. They are also
has noticed a few bruises on his limbs. Blood treated with ATRA (all-trans-retinoic-acid), as
tests reveals a white cell count (WCC) of 34 x the characteristic chromosomal abnormality
109/l, platelet count of 25 x 109/l, Hb of 7.8 renders them susceptible to this drug
g/dl, prothrombin time (PT) of 25 and activated
partial thromboplastin time (APPT) of 42,

Dr. Khalid Yusuf El-Zohry – Sohag Teaching Hospital (01118391123) Page | 414
El-Zohry MRCP Questions Bank (Part 2) – Medical Masterclass 2010

[ Q: 58 ] MasterClass Part2 In almost all cases of CLL the malignant


(2010) - Hematology lymphocytes are B cells. Amyloidosis is a
complication of myeloma.
A GP contacts you about one of his patients.
The following is the result of the patient's full
[ Q: 59 ] MasterClass Part2
blood count: Haemoglobin 10.2 g/dl, white cell
count 115x10^9/l, platelet count 112 x 10^9/l,
(2010) - Hematology
neutrophils 1.9x10^9/l, lymphocytes A medical registrar phones for your advice on a
112x10^9/l. Furthermore, a diagnosis of chronic 35-year-old woman who has been transferred
lymphocytic leukaemia (CLL) has been correctly to his care from another hospital. Her full blood
made by the laboratory. count has shown a raised MCV. A bone marrow
aspirate performed at the previous hospital has
Which of the following statements regarding
been reported to show no megaloblastic
CLL are incorrect?
changes.
A. CLL is a disease of the middle aged and
elderly. Which are the TWO LEAST likely underlying
causes of the macrocytosis?
B. Physical examination may be normal in
the early stages of the disease. A. Liver disease

C. Chlorambucil may be used as first line B. Pregnancy


therapy. C. B12 deficiency
D. Advanced disease may be associated with D. Hypothyroidism
hepatosplenomegaly.
E. Chronic obstructive airways disease
E. Smear cells are usually seen on the blood
film. F. Folate deficiency

F. Immunophenotyping is usually G. Scurvy


undertaken to confirm the diagnosis. H. Haemorrhage
G. There is an association with autoimmune I. Haemolysis
haemolytic anaemia.
J. Atypical pneumonia.
H. There is an associated increased
susceptibility to infection. Answer & Comments
I. In the majority of cases of CLL the
Correct answer: CF
malignant lymphocytes arise from the T
cell lineage.
The causes of macrocytosis can be divided into
J. 10% of patients with CLL develop those with a megaloblastic bone marrow and
amyloidosis during the course of their those with a normoblastic bone marrow.
illness. Megaloblastic changes are generally related to
vitamin B12 or folate deficiency (and
Answer & Comments syndromes causing this) or to drugs which
dierctly damage the DNA. A variety of medical
Correct answer: IJ
conditions result in a raised MCV with a normal
bone marrow.

Dr. Khalid Yusuf El-Zohry – Sohag Teaching Hospital (01118391123) Page | 415
El-Zohry MRCP Questions Bank (Part 2) – Medical Masterclass 2010

[ Q: 60 ] MasterClass Part2 C. Bisphosphonate therapy has no place in


(2010) - Hematology preventing bone disease.
D. Amyloid is a common complication.
A 56-year-old woman has a family history of
haemochromatosis and is homozygous for the E. Bence-Jones protein occurs without
C282Y mutation. Her ferritin is 927 mg/L serum bands in 15% of myeloma.
(normal range 15-150), haemoglobin 12.5 g/dL F. Immuneparesis is a typical.
(normal range 11.5-16) and aspartate
aminotransferase 87 U/L (normal range <40). G. Coombs' positive anaemia is typical.
H. An IgM monoclonal band is seen in many
Which is the best course of action? myeloma cases.
A. Weekly venesection until she is anaemic I. Lymphadenopathy is frequently seen.
B. Intravenous desferrioxamine J. Plasmapheresis should be considered for
C. Weekly venesection to drop her ferritin hypercalcaemia.
into the low-normal range
D. Watchful waiting, with 6 monthly checks Answer & Comments
of the ferritin level Correct answer: EF
E. Start aspirin
Bone pain is the commonest presentation.
Answer & Comments The origin and specificity of the monoclone is
Correct answer: C very difficult to determine.
Amyloid is not common.
Venesection, which may need to be performed The presence of monoclonal immunoglobulins
every 2-4 weeks, should be used to drop the suppresses normal immune function – some
ferritin level into the low-normal range. Do not patients require infusions of normal
be too vigorous and make the patient anaemic. immunoglobulin.
First-degree relatives are usually offered IgM bands are seen in Waldenström’s
testing. macroglobulinaemia, not myeloma.
Plasmapheresis is considered when the plasma
[ Q: 61 ] MasterClass Part2 viscosity threatens neurological or cardiac
(2010) - Hematology function; hypercalcaemia is treated with fluid
A 67-year-old man presents with heart and and bisphosphonate infusion.
renal failure. T he marrow is infiltrated with
plasma cells and confirms the diagnosis of [ Q: 62 ] MasterClass Part2
myeloma. (2010) - Hematology
Which of the following possibilities are features You are phoned by a local dentist. A 17-year-old
of this condition? girl has experienced prolonged bleeding
following a tooth extraction. On attending the
A. Renal failure is the commonest
Emergency Department she is found to have
presentation.
normal full blood count and clotting screen.
B. The antigen specificity of the monoclonal
band is easily established.

Dr. Khalid Yusuf El-Zohry – Sohag Teaching Hospital (01118391123) Page | 416
El-Zohry MRCP Questions Bank (Part 2) – Medical Masterclass 2010

Which of the following is the most likely The good prognostic features in childhood ALL
diagnosis? are:
A. Von Willebrand's disease  age between one and 10
B. Haemophilia B  female gender
C. Platelet storage pool disorder  low white cell count (below 50 x 109/l)
D. Haemophilia A  no evidence of central nervous system
E. Antiphospholipid syndrome. disease or particular chromosomal
abnormalities

Answer & Comments  complete response to early


chemotherapy.
Correct answer: A
Haemoglobin and platelet counts have not been
shown to significantly influence prognosis.
Von Willebrand's disease is by far the most
likely diagnosis. It often presents with mucosal
bleeding and affects about 1% of the [ Q: 64 ] MasterClass Part2
population. Haemophilia A and B are both X- (2010) - Hematology
linked diseases. Although patients with platelet
A 24-year-old woman presents to the
function defects may present in a similar way
Emergency Department with a history of a
(and have a normal platelt count), these are
painless swelling in her neck. She is otherwise
much rarer in the population. Antiphospholipid
well.
syndrome is associated with an increased
tendency to clot, not bleed. What is the most likely diagnosis?
A. Acute leukaemia
[ Q: 63 ] MasterClass Part2
(2010) - Hematology B. Infectious mononucleosis
C. Non-Hodgkin's lymphoma
A 16-year-old boy has been diagnosed with
acute lymphoblastic leukaemia (ALL). His white D. HIV seroconversion illness
cell count is 10 x 109/l, platelet count 100 x E. Hodgkin's disease
109/l and Hb 14 g/dl at diagnosis.

Which of these factors is associated with a Answer & Comments


better outcome? Correct answer: E
A. His age
B. His gender Hodgkin's disease and non-Hodgkin's
lymphoma can both present with isolated
C. His haemoglobin cervical lymphadenopathy, but this is more
D. His white cell count commonly seen in Hodgkin's disease in this age
group. The node(s) are classically painless and
E. His platelet count.
non-tender. The patient is often otherwise well,
but may have ‘B' symptoms (weight loss, fevers
Answer & Comments and night sweats).
Correct answer: D

Dr. Khalid Yusuf El-Zohry – Sohag Teaching Hospital (01118391123) Page | 417
El-Zohry MRCP Questions Bank (Part 2) – Medical Masterclass 2010

[ Q: 65 ] MasterClass Part2
(2010) - Hematology
A 45-year-old woman presents with tiredness
and weight loss. She has 4cm splenomegaly.
Her white cell count is 145.

Which of the following are typical features of


chronic myeloid leukaemia (CML)?
A. Thrombocytopenia
B. Lymphadenopathy
C. Eosinophilia Which of the following statements is NOT
correct?
D. Translocation between chromosome 9
and 22 A. A direct Coomb's test may be positive for
complement C3d.
E. Skeletal fracture
B. Cold agglutination in superficial vessels
F. Raised plasma viscosity which impede capillary blood flow are
G. Raised leucocyte alkaline phophatase responsible for the skin changes.
H. Low leucocyte alkaline phophatase C. A history of previous respiratory illness
may be significant.
I. Anaemia
D. Folic acid is of no benefit in this
J. Abdominal pain.
condition.

Answer & Comments E. The red cells show autoagglutination,


polychromasia and spherocytosis on
Correct answer: DH microscopy.

Thrombocytosis and basophilia are often seen Answer & Comments


in CML. Total blood viscosity is often raised, but
plasma viscosity is normal. Abdominal pain may Correct answer: D
be associated with splenomegaly, but this is not
typical. This is a classic presentation of cold agglutinin
disease which can be precipitated by
Mycoplasma, Ebstein Barr virus infection or
[ Q: 66 ] MasterClass Part2
associated with lymphomas. the antibody is
(2010) - Hematology usually IgM and can bind complement causing
A 67-year-old Caucasian male presents with haemolysis exacerbated by cold weather. The
skin ulcers, blue hands and feet and a ‘dusky autoagglutination in peripheral circulation
purple’ nose. He finds that he gets more tired in causes the skin changes of ulceration,
winter times. FBC shows an Hb 7.8g/dl, Wbc 11 acrocyanosis. Folic acid is required for the
x10^9/l, plts 489 x 10^9/l , MCV 105fl. increased haemolysis.

Dr. Khalid Yusuf El-Zohry – Sohag Teaching Hospital (01118391123) Page | 418
El-Zohry MRCP Questions Bank (Part 2) – Medical Masterclass 2010

[ Q: 67 ] MasterClass Part2 advanced disease), but Howell-Jolly bodies are


(2010) - Hematology not seen.

A young man has a full blood count performed.


[ Q: 68 ] MasterClass Part2
Although the blood counts are normal, the
blood film has a number of abnormalities (see (2010) - Hematology
image). The use of anti-D immunoglobulin has all but
wiped out cases of haemolytic disease of the
newborn in this country. Not all women need to
be given anti-D and it should not be given to
women who do not need it.

Which of the following women would require


further testing and anti-D following delivery?
For the scenarios below Father =father’s blood
group, Mother =mother’s blood group, Baby =
baby’s blood group.
A. Mother = A RhD -ve, Father = O RhD-ve,
Baby = O RhD -ve
What abnormality/ disease would NOT cause
these appearances? B. Mother = AB RhD+ve, Father = B RhD-ve,
Baby = O RhD -ve
A. Inflammatory bowel disease
C. Mother = B RhD-ve, Father = B RhD+ve,
B. Sickle cell disease
Baby = O RhD +ve
C. Iron deficiency
D. Mother = O RhD+ve, Father = B RhD-ve,
D. Coeliac disease Baby = O RhD +ve
E. Lymphoma E. Mother = O Rh -ve, Father = AB RhD+ve,
F. Previous splenectomy Baby = B RhD -ve

G. Liver disease F. Mother = B RhD+ve, Father = B RhD-ve,


Baby = AB RhD +ve
H. Sarcoidosis
G. Mother = AB RhD+ve, Father = AB RhD-
I. Amyloidosis ve, Baby = ARhD +ve
J. Essential thrombocythaemia. H. Mother = O RhD -ve, Father = B RhD-ve,
Baby = O RhD -ve
Answer & Comments
I. Mother = O RhD-ve, Father = B RhD+ve,
Correct answer: CG Baby = ORhD +ve
J. Mother = O RhD+ve, Father = O RhD +ve,
The prominent abnormalities in the blood film Baby = ORhD -ve.
are acanthocytes, Howell-Jolly bodies and
target cells. This is the classic appearance in a Answer & Comments
patient with hyposplenism (due to infiltration,
infarction or atrophy). Target cells may be Correct answer: CI
present in liver disease (as may acanthocytes in

Dr. Khalid Yusuf El-Zohry – Sohag Teaching Hospital (01118391123) Page | 419
El-Zohry MRCP Questions Bank (Part 2) – Medical Masterclass 2010

The father’s blood group is not relevant Which of the following are most likely to explain
(anyway, you should never assume paternity!). this finding?
The ABO blood group is not relevant. Anti D is A. Infectious mononucleosis
used when the mother is RhD-ve and the child
RhD+ve. B. HIV infection
C. Lymphoma
[ Q: 69 ] MasterClass Part2 D. Tuberculosis
(2010) - Hematology
E. malaria
You are the Haematology SHO. An adult with F. Acute leukaemia
acute lymphoblastic leukaemia requires
intrathecal methotrexate and your SpR asks you G. Food poisoning
to perform the lumbar puncture at 2pm and he H. Bacterial pneumonia
will come and administer the drug. You perform
I. Influenza
the LP at 2pm but the SpR does not appear.
J. Exercise.
What should you do?
A. Proceed with the methotrexate Answer & Comments
intrathecally
Correct answer: BE
B. Ask the nurse to give the methotrexate
C. Abandon the procedure Both malaria and HIV (and AIDS) will classically
present with a low lymphocyte count. All the
D. Phone the consultant for help
other options would tend rather to be
E. Send CSF for cytology and wait associated with a lymphocytosis

Answer & Comments [ Q: 71 ] MasterClass Part2


Correct answer: C (2010) - Hematology
A 71-year-old man is treated with fludarabine
First of all, you should not get into this situation for Waldenström's macroglobulinaemia. His
by yourself. There are VERY strict guidelines IgM band falls from 36 g/L to a plateau of 5 g/L
about intrathecal therapy and it should only be and he is stable for 5 years before suddenly
given by doctors on the intrathecal register. presenting with haemoglobin (Hb) 6.7 g/dL
Occasionally, SHOs are trained and certified to (normal range 12-16.5), platelets 79 x 109/L
do this. As SHO you can place the LP needle, but (normal range 150-400), creatinine 130
cannot administer the drug. You cannot wait micromol/L (normal range 70-110) and plasma
indefinitely with the needle in the CSF, so viscosity 2.5 mPa/s (normal range 1.5-1.75).
abandon the procedure and discuss the During blood transfusion he develops a
incident with the boss and the SpR tachycardia and chest pain, and is transferred
to the Coronary Care Unit. The next day results
[ Q: 70 ] MasterClass Part2 show Hb 7 g/dL, a positive Coombs' test and
(2010) - Hematology creatinine 377 micromol/L.

A young man presents to A&E with a history of What is the most likely diagnosis?
a febrile illness. FBC reveals lymphopenia. A. Renal failure due to hyperviscosity

Dr. Khalid Yusuf El-Zohry – Sohag Teaching Hospital (01118391123) Page | 420
El-Zohry MRCP Questions Bank (Part 2) – Medical Masterclass 2010

B. Pulmonary embolism with circulatory D. Stop warfarin and recheck her INR in 3
failure days
C. Fludarabine-induced haemolysis E. Admit her overnight for observation
D. Relapse of Waldenström's disease with
haemolytic transfusion reaction Answer & Comments

E. Transformation to acute leukaemia Correct answer: D

Answer & Comments The British Committee for Standards in


Haematology has published guidelines for the
Correct answer: D
reversal of warfarin effect. A patient who is not
bleeding, who has no additional risk factors for
Most patients with Waldenström's bleeding (eg older age, peptic ulceration) and
macroglobulinaemia or myeloma relapse, with has an INR of less than 8 may stop their
a rise in their paraprotein. When measured, this warfarin and wait for the level to come down
man's IgM had risen to 29 g/L and his anaemia without intervention.
was due to active disease. He developed chest
pain due to an acute haemolytic transfusion
[ Q: 73 ] MasterClass Part2
reaction that caused the Coombs' test to
become positive. This was due to a Jka red cell (2010) - Hematology
antibody that had not previously been A 78-year-old lady presents with a 3-month
detected.The plasma viscosity is only slightly history of worsening back pain. Her FBC shows
raised and is unlikely to produce renal failure at rouleaux with a Hb of 10.2g/dl, and she has
this level. Fludarabine can cause Coombs'- moderately impaired renal function. You
positive haemolysis, but not usually 5 years suspect myeloma but dipstick testing of her
after treatment. There is no evidence for acute urine is negative for protein and serum
myeloid leukaemia. electrophoresis fails to demonstrate a
monoclonal band.
[ Q: 72 ] MasterClass Part2
Which one of the following statements is true?
(2010) - Hematology
A. She does not have myeloma.
A 26-year-old woman attends the
anticoagulation clinic for monitoring of her B. Polymyalgia rheumatica is the most likely
warfarin, which she is taking because of diagnosis.
recurrent pulmonary embolism. She is quite C. She may have myeloma that produces
well with no evidence of bleeding or increased only free light chains.
bruising. She is on no other medication.
D. Bone pain is an unusual presentation of
Unexpectedly her INR is 7.5.
myeloma.
What is the correct course of action? E. Negative protein on dipstick testing of
A. Administer 10 mg of vitamin K urine rules out the presence of Bence-
intravenously Jones protein.

B. Administer 10 mg of vitamin K orally


Answer & Comments
C. Admit her for fresh frozen plasma
infusion Correct answer: C

Dr. Khalid Yusuf El-Zohry – Sohag Teaching Hospital (01118391123) Page | 421
El-Zohry MRCP Questions Bank (Part 2) – Medical Masterclass 2010

Bone pain is a common presenting feature of platelets 455 x 109/L is seen in the accident and
multiple myeloma. In 80% of patients with emergency department following a moderate
myeloma there is a paraprotein in the serum, gastrointestinal bleed. His blood presure is
usually of the IgG or IgA class. However in 20% 110/60 and he is tachycardic. He is on no
of patients, free light chains only are produced medication.
(Bence-Jones only myeloma). Dipstick testing of
urine is predominantly sensitive to albumin. Which of the following is the most appropriate
blood product to use?

[ Q: 74 ] MasterClass Part2 A. plasma protein fraction


(2010) - Hematology B. red cells in SAG-M

A 42-year-old man received blood transfusion in C. fresh frozen plasma (FFP)


London in 1988 following a serious road traffic D. colloids and crystalloids
accident. On recovery, he enlisted as a donor in
a selfless wish to show his appreciation for his E. iron dextran infusion
life.
Answer & Comments
Unfortunately following a new directive, his
name was struck off the register as a Correct answer: B
precautionary measure to prevent transmission
of which rare, blood-borne agent? The correct blood product in this case is red
cells in optimal additive solution, which can
A. HIV 1
increase the shelf life to 35 days. The patient is
B. Hepatitis B relatively haemodynamically stable. There is no
C. new variant Creutzfeld Jakob Disease (nV role for infusion of fresh frozen plasma unless a
CJD) coagulopathy is demonstrable. Iron infusion is
not recommended due to possiblity of
D. Syphilis anaphylaxis and his haemaglobin rise will not
E. Cytomegalovirus. be immediate. Albumin is also not required
here, and is neither cost effective nor risk free
Answer & Comments of blood borne infections.

Correct answer: C
[ Q: 76 ] MasterClass Part2
(2010) - Hematology
Following the demonstration of human to
human transmission of nVCJD by blood means, A 43-year-old man presented to the accident
the British Government in 2004 has banned and emergency department with swelling of the
blood donation from recipients of blood right leg. A venogram confirmed a thrombus in
transfusion from the year 1980. The other the deep veins. He had no previous history of
agents listed are transmitted by blood but not venous thromboembolism. A day later he
affected by the recent decree. develops right sided pleuritic chest pain. His
thrombophilia screen is negative.
[ Q: 75 ] MasterClass Part2
What is the correct therapeutic option?
(2010) - Hematology
A. Warfarin therapy for six months at a
A 47-year-old man with a haemoglobin (Hb) of target international normalized ratio
5.9 g/dL, white blood cells (WBC) 10 x 109/L and (INR) of 3.5 (range 3-4.5).

Dr. Khalid Yusuf El-Zohry – Sohag Teaching Hospital (01118391123) Page | 422
El-Zohry MRCP Questions Bank (Part 2) – Medical Masterclass 2010

B. Warfarin therapy for three months at a D. Salmonella typhimurium


target INR of 2.5 (range 2-3).
E. Candida glabrata
C. Warfarin therapy for six months at a
F. Capnocytophaga canimorsus
target INR of 2.5 (range 2-3).
G. Polio virus
D. No therapy is required after initial five
days heparin therapy. H. Coagulase-negative staphyloccoci

E. Lifelong anticoagulation with a target INR I. Shigella


of 2.5 (range 2-3). J. Listeria monocytogenes.

Answer & Comments Answer & Comments


Correct answer: C Correct answer: AF

Pulmonary embolism (PE) and deep vein People with an absent or dysfunctional spleen
thrombosis (DVT) occuring so close to each are prone to infections from encapsulated
other can assumed to be related. The initial organisms such as Streptococcus pneumonie,
management of this patient with one episode Haemophilus influenzae and Neisseria
of venous thromboembolism is six months meningitides. They are also prone to exotic
anticoagulation. This should follow initial bugs like Capnocytophaga canimorsus, a gram-
heparinisation with low molecular weight negative bacillus which is present in the mouth
(LMW) heparin, or five days unfractionated of up to 40% of dogs. This can cause purpura
heparin with an adjusted activated partial fulminans and can be rapidly fatal. Protozoal
thromboplastin time (APTT) ratio of 1.5-2.5. The illness like Falciparum malaria and babesiosis
management of a patient with heritable are also a real risk. The risk, estimated at
thrombophilia (i.e. protein C or S deficiency, or 0.42/year following splenectomy, is lifelong and
factor V Leiden mutation) and a first acute influenced by the indication for splenectomy.
thrombotic episode is similar. The target INR The risk can be reduced by a stringent
should be 2.5 (range 2-3). Three months vaccination strategy, prophylactic antibiotics
antiocoagulation may be sufficient for when the and health education. Travel advice and
thrombus is restricted to the calf veins alone. wearing a Medic Alert bracelet or a credit card
alert card may be potentially life saving.
[ Q: 77 ] MasterClass Part2
(2010) - Hematology [ Q: 78 ] MasterClass Part2
A 36-year-old woman is admitted with a (2010) - Hematology
temperature of 39.6ºC having been playfully A 56-year-old lady presented to A&E feeling
bitten by her pet dog. The patient had her ‘unwell’ and with headache. An MRI scan shows
spleen removed at the age of 11 following a bilateral middle cerebral artery thrombosis and
road traffic accident. bilateral homonymous hemianopia. The counts
are Hb 11g/dl, white blood count 15 x 109/l,
Which of the following organisms is she
platelets 540 x 109/l. The clotting was normal.
particularly susceptible to?
The renal function is deranged. The blood film
A. Streptococcus pneumonie shows fragments and reticulocytes.
B. Cryptococcus neoformans
C. Serious acute respiratory (SARS) virus
Dr. Khalid Yusuf El-Zohry – Sohag Teaching Hospital (01118391123) Page | 423
El-Zohry MRCP Questions Bank (Part 2) – Medical Masterclass 2010

The following are not recognized causes of 96% in room air. He was normotensive with BP
intravascular haemolysis? 110/65.
A. Mismatched blood transfusion
B. Sickle cell anaemia
C. Paroxysmal nocturnal haemoglobinuria
D. March haemoglobinuria
E. Micro-angiopathic haemolytic anaemia
F. Drugs
G. Disemminated intravascular coagulation
H. Blackwater fever
I. Hereditary elliptocytosis
J. Gram negative septicaemia.
What is the most likely cause of his
polycythaemia (see image)?
Answer & Comments
A. Polycythaemia Rubra Vera
Correct answer: BI
B. Secondary polycythaemia due to hypoxic
states
Intravascular haemolysis liberates free
haemoglobin into plasma which binds to C. Secondary polycythaemia due to excess
plasma haptoglobins. The complex is cleared androgen use
and free haemoglobin gets filtered by kidneys
D. Polycythemia due to renal secretion of
and reabsorbed by proximal tubular epithelium. erythropoietin (EPO) from cysts
This is shed with the iron into urine giving
haemoglobinuria, haemosiderinuria. The causes E. Stress polycythemia.
of intravascular haemolysis are usually acquired
and the list is exhaustive but should be learnt. Answer & Comments
From the above list, only sickle cell and Correct answer: C
hereditary elliptocytosis cause extravascular
haemolysis. Erythropoiesis is a carefully regulated process
driven by EPO from the kidneys. EPO is
[ Q: 79 ] MasterClass Part2 produced in response to hypoxia and some
(2010) - Hematology tumours may secrete inappropriate EPO levels.
Androgens and thyroid hormone are other
A 30-year-old gym enthusiast came to clinic. He humoral agents that can affect erythropoiesis
had been referred with the following blood either by causing EPO release or optimising the
count:- Hb 19.1g/dl, Wbc 8x10^9/l and platelets effect on erythroid progenitors. They are
of 379 ^109/l. He had been on nutritional high required in small doses to mediate their effect.
protein supplements of ‘Nutrical’ and also used
4-weekly periodic courses of Deca-Durabolin, The most likely cause of this man's secondary
an androgen. He smoked very occasionally and polycythaemia is excessive androgen use. This is
his oxygen saturation by pulse oximetry was a rarer cause of polycythaemia but should be
looked for. Stress polycythaemia is apparent
polycythaemia seen in young/middle aged

Dr. Khalid Yusuf El-Zohry – Sohag Teaching Hospital (01118391123) Page | 424
El-Zohry MRCP Questions Bank (Part 2) – Medical Masterclass 2010

overweight individuals with hectic lives. They [ Q: 81 ] MasterClass Part2


may smoke and lifestyle modification may help. (2010) - Hematology
A 26-year-old woman is referred to you by her
[ Q: 80 ] MasterClass Part2
GP after a routine INR was found to be high at
(2010) - Hematology 8.4. She is on warfarin for recurrent PEs.
A 69-year-old woman has seropositive Although she is not currently bleeding, she has
rheumatoid arthritis with very active previously suffered very heavy PV bleeds (once
arthropathy. She is on weekly methotrexate requiring a transfusion) and is due to begin
with folic acid. Routine counts by the nurse menstruating in the next few days.
shows her Hb to be 10.6g/dl with an elevated
What is the correct management?
serum ferritin at 300ug/l.
A. Administer 2.5 mg of Vitamin K.
Which of the following mechanisms are NOT
B. Administer 5 mg of Vitamin K.
causally implicated in her anaemia?
C. Stop the warfarin and observe, only
A. Chemotherapy from disease modifying
giving FFP if she begins bleeding.
drugs
D. Stop the warfarin and observe, only
B. Coombs positive Intravascular haemolysis
giving Vitamin K if she begins bleeding.
C. Blunted erythropoietin response by red
E. Stop the warfarin and observe, only giving
cell precursors in bone marrow
prothrombin complex concentrate if she
D. Reticuloendothelial iron blockade in begins bleeding.
macrophages
E. Abnormal cytokine profile including TNF- Answer & Comments
alpha
Correct answer: A

Answer & Comments


Even though this patient is young, she has an
Correct answer: B additional risk factor for bleeding and should
therefore be given Vitamin K even though this
Anaemia of chronic disease is common in complication is not currently present.
elderly people and relatively unappreciated.
Many mechanisms underlying the anaemia [ Q: 82 ] MasterClass Part2
have been postulated. Inhibitory cytokines such (2010) - Hematology
as TNF-alpha may play a major role in limiting
haemopoiesis and may blunt erythroid You are contacted regarding a 62-year-old
response to erythropoietin. Anaemia is typically woman who has a platelet count of 620x109/l
mild and normocytic and therapy includes (normal range 150x109/l - 400x109/l).
better disease control with anti TNF agents
such as Infliximab, or disease modifying drugs, Which of the following would not account for
which sometimes may even cause anaemia. this finding?
A. Osteomyelitis
B. Polycythemia rubra vera
C. Thrombotic thrombocytopenia purupra
D. 5 years post splenectomy

Dr. Khalid Yusuf El-Zohry – Sohag Teaching Hospital (01118391123) Page | 425
El-Zohry MRCP Questions Bank (Part 2) – Medical Masterclass 2010

E. 5 days post hip replacement Answer & Comments


F. Chronic myeloid leukaemia Correct answer: D
G. Acute lymphoblastic leukaemia
The British Committee for Standards in
H. Rheumatoid arthritis
Haematology (BCSH) has published guidelines
I. Essential thrombocythemia for reversal of warfarin effect. A patient who is
J. Mycoplasma pneumonia. not bleeding and who has no additional risk
factors for bleeding (e.g. older age, peptic
ulceration etc), with an INR less than 8 may
Answer & Comments
stop their warfarin and wait for the level to
Correct answer: CG come down without intervention.

Inflammatory and infectious processes can [ Q: 84 ] MasterClass Part2


cause thrombocytosis, as can recent trauma or (2010) - Hematology
surgery. Platelets may be persistently raised
following splenectomy. The myeloproliferative A 28-year-old man has a splenectomy after a
disorders (polycythemia rubra vera and road traffic accident.
essential thrombocythemia) are associated with
a raised platelet count, as is chronic myeloid Which one of the treatment strategies listed is
leukaemia. Acute lymphoblastic leukaemia NOT required?
would not cause a raised platelet count and A. Meningococcal vaccination
frequently (but not always) would be associated
B. Haemophilus influenzae vaccination
with a low platelet count. As the name
suggests, in thrombotic thrombocytopenic C. Prophylactic antibiotics with Penicillin V/
purpura, the platelet count is low. Erythromycin
D. Polyvalent pneumococcal vaccination
[ Q: 83 ] MasterClass Part2 E. Hepatitis B vaccination.
(2010) - Hematology
A 26-year-old woman attends the warfarin Answer & Comments
clinic that you are running. She is quite well Correct answer: E
with no evidence of bleeding or increased
bruising. She is on no other medication.
Patients splenectomised are prone to
Unexpectedly her INR is 7.5.
Overwhelming Post Splenectomy Infection
What is the correct course of action? (OPSI) that involves encapsulated organisms
including pneumococcus, meningococcus and
A. Administer 10 mg of Vitamin K Haemophilus influenzae. Individuals are also
intravenously prone to severe falciparum malarial infection
B. Administer 10 mg of Vitamin K orally and Capnocytophagia canimorsus bacterium.
The spleen is necessary for:
C. Admit for fresh frozen plasma (FFP)
infusion  filtering
D. Stop warfarin and recheck her INR in 3  phagocytosis
days
E. Admit overnight for observation.

Dr. Khalid Yusuf El-Zohry – Sohag Teaching Hospital (01118391123) Page | 426
El-Zohry MRCP Questions Bank (Part 2) – Medical Masterclass 2010

 production of antibodies required for Serologica testing for allergens is not a


opsonisation of these bacteria and mandatory test nor is it required at first
protozoa. contact.
Lifelong penicillin is recommended but AML M4Eo due to translocation t(16;16) is
compliance is very poor. Therefore, some uncommon but responds readily to all trans
physicians recommend 3 g amoxicillin to be retinoic Acid (ATRA).
taken at onset of febrile illness. In penicillin
sensitive individuals, erythromycin is desirable. [ Q: 86 ] MasterClass Part2
(2010) - Hematology
[ Q: 85 ] MasterClass Part2
This 63-year-old man had a short history of
(2010) - Hematology
heart failure and was commenced on lisinopril
You are asked to examine a 17-year-old girl and frusemide by his GP. He had a fatal cardiac
from Mauritius. Her family has a history of arrest three weeks later. The post mortem
atopy but showed a restrictive cardiomyopathy. The first
image shows cardiac histology stained with
she had recently returned from Africa on a
'haematoxylin and eosin', the second stained
voluntary mission in the church. She was noted
with a specialized stain.
to have a mild eosinophilia at 0.8 x 10^9/l.
Examination was entirely unremarkable.

Which of these statements is the most likely?


A. The commonest cause of eosinophilia
worldwide is hookworm infestation.
B. She has idiopathic hypereosinophilic
syndrome.
C. Clonal disorders of the bone marrow can
be excluded with certainty by a bone
marrow biopsy.
D. Serologic testing for allergens is a
mandatory initial investigation.
E. Acute myeloid leukaemia (M4Eo) is
common.

Answer & Comments

Correct answer: A

The commonest cause of eosinophilia


worldwide is parasitic infestation, of which Which of the following is the most likely
hookworm is the most prevalent. Clonal diagnosis?
disorders of the bone marrow account for 1% of
causes of eosinophilia and cannot be excluded A. Hypertension
with certainty by a marrow biopsy. B. Hypertrophic obstructive cardiomyopathy
C. Ischaemic heart disease

Dr. Khalid Yusuf El-Zohry – Sohag Teaching Hospital (01118391123) Page | 427
El-Zohry MRCP Questions Bank (Part 2) – Medical Masterclass 2010

D. Amyloidosis What is the diagnosis of our patient in lane 5?


E. Haemochromatosis. A. Hb SS - homozygous sickler, avoid flying
B. Hb AA - normal pattern
Answer & Comments
C. Hb AF - persistant Hb F, no problem flying
Correct answer: D
D. Hb AS - sickle heterozygote, safe for
flying (keep well hydrated)
The first image shows an amorphous
E. Hb FF - thalassaemia major, should avoid
eosinophilic substance within the specimen.
flying if very anaemic.
The second image shows the classical apple
green birefringence seen when the Congo red
stain is viewed with polarised light. This is Answer & Comments
diagnostic of amyloid. Note the staining in the
Correct answer: D
vessels as well as in the cardiac tissue. The man
had no evidence of myeloma or lymphoma, but
Lane 5 shows two hb bands. One runs on the
this was found to be an AL type of amyloid.
Hb A line and one on the Hb S line. The patient
The cardiac arrest was probably due to a therefore is a carrier for sickle cell but makes
dysrhythmia associated with amyloid rather enough Hb A to keep him from sickling. Flying is
than hypokalaemia. Coronary vessels were not a problem in this situation usually, unless
healthy for his age. the cabin becomes depressurized or
dehydration occurs
[ Q: 87 ] MasterClass Part2
(2010) - Hematology [ Q: 88 ] MasterClass Part2
This 32-year-old man has had a blood count and
(2010) - Hematology
haemoglobin electrophoresis (see image). He A young woman is diagnosed with
wants to take a long haul flight to New Zealand. antiphospholipid antibody syndrome, after
The Hb electrophoresis shows controls in lanes being investigated for recurrent DVTs.
3 and 6. The 4 significant bands run Hb C, S, F
and A (towards the anode). Lane 4 shows Hb A Which of the following statements regarding
only. treatment are valid?
A. She requires aspirin alone.
B. She requires long term warfarin therapy.
C. Steroids are better treatment then
heparin and aspirin.
D. Low molecular weight heparin should not
be used.
E. She should not use the oral contraceptive
pill.
F. Obesity will not increase her risks.
G. She requires long term heparin therapy.
H. In pregnancy, warfarin should be given.

Dr. Khalid Yusuf El-Zohry – Sohag Teaching Hospital (01118391123) Page | 428
El-Zohry MRCP Questions Bank (Part 2) – Medical Masterclass 2010

I. In pregnancy, aspirin is sufficient Answer & Comments


treatment.
Correct answer: BD
J. Regular plasma-exchange is indicated.
A macrocytosis, mild thrombocytopenia and
Answer & Comments neutrophilia are commonly seen in pregnancy
and are of no significance. However, always be
Correct answer: BE
alert for folate deficiency in pregnancy as this is
easily treated and has implications for the
Antiphospholipid antibody syndrome (APAS) fetus.
represents a spectrum of abnormalities which is
multi-system in nature. It may be idiopathic or
associated with a number of underlying causes, [ Q: 90 ] MasterClass Part2
including inflammatory arthritis, infections and (2010) - Hematology
medications. In view of the recurrent DVTs she
should be treated with lifelong warfarin with or In which of the following clinical scenarios
without aspirin. Recurrent fetal loss is a feature would you commonly consider giving a platelet
and pregnancy should be managed with transfusion?
heparin (low molecular weight) and aspirin. A. Idiopathic thrombocytopenia (ITP)
Aspirin alone is insufficient and warfarin is
teratogenic. Other risk factors for B. Acute myeloid leukaemia (AML)
thromboembolism should be removed. C. Thrombotic thrombocytopenia
D. Renal failure
[ Q: 89 ] MasterClass Part2
E. Myeloma
(2010) - Hematology
F. Disseminated intravascular coagulation
A 32-year-old woman is seen in the antenatal (DIC)
clinic. Her full blood count reveals: Hb 10.1 g/dl,
G. Chronic lymphoid leukaemia (CLL)
MCV 103 fl, Platelets 112, Neutrophils 9.3,
Lymphocytes 3.7, Monocytes 0.5. H. B12 deficiency
I. Warfarin overdose
Which TWO of the following are the most likely
cause for this picture? J. May–Hegglin abnormality.
A. Chronic granulocytic leukaemia
Answer & Comments
B. Folate deficiency
Correct answer: BF
C. Myelofibrosis
D. Normal haematology of pregnancy
Platelet transfusions are given for AML, before
E. Iron deficiency or after chemotherapy. It is rarely needed in
myeloma or CLL. DIC is usually treated with
F. Thrombotic thrombocytopenic purpura
blood products and platelets. The May–Hegglin
G. Pre-eclampsia abnormality is an inherited thrombocytopenia
H. HIV infection with little bleeding problems - it rarely requires
platelet infusions. Platelets should not be
I. Urinary tract infection infused in ITP as they are consumed by the
J. Acute leukaemia. patient's antibodies. Platelet transfusions are

Dr. Khalid Yusuf El-Zohry – Sohag Teaching Hospital (01118391123) Page | 429
El-Zohry MRCP Questions Bank (Part 2) – Medical Masterclass 2010

essentially contraindicated in thrombotic Answer & Comments


thrombocytopenia purpura.
Correct answer: B

[ Q: 91 ] MasterClass Part2 The clue is the raised reticulocyte count, this


(2010) - Hematology causes the raised mean corpuscular volume
A 24-year-old female student returns from a (MCV). This is due to Coomb's positive
holiday in Africa and is investigated for a fever. haemolysis - seen in 10% of CLL. Although B12,
folate and iron deficiency can occur in CLL, they
Which of the following would point away from a do not raise the retic count. A marrow aspirate
diagnosis of malaria? would not give the answer here.

A. Thrombocytopaenia
[ Q: 93 ] MasterClass Part2
B. Lymphopaenia
(2010) - Hematology
C. Anaemia
A 44-year-old man comes to the Emergency
D. Neutrophilia
Department after a nose bleed which lasted 10
E. Raised ESR mins that morning. He described having
moderately severe haematemesis the day prior
Answer & Comments to this, but thought this may have been related
to the two shots of whisky he drank before
Correct answer: D going to bed. He has noticed several bruises in
the last week, and on examination he has an
A raised neutrophil count is more in keeping extensive purpuric rash over the trunk.
with a bacterial infection. Anaemia and a raised
His FBC showed severe thrombocytopenia with
ESR are non-specific findings.
a platelet count of 2 x 10^9/l (normal 150-400).
A bone marrow is carried out by the on call
[ Q: 92 ] MasterClass Part2 haematologist. This shows normal numbers of
(2010) - Hematology megakaryocytes and a diagnosis of idiopathic
thrombocytopenic purpura is made.
A 68-year-old woman has had chronic
lymphocytic leukaemia (CLL) for 4 years without What is the most appropriate treatment for
marrow failure or any need for medication. At him?
follow up, her Hb dropped to 7.8 g/dl, MCV 122
A. Oral prednisolone 60mg
fl, platelets 211, lymphocytes 43, retics 12%.
B. Pulsed intravenous methyl prednisolone
Which test is most likely to give you the correct at a dose of 1g
diagnosis?
C. Intravenous immunoglobulin
A. B12 level
D. Platelet transfusions
B. Coomb's test
E. Intravenous thrombopoietin infusions.
C. Marrow aspirate
D. Red cell folate Answer & Comments

E. Ferritin. Correct answer: C

Dr. Khalid Yusuf El-Zohry – Sohag Teaching Hospital (01118391123) Page | 430
El-Zohry MRCP Questions Bank (Part 2) – Medical Masterclass 2010

The most appropriate treatment for this patient thrombosis. The HFE gene mutation causes
who is symptomatic from ITP is intravenous haemochromatosis and is not associated with
immunoglobulin. Oral steroids will work in 60- thrombosis. Fibrin degradation products (FDPs)
70% of cases but take about 2-3 days before an will be raised in a patient with a DVT, but are a
effect is noted. It is possible to give massive non-specific marker and will return to normal
platelet transfusions to swamp the antibody on treatment.
but the hazards of blood transfusions need to
be borne in mind. In ITP the thrombopoietin [ Q: 95 ] MasterClass Part2
levels are high and no commercially available
(2010) - Hematology
thrombopoietin is in routine use. Beware of the
side effects of high dose steroids such as pulsed A 45-year-old man, previously well, receives a
methyl prednisolone. unit of blood on a surgical ward. Shortly after
the transfusion commences you are bleeped to
[ Q: 94 ] MasterClass Part2 the ward and told that he has had a severe
reaction to the blood.
(2010) - Hematology
A young woman presents with a swollen leg Which of the following are the TWO most
and is proven to have a DVT. On questioning common causes of this scenario?
she admits to a strong family history of A. Incorrect blood unit being given (i.e. for
thrombosis. another patient)

Which TWO of the following tests are unlikely to B. Bacterial infection of blood
be helpful in determining genetic risk factors C. Reaction to HLA antibodies
she may have?
D. Circulatory overload
A. Protein C levels
E. Incorrect sample sent to blood bank
B. HFE gene studies (resulting in the incorrect blood for the
C. Protein S levels patient)

D. Antithrombin III levels F. Allergic reaction to white cells

E. Prothrombin gene studies G. Graft versus host disease

F. Homocytsteine levels H. Reaction to plasma proteins

G. Fibrin degradation products I. Immune sensitization

H. Antiphospholipid antibodies J. Allergic reaction to platelets.

I. Beta-2 glycoprotein 1 assay


Answer & Comments
J. Activated protein C resistance.
Correct answer: AE
Answer & Comments
Although all of the above listed causes can
Correct answer: BG result in a transfusion reaction (of varying
severity), it has been reported that the most
All of the above tests, except B and G, may be common reason for a severe reaction to blood
included in a ‘thrombophilia screen’ and is due to transfusion of the incorrect unit. In
identify factors which may predispose almost all cases this is due to human error,
individuals to venous and/or arterial from incorrect labelling of blood samples to

Dr. Khalid Yusuf El-Zohry – Sohag Teaching Hospital (01118391123) Page | 431
El-Zohry MRCP Questions Bank (Part 2) – Medical Masterclass 2010

failing to check the details on the unit being fibrinogen fall. Macrocytosis, eosinophilia and a
given. No addressograph labels are allowed on positive DRVVT do not occur as a result of DIC.
the sample bottle in many hospitals and the
Joint bleeding is characteristic of congenital
forms and sample bottle are signed by the
severe haemophilia.
person performing the venepuncture. This is
the latest guidance on blood safety. CRP is frequently raised as a result of the
underlying severe illness precipitating the DIC.

[ Q: 96 ] MasterClass Part2
(2010) - Hematology [ Q: 97 ] MasterClass Part2
(2010) - Hematology
A 19-year-old student is admitted with fever,
neck stiffness, photophobia and headache. He A 23-year-old man with sickle cell disease
has a Glasgow Coma Score of 13/15, a (HbSS) is seen for review in the Haematology
widespread purpuric rash and prolonged clinic. His long-term girlfriend has sickle cell
bleeding at the sites of venepuncture. A trait (HbAS) and they are considering having
presumptive diagnosis of meningococcal children. He has hyposplenism as a result of
infection is made. Laboratory investigations are recurrent sickling and infarction in his spleen.
in keeping with him having an associated
Which of the following are NOT true of
disseminated intravascular coagulation (DIC).
hyposplenism and sickle disease in this patient?
Which of the following would you expect in a A. His blood film shows Howell-Jolly bodies.
patient like this?
B. He is at increased risk of malaria infection
A. Prolonged prothrombin time if he visits a malaria endemic area.
B. Thrombocytosis C. Daily penicillin prophylaxis is
C. Raised fibrinogen recommended for him.

D. Eosinophilia D. Folate supplementation is recommended


for him.
E. Macrocytosis
E. Offspring produced by him and his
F. Bleeding into joints
current partner have a 25% chance of
G. Positive direct antiglobulin (DAG) test being born with sickle cell disease.
H. Raised C reactive protein (CRP) F. He should be vaccinated against
pneumococcus, meningococcus and
I. Normal activated partial prothrombin
haemophillus influenza B.
time (APTT)
G. Aplastic crisis may follow infection with
J. Positive dilute Russell's Viper Venom test
parvovirus.
(DRVVT).
H. Life expectancy for patients with
Answer & Comments homozygous sickle cell disease is reduced.

Correct answer: AH I. Renal impairment may occur as a result of


sickling in the kidney.

DIC occurs as a result of consumption of J. Non-steroidal anti-inflammatory drugs


components required for normal haemostasis. (NSAIDs) should not be used for analgesia
As a result PT and APTT rise, and platelets and because of the concerns regarding
impaired renal function in these patients.

Dr. Khalid Yusuf El-Zohry – Sohag Teaching Hospital (01118391123) Page | 432
El-Zohry MRCP Questions Bank (Part 2) – Medical Masterclass 2010

Answer & Comments [ Q: 99 ] MasterClass Part2


Correct answer: EJ (2010) - Hematology
A 65-year-old man is admitted with a 7-day
With his current partner there is a 50% chance history of easy widespread bruising and 24
that their children will have homozygous sickle hours of developing blood blisters in his mouth.
cell disease and 50% chance they will be He is having difficulty swallowing solids and
heterozygous sickle cell trait. Haematologists liquids. He is found to have a factor 8 inhibitor
commonly use NSAIDs for analgesia albeit and is correctly diagnosed with acquired
cognisant of the potential problems when these haemophilia. The on-call haematologist has
drugs are used in patients with impaired kidney been contacted immediately as this is a medical
function. emergency!

Which of the following statements are NOT true


[ Q: 98 ] MasterClass Part2
or are INAPPROPRIATE regarding this
(2010) - Hematology gentleman and his care?
A 76-year-old woman is referred to you by her A. Joint bleeding is a common problem in
GP who has performed a routine INR check. She acquired haemophilia.
is on warfarin for atrial fibrillation. She is quite
B. The haematologist recommends that
well with no bruising or bleeding. Unexpectedly
recombinant factor 7 treatment
her INR is 9.6.
90mcg/Kg, three doses each 2 hours
What is the correct action to take? apart should be given

A. Stop the warfarin and repeat the INR the C. The haematologist recommends that
following day. recombinant factor 7 should be given
through a central line.
B. Administer 2.5 mg of Vitamin K.
D. The haematologist recommends that
C. Administer 5 mg of Vitamin K.
Prednisolone 1mg/Kg daily should be
D. Administer 10 - 15 ml/kg of FFP. started.
E. Administer 50u/kg prothrombin complex. E. Regular frequent assessment of his
breathing and swallowing is essential.
Answer & Comments F. Consideration should be given to the
presence of a coexisting malignancy.
Correct answer: B
G. Consideration should be given to the
Guidelines produced by the British Committee presence of a coexisting vasculitis.
for Standards in Haematology recommend that H. There is no association between acquired
patients with an INR > 8 and an additional risk haemophilia and congenital haemophilia.
factor for bleeding (of which age > 70 years is
I. Mortality in this condition is in the region
one) should receive Vitamin K. 2.5 mg is
of 25%.
sufficient to reverse the anticoagulation and
not make it difficult to re-warfarinise the J. Acquired factor 8 deficiency is more
patient. common than acquired factor 9
deficiency.

Dr. Khalid Yusuf El-Zohry – Sohag Teaching Hospital (01118391123) Page | 433
El-Zohry MRCP Questions Bank (Part 2) – Medical Masterclass 2010

Answer & Comments Which of the following is the correct


management of an acute haemolytic
Correct answer: AC
transfusion reaction due to ABO blood group
incompatibility?
Joint bleeding though characteristic of
A. Ring blood bank for another correct unit
congenital haemophilia is hardly ever seen in
immediately.
acquired haemophilia. In acquired haemophilia
mucosal bleeding is a real problem. Bleeding in B. Give hydrocortisone 100mg
the mouth or around the airway constitutes an intravenously.
emergency and EXPERT help is needed C. Stop transfusion immediately.
immediately. This patient needs urgent clotting
factor treatment and option C is not D. Give 2 litres Normal saline.
unreasonable with reassessment of treatment E. Slow transfusion down.
after the third dose. More may need to be
given. Recombinant factor 7 has a short half life
Answer & Comments
(1-2 hours) and may need to be given
repeatedly. Some haematologists may Correct answer: C
recommend factor eight inhibitor bypassing
agent (FEIBA), which is a blood product. Clearly The immediate treatment of an acute
a central line is going to risk serious further haemolytic transfusion reaction due to a major
bleeding and recombinant factor 7 can be given blood group incompatibility is to STOP THE
through a venflon. It goes almost without TRANSFUSION IMMEDIATELY. This should be
saying that regular frequent assessment is followed by resuscitative measures such as
required. establishing venous access with infusion of
Acquired inhibitors (F8 more common than F9) crystalloids to maintain BP and encourage
may be associated with underlying medical diuresis. samples should be taken from patient
problems such as malignancy or vasculitis and and sent to the blood bank laboratory along
there is no association with congenital with the all the untransfused units. In extremely
haemophilia. Mortality is high. This appears to severe cases, an exchange transfusion may be
be an 'autoimmune' disorder and the mainstay necessary.
of therapeutic treatment is immune
suppression. Steroids should be prescribed in [ Q: 101 ] MasterClass Part2
the first place, then consider drugs such as (2010) - Hematology
cyclophosphamide.
A 24-year-old woman presents to A&E with a
history of a painless swelling in her neck. She is
[ Q: 100 ] MasterClass Part2
otherwise well.
(2010) - Hematology
What is the most likely diagnosis from the
A young girl with beta thalassaemia major
following list?
receives a blood transfusion. Her blood group is
A RhD+ve. The student nurse collected the first A. Acute leukaemia
unit of O RhD from the blood bank freezer. Two B. Infectious mononucleosis
minutes on putting up the unit, she feels faint
with a rapid heartbeat and complains of loin C. Non-Hodgkin’s lymphoma
pain, all features of an immediate transfusion D. HIV seroconversion illness
reaction.
E. Hodgkin’s disease.

Dr. Khalid Yusuf El-Zohry – Sohag Teaching Hospital (01118391123) Page | 434
El-Zohry MRCP Questions Bank (Part 2) – Medical Masterclass 2010

Answer & Comments Although all of the above listed causes can
result in a transfusion reaction (of varying
Correct answer: E
severity), it has been reported that the most
common reason for a severe reaction to blood
Hodgkin’s disease (HD) and non-Hodgkin’s is due to transfusion of the incorrect unit. In
lymphoma can both present with isolated almost all cases this is due to human error,
cervical lymphadenopathy, though this is more from incorrect labelling of blood samples to
commonly seen in HD in this age group. The failing to check the details on the unit being
node(s) are classically painless and non-tender. given. No addressograph labels are allowed on
The patient is often otherwise well, but may the sample bottle in many hospitals and the
have ‘B’ symptoms (weight loss, fevers and forms and sample bottle are signed by the
night sweats). person performing the venepuncture. This is
the latest guidance on blood safety.
[ Q: 102 ] MasterClass Part2
(2010) - Hematology [ Q: 103 ] MasterClass Part2
A 45-year-old man, previously well, receives a (2010) - Hematology
unit of blood on a surgical ward. Shortly after A 49-year-old man presents with malaena of a
the transfusion commences you are bleeped to week's duration. On investigation his platelet
the ward and told that he has had a severe count is found to be elevated.
reaction to the blood.
Which of the following is the most likely cause
Which of the following are the TWO most of the thrombocytosis?
common causes of this scenario?
A. Blood loss
A. Incorrect blood unit being given (i.e. for
another patient) B. Chronic myeloid leukaemia
C. Infection
B. Bacterial infection of blood
D. Essential Thrombocytopenia
C. Reaction to HLA antibodies
D. Circulatory overload E. Inflammatory bowel disease.

E. Incorrect sample sent to blood bank


Answer & Comments
(resulting in the incorrect blood for the
patient) Correct answer: A
F. Allergic reaction to white cells
Thrombocytosis is much more commonly a
G. Graft versus host disease
secondary phenomenon rather than a part of a
H. Reaction to plasma proteins primary malignant process. Blood loss results in
increased marrow activity and hence a raised
I. Immune sensitization
platelet count. Treatment is of the underlying
J. Allergic reaction to platelets. cause will serve to bring the platelet count
down gradually.
Answer & Comments

Correct answer: AE

Dr. Khalid Yusuf El-Zohry – Sohag Teaching Hospital (01118391123) Page | 435
El-Zohry MRCP Questions Bank (Part 2) – Medical Masterclass 2010

[ Q: 104 ] MasterClass Part2 rouleaux with a Hb of 10.2g/dl, and she has


(2010) - Hematology moderately impaired renal function. You
suspect myeloma but dipstick testing of her
A medical registrar phones for your advice on a urine is negative for protein and serum
35-year-old woman who has been transferred electrophoresis fails to demonstrate a
to his care from another hospital. Her full blood monoclonal band.
count has shown a raised MCV. A bone marrow
aspirate performed at the previous hospital has Which one of the following statements is true?
been reported to show no megaloblastic A. She does not have myeloma.
changes.
B. Polymyalgia rheumatica is the most likely
Which are the TWO LEAST likely underlying diagnosis.
causes of the macrocytosis? C. She may have myeloma that produces
A. Liver disease only free light chains.
B. Pregnancy D. Bone pain is an unusual presentation of
myeloma.
C. B12 deficiency
E. Negative protein on dipstick testing of
D. Hypothyroidism
urine rules out the presence of Bence-
E. Chronic obstructive airways disease Jones protein.
F. Folate deficiency
Answer & Comments
G. Scurvy
H. Haemorrhage Correct answer: C

I. Haemolysis
Bone pain is a common presenting feature of
J. Atypical pneumonia. multiple myeloma. In 80% of patients with
myeloma there is a paraprotein in the serum,
Answer & Comments usually of the IgG or IgA class. However in 20%
of patients, free light chains only are produced
Correct answer: CF (Bence-Jones only myeloma). Dipstick testing of
urine is predominantly sensitive to albumin.
The causes of macrocytosis can be divided into
those with a megaloblastic bone marrow and
[ Q: 106 ] MasterClass Part2
those with a normoblastic bone marrow.
Megaloblastic changes are generally related to
(2010) - Hematology
vitamin B12 or folate deficiency (and A 45-year-old woman presents with tiredness
syndromes causing this) or to drugs which and weight loss. She has 4cm splenomegaly.
dierctly damage the DNA. A variety of medical Her white cell count is 145.
conditions result in a raised MCV with a normal
bone marrow. Which of the following are typical features of
chronic myeloid leukaemia (CML)?
[ Q: 105 ] MasterClass Part2 A. Thrombocytopenia
(2010) - Hematology B. Lymphadenopathy
A 78-year-old lady presents with a 3-month C. Eosinophilia
history of worsening back pain. Her FBC shows

Dr. Khalid Yusuf El-Zohry – Sohag Teaching Hospital (01118391123) Page | 436
El-Zohry MRCP Questions Bank (Part 2) – Medical Masterclass 2010

D. Translocation between chromosome 9 Answer & Comments


and 22
Correct answer: A
E. Skeletal fracture
F. Raised plasma viscosity The commonest cause of eosinophilia
worldwide is parasitic infestation, of which
G. Raised leucocyte alkaline phophatase
hookworm is the most prevalent. Clonal
H. Low leucocyte alkaline phophatase disorders of the bone marrow account for 1% of
I. Anaemia causes of eosinophilia and cannot be excluded
with certainty by a marrow biopsy.
J. Abdominal pain.
Serologica testing for allergens is not a
mandatory test nor is it required at first
Answer & Comments
contact.
Correct answer: DH
AML M4Eo due to translocation t(16;16) is
uncommon but responds readily to all trans
Thrombocytosis and basophilia are often seen retinoic Acid (ATRA).
in CML. Total blood viscosity is often raised, but
plasma viscosity is normal. Abdominal pain may
[ Q: 108 ] MasterClass Part2
be associated with splenomegaly, but this is not
typical. (2010) - Hematology
A 65-year-old man is admitted with a 7-day
[ Q: 107 ] MasterClass Part2 history of easy widespread bruising and 24
(2010) - Hematology hours of developing blood blisters in his mouth.
He is having difficulty swallowing solids and
You are asked to examine a 17-year-old girl liquids. He is found to have a factor 8 inhibitor
from Mauritius. Her family has a history of and is correctly diagnosed with acquired
atopy but she had recently returned from Africa haemophilia. The on-call haematologist has
on a voluntary mission in the church. She was been contacted immediately as this is a medical
noted to have a mild eosinophilia at 0.8 x emergency!
10^9/l. Examination was entirely unremarkable.
Which of the following statements are NOT true
Which of these statements is the most likely? or are INAPPROPRIATE regarding this
A. The commonest cause of eosinophilia gentleman and his care?
worldwide is hookworm infestation. A. Joint bleeding is a common problem in
B. She has idiopathic hypereosinophilic acquired haemophilia.
syndrome. B. The haematologist recommends that
C. Clonal disorders of the bone marrow can recombinant factor 7 treatment
be excluded with certainty by a bone 90mcg/Kg, three doses each 2 hours
marrow biopsy. apart should be given

D. Serologic testing for allergens is a C. The haematologist recommends that


mandatory initial investigation. recombinant factor 7 should be given
through a central line.
E. Acute myeloid leukaemia (M4Eo) is
common. D. The haematologist recommends that
Prednisolone 1mg/Kg daily should be
started.

Dr. Khalid Yusuf El-Zohry – Sohag Teaching Hospital (01118391123) Page | 437
El-Zohry MRCP Questions Bank (Part 2) – Medical Masterclass 2010

E. Regular frequent assessment of his of therapeutic treatment is immune


breathing and swallowing is essential. suppression. Steroids should be prescribed in
the first place, then consider drugs such as
F. Consideration should be given to the
cyclophosphamide.
presence of a coexisting malignancy.
G. Consideration should be given to the
[ Q: 109 ] MasterClass Part2
presence of a coexisting vasculitis.
(2010) - Hematology
H. There is no association between acquired
haemophilia and congenital haemophilia. A 26-year-old woman attends the
anticoagulation clinic for monitoring of her
I. Mortality in this condition is in the region
warfarin, which she is taking because of
of 25%.
recurrent pulmonary embolism. She is quite
J. Acquired factor 8 deficiency is more well with no evidence of bleeding or increased
common than acquired factor 9 bruising. She is on no other medication.
deficiency. Unexpectedly her INR is 7.5.

Answer & Comments What is the correct course of action?


A. Administer 10 mg of vitamin K
Correct answer: AC
intravenously

Joint bleeding though characteristic of B. Administer 10 mg of vitamin K orally


congenital haemophilia is hardly ever seen in C. Admit her for fresh frozen plasma
acquired haemophilia. In acquired haemophilia infusion
mucosal bleeding is a real problem. Bleeding in
D. Stop warfarin and recheck her INR in 3
the mouth or around the airway constitutes an
days
emergency and EXPERT help is needed
immediately. This patient needs urgent clotting E. Admit her overnight for observation
factor treatment and option C is not
unreasonable with reassessment of treatment Answer & Comments
after the third dose. More may need to be
given. Recombinant factor 7 has a short half life Correct answer: D
(1-2 hours) and may need to be given
repeatedly. Some haematologists may The British Committee for Standards in
recommend factor eight inhibitor bypassing Haematology has published guidelines for the
agent (FEIBA), which is a blood product. Clearly reversal of warfarin effect. A patient who is not
a central line is going to risk serious further bleeding, who has no additional risk factors for
bleeding and recombinant factor 7 can be given bleeding (eg older age, peptic ulceration) and
through a venflon. It goes almost without has an INR of less than 8 may stop their
saying that regular frequent assessment is warfarin and wait for the level to come down
required. without intervention.

Acquired inhibitors (F8 more common than F9)


may be associated with underlying medical [ Q: 110 ] MasterClass Part2
problems such as malignancy or vasculitis and (2010) - Hematology
there is no association with congenital
A 70-year-old woman is admitted with anaemia
haemophilia. Mortality is high. This appears to
and fatigue. Her full blood count reveals Hb
be an 'autoimmune' disorder and the mainstay
6.7g/dl, MCV 101fl, WCC 76 x 10^9/l (90%

Dr. Khalid Yusuf El-Zohry – Sohag Teaching Hospital (01118391123) Page | 438
El-Zohry MRCP Questions Bank (Part 2) – Medical Masterclass 2010

lymphs), platelets 105 x 10^9/l, and Retics 14%. A. The patient should be commenced
She is slightly jaundiced and has splenomegaly. immediately on a therapeutic dose of low
The lab think she probably has chronic molecular weight heparin.
lymphatic leukaemia. She is on no medication.
B. Warfarin should be started and heparin
can be stopped when the INR is greater
Which are the TWO most likely causes of the
than 2.0.
anaemia?
A. B12 deficiency C. In pregnant women the majority of DVT's
occur in the right leg.
B. Bleeding
D. This patient requires 3 months
C. Folate deficiency anticoagulation.
D. Autoimmune haemolysis E. 5mg warfarin tablets are pink.
E. Acquired G6PD deficiency F. 1mg warfarin tablets are blue.
F. Iron deficiency G. She needs her APTT monitored whilst
G. Acute transformation taking warfarin to titrate dose.

H. Marrow infiltration with leukaemia H. Warfarin is contraindicated in patients


taking Penicillin.
I. Hyperviscosity
I. Low molecular weight heparin is given
J. Myelodysplasia.
intra-muscularly.
J. She should be advised not to become
Answer & Comments
pregnant again.
Correct answer: DH
Answer & Comments
The MCV is raised due to reticulocytosis - this is
Correct answer: AE
a feature of autoimmune haemolysis. A
reticulocytosis does not occur in haematinic
deficiency. Marrow infiltration also causes This patient needs to start LMW heparin.
anaemia in CLL and advances the staging of the Warfarin is teratogenic and should not be given
disease. in the first trimester. Because of bleeding
during the time of labour warfarin should not
be given in the third trimester. Although
[ Q: 111 ] MasterClass Part2
theoretically warfarin can be given in the
(2010) - Hematology second trimester generally speaking it is not
A 32-year-old woman who is 9 weeks pregnant used and pregnant patients get daily LMW
in her first pregnancy develops a painful heparin throughout pregnancy to treat DVT.
swollen calf. Ultrasound scan confirms the Ninety per cent of DVT's in pregnancy occur on
presence of extensive deep vein thrombosis the left.
extending into femoral and illiac vessels. This is
Anticoagulation should continue until at least 6
her first clot and there is no other obvious risk
weeks post partum in this patient.
factor.
The APTT is used to monitor unfrationated
Which of the following statements regarding heparin and the PT to monitor warfarin.
this lady are correct?
Warfarin tablets: 5mg = pink, 3mg = blue, 1mg =
brown.

Dr. Khalid Yusuf El-Zohry – Sohag Teaching Hospital (01118391123) Page | 439
El-Zohry MRCP Questions Bank (Part 2) – Medical Masterclass 2010

Penicillin will affect warfarin control but does [ Q: 113 ] MasterClass Part2
not preclude its use. (2010) - Hematology
Pregnancy should not be avoided in the future
A 44-year-old man comes to the Emergency
but prophylactic anticoagulation should be
Department after a nose bleed which lasted 10
offered during the next pregnancy.
mins that morning. He described having
moderately severe haematemesis the day prior
[ Q: 112 ] MasterClass Part2 to this, but thought this may have been related
(2010) - Hematology to the two shots of whisky he drank before
going to bed. He has noticed several bruises in
A 28-year-old man has a splenectomy after a
the last week, and on examination he has an
road traffic accident. extensive purpuric rash over the trunk.
Which one of the treatment strategies listed is His FBC showed severe thrombocytopenia with
NOT required? a platelet count of 2 x 10^9/l (normal 150-400).
A bone marrow is carried out by the on call
A. Meningococcal vaccination
haematologist. This shows normal numbers of
B. Haemophilus influenzae vaccination megakaryocytes and a diagnosis of idiopathic
C. Prophylactic antibiotics with Penicillin V/ thrombocytopenic purpura is made.
Erythromycin
What is the most appropriate treatment for
D. Polyvalent pneumococcal vaccination him?
E. Hepatitis B vaccination. A. Oral prednisolone 60mg
B. Pulsed intravenous methyl prednisolone
Answer & Comments at a dose of 1g
Correct answer: E C. Intravenous immunoglobulin
D. Platelet transfusions
Patients splenectomised are prone to
Overwhelming Post Splenectomy Infection E. Intravenous thrombopoietin infusions.
(OPSI) that involves encapsulated organisms
including pneumococcus, meningococcus and Answer & Comments
Haemophilus influenzae. Individuals are also
Correct answer: C
prone to severe falciparum malarial infection
and Capnocytophagia canimorsus bacterium.
The spleen is necessary for: The most appropriate treatment for this patient
who is symptomatic from ITP is intravenous
 filtering immunoglobulin. Oral steroids will work in 60-
 phagocytosis 70% of cases but take about 2-3 days before an
 effect is noted. It is possible to give massive
production of antibodies required for
platelet transfusions to swamp the antibody
opsonisation of these bacteria and
but the hazards of blood transfusions need to
protozoa.
be borne in mind. In ITP the thrombopoietin
Lifelong penicillin is recommended but levels are high and no commercially available
compliance is very poor. Therefore, some thrombopoietin is in routine use. Beware of the
physicians recommend 3 g amoxicillin to be side effects of high dose steroids such as pulsed
taken at onset of febrile illness. In penicillin methyl prednisolone.
sensitive individuals, erythromycin is desirable.

Dr. Khalid Yusuf El-Zohry – Sohag Teaching Hospital (01118391123) Page | 440
El-Zohry MRCP Questions Bank (Part 2) – Medical Masterclass 2010

[ Q: 114 ] MasterClass Part2 chronic myelomonocytic leukaemia) presents


(2010) - Hematology with easy bruising and unprovoked soft tissue
haematomas. The following results are
The laboratory call to inform you that a 43- obtained at presentation: Hb 10.6g/dl; Wbc
year-old lady on a surgical ward has a platelet 29.9 x 10^9/l; plts 67 x 10^9/l; PT ratio 0.9;
count of 790 x10^9/l. APTT ratio 1.1; fibrinogen 0.8g/l D-dimers
2467ng/ml ( 0-300).
Given only this information, which TWO
conditions would NOT form part of the Which of the following is NOT a possibility?
differential diagnosis at this stage?
A. Her disease has transformed.
A. Chronic myeloid leukaemia (CML)
B. She has developed a partially
B. Cholecystitis compensated consumptive coagulopathy.
C. 2 days post hemicolectomy C. A 'wait and see' approach should
D. Acute lymphoblastic leukaemia (ALL) continue to be the mainstay of
treatment.
E. Essential thrombocythemia (ET)
D. Treatment of her underlying condition
F. Recently admitted following a road may alleviate the coagulopathy.
accident
E. Replacement therapy with
G. Crohn’s disease cryoprecipitate, fresh frozen plasma and
H. Acute pancreatitis platelets should be instituted.
I. Myelofibrosis
Answer & Comments
J. Thrombotic thrombocytopenia purpura
(TTP) Correct answer: C

Answer & Comments The 'high risk' myelodysplastic syndromes have


a tendency to transform into acute myeloid
Correct answer: DJ leukaemia. The lady has a compensated
consumptive coagulopathy shown by a low
The platelet count is raised. Myeloproliferative fibrinogen, normal activated partial
disorders such as CML, ET and myelofibrosis prothrombin time (APTT) and a markedly
may all cause a raised platelet count. Platelet elevated D-dimer. She has become overtly
count may also rise in response to infection or symptomatic and this requires treatment as the
inflammation. Therefore all causes of trauma risk of intracranial haemorrhage is a real one.
(including surgery) or inflammation such as Treatment of her underlying disease may
infections or inflammatory disease (e.g. Crohn’s alleviate her coagulopathy by eradicating the
disease) may cause a raised platelet count. malignant clone secreting this procoagulant
Secondary causes of a raised platelet count are activity. Replacement therapy with
far more common than primary causes. cryoprecipitate, fresh frozen plasma and
platelets should be instituted.
[ Q: 115 ] MasterClass Part2
(2010) - Hematology
A previously asymptomatic 75-year-old lady
with myelodysplastic syndrome (subtype

Dr. Khalid Yusuf El-Zohry – Sohag Teaching Hospital (01118391123) Page | 441
El-Zohry MRCP Questions Bank (Part 2) – Medical Masterclass 2010

[ Q: 116 ] MasterClass Part2 The baby should have the platelet count
(2010) - Hematology checked at birth and a week later.

A previously well 34-year-old woman (gravida II,


[ Q: 117 ] MasterClass Part2
para II) who is 24 weeks pregnant presents with
a low platelet count of 84 x 109/L. She is (2010) - Hematology
asymptomatic and experienced similar You are the Haematology SHO. An adult with
problems in her last pregnancy, following which acute lymphoblastic leukaemia requires
she gave birth to a normal healthy boy. She intrathecal methotrexate and your SpR asks you
denies any previous medical history or family to perform the lumbar puncture at 2pm and he
history of a blood disorder. will come and administer the drug. You perform
the LP at 2pm but the SpR does not appear.
Which of the following statements is true?
A. A bone marrow aspirate will confirm What should you do?
chronic idiopathic thrombocytopenic A. Proceed with the methotrexate
purpura (ITP). intrathecally
B. An emergency caesarian section is B. Ask the nurse to give the methotrexate
immediately the preferred delivery
method. C. Abandon the procedure

C. The baby has a 10% risk of intracranial D. Phone the consultant for help
bleed. E. Send CSF for cytology and wait
D. No investigations are required and
platelets should normalize post partum. Answer & Comments

E. Platelet associated immunoglobulins will Correct answer: C


be high.
First of all, you should not get into this situation
Answer & Comments by yourself. There are VERY strict guidelines
about intrathecal therapy and it should only be
Correct answer: D
given by doctors on the intrathecal register.
Occasionally, SHOs are trained and certified to
This woman has gestational thrombocytopenia, do this. As SHO you can place the LP needle, but
which is seen in 1-4% of pregnancies. Idiopathic cannot administer the drug. You cannot wait
thrombocytopenic purpura (ITP) and indefinitely with the needle in the CSF, so
haemolysis/ elevated liver enzyme/low abandon the procedure and discuss the
platelets (HELLP) syndrome are unlikely due to incident with the boss and the SpR.
the:
 negative history of ITP or bleeding [ Q: 118 ] MasterClass Part2
 previous history of a similar picture (2010) - Hematology
related to pregnancy
An obese 50-year-old man attends his GP's
 lack of symptoms. surgery because he is getting headaches. He
HIV-infection can cause thrombocytopenia, but has a history of mild hypertension, for which he
absence of risk factors make it unlikely here. takes bendroflumethiazide 5 mg once daily, and
he smokes 15 cigarettes per day. He looks

Dr. Khalid Yusuf El-Zohry – Sohag Teaching Hospital (01118391123) Page | 442
El-Zohry MRCP Questions Bank (Part 2) – Medical Masterclass 2010

plethoric, and his full blood count reveals a of red cell mass and plasma volume. Treatment
haemoglobin concentration of 19.2 g/dl. is by modification / removal of associated
factors, but in some cases venesection may be
The most likely TWO causes of his abnormal needed, although this is a contentious issue.
blood count are:
A. Benign intracranial hypertension [ Q: 119 ] MasterClass Part2
B. Iron deficiency (2010) - Hematology
C. Primary poliferative polycythaemia A 15-year-old Greek Cypriot boy has the
D. Chronic obstructive pulmonary disease following blood results: haemoglobin (Hb) 8.2
g/dL, white blood cell count 11 x 109/L,
E. Renal disease causing inappropriate platelets 405 x 109/L, ferritin 3891 ug/L, iron 42
erythropoietin excretion umol/L, transferrin saturation 70%, Hb A2= 6.1.
F. Previously unsuspected cyanotic His liver biopsy is shown (see image).
congenital heart disease
G. Diuretic treatment
H. Liver disease causing inappropriate
erythropoietin excretion
I. Cerebellar haemangioblastoma
J. Stress polycythaemia.

Answer & Comments

Correct answer: GJ

Polycythaemias can usefully be classified into:


1. True primary - primary proliferative
Which one of the treatments listed would be
polycythaemia, also known as
best?
polycythaemia rubra vera
A. Oral desferrioxamine treatment
2. True secondary - due to hypoxia, which
stimulates appropriate erythropoietin B. Oral ascorbic acid
release, or to inappropriate C. Daily subcutaneous desferrioxamine
erythropoietin excretion, which can be infusion
caused by renal disease, hepatic
disease, uterine fibroids or cerebellar D. Bolus desferrioxamine therapy
haemangioblastoma E. Periodic transfusion therapy.
3. Apparent relative polycythaemia – due
to reduced plasma volume, caused by Answer & Comments
dehydration / diuretics or stress
Correct answer: C
polycythaemia, commonly seen in
middle-aged overweight smokers.
The boy has got beta thalassemia major and is
The diagnosis of apparent relative transfusion-dependent. His serum ferritin and
polycythaemia is established by measurement iron are high, with a low total iron binding

Dr. Khalid Yusuf El-Zohry – Sohag Teaching Hospital (01118391123) Page | 443
El-Zohry MRCP Questions Bank (Part 2) – Medical Masterclass 2010

capacity (TIBC) which is a measure of B12 and folate levels must be normal to
transferring saturation. establish the diagnosis. Blasts can be a feature
of evolving myelodysplasia. The retic count is
His liver biopsy shows excessive iron in the liver
variable.
with some degree of fibrosis.
Further transfusion is contraindicated, as is
[ Q: 121 ] MasterClass Part2
ascorbic acid which can increase iron
absorption from stomach. He requires chelation (2010) - Hematology
therapy with subcutaneous desferrioxamine A 55-year-old woman presents with tiredness.
given preferably daily at a dose of 20-40 mg/kg She had been given a diagnosis of rheumatoid
over 8-12 hours. It is not active orally and its arthritis at the age of 35 years, but has been fit
effectiveness depends on its ability to bind to and well for many years, without any joint
iron over time rather than as a bolus. problems. Her full blood count is as follows: Hb
9.3 g/dl, MVC 85 fl, MCH 28 pg, white blood cell
[ Q: 120 ] MasterClass Part2 count normal, platelet count normal.
(2010) - Hematology
Which of the following statements is correct?
A 73-year-old woman is admitted with Hb 7.5,
A. Sideroblastic anaemia is a likely diagnosis.
MCV 99fL, neuts 1.1, platelets 68, reticulocytes
1%. Her neutrophils appear hypogranular. B. Felty’s syndrome is a likely diagnosis.
Poikilocytosis and anisocytosis are present. She C. Acute blood loss is likely.
is only on atenolol for hypertension. She has
D. Anaemia of chronic disorders is the most
had no previous illnesses.
likely diagnosis.
What features do NOT support a diagnosis of E. A normal ferritin excludes iron deficiency
myelodysplasia? as a cause of her anaemia.
A. Low B12 level
B. Normal B12 level Answer & Comments

C. Ring sideroblasts in the marrow Correct answer: D

D. Monocytosis
Ferritin is an acute phase reactant, hence a
E. Raised ferritin normal value does not exclude iron deficiency
F. Cytogenetic abnormalities on marrow anaemia. A low ferritin would be a useful result,
aspiration strongly supporting the diagnosis of iron
deficiency.
G. 5% blasts in the marrow
Anaemia of chronic disorders is a feature
H. Normal folate levels
ofrheumatoid arthritis, most marked in the
I. Normal retic count acute phase of the illness. Felty’s syndrome is
very uncommon.
J. Negative Coombs' test.
The differential diagnosis of her anaemia is
Answer & Comments wide. Appropriate tests would include: B12,
folate, iron, TIBC, ferritin; inflammatory
Correct answer: AE markers (CRP); thyroid function; rheumatoid
factor; immunoglobulins (?myeloma); liver,
renal and bone function tests; chest radiograph.

Dr. Khalid Yusuf El-Zohry – Sohag Teaching Hospital (01118391123) Page | 444
El-Zohry MRCP Questions Bank (Part 2) – Medical Masterclass 2010

[ Q: 122 ] MasterClass Part2 [ Q: 123 ] MasterClass Part2


(2010) - Hematology (2010) - Hematology
A 47-year-old decorator has hereditary A previously asymptomatic 75-year-old lady
spherocytosis. His Hb is 11 g/dl, MCV 89 fl, with myelodysplastic syndrome (subtype
bilirubin 23 mmol/l. He copes with his job but chronic myelomonocytic leukaemia) presents
asks if splenectomy may help. with easy bruising and unprovoked soft tissue
haematomas. The following results are
Which are the most appropriate responses? obtained at presentation: Hb 10.6g/dl; Wbc
A. Splenectomy is without serious 29.9 x 10^9/l; plts 67 x 10^9/l; PT ratio 0.9;
consequences. APTT ratio 1.1; fibrinogen 0.8g/l D-dimers
2467ng/ml ( 0-300).
B. Folic acid replacement can usually
obviate the need for splenectomy. Which of the following is NOT a possibility?
C. Splenectomy is indicated for symptomatic A. Her disease has transformed.
anaemia.
B. She has developed a partially
D. Splenectomy should always be followed compensated consumptive coagulopathy.
by B12 replacement.
C. A 'wait and see' approach should
E. Splenectomy does not require vaccination continue to be the mainstay of
in adults. treatment.
F. Splenectomy should be routinely D. Treatment of her underlying condition
performed at the age of 7 years. may alleviate the coagulopathy.
G. Splenectomy is indicated for E. Replacement therapy with
splenomegaly in spherocytosis cryoprecipitate, fresh frozen plasma and
H. Splenectomy is indicated for platelets should be instituted.
uncompensated haemolysis.
Answer & Comments
I. Splenectomy always improves the Hb
level. Correct answer: C
J. Splenectomy reduces the number of
circulating spherocytes. The 'high risk' myelodysplastic syndromes have
a tendency to transform into acute myeloid
Answer & Comments leukaemia. The lady has a compensated
consumptive coagulopathy shown by a low
Correct answer: CH fibrinogen, normal activated partial
prothrombin time (APTT) and a markedly
Splenectomy is not without hazard. It is elevated D-dimer. She has become overtly
reserved for symptomatic anaemia which symptomatic and this requires treatment as the
affects quality of life. Vaccination and lifelong risk of intracranial haemorrhage is a real one.
penicillin are required in adults. Splenectomy Treatment of her underlying disease may
removes the source of red cell destruction and alleviate her coagulopathy by eradicating the
spherocytes may rise in number on the blood malignant clone secreting this procoagulant
film. Splenectomy is performed in older rather activity. Replacement therapy with
than young children, but is not routine. cryoprecipitate, fresh frozen plasma and
platelets should be instituted.

Dr. Khalid Yusuf El-Zohry – Sohag Teaching Hospital (01118391123) Page | 445
El-Zohry MRCP Questions Bank (Part 2) – Medical Masterclass 2010

[ Q: 124 ] MasterClass Part2 haematinics can make people feel a lot better
(2010) - Hematology very quickly.

You are given a full blood count report by the


[ Q: 125 ] MasterClass Part2
secretary in the gastroenterology department.
It is from a 67-year-old woman that was seen in (2010) - Hematology
last week’s clinic. It shows Hb 8.5 g/dl, MCV 122 An obese 50-year-old man attends his GP's
fl, platelets 98 x 10^9/l, and neutrophils 1.2 x surgery because he is getting headaches. He
10^9/l. has a history of mild hypertension, for which he
takes bendroflumethiazide 5 mg once daily, and
Your next TWO steps should be to:
he smokes 15 cigarettes per day. He looks
A. organise a Schilling test plethoric, and his full blood count reveals a
haemoglobin concentration of 19.2 g/dl.
B. phone the GP to ask him/her to check
B12 and folate levels
The most likely TWO causes of his abnormal
C. get a reticulocyte count blood count are:
D. phone the consultant gastroenterologist A. Benign intracranial hypertension
E. check to see what the blood film showed B. Iron deficiency
F. look in Medical Masterclass C. Primary poliferative polycythaemia
G. phone the consultant haematologist D. Chronic obstructive pulmonary disease
H. write to the patient and ask them to E. Renal disease causing inappropriate
commence oral folate erythropoietin excretion
I. make a note to check her Coomb's test at F. Previously unsuspected cyanotic
the next clinic congenital heart disease
J. leave the problem untill the next clinic. G. Diuretic treatment
H. Liver disease causing inappropriate
Answer & Comments erythropoietin excretion
Correct answer: BE I. Cerebellar haemangioblastoma
J. Stress polycythaemia.
Although it is important to seek help from
senior colleagues and Medical Masterclass, you
Answer & Comments
should be able to make a decision on this as a
medical SHO. Don't leave this sort of problem Correct answer: GJ
to a later date or to somebody else - if the
patient is lost to follow up they could become Polycythaemias can usefully be classified into:
quite ill. The blood film will show if there is
1. True primary - primary proliferative
polychromasia (immune haemolysis) or
polycythaemia, also known as
features of megaloblastic anaemia. The film will
polycythaemia rubra vera
give you more information than just a
reticulocyte count (is this high in megaloblastic 2. True secondary - due to hypoxia, which
anaemia?). The B12 and folate levels should be stimulates appropriate erythropoietin
checked quickly because restoring these release, or to inappropriate
erythropoietin excretion, which can be

Dr. Khalid Yusuf El-Zohry – Sohag Teaching Hospital (01118391123) Page | 446
El-Zohry MRCP Questions Bank (Part 2) – Medical Masterclass 2010

caused by renal disease, hepatic Multiple myeloma is a malignant proliferation


disease, uterine fibroids or cerebellar of clonal plasma cells in the bone marrow.
haemangioblastoma Presentation is often non-specific with bone
pain, symptoms of anaemia or symptoms due
3. Apparent relative polycythaemia – due
to renal failure. The ESR is most often raised
to reduced plasma volume, caused by
and a characteristic finding on the blood film is
dehydration / diuretics or stress
rouleaux (coin-like stacking up of red cells). This
polycythaemia, commonly seen in
should be distinguished from auto-agglutination
middle-aged overweight smokers.
where the red cells are matted together( 'akin
The diagnosis of apparent relative to clumped rice'). In most cases of
polycythaemia is established by measurement myelomatosis, treatment is with chemotherapy
of red cell mass and plasma volume. Treatment and supportive care.
is by modification / removal of associated
factors, but in some cases venesection may be
[ Q: 127 ] MasterClass Part2
needed, although this is a contentious issue.
(2010) - Hematology
[ Q: 126 ] MasterClass Part2 A 43-year-old man presented to the accident
(2010) - Hematology and emergency department with swelling of the
right leg. A venogram confirmed a thrombus in
the deep veins. He had no previous history of
venous thromboembolism. A day later he
develops right sided pleuritic chest pain. His
thrombophilia screen is negative.

What is the correct therapeutic option?


A. Warfarin therapy for six months at a
target international normalized ratio
(INR) of 3.5 (range 3-4.5).
B. Warfarin therapy for three months at a
target INR of 2.5 (range 2-3).
C. Warfarin therapy for six months at a
target INR of 2.5 (range 2-3).
Identify the most obvious feature on this blood D. No therapy is required after initial five
film (see image), taken from a patient days heparin therapy.
presenting with bone pain.
E. Lifelong anticoagulation with a target INR
A. Red cell clumping of 2.5 (range 2-3).
B. Platelet clumping
C. Rouleaux Answer & Comments

D. Hypochromia Correct answer: C

E. Cold agglutinins.
Pulmonary embolism (PE) and deep vein
thrombosis (DVT) occuring so close to each
Answer & Comments
other can assumed to be related. The initial
Correct answer: C management of this patient with one episode

Dr. Khalid Yusuf El-Zohry – Sohag Teaching Hospital (01118391123) Page | 447
El-Zohry MRCP Questions Bank (Part 2) – Medical Masterclass 2010

of venous thromboembolism is six months Most patients with Waldenström's


anticoagulation. This should follow initial macroglobulinaemia or myeloma relapse, with
heparinisation with low molecular weight a rise in their paraprotein. When measured, this
(LMW) heparin, or five days unfractionated man's IgM had risen to 29 g/L and his anaemia
heparin with an adjusted activated partial was due to active disease. He developed chest
thromboplastin time (APTT) ratio of 1.5-2.5. The pain due to an acute haemolytic transfusion
management of a patient with heritable reaction that caused the Coombs' test to
thrombophilia (i.e. protein C or S deficiency, or become positive. This was due to a Jka red cell
factor V Leiden mutation) and a first acute antibody that had not previously been
thrombotic episode is similar. The target INR detected.The plasma viscosity is only slightly
should be 2.5 (range 2-3). Three months raised and is unlikely to produce renal failure at
antiocoagulation may be sufficient for when the this level. Fludarabine can cause Coombs'-
thrombus is restricted to the calf veins alone. positive haemolysis, but not usually 5 years
after treatment. There is no evidence for acute
[ Q: 128 ] MasterClass Part2 myeloid leukaemia.

(2010) - Hematology
[ Q: 129 ] MasterClass Part2
A 71-year-old man is treated with fludarabine
(2010) - Hematology
for Waldenström's macroglobulinaemia. His
IgM band falls from 36 g/L to a plateau of 5 g/L A 49-year-old man presents with malaena of a
and he is stable for 5 years before suddenly week's duration. On investigation his platelet
presenting with haemoglobin (Hb) 6.7 g/dL count is found to be elevated.
(normal range 12-16.5), platelets 79 x 109/L
(normal range 150-400), creatinine 130 Which of the following is the most likely cause
micromol/L (normal range 70-110) and plasma of the thrombocytosis?
viscosity 2.5 mPa/s (normal range 1.5-1.75). A. Blood loss
During blood transfusion he develops a
B. Chronic myeloid leukaemia
tachycardia and chest pain, and is transferred
to the Coronary Care Unit. The next day results C. Infection
show Hb 7 g/dL, a positive Coombs' test and
D. Essential Thrombocytopenia
creatinine 377 micromol/L.
E. Inflammatory bowel disease.
What is the most likely diagnosis?
A. Renal failure due to hyperviscosity Answer & Comments

B. Pulmonary embolism with circulatory Correct answer: A


failure
C. Fludarabine-induced haemolysis Thrombocytosis is much more commonly a
secondary phenomenon rather than a part of a
D. Relapse of Waldenström's disease with
primary malignant process. Blood loss results in
haemolytic transfusion reaction
increased marrow activity and hence a raised
E. Transformation to acute leukaemia platelet count. Treatment is of the underlying
cause will serve to bring the platelet count
Answer & Comments down gradually.

Correct answer: D

Dr. Khalid Yusuf El-Zohry – Sohag Teaching Hospital (01118391123) Page | 448
El-Zohry MRCP Questions Bank (Part 2) – Medical Masterclass 2010

[ Q: 130 ] MasterClass Part2


(2010) - Hematology
A 47-year-old man with a haemoglobin (Hb) of
5.9 g/dL, white blood cells (WBC) 10 x 109/L and
platelets 455 x 109/L is seen in the accident and
emergency department following a moderate
gastrointestinal bleed. His blood presure is
110/60 and he is tachycardic. He is on no
medication.

Which of the following is the most appropriate


blood product to use?
A. plasma protein fraction
B. red cells in SAG-M
C. fresh frozen plasma (FFP)
D. colloids and crystalloids
E. iron dextran infusion

Answer & Comments

Correct answer: B

The correct blood product in this case is red


cells in optimal additive solution, which can
increase the shelf life to 35 days. The patient is
relatively haemodynamically stable. There is no
role for infusion of fresh frozen plasma unless a
coagulopathy is demonstrable. Iron infusion is
not recommended due to possiblity of
anaphylaxis and his haemaglobin rise will not
be immediate. Albumin is also not required
here, and is neither cost effective nor risk free
of blood borne infections.

Dr. Khalid Yusuf El-Zohry – Sohag Teaching Hospital (01118391123) Page | 449
El-Zohry MRCP Questions Bank (Part 2) – Medical Masterclass 2010

she has never had chickenpox herself before,


Infectious Disease and her varicella zoster IgG test is negative.

(150 Questions) Which one of the following is the correct


treatment?
(Medical Masterclass – Part 2) A. Reassure her, but advise her to report
promptly to her general practitioner if
she becomes ill
B. Commence intravenous aciclovir
[ Q: 1 ] MasterClass Part2
(2010) – Infectious disease C. Commence oral aciclovir
D. Give a single dose of varicella zoster
Which of the following CANNOT be attributed to immunoglobulin
ingestion of contaminated food or drink in an
affected traveller? E. Give a single dose of varicella zoster
immunoglobulin and commence oral
A. Anisakiasis aciclovir
B. West Nile fever
C. Cholera Answer & Comments

D. Gnathostomiasis Correct answer: D

E. Hepatitis A
There is a 1-2% additional risk of foetal
abnormalities if the mother gets chickenpox in
Answer & Comments
the first 20 weeks of gestation, and an
Correct answer: B increased risk of chickenpox pneumonitis in the
mother. If the mother doesn't think she has had
West Nile Fever is an arboviral infection chickenpox before, an urgent varicella zoster
(Arthropod Borne VIRAL) transmitted by IgG level should be performed on her. If this is
mosquitos that have bitten infected birds. positive, no further action is required. If this is
Though recognized in Africa, West Asia and the negative, varicella zoster immunoglobulin
Middle East for decades, it has recently arrived should be administered to reduce her risk of
in the US, leading to an exponential increase in developing chickenpox.
interest. Anisakiasis may be acquired from raw
fish, gnathostomiasis from contaminated fish, [ Q: 3 ] MasterClass Part2
shrimp, frog or chicken, hepatitis A classically (2010) – Infectious disease
from shellfish and V. cholerae (which more
usually is due to contaminated water) from A 40-year-old hepatitis B virus (HBV) carrier (e
contaminated seafood. antigen positive, e antibody negative) presents
with progressive liver fibrosis and an alanine
aminotransferase (ALT) of 30 U/L (normal <45).
[ Q: 2 ] MasterClass Part2
(2010) – Infectious disease Which drug would be of most value for long-
term treatment of his HBV?
A 25-year-old woman who is 16 weeks pregnant
presents because she had been in contact with A. Adefovir
a child with chickenpox the day before. She says B. Ganciclovir

Dr. Khalid Yusuf El-Zohry – Sohag Teaching Hospital (01118391123) Page | 451
El-Zohry MRCP Questions Bank (Part 2) – Medical Masterclass 2010

C. Interferon Travel across more than 6 time zones,


eastbound, leads to a shortened day, reducing
D. Ribavirin
insulin requirement. The opposite is true for
E. Zidovudine westbound travel, but short-lived
hyperglycaemia due to insulin underdosing is
Answer & Comments less dangerous than the risk of hypoglycaemia
while travelling.
Correct answer: A
Only about 1% of tropical travellers experience
significant infection, considerably more being
Adefovir would be of most value. The patient
affected by episodes of pre-existing cardiac or
needs some kind of treatment as he has
respiratory disease, or trauma.
progressive disease. Interferon alfa is usually
given short term and is not very effective in A healthy individual's oxygen saturation may
patients without an elevated ALT. Lamivudine drop as low as 88.6% at cruising altitude
would be an alternative, although resistance (studies on airline cabin staff).
develops commonly
[ Q: 5 ] MasterClass Part2
[ Q: 4 ] MasterClass Part2 (2010) – Infectious disease
(2010) – Infectious disease
A 40-year-old man with a history of congenital
An 48 yr woman with diabetes and mild chronic heart disease returns from a holiday in Africa
obstructive pulmonary disease is about to go with fever and a purpuric rash. He has had a
travelling abroad. She is extremely anxious number of sexual contacts, and swum in a
about the possible health risks. fresh-water lake.

Which one of the following statements about Which of the following is a likely cause of this
travel related disease is true: combination?
A. More than 30% of travellers can expect a A. Tick typhus
diarrhoeal illness on a trip of a month
B. Typhoid
B. West-bound long-haul flights are more
C. Cutaneous larva migrans
problematic for insulin-dependant
diabetics than east-bound D. Secondary syphilis

C. More travellers are hospitalised by E. Katayama syndrome


cardiovascular disease than infection F. Bacterial endocarditis
during tropical travel
G. HIV serocoversion illness
D. Cholera vaccine is currently not licensed
in the UK H. Glandular fever

E. A healthy adult can drop their arterial I. Measles


oxygen saturation below 90% at the J. Smallpox
normal cruising altitude of an
international airliner Answer & Comments

Correct answer: AF
Answer & Comments

Correct answer: C All of these infectious diseases can cause a rash.

Dr. Khalid Yusuf El-Zohry – Sohag Teaching Hospital (01118391123) Page | 452
El-Zohry MRCP Questions Bank (Part 2) – Medical Masterclass 2010

Tick typhus causes a purpuric rash (usually seroconversion illness in a sexually active
small petechiae) in the majority of cases, traveller, but the rash is usually macular.
although there is a variant of tick typhus which
causes a rash only in a minority, so its absence [ Q: 6 ] MasterClass Part2
does not exclude the diagnosis. Bacterial
(2010) – Infectious disease
endocarditis can cause purpuric patches if
multiple emboli are being flicked off and lodge A 40-year old man from Saudi Arabia (in UK 4
in the skin or digits. It is part of the differential days) presents with painful episodes of trismus.
for what clinically looks like meningococcal He has never been vaccinated against tetanus
septicaemia. and sustained a slight cut to his foot while
Cutaneous larva migrans presents a serpiginous walking barefoot in his garden in Saudi Arabia 3
red track on the skin, and should be obvious days before arriving in the UK. You suspect
clinically. tetanus and he then has an episode of
generalised tetanus with respiratory arrest.
Secondary syphilis can present a variety of
rashes (and most cases will have a macular or Which of the following is incorrect as part of his
papular rash of some sort) but purpuric rashes management?
are very unlikely.
A. Admit to side ward in intensive care unit
The rash of smallpox is pustular, coming at once
B. IV gentamicin to kill any Clostridium
rather than in crops.
tetani bacteria left in wound
Katayama syndrome, an immunological
C. Tetanus toxoid as he will not be immune
phenomenon which occurs about 6 weeks after
to tetanus even after recovery from
exposure to schistosomiasis causes fever and an
infection
urticarial rash.
D. Clean, debride and dress his foot lesion
The rash of typhoid is described as ‘rose spots’
which occur in the second week in a minority of E. IM tetanus antitoxin.
cases. As the name implies, they are not
purpuric. Answer & Comments
Many infectious diseases ranging from malaria Correct answer: B
to overwhelming gram-negative sepsis can, of
course, cause disseminate intravascular
These patients should be nursed in a quiet
coagulation in severe cases in which case
environment as noises often trigger generalized
purpuric or non-blanching petechial rashes may
spasms. If he is having generalized spasms he
occur. Many viral infections, including some
will require ventilation and muscle relaxants
which are quite trivial, can give rise to transient
with or without neuromuscular blockade. IM
capillary fragility. Petechial rashes may occur in
antitoxin should be given immediately to mop
these cases, especially on the arms below
up any unbound tetanus toxin. The antibiotic of
where blood pressure cuffs or tourniquets have
choice is IV metronidazole. After recovery from
been used (as in the tourniquet test for dengue
infection he will still have no immunity to
fever), and in gravitational areas.
tetanus and the first dose of tetanus toxoid (at
In children Henoch Schonlein purpura should be a different site to the antitoxin) is therefore
considered. given, with two more doses to follow.
The combination of fever and rash should
always raise the suspicion of acute HIV [ Q: 7 ] MasterClass Part2

Dr. Khalid Yusuf El-Zohry – Sohag Teaching Hospital (01118391123) Page | 453
El-Zohry MRCP Questions Bank (Part 2) – Medical Masterclass 2010

(2010) – Infectious disease


A 20 year old man in his first sexual relationship
complains of recurrent genital lesions. Lesions
tend to be unilateral, last between 2-4 days and
are mildly painful. He has experienced 4
episodes in the last 6 months. his partner is
asymptomatic.

Which investigation is likely to give the


diagnosis?
A. Treponemal serology
B. HLAB27 determination Which TWO of the statements are NOT correct?

C. Lesion biopsy A. The patient's roommate in his residential


home is at risk of catching shingles.
D. Lesion Viral Cultures
B. The pain may precede the appearence of
E. Lesion bacterial cultures the rash.
C. The patient remains infectious until the
Answer & Comments
rash has resolved.
Correct answer: D D. Postherpetic neuralgia is more common
in older patients.
1:25 males carry the herpes simplex type 2.
E. Vesicles appear first which become
Diagnosis can be made by identifying the virus
pustules and then crust.
from lesions by culture. Most individuals will
have contracted their infection from an F. Pneumonia and hepatitis are
asymptomatic source partner. HSV1 can cause complications.
genital infections although they tend not to G. Involvement of the V1 dermatome may
recur frequently. If troublesome, disease can be lead to ocular problems.
controlled with continous suppressive aciclovir
therapy H. Capsaicin may be useful in treating post-
herpetic neuralgia.

[ Q: 8 ] MasterClass Part2 I. Thoracic dermatomes are most commonly


(2010) – Infectious disease affected.
J. Early acyclovir treatment may reduce
An elderly man presents with the rash shown
post-herpetic neuralgia.
(see image).

Answer & Comments

Correct answer: AC

The rash is a typical dermatomal rash of


shingles caused by varicella zoster. Thoracic
dermatomes are the most commonly affected -
presumably for numerical reasons. The rash is
due to reactivation of the person's dormant

Dr. Khalid Yusuf El-Zohry – Sohag Teaching Hospital (01118391123) Page | 454
El-Zohry MRCP Questions Bank (Part 2) – Medical Masterclass 2010

virus, so while a nurse caring for him may Answer & Comments
develop chicken pox if there is no prior
Correct answer: E
exposure, his roommate will not develop
shingles. Like chicken pox, shingles is not
considered infectious once all the lesions are The hard palate is a common site for Kaposi’s
crusted. There is weak evidence that acyclovir sarcoma (KS) and this is the classical
may decrease post-herpetic neuralgia. Drugs appearance. The patient may be unaware of the
such as gabapentin or topical capsaicin may be lesion. Most commonly there will, in addition,
useful in treating the latter. be cutaneous KS lesions, but these may be
absent or inconspicuous. This presentation
would be most consistent with underlying HIV
[ Q: 9 ] MasterClass Part2 infection and the development of KS would be
(2010) – Infectious disease an AIDS-defining illness.
A 28-year-old man presents with a lesion in his KS may be seen on the legs of elderly men or
mouth (see image). complicating other causes of significant
immunodeficiency such as after
transplantation. Endemic KS is also seen in
parts of Africa where young HIV-negative
adults may be affected. Lymph nodes,
genito-urinary tract and lung involvement
may be seen in advanced cases. All types of
KS are closely linked to the presence of
human herpesvirus 8.

[ Q: 10 ] MasterClass Part2
(2010) – Infectious disease
A 30-year-old male recently returned from
trekking in Nepal presents with a 6-day
history of bloody diarrhoea with abdominal
cramps but no fevers. He has taken some
antibiotics, obtained in Nepal, with little
effect.

Which of the following would be an unlikely


cause?
What is the diagnosis? A. Entamoeba histolytica
A. HIV infection B. Trichuris
B. Squamous cell carcinoma C. Acute schistosomiasis
C. Oral syphilis D. Ulcerative colitis
D. Normal variant E. Clostridium difficile colitis.
E. Kaposi’s Sarcoma
Answer & Comments

Correct answer: C

Dr. Khalid Yusuf El-Zohry – Sohag Teaching Hospital (01118391123) Page | 455
El-Zohry MRCP Questions Bank (Part 2) – Medical Masterclass 2010

Schistosomiasis is not endemic in the Indian recent acquisition of infection – within the past
subcontinent, though would be a cause of the 12 months.
above elsewhere in the tropics. Heavy worm
Pregnancy may lead to a false positive VDRL but
loads with Trichuris can cause bloody diarrhoea.
her positive TPHA indicates syphilitic infection
Cosmopolitan causes of bloody diarrhoea
at some stage, and the high VDRL is most
should always always be considered as a cause
consistent with active infection.
despite the travel history and C. difficile disease
excluded when antibiotics have been taken.
[ Q: 12 ] MasterClass Part2
[ Q: 11 ] MasterClass Part2 (2010) – Infectious disease
(2010) – Infectious disease An 89-year-old man presents to the Emergency
Department with fever and a several week
A 32-year-old female is referred due to positive
history of headache. On examination he is
serology for syphilis. She gives a history of
confused, has neck stiffness and a right seventh
treatment for syphilis 8 years previously. Tests
cranial nerve palsy. He has no visible rash.
show a positive venereal disease reference
Investigations show: CSF Protein 4.0 g/L; CSF
laboratory (VDRL) titre of 1:128 and a positive
Glucose 1.2 mmol/l, plasma glucose 4.6 mmol/l;
Treponema pallidum haemagglutination assay
CSF Microscopy 300 white cells, predominantly
(TPHA) titre of 1:1024.
lymphocytes; Serum VDRL positive and TPHA is
negative.
Which of the following explanations is most
consistent with these data?
The most likely diagnosis is:
A. Inadequate previous therapy
A. Viral meningitis
B. Active syphilis acquired in the recent past
B. Neurosyphilis
C. Lyme disease
C. Herpes simplex encephalitis
D. Inadequate previous therapy
D. Tuberculous meningitis
E. Pregnancy.
E. Listeria meningitis.

Answer & Comments


Answer & Comments
Correct answer: B
Correct answer: D

Successful therapy for syphilis leads to a steady


The combination of long history, focal signs,
fall in the VDRL or rapid plasma reagin (RPR)
very high protein, low glucose and a
titre. Following primary disease the VDRL is
lymphocytic CSF make a diagnosis of
generally negative within 1 year, and within 2
tuberculous meningitis the most likely.
years for secondary syphilis. A small number of
treated patients have a persistently low The diagnosis of neurosyphilis cannot be made
detectable VDRL. Thus, her story of past syphilis with out a positive serum specific treponemal
is not consistent with these results. Even if she antibody (TPHA). The positive VDRL is a
had received no therapy the VDRL would fall biological false positive.
gradually and in latent syphilis the VDRL titre is
generally 1:16 or less. Therefore, her very high
VDRL titre of 1:128 most likely represents

Dr. Khalid Yusuf El-Zohry – Sohag Teaching Hospital (01118391123) Page | 456
El-Zohry MRCP Questions Bank (Part 2) – Medical Masterclass 2010

[ Q: 13 ] MasterClass Part2 occur within 24 hours of starting treatment.


(2010) – Infectious disease Organ failure and death are very rare. High
treponemal load as seen in secondary syphilis is
A 29-year-old woman is found to have positive responsible for the most severe cases. The
syphilis serology when she attends for course of the reaction can be modified with low
antenatal screening. Subsequent testing dose prednisolone.
suggests that she has active disease, with
Although classically described in the treatment
positive non-treponemal (rapid plasma reagin,
of syphilis, Jarisch–Herxheimer reactions have
RPR or Venereal Disease Reference Laboratory,
been described in other infections including
VDRL) and treponemal (Treponema Pallidum
brucellosis, borreliosis (relapsing fever) and
haemagglutination assay, TPHA or fluorescent
leptospirosis.
treponemal antibody, FTA) tests. A decision is
made to treat her with parenteral penicillin, but
you are worried that she might develop a [ Q: 14 ] MasterClass Part2
Jarisch–Herxheimer reaction. (2010) – Infectious disease

Which TWO of the following statements In addition to ceftriaxone, sometimes ampicillin


regarding the Jarisch–Herxheimer reaction are is added to 'blind' antibiotic regimens in cases
true? of meningitis.

A. It is a delayed hypersensitivity reaction to What organism is this intended to cover in such


antitreponemal therapy. a case?
B. It typically starts between 36-48 hours of A. Penicillin-resistant pneumococci
the initiation of therapy.
B. Meningococcus group B
C. It may be modified by steroids.
C. Meningococcus group C
D. It is characterized by fever, myalgia,
headache and hypotension. D. Listeria

E. It is worse in primary syphilis. E. Enterococci.

F. It is frequently fatal.
Answer & Comments
G. Immediate admission to inpatient
Correct answer: D
facilities is required if a Jarisch–
Herxheimer reaction is suspected.
Listeria can cause disease in the
H. It may occur in Yaws.
immunosuppressed (including pregnant
I. It is only seen in the treatment of syphilis. women). Ampicillin is the drug of choice. A-C
are covered well by ceftriaxone and E is not an
J.
important cause of meningitis except in special
cases.
Answer & Comments

Correct answer: CD [ Q: 15 ] MasterClass Part2


(2010) – Infectious disease
The Jarisch–Herxheimer reaction is due to the
generation of inflammatory cytokines A 25-year-old lady presents with chickenpox.
consequent on treponemal destruction after
Which of the following are true?
antimicrobial therapy. Most cases are mild and

Dr. Khalid Yusuf El-Zohry – Sohag Teaching Hospital (01118391123) Page | 457
El-Zohry MRCP Questions Bank (Part 2) – Medical Masterclass 2010

A. Varicella pneumonitis is more likely to immune responses play a role in long term
develop if she drinks alcohol control of this as in other herpesvirus infections
B. Varicella pneumonitis is less likely to
develop if she is a smoker [ Q: 16 ] MasterClass Part2
C. Varicella pneumonitis is more likely to (2010) – Infectious disease
develop if she is pregnant
What is the aetiological agent for this
D. She will not be infectious before the first asymptomatic lesion found in an HIV patient?
vesicles appear
E. The incubation period for chickenpox is 3
to 10 days
F. Varicella encephalitis rarely affects the
cerebellum
G. Protection against infection is provided
by humoral immunity
H. Infection during pregnancy is not
associated with congenital defects in the
foetus
I. Viral culture of vesicle fluid is unlikely to
isolate the varicella zoster virus A. Epstein Barr virus

J. Transmission of infection does not occur B. Herpes simplex virus


by the respiratory route. C. Candida albicans
D. Candida glabrata
Answer & Comments
E. Enterovirus.
Correct answer: CG
Answer & Comments
Varicella pneumonitis occurs more frequently in
adults who smoke or who are pregnant. Alcohol Correct answer: A
has no effect on the risk of developing
pneumonitis. Patients become infectious about This lesion is called oral hairy leucoplakia and is
48 hours before the appearance of the rash. caused by Epstein Barr virus. It is usually
The incubation period is 10-21 days. asymptomatic and occurs in patients with HIV
infection and very rarely in immunosuppressed
Varicella encephalitis commonly affects the
organ transplant recipients. It is useful because
cerebellum. The risk of embryopathy after
if present it is almost diagnostic of HIV infection
maternal varicella infection in the first 20 weeks
in the absence of an organ transplant.
of pregnancy is about 2%. The diagnosis of
chickenpox is usually clinical but if in doubt, can
be confirmed by viral culture or PCR of vesicle [ Q: 17 ] MasterClass Part2
fluid. Transmission actually occurs by the (2010) – Infectious disease
respiratory route and not by direct contact.
You are asked to see a UK healthcare worker
Detection of antibodies against Varicella Zoster who suffered a percutaneous injury in a high
is indicative of immunity, although cellular

Dr. Khalid Yusuf El-Zohry – Sohag Teaching Hospital (01118391123) Page | 458
El-Zohry MRCP Questions Bank (Part 2) – Medical Masterclass 2010

risk situation and wants to start HIV post- [ Q: 18 ] MasterClass Part2


exposure prophylaxis (PEP). (2010) – Infectious disease
Which of the following actions are A 25-year-old man has recently been diagnosed
inappropriate? as HIV-positive. In view of a high viral load and
A. Immediate testing of the source patient’s low CD4 count he was commenced on septrin
stored serum for HIV (does not require (960 mg alternate days), Zidovudine 250 mg
patient consent) b.d., Lamivudine 150mg b.d. and Abacavir
300mg b.d. Five weeks into this regimen he
B. Commencing dual agent therapy (e.g. becomes progressively unwell over 3 days – he
Zidovudine and lamivudine) for maximal develops a fever of 40ºC, an erythematous
efficacy macular rash affecting the limbs and trunk,
C. PEP should be continued for 28 days nausea, vomiting and abdominal pains. He has
lethargy and a marked headache. He noted
D. PEP related side effects such as tiredness,
some breathlessness and is significantly more
anemia, diarrhea and nephrolithiasis
breathless this evening.
should be anticipated
E. Full haematological, renal and liver What immediate course of action is advisable?
assessment at baseline and twice again at A. Stop septrin
2 and 4 weeks is advised.
B. Stop zidovudine
F. Safer sex should be advised
C. Stop lamivudine
G. Women should avoid pregnancy
D. Stop abacavir
H. If the healthcare worker is a surgeon,
invasive surgical procceedures may be E. Initiate steroids.
continued
Answer & Comments
I. HIV tests should be performed at 6 and 26
weeks. Correct answer: D

Answer & Comments Although hypersensitivity reactions can occur


on any antiretroviral regimen, some can be
Correct answer: AB
rapidly progressive and life-threatening.
Abacavir hypersensitivity occurs in 5% of
HIV testing requires consent whenever individuals. Patients should be counselled as to
possible. Current UK guidelines advise dual its features and advised to seek attention
therapy for 28 days as a minimum standard and immediately if suspected. Abacavir
most patients receive triple therapy. The risk of hypersensitivity may present without a rash
seroconversion is small but the patient should and should be considered in the differential
be advised to practice safe sex and avoid diagnosis of any febrile illness after
pregnancy if female. Surgeons may continue to commencing abacavir.
operate.
Abacavir should be stopped and an alternative
Complications from therapy are common and anti-retroviral agent commenced to maintain
compliance is generally low. Patients should be triple combination therapy. Rechallenge with
seen at 2-weekly intervals for full assessment. abacavir should never be attempted since it
may be rapidly fatal.

Dr. Khalid Yusuf El-Zohry – Sohag Teaching Hospital (01118391123) Page | 459
El-Zohry MRCP Questions Bank (Part 2) – Medical Masterclass 2010

[ Q: 19 ] MasterClass Part2 [ Q: 20 ] MasterClass Part2


(2010) – Infectious disease (2010) – Infectious disease
A 23-year-old medical student has just returned A 64-year-old Indian man presents with a 7-
from 8 weeks in the Congo on their elective. week history of fever, weight loss, worsening
They become hot and feverish and are sent to cough and exertional dyspnoea. He has a past
the Medical Admissions Unit, where their history of diabetes mellitus and has had an
temperature is recorded as being 40°C. Your angiogram for investigation of ischaemic heart
first thought is to exclude malaria, but three disease recently.
malarial films are negative and you begin to
consider alternative diagnoses. On enquiry the Which one of the following would not be an
student tells you that there had been several appropriate management step at this stage?
cases of haemorrhagic fever in the hospital that A. Chest radiography
they visited.
B. Blood cultures x3
Which TWO of the following are the routes by C. Empirical tuberculosis therapy
which viral haemorrhagic fevers are usually
transmitted? D. Echocardiography

A. Mosquito E. Urine dipstick and culture

B. Larval trombiculid mite


Answer & Comments
C. Sandfly
Correct answer: C
D. Body louse
E. Tick The patient may have developed endocarditis
secondary to his recent angiography.
F. Contact with bodily fluids from an
Tuberculosis is in the differential diagnosis, but
infected patient
more evidence should be obtained before
G. Bed bug commencing therapy.
H. Faecal-oral route
I. Reduviid bug [ Q: 21 ] MasterClass Part2
(2010) – Infectious disease
J. Rat urine.
A 50-year-old man has been ventilated on the
Answer & Comments intensive care unit for 12 days following a head
injury. He develops fever and worsening gas
Correct answer: EF exchange, accompanied by shadowing on his
CXR.
Viral haemorrhagic fevers are distributed
through Eastern Europe, the Middle East and What organism is most likely to have caused his
Africa. Transmission is mainly by tick bite, but nosocomial infection?
spread may occur by contact of healthcare A. Streptococcus pneumoniae
workers with bodily fluids from an infected
person. B. Coagulase-negative Staphylococcus
C. Escherichia coli
D. Pseudomonas aeruginosa

Dr. Khalid Yusuf El-Zohry – Sohag Teaching Hospital (01118391123) Page | 460
El-Zohry MRCP Questions Bank (Part 2) – Medical Masterclass 2010

E. Acinetobacter Clinical manifestations of drug


Onset
reactions
Answer & Comments
0–24
Anaphylaxis, urticaria
Correct answer: D hours

Haemolytic anaemia, neutropenia,


>72 hours
Pseudomonas is a very common cause of thrombocytopenia
ventilator-associated pneumonia. Streptococcus
Drug fever, serum sickness 7–14 days
pneumoniae is common in communityacquired
disease. Acinetobacter can be a major hazard Contact dermatitis Variable
on intensive care units.
Rash, fixed drug reactions 7–14 days

[ Q: 22 ] MasterClass Part2
(2010) – Infectious disease [ Q: 23 ] MasterClass Part2
(2010) – Infectious disease
A patient reports that he is allergic to
erythromycin. A 25-year-old woman complains of an
increasing offensive vaginal discharge over the
You would usually accept the following as being last 2 weeks. She is feeling feverish and has
compatible with an allergy, except: lower abdominal pains. She is not sexually
A. Fever active and has had a sexual health screen after
she finished her last relationship some 3
B. Widespread rash months ago, when a cervical erosion was noted.
C. Shortness of breath She has never had a sexually transmitted
infection. She is on no medication and her last
D. Diarrhoea
normal menstrual period was 17 days ago. She
E. Localized skin eruption. discontinued her oral contraceptive some 4
weeks ago.
Answer & Comments
What is the most likely diagnosis?
Correct answer: D
A. Candidal infection
B. Neisseria gonorrhoeae infection
Patients often report side effects of a drug that
are not related to an allergic phenomenon as an C. Cervical malignancy
allergy. These include nausea, diarrhoea and
D. Retained foreign body in the vagina
headache. If a patient reports a drug allergy you
should always determine what the allergy was, E. Pregnancy
how severe it was and who diagnosed and/or
documented it, to be able to make an informed Answer & Comments
decision as to whether they truely do.
Correct answer: D
Erythromycin is well known to cause nausea
and gastrointestinal symptoms as a side effect.
Sexually transmitted infections are highly
unlikely in view of the history.
Toxic shock syndrome is associated with
retained tampons and can present with fevers,
hypotension and rash. Unless diagnosed early

Dr. Khalid Yusuf El-Zohry – Sohag Teaching Hospital (01118391123) Page | 461
El-Zohry MRCP Questions Bank (Part 2) – Medical Masterclass 2010

patients may deteriorate rapidly and fatalities B would need to be considered but acute
have been reported. hepatitis C is generally subclinical. Syphilis and
disseminated gonorrhoea should be considered
[ Q: 24 ] MasterClass Part2 in any sexually active person presenting with
fever and rash, but in this patient an acute viral
(2010) – Infectious disease
infection is suggested by the reactive
A 17-year-old gay male has recently become lymphocytosis.
sexually active. He presents with a 1-week
history of fever, myalgia, sore throat and a [ Q: 25 ] MasterClass Part2
macular rash. A blood film shows a reactive
(2010) – Infectious disease
lymphocytosis and liver function tests are
abnormal. HIV-1 antibody is negative.
Which of the following is of LEAST concern to
your hospital infection control officer?
Which TWO of the following tests are most
likely to provide a diagnosis? A. Diarrhoea due to drug-sensitive
Clostridium difficile
A. Hepatitis C antibody
B. Pneumonia due to Mycobacterium bovis
B. Throat, urine and stool cultures for
viruses C. Wound infection with drug-sensitive
Enterococcus faecalis
C. Measurement of anti-CMV IgG
D. ITU ventilator-associated pneumonia
D. Paul–Bunnell test
with drug-resistant Klebsiella
E. HIV p24 antigen pneumoniae
F. Coxsackie virus serology E. Surveillance skin swabs with
G. HIV-2 antibody test Staphylococcus aureus resistant to
methicillin and vancomycin
H. Blood cultures
I. Parvovirus B 19 IgM Answer & Comments
J. Serum VDRL/TPHA. Correct answer: C

Answer & Comments Acquisition of C. difficile may occur by person-


Correct answer: DE to-person spread in outbreak situations in
institutions, such as nursing homes and
hospitals and thus a history of antibiotic use is
The picture here is of an acute glandular fever-
not an essential prerequisite. Mortality in the
like viral illness. Ebstein-Barr virus (EBV) and
elderly may be high and isolation is essential.
cytomegalovirus (CMV) would both need to be
Pneumonia due to M. bovis is TB (though most
considered and a likely diagnosis of EBV can be
is due to M. tuberculosis). Vancomycin resistant
quickly established by the Paul–Bunnell test.
enterococcal infections are more important
Acute CMV infection is confirmed by detection
than drug-sensitive infections, particularly as
of anti-CMV IgM. HIV seroconversion is likely
enterococcus is generally a pathogen of low
given the clinical picture and possible exposure.
virulence. Outbreaks of multidrug resistant
HIV antibody is often negative early during
Klebsiella can sweep through ITUs very quickly
seroconversion and the diagnosis can be
and to devastating effect. MRSA, which is
established by measuring P24 Ag or detection
resistant to glycopeptides such as vancomycin,
of high level viraemia with PCR. Acute hepatitis

Dr. Khalid Yusuf El-Zohry – Sohag Teaching Hospital (01118391123) Page | 462
El-Zohry MRCP Questions Bank (Part 2) – Medical Masterclass 2010

is a potentially untreatable organism and


intermediate-resistant isolates have already
been recognized in the UK.

[ Q: 26 ] MasterClass Part2
(2010) – Infectious disease
A 70-year-old man is receiving intravenous
antibiotics for severe sepsis. He has coexistent
renal impairment.

Which of the following antimicrobials does not TWO likely aetiological agents are:
require therapeutic drug level monitoring?
A. Streptococcus pyogenes
A. Gentamicin
B. Neisseria meningitidis
B. Linezolid
C. Escherichia coli
C. Amikacin
D. Neisseria gonorrhea
D. Vancomycin
E. Pseudomonas aeruginosa
E. Flucytosine
F. Enterococcus faecalis

Answer & Comments G. Candida albicans

Correct answer: B H. Staphylococcus aureus


I. Streptococcus pneumoniae
Linezolid is metabolised in the liver and is safe J. Salmonella typhi
in renal impairment. It does not require
therapeutic drug level monitoring. All other
Answer & Comments
agents may be toxic and require monitoring.
Correct answer: BD
[ Q: 27 ] MasterClass Part2
(2010) – Infectious disease The Gram stain shows Gram-negative diplococci
consistent with a neisseria species.
The image shows a Gram stain of pus aspirated
Both disseminated meningococcal and
from the knee of a 27-year-old man who
gonococcal infections can be complicated by
presented with an acute septic arthritis and a
acute septic arthritis and rash.
rash.
Meningococcaemia is usually accompanied by a
purpuric rash whereas the rash in disseminated
gonococcal infection classically consists of
discrete papules or pustules predominantly
found in the extremities. A third generation
cephalosporin, such as intravenous ceftriaxone,
is appropriate treatment for either condition.
Orthopaedic assessment is mandatory, as a
joint washout may be indicated.

Dr. Khalid Yusuf El-Zohry – Sohag Teaching Hospital (01118391123) Page | 463
El-Zohry MRCP Questions Bank (Part 2) – Medical Masterclass 2010

[ Q: 28 ] MasterClass Part2 blanching macular rash across the trunk. She is


(2010) – Infectious disease disorientated without meningism.

A 22-year-old woman presents with a fever and Which of the following diagnoses is most likely?
arthralgia. A. Salmonella enteritidis infection

Which of the following would not tie these TWO B. Pneumococcal meningitis
symptoms together? C. Anaphylaxis
A. Lyme disease D. Ecstasy overdose
B. Meningococcal sepsis E. Toxic shock syndrome.
C. Rubella
D. Measles Answer & Comments

E. Bacterial endocarditis Correct answer: E

F. Gonococcal infection
The presentation with fever, diarrhoea, shock
G. Yersinia infection and a macular rash are characteristic of
H. Brucellosis staphylococcal toxic shock syndrome due to
focal infection with a toxin-producing strain of
I. Parvovirus
S. aureus. Confusion, breathlessness (due to
J. Aspergillus infection. metabolic acidosis) and oliguria are commonly
present. The toxin acts as a superantigen
Answer & Comments bypassing the normal antigen-restricted
pathway of T-cell activation leading to
Correct answer: DJ widespread cytokine release shock and organ
failure. Approximately 50% of cases occur in
Reactive arthritis may follow gastrointestinal young women due to vaginal infection with S.
infections due to Salmonella, Shigella, aureus at the time of menstruation. A retained
Campylobacter or Yersinia as well as urethritis. vaginal tampon increases the risk and should be
Arthritis/arthralgia is a key feature of Lyme looked for in this type of presentation. A similar
disease, brucellosis, adult rubella and parvoviral syndrome may also be seen with toxin
infections and commonly occurs after 48-72 producing streptococci.
hours in patients recovering from
meningococcal sepsis and in patients with
[ Q: 30 ] MasterClass Part2
disseminated gonococcaemia. It is also well-
recognized in patients with bacterial
(2010) – Infectious disease
endocarditis (up to 30% in many case series). A 32-year-old 14/40 pregnant woman reports a
significant contact with a child with a fever and
[ Q: 29 ] MasterClass Part2 a rash.
(2010) – Infectious disease
Which of the following would you be concerned
A 26-year-old female presents with a short might have implications for her and more
history of confusion, diarrhoea and specifically her pregnancy?
breathlessness. On examination she is pyrexial A. Scarlet fever
at 38.5 C, pulse 120 bpm, Bp 80/60 and
respiratory rate 26 breaths/min. She has a faint B. Rubella

Dr. Khalid Yusuf El-Zohry – Sohag Teaching Hospital (01118391123) Page | 464
El-Zohry MRCP Questions Bank (Part 2) – Medical Masterclass 2010

C. Chicken pox [ Q: 31 ] MasterClass Part2


D. Mumps (2010) – Infectious disease
E. Impetigo A previously well 40-year-old woman presents
F. Measles to the Emergency Department with a 72-hour
history of increasing confusion and headaches.
G. Influenza She has no history of recent foreign travel and
H. Herpes simplex is not at high risk for HIV.

I. Toxoplasma gondii She is febrile at 38.5 C, and on examination is


drowsy with mild neck stiffness and her right
J. Listeria monocytogenes.
plantar is extensor in response. A CT brain
scan shows a low attenuation area in the left
Answer & Comments temporal region.
Correct answer: BC
What is the most likely diagnosis?

Any acute illness can be more severe in A. Meningococcal meningitis


pregnancy; however, only certain infections are B. Cerebral infarct
associated with congenital infection and/or
C. Herpes simplex encephalitis
congenital abnormalties. Of the listed
infections, only a few are associated with a D. Cerebral toxoplasmosis
childhood illness with a fever and a rash. E. Pyogenic brain abscess.
Maternal rubella invariably leads to fetal
infection in the first trimester causing severe Answer & Comments
defects such as cardiac abnormalities, deafness
and cataracts. There is a 15-12% risk of fetal Correct answer: C
infection between weeks 13-16, and after 20
weeks the risk is small. Herpes encephalitis presents with fever and
headache for several days, followed by
Maternal varicella infection can be associated
behavioural changes, seizure (40%),
with life-threatening complications such as
hemiparesis (33%) and a depressed level of
pneumonitis but can also lead to fetal infection
consciousness.
and abnormalities. The incidence of significant
fetal infection is highest between 13-20 weeks Infection occurs from exposure to infected
(2%). Complications include limb scarring, facial saliva or respiratory secretions with virus
lesions and developmental delay. ascending along the olfactory nerve into the
limbic lobe, or from reactivation of virus from
Although toxoplasma and genital herpes
the trigeminal ganglion. 90% of patients show
simplex infections have can be associated with
evidence of temporal lobe involvement.
specific complications for the foetus neither are
likely in this situation. Diagnostic test of choice is CSF PCR (ensure that
the lumbar puncture is not contra-indicated
Further information should be obtained about
because of mass effect). Sensitivity is 99%.
the child's illness, the exact nature of the
Treatment: aciclovir 10mg/kg q8h IV for 10-14
contact and maternal immunization and
days.
maternal childhood infection history. The case
should be referred to the microbiologist or
virologist for further advice.

Dr. Khalid Yusuf El-Zohry – Sohag Teaching Hospital (01118391123) Page | 465
El-Zohry MRCP Questions Bank (Part 2) – Medical Masterclass 2010

Having been previously well and not likely to be C. Intestinal amoebiasis


immunocompromised, other diagnosis can be
D. Giardiasis
excluded especially with CT findings.
E. Schistosoma mansoni.

[ Q: 32 ] MasterClass Part2
Answer & Comments
(2010) – Infectious disease
Correct answer: D
A 67 yr old man with chronic renal failure
(cause unknown) for which he receives regular
haemodialysis treatment has been admitted to Giardiasis presents with a small-bowel
the renal ward on many occasions with access diarrhoea, foul flatus and burping, bloating and
difficulties. He is now admitted with fever and nausea. It is not associated with bloody
malaise. Blood cultures grow a vancomycin- diarrhoea.
resistant enterococcus (VRE). It is important to remember that the majority
of problems a returning traveller will present
Which of the following drugs would be most with reflect underlying disease processes rather
suitable treatment? than exotic conditions acquired abroad; this is
A. Meropenem becoming increasingly important with an ageing
population travelling overseas.
B. Linezolid
C. Enterocid
[ Q: 34 ] MasterClass Part2
D. Tobramycin (2010) – Infectious disease
E. Septrin.
A 56-year-old man presents with a 5-day history
of fever and cough and has shadowing on his
Answer & Comments CXR consistent with community-acquired
Correct answer: B pneumonia.

Which of the following features is not a


VREs are an emerging problem in hospitals, recognised indicator of severe pneumonia?
where extensive glycopeptide use may select
A. Elevated urea
for resistant strains. Some are teicoplanin
sensitive. The new agents Linezolid and B. Age >55 years
Synercid may be active. Septrin is active against
C. Systolic BP <90 mmHg
some highly resistant gram negative bacteria
such as Stenotrophomonas maltophilia. D. Confusion
E. Respiratory rate >30 breaths/minute
[ Q: 33 ] MasterClass Part2
(2010) – Infectious disease Answer & Comments

Correct answer: B
Which of the following conditions are unlikely to
present with a 3-week history of loose bloody
stool in a traveller? The other features form part of the CURB score
(confusion, elevated urea, respiratory rate >30,
A. Ulcerative colitis BP 90 mmHg) for assessing the severity of
B. Colonic malignancy pneumonia. Age >65 years is a risk factor (CURB
65).

Dr. Khalid Yusuf El-Zohry – Sohag Teaching Hospital (01118391123) Page | 466
El-Zohry MRCP Questions Bank (Part 2) – Medical Masterclass 2010

[ Q: 35 ] MasterClass Part2 suspected to have pulmonary tuberculosis (TB).


(2010) – Infectious disease He has three sputum smears negative for acid
fast bacilli and is commenced on standard first
A HIV-positive patient with a CD4 count of 150 line anti-TB therapy. He continues to lose
is travelling to sub-Saharan Africa and requests weight on treatment and his cough persists.
vaccination advice.
Which of the following is true?
Which of the following would be
A. The diagnosis of TB is probably not
contraindicated?
correct.
A. Tetanus
B. He is not infectious to contacts.
B. Cholera
C. Carcinoma of the lung is a more likely
C. Meningococcal A/C vaccine diagnosis than TB.
D. Yellow fever D. A tuberculin skin test will not contribute
E. Hepatitis A. to making a diagnosis.
E. If he has received BCG it is highly unlikely
Answer & Comments that he has TB.

Correct answer: D F. It is essential to ask him about adherence


to the treatment regime.
Yellow fever vaccine is a live attenuated virus G. A CT scan of the thorax is mandatory.
and should be avoided in immunocompromised H. Drug susceptibility results are the single
individuals. Death from multi-organ failure has most important piece of data required.
been reported following yellow fever
vaccination to this group. I. Singing is a risk factor for TB.

The meningococcal A/C is a polysaccharide J. All contacts should receive isoniazid


vaccine, cholera and hepatitis A are both prophylaxis.
inactivated viral vaccines and tetanus is a
toxoid. If polio or typhoid vaccination is Answer & Comments
required then it is important to choose the
Correct answer: FH
correct formulation as both are available as live
and inactivated vaccines.
The most likely diagnosis by far is multi-drug
Proof of yellow fever vaccination is required by resistant tuberculosis (MDR-TB), although it is
some countries prior to entry. You may provide always important in patients failing to respond
your patient with a letter stating that they have to therapy to consider (a) whether the
a contraindication to vaccination, but this does treatment is being taken and (b) whether the
not guarantee entry. Advise against travel to treatment is correct. The treatment may be
any region in the midst of a yellow fever incorrect because the diagnosis is incorrect (in
outbreak. the case of smear-negative disease) or because
the infecting TB strain is resistant to some or all
[ Q: 36 ] MasterClass Part2 of the drugs used in the standard first line
(2010) – Infectious disease regimen (isoniazid, rifampicin, ethambutol,
pyrazinamide).
A 26-year-old singer from Estonia with weight
loss, fevers and cough for 2 months is In this age of international mobility it is
important to be aware of the ‘hotspots’ of

Dr. Khalid Yusuf El-Zohry – Sohag Teaching Hospital (01118391123) Page | 467
El-Zohry MRCP Questions Bank (Part 2) – Medical Masterclass 2010

certain diseases and MDR-TB is a particularly men. A history of previous hospital admissions
key example – Estonia, Latvia, and certain areas would increase the risk of MRSA infection,
of Russia and China have very high prevalences although community-acquired MRSA is also
of MDR-TB (37% of TB cases in Estonia for being reported with increased frequency. He is
example). also at higher risk of acquiring tuberculosis. The
smoking history puts him at increased risk of
The efficacy of BCG in preventing TB varies
lung neoplasm. Enterococcus faecalis does not
between geographical regions – conventional
usually cause cavitating lung lesions.
wisdom suggests that BCG protects against
miliary disease and TB meningitis in children.
Efficacy in preventing pulmonary disease in [ Q: 38 ] MasterClass Part2
adults is at best marginal in only a few studies. (2010) – Infectious disease
Smear-negative disease, whilst less infectious A 77-year-old man presents with sudden onset
than smear-positive disease, is believed to of weakness of his right arm on a background of
account for 15-20% of transmission. Singing is a 3-week history of thoracolumbar backpain,
not a risk factor for TB but it is a highly effective weight loss, fever and night sweats. Blood tests:
way to transmit the infection. Contacts of a full blood count (FBC) 9.8 g/dL, white cell count
patient with MDR-TB will obviously not derive (WCC) 12.0x109/l, platelets 450x109/l,
any benefit from isoniazid. erythrocyte sedimentation rate (ESR) 110
mm/hr, creatinine 180 micromol/L and
[ Q: 37 ] MasterClass Part2 microscopic haematuria.
(2010) – Infectious disease
What is the most likely diagnosis?
A 58-year-old alcoholic man presents to the
A. Spinal osteomyelitis
Emergency Department with a cough and fever.
He has a long history of smoking and is found to B. Myeloma
have a cavitating lesion on his CXR. C. Infective endocarditis

Which of the following is least likely to be the D. Mycobacterium tuberculosis infection


cause of his pneumonia? E. Tertiary syphilis
A. Meticillin-resistant Staphylococcus
aureus (MRSA) Answer & Comments
B. Klebsiella pneumoniae Correct answer: C
C. Enterococcus faecalis
Infective endocarditis is the only diagnosis that
D. Mycobacterium tuberculosis
would explain all the symptoms and results.
E. Lung cancer with secondary bacterial Vertebral osteomyelitis/discitis and stroke are
infection both recognised complications of infective
endocarditis and either can be the presenting
Answer & Comments feature of the disease.

Correct answer: C
[ Q: 39 ] MasterClass Part2
Cavitating pneumonia is associated with (2010) – Infectious disease
Staphylococcus aureus and Klebsiella You are called to see a 79-year-old urology
pneumoniae infection, particularly in alcoholic patient who is 6 hours post transurethral

Dr. Khalid Yusuf El-Zohry – Sohag Teaching Hospital (01118391123) Page | 468
El-Zohry MRCP Questions Bank (Part 2) – Medical Masterclass 2010

prostatectomy (TURP). He is afebrile, [ Q: 40 ] MasterClass Part2


hypotensive, hypoxic, vomiting and rigoring. (2010) – Infectious disease
Preoperative investigations included a chest
radiograph consistent with chronic obstructive A 54-year-old man presents with a headache
airway disease (COAD), a CSU showing profuse which came on over 3 weeks. He has mild neck
growth of mixed coliforms and urine stiffness. A CT of the head is reported as
methicillin-resistant staphylococcus aureus normal, and a lumbar puncture is performed.
(MRSA) screening was positive. Blood tests The opening pressure was 28 cm, and
showed mild renal impairment. laboratory investigations show 130 white cells
with 90% lymphocytes, and a protein of 1.3
Which TWO of the following are the least g/dl.
appropriate immediate management tasks?
Which of the following diagnoses are NOT
A. Oxygen administration via a facemask
typical for this picture?
B. Fluid resuscitation
A. Cerebral leukaemia
C. Commencement of oral ciprofloxacin
B. Listeria meningitis
500mg b.d.
C. Lyme disease
D. Commencement of intravenous
cefuroxime 1.5g t.d.s. with addition of D. Partially treated bacterial meningitis
500mg t.d.s. metronidazole E. Herpes simplex encephalitis
E. Commencement of intravenous F. Cerebral lymphoma
cefuroxime 1.5g t.d.s. with addition of
vancomycin 750mg b.d. G. Cryptococcal meningitis

F. Taking an ECG H. Cerebral sarcoid

G. Repeating a chest radiograph I. Tuberculous meningitis

H. Taking blood cultures J. Cerebral toxoplasmosis.

I. Placement on regular observations


Answer & Comments
J. Taking blood gases.
Correct answer: EJ

Answer & Comments


The diagnosis of lymphocytic meningitis is not
Correct answer: CD easy, and has a wide differential. The picture
described would be typical for tuberculous
This would be a typical picture of a Gram meningitis, but partially treated bacterial
negative or Staphylococcus aureus bacteraemia meningitis, cryptococcal meningitis (unlikely if
complicating surgical instrumentation. Often HIV negative) and several non-infectious causes
there is no febrile response until later. Any are possible. Listeria or Lyme need to be
antibiotic regimen should have good Gram- considered, but are very rare unless there are
negative but also MRSA cover given his clear signs pointing towards them. Cerebral
preoperative CSU. Other causes, such as a toxoplasmosis usually presents with fitting or
myocardial infarction, must be excluded and focal neurological signs, and the CSF is usually
standard resuscitation measures performed. normal with an abnormal CT scan. Herpes
simplex encephalitis usually presents with
reduced level of consciousness, fitting or focal

Dr. Khalid Yusuf El-Zohry – Sohag Teaching Hospital (01118391123) Page | 469
El-Zohry MRCP Questions Bank (Part 2) – Medical Masterclass 2010

signs, and the CSF picture (especially the high Which of the following statements is TRUE?
protein) would not be typical.
A. If otherwise well she could be treated as
an outpatient with oral quinine therapy
[ Q: 41 ] MasterClass Part2
B. The optimum treatment is IV quinine
(2010) – Infectious disease (20mg/kg) as a loading dose, followed by
A 48 yr old woman becomes febrile following a 10mg/kg tds as maintenance
blood transfusion. The fever does not settle, C. If seriously unwell, broad-spectrum
leading to consideration that the transfusion antibiotics should be used in addition to
may have transmitted infection. optimal antimalarial therapy

Which one of the following parasitic infections D. After acute treatment she should be
would NOT be transmitted by blood given a two-week course of primaquine
transfusion? to eliminate parasites from the liver

A. American trypanosomiasis E. The likely diagnosis is plasmodium vivax


infection due to poor compliance with
B. Leishmania antimalarial prophylaxis
C. Toxocara
D. Toxoplasma Answer & Comments

E. Malaria Correct answer: C

People chronically exposed to malaria develop


Answer & Comments
resistance and may have parasitaemia with
Correct answer: C little in the way of symptoms. Infections in
returning travellers should be treated seriously
There are case reports of transmission of as there is a significant risk of clinical
several parasitic infections by blood transfusion deterioration. Northern Thailand, Laos and
in non-endemic areas, and this should be Burma (Myanamar) have problems with quinine
considered if unexpected fever follows resistance and expert advice should be sought -
transfusion. combination therapy may be required. Always
Toxocara infection is spread to humans by consider whether patients could have other
ingestion of eggs in contaminated soil, which infections - if in doubt, cover with broad-
can occur whenever there are large populations spectrum antibiotics until they have been
of cats (T. catis) or dogs (T. canis) excluded. Primaquine is required for infection
with P. vivax or ovale to eliminate the liver
forms. A low platelet count and multiple ring
[ Q: 42 ] MasterClass Part2 forms are indicators of P. falciparum infection.
(2010) – Infectious disease
A 22-year-old female medical student presents [ Q: 43 ] MasterClass Part2
with a severe global headache, fever and (2010) – Infectious disease
sweats. She has recently returned from a
holiday in northern Thailand and has been A renal transplant patient develops fever and
taking mefloquine as antimalarial prophylaxis. haematuria.
Her full blood count reveals a moderately low
Which viral infection should be considered most
platelet count and 2% malarial parasitaemia
likely?
with some red cells having multiple ring forms.

Dr. Khalid Yusuf El-Zohry – Sohag Teaching Hospital (01118391123) Page | 470
El-Zohry MRCP Questions Bank (Part 2) – Medical Masterclass 2010

A. Polyoma J. Systemic lupus erythematosus (SLE).


B. Ebstein-Barr virus (EBV)
Answer & Comments
C. herpes simplex virus (HSV)
Correct answer: BI
D. Varicella zoster
E. Human herpes virus6 (HHV-6). All these could give a chronic fever, anaemia
and splenomegaly except:
Answer & Comments  Typhoid – fever generally resolves
within 4 weeks, although it can give
Correct answer: A
fever, splenomegaly and anaemia.
Polyoma virus BK is associated with interstitial  Schistosomiasis – S. mansoni and S.
nephritis, can cause fever and haematuria, but japonicum are causes of a massive
more commonly presents with impairment of spleen and secondary anaemia, but not
transplant function (rising creatinine). fever. Acute Schistosomiasis
Reduction of immunosuppression can reduce occasionally causes a fever (Katayama
viral replication. The other viruses listed can syndrome, generally with a rash) but no
reactivate during immunosuppression but do splenomegaly, which occurs with
not cause haematuria. Cytomegalovirus (CMV) chronic infection.
primary infection or reactivation is the most
Adult Still’s and Felty’s syndrome in rheumatoid
common and most feared infective
arthritis (RA) are well-recognized causes.
complication in the early post transplant
period.
[ Q: 45 ] MasterClass Part2
[ Q: 44 ] MasterClass Part2 (2010) – Infectious disease
(2010) – Infectious disease A 28-year-old fitness instructor develops fever,
jaundice and renal failure 10 days after
A 50-year-old man returns from the Sudan with
returning from a canoeing expedition in
chronic fever over 6 weeks. On examination he
Borneo. He reports that three others from the
is pyrexial and clinically anaemic with a large
expedition party of eight have recently had a
spleen. Haemoglobin is 7.8.
‘flu-like’ illness. It is suspected that he may have
Which of the following can NOT explain all leptospirosis.
these findings?
Which TWO of the following statements are
A. Malaria true?
B. Schistosomiasis A. A malaria blood film is mandatory
C. Tuberculosis B. Leptospiral infection can only be
D. Brucellosis confirmed by culture of blood or urine

E. Chronic lymphatic leukaemia C. If the diagnosis is leptospirosis,


supportive care is much more important
F. Leishmaniasis
than antibiotic therapy
G. Rheumatoid arthritis D. If his diagnosis is leptospirosis, it is
H. Adult onset Still's disease unlikely that his travel companions had
the same infection
I. Typhoid

Dr. Khalid Yusuf El-Zohry – Sohag Teaching Hospital (01118391123) Page | 471
El-Zohry MRCP Questions Bank (Part 2) – Medical Masterclass 2010

E. Conjunctival suffusion is diagnostic of Which of the following is NOT a likely cause of


Weil’s disease his genital ulceration?
F. Patient isolation is not necessary if he has A. Herpes simplex type 1 infection
not visited Africa B. Trichomonas vaginalis
G. Contact with water contaminated with
C. Haemophilus ducreyii
infected rat urine is unlikely to be the
cause D. Treponema pallidum

H. Leptospirosis is geographically limited to E. Chlamydia trachomatis infection.


tropical wetland areas
Answer & Comments
I. The incubation period of leptospirosis is 3-
5 days Correct answer: B
J. Chronic infection may be associated with
extensive intra-household transmission Trichomaonas vaginalis is a protozoan. It is a
very common sexually transmitted infection in
Answer & Comments the developing world, where it has been linked
closely to the acquisition of other genital
Correct answer: AC infections including HIV. Most men are only
mildly symptomatic with urethritis and it is not
An undiagnosed fever in any returning traveller a cause of genital ulceration. Chlamydia
from a malaria-endemic region demands a trachomatis species includes serovars L1, L2
malaria blood film regardless of whether a and L3 which cause Lyphogranuloma venereum.
second diagnosis is strongly suspected.
Serological testing is available for leptospirosis
though culture of the organism is used during [ Q: 47 ] MasterClass Part2
acute illness. Antibiotic therapy is often given, (2010) – Infectious disease
though there is controversy about the impact
An 84-year-old hospitalized woman receives co-
upon the natural history of disease. Leptospiral
amoxyclav treatment for a chest infection. After
infection can give rise to a wide range of clinical
5 days on the course she develops watery
presentations and given the similar
diarrhoea and a marked leucocytosis is noted. A
epidemiological exposure it is highly likely that
stool sample is positive for Clostrium difficile
his companions had the same infection – this
toxin.
would have been a common source outbreak
rather than human-to-human transmission. Which TWO of the following statements are
Conjunctival suffusion is a very useful clinical true regarding C. difficile diarrhoea?
sign but is not pathognomonic. There is no
chronic infectious disease state. A. Fluoroquinolones are more commonly
implicated than extended spectrum
penicillins.
[ Q: 46 ] MasterClass Part2
B. Advanced age is not a risk factor for C.
(2010) – Infectious disease
difficile infection.
A 25-year-man returns from a 3-week holiday in C. Enzyme immunassay is more sensitive
Thailand. He developed genital ulceration a few than cytotoxin assay using tissue culture.
days prior to his arrival back in the UK.
D. Most cases are due to toxin A.

Dr. Khalid Yusuf El-Zohry – Sohag Teaching Hospital (01118391123) Page | 472
El-Zohry MRCP Questions Bank (Part 2) – Medical Masterclass 2010

E. Oral vancomycin is significantly more Initial treatment involves stopping the


efficacious than oral metronidazole. offending antibiotic. However, if the diarrhoea
is severe, there is evidence of colitis or the
F. Intravenous metronidazole may be
original infection needs continuing antibiotics,
effective, whereas i.v. vancomycin is not.
then treatment with oral metronidazole or
G. Antiperistaltic agents (e.g. Lopermide) vanconycin should be started. They have similar
are safe. efficacy. If only i.v. antibiotics can be given,
H. Less than 5% of cases relapse. metronidazole will still result in effective
concentration of the drug in the colon.
I. Positive toxin assay after completion of
Antiperistaltic agents are contraindicated since
antibiotic course indicates need for a they may promote retention of toxin.
more prolonged course.
Unfortunately up to one quarter of cases will
J. Barrier nursing is not required to prevent
relapse. Another course in standard doses is
spread in hospitals. usually given before considering options such as
pulsed therapy. Repeat assays are not
Answer & Comments recommended after treatment since patients
may remain toxin-positive for some time.
Correct answer: DF
Recognized methods for C. difficile infection in
Risk factors for C. difficile infection are: hospitals are:

 antibiotic exposure  robust hand-washing

 advanced age  use of gloves

 hospitalization.  nursing patients in individual rooms

Overall however, only 10-20% cases of  restricted antibiotic policies.


antibiotic-associated diarrhoea are due to C.
difficile infection. Other causes include [ Q: 48 ] MasterClass Part2
accelerated gastric emptying, stimulated small (2010) – Infectious disease
bowel motility and infection with other enteric
pathogens. A 57-year-old woman develops a fever 7 days
after bone marrow transplantation. She is
Clindamycim, cephalosporins and penicillins are
placed empirically on broad-spectrum
most commonly indicated, with tetracyclines
antibiotics, but remains febrile. On the 11th day
and fluoroquinolones less so.
she develops a few painless, red, papular
There may be evidence of a colitis with fever lesions on her trunk and legs.
and marked leucocytosis. Endoscopy
characteristically shows pseudomembranes. What is the likely cause of these lesions?
These features suggest the condition but A. Aspergillus infection
diagnosis is generally now made by assays of C.
difficile toxin. Cytotoxin assays using tissue B. Candidal infection
culture are the most sensitive but take up to 48 C. Graft versus host disease
hours.
D. Recurrence of her haematological disease
Most laboratories offer enzyme immunoassay.
E. Staphylococcal infection
Most cases are due to toxin A.

Dr. Khalid Yusuf El-Zohry – Sohag Teaching Hospital (01118391123) Page | 473
El-Zohry MRCP Questions Bank (Part 2) – Medical Masterclass 2010

Answer & Comments of atopy and drug-related eosinophilia the most


likely cause in a person from the tropics is
Correct answer: B
invasive helminth disease (Strongyloides,
filarial, flukes, etc). Neither malaria nor
The persistence of fever, timing of occurrence amoebiasis are associated with eosinophilia.
and description of the lesions fits best with Malaria has no specific effects on the white cell
disseminated yeast infection, most likely count and amoebiasis is associated neutrophilia
Candida species. The diagnosis can usually be when invasive, though rarely with cyst carriage
made on blood cultures, but the organisms can alone. Other causes of eosinophilia include PAN
also be seen in the lesions on biopsy. Do not and malignancy.
forget to look in the fundi in such cases.
It would be too early for graft versus host [ Q: 50 ] MasterClass Part2
disease. (2010) – Infectious disease
A heterosexual woman has recently given birth.
[ Q: 49 ] MasterClass Part2
She has discovered that an ex-partner of her
(2010) – Infectious disease current sexual partner has chlamydial infection.
A 45-year-old male Tanzanian migrant is She is concerned about possible infection.
referred to outpatients with a persistent
Which of the following conditions or symptoms
eosinophilia of 1.3x109/l. His full blood count is
are not associated with Chlamydia trachomatis
otherwise normal. He is asymptomatic apart
infection?
from intermittent itching and reports no fevers.
He has been living in the UK for 9 months. A. Conjunctivitis
B. Scarring of the cornea
Which of the following would not be
appropriate in elucidating the cause? C. Epididymitis
A. Stool for ova, cysts and parasites D. Pneumonia in infants
B. Dermatological survey E. Pneumonia in adults
C. Strongyloides serology F. Genital itching
D. Blood film for malaria parasites G. Bartholinitis
E. Drug history H. Ureththral discharge
F. Filarial serology I. Perihepatitis
G. Amoebic serology J. Periappendicitis.
H. Serum cytoplasmic antibodies (ANCA)
Answer & Comments
I. Filarial blood film
Correct answer: EF
J. Urine for schistosome ova.

Chlamydia trachomatis serovars A, B and C


Answer & Comments
cause trachoma, the commonest worldwide
Correct answer: DG cause of blindness. Serovars D-K cause a variety
of genital syndromes. Serovars L1, L2 and L3
An eosinophil count of greater than 0.4x109/l cause lymphogranuloma venereum.
on repeat testing is significant. After exclusion

Dr. Khalid Yusuf El-Zohry – Sohag Teaching Hospital (01118391123) Page | 474
El-Zohry MRCP Questions Bank (Part 2) – Medical Masterclass 2010

Chlamydia pneumoniae is responsible for lymphocytes, protein 0.74 g/l and glucose 3.5
pneumonia in adults. mmol/l (plasma 5.6).

Which TWO of the following are most likely?


[ Q: 51 ] MasterClass Part2
(2010) – Infectious disease A. CMV encephalitis?
B. New variant Creutzfeldt–Jacob disease?
An intubated patient on the ICU for 8 days
following a road traffic accident has a persistent C. Cryptococcal meningitis?
fever and some lung shadowing. D. Cerebral lymphoma?

Which of the following organisms is most likely E. Depression?


to be involved? F. Neurosyphilis?
A. Streptococcus pneumoniae G. Progressive multi-focal
B. Staphylococcus epidermidis leucoencephalopathy?
C. Staphylococcus aureus H. Cerebral toxoplasmosis?
D. Pseudomonas aeruginosa I. HIV encephalopathy?
E. Legionella pneumophila. J. Tuberculous meningitis?

Answer & Comments Answer & Comments

Correct answer: D Correct answer: EI

Pseudomonas species are very common causes This presentation is typical of depression. HIV
of nosocomial pneumonia on the ICU. Strep encephalopathy may present with
pneumoniae is a very common cause of neuropsychiatric manifestations, focal
community-acquired pneumonia. neurology or dementia. Cerebral atrophy is
Staphylococcal disease due to methicillin- usually present but is also common in patients
resistant S. aureus (MRSA) or methicillin- with advanced HIV disease without clinical
susceptible S. aureus (MSSA) is also reasonably evidence of neurological involvement. MRI scan
common. Legionella is usually community- may show areas of increased signal but can also
acquired and can lead to ICU admission, but is be normal. There is a substantial increase in
now an uncommon nosocomial infection. psychiatric morbidity in patients with HIV/AIDS
and acute psychiatric illness may be
precipitated by a new HIV diagnosis.
[ Q: 52 ] MasterClass Part2
(2010) – Infectious disease Toxoplasmosis and lymphoma generally cause
space-occupying lesions seen best on MRI
A 42-year-old woman has recently been scanning. MRI abnormalities would also be
diagnosed HIV positive and has a CD4 count of expected in CMV encephalitis (unusual unless
180 and HIV viral load of 200 000 copies/ml. CD4 <75) and PML. nvCJD may present with a
She presents with behavioural change. She is depressive illness but this is a rare disease with
alert but withdrawn, uncommunicative, not no link to HIV. Cryptococcal and TB meningitis
eating and at times appears to be mute. CT will usually have a reduced CSF glucose and also
brain with contrast and MRI brain show a meningitis will not produce an alert but
moderate degree of cerebral atrophy but are withdrawn patient.
otherwise normal. CSF analysis reveals 20

Dr. Khalid Yusuf El-Zohry – Sohag Teaching Hospital (01118391123) Page | 475
El-Zohry MRCP Questions Bank (Part 2) – Medical Masterclass 2010

[ Q: 53 ] MasterClass Part2 some right upper quadrant tenderness. Blood


(2010) – Infectious disease tests: Na 141 mmol/L, K 3.9 mmol/L, urea 7.2
mmol/L, creatinine 111 micromol/L, bilirubin 48
A 42-year-old man presents with a 1-week micromol/L, alkaline phosphatase 500 IU/L,
history of fever and headache. A lumbar gamma-glutamyl transpeptidase (GGT) 220
puncture is performed, which reveals a IU/L, alanine aminotransferase (ALT) 80 IU/L.
lymphocytic infiltrate with raised protein and Arterial blood gases (breathing air) pH 7.28,
low glucose. PaO2 12.0 kPa, PCO2 3.2 kPa, base excess –11.9.
Chest radiograph normal. CD4 count 180
Which of the following is the least likely cause cells/mL, HIV viral load undetectable.
of his illness?
A. Listeria meningitis What is the most likely diagnosis?

B. Cryptosporidial meningitis A. Pneumocystis carinii pneumonia

C. Cerebral lymphoma B. Myocardial infarction

D. Mycobacterium tuberculosis C. Mycobacterium avium infection

E. Partially treated bacterial meningitis D. Lactic acidosis


E. Hepatitis C infection
Answer & Comments
Answer & Comments
Correct answer: B
Correct answer: D
Listeria and tuberculosis are both causes of
subacute lymphocytic meningitis and partially The undetectable HIV viral load implies that the
treated bacterial meningitis may present patient is receiving therapy. The blood tests
atypically with cerebrospinal fluid (CSF) point to a partially compensated metabolic
abnormalities including lymphocytosis. Fungal acidosis with abnormal liver function tests. This
infections, eg Cryptococcus, should be syndrome is seen with nucleoside reverse
considered, particularly if the patient is transcriptase inhibitors (AZT group of drugs)
immunocompromised. Cerebral lymphoma and and is thought to result from inhibition of
other intracranial malignancies may present as mitochondrial DNA. In addition to lactic
subacute meningitis with abnormal CSF, when acidosis, most patients have fatty infiltration of
cytology is usually helpful. Cryptosporidium is a the liver with a cholestatic pattern of liver
protozoan infection of the gastrointestinal tract enzymes. The initial clinical presentation is
that causes diarrhoea (which may be prolonged often non-specific with vague abdominal pain
and life-threatening in patients with AIDS), but and bloating. Alternative diagnostic possibilities
it does not cause meningitis. include bacterial sepsis and poisoning from
salicylate or methanol; but there is nothing else
[ Q: 54 ] MasterClass Part2 to suggest these. Lactic acidosis is one of the
most feared adverse effects of anti-HIV therapy
(2010) – Infectious disease
and may be fatal without prompt recognition.
An HIV-positive patient presents with Most patients gradually improve after
abdominal bloating and discomfort. He is withdrawal of anti-retroviral therapy, in severe
afebrile, with blood pressure 110/60, pulse 98, cases haemofiltration may be required to
respiratory rate 24/min. The chest is clear, control acidosis.
heart sounds normal, but palpation reveals

Dr. Khalid Yusuf El-Zohry – Sohag Teaching Hospital (01118391123) Page | 476
El-Zohry MRCP Questions Bank (Part 2) – Medical Masterclass 2010

[ Q: 55 ] MasterClass Part2 F. Loa Loa


(2010) – Infectious disease G. Ancylostoma duodenale
A 74 yr old woman with chronic leg ulceration H. Schistosoma mansoni
for which she has been admitted to hospital I. Taenia sagginatum
many times is admitted once again with fever
and malaise thought to be due to infection of J. Trichinella spiralis.
these ulcers. Swabs of the ulcer grow
Methicillin-resistant Staphylococcus aureus Answer & Comments
(MRSA).
Correct answer: CD
Which of the drugs listed can be used to treat
this condition? Eosinophils are part of our defence against
tissue invasion by multicellular organisms i.e.
A. Flucloxacillin
worms. Giardia lamblia is a single cell protozoa
B. Augmentin and so does not lead to eosinophilia. Enterobius
C. Vancomycin vermiculformis (thread worms or pin worms)
live entirely in the lumen of the gut and do not
D. Cefuroxime invade tissue. All of the other pathogens listed
E. Meropenem. have a phase of tissue invasion

Answer & Comments [ Q: 57 ] MasterClass Part2


Correct answer: C (2010) – Infectious disease
A 38-year-old man presents with breathlessness
MRSA is resistant to beta-lactam antibiotics. and cough. He is unwell, with high fever, and
Glycopeptides such as Vancomycin are has signs of consolidation in his right lower
commonly used. An alternative is Teicoplanin. lobe.
Other drugs such as gentamicin, rifampicin and
doxycycline may be active. The most likely pathogen is:
A. Legionella
[ Q: 56 ] MasterClass Part2 B. Neisseria meningitidis
(2010) – Infectious disease
C. Streptococcus pneumoniae
A 34-year-old city banker presents following a D. HIV
safari to Africa. He feels generally weak and
blood tests reveal a marked eosinophilia. E. Staphylococcus aureus.

Which of the following parasites would not Answer & Comments


present in this way?
Correct answer: C
A. Stronglyoides stercoralis
B. Ascaris lumbricoides Pneumococcus remains the commonest cause
C. Giardia lamblia of community acquired pneumonia. In giving
empirical treatment you would want to cover
D. Enterobius vermiculformis for the possibility of Legionella and other
E. Wucheria bancroftii atypicals (by giving clarithromycin or

Dr. Khalid Yusuf El-Zohry – Sohag Teaching Hospital (01118391123) Page | 477
El-Zohry MRCP Questions Bank (Part 2) – Medical Masterclass 2010

erythromycin) but these would be a less likely to stabilize the fracture. MRSA has been
cause of this man’s illness. isolated from surface swabs and is also likely to
be the causative agent of deeper infection.
[ Q: 58 ] MasterClass Part2
(2010) – Infectious disease [ Q: 59 ] MasterClass Part2
(2010) – Infectious disease
A 20-year-old man presents to a follow-up
outpatient clinic with severe pain in his right A 25-year-old HIV positive man presents with a
arm at the site of open reduction and internal 4-week history of bloody diarrhoea and weight
fixation of a humeral fracture. The wound has loss of 8 Kgs. He has had previous admissions to
broken down at the distal end and is oozing hospital with pneumocystis pneumonia but no
purulent fluid. He had been started empirically previous GI symptoms of note.
on flucloxacillin by his GP 2 days before. Wound
swabs taken at the GP surgery show a heavy Which TWO of the following diagnoses should
growth of Staphylococcus aureus, resistant to be considered?
penicillin, erythromycin, ciprofloxacin and A. Diverticular disease
flucloxacillin. A radiograph shows failure of
B. Ischaemic colitis
fracture healing.
C. Cytomegalovirus colitis
The most likely diagnosis is:
D. Colonic carcinoma
A. Superficial wound infection with
E. Irritable bowel syndrome
Methicillin-sensitive Staphylococcus
aureus F. Lactose intolerance
B. Deep seated Methicillin-sensitive G. Irritable bowel syndrome
Staphylococcus aureus infection including
H. Salmonella enteritis
infected internal metalwork
I. Malabsorption
C. Superficial wound infection with
Methicillin-resistant Staphylococcus J. Giardiasis.
aureus
D. Deep seated Methicillin-resistant Answer & Comments
Staphylococcus aureus infection including Correct answer: CH
infected internal metalwork
E. Fracture non-union. Cytomegalovirus colitis occurs in up to 10% of
patients with AIDS, and frequently presents
Answer & Comments with bloody diarrhoea and weight loss.
Salmonella is also common.
Correct answer: D
Lactose intolerance, irritable bowel syndrome,
Giardia and malabsorption may all present with
Given the picture of fracture non-union, wound
diarrhoea and weight loss, and are all
breakdown and purulent discharge the most
associated with HIV, but bleeding would be
likely scenario is deep-seated infection and
unusual.
contaminated metalwork. The gold standard
would be surgical exploration, debridement of Diverticular disease, ischaemic colitis and
infected material, removal of internal colonic carcinoma would all be unusual in this
metalwork and placement of an external fixator age group.

Dr. Khalid Yusuf El-Zohry – Sohag Teaching Hospital (01118391123) Page | 478
El-Zohry MRCP Questions Bank (Part 2) – Medical Masterclass 2010

[ Q: 60 ] MasterClass Part2 [ Q: 61 ] MasterClass Part2


(2010) – Infectious disease (2010) – Infectious disease
Blood drawn from a nurse immediately after a A previously well 55-year-old man is rushed to
needlestick injury on a Liver Unit is reported as the Emergency Department of a London
showing the presence of anti-HBc and anti-HBs hospital following a sudden collapse at work.
antibodies. He has no past medical history, excepting
splenectomy for a ruptured spleen following a
Which of the following is true? road traffic accident 10 years previously. On
A. She is at risk of acquiring hepatitis B from examination he is febrile (39.8°C) with evidence
the needlestick injury. of cardiovascular compromise (pulse 150 bpm
and BP 70/40 mmHg). In taking his history it
B. She may be at risk of acquiring hepatitis D
emerges that he was bitten by a dog 4 days
from the needlestick injury.
previously. Given his asplenia, it is thought that
C. She may have hepatitis D. he is septicaemic and urgent antibiotic
treatment is commenced pending results of his
D. She has received an adequately
blood cultures.
immunizing course of hepatitis B
vaccination.
Which of the following pathogens is likely to be
E. She has previously had and recovered responsible for his symptoms?
from hepatitis B. A. Capnocytophaga canimorsus (DF-2)
F. She should not be undertaking invasive B. Streptococcus suis
procedures.
C. Pseudomonas aeruginosa
G. She is at increased risk of cirrhosis.
D. Clostridium difficile
H. She is at risk of acquiring hepatitis C from
the needlestick injury. E. Staphylococcus aureus

I. She could have acquired antibodies from


the patient through the needlestick
injury. Answer & Comments
J. She should be advised to use barrier Correct answer: A
methods during intercourse to prevent
transmission of infection to her partner. In addition to the well-known risk of
overwhelming pneumococcal sepsis, asplenic
Answer & Comments patients are also prone to fulminant
septicaemia with other organisms such as
Correct answer: EH
Capnocytophaga canimorsus (previously known
as dysgonic fermenter, DF) and Streptococcus
Her serology demonstrates immunity acquired suis. Given that Capnocytophaga canimorsus is
through natural infection (anti-HBc antibodies found abundantly in canine saliva and that this
are not generated by vaccination) from which patient recently suffered a dog bite, this
she has fully recovered. She is therefore neither organism is most likely to be responsible for
infectious nor at risk of long term sequelae or this patient's septicaemia.
superinfection with the delta agent responsible
for hepatitis D.
[ Q: 62 ] MasterClass Part2

Dr. Khalid Yusuf El-Zohry – Sohag Teaching Hospital (01118391123) Page | 479
El-Zohry MRCP Questions Bank (Part 2) – Medical Masterclass 2010

(2010) – Infectious disease [ Q: 63 ] MasterClass Part2


A 48-year-old man presents on the medical take
(2010) – Infectious disease
with right lower lobe pneumonia. A 25 year-old soldier presents with a
rash on his leg following a tick bite in the New
In the acute assessment of an adult with Forest (a wooden are in the South East of
community-acquired pneumonia which one of
England, UK) a 2 weeks ago. The rash is
the following is NOT of prognostic importance?
spreading slowly in the form of a ring. He is
A. Urea >7mmol/l otherwise well.
B. Confusional state
What is the most useful clinical test to perform?
C. PaO2 <10kPa
A. Bartonella serology
D. Respiratory rate >30/min
B. Blood culture
E. Diastolic BP <60 mmHg.
C. Borellia serology
D. Rickettsial PCR
E. Skin biopsy
Answer & Comments

Correct answer: C Answer & Comments

Correct answer: C
The British Thoracic Society has recently
published guidelines for assessment and
management of adult community-acquired The patient is most likely to have Lyme disease,
pneumonia (Thorax 56, supplement 4). Adverse caused by Borrelia burgdorferi. The rash is
prognostic features include: acute disease and is termed erythema migrans.
Biopsy is not required. Serology should be
 Pre-existing factors:
positive and in this case IgM may also confirm
 Age > 50 years acute infection. Antibodies are slow to arise
 and may need to be repeated
Co-existing disease
 Core clinical adverse prognostic
[ Q: 64 ] MasterClass Part2
features (CURB criteria):
(2010) – Infectious disease
 Confusional state
A 38-year-old mother of two presents with a
 Urea > 7mmol/l
short history of headache, neck stiffness,
 Respiratory rate >30/min photophobia and fever. Her full blood count is
 Systolic BP <90mmHg and/or diastolic normal and her cerebrospinal fluid shows 50
BP < 60 mmHg white blood cells (all lymphocytes, normal <5),
with normal protein and glucose.
 Additional features:
 Hypoxaemia, SaO2 <92% or PaO2 <8kPa Which organism is the most likely cause of her
symptoms?
 Bilateral/multi-lobe disease
A. Enterovirus
Patients with two or more core features are at
B. Herpes simplex virus
high risk of death and should be managed as
severe pneumonia in hospital. C. Meningococcus

Dr. Khalid Yusuf El-Zohry – Sohag Teaching Hospital (01118391123) Page | 480
El-Zohry MRCP Questions Bank (Part 2) – Medical Masterclass 2010

D. Listeria [ Q: 66 ] MasterClass Part2


E. Mycobacterium tuberculosis (2010) – Infectious disease
A 26-year-old woman with epilepsy
Answer & Comments has been diagnosed with pulmonary
Correct answer: A tuberculosis and is about to be started on
quadruple therapy.

Enteroviral meningitis is the most likely. The following need to be discussed with her
Coxsackieviruses are common causes of except:
meningitis. Herpes simplex virus can cause this,
but more commonly causes an encephalitic A. Method of contraception
picture. B. The need for screening for ocular
complications with ethambutol
[ Q: 65 ] MasterClass Part2 C. Interactions with other medications
(2010) – Infectious disease D. Discoloration of urine due to isoniazid
A 25-year-old man presents with a 1-week E. Compliance.
history of genital ulceration. The ulcer is 1cm in
diameter.

Which of the following would exclude syphilis Answer & Comments


from the differential diagnosis?
Correct answer: D
A. History of a monogamous relationship for
the last 5 years
Isoniazid inhibits carbamazepine and phenytoin
B. Absence of lymphadenopathy and ethosuximide. Her anti-epileptic
medication will need reviewing. She should also
C. A negative venereal disease research
be reminded to check with a doctor or
laboratory (VDRL) test
pharmacist before starting other prescribed or
D. A negative dark ground microscopy test over the counter medications.
of specimens from the lesion
Rifampicin turns the body fluids including urine
E. A sexual history indicating no sexual orange, which can be alarming to a patient if
contact for over 2 years. they are not pre-warned. Methods of
contraception will be important for any sexually
Answer & Comments active female. Rifampicin is a hepatic enzyme
inducer. Accelerated metabolism leads to a
Correct answer: E
reduced contraceptive effect of both the
progesterone only and combined pill.
Over 50% of patients presenting with primary
Ethambutol has ocular toxicity and vision
syphilis have atypical clinical features. Classical
should be checked routinely at follow-up.
features of lymphadenopathy and painless
indurated lesions are absent. Serological tests
can be negative early on as can dark ground [ Q: 67 ] MasterClass Part2
microscopy. Chancres (primary and relapsing) (2010) – Infectious disease
are a feature of early syphilis (first 2 years after
acquisition). (1) A 68-year-old diabetic woman presents
three days following a penetrating injury to the

Dr. Khalid Yusuf El-Zohry – Sohag Teaching Hospital (01118391123) Page | 481
El-Zohry MRCP Questions Bank (Part 2) – Medical Masterclass 2010

right lower leg. She has an area of cellulitis MRSA is often sensitive to all of these
around the wound and signs of severe sepsis. antibiotics, excepting flucloxacillin. Rifampicin
Blood cultures grow a gram-negative rod and doxycycline may be used in combination in
subsequently identified as Pasteurella certain situations. Gentamicin can be added to
multocida. treat staphylococcal infections (e.g.
endocarditis) but is not usually used in cellulitis.
What was the most likely cause of the
penetrating injury?
[ Q: 69 ] MasterClass Part2
A. Insect bite (2010) – Infectious disease
B. Wood splinter
A 67 yr old man has significant heart valve
C. Snake bite disease.
D. Human bite
Which one of the following procedures does
E. Dog bite. NOT place him at significant risk of bacteraemia
and hence would not require antibiotic
Answer & Comments prophylaxis for infective endocarditis?

Correct answer: E A. Tooth descaling


B. Biopsy of an ulcerated skin lesion
Pasteurella multocida is found in the mouths of C. Cystoscopy
both dogs and cats with human infection
resulting from bites. Soft tissue infection results D. Sclerotherapy of oesophageal varices
and may progress to tenosynovitis, E. Root canal filling
osteomyelitis or lymphangitis depending on the
site of the bite. Bacteraemia with severe sepsis,
endocarditis, meningitis and pneumonia have
also been reported. Therapy is with penicillin. Answer & Comments

Correct answer: B
[ Q: 68 ] MasterClass Part2
(2010) – Infectious disease Skin biopsies and dental fillings above the gum-
line do not require prophylaxis, but all other
Which would be the best antibiotic regime to dental procedures do.
use in a patient who has spreading methicillin-
resistant Staphylococcus aureus (MRSA) [ Q: 70 ] MasterClass Part2
infection of a skin wound?
(2010) – Infectious disease
A. Flucloxacillin
An 80-year-old woman is admitted with a
B. Gentamicin history of falls to a general ward. After 3 days
C. Vancomycin she develops severe vomiting. Other patients
and members of staff are also affected.
D. Rifampicin
E. Doxycycline. What is the most likely organism?
A. Clostridium difficile
Answer & Comments
B. Norovirus
Correct answer: C

Dr. Khalid Yusuf El-Zohry – Sohag Teaching Hospital (01118391123) Page | 482
El-Zohry MRCP Questions Bank (Part 2) – Medical Masterclass 2010

C. Rotavirus D. HIV encephalopathy


D. Campylobacter E. Progressive multifocal
leucoencephalopathy
E. Helicobacter
F. Herpes simplex encephalitis
G. Cerebrovascular disease
Answer & Comments H. Tuberculosis
Correct answer: B I. Bacterial meningitis
J. Neurosyphilis.
Noroviruses are very common causes of
vomiting illnesses in hospitals. The other agents
Answer & Comments
can all cause gastrointestinal disease, but not in
this clinical context. Correct answer: CH

[ Q: 71 ] MasterClass Part2 The CT shows multiple contrast enhancing


(2010) – Infectious disease lesions. These are typical of the focal lesions of
tuberculosis (tuberculomas) or toxoplasmosis.
A 36-year-old HIV positive male presents with a Contrast enhancement may appear as a ring
grand mal seizure and a Glasgow coma score of (abscess) or central as in this case. Multiple
six. A contrast enhanced CT scan of the brain is bacterial brain abscesses, metastatic
shown (see image). malignancy or multifocal lymphoma may have a
similar appearance.
Toxoplasma encephalitis is the most likely
diagnosis in this situation and in 10-15% of
patients, toxoplasma serology is negative. Such
a patient would normally be commenced on a
therapeutic trial of pyrimethamine and
sulfadiazine for toxoplasma encephalitis. If
there was no improvement after two weeks,
brain biopsy may be appropriate and the most
likely diagnosis would be primary CNS
lymphoma.

[ Q: 72 ] MasterClass Part2
(2010) – Infectious disease
A 22-year-old woman has recently been
diagnosed with a genital infection. She is
reluctant to involve her partner with whom she
Which of the following are most likely? last had sexual contact 2 weeks ago.

A. Multifocal cerebral glioma Infection with which of the following agents


B. Cytomegalovirus encephalitis does not require contact tracing (finding and
treating infected sexual partners) to ensure the
C. Cerebral toxoplasmosis
prevention of reinfection of the index case?

Dr. Khalid Yusuf El-Zohry – Sohag Teaching Hospital (01118391123) Page | 483
El-Zohry MRCP Questions Bank (Part 2) – Medical Masterclass 2010

A. Trichomonas vaginalis
B. Giardia lamblia
C. Scabies mite
D. Chlamydia trachomatis
E. Pediculosis pubis
F. Herpes simplex virus type 2
G. Human papilloma virus type 6
H. Gardnerella vaginalis
I. Neisseria gonnorhoea
J. Treponema pallidum. Which of the following diagnoses is correct?
A. Active tuberculosis
Answer & Comments
B. Carcinoma of the bronchus
Correct answer: FG C. Pulmonary infarction
D. Aspergillus infection
Genital viral infections will generate protective
immunity that will not be compromised by E. Bacterial pneumonia.
subsequent re-challenge. Contact tracing is
similarly not needed to prevent reinfection in
bacterial vaginosis or vaginal candidal infection
Answer & Comments
where vaginal discharge results from the
temporary alteration of the local vaginal flora. Correct answer: D
In these cases although it may be indicated for
other reasons the treating partners is unhelpful The patient has scaring and cavitation from old
for ensuring long term cure in the index cases. tuberculosis. The lesion in the right apex is a
fungal ball within and existing pulmonary cavity
[ Q: 73 ] MasterClass Part2 – an aspergilloma. The diagnosis can be
(2010) – Infectious disease confirmed by CT scanning and growth of
aspergillus from respiratory specimens. Massive
A 74-year-old patient previously treated for haemoptysis in a person with known cavitary
pulmonary tuberculosis presents with cough lung disease is highly suggestive on the
and a 250 ml haemoptysis. The chest development of an aspergilloma.
radiograph is shown (see image). Aspergillomata respond poorly to antifungal
therapy. Definitive therapy is surgical and
bronchial artery embolization may be used to
stop haemorrhage.

[ Q: 74 ] MasterClass Part2
(2010) – Infectious disease
An asplenic traveller presents to you for advice
on prophylaxis for a trip to Ghana, West Africa.

Dr. Khalid Yusuf El-Zohry – Sohag Teaching Hospital (01118391123) Page | 484
El-Zohry MRCP Questions Bank (Part 2) – Medical Masterclass 2010

Which of the following would you not Which of the following diagnostic possibilities is
recommend? least likely?
A. Tetanus booster A. Japanese B encephalitis
B. Yellow fever vaccine B. Herpes simplex encephalitis
C. MMR (measles, mumps, rubella) C. Tuberculosis meningitis
D. Meningococcal vaccination D. Vivax malaria
E. Dengue vaccination E. Falciparum malaria
F. Antimalarial prophylaxis
Answer & Comments
G. Typhoid vaccination
Correct answer: D
H. Hepatitis A vaccination
I. Pneumococcal vaccination
Japanese B encephalitis can be acquired by
J. Rabies vaccination. travellers to rural areas of South-east Asia, but
is extremely rarely seen in the UK. Herpes
Answer & Comments simplex encephalitis is a devastating infection
that is treatable if intravenous aciclovir is given
Correct answer: CE early in the course of the disease. Tuberculosis
meningitis is an important differential in
Opinion about the relative merit of some of someone who has travelled to an area of high
these vaccinations varies but MMR would not prevalence. Cerebral malaria due to
be recommended specifically and there is no Plasmodium falciparum infection needs to be
vaccine available for dengue. excluded urgently with a blood film.
Plasmodium vivax infection is less serious and
Asplenic patients are at particular risk from
does not cause cerebral malaria.
malaria (and babesiosis) as well as infections
due to encapsulated organisms such as
Streptococcus pneumoniae, Haemophilus [ Q: 76 ] MasterClass Part2
influenzae and Neisseria meningitidis. Response (2010) – Infectious disease
to vaccination in asplenic patients is impaired
but not useless and should not be an A 74-year-old diabetic presents with impaired
impediment to vaccination. Prophylactic rabies consciousness and fever. Head CT scan is
vaccination provides a ‘headstart’ to a traveller normal. Cerebrospinal fluid (CSF) analysis is as
bitten by a rabid animal but in no way obviates follows: Opening pressure 28 cm/CSF, 1240
the essential need for wound toilet and post- WBC/micro.l which are 100% lymphocytes,
exposure vaccination. protein 1.8 g/dl and CSF glucose of 4.2 m.mol/l
compared to plasma glucose of 11.6 m.mol/l.

[ Q: 75 ] MasterClass Part2 Which of the following are most likely?


(2010) – Infectious disease A. Viral meningitis
A 25-year-old woman backpacker presents with B. Cryptococcal meningitis
fever and reduced conscious level shortly after
C. Listeria meningitis
returning from Thailand.
D. Meningococcal meningitis
E. Cerebral infarction

Dr. Khalid Yusuf El-Zohry – Sohag Teaching Hospital (01118391123) Page | 485
El-Zohry MRCP Questions Bank (Part 2) – Medical Masterclass 2010

F. Herpes simplex encephalitis  carniomatous meningitis


G. Cerebral toxoplasmosis  cerebral vasculitis.
H. Diabetic coma
[ Q: 77 ] MasterClass Part2
I. Tuberculous meningitis
(2010) – Infectious disease
J. Staphylococcal meningitis.
A 25-year-old man with a history of intravenous
Answer & Comments drug use is found on screening to have an
alanine aminotransferase (ALT) of 65 U/L
Correct answer: BI (normal <45).

The CSF findings are those of a lymphocytic What is the most likely infectious cause of his
meningitis with a very high CSF protein and low abnormal liver function?
CSF glucose (CSF: plasma ratio <0.4). This is A. Hepatitis A
typical of tuberculous and fungal meningitis.
B. Hepatitis B
Cryptococcal infection is most common in
C. Hepatitis C
patients with cell-mediated immunodeficiency,
particularly HIV infection, but may also be seen D. Hepatitis E
in diabetics and patients with chronic hepatic or E. Epstein-Barr virus
renal disease.
Viral central nervous system (CNS) infections
rarely cause a low CSF glucose (with the
Answer & Comments
exception of mumps), and the protein and cell
count are a little high for herpes simplex Correct answer: C
encephalitis.
Toxoplasmosis may cause a lymphocytic CSF Hepatitis C virus infection is very common in
but can usually be seen on contrast CT scan. this risk group and commonly presents with the
sole abnormality of a moderately raised ALT.
Other infectious causes of a lymphocytic CSF
and low CSF glucose include:
[ Q: 78 ] MasterClass Part2
 partially treated bacterial meningitis
(2010) – Infectious disease
 mumps meningitis
A 34-year-old intravenous drug user presents
 parameningeal focus of infection with a persistent fever and shortness of breath.
 secondary syphilis His radiograph shows bilateral discrete lesions.
Blood cultures taken on admission flag positive
 Lyme meningitis
after 24 hours. The Gram stain is shown (see
 brucellosis image).
 coccidiodomycosis
 histoplasmosis
 leptospirosis.
Non-infectious causes include:
 sarcoidosis

Dr. Khalid Yusuf El-Zohry – Sohag Teaching Hospital (01118391123) Page | 486
El-Zohry MRCP Questions Bank (Part 2) – Medical Masterclass 2010

How should she be treated?


A. Varicella Zoster immune globulin
B. Steroids
C. Acyclovir
D. Painkillers only
E. Immediate delivery of the child.

Answer & Comments

Correct answer: C
What is the organism and what is the most
likely underlying diagnosis? Acyclovir is safe in pregnancy and chickenpox is
A. Staphylococcus aureus dangerous. Steroids exacerbate chickenpox.
The immunosuppression of pregnancy puts the
B. Streptococcus bovis
mother as well as the fetus at risk. In early
C. Candida albicans pregnancy there is a risk of fetal abnormalities
D. Pneumocystis carinii (about 2%). At this late stage the main danger is
that a new born child would become infected
E. Mycobacterium tuberculosis with no transfer of antibody from the mother.
F. HIV infection In that case VZig should be given to the child,
but it has no role in therapy of the mother.
G. Left-sided endocarditis
H. Right-sided endocarditis
[ Q: 80 ] MasterClass Part2
I. Miliary tuberculosis (2010) – Infectious disease
J. Pneumonia.
A 40-year-old nun who has been working as a
nurse in a hospital in rural Nigeria returns to
Answer & Comments the UK. She is sent by the airport doctor
Correct answer: CH because she collapsed. She gives a history of
headache, fever, conjunctivitis and sore throat,
and looks unwell.
The clinical picture can be explained by a
diagnosis of right-sided endocarditis with Which of the following are true?
secondary septic pulmonary emboli. The Gram
stain shows a candidal species. Candidal A. Malaria is unlikely to be the cause.
endocarditis is seen in the intravenous drug B. She should be isolated and discussed with
using population where organism is a consultant before any blood is taken.
inadvertently introduced as a contaminant of
C. Influenza is unlikely to be the cause.
the intravenous cocktail.
D. Viral haemorrhagic fever (VHF) is unlikely
to be the cause.
[ Q: 79 ] MasterClass Part2
(2010) – Infectious disease E. If she had viral haemorrhagic fever she
should be admitted to your ITU.
A woman who is 36 weeks pregnant presents
with chickenpox.

Dr. Khalid Yusuf El-Zohry – Sohag Teaching Hospital (01118391123) Page | 487
El-Zohry MRCP Questions Bank (Part 2) – Medical Masterclass 2010

F. Medical students should be encouraged should (by law) be cared for in designated
to take a history. centres with full isolation facilities; general ITUs
do not have these facilities.
G. If she tries to leave you should detain her
under the Public Health Act. If the patient is mentally competent, doctors
and nurses have no legal powers to detain them
H. Viral haemorrhagic fever can present up
even if they have proven VHF (or any other
to 2 months after leaving an endemic
infectious disease), although public health
area.
should be informed. Detention under the Public
I. Viral haemorrhagic fever is more Health act needs a magistrate.
infectious than measles.
J. Ebola and Lassa fevers are almost [ Q: 81 ] MasterClass Part2
invariably fatal. (2010) – Infectious disease

Answer & Comments An 18-year-old man presents with a 24-hour


history of fever and malaise. He is confused,
Correct answer: BD breathless and hypotensive. There is no travel
history. The image shows his feet.
The management of those returning to the UK
from rural Africa with a fever within 21 days of
leaving an area endemic for viral haemorrhagic
fever is governed by statute (not just
guidelines). Most infectious disease physicians
think that the regulations are over the top and
difficult to apply, but they are there to protect
health workers and should be followed. In
practice viral haemorrhagic fever (VHF)
imported to the UK is extremely rare, whilst
malaria is commoner in rural Africa than
influenza is in the UK and both are several
hundred times more common than VHF, so
these diagnoses are far more likely.
However, if a patient has returned from rural What are the TWO most likely diagnoses?
Africa within 21 days (infections are very rare
after that), and especially if they have been A. Idiopathic thrombocytopenic purpura
nursing sick patients in rural Africa, senior B. Meningococcal septicaemia
advice should be sought before any blood
C. Rocky mountain spotted fever
specimens are taken. This lady has symptoms
which would certainly be in keeping with a D. Acute leukaemia with thrombocytopenia
diagnosis of VHF. Contacts should be kept to a E. Henoch Schonlein purpura
minimum until they have been classified as low
risk, and a list of all contacts kept. It is F. Toxic shock syndrome
important not to overstate the risk, however: G. Microscopic polyangiitis
only close contact with body fluids presents
H. Bacterial septicaemia with disseminated
serious hazard. It is also important not to delay
intravascular coagulation
treatment of potentially treatable diseases such
as malaria. Proven (or likely) cases of VHF I. Pneumococcal pneumonia

Dr. Khalid Yusuf El-Zohry – Sohag Teaching Hospital (01118391123) Page | 488
El-Zohry MRCP Questions Bank (Part 2) – Medical Masterclass 2010

J. Legionnaire’s disease. Advanced human immunodeficiency virus (HIV)


disease can present as an acute clinical
Answer & Comments problem. Persistant genital ulceration in a high
risk individual (this man had had syphilis in his
Correct answer: BH 20s) should raise the suspicion of
immunodeficiency. Ulcers when greater then
This man has meningococcal septicaemia until 2cm in diameter and if present for over 1
proven otherwise. month are indicative of extremely advanced
Management should be: disease. Such lesions will often intially respond
to aciclovir therapy but relapse is common and
1. Check ABC
in time occurs with resistant herpes simplex
2. Give high flow oxygen via reservoir bag virus. Managing the underlying HIV infection is
3. Call for assistance from ICU the most important aspect of care and early
initiation of triple antiretroviral therapy will be
4. Establish IV access, taking blood for of benefit.
culture and laboratory tests as you do
so
[ Q: 83 ] MasterClass Part2
5. Give colloid or saline as fast as possible (2010) – Infectious disease
to restore blood pressure
You are called to see a 78-year-old man who is
6. Whilst giving fluid fast, give cefotaxime
deteriorating rapidly following surgery for
1-2 g intravenously.
gallstones. He suffers from hypotension,
pyrexia, tachypnea and falling urine output.
[ Q: 82 ] MasterClass Part2
(2010) – Infectious disease Which of the following statements about severe
sepsis do you agree with?
A 35-year-old hospital pharmacist presents with
A. Severe sepsis accounts for 10% of all
a 6 week history of perianal soreness and
critical care admissions in England, Wales
ulceration, which he has not had before. He has
and Northern Ireland.
been in a monagamous relationship for 8 years,
and his partner has a history of genital herpes. B. No drugs are recommended by NICE for
He has a past history of syphilis in his early the treatment of severe sepsis.
twenties - current blood results show a TPHA -
C. Severe sepsis kills almost half the patients
positive, VDRL - negative.
affected.
What is the most likely diagnosis? D. Drotrecogin alpha (activated ) is a
A. Primary genital herpes infection recombinant IL-6 inhibitor.

B. Syphilitic chancre E. If the patient survives severe sepsis,


sequelae are rare.
C. Carcinoma of the anus
D. Human immunodeficiency virus infection Answer & Comments
with herpes simplex virus ulceration
Correct answer: C
E. Crohn's disease.
Severe sepsis accounts for over a quarter of
Answer & Comments admissions to critical care. Despite advances in
critical care, almost half of those patients die
Correct answer: D

Dr. Khalid Yusuf El-Zohry – Sohag Teaching Hospital (01118391123) Page | 489
El-Zohry MRCP Questions Bank (Part 2) – Medical Masterclass 2010

and those who survive often suffer permanent [ Q: 85 ] MasterClass Part2


organ damage. Drotrecogin alpha (activated) (2010) – Infectious disease
has been recommended by NICE for treatment
of adult patients with severe sepsis resulting in A 56-year-old man becomes unwell 2 days after
the failure of two or more organ systems. It is a returning from holiday in Spain. He is
recombinant activated protein C, but its main breathless, complains of pleuritic chest pain,
mechanism of action clinically appears to be and has a temperature of 39oC. A chest
anti-inflammatory radiograph shows patchy consolidation in both
lung fields.
[ Q: 84 ] MasterClass Part2 Which of the following is the most appropriate
(2010) – Infectious disease management?
(2) A 27 yr old man has just undergone an A. IV antibiotics after blood cultures are
emergency splenectomy following a motorcycle taken
accident. B. IV antibiotics after a sputum sample is
obtained
Which one of the following measures has NOT
been proven effective in reducing infection in C. Subcutaneous low molecular weight
splenectomised individuals? heparin before CT pulmonary
angiography
A. Meningococcal vaccine
D. IV diuretic after 12 lead ECG
B. Pneumococcal vaccine
E. Nebulized bronchodilators after peak
C. Daily penicillin V
expiratory flow rate (PEFR).
D. Interferon gamma infusion
E. Splenic remnant re-implantation Answer & Comments

Correct answer: A
Answer & Comments

Correct answer: D The history and presentation are suggestive of


Community Acquired Pneumonia, most
All splenectomised patients should be probably related to his recent travel. This raises
vaccinated against pneumococcus, the possibility of penicillin resistance and
meningococcus and haemophilus either before Legionella as confounding factors. Treatment
elective splenectomy, or as soon as possible should be commenced immediately after blood
after emergency splenectomy. It is cultures are obtained and not delayed to obtain
recommended that all patients receive yearly sputum specimens. A starting combination of
influenza vaccination and daily penicillin V (or co-amoxiclav or cefotaxime / ceftriaxone /
erythromycin) prophylaxis. There is some cefuroxime plus a macrolide to cover likely
evidence that reimplantation of a splenic organisms is indicated.
remnant in non-malignant cases may be of
some benefit. Despite documentation of low [ Q: 86 ] MasterClass Part2
interferon gamma levels in splenectomised (2010) – Infectious disease
patients there is no evidence that
supplementation reduces infection risk. A 37-year-old woman presents with fever and
jaundice two weeks after a trip to the Indian
subcontinent to visit her parents.

Dr. Khalid Yusuf El-Zohry – Sohag Teaching Hospital (01118391123) Page | 490
El-Zohry MRCP Questions Bank (Part 2) – Medical Masterclass 2010

What are the TWO most likely diagnoses? B. Botulism


A. Hepatitis C C. Brucellosis
B. Malaria D. Listeria
C. Hepatitis A E. E coli 0157
D. Leptospirosis
Answer & Comments
E. Amoebic liver abscess
Correct answer: B
F. Typhoid
G. Fascioliasis
Botulism is associated with poor home-
H. Relapsing fever preserved and bottled food, due to bacterial
contamination and toxin production. Wound-
I. Yellow fever
associated botulism is a well-recognised
J. Cytomegalovirus problem in intravenous drug abusers.
Other infections associated with unpasteurised
Answer & Comments milk include bovine TB, Salmonella,
Correct answer: BD Campylobacter and Staphylococcal food
poisoning.
Always consider malaria in a febrile traveller
who has or who may have been exposed in the [ Q: 88 ] MasterClass Part2
last 6 months. Acute viral hepatitis is a likely (2010) – Infectious disease
explanation for jaundice after travel but fever
usually precedes the onset of jaundice. A 78 yr old man has been ventilated on ICU for
Furthermore it is likley that this patient was 10 days following surgical repair of a ruptured
born in the Indian sub-continent and may well abdominal aortic aneurysm. His respiratory
be hepatitis A immune. Leptospirosis, although function is deteriorating and it is thought that
uncommon needs to be considered in the he has developed a ventilator-associated
differential diagnosis. Also be sure to consider pneumonia.
more cosmopolitan but potentially serious
causes such as cholecystitis or ascending Which of the following drugs might be most
cholangitis. Note that space occupying lesions suitable for treatment?
within the liver, such as bacterial or amoebic A. Benzyl penicillin
abscess, rarely present with jaundice.
B. Cefuroxime
C. Augmentin
[ Q: 87 ] MasterClass Part2
(2010) – Infectious disease D. Vancomycin

A 23 yr old student presents with a fever a few E. Piperacillin/tazobactam (Tazocin).


days after returning from a camping holiday
during which he drank unpastuerised milk from
a local farm. Answer & Comments

Which one of the following is NOTassociated Correct answer: E


with consumption of unpasteurised milk?
A. Q fever

Dr. Khalid Yusuf El-Zohry – Sohag Teaching Hospital (01118391123) Page | 491
El-Zohry MRCP Questions Bank (Part 2) – Medical Masterclass 2010

Important organisms to consider in this context [ Q: 90 ] MasterClass Part2


are resistant gram negative rods, including (2010) – Infectious disease
pseudomonas. The only drug with such cover in
this list is tazocin, but other anti-pseudomonal A 17-year-old caucasian woman presents with
regimens may be suitable. A and B cover post-coital bleeding of some three months
community acquired pneumonias but have standing. She has recently noticed an increase
weaker gram negative cover. in vaginal discharge. Her menstral cycle has
remained regular (she is on an oral
contraceptive), but menstral loss has increased.
[ Q: 89 ] MasterClass Part2
(2010) – Infectious disease On examination she has a marked cervicitis.
Direct microscopy of Gram-stained cervical
A report from abroad on an HIV-positive patient specimens shows the presence of Gram-
transferred to your care says that they have negative intracellular diplococci, consistent with
also tested positive for hepatitis G virus (GBV- a diagnosis of gonorrhoea.
C).
What other infection is she most likely to have?
Which of the following statements about
A. HIV 1
Hepatitis G virus (GBV-C) is true?
B. Syphilis
A. The virus is associated with progressive
liver disease. C. Chlamydial infection
B. It is closely related to hepatitis B virus. D. Group B streptococcal vaginitis
C. It is likely to worsen the patient's HIV E. Trichomoniasis
disease.
D. It is readily treated with a range of Answer & Comments
antivirals.
Correct answer: C
E. No further investigations or treatment
are needed. Co-infection of sexually transmitted agents is
common. Whenever one sexually transmitted
agent is found others should be looked for.
Although any two agents can occur together, in
Answer & Comments
a developed Western setting approximately
Correct answer: E 25% of those infected with Neisseria
gonorrhoeae will have chlamydial infection.
Hepatitis G virus or GBV-C does not appear to This is high enough to warrant the automatic
cause progressive liver disease, unlike its requirement for chlamydial therapy in any
relative, hepatitis C virus. Co-carriage of HGV individual with proven gonoccal disease.
appears to improve the progression of HIV.
There is no other clinically significant impact of [ Q: 91 ] MasterClass Part2
HGV known to date, and no large scale (2010) – Infectious disease
treatment trials. Blood products are not
screened for this virus in the UK. A 32-year-old woman, who might be pregnant,
recently returned from Africa. She is febrile and
drowsy with a Plasmodium falciparum malaria
parasite count of 1%, haemoglobin of 9.8 g/dl,

Dr. Khalid Yusuf El-Zohry – Sohag Teaching Hospital (01118391123) Page | 492
El-Zohry MRCP Questions Bank (Part 2) – Medical Masterclass 2010

platelets of 20, creatinine of 200 and mild intramuscular. Hypoglycaemia is a common


jaundice. complication in severe malaria, especially in
pregnant women, and those who are drowsy
Which of the following are essential parts of her should have their glucose measured repeatedly.
management?
A. CT of her head [ Q: 92 ] MasterClass Part2
B. Platelet transfusion (2010) – Infectious disease
C. Blood glucose monitoring (2) A 25-year-old woman has a history of well-
D. Exchange transfusion controlled systemic lupus erythematosus (SLE)
and is currently on a maintenance low dose of
E. Dialysis prednisolone. She presents with severe dysuria
F. Liver ultrasound that has been getting worse for seven days, and
she also has myalgia and headache.
G. Prophylactic phenobarbitone
Trimethoprim was prescribed three days ago
H. Steroids and she has taken the tablets, but they have
I. Antimalarial treatment given not helped. Today she has started to notice
intravenously some pharyngeal pain on swallowing. She is in a
monogamous relationship of some four years
J. Antibiotic cover. standing and her partner is asymptomatic. A
picture of her genital lesions is shown (see
Answer & Comments image).

Correct answer: CI

This woman has several pointers to potentially


severe disease – she is drowsy, jaundiced and
pregnant.
Even where patients have severe cerebral
malaria, CT seldom adds anything to diagnosis,
and jaundice is common in malaria due to red-
cell breakdown. Low platelets are almost
invariable in malaria, and require no treatment
unless a patient is bleeding. Prophylactic
antibiotics and phenobarbitone are seldom
helpful, even in severe cases. Steroids are
actively unhelpful. Exchange transfusion should
only be contemplated in those with very high
peripheral parasite loads (10% would be a
minimum, usually more – always ask for
specialist advice).
The key to managing malaria is an appropriate
antimalarial given early, and this takes priority
over all else. In potentially severe cases this
should always be parenteral and as she has low
platelets, intravenous is preferable to What is the most likely diagnosis?

Dr. Khalid Yusuf El-Zohry – Sohag Teaching Hospital (01118391123) Page | 493
El-Zohry MRCP Questions Bank (Part 2) – Medical Masterclass 2010

A. Stevens-Johnson syndrome Answer & Comments


B. Herpes zoster Correct answer: E
C. Behcet's disease
The classical triad of urethritis, arthritis and
D. Herpes simplex
conjunctivitis forms the commonest
E. Candidal infection. components of Reiter’s syndrome. The
syndrome can be triggered by infections in any
Answer & Comments system and is most commonly seen after
gastrointestinal infections. Patients with
Correct answer: D
HLAB27 are particularly prone to the syndrome.
Circinate balanitis, planter fasciitis and
Genital herpes simplex infection occurs in up to
keratoderma blennorrhagicum ( a nonfollicular
one in four women in the UK. Patients are
pustular eruption involving the soles) are also
frequently in monogamous relationships.
commonly seen. Very severe cardiac,
Partners are usually asymptomatic. Patients
cardiovascular and neurological problems may
frequently present with dysuria in the early
also develop, but rarely.
stages. Typically the dysuria is external.
Ten percent of patients will develop
[ Q: 94 ] MasterClass Part2
extragenital lesions - typically the hands and
mouth may be involved. (2010) – Infectious disease
A 21-year-old chef is admitted with a five day
[ Q: 93 ] MasterClass Part2 history of bloody diarrhoea, fever and
(2010) – Infectious disease confusion. He is febrile, normotensive, has a
pulse rate of 110, and has a soft diffusely
A 24-year-old man had urethritis some 2 weeks tender abdomen. Blood results: haemoglobin
ago. This was treated with antibiotics but has 7.2 g/dL, white cell count 22, platelets 20,
not settled. He has now developed a range of creatinine 380 mmol/L.
symptoms and has been found to be
HLAB27+ve. Which one of the following statements is NOT
true:
Which of the following clinical signs would not
A. A blood film may show fragmented red
fit with a clinical diagnosis of Reiter’s
blood cells.
syndrome?
B. This syndrome could be caused by
A. Tenderness over the planter surface of
Escherichia coli or shigella infection.
the ankle
C. The diagnosis has implications for the
B. Conjunctivitis
patient’s return to work.
C. Pustular lesions on the soles of the feet
D. There is an animal reservoir for the
D. A well demarcated rash on the penis with infection.
serpiginous edges
E. Most cases seen in the UK occur in
E. A mild fever with a generalised macular association with outbreaks.
rash.
Answer & Comments

Correct answer: E

Dr. Khalid Yusuf El-Zohry – Sohag Teaching Hospital (01118391123) Page | 494
El-Zohry MRCP Questions Bank (Part 2) – Medical Masterclass 2010

The chef has hemolytic uraemic syndrome H. Mumps


(HUS), most likely as a consequence of E. coli
I. Ebstein-Barr virus
0157:H7 infection.
J. Influenza.
HUS is a triad of renal failure,
thrombocytopaenia and microangiopathic
haemolytic anaemia +/- central nervous system Answer & Comments
symptoms (TTP). E. coli O157:H7 and Shigella Correct answer: HJ
dysenteriae type I are the organisms firmly
associated with the syndrome, though E. coli
Plague (caused by Yersinia pestis) and Cat
accounts for 85-95% of infective cases. The
Scratch Disease (caused by Bartonella henselae)
overall rate of HUS with a case of E. coli
are characterized by lymphadenopathy. The
O157:H7 infection with bloody diarrhoea is only
cervical swelling in mumps is parotid
5-10%.
enlargement and regional lymphadenopathy is
Cattle have been shown to be a reservoir of surprisingly rare. Uncomplicated influenza is
infection, and people whose occupation brings not associated with lymphadenopathy.
them into contact with farm animals are at
increased risk. Most cases in the UK are [ Q: 96 ] MasterClass Part2
sporadic in nature but outbreaks have been
(2010) – Infectious disease
associated with contaminated meat and dairy
products. A 28-year-old Afro-Caribbean female presents
The infection has public health implications, with a 1-week history of increasingly confused
particularly in this situation when the patient is behaviour. Investigations include a lumbar
a food handler. He should be excluded from puncture, which reveals 30 lymphocytes, no
work until two consecutive negative faecal neutrophils and protein 0.6 g/l.
specimens taken after recovery have been
Which of the following possible causes would be
obtained.
unlikely to cause the above cerebrospinal fluid
(CSF) picture?
[ Q: 95 ] MasterClass Part2
A. Parameningeal infection
(2010) – Infectious disease
B. Sarcoidosis
A patient presents with palpable
C. Herpes simplex encephalitis
lymphadenopathy.
D. Partially treated bacterial meningitis
Which of the following diagnoses need not be
E. Leptospirosis
considered as the likely cause?
F. Listeria meningitis
A. Yersinia pestis infection
G. sytemic lupus erythematous (SLE)
B. TB
H. HIV seroconversion illness
C. Bartonella henselae infection
I. Non-steroidal anti-inflammatory drug
D. Sarcoidosis
(NSAID) use
E. Syphilis
J. Enteroviral meningitis.
F. Toxoplasmosis
G. African trypanosomiasis

Dr. Khalid Yusuf El-Zohry – Sohag Teaching Hospital (01118391123) Page | 495
El-Zohry MRCP Questions Bank (Part 2) – Medical Masterclass 2010

Answer & Comments [ Q: 98 ] MasterClass Part2


Correct answer: DF (2010) – Infectious disease
A 20-year-old woman has recently given birth
While the clinical presentation would be to a well full-term baby delivered vaginally.
possible with any of the above, both partially Some 5 days after delivery she is told by her
treated bacterial meningitis and listeria regular sexual partner that he has recently been
meningitis more commonly give a CSF picture of treated for genital chlamydial infection. The
increased lymphocyte:neutrophil ratio rather doctor looking after her partner has advised
than pure lymphocytosis. Care must always be him to ensure that the baby is taken to a doctor
taken to determine whether antibiotics have for treatment of possible chlamydial infection.
been administered and are clouding the classic
CSF features of infection. This is because if untreated the baby may
develop all of the following EXCEPT:
[ Q: 97 ] MasterClass Part2 A. Neonatal opthalmia
(2010) – Infectious disease B. Sinusitis
A 24-year-old woman returns to the UK after a C. Pneumonia
7-day visit to a hypothetical region endemic for
D. Failure to thrive
the 5 diseases listed below. She has a fever that
began on the 6th day of travel. E. Pelvic inflammatory disease/
epididymitis.
Which of the following diseases can you
EXCLUDE as a cause of her fever with Answer & Comments
confidence?
Correct answer: E
A. Falciparum Malaria
B. Dengue Babies delivered through an infected genital
C. Yellow fever tract are susceptible to a variety of severe
manifestations of bacterial and viral STIs.
D. Leptospirosis
Infective agents can become established in any
E. Pneumococcal pneumonia mucous membrane site. Inhalation can lead to
upper and lower respiratory tract problems.
Answer & Comments Late presentations of sinusitis and failure to
thrive have been described.
Correct answer: A
Upper genital tract infection in neonates does
not occur.
Even if the traveller was bitten by a malaria-
infected mosquito on the first day of her visit,
the life cycle demands at least 8 days before [ Q: 99 ] MasterClass Part2
the first symptoms appear, thus excluding (2010) – Infectious disease
malaria. Dengue shows in 3-8 days, yellow fever
A 30-year-old woman presents with bilateral
in 3-6 days and leptospirosis in 5-14 days.
painful and swollen interphalyngeal joints and a
transient rash. She had a mild diarrhoeal illness
4 weeks previously but is otherwise well. She is
a mother of two small children the eldest of

Dr. Khalid Yusuf El-Zohry – Sohag Teaching Hospital (01118391123) Page | 496
El-Zohry MRCP Questions Bank (Part 2) – Medical Masterclass 2010

whom had a recent illness characterized by Although varicella infection can have major
fever and red cheeks. implications in pregnancy, the vaccine - and all
other live vaccines - should be avoided during
What is the most likely diagnosis? pregnancy.
A. Reiter's syndrome Inactivated and toxoid vaccines can be given
B. Rubella infection during pregnancy but are best withheld until
after the first trimester.
C. Acute parvovirus B19 infection
D. First presentation of rheumatoid arthritis [ Q: 101 ] MasterClass Part2
E. Adult-onset Stills disease. (2010) – Infectious disease
A 45-year-old man presents with right-sided
Answer & Comments
lower chest discomfort, shoulder-tip pain, dry
Correct answer: C cough and low-grade fever 2 weeks after
returning from a 3-month visit to relatives in
The child is likely to have had parvoviral India.
infection, which is characterized by a ‘slapped
Which TWO of the following would be NOT be
cheek’-like appearance. Adults often present
helpful in providing a diagnosis?
with fever, rash and arthritis as manifestations
of parvoviral infection. A. Thick and thin blood films for malaria

Reiter's syndrome is a triad of arthritis, B. Chest radiograph


conjunctivitis and urethritis but a similar C. Blood cultures
syndrome can occur after gastrointestinal
infections. Adult Still’s disease is rare and a D. Arterial blood gases
diagnostic challenge because of the lack of E. Amoebic serology
pathognomonic features.
F. Ventilation/perfusion scan
G. Widal test
[ Q: 100 ] MasterClass Part2
(2010) – Infectious disease H. CT thorax
I. Abdominal ultrasound
A 28 yr old woman is 23 weeks pregnant.
J. Stool for ova, cysts & parasites.
Which one of the following vaccines would be
unsafe for her?
Answer & Comments
A. Hepatitis A
Correct answer: GJ
B. Pneumococcus
C. Influenza The clinical picture is consistent with the
diagnosis of an amoebic liver abscess. A chest
D. Inactivated polio
radiograph may reveal a raised right hemi-
E. Varicella diaphragm and possibly reactive changes in the
right lung base. Amoebic serology has a
Answer & Comments sensitivity and specificity of greater than 95%
for amoebic liver abscess. Abdominal
Correct answer: E
ultrasound may be useful in confirming the

Dr. Khalid Yusuf El-Zohry – Sohag Teaching Hospital (01118391123) Page | 497
El-Zohry MRCP Questions Bank (Part 2) – Medical Masterclass 2010

abscess and assessing the likelihood of rupture The following would be unlikely to contribute
into surrounding viscera. Given the travel significantly to the evaluation of this patient:
history malaria must be excluded, though the A. A concise description of the nature of the
presentation is not typical. rash and photographs if available
The patient is still within the risk period for
B. A self-filled temperature chart
deep venous thrombosis and pulmonary
embolus following a long-haul flight. C. Genitourinary examination
D. Lifetime travel history
[ Q: 102 ] MasterClass Part2 E. HIV test
(2010) – Infectious disease
F. Drug history
Two days after returning from a 1-week trip G. Initial therapeutic trial of ciprofloxacin
around Thailand, a 25-year-old woman presents
with sudden onset of fever, headache and H. Echocardiogram even if clinical
severe myalgia. Three days after her symptoms examination is normal
started she develops a generalized I. Ventilation-perfusion scan
erythematous rash. Her Hb is 12g/dl, WCC
J. Abdominal imaging.
2.1x109/l and platelets 65x109/l.

What is the most likely diagnosis? Answer & Comments


A. Plasmodium vivax malaria Correct answer: GH
B. Typhoid fever
It is important to establish that the symptoms
C. Paratyphoid fever
described correspond to a pathological fever –
D. Dengue fever the first step is to obtain a self-recorded
E. Tick-borne encephalitis. temperature diary. Fever periodicity may
provide clues, e.g. to a benign malaria or
familial Mediterranean fever. The evanescent
Answer & Comments
rashes of adult Still’s disease or erythema
Correct answer: D chronicum migrans are highly characteristic.
The rash may, or may not, be part of the same
This is the typical presentation of someone with clinical condition causing the fever – if initial
dengue fever which has an incubation period of focus on detection of conditions causing both
5 to 8 days. Both malaria (usually falciparum in fails to yield results then the search should be
Thailand) and typhoid have to be excluded and broadened to include conditions not necessarily
the diagnosis of dengue is confirmed associated with rash.
serologically. Prostatitis or prostatic abscess, pelvic
inflammatory disease and tuberculous GU
[ Q: 103 ] MasterClass Part2 infection are often overlooked. Therapeutic
trials may be necessary after an exhaustive
(2010) – Infectious disease
unsuccessful search for the cause of a
A 20-year-old banker from Senegal reports continuing fever but need to be planned, well
fever and occasional rash dating back several thought out and directed at addressing specific
months. possible causes. Otherwise they simply muddy
the waters further.

Dr. Khalid Yusuf El-Zohry – Sohag Teaching Hospital (01118391123) Page | 498
El-Zohry MRCP Questions Bank (Part 2) – Medical Masterclass 2010

Echocardiography has low yield in the absence widely used treatment once the diagnosis has
of physical signs, though most work-ups include been confirmed.
it. In contrast, V/Q scans not infrequently reveal
Further information should be obtained about
the diagnosis, and abdominal imaging to
the nature of this man’s job. Though less likely,
discover occult sepsis is mandatory.
anthrax is also a possibility. Cases have been
reported in postal workers in the USA and those
[ Q: 104 ] MasterClass Part2 who work with contaminated hides and leather.
(2010) – Infectious disease Intravenous ciprofloxacin is the recommended
antibiotic, though mortality at this stage of
A 49-year-old factory worker presents unwell infection is high
with a fever and confusion. He is hypotensive
and hypoxic. The most notable finding on
clinical examination is a necrotic skin lesion on [ Q: 105 ] MasterClass Part2
his back. He is transferred to intensive care (2010) – Infectious disease
given supportive management and broad-
A 40-year-old man is found to be Hepatitis B
spectrum antibiotics and the surgical team
surface antigen and e antigen positive. He says
consulted. The next day blood cultures flag
he was told he had acute hepatitis B infection
positive with Gram-positive rods.
10 years before when he became jaundiced.
Liver biopsy reveals moderate hepatic
Which TWO organisms might you be concerned
about? inflammation and fibrosis.

A. Group A Streptococcus Which TWO of the following drugs could be


used singly in his treatment?
B. Staphylococcus aureus
A. Zidovudine
C. Clostridium tetani
B. Ritonavir
D. Listeria monocytogenes
C. Ribavirin
E. Bacteroides fragilis
D. Interferon gamma
F. Pseudomonas aeruginosa
E. Interferon alpha
G. Bacillus anthracis
F. Efavirenz
H. Clostridium perfringens
G. Lamivudine
I. Diptheroids
H. Acyclovir
J. Escherichia coli.
I. Interleukin-2
Answer & Comments J. Azithromycin.
Correct answer: GH
Answer & Comments
The most likely diagnosis is gas gangrene with Correct answer: EG
Clostridial infection. Bacteraemia complicates
about 15% of patients with gas gangrene.
Interferon alpha and lamivudine are both
Treatment includes emergency surgical
individually licensed for treatment of chronic
exploration and debridement. The combination
hepatitis B. Two other drugs not currently
of intravenous penicillin and clindamycin is
licensed that have potent activity against

Dr. Khalid Yusuf El-Zohry – Sohag Teaching Hospital (01118391123) Page | 499
El-Zohry MRCP Questions Bank (Part 2) – Medical Masterclass 2010

chronic hepatitis B include adefovir and Mycobacterium leprae cannot be cultured in


tenofovir. vitro. This patient has tuberculoid leprosy,
which is paucibacillary and characterised by a
[ Q: 106 ] MasterClass Part2 strong cell mediated immunity response.
Leprosy is transmitted by airborne droplet
(2010) – Infectious disease
spread of nasal secretions from patients with
A 45-year-old Nepalese trader is found to have multibacillary disease and is not highly
three anaesthetic pigmented macules on his contagious through physical contact as is widely
trunk during routine investigation of peripheral misconceived. Contact tracing is not usually
vascular disease. A skin biopsy is suggestive of fruitful partly because the rate of disease in
leprosy. exposed persons is low and partly because the
time lapse from infection to disease may be
Which TWO of the following statements are many years
true?
A. Culture of the biopsy would be helpful in [ Q: 107 ] MasterClass Part2
establishing the diagnosis of (2010) – Infectious disease
leprosyCulture of the biopsy would be
helpful in establishing the diagnosis of A 32-year-old male presents with fever, dry
leprosy. cough and breathlessness. Resting oxygen
saturation on air is 97% and his chest
B. Ziehl–Nielsen stain of the biopsy is likely
radiograph was reported as normal. This
to be positive if the diagnosis is leprosy.
appearance of his tongue was noted (see
C. The patient should be managed in image).
isolation until the diagnosis is established
or excluded.
D. Thickened nerves may be palpated on
physical examination.
E. Screening of the patient’s household
contacts is mandatory.
F. Recovery of any neural damage is to be
expected early in therapy.
G. Rifampicin is a key component of leprosy
treatment.
H. Leprosy is spread predominantly through
physical contact with infectious cases.
I. Leprosy has been eradicated from South
America.
What is the most likely diagnosis?
J. Response to therapy is predictable from
baseline assessment. A. Pneumocytsis carinii pneumonia
B. Q fever
Answer & Comments C. Legionnaire’s disease
Correct answer: DG D. Infective endocarditis
E. Pulmonary candidiasis.

Dr. Khalid Yusuf El-Zohry – Sohag Teaching Hospital (01118391123) Page | 500
El-Zohry MRCP Questions Bank (Part 2) – Medical Masterclass 2010

Answer & Comments I. Leishmaniasis is the cause of sleeping


sickness in Africa.
Correct answer: A
J. Recrudescence of the lesion after
There is a white patch on the side of the tongue apparent treatment cure is common.
indicating oral hairy leukoplakia (OHL). OHL is
almost pathognomonic of underlying Answer & Comments
immunodeficiency due to HIV infection. OHL is
Correct answer: BG
generally seen when the CD4 count is less than
350 and is due to local Ebstein Barr virus
South American cutaneous leishmaniasis is
infection of the skin. Pneumocytsis carinii
pneumonia (PCP) is one of the commonest transmitted by sandflies (as are all
causes of breathlessness in patients with a CD4 leishmaniases). Patients are therefore non-
infectious, except to sandflies (direct person to
count of less than 200. The presentation
person spread does not occur). Diagnosis of
described is typical of early PCP when the
leishmaniasis without speciation (as in the vast
patient may still have a normal chest
majority of cases worldwide) can be achieved
radiograph and only becomes hypoxic on
exercise. by smear of a lesion and microscopy or by
histological examination of a margin biopsy.
Spontaneous resolution does occur and is
[ Q: 108 ] MasterClass Part2 probably common in endemic populations.
(2010) – Infectious disease Treatment will depend on speciation (if
available this is usually by PCR) but usually will
A 19-year-old student complains of an irritating
involve parenteral treatment for several weeks
crusting lesion on his face that has been
with sodium stibogluconate or a similar agent.
increasing in size for 6 weeks and has been
This is important as potentially devastating
present since he returned from a 3-week
mucocutaneous relapse can lead to destructive
holiday in the jungles of Belize. It is thought
naso-facio-oral lesions which are much more
likely that he has developed cutaneous
difficult to treat. Sleeping sickness is African
leishmaniasis.
trypanasomiasis and Kala-azar is visceral
Which of the following are true? leishmaniasis which is caused by 'old world'
species of Leishmania and is mainly seen in
A. He must have been bitten by an infected Africa.
mosquito whilst in Belize.
B. The lesion might resolve spontaneously. [ Q: 109 ] MasterClass Part2
C. He should be nursed in isolation. (2010) – Infectious disease
D. Topical treatment will suffice. A 54-year-old woman received an allogeneic
E. The diagnosis cannot be confirmed by bone marrow transplant for acute myeloid
microscopy of a slide scraped over the leukaemia. Eight weeks post transplant she
edge of the lesion. complains of blurred vision and the lesion
shown in the image is seen on fundoscopy.
F. The diagnosis requires PCR.
G. Biopsy and histology of the lesion would
aid in diagnosis.
H. Without treatment he is at risk of Kala-
azar.

Dr. Khalid Yusuf El-Zohry – Sohag Teaching Hospital (01118391123) Page | 501
El-Zohry MRCP Questions Bank (Part 2) – Medical Masterclass 2010

[ Q: 110 ] MasterClass Part2


(2010) – Infectious disease
A 48 yr old man presents with arthritis
following a diarrhoeal illness.

Which one of the following enteric infections is


LEAST LIKELY to be associated with reactive
arthritis?
A. Shigella spp
B. Campylobacter spp
C. E. coli
Which of the following is the likely diagnosis? D. Yersinia spp
A. Cytomegalovirus retinitis E. Salmonella spp
B. Bacterial endophthalmitis
Answer & Comments
C. Reactivation of toxoplasmosis
D. Leukaemic deposits in the retina Correct answer: C

E. Candida endophthalmitis.
Reactive arthritis is also well described after C.
difficile infection, as well as all its associations
Answer & Comments with sexually-transmitted infections
Correct answer: C (Chlamydia, gonococcus, ureaplasma).

The image shown includes a black scar on the [ Q: 111 ] MasterClass Part2
retina typical of old toxoplasma (2010) – Infectious disease
choroidoretinitis. Such scars are often found as
incidental findings on fundoscopy and indicate A 30-year-old man presents with dual hepatitis
a risk for subsequent reactivation. In this case B virus (HBV) and HIV infection. These were
there are also exudates within the old scar plus acquired some time in the past. He is HBV
a new lesion (underneath old scar) with florid surface antigen positive and e antigen
exudate indicating reactivation of positive/e antibody negative. He is on no
toxoplasmosis as a consequence of intense therapy currently. He is clinically stable.
immunosuppression.
Which drug would be active against both
None of the other diagnostic options given will infections?
have black areas. CMV retinitis typically consists
A. Lamivudine
of haemorrhage plus exudates (cheese and
tomato pizza appearance). Candida appears as B. Zidovudine
white lesions extending from the retina into the C. Stavudine
vitreous. Bacterial endophthalmitis generally
has signs of inflammation throughout the eye. D. Interferon alfa
E. Ribavirin

Dr. Khalid Yusuf El-Zohry – Sohag Teaching Hospital (01118391123) Page | 502
El-Zohry MRCP Questions Bank (Part 2) – Medical Masterclass 2010

Answer & Comments [ Q: 113 ] MasterClass Part2


Correct answer: A (2010) – Infectious disease
A 25-year-old girl presents to renal transplant
Lamivudine is active against HIV, given in clinic with fever, diarrhoea and worsening renal
combination with other antiretrovirals. function. She is 7 weeks post live-related-donor
Lamivudine is also active against HBV. Other transplantation. Her initial course had been
dually active reverse transcriptase inhibitors complicated by an episode of acute rejection
include adefovir. HBV possesses a reverse which responded to treatment with anti-
transcriptase even though it is not a classical thymocyte globulin (ATG). She is currently
retrovirus. maintained on prednisolone, cyclosporin and
azathioprine and has just finished a course of
[ Q: 112 ] MasterClass Part2 oral ciprofloxacin for an E. coli urinary tract
infection. Blood tests reveal a mild hepatitis
(2010) – Infectious disease
and leucopenia.
A 45-year-old female presents with low grade
fevers, cramping abdominal pains and bloody The most likely diagnosis is:
diarrhoea shortly after returning from a A. Clostridium difficile diarrhoea
business trip to Mexico City.
B. E. coli bacteraemia
Which of the following would be inappropriate? C. Cytomegaloviral infection
A. Blind treatment with ciprofloxacin D. Cyclosporin toxicity
B. Rehydration with oral rehydration E. Graft-versus-host disease.
solution
C. Use of loperamide to reduce stool Answer & Comments
frequency
Correct answer: C
D. IV fluids if unable to maintain oral
hydration
Seven weeks post transplantation would be
E. Notification of Public Health authorities. classic timing for presentation of
cytomegaloviral (CMV) infection. It is
Answer & Comments commonest 1–3 months post transplantation
and the risk of severe infection is increased by
Correct answer: C the use of immunosuppressants such as ATG. A
number of complications can occur with CMV
Loperamide is contraindicated in acute bloody infection including pneumonitis, colitis,
diarrhoea as it increases the risk of toxic hepatitis, retinitis and neurological
megacolon associated with both ulcerative manifestations. Leucopenia and
colitis and bacterial colitis. The likely microbial thrombocytopenia is also common. Diagnosis
cause of bloody diarrhoea in this situation is can be made by detection of CMV antigen or
Shigella and treatment with ciprofloxacin is polymerase chain reaction (PCR) positivity from
appropriate before culture results are available. blood. Tissue biopsy can also show inclusion
Episodes of food poisoning and dysentery, bodies.
whether acquired in the UK or overseas, are
subject to statutory notification by the [ Q: 114 ] MasterClass Part2
managing physician.

Dr. Khalid Yusuf El-Zohry – Sohag Teaching Hospital (01118391123) Page | 503
El-Zohry MRCP Questions Bank (Part 2) – Medical Masterclass 2010

(2010) – Infectious disease [ Q: 115 ] MasterClass Part2 (2010) –


A mother reports that her 6-year-old daughter
Infectious disease
was bitten on the leg 6 weeks earlier by a A 32-year-old HIV positive woman with a CD4
monkey whilst on holiday in Thailand. count of 100 presents with breathlessness and
abdominal pain. Six weeks earlier she had been
The correct action is: commenced on didanosine, stavudine and
A. reassurance – there is minimal risk of nevirapine combination anti-HIV therapy. On
rabies examination she has a respiratory rate of
24/min, chest is clear and there is epigastric
B. administer both anti-rabies
tenderness. Investigations reveal alanine
immunoglobulin and commence rabies
aminotransferase (ALT) 115 iu/ml, GGT 680
vaccination
iu/ml, bilirubin 24 micro.mol/l. Arterial blood
C. reassurance plus antibiotics to prevent gases on air: Ph 7.15, PaO2 11.4 Kpa, PaC02 2.8
local infection Kpa and base excess -15.
D. commence a course of rabies vaccine
What are the TWO most likely diagnoses?
E. consider post-exposure prophylaxis for
A. Diabetic ketoacidosis
hepatitis B.
B. Hypersensitivity to nevirapine
Answer & Comments C. Pneumocystis carinii pneumonia (PCP)
Correct answer: B D. Duodenal ulceration
E. Acute pancreatitis
Thailand is considered high risk for rabies and
F. Drug overdose
post-exposure prophylaxis is indicated if
significant exposure has occurred. Vaccination G. Lactic acidosis
alone is insufficient, particularly when
H. Cytomegalovirus (CMV) enteritis
treatment has been delayed as in the case given
above. Anti-rabies immunoglobulin can be I. Pyelonephritis
obtained from the Public Health Laboratory J. Immune reconstitution syndrome.
Service (PHLS). Fifty per cent of the dose is
infiltrated around the wound with the rest
Answer & Comments
being given intramuscularly at a separate site. A
full course of vaccine is then administered over Correct answer: EG
the next two weeks.
Antibiotics would be indicated immediately The key features of this case are abnormal liver
following such a bite to prevent wound function tests and a severe metabolic acidosis.
infection, but are unnecessary at this stage. Lactic acidosis is a potentially life-threatening
Hepatitis B is a human-only virus; monkeys may complication of anti-HIV therapy. It appears to
transmit herpes B virus that can be treated with be related to the use of nucleoside reverse
aciclovir if infection is suspected. transcriptase inhibitors – most particularly with
stavudine (D4T) and didanosine (DDI). Patients
present with vague abdominal pain, hepatic
steatosis and marked acidosis. The syndrome is
due to inhibition of mitochondrial DNA and may
also be associated with a peripheral

Dr. Khalid Yusuf El-Zohry – Sohag Teaching Hospital (01118391123) Page | 504
El-Zohry MRCP Questions Bank (Part 2) – Medical Masterclass 2010

neuropathy. The use of DDI may also be organism also causes pneumonia and skin
complicated by life-threatening pancreatitis, infections.
which may present with metabolic acidosis and
The organism can be cultured easily in a routine
abdominal pain.
microbiology laboratory, for example it will
PCP is very unlikely, in view of the normal grow on the standard agar plates that are used
oxygenation. to detect pathogens in the CSF from patients
with meningitis. It can also be seen on Gram
Nevirapine may cause hepatic failure, but in this
staining or India ink preparation of the CSF. The
patient, the relatively mild abnormality in liver
India ink stained shows a characteristic
function contrasts with a severe metabolic
spherical cell with a distinctly outlined capsule
acidosis, making this diagnosis unlikely.
that excludes the stain. India ink staining is not
very sensitive and organisms may only be seen
[ Q: 116 ] MasterClass Part2 25-50% of patients with cryptococcal
(2010) – Infectious disease meningitis. Detection of the cryptococcal
polysaccharide antigen is more sensitive and is
Which of the following statements concerning highly specific. The antigen can be detected in
Cryptococcus neoformans is true? the CSF in over 95% of cases and in >99% of
A. The India ink preparation of the cerebral patients it is also present the serum.
spinal fluid in meningoenchephalitis The choice of treatment of the disease depends
reveals filamentous yeast on the anatomical sites of involvement and the
B. Detection of the polysaccharide antigen host immune status. The commonly used anti-
in CSF is sensitive but nonspecific in fungals include amphotericin, fluconazole and
meningoencephalitis flucytosine. There is a high incidence of relapse
(15- 35%) in the HIV population in the absence
C. The organism cannot be easily cultured in of maintenance therapy. The key elements in
the routine microbiological laboratory preventing relapse in these patients are 1)
D. It can cause disease in patients with no control of HIV replication with anti virals and 2)
known immunological defect the use of chronic antifungal therapy.

E. Early initiation of therapy usually leads to


complete eradication of the organism in [ Q: 117 ] MasterClass Part2
HIV positive individuals allowing (2010) – Infectious disease
ceasation of therapy
A 26-year-old Bangladeshi male presents with
several depigmented macular lesions on his
Answer & Comments
trunk.
Correct answer: D
Which would be an unlikely diagnosis?

Cryptococcus neoformans is an encapsulated A. Seborrheic dermatitis


yeast-like fungus. Patients with immunological B. Hansen's disease
defects in T cell mediated host defense
C. Lupus vulgaris
mechanisms are at increased risk of infection,
but disease can occur in those with no D. Vitiligo
identifiable predisposing immunological defect.
E. Tinea versicolor.
CNS infection (meningitis and
meningoencephalitis) is the commonest
manifestation of cryptococcosis but the
Dr. Khalid Yusuf El-Zohry – Sohag Teaching Hospital (01118391123) Page | 505
El-Zohry MRCP Questions Bank (Part 2) – Medical Masterclass 2010

Answer & Comments [ Q: 119 ] MasterClass Part2


Correct answer: C (2010) – Infectious disease
A 58-year-old alcoholic man presents to the
Lupus vulgaris presents as an infiltrated patch Emergency Department with a cough and fever.
rather than a non-palpable macular lesion and He has a long history of smoking and is found to
is more commonly a solitary lesion. Hansen's have a cavitating lesion on his CXR.
disease (leprosy) may be differentiated from
other causes of depigmented lesions by Which of the following is least likely to be the
detection of anaesthesia in the lesion. All such cause of his pneumonia?
lesions in persons from the tropics and sub- A. Meticillin-resistant Staphylococcus aureus
tropics should be examined for anaesthesia. (MRSA)
B. Klebsiella pneumoniae
[ Q: 118 ] MasterClass Part2
C. Enterococcus faecalis
(2010) – Infectious disease
D. Mycobacterium tuberculosis
A 57-year-old woman develops a fever 7 days
after bone marrow transplantation. She is E. Lung cancer with secondary bacterial
placed empirically on broad-spectrum infection
antibiotics, but remains febrile. On the 11th day
she develops a few painless, red, papular Answer & Comments
lesions on her trunk and legs.
Correct answer: C
What is the likely cause of these lesions?
Cavitating pneumonia is associated with
A. Aspergillus infection
Staphylococcus aureus and Klebsiella
B. Candidal infection pneumoniae infection, particularly in alcoholic
C. Graft versus host disease men. A history of previous hospital admissions
would increase the risk of MRSA infection,
D. Recurrence of her haematological disease
although community-acquired MRSA is also
E. Staphylococcal infection being reported with increased frequency. He is
also at higher risk of acquiring tuberculosis. The
Answer & Comments smoking history puts him at increased risk of
lung neoplasm. Enterococcus faecalis does not
Correct answer: B usually cause cavitating lung lesions.

The persistence of fever, timing of occurrence [ Q: 120 ] MasterClass Part2


and description of the lesions fits best with
(2010) – Infectious disease
disseminated yeast infection, most likely
Candida species. The diagnosis can usually be A nun returns from a trip to Bangladesh. She
made on blood cultures, but the organisms can has had full travel vaccinations and is well
also be seen in the lesions on biopsy. Do not whilst there but nevertheless develops acute
forget to look in the fundi in such cases. hepatitis shortly after her return.
It would be too early for graft versus host
The most likely virus is:
disease.
A. hepatitis A

Dr. Khalid Yusuf El-Zohry – Sohag Teaching Hospital (01118391123) Page | 506
El-Zohry MRCP Questions Bank (Part 2) – Medical Masterclass 2010

B. hepatitis B Which of the following is the likely diagnosis?


C. hepatitis C A. Cerebral tuberculosis
D. hepatitis D B. Cerebral metastases
E. hepatitis E. C. Schistosomiasis
D. Cystercicosis
Answer & Comments
E. Cerebral toxoplasmosis.
Correct answer: E
Answer & Comments
A is possible if a vaccine failure occured but E
Correct answer: D
would be more likely overall. The others are
spread parenterally.
The CT scan shows small calcified lesions with
no surrounding mass effect or oedema, this is
[ Q: 121 ] MasterClass Part2
the typical appearance of neurocystercicosis.
(2010) – Infectious disease Cystercicosis due to the pork tapeworm Taenia
A 32-year-old man presents to the Emergency solium is prevalent in Central and South
Department following a grand mal seizure. He America and parts of Asia including the Punjab.
gives a history of occasional blackouts over the Tissue cysts may be found in many tissues
previous five years. He entered the UK from including skeletal and cardiac muscle, retina
India 3 years earlier and has otherwise and brain. Active cysts may be surrounded by
been well with no fever and normal blood tests. oedema but as the organisms die the cysts
An unenhanced CT brain scan is shown (see become inactive and calcified. Many cases are
image). asymptomatic.
Epilepsy is the commonest clinical problem but
encephalitis, mass lesions and hydrocephalus
may also occur.
Toxoplasmosis and tuberculosis form cerebral
abscesses with a low-density area, surrounding
oedema and enhancement with contrast.
Schistosomiasis may rarely produce focal CNS
lesions.
Schistsomiasis is found in Africa and parts of the
far East and this patient has no epidemiological
exposure.

[ Q: 122 ] MasterClass Part2


(2010) – Infectious disease
A 39-year-old woman develops tuberculosis
resistant to rifampicin, isoniazid and
ethambutol.

Dr. Khalid Yusuf El-Zohry – Sohag Teaching Hospital (01118391123) Page | 507
El-Zohry MRCP Questions Bank (Part 2) – Medical Masterclass 2010

Which of the following drugs would not be A. Paracoccidiodomycosis and


considered as part of a new regimen? histoplasmosis should be considered in
A. Cycloserine the differential diagnosis.

B. Streptomycin B. Bone marrow examination is unlikely to


be helpful.
C. Azithromycin
C. Tuberculosis is the second most likely
D. Amikacin diagnosis.
E. Para amino salicylic acid D. Empirical treatment for bacterial
F. Gentamicin pneumonia should be the first step in
management.
G. Ethionamide
E. Pneumocystis carinii pneumonia need not
H. Clarithromycin
be considered in Latin America.
I. Ofloxacin
F. Cryptococcal infection is unlikely in the
J. Norfloxacin. absence of meningeal irritation.
G. Respiratory isolation should be
Answer & Comments undertaken.
Correct answer: FJ H. A tuberculin skin test is unlikely to be
helpful due to cross reactivity with BCG
Multidrug-resistant tuberculosis (MDR-TB) is and environmental mycobacteria.
becoming more of a problem worldwide, I. Malaria could be expected to account for
including in the UK. Treatment should be this clinical presentation.
initiated with at least four active drugs, one of
which should be injectable and should be J. Dengue co-infection should always be
continued for 12-18 months (3-6 months of considered in cases of prolonged fever.
injectable). Quinolones have good activity
including ciprofloxacin, moxifloxacin and Answer & Comments
ofloxacin – norfloxacin however is inadequate.
Correct answer: AG
Of the aminoglycosides, streptomycin,
kanamycin and amikacin have useful anti-TB
activity. The other drugs listed are all suitable The first step should be a sputum smear for AFB
drugs where indicated by sensitivity testing, but of Mycobacterium tuberculosis, which is the
some are becoming increasingly difficult to most likely diagnosis given the prolonged
obtain in the UK. history. If this is negative then a reasonable
approach would be an empirical therapeutic
trial of antibiotics for bacterial pneumonia.
[ Q: 123 ] MasterClass Part2 Though facilities are rarely available, respiratory
(2010) – Infectious disease isolation should be undertaken if possible. Co-
morbidity is common in this patient group so
An HIV-infected 25-year-old woman from the
diagnosis of TB does not preclude a second
Peruvian Highlands presents with 3 weeks of
infection. Bone marrow infection could
cough and dyspnoea with fever. Chest
potentially yield a number of otherwise difficult
radiograph shows scattered infiltrates.
to make diagnoses including the deep-seated
Which of the following is true? mycoses and TB. Cryptococcal
pneumonia/pneumonitis is often asymptomatic

Dr. Khalid Yusuf El-Zohry – Sohag Teaching Hospital (01118391123) Page | 508
El-Zohry MRCP Questions Bank (Part 2) – Medical Masterclass 2010

so the symptom complex rather than the necrophorum is the cause of Lemierre's disease,
absence of meningeal involvement probably with initial throat infection followed by lung
precludes this diagnosis. PCP does need to be abscesses and septic thrombosis of the internal
excluded of course. Many HIV patients are jugular vein.
anergic to tuberculin, which is the main reason
that utility is limited. Cough and dyspnoea are [ Q: 125 ] MasterClass Part2
rarely prominent in malaria (which in any case
(2010) – Infectious disease
is not found in mountainous regions as the
vector mosquitoes tolerate high altitude This chest radiograph (see image) is from a 25-
poorly). The fever of dengue is characteristically year-old intravenous drug user who presented
short-lived. with a 10-day history of fever and a cough
productive of copious amounts of green
[ Q: 124 ] MasterClass Part2 sputum.
(2010) – Infectious disease
A 20-year-old girl is admitted with hypotension
and a blanching erythema. She is thought to
have a toxic shock syndrome.

Which TWO of the following organisms are the


most likely causes of her toxic shock syndrome?
A. Staphylococcus aureus
B. Moraxella catarrhalis
C. Streptococcus viridans
D. Klebsiella pneumoniae
E. Group A Streptococcus pyogenes
F. Candida albicans
G. Bartonella henselae
H. Pseudomonas aeruginosa
I. Fusobacterium necrophorum What are the TWO most likely aetiological
agents?
J. Staphylococcus epidermidis
A. Streptococcus pneumoniae
Answer & Comments B. Staphylococcus epidermidis
Correct answer: AE C. Haemophilus influenzae
D. Mycoplasma pneumoniae
Toxin-producing Staphylococcus aureus (often
E. Staphylococcus aureus
associated with retained tampon use) and
Group A Streptococcus pyogenes (usually F. Legionella pneumophila
associated with skin, soft tissue or throat G. Klebsiella pneumoniae
infections) are the most common causes of
toxic shock syndrome. Bartonella henselae is H. Mycobacterium avium intracellulare
the cause of cat scratch fever. Fusobacterium I. E. coli

Dr. Khalid Yusuf El-Zohry – Sohag Teaching Hospital (01118391123) Page | 509
El-Zohry MRCP Questions Bank (Part 2) – Medical Masterclass 2010

J. Salmonella typhi. I. He requires mandatory exchange


transfusion.
Answer & Comments J. It is highly unlikely that he used
Correct answer: EG mefloquine prophylaxis.

Answer & Comments


The fluid levels within these cavities confirms
that these are lung abscesses. In an intravenous Correct answer: GJ
drug user, Staphylococcus aureus would be the
most likely aetiological agent and was grown
The patient received the correct treatment for
from this patient. Klebsiella pneumoniae is
uncomplicated falciparum malaria currently
another common aetiological agent for lung
recommended in the UK for patients with an
abscess. Other bacteria can cause lung abscess
initial parasitaemia of less than 2%. The
but less commonly than those mentioned
diagnosis is certain and the evolving clinical
above
picture is well-recognised though uncommon.
An increase in parasitaemia despite appropriate
[ Q: 126 ] MasterClass Part2
quinine therapy can occur following schizogeny
(2010) – Infectious disease of schizonts (a form of the life-cycle which are
An ambulant 57-year-old man returning to the not sensitive to quinine), releasing large
UK from Tanzania is diagnosed with falciparum numbers of merozoites. These schizonts may be
malaria with a parasitaemia of 1.1%. He apparent on the original blood film but are
receives treatment with oral quinine. After 24 rarely reported. This surge in parasitaemia may
hours he becomes confused and has a fit. His be accompanied by clinical deterioration and in
repeat blood film reveals a parasitaemia of 7%. this case neurological complications may be
accompanied by renal and/or respiratory
Which TWO of the following statements about involvement.
the management of this patient are true? Quinine-induced hypoglycaemia is an important
A. Initial treatment was incorrect. avoidable complication of parenteral but rare
with oral quinine therapy. Treatment with
B. The diagnosis of falciparum malaria is
quinine is correct and should be continued;
open to question.
artemesinin derivatives act rapidly, but
C. The most likely cause of the deterioration superiority over quinine in this setting remains
is quinine-induced hypoglycaemia. as yet unproven. Exchange transfusion is rarely
D. The first parasitaemia estimate was considered in the absence of hyperparasitaemia
incorrect. (at least 10% and usually more than 20%).

E. The parasite is probably quinine-resistant. If used correctly, mefloquine is a highly


efficacious agent for antimalarial prophylaxis.
F. Only artemether should be used for the
remainder of his treatment.
[ Q: 127 ] MasterClass Part2
G. An increase in parasitaemia can be seen
(2010) – Infectious disease
despite correct initial therapy.
A 23-year-old man requires urgent splenectomy
H. The likely reason for the lack of clinical
following trauma sustained in a road traffic
response is a mixed plasmodium
accident.
infection.

Dr. Khalid Yusuf El-Zohry – Sohag Teaching Hospital (01118391123) Page | 510
El-Zohry MRCP Questions Bank (Part 2) – Medical Masterclass 2010

Which of the following prophylactic measures H. Ask about the use of herbal medicines
does not have proven benefit?
I. Examine and investigate for co-existent
A. Haemophilus influenzae b immunisation opportunistic infections
B. Pneumococcal vaccine J. Measure therapeutic drug levels of
nevirapine and indinavir.
C. Penicillin V
D. BCG immunisation
Answer & Comments
E. Meningococcal vaccine
Correct answer: BF

Answer & Comments


The blood tests indicate loss of HIV control and
Correct answer: D a significant deterioration in immune function
indicating treatment failure, which may have
The other measures are all recommended in multiple possible causes. Firstly confirm the
patients with an absent spleen. results by re-testing particularly as previous
results were so good. Poor adherence is the
commonest cause of therapeutic failure, so
[ Q: 128 ] MasterClass Part2
discuss adherence issues and try to objectively
(2010) – Infectious disease assess compliance by checking with pharmacy.
A 54-year-old HIV positive man has received Drug levels of protease inhibitors and non-
anti-retroviral therapy with several different nucleoside agents can be measured and if low
regimens over the past 5 years. For the past suggest poor adherence or a pharmacologic
year he has been taking a four-drug regimen of problem. Treatment failure is often associated
zidovudine, didanosine and nevirapine and with mutations in HIV leading to resistance.
indinavir. Three months ago blood tests showed These can be measured with genotypic
a CD4+ lymphocyte count of 300 and an HIV resistance tests which can aid selection of the
viral load of less than 50 copies per ml. Now he next drug regimen. Complicating infections
has a CD4 count of 150 and HIV viral load of such as tuberculosis may lead to loss of viral
11000. control. Some ‘herbal’ remedies such as St
John’s Wort can significantly reduce levels of
Which TWO of the following possible actions some HIV drugs including indinavir.
are not appropriate? Resistance to nevirapine is quite likely and is
A. Repeat CD4 and HIV viral load commonly associated with cross-resistance to
efavirenz (K103N mutation in HIV reverse
B. Discontinue all anti-HIV drugs as he now
transcriptase gene). Although this anti-HIV
has resistance and further therapy is
regimen is failing it is still likely to be conferring
futile
some clinical benefit and there is still a good
C. Commence prophylaxis for pneumocystis chance that control can be re-established with a
carinii new regimen.
D. Change to an entirely new drug regimen
E. Detailed discussion and check on [ Q: 129 ] MasterClass Part2
adherence (2010) – Infectious disease
F. Switch nevirapine to efavirenz A 24-year-old bricklayer presents with a short
history of a swollen lymph gland in his neck. He
G. Arrange for genotypic resistance testing
is otherwise well. The following results are
Dr. Khalid Yusuf El-Zohry – Sohag Teaching Hospital (01118391123) Page | 511
El-Zohry MRCP Questions Bank (Part 2) – Medical Masterclass 2010

received from the virology laboratory: C. Cerebral toxoplasmosis


cytomegalovirus (CMV) IgG positive, CMV IgM
D. Neurosyphilis
negative, Epstein-Barr virus (EBV) nuclear
antigen positive, Toxoplasma IgG positive and E. Primary central nervous system
Toxoplasma IgM positive. lymphoma

On the basis of these results, what would be the Answer & Comments
most appropriate drug treatment?
Correct answer: A
A. Ganciclovir
B. Aciclovir The combination of space-occupying lesions,
C. Spiramycin high CSF protein, low CSF glucose and CSF
lymphocytosis point to this case being
D. Septrin
tuberculosis meningitis with cerebral
E. No drug treatment tuberculomas. The organism is not seen on
microscopy in 20-50% of cases. Cryptococcal
Answer & Comments meningitis can give a similar picture, although
the protein and glucose changes are not usually
Correct answer: E so marked; cerebral cryptococcomas are
uncommon and the organism is normally seen
The serology is consistent with acute on microscopy in patients as ill as this.
toxoplasmosis, as well as past CMV and EBV. No Toxoplasmosis, syphilis and lymphoma would
treatment is required. Spiramycin is sometimes not cause such significant changes in the
used in pregnancy and a range of treatments protein, glucose or white cell count.
may be used for the prevention or treatment of
reactivation of toxoplasmosis in the [ Q: 131 ] MasterClass Part2
immunosuppressed.
(2010) – Infectious disease

[ Q: 130 ] MasterClass Part2 A 49-year-old businessman has just returned


from a trip to the Far East during an outbreak of
(2010) – Infectious disease
severe acute respiratory syndrome (SARS). He is
An HIV-positive man is brought to the referred with a respiratory illness.
Emergency Department having had a fit. A
friend says that he has complained of Which TWO of the following would not be
headaches for the last week. An emergency CT consistent with a diagnosis of SARS.
brain scan shows two 1-cm ring-enhancing A. Abrupt onset of high fever?
lesions in the cerebral cortex. A lumbar
B. Neutrophil leucocytosis?
puncture shows a cerebrospinal fluid (CSF)
pressure of 35 mmH2O, protein 2.5 g/L, glucose C. Non-productive cough?
1 mmol/L (plasma 6 mmol/L) and 100 white
D. Normal chest X ray?
cells/high powered field (all lymphocytes). No
organisms are seen. E. Thrombocytopaenia?
F. Elevated liver transaminases?
What is the most likely diagnosis?
G. Prodromal coryzal symptoms?
A. Tuberculosis
H. Clear chest examination?
B. Cryptococcal meningitis

Dr. Khalid Yusuf El-Zohry – Sohag Teaching Hospital (01118391123) Page | 512
El-Zohry MRCP Questions Bank (Part 2) – Medical Masterclass 2010

I. Alveolar infiltrates on chest X-ray? D. Stool microscopy would not aid diagnosis.
J. Elevated creatinine phosphokinase (CPK)? E. HIV infection is the most potent risk
factor for Strongyloides hyperinfection
Answer & Comments syndrome.

Correct answer: BG F. Strongyloides infection can persist for


decades.

SARS has recently emerged as a severe G. No treatment is necessary.


respiratory illness originating in the Far East. It H. The patient is highly infectious.
is probably due to a virus with a novel
coronavirus as the most likely causative agent. I. Strongyloides infection is acquired by the
faecal-oral route.
SARS is characterized by an abrupt onset of
fever >38C, dry cough and myalgia. J. Strongyloides cannot be cultured.
Approximately 40% progress to pneumonia and
respiratory distress. In contrast to many other Answer & Comments
respiratory infections upper respiratory
Correct answer: BG
symptoms are generally absent.
Initial chest X-ray may be normal but then This is the characteristic history of larva
develops an alveolar pattern of consolidation as currens, a pathognomonic symptom of
the illness progresses. Despite consolidation on infection with Strongyloides stercoralis, an
X-ray most patients have only subtle or no enteric parasite. Strongyloides larvae are
abnormalities on chest examination. Laboratory identifiable on stool microscopy in
findings typically reveal leucopaenia and uncomplicated infection. Eosinophilia is
thrombocytopaenia with elevated CPK and liver common and is absent in the potentially life-
enzymes. threatening hyperinfection syndrome, which is
clinically completely distinct with systemic
[ Q: 132 ] MasterClass Part2 malaise, diarrhoea, cough and sometimes
(2010) – Infectious disease haemoptysis. Immunological compromise due
to chemotherapy, malignancy, steroid use and
A businessman reports the appearance of an diabetes mellitus are recognised risk factors but
extremely itchy urticaria-like rash on the chest (perhaps surprisingly) to date no association
wall, lasting for 2-3 hours on three occasions in has been demonstrated for HIV infection.
the preceding month. He has travelled widely in
Africa and the Caribbean. Clinical examination is
[ Q: 133 ] MasterClass Part2
unremarkable.
(2010) – Infectious disease
Which TWO of the following statements are
A 35-year-old Peruvian woman develops fever,
true?
headache and vomiting five weeks after
A. The rash is known as cutaneous larva commencing highly active antiretroviral therapy
migrans. (HAART) for HIV infection.
B. Eosinophilia would support a diagnosis of
Which of the following is LEAST likely to explain
infection with Strongyloides stercoralis.
this clinical picture?
C. In the absence of eosinophilia
A. Adverse reaction to HAART
Strongyloides hyperinfection syndrome is
likely.
Dr. Khalid Yusuf El-Zohry – Sohag Teaching Hospital (01118391123) Page | 513
El-Zohry MRCP Questions Bank (Part 2) – Medical Masterclass 2010

B. Immune reconstitution inflammatory Tetanus and strychnine poisoning both produce


syndrome (IRIS) to M tuberculosis muscle spasm which may lead to respiratory
failure but not muscle weakness.
C. Immune reconstitution inflammatory
syndrome (IRIS) to Cryptococcus Rabies produces a uniformly fatal encephalitis
neoformans characterized by pharyngeal spasm triggered by
water.
D. Progressive multifocal
leukoencephalopathy (PML) Diphtheria presents with a pharyngitis and a
membrane over the tonsils. The toxin may
E. Cerebral toxoplasmosis
cause myocarditis and neurotoxicity with
palatal paralysis and cranial nerve palsies may
Answer & Comments occur.
Correct answer: D Botulism typically produces a descending
paralysis which starts with diplopia or blurred
Adverse reactions to any of the components of vision due to difficulty with accommodation
combination antiretroviral therapy or HAART and progresses to weakness of the neck, arms
are common and varied. IRIS to most common and respiratory muscles.
HIV-associated pathogens is emerging as a
considerable issue in the provision of HAART to [ Q: 135 ] MasterClass Part2
developing world populations, with multiple co-
(2010) – Infectious disease
existent opportunistic infections and low
starting CD4 counts. Delay in PML progression A 28-year-old English female is referred with a
has been reported on HAART though the effect 5-week history of persistent diarrhoea and
is modest. abdominal discomfort following a 1-month
trekking holiday in Mexico.
[ Q: 134 ] MasterClass Part2
Which of the following diagnoses would not be
(2010) – Infectious disease consistent with her history?
A 30-year-old man presents with diplopia, A. Giardiasis
dysphagia and dysarthria. After 12 hours he has
B. Crohn's disease
weakness of his arms but remains afebrile and
is not confused. Over the next 12 hours he C. HIV disease
develops respiratory failure and requires
D. Hydatid disease
artificial ventilation.
E. Post-enteric infection disaccharide
What is the likely diagnosis? intolerance.
A. Tetanus
Answer & Comments
B. Diphtheria
Correct answer: D
C. Botulism
D. Strychnine poisoning
Persistent diarrhoea following enteric infection
E. Rabies. in the returning traveller is a common problem.
Several groups of disease should be considered:
Answer & Comments  Persistent infection or infestation, e.g.
Giardia, cryptosporidia, amoebic
Correct answer: C

Dr. Khalid Yusuf El-Zohry – Sohag Teaching Hospital (01118391123) Page | 514
El-Zohry MRCP Questions Bank (Part 2) – Medical Masterclass 2010

disease, bacillary dysentery, tropical Current guidelines advise the treatment of PID
sprue. and gonorrhoea with two antibiotics. PID

should be treated with ofloxacin plus
Post-infectious malabsorptive
metronidazole or doxycline plus metronidazole.
processes, e.g. disaccharide
Gonorrhoea requires initially an antibiotic to
intolerance, bacterial overgrowth.
treat the neisserial infection but this should
 Post-infectious irritable bowel. immediately be followed with a tetracycline or
 Chronic GI disease unmasked by macrolide to prevent the emergence of post-
infection, e.g. inflammatory bowel gonnococcal urethritis and treat any co-existing
disease, coeliac disease, colorectal chlamydial infection (present in over 50% of
carcinoma, HIV infection. cases).

Hydatid disease is often discovered incidentally Uncomplicated chlamydial infections and


by imaging and is rarely associated with GI syphilis respond well to single-agent therapy.
symptoms per se. It is rare in travellers.
[ Q: 137 ] MasterClass Part2
[ Q: 136 ] MasterClass Part2 (2010) – Infectious disease
(2010) – Infectious disease A 25-year-old man was admitted via the
You are asked to see a 24-year-old woman who Emergency Department some 12 months ago
presents after possible exposure to sexually with shortness of breath, a dry persistent cough
transmitted infection. and fevers. He was HIV positive but on no
treatment. He was treated for presumptive
Which of the following genital infections (when Pneumocystis carinii pneumonia (PCP) with i.v.
acquired in the UK) requires combination Co-trimoxazole, made a full recovery, and has
antimicrobial therapy with TWO agents or more remained on Co-trimoxazole as secondary
to ensure complete resolution of all symptoms prophylaxis since this episode.
and the prevention of long-term complications?
He has subsequently started on HAART. His last
A. Pelvic inflammatory disease (PID) two HIV viral load assays (3 and 6 months ago)
B. Candidiasis have been <50 copies /ml detected. His last two
CD4 counts have been (3 and 6 months ago)
C. Lymphogranuloma venereum 210 /μL and 240 /μL.
D. Chlamydial cervicitis
Concerning his secondary prophylaxis with Co-
E. Bacterial vaginosis trimoxazole for PCP, which of the following
F. Syphilis statements is correct?

G. Gonorrhoea A. This should be discontinued since he


never had laboratory confirmed PCP.
H. Chancroid
B. This should be discontinued since he is
I. Scabies
stabilised on highly active antiretroviral
J. Human papilloma virus infection. therapy (HAART).
C. This should be discontinued since his CD4
Answer & Comments count is persistently over 200 and his viral
load is stable on HAART.
Correct answer: AG
D. This should never be discontinued for
fear of PCP recurring.
Dr. Khalid Yusuf El-Zohry – Sohag Teaching Hospital (01118391123) Page | 515
El-Zohry MRCP Questions Bank (Part 2) – Medical Masterclass 2010

E. This should now be changed to nebulised I. Ocular cysticerci are often seen in patients
pentamidene once monthly to reduce his with neurological involvement.
pill burden.
J. Bladder involvement is a frequent cause
of haematuria in endemic regions.
Answer & Comments

Correct answer: C Answer & Comments

Correct answer: AH
Secondary prophylaxis with Co-trimoxazole is
advisable after PCP pneumonia because of the
Cysticercosis is caused by infection with the
high risk of recurrence. However, this may be
larval form of the pork tapeworm Taenia solium
safely discontinued if there is sufficient immune
(but not the beef tapeworm T saginata).
reconstitution on starting HAART therapy. Even
Infection is acquired not by ingestion of
on HAART, prophylaxis should be continued if
cysticerci in infected meat (which is how
the CD4 count remains below 200. Nebulized
tapeworm infection is acquired), rather by
pentamidene is not as effective as oral therapy
ingestion of eggs produced by the adult
is only used if other agents are contraindicated.
tapeworm – as these may contaminate
vegetable matter (faeco-oral), vegetarians can
[ Q: 138 ] MasterClass Part2 be infected. Tapeworms do not cause weight
(2010) – Infectious disease loss. Neurological involvement results in
convulsions in a proportion of subjects but
An 18-year-old Bolivian man with convulsions is asymptomatic infection is common.
diagnosed with neurocysticercosis. With regard Management of neurocysticercosis is both
to cysticercosis, which of the following are true? symptomatic (with anticonvulsants) and
specifically antiparasitic (with albendazole
A. Neurocysticercosis is the commonest
and/or praziquantal), though whether
cause of convulsions in young adults in
antiparasitic treatment improves outcomes is
Africa and Latin America.
still under investigation, and some authorities
B. Cysticercosis is a manifestation of advocate caution in their use as florid
infection with the larval stage of the pork inflammatory reactions with worsening of
tapeworm, Taenia saginata. symptoms, may also occur. Ocular involvement
C. Neurocysticercosis has not been reported is rare and often causes blindness.
in vegetarians.
D. Infection with an adult tapeworm is an [ Q: 139 ] MasterClass Part2
essential prerequisite to cysticercosis. (2010) – Infectious disease
E. Adult tapeworm infection is often A 35-year-old male Caucasian presents 1 week
associated with weight loss. after returning from a safari holiday in
Tanzania. He has several inflamed oedematous
F. Treatment with praziquantal and
albendazole is safe and effective. papular lesions on his right thigh with local
tender lymphadenopathy and has a low-grade
G. Albendazole should not be used with fever.
anticonvulsants.
Which of the following would not be a possible
H. Cysticerci may be identified by plain
diagnoses?
radiography.
A. Cutaneous myiasis

Dr. Khalid Yusuf El-Zohry – Sohag Teaching Hospital (01118391123) Page | 516
El-Zohry MRCP Questions Bank (Part 2) – Medical Masterclass 2010

B. Group-A Streptococcal infection of insect E. Ceftriaxone


bites
C. Acute African trypanosomal chancres
D. Cutaneous larva migrans Answer & Comments

E. Rickettsial chancre. Correct answer: B

Answer & Comments Penicillin-based antibiotics such as co-amoxiclav


and Tazocin (piperacillin) are contraindicated.
Correct answer: D
There is 5-10% cross-reactivity with
cephalosporins (cefuroxime and ceftriaxone)
Cutaneous myiasis is a condition caused by which should be avoided if there is a clear
various fly species whose eggs are deposited on history of anaphylaxis. Macrolides
clothing and then hatch, the larval form (clarithromycin) should be safe.
penetrating the skin to form a boil-like lesion
that may suppurate. Treatment is by careful
[ Q: 141 ] MasterClass Part2
removal of the larva. Insect bites will often
become secondarily infected by Streptococci (2010) – Infectious disease
and Staphylococci and can cause significant A 48-year-old man presents with a 5-day history
sepsis. of fever and cough. He has no significant past
African trypanosomiasis and rickettsial (tick- medical history but is very unwell. His chest
borne) typhus are both encountered by radiograph shows patchy shadowing, mainly in
travellers to game parks. The bite of the tsetse the right lower lobe.
fly, which transmits trypanosomiasis, is
exquisitely painful, while tick bites are painless What TWO antibiotic regimen from the list
but the tick may be found still attached by the below would it be most appropriate to give
patient. him?

Cutaneous larva migrans, caused by various A. Cotrimoxazole 120 mg/kg daily in 2-4
Ancylostoma (hookworm) species is divided doses
characterised by a slowly lengthening, B. Ciprofloxacin 400mg 12-hourly
serpiginous, intensely itchy rash. intravenously
C. Erythromycin 1g 6-hourly intravenously
[ Q: 140 ] MasterClass Part2
D. Clarithromycin 250mg 8-hourly orally
(2010) – Infectious disease
E. Ciprofloxacin 500mg 12-hourly orally
A patient states that he has an allergy to
F. Erythromycin 500mg 6-hourly
penicillin with a previous anaphylactic reaction.
intravenously
Which of the following antibiotics is likely to be G. Benzylpenicillin 1.2-2.4g 6-hourly
safest? intravenously
A. Cefuroxime H. Rifampicin 600mg 12-hourly
B. Clarithromycin intravenously

C. Co-amoxiclav (Augmentin) I. Amoxicillin 250mg 8-hourly orally

D. Tazocin J. Cefotaxime 1g 8-hourly intravenously.

Dr. Khalid Yusuf El-Zohry – Sohag Teaching Hospital (01118391123) Page | 517
El-Zohry MRCP Questions Bank (Part 2) – Medical Masterclass 2010

Answer & Comments J. Malaria in pregnancy is a risk factor for


premature delivery and low birth weight.
Correct answer: CJ

Answer & Comments


British Thoracic Society guidelines for the
treatment of severe community- acquired Correct answer: BH
pneumonia suggest parenteral treatment with a
second- or third-generation cephalosporin, with Side effects of quinine include cinchonism
high dose parenteral erythromycin in suspected (tinnitus, headache, nausea and visual blurring).
Legionnaire’s disease, which proved to be the This almost always occurs at therapeutic drug
diagnosis in this case. levels and does not necessarily indicate drug
toxicity. Quinine can induce dysrhythmia in the
[ Q: 142 ] MasterClass Part2 presence of pre-existing cardiac conduction
(2010) – Infectious disease defects, particularly long QT syndrome, and
following pre-treatment with mefloquine. Both
A 22-year-old pregnant Ugandan female has malaria infection and quinine can induce
been diagnosed with P. falciparum malaria. She hypoglycaemia and this should be excluded
is vomiting and unable to take oral medication when there is a declining level of consciousness
but is otherwise well. Intravenous quinine before a diagnosis of cerebral malaria is made.
600mg bd is commenced. A loading dose of quinine is only suggested in
the presence of severe malaria (cerebral
Which of the following statements are disease, hyperparasitaemia etc.).
incorrect?
Malaria in pregnancy is potentially fatal for
A. Mild tinnitus and headache are normal both mother and fetus and the optimum
side effects of treatment. treatment should be instituted, which
B. A loading dose of 1200mg quinine should continues to be quinine in falciparum malaria,
be given initially. despite potential drug complications in the
mother and fetus.
C. A baseline ECG to exclude rhythm
abnormalities should be performed.
[ Q: 143 ] MasterClass Part2
D. Thrombocytopenia is a recognised drug
side effect. (2010) – Infectious disease

E. A worsening level of consciousness may A 67-year-old woman is 14 days post


be due to drug-induced hypoglycaemia. emergency laparotomy for generalized
peritonitis. The surgeon had performed a right
F. Pre-treatment with mefloquine places the
hemi-colectomy for a perforated colonic
patient at risk of dysrhythmias.
tumour. Her post-operative course has been
G. When vomiting resolves the patient can characterized by a wound infection and
be switched to quinine sulphate 600mg swinging fevers that have not responded to 7
po tds. days of intravenous metronidazole and
cefuroxime. She has remained
H. Quinine is contraindicated due to
haemodynamically stable.
teratogenicity in pregnancy.
I. Use of a second drug e.g. fansidar should Which is the LEAST helpful action?
be avoided if possible during pregnancy.
A. Blood cultures
B. Abdominal imaging

Dr. Khalid Yusuf El-Zohry – Sohag Teaching Hospital (01118391123) Page | 518
El-Zohry MRCP Questions Bank (Part 2) – Medical Masterclass 2010

C. Empirical change in antibiotics Answer & Comments


D. Discontinuation of antibiotics Correct answer: B
E. Removal of unnecessary intravenous
lines. Significant hypoxia with only a mild
radiographic abnormality is most likely to be
Answer & Comments due to Pneumocystis carinii (jiroveci)
pneumonia. An urgent HIV test would aid the
Correct answer: C diagnosis, and bronchoscopy should be
arranged as soon as possible. Hypoxia caused
Discontinuation of the antibiotics and re- by bacterial pneumonia or tuberculosis would
assessment is reasonable. Prolonged courses on be associated with significant radiological
antibiotics only select for resistant organisms. A abnormality. Malaria can cause such a clinical
full septic screen should be taken including picture but is less likely. Withholding empirical
blood cultures, urine culture and wound swab. therapy in someone so ill would not be
A chest radiograph may help exclude a appropriate.
respiratory focus. Abdominal imaging is
particularly useful as an intra-abdominal [ Q: 145 ] MasterClass Part2
collection complicating the perforation must be
(2010) – Infectious disease
high on the differential diagnoses. Such a
collection if found is unlikely to respond to A 68 year-old man is admitted to hospital with
antibiotics alone and would require drainage. pneumonia and prescribed intravenous
antibiotics. He makes a steady recovery but 5
[ Q: 144 ] MasterClass Part2 days later he develops further fever and rigors.
The intravenous canula site in the antecubital
(2010) – Infectious disease
fossa appears inflamed.
A 30-year-old African woman presents with
cough and breathlessness for the last 5 days. What is the most likely organism responsible for
She has been losing weight for the last year. the new fever?
Her CXR is mildly abnormal with bilateral mid- A. Acinetobacter baumii
zone changes. Arterial blood gas analysis on air
B. Candida albicans
shows Pao2 8 kPa.
C. Staphylococcus aureus
What is the appropriate treatment for the most
D. Staphylococcus epidermidis
likely diagnosis?
E. Streptococcus pneumonia
A. Intravenous cefotaxime and erythromycin
B. Intravenous co-trimoxazole 120 mg/kg
daily
Answer & Comments
C. Oral
rifampicin/isoniazid/pyrazinamide/etham Correct answer: C
butol
D. No treatment: await further test results The patient is most likely to have a line-
associated infection, typically caused by
E. Intravenous quinine Staphylococcus aureus in this setting.
Acinetobacter species can cause nosocomial
infections, usually on an ICU setting and may be

Dr. Khalid Yusuf El-Zohry – Sohag Teaching Hospital (01118391123) Page | 519
El-Zohry MRCP Questions Bank (Part 2) – Medical Masterclass 2010

very resistant to antibiotics. Staphylococcus Answer & Comments


epidermidis and Candida species can cause line
Correct answer: B
infections in the immunosuppressed, in ICU
care or in those with long term lines.
Erythema multiforme is a common
complication of genital herpes infection. The
[ Q: 146 ] MasterClass Part2 target-like lesions appear either as the genital
(2010) – Infectious disease lesion is evolving or soon after. Attacks can be
A 28-year-old woman has noticed this blistering controlled with continuous oral aciclovir to
eruption appear over her body every 2-3 prevent HSV reactivation, but aciclovir has no
months for the last year (see image). effect on the erythema multiforme during an
acute attack.

[ Q: 147 ] MasterClass Part2


(2010) – Infectious disease
A 19-year-old Indian man presents with a 6-
week history of cough, fever and weight loss.
Acid fast bacilli are seen on sputum smear and
he is commenced on antituberculous therapy.
His condition deteriorates and his culture
subsequently grows M. tuberculosis resistant to
rifampicin, isoniazid and ethambutol.

Which of the following is incorrect regarding


subsequent management?
A. He should be admitted to a negative
pressure isolation cubicle
B. He should be isolated until he has had 2
weeks therapy with at least three active
drugs

On each occasion she has had symptoms of C. During isolation, all visitors should wear
genital soreness and has taken some Potassium appropriate masks to filter out M.
citrate and cranberry juice. tuberculosis
D. All close contacts should be screened for
What is the most likely diagnosis? tuberculosis
A. Fixed drug eruption E. On discharge he should receive directly
B. Erythema multiforme observed therapy.

C. Disseminated gonoccacal disease


Answer & Comments
D. Bullous phemphigus
Correct answer: B
E. Urticaria.

Patients with multidrug resistant tuberculosis


should be isolated in a negative pressure

Dr. Khalid Yusuf El-Zohry – Sohag Teaching Hospital (01118391123) Page | 520
El-Zohry MRCP Questions Bank (Part 2) – Medical Masterclass 2010

isolation room. These patients should be I. Aeromonas hydrophila


treated with five active drugs including an
J. Vibrio spp.
injectable preparation such as amikacion or
streptomycin. He should remain isolated until
his sputum smears have become negative or Answer & Comments
culture of a smear-positive sputum is negative - Correct answer: AF
this often means isolation for 3-4 months.
Approximately 1% of UK isolates of MTB are Your first line antibiotic treatment should cover
multidrug resistant and about 6% are isoniazid- against streptococci and staphylococci.
resistant. Resistance is more likely in patients Intravenous benzylpenicillin (1.2-2.4 g 6-hourly)
who have been previously treated (but not and flucloxacillin (0.5-1 g 6-hourly) would be
complied) and those who acquired disease appropriate in this case.
abroad.
[ Q: 149 ] MasterClass Part2
[ Q: 148 ] MasterClass Part2 (2010) – Infectious disease
(2010) – Infectious disease
A 65-year-old man with a history of controlled
The image shows the legs of a 50-year-old chronic myeloid leukaemia presents confused
woman. and is unwell with a high fever but no other
localizing signs of infection.

Which combination would constitute the most


appropriate treatment pending CSF analysis and
blood culture results?
A. Third generation cephalosporin such as
cefotaxime of ceftriaxone?
B. High dose intravenous ampicillin?
C. High dose intravenous penicillin?
D. Anti-tuberculous therapy?
There was no improvement in the condition
E. Oral acyclovir?
after 3 days of oral antibiotic treatment.
F. Intravenous ciprofloxacin
What are the TWO most likely aetiological
G. High dose intravenous flucloxacillin?
agents?
H. Oral metronidazole?
A. Streptococcus pyogenes
I. Intravenous vancomycin?
B. Clostridium perfringens
J. No antimicrobials?
C. Streptococcus viridans
D. Pseudomonas Answer & Comments
E. Staphylococcus albus
Correct answer: AB
F. Staphylococcus aureus
G. Streptococcus pneumoniae The differential diagnosis for this gentleman is
extensive. However, in this early stage, before
H. Anaerobic bacteria
an absolute diagnosis has been made, he needs
Dr. Khalid Yusuf El-Zohry – Sohag Teaching Hospital (01118391123) Page | 521
El-Zohry MRCP Questions Bank (Part 2) – Medical Masterclass 2010

good broad-spectrum antibiotics aimed at It should be remembered that a large


covering immediate life-threatening infections proportion of travellers with fever would have
such as meningococcal, pneumococcal or gram- cosmopolitan rather than exotic causes for their
negative sepsis. Drug, mode of delivery and fever, but that a careful history and
dosing should allow adequate CSF penetration examination should be undertaken to exclude
in the event that he has meningitis. Given his the exotic causes.
age and predisposing haematological condition
listeria must be considered, hence ampicillin
should be given in addition to a broad-spectrum
agent such as ceftriaxone. The addition of
acyclovir whilst awaiting PCR on CSF is not
inappropriate but this should be administered
intravenously.

[ Q: 150 ] MasterClass Part2


(2010) – Infectious disease
A 40-year-old female traveller recently returned
from a 2-month trip to India presents with a
week’s history of fever, malaise, myalgia and
headache. Examination is non-localising and a
full blood count shows Hb 13.6, WCC 14.2 Plt
148 neutrophils 12.4. A malaria film is negative.

Which of the following would be unlikely?


A. Rickettsial disease
B. Amoebic liver abscess
C. Leptospirosis
D. Urinary tract infection
E. Sepsis from pyodermic insect bites.

Answer & Comments

Correct answer: A

Other than rickettsial infection, which usually


presents with a localised lymphadenopathy +/-
eschar and rash and normal white blood count,
all the other conditions listed can be associated
with peripheral neutrophilia. Malaria is rarely
associated with raised neutrophils but should
always be excluded in travellers returning from
endemic areas. Amoebic liver abscess is
associated with neutrophilia and not
eosinophilia.

Dr. Khalid Yusuf El-Zohry – Sohag Teaching Hospital (01118391123) Page | 522
El-Zohry MRCP Questions Bank (Part 2) – Medical Masterclass 2010

[ Q: 2 ] MasterClass Part2

Respiratory (2010) - Respiratory


A businessman of 60 presents with several
months of increasing dyspnoea. He is a non-
(160 Questions) smoker and has always felt fit. His past medical
history includes only a broken leg playing rugby
(Medical Masterclass – Part 2) in 1980, complicated by a deep vein thrombosis
(DVT). His chest radiograph is shown.

[ Q: 1 ] MasterClass Part2
(2010) - Respiratory
A 62-year-old woman is admitted with an
infective exacerbation of chronic obstructive
pulmonary disease (COPD). Admission blood
gases taken while breathing 28% oxygen are as
follows: pH 7.31, pO2 8.4 kPa and pCO2 7.4 kPa.

Which of the following best describes the blood


gas picture?
A. Compensated Type I respiratory failure
B. Compensated Type II respiratory failure
C. Decompensated Type I respiratory failure
D. Decompensated Type II respiratory What is the likely diagnosis?
failure A. Left ventricular failure secondary to silent
E. Partially compensated respiratory myocardial infarction
alkalosis. B. Primary pulmonary hypertension
C. Atrial septal defect
Answer & Comments
D. Chronic thromboembolic pulmonary
Correct answer: D hypertension
E. Systemic sclerosis with interstitial lung
These blood gases show the patient is hypoxic disease.
on oxygen and the elevated pCO2 combined
with the low pH confirm a respiratory acidosis.
Answer & Comments
Therefore, the blood gas picture is of
decompensated Type II (respiratory) failure and Correct answer: D
the prognosis is poor unless treated
appropriately, including consideration of non- The chest radiograph shows changes of severe
invasive positive pressure ventilation (NIPPV). pulmonary hypertension, with marked
enlargement of the proximal pulmonary
arteries and attenuation of the smaller
pulmonary vessels. There is no interstitial
pulmonary oedema to suggest left ventricular

Dr. Khalid Yusuf El-Zohry – Sohag Teaching Hospital (01118391123) Page | 523
El-Zohry MRCP Questions Bank (Part 2) – Medical Masterclass 2010

failure. With a prior history of DVT, the most [ Q: 3 ] MasterClass Part2


likely diagnosis is chronic thromboembolic (2010) - Respiratory
pulmonary hypertension (CTEPH). Primary
pulmonary hypertension can occur at any age, A 50-year-old Indian male accountant gives a
but the mean age of diagnosis is 36 years and history of 3-months cough, weight loss and
there is a female preponderance of 1.7:1. haemoptysis. He was found to have some nasal
crusting but otherwise examination was
Chronic thromboembolic pulmonary
normal. His CRP and ESR are elevated but he
hypertension has an unknown incidence, but
has normal biochemistry except for an elevated
probably occurs in no more than 0.1% of
creatinine of 400. His chest radiograph showed
survivors of pulmonary embolus. The
a few nodules. A CT thorax is shown (see
pathophysiology is not understood.
image).
Antiphospholipid antibodies are found in 10-
20% of patients referred for surgical treatment.
Progression to pulmonary hypertension and
death occurs not because of repeated emboli,
as was originally thought – serial perfusion
scans on patients in follow-up do not
demonstrate new perfusion defects. Rather, it
appears that there develop progressive
pulmonary hypertensive changes in the small
vessels of the pulmonary vascular bed. The
clinical course of CTEPH is usually one of an
asymptomatic (or “honeymoon”) period, during
which right ventricular hypertrophy develops,
followed by progressively increasing dyspnoea
and exercise intolerance, during which the right
ventricle dilates and fails. The only effective
treatment is surgical. Pulmonary What is the most likely diagnosis?
thromboendarterectomy is a difficult and
specialised operation to remove the organized A. Tuberculosis
thrombus, which is not endoluminal but B. Bronchial carcinoma
incorporated into the walls of the pulmonary
C. Staphylococcal pneumonia
arterial tree. The surgeon has to dissect down
to the level of the original arterial intima, D. Wegener’s granulomatosis
performing a true endarterectomy, rather than E. Systemic lupus erythematosus (SLE)
a thrombectomy. In San Diego, where the
number of cases performed now exceeds 1200,
Answer & Comments
the operative mortality is about 7% and the
results following surgery excellent. The only UK Correct answer: D
centre offering this procedure is Papworth
Hospital. The two most appropriate diagnoses were
Wegener’s and tuberculosis. However, given
the renal dysfunction and nasal disease, a
vasculitis is most likely. The CT shows multiple
and occasionally cavitating nodules. Although
staphylococcal infection cavitates, the history is

Dr. Khalid Yusuf El-Zohry – Sohag Teaching Hospital (01118391123) Page | 524
El-Zohry MRCP Questions Bank (Part 2) – Medical Masterclass 2010

too long for this. Appropriate investigations works in a ship yard and has smoked 20
would involve bronchoscopy and serum ANCA. cigarettes per day for the last four years.

The most likely diagnosis is:


[ Q: 4 ] MasterClass Part2
(2010) - Respiratory A. Asthma
B. Asbestosis
The image shows the flow volume loop of a
patient with exertional dyspneoa. C. Lung cancer
D. Chronic obstructive pulmonary disease
E. Bronchiectasis.

Answer & Comments

Correct answer: E

Of the diagnoses listed, only bronchiectasis


could explain the amount of sputum that this
man coughs up each day. Possible causes to
consider would include cystic fibrosis, alpha-1
antitrypsin deficiency and
hypogammaglobulinaemia.
The diagnosis is:
A. asthma [ Q: 6 ] MasterClass Part2
B. emphysema (2010) - Respiratory

C. restrictive lung defect In which of the following emergency medical


D. fixed upper airway obstruction presentations is non-invasive ventilation an
established treatment?
E. variable extrathoracic obstruction.
A. Tension pneumothorax
Answer & Comments B. Acute asthma

Correct answer: D C. Community acquired pneumonia


D. Acute exacerbation of chronic obstructive
The marked limitation of both inspiratory and pulmonary disease
expiratory flow rates is characteristic of fixed
E. Pulmonary oedema.
upper airway obstruction, either intra- or extra-
thoracic.
Answer & Comments

[ Q: 5 ] MasterClass Part2 Correct answer: D


(2010) - Respiratory
Although non-invasive ventilation (NIV) is
A 20-year-old man gives a six-month history of currently being evaluated in an number of
worsening breathlessness and coughs up half a emergency situations, the best evidence relates
cupful of sputum daily. He was occasionally to exacerbations of chronic obstructive
wheezy with viral illnesses as a young child, pulmonary disease (COPD), and in particular,
Dr. Khalid Yusuf El-Zohry – Sohag Teaching Hospital (01118391123) Page | 525
El-Zohry MRCP Questions Bank (Part 2) – Medical Masterclass 2010

patients with decompensated Type II D. Like Mycoplasma it is accompanied by


respiratory failure. In these patients, the presence of cold agglutinins.
physiological responses (heart and respiratory
E. A pale macular rash may be present.
rate, and arterial blood gases) improve more
quickly with NIV compared to standard F. Haemolysis is a well known feature.
treatment. Intubation is also less frequently G. It typically affects the upper lobes.
required.
H. Cavitations are usually seen on the chest
radiograph.
[ Q: 7 ] MasterClass Part2
I. Patients should be barrier nursed.
(2010) - Respiratory
J. Endocarditis is not a complication.
A 68-year-old lady has been admitted to the
hospital with a severe pneumonia and is
Answer & Comments
presently intubated and ventilated. She was
previously fit and on no medication. There is no Correct answer: BE
history of travel abroad. Her partner says that 3
weeks ago she visited a turkey farm. A chest Psittacosis is due to C. psittaci. It is acquired by
radiograph is shown. exposure to infected birds that are typically ill.
It is an occupational hazard in poultry business
workers and turkey farm workers. It has an
incubation period of 1-2 weeks and presents
with high-grade fever, severe myalgias
especially in the neck. Splenomegaly and a pale
macular rash (Horder's spots) are a well known
feature. It usually affects the lower lobes and
both lungs may be affected. Diagnosis is best
made by demonstrating a fourfold rise in
complement-fixing antibodies in acute and
convalescent sera. Tetracycline is the drug of
choice and should be given in a dose of 2 gm
daily for a minimum of 2 weeks after fever has
subsided. With treatment prognosis is good.

[ Q: 8 ] MasterClass Part2
(2010) - Respiratory
A 52-year-old married heathcare assistant gives
a 6-week history of a dry hacking cough and
You make a working diagnosis of Psittacosis. progressive shortness of breath, which has
failed to respond to two courses of antibiotics
Which TWO of the following are true regarding (penicillin and macrolide). Her CXR is reported
psittacosis? as showing bilateral patchy consolidation,
A. It is caused by Chlamydia pneumoniae. predominantly in the lower lobes. Routine
blood tests are normal, except for elevated C-
B. Splenomegaly can occur.
reactive protein. Her antinuclear antibodies and
C. It is best treated with septrin.

Dr. Khalid Yusuf El-Zohry – Sohag Teaching Hospital (01118391123) Page | 526
El-Zohry MRCP Questions Bank (Part 2) – Medical Masterclass 2010

antineutrophil cytoplasmic antibodies are also E. Goodpasture's syndrome is commonly


normal. seen in smokers.
F. Haemoptysis is commonly seen in
Which of the listed diagnoses is most likely?
lymphangitic carcinoma.
A. Pulmonary tuberculosis
G. A raised serum angiotensin-converting
B. Extrinsic allergic alveolitis enzyme (ACE) level is diagnostic of
C. Cryptogenic organising pneumonia sarcoidosis.

D. Bronchiolitis obliterans H. Hypercalcaemia is seen in 11-18% 0f


patients with sarcoidosis.
E. Sarcoidosis
I. Transbronchial lung biopsy (TBLB) can
result in pneumothorax in over 25% of
Answer & Comments
patients.
Correct answer: C
J. The presence of traction bronchiectasis on
high resolution CT scanning in a patient
Pulmonary tuberculosis, extrinsic allergic with CFA suggests reversibility.
alveolitis and sarcoidosis are unlikely as in these
conditions upper lobe predominance would be
Answer & Comments
expected. Consolidation is not a feature of
bronchiolitis obliterans, which usually presents Correct answer: EH
either with a normal CXR or with signs of
hyperinflation. Bilateral airspace radiological Diffuse parenchymal lung disease (DPLD) is the
changes not responding to an antibiotic may name given to the group of diseases that affect
suggest cryptogenic organising pneumonia the lung parenchyma in a diffuse manner.
(COP). A tissue diagnosis (video-assisted Clubbing is seen in 49-66% of patients with CFA
thoracoscopic lung biopsy) should be obtained, and in up to 75% of patients with rheumatoid
as COP requires long-term oral corticosteroid arthritis related DPLD. Bilateral fine end
treatment, initially in high dose. inspiratory crackles have been reported in up to
90% of patients with CFA. Pleurisy is common in
[ Q: 9 ] MasterClass Part2 DPLD due to rheumatoid arthritis and systemic
(2010) - Respiratory sclerosis. It is rare in CFA.
Lymphangioleiomyomatosis is a disease of
A 55-year-old gentleman is under investigation
young pre-menopausal women and is
for breathlessness. A chest radiograph suggests
characterised by proliferation of smooth muscle
a diffuse parenchymal lung disease.
in the walls of pulmonary lymphatics, which can
Which of the following statements are correct? lead to obstruction and formation of achylous
pleural effusion. It can also lead to airflow
A. Clubbing is seen in over 90% of patients limitation. Pneumothorax also occurs
with cryptogenic fibrosing alveolitis (CFA). commonly.
B. Bilateral fine end inspiratory crackles Goodpasture's syndrome characterised by
occur in 40-50 % of patients with CFA. alveolar haemorrhage and renal impairment
C. Pleurisy is a common feature in CFA. occurs more commonly in smokers.

D. Lymphangioleiomymatosis is commonly Haemoptysis is commonly seen in pulmonary


seen in tall young men. vasculitis and not in lymphangitic carcinoma.

Dr. Khalid Yusuf El-Zohry – Sohag Teaching Hospital (01118391123) Page | 527
El-Zohry MRCP Questions Bank (Part 2) – Medical Masterclass 2010

Raised serum ACE levels are very non-specific Non invasive positive pressure ventilation
and not diagnostic of sarcoidosis. TBLB can should be considered when there is a need for
result in pneumothorax in 0.7-2% of cases, this ventilatory assistance as indicated by worsening
is more common if samples are taken from the dyspnoea, acute respiratory acidosis and
right middle lobe or the lingula. Tranction worsening oxygenation.
bronchiectasis on HRCT suggests established
fibrosis and irreversibility. [ Q: 11 ] MasterClass Part2
(2010) - Respiratory
[ Q: 10 ] MasterClass Part2
A 26-year-old man is admitted to hospital with
(2010) - Respiratory
a history of dyspnoea of 4-6 weeks duration. He
An 86-year-old lady, generally fit and self- initially had flu-like symptoms and was treated
caring, is admitted with an acute exacerbation by his doctor with a 10-day course of
of chronic obstructive pulmonary disease ciprofloxacin. However, he then started
(COPD). One hour after admission she remains coughing up blood, leading to urgent referral.
distressed with a respiratory rate of 30/minute On examination he was dyspnoeic at rest, with
and is peripherally cyanosed. Repeated arterial bilateral crackles on auscultation of the lungs.
blood gases show a severe respiratory acidosis Investigation revealed anaemia and impaired
with a pH of <7.3. She is becoming exhausted, renal function (creatinine 220 micromol/l).
oxygen saturations are falling, and the on call Pulmonary function tests were normal apart
anaethetist says there are no intensive care from an abnormally high diffusion factor. Urine
beds available for mechanical ventilation. dipstick testing showed the presence of red
blood cells.
Which of the following treatments is correct?
What is the most likely diagnosis?
A. Repeat bronchodilator therapy and
arrange repeat arterial gases in one hour. A. Lymphangiomyomatosis
B. Give intravenous infusion of B. Goodpasture's syndrome
aminophylline.
C. Chronic eosinophilic pneumonia
C. Intubate the patient at the bedside and
D. Bronchiolitis obliterans
manually ventilate until a bed is available.
E. Invasive Aspergillosis.
D. Arrange for noninvasive positive pressure
ventilation.
Answer & Comments
E. Give intravenous hydrocortisone.
Correct answer: B
Answer & Comments
This patient has both lung and kidney
Correct answer: D involvement typical of a 'pulmonary renal
syndrome'.
Enhancing ventilation by unloading fatigued
Goodpasture described the association of
ventilatory muscles is an important goal in
pulmonary haemorrhage with renal failure
treating exacerbations of acute COPD. Non
(Goodpasture's syndrome) in 1919, and the
invasive positive pressure ventilation has been
'classic' cause of this, where the condition is
shown to reduce rates of intubation, lower
due to the presence of circulating anti-
hospital mortality rates and lead to shorter
glomerular basement membrane antibodies
hospital stays.

Dr. Khalid Yusuf El-Zohry – Sohag Teaching Hospital (01118391123) Page | 528
El-Zohry MRCP Questions Bank (Part 2) – Medical Masterclass 2010

(anti-GBM antibodies) is termed Goodpasture's [ Q: 13 ] MasterClass Part2


disease (although this wasn't the cause of the (2010) - Respiratory
cases he described).
Non-invasive methods of ventilation (NIV) are
Other causes of pulmonary haemorrhage and
increasingly used for patients with a variety of
renal failure include Wegener's granulomatosis,
acute and chronic medical conditions. The
microscopic polyangiitis and systemic lupus
Emergency Department staff have acutely
erythematous (SLE).
admitted a 75-year-old smoker with endstage
chronic obstructive pulmonary disease who has
[ Q: 12 ] MasterClass Part2 been using non-invasive ventilation (NIV) at
(2010) - Respiratory home. They are seeking advice about
management before transferring him to the
A 20-year-old woman was found unconscious. respiratory physicians.
Nearby were an empty bottle of whisky and an
empty bottle of tablets. An ECG revealed a Which of the following statements regarding
broad complex tachcardia at the rate of 130 non-invasive ventilation are correct?
beats per minute.
A. It is not indicated for patients with
Which drug is she MOST likely to have taken? terminal conditions.

A. Diazepam B. It should only be used with a nasogastric


tube in situ.
B. Dihydrocodeine
C. It reduces expiratory effort.
C. Citalopram
D. It can cause facial skin necrosis.
D. Fluoxetine
E. It can be used for comatose patients.
E. Amitriptyline
F. It may be of benefit for patients with
sleep apnoea.
Answer & Comments
G. It increases the work of breathing.
Correct answer: E
H. It should only be used if the patient has a
definitive airway.
Diazepam and codeine overdose generally do
not cause the ECG changes described although I. It can be used only for type 1 respiratory
they can cause sedation. Coma, convulsions and failure.
arrhythmias are the most serious complication
J. It reduces functional residual capacity.
of tricyclic antidepressant overdose.
Tachycardia and QRS prolongation (>100ms) Answer & Comments
signify a significant overdose and may require
the use of intravenous sodium bicarbonate to Correct answer: DF
alkalinise the urine and promote excretion.
Modern NIV can provide support for patients
Fluoxetine and citalopram are selective
with both type 1 and type 2 respiratory failure
serotonin re-uptake inhibitors (SSRIs), have
and is increasingly being used to support
fewer antimuscarinic side effects than the older
patients with conditions such as motor neurone
tricyclics (eg amitriptyline) and are also less
disease where it is mainly CO2 clearance that is
cardiotoxic in overdose.
the problem. It should not be used for patients

Dr. Khalid Yusuf El-Zohry – Sohag Teaching Hospital (01118391123) Page | 529
El-Zohry MRCP Questions Bank (Part 2) – Medical Masterclass 2010

with immediate life-threatening hypoxia or Answer & Comments


ventilatory failure.
Correct answer: E
NIV should not be used for comatose patients
as it does not provide a definitive airway and Although a Ventiliation-Perfusion (VQ) scan
they are at risk of aspiration.
may show mismatches, this can be confounded
Patients with sleep apnoea suffer from by someone with co-existent COPD and
recurrent episodes of arterial desaturation. The therefore the most specific test is still a CTPA.
application of continuous positive airway The D-Dimer, ECG and CXR changes are non-
pressure helps to reduce these. specific although the D-Dimer is extremely
sensitive and a negative value is highly
There is an increased risk of aspiration with NIV
predictive of the absence of a PE.
as the increased inspiratory pressure can result
in aerophagia.
[ Q: 15 ] MasterClass Part2
If the patient has problems with gastric
distension a nasogastric tube may be beneficial. (2010) - Respiratory
Expiratory effort is increased as the patient is A 46-year-old gentleman is referred with a left
expiring against resistance. Inspiratory effort pleural effusion.
and the overall work of breathing are reduced.
Functional residual capacity is increased. Which of the following conditions do NOT cause
pleural effusions?
Common complications include claustrophobia,
gastric distension and eye irritation. Continual A. Asbestos exposure
pressure from a tight fitting mask may result in B. Dressler's syndrome
skin necrosis.
C. Meigs' syndrome
D. Bird fanciers disease
[ Q: 14 ] MasterClass Part2
(2010) - Respiratory E. Sarcoidosis
F. Cryptogenic fibrosing alveolitis
A 55-year-old woman with chronic obstructive
pulmonary disease (COPD) presents with a G. Q fever
history of acute breathlessness and pleuritic H. Kaposi's sarcoma
chest pain. Her arterial blood gases show the
following recordings when taken at room air: I. Nitrofurantoin ingestion
pH 7.4, pO2 6.5kPa, pCO2 3.4 kPa, bicarbonate J. Endoscopic variceal sclerotherapy.
25mmol/L.
Answer & Comments
Which of the following tests is most specific to
acute pulmonary embolism? Correct answer: DF
A. A chest x-ray
Pleural effusions can occur as a complication of
B. An ECG
numerous diseases. These can be classified as
C. A Ventilation perfusion scan transudative or exudative, based on the protein
D. A D-Dimer and lactate dehydrogenase (LDH) level as
compared to the protein and LDH level in the
E. A CT pulmonary angiogram. blood.

Dr. Khalid Yusuf El-Zohry – Sohag Teaching Hospital (01118391123) Page | 530
El-Zohry MRCP Questions Bank (Part 2) – Medical Masterclass 2010

[ Q: 16 ] MasterClass Part2 Which of the following statements is correct?


(2010) - Respiratory A. Radiotherapy is the primary treatment.

A 76-year-old retired builder presents with a B. Patients should be advised about state
history of increasing breathlessness and dull compensation and entitlement to a
ache in his right lateral chest wall for 4 months. disability pension.
He is a smoker of 20 pack years. He is clubbed C. Chemotherapy is the most appropriate
but besides the finding of a pleural effusion treatment.
there are no other abnormal features on
examination. A chest radiograph shows a D. Pleural aspiration is usually adequate for
moderately large effusion. a diagnosis.
E. Sarcomatous mesothelioma has a good
The TWO likeliest diagnoses are: prognosis.
A. Tuberculosis
B. Liver cirrhosis Answer & Comments

C. Hypothyroidism Correct answer: B

D. Bronchial carcinoma
The correct answer is B. As asbestos is
E. Mesothelioma associated with the development of
F. Empyema mesothelioma, all patients should be advised
that they may be eligible for sate compensation
G. Disseminated prostate carcinoma
and disability benefits. These claims must be
H. systemic lupus erythematosus (SLE) made within 3 years of initial diagnosis.
I. Benign asbestos pleural plaques
J. Cardiac failure.
[ Q: 18 ] MasterClass Part2
(2010) - Respiratory
Answer & Comments A 40-year-old man smoker of 40 pack years
presents with a 3- month history of shortness of
Correct answer: DE
breath. His past medical history includes
hypertension, cervical spondylosis and
The two most appropriate diagnoses are depression. His spirometry shows FEV1 of 1.03
mesothelioma or a malignant pleural effusion
litres - 67% predicted, FVC 1.03 litres - 53%
from a primary bronchial carcinoma. The other
predicted and FEV1%VC of 96.
causes of effusions listed are not associated
with chest pain, or the history would be too How would you interpret his spirometry results?
long.
A. Normal for his age

[ Q: 17 ] MasterClass Part2 B. Obstructive defect

(2010) - Respiratory C. Mixed defect

A 62-year-old man presented with increasing D. Restrictive defect


shortness of breath. He was a current smoker E. Unable to interpret as bronchial
with over 20 packs per year exposure. He was reversibility not done.
found to have a right-sided pleural disease. A
final diagnosis of mesothelioma was made.

Dr. Khalid Yusuf El-Zohry – Sohag Teaching Hospital (01118391123) Page | 531
El-Zohry MRCP Questions Bank (Part 2) – Medical Masterclass 2010

Answer & Comments fibrosis and any form of extrapulmonary


restriction) with decreased gas transfer. This
Correct answer: D
pattern is highly suggestive of a pulmonary
vascular defect. Anaemia tends to cause a 3%
Reduced FEV1 and FVC with normal FEV1 ratio drop in KCO for every g/dl Hb lost and therefore
is compatible with restrictive defect this picture would not fit with MILD anaemia.
Cardiac failure can cause a mildly restrictive
[ Q: 19 ] MasterClass Part2 lung defect with decreased gas transfer, but
(2010) - Respiratory this would not be the most likely diagnosis in
this scenario.
A 44-year-old woman with a diagnosis of
limited scleroderma has become increasingly
breathless. She has some lung function tests, [ Q: 20 ] MasterClass Part2
which reveal the following: (2010) - Respiratory
FEV1 95% predicted A 35-year-old postman presents with a history
FVC 98% predicted of breathlessness that has been getting
gradually worse over 6 months. He coughs up
TLC 93% predicted
sputum regularly, but is otherwise well, with no
RV 101% predicted significant past medical history. He smoked five
to ten cigarettes per day from the age of 18 to
TLCO 65% predicted
26 years.
KCO 69% predicted
The TWO most likely diagnoses are:
What is the most likely diagnosis?
A. Cystic fibrosis
A. Pulmonary fibrosis
B. Emphysema
B. Mild anaemia
C. Congestive cardiac failure
C. Extrapulmonary restriction due to
D. alpha 1 antitrypsin deficiency
scleroderma
E. Pneumoconiosis
D. Cardiac failure
F. Asbestosis
E. Pulmonary vascular disease
G. Hypogammaglobulinaemia
Answer & Comments H. Chronic obstructive pulmonary disease
Correct answer: E I. Bronchial asthma
J. Postman’s lung.
Limited scleroderma is usually associated with
the anticentromere antibody and CREST
Answer & Comments
syndrome. The pulmonary complication is
usually pulmonary hypertension, as opposed to Correct answer: AD
pulmonary fibrosis, which is usually associated
with diffuse scleroderma and the anti-Scl-70 The most likely diagnosis to explain gradually
antibody. worsening breathlessness with regular sputum
The pulmonary function tests show essentially production is bronchiectasis. This could arise as
normal lung volumes (against pulmonary a consequence of childhood infection (measles,

Dr. Khalid Yusuf El-Zohry – Sohag Teaching Hospital (01118391123) Page | 532
El-Zohry MRCP Questions Bank (Part 2) – Medical Masterclass 2010

pneumonia, pertussis), cystic fibrosis, alpha 1 would be to give prednisolone 30 mg daily for 2
antitrypsin deficiency or weeks, regarding an increase in FEV1 of >10%
hypogammaglobulinaemia (which would be and >200 ml as a positive response. Given the
suggested by recurrent pneumonia or sinusitis). non-specific effects and many side effects of
steroids, it is crucial to demonstrate functional
[ Q: 21 ] MasterClass Part2 improvement: many patients with COPD have
sustained severe complications of steroid
(2010) - Respiratory
treatment, e.g. vertebral fracture, without any
A 58-year-old man, a smoker for many years evidence that the steroids were beneficial for
despite repeated advice that he should stop, their chest.
has chronic obstructive pulmonary disease that
is increasingly limiting his exercise capacity. You [ Q: 22 ] MasterClass Part2
wish to conduct a trial of steroid therapy.
(2010) - Respiratory
Which of the following is the correct way to do (1) A 25-year-old man is admitted with a
this and interpret the outcome? history of haemoptysis, cough and dyspnoea.
A. Give oral prednisolone, 60 mg daily for 4 He has been previously fit, but has smoked 20
weeks, regarding a clear statement of cigarettes a day for the last 5 years. A chest
subjective improvement by the patient as radiograph shows diffuse alveolar infiltrates. He
a positive response. has a microcytic, hypochromic anaemia; urine
dipstick confirms haematuria and proteinuria;
B. Give oral prednisolone, 10 mg daily for 2
lung function tests show normal spirometry and
weeks, regarding a clear statement of
a TLCO of 135% predicted.
subjective improvement by the patient as
a positive response if accompanied by a What is the most likely diagnosis?
rise in FEV1.
A. Pulmonary tuberculosis
C. Give oral prednisolone, 60 mg daily for 4
B. Recurrent pulmonary emboli
weeks, regarding a clear statement of
subjective improvement by the patient as C. Community acquired pneumonia
a positive response if accompanied by a
D. Congestive cardiac failure
rise in FEV1 of >10%.
E. Goodpasture's syndrome.
D. Give oral prednisolone, 30 mg daily for 2
weeks, regarding an increase in FEV1 of
>10% and >200 ml as a positive response. Answer & Comments

E. Give oral prednisolone, 30 mg daily for 2 Correct answer: E


weeks, regarding a clear statement of
subjective improvement by the patient as Goodpasture's syndrome is characterised by
a positive response. diffuse alveolar haemorrhage and glomerulitis.
Men are commonly affected with most cases
Answer & Comments occurring between the ages of 20-30 years. It is
more likely to occur in smokers. Exposure to
Correct answer: D smoke and volatile hydrocarbons is associated
with initiation as well as exacerbation of the
Steroid challenge is indicated in chronic disease. The anti glomerular basement
obstructive pulmonary disease (COPD) of more antibody is present in up to 90% of the patients.
than moderate severity. Standard practice Renal histology usually shows a focal segmental

Dr. Khalid Yusuf El-Zohry – Sohag Teaching Hospital (01118391123) Page | 533
El-Zohry MRCP Questions Bank (Part 2) – Medical Masterclass 2010

necrotizing glomerulitis with crescent the glucose level is above 1.6. These effusions
formation. tend to have high cholesterol levels
The TLCO is increased during active bleeding
and can be used to monitor the disease. An [ Q: 24 ] MasterClass Part2
increase above 30% of baseline is highly (2010) - Respiratory
suggestive of an intra-alveolar bleed.
A 40-year-old woman is admitted to the
hospital with a parapneumonic effusion. A
[ Q: 23 ] MasterClass Part2 diagnostic pleural tap has been done.
(2010) - Respiratory
Which of the following is an indication for
A 65-year-old man with a history of rheumatoid
inserting a chest drain?
arthritis (RA) develops a left pleural effusion.
A. Temperature above 39°C
Which of the following are typical of B. A rising white cell count and C-reactive
rheumatoid pleural effusions?
protein
A. Over 50% of patients with RA develop a
C. Pleural pH <7.2
pleural effusion at some stage.
D. Blood-stained pleural fluid
B. It is more commonly seen in female
patients. E. Pleural fluid lactate dehydrogenase >200
U/L
C. The glucose level in the effusion fluid is
less than 1.6mmol/l.
Answer & Comments
D. The effusions mainly occur in patients
without rheumatoid nodules. Correct answer: C

E. The effusions are usually transudates.


Indications for the urgent insertion of a chest
F. Intrapleural corticosteroids is the drain for a pleural effusion include empyema
treatment of choice. (pus in the pleural cavity), haemothorax (not
G. They are usually bilateral. the same as blood-stained fluid) or a pH <7.3,
which indicates a highly metabolically active
H. They have high levels of cholesterol. collection (almost always due to infection or
I. The pH of pleural fluid in an effusion due malignancy). In all cases, the fluid is likely to
to RA is usually > 7.30. form locules with thick fibrous septa, and hence
early intervention to prevent this is required.
J. They should be treated with a chest drain.

Answer & Comments [ Q: 25 ] MasterClass Part2


(2010) - Respiratory
Correct answer: CH
A 24-year-old man presents with fever,
Twenty per cent of patients with RA develop breathlessness, cough and sputum production.
pleurisy and only 5% develop pleural effusions. His only medical history of note is long-standing
It is commonly seen in older men with heavy alcohol consumption, but he had no
rheumatoid nodules and is usually an exudate. respiratory complaint whatever until 6 weeks
RA is unlikely to be the cause of an effusion if ago when he developed breathing difficulty
with high fever and rigors. He was given
antibiotics by his general practitioner and

Dr. Khalid Yusuf El-Zohry – Sohag Teaching Hospital (01118391123) Page | 534
El-Zohry MRCP Questions Bank (Part 2) – Medical Masterclass 2010

began to improve, but this improvement has D. Empyema


not been sustained.
E. Reactive pleural effusion.
The most likely diagnosis is:
Answer & Comments
A. Lung abscess
Correct answer: D
B. Bronchiectasis
C. Chronic obstructive pulmonary disease
The most likely diagnosis is D; an empyema
(COPD)
complicating a pneumonia. The other causes
D. Asthma precipitated by chest infection are less likely in the described case but should
be considered in the differential diagnosis.
E. Empyema.

Answer & Comments [ Q: 27 ] MasterClass Part2


(2010) - Respiratory
Correct answer: A
A 46-year-old lady has a pneumonic illness
Bronchiectasis typically causes chronic complicated by a pleural effusion.
breathlessness and sputum production, often
Which of the following are correct?
with febrile infective exacerbations, but will not
be the diagnosis in a patient with a short A. All effusions associated with pneumonias
respiratory history. should be drained.

The history strongly suggests pneumonia 6 B. Chest drain should be inserted if the pH
weeks previously that has failed to clear. of the fluid is <7.2.
Empyema could cause persistent fever, malaise C. The presence of organisms in a Gram
and breathlessness, but would not explain stain of the fluid is not an indication for
continued sputum production. chest drain insertion.
Alcoholism is a risk factor for aspiration and D. Bronchoscopy should be performed to
cavitating pneumonia. exclude a proximal tumour.
E. A complicated effusion has a low lactate
[ Q: 26 ] MasterClass Part2 dehydrogenase level.
(2010) - Respiratory
F. Once the effusion is drained she should
A 47-year-old man presented with a 12-day undergo pleurodesis to prevent
history of coughing, purulent green sputum and recurrence.
right-sided pleuritic chest pain. On examination
G. A large bore tube should be used for
he was found to be pyrexial with the clinical
drainage.
features of a right-sided pleural effusion. His
chest radiograph confirmed the presence of a H. If a tube becomes blocked it should be
moderate right-sided pleural effusion. removed and a new one inserted.
I. A loculated pleural collection should not
The most likely diagnosis is:
be drained.
A. Malignant pleural effusion
J. Intrapleural fibrinolytic drugs may
B. Congestive cardiac failure improve radiological outcomes.
C. Haemothorax

Dr. Khalid Yusuf El-Zohry – Sohag Teaching Hospital (01118391123) Page | 535
El-Zohry MRCP Questions Bank (Part 2) – Medical Masterclass 2010

Answer & Comments Light's criteria for differentiating an exudative


from transudative pleural effusion are the
Correct answer: BJ
presence of one or more of fluid protein/serum
protein >0.5, fluid LDH/serum LDH >0.6 and
Up to 40 % of pneumonias may be fluid LDH more than two-thirds over the upper
accompanied by a pleural effusion. Not all limit of serum normal.
effusions associated with pneumonias need
drainage. The indications to insert a chest drain
in an infected pleural effusion are frank pus on
[ Q: 29 ] MasterClass Part2
aspiration, organisms seen in Gram stain or (2010) - Respiratory
culture of pleural fluid, pleural fluid pH<7.2 and A 65-year-old male presents to A&E with acute
poor clinical progress during treatment with onset shortness of breath. Arterial blood gases
antibiotics. Bronchoscopy should be performed show PO2 7.3, PCO2 3.9 and pH 7.45 breathing
only if there are features suggestive of an air. Blood pressure is 130/70 and pulse is 120,
underlying tumour. Lactate dehydrogenase regular. JVP is elevated 3 cm and there is no
(LDH) is raised in infected pleural effusions and ankle oedema. Chest is clear. No past history of
pleurodesis is not indicated. A small bore tube thromboembolic disease, but has a positive
(8-14F) are equally good as large bore family history. He has no other known risk
tubes(>24F). Loculated pleural collections factors for venous thromboembolic disease. His
should be drained. CT pulmonary angiogram is shown (see image).

[ Q: 28 ] MasterClass Part2
(2010) - Respiratory
A 56-year-old woman has a pleural effusion.
The diagnostic tap shows a pleural fluid protein
of 32 g/L.

Which of the following is one of Light's criteria


for differentiating pleural effusions into
exudates and transudates?
A. Pleural fluid protein divided by serum
protein >0.5
B. Pleural lactate dehydrogenase (LDH)
divided by serum LDH >0.9
C. Pleural fluid glucose divided by serum
glucose >0.3
D. Pleural fluid osmolality divided by serum Which of the following statements is true?
osmolality >0.7
A. Providing there are no contraindications,
E. Pleural fluid LDH less than two-thirds of he should be thrombolysed in view of the
the upper limit of normal serum LDH size of the clot.
B. He should be anticoagulated for life in
Answer & Comments
view of his family history (provided no
Correct answer: A contraindications).

Dr. Khalid Yusuf El-Zohry – Sohag Teaching Hospital (01118391123) Page | 536
El-Zohry MRCP Questions Bank (Part 2) – Medical Masterclass 2010

C. He should receive prophylactic heparin [ Q: 30 ] MasterClass Part2


for long-haul flights in future. (2010) - Respiratory
D. He should be screened for common
A 55-year-old woman is referred with a 10-day
cancers.
history of productive cough, fever and
E. It is better to use unfractionated heparin increasing shortness of breath.
acutely if he has lupus anticoagulant to
achieve higher levels of anticoagulation.

Answer & Comments

Correct answer: C

He has suffered a non-massive pulmonary


embolus (PE). Massive pulmonary embolus is
defined as pulmonary embolus associated with
hypotension (systolic BP <90mmHg) or other
adverse features such as cardiac arrest.
Thrombolysis is only currently indicated for
massive PE. Life-long prophylaxis after first
idiopathic PE should only be considered with
the results of thrombolphilia testing and must
be balanced against the risk of bleeding. Life-
long prophylaxis remains debatable after first
PE, except in the context of lupus
anticoagulant. He should receive low molecular Her chest radiograph shows:
weight heparin and venous compression
A. widened mediastinum
stockings for future long-haul flights.
B. right upper zone consolidation
There is an increased risk of cancer being
detected within 6–12 months of a first episode C. right upper lobe collapse
of venous thromboembolism (VTE), particularly D. superior mediastinal mass
in those with no other risk factors and/or
recurrent episodes. Previously unrecognised E. anomalous superior vena cava.
cancer, present in 7–12% of those with
idiopathic VTE, can usually be detected by a Answer & Comments
combination of careful clinical assessment,
Correct answer: C
routine blood tests, and chest radiography and,
if these are satisfactory, the current consensus
is that it is not appropriate to proceed to tests Note also the tenting of the right
such as ultrasound, CT scanning or endoscopy. hemidiaphragm and the raised right hilum.

He should not be anticoagulated with Depending on the clinical context the


unfractionated heparin if he has lupus commonest causes to consider would be
anticoagulant since this abnormality interferes infection, carcinoma of the lung and foreign
with activated partial thromboplastin time body. Other tumours, e.g. carcinoid, benign
(APTT) monitoring. adenoma, and pulmonary infarct would be
much less likely.

Dr. Khalid Yusuf El-Zohry – Sohag Teaching Hospital (01118391123) Page | 537
El-Zohry MRCP Questions Bank (Part 2) – Medical Masterclass 2010

[ Q: 31 ] MasterClass Part2 What is the most likely diagnosis?


(2010) - Respiratory A. Bronchiectasis

A 63-year-old man attends the hospital with a B. Tuberculosis


history of proximal muscle weakness. He also C. Cryptogenic fibrosing alveolitis
gives a history of cough of 8 weeks duration
and complains of pain of the small joints of the D. Lymphangioleiomyomatosis
hands. He has small haemorrhages in the nail E. Sarcoidosis
folds, but is not clubbed. In the chest he has
bibasal crackles, and a chest radiograph reveals Answer & Comments
diffuse reticular infiltrates. Lung function tests
confirm a restrictive pattern. Correct answer: E

What is the underlying cause of his interstitial Bilateral hilar lymphadenopathy and erythema
lung disease? nodosum is a typical presentation of
A. Cryptogenic fibrosing alveolitis sarcoidosis. The diagnosis should be confirmed
by tissue biopsy, and the other main
B. Ankylosing spondylitis
differential, tuberculosis, excluded.
C. Polymyositis and dermatomyositis
D. Rheumatoid arthritis [ Q: 33 ] MasterClass Part2
E. Mixed connective tissue disorder. (2010) - Respiratory
A 22-year-old man presents to A&E claiming to
Answer & Comments have taken a large quantity of paracetamol 24
Correct answer: C hours previously. He washed the tablets down
with vodka.

Polymyositis and dermatomyositis are Which of the following statements are correct?
inflammatory conditions involving the muscle
A. Measuring paracetamol levels at 24 hours
and skin. Patients often complain of proximal
muscle weakness and of pain in the small joints is of no use.
of the fingers. They may have ragged cuticles B. N-acetyl cysteine should be witheld until
and haemorrhages at the finger nail folds. plasma paracetamol levels are known.
Interstitial lung disease can occur. Underlying
C. The prognostic accuracy of the treatment
malignancy (lungs, ovaries, breasts and
nomogram is less certain at 24 hours
stomach) is present in 5-8% of cases.
post-ingestion.
D. Methionine can be given if the patient is
[ Q: 32 ] MasterClass Part2
intolerant of N-acetyl cysteine (NAC).
(2010) - Respiratory
E. Clinical symptoms may occur > 24 hours
A 33-year-old previously fit woman is admitted post-ingestion.
to the hospital with a 10-week history of
F. If the urea and electrolytes are normal,
progressive breathlessness and dry cough. She
the patient can be sent home.
has never smoked and works in a supermarket.
In the last 3 days she has developed painful G. Even at 24 hours, activated charcoal
dusky-coloured nodules on her shins. A CXR should be given as gastric transit time is
reveals bilateral hilar shadows. prolonged by paracetamol.

Dr. Khalid Yusuf El-Zohry – Sohag Teaching Hospital (01118391123) Page | 538
El-Zohry MRCP Questions Bank (Part 2) – Medical Masterclass 2010

H. The measurement of alanine transferase A. It is essential to check his BM stix and


is of prognostic value. give dextrose if this is low.
I. If he drank more than two units of alcohol B. He should be given naloxone 400mcg iv.
with the overdose, he is at greater risk.
C. Treatment with 50g of activated charcoal
J. If he is obese, he is at greater risk from is not indicated.
the effects of the overdose.
D. Gastric lavage is not indicated.
E. There is no benefit in taking blood for
Answer & Comments
paracetamol and salicylate levels.
Correct answer: CE
Answer & Comments
Levels should always be measured. If
Correct answer: E
paracetamol is still present it may suggest that
the overdose was taken over a period of time
making interpretation of the nomogram very Basic management should always include
difficult. If a clinically significant quantity of maintenance of airway, fluid resuscitation
paracetamol has been taken, start NAC whilst where appropriate and correction of
waiting for the results - it can always be hypoglycaemia.
stopped (in most hospitals it takes a significant He is known to have a history of opiate abuse
amount of time for blood tests to be processed and has a reduced GCS and pinpoint pupils so it
and the results retrieved). Most reactions to would be reasonable to give naloxone iv. This is
NAC are mild and can be overcome by giving it the correct dose and can be repeated as often
more slowly. For patients who are truly as necessary.
intolerant, methionine can be given up to 12
hours after ingestion. Clotting, liver function Activated charcoal does not adsorb lithium,
and U&E should all be checked before opiates or alcohol.
consideration is made as to whether the patient Gastric lavage has not been shown to be of
could go home. The best prognostic indicator is benefit, indeed there is some evidence that it
the INR. Patients who are anorexic for any increases absorption by forcing the drugs
reason, who drink significantly (>21 units/wk through the pylorus.
for men, >14 units/wk for women) or who are
E is not true. Overdoses are usually of multiple
on enzyme-inducing drugs are at greater risk
medications and this should be done.
from paracetamol overdose.

[ Q: 35 ] MasterClass Part2
[ Q: 34 ] MasterClass Part2
(2010) - Respiratory
(2010) - Respiratory
A 67-year-old man has been diagnosed with
A 25-year-old man with a history of bipolar
chronic obstructive pulmonary disease (COPD).
affective disorder and opiate abuse is brought
Spirometry confirms severe COPD with a FEV1
into A&E by ambulance. His GCS is 9/15. He
of less than 30% predicted. In the last 12
smells strongly of alcohol and has pinpoint
months he has been admitted to hospital on
pupils.
four occasions with COPD exacerbation.
Which of the following statements about his
Which one of the following statements is true?
management is NOT correct?

Dr. Khalid Yusuf El-Zohry – Sohag Teaching Hospital (01118391123) Page | 539
El-Zohry MRCP Questions Bank (Part 2) – Medical Masterclass 2010

A. Exacerbations can be reduced by daily Answer & Comments


oral steroids.
Correct answer: D
B. Nebulisers help to reduce exacerbations.
C. Long-acting beta 2 agonists reduce It is important to make an accurate diagnosis,
exacerbations. and to treat the effusion. Pleural tap has about
a 60% sensitivity for pleural malignancy,
D. Oral theophyllines can reduce rate of
whereas thoracoscopic biopsy will make the
exacerbations.
diagnosis in 98% of cases. In addition, the fluid
E. Long-acting anticholinergics can reduce can be drained and pleurodesis performed in
exacerbation rate. the same procedure. If mesothelioma is
confirmed, the patient will need to have
Answer & Comments radiotherapy to the site to prevent extension of
tumour along the tract.
Correct answer: E

[ Q: 37 ] MasterClass Part2
Severe COPD is diagnosed if the FEV1 is less or
equal to 30% predicted. Studies have shown (2010) - Respiratory
that patients treated with long acting A 62-year-old lady has had a right sided
anticholinergic (tiotropium) had fewer posterior chest pain for the last three months.
exacerbations per year. Use of rescue It is exacerbated by deep inspiration. Over the
medication was often less in patients on past few weeks she has also noticed increasing
tiotropium. dyspnoea. She is a smoker with a history of 30
pack years. There are no systemic symptoms
[ Q: 36 ] MasterClass Part2 but she does have pain in both wrists and
(2010) - Respiratory ankles. Her chest radiograph is shown.

A 65-year-old retired builder is admitted with a


left pleural effusion. He is an ex-smoker with a
smoking history of 50 pack-years. As part of his
employment he was exposed to asbestos 40
years ago. You suspect the underlying diagnosis
to be mesothelioma.

Which of the following is the investigation of


choice?
A. High-resolution CT scan of the lungs
B. Bronchoscopy
C. Pulmonary function test
D. Video-assisted thoracoscopy and pleural
biopsy
E. Diagnostic pleural tap and insertion of a The TWO most likely causes of her condition
chest drain are:
A. empyema
B. pulmonary emboli

Dr. Khalid Yusuf El-Zohry – Sohag Teaching Hospital (01118391123) Page | 540
El-Zohry MRCP Questions Bank (Part 2) – Medical Masterclass 2010

C. para-pneumonic effusion B. The short time lag between the start of


symptoms and hospital admission is a risk
D. tuberculosis
factor.
E. right heart failure
C. Female patients are especially at risk.
F. rheumatoid arthritis
D. The risk increases in obese patients.
G. malignant effusion secondary to lung
E. Family history of asthma is common.
cancer
F. Allergy to peanut is commoner in patients
H. mesothelioma
with near fatal asthma.
I. wegners granulomatosis
G. Near fatal asthma occurs more frequently
J. pancreatitis. during winter season.
H. Long acting beta2 agonists are beneficial
Answer & Comments in preventing asthma attacks.
Correct answer: FG I. Childhood eczema is a risk factor.
J. Parental smoking is a risk factor.
The key symptoms suggest a slowly developing
pleural effusion. In addition the wrist and ankle
pain may be in keeping with either hypertrophic Answer & Comments
pulmonary osteo-arthropathy (HPOA) Correct answer: AB
secondary to lung cancer, or rheumatoid
arthritis. The essential tests are examination of
Near fatal asthma or brittle asthma is
the pleural fluid for MC&S, auramine stain and
responsible for around 1000 deaths every year
TB culture, cytology, protein, glucose and
in the UK. It mainly occurs in young patients.
lactate dehydrogenase (LDH) concentrations,
Previous attacks and the short time between
pH, and rheumatoid factor.
the start of symptoms and hospital admission
are the two main risk factors. The treatment of
[ Q: 38 ] MasterClass Part2 choice is hospitalisation with early start of
(2010) - Respiratory nebulised bronchodilators, oxygen and
intravenous corticosteroids. The recovery rate
A 27-year-old woman with asthma was is normally quick. Long acting beta2 agonists
admitted with an acute attack. Her symptoms and leukotrine receptor antagonists are not
started around 8.00am, but by 10.00am she successful in preventing near fatal asthma.
was so unwell that she needed admission to the
hospital. Shortly after arriving to the accident There is no known relationship between near
and emergency she was admitted to the fatal asthma and personal history of allergy.
intensive care unit. Similar incidents had The incidence of the disease does not differ
occurred twice in the past. according to gender and there are no seasonal
variations for the rate of hospitalisation.
Which TWO of the following statements are
correct for near fatal asthma? [ Q: 39 ] MasterClass Part2
A. Previous severe asthma attacks are a risk (2010) - Respiratory
factor.
A 64-year-old lady is reviewed in outpatients
with the histopathology results from a biopsy
taken during bronchoscopy. The report shows

Dr. Khalid Yusuf El-Zohry – Sohag Teaching Hospital (01118391123) Page | 541
El-Zohry MRCP Questions Bank (Part 2) – Medical Masterclass 2010

cells consistent with a small cell carcinoma. She


has been referred to the local multi-disciplinary
team but her case as yet has not been
discussed. Both the patient and her son want to
know what future treatments she might be
offered.

Which of the following statements are correct?


A. Small cell carcinoma does not
metastasise early.
B. Small cell carcinoma has a slow doubling
time.
What is the likeliest diagnosis?
C. Limited small cell carcinoma is defined as
tumour confined to the mediastinum. A. Testicular carcinoma with pulmonary
metastases
D. Bone marrow evaluation is essential for
staging. B. Sarcoidosis
E. Patients with small cell should undergo a C. Disseminated adenocarcinoma
chest CT, bone scanning and a brain CT. D. Miliary tuberculosis
E. Lymphoma.
Answer & Comments

Correct answer: E Answer & Comments

Correct answer: D
Small cell lung cancer is characterised by its
propensity for early metastases and rapid
doubling time. Limited disease is defined as This patient has classical miliary tuberculosis,
tumour that can be encompassed within a given his chest radiograph and CT findings of
single tolerable radiation port. Staging should miliary nodular shadowing with possible
include a chest CT, a brain CT and bone evidence of tuberculous epididymitis.
scanning. Bone marrow evaluation adds little Sarcoidosis is a possibility but is less likely given
information and is not required. the lack of lymphadenopathy. It is unlikely to be
testicular carcinoma given his age and that this
causes larger pulmonary nodules. The other
[ Q: 40 ] MasterClass Part2 conditions do not normally present with
(2010) - Respiratory testicular pain. Pulmonary lymphoma causes
A 60-year-old Somalian man presents with a 2- hilar lymphadenopathy
month history of weight loss, night sweats and
back pain. A chest radiograph shows bilateral [ Q: 41 ] MasterClass Part2
fine nodular shadowing but no (2010) - Respiratory
lymphadenopathy. A high resolution CT is
performed (see image). A 72-year-old man has a recurrent pleural
effusion of unknown cause.

Which one of the following statements


regarding chemical pleurodesis is true?

Dr. Khalid Yusuf El-Zohry – Sohag Teaching Hospital (01118391123) Page | 542
El-Zohry MRCP Questions Bank (Part 2) – Medical Masterclass 2010

A. Talc has a success rate of less than 50% D. Pulse oximetry classically shows low
oxygen saturation.
B. Acute respiratory failure is a known
complication with talc E. Carbon monoxide cannot be measured in
expired air.
C. Talc causes fever in over 95% of cases
D. Tetracycline has a success rate of over
Answer & Comments
90%
Correct answer: A
E. It should be performed under general
anaesthesia
Carbon monoxide poisoning results in the death
of about 50 people in the UK each year. It is
Answer & Comments
produced by the incomplete combustion of
Correct answer: B carbon containing fuel: gas, coal, oil, wood and
coke. Inadequate maintenance leading to poor
Chemical pleurodesis can be performed under combustion of fuel and inadequate removal of
sedation. Talc has a success rate of over 93% waste products as a result of blocked chimney
while tetracycline has a success rate of 67%. are the main causes of poisoning.
Chest pain and fever are the most common Headache is the most common symptom (90%)
adverse effects seen with all chemical agents. followed by nausea & vomitting (50%), vertigo
The fever due to talc usually occurs 4-12 hours (50%), alteration in consciousness (30%) and
after instillation and does not last longer than weakness (20%).
72 hours. A few cases of acute respiratory
The cherry red skin colour occurs when COHb
failure, empyema & arrythmia have been
concentration exceeds 20% but it is rarely seen
described with talc
in life.

[ Q: 42 ] MasterClass Part2 Pulse oximetry gives false high oxygen


saturation and it is not recommended.
(2010) - Respiratory
CO levels can be measured in expired air and
A 24-year-old man with a history of depressive this must be done soon after the exposure.
illness is brought to the accident and
emergency department after being found
unconscious in a garage. Carbon monoxide [ Q: 43 ] MasterClass Part2
poisoning is considered. (2010) - Respiratory
A 65-year-old man presents with a history of
Which one of the following statements
worsening breathlessness and cough. His
regarding carbon monoxide and carbon
monoxide poisoning is true? arterial blood gases show the following
recordings when taken at room air: pH 7.26
A. It is produced by the incomplete /pO2 6.5kPa/pCO2 9.5 kPa /Bicarbonate
combustion of carbon containing fuel. 32mmol/L.
B. Alteration in consciousness is the most
frequently seen symptom in carbon Which of the following is the most likely
diagnosis?
monoxide poisoning.
A. Obstructive sleep apnoea (OSA)
C. The presence of a cherry red skin colour is
found in about 50% of cases presenting to B. Acute asthma
hospital.
C. Pulmonary embolus

Dr. Khalid Yusuf El-Zohry – Sohag Teaching Hospital (01118391123) Page | 543
El-Zohry MRCP Questions Bank (Part 2) – Medical Masterclass 2010

D. Acute exacerbation chronic obstructive [ Q: 45 ] MasterClass Part2


pulmonary disease (COPD) (2010) - Respiratory
E. Pulmonary oedema.
A 43-year-old woman was admitted with
infective exacerbation of asthma. She
Answer & Comments responded to treatment but the house officer
Correct answer: D was concerned that Aspergillus fumigatus was
cultured from her sputum. Subsequently
arranged serum total, IgE level was elevated at
Although OSA can cause chronic respiratory
438 ng/ml (normal 40-180 ng/ml), RAST to
failure, it is unusual to have chronic type 2
Aspergillus fumigatus was class III, Aspergillus
respiratory failure except in combination with
fumigatus precipitins were negative.
some other cardiopulmonary illness. This blood
gas would be most compatible with a patient What would be the most appropriate action?
with severe COPD and chronic type 2
respiratory failure with an acute exacerbation. A. High dose oral corticosteroids
B. Itraconazole
[ Q: 44 ] MasterClass Part2 C. High dose oral corticosteroids and
(2010) - Respiratory Itraconazole

A 75-year-old gentleman presents with a 6- D. Chest X ray


month history of worsening breathlessness and E. No change in medication.
a non-productive cough. On examination he has
evidence of clubbing and basal crackles. A chest
Answer & Comments
X-ray shows bibasal shadowing.
Correct answer: E
Which of the following diagnoses is most likely?
A. Organising pneumonia With negative Aspergillus fumigatus precipitins
and serum total IgE less than 1000 ng/ml, this
B. COPD
patient is unlikely to have Allergic
C. Bronchiectasis Bronchopulmonary Aspergillosis (ABPA).
D. Bronchoalveolar cell carcinoma Many asthmatics have one or more findings of
E. Cryptogenic fibrosing alveolitis ABPA but do not meet full criteria for the
diagnosis. The presence of isolated ABPA
features may cause diagnostic confusion but
Answer & Comments
does not appear to affect prognosis. Features of
Correct answer: E ABPA which are found commonly in asthmatics
without ABPA include:
Although clubbing and crackles can also  Positive immediate skin reactivity to
accompany patients with bronchiectasis and Aspergillus fumigatus, which is present
alveolar cell carcinoma, these 2 conditions will in 20-30% of all asthmatics
tend to be productive on coughing. CFA is the
 Positive serum precipitins to
most likely diagnosis in this case. COPD and
Aspergillus, which occur in 10% of
organising pneumonia do not cause clubbing.
asthmatics without ABPA and in 10% of
non-asthmatic patients with chronic
lung disease

Dr. Khalid Yusuf El-Zohry – Sohag Teaching Hospital (01118391123) Page | 544
El-Zohry MRCP Questions Bank (Part 2) – Medical Masterclass 2010

 Recurrent mucoid impaction and [ Q: 47 ] MasterClass Part2


atelectasis (2010) - Respiratory
 Peripheral blood eosinophilia and
A 24-year-old woman gives a history of cough
elevation of serum total IgE.
and dyspnoea for many years. She also gets
frequent chest infections treated with
[ Q: 46 ] MasterClass Part2 prolonged courses of antibiotics and coughs up
(2010) - Respiratory an eggcupful of phlegm daily. Three weeks ago
she had an episode of haemoptysis. She has
A 24-year-old woman complains of excessive
never smoked and works in an office.
daytime sleepiness, and also mentions that she
has fallen asleep at work. Her partner says that Which of the following investigations will
he has sometimes found her asleep during establish the diagnosis?
supper. A diagnosis of narcolepsy is made.
A. V/Q scan
Which of the following is true? B. Bronchoscopy
A. Nocturnal apnoea spells are characteristic C. Spirometry
of this condition.
D. High resolution CT chest scan
B. Nasal Continuous Positive Airway
E. Pek expiratory flow recording
Pressure (CPAP) is the treatment of
choice.
Answer & Comments
C. Patients with narcolepsy have raised
hypocretin levels in the cerebrospinal Correct answer: D
fluid (CSF).
D. Cataplexy is pathognomic. The history is suggestive of bronchiectasis and
high resolution CT of the chest will establish the
E. Narcoleptic patients have normal correct diagnosis.
nocturnal sleep pattern.

[ Q: 48 ] MasterClass Part2
Answer & Comments
(2010) - Respiratory
Correct answer: D
A 61-year-old woman with rheumatoid arthritis
is referred with a history of recurrent chest
Narcolepsy is characterised by cataplexy, sleep
infections, intermittent wheeze and production
paralysis, hypnagogic hallucinations and
of half a teacupful of phlegm daily, on occasions
excessive daytime sleepiness. Over 24 hours,
with a streak of blood. She is a retired secretary
patients with narcolepsy do not sleep more
and has never smoked.
than normal controls, but they are prone to fall
asleep throughout the day, often at What is the most likely diagnosis?
inappropriate times. Nocturnal apnoea spells
are not a feature. Such patients lack hypocretin A. Chronic bronchitis and emphysema
in the CSF. In patients with narcolepsy, night B. Lung cancer
sleep is often interrupted by repeated
C. Diffuse interstitial lung fibrosis
awakenings and terrifying dreams.
D. Bronchiectasis
E. Tuberculosis.

Dr. Khalid Yusuf El-Zohry – Sohag Teaching Hospital (01118391123) Page | 545
El-Zohry MRCP Questions Bank (Part 2) – Medical Masterclass 2010

Answer & Comments C. Reduction in inspired oxygen


concentration to 28% via a Venturi mask
Correct answer: D
and recheck arterial blood gases in 20
minutes
Bronchiectasis is associated with rheumatoid
D. Reduction in inspired oxygen
arthritis, occurring in 3-4% of patients with this
condition. As with all other causes of concentration to 60% via a Venturi mask
bronchiectasis, it presents with recurrent chest and recheck arterial blood gases in 20
infections and excessive phlegm. Recurrent minutes
haemoptysis is a common feature. E. Application of non-invasive ventilation via
Chronic obstructive pulmonary disease (COPD) a tight-fitting facemask
and lung cancer are unlikely in a non-smoker.
Tuberculosis should be excluded as it can cause Answer & Comments
bronchiectasis and haemoptysis.
Correct answer: D
Interstitial fibrosis occurs in up to 20% of
patients with rheumatoid arthritis but is not The patient has evidence of type II respiratory
associated with sputum production or failure with elevation of the pCO2. Fortunately
haemoptysis (unless there is an associated he is alert and orientated which allows you
cancer). some time to initiate treatment and try to avoid
The woman should undergo pulmonary intubation and ventilation (once intubated it
function tests to assess the presence of airway often proves extremely difficult to wean such
obstruction, which is associated with patients from the ventilator). Part of the
bronchiectasis. A HRCT scan of the lungs is the problem may be related to administration of
investigation of choice to confirm presence of high flow oxygen in someone who normally
bronchiectasis. relies on hypoxic drive to stimulate respiration.
The ambulance crew may well have been right
to give high flow O2 initially as hypoxia kills far
[ Q: 49 ] MasterClass Part2
quicker than hypercarbia: the oxygen
(2010) - Respiratory concentration should now be reduced slowly
A 66-year-old man with known chronic whilst rechecking blood gases regularly.
obstructive pulmonary disease (COPD) presents Switching abruptly to low concentration oxygen
via the emergency department with an acute may render the patient both hypoxic and
exacerbation. The ambulance crew have put hypercarbic (a bad combination).
him on high flow oxygen using a reservoir mask. IV aminophylline is part of the armament of
His initial arterial blood gases are: pH 7.32, treatment in such cases but other treatments
pCO2 7.1 kPa, pO2 15.2kPa, standard base should have been tried first. Similarly, non-
excess -1.7mmol/l, bicarbonate 32mmol/l. The invasive ventilation may be extremely useful
patient is alert and orientated. and avoid intubation. It is increasingly used
outside the ICU setting.
Appropriate initial management would be:
A. Immediate rapid sequence induction, [ Q: 50 ] MasterClass Part2
intubation and ventilation
(2010) - Respiratory
B. Intravenous administration of a bolus of
You are asked to speak to a 58-year-old man
aminophylline (250mg) followed by a
and his wife on the ward. He has been admitted
maintenance infusion
for routine surgery (cholecystectomy) and
Dr. Khalid Yusuf El-Zohry – Sohag Teaching Hospital (01118391123) Page | 546
El-Zohry MRCP Questions Bank (Part 2) – Medical Masterclass 2010

when clerked says that he worked in shipyards Mesothelioma is a tumour of the mesothlial
for many years. The house surgeon enquires cells or the pro-mesothelial cells. It affects the
about asbestos exposure, and this leads to a pleura and, to a lesser extent, the pericardium
great deal of anxiety. His wife is very keen to and the peritoneum. The incidence of
know what medical risks this poses for him. mesothelioma increases in patients with heavy
asbestos exposure and in those who were
Which one of the following statements exposed to asbestos at a young age. Cigarette
regarding asbestos exposure or asbestos- smoking does not increase the incidence of
related disease is true? mesothelioma.
A. pleural plaques are often associated with Mesothelioma is a fatal disease. The majority of
restrictive defect in respiratory function patients die within 18 months of diagnosis. The
tests. disease is resistant to current treatment
B. the risk of mesothelioma increases with modalities such as chemotherapy and
smoking. radiotherapy. Tumor resection is indicated in
only a small proportion of patients.
C. the risk of lung cancer increases by 50-
fold in asbestos workers who smoked Lung cancer is associated with heavy asbestos
compared to persons who neither exposure. The effects of asbestos exposure and
smoked nor worked with asbestos. smoking are multiplicative. The risk of lung
cancer in asbestos workers who never smoke is
D. chemotherapy is the treatment of choice
increased by 5-fold. In smokers the risk is about
in patients with mesothelioma.
55-fold that in non-smoking persons who never
E. adenocarcinoma is the predominant worked with asbestos.
histological type in asbestos workers with
lung cancer.
[ Q: 51 ] MasterClass Part2
(2010) - Respiratory
Answer & Comments
A 78-year-old man is referred to the chest clinic
Correct answer: C with history of dyspnoea for over 3 months. He
is a retired teacher and has never smoked. He
Asbestos exposure is associated with: has a past medical history of atrial fibrillation,
 pleural plaques which is well controlled on digoxin. He also
takes warfarin and the occasional paracetamol.
 diffuse pleural thickening He is clubbed and hypoxic on air with a SaO2 of
 lung fibrosis (also known as asbestosis) 89%. He has bilateral crackles and a chest
radiograph confirms fibrosing alveolitis.
 lung cancer
 mesothelioma. Which of the following combination of lung
function is typical of fibrosing alveolitis?
The commonest abnormality seen in asbestos
workers is pleural plaques, which are well- A. Reduced FEV1 & FVC, FEV1/FVC<70%,
circumscribed areas of thickening affecting the raised TLC & RV, reduced TLCO
parietal pleura. Pleural plaques are even seen in B. Reduced FEV1 & FVC, FEV1/FVC >70%,
patients with mild asbestos exposure. In the raised TLC & RV, reduced TLCO
majority of patients, these plaques are
asymptomatic and do not cause any changes in C. Normal FEV1 & FVC, FEV1/FVC >70%,
respiratory function tests. raised TLC & RV, reduced TLCO

Dr. Khalid Yusuf El-Zohry – Sohag Teaching Hospital (01118391123) Page | 547
El-Zohry MRCP Questions Bank (Part 2) – Medical Masterclass 2010

D. Reduced FEV1 & FVC, FEV1/FVC>70%, Answer & Comments


reduced TLC & RV, reduced TLCO
Correct answer: C
E. Reduced FEV1 & FVC, FEV1/FVC >70%,
raised TLC & RV, normal TLCO. Type 1 respiratory failure is defined as pO2 <8
kPa with pCO2 <6.5 kPa.
Answer & Comments
Type 2 respiratory failure is defined as pO2 <8
Correct answer: D kPa with pCO2 >6.5 kPa.
The base excess is a calculated value that
Restrictive lung disorders are characterised by provides a measure of the degree of ‘metabolic’
reduced FEV1 & FVC, FEV1/FVC >70%, reduced as opposed to ‘respiratory’ disturbance of pH. A
TLC & RV and reduced TLCO. normal value is between –2 and +2. If this man
Obstructive disorders are characterised by had metabolic acidosis as a result of his
reduced FEV1 & FVC, FEV1/FVC < 70%, raised diabetes, then a significantly negative ‘base
TLC & RV (gas trapping) and reduced TLCO excess’ would be expected.
(emphysema) or normal or raised TLCO (non-
smoking asthmatics). [ Q: 53 ] MasterClass Part2
(2010) - Respiratory
[ Q: 52 ] MasterClass Part2
A 21-year-old patient with cystic fibrosis (CF)
(2010) - Respiratory presented with weight loss and repeated
A 55-year-old man with insulin-dependent infective exacerbations. Her fasting blood
diabetes mellitus and chronic obstructive glucose was found to be 18 mmol/litre (324
pulmonary disease presents with five days of mg/dl).
worsening breathlessness, fever and cough. On
Which of the following statements is correct?
examination he looks very unwell, with a
temperature of 38.7ºC, a respiratory rate of A. She will be prone to repeated infected
18/min and widespread coarse crackles and exacerbations of her lung disease.
wheezes in his chest. A finger prick test reveals
B. She must start a glucose-reduced diet and
a glucose level of 18 mmol/l. His arterial blood
oral hypoglycaemic agents as a first-line
gases (breathing air) are as follows: pH 7.30,
treatment.
pO2 7.8 kPa, pCO2 6.8 kPa, base excess –2
mmol/l. C. She is likely to develop retinopathy within
2 years if her disease is untreated.
Which of the following is the most likely
D. Gaining weight should be avoided to
explanation for these findings?
reduce insulin resistance.
A. He has a metabolic acidosis with
E. CF-Related diabetes is a contraindication
respiratory compensation.
for heart lung transplant.
B. He has type 1 respiratory failure.
C. He has type 2 respiratory failure. Answer & Comments

D. He has type 1 respiratory failure and Correct answer: A


significant metabolic acidosis.
E. He has type 2 respiratory failure with CF-related diabetes is seen in up to 20% of
compensatory metabolic alkalosis. adult patients with CF. The manifestations of

Dr. Khalid Yusuf El-Zohry – Sohag Teaching Hospital (01118391123) Page | 548
El-Zohry MRCP Questions Bank (Part 2) – Medical Masterclass 2010

the disease are with weight loss and rapid G. A normal arterial carbon dioxide
decline in lung function. concentration suggests the alternative
diagnosis of hysterical hyperventilation.
Glucose levels fluctuate for the first few years,
but are consistently raised during H. Low dose diazepam is useful for
exacerbations. Diet is not a suitable alleviating anxiety.
management for CF-related diabetes. The use
I. The degree of pulsus paradoxus correlates
of oral hypoglycaemic agents is controversial.
with the severity of the attack.
The treatment of choice is injectable insulin.
J. Absence of wheeze excludes the
Due to the short life latency of CF patients and
diagnosis.
that this may be due to the different natural
history of the disease, microvascular
complications of CF-related diabetes are not Answer & Comments
common. The condition is not a Correct answer: CI
contraindication for organ transplantation
when it is well-controlled.
Aminophylline is a pulmonary vasodilator and
can rapidly worsen the VQ mismatch.
[ Q: 54 ] MasterClass Part2
Beta-2 agonists are a commonly used treatment
(2010) - Respiratory for hyperkalaemia as they drive potassium into
A 22-year-old woman with known severe the cells.
asthma presents to the Emergency Department Acute asthmatics are usually dry and because of
complaining of acute breathlessness for the last the high intrathoracic pressure they need a high
4 hours. She has had no response from her RV filling pressure to maintain cardiac output.
ordinary medication and is getting distressed.
Never use low flow oxygen for these patients -
The ambulance crew have given 100% oxygen
hypoxia kills, hypercarbia merely intoxicates. If
and a salbutamol nebuliser.
they are hypercarbic they need ventilation.
Which TWO of the following statements IV beta-2 agonists can be life saving in patients
concerning the management of acute severe who are too breathless to be helped by
asthma are correct? nebulisers.
A. Aminophylline improves the ventilation / The most important investigation is an arterial
perfusion mismatch. blood gas as it will give information on carbon
B. Hyperkalaemia commonly follows dioxide concentration and pH. A normal carbon
treatment with beta-2 agonists. dioxide level is a worrying sign - it suggests a
tiring asthmatic. Hyperventilation should lower
C. Patients require high right ventricular the carbon dioxide concentration.
filling pressures.
Low dose diazepam is likely to result in your
D. 28-35% oxygen should be used for appearance in a coroner's court - sedation is
patients with hypercapnia to avoid contraindicated, unless mechanical ventilation
progressive hypoventilation. is indicated.
E. Intravenous beta-2 agonists are The degree of pulsus paradoxus (fall in systolic
contraindicated as they exacerbate blood pressure during inspiration) does
tachycardia. correlate with the severity of attack.
F. The most important investigation is the
peak expiratory flow rate.

Dr. Khalid Yusuf El-Zohry – Sohag Teaching Hospital (01118391123) Page | 549
El-Zohry MRCP Questions Bank (Part 2) – Medical Masterclass 2010

[ Q: 55 ] MasterClass Part2 down to the segmental arteries. It is the


(2010) - Respiratory investigation of choice for patients with pre-
existing lung disease, which renders the
A 70-year-old man presents with shortness of interpretation of ventilation-perfusion scans
breath that has developed over a few days. He difficult or impossible.
has no other symptoms. One week ago he
returned on a transatlantic flight from a holiday
[ Q: 56 ] MasterClass Part2
in Florida.
(2010) - Respiratory
Which TWO investigations are most likely to
A breathless 23-year-old woman has the
reveal the diagnosis?
following lung function tests: FEV1 1.1L (60%)/
A. ECG FVC 1.3 L (55%)/ FEV1/FVC ratio = 84%/ TLC =
B. Ventilation-Perfusion isotope scanning 66% predicted/ RV = 57% predicted/ TLCO =
55% predicted/ KCO = 110% predicted
C. Echocardiogram
What is the most likely diagnosis?
D. Serum troponin I
A. Acute sickle crisis
E. Chest radiograph
B. systemic lupus erythematosus (SLE)
F. Spiral computed tomography scanning
pneumonitis
G. Chest radiograph taken in expiration
C. Scoliosis
H. Spirometry
D. Asthma
I. Arterial blood gases
E. Cystic fibrosis.
J. Ultrasound scanning of the leg veins.
Answer & Comments
Answer & Comments
Correct answer: C
Correct answer: BF
The lung function tests show a significant
The most likely diagnosis is pulmonary restrictive defect. Only kyphoscoliosis or a
embolism, the two most specific tests for this pneumonitis may fit this picture but given the
being ventilation-perfusion isotope scanning normal/high KCO (i.e. after correcting for
and spiral computed tomography scanning. alveolar volumes), the most likely answer is
Ventilation scans are obtained using krypon- kyphoscoliosis as the gas exchange after
81m, technegas or xenon-133 and perfusion correcting for the alveolar volume would in fact
scans with intravenous 99m-Tc-labelled be high.
macroaggregates of albumin. Scans are
reported as being normal or of low, medium or [ Q: 57 ] MasterClass Part2
high probability, but it is very important to (2010) - Respiratory
remember that reports must ALWAYS be
interpreted in the light of the clinical context, A 72-year-old is stung by a wasp at a garden
and that a low probability scan does NOT mean party and becomes breathless and wheezy. She
that ‘pulmonary embolism is excluded’. is brought in to A&E within 15 minutes by her
nephew and is distressed and hypotensive.
Spiral computed tomography can detect
intravascular clot from the pulmonary trunk Which TWO statements are INCORRECT?

Dr. Khalid Yusuf El-Zohry – Sohag Teaching Hospital (01118391123) Page | 550
El-Zohry MRCP Questions Bank (Part 2) – Medical Masterclass 2010

A. Adrenaline is the first line drug in [ Q: 58 ] MasterClass Part2


anaphylaxis. (2010) - Respiratory
B. As she has a history of ischaemic heart
A 43-year-old Portugese man presents with a 2
disease, adrenaline should be avoided.
week history of feeling short of breath, fever
C. Adrenaline should be not be given IV and a rash. He is found to be hypoxic with a PO2
initially. of 6 on air but with a few sparse crackles. He
D. Adrenaline stimulates alpha, beta 1 and has a rash over his back and an abnormal chest
beta 2 receptors. radiograph (see images).

E. If she is on beta blockers the dose of


adrenaline should be halved.
F. Adrenaline should not be given until her
blood gas analysis is available.
G. Use of an adrenaline auto injector should
be considered for the future.
H. Tricyclic antidepressants increase the risk
of cardiac arrythmias.
I. The severity of this reaction does not
determine the severity of future
reactions.
J. If there is no improvement, a repeat dose
of adrenaline should be given after 5
minutes.

Answer & Comments

Correct answer: BF

IM adrenaline is the first line drug for


anaphylaxis, even in patients with ischaemic
heart disease, but IV administration is more
dangerous and should only be given in those
unresponsive to a second dose of IM adrenaline What is the most likely pathogen?
or in extremis. Early use is associated with A. Mycobacterium tuberculosis
improved outcome. The dose is reduced in
B. Pneumocystis carinii
those on beta-blockers because of unopposed
alpha effects. Subsequent reactions can be C. cytomegalovirus
more or less severe. If an auto injector is D. Streptococcus
prescribed, the patient or family member must
be given proper instruction. E. Cryptococcus

Answer & Comments

Correct answer: B

Dr. Khalid Yusuf El-Zohry – Sohag Teaching Hospital (01118391123) Page | 551
El-Zohry MRCP Questions Bank (Part 2) – Medical Masterclass 2010

The skin lesions are characteristic of Kaposi’s A. Refer for an urgent respiratory outpatient
sarcoma and the chest radiograph (CXR) clinic appointment
suggestive of a perihilar infiltrate/alveolar
B. Admit and monitor with daily chest
shadowing.
radiographs
This patient is likely to have a human
C. Intercostal tube drainage
immunodeficiency virus related disease.
D. Simple aspiration
The likeliest cause for this patient's LRTI is
pneumocystis given the clinical history, E. Review with daily chest radiographs
examination and CXR.
Answer & Comments
[ Q: 59 ] MasterClass Part2 Correct answer: C
(2010) - Respiratory
A 35-year-old woman is seen in A&E The lung edge measurement suggests a greater
department giving 2-day history of left sided than 50% pneumothorax. In the context of
chest pain pleuritic in nature. Pulmonary chronic underlying respiratory disease such as
embolism (PE) is suspected. chronic obstructive pulmonary disorder, this is
best managed by admission and pleural
Which of the following statements is FALSE? drainage.

A. Absence of tachycardia excludes PE


[ Q: 61 ] MasterClass Part2
B. Presence of chest wall tenderness
excludes PE
(2010) - Respiratory

C. Normal arterial blood gases excludes PE (2) A 71-year-old man with idiopathic
pulmonary fibrosis presents to the Emergency
D. All of them
Department breathless with a first episode of a
E. None of them spontaneous left pneumothorax. His SaO2 is
97% on room air, with respiratory rate 16 / min.
Answer & Comments The chest radiograph shows a left-sided
pneumothorax with the lung edge 4 cm from
Correct answer: D the edge of the thoracic cavity.

Statements A, B and C are common What is the most appropriate management?


misconceptions. A. Observe and repeat the radiograph in 4-8
hours
[ Q: 60 ] MasterClass Part2 B. Aspirate the chest and observe
(2010) - Respiratory
C. Place an intercostal drain and observe
A 56-year-old man with emphysema presents D. Refer to thoracic surgeons for
to the Emergency Department with increasing consideration of surgical pleurectomy or
dyspnoea and left-sided pleuritic chest pain. A pleurodesis
chest radiograph reveals a left-sided
pneumothorax and that the lung edge E. Treat with maximal oxygen to increase
measured 4 cm away from the chest wall. resorption of the pneumothorax

The most appropriate management is:

Dr. Khalid Yusuf El-Zohry – Sohag Teaching Hospital (01118391123) Page | 552
El-Zohry MRCP Questions Bank (Part 2) – Medical Masterclass 2010

Answer & Comments common in males and is characterised by


snoring, apnoea spells, choking and gurgling
Correct answer: C
sounds during sleep and excessive daytime
somnolence. The Epworth Sleepiness Score
This is a secondary spontaneous evaluates general levels of sleepiness and is not
pneumothorax, in which case drainage is the diagnostic. A normal overnight pulse oximetry
most appropriate form of management. The does not exclude OSA. The gold standard for
only indication to aspirate would be a patient diagnosis is polysomnography. Multiple sleep
who was not breathless, aged less than 50 and latency test measures sleepiness objectively.
a smaller pneumothorax (<2cm) (BTS guidelines Fibreoptic nasoendoscopy is performed in
2003). Therefore, as a general rule, these types selected patients to locate the site of
of pneumothorax require drainage. There is no obstruction.
indication to refer to the surgeons unless the
pneumothorax does not resolve with tube
drainage or if this is a repeat episode.
[ Q: 63 ] MasterClass Part2
(2010) - Respiratory
[ Q: 62 ] MasterClass Part2 A 24-year-old homosexual man presents with a
(2010) - Respiratory 2-day history of progressive breathlessness, dry
cough and slight fever. On examination he was
A 55-year-old lorry driver has been referred tachypnoeic, but there were no abnormalities
because of daytime somnolence. His wife on auscultation. See image for chest
reported that he snores and is very concerned radiograph.
that he stops breathing for a few seconds while
he sleeps. He has no significant past medical
history. Apart from being obese (BMI 31) his
examination is normal. Obstructive sleep
apnoea is suspected.

Which of the following is most likely to be true?


A. An Epworth Sleepiness Score will be
diagnostic.
B. A normal overnight pulse oximetry
excludes the diagnosis.
C. Polysomnography will be diagnostic.
D. Multiple sleep latency test is indicated.
E. Fibreoptic nasoendoscopy will be
diagnostic.
The most likely diagnosis is:
Answer & Comments A. Miliary tuberculosis
Correct answer: C B. Pneumocystis carnii pneumonia
C. Congestive cardiac failure
Obstructive sleep apnoea (OSA) is a common
sleep disorder. The risk of developing OSA D. Atypical pneumonia
increases with age and obesity. It is more E. Intersitial pneumonitis.

Dr. Khalid Yusuf El-Zohry – Sohag Teaching Hospital (01118391123) Page | 553
El-Zohry MRCP Questions Bank (Part 2) – Medical Masterclass 2010

Answer & Comments C. Bilateral pulmonary emboli

Correct answer: B D. Mediastinal emphysema


E. Megacolon
There is bilateral perihilar interstitial shadowing
and paucity of clinical signs which is classical for Answer & Comments
pneumocystis carnii pneumonia. Atypical
pneumonia does not cause this degree of Correct answer: C
breathlessness without clinical signs.
Severe hypoxia accompanied by tachycardia,
[ Q: 64 ] MasterClass Part2 RBBB and bilateral atelectasis strongly suggest
pulmonary embolism. None of the other
(2010) - Respiratory
diagnoses could explain all of these findings.
A 72-year-old man presents with sudden onset
epigastric pain (10/10), not relieved by [ Q: 65 ] MasterClass Part2
vomiting, or by a GTN spray. His past medical
(2010) - Respiratory
history includes ischaemic heart disease and
ulcerative colitis. On examination his abdomen A 37-year-old man with asthma comes to your
is soft with slight epigastric tenderness, but clinic. His current medication consists of a low
there is no rebound and bowel sounds are dose of inhaled corticosteroids and inhaled
present. The ECG shows sinus tachycardia and short-acting beta 2 agonist which he takes, on
right bundle branch block (RBBB). A chest average, three to four times a day.
radiograph shows widening of the mediastinum
and bilateral atelectasis. Breathing air, his What would you do?
oxygen saturation is 86%, and arterial blood gas A. Advise him to continue with his current
analysis reveals pH 7.42, pCO2 6.5 kPa, p02 medication.
6.8kPa.
B. Commence a high dose of
inhaled corticosteroids.
C. Add an inhaled long-acting beta
2 agonist.
D. Add a leucotriene receptor
antagonist.
E. Add a long-acting
anticholinergic.

Answer & Comments

Correct answer: C

This patient is in Step Two of asthma


treatment. His asthma is not well-controlled, as
What is the most likely diagnosis diagnosis? he needs to take his rescue medication more
A. Dissection of the aorta than twice a day. Therefore, his treatment
should be stepped up. In Step Three of asthma
B. Ruptured oesophagus
treatment, the current British Thoracic Society

Dr. Khalid Yusuf El-Zohry – Sohag Teaching Hospital (01118391123) Page | 554
El-Zohry MRCP Questions Bank (Part 2) – Medical Masterclass 2010

guideline advises first adding inhaled long- [ Q: 67 ] MasterClass Part2


acting beta 2 agonist (LABA) and then assessing (2010) - Respiratory
the situation.
If there is a good response to LABA, this Which of the following will not result in an
medication should be continued. If there is abnormally raised central venous pressure
benefit from LABA but control is still (CVP)?
inadequate, LABA should be continued, and the A. Tension pneumothorax
inhaled corticosteroids should be increased to a
high dose. If there is no response to LABA, then B. Right ventricular infarction
that treatment should be stopped and inhaled C. Cardiac tamponade
corticosteroids should be increased to a high
D. Occlusion of the central line
dose. Anticholinergics have very little role in the
management of stable asthma. E. Anaphylaxis.

[ Q: 66 ] MasterClass Part2 Answer & Comments


(2010) - Respiratory Correct answer: E
A 72-year-old woman with known COPD on
maximal medication is referred with a 4-month Tension pneumothorax raise intrathoracic
history of morning headaches. ABG showed pH pressure resulting in an artificially elevated CVP.
of 7.34 pCO2 of 5.8 kPa, pO2 of 7.4kPa and Right ventricular infarction will also cause
HCO3 27 mmHg. elevation of the CVP - importantly, part of the
treatment for RV infarction is to keep the filling
What would you recommend? pressure very high to ensure adequate cardiac
output. In cardiac tamponade, RA and RV
A. Long-term oxygen therapy.
pressures are elevated significantly. A blocked
B. Overnight SaO2 and CO2 monitoring. catheter results in a high CVP with a damped
C. Repeated ABG in 3-weeks time. waveform. Anaphylaxis is associated with
cardiovascular collapse.
D. CT brain scan.
E. Ambulatory oxygen therapy. [ Q: 68 ] MasterClass Part2
(2010) - Respiratory
Answer & Comments
A 38-year-old woman who was previously fit
Correct answer: B and well presents with breathlessness that has
been getting worse over 3 or 4 months. You
This patient is likely to have nocturnal hypoxia suspect that she might have primary pulmonary
with overnight or early morning hypercapnia hypertension.
and may require nocturnal oxygen
supplementation. Overnight SaO2 with a pulse Which of the following physical signs would
oximeter and transcutaneous CO2 NOT be consistent with this diagnosis?
measurement should be carried out. A. Elevated jugular venous pressure
B. Clubbing
C. Palpable heave at left sternal edge
D. Added heart sound over right ventricle

Dr. Khalid Yusuf El-Zohry – Sohag Teaching Hospital (01118391123) Page | 555
El-Zohry MRCP Questions Bank (Part 2) – Medical Masterclass 2010

E. Loud P2. E. Inhaled corticosteroid treatment

Answer & Comments Answer & Comments

Correct answer: B Correct answer: E

Primary pulmonary hypertension can present Inhaled corticosteroids should be prescribed for
with breathlessness, fatigue, angina (due to patients with a forced expiratory volume in 1
right ventricular ischaemia) or presyncope / second less than or equal to 50% of predicted,
syncope. and who have had two or more exacerbations
of chronic obstructive pulmonary disease
Physical signs include elevated JVP (sometimes
(COPD) requiring treatment with antibiotics or
with a giant v wave of tricuspid regurgitation
oral corticosteroids in a 12-month period. The
and pulsatile hepatomegaly), left parasternal
aim of treatment is to reduce exacerbation
heave, pansystolic murmur (tricuspid
rates and slow the decline in health status, not
regurgitation) and right ventricular S4,
to improve lung function per se. Maintenance
peripheral oedema. Clubbing is not a feature
use of oral corticosteroid therapy in COPD is not
and suggests chronic lung disease or cyanotic
normally recommended.
congenital heart disease in this context.
An important differential diagnosis of primary
[ Q: 70 ] MasterClass Part2
pulmonary hypertension is chronic pulmonary
thromboembolism. (2010) - Respiratory
A 27-year-old previously fit woman has been
[ Q: 69 ] MasterClass Part2 referred to the sleep clinic with excessive
(2010) - Respiratory somnolence. There is no history of snoring. Her
partner has noticed that on occasions when she
A 62-year-old woman with chronic obstructive is watching a comedy show on the television,
pulmonary disease continues to complain of she tends to drop whatever she is holding. A 2-
shortness of breath on exertion and presents week sleep diary shows that she sleeps for 8
with frequent lower respiratory tract infections hours every night and has numerous daytime
requiring antibiotics and oral steroid treatment. naps.
Her medication consists of an inhaled short-
acting anticholinergic and aminophylline What is the most likely diagnosis?
tablets. She has been unable to tolerate inhaled
A. Sleep-disordered breathing
long-acting β2 agonists because of tremor. A
trial of long-acting anticholinergic has proved B. Narcolepsy
unhelpful. Her forced expiratory volume in 1 C. Insufficient sleep syndrome
second is 0.8 L (38% of predicted) and her Sao2
D. Restless leg syndrome
is 95%.
E. Kleine-Levin syndrome
What treatment would you recommend?
A. Long-term oral corticosteroid treatment Answer & Comments
B. Nebulised bronchodilators Correct answer: B
C. Leukotriene receptor antagonist
Narcolepsy is characterised by excessive
D. Prophylactic antibiotic treatment
daytime somnolence (can be measured by the

Dr. Khalid Yusuf El-Zohry – Sohag Teaching Hospital (01118391123) Page | 556
El-Zohry MRCP Questions Bank (Part 2) – Medical Masterclass 2010

Epworth Sleepiness Scale), cataplexy (sudden 120mg/kg for at least 3 weeks. Early
onset of muscle weakness, which may be focal administration of glucocorticoids to patients
or generalised), hypnagogic hallucinations (vivid with moderate to severe hypoxia, decreases the
hallucinations occurring at the onset of sleep) risk of respiratory failure and death by over
and sleep paralysis (inability to move on falling 50%. Pulmonary function tests demonstrate a
asleep or wakening, often accompanied by decreased vital capacity, increased residual
hallucinations). capacity, normal flow rates and a reduced
DLCO. SIADH is known to occur with PCP.
[ Q: 71 ] MasterClass Part2
(2010) - Respiratory [ Q: 72 ] MasterClass Part2
(2010) - Respiratory
A 34-year-old previously fit homosexual man is
admitted to the hospital with a one-week A 60-year-old woman has been diagnosed as
history of dyspnoea. His chest radiograph suffering from asthma by her doctor for over 12
shows bilateral alveolar infiltrates and apart months. Her initial symptoms were of non-
from a PaO2 of 6Kpa on breathing room air, his productive cough, worse at night, intermittent
routine blood tests are normal. The admitting wheeze and dyspnoea on exertion. She is on
doctor makes a diagnosis of Pneumocystis high dose inhaled steroids, a long acting beta
carinii pneumonia (PCP). agonist and a short acting beta agonist. She still
complains of nocturnal symptoms and has been
Which of the following is NOT true? referred to the chest clinic. Her chest
A. Unilateral pleural effusions are commonly radiograph is shown (see image).
seen in this condition.
B. The patient should be treated with
intravenous co-trimoxazole.
C. The patient should be treated with
prednisolone.
D. The patients lung function tests will show
a reduced diffusing capacity for carbon
monoxide (DLCO).
E. PCP is associated with the syndrome of
inappropriate ADH (SIADH) secretion.

Answer & Comments

Correct answer: A

PCP is commonly seen in the


immunosupressed. Patients usually complain of She has never smoked and has never kept any
cough and dyspnoea. The cough is usually non- pets.
productive. Patients are usually hypoxic and
What is the next appropriate step?
chest radiograph characteristically shows
bilateral alveolar infiltrates. Pleural effusions A. Add an anticholinergic
are very rare. The patient should receive B. Add oral steroids with a biphosphonate
intravenous co-trimoxazole in a dose of

Dr. Khalid Yusuf El-Zohry – Sohag Teaching Hospital (01118391123) Page | 557
El-Zohry MRCP Questions Bank (Part 2) – Medical Masterclass 2010

C. Add a proton pump inhibitor I. Traumatic chylothorax usually heal with


conservative management.
D. Add a leukotriene antagonist
J. Chylothorax can occur following
E. Add theophylline.
cardiovascular surgery.

Answer & Comments


Answer & Comments
Correct answer: C
Correct answer: AG

The chest radiograph shows a hiatus hernia.


Milky pleural fluid with a high lipid content is
The underlying cause of her nocturnal
seen in two different conditions.
symptoms may be gastrooesophageal reflux
which can cause cough as well as intermittent 1. Chylothorax: in this condition, chyle
wheeze. She should be started on a proton enters the pleural cavity as a result of
pump inhibitor and at the same time she should disruption of the thoracic duct. This is
maintain a peak expiratory flow rate (PEFR) usually an acute situation and may be
record. If her PEFR are acceptable, she should due to lymphoma, trauma,
be gradually weaned off some of her extensive cardiovascular surgery, filariasis,
asthma treatment. lymphangitis of the thoracic duct,
lymphangiolyomyomatosis. It is
[ Q: 73 ] MasterClass Part2 characterized by a raised triglyceride
level (above 110mg/dl). Chyle is
(2010) - Respiratory
bacteriostatic. The thoracic duct carries
A 36-year-old woman with a left pleural around 1,500 to 2,500 mls of chyle daily
effusion undergoes a diagnostic pleural tap. The from the intestine to the left subclavian
pleural fluid is milky in consistency and vein. Traumatic chylothorax is initially
laboratory tests confirm it to be a chylothorax. managed by reducing chyle formation,
relieving dyspnoea by tube drainage
Which TWO of the following are not correct? and preventing malnutrition, hoping for
A. The most likely diagnosis is secondaries the site of the leak to heal.
from the breast. 2. Pseudochylothorax or chyliform
B. The most likely cause is lymphoma. effusion: in this condition large
amounts of cholesterol accumulate in a
C. The fluid has a raised triglyceride level. long-standing pleural effusion.
D. Lymphangiomyomatosis is a possible
diagnosis if the chest radiograph shows [ Q: 74 ] MasterClass Part2
parenchymal infiltrates.
(2010) - Respiratory
E. Patient is at risk of becoming cachexic.
A 44-year-old woman with a history of nasal
F. Trauma is a common cause of polyps, sinusitis and steroid-dependent asthma
chylothorax. attends the chest clinic. A leukotriene receptor
G. The fluid contains large amounts of antagonist is added to her medications and her
cholesterol. oral steroid dose is reduced.

H. Chyle is bacteriostatic. Four weeks later she is sent back to the clinic
for urgent review because she has developed

Dr. Khalid Yusuf El-Zohry – Sohag Teaching Hospital (01118391123) Page | 558
El-Zohry MRCP Questions Bank (Part 2) – Medical Masterclass 2010

subcutaneous nodules on the extensor surfaces A. Prednisolone 0.5 mg/kg/day plus


of the arms azathioprine 1.5 mg/kg/day for 4 weeks
She has a mild normochromic normocytic B. Prednisolone 0.5 mg/kg/day for 4 weeks
anaemia, raised erythrocyte sedimentation
C. Methylprednisolone 0.5-1 g IV/day for 3
rate, peripheral eosinophilia and a positive
days, then prednisolone 1 mg/kg/day for
rheumatoid factor. Review of her chest
4 weeks
radiographs performed over the last 8 years
shows fleeting interstitial patchy shadowing. D. Methylprednisolone 0.5-1 g IV/day for 3
days, then prednisolone 1 mg/kg/day,
What is the most likely diagnosis? plus cyclophosphamide 1.5-2 mg/kg/day
for 4 weeks
A. Churg-Strauss syndrome
E. Observe
B. Allergic bronchopulmonary aspergillosis
C. Extrinsic allergic alveolitis
Answer & Comments
D. Wegener's granulomatosis
Correct answer: E
E. Rheumatoid lung disease
The International Congress on Sarcoidosis
Answer & Comments defines stages based on chest radiography as
follows:
Correct answer: A
Stage 0, normal chest radiograph
The use of leukotriene receptor antagonists has Stage I, hilar lymph node enlargement only
been associated with the development of
Stage IIA, lymph node enlargement and diffuse
Churg-Strauss syndrome in asthma, although
pulmonary disease
almost always in the context of a reduction in
oral steroid use, which is the more likely Stage IIB, diffuse pulmonary disease without
precipitant. It should be considered in all lymph node enlargement
patients with deteriorating asthma, especially
Stage III, pulmonary fibrosis
where there is blood eosinophilia and
interstitial shadows on the CXR. Systemic Stages 0 and I commonly resolve
vasculitis with skin, cardiac and renal spontaneously. Stage II disease without any
involvement may follow. symptoms should be monitored for at least six
months, with treatment offered if symptoms -
eg. breathlessness, cough – develop.
[ Q: 75 ] MasterClass Part2
(2010) - Respiratory If treatment is indicated (not in this case), then
oral prednisolone (0.5 mg/kg/d) is the
A 28 yr old man is found to have hilar treatment of choice, given for four weeks and
lymphadenopathy on a chest radiograph, which then reduced in a stepwise fashion to a
is otherwise normal. He is well, without maintenance dose of 5-15 mg/d as dictated by
respiratory symptoms or any abnormalities on clinical and radiological response. Other
physical examination. His full blood count and immunosuppressants are used in refractory
biochemical screen (including serum calcium) cases.
are all normal. A diagnosis of sarcoidosis is
made.

What is the correct initial treatment?

Dr. Khalid Yusuf El-Zohry – Sohag Teaching Hospital (01118391123) Page | 559
El-Zohry MRCP Questions Bank (Part 2) – Medical Masterclass 2010

[ Q: 76 ] MasterClass Part2 Which TWO of the following features would


(2010) - Respiratory NOT be expected in a case of sarcoidosis?
A. Facial skin rash
A 48-year-old lorry driver has been referred to
the sleep clinic with a history of snoring, B. Arthritis in the small feet joint
daytime somnolence and nocturnal apnoeic C. Anterior uveitis
spells. His score on the Epworth Sleepiness
Scale is 14/24. You suspect obstructive sleep D. Superior vena cava obstruction
apnoea (OSA) and arrange for him to have a E. Lung nodules
polysomnogram.
F. Splenomegaly
Which of the following is true? G. Renal stones
A. OSA is usually more severe during rapid H. Ulcerated cervical lymph nodes
eye movement (REM) sleep
I. Complete heart block
B. OSA is worse in the prone sleeping
J. Cardiomyopathy.
position
C. Alcohol increases REM sleep Answer & Comments
D. Normally REM sleep occurs during the
Correct answer: DH
first half of the night
E. Benzodiazepines result in an increase in Sarcoidosis is characterised by formation of
stages 3 and 4 (slow-wave sleep) of non- granulomas, often in many organs.
REM sleep
Lymphadenopathy is the most common feature
of sarcoidosis, in which the glands are rubbery,
Answer & Comments
soft and tend not to compress adjacent organs.
Correct answer: A Features such as superior vena cava
obstruction, laryngeal nerve palsy and phrenic
Obstructive sleep apnoea (OSA) is usually worse nerve palsy due to mediastinal lymph gland
in the supine position and during rapid eye enlargement are very rare in sarcoidosis, and if
movement (REM) sleep. Alcohol worsens OSA present they suggest another diagnosis such as
as a result of its sedative effect (although it lymphoma or lung cancer.
does not increase REM sleep) and by relaxing Biochemical features of sarcoidosis include
the pharyngeal muscles hypercalcaemia and hypercalciuria which are
due to secretion of vitamin D-like material from
[ Q: 77 ] MasterClass Part2 the epithelioid cells in granulomata.
Hypercalciurea may lead to renal stone
(2010) - Respiratory
formation.
A 29-year-old woman presents with a 3-month
The heart can be involved in sarcoidosis. The
history of malaise, joint pain and arthralgia.
conduction system may be infiltrated with
Three weeks ago, she noticed a painful rash on
granulomas leading to branch blocks or to
her shins. Her chest radiograph reveals large
complete heart block. Cardiomayopathy is also
lymph nodes in the hilar and paratracheal
seen.
areas.
Hepatomagaly or splenomagaly are reported in
a quarter of patients with sarcoidosis. The

Dr. Khalid Yusuf El-Zohry – Sohag Teaching Hospital (01118391123) Page | 560
El-Zohry MRCP Questions Bank (Part 2) – Medical Masterclass 2010

spleen is directly involved with the disease in Answer & Comments


most cases, with enlargement rarely due to
Correct answer: B
portal hypertension.

This man with severe kyphoscoliosis has


[ Q: 78 ] MasterClass Part2 probably developed chronic respiratory failure.
(2010) - Respiratory All patients with significant respiratory disease
The wife of this 63-year-old man complained should be offered annual 'flu vaccination. He
that he was sleepy and increasingly aggressive. should be assessed for non-invasive ventilation
and long-term oxygen therapy and is likely to

have cor pulmonale with pitting oedema, which


Which of the following is LEAST likely to be will be improved symptomatically with careful
helpful in his management? use of diuretics. He might have reversible
A. Non-invasive ventilation airways obstruction (as this is a common
condition,) but this would be an additional
B. Salbutamol nebules problem.
C. Loop diuretics
D. Long-term oxygen therapy (LTOT) [ Q: 79 ] MasterClass Part2
(2010) - Respiratory
E. 'Flu vaccination.
A 38-year-old woman is admitted with a 2-day
history of fever, rigors and breathlessness. She
looks extremely unwell and is confused,

Dr. Khalid Yusuf El-Zohry – Sohag Teaching Hospital (01118391123) Page | 561
El-Zohry MRCP Questions Bank (Part 2) – Medical Masterclass 2010

cyanosed, has a respiratory rate of 24/min and not improved despite this emergency
a systolic blood pressure of 80 mmHg. There is treatment. You consider starting him on bi-level
bronchial breathing at her right base, where a positive airways pressure (BiPAP).
chest radiograph reveals consolidation.
Which of the following arterial blood gas
Which would be the most appropriate antibiotic readings is the clearest indication for BiPAP?
regimen to use? (Normal values: pH 7.35-7.45, Paco2 4.7-6.0
kPa, Pao2 >10.6 kPa, bicarbonate 22-28
A. Oral amoxicillin
mmol/L)
B. Oral amoxicillin and oral clarithromycin
A. pH 7.36, Paco2 7.3 kPa, Pao2 6.6 kPa,
C. Intravenous amoxicillin and intravenous bicarbonate 30 mmol/L
clarithromycin
B. pH 7.39, Paco2 5.0 kPa, Pao2 7.1 kPa,
D. Intravenous cefotaxime and intravenous bicarbonate 32 mmol/L
erythromycin
C. pH 7.56, Paco2 3.7 kPa, Pao2 8.9 kPa,
E. Intravenous ceftazidime and intravenous bicarbonate 38 mmol/L
vancomycin.
D. pH 7.30, Paco2 4.0 kPa, Pao2 6.9 kPa,
bicarbonate 19 mmol/L
Answer & Comments
E. pH 7.29, Paco2 8.9 kPa, Pao2 6.1 kPa,
Correct answer: D bicarbonate 32 mmol/L

This woman has severe pneumonia as defined Answer & Comments


by the British Thoracic Society guidelines, which
require any two of the following features: Correct answer: E
confusion, urea >7mmol/l, respiratory rate
>30/min, and hypotension (SBP <90mmHg, DBP Bi-level positive airways pressure is indicated
<60mmHg). for acute type II respiratory failure with a pH
<7.35, although there is less evidence for
Appropriate treatment (as recommended by
benefit if pH <7.25.
the British Thoracic Society) is with intravenous
antimicrobials: co-amoxiclav 1.2g three times
daily or cefuroxime 1.5g three times daily or [ Q: 81 ] MasterClass Part2
cefotaxime 1g three times daily or ceftriaxone (2010) - Respiratory
2g once daily plus erythromycin 500 mg four
times daily or clarithromycin 500mg twice daily. A 73-year-old man has been referred to the
Acute Medical Team by his GP. He has become
increasingly tired and drowsy over the last 2
[ Q: 80 ] MasterClass Part2 weeks. In the past 2 months he has lost 4kg in
(2010) - Respiratory weight. Investigations reveal a sodium reading
of 118mmol/l, potassium of 3.6mmol/l, urea of
An 83-year-old man is admitted with a history
6.2mmol/l and creatinine of 110micromol/l.
of chronic obstructive pulmonary disease
exacerbation. His forced expiratory volume in 1 Which of the following statements regarding
second done 2 months ago, when he was well, hyponatraemia are correct?
was 1.1 L (36% of predicted). He has been
treated with nebulisers, intravenous A. Water intake should be restricted to 1
hydrocortisone, intravenous infusion of litre/day in all patients with
aminophylline and controlled oxygen. He has hyponatraemia.

Dr. Khalid Yusuf El-Zohry – Sohag Teaching Hospital (01118391123) Page | 562
El-Zohry MRCP Questions Bank (Part 2) – Medical Masterclass 2010

B. Hypertonic saline (1.8%) should be used water clearance producing profound


to bring the serum sodium into the hyponatraemia. Diuretics are a common cause
normal range. of important hyponatraemia.
C. The sodium concentration should be Small cell (oat cell) carcinoma of the lung is
raised by 1-2mmol/hour. associated with SIADH. Hyponatraemia is
almost always a result of too much water rather
D. Assessment of the patient's volume
than lack of sodium. In hospital, the commonest
status may aid diagnosis.
cause of hyponatraemia is inappropriate fluid
E. Desmopressin is useful in the treatment administration - especially 5% dextrose.
of SIADH (syndrome of inappropriate ADH
production).
[ Q: 82 ] MasterClass Part2
F. The patient's metabolic picture may be (2010) - Respiratory
caused by severe hyperthyroidism.
A 37-year old white south African man presents
G. Clinically important hyponatraemia rarely
with a 3-week history of progressive shortness
results from diuretic therapy.
of breath that has not improved following two
H. A history of heavy smoking suggests a courses of antibiotics (amoxicillin and
diagnosis of adenocarcinoma of the lung. erythromycin). He does not keep any pets and
I. Lack of total body sodium is common in is on no other medication. On auscultation his
hyponatraemia. chest is clear, but he is profoundly hypoxic.

J. The commonest cause of hyponatraemia


in hospital is diuretic therapy.

Answer & Comments

Correct answer: CD

Some causes of hyponatraemia (such as GI fluid


losses, Addison's disease, diuretics) result in
decreased extracellular volume, whereas others
(such as cardiac failure, cirrhosis, nephrotic
syndrome, chronic renal failure) may be
associated with increased extracellular volume.
Assessment of volume status is helpful - those
What is the most likely diagnosis?
with normal or increased extracellular volume
should have their water intake restricted. A. Extrinsic allergic alveolitis
Hypertonic saline solution is used if the sodium B. Sarcoidosis
is less than 125mmol/L with the aim of raising
the sodium by 1-2mmol/hr. Once the sodium is C. Pneumocystis carini pneumonia
above 125mmol/L, normal saline solution D. Pulmonary tuberculosis
should be used. Too rapid an increase in sodium
E. Wegener's granulomatosis.
can cause central pontine myelinolysis.
Desmopressin is used to treat cranial diabetes
insipidus, demeclocycline may be useful in Answer & Comments
SIADH. Severe hypothyroidism can impair free Correct answer: C

Dr. Khalid Yusuf El-Zohry – Sohag Teaching Hospital (01118391123) Page | 563
El-Zohry MRCP Questions Bank (Part 2) – Medical Masterclass 2010

Severe hypoxia with relatively minimal Answer & Comments


radiological changes makes Pneumocystis carinii
Correct answer: B
the most likely diagnosis

The chest radiograph shows signs of early


[ Q: 83 ] MasterClass Part2 collapse and consolidation in the right apex. An
(2010) - Respiratory endobronchial lesion must be excluded.
A 62-year-old man is referred to the chest clinic
because of a productive cough, which has [ Q: 84 ] MasterClass Part2
persisted for 6 weeks since his return from a (2010) - Respiratory
holiday in India. He is an ex-smoker, having
stopped 4 years ago. Physical examination is A 38-year-old woman presents with a 2-year
unremarkable, and his chest radiograph is as history of worsening breathlessness. Results of
shown (see image). preliminary investigation are as follows:
spirometry – FEV1 83% predicted, Vital Capacity
(VC) 79% predicted, FEV1/VC ratio 78%; chest
radiograph reported as being within normal
limits; blood gases (breathing air) - PaO2 9.6
kPa, PaCO2 4.1 kPa.

What are the TWO most likely diagnoses?


A. Chronic obstructive pulmonary disease
B. Cardiac failure
C. Diffuse parenchymal lung disease
D. Pneumocystis carinii pneumonia
E. Anaemia
F. Bronchiectasis
G. Asthma
H. Pulmonary vascular disease
I. Diabetes mellitus
J. Primary hyperventilation.
What is the most appropriate investigation
to arrange next? Answer & Comments

A. Full lung function tests Correct answer: CH


B. Fibreoptic bronchoscopy
The spirometry effectively rules out obstructive
C. Mantoux test
lung disease / asthma. Hypoxia excludes
D. Sputum direct staining and culture for hyperventilation and anaemia as primary
acid-fast bacilli causes. The differential diagnosis lies between
E. Angiotensin-converting enzyme level diffuse parenchymal lung disease and a
problem with the pulmonary vasculature.

Dr. Khalid Yusuf El-Zohry – Sohag Teaching Hospital (01118391123) Page | 564
El-Zohry MRCP Questions Bank (Part 2) – Medical Masterclass 2010

In diffuse parenchymal lung disease the patient Thrombolysis removes the clot obstructing the
may have a dry cough, but there may be no large pulmonary arteries as well as any clot in
specific features. It will be important to ask the pelvic or deep leg veins. It reduces the
about systemic / iatrogenic disorders associated release of serotonin and other neurohumoral
with lung disease, and specific employment factors, which would tend to exacerbate
history / recreational interests may also be pulmonary hypertension.
relevant. Note that the chest radiograph can
appear entirely normal in patients with diffuse [ Q: 86 ] MasterClass Part2
parenchymal lung disease.
(2010) - Respiratory
Regarding diseases of the pulmonary
vasculature: a small number of patients with A 43-year-old woman presents with
pulmonary embolism present with breathlessness that has been getting gradually
breathlessness alone, but primary pulmonary worse over a few weeks and now makes it
hypertension needs careful consideration in this difficult for her to walk upstairs. On physical
case. examination she is found to have a large left
sided pleural effusion but no other
abnormalities. The presence of the effusion is
[ Q: 85 ] MasterClass Part2 confirmed by chest radiography.
(2010) - Respiratory
The most appropriate initial investigation would
A 50-year-old man presents with sudden onset
be:
shortness of breath and pleuritic chest pain. He
has a CT pulmonary angiogram, which shows a A. CT chest
large pulmonary embolus. B. Diagnostic aspiration of pleural fluid
followed by drainage of effusion to
Which of the following is NOT an indication for dryness
thrombolysis in this patient?
C. Diagnostic aspiration of pleural fluid with
A. Cardiac arrest
pleural biopsy
B. Falling blood pressure
D. Sputum cytology
C. D dimer greater than 4000
E. Diagnostic aspiration of pleural fluid.
D. Engorgement of neck veins
E. Right ventricular gallop. Answer & Comments

Correct answer: E
Answer & Comments

Correct answer: C The first investigation should be diagnostic


aspiration of pleural fluid for biochemical,
microbiological and cytological analysis.
Thrombolysis improves the outcome of patients
with large pulmonary embolus with signs of Light’s criteria can be used to distinguish
right heart failure. Any signs of incipient right transudates from exudates: in exudates at least
heart failure are indications for thrombolysis. D one of the following three criteria are met –
dimer is really more useful as a negative pleural fluid protein concentration greater than
predictor and is not helpful as a measure of the 50% of that in plasma; pleural fluid LDH greater
size of an embolus. Contraindications to than 60% of that in plasma; pleural fluid LDH
thrombolysis include recent surgery, trauma more that two thirds the upper limit of normal
and intracranial pathology. in plasma.
Dr. Khalid Yusuf El-Zohry – Sohag Teaching Hospital (01118391123) Page | 565
El-Zohry MRCP Questions Bank (Part 2) – Medical Masterclass 2010

Transudative pleural effusions are most J. Eosinophilic pleural effusions are always
commonly due to congestive cardiac failure but benign.
are sometimes associated with
hypoproteinaemic states such as cirrhosis or Answer & Comments
nephrotic syndrome. Most other causes of
pleural effusion are exudative. Correct answer: EI

[ Q: 87 ] MasterClass Part2 The ANA level in pleural fluid mirrors that in the
serum and is therefore unhelpful in diagnosis.
(2010) - Respiratory
Benign asbestos-related pleural effusions are
A 38-year-old post-doctoral scientist working commonly small, asymptomatic and often
for a biotechnology company presents with haemorrhagic. They occur most commonly in
breathlessness and is found to have a pleural the first two decades after asbestos exposure
effusion. He reads several review articles on the and resolve within 6 months.
subject before coming to see you in clinic.
Malignant pleural effusions can be diagnosed
Which TWO of the following statements that he by pleural fluid cytology alone in 60% of cases,
makes regarding pleural effusions are correct? so pleural aspiration for cytology is
recommended as the initial investigative step.
A. The pleural fluid ANA level should be
measured if systemic lupus Pleural fluid for pH should be collected
erythematosus is suspected. anaerobically with heparin and then measured
in a blood gas analyser.
B. Benign asbestos pleural effusion is never
haemorrhagic. Aminoglycosides should be avoided as they
have poor penetration into the pleural space
C. Routine bronchoscopy should be carried
and may be inactive in the presence of pleural
out in any case of pleural effusion to
fluid acidosis.
exclude any possibility of an
endobronchial lesion. The presence of pleural fluid eosinophilia is of
little use in the differential diagnosis of pleural
D. Malignant pleural effusions can be
effusion, as malignancy is also a common cause.
diagnosed by pleural fluid cytology alone
in only 25% of cases so pleural biopsy
should be always carried out. [ Q: 88 ] MasterClass Part2
E. In an infected pleural effusion a pH of
(2010) - Respiratory
<7.2 indicates the need for tube drainage. A 55-year-old man presents with a history of
F. Pleural fluid pH should be analysed by pH cough and haemoptysis of 2 weeks' duration.
litmus paper or a pH meter. Six months ago he was diagnosed with sinusitis
and started on some nasal drops. His CXR
G. Pleural fluid pH should be analysed by a
shows bilateral infiltrates and nodules with
pH meter.
cavitations. He has never smoked and works in
H. Aminoglycosides should be used if a zoo.
empyema is suspected.
What is the most likely diagnosis?
I. Rheumatoid arthritis is unlikely to be a
cause of pleural effusion if the glucose A. Bronchogenic carcinoma
level in the pleural fluid is above 1.6 B. Pulmonary tuberculosis
mmol/l.
C. Cryptogenic fibrosing alveolitis

Dr. Khalid Yusuf El-Zohry – Sohag Teaching Hospital (01118391123) Page | 566
El-Zohry MRCP Questions Bank (Part 2) – Medical Masterclass 2010

D. Sarcoidosis should be therefore followed and regular


inhaled corticosteroids prescribed.
E. Wegener's granulomatosis

Answer & Comments [ Q: 90 ] MasterClass Part2


(2010) - Respiratory
Correct answer: E
A 34-year-old lady is referred to the chest clinic
Wegener's granulomatosis affects the upper with history of frequent chest infections,
and lower respiratory tract and kidneys. Further wheeze and production of phlegm daily. She
investigation includes urgent measurement of has never smoked and has no significant past
serum antineutrophil cytoplasmic antibodies medical history. A clinical diagnosis of
and mucosal biopsy (nasal is the easiest), bronchiectasis with air flow obstruction is
preferably prior to commencement of made.
immunosuppression.
Which of the following tests will confirm the
diagnosis?
[ Q: 89 ] MasterClass Part2
A. Serial peak expiratory flow recordings
(2010) - Respiratory (PEFR)
(2) A 54-year-old smoker with chronic B. Bronchoscopy
obstructive pulmonary disease is referred to
C. CT pulmonary angiogram
you by his GP for advice regarding further
management. His spirometry is as follows: FEV1 D. Bronchogram
1.7 litres -64% predicted, FVC 2.40 litres - 75%
E. High Resolution CT chest scan
predicted, FEV1/VC 56%. Post bronchodilator
FEV1 is 2.5 litres - 92% predicted. His only F. Pulmonary function test
medication is a Salbutamol Evohaler that he G. Ventilation/Perfusion scan
uses on average 4 times a day.
H. Serum immunoglobulin level
What would you prescribe? I. Alpha 1 antitrypsin level
A. Inaled corticosteroid J. PET Scan
B. Long acting beta2 agonist
C. Long acting anti-cholinergic Answer & Comments

D. Regular short acting beta2 agonist and Correct answer: AE


short acting anti-cholinergic
The clinical history is suggestive of
E. Oral methyloxanthine
bronchiectasis and airflow obstruction. High
resolution CT chest scan is the investigation of
Answer & Comments
choice for bronchiectasis, while serial PEFR
Correct answer: A recording will help in diagnosing asthma, which
can co-exist.
A significant bronchial reversibility of 44% with
normal postbronchodilator FEV1 is consistent
with asthma not COPD. Asthma guidelines

Dr. Khalid Yusuf El-Zohry – Sohag Teaching Hospital (01118391123) Page | 567
El-Zohry MRCP Questions Bank (Part 2) – Medical Masterclass 2010

[ Q: 91 ] MasterClass Part2 The duration of treatment recommended for


(2010) - Respiratory patients with AIDS is 21 days, based upon the
burden of organisms, slower response time and
A 28-year-old man with end stage renal failure observed early relapses after standard
due to IgA nephropathy received a renal treatment for 14 days. Non-AIDS patients with
transplant 9 months previously. Progress was PCP generally respond to therapy within four to
complicated by several episodes of rejection five days.
that required high-dose steroid treatment. He
now presents with malaise, fever and
[ Q: 92 ] MasterClass Part2
breathlessness. He is hypoxic (arterial oxygen
saturation 88% on air) and his chest radiograph
(2010) - Respiratory
shows patchy interstitial shadowing. A 67-year-old man is on coronary care following
an inferior myocardial infarct. He has had a
Which one of the following statements
temporary transvenous pacemaker inserted for
regarding Pneumocystis carini pneumonia (PCP)
Mobitz type II heart block. He suddenly
in non-HIV infected patients is NOT correct?
becomes light-headed with chest pain. The
A. Patients who are receiving more than two cardiac monitor shows complete heart block
weeks of high dose corticosteroids with with a rate of 20 bpm.
or without other immunosuppressive
agents are at risk of PCP. What should your first TWO actions be?

B. Non-AIDS patients generally have a more A. Call the cardiac arrest team
fulminant onset of symptoms than HIV- B. Ask him to cough repeatedly
infected patients.
C. Give a precordial thump
C. In contrast to AIDS patients, induced
sputum and broncho-alveolar lavage D. Check the lead connections on the
specimens are less often diagnostic. pacemaker

D. The duration of therapy in non-AIDS E. Institute external pacing


patients is 14 days compared to 21 days F. Obtain a 12 lead electrocardiogram
for AIDS patients.
G. Increase the pacemaker voltage to
E. The outcome of PCP in non-AIDS patients maximum
is generally better than in those with
H. Start an isoprenaline infusion
AIDS.
I. Give streptokinase 1.5 MU intravenously
Answer & Comments J. Give morphine 5-10 mg intravenously.
Correct answer: E
Answer & Comments
In contrast to AIDS patients who can have a Correct answer: DG
high burden of organisms, induced sputum or
even specimens obtained by broncho-alveolar The patient's symptoms result from his
lavage (BAL) are less often diagnostic in other bradycardia. The pacemaker is failing to
immunocompromised patients who have a capture. The first thing to do is to check the
lesser burden of organisms. Hence when clinical leads. The pacemaker voltage should then be
suspicion for PCP is high and sputum or BAL increased to maximum which may enable it to
negative, tissue should be obtained. capture.

Dr. Khalid Yusuf El-Zohry – Sohag Teaching Hospital (01118391123) Page | 568
El-Zohry MRCP Questions Bank (Part 2) – Medical Masterclass 2010

The threshold for pacing increases with time [ Q: 94 ] MasterClass Part2


and should be checked at least daily. If capture (2010) - Respiratory
occurs, arrangements should be made to
reposition the pacemaker lead. If it does not, A 75-year-old man with motor neurone disease
external pacing should be instituted. presents with increasing confusion. Clinical
examination reveals signs of motor neurone
Cough and percussion pacing are emergency
disease, a CO2 retention flap and central
measures that occasionally work.
cyanosis, but nothing else abnormal.
Isoprenaline is rarely found even on coronary
Arterial blood gases performed while breathing
care units nowadays.
air reveal PO2 4.5 kPa, PCO2 10.0 kPa, pH 7.15
kPa.
[ Q: 93 ] MasterClass Part2
(2010) - Respiratory What is the most likely diagnosis?
A. Respiratory failure secondary to motor
A 49-year-old man who has just returned from
neurone disease
Barbados presents with a 2-day history of
shortness of breath. His only medication is a B. Aspiration pneumonia
Salbutamol inhaler PRN for his asthma, which is
C. Aspiration pneumonia and respiratory
normally well controlled. His respiratory rate is
failure secondary to motor neurone
26 per minute, heart rate 122 per minute,
disease
oxygen saturation (on air) 92%, peak flow
450L/min (best is 500 L/min). His chest is clear D. Pulmonary embolism
and his chest radiograph is normal. E. Pulmonary embolism and respiratory
failure secondary to motor neurone
Which of the following investigations would be
disease
most appropriate?
A. D-dimer Answer & Comments
B. V/Q scan
Correct answer: A
C. CT pulmonary angiogram
D. ECG Motor neuron disease causes type 2 respiratory
failure. In this case, and in many others, it is
E. Troponin I
very important to be able to determine
whether a patient has respiratory failure due to
Answer & Comments muscle weakness alone, or whether there is an
additional problem that confounds the muscle
Correct answer: C
weakness or may have triggered it. Knowledge
of the simplified alveolar gas equation is key to
This patient's clinical probability of pulmonary
working this out.
embolism is high, hence testing for D-Dimer is
not appropriate - he requires radiological The partial pressure of oxygen in the alveoli
imaging (and would do so even if a serum D- (PAO2, kPa) can be determined from the
dimer level were normal). A CTPA should be inspired oxygen concentration (FiO2, expressed
requested - a V/Q scan is likely to be as a percentage) and arterial carbon dioxide
inconclusive in a patient with asthma. (PaCO2, kPa) using the equation:
PAO2 = FiO2 - (PaCO2 x 1.25), which in this case
is

Dr. Khalid Yusuf El-Zohry – Sohag Teaching Hospital (01118391123) Page | 569
El-Zohry MRCP Questions Bank (Part 2) – Medical Masterclass 2010

PAO2 = 21 - (10 x 1.25) = 8.5 kPa indicate deteriorating liver function and need
for specialist advice/transfer. Close monitoring
The measured PaO2 is 4.5 kPa, meaning that
of clinical state, routine blood chemistry and
the alveolar-arterial (Aa) oxygen gradient is 4
haematology is essential as well as monitoring
kPa, which is within the normal range of 3-5
for development of encephalopathy.
kPa, hence there is no reason to suspect
additional pathology other than pure muscle
weakness for this man's severe hypoxaemia and [ Q: 96 ] MasterClass Part2
hypercapnia. (2010) - Respiratory
A 56-year-old man has been diagnosed with
[ Q: 95 ] MasterClass Part2 chronic obstructive pulmonary disease. His
(2010) - Respiratory FEV1 is 46% predicted. He continues to smoke
20 cigarettes every day. His PaO2 on air is 6.0
A 40-year-woman presents after a deliberate
KPa and PaCO2 is 4.8KPa. He is a retired civil
overdose of paracetamol.
servant and has no other significant past
Which TWO of the following are suggestive of medical history.
the development of acute liver failure 48 hours
Which of TWO of the following will delay
post a paracetamol overdose?
deterioration of his condition?
A. Prothrombin time > 60 seconds, control 9
A. Nebulised beta 2 agonist
seconds
B. Nebulised anticholinergic
B. Albumin > 30g/ litre
C. Long term oxygen treatment
C. Alanine Aminotransferase(ALT) ten times
upper limit of laboratory normal range D. Tiotropium
D. Evidence of metabolic acidosis E. Pulmonary rehabilitation
E. Hyperglycaemia F. Oral Theophyline
F. Hypokalaemia G. Mucolytics
G. Elevated C-reactive protein (CRP) H. Smoking cessation
H. Leucocytosis I. Regular antibiotics
I. ESR > 50mm J. Regular oral prednisolone
J. Thrombocytosis.
Answer & Comments
Answer & Comments Correct answer: CH
Correct answer: AD
The two therapies that will delay deterioration
of his COPD and hence prolong survival are
The elongation of the prothrombin time is a
smoking cessation and long term oxygen
poor prognostic factor and should result in
treatment (LTOT).
discussions with the nearest liver unit and
toxicology advice as there is a definite risk of Smoking cessation slows the rate of decline in
acute liver failure developing. A metabolic FEV1. LTOT is recommended once the PaO2 is <
acidosis whether directly measures on blood 7.3 KPa on two occasions while the patient is
gases, rising lactate or a falling bicarbonate also stable 3 weeks apart. Maintaining a PaO2 above

Dr. Khalid Yusuf El-Zohry – Sohag Teaching Hospital (01118391123) Page | 570
El-Zohry MRCP Questions Bank (Part 2) – Medical Masterclass 2010

7.3KPa prevents pulmonary hypertension and planned within 6 weeks of discharge from
cor pulmonale and improves survival. hospital for acute respiratory illness.

[ Q: 97 ] MasterClass Part2 [ Q: 98 ] MasterClass Part2


(2010) - Respiratory (2010) - Respiratory
A 78-year-old woman with chronic obstructive A breathless 70-year-old smoker presents with
pulmonary disease (who lives in London in the the following lung function tests.
United Kingdom) is planning to visit her
FEV1 1.5L (60%)/ FVC 1.8 L (55%)/ FEV1/FVC
daughter in the USA. Her forced expiratory
ratio = 84%/ TLC = 66% predicted/ RV = 57%
volume in 1 second is 1.2 L (55% of predicted)
predicted/ TLCO = 55% predicted/ KCO = 60%
and Sao2 on air is 96%.
predicted
What would you advise her to do?
What is the most likely diagnosis?
A. Say she should not fly
A. Emphysema
B. Say she can fly but will need in-flight
B. Fibrosing alveolitis
oxygen at a rate of 2 L/min
C. Anaemia
C. Say she can fly but will need in-flight
oxygen at a rate of 4 L/min D. Asthma

D. Say there is no contraindication to air E. Obesity


travel
E. Say an altitude simulation test is required Answer & Comments

Correct answer: B
Answer & Comments

Correct answer: D The lung function tests show a significant


restrictive defect. Only fibrosing alveolitis or
obesity may fit this picture but given the
There are no contraindications to air travel if
decrease in KCO (i.e. after correcting for
Sao2 is over 95%, or if it is between 92 and 95%
alveolar volumes), the most likely answer is
and there are no risk factors. In-flight oxygen is
fibrosis, as pure obesity would not affect the
indicated if Sao2 is below 92%. If Sao2 is
KCO as the gas exchange after correcting for
between 92 and 95% and risk factors are
the alveolar volume would in fact be high.
present, then hypoxic challenge should be
carried out. Risk factors include severe chronic
obstructive pulmonary disease/asthma, severe [ Q: 99 ] MasterClass Part2
restrictive disease, cystic fibrosis, pulmonary (2010) - Respiratory
tuberculosis, comorbidity with other conditions
A 73-year-old man is admitted to hospital with
worsened by hypoxaemia (cerebrovascular
a persistent fever, cough with purulent sputum
accident, ischaemic heart disease and
and right-sided pleuritic chest pain. He
congestive cardiac failure), risk of or previous
presented to his GP with a cough and fever
venous thrombosis, recent pneumothorax, pre-
about 2 weeks ago and was treated with
existing requirement of oxygen or ventilatory
amoxicillin 250 mg tds.
support, history of travel intolerance with
respiratory symptoms and any travelling The TWO most likely diagnoses are:

Dr. Khalid Yusuf El-Zohry – Sohag Teaching Hospital (01118391123) Page | 571
El-Zohry MRCP Questions Bank (Part 2) – Medical Masterclass 2010

A. Empyema deformities typical of rheumatoid, with nodules


at the elbows. Her lung fields were clear on
B. Pulmonary infarction
auscultation. A chest radiograph is shown (see
C. Lung Cancer image).
D. Rheumatoid Arthritis
E. Metastatic Pleural disease
F. Lung Abscess
G. Systemic Lupus erythematosus
H. Vasculitis
I. Exacerbation of COPD
J. Aspiration Pneumonia

Answer & Comments

Correct answer: AF

The history of cough with purulent sputum and


fevers suggest an infective aetiology. The likely
diagnosis is a pneumonia complicated by either What are the TWO possible diagnoses?
an empyema or lung abscess. The low dose of A. Asthma
antibiotics administered may have resulted in
partial treatment of the pneumonia. He B. Emphysema
requires further investigation with a chest C. Lung fibrosis secondary to rheumatoid
radiograph, ultrasound scan of the thorax with arthritis
pleural aspiration and a CT scan of the thorax.
D. Bronchiolitis obliterans
The basic treatment includes intravenous
antibiotics. Insertion of an intercostals drain E. Bronchiectasis
with pleural drainage will be required for an F. Aspergillosis
empyema. Whereas if he has developed a lung
abscess he will require regular chest G. Atypical pneumonia
physiotherapy and airway clearance with at H. Pulmonary eosinophilia
least 2 weeks of intravenous antibiotics
I. Occult lung cancer
followed by further oral treatment.
J. Bilateral hilar lymphadenopathy due to
sarcoidosis
[ Q: 100 ] MasterClass Part2
(2010) - Respiratory
Answer & Comments
A 55-year-old woman with long-standing
Correct answer: AD
rheumatoid arthritis attends the chest clinic
with a history of non-productive cough and
dyspnoea of 8 weeks duration. There is no The chest radiograph is normal.
wheeze, weight loss or haemoptysis. She has Asthma is possible, but emphysema is unlikely
never smoked and has not been exposed to any in a non-smoker, and the history of respiratory
organic dust. On examination she had hand

Dr. Khalid Yusuf El-Zohry – Sohag Teaching Hospital (01118391123) Page | 572
El-Zohry MRCP Questions Bank (Part 2) – Medical Masterclass 2010

symptoms is much too short to entertain this Initially the history might suggest a number of
diagnosis. diagnoses, including cardiac tamponade,
massive pulmonary embolism, haemothorax or
Bronchiolitis obliterans, first described in 1835,
aortic dissection; however the respiratory
can be due to toxic fume inhalation, viral
examination findings indicate that he almost
infections, connective tissue disorders, bone-
certainly has sustained a spontaneous
marrow and lung or heart-lung transplantation,
pneumothorax, which has now developed into
or drug toxicity (penicillamine). It is
a tension pneumothorax.
characterised by cough and dyspnoea without
wheeze. Diagnosis can be confirmed by lung This is now a peri-arrest situation, and there is
biopsy. no time to arrange for portable chest
radiograph, before attempting to reduce the
[ Q: 101 ] MasterClass Part2 pressure in the right hemithorax with the
insertion of a large bore needle. If the diagnosis
(2010) - Respiratory
is correct, insertion may be accompanied by a
You are called to the resuscitation room to see loud ‘hiss’.
a 25-year-old man whose condition has Positive pressure ventilation is relatively
suddenly deteriorated. He had arrived 30 contraindicated in this situation, and will
minutes earlier with a 2-hour history of central probably not be required once the lung has re-
pleuritic chest pain and breathlessness. He inflated.
collapsed while awaiting radiograph and now is
agitated and cyanosed with pulse 120/min and
BP 80/40. Oxygen saturation is reading 79%, [ Q: 102 ] MasterClass Part2
with the patient breathing high flow oxygen via (2010) - Respiratory
a re-breathe mask. Respiratory examination
A 77-year-old man is admitted as an emergency
reveals reduced breath sounds in the right lung
with increasing shortness of breath and a
field with deviation of the trachea towards the
productive cough with green purulent sputum.
left. Percussion is resonant bilaterally.
His chest radiograph shows hyperinflation but
no consolidation. Arterial blood gases shows an
What immediate course of action should you
acute respiratory acidosis.
take?
A. Arrange for urgent portable chest Which of the following statements is true?
radiograph
A. Oxygen should not be given if he has
B. Contact ITU to arrange for the patient to significant hypercapnia.
be ventilated
B. Bronchodilators in the acute exacerbation
C. Insert large bore needle into left have no effect on forced vital capacity.
hemithorax
C. Beta-adrenergic agonists are superior to
D. Insert large bore needle into right anticholinergic agents.
hemithorax
D. There are fewer side effects with
E. Check arterial blood gases and commence ipratropium bromide than with beta
Bipap if hypoxia is confirmed. adrenergic agonists.
E. Metered dose inhalers are more
Answer & Comments expensive than nebulisers.
Correct answer: D

Dr. Khalid Yusuf El-Zohry – Sohag Teaching Hospital (01118391123) Page | 573
El-Zohry MRCP Questions Bank (Part 2) – Medical Masterclass 2010

Answer & Comments Answer & Comments

Correct answer: D Correct answer: D

Oxygen should be given for exacerbation of Obesity hypoventilation eventually leads to


chronic obstructive pulmonary disease, aiming chronic type II respiratory failure and cor
for oxygen saturation values of 90-92% with pulmonale. Patients often have coexisting
corresponding partial pressures of arterial obstructive sleep apnoea syndrome.
oxygen of 60-65 mmHg. These targets ensure
acceptable haemoglobin saturation whilst [ Q: 104 ] MasterClass Part2
reducing the liklihood of the hypercapnia that
(2010) - Respiratory
can accompany the use of supplemental
oxygen. A 35-year-old woman recently diagnosed to be
Bronchodilators increase the FEV1 and forced suffering from sarcoidosis consults you for
vital capacity (FVC) by 15 to 29% over 60 to 120 information about the disease.
minutes.
Of the following statements, which TWO are
Beta adrenergic agonists are often used as first- correct?
line treatment for acute exacerbations of
A. It is hereditary and can affect her
chronic obstructive pulmonary disease, but
offspring.
have not been shown to be superior to
anticholinergic agents and the two classes of B. It can cause facial nerve palsy.
agent are often used in combination. Side C. Raised serum ACE levels are diagnostic.
effects are less frequent and milder with
ipratropium bromide than with beta agonists. D. It does not cause pleural effusions.

Metered dose inhalers are cheaper than E. It always needs an open lung biopsy to
nebulisers. reach a diagnosis.
F. It is best treated with cyclosporin.
[ Q: 103 ] MasterClass Part2 G. It can present with polydipsia.
(2010) - Respiratory H. It is associated with clubbing of the
A 65-year-old woman attends the chest clinic fingers.
with a history of dyspnoea on exertion and I. It is not a cause of pericardial effusions.
morning headaches. She also mentions daytime
J. It is best treated with lung
tiredness. On examination she is normal apart
transplantation.
from pedal oedema and a BMI of 41. She has
never smoked and is on no medications.
Answer & Comments
What is the most likely diagnosis?
Correct answer: BG
A. Narcolepsy
B. Obstructive sleep apnoea syndrome Sarcoidosis is a multisystem non-caseating
granulomatous disorder of unknown aetiology.
C. Chronic obstructive pulmonary disease
It is more common in females and can cause
D. Obesity hypoventilation syndrome Bell's palsy. It can present as polydipsia due to
E. Congestive cardiac failure hypercalcaemia.

Dr. Khalid Yusuf El-Zohry – Sohag Teaching Hospital (01118391123) Page | 574
El-Zohry MRCP Questions Bank (Part 2) – Medical Masterclass 2010

[ Q: 105 ] MasterClass Part2 chest scan, because this group of patients is at


(2010) - Respiratory high risk of death after surgery and is also
unlikely to benefit from LVRS.
A 72-year-old man with emphysema, an ex-
smoker for 4 years, continues to be
[ Q: 106 ] MasterClass Part2
symptomatic on minimal exertion despite
maximal medical therapy, including long-term
(2010) - Respiratory
oxygen treatment. A 6-week pulmonary A 60-year-old man with chest pain is anxious of
rehabilitation programme has also failed to the effect of asbestos exposure that he might
palliate his symptoms. His forced expiratory have incurred in his previous jobs. He worked
volume in 1 second is 0.6 L (28% of predicted) as a thermal-insulating engineer for 10 years
and his gas transfer is 45%. His CT chest scan from the age of 16. He has smoked 20
shows severe heterogeneous emphysema with cigarettes a day since he was 16.
almost completely destroyed upper
lobes. Which one of the following statements is
correct?
Which one of the following treatment options
A. The fact that he worked with asbestos at
should be considered?
young age is a risk factor for
A. Single lung transplantation mesothelioma.
B. Bilateral lung transplantation B. Amosite (brown asbestos) and crocidolite
C. Heart-lung transplantation (blue asbestos) are less potent
carcinogens than chrysotile (white
D. Lung volume reduction surgery
asbestos).
E. Nebulised morphine
C. His smoking increases the possibility of
mesothelioma.
Answer & Comments
D. His asbestos exposure is a risk factor for
Correct answer: D bladder cancer.
E. Chemotherapy is the main stay of
This patient is too old for either bilateral lung treatment of patient with mesothelioma.
transplantation or heart-lung transplantation.
Single lung transplantation is highly unlikely
Answer & Comments
because of the shortage of donors. He should
be considered for lung volume reduction Correct answer: A
surgery (LVRS). The goal of LVRS is to reduce
lung volume by 20-30%, which probably Asbestos exposure is known to be a risk factor
improves pulmonary and chest wall mechanics for mesothelioma and lung cancer. The
at rest and during exercise. Several randomised relationship with other cancers is not certain.
trials have compared LVRS with optimal medical The rate of mesothelioma is increasing. This is
treatment and have shown that patients with mainly due to the long latent period between
upper lobe-predominant emphysema and a low exposure and the clinical development of the
exercise capacity benefit the most from LVRS. disease. Amosite and crocidolite are more
Contraindications include forced expiratory potent carcinogens than chrysotile. Exposure at
volume in 1 second <20% of predicted, diffusing an early age is another risk factor. Cigarette
capacity of the lungs for carbon monoxide <20% smoking does not increase the risk of
of predicted and homogeneous changes on CT mesothelioma.

Dr. Khalid Yusuf El-Zohry – Sohag Teaching Hospital (01118391123) Page | 575
El-Zohry MRCP Questions Bank (Part 2) – Medical Masterclass 2010

The majority of patients with mesothelioma [ Q: 108 ] MasterClass Part2


present with advanced disease. Chemotherapy (2010) - Respiratory
and radiotherapy alone are not helpful. Tri-
modality treatment with surgery, You have been asked to see a 56-year-old
chemotherapy and radiotherapy may be helpful gentleman in A&E. He was brought in with type
in selected cases. I respiratory failure, has already been intubated
and is waiting to be transferred to ITU. His wife
says that he is quite a fit man and is not on any
[ Q: 107 ] MasterClass Part2
medication. He developed cough, fever,
(2010) - Respiratory headaches and body aches 1 week ago. Since
A 74-year-old man was treated in hospital for the last 24 hours he has developed dyspnoea
exacerbation of Chronic Obstructive Pulmonary on exertion. His wife found him unresponsive
Disease (COPD). His condition has improved today. He is a car salesman by profession and
significantly and he is keen to go home. His keeps racing pigeons. His wife also mentions
repeated arterial blood gas (ABG) reading (on that two of his favourite pigeons died 2 weeks
air) showed a pH of 7.35, pCO2 of 4.5kPa, pO2 ago.
of 7.1kPa, HCO3 of 26 mmo/L. Examination is consistent with a pneumonic
illness. He has mild splenomegaly and a pale
What action would be most appropriate?
macular rash on the abdomen, anterior chest
A. Discharge the patient, prescribing an wall and legs.
oxygen cylinder.
Which of the following is the likely causative
B. Discharge the patient once his ABG organism?
returns to normal.
A. Staphyloccocus aureus
C. Request an oxygen concentrator and
discharge the patient once an oxygen B. Legionella pneumophila
concentrator is fitted. C. Mycoplasma pneumoniae
D. Discharge the patient and arrange a D. Chlamydia psittaci
follow up in 6 weeks' time with a
E. Haemophilus influenzae.
repeated ABG.
E. Discharge the patient and advise a follow
Answer & Comments
up by his GP.
Correct answer: D
Answer & Comments
Psittacosis is a systemic illness with primarily
Correct answer: D
pulmonary manifestations. The diagnosis is
suggested by exposure to pigeons. The rash
Long-term oxygen treatment is recommended if (Horder's spots) and splenomegaly are
pO2 is less than 7.3kPa on two occasions in characteristic. Diagnosis can be made by a four-
remission of COPD. This patient's p02 is likely to fold increase in the complement-fixing
improve when he recovers from his current antibodies. Tetracycline for 2 weeks is the
exacerbation and his ABG should be re-checked treatment of choice.
in about 6 weeks time. As he seems able to
tolerate his mild hypoxia, he would not require
any oxygen supplementation at home at this
stage.

Dr. Khalid Yusuf El-Zohry – Sohag Teaching Hospital (01118391123) Page | 576
El-Zohry MRCP Questions Bank (Part 2) – Medical Masterclass 2010

[ Q: 109 ] MasterClass Part2


What are the differential diagnoses?
(2010) - Respiratory
A. Mesothelioma
A 78-year-old man attends the chest clinic with
a 4-month history of cough and intermittent B. Carcinoma right lung
haemoptysis. He has a smoking history of 40 C. Bronchiectasis
pack years and previously worked as a coal
D. Sarcoidosis
miner for 15 years. He also suffers from
rheumatoid arthritis and has been on E. Bronchial adenoma
methotrexate and prednisolone for the last 14
F. Caplan's syndrome
years. Examination is normal apart from
wheeze on the right lower zone. PA and lateral G. Right lower pneumonia
chest radiographs are displayed. H. Tuberculosis
I. Aspiration pneumonia
J. Pulmonary fibrosis.

Answer & Comments

Correct answer: BF

The main feature on the chest radiograph is


a cavitating lesion in the right lower lobe.
Cavitating lesions can be due to squamous
cell carcinoma, rheumatoid nodules,
tuberculosis, Wegener's granulomatosis and
gram negative infection (Klebsiella).
Rheumatoid nodules are specific for
rheumatoid arthritis. These nodules are
located in the interlobular septa, underneath
the pleura, or in the lung parenchyma. They
may antedate the onset of arthritis and can
be single or multiple. Cavitation may occur
and the nodules can be difficult to
differentiate from lung malignancy.
Haemoptysis can occur if the nodules
cavitate and pneumothorax if they erode
into the pleural space.
Patients with rheumatoid arthritis and
exposure to coal dust may develop
pneumoconiotic nodules which can vary in
size from (0.5-5.0 cm). This combination is
called Caplan's syndrome.

Dr. Khalid Yusuf El-Zohry – Sohag Teaching Hospital (01118391123) Page | 577
El-Zohry MRCP Questions Bank (Part 2) – Medical Masterclass 2010

[ Q: 110 ] MasterClass Part2 for cytology. Measurement of the pleural fluid


(2010) - Respiratory and blood concentrations of protein and lactate
dehydrogenase (LDH) can be used to distinguish
A 46-year-old woman is sent to the medical exudates from transudates: exudative effusions
assessment unit because, following a chest meet at least one of the following (Light’s)
infection three weeks ago, she has been criteria:
persistently unwell with a fever that has not
1. Pleural fluid protein divided by serum
responded to a second course of oral
protein >0.5
antibiotics.
2. Pleural fluid LDH divided by serum LDH
Her chest radiograph shows a right sided >0.6
pleural effusion, the TWO most likely causes of
3. Pleural fluid LDH > two thirds the upper
which are:
limit of normal of the serum LDH.
A. Tuberculosis
B. Empyema [ Q: 111 ] MasterClass Part2
C. Rheumatoid arthritis (2010) - Respiratory
D. Pulmonary emboli A 23-year-old man with cystic fibrosis presents
E. Familial Mediterranean Fever with a large right-sided pneumothorax.

F. Systemic lupus erythematosus What would be the best management?


G. Lung cancer A. Bed rest
H. Mesothelioma B. Overnight observation
I. Meig’s syndrome C. High flow (10 l/min) oxygen treatment
J. Pneumonia with effusion. D. Simple aspiration
E. Intercostal tube drainage and intravenous
Answer & Comments antibiotics
Correct answer: BJ F. Intercostal tube drainage with immediate
suction
The main symptom is fever, suggesting an G. Plan for talc pleurodesis
underlying inflammatory or infective process,
and the history is short, indicating an acute H. Plan for tetracycline pleurodesis
problem. Hence the two most likely diagnoses I. Plan for bleomycin pleurodesis
are pneumonia with effusion and empyema,
J. Plan for partial pleurectomy.
although all of the other conditions listed can
cause pleural effusions.
Answer & Comments
The critical test is examination of the pleural
fluid. If it is opaque / turbid and with a foul Correct answer: EJ
smell then it is clearly an empyema. In cases
that are less clear-cut the pleural fluid pH is The treatment of pneumothorax for patients
probably the single most useful test, with with CF is similar to that for non-CF patients.
empyema strongly suggested by pH<7.22. A
A large pneumothorax requires immediate
sample should be sent for microscopy and
treatment with intercostal tube drainage. In
culture (aerobic and anaerobic, and for TB), and

Dr. Khalid Yusuf El-Zohry – Sohag Teaching Hospital (01118391123) Page | 578
El-Zohry MRCP Questions Bank (Part 2) – Medical Masterclass 2010

this context it is important to commence Answer & Comments


antibiotics at the same time to prevent sputum
Correct answer: BJ
retention, which can delay re-expansion of the
collapsed lung.
The most important differential is clearly lung
Surgical intervention should be considered,
cancer, with a secondary metastasis next on the
even after a first episode of pneumothorax, in a list.
patient with chronic lung disease because the
recurrence rate is high (50%). Partial Appropriate investigation would involve:
pleurectomy is the treatment of choice with a 1. CT scan of chest, liver and adrenals
success rate of 95%.
2. Bronchoscopy

[ Q: 112 ] MasterClass Part2 3. Sputum for cytology and microbiology


(including Acid-fast bacillus (AFB))
(2010) - Respiratory
4. Blood tests for routine haematology,
A 52-year-old man has this chest radiograph coagulation screen, electrolytes
taken as part of a health screening program. (including calcium) and liver function
tests.

The TWO most likely diagnoses are:


A. Sarcoidosis [ Q: 113 ] MasterClass Part2
(2010) - Respiratory
B. Lung cancer
A 35-year-old man presents with a first episode
C. Rheumatoid nodule
of haemoptysis. He has felt a bit feverish for the
D. Tuberculosis last few days, but has not had sweats or rigors.
E. Bronchial adenoma He is generally fit and well and gives no history
of previous respiratory problems. On
F. Pneumonia
examination his temperature is 37.8ºC and his
G. Lung abscess chest is clear.
H. Arteriovenous malformation Which TWO of the following are the most likely
I. Wegener’s granulomatosis diagnoses?
J. Secondary metastasis. A. Pulmonary tuberculosis

Dr. Khalid Yusuf El-Zohry – Sohag Teaching Hospital (01118391123) Page | 579
El-Zohry MRCP Questions Bank (Part 2) – Medical Masterclass 2010

B. Bronchiectasis A. Rheumatoid arthritis is unlikely to be the


cause of the effusion if pleural fluid
C. Pneumonia
glucose is above 1.6mmol/L.
D. Lung cancer
B. In pleural effusion secondary to RA,
E. Benign bronchial tumour pleural pH is usually >7.35.
F. Pulmonary arteriovenous malformation C. Rheumatoid pleural effusions have high
G. Asthma C4 complement levels.

H. Chronic obstructive pulmonary disease D. Up to 40% of patients with rheumatoid


arthritis develop pleural effusions.
I. Goodpasture’s syndrome
E. The majority of patients with rheumatoid
J. Pulmonary embolism. pleural effusions are women.

Answer & Comments Answer & Comments


Correct answer: CJ Correct answer: A

The most common cause of haemoptysis in a


Pleural involvement occurs in 5% of patients
young patient is pulmonary infection, either with rheumatoid arthritis. It usually affects men
with pyogenic bacteria or mycobacterium and it can be serous, turbid, yellow, milky or
tuberculosis depending on the clinical context.
haemorrhagic. RA is unlikely to be the cause of
The short history with previous good health a pleural effusion if the pleural fluid glucose is
makes TB unlikely in this case. A clear chest
more than 1.6mmol/L. Eighty percent of
examination is not unusual in pneumonia,
rheumatoid pleural effusions have a pleural
particularly in the context of atypical or viral fluid pH of less than 7.30. In rheumatoid pleural
pathogens. effusions, C4 complement level is below 0.04
The next most common causes would be g/L.
bronchiectasis (made unlikely in this case by the
absence of any previous respiratory problems) [ Q: 115 ] MasterClass Part2
or pulmonary embolism.
(2010) - Respiratory
Tumour (benign or malignant), Goodpasture’s
syndrome or pulmonary vasculitis (often as part A 65-year-woman, a lifelong non-smoker, has
of a pulmonary-renal syndrome) also need to presented with an 8-month history of shortness
be considered but are much less likely of breath and daily sputum production. Physical
diagnoses. examination shows a left pleural effusion and
yellow discoloration of nails. Diagnostic pleural
aspiration reveals that the fluid is an exudate
[ Q: 114 ] MasterClass Part2 and contains no malignant cells.
(2010) - Respiratory
What would be the most appropriate action to
A 55-year-old man known to suffer from take next?
rheumatoid arthritis (RA) is admitted with a left
pleural effusion. He has never smoked and A. Bronchoscopy
apart from dyspnea, he is asymptomatic. B. CT chest scan

Which of the following is true?? C. Pleural biopsy


D. Simple pleural aspiration

Dr. Khalid Yusuf El-Zohry – Sohag Teaching Hospital (01118391123) Page | 580
El-Zohry MRCP Questions Bank (Part 2) – Medical Masterclass 2010

E. Pleural drainage with talc pleurodesis [ Q: 117 ] MasterClass Part2


(2010) - Respiratory
Answer & Comments
A 56-year-old retired plumber had a chest
Correct answer: E radiograph that was arranged as a part of
routine investigations before elective
Yellow nails and pleural effusion suggest yellow cholecystectomy. This showed bilateral pleural
nail syndrome. She most probably also has thickening. CT chest scan confirms calcified
bronchiectasis, which is part of the syndrome. pleural plaques suggestive of previous asbestos
This would explain her daily sputum production. exposure. He smokes 20 cigarettes daily, but
No further investigations are required. As denies any shortness of breath or chest pain,
pleural effusion is likely to recur, pleural and full lung function tests are normal.
drainage with talc pleurodesis is
recommended. Which TWO of the following are correct?
A. Regular follow up with a repeated chest
[ Q: 116 ] MasterClass Part2 radiograph is required.
(2010) - Respiratory B. Pleural biopsy is essential to make a final
diagnosis.
A middle-aged meat factory worker is found to
have Q fever pneumonia. C. Bronchoscopy should be arranged to rule
out lung cancer.
Which of the following statements is correct?
D. No further investigations are required
A. His occupation is important. apart from annual spirometry.
B. He requires high dose penicillin. E. Asbestos related pleural plaques are
precursors of malignant change.
C. There are no long-term sequelae.
F. In asbestos related pleural plaques
D. The organism responsible is Chlamydia
calcification of the pleural plaques is
pneumoniae.
always seen on a chest radiograph.
E. Q fever is a notifiable disease.
G. The risk of lung cancer with combined
exposure to asbestos and cigarette smoke
Answer & Comments
appears to be multiplicative.
Correct answer: A H. Asbestos-related pleural plaques entitle
to Industrial Injuries Disablement benefit.
Q fever is due to Coxiella burnetii and is
I. Normal chest radiograph excludes
acquired through contact with animals. It is not
asbestosis.
notifiable, but can occur in outbreaks in farming
communities and in abbatoirs. Treatment is J. Normal high-resolution computed
with prolonged courses of tetracyclines. Rarely tomography (HRCT) scan rules out
infection can be persistent leading to chronic asbestosis.
symptoms including fatigue, malaise and
sweats. Lengthy antibiotic courses can be Answer & Comments
curative.
Correct answer: DH

Dr. Khalid Yusuf El-Zohry – Sohag Teaching Hospital (01118391123) Page | 581
El-Zohry MRCP Questions Bank (Part 2) – Medical Masterclass 2010

Approximately 50% of persons exposed to count, urea and electrolytes and liver function
asbestos develop pleural plaques. Up to 30% of tests are normal, excepting a leukocytosis of 16.
asbestos-exposed individuals have an abnormal His chest radiograph is shown (see image).
HRCT in spite of a normal chest radiograph.
However, HRCT may still appear normal or near
Which one of the following statements is true?
normal in cases of histopathologically-proven
asbestosis. Calcification is identified by chest A. Because of the lobar distribution, it can
radiography in 20%, on CT scanning in 50%, and be assumed that the infecting organism is
at morphologic examination in 80%. unlikely to be an atypical one.

Benign pleural plaques do not require a regular B. He requires a bronchoscopy to rule out
follow up, but a patient should be advised to cancer causing an obstructive pneumonia
report any new symptoms as that may warrant and haemoptysis.
further investigations. C. He has non-severe pneumonia.
As this patient is a smoker, he should have D. If his symptoms and signs resolve rapidly
spirometry annually as a screening for chronic he need not have a follow-up chest
obstructive pulmonary disease. radiograph.
E. He should be given full dose
[ Q: 118 ] MasterClass Part2 anticoagulation to cover the possibility
(2010) - Respiratory that this may be a pulmonary embolus
causing pulmonary infarction and
A 65-year-old smoker presents to A&E with a
haemoptysis.
cough productive of green sputum and blood,
fever, right-sided pleuritic pain and
breathlessness. He is confused, has a Answer & Comments
temperature of 39.8C, BP 120/70, pulse 120 Correct answer: C
regular, respiratory rate 20. His cardiovascular
examination is normal and he has bronchial
It is not possible to predict the organism from
breath sounds at the right apex. His full blood
radiographic features.
Pneumonia itself can cause
haemoptysis and bronchoscopy
should only be considered if
pneumonia or haemoptysis fails to
resolve at follow-up.
Severity of pneumonia is defined
according to the (new) Confusion
(abbreviated mental test score <8),
Urea >7 mmol, Respiratory rate
(>30) and Blood Pressure (systolic <
90, diastolic <60) (CURB) criteria.
More than two of these features
suggests severe pneumonia and
these patients should be assessed
by a respiratory physician.
All smokers, patients over 50 or

Dr. Khalid Yusuf El-Zohry – Sohag Teaching Hospital (01118391123) Page | 582
El-Zohry MRCP Questions Bank (Part 2) – Medical Masterclass 2010

those with non-resolving symptoms or signs [ Q: 120 ] MasterClass Part2


should have a follow-up chest film at 6 weeks. (2010) - Respiratory
Although pulmonary infarction should be on
A 48-year-old woman presents with a pleural
the differential diagnosis, the clinical probability
effusion 3 weeks after receiving antibiotics
strongly favours pneumonia. There are no
from her general practitioner for a chest
atypical features incompatible with pneumonia.
infection. You suspect a parapneumonic
There is lobar consolidation and he is coughing
effusion.
green sputum. There are no cardiovascular
features to suggest PE. He should receive Which one of the following statements
regular DVT prophylaxis. regarding parapneumonic effusions is true?
A. is any effusion associated with bacterial
[ Q: 119 ] MasterClass Part2 pneumonia, lung abscess or
(2010) - Respiratory bronchiectasis.
A 72-year-old man is referred to the chest clinic B. always need to be drained.
by the cardiologists with a history of shortness C. is commonly due to mycobacteria.
of breath two months following an uneventful
aortic valve repair. An echocardiogram is D. should be treated surgically.
normal. and a CXR shows a mildly elevated left E. should never be aspirated.
hemidiaphragm. A CXR done prior to his surgery
was normal.
Answer & Comments
Which of the following investigations is most Correct answer: A
appropriate?
A. High-resolution CT chest scan Any pleural effusion associated with bacterial
pneumonia, lung abscess or bronchiectasis is a
B. Regular peak expiratory flow rate
parapneumonic effusion. Up to 40% of patients
monitoring
with pneumonia have an accompanying pleural
C. Sitting and lying spirometry effusion and the majority resolve without any
D. Arterial blood gases specific therapy directed towards the pleural
fluid. The evolution of these effusions can be
E. Bronchoscopy divided into 3 stages:
 exudative
Answer & Comments
 fibropurulent
Correct answer: C
 organistaion.
The diaphragm has to work harder to inflate the During these stages the pH and glucose level of
lungs when lying down, because there is no the pleural fluid becomes progressively lower.
assistance from gravity. Normally no functional These effusions can be monitored by the pH:
effect is seen, but in diaphragmatic weakness  pH<7 - absolute indication to insert an
there will be a relative reduction in forced
intercostal drain
inspiratory flow (and pressure, which is more
difficult to measure) when lying down  pH>7.20 - usually do not need drainage
compared with standing or sitting.

Dr. Khalid Yusuf El-Zohry – Sohag Teaching Hospital (01118391123) Page | 583
El-Zohry MRCP Questions Bank (Part 2) – Medical Masterclass 2010

 pH 7-7.20 - should be monitored and The history is most compatible with erythema
intercostal drain inserted if the pH nodosum and therefore in combination with
swings towards 7. night sweats one should consider either TB or
sarcoidosis. Given the chest radiograph of
Prior to the antibiotic era Streptococcus
bilateral hilar lymphadenopathy, the likeliest
pneumoniae or haemolytic streptococci were
diagnosis is sarcoidosis.
the commonest organisms. Anaerobic
organisms are commonly seen nowadays.
[ Q: 122 ] MasterClass Part2
[ Q: 121 ] MasterClass Part2 (2010) - Respiratory
(2010) - Respiratory (1) A 26-year-old patient with known cystic
fibrosis (CF) presents with weight loss, urinary
A 42-year-old West Indian woman presents
frequency and frequent infective exacerbation
with a painful rash on her legs, weight loss,
of his chest problem.
night sweats and the following chest radiograph
(see image).
The most likely complication of his CF is:
A. Liver cirrhosis
B. Diverticulitis
C. Secondary amyloidosis
D. Diabetes mellitus
E. Renal tubular acidosis.

Answer & Comments

Correct answer: D

Approximately 20% of adult patients with cystic


fibrosis develop diabetes mellitus (DM). The
reason for this is direct damage to the insulin-
producing islets. The commonest
manifestations of DM in CF patients are weight
What is the most likely diagnosis? loss, increased frequency of infective
exacerbation, as well as decline of lung function
A. Carcinoma bronchus
tests. Increased urinary frequency is not specific
B. Tuberculosis for DM in adult CF patients.
C. Sarcoidosis For unknown reasons, renal tubular acidosis
D. Lymphoma and secondary amyloidosis are quite rare in CF
patients, despite the highly damaged lung
E. Streptococcal infection. which is frequently colonized with bacteria
causing repeated clinical exacerbations.
Answer & Comments
Liver cirrhosis occur in 5-10% of adult CF
Correct answer: C patients. The manifestations are similar to
those in non-CF patients.
Diverticulits is very rare in treated CF patients.

Dr. Khalid Yusuf El-Zohry – Sohag Teaching Hospital (01118391123) Page | 584
El-Zohry MRCP Questions Bank (Part 2) – Medical Masterclass 2010

[ Q: 123 ] MasterClass Part2 [ Q: 124 ] MasterClass Part2


(2010) - Respiratory (2010) - Respiratory
A 72-year-old gentleman with a 10-year history A 45-year-old female non-smoker presents with
of progressive breathlessness presents. He has a 2-week history of fevers, cough and
had a marked deterioration over the past six productive sputum. She has been treated with
months and is now breathless after walking 50 amoxicillin 250 mg tds for 7 days by her GP, but
yards on the flat. He is a smoker with an she continues to have persistent fevers. Her
equivalent exposure to 40 pack years. See Respiratory rate is 30/min, BP 90/60 mmHg,
image for chest radiograph. temperature is 38oC, and oxygen saturations
are 96% on air.

Which of the following is her most appropriate


management?
A. Change the antibiotic to Augmentin
B. Add a macrolide antibiotic
C. Stop antibiotics, culture sputum and
blood, and then treat accordingly
D. Admit and manage as an in-patient with
intravenous antibiotics
E. Arrange chest radiograph

Answer & Comments

Correct answer: D

She has not responded to basic treatment for


community-acquired pneumonia. She also has a
CURB-65 score (BTS recommended severity
The most likely diagnosis is:
score) of 2, where C= confusion, U= urea >
A. Pulmonary embolism 7mmol/L, R= resp rate ≥30, B= systolic<90 or
B. Right hilar mass with malignancy diastolic≤60 mmHg, 65= age over 65 years

C. Emphysema A patient with one or more of these features in


the community should be referred and assessed
D. Cardiac failure in hospital and is usually considered for
E. Asthma. inpatient treatment.

Answer & Comments [ Q: 125 ] MasterClass Part2


Correct answer: C (2010) - Respiratory
A 43-year-old woman with long-standing
The chest radiograph demonstrates asthma is admitted with an exacerbation. She is
hyperinflation with low flat diaphragms and cyanosed and unable to speak more than three
fewer lung markings at the bases. Lung function words at a time. She is using her accessory
would help confirm the diagnosis. muscles, chest expansion is reduced but the

Dr. Khalid Yusuf El-Zohry – Sohag Teaching Hospital (01118391123) Page | 585
El-Zohry MRCP Questions Bank (Part 2) – Medical Masterclass 2010

same on both sides, and a wheeze can be heard [ Q: 126 ] MasterClass Part2
bilaterally. (2010) - Respiratory
Which one of the following is the best initial A 25-year-old woman presents to A&E 1 hour
treatment? after consuming 28 x 500mg paracetamol
A. Maximum inspired oxygen by face mask; tablets.
nebulised salbutamol (5 mg) driven by
Which of the following statements is true?
oxygen
A. If the INR is normal on a sample taken
B. Nebulised salbutamol (5 mg) driven by air
four hours from the time of ingestion,
C. Nebulised salbutamol (50 mg) driven by liver damage is unlikely to occur.
oxygen
B. Alcohol ingestion at the time of
D. Oxygen 35% by face mask; nebulised consumption of paracetamol is an
salbutamol (10 mg) driven by 35% oxygen indication for N-acetyl-cysteine treatment
E. Maximum inspired oxygen by face mask; if paracetamol level at 4 hours exceeds
decompress both sides of the chest by the ‘high-risk’ line.
inserting venflons into the second C. Activated charcoal may be beneficial if
intercostal spaces in the mid-clavicular given immediately.
line bilaterally.
D. Onset of tinnitus may be an early
symptom of liver failure.
Answer & Comments
E. Deterioration in conscious level within
Correct answer: A the first 24 hours usually suggests hepatic
encephalopathy.
Resuscitation is the first priority. Maximum
inspired oxygen should be given by facemask: Answer & Comments
this is best achieved using a reservoir bag at a
flow rate of 15 l/min, which can generate an Correct answer: C
FiO2 of about 85%. Nebulised salbutamol (5-10
mg) driven by oxygen should be given, and Abnormal blood clotting following paracetamol
many would add ipatropium bromide (Atrovent, overdose results from loss of production of
500 microg) to the nebuliser chamber at the clotting factors by hepatocytes, and is therefore
same time as the salbutamol. If the woman a good early marker of synthetic liver function.
does not improve, then call for assistance from The INR rises first because Factor VII has the
ICU sooner rather than later. shortest half-life and is therefore the first to
disappear from the blood. However it is unusual
Although it is always important to consider
to see any abnormality in blood clotting less
pneumothorax in any breathless patient, there
than 18 hours from ingestion, so normal INR at
is no evidence at all to suggest that this woman
4 hours is unhelpful. Abnormal INR at the time
has bilateral pneumothoraces and she would
of admission may indicate prior chronic liver
not be well served by chest decompression.
disease, warfrin ingestion or suggest that
ingestion of the drug occurred earlier than the
patient reports.
Because acute alcohol intoxication inhibits liver
enzyme function, less paracetamol will be
metabolised to the toxic metabolite, so that

Dr. Khalid Yusuf El-Zohry – Sohag Teaching Hospital (01118391123) Page | 586
El-Zohry MRCP Questions Bank (Part 2) – Medical Masterclass 2010

liver damage is, if anything, less likely. Chronic Infected pleural effusions should be drained.
alcohol abuse causes liver enzyme induction Indications of infection include fluid that is
and is an indication for treatment with N-acetyl frankly purulent, Gram stain showing
cysteine if paracetamol level exceeds the ‘high organisms, or fluid pH<7.2. Pleural fluid should
risk’ line. also be drained if clinical improvement is slow
despite antibiotics.
Activated charcoal is only likely to be beneficial
if given within 1 hour of ingestion of
paracetamol. [ Q: 128 ] MasterClass Part2
If the patient complains of tinnitus, this (2010) - Respiratory
suggests concurrent salicylate consumption, A 73-year-old man with known chronic
which requires specific treatment according to obstructive pulmonary disease (COPD) is
the plasma level. admitted with symptoms of worsening
Hepatic encephalopathy rarely occurs less than breathlessness and confusion. His arterial blood
48 hours from consumption; reduction in level gases are as follows: pH 7.20, PO2 6.0 kPa (45
of consciousness in the first 24-48 hours is mmHg), PCO2 9.1 kPa (68 mmHg).
usually a result of concurrent consumption of
sedative drugs (particularly opiates in Which one of the following statements
combination drugs such as co-dydramol of co- regarding his management is correct?
proxamol) or hypoglycaemia. Regular A. Mechanical ventilation with tracheal
monitoring of blood glucose is recommended intubation should be considered
for patients with elevated paracetamol level
B. Patients with COPD are particularly
requiring treatment with N-acetyl cysteine, or
difficult to wean from mechanical
following any change in conscious level.
ventilation
C. Non-invasive positive pressure ventilation
[ Q: 127 ] MasterClass Part2
(NIPPV) is not appropriate for patients
(2010) - Respiratory with COPD
A 65-year-old woman is admitted with left D. Low arterial pH in COPD is associated
sided pneumonia and pleural effusion. Pleural with a high mortality rate
fluid is aspirated and sent for tests.
E. Emphysema is a contraindication for nasal
Which of the following is an indication for ventilation because of the risk of
inserting a chest drain? pneumothorax

A. Pleural fluid pH <7.2


Answer & Comments
B. Serous pleural fluid
Correct answer: A
C. Blood stained pleural fluid
D. Pleural fluid glucose >2mmol/l This patient should be considered for
E. Pleural fluid lactate dehydrogenase > mechanical ventilation. Non-invasive
200IU/l. intermittent positive pressure ventilation
(NIPPV) is a useful method of treatment of
acute or chronic respiratory failure. The results
Answer & Comments
are best in patients with type II respiratory
Correct answer: A failure without acidosis. NIPPV should be either
a first step of management in those who are

Dr. Khalid Yusuf El-Zohry – Sohag Teaching Hospital (01118391123) Page | 587
El-Zohry MRCP Questions Bank (Part 2) – Medical Masterclass 2010

suitable for mechanical ventilation or the HRCT has a high sensitivity for PCP among HIV-
management of choice in patients who are positive patients: a negative HRCT virtually
deemed to be unsuitable for intubation and excludes PCP.
ventilation. The rate of pneumothorax is small,
In HIV-infected patients the two blood tests
but is higher in machines using volume-cycled
that are most commonly abnormal in
mode rather than pressure cycled mode.
association with PCP are a CD4 count below 200
The rate of successful weaning from mechanical cells/mm3 and an elevated LDH, which is
ventilation of appropriately selected patients present in 90% of cases. A rising LDH level
with COPD is similar to that for other diseases. despite appropriate treatment portends a poor
prognosis.
[ Q: 129 ] MasterClass Part2
(2010) - Respiratory [ Q: 130 ] MasterClass Part2
(2010) - Respiratory
A 38-year-old homosexual man known to be
HIV positive gives a 3-week history of A 69-year-old man with chronic obstructive
progressive shortness of breath. No significant pulmonary disease (COPD) on long-term oxygen
abnormality is found on physical examination therapy (LTOT) is reviewed in clinic. He
and his chest radiograph is normal. Oxygen complains of persistent leg swelling. His arterial
saturation at rest (on air) is 96%. blood gases, carried out by COPD nurses 3-
weeks ago on a supplemental oxygen flow rate
Which one of the following would be of LEAST of 2 l/min, showed a pH 7.34, pCO2 5.6kPa, pO2
use in excluding the possibility of Pneumocystis 8.0kPa and HCO3 28 mmol/l.
carini pneumonia (PCP)?
A. The finding of normal oxygenation on What would you recommend?
exercise. A. Start diuretic
B. A normal high resolution CT (HRCT) scan B. Echocardiogram
of the lungs.
C. ECG
C. A normal serum lactate dehydrogenase
D. Overnight SaO2 monitoring
level (LDH).
E. Chest radiograph
D. Normal peak expiratory flow rate.
E. A blood CD4 count of 300 cells/mm3. Answer & Comments

Correct answer: D
Answer & Comments

Correct answer: D The presence of persistent oedema or


secondary polycythaemia suggests that the
In PCP a normal chest examination occurs in correction of overnight SaO2 may be
50% cases and a chest radiograph is initially inadequate. To exclude nocturnal hypoxaemia
normal in up to 25%. an overnight SaO2 monitoring should be carried
The presence of PCP is highly unlikely if the out.
response to exercise is normal or if
measurement of the diffusing capacity for
carbon monoxide (DLCO) is normal, defined by
70% of the predicted value or greater.

Dr. Khalid Yusuf El-Zohry – Sohag Teaching Hospital (01118391123) Page | 588
El-Zohry MRCP Questions Bank (Part 2) – Medical Masterclass 2010

[ Q: 131 ] MasterClass Part2 J. Ultrasound abdomen.


(2010) - Respiratory
Answer & Comments
Which of the following statements concerning
Correct answer: CF
diabetic ketoacidosis is incorrect?
A. It may present as an acute abdomen. The two most specific tests in this instance are
B. Serum amylase may be elevated without the transbronchial biopsy and a
evidence of pancreatitis. mediastinoscopy, as these will potentially give
firm histological evidence of non-caseating
C. The white cell count may be elevated
granuloma and the absence of lymphoma. In
without evidence of infection.
this setting, finding mycobacterial organisms is
D. The anion gap is normal. often of low yield and therefore with a strong
E. Although serum potassium is often raised, Heaf Test and indicative histology, a diagnosis
total body potassium is reduced. of Tuberculosis can be made in the absence of
firm microbiology. The serum ACE (angiotensis
converting enzyme) is of low sensitivity and
Answer & Comments
specificity. A PET scan can give evidence of
Correct answer: D activity in all three conditions.

DKA is associated with a high anion gap [ Q: 133 ] MasterClass Part2


acidosis. (2010) - Respiratory
A 77-year-old man is admitted as a medical
[ Q: 132 ] MasterClass Part2
emergency with a history of progressive
(2010) - Respiratory shortness of breath. His chest radiograph is
(2) A 27-year-old man gives a 3-month history shown (see image).
of weight loss, coughing and night sweats. His
chest radiograph suggests bilateral hilar
lymphadenopathy.

What are the TWO most specific investigations


that will help differentiate sarcoidosis from
tuberculosis or lymphoma?
A. CT thorax
B. Heaf test
C. Transbronchial biopsy
D. Bone marrow examination
E. Positron emission tomography
F. Mediastinoscopy
G. Serum Angiotensin Converting Enzyme
H. Gallium scan Apart from scoliosis, what other radiological
abnormality is seen?
I. White cell scan

Dr. Khalid Yusuf El-Zohry – Sohag Teaching Hospital (01118391123) Page | 589
El-Zohry MRCP Questions Bank (Part 2) – Medical Masterclass 2010

A. Left upper lobe consolidation and C. Both external and transvenous


collapse pacemakers should be initially set at a
fixed rate of 60 beats per minute.
B. Localised left upper pneumothorax
D. The tip of a temporary transvenous
C. Left upper lobectomy
pacemaker should lie at the apex of the
D. Left thoracoplasty left ventricle.
E. Impossible to comment as the film is E. The threshold of a temporary
rotated. transvenous pacemaker should be
checked at least daily.
Answer & Comments
F. A temporary transvenous pacemaker
Correct answer: D generates a pacing spike followed by a
narrow QRS complex.
Thoracoplasty is a surgical procedure in which, G. The voltage of both types of temporary
to decrease thoracic volume, resected ribs are pacemaker should be set at the minimum
either removed or are replaced with their that achieves capture.
convexity inward.
H. Increasing threshold is an indication for
The only current indication for thoracoplasty is repositioning of a temporary pacing wire.
to close a persistent pleural space, but before
I. The subclavian route is the preferred
1952 thoracoplasty was frequently used as a
route for temporary pacing in all such
form of collapse therapy for cavitary pulmonary
patients.
tuberculosis.
J. The patient will need a permanent pacing
Scoliosis can develop after thoracoplasty in pre-
system within 40 minutes.
pubertal patients (its severity is related to the
number of ribs removed), eventually leading to
nocturnal hypoventilation, respiratory failure Answer & Comments
and a requirement for oxygen therapy / non- Correct answer: EH
invasive ventilation at night.
External pacing is a bridging technique to
[ Q: 134 ] MasterClass Part2 transvenous pacing only; a temporary pacing
(2010) - Respiratory wire should be inserted as soon as possible.
External pacing is painful and usually requires
A 56-year-old man is receiving coronary care intravenous analgesia and sedation.
following a recent inferior myocardial infarct for Pacemakers should be set on demand to ensure
which he received intravenous thrombolytic that they do not compete with any underlying
therapy. 24 hours later he develops rhythm. The temporary wire should lie at the
symptomatic bradycardia. apex of the right ventricle. As such, it generates
a broad QRS complex. The threshold to achieve
Which of the following statements regarding
capture should be checked daily and the
temporary pacing are correct?
voltage should then be set at at least 2 volts
A. External pacing may avoid the need for above threshold.
transvenous pacing.
An increasing threshold suggests that the wire
B. External pacing is well tolerated by has either moved or that there is damage to the
patients. myocardium requiring repositioning. Patients
who have recently received thrombolytics

Dr. Khalid Yusuf El-Zohry – Sohag Teaching Hospital (01118391123) Page | 590
El-Zohry MRCP Questions Bank (Part 2) – Medical Masterclass 2010

should have pacing wires situated via the Answer & Comments
femoral route to allow compression in case of
Correct answer: BD
bleeding. In these situations, a permanent
system may not be required as normal
conduction may return over a few hours or Small bore (10-14 F) intercostal tubes should be
days. the initial choice for effusion drainage and
pleurodesis. The intercostal tube should be
clamped for 1 hour after sclerosant
[ Q: 135 ] MasterClass Part2 administration. Patient rotation is required
(2010) - Respiratory when using talc slurry, but is not necessary
An elective admission for chemical pleurodesis after intrapleural instillation of tetracycline.
is arranged for a 77-year-old man with Talc is the most effective sclerosant, but a few
malignant pleural effusion secondary to patients (1%) develop acute respiratory failure
adenocarcinoma of the right lung. following its administration. The success rate
(complete and partial response) for talc slurry is
Which TWO of the following statements about 90%, for tetracycline 65%, for bleomycin
regarding chemical pleurodesis are correct? 61%.
A. Large bore (32-38 F) intercostals tubes When excessive fluid drainage persists
are preferable to small bore tubes (10-14 (>250ml/day), repeat pleurodesis may be
F). attempted with an alternative sclerosant.
B. Talc is the most effective sclerosant.
C. Tetracycline and bleomycin are as [ Q: 136 ] MasterClass Part2
effective as talc. (2010) - Respiratory
D. Tetracycline is modestly effective, has A 28-year-old man with long-standing asthma is
few severe side effects, and is the admitted with an acute exacerbation.
preferred sclerosant to minimise adverse
event rates. Which one of the following would suggest that
he has a life-threatening attack?
E. Pleuritic chest pain and fever are
uncommon side effects of sclerosant A. His peak flow is 40% of predicted /
administration. personal best.
F. Intrapleural administration of lignocaine B. He is confused.
is no longer recommended. C. He is using accessory muscles of
G. The intercostal tube should be clamped respiration.
for 24 hours after sclerosant D. His pulse rate is 120 / min.
administration.
E. His respiratory rate is 40 / min.
H. Patient rotation is not required.
I. Pleurodesis should not be attempted if Answer & Comments
only partial lung apposition is achieved.
Correct answer: B
J. If pleurodesis fails (excessive fluid
drainage persist) alternative sclerosants
An attack of asthma is life-threatening asthma if
are also unlikely to be helpful.
there is:

Dr. Khalid Yusuf El-Zohry – Sohag Teaching Hospital (01118391123) Page | 591
El-Zohry MRCP Questions Bank (Part 2) – Medical Masterclass 2010

1. peak flow <33% predicted / personal Congestive cardiac failure would clearly need to
best; be considered in a man with known cardiac
disease, evidenced in this case by atrial
2. silent chest;
fibrillation, but this is not a sustainable
3. cyanosis; diagnosis in the absence of signs other than
4. altered consciousness, confusion or basal crackles. However, you would obviously
coma; look carefully for raised JVP, displaced apex, LV
or RV heaves, added heart sounds and
5. exhaustion, inability to speak; peripheral oedema before excluding this
6. hypotension or bradycardia. diagnosis.
An acute attack of asthma is severe if there is: Chronic pulmonary embolism could present
with insidious breathlessness but is not likely in
1. peak flow <50% predicted / personal
a man who is on warfarin, and there are no
best;
signs to support the diagnosis (raised JVP, RV
2. pulse rate >110/min; heave, loud P2, RV gallop).
3. respiratory rate >25/min; The diagnosis of idiopathic interstitial lung
4. cannot complete sentences in one disease cannot be made in the presence of a
breath; known secondary cause of interstitial lung
disease.
5. use of accessory muscles of respiration
/ intercostal recession.
[ Q: 138 ] MasterClass Part2
(2010) - Respiratory
[ Q: 137 ] MasterClass Part2
(2010) - Respiratory A 40-year-old female is breathless and has a full
set of lung function performed: FEV1 1.1L (60%)
A 50-year-old man presents with breathlessness FVC 1.3 L (55%) FEV1/FVC ratio = 84% TLC =
that has got gradually worse over three 65% predicted RV = 58% predicted TLCO = 55%
months. He has long-standing atrial fibrillation, predicted KCO = 105% predicted
for which he takes warfarin and amiodarone.
On examination his pulse is 80/min in AF and he What is the most likely diagnosis?
has fine bibasal crackles, but there are no other
A. Fibrosing alveolitis
abnormal physical signs.
B. COPD
The most likely diagnosis is:
C. Obliterative bronchiolitis
A. amiodarone-induced interstitial lung
D. Asthma
disease
E. Obesity
B. congestive cardiac failure
C. pulmonary embolism
Answer & Comments
D. idiopathic interstitial lung disease
Correct answer: E
E. pulmonary haemorrhage.
The lung function tests show a significant
Answer & Comments restrictive defect. Only obesity or a fibrosing
disease may fit this picture given the
Correct answer: A

Dr. Khalid Yusuf El-Zohry – Sohag Teaching Hospital (01118391123) Page | 592
El-Zohry MRCP Questions Bank (Part 2) – Medical Masterclass 2010

normal/high KCO (i.e. after correcting for transfer are most likely due to a vasculitic
alveolar volumes). process in pulmonary circulation associated
with an autoimmune rheumatic disorder.
The most likely answer is obesity, as the gas
exchange after correcting for the alveolar
volume (TLCO) would tend to be low in [ Q: 140 ] MasterClass Part2
fibrosing alveolitis. Both COPD and obliterative (2010) - Respiratory
bronchiolitis show an obstructive picture but
with low TLCO whereas asthma will also The chest CT is shown from a woman of 50 who
produce an obstructive pattern but with normal underwent lumpectomy, followed by
or high TLCO. radiotherapy to her right breast and axilla for
breast carcinoma. She presents with cough and
breathlessness.
[ Q: 139 ] MasterClass Part2
(2010) - Respiratory
A 56-year-old women, a smoker of 20 cigarettes
daily, presents with a 6-month history of
progressive shortness of breath. Her past
medical history is unremarkable apart from
Raynaud’s syndrome for which she takes a
calcium channel blocker. On examination no
significant abnormality is found apart from
telangiectasia. Her chest radiograph shows
clear lung fields, prominent pulmonary arteries
and mildly enlarged heart. Spirometry is
normal, but gas transfer is reduced to 50%
predicted.
What is the likely diagnosis
What is the most likely diagnosis?
A. Allergic bronchopulmonary aspergillosis
A. Cor pulmonale secondary to chronic (ABPA)
obstructive pulmonary disease
B. Eosinophilic pneumonia
B. Multiple pulmonary emboli
C. Recurrent breast cancer
C. Pulmonary arterial hypertension
D. Radiation pneumonitis
D. Sarcoidosis
E. Bacterial pneumonia.
E. Congestive cardiac failure .
Answer & Comments
Answer & Comments
Correct answer: D
Correct answer: C
The CT shows sharply demarcated consolidation
Normal spirometry excludes chronic obstructive in the right upper lobe in a region within the
pulmonary disease. likely field of radiotherapy that would
Raynaud’s syndrome with telangiectasia and encompass the breast and axilla. The sharp
radiological appearances suggestive of border to the area of abnormality is
pulmonary hypertension with impaired gas characteristic of radiation pneumonitis.

Dr. Khalid Yusuf El-Zohry – Sohag Teaching Hospital (01118391123) Page | 593
El-Zohry MRCP Questions Bank (Part 2) – Medical Masterclass 2010

Radiation can produce organising pneumonia or foul then it is clearly an empyema,


interstitial fibrosis within the lung. which would also be suggested by
pH<7.2
ABPA is characterized by proximal
bronchiectasis. 2. If not associated with pneumonia then
check protein and LDH to distinguish
Eosinophilic pneumonia produces patchy
exudate from transudate using Light’s
peripheral pulmonary infiltrates and is
criteria
characteristically very responsive to steroid
treatment. 3. Cytology
Recurrent breast metastases are unlikely in 4. Microscopy and culture, both aerobic
view of the air bronchograms within the and anaerobic, and for TB.
abnormal area.
Segmental pneumonia is a possible diagnosis [ Q: 142 ] MasterClass Part2
but less likely than radiation pneumonitis. (2010) - Respiratory
A 70-year-old man presented with increasing
[ Q: 141 ] MasterClass Part2 shortness of breath. He had suffered a
(2010) - Respiratory myocardial infarction complicated by
arrhythmias two years earlier. Investigations
A 40-year-old woman, generally fit and well, is
revealed: Ward oximetry - O2 saturation 84%
admitted with malaise and fever. Four weeks
on air; Chest radiograph - Bilateral patchy
previously she had suffered a chest infection for
infiltration of lung fields, cardiothoracic ratio of
which she was given a course of oral antibiotics.
20/31 cm. No evidence of pulmonary oedema.
She felt better initially, but never recovered
fully and is now getting worse. Her chest
Which one of the following drugs that he was
radiograph shows a right-sided pleural effusion.
prescribed might explain these findings?
The most likely diagnosis is: A. Aspirin
A. Connective tissue disorder B. Amiodarone
B. Lung cancer C. Lisinopril
C. Tuberculosis D. Furosemide
D. Pulmonary embolism E. Atorvastatin.
E. Pneumonia with effusion / empyema.
Answer & Comments
Answer & Comments Correct answer: B
Correct answer: E
Amiodarone, an antiarrhythmic drug, can cause
pulmonary toxicity – an acute alveolitis that
Although all of the diagnoses listed could
may or may not be followed by fibrosis. It is
present in this way, pneumonia with effusion /
estimated that approx 6% of patients taking
empyema is much the most likely.
400mg or more per day for 2 months or more
Examination of the pleural fluid is critical: will develop overt pulmonary toxicity. The
1. If associated with pneumonia and the mechanism may include both immunologically
effusion is opaque / turbid and smells mediated and direct toxic effects. Radiographic
appearances are varied: most frequently there

Dr. Khalid Yusuf El-Zohry – Sohag Teaching Hospital (01118391123) Page | 594
El-Zohry MRCP Questions Bank (Part 2) – Medical Masterclass 2010

is a diffuse nodular or alveolar pattern [ Q: 144 ] MasterClass Part2


sometimes with upper lobe predominance. (2010) - Respiratory
Drug cessation will normally be required.
A 25-year-old gentleman with a history of
asthma attends the casualty department with a
[ Q: 143 ] MasterClass Part2
history of acute dyspnoea. He is hypoxic with a
(2010) - Respiratory PaO2 of 7.8 Kpa. The chest radiograph is shown
A 55-year-old man with no significant past (see image).
medical history presents with breathlessness of
gradual onset. He is found to have a
predominantly lymphocytic pleural effusion.

Which TWO of the following diagnoses are NOT


likely to cause a predominantly lymphocytic
effusion?
A. Systemic lupus erythematosus
B. Tuberculosis
C. Lymphoma
D. Sarcoidosis
E. Rheumatoid arthritis
F. Pulmonary embolism
G. Chylothorax
H. Coronary artery bypass grafting (CABG)
I. Drug-induced
J. Oesophageal perforation.
What is the appropriate management?
Answer & Comments A. Anti-coagulation and urgent spiral CT
thorax
Correct answer: FJ
B. Admission to ITU and ventilatory support
Parapneumonic pleural effusions are C. Continuous positive airway pressure
predominantly neutrophilic, as are those (CPAP)
associated with pulmonary embolism, acute
D. High flow oxygen, nebulised
tuberculosis and benign asbestos pleural
bronchodilators and oral steroids
effusion. In oesophageal perforation
neutrophilic effusion with elevated amylase and E. None of the above.
low pH are expected.
CABG often causes a lymphocytic pleural Answer & Comments
effusion, which can usually be treated Correct answer: E
conservatively.

The chest radiograph shows a right


pneumothorax. He needs a needle aspiration

Dr. Khalid Yusuf El-Zohry – Sohag Teaching Hospital (01118391123) Page | 595
El-Zohry MRCP Questions Bank (Part 2) – Medical Masterclass 2010

and may need an intercostal chest drain if J. In the UK, LTOT can only be prescribed by
needle aspiration is not successful. a respiratory physician.

[ Q: 145 ] MasterClass Part2 Answer & Comments


(2010) - Respiratory Correct answer: FH
You are reviewing a 76-year-old man in clinic.
He has been diagnosed as having chronic LTOT for at least 15 hours/day improves
obstructive pulmonary disease (COPD) and survival in patients with severe hypoxemia LTOT
results of arterial blood gases (according to the does not increase survival in patients with mild
last Consultant letter in the case notes) suggest to moderate hypoxemia Nocturnal oxygen
the patient might benefit from home oxygen. therapy does not improve survival in patients
with mild to moderate daytime hypoxemia
Which TWO of the following are correct in Patients should stop smoking before being
relation to domicillary long-term oxygen prescribed LTOT
therapy (LTOT)?
 Nocturnal oxygen therapy does not
A. LTOT for 10 hours/day improves survival improve survial in patients with
in patients with severe hypoxemia nocturnal sleep deprivation
(pO2<8.0kPa).  LTOT should be prescribed for patients
B. LTOT increases survival in patients with with COPD who, when stable, have a
moderate hypoxemia. resting PaO2<7.3kPa or between 7.7 to
8.0kPa plus at least one of the
C. Nocturnal oxygen therapy improves following:-secondary polycythaemia,
survival in mild to moderate daytime nocturnal hypoxemia, peripheral
hypoxemia. odema or pulmonary hypertension
D. Patients should stop smoking after being  LTOT is most economically provided via
prescribed LTOT. an oxygen concentrator, which can be
prescribed on the NHS
E. Nocturnal oxygen therapy improves
 Prescriptions should specify the amount
survival in patients nocturnal sleep
of oxygen and the rate of flow aimed at
desaturation.
keeping PaO2 at rest above 8.0kPa
F. LTOT should be prescribed for COPD  LTOT should be considered if patients
patients who when stable have a resting have PaO2 7.3-8.0kPa if they also have
PaO2<7.3kPa. peripheral oedema
G. LTOT is most economically provided by  In Scotland LTOT can only be prescribed
an oxygen concentrator which cannot be by a Respiratory Physician however in
prescribed on the NHS. England, Wales and Northern Ireland
LTOT can be prescribed by general
H. Prescriptions should specify both the practitioners
amount of oxygen required and a flow
aimed at keeping PaO2 at rest above
8.0kPa. [ Q: 146 ] MasterClass Part2
(2010) - Respiratory
I. LTOT should be prescribed for patients
with peripheral oedema and a PaO2 A 42-year-old asthmatic treated with high dose
of<10kPa. of inhaled corticosteroids developed a skin
rash. His routine blood tests showed mild renal

Dr. Khalid Yusuf El-Zohry – Sohag Teaching Hospital (01118391123) Page | 596
El-Zohry MRCP Questions Bank (Part 2) – Medical Masterclass 2010

dysfunction and blood eosinophilia. His chest What would be the correct course of action?
radiograph is normal.
A. Re-assure and discharge.
Which blood test would you arrange next? B. Arrange a bronchoscopy.
A. Aspergillus fumigates precipitins C. Arrange a CT chest scan.
B. Total Ig E level D. Discharge, but explain that her
pneumonia left her with permanent lung
C. Antineutrophil cytoplasmic antibodies(c-
scarring.
ANCA)
E. Arrange a percutaneous lung biopsy.
D. Anti-nuclear antibody (ANA)
E. Anti GB antibody.
Answer & Comments

Answer & Comments Correct answer: A

Correct answer: C
Right upper lobe consolidation has resolved
completely. A mild increase in shadowing in the
Skin rash and renal dysfunction in an asthmatic left lower lobe is due to a soft tissue breast
suggest vasulitis or Churg–Strauss syndrome. shadow. There are radiological signs of previous
right mastectomy.
[ Q: 147 ] MasterClass Part2
(2010) - Respiratory [ Q: 148 ] MasterClass Part2
This 70-year old woman presented with right
(2010) - Respiratory
upper lobe pneumonia 6 weeks ago and is now A 44-year-old woman who is a lifelong non-
seen in outpatient for a review with a repeated smoker presents with a 5-month history of
chest X-ray that is shown below. progressive exertional shortness of breath. Her
GP diagnosed asthma,
but she failed to respond
to antiasthma
medication, including a
prolonged trial of oral
corticosteroids. Her chest
radiograph shows
hyperinflated lung fields.
Spirometry confirms
irreversible airway
obstruction. A CT chest
scan shows no evidence
of emphysema.

Which is the most likely


diagnosis?
A. Pulmonary
sarcoidosis
B. Usual interstitial

Dr. Khalid Yusuf El-Zohry – Sohag Teaching Hospital (01118391123) Page | 597
El-Zohry MRCP Questions Bank (Part 2) – Medical Masterclass 2010

pneumonia Bronchiectasis may be due to many causes.


Post infection (TB, measles, whooping cough
C. Cryptogenic organising pneumonia
etc), foreign body aspiration, adenomas,
D. Bronchiolitis obliterans hypogammaglobulinaemia, Kartagener's
E. Multiple pulmonary emboli syndrome, Young's syndrome, cystic fibrosis,
allergic bronchopulmonary aspergillosis are the
main causes. Up to 4 % of patients with
Answer & Comments
rheumatoid arthritis develop bronchiectasis.
Correct answer: D
[ Q: 150 ] MasterClass Part2
The absence of emphysema on the CT chest (2010) - Respiratory
scan of a patient who has severe airway
limitation and no clinical manifestation of A 72-year-old female patient has been admitted
asthma may suggest bronchiolitis obliterans. acutely to the ward via the Emergency
Mosaic pattern is usually seen on high- Department. She was found collapsed at home,
resolution CT chest scans carried out during semiconscious. Her Glasgow coma score (GCS)
expiration. Lung biopsy may be required to is currently 10 and she is receiving oxygen via a
establish a diagnosis. venturi mask. You notice that she has stridor
with marked inspiratory effort.
[ Q: 149 ] MasterClass Part2 Which of the following statements regarding
(2010) - Respiratory basic airway management are correct?
A 35-year-old woman known to suffer from A. The length of an oropharyngeal airway
rheumatoid arthritis gives a history of cough should correspond to the length of the
with copious phlegm with intermittent patient's little finger.
haemoptysis. She also gets recurrent chest B. Oropharyngeal airways are
infections. She is on weekly methotrexate along contraindicated in patients with a base of
with a non-steroidal anti-inflammatory drug
skull fracture.
(NSAID). Her chest radiograph shows linear
radioluciences at both bases. A high-resolution C. Patients with preserved laryngeal reflexes
computed tomography scan (HRCT) of the chest will not tolerate a nasopharyngeal tube.
confirms bronchiectasis. D. Insertion of an oropharyngeal airway can
trigger laryngospasm.
What is the most likely cause?
E. A nasopharyngeal airway cannot be used
A. Foreign body aspiration
if the nose is fractured.
B. Kartagener's syndrome
F. Both nasopharyngeal and oropharyngeal
C. Old tuberculosis airways can provide a definitive airway
for comatose patients.
D. Rheumatoid arthritis
G. Nasopharyngeal and oropharyngeal
E. Cystic fibrosis.
airways should be used when a jaw thrust
is contraindicated.
Answer & Comments
H. The diameter of a nasopharyngeal tube
Correct answer: D should be similar to the diameter of the
patient's little finger.

Dr. Khalid Yusuf El-Zohry – Sohag Teaching Hospital (01118391123) Page | 598
El-Zohry MRCP Questions Bank (Part 2) – Medical Masterclass 2010

I. A head-tilt chin lift produces less neck C. Calcium tablets


movement than a jaw thrust manoeuvre.
D. Prednisolone
J. Vomitus should be removed from the
E. Heroin
airway using a blind finger-sweep.
F. Insulin
Answer & Comments G. Cyclophosphamide

Correct answer: DH H. Busulfan


I. Sulfasalazine
Basic airway management involves initially
J. Paracetamol.
opening the airway. This should be done with a
head-tilt chin lift unless there is suggestion of a
neck injury when a jaw-thrust manoeuvre is Answer & Comments
preferred. Material in the oropharynx should be Correct answer: AI
removed under direct vision. Oropharyngeal
(OPA) and nasopharyngeal (NPA) airways are
Drug-induced pulmonary eosinophilia is the
useful adjuncts but do not provide a definitive
most common type of pulmonary eosinophilia
airway for unconscious patients.
seen in the western world. Drugs that can be
A quick way to size a NPA is to choose one with responsible include ampicillin, aspirin, captopril,
an external diameter similar to the patient’s bleomycin, carbamazepine, dapsone,
little finger. NPAs are contraindicated in base of ethambutol, gold, methotrexate, penicillin,
skull fractures. If there is a nasal fracture, the penicillamine, sulphonamides (including
most patent nostril should be chosen. NPAs are sulfasalazine), tamoxifen and tetracycline.
generally better tolerated than OPAs and can
be used in patients where the laryngeal reflexes
[ Q: 152 ] MasterClass Part2
are preserved.
(2010) - Respiratory
OPAs should be sized with the length
corresponding to the distance from the angle of A 43-year-old woman has been diagnosed as
the patient's mouth to the angle of the jaw. suffering from bronchiectasis on a high-
Inserting OPAs can trigger vomiting and resolution computed tomography (HRCT) scan
laryngospasm. of the lung.

Which one of the following statement is NOT


[ Q: 151 ] MasterClass Part2 true?
(2010) - Respiratory A. Her immunoglobulin levels should be
A 36-year-old woman is admitted with a history checked.
of cough and shortness of breath of 8 weeks' B. She is at risk of developing a
duration. Her chest radiograph shows bilateral pneumothorax.
mid-zone infiltrates and blood count shows a
C. Massive haemoptysis is the commonest
raised eosinophil count.
cause of death in her age group.
Which TWO of the following drugs might be D. She is at risk of developing brain abscess .
responsible for her condition?
E. Recurrent chest infections are unlikely at
A. Bleomycin this age.
B. Amiodarone

Dr. Khalid Yusuf El-Zohry – Sohag Teaching Hospital (01118391123) Page | 599
El-Zohry MRCP Questions Bank (Part 2) – Medical Masterclass 2010

Answer & Comments M. avium complex (MAC) is a saprophyte and


can be found in soil, water, dust etc. It typically
Correct answer: E
affects middle age and old men with underlying
lung diseases such as COPD, old TB,
Congenital and acquired immunodeficiency are bronchiectasis. It may be contaminant, hence to
well known causes of bronchiectasis, hence make a diagnosis there should be radiographic
serum immunoglobulins should be checked, evidence of disease (cavities etc), and it should
particularly the IgG subclass (IgG1,2,3 & 4). be isolated in at least three sputum samples.
Patients with immunoglobulin deficiency should
be treated with regular immunoglobulin Extrapulmonary disease is seen in
infusion. immunosupressed patients. Recommended
treatment is with rifampicin and ethambutol for
Pneumothorax, recurrent chest infections and 24 months.
brain abscess are well recognised
complications. In fit patients, surgery resection of the diseased
lung along with chemotherapy can cure.
Haemoptysis is a common symptom but rarely
causes death.
[ Q: 154 ] MasterClass Part2
(2010) - Respiratory
[ Q: 153 ] MasterClass Part2
(2010) - Respiratory A 67-year-old bus driver attends the chest clinic
with his wife. He feels quite fine, but his wife
A 60-year-old gentleman is referred to the says that he snores at night and has numerous
hospital as a routine chest radiograph has daytime naps. He has hypertension despite
shown bilateral apical scars with possibility of taking atenolol 100 mg od,
cavity formation. He is complaining of some bendroflumethiazide 2.5 mg od and amlodipine
cough for over 8 months and is known to suffer 5 mg od. He has never smoked and his BMI is
from mild chronic obstructive pulmonary 39. His screening CXR and lung function tests
disease (COPD). There is no history of are normal. On further questioning his wife
immunosuppression and repeated sputum mentions that he makes choking and gurgling
samples have grown Mycobacterium avium sounds at night.
complex.
Which of the following would be the best
Which of the following is true? screening tool to apply to this man?
A. Treatment should be the same as A. Short form 36 questionnaire
Mycobacterium tuberculosis but with an
B. Hospital anxiety and depression score
8-month continuation phase using
questionnaire
isoniazid and rifampicin.
C. Epworth Sleepiness Scale
B. Patient should be notified within 1 week.
D. Medical Research Council dyspnoea scale
C. Extrapulmonary disease is uncommon in
questionnaire
immunocompetent patients.
E. St George's respiratory questionnaire
D. Surgery has no role in treatment.
E. It commonly affects young fit men.
Answer & Comments

Answer & Comments Correct answer: C

Correct answer: C
Dr. Khalid Yusuf El-Zohry – Sohag Teaching Hospital (01118391123) Page | 600
El-Zohry MRCP Questions Bank (Part 2) – Medical Masterclass 2010

The Epworth Sleepiness Scale is a validated tool [ Q: 156 ] MasterClass Part2


for screening for obstructive sleep apnoea (2010) - Respiratory
syndrome. It comprises seven questions scoring
a maximum of 3 points each. A total score of A 64-year-old woman with newly-diagnosed
over 11 in an appropriate clinical scenario is small cell carcinoma discusses her further
highly supportive of the diagnosis. treatment options with the respiratory
consultant, the cardiothoracic surgeon and the
local oncologist.
[ Q: 155 ] MasterClass Part2
(2010) - Respiratory Which of the following statements are correct?
A 55-year-old gentleman with a smoking history A. Patients with small cell lung cancer
of 50 pack years has been diagnosed to be always benefit from surgery.
suffering from a lung mass based on a chest
B. Small cell carcinoma is rarely responsive
radiograph. He is presently waiting a
to chemotherapy.
bronchoscopy. You predict that this is a non-
small cell lung cancer. C. Radiotherapy has no role to play in the
treatment of small cell lung cancer.
Which one of the following does not support
D. Management of small cell carcinoma
your prediction?
should include combined
A. Hypertrophic pulmonary chemotherapy/radiotherapy.
osteoarthropathy (HPOA)
E. Brain metastases are uncommon in
B. Hypercalcaemia with a normal patients with small cell lung cancer.
radioisotope bone scan
C. Syndrome of inappropriate antidiuretic Answer & Comments
hormone secretion (SIADH)
Correct answer: D
D. Disseminated intravascular coagulation
(DIC) Because small cell lung cancer has a propensity
E. Thrombocytosis. to spread early, surgical resection is not usually
considered as part of the routine treatment.
Patients diagnosed by biopsy are treated by
Answer & Comments
combined radiotherapy and chemotherapy.
Correct answer: C Concurrent radiotherapy and chemotherapy
provides better 5-year survival rates than
Paraneoplastic syndromes are commonly seen sequential therapy. Patients are at risk of brain
with lung cancer. SIADH is most comonly seen metastases and one study suggested a 50%
with small cell carcinoma. HPOA, incidence 2 years after diagnosis.
hypercalcaemia without bone metastasis is
more common in squamous cell carcinoma. DIC [ Q: 157 ] MasterClass Part2
and thrombocytosis are more common with (2010) - Respiratory
adenocarcinoma.
An 18-year-old woman has asthma for which
she is using her salbutamol inhaler two or three
times a day.

Dr. Khalid Yusuf El-Zohry – Sohag Teaching Hospital (01118391123) Page | 601
El-Zohry MRCP Questions Bank (Part 2) – Medical Masterclass 2010

What should be the next step in her [ Q: 158 ] MasterClass Part2


management? (2010) - Respiratory
A. Add regular long-acting inhaled beta
A 35-year-old woman is referred with a history
agonist
of red, painful legs of 3 weeks duration that
B. Advise her to use the salbutamol inhaler have not responded to a course of amoxycillin
regularly three times a day and flucloxacillin given for presumed cellulitis.
C. Add regular inhaled steroid, e.g. She is afebrile, does not have any other
beclometasone 100 microg twice daily symptoms, and has never smoked. Examination
reveals tender purple / red nodules on her
D. Add regular inhaled steroid, e.g. shins. Her full blood count, kidney and liver
beclometasone 1000 microg twice daily, function tests are normal. A chest radiograph
plus regular long-acting inhaled beta shows prominent hilae.
agonist
E. Add regular inhaled steroid, e.g. What is the appropriate management?
beclometasone 1000 microg twice daily. A. Arrange bronchoscopy and
bronchoalveolar lavage to exclude
Answer & Comments malignancy

Correct answer: C B. Start azathioprine and prednisolone;


follow up in clinic
The British asthma guidelines for the stepwise C. Start prednisolone; follow up in clinic
management of chronic asthma in adults are as D. Start simple analgesics; follow up in clinic
follows:
E. Arrange CT scan of the lungs and lung
Step 1: occasional use of inhaled short-acting biopsy
beta agonists
Step 2: regular inhaled anti-inflammatory Answer & Comments
agent, e.g. standard dose inhaled steroid
Correct answer: D
Step 3: high-dose inhaled steroids, or low-dose
inhaled steroids plus long-acting beta agonist
The combination of bilateral hilar
Step 4: high-dose inhaled steroid and regular lymphadenopathy and erythema nodosum is
bronchodilators (sequential therapeutic trial of diagnostic of sarcoidosis. This is usually self-
inhaled long-acting beta agonist, sustained limiting. She should however be followed up in
release theophylline, inhaled ipatropium / clinic with full lung function tests including
oxitropium, oral long-acting beta agonist, high- transfer factor and lung volumes. Serum
dose inhaled bronchodilators, cromoglycate / angiotensin-converting enzyme (ACE) level and
nedocromil) lung functions can be used to monitor disease.
Step 5: addition of regular steroid tablets Worsening disease should be treated with
prednisolone.
This woman’s treatment needs to be escalated
from Step 1 to Step 2.
[ Q: 159 ] MasterClass Part2
(2010) - Respiratory
You follow up a 55-year-old male smoker in
your clinic with stable emphysema. His FEV1 is

Dr. Khalid Yusuf El-Zohry – Sohag Teaching Hospital (01118391123) Page | 602
El-Zohry MRCP Questions Bank (Part 2) – Medical Masterclass 2010

0.6 (22% predicted) and FVC is 2.3 (63% rehabilitation courses offer smoking cessation
predicted) giving an FEV1:FVC ratio of 26%. His therapy.
arterial blood gases show a PO2 of 7.1 and 7.2
There is no evidence to suggest annual chest
on two separate occasions. He is mildly
radiographs will benefit any group of patients in
breathless at rest, but severely breathless on
terms of primary prevention.
exertion and his saturation drops from 91% to
88%.
[ Q: 160 ] MasterClass Part2
Which of the following statements is true? (2010) - Respiratory
A. The major cause for his breathlessness is A 55-year-old woman attends the chest clinic
hyperinflation not hypoxia. with a history of dry nocturnal cough for over 6
B. He has severe emphysema because his months. She has never smoked and is a retired
FEV1:FVC ratio is less than 40%. hotel receptionist. There is no history of
haemoptysis, wheeze or weight loss. There are
C. There is good evidence to suggest that he
no nasal symptoms. She suffers from
will benefit from long-term oxygen
hypertension and is on perindopril 4 mg and
therapy.
bendroflumethiazide 2.5 mg daily. Examination
D. He should not be considered for is normal.
pulmonary rehabilitation because he is a
current smoker. Which of the following is most likely to be
responsible for her symptoms?
E. He should have a yearly chest radiograph
to look for lung cancer. A. Bronchogenic carcinoma
B. Cryptogenic pulmonary fibrosis
Answer & Comments
C. Drug-induced cough
Correct answer: A D. Late-onset asthma
E. Bronchiectasis
Although hypoxic at rest, this is mild and not
causing significant dyspnoea. His breathlessness
worsens considerably without much change in Answer & Comments
oxygen tension, suggesting that the cause of his Correct answer: C
dyspnoea is hyperinflation of his chest, which
worsens on exertion.
A chronic cough is defined as one persisting for
Severity of emphysema is defined by the British at least 8 weeks. In approximately 90% of cases
Thoracic Society (BTS) in relation to FEV1, not presenting to secondary care the cause is one
FEV1:FVC ratio. Mild is 60-80% predicted; of asthma, rhinitis/sinusitis or gastro-
moderate 40-60% and severe <40%. oesophageal reflux disease. However, in this
Evidence for long-term oxygen therapy group of patients the GP has usually considered
(Medical Research Council and Nocturnal any drug-related causes prior to referral; in this
Oxygen Therapy Trials) is only available in non- woman it is likely that the angiotensin-
smokers. Therefore, first, he may not benefit, converting enzyme inhibitor is responsible, and
and second he is at risk of burns if he carries on should be stopped.
smoking. He will benefit from pulmonary
rehabilitation. Furthermore, some

Dr. Khalid Yusuf El-Zohry – Sohag Teaching Hospital (01118391123) Page | 603
El-Zohry MRCP Questions Bank (Part 2) – Medical Masterclass 2010

is very likely that the carotid stenosis is the

Neurology source of the emboli.

[ Q: 2 ] MasterClass Part2
(154 Questions) (2010) - Neurology
A 70-year-old man presents with left-sided
(Medical Masterclass – Part 2) hemiparesis associated with a gaze paralysis to
the right.

Which part of the brain is damaged?


[ Q: 1 ] MasterClass Part2
A. Left frontal lobe
(2010) - Neurology
B. Left pons
You are referred a 74-year-old woman who had
multiple episodes of unilateral transient loss of C. Right frontal lobe
vision in the right eye described as ‘a curtain D. Right medulla oblongata
coming down’ lasting for several minutes. She is
E. Right pons.
a heavy smoker and has hypertension. She is on
aspirin, perindopril, indapamide and
simvastatin. She is found to have atrial Answer & Comments
fibrillation with an apical rate of 78 beats per Correct answer: E
minute. Investigations reveal a 90% ipsilateral
carotid stenosis and 35% contralateral disease
The lateral gaze centre is situated in the pons. A
and a normal transthoracic echocardiogram.
lesion in this area will cause impaired conjugate
What is the most appropriate course of action? gaze to the side of the lesion, with consequent
deviation away from the side of the lesion.
A. Add clopidogrel to aspirin
B. Anticoagulation with heparin, then [ Q: 3 ] MasterClass Part2
warfarin
(2010) - Neurology
C. Continue present medication without
change A 37-year-old man presents with foot drop.

D. Referral for consideration of carotid In the assessment of this patient:


endarterectomy, with delayed post-
A. assessment of foot inversion and
operative warfarin
eversion is critical
E. Start prednisolone 60mg and arrange
B. assessment of hip flexion tests the L5
temporal artery biopsy.
nerve root

Answer & Comments C. brisk reflexes are an incidential finding


D. compression of the obturator nerve
Correct answer: D
should be considered

If carotid stenosis is greater than 70%, caratid E. pain to the anterior thigh is suggestive of
endartectomy confers a beneficial effect and L4 compression
should be considered. Emboli from fibrillating F. lumbar plexopathy is a common cause
hearts tends to cause large strokes, therefore it

Dr. Khalid Yusuf El-Zohry – Sohag Teaching Hospital (01118391123) Page | 605
El-Zohry MRCP Questions Bank (Part 2) – Medical Masterclass 2010

G. common peroneal palsy is associated B. Some forms of migraine are familial.


with numbness over the lateral aspect of
C. The presence of aura, e.g. visual
the leg and dorsum of the foot
symptoms, is required for the diagnosis.
H. ankle jerk should be absent
D. Lifestyle modification is not helpful.
I. marked weakness is more suggestive of a
E. The pain does not respond to non-
lumber radiculopathy
steroidal anti-inflammatory drugs
J. sciatica nerve lesions most commonly lead (NSAIDs).
to equal weakness of muscle supplied by
F. Ergot derivatives have a favourable side
the peroneal and tibial nerve.
effect profile.
G. The 'Triptans', e.g. sumatriptan, are 5-HT
Answer & Comments
receptor antagonists.
Correct answer: AG
H. Triptans should be avoided in ischaemic
heart disease.
Assessment of foot eversion and inversion
helps distinguish a common peroneal palsy and I. Pizotifen is used to treat acute attacks.
multi-level radiculopathy. J. There are no effective preventative
Hip flexion is test of the L1/2 root. treatments.

Brisk reflexes suggest upper motor neurones


Answer & Comments
(UMN) which can cause foot drop.
Correct answer: BH
Obturator nerve supplies the hip adductors.
L4 compression is associated with pain in the
Migraine can start at any age, but incidence
medial aspect of the lower leg.
peaks in adolescence.
Brachial and lumbar/sacral plexopathy are rare.
In familial hemiplegic migraine there is
The ankle jerk will be present in L5 lesions. dysfunction of ion channels due to mutations.
Lumbar radiculopathy can present with little Classic migraine is preceded or accompanied by
weakness because of the dual innervation of aura. However, most patients have migraine
most muscles in the lower leg. without aura (previously known as common
migraine).
[ Q: 4 ] MasterClass Part2 Migraine is characterized by the presence of at
(2010) - Neurology least some of a number of associated features,
e.g. nausea or vomiting, aggravation on
A 70-year-old man complains of episodic severe movement, sensitivity to light or sound.
throbbing headaches, worse on movement and
associated with nausea. He has had one or two Non-pharmacological measures such as
episodes per month for the last year, lasting avoidance of precipitants and regular sleep and
between 4 to 24 hours. You diagnose migraine. meals might help. Dry therapy can be divided
into those used to treat acute attacks and those
Which TWO statements are true about this taken daily to reduce the frequency and
condition? severity of attacks. Aspirin, paracetamol and
NSAIDs may be useful with an anti-emetic for
A. Migraine may start at any age, but
acute attacks. Ergot derivatives have a long
incidence peaks in old age.
history of use for acute attacks; however, there

Dr. Khalid Yusuf El-Zohry – Sohag Teaching Hospital (01118391123) Page | 606
El-Zohry MRCP Questions Bank (Part 2) – Medical Masterclass 2010

is a significant risk of adverse vascular events. Answer & Comments


The 'Triptans', e.g. sumatriptan, are 5-HT
Correct answer: C
receptor AGONISTS also used to treat acute
attacks. They have a much better safety record
then ergot derivatives. However, they may The left-sided weakness is due to right cerebral
cause vasoconstriction and are therefore not hemisphere dysfunction, and with the sudden
recommended in patients with ischaemic heart onset is likely to represent a transient ischaemic
disease and cerebrovascular disease. attack.

In those who have very frequent attacks, very The combination of these two symptoms
disabling attacks, or little response to acute strongly suggests a right internal carotid artery
medications, preventative therapy should be stenosis. Cardioembolism is unlikely in the
considered. Pizotifen is an example of absence of either atrial fibrillation or previous
preventative therapy which can be effective for cardiac symptoms of either cardiac ischaemia
some people. or cardiac failure. Transient occlusion of a small
penetrating vessel i.e. a lacunar syndrome
could cause the hemiparesis but not the
[ Q: 5 ] MasterClass Part2 amaurosis fugax. Giant cell arteritis should
(2010) - Neurology always be considered in patients over 6o years
You are asked to give a medical opinion on a old, but there are no specific features to
67-year-old man who attends A&E after support the diagnosis, although it is always
experiencing sudden onset left-sided weakness worthwhile checking the erythrocyte
lasting for 20 minutes. Three days previously he sedimentation rate (ESR). Lastly, migraine
had suffered an episode of blurred vision in his equivalents (aura-like symptoms without
right eye that lasted for 5 minutes and was headache) can provide diagnostic difficulties
associated with a headache lasting for an hour. but there is no suggestion of the characteristic
Previously he had been seeing his GP for slow spread of symptoms in this patient.
elevated blood pressure and for help in
stopping smoking. Neurological examination [ Q: 6 ] MasterClass Part2
was normal. His blood pressure was 150/90 (2010) - Neurology
mmHg, heart rate 84 and regular. There were
no audible carotid bruits. A patient has: 1. Wide based ataxic gait 2.
Relatively normal heel-knee shin coordination
Which of the following is the most likely whilst lying down 3. Normal arm coordination
diagnosis? 4. Normal eye movements without nystagmus
A. Transient ischaemic attack secondary to
Where is the lesion most likely to be located?
giant cell arteritis
A. Midline pons
B. Migraine equivalent
B. Intrinsic upper cervical cord
C. Transient ischaemic attack secondary to
carotid artery disease C. Caudal cerebellar vermis

D. Transient ischaemic attack secondary to D. Rostral cerebellar vermis


small vessel disease E. Periventricular white matter
E. Transient ischaemic attack secondary to
cardioembolism. Answer & Comments

Correct answer: C

Dr. Khalid Yusuf El-Zohry – Sohag Teaching Hospital (01118391123) Page | 607
El-Zohry MRCP Questions Bank (Part 2) – Medical Masterclass 2010

These are characteristic of damage limited to A. Headache is no more likely to occur if a


the anterior superior part of the cerebellar large amount of CSF is taken
vermis, most characteristically seen with early
B. The headache is a typical low-pressure
alcohol induced cerebellar degeneration.
headache
C. Inserting the needle with the bevel
[ Q: 7 ] MasterClass Part2
horizontal is less likely to cause headache
(2010) - Neurology
D. Prolonged bed rest following the
A 19-year-old woman presents with optic procedure reduces the incidence of
neuritis and the T2-weighted brain MRI headache
demonstrates multiple high signal lesions in the
E. Headache occurs after around 10% of
white matter.
lumbar punctures
The risk of a further episode of neurogical
dysfunction is: Answer & Comments
A. 75% Correct answer: D
B. 10%
Post LP headache occurs to some degree in
C. 50%
around 10% of cases. It is not related to the
D. 25% amount of CSF taken, but is more likely to be
E. 95%. due to continuous leak of CSF from the hole
made in the dura. Hence the incidence of post
LP headache is related to the size of the hole
Answer & Comments
and a larger gauge needle, or one with the
Correct answer: A bevel inserted vertically (which slices through
dural fibres) are associated with an increased
The risk of a further episode of neurogical risk of headache. The headache is low pressure,
dysfunction, i.e. developing multiple sclerosis, is being worse on standing and ameliorated by
moderately high after the occurrence of optic lying down. There is no need to rest for more
neuritis in the context of multiple white matter than 30 minutes after the procedure since
lesions. prolonged bed rest has not been shown to
reduce the incidence of headache.

[ Q: 8 ] MasterClass Part2
[ Q: 9 ] MasterClass Part2
(2010) - Neurology
(2010) - Neurology
A 30-year-old woman with idiopathic
intracranial hypertension is admitted for a A 43 -year-old man complains of sensory loss in
lumbar puncture. The opening pressure is his left arm and hand. On examination he has
measured at 40cm of water and 50mls of subjectively diminished light touch and pinprick
cerebrospinal fluid are collected. Six hours later sensation in the left hand extending to above
the patient complains of headache. the elbow. Joint position sense appears intact.
He has difficulty distinguishing his lighter from a
Which of the following statements concerning pen using the left hand (with eyes closed), and
post lumbar puncture headaches is NOT his two-point discrimination is 11 mm.
correct?
What will your first investigation be?

Dr. Khalid Yusuf El-Zohry – Sohag Teaching Hospital (01118391123) Page | 608
El-Zohry MRCP Questions Bank (Part 2) – Medical Masterclass 2010

A. CT head commonest causes of dementia, although all of


the conditions listed can cause dementia.
B. Electromyogram (EMG) and nerve
conduction studies Gradual progression of symptoms would be
typical of Alzheimer’s, as would an absence of
C. MRI cervical spine
so-called ‘subcortical’ features (where the
D. Chest radiograph patient is slow with disturbed attention and
E. Vitamin B12 levels. motivation). There may be a family history of
dementia in some cases.
Answer & Comments The presence of history, symptoms or signs of
vascular disease (or its risk factors) would all
Correct answer: A
increase the likelihood of a multi-infarct
dementia, as would step-wise progression.
The sensory signs described here are cortical
Dementia with Lewy bodies often presents with
sensory signs, and are indicative of a lesion of
fluctuating cognitive impairment affecting both
the anterior parietal cortex, particularly mid
memory and higher cortical function. The
postcentral gyrus.
presence of delusions, visual hallucinations,
abnormal sleep pattern and motor features of
[ Q: 10 ] MasterClass Part2 parkinsonism would all support this diagnosis.
(2010) - Neurology
A 57-year-old teacher presents with a 6-month [ Q: 11 ] MasterClass Part2
history of increasing forgetfulness. (2010) - Neurology

The TWO most likely diagnoses are: A 34-year-old woman presents with a 1-year
history of headaches. The pain was initially
A. Alcohol-related dementia
episodic, lasting for a few hours, but is now
B. Multi-infarct dementia present everyday, particularly on waking and
only lessens slightly towards the evening. She
C. Parkinson’s disease
describes the pain as tight, generalised and
D. Cerebral tumour constant, worsening in bright light. There is no
E. Huntington’s disease associated nausea. The results of her
examination are normal.
F. Hypothyroidism
G. AIDS dementia complex She needs to take at least TWO codydramol
tablets per day.
H. Chronic subdural haematoma
A. Cervicogenic headache
I. Hydrocephalus
B. Spontaneous intracranial hypotension
J. Alzheimer’s disease.
C. Analgesic-induced chronic headache
Answer & Comments D. Idiopathic intracranial hypertension

Correct answer: BJ E. Temperomandibular joint dysfunction.

Alzheimer’s disease, multi-infarct dementia and Answer & Comments


dementia with Lewy bodies are the three Correct answer: C

Dr. Khalid Yusuf El-Zohry – Sohag Teaching Hospital (01118391123) Page | 609
El-Zohry MRCP Questions Bank (Part 2) – Medical Masterclass 2010

Although it is not clear what initiated this inhibitors, HIV non-nucleoside reverse
patient’s headaches, it is likely that the use of a transcriptase inhibitors, the
simple analgesic, particularly one containing an immunosuppressants ciclosporin and
opiate, is causing them to persist. Cervicogenic tacrolimus, and amitriptyline.
headache is possible, but there is no history of
neck pain or trauma. Intracranial hypertension [ Q: 13 ] MasterClass Part2
is less likely (no specific features of raised
(2010) - Neurology
intracranial pressure, no papilloedema), and
intracranial hypotension is characteristically A 46-year-old man presents with a headache
postural (worsening on standing, relieved by over the left eye spreading across the forehead.
lying down). There are no symptoms to support It started suddenly and built up over three
a diagnosis of temperomandibular joint hours. He smokes 25 a day. On examination, he
dysfunction, an uncommon cause of has a mild left ptosis and a small reactive left
generalised headache. pupil. There are no other abnormal signs.

What is the most likely diagnosis?


[ Q: 12 ] MasterClass Part2
(2010) - Neurology A. Basilar artery aneurysm
B. Internal carotid artery dissection
A 38-year-old woman with a long history of
migraine is receiving prophylactic treatment C. Migraine
with amitriptyline 100 mg at night. She is
D. Subarachnoid haemorrhage
referred to the outpatient clinic with worsening
migraine control despite good compliance with E. Vertebral artery dissection.
her medication. Her GP had recently started
another medication for an unrelated condition. Answer & Comments

Which one of the following may cause a Correct answer: B


reduction in plasma amitriptyline levels when
co-prescribed? The internal carotid artery dissection caused
A. Warfarin the left Horner’s syndrome.

B. Sodium valproate
[ Q: 14 ] MasterClass Part2
C. Fluoxetine (2010) - Neurology
D. St John's wort
You are asked to see a 53-year-old publican
E. Trimethoprim who is brought to the Emergency
Department after being found slumped in her
Answer & Comments chair. She reports that she is drowsy because
she has been assaulted, although those that
Correct answer: D have brought her do not think this is possible.
Her GCS is 13/15. Her pupils and eye
St John's wort is proven to be effective in mild
movements are normal. She has no focal
to moderate depression, but it induces the
neurological deficits. There is no evidence of
cytochrome P450 3A4 enzyme system and the
trauma to her face or head. She has had an
P-glycoprotein transporter and thereby reduces
emergency CT brain scan that has been verbally
the plasma levels of drugs metabolised via
these pathways. These include HIV protease

Dr. Khalid Yusuf El-Zohry – Sohag Teaching Hospital (01118391123) Page | 610
El-Zohry MRCP Questions Bank (Part 2) – Medical Masterclass 2010

reported as normal. She has a raised MCV, ALP Answer & Comments
and low platelets.
Correct answer: C
The most likely diagnosis is:
C5/C6 supplies the following muscles: deltoid,
A. Wernike's encephalopathy
biceps, brachioradialis, supra and infra spinati,
B. Subdural haematoma pectoralis major (clavicular head), serratus
C. Korsakoff's psychosis anterior (causes winging of scapula if weak),
extensor carpi radialis longus.
D. Alcohol withdrawal
Both biceps and brachioradialis reflexes may be
E. Postictal drowsiness. affected in a C5/C6 lesion
Sensory loss can be variable but typically
Answer & Comments
involves the lateral (radial) aspect of the upper
Correct answer: C and lower arm including the thumb. C8 typically
supplies the ring and little finger.
This woman is clearly at high risk of alcohol-
related disease as she has a high-risk profession [ Q: 16 ] MasterClass Part2
and abnormal blood tests. (2010) - Neurology
The fact that she seems to be confabulating
A 39-year-old keen camper presents with a
makes Korsakoff's the best diagnosis. She has
two-week history of headaches, a 'droopy face',
no evidence of nystagmus or ataxia suggestive
joint pains and a low grade temperature. He
of Wernicke's encephalopathy. Subdural
had returned from a holiday in the USA 3
haematoma is usually easily diagnosed by CT
months earlier. The only abnormality on
scan. Alcohol withdrawal usually occurs 48-72
neurological examination is shown (see image).
hours after the last alcoholic drink (and we do
not have any timings here).
Do not forget to exclude hypoglycaemia,
hyponatraemia and hypomagnesaemia as
causes of fits in alcoholics.

[ Q: 15 ] MasterClass Part2
(2010) - Neurology

A 65-year-old man presents with pain and


weakness in his left arm. The following features
are consistent with a C5/C6 radiculopathy (root
problem) except:
A. weakness of deltoid muscle
B. reduced biceps reflex
C. sensory loss over the ring and index
finger
D. weakness of supraspinatus muscle
E. winging of the scapula.

Dr. Khalid Yusuf El-Zohry – Sohag Teaching Hospital (01118391123) Page | 611
El-Zohry MRCP Questions Bank (Part 2) – Medical Masterclass 2010

A lumbar puncture was carried out and the Paraneoplastic conditions can occur up to
results of the CSF analysis are as follows: 200 several years before the emergence of the
cells/µl, over 95% lymphocytes, protein 1.4 g/l, underlying tumour. The pathophysiological
glucose 2.0 mmol/l (serum glucose 8.0 mmol/l). basis is thought to be autoimmune. A wide
variety of clinical syndromes have been
What treatment would you institute in this described affecting central structures, such as
patient? the mesial temporal lobes or spinal cord, or
A. Chemotherapy agents peripheral elements including nerve and
muscle.
B. Intravenous acyclovir
C. Intravenous ceftriaxone [ Q: 18 ] MasterClass Part2
D. Intravenous gentamicin (2010) - Neurology
E. Oral rifampicin, isoniazid, pyrazinamide A 30-year-old female patient presents with a 5-
and ethambutol. day history of ascending sensorimotor deficit of
all four limbs. You suspect the diagnosis of
Answer & Comments Guillain–Barré syndrome (GBS).
Correct answer: C Which one of the following responses is true:
A. In 90% of individuals there is a history of
The treatment of choice for meningitis of Lyme
a preceding upper respiratory or
disease is intravenous penicillin or ceftriaxone
gastrointestinal infection
B. Hyperreflexia is typical
[ Q: 17 ] MasterClass Part2
(2010) - Neurology C. Autonomic dysfunction is rare
D. The cerebrospinal fluid is typically normal
A 60-year-old patient was admitted with
throughout the course of the illness
fluctuating confusion, inappropriate laughter,
and seizures. Small-cell lung carcinoma is E. Treatment with intravenous
subsequently identified. You suspect the immunoglobulin is as effective as plasma
presenting symptomatology is due to exchange
paraneoplastic limbic encephalitis.
Answer & Comments
The following are paraneoplastic conditions
except: Correct answer: E
A. Encephalomyelitis
GBS affects people of all ages. Approximately
B. Cerebellar degeneration
60–70% of patients have a preceding infective
C. Retinopathy illness approximately 1–4 weeks before onset
of neurological deficit. These include: CMV,
D. Lambert-Eaton syndrome
EBV, viral hepatitis, HIV, mycoplasma and most
E. Parkinsonism commonly, Campylobacter jejuni. Initial
features may be sensory or motor deficit or
Answer & Comments weakness alone. Hyporeflexia is an invariable
feature. Assisted ventilation is required in up to
Correct answer: E
20% of patients. Autonomic dysfunction is

Dr. Khalid Yusuf El-Zohry – Sohag Teaching Hospital (01118391123) Page | 612
El-Zohry MRCP Questions Bank (Part 2) – Medical Masterclass 2010

common, therefore patients should be placed Answer & Comments


on a cardiac monitor. Steroids are ineffective.
Correct answer: BJ

[ Q: 19 ] MasterClass Part2 Benign (a misnomer since it can be disabling )


(2010) - Neurology essential tremor (BET) predominantly affects
A 72-year-old man is referred with suspected those over the age of 70. The tremor is
Parkinson’s disease. He complains of shaking of pathological, unlike the physiological tremor we
his hands which is worse when he tries to do all have which may become noticeable in
things such as drinking a cup of tea, which has stressful situations. It is an ACTION tremor
got progressively worse over the last 5 years. which occurs during voluntary motion and
He comments that his father had a similar when the arms are outstretched. An intention
condition. He is otherwise well and has a tremor is coarse and occurs as a limb
normal posture and gait. You diagnose benign approaches a target. It is a sign of cerebellar
essential tremor. disease. It is important to take a drug history
since several medications may enhance
Which TWO statements about this condition are physiological tremor and lead to confusion.
correct? Wilson’s disease is rare. There is a rest and
A. The tremor is an intention tremor. action tremor. There are usually other
neurological abnormalities. The serum
B. Drugs such as sodium valproate and
caeruloplasmin is LOW.
lithium may produce a similar tremor.
The familial form (30-50% cases) of BET has an
C. In young people, Wilson's disease,
autosomal DOMINANT form of inheritance.
indicated by a high serum
caeruloplasmin, should be excluded. The diagnosis is indeed a clinical one and there
are no validated laboratory markers, but at
D. Many cases are familial with autosomal
least thyroid function tests should be done to
recessive inheritance.
exclude hyperthyroidism which may present
E. This is a clinical diagnosis and no baseline with a sustained enhanced physiological
investigation is required. tremor.
F. Medication should always be offered Medication should only be used for patients
since it may halt progression of the who are disabled and /or embarrassed by their
condition. tremor. Primidone is often effective, but there
G. Primidone is well tolerated initially but is a high incidence of nausea and vomiting
has a high rate of long-term when initiating treatment, requiring
complications. discontinuation in up to one fifth of patients.
Propranol has similar efficacy, but although
H. Gabapentin, a GABA-antagonist, is a better tolerated initially may be less well
useful second-line agent. tolerated long term. Gabapentin is structurally
I. Over 90% of patients will have marked similar to GABA (i.e. not an inhibitor ). There is
improvement with pharmacotherapy some evidence of disturbance of the GABA –
therapy. ergic system in essential tremor and it can be
an effective alternative treatment.
J. Thalamic stimulation may be
recommended in those with disabling Unfortunately a large proportion of patients do
tremor unresponsive to medication. not respond to pharmacotherapy. In such
patients, if warranted, thalamotomy or deep

Dr. Khalid Yusuf El-Zohry – Sohag Teaching Hospital (01118391123) Page | 613
El-Zohry MRCP Questions Bank (Part 2) – Medical Masterclass 2010

brain stimulation by an electrode placed in the D. L5


thalamus may be tried.
E. S1

[ Q: 20 ] MasterClass Part2 Answer & Comments


(2010) - Neurology
Correct answer: C
A 64-year-old man was referred to the
outpatient clinic with a 6-month history of The description is that of L4 nerve root pain. L2
episodes of loss of awareness without collapse. and L3 pain radiates to the anterior thigh; L5
A witness account mentioned the occurrence of pain radiates through the buttock, down the
chewing movements during the attacks. He also posterolateral aspect of the thigh and lateral
has diabetes mellitus and hypertension. aspect of the calf, and across the dorsum of the
foot to the big toe; and S1 pain radiates
What one of the following is the most likely
through the inner buttock to the posterior
diagnosis?
aspect of the thigh, and then through the
A. Hypoglycaemia posterolateral aspect of the calf to the lateral
B. Transient ischaemic attack border of the foot.

C. Postural hypotension
[ Q: 22 ] MasterClass Part2
D. Cardiac arrhythmia
(2010) - Neurology
E. Complex partial seizure
A patient has: 1. A staggering gait 2. Truncal
instability 3. Normal arm and leg coordination
Answer & Comments 4. Spontaneous nystagmus 5. A rotated head
Correct answer: E posture

Where is the lesion most likely to be located?


The diagnosis of transient ischaemic attack (TIA)
should only be made if there is a clear history of A. Lateral pons
focal neurological deficit. ‘Positive' symptoms, B. Midline pons
such as chewing movements, would not be
C. Caudal cerebellar vermis
expected in TIA or any of the other diagnoses
listed. D. Rostral cerebellar vermis
E. Foramen magnum
[ Q: 21 ] MasterClass Part2
(2010) - Neurology Answer & Comments
A 50-year-old woman complains of pain Correct answer: C
radiating through the knee and down the
medial side of the calf to the medial malleolus. These signs are typical of an inferior cerebellar
vermis lesion, particularly the flocculonodular
Which one of the following nerve roots would
lobe. In children the commonest cause is a
give rise to such pain?
medulloblastoma.
A. L2
B. L3
C. L4

Dr. Khalid Yusuf El-Zohry – Sohag Teaching Hospital (01118391123) Page | 614
El-Zohry MRCP Questions Bank (Part 2) – Medical Masterclass 2010

[ Q: 23 ] MasterClass Part2 overall pattern is more in keeping with cluster


(2010) - Neurology headache.
Persistent daily headache always raises the
A 60-year-old man returns to his GP with a 12-
possibility of intracranial space occupying
month history of intense, boring pain around
lesions but the chronicity of pain and lack of
his right eye and temple. This was initially
neurological signs go against this.
occurring intermittently for a half hour or so ,
several times a day, for a few weeks at a time.
Now however, the pain has become chronic [ Q: 24 ] MasterClass Part2
and recurrent. It is associated with swelling of (2010) - Neurology
his eyelid and a feeling that his nose is 'blocked
A 50-year-old woman presents to the
up'. There are no visual symptoms. He does not
feel that the regular codeine/paracetamol emergency department with short history of
severe occipital headache with vomiting and
preparation previously prescribed has helped.
He is otherwise well and the GP elicits no impaired balance. Her past medical history
neurological signs. includes hypertension. On examination she has
nystagmus to the right, ataxia of her right limbs
What is the most likely diagnosis? and gait ataxia.

A. Headache due to analgesic overuse What is the most likely diagnosis?


B. Chronic trigeminal neuralgia A. Basal ganglia haemorrhage
C. Sinus problems B. Subdural haemorrhage
D. Chronic cluster headache C. Left temporal lobe haemorrhage
E. Intracranial tumour D. Pontine haemorrhage
E. Cerebellar haemorrhage.
Answer & Comments

Correct answer: D Answer & Comments

Correct answer: E
The initial pattern of the headache is
characteristic of cluster headache. Features
such as nasal congestion, lacrimation, She has posterior circulation signs, in particular
right cerebellar signs, and with the fact she has
rhinorrhea and Horner's syndrome are also
hypertension we need to exclude an acute
typical. A chronic form can develop on from
episodic cluster. cerebellar haemorrhage, which may need
surgical intervention
Daily analgesia can cause or maintain chronic
daily headache but the features here are more
typical of cluster headache, which makes this
[ Q: 25 ] MasterClass Part2
more likely. Similarly, although trigeminal (2010) - Neurology
neuralgia can leave the patient with chronic
residual pain, the pain typically starts around For how long is a patient required to avoid
the lower jaw, is lancinating and lasts seconds. driving (standard Group 1 licence) following full
recovery from a single transient ischaemic
Sinus disease can cause head/facial pain and attack (TIA)?
the nasal symptoms might support this but the
A. No restriction at all

Dr. Khalid Yusuf El-Zohry – Sohag Teaching Hospital (01118391123) Page | 615
El-Zohry MRCP Questions Bank (Part 2) – Medical Masterclass 2010

B. 1 week drink (unlike benign essential tremor, where


action such as drinking a cup of tea makes the
C. 1 month
tremor worse). Marked chewing movements
D. 3 months and lip-smacking suggest drug- induced
E. 6 months Parkinsonism and an extended neck and raised
eyebrows suggest progressive supranuclear
palsy; the neck is characteristically flexed in
Answer & Comments
Parkinson's disease. A positive glabeller tap is
Correct answer: C common in normal older people and is falling
out of favour as a diagnostic sign in younger
Driving advice should not be forgotten when a patients too.
patient is discharged from hospital following a
TIA. Patients must not drive for one month. [ Q: 27 ] MasterClass Part2
Notification of the DVLA is not necessary unless (2010) - Neurology
there is residual neurological deficit at the end
of this time. In the case of recurrent TIAs, a
symptom-free period of three months may be A 23-year-old is admitted with a heroin
required. overdose. After treatment with a naloxone
infusion he wakes up, but says that he cannot
[ Q: 26 ] MasterClass Part2 get out of bed because his legs are weak and he
(2010) - Neurology is unable to walk. On examination he has
marked weakness of all movements at the
An 82-year-old man is admitted following a fall. ankles and of knee flexion. He has preserved
The physiotherapist thinks he looks knee jerks, but the ankle jerks are absent. He
Parkinsonian and asks for your opinion. has sensory loss to pinprick in the distal legs,
but it is difficult to clarify the exact distribution.
Which of the following is most supportive of a
A lumbosacral MRI scan demonstrates mild
diagnosis of Parkinson's disease?
degenerative changes only.
A. His tremor is most disabling when he is
drinking his tea. What is the most probable diagnosis?

B. His neck is extended and he has a A. Diffuse cortical ischaemia


surprised expression, despite paucity of B. Bilateral lumbosacral plexopathy
facial movement.
C. A thoracic cord lesion
C. The tremor is worse in his left arm and
D. Bilateral sciatic nerve palsies
leg.
E. Myopathy due to rhabdomyolysis
D. You elicit a positive glabellar tap.
E. You notice marked oro-facial dyskinesia.
Answer & Comments

Answer & Comments Correct answer: D

Correct answer: C
The sciatic nerves supply all the muscles below
the knee and the knee flexors, and they also
Parkinson's disease is typically asymmetrical at provide sensory innervation of the lateral
presentation. The tremor is typically a rest border of the lower leg and entire foot, except
tremor so it is usually possible for the patient to

Dr. Khalid Yusuf El-Zohry – Sohag Teaching Hospital (01118391123) Page | 616
El-Zohry MRCP Questions Bank (Part 2) – Medical Masterclass 2010

the medial malleolus, which is supplied by the Which TWO responses are not recognised
saphenous nerve. The femoral nerve is features of sarcoidosis?
responsible for the knee jerk. A. optic disc pallor
B. diabetes insipidus
[ Q: 28 ] MasterClass Part2
(2010) - Neurology C. parkinsonism
D. matched oligoclonal bands in serum and
A 26-year-old woman presents with a week's
CSF
history of progressive numbness and weakness
in her legs. E. reduced transfer factor on respiratory
function tests
Which of the following suggests a diagnosis of
F. myopathy
Guillain-Barré syndrome (GBS)?
G. anterior uveitis
A. Optic atrophy on fundoscopy
H. peripheral neuropathy
B. A sensory level
I. multiple white matter lesions on MRI
C. Ankle weakness with saddle area sensory
loss J. amenorrhoea
D. Autonomic dysfunction
Answer & Comments
E. Proximal weakness greater than distal
weakness Correct answer: CD

Answer & Comments Sarcoidosis is a primary granulomatous


disorder. Neurological complications occur in
Correct answer: D approximately 5% of patients. Cranial
neuropathies from chronic basal meningitis are
Optic atrophy suggests a diagnosis of the main neurological complications of
demyelination. Clear-cut sensory levels sarcoidosis; facial palsies, commonly bilateral,
classically occur with cord compression. Ankle are most common. Other abnormalities include
weakness with saddle area sensory loss would hypothalamic disturbances, optic nerve
point towards a cauda equina lesion (always inflammation, peripheral neuropathy and
examine the saddle area!). In Guillain Barré myopathy. Meningeal enhancement on MRI
syndrome there are often cardiovascular with gadolinium is typical. Unmatched
abnormalities reflecting autonomic dysfunction. oligoclonal bands are typically seen in the CSF
Indeed the autonomic dysfunction can result in of patients with neurosarcoidosis.
sudden death and can manifest as extreme
hyper or hypotension, tachycardia and
[ Q: 30 ] MasterClass Part2
bradycardia. In GBS the weakness is greater
distally.
(2010) - Neurology
A 78-year-old man presents with left sided
[ Q: 29 ] MasterClass Part2 facial weakness, thought to be due to a lesion
(2010) - Neurology of the seventh cranial nerve.

A 40-year-old Afro-Caribbean lady presents Which one of the following statements about
with bilateral facial nerve palsy. You suspect the the seventh cranial nerve or its diseases is NOT
diagnosis of neurosarcoidosis. true?

Dr. Khalid Yusuf El-Zohry – Sohag Teaching Hospital (01118391123) Page | 617
El-Zohry MRCP Questions Bank (Part 2) – Medical Masterclass 2010

A. Five per cent of patients with Bell's palsy digastric and platysma muscles. The remaining
develop aberrant regeneration of fibres enter the parotid gland and fan out to
parasympathetic fibres which results in supply the muscles of facial expression -
lacrimation during eating. orbicularis oculi and oris, buccinator,
zygomaticus, frontalis, occipitalis etc.
B. It enters the petrous temporal bone at
the internal auditory meatus and exits Bell’s palsy is defined as a lesion of the facial
through the stylomastoid foramen. nerve in the petrous temporal bone of
unknown cause. It is characterized by a lower
C. It arises in the lower pons adjacent to the
motor neurone pattern of weakness that
sixth cranial nerve nucleus.
progresses over 3-72 hours, pain behind or in
D. Ten per cent of patients affected by Bell's the ear in 50% of patients, Bell's phenomenon
palsy have permanent residual facial (upward rotation of the globe in the orbit on
weakness. attempted eyelid closure), loss of taste and
E. It supplies the muscles of the second hyperacusis. Thirty per cent of patients will
branchial arch. have permanent facial weakness following
Bell's palsy.
Answer & Comments Aberrant regeneration of motor fibres explains
the phenomenon of synkinesis (seen in 50% of
Correct answer: D
patients) where eyelid closure is accompanied
by contraction of other facial muscles. A similar
The facial nerve is comprised of two 'roots': process occurs in the regenerating
1. Motor component which supplies parasympathetic fibres where aberrant
muscles of second branchial arch innervation produces 'crocodile tears' when
(muscles of facial expression) eating (5% of cases).

2. intermediate nerve which carries taste


fibres, parasympathetic efferents to the [ Q: 31 ] MasterClass Part2
lacrimal and salivary glands and a minor (2010) - Neurology
cutaneous sensory branch.
A 78-year-old woman presents to clinic with
The facial nerve arises in lower pons, and then forgetfulness and failure to look after herself at
loops around the sixth cranial nerve nucleus. home. She recently got lost at her local shops
On leaving the brainstem, it is closely opposed and had to be taken home by a friend. She feels
to the sixth and eighth cranial nerves in the there is very little the matter. Her Mini-Mental
cerebellopontine angle. The nerve enters the State Examination score is 22/30.
petrous temporal bone via the facial canal at
the internal auditory meatus. In the facial canal, What is the most likely diagnosis?
the nerve gives and receives several branches: A. Bilateral subdural haemorrhages
1. greater superficial petrosal nerve (taste B. Normal pressure hydrocephalus
from palate)
C. Alzheimer's disease
2. chorda tympani (taste from anterior
two-thirds of the tongue) D. Vascular dementia

3. nerve to stapedius E. Pseudo-dementia

It then exits through the stylomastoid foramen


where it supplies branches to the stylohyoid,

Dr. Khalid Yusuf El-Zohry – Sohag Teaching Hospital (01118391123) Page | 618
El-Zohry MRCP Questions Bank (Part 2) – Medical Masterclass 2010

Answer & Comments C. C6

Correct answer: C D. C7
E. C8.
This presentation would be absolutely typical of
Alzheimer's disease where forgetfulness is a Answer & Comments
common early feature, as is the patient's belief
that there is nothing wrong. Gait disturbance Correct answer: E
(apraxia) and urinary incontinence would
suggest normal-pressure hydrocephalus; Wasting of the first dorsal interosseous will
stepwise progression of symptoms would be occur with lesions affecting nerve roots C8, T1
typical of vascular dementia. which give rise to the ulnar nerve

[ Q: 32 ] MasterClass Part2 [ Q: 33 ] MasterClass Part2


(2010) - Neurology (2010) - Neurology
You see a patient in clinic with the abnormality A 60-year-old man presents to clinic with
marked ‘x’ in the image. muscle weakness and mild dysphagia. On
examination, he is noted to have
disproportionate weakness in his finger flexors
relative to the corresponding extensors, and
disproportionate weakness of knee extensors
compared to hip flexors.

What is the most likely diagnosis in this case?


A. Polymyositis
B. Motor neuron disease
C. Oculopharyngeal muscular dystrophy
D. Inclusion body myositis
E. Myasthenia gravis

Answer & Comments

Correct answer: D

The pattern of weakness described is


characteristic of inclusion body myositis.

[ Q: 34 ] MasterClass Part2
(2010) - Neurology
A lesion affecting which of the following nerve
roots could account for the patient’s condition? The image shows a T1-weighted axial MRI slice
of a 32-year-old male patient with band
A. C4
heterotopia, a malformation of cortical
B. C5 development.

Dr. Khalid Yusuf El-Zohry – Sohag Teaching Hospital (01118391123) Page | 619
El-Zohry MRCP Questions Bank (Part 2) – Medical Masterclass 2010

associated with seizure freedom in only


approximately 40%.
The image shows band heterotopia, also called
double cortex syndrome because of its
appearance on MRI. It is thought to represent
arrest of migrating neurons during cortical
development as they move from the
periventricular region to the cortex.

[ Q: 35 ] MasterClass Part2
(2010) - Neurology
A 42-year-old man presents with a sudden
onset of headache followed by collapse. On
arrival in A&E he has a heart rate of 76 bpm, a
blood pressure of 220/140 and a Glasgow Coma
Score of 9 (E2, M5, V2).

Which of the following should be done


Which of the following statements is incorrect? immediately?

A. Malformations of cortical development A. Give nimodipine 60 mg 4-hourly via a


(MCDs) frequently cause epilepsy. nasogastric tube

B. MCDs can be found incidentally on MRI in B. Start a labetalol infusion 15-30 mg/hour
otherwise apparently normal subjects. intravenously

C. MCDs are almost invariably associated C. Give codeine phosphate 60 mg IM


with skin lesions. D. Give high flow oxygen via a non-
D. MCDs do not become malignant. rebreather mask

E. MCDs can be genetically determined. E. Give 50 ml 50% dextrose intravenously.

Answer & Comments

Correct answer: C Answer & Comments

Correct answer: D
MCDs are a common cause of refractory focal
epilepsy, although they may be found He has likely suffered a subarachnoid
incidentally on MRI scans performed for other haemorrhage or an intracerebral bleed. The
reasons. Patients may be cognitively impaired priority is to prevent secondary brain injury. His
or completely normal. The abnormalities are airway is likely to be protected with a GCS of 9
non-progressive developmental lesions, and as (although he may benefit from a nasal or oral
such are not malignant or invasive. Skin lesions airway, and close consideration paid to his
are rare. MCDs may be acquired in utero during airway if going for a CT scan). He should receive
cerebral development or genetically high flow oxygen. His blood pressure should not
determined. Overall, surgical treatment is be treated acutely as it is often appropriate to
compensate for a rise in intracranial pressure.

Dr. Khalid Yusuf El-Zohry – Sohag Teaching Hospital (01118391123) Page | 620
El-Zohry MRCP Questions Bank (Part 2) – Medical Masterclass 2010

Nimodipine should be given if a subarachnoid C. preganglionic trigeminal nerve root


haemorrhage is proven. Attention should be involvement
paid to maintaining a normal blood sugar as
D. gasserion ganglion lesions
hyperglycaemia worsens outcome.
E. cavernous sinus syndrome

[ Q: 36 ] MasterClass Part2 F. superior orbital fissure syndrome


(2010) - Neurology G. foramen magnum
A patient presents with high stepping gait. H. foramen ovale
I. foramen rotundum
Which of the following is the most likely
diagnosis? J. peripheral branches of the trigeminal
nerve.
A. Cerebellar lesion
B. Diffuse cerebrovascular disease Answer & Comments
C. Parkinson’s disease
Correct answer: BD
D. Peripheral neuropathy
E. Proximal myopathy. This woman probably has dysfunction of the
mandibular and maxillary division of the
Answer & Comments trigeminal nerve. The loss of the corneal reflex
does not necessarily imply involvement of the
Correct answer: D ophthalmic division, as the sensory innervation
to the cornea is shared by the ophthalmic and
The high stepping gait reflects bilateral foot maxillary branches. There is no motor
drop. involvement.
Having determined this, one can begin to
[ Q: 37 ] MasterClass Part2 determine the possible site of the lesion.
(2010) - Neurology Lesions of the trigeminal system are said to be
diagnosed by ‘the company they keep’, i.e.
A 49-year-old Afro-Caribbean woman sees you associated signs.
in outpatients because three weeks previously
Loss in such a discrete sensory distribution
she noticed that the right side of her face was
numb. She felt otherwise well, with no other would not be caused by a cortical lesion,
although parietal lesions can cause isolated
symptoms. On examination she had diminished
perception of light touch and pinprick in the depression of the contralateral corneal reflex,
but hemifacial anaesthesia could result from a
lower 2/3 of her face, and in the intaoral cavity.
small thalamic lesion.
In addition, the right corneal reflex was slow.
There were no other abnormalities on Involvement of the rostral (upper) portion of
examination. the spinal trigeminal nucleus could result in
sensory loss in an onion-skin distribution (upper
Possible anatomical sites of trigeminal system portion / peri- and intaoral; lower portion /
involvement include: lateral face), that could mimic this patient’s
A. cortical lesion facial sensory loss. The spinal trigeminal
nucleus is often involved as part of a lateral
B. trigeminal nuclear lesion
medullary syndrome, and either complete,
upper, or lower facial hemianaesthesia may be

Dr. Khalid Yusuf El-Zohry – Sohag Teaching Hospital (01118391123) Page | 621
El-Zohry MRCP Questions Bank (Part 2) – Medical Masterclass 2010

associated, but intra-oral sensation is spared, as [ Q: 38 ] MasterClass Part2


this is supplied by the uppermost portion of the (2010) - Neurology
spinal trigeminal nucleus, which extends into
the pons. You are asked to see a 36-year-old woman with
headaches.
Lesions of the preganglionic trigeminal nerve
roots may be damaged by tumour, infection, Which TWO of the following features would
trauma or aneurysm. Lesions in this location are suggest a diagnosis of tension-type headache?
suggested by associated involvement of other
cranial nerves (VI, VII, VIII). A. Aggravated by physical activity

The trigeminal ganglion may be involved in B. Associated with ptosis


middle cranial fossa lesions. Pain is often the C. Attacks commonly occur at a frequency of
presenting feature, but sometimes numbness. 20-40 times a day
The distribution is variable and might affect
D. Aura prior to headache
only the lower part of the face. These
symptoms may precede several connective E. Bifronto-temporal pain
tissue diseases, e.g. Sjogren's syndrome, F. Commonly associated with vomiting
rheumatoid arthritis, systemic sclerosis. In
these cases, facial sensory loss is the first G. Headache relieved by alcohol
symptom in about 10% of cases. H. Lancinating pain
The cavernous sinus could not be the site of the I. Regular aspirin is beneficial for pain relief
lesion, as the ophthalmic and maxillary
J. Throbbing pulsatile pain.
branches, but not the mandibular branches run
through it. Similarly, the superior orbital fissure
contains the ophthalmic branch of the Answer & Comments
trigeminal nerve, together with CN III, IV, VI. Correct answer: EG
A small single lesion could not affect the
peripheral branches of the trigeminal nerve. Tension-type headache is characterized by a
Meningeal involvement at the skull base would constant, non-pulsatile, band-like, bi-fronto-
produce more diffuse signs, and carcinomatous temporal pressure on top of the head. The
deposits compressing individual braches (e.g. headache is not worsened by physical activity.
infraorbital branch of the maxillary division) is Associated mild nausea, but not vomiting, is
to occur singly, and so this combination of the common. Chronic forms are worsened by
two lower divisions of the trigeminal nerve anxiety and stress. Alcohol may relieve the
being affected is unlikely. headache. Regular simple analgesia may be the
Anatomical diagnosis of lesions involving the commonest cause of chronic daily headache.
trigeminal nerve is difficult, and relies on being There are no abnormal features on
absolutely clear about the signs in the face, and examination. Amitryptyline is the drug
associated signs. Isolated trigeminal sensory treatment of choice.
neuropathy is the commonest diagnosis in this
situation, but is a diagnosis of exclusion. [ Q: 39 ] MasterClass Part2
(2010) - Neurology
A 25-year-old woman attends A&E with a 48-
hour history of unilateral headache of gradual
onset associated with visual blurring and

Dr. Khalid Yusuf El-Zohry – Sohag Teaching Hospital (01118391123) Page | 622
El-Zohry MRCP Questions Bank (Part 2) – Medical Masterclass 2010

nausea. The headache is similar to her previous Third cranial nerve palsy due to compression by
migraine, but examination reveals that her right a posterior communicating artery aneurysm
pupil is larger than the left. Eye movements are may cause dilation of the ipsilateral pupil;
normal and there is no diplopia. however it is uncommon for eye movements to
be spared.
Which of the following statements is correct?
A. If the left pupil fails to respond to light it [ Q: 40 ] MasterClass Part2
is probably a Holmes–Adie pupil. (2010) - Neurology
B. Maintenance of the degree of inequality
An elderly man is brought into A&E by
in different light intensities is suggestive
ambulance because he had been found
of physiological anisocoria.
wandering down his street early in the morning.
C. She may have a right Horner's syndrome. He gives a fluent history of his past life, but is
D. A left Argyll-Robertson pupil is a likely unable to explain what he had been doing. On
explanation. examination he smells of alcohol. He has
nystagmus and bilateral lateral gaze palsies.
E. The features are strongly suggestive of
3rd cranial nerve palsy caused by Which of the following statements is incorrect?
posterior communicating artery
A. The lesions are in the mamillary bodies
aneurysm.
and thalamus.

Answer & Comments B. His red cell transketolase is low.


C. Examination of his pupils is normal.
Correct answer: B
D. All of his deficits will resolve with 3 days
Minor discrepancies in pupillary diameters are of parenteral thiamine.
common and usually represent physiological E. A CT scan of his head is likely to be
anisocoria, confirmed by the maintenance of normal.
the degree of inequality in different light
intensities. Answer & Comments
A Holmes–Adie pupil is usually larger, and fails
Correct answer: D
to react to light, with sluggish reaction to
accommodation.
This man has Wernicke–Korsakoff syndrome. It
Argyll–Robertson pupils are small, fail to react is caused by thiamine deficiency and is common
to light or accommodation, and may be in alcoholic patients. However, it should be
unilateral as in this case; however since the considered in all patients with malnutrition. The
cause of this condition is neurosyphilis, a classical presentation is of confusion, ataxia,
condition now almost unheard of in the UK, this ophthalmoplegia and nystagmus. The
would be extremely unlikely in a woman of this neurological signs do tend to improve with 3
age. days of parenteral thiamine, but there are often
When pupillary abnormalities are identified, residual memory problems. CT of the head is
always consider the possibility of a Horner's likely to be normal. An MRI may show evidence
syndrome on the side of the smaller pupil - of neuronal loss and demyelination in the
there will be ipsilateral partial ptosis and may midbrain structures.
be anhydrosis of the same side of the face.

Dr. Khalid Yusuf El-Zohry – Sohag Teaching Hospital (01118391123) Page | 623
El-Zohry MRCP Questions Bank (Part 2) – Medical Masterclass 2010

[ Q: 41 ] MasterClass Part2 I. Sjogren’s syndrome


(2010) - Neurology J. Vitamin B12 deficiency.
A 67-year-old woman recently diagnosed with
small cell carcinoma of the lung presents with a Answer & Comments
two-month history of progressive ataxia, Correct answer: DH
dysarthria and nystagmus.

Which of the following onconeuronal antibodies Lead toxicity and porphyria typically cause
would you expect to be positive? motor peripheral neuropathy.

A. Anti-Hu
[ Q: 43 ] MasterClass Part2
B. Anti-Yo
(2010) - Neurology
C. Anti-acetylcholine receptor
A 58-year-old man presents complaining of
D. Anti-amphiphysin unsteadiness.
E. Anti-voltage gated calcium channel
Which of the following symptoms or signs is
usually inconsistent with a cerebellar lesion?
Answer & Comments
A. Symptoms worse in dark
Correct answer: B
B. Symptoms have got worse over several
years
The patient has developed paraneoplastic
cerebellar degeneration secondary to her lung C. Symptoms are very brief and associated
cancer. This condition is associated with the with head movement
anti-Yo antibody, the antigen of which is the D. Sustained horizontal nystagmus that
Purkinje cell cytoplasm. doses not fatigue
E. A wide-based gait
[ Q: 42 ] MasterClass Part2
F. Bilateral upgoing plantars
(2010) - Neurology
G. Papilloedema
A 40-year-old man presents with a sensory
peripheral neuropathy. H. Headache
I. Diplopia
Which TWO of the following are unlikely to be
the cause of this man’s symptoms? J. Pain in the neck and wasting of hand
muscles.
A. Alcohol
B. Amyloid Answer & Comments
C. Human immunodeficiency syndrome Correct answer: CJ
D. Lead
E. Leprosy Cerebellar lesion can be acute or chronic,
symmetrical or asymmetric, isolated or part of a
F. Paraneoplastic more widespread degenerative condition. The
G. Paraproteinaemic symptoms are characteristically worse in the
dark and can come on slowly in degenerative
H. Porphyria

Dr. Khalid Yusuf El-Zohry – Sohag Teaching Hospital (01118391123) Page | 624
El-Zohry MRCP Questions Bank (Part 2) – Medical Masterclass 2010

conditions. Headache and double vision are Anterior dysphasia is characterized by non-
common in pontine-cerebellar lesion. On fluent hesitant speech with an agrammatic or
examination the classical findings are horizontal telepathic element, the substitution of words
nystagmus, with gait and limb ataxia.
or syllables, poor writing with errors similar to
The findings of papilloedema suggest a speech, impaired naming of things and
posterior fossa space-occupying lesion (SOL) preserved comprehension, repetition and
and the upgoing plantar may be indicative of a reading.
brain stem lesion, or MSA. Very brief symptoms
Posterior dysphasia is characterized by fluent
associated with head movement are more
speech with normal rhythm, neologisms,
suggestive of benign paroxysmal positional
paraphrasias, substitutions and poor
vertigo (BPPV); neck pain and wasting of hand
comprehension, repetition and reading.
muscles should raise the possibility of a cervical
myelopathy.
[ Q: 45 ] MasterClass Part2
[ Q: 44 ] MasterClass Part2 (2010) - Neurology
(2010) - Neurology A 76-year-old woman is admitted with sudden
onset left sided weakness. CT brain scan shows
You are referred a 60-year-old man complaining
a right parietal infarct. Nursing staff notice she
of increasing difficulty with speech.
coughs and appears to choke when drinking
fluids.
Which of the following TWO features would
make you characterize it as a posterior
Which of the following statements is correct?
(receptive) type of dysphasia?
A. A gastroscopy should be organized as
A. Agrammatic speech
soon as possible.
B. Defective voice production
B. Swallowing difficulties are likely to persist
C. Fluent speech long term.
D. Impaired articulation C. The speech and language therapist may
E. Impaired naming of objects recommmend fluid thickeners.

F. Non-fluent speech D. Nasogastric (NG) feeding will protect


against aspiration.
G. Poor comprehension
E. Percutaneous endoscopic gastrostomy
H. Poor writing (PEG) feeding should be considered if
I. Preserved repetition dysphagia persists for greater than one
month.
J. Silent reading unaffected.

Answer & Comments


Answer & Comments
Correct answer: C
Correct answer: CG

Dysphagia associated with stroke is generally


Defective voice production results in dysphonia.
due to abnormalities of the oropharyngeal
Impaired articulation results in dysarthria.
phase of swallowing. Unlike patients with
dysphagia due to oesophageal tumours or
strictures, patients with dysphagia due to

Dr. Khalid Yusuf El-Zohry – Sohag Teaching Hospital (01118391123) Page | 625
El-Zohry MRCP Questions Bank (Part 2) – Medical Masterclass 2010

neurological disorders find fluids more difficult pain to subside as pain is increasing.
to swallow than solids. Bedside assessment and Approximately one third recover after 12
videofluoroscopy are the basis of evaluation. months, 75% by 2 years and 89% by the end of
3 years.
Swallowing difficulties associated with stroke
are usually transient and recovery occurs within
two weeks in the majority of cases. Adequate [ Q: 47 ] MasterClass Part2
intake can often be safely maintained using (2010) - Neurology
fluid thickeners
A 78-year-old man presents with a stroke.
to increase the viscocity of ingested fluids. When conducting your physical examination of
Although there is no firm evidence as yet to him you are trying to localize this.
support a particular approach, most physicians
would consider PEG feeding if there were no Which one of the following features would NOT
signs of recovery after one to two weeks be consistent with the diagnosis of a subcortical
lacunar stroke?

[ Q: 46 ] MasterClass Part2 A. Pure motor stroke


(2010) - Neurology B. Pure sensory stroke

A 30-year-old man presents with a three-day C. Ataxic hemiparesis


history of spontaneous, deep, boring, right D. Dysarthria and a clumsy hand
shoulder/neck pain, which is followed a few
E. Dysphasia.
days later by weakness and then wasting of the
right deltoid, spinati and triceps muscles.
Answer & Comments
The likely diagnosis is:
Correct answer: E
A. C5/C6 radiculopathy
B. Mononeuritis multiplex Evidence of higher cortical involvement, e.g.
dysphasia, dyscalculia or visuospatial disorder,
C. Rotator cuff tendonitis
or disturbance of consciousness, excludes a
D. Brachial neuritis lacunar syndrome. All of the other features are
E. Syringomyelia consistent with this diagnosis.

Answer & Comments [ Q: 48 ] MasterClass Part2


(2010) - Neurology
Correct answer: D
A 28-year-old woman with epilepsy attends
Idiopathic brachial neuritis/plexopathy is an your clinic for advice regarding contraception.
immune-mediated disorder. Half of the cases
are preceded by an upper respiratory tract Which of the following statements are FALSE?
infection, a flu-like illness, or immunisation. The A. Lamotrigine does not affect the efficacy
initial feature is the abrupt onset of unilateral of the combined oral contraceptive pill
arm pain, usually located in the shoulder. (OCP)
Lessening of pain is associated with evolving
B. The progesterone only pill (POP) is not an
weakness, which peaks 2-3 weeks after the
effective alternative to the OCP in
onset of pain. Cervical radiculopathy is the main
patients taking carbamazepine.
differential diagnosis, however, it is unusual for

Dr. Khalid Yusuf El-Zohry – Sohag Teaching Hospital (01118391123) Page | 626
El-Zohry MRCP Questions Bank (Part 2) – Medical Masterclass 2010

C. Metabolism of the OCP is accelerated by D. For patients on enzyme-inducing


topiramate medication, an OCP containing 50mcg of
oestrogen is as effective as a low
D. For patients on enzyme-inducing
oestrogen dose OCP for women on no
medication, an OCP containing 50mcg of
anti-epileptic medication.
oestrogen is as effective as a low
oestrogen dose OCP for women on no E. Metabolism of depo-provera is
anti-epileptic medication. unaffected by anti-epileptic medication.
E. Metabolism of depo-provera is
unaffected by anti-epileptic medication. Answer & Comments

Correct answer: D
Answer & Comments

Correct answer: D Metabolism of the OCP and the POP is


accelerated by: carbamazepine, phenytoin,
phenobarbitone, oxcarbazepine, and
Metabolism of the OCP and the POP is
topiramate. The higher oestrogen (50mcg)
accelerated by: carbamazepine, phenytoin,
containing OCP preparations are more effective
phenobarbitone, oxcarbazepine, and
than the typical lower dose preparations
topiramate. The higher oestrogen (50mcg)
(30mcg) in patients on enzyme inducing
containing OCP preparations are more effective
medication, but are still approximately 10 times
than the typical lower dose preparations
less effective than in women on the 30mcg
(30mcg) in patients on enzyme inducing
preparations who do not take anti-epileptic
medication, but are still approximately 10 times
medication. There is no evidence that the
less effective than in women on the 30mcg
metabolism of Depo-provera is affected by anti-
preparations who do not take anti-epileptic
epileptic medication.
medication. There is no evidence that the
metabolism of Depo-provera is affected by anti-
epileptic medication. [ Q: 50 ] MasterClass Part2
(2010) - Neurology
[ Q: 49 ] MasterClass Part2 When testing the pupillary light reflex, you
(2010) - Neurology observe that the patient has a direct response
to light shone in the right eye, but no
A 28-year-old woman with epilepsy attends
consensual response. Light shone in the left eye
your clinic for advice regarding contraception.
elicits a consensual response, but not a direct
Which of the following statements are FALSE? response. In addition, on examining eye
movements, the left eye appeared to be fixed in
A. Lamotrigine does not affect the efficacy
an inferior and lateral position.
of the combined oral contraceptive pill
(OCP) Where is the lesion likely to be?
B. The progesterone only pill (POP) is not an A. Right optic nerve
effective alternative to the OCP in
B. Left optic nerve
patients taking carbamazepine.
C. Right oculomotor nerve
C. Metabolism of the OCP is accelerated by
topiramate D. Left oculomotor nerve
E. Right abducens nerve

Dr. Khalid Yusuf El-Zohry – Sohag Teaching Hospital (01118391123) Page | 627
El-Zohry MRCP Questions Bank (Part 2) – Medical Masterclass 2010

Answer & Comments she did not take any prophylactic drugs due to
concerns regarding side effects.
Correct answer: D
Which of the following are correct?
A left third nerve palsy will cause a dilated left
A. The headache from migraine is usually
pupil, with the eye in a 'down and out' position.
generalised from the onset.
For the light reflex, the afferent pathway is
controlled by the optic nerve and the efferent B. Distinction from a tension headache is
pathway by the oculomotor nerve. always possible clinically.
C. Patients with migraine may present with
[ Q: 51 ] MasterClass Part2 a 7th cranial nerve palsy.
(2010) - Neurology D. The superficial temporal artery is never
engorged or pulsatile in migraine.
A patient presents with weakness of
dorsiflexion of the right big toe. E. Prodromal symptoms are usually systemic
and related to the vomiting centre.
Which nerve root would you expect to be
affected in this case? Answer & Comments
A. L2
Correct answer: C
B. L3
C. L4 Prodromal symptoms are usually visual and
related to ischaemia of the intracranial arteries.
D. L5
The headache in migraine is usually unilateral
E. S1
to start with but can become generalised.

Answer & Comments The superficial temporal artery can be engorged


and pulsating.
Correct answer: D
It may be difficult to distinguish from a tension
headache.
An L5 root lesion causes weakness of ankle
dorsiflexion, inversion and eversion, and Although rare, a 7th cranial nerve palsy may
dorsiflexion of the great toe. L2 weakness develop during a migraine attack.
affects hip flexion and thigh adduction; L3
weakness affects thigh adduction and knee [ Q: 53 ] MasterClass Part2
extension; L4 weakness affects knee extension (2010) - Neurology
and ankle inversion; and S1 weakness affects
plantar flexion, eversion and knee flexion. A 56-year-old woman presents with progressive
leg weakness over 3 days. She has noted minor
urinary incontinence in addition. She has a past
[ Q: 52 ] MasterClass Part2
medical history of breast cancer treated 10 year
(2010) - Neurology ago with lumpectomy and radiotherapy.
A 74-year-old lady was reviewed on the post-
Which is the investigation of choice?
take ward round having been admitted via her
general practitioner with a severe headache, A. Myelography
nausea and vomiting. Further enquiry revealed B. Plain spinal radiographs
a long-standing history of migraine, although

Dr. Khalid Yusuf El-Zohry – Sohag Teaching Hospital (01118391123) Page | 628
El-Zohry MRCP Questions Bank (Part 2) – Medical Masterclass 2010

C. Lumbar puncture metabolism. The 'second wind' phenomenon is


characteristically seen in McArdle's disease. It is
D. Computed tomography (CT)
thought to occur because of diversion of blood
E. Magnetic resonance imaging (MRI) flow to muscle and the onset of fatty acid
metabolism.
Answer & Comments

Correct answer: E [ Q: 55 ] MasterClass Part2


(2010) - Neurology
She is likely to have spinal cord compression A 45-year-old man complains of an unsteady
from metastases. Plain radiographs and CT may gait. He appears to have a wide based ataxic
give some information but the imaging gait. However, on testing his coordination
modality of choice is MRI. Lumbar puncture and whilst lying down, it seems a little inconsistent
myelography carry the risk of clinical but within normal limits. His arms are normal,
deterioration. as are his eye movements and speech. The
ankle reflexes are difficult to obtain, but knee
[ Q: 54 ] MasterClass Part2 reflexes are present.
(2010) - Neurology
What is the likely cause of his signs?
A 25-year-old man presents with muscle pains
A. Pontine infarction
and fatigue, which typically occur within
minutes of exercising. He has noticed that if he B. Multiple sclerosis
gently exercises through the initial barrier, he is C. Friedreich's ataxia
able to continue normally again for two to
D. Conversion disorder
three hours. Neurological examination is
unremarkable in clinic. E. Alcoholic cerebellar degeneration

Which of the following conditions is this patient


Answer & Comments
likely to have?
Correct answer: E
A. Myasthenia gravis
B. Myotonic dystrophy
The signs represent superior cerebellar vermis
C. Carnitine palmitoyl transferase deficiency dysfunction, which is the usual site of cerebellar
damage with alcohol abuse.
D. Acid maltase deficiency
He is too old for Friedreich's ataxia, and the
E. Myophosphorylase deficiency (McArdle's
absence of ankle reflexes could be neuropathic
disease)
(secondary to alcohol) in origin.

Answer & Comments


[ Q: 56 ] MasterClass Part2
Correct answer: E (2010) - Neurology

McArdle's disease is a disorder of carbohydrate A young woman attends outpatients following


metabolism caused by myophosphorylase several witnessed convulsive seizures. Her GP
deficiency. Carbohydrate stores within muscles has already started her on anti-epileptic
are needed in early exercise prior to further monotherapy. She reports that she wishes to
energy supply being provided by lipid get pregnant.

Dr. Khalid Yusuf El-Zohry – Sohag Teaching Hospital (01118391123) Page | 629
El-Zohry MRCP Questions Bank (Part 2) – Medical Masterclass 2010

Which of the following statements is NOT Answer & Comments


correct?
Correct answer: D
A. Folic acid supplementation is important.
B. The risk of a major fetal malformation is New variant CJD commonly presents in young
approximately 2-3 times higher than the adult with psychiatric symptoms, followed by
background risk. non-specific painful sensory symptoms, most
often in the lower limbs. Cognitive impairment,
C. The risk to the mother and fetus during
pyramidal signs, myoclonus and primitive
convulsive seizures generally outweighs
reflexes then develop. Mean disease duration is
the deleterious effects of continuing anti-
approximately 14 months. MRI commonly
epileptic medication.
shows high signal on T2-weighted images in the
D. Seizure frequency deteriorates during pulvinar (posterior aspect of thalamus). EEG is
most pregnancies. often normal, compared to sporadic CJD in
E. Vitamin K supplements are required in which triphasic waves are observed. Diagnosis
the last month of pregnancy in those is made ante-mortem by brain biopsy or more
taking enzyme-inducing medication. commonly tonsillar biopsy. CSF contains 14-3-3
protein. A clear link with BSE has now been
established.
Answer & Comments

Correct answer: D [ Q: 58 ] MasterClass Part2


(2010) - Neurology
Appoximately one third of patients experience
an increase in seizure frequency during A 58-year-old man presents with back pain that
pregnancy. radiates through the knee and down the medial
side of the calf to the medial malleolus.
[ Q: 57 ] MasterClass Part2 The nerve root involved is:
(2010) - Neurology
A. L2
A 29-year-old man presents with a 9-month
B. L3
history of depression, 4-month history of
painful sensory disturbance in both legs and C. L4
most recently cognitive impairment with D. L5
myoclonus. His MRI scan reveals thalamic
E. S1.
hyperintensity on T2-weighted images. His EEG
is normal.
Answer & Comments
The most likely diagnosis is:
Correct answer: C
A. corticobasal degeneration
B. progressive multifocal L2,3 radiates to the anterior thigh.
leucoencephalopathy
L5 radiates through the buttock, down the
C. juvenile myoclonic epilepsy posteriolateral aspect of the thigh, lateral
aspect of calf and across the dorsum of the foot
D. new variant Creutzfeldt-Jakob disease
to the big toe.
E. Wilson's disease.

Dr. Khalid Yusuf El-Zohry – Sohag Teaching Hospital (01118391123) Page | 630
El-Zohry MRCP Questions Bank (Part 2) – Medical Masterclass 2010

S1 radiates through the inner buttock to the D. Fluctuating blood pressure


posterior aspect of the thigh, posteriolateral
E. Sluggish pupillary reactions
aspect of the calf to the lateral border of the
foot. F. Normal arm and leg relexes
G. Marked fatiguability of leg movement
[ Q: 59 ] MasterClass Part2 H. Subjective sensory disturbance
(2010) - Neurology
I. Dysarthria
An 84-year-old man presents to clinic with a 6- J. Shortness of breath on exertion.
month history of increasing confusion, visual
hallucinations, reduced mobility and falls.
Answer & Comments
What is the most likely cause for his problems? Correct answer: CF
A. Alzheimer's disease
B. Frontotemporal dementia Guillain–Barré classically presents with an
ascending paralysis beginning in the legs.
C. Dementia with Lewy bodies Sensory involvement is variable, but usually less
D. Parkinson's disease with dementia than motor features. Reflexes are lost early in
the illness. Demyelinating neuropathies
E. Vascular dementia
characteristically cause fatigue with repeated
use. The cranial nerves are commonly affected,
Answer & Comments most
Correct answer: C commonly the facial nerves. The autonomic
nerves can be involved leading to pupillary
The presentation, particularly visual abnormalities and haemodynamic instability.
hallucinations, is typical of dementia with Lewy The bladder and bowel can be affected, but
bodies. Patients with Alzheimer's disease tend early involvement would be more suggestive of
to present with forgetfulness for recent events a conus lesion. Guillain–Barré is very painful,
and a tendency to repeat conversations, but and can present with back pain.
early motor impairment would be unusual.
Visual hallucinations would be less usual for the [ Q: 61 ] MasterClass Part2
other diagnoses.
(2010) - Neurology

[ Q: 60 ] MasterClass Part2 The wife of a 32-year-old man admitted to


intensive care unit (ITU) with status epilepticus
(2010) - Neurology
(SE), wants to discuss his treatment and
A 40-year-old patient presents with progressive prognosis with the medical registrar on call. She
leg weakness. is very distressed but has a list of questions that
she wants answered.
Which features would be against a diagnosis of
Guillain–Barré syndrome? Which of the following statements is CORRECT?
A. Severe back pain A. Most patients with status epilepticus
have a background history of epilepsy
B. Recent chest infection
B. The mortality in patients admitted to ITU
C. Urinary incontinence at the beginning of
with status epilepticus is more than 50%
the illness

Dr. Khalid Yusuf El-Zohry – Sohag Teaching Hospital (01118391123) Page | 631
El-Zohry MRCP Questions Bank (Part 2) – Medical Masterclass 2010

C. SE occurring in patients with epilepsy is C. Acetyl choline receptor antibodies


more likely with a temporal lobe
D. Chest radiograph
abnormality
E. MRI scan of the brain
D. If SE persists despite medical therapy,
they should have an anaesthetic and be F. Blood glucose
ventilated G. Erythrocyte sedimentation rate (ESR)
E. The duration of the attack has no effect H. Temporal artery biopsy
on prognosis
I. Random cholesterol measurement

Answer & Comments J. Cervical spine radiograph.

Correct answer: D
Answer & Comments

About 60% of all episodes of SE occur in Correct answer: CD


patients with no previous history of epilepsy.
The patient’s symptoms suggest myasthenia,
Mortality of patients with SE admitted to ITU is
and bezafibrate is a recognised cause of this,
5 to 10%.
indeed the timing of the onset of his symptoms
In a patient with a history of epilepsy, SE is suggests this may be the most likely cause.
more likely if there is some frontal lobe
The presence of acetylcholine receptor
abnormality.
antibodies would be suggestive of myasthenia
If SE persists despite medical therapy, then the gravis, and a chest radiograph is necessary to
patient should have a general anaesthetic and rule out an underlying primary lung tumour.
be ventilated.
A tensilon test, whereby a small dose of
The longer the attack lasts, the greater the edrophonium is injected and produces rapid
chance of neuronal damage and the greater the resolution of symptoms caused by myasthenia,
chance that the attack will resist treatment. may be helpful.
It is unlikely that the other investigations listed
[ Q: 62 ] MasterClass Part2 would affect his initial management, although
(2010) - Neurology imaging of the brain, preferably with MRI,
should be considered if the symptoms progress
A 75-year-old man with no recent history of
despite discontinuation of the bezafibrate with
head trauma presents with a one-week history
negative antibodies and tensilon test.
of intermittent dysarthria. This progresses
during the day and is usually relieved following Random measurement of lipids will not
a period of rest. He gives a past history of influence initial management, although fasting
cerebrovascular disease and has recently lipids after discontinuation of the bezafibrate
started bezafibrate for the treatment of may help in the decision of whether treatment
hyperlipidaemia, in addition to regular aspirin. with a statin is justified.
He has been a lifelong smoker.
[ Q: 63 ] MasterClass Part2
Which TWO of the following investigations
would be most helpful in his management? (2010) - Neurology
A. Carotid doppler studies A 30-year-old woman presents with a 3-day
history of left-sided retro-orbital pain and
B. CT brain scan

Dr. Khalid Yusuf El-Zohry – Sohag Teaching Hospital (01118391123) Page | 632
El-Zohry MRCP Questions Bank (Part 2) – Medical Masterclass 2010

progressive visual loss. She has been previously magnetic resonance angiography.
fit and well.
Which of the following would you expect to find
The following are recognised features of optic in this patient?
neuritis, except:
A. Constriction of the right pupil
A. central scotoma
B. Decreased right blink reflex
B. relative afferent pupillary defect
C. Inability to depress right eyeball
C. colour desaturation
D. Inability to elevate right eyeball
D. ptosis
E. Inability to move the right eyeball
E. sheathed retinal vasculature laterally.
F. peripheral visual field loss
Answer & Comments
G. pupil unreactive to light and
accommodation Correct answer: D
H. swollen optic disc on fundoscopy
A posterior communicating artery aneurysm
I. normal optic disc on fundoscopy will cause compression of the third nerve, and
J. delayed visual evoked potential. therefore pupillary involvement from
compression of the parasympathetic fibres that
Answer & Comments run on the outside of the third nerve. Other
features of a third nerve palsy include ptosis,
Correct answer: DF and a ‘down and out’ eye.

Optic neuritis presents with rapid, progressive [ Q: 65 ] MasterClass Part2


visual loss, often associated with orbital or
(2010) - Neurology
retro-orbital pain. Examination reveals reduced
visual acuity, impaired colour vision, a central A 68-year-old woman is referred having had a
scotoma, relative afferent pupillary defect and ‘funny turn’, the question asked by the general
retinal vein sheathing. It may be associated practitioner being ‘was this a transient
with either a normal appearing optic disc ischaemic attack (TIA)?
(retrobulbar neuritis) or a swollen optic disk in
typical optic neuritis. Visual evoked potentials Which TWO features listed below would be
are commonly delayed. acceptable to sustain the diagnosis of TIA?

There are a number of causes including: A. Vertigo


sarcoidosis, Wegener's granulomatosis, B. Diplopia
tuberculosis, syphilis, meningitis or most
commonly idiopathic (either in isolation or in C. Dysphagia
association with multiple sclerosis). D. Dysphasia
E. Dysphonia
[ Q: 64 ] MasterClass Part2
F. Loss of balance
(2010) - Neurology
G. Tinnitus
A 55-year-old man is found to have a right
H. Impairment of the function of one hand
posterior communicating artery aneurysm on

Dr. Khalid Yusuf El-Zohry – Sohag Teaching Hospital (01118391123) Page | 633
El-Zohry MRCP Questions Bank (Part 2) – Medical Masterclass 2010

I. Amnesia What is the most likely diagnosis?


J. Sensory symptoms confined to one part of A. Basilar artery thrombosis
limb or face.
B. Multiple sclerosis
C. Miller Fisher syndrome
Answer & Comments
D. Amyotrophic lateral sclerosis
Correct answer: DH
E. Wernicke's encephalopathy
A transient ischaemic attack (TIA) is defined as
an episode of acute loss of focal cerebral or Answer & Comments
monocular function lasting less than 24 hours.
Correct answer: D
Since any signs will almost certainly have
disappeared before the patient is assessed, the Amyotrophic lateral sclerosis is characterised by
diagnosis depends entirely upon the history. a combination of upper and lower motor
Non-focal symptoms such as loss of neuron signs as described in this scenario.
consciousness, dizziness, mental confusion,
generalised weakness and incontinence are not
[ Q: 67 ] MasterClass Part2
acceptable as evidence of TIA. In addition, some
focal signs occurring in (2010) - Neurology
isolation should not be interpreted as TIA A 67-year-old man presents with ptosis of the
because they can so often mislead. These left eye. You explain that this could be due to
include: Horner’s syndrome. A medical student asks you
how the sympathetic fibres reach the pupil.
 vertigo
 diplopia You reply that they travel in a branch of the:
 dysphagia A. third nerve
 dysphonia B. sixth nerve
 loss of balance C. seventh nerve
 tinnitus D. fifth nerve
 amnesia E. fourth nerve.
 sensory symptoms with restricted
distribution. Answer & Comments

Correct answer: D
[ Q: 66 ] MasterClass Part2
(2010) - Neurology Sympathetic fibres reach the pupil as the long
ciliary nerves from the naso-ciliary branch of
the fifth nerve.
A 65-year-old man presents with a 4-month Sympathetic fibres to the eyelid travel in the
history of dysarthria and progressive difficulty branch of the third nerve to levator palpebrae
in swallowing. Examination reveals a weak superioris.
fasciculating tongue, a brisk gag reflex, and jaw
jerk.

Dr. Khalid Yusuf El-Zohry – Sohag Teaching Hospital (01118391123) Page | 634
El-Zohry MRCP Questions Bank (Part 2) – Medical Masterclass 2010

[ Q: 68 ] MasterClass Part2 [ Q: 69 ] MasterClass Part2


(2010) - Neurology (2010) - Neurology
A 16-year-old girl presents with a history of On examination of the eye movements, you
episodes of bizarre behaviour following her discover poor upgaze.
parents' recent divorce. Her mother describes
these attacks to you. In which of the following clinical situations is
this least likely to be of clinical significance?
Which TWO of the following features would A. An 87-year-old with severe angina
suggest a true epileptic seizure rather than a
non-epileptic attack? B. A 46-year-old with atrial fibrillation on
carbimazole
A. Limb flailing
C. A 67-year-old with worsening headaches
B. Opisthotonus
D. A 73-year-old with tremor and increased
C. Carpet burns
tone
D. Tongue biting
E. A 52-year-old with muscular dystrophy
E. Pelvic thrusting
F. Duration of several hours Answer & Comments

G. Cyanosis Correct answer: A


H. Rapid recovery
The patients in B, C, D & E are all likely to have
I. Directed violent behaviour
pathological reasons for this finding ie
J. Repetitive chewing movements thyrotoxic myopathy, parinaud's syndrome,
progressive supranuclear palsy and
Answer & Comments ophthalmoplegia in association with one of the
forms of MD.
Correct answer: GJ
However, older people with no overt
neurological disorder are sometimes found to
Non-epileptic attacks can be distinguished fairly
have poor upgaze, which does not carry
reliably on clinical grounds; however difficulty
diagnostic significance.
often arises with frontal lobe seizures which can
be bizarre and occur without EEG change.
[ Q: 70 ] MasterClass Part2
Cyanosis and repetitive chewing movements
(2010) - Neurology
are more suggestive of epileptic seizures.
Recovery usually occurs over a few minutes A 70-year-old man with Parkinson's disease
rather than immediately. Carpet burns, complains that he gets a lot of peak-dose
opisthotonus and pelvic thrusting are classically dyskinesias. He is taking L-dopa medication.
associated with non-epileptic attacks. Tongue-
biting is seen with both types of attack and is If it was impossible to reduce his total L-dopa
non-discriminotory. dose because of the dramatic increase in his
'off' time, which of the following would you
consider adding to his treatment regime?
A. Amantadine
B. Benzhexol

Dr. Khalid Yusuf El-Zohry – Sohag Teaching Hospital (01118391123) Page | 635
El-Zohry MRCP Questions Bank (Part 2) – Medical Masterclass 2010

C. Entacapone Treatment is usually symptomatic, but


remember that neuroleptic drugs such as
D. Ropinorole
haloperidol will worsen Parkinsonian features,
E. Selegiline. so consider using atypical antipsychotic agents
such as quetiapine. There is some evidence that
Answer & Comments the dementia may respond to
anticholinesterase inhibitors.
Correct answer: A
Dementia associated with Parkinson's disease
tends to occur much later in the course of the
This is a difficult and common problem. There is
disease.
currently interest in amantadine, an NMDA-
receptor antagonist, and its possible role in
temporarily reducing dyskinesias in patients [ Q: 72 ] MasterClass Part2
with Parkinson's disease without worsening (2010) - Neurology
Parkinsonian symptoms. The evidence at
present is limited, but it is a promising A 37-year-old man is admitted one hour after
therapeutic option. The alternative is to reduce the sudden onset of severe headache, which he
the dose of L-dopa, and replace it with described as like 'being hit on the head with a
dopamine agonists, although in practice these hammer'. You suspect subarachnoid
drugs can also induce dyskinesias. haemorrhage, but a CT scan of the brain is
reported as normal.

[ Q: 71 ] MasterClass Part2 To pursue the diagnosis you should:


(2010) - Neurology A. wait for 24 hours after the onset of
An 84-year-old man presents with a 6-month headache, then perform a lumbar
history of increasing confusion, visual puncture to look for xanthochromia
hallucinations, reduced mobility and falls. B. repeat the CT scan

Which type of dementia fits this history best? C. perform a lumbar puncture to look for
xanthochromia as soon as possible
A. Alzheimer's disease
D. perform a lumbar puncture to look for
B. Pick's disease
red blood cells in the CSF as soon as
C. Dementia with Lewy bodies possible
D. Parkinson's disease E. refer to neuroradiologist for cerebral
angiography.
E. Vascular dementia

Answer & Comments


Answer & Comments
Correct answer: A
Correct answer: C

The investigation of choice in suspected


There is some overlap between the different
subarachnoid haemorrhage (SAH) is immediate
types of dementia, but in this case there are
CT scan without contrast, taking very thin cuts
clues that this is dementia with Lewy bodies
through the base of the brain to optimise the
(DLB) with the early development of instability,
chances of seeing small collections of blood.
falls and hallucinations. Other features include
fluctuating cognition, depression and delusions.

Dr. Khalid Yusuf El-Zohry – Sohag Teaching Hospital (01118391123) Page | 636
El-Zohry MRCP Questions Bank (Part 2) – Medical Masterclass 2010

Imaging within 12 hours using modern scanners Motor neurone disease results in only motor
has a 98-100% sensitivity for SAH. deficit. Muscle weakness is the commonest
presenting symptom and may be limb or bulbar
Lumbar puncture should be performed in
or both at onset. Clinical features include
suspected SAH if the CT scan is not diagnostic.
wasting, fasciculation and normal sensation.
The CSF specimen should be centrifuged
Reflexes may be pathologically brisk.
without delay and examined by
spectrophotometry for the presence of Porphyria and lead toxicity cause a purely
xanthochromia due to the presence of motor peripheral neuropathy.
oxyhaemoglobin and bilirubin. Note, however,
that xanthochromia may not be present if the [ Q: 74 ] MasterClass Part2
CSF is examined within 12 hours of
(2010) - Neurology
haemorrhage occurring, so lumbar puncture
should be delayed for 24 hours. A 48-year-old man has attacks of facial/head
pain and you are considering the diagnosis of
[ Q: 73 ] MasterClass Part2 cluster headache.
(2010) - Neurology Which one of the following statements is NOT
A 50-year-old woman presents with a history of true of this condition?
progressive distal weakness of her upper and A. Effective acute treatment includes 100%
lower limbs. Examination also reveals reduced oxygen, inhaled for 15-20 minutes
tone and impaired sensation to all modalities in
B. The pain commonly follows the
a glove and stocking distribution to the
distribution of the trigeminal nerve
metacarpophalangeal joint (MCPJ) and knees
bilaterally. Ankle and knee reflexes are absent. C. The acute attack is frequently alleviated
by alcohol
Possible underlying diagnoses include the
D. Often occurs in the same month each
following, except:
year
A. Diabetes mellitus
E. Cluster headache is five times more
B. Alcoholism prevalent in males than females.
C. Porphyria
D. Chronic inflammatory demyelinating Answer & Comments
neuropathy Correct answer: C
E. Hereditary motor and sensory
neuropathy Cluster headache is a strictly unilateral,
excruciating headache with an abrupt onset
F. Myeloma
and cessation. The pain commonly follows the
G. Small cell carcinoma of the lung distribution of the trigeminal nerve, particularly
H. Guillain-Barre syndrome the orbital or temporal divisions. Prevalence is
one per 1000 with a male to female ratio of 5:1,
I. HIV infection and patients typically present in their 30s or
J. Motor neurone disease. 40s.
Pain is associated with cranial autonomic
Answer & Comments dysfunction. The International Headache
Society classification diagnostic criteria require
Correct answer: CJ

Dr. Khalid Yusuf El-Zohry – Sohag Teaching Hospital (01118391123) Page | 637
El-Zohry MRCP Questions Bank (Part 2) – Medical Masterclass 2010

the cluster attacks to be accompanied by at lignocaine (nasal drops or spray can


least one of the following, which have to be also be effective)
present on the pain side:  Prevention: to prevent further attacks
 Conjunctival injection during a cluster, steroids, methysergide
 (short-term only) or ergotamine can be
Lacrimation
used
 Miosis
 Long-term prevention: to prevent
 Ptosis clusters from occurring, verapamil and
 Eyelid oedema lithium can be used, or surgery can be
performed to interrupt the trigeminal
 Rhinorrhoea sensory or autonomic pathways.
 Nasal blockage
 Forehead or facial sweating. [ Q: 75 ] MasterClass Part2
Although these are typically transient (2010) - Neurology
symptoms and last only as long as the attack, A 30-year-old previously fit and well woman
ptosis or miosis may occasionally persist. presents with a 2-day history of progressive leg
Cluster headache has a striking circadian and weakness and bladder dysfunction.
circannual periodicity. Each attack may occur at Examination reveals a spastic paraparesis.
the same time of day or night and each cluster
may occur in the same month of each year. The What is the most important investigation for
attacks within a cluster usually last a few this patient?
minutes to several hours, and each cluster may A. Cerebrospinal fluid examination
last a few weeks to several months. Attack
B. MRI scan of the spine
precipitants include alcohol, nitroglycerine,
exercise, and an elevated environmental C. Electromyography and nerve conduction
temperature. studies

Symptomatic cluster headache occurs in D. Anti-ganglioside antibodies


infectious, vascular and neoplastic intracranial E. Creatine kinase
lesions.
Differential diagnoses of cluster headache Answer & Comments
include migraine and paroxsysmal hemicrania,
which is a syndrome that is clinically similar Correct answer: B
except more common in females, attacks are
briefer and more frequent, and it completely It is important to exclude acute spinal cord
responds to indomethacin. compression with an MRI scan of the spine with
this history and examination findings.
Treatment of cluster headache includes:
 General measures: alcohol avoidance
[ Q: 76 ] MasterClass Part2
during cluster bout
(2010) - Neurology
 Abortive agents: triptans (e.g.
subcutaneous or nasal sumatriptan), You are examining the pupils of a 48-year-old
oxygen (100% at 7-12 litres per minute woman in whom the diagnosis of multiple
for at least 15-20 minutes), topical sclerosis is suspected.

Dr. Khalid Yusuf El-Zohry – Sohag Teaching Hospital (01118391123) Page | 638
El-Zohry MRCP Questions Bank (Part 2) – Medical Masterclass 2010

Which of the following observations would down to his current and previous neuroleptics.
make you conclude that she has a left relative Recently, the man is failing to mobilise, is
afferent pupillary defect? becoming even slower and is starting to shake.
A. You move a light from her left eye to her
What clinical feature would be most suggestive
right eye, and the left pupil constricts.
that he has developed super-added idiopathic
B. You move a light from her right eye to her Parkinson's disease?
left eye, and the left pupil dilates.
A. His tremor is most disabling when he
C. You flash a light on and off in her right drinks his tea
eye, and the left pupil constricts, but it
B. The glabellar tap is positive
does not when you flash it on and off in
her left eye. C. He has marked orofacial dyskinesias

D. You move a light from her right eye to D. He has a marked left-sided arm, chin and
her left eye, and the left pupil constricts. leg tremor at rest

E. You move a light from her left eye to her E. He has severe symmetrical bradykinesia
right eye, and the left pupil dilates.
Answer & Comments
Answer & Comments Correct answer: D
Correct answer: B
Parkinson's disease is almost always
A relative afferent pupillary defect (RAPD) is asymmetrical at onset. A tremor that is worse
detected using the 'swinging light test'. It allows while drinking tea is an intention tremor; the
comparison of pupillary constriction generated glabellar tap is not a useful sign; and orofacial
through the afferent pathway (within the retina dyskinesias are features most likely due to
or optic nerve anterior to the chiasm) of each neuroleptics in this case.
eye with that produced consensually.
After checking for direct and consensual [ Q: 78 ] MasterClass Part2
pupillary responses in each eye, proceed as (2010) - Neurology
follows. Shine a bright light into the right eye
A 28-year-old woman is found to have
for five seconds. Look at the pupil of the left
nystagmus.
eye. Move the light swiftly to shine in the left
eye. If the left pupil dilates, there is a left RAPD. Which one of the following statements about
Now observe the pupil of the right eye. Move nystagmus is NOT true?
the light back to the right eye. If the right pupil
dilates there is a right RAPD A. It is described by the direction of the fast
phase

[ Q: 77 ] MasterClass Part2 B. Cerebellar disease commonly causes


pendular nystagmus
(2010) - Neurology
C. When occurring as a result of a peripheral
An 82-year-old man has suffered from severe
lesion, e.g. acute vestibular neuronitis, it
depression for most of his life and has recently
is reduced by fixation and is associated
been treated with quetiapine for psychosis. His
with symptoms of vertigo
psychiatrist has noticed that he has been
parkinsonian for several years and had put this D. Downbeat nystagmus indicates a
medullary-cervical junction lesion
Dr. Khalid Yusuf El-Zohry – Sohag Teaching Hospital (01118391123) Page | 639
El-Zohry MRCP Questions Bank (Part 2) – Medical Masterclass 2010

E. Causes of multidirectional gaze-evoked Four vessel angiography will be needed later


nystagmus include multiple sclerosis and once the patient is stabilised to identify the
excess anti-epileptic medication. source of the bleeding and surgical or
endovascular treatment.
Answer & Comments In the acute stages of SAH (because of the
Correct answer: B speed of investigation and availability) CT
imaging is superior to MRI. MRI imaging
becomes much more useful if presentation is
Pendular nystagmus is caused by an inability to
delayed after the first few days (>4) when CT
fixate, commonly congenital blindness.
sensitivity for subarachnoid blood rapidly
declines.
[ Q: 79 ] MasterClass Part2
MR angiography is without risks and reasonably
(2010) - Neurology sensitive (90%) – useful for screening people at
A 35 year-old women presented 2 hours after risk of intracranial aneurysms but less suitable
collapsing at home with severe headache. On for patients with subarachnoid haemorrhage.
examination she was drowsy and had neck
stiffness. Her temperature was 37.5. She had a [ Q: 80 ] MasterClass Part2
mild right hemiparesis. (2010) - Neurology
Which of the following is the most appropriate A 39-year-old keen camper presents with a two
first diagnostic investigation? week history of headaches, a 'droopy face',
A. CT brain scan joint pains and a low grade temperature. He
had returned from a holiday in Amsterdam a
B. Lumbar puncture and examination of the month earlier. The only abnormality on
cerebrospinal fluid neurological examination is shown (see image).
C. Four-vessel angiography
D. MR angiogram
E. MR brain scan.

Answer & Comments

Correct answer: A

The history is suggestive of a subarachnoid


haemorrhage. Urgent CT brain scan will identify
more than 95% of patients with suspected
subarachnoid haemorrhage if performed within
1-2 days after headache onset (van Gijn 1982).
Lumbar puncture is potentially dangerous and
will add no extra information if brain CT shows
definite evidence of extravasated blood. If CT is
negative and there are no contraindications, LP
should be performed.

Dr. Khalid Yusuf El-Zohry – Sohag Teaching Hospital (01118391123) Page | 640
El-Zohry MRCP Questions Bank (Part 2) – Medical Masterclass 2010

A lumbar puncture was carried out and the A. Cryptococcal meningitis


results of the CSF analysis are as follows: 200
B. Herpes simplex encephalitis
cells/µl, over 95% lymphocytes, protein 1.4 g/l,
glucose 2.5 mmol/l (serum glucose 8.0 mmol/l). C. Herpes simplex meningitis
D. Japanese B encephalitis
What is the most likely diagnosis?
E. Listeria monocytogenes meningitis
A. Borrelia burgdorferi meningitis
F. Meningococcal meningitis
B. Carcinomatous meningitis
G. Mumps meningitis
C. Herpes simplex encephalitis
H. Ruptured mycotic aneurysm
D. Listeria monocytogenes meningitis
I. Subarachnoid haemorrhage
E. Tuberculous meningitis.
J. Tuberculous meningitis.
Answer & Comments
Answer & Comments
Correct answer: A
Correct answer: AJ
Lyme disease occurs throughout the USA,
Europe and the former Soviet Union. It is a tick- The clinical features of insidious onset,
borne disorder that results from systemic meningeal irritation and cerebrospinal fluid
infection with a spirochaete. Lymphocytic (CSF) findings of a lymphocytic pleocytosis, high
meningitis is a common neurological protein and low glucose make fungal and
abnormality occurring several months after the tuberculous difficult to
initial tick bite. Other neurological distinguish. Therefore Ziehl-Neelsen stain,
abnormalities may occur, e.g. a left lower polymerase chain reaction for Mycobacterium
motor neurone seventh nerve palsy, as in this tuberculosis, cryptococcal antigen and India ink
case. The CSF usually shows a lymphocytic stain should be carried out on the CSF. A chest
pleocytosis, slightly elevated protein and radiograph to look for pulmonary tuberculosis
normal glucose. will also be helpful.

[ Q: 81 ] MasterClass Part2 [ Q: 82 ] MasterClass Part2


(2010) - Neurology (2010) - Neurology
A 60-year-old alcoholic presents with a 4-week A 30-year-old man presents with a 3-year
history of increasing lethargy, headache and history of severe, episodic, unilateral
confusion. On examination, he was confused, auriculotemporal pain. Paroxysmal hemicrania
had signs of mild neck stiffness and is suspected.
photophobia. Cerebrospinal fluid analysis
revealed the following results: Opening Which of the following is NOT a feature of
pressure: 35 cm H2O White cells: 250 per mm3 paroxysmal hemicrania?
(80% lymphocytes, 20% polymorphs) Protein:
A. Episodes occurring up to 30 times per day
1.5 g/l Glucose: 2.0 mmol/l (serum glucose 8.0
mmol/l) Gram stain: Negative B. Attacks aborted by high flow oxygen
C. Clear response to indomethacin
Which of the following are the TWO most likely
diagnoses? D. Attacks lasting 10 minutes

Dr. Khalid Yusuf El-Zohry – Sohag Teaching Hospital (01118391123) Page | 641
El-Zohry MRCP Questions Bank (Part 2) – Medical Masterclass 2010

E. Ipsilateral lacrimation during attacks Answer & Comments

Correct answer: A
Answer & Comments

Correct answer: B The rest tremor of Parkinson's disease


commonly starts asymmetrically and remains
Paroxysmal hemicrania is characterized by so. Benign essential tremor is usually fairly
multiple, brief, intense, daily focal head pain symmetrical, worse on using the affected limb,
attacks. The pain is unilateral and always affects has a younger age of onset and tends to be
the same side. The pain is usually most severe reduced by alcohol.
in the auriculotemporal area, the forehead, and
above or behind the ear. The pain is described [ Q: 84 ] MasterClass Part2
as excruciating, throbbing, boring, or pulsating. (2010) - Neurology
The attack profile of paroxysmal hemicrania is
highly characteristic. The frequency of attacks A recently married 26-year-old woman presents
ranges from 2 to 30 per 24-hour period. Attacks with a 2-month history of recurrent episodes of
usually last between 2 and 25 minutes, and a rising epigastric sensation followed by lip-
occasionally as long as 60 minutes. smacking and chewing movements with loss of
awareness.
Ipsilateral lacrimation is reported in 62% of
attacks. Conjunctival injection, ipsilateral nasal The most likely diagnosis and the most
congestion and rhinorrhea are also seen. appropriate investigation are:
Photophobia may accompany some attacks,
and increased forehead sweating, especially on A. frontal lobe epilepsy and EEG
the symptomatic side, is observed in a few B. migraine and anti-cardiolipin antibodies
patients. The treatment of chronic paroxysmal
C. transient ischaemic attacks and CT scan
hemicrania is prophylactic; oxygen is ineffective
as acute treatment. Indomethacin is the drug of D. multiple sclerosis and MRI brain scan
choice and has an absolute effect on symptoms. E. temporal lobe epilepsy and MRI brain
scan.
[ Q: 83 ] MasterClass Part2
(2010) - Neurology Answer & Comments

A man presents in clinic with tremor. Correct answer: E

Which of the following makes Parkinson's The diagnosis is temporal lobe epilepsy, most
disease more likely than benign essential
likely of mesial temporal lobe origin. The
tremor?
commonest cause is hippocampal sclerosis
A. Tremor worse on the left side which is readily identified on MRI. An EEG
would also be useful, but the clinical seizure
B. Normal writing
semiology is not suggestive of frontal lobe
C. Reduction of tremor with alcohol epilepsy.
D. Tremor worse on using the affected limb
E. Young age of onset

Dr. Khalid Yusuf El-Zohry – Sohag Teaching Hospital (01118391123) Page | 642
El-Zohry MRCP Questions Bank (Part 2) – Medical Masterclass 2010

[ Q: 85 ] MasterClass Part2 A. Polymyositis


(2010) - Neurology B. Inclusion body myositis
An 83-year-old man is admitted complaining of C. Myasthenia gravis
acute onset weakness in both legs and low back D. Dermatomyositis
pain for the previous 2 months for which he
had been taking codeine phosphate. On E. Myotonic dystrophy
examination, he was cachectic and pale. His
cranial nerves and upper limbs were normal. Answer & Comments
His hips and knees were weak. He had a sensory
Correct answer: B
level just below the umbilicus with extensor
plantar reflexes. Rectal tone was normal, but he
had a large, craggy prostate. Painless weakness and wasting with selective
involvement of long finger flexors and
Which is the most important investigation to quadriceps is characteristic of inclusion body
perform? myositis.
A. Bone scintigram
[ Q: 87 ] MasterClass Part2
B. Prostate specific antigen
(2010) - Neurology
C. Transrectal prostatic biopsy
A 45-year-old man is referred to you with an
D. MRI thoracic spine
assortment of neurological symptoms and a
E. Radiograph thoracolumbar spine. family history of Huntingdon's disease (HD).

Which TWO of the following are not typically


Answer & Comments
seen in HD?
Correct answer: D
A. paranoia
B. emotional lability
This man has cord compression and therefore
needs urgent imaging of his spine to facilitate C. upper motor neurone / pyramidal
early treatment with radiotherapy. The other weakness
tests are all useful, and may help with later
D. athetosis
management
E. homonymous hemianopia with macular
sparing
[ Q: 86 ] MasterClass Part2
(2010) - Neurology F. self-neglect
G. dysarthria
A 59-year-old man is referred to clinic with
painless muscle wasting and weakness. On H. memory impairment
examination, he has atrophy of his flexor I. bradykinesia
forearm muscles and quadriceps. Power is
diminished in wrist and finger flexors, but not J. psychosis
the extensors. In addition he has weakness of
hip flexion. The rest of his muscle power is Answer & Comments
preserved.
Correct answer: CE
What is the most likely diagnosis?

Dr. Khalid Yusuf El-Zohry – Sohag Teaching Hospital (01118391123) Page | 643
El-Zohry MRCP Questions Bank (Part 2) – Medical Masterclass 2010

Huntington's disease is an autosomal dominant B. The patient has an extradural


condition characterized by neurological and haematoma.
psychiatric disturbances. Age of onset is usually
C. The patient has meningitis.
in the fourth or fifth decade. Personality
change, depression, and psychosis with D. The patient has a subarachnoid
progressive dementia in association with haemorrhage (SAH).
clumsiness, twitching, chorea and gait E. The patient has brain tumour.
disturbance are typically seen. Parkinsonian
features become more prominent as the
Answer & Comments
disease progresses. Genetic abnormality (CAG
trinucleotide repeat) resides on chromosome 4. Correct answer: D
Brisk reflexes with clonus may be seen but
pyramidal / upper motor neurone weakness is The scan show hyperdensity throughout the
rare. Ocular motor dysfunction is common but subarachnoid space which is most consistent
field defects are not seen in HD. with a SAH.

[ Q: 88 ] MasterClass Part2 [ Q: 89 ] MasterClass Part2


(2010) - Neurology (2010) - Neurology
A 25-year-old man presents to A&E with severe A 17-year-old girl presents to her GP with a
headache. This headache is unusual and he is fever. She is diagnosed with a viral illness. The
investigated with a CT of his brain (see image). following day, her condition deteriorates. Her
temperature persists and she now complains of
a bad headache. She is sent to A&E but doctors
find it difficult to take her history because she
has difficulty expressing herself. She becomes
agitated. A CT scan is performed, but as this is
finishing she has a generalised seizure. The
radiographer reports seeing jerking of the right
arm before the seizure. The CT is normal, and
subsequent CSF examination demonstrates only
mild lymphocytosis.

What is the most likely diagnosis?


A. Listeria meningitis
B. Herpes simplex encephalitis
C. Meningococcal meningitis
D. Cerebral abscess
E. Acute disseminated
encephalomyelopathy.

Answer & Comments


Which of the following statements is true? Correct answer: B
A. The patient has a subdural haematoma.

Dr. Khalid Yusuf El-Zohry – Sohag Teaching Hospital (01118391123) Page | 644
El-Zohry MRCP Questions Bank (Part 2) – Medical Masterclass 2010

Although it is perfectly reasonable to consider The family history suggests an autosomal


all of these as a cause, herpes simplex dominant disease with anticipation, meaning
encephalitis should receive the strongest that the condition presents earlier and more
consideration. The difficulty with the history severely in succeeding generations.
taking may be due to aphasia, and the Huntington's is the only one of the diagnoses
likelihood is that the seizure started as a focal listed that is plausible.
motor
seizure (originating in the left hemisphere) [ Q: 91 ] MasterClass Part2
progressing to secondary generalisation. These (2010) - Neurology
focal features are characteristic of herpes
simplex encephalitis or possibly a brain abscess. (1) A 19-year-old man presents with a history
The normal looking CT makes the latter less of a single tonic-clonic seizure, the morning
likely. The CSF findings make meningitis less after a party. On direct questioning, he also
likely. Listeria should be considered in reports the occurrence of occasional blank
immunocompromised patients, and during spells and brief jerking of his upper limbs
pregnancy. several times a month.

Which of the following drugs is most suitable?


[ Q: 90 ] MasterClass Part2
A. Phenytoin
(2010) - Neurology
B. Carbamazepine
You are asked to see a 35-year-old man, a
C. Sodium Valproate
heavy smoker, who is currently being treated
with clozapine by the psychiatrists for a major D. Gabapentin
depressive illness. He has had several other
E. Clonazepam.
neuroleptics in the past. The psychiatrists have
noticed that over the past 4 months he has
developed a strange jerking of his arms and Answer & Comments
neck. He never knew his father, who died in a Correct answer: C
psychiatric hospital when he was only 2 years
old, and his paternal grandfather was also said
The diagnosis here is juvenile myoclonic
to be ‘twitchy', although he died of a heart
epilepsy (JME), and valproate is the treatment
attack in his sixties. He has lost contact with his
of choice. The chance of seizure recurrence
two sons and is divorced.
with this type of epilepsy is high so treatment is
warranted. Phenytoin and carbamazepine are
What is the likely diagnosis
useful only for partial seizures or generalised
A. Tardive dyskinesia tonic-clonic seizures. They are ineffective, or
B. Huntington's chorea may in fact worsen, myoclonic jerks and
absence seizures. Gabapentin is licensed only
C. Inherited Creutzfeldt-Jakob disease with
for use in partial epilepsy. Clonazepam is
myoclonus
effective treatment for myoclonic jerks, but less
D. Wilson's disease effective and more sedating than valproate in
absences and generalised tonic-clonic seizures.
E. Sydenham's chorea
An alternative to valproate in JME, particularly
in women of child-bearing age, is lamotrigine
Answer & Comments

Correct answer: B

Dr. Khalid Yusuf El-Zohry – Sohag Teaching Hospital (01118391123) Page | 645
El-Zohry MRCP Questions Bank (Part 2) – Medical Masterclass 2010

[ Q: 92 ] MasterClass Part2 [ Q: 93 ] MasterClass Part2


(2010) - Neurology (2010) - Neurology
A 65-year-old gentleman is reviewed on the The 45-year-old man shown (see image) is
post take ward round. He has had a severe admitted with ptosis, diplopia and dysphagia.
headache for the last 3 weeks with no response On examination he is found to have fatigable
to regular paracetamol. He has lost weight and muscle weakness and normal deep tendon
generally felt unwell for the last 6 weeks. He reflexes.
has noted a lot of jaw pain whilst chewing his
food over the last 4 weeks. He did not want to
bother his general practitioner but had
suddenly noted problems with his right vision
and felt he had no choice but to seek help. His
ESR on admission is 96mm/hr.

Which of the following is correct?


A. It is unlikely that his pain will improve in
response to steroids within the first 24
hours.
B. Visual loss in temporal arteritis occurs in
less than 5% of cases.
C. Weight loss, sweating and general
malaise do not occur in temporal arteritis.
D. Pain is characteristically worse when
eating (jaw claudication).
E. Anaemia does not occur in temporal
arteritis.
Which of the following blood tests is most likely
Answer & Comments to confirm the underlying diagnosis?

Correct answer: D A. Acetylcholine receptor antibodies


B. Anti GM1 antibodies
Normochromic normocytic anaemia does occur C. Anti GQ1b antibodies
in temporal arteritis.
D. Anti Hu antibodies
Visual loss occurs in 25% of untreated cases.
E. Anti Purkinje cell antibodies.
Weight loss, sweating and general malaise
occurs in temporal arteritis.
Answer & Comments
Pain is characteristically worse on eating (jaw
Correct answer: A
claudication).
Characteristically the headache subsides within
Acetylcholine receptor antibodies are present
hours of the first large dose of steroid.
in about 80% of patients with generalized
myasthenia gravis

Dr. Khalid Yusuf El-Zohry – Sohag Teaching Hospital (01118391123) Page | 646
El-Zohry MRCP Questions Bank (Part 2) – Medical Masterclass 2010

[ Q: 94 ] MasterClass Part2 A sixth nerve palsy inactivates the lateral rectus


(2010) - Neurology muscle, causing double vision with horizontal
separation of images that is most apparent
A 22-year-old man presents with a four-day when the patient attempts to look to the
history of worsening diplopia. affected side.

In the affected eye, which TWO of the following


features would support the diagnosis of a third [ Q: 95 ] MasterClass Part2
nerve palsy? (2010) - Neurology
A. Eye is directed downwards and medially A 35-year-old woman presents with gradual
at rest onset headache and visual obscurations. On
B. Loss of consensual light reflex when light examination, she has mild bilateral
is shone into affected eye papilloedema.

C. Inward rotation of the eye on attempted Which of the following is not a cause of the
down-gaze patient’s condition?
D. Pain in the eye A. Acetazolamide
E. Eye is directed upwards and laterally at B. Cyclosporine
rest
C. Minocycline
F. Dilated pupil
D. Prednisolone
G. Eye is directed upwards and medially at
E. Vitamin A.
rest
H. Nystagmus on attempted lateral gaze Answer & Comments
I. Constricted pupil
Correct answer: A
J. Lack of ptosis
The patient has benign intracranial
Answer & Comments hypertension which can be induced by all of the
above except for acetazolamide which is in fact
Correct answer: CF
a treatment for the condition.

In a third nerve palsy there is ptosis, the eye


[ Q: 96 ] MasterClass Part2
points ‘down and out’ and the pupil is dilated.
On attempted downgaze the eye rotates (2010) - Neurology
inwards (best seen by looking at a conjunctival A 30-year-old patient presents with episodes of
vessel) due to action of the superior oblique muscle pain after several minutes of exercise
muscle (innervated by the IVth cranial nerve) and poor exercise tolerance. However, he
on an eye that is abducted. describes a ‘second wind’ phenomenon if he
A fourth nerve palsy inactivates the superior exercises through the initial barrier.
oblique muscle, causing double vision with
vertical separation of images that is most Which TWO of the following are the most likely
marked when the patient attempts to look diagnoses?
down, e.g. when reading or walking down A. Acid maltase deficiency
stairs.
B. Becker’s muscular dystrophy

Dr. Khalid Yusuf El-Zohry – Sohag Teaching Hospital (01118391123) Page | 647
El-Zohry MRCP Questions Bank (Part 2) – Medical Masterclass 2010

C. Carnitine palmitoyl transferase deficiency D. A common peroneal nerve lesion will


cause weakness of ankle dorsiflexion and
D. Inclusion body myositis
eversion but will not affect inversion,
E. Lambert-Eaton syndrome which will be affected in an L5 root lesion.
F. Motor neurone disease E. The two conditions cannot be
G. Myasthenia gravis distinguished clinically, but can by nerve
conduction studies.
H. Myophosphorylase deficiency
I. Myotonic dystrophy Answer & Comments
J. Phosphorylase b kinase deficiency. Correct answer: D

Answer & Comments A common peroneal nerve lesion leads to foot


Correct answer: HJ drop with loss of ankle and toe dorsiflexion and
ankle eversion. Numbness occurs over the
lateral aspect of the lower leg and dorsum of
Myophosphorylase deficiency (McArdle’s
the foot. It is usually due to pressure over the
disease) typically presents in the second and
fibular head.
third decade. The ‘second wind’ phenomena is
characteristic and is the result of diversion of An L5 root lesion causes weakness of ankle
blood flow to muscle and the onset of fatty acid dorsiflexion, inversion and eversion and
metabolism. The same clinical picture is seen dorsiflexion of the great toe. Numbness occurs
with phosphorylase b kinase deficiency down the posteriolateral aspect of the thigh,
lateral aspect of the calf and across the dorsum
of the foot to the great toe.
[ Q: 97 ] MasterClass Part2
(2010) - Neurology Loss of the ankle jerk occurs in lesions of S1.

A 38-year-old woman has weakness of her right


foot. You are trying to decide whether she has a
[ Q: 98 ] MasterClass Part2
common peroneal nerve lesion or an L5 root (2010) - Neurology
lesion. You are called to see a 65-year-old patient with
hypertension and elevated urea and creatinine
Which of the following statements is true?
who is undergoing dialysis.
A. An L5 root lesion will cause sensory and
motor symptoms, but a common Which TWO responses are not recognized
peroneal nerve lesion will only cause neurological sequelae of chronic renal disease?
motor symptoms. A. altered mental status
B. A common peroneal nerve lesion will B. seizures
cause weakness of ankle dorsiflexion,
eversion and inversion, but an L5 root C. muscle cramps
lesion not affect inversion. D. restless legs
C. An L5 root lesion will cause loss of the E. autonomic dysfunction
ankle jerk, but a common peroneal nerve
F. muscle weakness
lesion will not.
G. pure motor neuropathy

Dr. Khalid Yusuf El-Zohry – Sohag Teaching Hospital (01118391123) Page | 648
El-Zohry MRCP Questions Bank (Part 2) – Medical Masterclass 2010

H. foot drop B. Immediate lumbar puncture, to exclude


meningitis and subarachnoid
I. impotence
haemorrhage
J. optic atrophy
C. CT brain scan, followed by discharge if
normal
Answer & Comments
D. CT brain scan followed by lumbar
Correct answer: GJ puncture if CT normal
E. Immediate referral for cerebral
Chronic renal failure is associated with
angiography.
encephalopathy, the mechanism of which
remains unclear. Sensorimotor peripheral
neuropathy is a common complication leading Answer & Comments
to dysaesthesias, muscle cramps, restless legs Correct answer: D
and distal limb weakness. Autonomic
dysfunction leads to postural hypotension,
Despite the previous history of migraine, the
impotence and gastointestinal disturbances.
patient displays two concerning features:
Dialysis may cause rapid shifts of water into the
sudden onset and greater severity, which
brain resulting in headache, seizures, muscle
requires that subarachnoid haemorrhage (SAH)
cramps and nausea. Patients undergoing
be excluded. CT brain scan is normal in up to
dialysis for more than 1 year may develop a
30% of patients with SAH, so that a negative
fatal encephalopathy characterised by speech
scan cannot be used to exclude the diagnosis if
problems, hallucinations, gait disturbances etc.,
suspected clinically. However, a positive CT
possibly due to aluminium toxicity.
brain scan enables the diagnosis to be made
non-invasivally.
[ Q: 99 ] MasterClass Part2
(2010) - Neurology Absence of red blood cells, or the absence of
xanthochromia is sensitive for the exclusion of
A 35-year-old man presents to the A&E SAH. Where lumbar puncture (LP) is traumatic,
department complaining of a sudden onset of the presence of large numbers of red cells may
occipital headache, associated with vomiting, complicate interpretation. In this situation,
which started within the previous two hours spectophotometric analysis of the supernatant
and is continuing. The headache is much more for red cell pigments may be helpful.
severe than his usual migraine, which has not
Bilirubin and oxyhaemoglobin take 12 hours to
previously started suddenly. Examination,
appear in the cerebrospinal fluid (CSF) following
including fundoscopy is unremarkable, with no
a bleed, and for this reason LP should usually be
neck stiffness or photophobia.
delayed until more than 12 hours from the
Following prescription of analgesia and an onset of symptoms.
antiemetic, which of the following management Because of the invasive nature of cerebral
options would be the most appropriate? angiography, and relatively high complication
A. Reassurance that migraine is the most rate, it is usually reserved for patients whose
likely and discharge from the A&E other investigations are positive.
department after a short period of
observation

Dr. Khalid Yusuf El-Zohry – Sohag Teaching Hospital (01118391123) Page | 649
El-Zohry MRCP Questions Bank (Part 2) – Medical Masterclass 2010

[ Q: 100 ] MasterClass Part2 [ Q: 101 ] MasterClass Part2


(2010) - Neurology (2010) - Neurology
A 28-year-old man presents with pain radiating
to the front of his right thigh.

Which TWO nerve roots could be involved?


A. S1
B. L2
C. S3
D. L3
E. L5
F. S4
G. L1
H. L6
I. L4
J. S2

Answer & Comments

Correct answer: BD
A patient who has a scan as shown is most likely
to have which one of the following signs or
L2,3 radiates to the anterior thigh.
symptoms?
L4 radiates through the knee and down the
A. Pain on the outer aspect of the feet
medial side of the calf to the medial malleolus.
B. Pain radiating down both thighs
L5 radiates through the buttock, down the
C. Altered sensation in the pelvic area posteriolateral aspect of the thigh, lateral
D. Loss of the right ankle jerk aspect of calf and across the dorsum of the foot
to the big toe.
E. Loss of anal sensation.
S1 radiates through the inner buttock to the
posterior aspect of the thigh, posteriolateral
Answer & Comments
aspect of the calf to the lateral border of the
Correct answer: B foot

Outer aspect of the feet is S1 dermatome. The [ Q: 102 ] MasterClass Part2


ankle jerk is transmitted through S1. Anal (2010) - Neurology
sensation is S3 dermatome.
A 30-year-old man presents with a tremor. He
has had this for many years, but it has become
worse recently such that he now finds it socially
embarassing. His father had a similar problem.
His gait is normal.

Dr. Khalid Yusuf El-Zohry – Sohag Teaching Hospital (01118391123) Page | 650
El-Zohry MRCP Questions Bank (Part 2) – Medical Masterclass 2010

The most likely diagnosis is: C. Sub-arachnoid haemorrhage


A. Wilson's disease D. Sub-dural
B. Familial cerebellar degeneration E. Transient ischaemic attack.
C. Parkinson's disease
Answer & Comments
D. Huntington's disease
Correct answer: D
E. Benign essential tremor.

The recent minor head injury and fall mean that


Answer & Comments
a sub-dural must be excluded especially as the
Correct answer: E symptoms were intermittent. A CT scan of head
should be performed.
Benign essential tremor is a fine tremor that
often starts in childhood or adolesence, but [ Q: 104 ] MasterClass Part2
only presents later when it becomes (2010) - Neurology
functionally debilitating. The tremor is typically
worse on using the affected limb, e.g. using a A 40-year-old obese man is referred with
knife and fork or holding a cup. It can be excessive daytime sleepiness.
sporadic or inherited (autosomal dominant with
variable penetrance). The tremor is improved Which of the following is NOT a feature of
by alcohol. obstructive sleep apnoea?

Tremor is rarely the only symptom of cerebellar A. Intellectual deterioration


disease, and gait disorder would certainly be B. Morning headache
present if tremor was caused by cerebellar
C. Snoring
disorder.
D. Awareness of sleep disturbance
[ Q: 103 ] MasterClass Part2 E. Nocturnal enuresis.
(2010) - Neurology
Answer & Comments
An 72-year-old female is admitted by her GP
with intermittent confusion over the last 48 Correct answer: D
hours. She is on no regular medication. There is
no past medical history of note apart from a Snoring occurs in all patients with obstructive
simple fall complicated by a Colles fracture and sleep apnoea (OSA), and intellectual
minor head injury 3 weeks before. Clinical deterioration in 75%. Morning headache and
examination is normal, including mental state nocturnal enuresis occurs in approximately a
assessment. The night following admission she third of adult patients. Patients are not aware
is noted to be slightly confused. Routine bloods, of their sleep disturbance and a witness
resting ECG and chest radiograph are normal. description is necessary for the diagnosis to be
made.
Which of the following conditions must be
excluded?
[ Q: 105 ] MasterClass Part2
A. Alzheimer's disease
(2010) - Neurology
B. Intercurrent urinary tract infection
A patient presents with sensorineural deafness.

Dr. Khalid Yusuf El-Zohry – Sohag Teaching Hospital (01118391123) Page | 651
El-Zohry MRCP Questions Bank (Part 2) – Medical Masterclass 2010

Which of the following is not associated with brain is the imaging modality of choice and if
sensorineural hearing loss? positive, is sufficient to make the diagnosis
A. Basilar meningitis without CSF examination.

B. Benign positional vertigo


[ Q: 107 ] MasterClass Part2
C. Meniere’s disease (2010) - Neurology
D. Otosclerosis
A 68-year-old woman is referred to your clinic
E. Paget’s disease. following a single episode of loss of memory
lasting several hours, witnessed by her
Answer & Comments daughter. The patient has no recollection of the
episode but her daughter tells you her mother
Correct answer: B seemed bewildered and was constantly asking
questions such as "Where am I?" and "Why are
Benign positional vertigo is not associated with we here?". Consciousness was not impaired
hearing loss. Although otosclerosis commonly and there was no obvious disturbance of limb
causes conductive hearing loss, sensorineural power or sensation.
deafness is also seen.
What is the most likely diagnosis?
[ Q: 106 ] MasterClass Part2 A. Alzheimer's disease
(2010) - Neurology B. Vertebrobasilar transient ischaemic
attack
(2) A 35-year-old woman presents with
diplopia and unsteadiness. On examination she C. Simple partial seizure
has impaired adduction of the right eye looking
D. Transient global amnesia
left and horizontal nystagmus in the left eye.
E. Migraine
Which of the following is MOST likely to yield a
diagnosis? Answer & Comments
A. CT scan of the brain
Correct answer: D
B. Serum copper and caeruloplasmin
C. Nerve conduction studies This is a classic description of transient global
amnesia (TGA) which occurs most often in
D. Anti-acetylcholine receptor antibody titre
middle aged or elderly people. The patient is
E. Paired CSF and serum for oligoclonal alert and capable of high-level intellectual
bands activity and language function during the
attack. The pathogenesis of TGA has not been
Answer & Comments settled, it has been suggested that it represents
an unusual form of epilepsy (although seizure
Correct answer: E activity on EEG is rarely demonstrated), or has
an ischaemic basis (although the attacks rarely
This woman has internuclear ophthalmoplegia, progress to stroke). The benign nature of TGA in
which is most commonly seen in multiple most patients should be noted. Once
sclerosis. Oligoclonal bands are detected in the cerebrovascular disease and epilepsy have been
CSF but not serum in 90% of cases with ruled out, no
clinically definite multiple sclerosis. MRI of the

Dr. Khalid Yusuf El-Zohry – Sohag Teaching Hospital (01118391123) Page | 652
El-Zohry MRCP Questions Bank (Part 2) – Medical Masterclass 2010

treatment is required other than explanation of [ Q: 109 ] MasterClass Part2


the nature of the attack and reassurance. (2010) - Neurology

[ Q: 108 ] MasterClass Part2 In the clinical assessment of a 55-year-old man


(2010) - Neurology presenting with a tremor, which features would
suggest idiopathic Parkinson’s disease (PD)?
The 45-year-old man shown (see image) is
A. Symmetrical onset
admitted with ptosis, diplopia and dysphagia.
B. Positive family history
C. Improvement with alclohol
D. Worse when reaching for objects
E. Sudden onset of tremor
F. Increased tone with distraction
G. Progressive difficulty with handwriting
H. Tremor restricted to arms
I. Dizziness when standing
J. Nystagmus on horizontal gaze.

Answer & Comments

Correct answer: FG

PD is typically asymmetric at onset, and may


remain so for many years.

On examination he is found to have fatigable Rarely PD is familial, but benigh essential


muscle weakness and normal deep tendon tremor (BET) is typically familial.
reflexes. Improvement with alcohol is more suggestive of
BET.
What is the most likely diagnosis?
Intention tremor suggests cerebellar tremor.
A. Bell’s palsy
Sudden onset tremor suggests cerebrovascular
B. Guillain-Barre syndrome
disease.
C. Motor neurone disease
Increased tone (rigidity) is typical of PD.
D. Multiple sclerosis
Micrographia is often the presenting complaint
E. Myasthenia gravis. in PD.
Tremor restricted to arms is more suggestive of
Answer & Comments BET, and tremor involving the legs is highly
Correct answer: E suggestive of PD
Dizziness when standing suggests autonomic
Fatigable muscle weakness is found in involvement which would be more suggestive
myasthenia gravis. of multi-system atrophy.

Dr. Khalid Yusuf El-Zohry – Sohag Teaching Hospital (01118391123) Page | 653
El-Zohry MRCP Questions Bank (Part 2) – Medical Masterclass 2010

Nystagmus is not a feature of PD. Optic nerve involvement is not a feature of


motor neurone disease.
[ Q: 110 ] MasterClass Part2
(2010) - Neurology [ Q: 112 ] MasterClass Part2
(2010) - Neurology
A 70-year-old man is referred for increasing
forgetfulness. On closer questioning, he admits A 47-year-old patient with a past history of
to some urinary incontinence and unsteadiness relapsing-remitting multiple sclerosis presents
on walking. He smokes 40 cigarettes a day and to the acute medical take following a flare-up of
has been a heavy drinker in the past. his condition.

What is most likely diagnosis? Which of the following statements regarding


steroid treatment is untrue?
A. Alcoholic cerebellar degeneration
A. Corticosteroid treatment may limit the
B. Alzheimer’s disease
duration of visual loss due to optic
C. Frontotemporal dementia neuritis.
D. Multi-infarct dementia B. Avascular necrosis of the femoral head is
E. Normal pressure hydrocephalus. a recognised complication of
corticosteroid treatment.
Answer & Comments C. Corticosteroid treatment may reduce the
frequency of future relapses.
Correct answer: E
D. Oral corticosteroid treatment has no
impact on rate of recovery from a
The triad of dementia, urinary incontinence and
relapse.
gait disturbance is classically associated with
normal pressure hydrocephalus. E. 1g methylprednisolone daily for 3 days is
an appropriate dose.
[ Q: 111 ] MasterClass Part2
(2010) - Neurology Answer & Comments

A 68-year-old woman is diagnosed with motor Correct answer: C


neurone disease.
Corticosteroids reduce the duration of a
Which of the following would not be expected in relapse, including optic neuritis, but have no
this patient? effect on the disease progression or frequency
A. Atrophy of the tongue of relapses. Short courses of oral or intravenous
methylprednisolone have been shown to be
B. Emotional lability equally effective. Avascular necrosis is a
C. Footdrop recognised side effect.

D. Muscle cramps
[ Q: 113 ] MasterClass Part2
E. Optic atrophy.
(2010) - Neurology
Answer & Comments (2) A 72-year-old man with type 2 diabetes
mellitus complains of difficulty walking and
Correct answer: E
trouble with his hands. It began with a tingling

Dr. Khalid Yusuf El-Zohry – Sohag Teaching Hospital (01118391123) Page | 654
El-Zohry MRCP Questions Bank (Part 2) – Medical Masterclass 2010

sensation in his soles, which later extended up D. It has a higher frequency (Hz) than
to his ankles. He now feels unsteady when Parkinsonian tremor.
walking and more recently has noticed
E. It is helped by anticholinergics.
numbness and tingling in the fingers of both
hands. On examination he has absent ankle
reflexes, a high steppage gait and altered Answer & Comments
sensation to his mid-calves. Correct answer: E

What is the underlying pathological process?


Essential tremor (ET) is a common movement
A. Wallerian degeneration disorder of unknown pathogenesis. Its
B. Osteophyte formation and spinal canal prevalence has been quoted to range from one
stenosis to 22%, probably due to variations in definitions
of ET. It is more common than Parkinson's
C. Demyelination
disease (PD). It has a bimodal distribution, with
D. Axonal degeneration peaks in the second and sixth decades of life. A
family history has been reported in 17 to 100%
E. Anterior horn disease
of cases, the reason for the discrepancy being
that unless all symptomatic and asymptomatic
Answer & Comments members of a family are examined, the number
Correct answer: D of affected relatives will be underestimated. ET
has been mapped to several gene loci and more
are expected to be identified in the future.
This man has a distal, symmetrical sensorimotor
Recently, core criteria have been suggested for
neuropathy likely secondary to diabetes
the diagnosis of ET: (1) bilateral action tremor
mellitus. The ' dying back' pattern from distal to
of the hands and forearms, (2) absence of other
proximal is characteristic of axonal
neurological signs, except the cogwheel
degeneration. Wallerian degeneration occurs
phenomenon, and (3) may have isolated head
when the distal part of a nerve degenerates
tremor with no signs of dystonia. Secondary
after injury. Demyelination can occur anywhere
criteria include a long disease duration (more
on the nerve with concurrent distal and
than three years), a positive family history and
proximal signs and symptoms. Anterior horn
beneficial response to alcohol. Features which
disease would cause motor symptoms and signs
are against the diagnosis of ET, although not
only.
exclusively so, include unilateral, focal or leg
tremor, gait disturbance, rest tremor, rigidity,
[ Q: 114 ] MasterClass Part2 bradykinesia, sudden or rapid onset of the
(2010) - Neurology tremor, current drug treatment causing or
exacerbating the tremor and isolated head
A 48 yr old man is referred because he has a
tremor with abnormal posture (suggesting
tremor.
dystonia). However, there are suggestions that
Which one of the following features would NOT dystonia, parkinsonism and deafness are more
support the diagnosis of essential tremor? common in ET. The frequency of the tremor is
4-10 Hz (compared to 4-6 Hz in PD). Propranolol
A. It is an action tremor. and primidone are effective in the treatment of
B. It is ameliorated by alcohol. ET. If one drug does not control the tremor, a
combination of both drugs may be beneficial. In
C. It is associated with a positive family
severe cases refractory to medical treatment,
history.

Dr. Khalid Yusuf El-Zohry – Sohag Teaching Hospital (01118391123) Page | 655
El-Zohry MRCP Questions Bank (Part 2) – Medical Masterclass 2010

surgical options including thalamic stimulation feature of third nerve palsy, being absent in
and thalamotomy may be needed. some medical (e.g. diabetes) causes, but is seen
with compressive damage – and an aneurismal
[ Q: 115 ] MasterClass Part2 cause is likely in this woman.

(2010) - Neurology
[ Q: 116 ] MasterClass Part2
A 38-year-old woman presents with severe
(2010) - Neurology
headache. Your house physician tells you that
her ‘right eye is not normal’, and you suggest A known alcoholic presents to A&E with
that the patient might have a third nerve palsy. confusion. There are no signs of acute alcohol
intoxication. You suspect Wernicke's
The houseman asks you what signs he should encephalopathy.
look for to confirm this diagnosis. You reply:
A. there is ptosis, a small pupil, and the eye Which of the following features are not
compatible with this diagnosis?
is positioned ‘down and out’
A. bilateral lateral recti gaze palsy
B. there is a small pupil and the eye cannot
abduct B. disorientation
C. there is ptosis, a dilated pupil, and the C. hypothermia
eye is positioned ‘down and out’, but will
D. markedly elevated blood ammonia level
rotate inwards if the patient attempts to
look down E. seizures

D. there is a small pupil and the eye is F. postural hypotension


positioned ‘down and out’ G. apathy
E. there is ptosis, a dilated pupil, and the H. multidirectional nystagmus
eye is positioned ‘down and in’, and will
not move in any direction. I. memory loss
J. pupillary abnormalities.
Answer & Comments
Answer & Comments
Correct answer: C
Correct answer: DE
The third (oculomotor) nerve innervates all the
muscles controlling movement of the eyeball Wernicke's encephalopathy (WE) is seen in
excepting the superior oblique (fourth nerve) patients with a poor nutritional state, due to
and lateral rectus (sixth nerve). The superior either poor intake or absorption, or increased
oblique acts to depress the eyeball and the metabolic requirement of thiamine. It may be
lateral rectus to abduct it, hence the precipitated acutely in at-risk patients by
unopposed action of these muscles leads to the intravenous glucose administration. The main
eye being ‘down and out’. Action of the features are confusion, developing over days or
superior oblique on the abducted eye leads to weeks with inattention, apathy, disorientation,
inward rotation of the eye if the patient and memory loss. Coma is rare. Most patients
attempts to look down. have bilateral lateral recti palsies, occasionally
The third nerve carries parasympathetic fibres with other muscles involved, multidirectional
to the pupil, paralysis of which leads to nystagmus, and pupillary abnormalities. Truncal
papillary dilatation. This is not an invariable ataxia, and brain stem autonomic dysfunction

Dr. Khalid Yusuf El-Zohry – Sohag Teaching Hospital (01118391123) Page | 656
El-Zohry MRCP Questions Bank (Part 2) – Medical Masterclass 2010

are also seen. There are no distinctive developed a clumsy right hand and difficulty
laboratory findings; an elevated ammonia is speaking, the problem with his hand having
seen in hepatic encephalopathy. The mortality now persisted for 12 hours. He is known to
rate of treated WE is 10-20%. Often patients have hypertension, asthma and rheumatoid
are left with a disorder of impaired memory arthritis. In addition he suffered from migraines
and learning - Korsakoff's syndrome. Thiamine as a young man. He smokes a pipe. On
is the only treatment known to alter the examination, he has some weakness of the
outcome, and should be given parenterally in intrinsic hand muscles on the right only. Visual
the acute setting. fields, speech and sensation are normal. Blood
pressure is 190/90 mmHg.
[ Q: 117 ] MasterClass Part2
Which of the following is the most likely
(2010) - Neurology diagnosis?
A 73-year-old man is admitted with a 3-day A. Left hemisphere lacunar stroke
history of paraesthesiae in his hands and feet
B. Migraine equivalent
followed by progressive symmetrical ascending
weakness in his lower limbs. One week prior to C. Left middle cerebral artery territory
his current symptoms he had several days of cardioembolic stroke
diarrhoea. D. Left pontine microhaemorrhage

Which of the following treatments should be E. Neck-tongue syndrome.


started if the patient continues to deteriorate?
A. Intramuscular human tetanus Answer & Comments
immunoglobulin Correct answer: A
B. Intravenous immunoglobulin
C. Intravenous methylprednisolone The history suggests the dysarthria-clumsy
hand syndrome, one of the classic lacunar
D. Intravenous antibiotics syndromes that are strokes in the subcortical
E. Thrombolysis regions (or brain stem) secondary to small
vessel disease. The usual site of damage in the
Answer & Comments dysarthria-clumsy hand syndrome is the
internal capsule or pons, infarction being more
Correct answer: B common than haemorrhage although both are
caused by disease of small perforating
The patient is likely to have Guillain-Barré arterioles. Contributory risk factors include
syndrome. Intravenous immunoglobulin or smoking, hypertension and
plasmapheresis is the treatment of choice. hypercholesterolaemia.
Supportive measures such as intubation and Migraine equivalents (aura-like symptoms
ventilation may be needed as well. without headache) can provide diagnostic
difficulties and should always be considered in
[ Q: 118 ] MasterClass Part2 those with a previous history of migraine, but
(2010) - Neurology there is no suggestion of the characteristic slow
spread of symptoms in this patient.
The daughter of a 62-year-old man takes him to
The neck tongue syndrome consists of pain and
A&E where you are asked to see him to give a
parasthesias in one half of the tongue
medical opinion. Early that morning he

Dr. Khalid Yusuf El-Zohry – Sohag Teaching Hospital (01118391123) Page | 657
El-Zohry MRCP Questions Bank (Part 2) – Medical Masterclass 2010

precipitated by neck movement, often What first line treatment would you prescribe?
associated with occipital pain and ipsilateral
A. Carbamazepine
hand parasthesias.
B. Ethosuximide

[ Q: 119 ] MasterClass Part2 C. Phenytoin


(2010) - Neurology D. Sodium valproate

A 66-year-old man with diabetes crashes his car E. Topiramate.


whilst trying to park at the shops. He is taken to
A&E with minor cuts and bruises, but a Answer & Comments
thorough SHO finds a left superior
homonymous quadrantanopia. Correct answer: D

Where is the lesion? Approximately 90% of patients with juvenile


A. Optic chiasm myoclonic epilepsy become seizure free on
sodium valproate.
B. Left optic nerve
C. Lower fibres of the right optic tract [ Q: 121 ] MasterClass Part2
D. Upper fibres of the right optic tract (2010) - Neurology
E. Left occipital lobe A patient presents with weakness of knee
extension and ankle inversion.
Answer & Comments
Which of the following nerve roots is most likely
Correct answer: C affected?
A. L2
The lower fibres of the optic tract from the
inferior retinas swing in a wide arc over the B. L3
temporal horn of the lateral ventricle and C. L4
proceed posteriorly to join the upper fibres of
D. L5
the pathway on their way to the calcarine
cortex. A lesion that interrupts these fibres E. S1.
produces a superior homonymous
quadrantanopia. Lesions in the optic chiasm Answer & Comments
cause a bitemporal hemianopia and damage or
disease of one optic nerve causes a monocular Correct answer: C
visual impairment. This man needs
ophthalmalogical visual field testing and, if the L4 is involved in knee extension and ankle
visual field defect is confirmed, should be inversion.
advised not to drive and to inform the DVLA.
[ Q: 122 ] MasterClass Part2
[ Q: 120 ] MasterClass Part2 (2010) - Neurology
(2010) - Neurology
A 60-year-old man is referred to you with
A teenager is diagnosed with juvenile myoclonic stiffness and fluctuating confusion. His
epilepsy. symptoms began about three years ago when
he noticed stiffness in his legs. The stiffness

Dr. Khalid Yusuf El-Zohry – Sohag Teaching Hospital (01118391123) Page | 658
El-Zohry MRCP Questions Bank (Part 2) – Medical Masterclass 2010

gradually progressed and now affects all limbs. she uses nonsense and inappropriate words,
He also has an associated mild tremor affecting such that much of what she is saying does not
all limbs. He has noted that his writing has make sense.
become illegible. Over the last six months, he
has developed hallucinations and reports Which of the following would NOT cause such
seeing insects on the walls of his house. He is speech disturbance?
also increasingly forgetful at home. At night, his A. Herpes simplex encephalitis
wife says that he is often restless and agitated.
B. Brain metastases
On examination, his mini-mental score was
C. Myasthenia Gravis
15/30. Blood pressure fluctuated between 140-
160/80-100 with no consistent postural drop. D. Middle cerebral territory artery infarct
He is symmetrically rigid and slow with a tremor
E. Dominant hemisphere intracranial
in all limbs. Cranial nerve examination reveals
haemorrhage
mild restriction of conjugate upgaze eye
movement. His gait was shuffling with a
Answer & Comments
tendency to fall backwards.
Correct answer: C
What is the most likely diagnosis?
A. Benign essential tremor This woman has Dysphasia, which can be
B. Idiopathic Parkinson’s disease further classified as posterior or receptive
dysphasia. Dysphasia is a speech disorder
C. Lewy body dementia
caused by cortical lesions of the dominant
D. Multiple system atrophy hemisphere. Myasthenia gravis is an antibody
mediated autoimmune disease. It is
E. Progressive supranuclear palsy.
characterised by weakness and fatiguability of
proximal limb, ocular and bulbar muscles. The
Answer & Comments bulbar palsy causes difficulties with articulation
Correct answer: C - a disturbance of speech known as Dysarthria.

The combination of progressive cognitive [ Q: 124 ] MasterClass Part2


decline, fluctuating symptoms, visual (2010) - Neurology
hallucinations, extrapyramidal signs (rigidity
and bradykinesia more prominent than tremor) A 66-year-old man presents with a twelve-
and sleep pattern reversal suggest a diagnosis month history of dysarthria and dysphagia.
of Lewy body dementia.
Which TWO of the following statements are
true?
[ Q: 123 ] MasterClass Part2
A. A tensilon test is unlikely to be helpful.
(2010) - Neurology
B. The most likely diagnosis is a brain stem
You review a 72-year-old woman admitted on cerebrovascular accident (CVA).
the acute take with recent onset speech
C. Normal muscle strength in the arms
disturbance. As you speak with her, it quickly
excludes motor neurone disease (MND).
becomes apparent that she does not
understand what you are saying. The quality of D. Fasiculations in the tongue strongly
her voice is normal and her speech is fluent but suggest myaesthenia gravis.

Dr. Khalid Yusuf El-Zohry – Sohag Teaching Hospital (01118391123) Page | 659
El-Zohry MRCP Questions Bank (Part 2) – Medical Masterclass 2010

E. Assessment of respiratory function is D. He should be given a loading dose of


important. phenytoin 50mg/kg
F. NCS and EMG could be diagnostic. E. Physical examination is likely to show
flexor plantar response
G. MRI of the cervical spine is mandatory.
H. LP is mandatory.
Answer & Comments
I. The diagnosis could be Guillian Barre
Correct answer: C
syndrome.
J. Clinical tests of sensation are probably not Status epilepticus can be defined as either 2 or
relevant in this case. more seizures occurring consecutively without
an intervening period of full recovery of
Answer & Comments consciousness, or as a recurrent epileptic
seizure lasting for more than 30 minutes.
Correct answer: AF
The airway should be secured and oxygen
This patient turned out to have bulbar onset given, but no attempt to intubate should be
MND. The differential does include MG, arm made when the jaw is clenched.
strength can be retained, and tongue The patient should be transferred to an
fasciculations are highly suggestive of MND. intensive care unit.
The patient could have marked respiratory
The recommended loading dose for phenytoin
muscle weakness which would not be obvious
is 15mg/kg given IV. Further boluses up to a
clinically; this will influence the prognosis
loading dose of 30mg/kg may be given if
markedly. NCS/EMG after a tensilon test is the
seizures persist.
most useful test.
Physical examination will show extensor plantar
GBS is defined by a 4-week history.
response.
MRI and LP are more use in patients with signs
below the neck. Testing sensation is vital as it
[ Q: 126 ] MasterClass Part2
radically alters the differential diagnosis.
(2010) - Neurology
[ Q: 125 ] MasterClass Part2 A 60-year-old woman is asked to dorsiflex both
(2010) - Neurology her feet (see image). On examination, her
reflexes in the lower limbs are all normal.
A 32-year-old man has been fitting for more
than 35 minutes despite treatment with
intravenous lorazepam.

Which of the following statements is CORRECT?


A. The patient does not have status
epilepticus
B. The patient should be intubated to
establish a patent airway whilst fitting
C. The patient should be transferred to an
intensive care unit

Dr. Khalid Yusuf El-Zohry – Sohag Teaching Hospital (01118391123) Page | 660
El-Zohry MRCP Questions Bank (Part 2) – Medical Masterclass 2010

An abnormality affecting which of the following E. Chances of complete seizure control with
root lesions would be expected to give the antiepileptic drugs are small.
above clinical picture?
A. L2 Answer & Comments

B. L3 Correct answer: C
C. L4
Studies have shown that the age-specific
D. L5
incidence and prevalence of treated epilepsy is
E. S1. higher in elderly people than in any other age
group. Cerebrovascular disease (overt and
Answer & Comments occult) is by far the commonsest cause in this
age group. Cerebral tumours account for only a
Correct answer: D minority. A reliable history from the patient and
witness are more valuable than an EEG. Up to
A lesion of L5 causes weakness of ankle 38% of healthy old people may have abnormal
dorsiflexion, inversion and eversion and EEGs whilst few old people with seizures have
dorsiflexion of the great toe. A lesion of S1 abnormal inter-ictal EEGs. An EEG should
causes weakness of plantar flexion, eversion therefore be interpreted with caution.
and knee flexion, and an absent ankle jerk. Appropriate treatment will control seizures in
Therefore a foot drop in association with the more than 70% of affected patients.
loss of the ankle jerk will only occur with
involvement of both L5 and S1.
[ Q: 128 ] MasterClass Part2
(2010) - Neurology
[ Q: 127 ] MasterClass Part2
An 81-year-old man is brought to A&E by blue
(2010) - Neurology
light ambulance having collapsed in the
An 82-year-old man with a history of supermarket.
hypertension and atrial fibrillation is admitted
after two episodes of loss of conciousness
associated with generalized shaking. After a
period of drowsiness and confusion he is now
alert and orientated. There are no neurological
signs. There is no evidence of intercurrent
illness.

Which of the following statements is correct?


A. It is unusual for seizures to start for the
first time at this age.
B. A brain tumour is the most likely
underlying cause.
C. The most likely aetiology is occult
cerebrovascular disease.
D. An EEG can reliably differentiate seizures
from other causes of syncope.

Dr. Khalid Yusuf El-Zohry – Sohag Teaching Hospital (01118391123) Page | 661
El-Zohry MRCP Questions Bank (Part 2) – Medical Masterclass 2010

Urgent CT scan of the brain shows: D. Unilateral lower limb weakness with
contralateral loss of proprioception.
A. a crescent-shaped infarct
E. Bilateral distal weakness with loss of pain
B. lesion suggestive of a brain tumour
sensation in a “stocking” distribution.
C. haemorrhage in the putamen
D. primary brain haemorrhage and chronic Answer & Comments
subdural haematoma
Correct answer: A
E. a crescent-shaped infarct and signs of
cerebral atrophy.
Hemisection of the spinal cord, for example,
with transverse myelitis or following trauma,
Answer & Comments results in loss of appreciation of pain and
temperature contralateral to the lesion, with
Correct answer: C
loss of sensation for position and vibration, and
upper motor neuron paralysis ipsilateral to the
Remember blood appears white on a CT scan.
lesion. The dissociated sensory loss occurs
The putamen is the commonest site for
because of the different decussation levels of
hypertensive intracerebral haemorrhage. There
the dorsal column sensory fibres (brainstem)
is some space around the front of the brain
compared to the spinothalamic tracts (within
(especially contralateral to the bleed as the
spinal cord shortly after entering).
latter is causing some mass effect) but the
sulcal pattern is clear and the space CSF
density, suggesting that this is due to atrophy [ Q: 130 ] MasterClass Part2
rather than the presence of a subdural. (2010) - Neurology
You request an MRI brain scan on a patient with
[ Q: 129 ] MasterClass Part2 a neurodegenerative condition. Her husband
(2010) - Neurology asks for additional information on the safety of
MRI scanning.
You are on-call and a GP telephones you to
refer a patient with known relapsing-remitting Which of the following statements is NOT true?
multiple sclerosis and with what he believes
A. Modern metallic hip prostheses are MRI-
may be transverse myelitis. The GP reports his
compatible.
clinical findings after examining the lower
limbs. B. Prosthetic heart valves are MRI-
compatible.
The following suggests the presence of the
Brown-Sequard syndrome: C. Earrings should be removed before brain
scanning as they cause considerable
A. Unilateral lower limb weakness and loss image artefact.
of proprioception, with contralateral loss
of pain sensation. D. Cerebral aneurysm clips are an absolute
contra-indication to MRI scanning.
B. Bilateral lower limb weakness with loss of
sensation to all modalities and sphincter E. A maximum of three MRI scans should be
performed on any one patient per year.
disturbance.
C. Bilateral lower limb weakness with
Answer & Comments
unilateral loss of light touch and pain
sensation. Correct answer: E

Dr. Khalid Yusuf El-Zohry – Sohag Teaching Hospital (01118391123) Page | 662
El-Zohry MRCP Questions Bank (Part 2) – Medical Masterclass 2010

Absolute contra-indications to MRI include: The causes of proximal myopathy can be


 divided into inherited (e.g. Duchenne's,
aneurysm clips
Becker's, facio-scapular-humeral and myotonic
 cardiac pacemakers which can not be dystrophies), inflammatory (e.g. polymyositis,
safely switched off dermatomyositis, polymyalgia rheumatica,
 the presence of shrapnel or an inclusion body myositis), endocrine (e.g.
intraocular metallic foreign object. steroid-induced, thyroid disorders), metabolic
(e.g. glycogen sotrage disorders), toxic (e.g.
All modern metallic joint prostheses, and alcohol, chloroquine, clofibrate).
prosthetic heart valves (including metallic) are
MRI-compatible. Myasthenia gravis causes fatiguability and
commonly occurs in women in the second or
Jewellery is MRI-compatible but causes
third decade of life. B12 deficiency, and CIDP
significant image artefact if close to the area
are causes of neuropathy. Polymyalgia
being scanned. rheumatica presents with pain. Inclusion-body
MRI does not use ionising radiation and myositis presents in this age group with a
numerous scans are therefore safe. myopathy principally affecting the thigh and
forearm muscles, but is less common than
polymyositis and thyroid disorders.
[ Q: 131 ] MasterClass Part2
(2010) - Neurology
[ Q: 132 ] MasterClass Part2
A 60-year-old woman presents with difficulty (2010) - Neurology
climbing stairs and standing from a sitting
position. You examine her and find she has A GP refers to you a 30-year-old woman with a
evidence of painless proximal myopathy. 3-month history of diplopia and fatiguable
weakness.
The TWO most likely causes are:
Which of the following are not features of
A. Myasthenia gravis
myasthenia gravis:
B. Motor neuron disease
A. difficulty swallowing
C. Inclusion-body myositis
B. ptosis
D. Polymyositis
C. nasal speech
E. B12 deficiency
D. worsening of symptoms with
F. Thyroid disorder angiotensin-converting enzyme (ACE)
G. Spinal stenosis inhibitors

H. Myotonic dystrophy E. positive Tensilon test in 90% of patients

I. Chronic inflammatory demyelinating F. abnormal single-fibre electromyography


polyneuropathy G. pupillary abnormalities
J. Polymyalgia rheumatica. H. worsening of symptoms on initiating
high-dose steroids
Answer & Comments I. thymoma
Correct answer: DF J. worsening of symptoms with beta-
blockers.

Dr. Khalid Yusuf El-Zohry – Sohag Teaching Hospital (01118391123) Page | 663
El-Zohry MRCP Questions Bank (Part 2) – Medical Masterclass 2010

Answer & Comments A. Fluorescein angiography confirms the


presence of myelinated nerve fibres
Correct answer: DG
B. There is evidence of peripapillary
Myasthenia gravis (MG) is caused by an haemorrhage
antibody-mediated autoimmune attack against C. Common complications of fluorescein
acetylcholine receptor receptors at angiography include nausea and vomiting
neuromuscular junctions. Antiacetylcholine
D. Fluorescein angiography confirms
receptor antibodies are present in
leakage of the retinal vasculature
approximately 80% of patients with generalised
MG and approximately 50% of patients with E. The appearances on fundoscopy and
ocular MG. Seventy-five percent of patients fluorescein angiography are consistent
have thymic abnormalities, either hyperplasia with papilloedema
or less commonly, thymoma. Patients usually F. Retrobulbar neuritis is a common cause of
present with ptosis, diplopia, bulbar symptoms this abnormality in young adults
or limb weakness, but not fatigue per se.
Exacerbations are seen with intercurrent G. Anaphylaxis is a rare but recognised
infections, pregnancy, and certain drugs, such complication of fluorescein angiography
as aminoglycosides, beta-blockers, calcium H. An enlarged blind spot may be the only
antagonists etc. Steroids are an effective visual field defect
treatment but should be started at low doses
I. Possible causes of the abnormality shown
and increased slowly to avoid worsening of
include ischaemic optic neuropathy and
symptoms. The Tensilon test is positive in 90%
idiopathic intracranial hypertension
of patients but has been largely superseded by
single-fibre EMG and testing for the presence of J. a CT or MRI brain scan should be
antiacetylcholine receptor antibodies. Tendon performed before a lumbar puncture is
reflexes are normal, and there are no pupillary performed
abnormalities.
Answer & Comments
[ Q: 133 ] MasterClass Part2 Correct answer: AF
(2010) - Neurology
A 30-year-old woman presents with headaches. The images show optic disc swelling
See image for the fundoscopy (A) and (papilloedema) of which there are a number of
fluorescein angiography (B) that you perform. causes including: raised intracranial pressure
due to a space-occupying
lesion or idiopathic
intracranial hypertension,
ischaemic or inflammatory
optic neuropathy, or increased
CSF protein such as in Buillain-
Barre syndrome. There is also
evidence of a peripapillary
haemorrhage on the image.
Which of the following statements are NOT The cause in this patient was POEMS syndrome
true? (peripheral neuropathy, organomegaly,
endocrinopathy, M-protein, skin changes).

Dr. Khalid Yusuf El-Zohry – Sohag Teaching Hospital (01118391123) Page | 664
El-Zohry MRCP Questions Bank (Part 2) – Medical Masterclass 2010

Fluorescein angiography shows leakage of the unavailable. Carotid stenting may provide an
retinal vasculature. The blood-retinal barrier is alternative to endarterectomy in symptomatic,
usually impervious to fluorescein. Contra- severe ICA stenosis, especially in medically unfit
indications include recent myocardial patients such as those with cardiac failure.
infarction, pregnancy or allergic reaction to However, it has not yet been shown in
fluorescein. Resuscitation facilities must be randomised trials to be as safe or effective as
available. Common side effects include nausea endarterectomy therefore its use should be
and vomiting, local extravasation and restricted to ongoing trials. This man should be
thrombophlebitis. advised to stop smoking and his blood pressure
should be gently lowered to around 140/80
[ Q: 134 ] MasterClass Part2 mmHg by the addition of an ACE inhibitor to
the diuretic in the first instance.
(2010) - Neurology
A GP refers you a 72-year-old man with a [ Q: 135 ] MasterClass Part2
history of cardiac failure following detection of
(2010) - Neurology
a bruit on the right side of the neck. He is a
heavy smoker and is on aspirin, frusemide and A 67-year-old man is seen in A&E after falling
simvastatin. Investigation with Doppler down his stairs. He is noted to have some
ultrasound and MRA reveals a 90% stenosis of Parkinsonian features, but the astute SHO
the right external carotid artery and a 60% considers the possibility of progressive
stenosis of the left internal carotid artery. His supranuclear palsy (PSP).
blood pressure is 190/100mmHg.
Which of the following findings DO NOT support
What is the most appropriate management her diagnosis?
option?
A. Conjunctivitis
A. Right carotid endarterectomy
B. Reduction of upward gaze
B. Left carotid endarterectomy
C. Poor short term memory
C. Right carotid stent
D. Startled expression
D. Optimise medical treatment
E. Symmetrical akinesia.
E. Formal carotid angiogram
Answer & Comments
Answer & Comments
Correct answer: B
Correct answer: D
The primary diagnostic indicators include the
The 90% stenosis is in the right external carotid early onset of postural instability and vertical
artery (ECA) and is probably the cause of the gaze palsy. However, reduction in upgaze is
bruit. ECA stenosis does not require surgical non-specific as it may be present in a number of
intervention. The left ICA disease is other conditions or even normal ageing.
asymptomatic, is less than 70% stenosis and Paralysis of downgaze is very highly suggestive
should not be considered for intervention. of PSP although even this is not
Formal angiography carries approximately a 1% pathognomonic, as it has been reported in
risk of causing a stroke and should only be frontotemporal dementia. Other features of
performed where ultrasound and MRA results PSP include very reduced blinking, which may
are non-concordant, or when MRA is lead to dry eye and conjunctivitis, and

Dr. Khalid Yusuf El-Zohry – Sohag Teaching Hospital (01118391123) Page | 665
El-Zohry MRCP Questions Bank (Part 2) – Medical Masterclass 2010

retrocollis and frontalis over activity, which Answer & Comments


leads to a startled expression. The akinesia is
Correct answer: GJ
usually symmetrical from its onset, contrasting
with the asymmetrical findings in early
Parkinson's disease. There is a positive linear relationship between
both systolic and diastolic BP and the incidence
of any subtype of ischaemic or haemorrhagic
[ Q: 136 ] MasterClass Part2 stroke, at any age and in both sexes.
(2010) - Neurology
No threshold distinguishes patients who will
A 76-year-old man is admitted to hospital with have a vascular event from those who will not.
a transient ischaemic attack. He is found to be
The risk of stroke doubles for every 7.5mmHg
hypertensive with blood pressure 222/104
increase in diastolic BP. Hypertension increases
mmHg in outpatients 6 weeks later. He does
the risk of dementia from both vascular
not like taking tablets and wants to know why
dementia and Alzheimer’s.
he should be taking them.
Isolated systolic BP and pulse pressure are risk
Which TWO of the following statements are factors for stroke in older people. Angiotensin-
correct? converting enzyme inhibitors have been shown
A. Hypertension increases the risk of stroke to reduce the risk of stroke (HOPE STUDY).
and vascular dementia but not Antihypertensive treatment in patients who
Alzheimer's diseaseni have had a stroke has been shown to reduce
B. There is a linear relationship between the risk of further stroke (PROGRESS STUDY).
systolic and diastolic BP and stroke in Mid life hypertension predisposes to late life
men only. cognitive decline and dementia.
C. There is a linear relationship between
systolic and diastolic BP and stroke in [ Q: 137 ] MasterClass Part2
people over the age of 75 years (2010) - Neurology
D. A threshold of > 250 systolic BP
A 54-year-old woman presents with asymmetric
distinguishes patients who will stroke.
hand muscle bulk.
E. Pulse pressure is not important as a risk
factor in older people. Which TWO of the clinical signs listed would
make you suspect a T1 root lesion:
F. Isolated systolic BP is not a risk factor for
stroke in older people. A. combined wasting of 1st dorsal
interosseus and abductor pollicis brevis
G. Risk of stroke doubles for every 7.5mmHg muscles
increase in diastolic BP.
B. sensory loss over the whole palm
H. Reducing BP after a stroke does not
reduce the risk of further stroke. C. diminished finger reflexes

I. Angiotensin converting enzyme inhibitors D. loss of pain sensation over the shoulder
do not reduce the risk of stroke. and across the upper chest and back

J. Hypertension predisposes to late life E. muscle wasting in shoulder girdle


cognitive decline. F. ipsilateral Horner's syndrome
G. brisk triceps reflexes

Dr. Khalid Yusuf El-Zohry – Sohag Teaching Hospital (01118391123) Page | 666
El-Zohry MRCP Questions Bank (Part 2) – Medical Masterclass 2010

H. inverted biceps reflex A. It is an autosomal dominant condition


that shows anticipation
I. contralateral Horner's Syndrome
B. Patients may develop nodular thyroid
J. extensor plantars.
enlargement

Answer & Comments C. Diagnosis is made by DNA analysis

Correct answer: AF D. Grip myotonia is invariably present


E. Weakness can affect proximal and distal
A T1 root lesion will lead to wasting of all muscles
intrinsic hand muscles, diminished sensation
along the medial aspect of the arm, and will be Answer & Comments
associated with an ipsilateral Horner’s
Correct answer: D
syndrome. In all such patients a chest
radiograph is mandatory, whether they are
smokers or not, although this would heighten Myotonic dystrophy is an autosomal dominant
your suspicion that an apical lung tumour has condition where the gene has been identified at
caused the root compression. 19q13.1 in which there is an abnormal large
expansion of CTG trinucleotide repeats. This
Sensory loss over the whole palm in this
allows DNA analysis to be the first line
situation would suggest combined ulnar and
investigation in patients in whom the condition
median nerve involvement, and dissociated
is suspected. The disorder is worse in successive
cape distribution sensory loss is seen in intrinsic
generations (ie shows anticipation) and
cord lesions at around T1, e.g. syringomyelia,
previous generations may show mild
with the muscle wasting in this diagnosis
manifestations of the condition (eg this
caused by compression of the anterior horn
patient's father may have cataracts).
cells by the syrinx.
There are many complications of the condition
Finger reflexes are a C8 reflex, but are most
including glucose intolerance, cardiac
useful when brisk, rather than absent, which is
conduction abnormalities, cardiomyopathy and
difficult to distinguish from normal.
learning difficulties. A less well know
An inverted biceps reflex is a feature of a cord complication is the development of nodular
lesion at C5,6. thyroid enlargement in some patients. There
may be absence of grip myotonia in advanced
[ Q: 138 ] MasterClass Part2 disease because of progressive muscle wasting.

(2010) - Neurology
[ Q: 139 ] MasterClass Part2
An 18-year-old man presents with difficulty
(2010) - Neurology
using his hands. For the last four years he has
been working as a decorator but is finding this A patient presents to clinic with intention
increasingly difficult. The only family history of tremor of the left arm and a wide-based
note is that his father has very poor eyesight. unsteady gait.
On examination you find facial and hand muscle
weakness. He has difficulty opening and closing Where is the lesion likely to be?
his fists. You suspect myotonic dystrophy. A. Left pons

Which of the following is NOT a recognised B. Right globus pallidus


feature of myotonic dystrophy? C. Right primary cortex

Dr. Khalid Yusuf El-Zohry – Sohag Teaching Hospital (01118391123) Page | 667
El-Zohry MRCP Questions Bank (Part 2) – Medical Masterclass 2010

D. Right subthalamic nucleus E. Hereditary motor sensory neuropathy


E. Left cerebellar hemisphere.
Answer & Comments
Answer & Comments Correct answer: C
Correct answer: E
The short history, lack of reflexes and abnormal
ECG are the clues here. It can be difficult to
Lesions of the cerebellum lead to motor
objectively assess sensory symptoms (which are
disabilities ipsilateral to the side of the lesion.
often attributed to anxiety) and in GBS the
The tremor of Parkinson's disease is most
sensory examination may be normal early on or
prominent at rest, although postural tremors
if the condition is mild. The lack of reflexes
may be present. The gait is shuffling with the
however, is a hard sign that should not be
centre of gravity shifted forwards beyond the
ignored. In addition, autonomic involvement
base of the feet.
occurs in approximately 50% of patients with
Patients with extensive ischemic small vessel GBS and together with the respiratory
disease may present with a similar gait, often complications of the disease are the cause of
termed 'marche-a-petit-pas.' The steps are most morbidity and mortality.
short and, because patients are unsteady, the
The patient needs to be admitted and
feet may be a little more widely separated than
monitored for worsening autonomic problems
normal. However, the centre of gravity is over
and to ensure the early detection of paralysis or
the base of the feet. These patients do not
respiratory problems. Mild GBS does not
usually have tremors, but may have rigidity
require treatment, but if the patient becomes
similar to that seen in idiopathic Parkinson's
unable to walk, treatment is with a 5-day
disease.
course of intravenous immunoglobulin.

[ Q: 140 ] MasterClass Part2


[ Q: 141 ] MasterClass Part2
(2010) - Neurology
(2010) - Neurology
(1) An anxious 26-year-old woman attends the
A 40-year-old woman is referred with a 15-year
Emergency Department complaining of a 48-
history of unilateral throbbing headaches
hour history of tingling in both feet and all
associated with photophobia, phonophobia,
fingers, and clumsiness when walking. The
osmophobia, nausea, and visual aura.
doctor who first assessed her thought that the
neurological examination was normal,
Which of the following medications is NOT
excepting that they were unable to elicit any
effective in migraine?
tendon reflexes. Blood tests are normal and an
ECG shows sinus tachycardia only. A. Topiramate
B. Sodium Valproate
Which of the following is the MOST likely
diagnosis? C. Propranolol

A. Hyperventilation secondary to anxiety D. Sertraline

B. Multiple sclerosis E. Gabapentin

C. Guillian-Barre syndrome
Answer & Comments
D. Vitamin B12 deficiency
Correct answer: D

Dr. Khalid Yusuf El-Zohry – Sohag Teaching Hospital (01118391123) Page | 668
El-Zohry MRCP Questions Bank (Part 2) – Medical Masterclass 2010

Proven medications include: propranolol, [ Q: 143 ] MasterClass Part2


amitriptyline, pizotifen, gabapentin, (2010) - Neurology
topiramate, valproate, methysergide,
flunarizine and phenelzine. Although SSRIs are An 80-year-old woman had a funny turn and is
frequently prescribed, there is no evidence that sent by her GP to the rapid access transient
they are effective preventatives. ischaemic attack (TIA) clinic. When you take her
history she had only noticed one symptom.
[ Q: 142 ] MasterClass Part2 Which of the symptoms listed below occurring
(2010) - Neurology in isolation suggests a TIA?
A GP rings you about a 36-year-old man who A. Dysarthria
has presented with lower back pain, radiating B. Sensory symptoms confined to one part
to his buttocks, associated with lower limb of the face
parathesia over the last few days. He now has
difficulty walking, particularly on uneven C. Diplopia
surfaces. The GP is not sure he can elicit the D. Vertigo
reflexes.
E. Transient loss of consciouness.
The most likely cause is:
Answer & Comments
A. Acute lumbar disc prolapse
B. Vertebral collapse secondary to Correct answer: A
malignant infiltration
The history is essential in making the diagnosis
C. Acute porphyria
of a TIA, but non-focal signs such as loss of
D. Guillain Barré syndrome consciousness and some focal signs in isolation
E. Post infectious transverse myelitis. should not be interpreted as TIAs. Isolated
dysarthria is suggestive of TIA, but like an
isolated homonymous hemianopia may be
Answer & Comments
caused by TIAs in either the carotid or
Correct answer: D vertebrobasilar territory.

Guillain Barré syndrome is preceded by [ Q: 144 ] MasterClass Part2


respiratory or gastrointestinal symptoms in (2010) - Neurology
two-thirds, but not all, cases. Lower back pain,
often radiating to the buttocks occurs in 20- A 60-year-old man presents with a sudden
30% of cases, and results in a diagnosis of onset left hemiplegia. A CT scan of the head is
lumbar vertebral or disc disease being made in performed (see image).
many cases. Difficulty walking in this case is due
to distal weakness as suggested by the history Which of the following statements is true?
of difficulty walking or tripping on uneven A. The patient has a cerebellar infarct.
ground. This, together with the probable
B. The patient has a cerebellar
areflexia, suggest a peripheral neuropathy of
haemorrhage.
acute onset. Acute transverse myelitis may be
very difficult to distinguish in the first few days C. The patient has cerebellar tumour.
before upper motor neuron signs develop. D. No specific treatments are available if the
patient deteriorates.

Dr. Khalid Yusuf El-Zohry – Sohag Teaching Hospital (01118391123) Page | 669
El-Zohry MRCP Questions Bank (Part 2) – Medical Masterclass 2010

E. The patient should be commenced on D. The Apolipoprotein E (APOE) genotype is


aspirin. a risk factor for vascular dementia.
E. Only patients with large cerebral infarcts
Answer & Comments develop vascular dementia.
Correct answer: B F. About 50% of patients with ischaemic
stroke develop dementia.
The CT shows a high attenuation lesion most G. Patients with coronary artery bypass
consistent with a primary haemorrhage. grafting are not at increased risk of
An infarct would be hypodense. cognitve decline.

A tumour could underlie the bleed but is less H. Patients with cerebral infarction will
likely. always have evidence of neurological
deficit on examination.
The patient is at high risk of hydrocephalus.
Neurosurgery would be life-saving. All anti- I. Controlling cholesterol level does not
platelets and anti-coagulates should be reduce the prevalence of Alzheimer's
stopped. disease.
J. Vascular dementia may cause attention
[ Q: 145 ] MasterClass Part2 deficit and depression.
(2010) - Neurology
Answer & Comments
A previously fit 78-year-old woman is admitted
having been found to be ‘confused’. There is a Correct answer: DJ
past history of ischaemic heart disease but nil
else of note. Physical examination is unhelpful. Vascular dementia (VaD) and Alzheimer’s
Her 12 lead electrocardiogram shows an old disease (AD) are the most common causes of
anteroseptal myocardial infarction and left dementia. In many patients these conditions
ventricular hypertrophy. After a week’s co-exist.
observation and baseline investigations she has
Apolipoprotein E (APOE) genotype is a risk
a brain CT scan which shows periventricular
factor for VaD.
ischaemic changes and an old infarct. A
diagnosis of vascular dementia is relayed to the VaD patients suffer from global cognitive deficit
family, who want to speak to you in person and lead to attention deficit and depression.
about this. VaD encompasses patients with large
intracerebral infarctions, multiple small
Which of the following statements are correct?
infarctions, lacunar infarcts or leukoariosis.
A. Vascular dementia as a cause of
About 26% of patients with ischaemic stroke
dementia in older patients is very
develop dementia.
uncommon.
Up to 40% of patients have cognitive decline
B. Vascular dementia rarely co-exists with
following coronary artery bypass grafting
Alzheimer's disease.
(CABG).
C. Patients with vascular dementia have
Patients may have silent strokes with no
specific rather than global cognitive
neurological deficit on examination.
deficit.
Controlling cholesterol levels may reduce the
prevalence of AD.

Dr. Khalid Yusuf El-Zohry – Sohag Teaching Hospital (01118391123) Page | 670
El-Zohry MRCP Questions Bank (Part 2) – Medical Masterclass 2010

[ Q: 146 ] MasterClass Part2 E. Tectal lesion.


(2010) - Neurology
Answer & Comments
A 60-year-old man is found to have a parietal
lobe tumour. Correct answer: D

Which of the following would you expect to find A pontine lesion will cause bilateral miosis.
in this patient?
A. Ataxia [ Q: 148 ] MasterClass Part2
B. Bitemporal hemianopia (2010) - Neurology
C. Contralateral motor weakness A 14-year-old boy is admitted to intensive care
D. Contralateral sensory disturbance unit (ITU) having presented with status
epilpeticus. He continues to be treated with a
E. Dysphasia
combination of phenytoin and lorazepam and is
F. Memory disturbance closely monitored.
G. Neglect
Which of the following statements is CORRECT?
H. Nystagmus
A. EEG recording has no immediate role in
I. Personality change his ITU assessment
J. Spastic paraparesis. B. The finding of raised intracranial pressure
does not require treatment
Answer & Comments C. The normal maintenance dose of
phenytoin is about 100mg IV every 6 - 8
Correct answer: DG
hours

Common presentations of a parietal lobe D. The infusion rate for phenytoin is


tumour include contralateral sensory or cortical 100mg/minute as a maintenance dose
sensory loss, neglect, apraxia and contralateral E. Hyperthermia has no effect on prognosis
homonymous field defect (that sometimes
consists solely of lower quadrantanopia).
Answer & Comments

[ Q: 147 ] MasterClass Part2 Correct answer: C

(2010) - Neurology
Continous EEG recording can be used to assess
A 60-year-old man is brought into A&E after the effect of treatment on abnormal electrical
being found unconscious. Examination reveals activity.
bilateral pinpoint pupils.
Raised intracranial pressure may need to be
What would this suggest? lowered by using intermittent positive pressure
ventilation, dexamethasone, mannitol or
A. Bilateral oculomotor lesion sometimes surgery depending on the
B. Bilateral uncal herniation underlying cause.

C. Midbrain lesion The maintenance dose of phenytoin is about


100mg IV every 6 - 8 hours.
D. Pontine lesion

Dr. Khalid Yusuf El-Zohry – Sohag Teaching Hospital (01118391123) Page | 671
El-Zohry MRCP Questions Bank (Part 2) – Medical Masterclass 2010

The infusion rate should not exceed What is the diagnosis?


50mg/minute because rapid infusion can cause
A. Cervical spondylosis
cardiac arrhythmias.
B. Syringomyelia
Hyperthermia increases the risk of irreversible
brain damage and death. C. Epidural metastases from a primary
neoplasm

[ Q: 149 ] MasterClass Part2 D. Transverse myelitis


(2010) - Neurology E. Acute disc herniation.

A 50-year-old heavy smoker is referred to the


clinic with a six-month history of paraesthesiae Answer & Comments
in his fingertips, progressive weakening of grip Correct answer: A
strength in the upper limbs and general
stiffness in his arms and legs. He is a manual
The MRI scan shows multilevel degenerative
labourer and often involved in heavy lifting. On
change in the cervical region. There is severe
examination, he has increased tone in the
cord compression with myelopathic change due
upper and lower limbs, power of 4/5 for finger
to osteophytic indentations at the C3/4 and
flexion, finger extension and finger abduction
C4/5 levels.
and pathologically brisk reflexes with inverted
C5/6 jerks and extensor plantar responses. An Cervical spondylotic myelopathy is the most
MRI of his cervical spine is shown (see image). frequent cause of myelopathy in those over the
age of 50. It usually develops insidiously, but
may be associated with sudden acceleration,
especially following a slight head and neck
injury. Clinical findings include bilateral upper
motor neurone weakness of the legs
(paraparesis, paraplegia) or legs and arms
(quadriparesis, quadriplegia), bilateral
impairment of sensation with a 'level'
separating a region of normal sensation from a
region of impaired sensation (either
spinothalamic-mediated or posterior column-
mediated sensory modalities may be impaired),
and bowel or bladder sphincter dysfunction.
Sphincter dysfunction is usually preceded by
motor or sensory findings. Less commonly, neck
pain and Lhermitte's sign (electric shock-like
sensation down the spine with neck flexion)
may be present.
Cervical spondylotic myelopathy is more
common in congenitally narrow spinal canals
where disc protrusion, osteophytes,
ligamentum flavum hypertrophy, posterior
longitudinal ligament ossification and vertebral
body subluxations may all play a role in
compressing the cord.

Dr. Khalid Yusuf El-Zohry – Sohag Teaching Hospital (01118391123) Page | 672
El-Zohry MRCP Questions Bank (Part 2) – Medical Masterclass 2010

Decompressive surgery is usually considered to I. Nocturnal wandering


be an effective treatment in severe cases or
J. Concomitant diabetes.
rapid progression, but treatment is less clear-
cut in mild and moderate cases without rapid
progression. Some patients improve with Answer & Comments
conservative treatment such as intermittent Correct answer: BC
cervical immobilization with a soft collar, use of
anti-inflammatory medications and intermittent
Dementia with Lewy bodies is probably the
bed rest in patients with pain. Many variables
second commonest dementia after Alzheimer's
play a role in decompressive surgery, including
disease. The patients show features of
the risk of operating on an irrelevant stenotic
dementia, with Parkinsonism, psychiatric
process (e.g. in patients with motor neurone
symptoms and sometimes striking variation in
disease) and surgery-related morbidity and
performance or even conscious level. All
mortality. In slowly evolving cases, the best
dementias tend to produce loss of short-term
timing for surgery is also difficult to determine.
memory and nocturnal wandering, and may be
In a recently reported two-year prospective
associated with weight loss if the patient is
randomized trial of patients with slowly
supposedly self-caring. Stepwise deterioration,
progressive mild and moderate cervical
brisk reflexes and diabetes may be associated
spondylotic myelopathy, surgical treatment did
with vascular dementia. Early impairment of
not show better results than conservative
language is most marked in Primary Progressive
treatment over the follow-up period. In general,
Aphasia. Patients with progressive supranuclear
best surgical and non-surgical treatment results
palsy (PSP) are particularly prone to fall down
are seen in cases with mild neurological
stairs because of their impaired eye
impairment that is present for less than six
movements.
months, and when the age of the patient is
below 70.
[ Q: 151 ] MasterClass Part2
[ Q: 150 ] MasterClass Part2 (2010) - Neurology
(2010) - Neurology A 17-year-old man is wheelchair bound and has
respiratory difficulty. He has enlarged calf
A 77-year-old woman presents with her
muscles. Blood tests reveal an elevated creatine
husband who describes her dementing illness.
kinase level.
You diagnose dementia with Lewy bodies.
What is a muscle biopsy likely to show?
Which TWO features are the most helpful in
making this diagnosis? A. Excessive lipid storsge

A. Loss of short-term memory B. Absence of dystrophin

B. Markedly fluctuating conscious level C. Excessive glycogen storage

C. Hallucinations D. Vasculitic changes

D. Stepwise deterioration E. Necrotic muscle fibres

E. Brisk reflexes
Answer & Comments
F. Early impairment of language
Correct answer: B
G. A tendency to fall on stairs
H. Weight loss

Dr. Khalid Yusuf El-Zohry – Sohag Teaching Hospital (01118391123) Page | 673
El-Zohry MRCP Questions Bank (Part 2) – Medical Masterclass 2010

The patient has Duchenne's muscular B. Treatment with high dose oral thiamine is
dystrophy, which is associated with the absence likely to reverse the symptoms.
of dystrophin. Pseudohypertrophy of the calf
C. If the patient is hypoglycaemic,
muscles is characteristic of the condition.
intravenous thiamine should be
administered before administration of
[ Q: 152 ] MasterClass Part2 glucose.
(2010) - Neurology D. Hepatic encephalopathy is the most likely
A patient presents with weakness of elbow explanation for his symptoms.
extension. E. A single dose of intravenous thiamine is
usually sufficient for treatment of acute
Which of the following nerve roots is likely to be wernicke's encephalopathy.
affected?
A. C5 Answer & Comments
B. C6 Correct answer: C
C. C7
D. C8 The classical triad of confusion, ataxia and
ophthalmoplegia is present in only 10% of
E. T1 patients with Wernicke's encephalopathy;
confusion is the commonest symptom, present
Answer & Comments in over 80% of cases, and the condition may
mimic and complicate 'simple' alcohol
Correct answer: C
withdrawal. Treatment requires high doses of
intravenous thiamine to be given repeatedly, in
C7 supplies triceps and wrist extensors. A lesion order for passive diffusion to occur across the
of C5 causes weakness of the deltoid (shoulder blood brain barrier into the brain cells depleted
abduction) and infraspinatus; of C6 weakness of of B-vitamins. Oral absorption is unreliable,
biceps (elbow flexion) and brachioradialis; and particularly in patients who are nutritionally
of C8 and T1 weakness of intrinsic hand deficient and those who are continuing to drink
muscles. alcohol. Administration of glucose increases the
metabolic activity in brain cells and may
[ Q: 153 ] MasterClass Part2 exacerbate or precipitate Wernicke’s in patients
(2010) - Neurology with chronic thiamine deficiency.
Hepatic encephalopathy is unlikely in the
A 56-year-old alcoholic patients presents with a
absence of clinical evidence of liver disease.
1-week history of progressive confusion and
unsteadiness of his gait. On examination he has
no stigmata of chronic liver disease and no [ Q: 154 ] MasterClass Part2
evidence of portal hypertension. Eye (2010) - Neurology
movements and cerebellar function appear
normal. A 48-year-old man presents with a history of
excruciating pain in his left shoulder that woke
Which of the following statements is correct? him early one morning 2 weeks ago. Since then,
he has noticed difficulty abducting the right
A. Wernicke's encephalopathy is unlikely in
shoulder due to weakness. There was wasting
the absence of ophthalmoplegia.
of his right deltoid when he was examined.

Dr. Khalid Yusuf El-Zohry – Sohag Teaching Hospital (01118391123) Page | 674
El-Zohry MRCP Questions Bank (Part 2) – Medical Masterclass 2010

What is the most likely diagnosis?


A. Neuralgic amyotrophy
B. Guillain-Barré syndrome
C. Cervical disc prolapse
D. Syringomyelia
E. Polymyalgia rheumatica

Answer & Comments

Correct answer: A

The history of sudden onset of excruciating arm


(usually shoulder) pain, followed by shoulder
and/or parascapular muscle weakness several
days later is typical of neuralgic amyotrophy.
The pain is so severe it is often confused with
the pain of a myocardial infarction

Dr. Khalid Yusuf El-Zohry – Sohag Teaching Hospital (01118391123) Page | 675
El-Zohry MRCP Questions Bank (Part 2) – Medical Masterclass 2010

mg/d), allergic dermatitis / photosensitivity and

Psychiatry neuroleptic malignant syndrome

[ Q: 2 ] MasterClass Part2
(42 Questions) (2010) - Psychiatry
A 40-year-old woman is referred to the general
(Medical Masterclass – Part 2) medical clinic with a 9-month history of feeling
exhausted. She is unable to do anything that
requires physical exertion and feels miserable
because of this. A comprehensive battery of
[ Q: 1 ] MasterClass Part2
tests fails to establish any clear medical cause
(2010) - Psychiatry for her symptoms and you think that she has
A 23-year-old man, studying for a higher degree chronic fatigue syndrome.
in pharmacology, develops schizophrenia. You
Which one of the following treatments is of
suggest that he would benefit from treatment
definite benefit in this condition?
with thioridazine. He is very concerned about
the possibility of drug toxicity and produces a A. Graded exercise programme
long list of possible side effects. B. Prolonged rest
Which one of the following is NOT a recognised C. Corticosteroids
side effect of thioridazine? D. Benzodiazepines
A. Sedation E. Allergen avoidance
B. Jaundice
C. Leucopenia Answer & Comments

D. Orthostatic hypotension Correct answer: A


E. Diarrhoea.
Graded exercise programmes and cognitive-
behavioural therapy are the only two
Answer & Comments
treatments shown to be of definite benefit in
Correct answer: E chronic fatigue syndrome. Prolonged rest is
ineffective and tends to be harmful.
The side effects of antipsychotics include
extrapyramidal symptoms (stiffness, tremor, [ Q: 3 ] MasterClass Part2
hypersalivation, acute dystonia, akathisia, (2010) - Psychiatry
tardive dyskinesia), anticholinergic symptoms
(blurred vision, constipation, urinary retention, A 27-year-old anaesthetic nurse is suspected of
dry mouth, confusion, agitation, seizures), drug abuse. He is found at work delirious,
antihistaminergic symptoms (sedation), alpha- euphoric and talking about being able to “smell
blocking effects (orthostatic hypotension), sounds” he was hearing from the anaesthetic
leucopenia, increased prolactin secretion machine.
(amenorrhoea, galactorrhoea, sexual
dysfunction), weight gain, obstructive jaundice, Which of the following drugs is the most likely
retinitis pigmentosa (with thioridazine >600 to produce this clinical picture?
A. Pethidine

Dr. Khalid Yusuf El-Zohry – Sohag Teaching Hospital (01118391123) Page | 677
El-Zohry MRCP Questions Bank (Part 2) – Medical Masterclass 2010

B. Nitrous oxide Answer & Comments


C. Heroin Correct answer: EH
D. Cocaine
It can be difficult to differentiate between
E. Ketamine
depression and the sadness associated with an
impending and untimely death. Thoughts of
Answer & Comments guilt and self-worthlessness are usually found in
Correct answer: E those who are depressed, and it is unusual for
those who are sad not to gain some enjoyment
from some activities. Other questions should
Ketamine is commonly used as a recreational
include those about suicidal thoughts.
drug. Its sought-after effects are stimulation,
euphoria, depersonalisation, a floating feeling
and hallucinations. Other adverse effects [ Q: 5 ] MasterClass Part2
include synaesthesia (a sensory stimulus in one (2010) - Psychiatry
modality is perceived as a sensation in
another), delirium, vivid dreams and You are asked to see a very thin 18-year-old girl.
hallucinations. You suspect that she has anorexia nervosa.

On examination, which one of the following


[ Q: 4 ] MasterClass Part2 features would NOT be expected if this
(2010) - Psychiatry diagnosis were correct?

A 46-year-old with pancreatic cancer attends A. Tachycardia


clinic. Her pain is now well controlled on B. Hypotension
methadone; however, she spends much of the
C. Oedema
day in bed and is refusing food.
D. Lanugo
What are the most helpful factors in
E. Hypothermia
distinguishing the disease process and
appropriate sadness from depression?
Answer & Comments
A. Anorexia
Correct answer: A
B. Weight loss
C. Hypersomnia When examining this patient you should:
D. Social withdrawal  Establish her height and weight and
E. Feelings of worthlessness calculate her BMI
F. Nocturnal wakening  Look for signs of starvation:
hypothermia, lanugo, loss of muscle
G. Fatigue
mass, dependent oedema, bradycardia,
H. Anhedonia hypotension, neuropathy.
I. Reduction in normal activities  Look for signs that she is inducing
J. Thoughts of death. vomiting: salivary gland enlargement,
dental erosion, calluses on the fingers /
knuckles.

Dr. Khalid Yusuf El-Zohry – Sohag Teaching Hospital (01118391123) Page | 678
El-Zohry MRCP Questions Bank (Part 2) – Medical Masterclass 2010

[ Q: 6 ] MasterClass Part2 recommended in the National Institute for


(2010) - Psychiatry Health and Clinical Excellence (NICE) guidelines.
There is no evidence that dietary supplements,
A 30-year-old schoolteacher has been on sick exclusion diets or immunotherapy will treat
leave for 7 months since she had a bout of chronic fatigue syndrome, although medication
gastroenteritis. Her diarrhoea and vomiting had may be used to treat associated conditions, eg
stopped after 2 days, but she has felt exhausted depression.
ever since and is spending increasing amounts
of time resting in bed. She sleeps badly, her
[ Q: 7 ] MasterClass Part2
concentration is poor and she is worried about
swollen glands in her neck. A nutritionist has
(2010) - Psychiatry
advised her to eat no dairy or wheat products, A 74-year-old woman has been taking lithium
whilst her reflexologist has recommended a for 15 years. She developed acute
course of trace elements and other dietary gastroenteritis with vomiting and diarrhoea.
supplements to correct an alleged postviral Her family brought her to casualty and lithium
imbalance of her adrenal function. Extensive toxicity is suspected.
medical tests and investigations have failed to
demonstrate any significant abnormality. Which of the following would NOT be consistent
with lithium toxicity?
Which one of the following would contribute to
A. Muscle weakness
effective medical management in this case?
A. A conservative approach of waiting until B. Drowsiness
she makes a spontaneous recovery C. Ataxia
B. Encouragement to join an ME self-help D. A serum lithium level of 1.0 mmol/
group on the grounds that this would
E. Coarse tremor
improve the prognosis
C. Advise her to give up her teaching career Answer & Comments
because her incapacity is likely to be
permanent Correct answer: D

D. Recommendation for cognitive behaviour


In acute lithium toxicity there are two main
therapy and graded exercise
groups of symptoms, gastrointestinal and
E. Referral for immunotherapy neurological. Gastrointestinal symptoms
include nausea and vomiting. Neurological
Answer & Comments symptoms include muscle weakness, ataxia,
coarse tremor, muscle twitching, drowsiness,
Correct answer: D
slurred speech and disorientation. Severe
toxicity may result in seizures, myoclonus,
This is a fairly typical presentation of chronic choreiform movements, incontinence and
fatigue syndrome (myalgic encephalomyelitis or ultimately coma.
ME). A proportion of patients with chronic
fatigue syndrome develop this without a
preceding infection. The only treatments for
[ Q: 8 ] MasterClass Part2
which there is significant evidence of (2010) - Psychiatry
effectiveness are cognitive behaviour therapy A 22-year-old man is brought to hospital by his
and graded exercise treatment, and these are family after he threatened his father with a

Dr. Khalid Yusuf El-Zohry – Sohag Teaching Hospital (01118391123) Page | 679
El-Zohry MRCP Questions Bank (Part 2) – Medical Masterclass 2010

knife. This happened when his father [ Q: 9 ] MasterClass Part2


confronted him about the filthy state of his (2010) - Psychiatry
room. It transpires that for the past 3 years he
has been collecting and keeping his own hair, A 28-year-old man is brought to the Emergency
nail clippings and skin. He refuses to wash. Department from a nightclub where he had
When you interview him he tells you that he collapsed. Over the course of 4 hours he had
has intrusive thoughts that terrible harm will taken eight tablets of ecstasy.
come to him and his family. He spends hours
each day meticulously collecting his hair and Which of the following clinical observations is
skin in a ritualised way, which he says will likely to be found in a person who has taken
prevent these awful things happening. He has ecstasy?
no other abnormal beliefs or perceptions. A. Hyperthermia

Which one of the following is true? B. Hypotension

A. This man is more likely to be having C. Bradycardia


delusions than obsessions D. Slowed breathing
B. Patients typically describe obsessions as E. Dry skin
being alien thoughts inserted into their
minds Answer & Comments
C. It is unlikely that his man has obsessive-
Correct answer: A
compulsive disorder because the
thoughts he is having and the ritualised
behaviours do not have the same theme: Ecstasy is a stimulant and the main
this is more likely to be schizophrenia physiological effects of intoxication are
therefore sympathomimetic, namely,
D. Cognitive behaviour therapy would be hypertension, tachycardia, increased
the treatment of choice in this case, plus respiratory rate, hyperthermia and increased
a selective serotonin reuptake inhibitor sweating. The most frequent serious
E. Antipsychotic drugs are as effective in complications are dehydration and malignant
treating obsessions as delusions hyperthermia. Rare complications are cardiac
problems, liver failure and cerebral oedema.
Answer & Comments
[ Q: 10 ] MasterClass Part2
Correct answer: D
(2010) - Psychiatry
In obsessive-compulsive disorder, ritualised A frail 70-year-old woman who lives with her fit
behaviours serve to neutralise the obsessive husband has missed several outpatient
thoughts and thus do not necessarily have the appointments. Her husband rang to cancel
same theme, eg counting to prevent an them with a variety of explanations. She has a
accident to a loved one. Typically patients residual right-sided weakness and speech
recognise the obsessive thoughts are a product difficulties from a stroke. She is admitted to
of their own minds. Selective serotonin A&E with a fall and suspected hip fracture. The
reuptake inhibitors can be effective especially Casualty officer notices bruises of various ages
when combined with cognitive behaviour on her arms, particularly wrists, and suspects
therapy. elder abuse.

Dr. Khalid Yusuf El-Zohry – Sohag Teaching Hospital (01118391123) Page | 680
El-Zohry MRCP Questions Bank (Part 2) – Medical Masterclass 2010

Which TWO statements concerning this have multiple chronic illnesses which can mask
problem are FALSE? or masquerade as signs of abuse. There are no
A. Most physicians fail to screen for this diagnostic symptoms or signs of abuse but
problem. certain patterns should raise suspicions and
prompt further investigation. Potential clinical
B. Adult protection services are available to presentations include:
assist physicians deal with suspected
 Description of problem is inconsistent
cases.
with clinical findings
C. It is virtually confined to the affluent.
 Delay between development of injury
D. Physical disability is a risk factor. and seeking medical advice
E. The vast majority of perpetrators are not  Multiple bruises, particularly of
professional carers. different ages, and around the wrists,
F. Victims often remain silent. neck or inner thighs

G. Victims abused by spouses or intimate  Depression, anxiety, withdrawal.


partners have been chronically battered When addressing abuse by a spouse or partner
throughout the relationship. one needs to distinguish between persistent
H. Sexual abuse of older people is not violence perpetrated by a chronic batterer and
unknown. that associated with caregiver stress. Whilst the
physician must always validate the experience
I. Bruising around the wrists is not
of the victim, their autonomy must be
diagnostic of elder abuse.
respected. Action should be taken however if
J. Anxiety and withdrawal are potential the elderly victim is felt to be unable to make
indicators of abuse. rational decisions or felt to be in imminent
danger.
Answer & Comments

Correct answer: CG
[ Q: 11 ] MasterClass Part2
(2010) - Psychiatry
Abuse of older people is a vastly underreported A 34-year-old woman develops chest pain after
problem. Victims often fail to report because of an argument with her 17-year-old daughter.
fear of retaliation from the abuser, shame, She is brought to the Accident and Emergency
cultural or language barriers or a sense of their department where you are asked to see her.
economic vulnerability. Physicians are often She is hyperventilating and looks very anxious.
unaware of the prevalence of the problem or She is tender to light pressure on the front of
lack training in its recognition and her chest, but examination is otherwise
management. Risk factors for abuse include unremarkable. Breathing room air, her oxygen
cognitive impairment, social isolation and saturation (finger probe) is 99%. Her ECG is
physical dependency. However, there is no normal.
classic profile of a victim. Interdisciplinary
working, adult protection services and the You should:
police can all assist physicians to help patients A. Check serum troponin
surmount abuse.
B. Check serum D-dimer
Abuse can be physical, psychological, sexual or
financial. It should be noted the elderly often C. Organise a lung ventilation perfusion scan

Dr. Khalid Yusuf El-Zohry – Sohag Teaching Hospital (01118391123) Page | 681
El-Zohry MRCP Questions Bank (Part 2) – Medical Masterclass 2010

D. Organise a CT pulmonary angiogram  Restlessness


E. Organise no further investigations  Sighing.

Answer & Comments [ Q: 12 ] MasterClass Part2


Correct answer: E (2010) - Psychiatry
A 27-year-old man with a body builder’s
It was essential to check the ECG and pulse physique is brought to you handcuffed by the
oximetry, but the clinical context and police. He is agitated, sweating and lashing out
examination findings all point to the diagnosis at the police. He has pressured speech that
of a panic attack, and – given that these tests does not really make sense.
gave normal results – it would be appropriate
to explain the diagnosis to the patient and What is the first thing you would do? Choose
reassure accordingly. the best option from the following:

Psychological symptoms of an anxiety state A. Ask the police to release him so you can
include: do a physical examination
 irritability B. Sedate him with Haloperidol 10mg IMI
 Intolerance of noise C. Quickly get as much collateral
information as you can from all sources
 Poor concentration / memory
D. Try talking to him in a calm way to find
 Fearfulness
out what is wrong with him
 Apprehensiveness
E. Immediately transfer him to a psychiatric
 Restlessness ward.
 Continuous worrying thoughts.
Answer & Comments
Physical symptoms of an anxiety state include:
 Dry mouth Correct answer: C

 Difficulty in swallowing
At this point information is what you most
 Chest pain need. Get collateral information, including from
 Shakiness the police, A&E staff who might recognize him
or family if you have a phone number. The five
 Diarrhoea minutes doing this will be well spent. Has he
 Urinary frequency recently been physically ill with a fever? Does
he have a psychiatric or substance use history?
 Paraesthesiae
Is he habitually violent? Having background
 Hot flushes. information will guide you to the more likely
conditions. Unfortunately, sometimes no
Physical signs of an anxiety state include:
collateral information is available. Of course,
 Tenseness the second step that should be started urgently
 Sweating is to talk to the patient and begin assessing him.
 Shaking
 Pallor

Dr. Khalid Yusuf El-Zohry – Sohag Teaching Hospital (01118391123) Page | 682
El-Zohry MRCP Questions Bank (Part 2) – Medical Masterclass 2010

[ Q: 13 ] MasterClass Part2 consultant psychiatrist, the patient’s own


(2010) - Psychiatry general practitioner and an approved social
worker.
A 58-year-old man with no known past medical
or psychiatric history is admitted to coronary
[ Q: 14 ] MasterClass Part2
care with an inferior myocardial infarct. He
receives treatment with thrombolysis and is
(2010) - Psychiatry
started on a beta-blocker, aspirin and a statin. A 34-year-old woman is brought to the
He is recovering well and is transferred to one Emergency Department after taking an
of the general medical wards. On day 5 he uncertain quantity of paracetamol two hours
becomes mentally disturbed, starts to hear previously and trying to hang herself. She
voices in his head, and accuses staff on the becomes agitated and insists that she wants to
ward of trying to kill him. He says that he wants go home immediately. You judge that she is at
to go home and you cannot convince him to high risk of suicide.
agree to stay.
How should you proceed?
It would be appropriate for you to explain to
him that you: A. Call the duty psychiatrist, and with other
staff in the Emergency Department
A. have no right to detain him, but will ask attempt to restrain her under Common
him to sign to say that he is discharging Law until they arrive
himself against medical advice
B. Ask her to sign a ‘discharge against
B. have the right to detain him under medical advice’ form and let her go
Section 2(2) of the Mental Health Act
C. Call the duty psychiatrist, but let the
C. have the right to detain him under patient go if she insists and the duty
Section 3(1) of the Mental Health Act psychiatrist does not arrive in time to see
D. have the right to detain him under her
Section 5(2) of the Mental Health Act D. Detain her under section 5(2) of the
E. have the right to detain him under Mental Health Act
Section 7(1) of the Mental Health Act. E. Call the hospital security services, restrain
her and sedate her
Answer & Comments

Correct answer: D
Answer & Comments
If a patient is already being nursed on a
Correct answer: A
medical, surgical or obstetric ward, or in an
intensive care or high-dependency unit, and
they develop a mental illness de novo, or have In an Emergency Department the suicidal
an exacerbation or relapse of a pre-existing patient who declines to be admitted for
disorder, their physician or surgeon can observation and treatment should be managed
authorise their compulsory detention for up to as follows:
72 hours under section 5(2) of the Mental  Ensure that a member of staff stays
Health Act. with them at all times
During those 72 hours the medical or surgical  Call the duty psychiatrist
team must request a formal assessment by a

Dr. Khalid Yusuf El-Zohry – Sohag Teaching Hospital (01118391123) Page | 683
El-Zohry MRCP Questions Bank (Part 2) – Medical Masterclass 2010

 If they attempt to abscond before or excluded. The PSA may give you some
during psychiatric assessment, the staff information about the extent of the prostatic
of the Emergency Department have a disease, but not the cause of the confusion.
duty under Common Law to restrain
the patient [ Q: 16 ] MasterClass Part2
If a patient who is already being nursed on (2010) - Psychiatry
medical, surgical or obstetric ward, or in a high
dependency or intensive care unit, develops a A 54-year-old man is admitted to hospital in a
mental illness (or has an exacerbation of a pre- neglected state. He appears anxious, agitated,
existing disorder), their physician or surgeon shaky and sweaty. He reports that he stopped
can authorise their compulsory detention for up drinking alcohol two days previously. After
to 72 hours under section 5(2) of the Mental initial treatment to control his withdrawal
Health Act. symptoms, you talk to him about his usage of
alcohol.

[ Q: 15 ] MasterClass Part2 Which of the following is NOT one of the DSM-


(2010) - Psychiatry IV criteria for alcohol dependence?

An 85-year-old man is admitted from home A. Inability to cut down


because he has become increasingly confused B. Repeated efforts to control drinking
and is not coping. He is known to have
C. Serum gamma-GT over three times the
metastatic carcinoma of the prostate and takes
upper limit of normal
Zoladex 3 monthly. You note that he has
bruising over the left side of his forehead. D. Drinking increasing amounts
E. Withdrawal symptoms.
Which of the following investigations will not
help you diagnose and treat his confusion?
A. CT scan of the head
Answer & Comments
B. Midstream urine sample
Correct answer: C
C. Calcium
D. Urea and electrolytes
The six DSM-IV criteria for alcohol dependence
E. Prostatic specific antigen (PSA). are:
1. inability to cut down
2. repeated efforts to control drinking
Answer & Comments
3. amnesic periods
Correct answer: E
4. drinking increasing amounts
5. ongoing drinking despite detrimental
This patient deserves a CT scan of his head to
exclude not only coexisting vascular dementia consequences
but also a possible subdural haemorrhage, as 6. withdrawal symptoms.
there is evidence that he has had at least one
fall. It is important to exclude intercurrent
infections including a UTI and chest infection.
Dehydration and hypercalcaemia should also be

Dr. Khalid Yusuf El-Zohry – Sohag Teaching Hospital (01118391123) Page | 684
El-Zohry MRCP Questions Bank (Part 2) – Medical Masterclass 2010

[ Q: 17 ] MasterClass Part2 medications, has become increasingly forgetful


(2010) - Psychiatry over the last 18 months, to the point where he
now finds it difficult to remember the names of
You are asked to see a 52-year-old woman with some members of his family. He has recently
metastatic breast cancer in clinic who is tearful been having visual hallucinations. On
and feels that she can no longer cope. examination he seems rather expressionless
and has cogwheel rigidity of both arms.
What would be the most appropriate course of
action? The most likely diagnosis is:
A. Screen for depression / suicidal thoughts A. Alzheimer’s disease
and liaise with primary care team
B. Lewy body dementia
B. Prescribe antidepressants
C. Vascular dementia
C. Reassure her that these feelings are
D. Frontal lobe dementia
normal in this situation
E. Alzheimer’s disease and Parkinson’s
D. Listen to her concerns and refer her for
disease.
counselling
E. Refer her for psychiatric assessment.

Answer & Comments


Answer & Comments
Correct answer: B
Correct answer: A

Lewy body dementia is characterised by


Depression is common in patients with
fluctuating cognition, visual hallucinations,
advanced cancer (20-25% incidence) and can
parkinsonism, neuroleptic sensitivity, falls /
respond to treatment. Diagnosing depression
transient loss of consciousness / syncope and
with screening tools such as the Hospital
delusions.
Anxiety and Depression scale (HAD) or basic
clinical interview is within the capabilities of all Vascular dementia typically occurs in those with
doctors. In a clinic setting you should screen for widespread vascular disease. The course is
depression, which includes listening to her typically fluctuating with stepwise progression.
concerns. If her feelings overwhelm her it is Differentiation from Alzheimer’s disease can be
important to ask if she has thought about difficult, although a history of strokes or the
taking her own life. Studies suggest that presence of focal neurological signs are very
patients don’t often volunteer this but suggestive.
appreciate being asked and it does not ‘put
ideas into their head’. Good communication [ Q: 19 ] MasterClass Part2
with the primary care team about appropriate
(2010) - Psychiatry
management, that might include
antidepressants, is vital. A very thin 17-year-old girl is referred for
medical opinion. A comprehensive battery of
[ Q: 18 ] MasterClass Part2 tests fails to establish any clear medical cause
for her condition and you think that she has an
(2010) - Psychiatry
eating disorder.
A 78-year-old man, without significant past
medical history and taking no regular

Dr. Khalid Yusuf El-Zohry – Sohag Teaching Hospital (01118391123) Page | 685
El-Zohry MRCP Questions Bank (Part 2) – Medical Masterclass 2010

Which one of the following statements about


anorexia nervosa and bulimia nervosa is true?
A. In both anorexia nervosa and bulimia [ Q: 20 ] MasterClass Part2
nervosa body weight is significantly (2010) - Psychiatry
reduced.
A 30-year-old woman, with a long history of
B. Patients with anorexia nervosa judge self- intravenous drug abuse, is admitted with an
worth largely or exclusively in terms of abscess in her right groin related to the use of
shape and weight, but those with bulimia non-sterile needles. This is drained, and she is
nervosa do not. admitted to a medical ward for intravenous
C. In anorexia nervosa body weight is antibiotic treatment. She ‘goes missing’ from
significantly reduced, but in bulimia the ward at 14.00h, but is found outside a back
nervosa it is often normal. entrance of the hospital smoking a cigarette at
15.30h and then returns to the ward. At 21.30h
D. Patients with anorexia nervosa and
you are asked to see her by the ward nurses
bulimia nervosa are both in control of
because she says she feels very unwell. She is
their eating, but exert this control in
nauseated, has had some diarrhoea, and yawns
different ways.
as you talk to her. Her temperature is 37.6 C
E. Patients with anorexia nervosa tend to and her pupils are dilated.
give a reliable history of how much (and
how little) they eat, whereas patients The most likely diagnosis is:
with bulimia nervosa tend to conceal A. Opioid toxicity
their history of vomiting.
B. Alcohol withdrawal

Answer & Comments C. Clostridium difficile infection

Correct answer: C D. Opioid withdrawal reaction


E. Toxic shock syndrome.
The ICD-10 critera for anorexia nervosa require
BMI <17.5, self-induced weight loss, body Answer & Comments
image distortion, and abnormalities of the
hypothalamic-pituitary-gonadal axis. Correct answer: D

The DSM-IV criteria for bulimia nervosa require


The symptoms and signs of opioid withdrawal
recurrent episodes of binge eating, recurrent
include dysphoric mood, yawning, insomnia,
inappropriate compensatory behaviour to
nausea, vomiting, diarrhoea, muscle aches,
prevent weight gain, with these features
lacrimation / rhinorrhoea, pupillary dilatation,
occurring more than twice weekly for three
piloerection, sweating and fever.
months. There is no diagnostic requirement for
weight loss. Initially give 10 mg of methadone syrup and
wait about 60 min to determine its effect.
In both anorexia nervosa and bulimia nervosa
Continue administering in 10 mg doses until
self-worth is judged largely or exclusively in
symptoms are under control. It is rare to exceed
terms of shape and weight, and in both
a total dose of 40 mg over 24 hours.
conditions a reliable dietary history is unlikely
to be obtained from the patient. A key feature
in the binge eating of bulimia nervosa is a
feeling of loss of control.

Dr. Khalid Yusuf El-Zohry – Sohag Teaching Hospital (01118391123) Page | 686
El-Zohry MRCP Questions Bank (Part 2) – Medical Masterclass 2010

[ Q: 21 ] MasterClass Part2 E. Lithium is safe during the first trimester,


(2010) - Psychiatry but contraindicated during the second
and third trimesters and when breast-
A 52-year-old man, with a history of heavy feeding.
alcohol consumption, is admitted to hospital
with a chest infection. He is tremulous and Answer & Comments
confused due to alcohol withdrawal.
Correct answer: B
Along with treating his chest infection, giving
thiamine / other vitamins, and preventing In the first trimester lithium can cause
hypoglycaemia, you will prescribe atrialisation of the right ventricle.
chlordiazepoxide at a dose of:
During the second and third trimesters lithium
A. 500 mg po four times daily can be used, but dose requirements are
B. 10 mg po four times daily increased.
C. 50 mg IM as needed Immediately after delivery lithium dose
requirements return to normal abruptly.
D. 30 mg po four times daily
Lithium levels can rise dangerously if a high
E. 10 mg IM as needed. dose is continued. Lithium is excreted in breast
milk and if the infant becomes dehydrated,
Answer & Comments then toxic lithium levels develop rapidly.

Correct answer: D
[ Q: 23 ] MasterClass Part2
If in doubt regarding a drug dose, always look in (2010) - Psychiatry
the British National Formulary (BNF). A 22-year-old woman is brought into the
Emergency Department after taking half a
[ Q: 22 ] MasterClass Part2 dozen paracetamol tablets in front of her
(2010) - Psychiatry partner, with whom she has been quarrelling.
She has also threatened to cut her wrists and
A 28-year-old woman with a history of manic has a tendency to misuse alcohol.
depression has been well controlled on lithium
for several years. She wishes to get pregnant Which one of the following statements is
and asks for advice regarding the safety of correct?
lithium in pregnancy and when breast-feeding. A. A past history of similar self-harming
episodes reduces the risk of a future fatal
Which one of the following statements is true?
outcome
A. Lithium is safe during pregnancy and
B. People who threaten to harm themselves
when breast-feeding.
do not actually do any serious damage
B. Lithium is contraindicated during the first
C. The prognosis is not improved by the
trimester and when breast-feeding.
patient being interviewed by a mental
C. Lithium is contraindicated in the first health professional in the Emergency
trimester but is safe when breast-feeding. Department
D. Lithium is safe during pregnancy but is D. Her misuse of alcohol is a significant
contraindicated when breast-feeding. additional risk factor

Dr. Khalid Yusuf El-Zohry – Sohag Teaching Hospital (01118391123) Page | 687
El-Zohry MRCP Questions Bank (Part 2) – Medical Masterclass 2010

E. The police have to be informed because [ Q: 25 ] MasterClass Part2


attempted suicide is a criminal offence (2010) - Psychiatry

Answer & Comments An 81-year-old, who was admitted with


recurrent falls and leg ulcers, wants to go home
Correct answer: D but her daughter is concerned. You assess her
mental state and find she has an MMSE of
Alcohol has a short-term disinhibiting effect 21/30. The daughter has many questions about
that increases the risk of violence towards one's what medication may be helpful and how able
self and others. The clinical picture she will be to manage at home.
described may be very different once the
effects of the alcohol have worn off. Which of the following statements do you feel is
LEAST applicable to this situation?

[ Q: 24 ] MasterClass Part2 A. Calcium and vitamin D should be


prescribed
(2010) - Psychiatry
B. The score on her MMSE can be used to
An 84-year-old man with Bipolar Affective
advise her daughter on whether she can
Disorder has been on lithium for 20 years. He make a will
presents with lithium toxicity. The family report
the General Practitioner prescribed a new C. You need to know her background to
medication the previous week. determine the significance of the MMSE
score
Which among the following drugs DO NOT
D. High dose vitamin C is unlikely to be
increase the levels of Lithium?
helpful here
A. Bendroflumethiazide
E. The score on her MMSE does not indicate
B. Ibuprofen whether or not she is likely to cope at
home
C. Perindopril
D. Losartin
Answer & Comments
E. Atenolol
Correct answer: B

Answer & Comments


Scores on the MMSE are affected by
Correct answer: E educational background. A score may suggest
the need for further diagnosis or assessment
Drugs that can cause lithium toxicity by but does not indicate the presence or absence
reducing lithium excretion include ACE of ‘capacity’, which is task (and time) specific.
inhibitors, angiotensin II antagonists, non- Calcium / vitamin D is likely to be beneficial
steroidal anti-inflammatories and diuretics. whereas there is much weaker, if any, evidence
Drugs that can cause lithium toxicity without for vitamin C.
increased serum lithium levels include selective The best way of determining whether she will
serotonin reuptake inhibitors, antiepileptics, manage at home, is a home visit carried out by
methyldopa, antipsychotics and calcium an Occupational Therapist, and in this situation
channel blockers. it may be helpful if the daughter can be
present.

Dr. Khalid Yusuf El-Zohry – Sohag Teaching Hospital (01118391123) Page | 688
El-Zohry MRCP Questions Bank (Part 2) – Medical Masterclass 2010

[ Q: 26 ] MasterClass Part2 What is the best management plan for this


(2010) - Psychiatry man?
A. Haloperidol 0.5 mg orally, given 2 hourly
A 58-year-old man, with a long history of
to a maximum of 5 mg in 24 hours
alcohol abuse, is admitted in a dishevelled and
confused state. He is very unsteady on his feet. B. Bedrails on both sides of the bed
C. Risperidone 0.5 mg orally, and repeated 2
Which of the following findings would most
hourly up to 3 mg in 24 hours
strongly support a diagnosis of Wernicke’s
encephalopathy? D. Regular reassurance, being placed in a
calm environment
A. Fluctuating conscious level with focal
neurological signs E. Lorazepam 0.5 mg as required, but no
more than tds.
B. Nystagmus and intention tremor
C. Nystagmus and ataxia of gait Answer & Comments
D. Tachycardia and tremulousness
Correct answer: D
E. Drowsiness, jaundice and metabolic flap.
There is a wealth of evidence that shows
patients with delirium are best managed in a
Answer & Comments calm environment, with constant reassurance
and re-orientation. It is also helpful if the
Correct answer: C patient has their glasses and hearing aids as
sensory deprivation aggravates the situation.
Wernicke’s encephalopathy typically presents The underlying cause of the delirium should be
with ophthalmoplegia (horizontal and vertical sought and treated: consider infection,
nystagmus, weakness / paralysis of the lateral inappropriate medication, electrolyte
rectus muscles, weakness / paralysis of imbalance, constipation and alcohol withdrawal
conjugate gaze), ataxia (predominantly etc.
affecting stance or gait, and often without
clear-cut intention tremor) and confusion. If the patient really needs to be medicated then
consider lorazepam 0.5mg to 1mg orally, up to
Presentation A would suggest subdural a maximum of 3mg in 24 hours. Haloperidol
haematoma, presentation D alcohol causes extrapyramidal side effects and should
withdrawal, and presentation E hepatic be avoided in patients with known Lewy Body
encephalopathy. Dementia. Risperidone has been linked with
stroke disease when used long-term in patients
[ Q: 27 ] MasterClass Part2 with dementia, but may be used short-term.
(2010) - Psychiatry The above are reasonable dosing regimens, but
always use the lowest possible effective doses.
An 84-year-old man is admitted via the
Emergency Department after being found
[ Q: 28 ] MasterClass Part2
wandering outside in his pyjamas. He is
obviously confused and is talking to people who (2010) - Psychiatry
are not there. The nurses are concerned when A 48-year-old man with a long history of alcohol
he starts wandering about the ward and feel he abuse is admitted after being found collapsed in
is going to prove difficult to look after.

Dr. Khalid Yusuf El-Zohry – Sohag Teaching Hospital (01118391123) Page | 689
El-Zohry MRCP Questions Bank (Part 2) – Medical Masterclass 2010

the street. He is agitated and tremulous. You D. Yawning


make a diagnosis of alcohol withdrawal.
E. Diarrhoea.
Which one of the following is appropriate
management for this condition?
A. Nurse in slightly darkened room Answer & Comments

B. Monitor for hyperglycaemia and control Correct answer: B


blood glucose using an insulin infusion if
necessary Symptoms of opioid withdrawal include
C. Give antipsychotics to reduce agitation dysphoric mood, yawning, insomnia, muscle
and likelihood of seizures aches, lacrimation / rhinorrhoea, papillary
dilatation, piloerection, fever, sweating, nausea
D. Avoid benzodiazepines to reduce
/ vomiting, diarrhoea.
likelihood of respiratory depression
If the patient is having an opioid withdrawal
E. High-calorie, high-carbohydrate diet.
reaction, then give 10 mg of methadone syrup
and wait about 60 min to determine its effect.

Answer & Comments [ Q: 30 ] MasterClass Part2


Correct answer: E (2010) - Psychiatry
A 23-year-old man is brought to the Emergency
Important aspects of immediate management Department by ambulance. He is known to have
of alcohol withdrawal include benzodiazepines schizophrenia and is being prescribed oral
(reducing schedule), nursing in well-lit quiet trifluoperazine. He is mute. His head is turned
environment, thiamine supplementation, to the right in an awkward and extreme
monitoring for and treating hypoglycaemia, position. He appears distressed. The ambulance
rehydration, high-calorie high-carbohydrate crew tell you he has been like this for about 30
diet, and avoiding antipsychotics (which lower minutes. Other than this odd posture his
seizure threshold). physical examination is normal.

From the list below choose the immediate


[ Q: 29 ] MasterClass Part2
intervention that is most likely to be
(2010) - Psychiatry appropriate:
A 32-year-old man with a history of opioid A. Administer an extra dose of
abuse is admitted to hospital after a road traffic trifluoperazine.
accident in which he fractures his femur. He
B. Reduce the dose of trifluoperazine.
becomes unwell and you are asked to assess
him on the ward. C. Administer intramuscular dose of
procyclidine.
Which one of the following features would NOT
D. Refer to on call psychiatrist.
be compatible with a diagnosis of an opioid
withdrawal reaction? E. Prescribe dose of oral lorazepam.
A. Nausea / vomiting
Answer & Comments
B. Hypothermia
Correct answer: C
C. Lacrimation
Dr. Khalid Yusuf El-Zohry – Sohag Teaching Hospital (01118391123) Page | 690
El-Zohry MRCP Questions Bank (Part 2) – Medical Masterclass 2010

This is most likely an acute dystonic reaction to evidence of signs of withdrawal, including
the trifluoperazine. Acute dystonia as an lacrimation, runny nose, agitation, sweating,
adverse effect of antipsychotic drugs affects piloerection, tachycardia, vomiting, shivering,
about 2% of patients. It occurs abruptly and yawning and widely dilated pupils.
early in the course of treatment. It is generally
associated with piperazine phenothiazines (e.g. [ Q: 32 ] MasterClass Part2
trifluoperazine) and butyrophenones (e.g.
(2010) - Psychiatry
haloperidol). Males, young adults and children
are more susceptible. It is best treated with A 30-year-old man is involved in a multiple car
parenteral anticholinergic drugs and relief of crash in which several people die. He is not
symptoms should be swift. physically injured himself, but in the days that
immediately follow he feels numb and
[ Q: 31 ] MasterClass Part2 detached, dazed and disorientated, with
physical symptoms of sweating and shakiness.
(2010) - Psychiatry
A 24-year-old man from Spain is admitted to The most likely diagnosis is:
your ward. He tells you that he has been a A. Acute stress disorder
heroin user for 3 years. He says he is attending
B. Post-traumatic stress disorder
a drug-treatment unit in Spain where he is
being prescribed 40 mg of methadone per day. C. Adjustment disorder
D. Panic attack
Which one of the following is the first thing to
do? E. Depression.
A. Prescribe methadone 40 mg/day in
divided doses Answer & Comments

B. Prescribe methadone 10 mg twice a day Correct answer: A


C. Confirm that he is taking opioids by
checking his urine with a drug-testing kit Acute stress disorder is a short-lived but severe
disorder caused by an overwhelming,
D. Use naloxone to control withdrawal
psychologically traumatic experience. The
symptoms
symptoms develop rapidly, but tend to resolve
E. Reassure him that any heroin withdrawal within a matter of days. These include
symptoms will resolve spontaneously psychological symptoms, such as feeling numb
and detached, dazed and disorientated, and
physical symptoms, such as sweating,
shakiness, palpitations and insomnia. Some
Answer & Comments
patients will go on to develop post-traumatic
Correct answer: C stress disorder.
If there is persistent denial that the event has
Giving methadone to someone who is not occurred, the patient should be cautiously
dependent on opioids can cause death. Also, if prompted to recall the facts. A short course of a
drug users exaggerate their methadone or benzodiazepine tranquilliser and/or hypnotic
heroin use and you use this dose to calculate may help severe agitation or insomnia.
the equivalent amount of methadone, you can
kill them. Look for the objective use of opioids
with urine drug screens. Also look for objective
Dr. Khalid Yusuf El-Zohry – Sohag Teaching Hospital (01118391123) Page | 691
El-Zohry MRCP Questions Bank (Part 2) – Medical Masterclass 2010

[ Q: 33 ] MasterClass Part2  Unrefreshing sleep


(2010) - Psychiatry  Postexertional malaise
A 42-year-old woman is referred to the medical
outpatient clinic because she is ‘always [ Q: 34 ] MasterClass Part2
exhausted’. She says that she cannot do (2010) - Psychiatry
anything that requires any physical effort and
A very thin 18-year-old girl is referred for
that she sleeps all the time. Her general
practitioner cannot find any explanation for her investigation of weight loss. You suspect that
symptoms. You consider the diagnosis of she has anorexia nervosa, but perform a range
of screening tests.
chronic fatigue syndrome.

Which one of the following findings would NOT Which one of the following would be
be consistent with this diagnosis? compatible with the diagnosis of anorexia
nervosa?
A. Subjective memory impairment
A. Hyperkalaemia
B. Tender lymph nodes
B. Low serum bicarbonate
C. Muscle pain
C. Low serum cholesterol
D. Joint pain
D. Low white cell count
E. Weight loss.
E. Elevated ESR.

Answer & Comments


Answer & Comments
Correct answer: D
Correct answer: E
A wide range of biochemical / haematological
Chronic fatigue syndrome is defined as clinically abnormalities can be found in anorexia nervosa.
evaluated, medically unexplained fatigue of at These include hypokalaemia, hypochloraemic
least six months’ duration that is: alkalosis (both due to vomiting and/or
 Of new onset diuretic/laxative abuse) and
hypercholesterolaemia (mechanism unknown).
 Not a result of ongoing exertion
The ESR is normal or reduced.
 Not substantially alleviated by rest
 A substantial reduction in previous [ Q: 35 ] MasterClass Part2
levels of activity. (2010) - Psychiatry
Together with the occurrence of four or more A 48-year-old with recurrent carcinoma of the
of the following symptoms: colon admits to poor sleeping, anhedonia and
 Subjective memory impairment feelings of hopelessness.
 Tender lymph nodes Which of the following would you do?
 Muscle pain A. Reassure him that this is a normal
 Joint pain response to his predicament
 Headache B. Start anti-anxiolytic drugs

Dr. Khalid Yusuf El-Zohry – Sohag Teaching Hospital (01118391123) Page | 692
El-Zohry MRCP Questions Bank (Part 2) – Medical Masterclass 2010

C. Suggest counselling doses it can cause perceptual disturbances,


including hallucinations.
D. Start an anti-depressant
E. Suggest relaxation.
[ Q: 37 ] MasterClass Part2
(2010) - Psychiatry

Answer & Comments A 59-year-old school teacher was diagnosed


with depression and anxiety when he felt
Correct answer: D unable to cope with the pressures of work.
When you saw him his mood and anxiety were
Depression is the most likely diagnosis. This improved. His wife reported that he was
needs to be treated along conventional lines. increasingly absent-minded. On cognitive
Other modalities may be of additional benefit. testing his recall was poor and he made errors
on the serial seven subtraction exercise.
[ Q: 36 ] MasterClass Part2 Choose the TWO most appropriate
(2010) - Psychiatry management options listed below:
A 26-year-old woman is brought to the A. Prescribe an antidepressant.
Emergency Department from an office party by B. Advise retirement on medical grounds.
her friends, who report she became disinhibited
and anxious. They suspect she took an ecstasy C. Arrange electroconvulsive therapy.
tablet. D. Arrange psychometric testing.

Which of the following is a psychological effect E. Refer for cognitive behaviour therapy.
of acute ecstasy intoxication? F. Arrange a dementia screen, including
A. Apathy neuroimaging.

B. Decreased aggression G. Refer for marital therapy.

C. Enhanced intellectual performance H. Observe for a few months and then


review.
D. Irritability
I. Prescribe lithium.
E. Lethargy
J. Prescribe a benzodiazepine.
Answer & Comments
Answer & Comments
Correct answer: B
Correct answer: DF
Ecstasy is 3,4 Methylenedioxy
Methamphetamine whose mechanism of action In the early stages of dementia, patients may
is mediated by the release of serotonin, and to present with low mood and anxiety symptoms.
a lesser extent the blockade of dopamine Depressive symptoms affect up to 50% of
reuptake. The psychological effects of ecstasy patients with dementia. This is confounded by
intoxication include increased empathy and the fact that patients with depression can
emotional expression, decreased desire and present with a ‘pseudodementia’. Cognitive
ability to perform mental tasks, increased impairment must always be actively
anxiety and restlessness, increased energy, investigated and a diagnosis made so that
decreased aggression, disinhibition. At high appropriate treatment options can be explored.

Dr. Khalid Yusuf El-Zohry – Sohag Teaching Hospital (01118391123) Page | 693
El-Zohry MRCP Questions Bank (Part 2) – Medical Masterclass 2010

[ Q: 38 ] MasterClass Part2 last resort and preferably only once the cause
(2010) - Psychiatry of the delirium has been established

A 72-year-woman is admitted after a fall. She


[ Q: 39 ] MasterClass Part2
lives alone and works in the hospital
benevolent society shop. You are called to see (2010) - Psychiatry
her because she is agitated, accusing staff of A large man, in his early 20’s, is brought to the
stealing her jewellery and wanting to leave. Accident and Emergency department by his
friends after he suddenly developed a headache
Which of the following would be your first
and collapsed when drinking in a pub. He has
intervention?
become very aggressive and attempted to hit
A. Prescribe haloperidol 0,5mg to calm her one of the nurses, but is now in a side room
down and call the psychiatrist on call to with one of his friends. You are asked to assess
assess whether she should be detained him.
under the Mental Health Act.
What should you do?
B. Order an urgent CT head scan.
A. Adopt a professional but firm manner,
C. Do a physical examination and review the
walk in to the side room, and speak to
blood and urine test results
him as you would to any other patient
D. Interview the patient to establish her
B. Explain to his friends that you are calling
concerns, take a history, assess her
a psychiatrist so that he can be detained
mental state and try to reassure her.
and treated under section 5(2) of the
E. Ask the nurses to move her to a quiet, Mental Health Act
well lit room and try to get a family
C. Get as much information as possible
member to come and sit with her.
about him from his friends, call the
security porters / police, and when they
Answer & Comments have arrived try to speak to him from the
Correct answer: D doorway of the side room
D. Explain to his friends that you will not
A sudden change in the mental state or sudden speak to him or see him until he has
onset of behaviour that is out of character in an settled down
older person is most likely to be due to an acute E. Tell his friends that you are going to call
confusional state (delirium). Even if a patient is the security porters and an anaesthetist,
confused it is paramount to take a history, and then restrain and sedate him.
which may give clues to the cause of the
delirium. It also gives you the opportunity to
Answer & Comments
assess the severity of confusion. The patient
will benefit from reassurance. Of course, don’t Correct answer: C
dismiss the possibility that the patient is not
confused and her jewellery is indeed missing! A It would be very tempting to pick option ‘D’, but
physical review and review of investigations doctors have a duty of care that they cannot
must be done as soon as is possible as delirium neglect because the situation is difficult.
is a serious condition with a high morbidity and However, only boxers are paid to get beaten
mortality. Sedation should only be used as a up.

Dr. Khalid Yusuf El-Zohry – Sohag Teaching Hospital (01118391123) Page | 694
El-Zohry MRCP Questions Bank (Part 2) – Medical Masterclass 2010

Do not take any risks with a patient who might Never give a methadone dose equivalent to
be violent. Never see such patients alone, do what the patient reports they are taking.
not let back-up (hospital security and / or Initially give 10 mg, and continue administering
police) leave until you think the situation is in 10 mg increments each hour until symptoms
safe, remove your necktie or scarf, and make are under control. It is rare to need more than
sure that you and other staff always have easy 40 mg per 24 hours: beware of overdosing
access to an exit door. which can lead to respiratory arrest.
Section 5(2) of the Mental Health Act allows a
physician or surgeon to detain a patient for up [ Q: 41 ] MasterClass Part2
to 72 hours if they are already being nursed in (2010) - Psychiatry
hospital when they develop (or have an
exacerbation of) a mental illness. It does not A 40-year-old woman was referred to medical
apply to Accident and Emergency departments. outpatients with breathlessness on exertion
and palpitations. Her GP has observed that she
was very pale. She has suspected anaemia, but
[ Q: 40 ] MasterClass Part2 has refused to have any blood tests. You think
(2010) - Psychiatry she may have a blood/injection/injury phobia.
A 28-year-old woman is admitted to a surgical
Which one of the following would confirm your
ward for drainage of an abscess in her forearm.
diagnosis?
She uses heroin intravenously and says that she
is about to go into opioid withdrawal and shows A. Hallucinations of corpses
early signs of doing so. B. Paranoia

What should you do? C. Slowing of her heart rate when you show
her a syringe
A. Ask her how much heroin she uses per
day and give her a dose of methadone D. Impairment of short-term memory
syrup equivalent to this E. Compulsive checking
B. Give her 40 mg of methadone syrup and
wait 60 min to determine its effect
C. Ask her how much heroin she uses per Answer & Comments
day and arrange for her to receive
Correct answer: C
methadone syrup equivalent to this
divided into four doses per day
Blood/injection/injury phobia is unusual and is
D. Give her 10 mg of methadone syrup and
counterintuitive in its autonomic nervous
wait 60 min to determine its effect
system response in that it is associated with
E. Give her a bottle containing 100 mg of bradycardia and hypotension. This can lead to
methadone syrup and tell her to adjust fainting.
the dosage herself.
[ Q: 42 ] MasterClass Part2
Answer & Comments (2010) - Psychiatry
Correct answer: D A 40-year-old man from Uganda is brought in
by the police under section 136 of the Mental
Health Act. He was arrested because he was

Dr. Khalid Yusuf El-Zohry – Sohag Teaching Hospital (01118391123) Page | 695
El-Zohry MRCP Questions Bank (Part 2) – Medical Masterclass 2010

standing naked in a busy street attempting to


‘direct’ the traffic. On examination he declared
that he was the Minister of Transport. He spoke
rapidly and paced around the room issuing
orders in a loud voice. He was found to have
oral thrush.

From the following choose the TWO LEAST likely


diagnostic possibilities:
A. Mania
B. Tertiary Syphilis
C. Amphetamine use
D. Gonorrhoea
E. Adverse effects of antiretroviral drugs
F. Neuropsychiatric sequelae of HIV
G. Delirium due to secondary infection
H. Cocaine use
I. Post traumatic stress disorder
J. Psychological reaction to the diagnosis of
AIDS.

Answer & Comments

Correct answer: DI

The most common aetiological factors in


HIV/AIDS-associated psychotic disorders are:
prexisiting psychotic disorders, substance
misuse, psychogenic reaction, iatrogenic factors
and HIV related brain disease

Dr. Khalid Yusuf El-Zohry – Sohag Teaching Hospital (01118391123) Page | 696
El-Zohry MRCP Questions Bank (Part 2) – Medical Masterclass 2010

B. Androgenetic alopecia

Dermatology C. Discoid lupus erythematosus


D. Lichen planopilaris
(85 Questions) E. Telogen effluvium.

(Medical Masterclass – Part 2) Answer & Comments

Correct answer: B

[ Q: 1 ] MasterClass Part2 Androgenetic alopecia is a common cause of


(2010) - Dermatology alopecia in men and women. It classically
causes diffuse hair loss over the vertex with
A 7-year-old girl presents to her GP with an
sparing over the occiput, with no scarring of the
itchy scalp and rash on the dorsum of the neck.
scalp.
Which of the following is the most likely Discoid lupus and lichen planus can cause
diagnosis? patchy, scarring alopecia.
A. Head lice infestation Alopecia areata is non-scarring but normally
B. Atopic eczema causes patchy hair loss.

C. Lichen planus Telogen effluvium is a cause of diffuse non-


scarring alopecia but normally occurs
D. Discoid lupus approximately 4 months after an event such as
E. Allergic contact dermatitis childbirth or severe illness.

Answer & Comments [ Q: 3 ] MasterClass Part2


Correct answer: A (2010) - Dermatology
A 15-year-old girl presents with an itchy rash
All of the above can cause an itchy scalp and around her umbilicus at the site of her jeans
rash. Infestation with head lice is however the stud. She is patch tested and found to be
most common diagnosis. An eczematous rash allergic to nickel.
affecting the dorsum of the neck often
accompanies head lice infestation, particularly Which of the following is NOT true?
in girls with long hair. A. She has a type-4 allergy to nickel
B. She is likely to become desensitised to
[ Q: 2 ] MasterClass Part2 nickel if she avoids it for a long time
(2010) - Dermatology
C. Topical steroids will improve the rash
A 60-year-old lady presents with a 3-year D. Painting the stud with nail varnish may
history of progressive hair loss. Examination improve the rash
reveals a diffuse thinning of hair over the
vertex, with no scarring of the scalp. E. Nickel allergy is the most common patch
test finding in women
The most likely diagnosis is
A. Alopecia areata

Dr. Khalid Yusuf El-Zohry – Sohag Teaching Hospital (01118391123) Page | 697
El-Zohry MRCP Questions Bank (Part 2) – Medical Masterclass 2010

Answer & Comments Which would be the TWO most helpful


investigations in reaching a diagnosis?
Correct answer: B
A. Patch tests
Contact dermatitis is a type-4 allergy that can B. Direct immunofluorescence
be treated with topical steroids. Nail varnish
C. Pathergy test
can reduce contact of studs with the skin and
improve this problem (although subsequent D. Autoantibody screen
allergy to the nail varnish has been reported). E. Skin prick tests
Once type-4 allergy to a substance has
F. Skin biopsy
developed, it is usually life-long, even if the
allergen is strictly avoided. G. Skin scrapings
H. Skin swab
[ Q: 4 ] MasterClass Part2
I. Tzanck smear
(2010) - Dermatology
J. IgE level.
A 60-year-old man presents with a palpable,
non-blanching rash on the lower legs. Answer & Comments
Which of the following investigations is least Correct answer: BF
likely to help with the underlying diagnosis?
A. Antistreptolysin-O test (ASOT) Oral and genital ulceration may be part of
Behçet’s disease, for which a pathergy test is
B. Antinuclear antibodies (ANA)
helpful, but cutaneous ulcers makes this
C. Hepatitis B antibodies diagnosis unlikely. Skin scrapings, which are
D. Thyroid function tests generally sent for mycology and thus for
diagnosis of fungal infections, would not be
E. Cryoglobulins. helpful. The history is not suggestive of an
allergic contact dermatitis or atopic dermatitis
Answer & Comments so investigations such as patch tests, skin prick
tests and IgE levels would not be helpful.
Correct answer: D
Ulceration of the skin, genitalia and mouth can
A palpable, purpuric rash clinically represents occur in autoimmune blistering diseases such as
an underlying vasculitis. Common causes for pemphigus vulgaris (PV). The patient is the right
cutaneous vasculitis are infections (e.g. age and ethnic group for PV that tends to
Streptococcal, Hepatitis), drug reactions and present more commonly in the 3rd-6th decades
autoimmune diseases. and in Jewish and Indian patients. A skin biopsy
for routine histology is a useful investigation
Thyroid function abnormalities are not usually but the gold standard is direct
associated with vasculitis. immunofluorescence (performed on a skin
biopsy), which demonstrates the intercellular
[ Q: 5 ] MasterClass Part2 IgG in the epidermis, which characterises PV. A
(2010) - Dermatology tzanck smear can be helpful but is rarely done
and not as informative as a biopsy.
A 40-year-old Indian male presents with mouth
ulcers, a sore penis and ulcers on the skin.

Dr. Khalid Yusuf El-Zohry – Sohag Teaching Hospital (01118391123) Page | 698
El-Zohry MRCP Questions Bank (Part 2) – Medical Masterclass 2010

[ Q: 6 ] MasterClass Part2 [ Q: 7 ] MasterClass Part2


(2010) - Dermatology (2010) - Dermatology
A 60-year-old cleaner presents with a rash on This 30-year-old woman presents with a
both hands. An irritant hand dermatitis is scarring hypopigmented area on the right
suspected. cheek, surrounded by hyperpigmentation (see
image). Close inspection reveals follicular
Which one of the following statements is true plugging. It is asymptomatic and has been
regarding this condition? slowly enlarging over 6 months.
A. It classically causes a finger tip dermatitis.
B. It is diagnosed by patch testing.
C. It is more common in atopic patients.
D. It can be differentiated from allergic
contact dermatitis histologically.
E. It should not be treated with topical
steroids.

Answer & Comments

Correct answer: C

Irritant hand dermatitis classically causes


dermatitis in the finger webs, beneath rings and
over the dorsum of the hands. It can be caused
acutely by contact with stong alkalis or acids
but is usually a chronic problem caused by
repeated exposure to detergents and wet work.
Atopic patients are more prone to develop
irritant dermatitis.
Patch tests demonstrates type IV allergic What is the most likely diagnosis?
reactions and are used to diagnose an allergic
contact dermatitis. Patch tests are negative in A. Discoid lupus erythematosus
an irritant dermatitis. Histology is similar in B. Systemic lupus erythematosus
allergic and irritant dermatitis. Therefore, the
C. Tinea incognito
diagnosis of an irritant contact dermatitis is
based on the the history, clinical signs and D. Contact dermatitis
negative patch tests. E. Sarcoidosis
The treatment of irritant and allergic dermatitis
is similar and involves avoidance of Answer & Comments
irritants/allergens, use of soap substitutes,
regular emollients and topical steroids. Correct answer: A

Dr. Khalid Yusuf El-Zohry – Sohag Teaching Hospital (01118391123) Page | 699
El-Zohry MRCP Questions Bank (Part 2) – Medical Masterclass 2010

Discoid lupus erythematosus and sarcoid can [ Q: 9 ] MasterClass Part2


both cause scarring skin lesions, but the (2010) - Dermatology
location here would be typical of lupus.
A 45-year-old female presents with oral
ulceration and a positive Nikolsky sign.
[ Q: 8 ] MasterClass Part2
(2010) - Dermatology In which of the following may the Nikolsky sign
be positive?
A 21-year-old female presents with an acute
onset of widespread, small, scaly plaques over A. Psoriasis
her trunk and limbs. Her GP diagnoses her as B. Eczema
having guttate psoriasis.
C. Toxic epidermal necrolysis
Which of the following management options is D. Lichen simplex
least appropriate?
E. Pemphigus vulgaris
A. Referral to hospital for ultraviolet B (UVB)
light treatment F. Mycosis fungoides

B. Prescription of topical vitamin D G. Secondary syphilis


analogues H. Chicken pox
C. Reassure patient that it should I. Porphyria
spontaneously improve
J. Lichen planus.
D. Reassure patient that once treated it is
unlikely to occur again
Answer & Comments
E. Prescription of oral antibiotics if throat
Correct answer: CE
swab is positive

Nikolsky sign is positive if a shearing force is


Answer & Comments
applied to apparently normal skin and results in
Correct answer: D partial epidermal detachment. It is traditionally
a sign of pemphigus vulgaris, but may also be
Guttate psoriasis is a pattern of psoriasis, which positive in toxic epidermal necrolysis and
classically occurs in response to a streptococcal staphylococcal scalded skin syndrome. It is
infection. thought to be due to damage to the
desmosomes (eg by autoantibodies) which
Guttate psoriasis typically clears very rapidly
leads to an intra-epidermal split.
with UVB light treatment.
Topical vitamin D analogues can help, and left [ Q: 10 ] MasterClass Part2
alone some patients improve and may clear.
(2010) - Dermatology
Recurrent episodes of streptococcal infections
are classically associated with a further episode A 28-year-old man presents with a 3-month
of guttate psoriasis. history of the daily development of itchy
erythematous weals each of which lasts several
Ongoing streptococcal infection can prevent hours and then resolve without scaling.
resolution of the rash.
Which of the following are true of chronic
urticaria?

Dr. Khalid Yusuf El-Zohry – Sohag Teaching Hospital (01118391123) Page | 700
El-Zohry MRCP Questions Bank (Part 2) – Medical Masterclass 2010

A. With a careful history, it is possible to with moderate-severe disease. Non-steroidals


identify the trigger in virtually all cases. are a common cause of urticaria exacerbation.
B. It is thought to be predominantly IgE-
mediated. [ Q: 11 ] MasterClass Part2
C. It is associated with internal malignancy. (2010) - Dermatology
D. It can be associated with systemic lupus A 68-year-old woman presents with a skin rash
erythematous (SLE). (see image).

E. Vasculitic changes are often seen on


biopsy.
F. H1-antagonists provide an important
therapeutic option.
G. H2-antagonists have no significant role in
therapy.
H. Most patients have disease lasting longer
than 10 years.
I. Essential investigations would include a
chest radiograph.
J. non-steroidal anti-inflammatory drugs are
often useful therapeutically.

Answer & Comments

Correct answer: DF

In approximately 50% of patients with chronic


urticaria, a precipitant is identified, but IgE-
mediated chronic urticaria is thought to be a
relatively minor cause. It is now not thought to
What are the TWO most likely diagnoses?
be associated with internal malignancy but can
rarely be associated with systemic vasculitides. A. Mycosis fungoides
However, in the vast majority of cases there are B. Atopic eczema
no vasculitic changes on biopsy. H1-antagonists
can be very helpful and indeed H2-antagonists C. Dermatitis herpetiformis
can help in some patients. Most patients with D. Contact dermatitis
chronic urticaria will have improved within a
E. Pityriasis versicolor
year, but relapses are not infrequent. For
uncomplicated (with no clear clues from the F. Discoid lupus erythematosus
history or examination of systemic disease or
G. Psoriasis
an exogenous precipitant) chronic urticaria, the
current UK guidelines (2003) recommend full H. Erythrodermic drug eruption
blood count, ESR and thyroid antibodies as I. Pemphigus
reasonable screening investigations for those
J. Pemphigoid.

Dr. Khalid Yusuf El-Zohry – Sohag Teaching Hospital (01118391123) Page | 701
El-Zohry MRCP Questions Bank (Part 2) – Medical Masterclass 2010

Answer & Comments Answer & Comments

Correct answer: AG Correct answer: HI

This woman was initially thought to have ACE-inhibitor-associated angio-oedema is an


psoriasis but turned out to have mycosis important cause of angio-oedema. It typically
fungoides. A clinical clue that would lead you to starts within 3 weeks of treatment but can be
the correct diagnosis would be that the delayed, which can clearly contribute to
individual patches showed wide variation in diagnostic difficulty. The mechanism is not
colour, size and shape. thought to be IgE-mediated but possibly to the
potentiation of bradykinin effects by ACE-
Mycosis fungoides is the commonest type of
inhibitors. The angio-oedema does not improve
primary cutaneous lymphoma.
on continued therapy and he should not be
given an alternative ACE-inhibitor. The
[ Q: 12 ] MasterClass Part2 reactions can be life threatening. Skin testing
(2010) - Dermatology currently has no role to play in investigation of
ACE-inhibitor-associated angio-oedema.
A 55-year old man with hypertension presents
Acquired C1 esterase deficiency can be
with new onset angio-oedema.
associated with systemic disease including
Which of the following are true? lymphoma and SLE. The C4 is typically low
during the attacks and is often low between
A. Angiotensin-converting enzyme (ACE) attacks.
inhibitor-associated angio-oedema
follows within minutes after tablet
ingestion.
[ Q: 13 ] MasterClass Part2
(2010) - Dermatology
B. ACE-inhibitor-associated angio-oedema is
associated with fall in complement levels. A 40-year-old female presents with redness and
swelling of the eyelids. There is no itching or
C. ACE-inhibitor-associated angio-oedema is
scaling and the swelling is persistent.
associated with drug-specific IgE.
D. ACE-inhibitor-associated angio-oedema Which TWO investigations would be most
usually improves on drug continuation. helpful in reaching a diagnosis?
E. ACE-inhibitor-associated angio-oedema is A. Chest radiograph
best investigated with skin testing. B. Patch tests
F. ACE-inhibitor-associated angio-oedema is C. Autoantibody screen
associated with atopy.
D. Skin biopsy for direct
G. It is likely to be safe to switch him to an immunofluorescence
alternative ACE-inhibitor.
E. Creatinine kinase level
H. Acquired C1 esterase deficiency is
associated with lymphoma. F. Thyroid function tests

I. Individuals with acquired C1 esterase G. Indirect immunofluorescence


deficiency can have low C4 between H. Prick tests
attacks.
I. CA125 level
J. Hereditary angio-oedema typically
J. Electromyography (EMG).
responds well to anti-histamines.

Dr. Khalid Yusuf El-Zohry – Sohag Teaching Hospital (01118391123) Page | 702
El-Zohry MRCP Questions Bank (Part 2) – Medical Masterclass 2010

Answer & Comments Chronic idiopathic urticaria is a very common


condition which is self-limiting and is usually
Correct answer: EJ
controlled by oral antihistamines. It is known to
be exacerbated by aspirin, NSAIDS, heat and
The lack of scaling and itching rules out eczema alcohol.
(atopic and contact allergic) and hay fever thus
patch tests and prick tests would be unhelpful. Most cases are idiopathic with no underlying
Hypo and hyperthyroidism, particularly allergic cause. Referral for allergy tests is
Graves’s disease, can cause periorbital oedema therefore not appropriate for the majority of
but usually without erythema. Dermatomyositis cases. Those cases of urticaria which are allergic
causes erythema and oedema of the eyelids in aetiology are thought to be type 1
therefore a creatinine kinase level and EMG hypersensitivity reactions, but patch testing
would be helpful in confirming the diagnosis. A identifies type 4 delayed hypersensitivity
chest radiograph and CA125 level would be reactions (response D is therefore wrong for
helpful in screening a patient with two reasons).
dermatomyositis for an underlying malignancy.
ANA may be positive in dermatomyositis, and [ Q: 15 ] MasterClass Part2
more specifically anti-Jo1 antibodies may be (2010) - Dermatology
detected, but a negative result does not
exclude dermatomyositis. Direct and indirect A 30-year-old black woman with epilepsy
immunofluorescence are the investigations of presents with a short history of fever, facial
choice to diagnose autoimmune blistering oedema, lymphadenopathy and a widespread
diseases such as bullous pemphigoid but would rash.
be of little value in this case.
Which of the following abnormalities is most
likely to be revealed by blood tests?
[ Q: 14 ] MasterClass Part2
A. Neutropenia
(2010) - Dermatology
B. Eosinophilia
A 30-year-old male presents with a 6-month
history of a recurring itchy rash. Clinically, you C. Raised bilirubin
suspect chronic idiopathic urticaria. D. Neutrophilia

Which of the following is NOT appropriate E. Thrombocytopenia


management for him?
Answer & Comments
A. Prescribing oral antihistamines
Correct answer: B
B. Advising avoidance of aspirin and NSAIDS
C. Reassuring patient that it should be self-
The history is suggestive of a drug
limiting
hypersensitivity reaction, also known as DRESS
D. Referring for patch testing syndrome (drug rash with eosinophilia and
systemic symptoms). Features include a
E. Advising keeping cool and avoiding excess
maculopapular or erythrodermic rash with
alcohol when the rash is present.
fever, eosinophilia, lymphadenopathy, facial
oedema with or without hepatitis, pneumonitis
Answer & Comments
and myocarditis. Anticonvulsants are common
Correct answer: D culprits and the onset is typically 2-6 weeks
after the drug is started, sometimes longer.
Dr. Khalid Yusuf El-Zohry – Sohag Teaching Hospital (01118391123) Page | 703
El-Zohry MRCP Questions Bank (Part 2) – Medical Masterclass 2010

[ Q: 16 ] MasterClass Part2 B. Pyoderma gangrenosum


(2010) - Dermatology C. Necrobiosis lipoidica
A 60-year-lady with rheumatoid arthritis D. Cutaneous vasculitis
presents with a rapidly enlarging, painful, E. Squamous cell carcinoma.
sloughy leg ulcer on the anterior shin. Doppler
examination is normal. There has been no
Answer & Comments
improvement with dressings and bandages.
Correct answer: B
What would be the most appropriate next
management step?
All of the above can cause leg ulceration.
A. Referral to surgeons for debridement
A squamous cell carcinoma would be very
B. Referral to surgeons for grafting unusual in a 30-year-old, and they tend to be
C. Maggot therapy slow growing and are often painless.

D. Compression Venous leg ulceration would also be unusual in


a young person.
E. Immunosupression.
Necrobiosis lipoidica has a predilection for the
shins but ulceration is not very common. It is
Answer & Comments
often associated with diabetes.
Correct answer: E The clinical history including the site would fit
the diagnosis of pyoderma gangrenosum, and
Rheumatoid arthritis is a risk factor for the history of arthritis in a young patient may
pyoderma gangrenosum (PG). The anterior shin indicate rheumatoid arthritis, tying the case
is an unusual site for venous ulceration. together.
Dopplers are normal, making arterial disease
Vasculitis may also occur in rheumatoid arthritis
unlikely. PG ulcers are often painful and do not
and necrotic areas may ulcerate, but one would
respond to conventional treatment.
expect to see areas of purpura in addition to
PG is associated with pathergy, such that the leg ulcer.
trauma to the ulcer via debridement, or
removal of skin at a distant site for grafting,
[ Q: 18 ] MasterClass Part2
would be contraindicated.
(2010) - Dermatology
PG is diagnosed by exclusion of other causes of
ulceration and by its improvement with A 16-year-old girl with atopic eczema presents
immunosuppression. with a painful eruption on her left face and
neck. There is erythema, eroded papules and
small vesicles. She is febrile and unwell.
[ Q: 17 ] MasterClass Part2
(2010) - Dermatology What is the most likely diagnosis?
A 30-year-old woman with arthritis presents A. Impetigo
with a painful, rapidly enlarging ulcer on the B. Herpes Zoster
lower leg.
C. Eczema herpeticum
What is the most likely diagnosis? D. Varicella-Zoster
A. A venous leg ulcer E. Cellulitis

Dr. Khalid Yusuf El-Zohry – Sohag Teaching Hospital (01118391123) Page | 704
El-Zohry MRCP Questions Bank (Part 2) – Medical Masterclass 2010

Answer & Comments The clinical history is highly suggestive of


erythema nodosum (EN) which may be
Correct answer: C
associated with options A-C and E. Myeloid
malignancies are not associated with EN, but
Eczema herpeticum is a secondary infection of are associated with other skin conditions such
the skin in a patient with herpes simplex and as pyoderma gangrenosum and Sweet's
eczema. It often follows a cold sore or contact syndrome (hot, red, tender plaques often in a
with someone who has a cold sore. It is typically pyrexial patient and usually on the upper body).
painful and, unlike in a patient with normal
skin, the herpes infection spreads quickly and
can be extensive and life-threatening if
[ Q: 20 ] MasterClass Part2
untreated. It can sometimes be difficult to (2010) - Dermatology
distinguish from secondary bacterial infection, A 5-year-old girl, who appears to be well,
but the giveaway sign is the presence of small, presents to her GP with a 1-year history of non-
monomorphic vesicles, which are best seen at itchy, non-scaly, red, ringed areas on the
the spreading edge of the infection. In other dorsum of both feet.
areas they may be confluent and eroded on a
background of oedema and erythema. What is the most likely diagnosis?
Impetigo may complicate eczema but is usually A. Granuloma annulare (GA)
less painful and patients are generally well.
B. Ringworm (tinea)
Pain and vesicles would occur in shingles
C. Atopic eczema
(herpes zoster) but the clinical signs would
usually be confined to one dermatome. D. Psoriasis
Chicken pox (varicella-zoster) would be E. Contact dermatitis
widespread and symmetrical.
Answer & Comments
[ Q: 19 ] MasterClass Part2 Correct answer: A
(2010) - Dermatology
A 30-year-old female develops multiple hot red Many skin disorders can form ringed (annular)
tender nodules on both lower legs. patterns, however the two classical disorders
are GA and ringworm (tinea). GA is
Which of the following is least likely in your asymptomatic and non-scaly, while ringworm is
differential diagnosis? typically itchy and scaly. Atopic eczema,
A. Sarcoidosis psoriasis and contact dermatitis are also usually
scaly and symptomatic.
B. TB
C. Lymphoma [ Q: 21 ] MasterClass Part2
D. Chronic myeloid leukaemia (2010) - Dermatology
E. Crohn's disease. A 23-year-old woman presents with a rash on
her eyelids (see image). She has no other
Answer & Comments problems with her skin.

Correct answer: D

Dr. Khalid Yusuf El-Zohry – Sohag Teaching Hospital (01118391123) Page | 705
El-Zohry MRCP Questions Bank (Part 2) – Medical Masterclass 2010

C. Dermatomyositis
D. Hypothyroidism
E. Polyarteritis nodosa

Answer & Comments

Correct answer: C

Dermatomyositis (DM) clasically presents with a


The likely diagnosis is: periorbital heliotrope rash, but at times the
A. atopic eczema purple discoloration is not seen and the
diagnosis of DM is one to consider in a patient
B. contact dermatitis with periorbital oedema. The finding of
C. psoriasis erythematous papules over the knuckes
(Gottren's papules), linear erythema along the
D. systemic lupus erythematosus
dorsal surface of the fingers, and ragged,
E. sarcoidosis. erythematous nailfolds indicates DM.
There is often a detectable proximal myopathy
Answer & Comments and patients will struggle with activities that
Correct answer: B involve lifting their arms above the shoulders,
as in this case. There is an association with
malignancy, particularly lung and upper
Her erythematous, scaly, lichenified eyelids
aerodigective tract. The patient's cough
were due to contact dermatitis caused by
suggests he may have an underlying lung
allergy to the eye make-up she had been using.
cancer.
Atopic eczema could produce a rash on the
Hypothyroidism can cause periorbital oedema
eyelids like this, but not in isolation. Look in
and a proximal myopathy but doesn't explain
particular for a history of atopy and
the rash.
involvement of the limb flexures when
considering this diagnosis. PAN often presents with middle aged men and
could explain his cough but causes myalgia and
Systemic lupus erythematosus characteristically
a vasculitic rash.
causes a photosensitive rash, which would
spare the upper eyelid and not affect it.
[ Q: 23 ] MasterClass Part2
[ Q: 22 ] MasterClass Part2 (2010) - Dermatology
(2010) - Dermatology A 70-year-old lady is an inpatient with a
diagnosis of bullous pemphigoid. She has many
A 50-year-old male is referred with periorbital
new, intact blisters over her trunk and limbs.
oedema and a rash on the back of his hands. He
She is started on 40mg of prednisolone as
also has a cough and is having difficulty stacking
treatment.
the high shelves at work in a supermarket.
Which of the following additional management
What is the most likely diagnosis?
suggestions are not recommended or
A. Angioedema unnecessary?
B. Allergic contact dermatitis A. Regular dipstick urine for glucose
Dr. Khalid Yusuf El-Zohry – Sohag Teaching Hospital (01118391123) Page | 706
El-Zohry MRCP Questions Bank (Part 2) – Medical Masterclass 2010

B. Regular blood pressure Answer & Comments


C. Oral bisphosphonate Correct answer: B
D. Leave blisters intact to prevent infection
Her history of proximal muscle weakness and
E. Regular count of new blisters.
rash is compatible with an inflammatory
myositis - either complicating SLE or
Answer & Comments dermatomyositis. Although she has a positive
Correct answer: D ANA, her negative ENA and DNA and normal
complement make dermatomyositis more
likely. Patients with idiopathic inflammatory
Any patient started on oral steroids, and
myositis frequently have a positive ANA
particularly the elderly, should be monitored
without ENA positivity. The ANA is usually
for steroid side effects. This includes
directed against other nuclear antigens such as
hypertension, diabetes and gastric erosions. An
PM-Scl and U1RNP.
oral bisphosphonate will protect against
osteoporosis.
[ Q: 25 ] MasterClass Part2
A blister chart is useful for inpatients to monitor
disease progression and control. If no new (2010) - Dermatology
blisters are occurring then steroid doses can be A 52-year-old woman is referred to the
reduced. Dermatology Clinic with several months' history
Current practice is to burst tense blisters with a of generalised pruritus. On examination, apart
sterile needle to release fluid. The roof of the from non-specific excoriations, she is noted to
blister is left in situ over the eroded surface. No have yellowish plaques on her upper eyelids
increase in infection rate is seen. It is more and a smooth skin-coloured nodule overlying
comfortable for the patient and healing is felt her left Achilles tendon.
to be enhanced.
What is the most likely diagnosis?

[ Q: 24 ] MasterClass Part2 A. Familial hypercholesterolaemia


(2010) - Dermatology B. Primary biliary cirrhosis

A 35-year-old woman recounts a 2-month C. Dysbetalipoproteinaemia


history of weakness; she is having problems D. Diabetes mellitus
climbing the stairs and rising from a chair. On
E. Lipoprotein lipase deficiency
examination, you observe that she has an
erythematous rash on her cheeks. An ANA
comes back as strongly positive with a negative Answer & Comments
ENA and DNA. Correct answer: B
Which is the most likely diagnosis?
Hepatic cholestasis may be associated with
A. SLE tendinous and plane xanthomas (including
B. Dermatomyositis xanthelasma) and would also explain her
pruritus.
C. Mixed connective tissue disease
D. Anti-phospholipid syndrome
E. Primary Raynaud’s disease.

Dr. Khalid Yusuf El-Zohry – Sohag Teaching Hospital (01118391123) Page | 707
El-Zohry MRCP Questions Bank (Part 2) – Medical Masterclass 2010

[ Q: 26 ] MasterClass Part2 What is the most likely diagnosis?


(2010) - Dermatology A. Systemic sclerosis

A 60-year-old man presents with a 1-month B. Psoriasis


history of a rash (see image). C. Sezary syndrome
He believes that it has been caused by exposure D. Atopic eczema
to sunlight while gardening. He is taking many
new medications and you wonder whether he E. Staphylococcal scalded skin syndrome.
has developed a photosensitive drug eruption.

Which of the following drugs do you think may


Answer & Comments
have caused this rash?
Correct answer: C
A. Codeine
B. Beta-blocker
Options B, C and D can cause erythroderma
C. Thiazide diuretic which describes widespread erythema of the
skin affecting at least 90% of the surface area.
D. Angiotensin-converting enzyme inhibitor
Systemic sclerosis produces diffuse fibrosis of
E. Warfarin the skin but not erythema.
Staphylococcal scalded skin syndrome is due to
Answer & Comments haematogenous dissemination of a toxin
Correct answer: C (exfolative toxin) produced by certain strains of
staphylococcus aureus. Its substrate is a protein
in the epidermis called desmoglein 1. It
The commonest drugs that cause a
presents acutely, usually in children, causes a
photosensitive rash are antibiotics (especially
burning sensation in the skin and superficial
tetracyclines), NSAIDs, amiodarone,
blistering.
phenothiazines and thiazides.
Psoriasis, atopic eczema and the sezary
[ Q: 27 ] MasterClass Part2 syndrome can all present with a scaly
erythroderma. Erythrodermic psoriasis tends to
(2010) - Dermatology
be sore rather than itchy. Modest peripheral
A 60-year-old female has been erythrodermic, lymphadenopathy can occur in erythroderma,
scaly and itchy for several months. There is whatever the cause, as a reactive phenomenon.
peripheral lymphadenapathy and a blood film However, it is a diagnostic feature of the sezary
shows atypical lymphocytes. syndrome, a type of cutaneous T-cell lymphoma

Dr. Khalid Yusuf El-Zohry – Sohag Teaching Hospital (01118391123) Page | 708
El-Zohry MRCP Questions Bank (Part 2) – Medical Masterclass 2010

in which there is systemic involvement. The Answer & Comments


lymph nodes may be large.
Correct answer: C
The finding of atypical lymphocytes in the
peripheral blood and lymphadenopathy This is a commonly recounted history in the
indicates Sezary syndrome is the most likely
elderly with scabies. The hospital admission is
diagnosis in this scenario. an important clue because institutions are a
common source of infestation, particularly if
[ Q: 28 ] MasterClass Part2 intensive nursing was involved. Symptoms do
(2010) - Dermatology not occur for several weeks after the initial
infestation because the symptoms and signs are
A 25-year-old female presents with due to a hypersensitivity reaction. This can
hypopigmentation on the knees and elbows. build up to produce a widespread eczema. It is
important to examine the hands and feet where
What is the most likely diagnosis?
the typical burrows of scabies may be seen
A. Leprosy most easily.
B. Vitiligo Bullous pemphigoid can be mistaken for
C. Post-inflammatory hypopigmentation eczema in its early stages, prior to the
development of blisters, and is most commonly
D. Pityriasis versicolor seen in the elderly. Varicose eczema is also
E. Halo naevi. common in the elderly. It is usually sited on the
lower legs but can become widespread. Atopic
eczema is also possible in this case but less
Answer & Comments
likely given the recent onset with no history of
Correct answer: B atopy. Xerosis is dry skin, is common in the
elderly and is a cause of itching.
All of the above are hypopigmented. However,
the symmetrical distribution on pressure points [ Q: 30 ] MasterClass Part2
is very typical of vitiligo. (2010) - Dermatology
A 25-year-old man with atopic eczema has a
[ Q: 29 ] MasterClass Part2
positive skin-prick test reaction to house dust
(2010) - Dermatology
mites.
An 80-year-old female is referred with
widespread, refractory eczema of 3 months’ Which of the following is true?
duration. There is no past medical history other A. House dust mite reactivity is definitely
than a hospital admission for a fractured neck the cause of his eczema.
of femur six months ago.
B. He should spray acaricide throughout the
What is the most likely diagnosis? house.

A. Bullous pemphigoid C. He should remove bedroom carpets.

B. Atopic eczema D. He should install a de-humidifier.

C. Scabies E. None of the above.

D. Varicose eczema
E. Xerosis.

Dr. Khalid Yusuf El-Zohry – Sohag Teaching Hospital (01118391123) Page | 709
El-Zohry MRCP Questions Bank (Part 2) – Medical Masterclass 2010

Answer & Comments should be treated with a topical scabecide.


General advice is to apply treatment topically
Correct answer: E
from neck down in adults and children, but
from head to toe in babies where the scalp can
Individuals with atopic dermatitis may show be affected.
reactivity to many different environmental
antigens (e.g. house dust mites, grass pollen, The rash and itch associated with scabies is
cat and dog dander) on skin prick testing, but it usually delayed from time of infestation, as
is unclear to what extent this causes disease. symptoms occur due to an allergic type
response to the mite eggs or faeces. The father
House dust mite avoidance for individuals with must therefore be treated even if
atopic dermatitis is still controversial, but may asymptomatic at present.
benefit some patients. A number of measures
do help to reduce house dust mite levels but it Mites do not usually survive in clothing and
is difficult to predict who will benefit. bedding, therefore boil washes are not needed.
Impermeable mattress, pillow and duvet covers However, normal washing of linen is usually
are thought to be relatively effective at recommended.
reducing house dust mite exposure.
[ Q: 32 ] MasterClass Part2
[ Q: 31 ] MasterClass Part2 (2010) - Dermatology
(2010) - Dermatology A 30-year-old man with a history of atopic
A mother and her three children, one a 6- eczema presents a painful widespread vesicular
month-old baby, present to their GP with an rash (which he has had for two days) and
itchy rash that has been present for two weeks. constitutional upset.
The father has no rash and no symptoms of
What is the most likely diagnosis?
itch. Clinically scabies infestation is suspected.
A. Eczema herpeticum
Which of the following advice is not
B. Severe exacerbation of atopic eczema
appropriate?
C. Bacterial infection
A. The whole family should be treated with
a topical scabecide. D. Drug reaction

B. Application of the scabecide from the E. Molluscum contagiosum.


neck down is only needed for adults.
C. No scabecide is needed for father as he is Answer & Comments
unaffected. Correct answer: A
D. Scabecide should be applied head to toe
for the baby. Eczema herpeticum can develop in the absence
E. Wash the bedclothes and linen. of a history of herpes simplex reactivation or
contact. The other diagnoses are all seen in
individuals with atopic eczema, but the
Answer & Comments
hallmark features of pain and vesicals are highly
Correct answer: C suggestive.

Scabies infestation is extremely common. All


people living in a house with an affected person

Dr. Khalid Yusuf El-Zohry – Sohag Teaching Hospital (01118391123) Page | 710
El-Zohry MRCP Questions Bank (Part 2) – Medical Masterclass 2010

[ Q: 33 ] MasterClass Part2 reported and examined dermatitis, and similar


(2010) - Dermatology trends have been observed in skin prick test
reactivity.
A very anxious 28-year-old woman with atopic
dermatitis comes to see you in outpatients. She
[ Q: 34 ] MasterClass Part2
is concerned about the chances of her children
inheriting the condition, and of whether
(2010) - Dermatology
'lifestyle' issues have any impact on the A 28-year-old woman presents with a red rash
condition. on her face (see image).

Which one of the following statements about


atopic dermatitis is true?
A. It affects females far more frequently
than males.
B. It has a monozygotic twin concordance of
less than 50%.
C. It has greatly increased prevalence in
rural areas compared to urban areas.
D. has had a two to three-fold increase in
prevalence in developed countries in the
last 30 years.
E. Paternal atopy carries a greater risk of
offspring disease than maternal atopy.

Answer & Comments

Correct answer: D

Some studies have shown a slight increase in


prevalence in females compared to males in
childhood. The monozygotic twin concordance
rate is as high as 85%, with the dizygotic rate
being approximately 20%. Maternal atopy What are the TWO most likely diagnoses?
carries greater risk of disease in the offspring
than paternal atopy. A. Seborrhoeic dermatitis

Environmental factors are thought to be B. Contact dermatitis


important in explaining the increased C. Atopic eczema
prevalence in developed countries over the last
D. Acne
century. For example, migrant individuals from
countries with a low prevalence tend to assume E. Rosacea
rates of disease which are closer to that found F. Tinea faciei
in the adopted country. There is also a strong
link between atopic dermatitis and higher G. Systemic lupus erythematosus
socio-economic group and smaller family size. H. Discoid lupus erythematosus
These findings have been confirmed for

Dr. Khalid Yusuf El-Zohry – Sohag Teaching Hospital (01118391123) Page | 711
El-Zohry MRCP Questions Bank (Part 2) – Medical Masterclass 2010

I. Dermatomyositis Which of the following is a sign that her


prognosis for complete hair regrowth / lack of
J. Sarcoidosis.
progression may not be good.

Answer & Comments A. Hair loss at the occiput (ophiasis)


B. Hair loss at the anterior scalp margin
Correct answer: AC
C. Normal nails
All of the conditions listed can present as a D. Lack of 'exclamation mark' hairs at edge
chronic red facial rash. This image, however, of patches
shows seborrheic dermatitis characterized by
E. Presence of seborrheic dermatitis.
poorly defined, erythematous, scaly macules
and patches, with a predeliction for the
nasolabial folds, eyebrows and scalp. It is this Answer & Comments
distribution that allows it to be differentiated Correct answer: A
from atopic eczema, which is the other
common diagnosis that would require serious
Hair loss in an 'ophiasis' pattern is associated
consideration in this case
with a poor prognosis. Pitted nails are
sometimes associated with AA, and maybe a
[ Q: 35 ] MasterClass Part2 marker of severity. ‘Exclamation mark’ hairs are
(2010) - Dermatology short stubby hairs found at the edge of all
'active' patches of hair loss in AA. Seborrheic
A 72-year-old man has toxic epidermal dermatitis has no association with AA.
necrolysis. Potential complications do NOT
typically include:
[ Q: 37 ] MasterClass Part2
A. Infection (2010) - Dermatology
B. Dehydration
A 30-year-old woman presents with intensely
C. Temperature dysregulation itchy grouped vesicles on both buttocks. You
suspect dermatitis herpetiformis clinically.
D. Protein loss
E. Gastrointestinal haemorrhage Which of the following is NOT true:
A. Gluten-sensitive enteropathy is
Answer & Comments commonly associated
Correct answer: E B. IgA is deposited in the superficial dermis
C. A gluten-free diet is rarely helpful in
Although GI haemorrhage can occur, it is not practice
thought to be a typical feature.
D. Dapsone is often helpful

[ Q: 36 ] MasterClass Part2 E. Histology shows a subepidermal blister


with a neutrophilic infiltrate
(2010) - Dermatology
A 30-year-old woman presents with three Answer & Comments
patches of hair loss. Clinically, the scalp is
normal within the patches and you suspect a Correct answer: C
diagnosis of alopecia areata (AA).

Dr. Khalid Yusuf El-Zohry – Sohag Teaching Hospital (01118391123) Page | 712
El-Zohry MRCP Questions Bank (Part 2) – Medical Masterclass 2010

Although the skin can be slow to respond, a plaques. Individual lesions come and go over
gluten-free diet is recommended. several hours, leaving no trace.

What is the most likely diagnosis?


[ Q: 38 ] MasterClass Part2
(2010) - Dermatology A. Epstein Barr virus (EBV) infection
B. Drug hypersensitivity syndrome
A 35-year-old male runner has thickened,
discoloured toenails on three toes with C. Scarlet fever
onycholysis and subungual hyperkeratosis. D. Cutaneous vasculitis

What is the most likely diagnosis? E. Urticaria.


A. Psoriasis
Answer & Comments
B. Lichen planus
Correct answer: E
C. Onychomycosis
D. Alopecia areata Lesions which come and resolve without trace,
E. Darier's disease typically within 24 hours, are characteristic of
urticaria. The other diagnoses would all fit the
Answer & Comments clinical scenario but individual lesions would
take several days minimum to resolve.
Correct answer: C
Amoxycillin in the context of EBV infection can
give a florid maculo-papular eruption. Penicillin
The clinical signs are consistent with antibiotics may trigger a variety of drug
onychomycosis (fungal nail infection). These eruptions including a cutaneous vasculitis and
clinical signs could also be seen in psoriasis but the drug hypersensitivity syndrome. The latter
there is usually additional pitting and term should be reserved for a multi-system
involvement of fingernails. Pitting does not drug reaction consisting of a widespread
occur in onychomycosis. eruption which may progress to erythroderma,
Being a runner, he is at risk of tinea pedis often with facial oedema, plus peripheral
(athlete’s foot), which is commonly seen in lymphadenopathy, peripheral blood
association with onychomycosis and is caused eosinophilia, atypical lymphocytosis and
by the same organisms. All the other disorders visceral involvement which may include the
are associated with nail dystrophy, but the signs liver and lungs.
are different. In alopecia areata - pitting is seen; Scarlet fever is produced by a streptococcal
in lichen planus - longitudinal ridges and toxin
thinning, and in Darier’s disease - red and white
longitudinal lines and notches in the end of the
nail. [ Q: 40 ] MasterClass Part2
(2010) - Dermatology
[ Q: 39 ] MasterClass Part2 A 25-year-old woman presents with scaly red
(2010) - Dermatology plaques on the face, scalp and upper torso. She
reports that it is worse in the summer.
A 16-year-old male is given amoxycillin for a
sore throat. Five days later, he develops a rash Which of the following is the best option?
consisting of erythematous papules and
A. Nail pitting may be detected.

Dr. Khalid Yusuf El-Zohry – Sohag Teaching Hospital (01118391123) Page | 713
El-Zohry MRCP Questions Bank (Part 2) – Medical Masterclass 2010

B. A trial of topical corticosteroids is D. Tinea capitis can be successfully treated


indicated. with topical terbinafine.
C. Direct immunofluorescence would be E. Tinea capitis is commoner in children
unhelpful. than adults.
D. Patch tests are indicated. F. Direct microscopy of plucked hairs is
helpful in tinea capitis.
E. A skin biopsy is necessary.
G. Onychomycosis can be treated
Answer & Comments successfully with oral terbinafine.

Correct answer: E H. Melanychia (brown/black nail


discoloration) can occur in
onychomycosis.
The history suggests discoid lupus
erythematosus (DLE). Individual lesions of I. Regional lymphadenopathy can occur in
psoriasis and DLE may look similar but the tinea capitis.
distribution in this case is typical of DLE, not J. Onycholysis is a sign of onychomycosis.
psoriasis. Therefore nail pitting, a feature of
psoriasis, would be unlikely. DLE is
Answer & Comments
photoaggravated and therefore worse in the
summer, whereas psoriasis usually improves Correct answer: BD
with sun exposure. In cases of suspected DLE,
the diagnosis should be confirmed with a skin Nail and hair infections cannot be successfully
biopsy prior to commencing treatment. If eradicated with topical therapy and systemic
lesional skin is also sent for direct treatment is required. Terbinafine is an
immunofluorescence, granular deposits of effective treatment in most cases.
immunoglobulins and complement will be seen
along the basement membrane zone (a lupus Wood’s light examination has a role in tinea
capitis, when microsporum infected hairs glow
band) in the majority of cases. Patch tests are
green. However, many infections nowadays are
unlikely to help in this case.
with Trichophyton tonsurans which does not
show up under Wood’s light. Direct microscopy
[ Q: 41 ] MasterClass Part2 and culture of hairs and skin scrapings is
(2010) - Dermatology required to confirm the diagnosis.
A patient presents with thickened, discoloured Tinea capitis is rare in adults but seen most
toenails and scaly feet. commonly in inner city children, particularly
black children. Regional lymphadenopathy is
Which of the following statements about common in tinea capitis.
superficial fungal infections are false?
Onychomycosis most commonly presents with
A. Tinea pedis may be successfully treated nail thickening, yellowish discoloration,
with topical terbinafine. subungual hyperkeratosis and onycholysis
B. Wood's light examination is helpful in the (these are also features of nail psoriasis).
investigation of tinea pedis. Occasionally, the fungi can produce melanin,
which results in melanychia
C. Wood's light examination is helpful in the
investigation of tinea capitis.

Dr. Khalid Yusuf El-Zohry – Sohag Teaching Hospital (01118391123) Page | 714
El-Zohry MRCP Questions Bank (Part 2) – Medical Masterclass 2010

[ Q: 42 ] MasterClass Part2 Answer & Comments


(2010) - Dermatology Correct answer: BF
An elderly man complains of a rash that
developed after a hot weekend in April. The distribution and seasonal timing of this rash
Although his medication had not been changed suggest photosensitivity. Although occasional
since the previous November, his GP suspected reports may have been documented for some
a drug reaction. of the other drugs listed, amiodarone and
thiazides are the most likely culprits. The
problem is very common with amiodarone so
that all patient started on the drug must be
warned and given advice about avoiding sun
exposure. Photosensitivity is much less
common with thiazides, but these are very
widely prescribed so it is important to recognise
this.

[ Q: 43 ] MasterClass Part2
(2010) - Dermatology
A 70-year-old female with cardiac failure has
dry, red, itchy and swollen legs.

What is the most likely diagnosis?


A. Allergic contact dermatitis
B. Cellulitis
C. Erythema nodosum
D. Worsening cardiac failure
E. Gravitational eczema
Which TWO of the following are likely causes?
A. Frusemide Answer & Comments
B. Bendrofluazide
Correct answer: E
C. Temazepam
D. Amlodipine Gravitational eczema is common in elderly
patients with venous hypertension and
E. Digitalis
peripheral oedema. It is also referred to as
F. Amiodarone stasis eczema and in the context of varicose
G. Dihydrocodeine veins, is also referred to as varicose eczema or
venous eczema. The skin is typically red, dry,
H. Sotalol scaly and itchy.
I. Nitrazepam Making the diagnosis of cellulitis is a common
J. Atenolol. mistake. Cellulitis is not scaly nor itchy and is
rarely bilateral. It is usually painful and patients
are unwell with a fever.

Dr. Khalid Yusuf El-Zohry – Sohag Teaching Hospital (01118391123) Page | 715
El-Zohry MRCP Questions Bank (Part 2) – Medical Masterclass 2010

Treatment of gravitational eczema involves [ Q: 45 ] MasterClass Part2


emollients and topical corticosteroids plus (2010) - Dermatology
treatment of the underlying oedema and
venous hypertension with compression hosiery. A 60-year-old man with hypertension develops
Allergic contact dermatitis can complicate episodes of lip swelling that occur at any time
gravitational eczema if patients are allergic to and resolve within 24 hours.
one of the creams they are using to treat
themselves, or to the bandages being applied. Which of the following diagnoses would be the
most likely:

[ Q: 44 ] MasterClass Part2 A. ACE inhibitor associated angioedema


(2010) - Dermatology B. Oral candida

A 50-year-old man presents with hair loss. C. Perioral dermatitis


Examination reveals patches of scarring D. Melkersson-Rosenthal syndrome
alopecia with surrounding inflammation.
E. Nut allergy
Which is the most likely diagnosis?
Answer & Comments
A. Lichen planopilaris
B. Androgenetic alopecia Correct answer: A

C. Alopecia areata
ACE inhibitor associated angioedema is a well-
D. Traction alopecia recognised complication of therapy and may
E. Trichotillomania. not develop straight away. It is therefore often
inappropriately ruled out as a potential cause
for the angioedema. It usually resolves rapidly
Answer & Comments
on stopping the ACE inhibitor.
Correct answer: A
Nut allergy would be rare to develop at this age
and the episodes were documented to be
The two most common causes of scarring occurring at any time
alopecia are lichenplanopilaris (lichen planus
affecting the scalp hair follicles) and discoid
[ Q: 46 ] MasterClass Part2
lupus erythematosus. Both conditions cause
inflammation, and can be difficult to (2010) - Dermatology
differentiate clinically. A 43-year-old man presents with intensely itchy
B and C are both non-scarring causes of papules on ventral aspects of his wrists
alopecia. bilaterally.

Traction alopecia can cause scarring but is not A differential diagnosis would be unlikely to
usually associated with inflammation. Alopecia include:
occurs at sites of traction and is usually caused
by hair styling practices, such as braiding. A. Lichen planus
B. Scabies
C. Dermatitis
D. Urticaria
E. Psoriasis

Dr. Khalid Yusuf El-Zohry – Sohag Teaching Hospital (01118391123) Page | 716
El-Zohry MRCP Questions Bank (Part 2) – Medical Masterclass 2010

Answer & Comments cryoglobulinaemia, although there is an overlap


of histological features.
Correct answer: E
DIC is usually acute with multi-system
Although psoriasis can be itchy, this is rarely a manifestations but the cutaneous features and
histology would be similar to this case.
major feature and when present is not usually
intense. Itchy lesions on the wrist are common Venous and arterial leg ulcers are not
in lichen planus, scabies and atopic dermatitis. associated with purpura.
Urticaria is itchy and can occur anywhere.
[ Q: 48 ] MasterClass Part2
[ Q: 47 ] MasterClass Part2 (2010) - Dermatology
(2010) - Dermatology
A 16-year-old girl presents with an itchy rash 3
A 40-year-old male, who is an IV drug-user, days after arriving in Majorca on holiday.
presents with a 3-month history of purpura and Examination reveals an erythematous papular
ulcers on the feet and lower legs. A skin biopsy rash over the arms and trunk, sparing sites
shows occlusion of the vascular lumini with under her swimming costume and sparing her
thrombi. face and hands.

What is the MOST likely diagnosis? What is the most likely diagnosis?
A. Venous leg ulcers A. Systemic lupus erythematosus
B. Vasculitis B. Polymorphic light eruption
C. Arterial leg ulcers C. Photoallergic contact dermatitis
D. Disseminated intravascular coagulation D. Scabies
E. Cryoglobulinaemia E. Xeroderma pigmentosum.

Answer & Comments Answer & Comments

Correct answer: E Correct answer: B

Cryoglobulins are serum proteins that are Polymorphic light eruption is a common
soluble at 37oC but precipitate when cooled. photosensitivity disorder which particularly
Hence the clinical manifestations tend to be affects young females. The skin shows
seen on the extremities e.g. hands, feet, nose 'hardening', such that areas frequently exposed
and ears, particularly in winter. Cryoglobulins to the sun, such as the face and hands, may not
precipitate in and occlude blood vessels such be affected whilst newly exposed sites are most
that the signs include livedo, purpura, skin severely affected. The rash often develops after
necrosis and ulceration and digital gangrene. A a few days of sun exposure and is most severe
skin biopsy will show occluded vessels and at the beginning of the summer and improves
there may be an associated vasculitis. Hepatitis at the end of the summer as the skin 'hardens'.
C infection is a recognised association and is the
Systemic lupus erythematosus is a
risk factor in this case.
photoaggravated disorder in which the face is
A primary vasculitis could present like this case, usually affected.
but the histology would help distinguish from

Dr. Khalid Yusuf El-Zohry – Sohag Teaching Hospital (01118391123) Page | 717
El-Zohry MRCP Questions Bank (Part 2) – Medical Masterclass 2010

Photoallergic contact dermatitis can occur due biopsies are only performed under certain
to sunscreen allergy. A rash would appear at all circumstances e.g. confirmation of the
sites where sunscreen had been applied and diagnosis if there is doubt, particularly when an
subsequently exposed to the sun. excisional biopsy is not straightforward.
Scabies is not photoaggavated. As a general rule, acne is a clinical diagnosis and
a biopsy is never required, even in persistent
Patients with xeroderma pigmentosum describe
cases. A biopsy would only be considered in
easy burning after minimal sun exposure. They
cases where there is diagnostic doubt.
subsequently develop freckling, chronic solar
damage and skin tumours.
[ Q: 50 ] MasterClass Part2
[ Q: 49 ] MasterClass Part2 (2010) - Dermatology
(2010) - Dermatology (2) A 60-year-old woman with a recent onset of
gout presents with a 5-day history of a
A patient is due to have a skin biopsy.
widespread erythematous skin eruption and
Which of the following statements about an mouth ulcers. She has lesions on the palms and
incisional skin biopsy are incorrect? some areas of skin are beginning to blister and
ulcerate.
A. It is essential in the diagnosis of
pemphigus vulgaris. What is the most likely diagnosis?
B. It is essential in the management of A. Bullous pemphigoid
malignant melanoma.
B. Fixed drug eruption
C. Lignocaine with adrenaline can be used to
C. Stevens-Johnson syndrome
anaesthetise the nose.
D. Pemphigus vulgaris
D. Keloid scar formation may occur.
E. Erythema multiforme.
E. Is not helpful in persistent cases of acne.

Answer & Comments


Answer & Comments
Correct answer: C
Correct answer: B

Allopurinol, prescribed for gout, is a well-


Scarring will follow any skin biopsy and patients
recognised trigger of Stevens-Johnson
must be warned. Keloid scars are also a risk,
syndrome (SJS). The palmar lesions, mouth
particularly in black skin, and the highest risk
ulcers and short history suggest this diagnosis.
sites are the upper torso and shoulders.
Erythema multiforme is also a possible
Adrenaline aids haemostasis but should be diagnosis but the eruption tends to be less
avoided on the digits where vasoconstriction of widespread than SJS, confined mainly to the
the digital arteries could cause ischaemia. distal limbs, with less tendency blister. An
A skin biopsy is needed to confidently make the autoimmune blistering disease, such as bullous
diagnosis of pemphigus vulgaris and some skin pemphigoid or pemphigoid vulgaris, would tend
should be sent for direct immunofluorescence, to have a longer history before presenting in a
the gold standard investigation. widespread form and palmar lesions are not
typical of either. A fixed drug eruption usually
Optimal management of a suspected malignant
presents with a small number of erythematous
melanoma is an excisional biopsy. Incisional

Dr. Khalid Yusuf El-Zohry – Sohag Teaching Hospital (01118391123) Page | 718
El-Zohry MRCP Questions Bank (Part 2) – Medical Masterclass 2010

plaques, which always recur at the same site [ Q: 52 ] MasterClass Part2


upon re-exposure to the drug (2010) - Dermatology
A 20-year-old trainee hairdresser develops an
[ Q: 51 ] MasterClass Part2
intensely itchy, erythematous scaly rash on her
(2010) - Dermatology hands.
A 48-year-old man presents with a necrotic
The TWO most common diagnoses would be:
lesion on his skin. A diagnosis of pyoderma
gangrenosum is made and you need to consider A. Irritant hand dermatitis
whether it might be secondary to some
B. Contact allergic dermatitis
underlying condition.
C. Psoriasis
Which one of the following conditions is NOT
D. Lichen planus
associated with pyoderma gangrenosum?
E. Urticaria
A. Leukaemia
F. Porphyria
B. Multiple sclerosis
G. Mycosis fungoides
C. Monoclonal gammopathies
H. Erythema multiforme
D. Inflammatory bowel disease
I. Bullous pemphigoid
E. Rheumatoid arthritis.
J. Hand, foot and mouth disease.
Answer & Comments
Answer & Comments
Correct answer: B
Correct answer: AB
Pyoderma gangrenosum is a destructive
necrotising non-infective ulceration of the skin. Although many of the diagnoses listed above
It is associated with: can affect the hands, the commonest diagnoses
in this setting would be an irritant hand
1. Gastrointestinal disease e.g. ulcerative
dermatitis or a contact allergic dermatitis.
colitis, Crohn's disease
Possible precipitants for the latter could include
2. Hepatic disease e.g. chronic active rubber (including latex), hair-dye, and nickel.
hepatitis, primary biliary cirrhosis, However, it would be important to undertake
sclerosing cholangitis patch testing to a wide range of potential
contact allergens in order to precisely identify
3. Joint disease e.g. rheumatoid arthritis,
the precipitant and advise on her future career.
seronegative arthritis
4. Haematological disease e.g.
[ Q: 53 ] MasterClass Part2
leukaemias, lymphoma, monoclonal
gammopathies (2010) - Dermatology
5. Neoplasia e.g. carcinoma of breast,
bronchus, colon A 30-year-old woman presents with an
Others such as infections, diabetes, thyroid intermittent pruritic eruption which has been
disease, lupus erythematosus occurring for several months. Lesions last a few
hours only and disappear leaving no mark,

Dr. Khalid Yusuf El-Zohry – Sohag Teaching Hospital (01118391123) Page | 719
El-Zohry MRCP Questions Bank (Part 2) – Medical Masterclass 2010

suggesting a diagnosis of urticaria. She is atopic Acanthosis nigricans is a warty, velvety,


and had a nut allergy as a child. hyperpigmented thickening of the skin, which
occurs most commonly in the flexures. It can be
Which one of the following is true? idiopathic, familial or drug induced. Many cases
A. Acute urticaria typically responds to are associated with insulin resistance
topical steroids syndromes, including the HAIR-AN syndrome
(HyperAndrogenism, Insulin Resistance and
B. Chronic urticaria is strongly associated
Acanthosis Nigricans). Acne can be one of the
with atopy
features of hyperandrogenism. Malignancy is
C. Infant nut allergy persists into adult life in another cause, particularly adenocarcinomas of
most patients GI and GU tract.
D. Most patients with chronic urticaria do This case is most likely to be paraneoplastic
not have an exogenous trigger given the age of the patient, recent onset and
history of weight loss.
E. Skin prick tests are not useful in diagnosis
of the causes of urticaria Rosacea is not associated with acanthosis
nigricans.
Answer & Comments

Correct answer: D
[ Q: 55 ] MasterClass Part2
(2010) - Dermatology
Most individuals with chronic long-term A 30-year-old woman presents with a 5-year
urticaria do not have an exogenous trigger. IgE- history of flushing of her facial skin and a spotty
mediated urticarial reactions to foods usually rash. Examination reveals a pustular rash on the
follow a clear pattern and can be investigated cheeks with no comedones.
by skin-prick tests or serum IgE quantification.
What is the most likely diagnosis?
[ Q: 54 ] MasterClass Part2 A. Rosacea
(2010) - Dermatology B. Acne vulgaris
A 60-year-old male with weight loss complains C. Carcinoid syndrome
of recent darkening of the skin under his arms.
D. Systemic lupus erythematosus (SLE)
Which of the following statements about E. Allergic contact dermatitis.
acanthosis nigricans is incorrect?
A. It is associated with acne. Answer & Comments

B. It is a cause of hyperpigmentation. Correct answer: A


C. It is associated with gastric
adenocarcinomas. Rosacea often presents with easy facial flushing
before the onset of a pustular rash. Rosacea is
D. It is associated with insulin resistance.
often mistaken for acne, but comedones are
E. It is associated with rosacea. the clinical hallmark of acne and are absent in
rosacea.
Answer & Comments Carcinoid syndrome often presents with
Correct answer: E flushing but is not associated with a pustular

Dr. Khalid Yusuf El-Zohry – Sohag Teaching Hospital (01118391123) Page | 720
El-Zohry MRCP Questions Bank (Part 2) – Medical Masterclass 2010

rash. SLE can cause erythema of the cheeks but E. Alopecia areata.
pustules are not seen.
Allergic contact dermatitis would most Answer & Comments
commonly present with a scaly, eczematous Correct answer: E
rash.

All of the above can cause loss of scalp hair.


[ Q: 56 ] MasterClass Part2 Discoid lupus erythematosus and lichen planus
(2010) - Dermatology are inflammatory skin diseases that can cause
permanent hair loss due to scarring of the hair
A 30-year-old insulin-dependent diabetic
follicles (scarring alopecia). The scalp would
presents with a rash on her shins.
look red and scaly in these two conditions.
Which is the most likely diagnosis? Tinea capitis can produce patchy hair loss but
the scalp would not look normal; there would
A. Erythema nodosum be scaling and erythema. Examination of
B. Pretibial myxoedema plucked hairs or skin scrapings would confirm
the diagnosis.
C. Candidiasis
Alopecia areata and alopecia totalis are
D. Necrobiosis lipoidica
believed to be organ-specific autoimmune
E. Granuloma annulare. diseases targeting hair follicles. Alopecia areata
characteristically presents with round patches
Answer & Comments of complete hair loss and the scalp looks
normal. There is often a short history. Alopecia
Correct answer: D totalis is a more extensive form of alopecia
areata in which the entire scalp hair is lost.
Answers A and B are both rashes with a
predeliction for the shins, but they do not occur
[ Q: 58 ] MasterClass Part2
more commonly in diabetics. Answers C and E
do occur more commonly in diabetics but do
(2010) - Dermatology
not classically affect the shins. When a 21-year-old female returns from
vacation abroad, she notices an eruption
[ Q: 57 ] MasterClass Part2 composed of hypopigmented macules over her
(2010) - Dermatology upper central back and chest.

A 25-year-old female presents with a 3-week What is the most likely diagnosis?
history of bald patches in the scalp. On A. Post-inflammatory hyperpigmentation
examination, there are several round patches
(2-3cm), of completely hairless skin. The skin B. Vitiligo
looks normal. C. Pityriasis versicolor

What is the most likely diagnosis? D. Leprosy

A. Alopecia totalis E. Hypopituitarism.

B. Tinea capitis
Answer & Comments
C. Discoid lupus erythematosus
Correct answer: C
D. Lichen planus

Dr. Khalid Yusuf El-Zohry – Sohag Teaching Hospital (01118391123) Page | 721
El-Zohry MRCP Questions Bank (Part 2) – Medical Masterclass 2010

All of the above can cause hypopigmentation, D. Systemic lupus erythematosus


and options A-D produce localized
E. Seborrhoeic dermatitis.
hypopigmentation. However, the distribution is
very characteristic of pityriasis versicolor, an
infection with the yeast malassezia furfur. Answer & Comments
Warm, moist conditions favour its growth. Correct answer: A
Furthermore, infected skin does not pigment
normally so tanning of the normal, surrounding
The V-shaped cut-off at the neck indicates that
skin makes the infection more obvious. For
the rash is photo-induced. Look also for sparing
these two reasons, patients may not be aware
behind the ears, under the chin and nose, and
of the problem until they take a vacation.
in the upper eyelid fold.
Another clue to the diagnosis is scaling of the
skin which may not be obvious until the skin is Seborrhoeic dermatitis typically affects the
gently abraded. eyebrows and nasolabial folds.
Systemic lupus erythematosus can cause a
[ Q: 59 ] MasterClass Part2 photosensitive rash, but is not a likely diagnosis
(2010) - Dermatology in a middle-aged man.

A 62-year-old man presented with a scaly rash


[ Q: 60 ] MasterClass Part2
on his face and hands (see image).
(2010) - Dermatology
A 79-year-old woman complains of a rash (see
image).

What is the likely diagnosis?


A. Photosensitive eczema
B. Psoriasis
Which of the following statements is NOT
C. Contact dermatitis correct?

Dr. Khalid Yusuf El-Zohry – Sohag Teaching Hospital (01118391123) Page | 722
El-Zohry MRCP Questions Bank (Part 2) – Medical Masterclass 2010

A. The patient complains of itch. Answer & Comments


B. A diagnosis of pemphigus vulgaris is Correct answer: BF
likely.
C. A drug may be responsible. Oral and genital ulceration may be part of
Behçet’s disease, for which a pathergy test is
D. A biopsy for direct immunofluorescence
helpful, but cutaneous ulcers makes this
(DIF) should be taken from normal skin.
diagnosis unlikely. Skin scrapings, which are
E. There is likely to be IgG and C3 deposition generally sent for mycology and thus for
along the dermoepidermal junction. diagnosis of fungal infections, would not be
helpful. The history is not suggestive of an
Answer & Comments allergic contact dermatitis or atopic dermatitis
so investigations such as patch tests, skin prick
Correct answer: B
tests and IgE levels would not be helpful.
Ulceration of the skin, genitalia and mouth can
This is a bullous disorder, and as the blister is
occur in autoimmune blistering diseases such as
intact, pemphigoid is much more likely than
pemphigus vulgaris (PV). The patient is the right
pemphigus. Drugs can cause bullous eruptions.
age and ethnic group for PV that tends to
DIF is the investigation of choice, and the
present more commonly in the 3rd-6th decades
findings in pemphigoid are as described in E,
and in Jewish and Indian patients. A skin biopsy
but unaffected skin should be sampled as false
for routine histology is a useful investigation
negatives can be obtained if blistered skin is
but the gold standard is direct
sampled.
immunofluorescence (performed on a skin
biopsy), which demonstrates the intercellular
[ Q: 61 ] MasterClass Part2 IgG in the epidermis, which characterises PV. A
(2010) - Dermatology tzanck smear can be helpful but is rarely done
and not as informative as a biopsy.
A 40-year-old Indian male presents with mouth
ulcers, a sore penis and ulcers on the skin.
[ Q: 62 ] MasterClass Part2
Which would be the TWO most helpful (2010) - Dermatology
investigations in reaching a diagnosis?
A 50-year-old male presented with thickening,
A. Patch tests
discolouration and onycholysis of several
B. Direct immunofluorescence toenails. There was erythema and scaling in the
toewebs.
C. Pathergy test
D. Autoantibody screen What is the most likely cause of his nail
dystrophy?
E. Skin prick tests
A. Lichen planus
F. Skin biopsy
B. Psoriasis
G. Skin scrapings
C. Twenty nail dystrophy
H. Skin swab
D. Onychomycosis
I. Tzanck smear
E. Alopecia areata
J. IgE level.

Dr. Khalid Yusuf El-Zohry – Sohag Teaching Hospital (01118391123) Page | 723
El-Zohry MRCP Questions Bank (Part 2) – Medical Masterclass 2010

Answer & Comments diarrhoea. However, it does not have a


predilection for the buttocks.
Correct answer: D
Intestinal threadworms are associated with
All of the above can cause nail dystrophy. peri-anal itching due to migration of worms
onto the peri-anal skin.
Onycholysis describes distal separation of the
nail from the nail bed and is most commonly Cutaneous larva migrans is a characteristic itchy
seen in psoriasis and onychomycosis. It is not a eruption of creeping tortuous tracks produced
feature of the other diseases above. by hookworms as they migrate through the
Thickening and discolouration are also signs skin.
seen in both onychomycosis and psoriasis. The Hookworms are acquired through direct
presence of erythema and scaling between the contact with faeces-contaminated sand or soil.
toes could be psoriasis but is more likely to be The feet, hands and buttocks are commonly
tinea pedis (athlete's foot) and the organisms affected.
responsible may infect the nails to cause
Folliculitis is common on the buttocks and may
onychomycosis.
be itchy. The papules would be centred on hair
follicles. It does not tie in with the history of
[ Q: 63 ] MasterClass Part2 diarrhoea.
(2010) - Dermatology
A 30-year-old male complains of itchy buttocks. [ Q: 64 ] MasterClass Part2
There is a papular rash on examination. On (2010) - Dermatology
further questioning, he complains of diarrhoea
A 22-year-old woman presents with atopic
and weight loss.
dermatitis.
What is the most likely diagnosis?
Investigations in individuals with this condition
A. Folliculitis show:
B. Dermatitis herpetiformis A. most patients have elevated total IgE
levels.
C. Intestinal threadworms
B. IgE titres do not correlate with disease
D. Cutaneous larva migrans
activity.
E. Urticaria.
C. most have positive radioallergenosorbent
(RAST) tests to food allergens such as nut,
Answer & Comments
wheat or milk.
Correct answer: B D. few have positive RAST tests to house
dust mite, cat, dog, grass.
Dermatitis herpetiformis has a predilection for
E. skin prick tests show poor correlation
the extensor surfaces including the elbows,
with RAST tests to defined allergens.
knees and buttocks. It is characteristically very
itchy. It is associated with gluten-sensitive
enteropathy, which may explain this man’s Answer & Comments
diarrhoea. Correct answer: A
Urticaria is itchy and is occasionally associated
with intestinal parasites, which would link with

Dr. Khalid Yusuf El-Zohry – Sohag Teaching Hospital (01118391123) Page | 724
El-Zohry MRCP Questions Bank (Part 2) – Medical Masterclass 2010

Total IgE titre does show some correlation with Answer & Comments
disease activity and it can be a useful screening
Correct answer: E
tool in patients in whom a diagnosis of atopic
dermatitis is being considered. Positive
radioallergenosorbent (RAST) tests to foods Many systemic therapies have been used in
such as nut, wheat and milk are rare in patients individuals with severe atopic eczema. Tumour
with atopic dermatitis, but in Northern necrosis factor inhibitors (e.g. infliximab) can be
European affected adults there are positive helpful in patients with psoriasis, but there is no
RAST tests to: data on their use for treatment of eczema.

 house dust mite in 50-60%


[ Q: 66 ] MasterClass Part2
 cat dander in 50%
(2010) - Dermatology
 grass in 75%
A 70-year-old woman with eczema complains of
 birch pollen in 65%. ‘bruising'. Examination reveals purpura on the
There is generally a good correlation between forearms only.
skin prick test reactivity and RAST test to these
allergens. Which is the most likely diagnosis?

Prick tests involve the intradermal injection of A. Liver disease


allergen with assessment of the immediate B. Thrombocytopenia
urticarial reaction and are believed to reflect
C. Senile purpura
type I allergic reactions. In contrast patch tests
involve applying antigen to the skin for 48 hours D. Cutaneous vasculitis
and assessing the subsequent delayed E. Systemic amyloidosis
eczematous reaction. This is believed to more
closely reflect type IV delayed hypersensitivity
Answer & Comments
reactions and is used clinically to investigate
patients with suspected contact allergic Correct answer: C
dermatitis to environmental antigens such as
nickel. All can cause purpura. Localisation on the
forearms is typical of senile purpura and
[ Q: 65 ] MasterClass Part2 reflects solar damage to the connective tissue
(2010) - Dermatology such that the skin often appears atrophic.
Corticosteroid use may exacerbate senile
A 40-year-old man with severe atopic eczema purpura.
fails to respond to potent topical steroids.

Possible treatment options do not currently [ Q: 67 ] MasterClass Part2


include: (2010) - Dermatology
A. Azathioprine A 40-year-old female presents with a painful,
B. Cyclosporin rapidly enlarging ulcer on the shin at the site of
minor trauma. She has a history of Crohn’s
C. Ultraviolet light disease.
D. Mycophenolate mofetil
What is the MOST likely diagnosis?
E. Tumour necrosis factor inhibitors.
A. Varicose ulcer

Dr. Khalid Yusuf El-Zohry – Sohag Teaching Hospital (01118391123) Page | 725
El-Zohry MRCP Questions Bank (Part 2) – Medical Masterclass 2010

B. Arterial ulcer Answer & Comments


C. Infection with Mycobacterium ulcerans Correct answer: C
D. Pyoderma gangrenosum
Guttate psoriasis is a relatively common
E. Neuropathic ulcer
condition where small 'rain drop' - type plaques
of psoriasis appear rapidly following an acute
Answer & Comments streptococcal infection. The patient may
Correct answer: D already have chronic plaque psoriasis, or the
rash may appear with no previously known
history of psoriasis.
Pyoderma gangrenosum (PG) exhibits the
Koebner phenomenon i.e. it may begin at sites CPP classically affects the extensor surfaces and
of trauma, including surgical scars, so it is a by definition is chronic.
diagnosis to consider in surgical wounds which
AE and lupus can cause red and scaly rashes,
break down. PG ulcers are typically painful, with
which can flare when a patient is systemically
a blue/purple edge and may enlarge rapidly. PG
unwell. AE is however usually a chronic
most commonly occurs on the lower legs.
condition and clinically does not form small
Associated diseases include inflammatory plaques as seen in guttate psoriasis. Lupus is
bowel disease, rheumatoid arthritis and much less common and also will clinically not
haematological malignancies particularly have the small plaques of guttate psoriasis.
myeloid ones. Neuropathic ulcers typically Lichen planus can occur acutely, however the
occur at sites of trauma e.g. over the metatarsal classic presentation is of violaceous, itchy
heads. Varicose ulcers occur over the medial papules at the wrists
ankle in patients with venous hypertension.
Infection with Mycobacterium ulcerans causes
ulceration termed a Baruli ulcer. It is seen in [ Q: 69 ] MasterClass Part2
rural, tropical west Africa. (2010) - Dermatology
A 30-year-old man presents with a 3-month
[ Q: 68 ] MasterClass Part2 history of a sore mouth and a 2-week history of
(2010) - Dermatology ulcers on the upper chest and back.

A 12-year-old boy with no previous skin What is the most likely diagnosis?
problems presents to his doctor with a 3-day
A. Behçet’s disease
history of a red, scaly, widespread rash over his
trunk. He is otherwise well, other than a recent B. Crohn's disease
severe sore throat.
C. Pemphigus vulgaris
What is the most likely diagnosis? D. Pyoderma gangrenosum
A. Atopic eczema (AE) E. Bullous pemphigoid.
B. Chronic plaque psoriasis (CPP)
Answer & Comments
C. Guttate psoriasis
Correct answer: C
D. Lupus
E. Lichen planus
Oral ulceration can be a feature of both
Behçet’s and Crohn’s disease. Both have

Dr. Khalid Yusuf El-Zohry – Sohag Teaching Hospital (01118391123) Page | 726
El-Zohry MRCP Questions Bank (Part 2) – Medical Masterclass 2010

cutaneous manifestations but ulceration over A. Sezary syndrome


the upper torso would not be typical. Both are
B. Erythema Nodosum
associated with pyoderma gangrenosum, which
does present with cutaneous ulceration but the C. Nodular vasculitis
upper torso would be an unusual site. The legs D. Necrobiosis lipoidica
are most common. Pyoderma gangrenosum
E. Mycosis fungoides
very rarely produces oral ulceration.
F. Discoid lupus erythematosus
The clinical history is very typical of pemphigus
vulgaris. Oral ulceration is almost universal and G. Contact dermatitis
typically precedes cutaneous involvement by
H. Pre-tibial myxoedema
weeks to months. The upper torso is a site of
predilection. Patients can present at any age I. Insect bites
but most commonly in the 3rd-6th decades. J. Psoriasis.
Bullous pemphigoid is most often seen in the
elderly and oral ulceration is seen in a minority
Answer & Comments
of cases.
Correct answer: BI
[ Q: 70 ] MasterClass Part2
(2010) - Dermatology This is a typical case of erythema nodosum, but
insect bites can give a similar picture, as can
A 28-year-old woman presents with painful nodular vasculitis, which is much rarer.
lumps on her legs (see image).
Erythema nodosum can be caused by a variety
of infections (streptococci, tuberculosis,
yersinia, chlamydia, Epstein-Barr virus,
Trichophyton, coccidiomycosis), drugs
(sulphonamides, oral contraceptives), as well as
other conditions such as sarcoidosis, Crohn’s
disease, Ulcerative colitis, Behcet’s disease and
malignancy.

[ Q: 71 ] MasterClass Part2
(2010) - Dermatology
A 24-year-old keen hiker presents to his GP
with a rash on his leg and 'flu-like symptoms.
He reports that the rash started as a small
raised red area 2 days ago, which has now
spread. On examination he is apyrexial and has
mild lymphadenopathy in his right groin. On
examination of his right lower leg, he has a ring
of erythema surrounding a central area of
normal skin.

What is the most likely diagnosis?


What are the TWO most likely diagnoses? A. Weil's disease

Dr. Khalid Yusuf El-Zohry – Sohag Teaching Hospital (01118391123) Page | 727
El-Zohry MRCP Questions Bank (Part 2) – Medical Masterclass 2010

B. Lyme disease [ Q: 73 ] MasterClass Part2


C. Coeliac disease (2010) - Dermatology
D. Graves' disease A 75-year-old man presents with a widespread
E. Erythema nodosum blistering rash.

Which TWO of the following are true for bullous


Answer & Comments pemphigoid?
Correct answer: B A. Tense pruritic blisters are common
B. Oral lesions are common
The rash is erythema chronicum migrans, which
is a feature of early Lyme disease (Stage 1) in C. Desmosomes are an important
70% of cases. Regional lymphadenopathy may autoantibody target
also be a feature. The diagnosis is confirmed by D. Target lesions are common
serology. The treatment for early disease is 2-3
E. Pruritic urticaria-like plaques are common
weeks of doxycycline or amoxicillin.
F. Methotrexate is the treatment of choice
[ Q: 72 ] MasterClass Part2 G. Antinuclear antibody is usually of raised
(2010) - Dermatology titre
H. Serum complement is typically low
A previously well 18-year-old young man
presents with a 1-week history of multiple I. The disease rarely remits
erythematous scaly plaques over his trunk. This
J. Lesions usually scar.
eruption followed a severe throat infection that
was treated with penicillin.
Answer & Comments
What is the most likely diagnosis?
Correct answer: AE
A. Erythema multiforme
B. Erythema nodosum Initially erythematous pruritic urticaria-like
plaques develop, on which tense blisters may
C. Guttate psoriasis
arise. Oral lesions are uncommon, but can
D. Pustular psoriasis occur.
E. Lichen planus The disease can start at any age, but is most
common in the elderly. The lesions usually heal
Answer & Comments without scarring and in most patients the
disease remits after several years.
Correct answer: C
The principle autoantibody targets are collagen
type XVII (BP180) and BP230, which are
If an acute generalised rash is scaly, consider
components of the hemidesmosomes and are
guttate psoriasis, pityriasis rosea and secondary
involved in adhesion of the basal layer of the
syphilis. Guttate psoriasis is often triggered by
epidermis to the basement membrane.
infection.
The first line treatment is topical and/or
systemic corticosteroids (dependent on the
severity and co-morbidity of the patient).

Dr. Khalid Yusuf El-Zohry – Sohag Teaching Hospital (01118391123) Page | 728
El-Zohry MRCP Questions Bank (Part 2) – Medical Masterclass 2010

[ Q: 74 ] MasterClass Part2 E. Thyroid function tests.


(2010) - Dermatology
Answer & Comments
A 7-year-old black male presents with enlarged
cervical lymph nodes and scaly patches in the Correct answer: D
scalp.
B12 deficiency does not normally cause
What is the MOST likely diagnosis? pruritus.
A. Psoriasis Iron deficiency, investigated with the serum
B. Tinea capitis ferritin, is a cause of generalized pruritus.

C. Rubella Chronic renal impairment and cholestasis can


both cause itching. Hyperthyroidism and
D. Non-Hodgkins lymphoma
hypothyroidism are also systemic causes of itch.
E. Seborrhoeic dermatitis Other possible systemic causes of itch are
polycythaemia rubra vera and underlying
Answer & Comments malignancy.

Correct answer: B
[ Q: 76 ] MasterClass Part2
Tinea capitis is more commonly seen in black
(2010) - Dermatology
children than other races and there have been A patient presents with a rash and demands
inner city epidemics. Regional testing for allergies.
lymphadenopathy often occurs. Seborrhoeic
dermatitis and psoriasis can both produce scalp In which of the following scenarios would patch
scaling, but regional lymphadenopathy is not a tests be least useful?
feature. Cervical lymphadenopathy may occur
A. Facial eczema in a beautician
in rubella and lymphoma but a scaly scalp
would not be expected. B. Angioedema and urticaria after eating
seafood

[ Q: 75 ] MasterClass Part2 C. A rash after taking penicillin


(2010) - Dermatology D. Hand eczema in a nurse

A 70-year-old man presents to a Dermatology E. A facial rash after dying the hair black
clinic with generalized itching for 6 months. On F. Hand eczema in a hairdresser
examination no rash is seen. Several blood tests
are taken to exclude a systemic cause for his G. Facial eczema in an atopic patient
pruritus. H. A rash around a chronic leg ulcer

Which of the following is NOT likely to be I. Earlobe eczema


helpful? J. Hand eczema in a cabinetmaker.
A. Urea and electrolytes
Answer & Comments
B. Liver function tests
C. Ferritin Correct answer: BC

D. B12

Dr. Khalid Yusuf El-Zohry – Sohag Teaching Hospital (01118391123) Page | 729
El-Zohry MRCP Questions Bank (Part 2) – Medical Masterclass 2010

Patch tests detect type IV (delayed) Answer & Comments


hypersensitivity reactions. Type IV
Correct answer: A
hypersensitivity presents as eczema and is
termed allergic contact dermatitis (ACD) (or
eczema). Any patient who presents with The Koebner phenomenon is the localization of
localised eczema that does not conform to the cutaneous disease to sites of trauma and is
typical distribution of an endogenous form, e.g. shown by several disorders including psoriasis,
atopic or seborrhoeic eczema, could have ACD lichen planus, viral warts and vitiligo.
and should be patch tested. Nail involvement in psoriasis is common and is
ACD may be occupational. Allergens in ACD are characterized by the presence of “thimble
encountered on the skin (so not relevant in pitting”, onycholysis (separation of the nail
food allergy). The beautician could be allergic to from the nail bed) and subungual
her make-up, the nurse to rubber gloves, the hyperkeratosis. Chronic plaque psoriasis
cabinetmaker to colophony in pine, varnishes localizing to the extensor surfaces is by far the
and glues and the hairdresser to one of commonest form.
numerous chemicals used in hairdressing Widespread pustules within lesions or on
products. One of the commonest hairdressing erythematous skin should raise the possibility
allergens is PPD, present in many hair dyes, of generalized pustular psoriasis which is a rare
particularly dark colours. Patients with leg but serious complication with a significant
ulcers are at risk of developing allergy to mortality.
chemicals in medicaments and bandages used
Although pruritus can be feature of psoriasis, it
in treatment. Nickel is a common allergen,
is rarely intense.
present in jewellery e.g. earrings.
Patch tests are not usually helpful in
[ Q: 78 ] MasterClass Part2
investigating angioedema/urticaria, which is
due to type I allergy, nor in suspected drug (2010) - Dermatology
eruptions. A 65-year-old woman develops oral ulceration
and a widespread painful exfoliative rash three
[ Q: 77 ] MasterClass Part2 days after starting penicillin.
(2010) - Dermatology
Your differential would NOT include:
A 58-year-old man presents with a scaly rash.
A. Toxic epidermal necrolysis
You consider the diagnosis of psoriasis.
B. Toxic shock syndrome
Which one of the following statements is true of
C. Pemphigus
this condition?
D. Pemphigoid
A. Psoriasis shows the Koebner
phenomenon. E. Stevens-Johnson syndrome

B. Nail involvement in psoriasis is rare.


Answer & Comments
C. Guttate psoriasis is the commonest form.
Correct answer: D
D. Sterile pustules are frequently seen
within lesions.
Some believe that Stevens-Johnson syndrome
E. Intense pruritus is a typical symptom. can evolve into toxic epidermal necrolysis but
others believe that they are separate entities.

Dr. Khalid Yusuf El-Zohry – Sohag Teaching Hospital (01118391123) Page | 730
El-Zohry MRCP Questions Bank (Part 2) – Medical Masterclass 2010

Bullous pemphigoid typically presents with [ Q: 80 ] MasterClass Part2


tense blisters, itching and minimal mucosal (2010) - Dermatology
involvement.
A 76-year-old woman is referred by her GP with
a worsening ulcer on the left medial lower leg.
[ Q: 79 ] MasterClass Part2
On examination she has changes of venous
(2010) - Dermatology insufficiency and oedema, and a shallow
A 30-year-old man presents with intermittent sloughy ulcer overlying the medial malleolus.
episodes of lip swelling, at random times. There Her arterial pulses are all palpable.
is no associated wheeze, swallowing problems
What is the most appropriate therapeutic
or abdominal pain. He is well other than a
intervention?
history of migraine.
A. A high-protein diet
What is the most likely diagnosis?
B. Systemic immunosuppression
A. Hereditary angioedema
C. Intravenous antibiotics
B. Food allergy
D. Antiseptic dressings
C. Chronic idiopathic angioedema
E. Elevation and compression bandaging
D. Angiotensin-converting enzyme (ACE)
inhibitor-induced angioedema
Answer & Comments
E. Hypothyroidism.
Correct answer: E

Answer & Comments


This is the mainstay of management for venous
Correct answer: C ulceration. Venous ulcers will never heal if the
leg remains oedematous and dependent.
Chronic idiopathic angioedema is a common
condition. It is often exacerbated by analgesics [ Q: 81 ] MasterClass Part2
such as aspirin, codeine and non-steroidal anti- (2010) - Dermatology
inflammatory drugs (NSAIDs), which act as
general mast cell destabilisers. A 50-year-old man presents with erythroderma.

Hereditary angioedema is less common and Which of the following is least likely to be the
caused by C1 esterase inhibitor deficiency. underlying cause?
Attacks are often associated with abdominal
pain (due to bowel oedema). A. Eczema

Type 1 food allergy can present with lip swelling B. Psoriasis


but the timing is not random, occurring within C. Drug eruption
30 minutes of contact with the food.
D. Sezary syndrome
ACE-inhibitors can trigger angioedema.
E. Lichen planus.
Hypothyroidism is associated with angioedema,
however our patient was well.
Answer & Comments

Correct answer: E

Dr. Khalid Yusuf El-Zohry – Sohag Teaching Hospital (01118391123) Page | 731
El-Zohry MRCP Questions Bank (Part 2) – Medical Masterclass 2010

Answers A-D are all well recognized causes of A. He should avoid fresh fruits as these
erythroderma. Lichen planus has many contain cross-reactive antigens
different clinical presentations. It classically
B. Coexistant asthma increases the risk of
causes itchy, violacious papules at the wrists
more significant reactions
and very rarely causes erythroderma.
C. His reactions will become more severe
with time
[ Q: 82 ] MasterClass Part2
(2010) - Dermatology D. He can be advised that other nuts are
fine to eat
A 15-year-old boy presents with white patches
E. He should avoid NSAIDs
classical of vitiligo affecting his hands, eyelids
and feet.
Answer & Comments
Which of the following statements from his GP
Correct answer: B
are correct?
A. There are no disease associations The diagnosis would be consistent with peanut
B. Vitiligo is not helped by topical steroids allergy. The oral allergy syndrome is a
sensitivity to particular fresh fruits (especially
C. Vitiligo is unlikely to progress
tree fruits eg apples, peaches, pears) which can
D. Vitiligo will not spontaneously improve be linked to nut allergy. However if he does not
E. Vitiligo patches are at increased risk of describe reactions to these fruits then most
sunburn would not advise him to avoid them.
Coexistant asthma has been linked to death
Answer & Comments from anaphylaxis and most would recognise
this as one of the indications to prescribe self-
Correct answer: E injectable adrenaline for use in emergencies. It
is difficult to predict with much certainty how
Vitiligo is a common autoimmune condition, the severity of the reactions will change over
which is associated with other autoimmune time. Cross-reactivities with other nuts are well
problems such as thyroid disease and described, and there are frequent multi-nut
pernicious anaemia. Potent topical steroids can contaminants in products containing nuts.
sometimes induce repigmentation. The Therefore most would advise avoiding all nuts.
prognosis of vitiligo is difficult to predict - some
areas may improve and new areas may appear.
[ Q: 84 ] MasterClass Part2
Patients should be warned about the increased (2010) - Dermatology
risk of sunburn within vitiligo patches.
A 37-year-old woman with hypothyroidism
presents with white patches of skin on her
[ Q: 83 ] MasterClass Part2
elbows, knees and eyelids.
(2010) - Dermatology
What is the most likely diagnosis?
A 24-year-old man develops urticaria on eating
peanuts, and has a positive skin prick test to A. Pityriasis versicolor
peanuts. B. Post-inflammatory hypopigmentation

Which one of the following is most likely: C. Psoriasis

Dr. Khalid Yusuf El-Zohry – Sohag Teaching Hospital (01118391123) Page | 732
El-Zohry MRCP Questions Bank (Part 2) – Medical Masterclass 2010

D. Vitiligo firm, thickening of the skin of the lower legs


that can occur in patients with venous
E. Leprosy
hypertension. Necrobiosis lipoidica is usually
asymptomatic and typically occurs on the shins
Answer & Comments of younger patients. Lymphedema is not red
Correct answer: D and itchy unless complicated by eczema.

Vitiligo is typically symmetrical and causes


complete pigment loss so that the skin is white
(depigmented) rather than pale
(hypopigmented). Hypopigmentation is seen in
post-inflammatory hypopigmentation, pityriasis
versicolor and leprosy. The latter two disorders
are also scaly but the skin in vitiligo is smooth.
The symmetrical distribution in this case is
typical of vitiligo. Post-inflammatory
hypopigmentation following psoriasis could
occur on the elbows and knees but would be
unusual on the eyelids whereas vitiligo is
commonly seen in this area.

[ Q: 85 ] MasterClass Part2
(2010) - Dermatology
A 70-year-old woman complains of swollen,
red, itchy legs.

Which is the most likely diagnosis?


A. Lymphedema
B. Lipodermatosclerosis
C. Varicose eczema
D. Cellulitis
E. Necrobiosis lipoidica

Answer & Comments

Correct answer: C

Varicose eczema is a common problem,


particularly in elderly patients with peripheral
edema, which tends to exacerbate the problem.
It is often mistaken for cellulitis, but cellulitis is
rarely bilateral and is painful rather than itchy.
Lipodermatosclerosis describes the woody,

Dr. Khalid Yusuf El-Zohry – Sohag Teaching Hospital (01118391123) Page | 733
El-Zohry MRCP Questions Bank (Part 2) – Medical Masterclass 2010

Polyps increase the risk of colorectal cancer,

Oncology which usually develops by age 40.


Duodenal adenomas are found in almost all FAP
patients and are pre-malignant.

(80 Questions) There is an increased risk of extraintestinal


tumours including brain tumours and papillary
(Medical Masterclass – Part 2) carcinoma of the thyroid. Extraintestinal
features of FAP include multiple subcutaneous
cysts and osteomas (Gardner Syndrome).

[ Q: 1 ] MasterClass Part2 [ Q: 2 ] MasterClass Part2


(2010) - Oncology (2010) - Oncology
A 52-year-old man is admitted with a history of A 28-year-old woman presents with an
bleeding per rectum, tenesmus and weight loss. enlarging mole on her calf. Excision biopsy
His father died of colorectal carcinoma aged 43 shows a superficial spreading melanoma of
years and two of his siblings died of colorectal 0.5mm thickness.
carcinoma aged 36 and 37 years of age. An
older sister had died aged 64 from thyroid Subsequent treatment should be:
carcinoma. Barium enema showed an apple A. Wide excision and adjuvant
core lesion in his sigmoid colon in keeping with immunotherapy
carcinoma, also multiple polyps throughout the
colon and rectum. B. Wide excision with 1 cm margin
C. Wide excision and sentinel node biopsy
Which of the following statements is correct?
D. Wide excision and elective inguinal node
A. A diagnosis of familial adenomatous dissection
polyposis (FAP) is unlikely.
E. Wide excision with 5 cm margin.
B. FAP is a poorly penetrant autosomal
recessive condition .
Answer & Comments
C. There is an increased incidence of
papillary carcinoma of the thyroid in FAP. Correct answer: B

D. Duodeal adenomas do not develop in


This is a low risk melanoma. Excision margins
FAP.
greater than 1-2 cm have no advantage and the
E. There are no extraintestinal features of role of adjuvant immunotherapy is debatable
FAP. even in high risk patients, although it may
prolong disease-free survival. Sentinel node
Answer & Comments biopsy is in clinical trial for thicker melanomas.

Correct answer: C
[ Q: 3 ] MasterClass Part2
FAP is a highly penetrant autosomal dominant (2010) - Oncology
condition in which multiple adenomatous A 76-year-old woman with metastatic breast
polyps develop throughout the colon and cancer developed nausea and vomiting. She
rectum during adolescence. was known to have bone and liver metastases

Dr. Khalid Yusuf El-Zohry – Sohag Teaching Hospital (01118391123) Page | 735
El-Zohry MRCP Questions Bank (Part 2) – Medical Masterclass 2010

and her pain was well controlled on a stable [ Q: 4 ] MasterClass Part2


dose of slow release morphine sulphate for the (2010) - Oncology
last 6 months. She was also on diuretics and
digoxin for chronic heart failure and atrial A 58-year-old man with extensive stage small
fibrillation and her GP had prescribed cell lung cancer is enrolled onto a phase I trial
antibiotics 3 days previously for a respiratory of carboplatin, etoposide and topotecan
tract infection. chemotherapy.

Which of the following is least likely to be What is the principal aim of a phase I study?
contributing to her nausea & vomiting? A. Establish the maximum tolerated dose
A. Hypercalcaemia B. Determine drug interactions
B. Uraemia C. Compare results with gold standard
C. Opiates treatment
D. Erythromycin D. Normal volunteer study for drug toxicity
E. Squashed stomach secondary to liver E. Determine tumours types that respond to
metastases. the therapy.

Answer & Comments Answer & Comments

Correct answer: C Correct answer: A

Side effects of opiates: Experimental therapies:


 Constipation: This affects almost all  Phase I studies determine the
patients and all patients require relationship between toxicity and dose
prophylaxis with a stimulant laxative schedules of treatment
(e.g. senna, bisacodyl) and a softener  Phase II studies identify tumour types
(e.g. docusate sodium) or as a for which the treatment appears
combined preparation (e.g. promising
codanthramer, codanthrusate)
 Phase III studies assess the efficacy of
 Drowsiness: Generally remits after a
few days treatment compared to standard
 Nausea: Affects one third of opioid- treatment, including toxicity
naïve patients but usually resolves
within one week. Consider prophylaxis [ Q: 5 ] MasterClass Part2
for 1 week (2010) - Oncology
 Hallucinations: An uncommon side
effect that often features images in the A 68-year-old lady attended her GP with a rash
peripheral vision around her eyes and over her knuckles and
 Nightmares: Vivid and unpleasant but chest. She also complains of a generalized body
rare ache. On direct questioning she has recently
 Myoclonic jerks: Occur usually with noticed painful upper arms and difficulty in
excess doses and may be mistaken for brushing her hair.
fits
 Respiratory depression: Not a problem Which of the following statements are untrue?
in patients with pain. A. A history of smoking may be relevant.

Dr. Khalid Yusuf El-Zohry – Sohag Teaching Hospital (01118391123) Page | 736
El-Zohry MRCP Questions Bank (Part 2) – Medical Masterclass 2010

B. Examination of her breasts may be inconsistent course, independent of the


prudent. tumour.
C. Serum muscle enzymes are usually Eaton-Lambert myasthenic syndrome is a
elevated. disorder of the myoneural junction that results
in a proximal muscle weakness. Nerve
D. It may precede a diagnosis of lung cancer.
conduction studies show characteristic normal
E. A myopathic process is seen on conduction velocities and initially low
electromyography. amplitude compound muscle action potentials.
F. Muscle biopsy shows inflammatory Repetitive nerve conduction studies show a
degeneration of muscle. decrease of compound muscle action potentials
at low stimulation rates and an increase at high
G. The course of skin and muscle symptoms
stimulation rates.
and signs follow the underlying disease.
H. Treatment with immunosuppressants is [ Q: 6 ] MasterClass Part2
standard.
(2010) - Oncology
I. Repetitive nerve conduction studies show
a decrease of compound muscle action A 20-year-old man had a high grade B-cell Non-
potentials at low stimulation rates and an Hodgkin’s lymphoma for which he received six
increase at high stimulation rates. cycles of CHOP chemotherapy and went into
complete remission. At a routine follow-up visit
J. Muscle tenderness is a usual feature. 2 years later he reports that he feels perfectly
well, but has developed pain in his right hip that
Answer & Comments prevents him from playing football.

Correct answer: GI What is the most likely cause of his hip pain
(select one diagnosis), and what is the most
Dermatomyositis and polymyositis are sensitive and specific investigation for this
inflammatory myopathies. Subacute proximal (select one investigation)?
muscle weakness may occur with or without
A. Avascular necrosis of the right femoral
muscle tenderness. In dermatomyositis skin
head
changes are seen with the muscle weakness.
The skin signs are erythema or telangiectasias B. Isotope bone scan
of the knuckles, chest and periorbital region. C. Rheumatoid arthritis of the right hip
Both conditions are idiopathic. Malignant D. Lymphomatous deposit in the hip / pelvis
disease is reported in up to 50% of cases; it may
precede the diagnosis of carcinoma by days or E. Ultrasonography
years. F. Plain radiography
The most commonly associated cancers are G. Osteoarthritis of the right hip
breast and lung tumours.
H. Magnetic Resonance Imaging
Muscle enzymes are usually elevated.
I. CT scan of pelvis / right hip
Electromyography (EMG) is consistent with a
myopathic process, and muscle biopsy shows J. Osteomyelitis in the hip / pelvis
inflammatory degeneration of muscle.
Immunosuppressive agents, including steroids,
are standard. The syndrome often follows an

Dr. Khalid Yusuf El-Zohry – Sohag Teaching Hospital (01118391123) Page | 737
El-Zohry MRCP Questions Bank (Part 2) – Medical Masterclass 2010

Answer & Comments E. A raised alkaline phosphatase would be


helpful in the differential diagnosis.
Correct answer: AH

Answer & Comments


This presentation would be typical of avascular
necrosis of the femoral head as a complication Correct answer: C
of steroid treatment.
Plain radiology may reveal sclerosis of the Radiation-induced sarcomas are very rare. They
femoral head, also subchondral lucency and occur from 7 years onwards after radiotherapy.
flattening, but these features may not be seen
in early cases. Isotope bone scanning can be [ Q: 8 ] MasterClass Part2
used to make the diagnosis of avascular (2010) - Oncology
necrosis, but the appearance of increased
uptake is not specific for this condition. the A 59-year-old man has lung and bone
most sensitive and specific test for avascular metastases from renal adenocarcinoma.
necrosis is MRI, which initially shows medullary
cavity oedema and bone ischaemia. Which of the following paraneoplastic features
are not characteristic of renal cancer?
Other late complications of treatment for
lymphoma include: hypothyroidism (following A. SIADH
mantle irradiation), infertility / hypogonadism B. Pyrexia
(following some chemotherapeutic regimen),
C. Hypercalcaemia
peripheral neuropathy (vincristine), and
secondary acute leukaemia (usually AML). D. Anaemia
E. Polycythaemia
[ Q: 7 ] MasterClass Part2 F. Lambert-Eaton syndrome
(2010) - Oncology
G. Hepatic dysfunction
A 58-year-old woman presents with left chest
H. Polymyositis
wall pain after a coughing fit. Eight years ago
she had a mastectomy, adjuvant chemotherapy I. Hypertension
and endocrine therapy and chest wall J. Thrombocytosis
radiotherapy of a T2N1 cancer of the left
breast. Her bone scan now shows a solitary hot
Answer & Comments
spot in the anterior left 5th rib.
Correct answer: AF
Which one of the following statements is
INCORRECT?
SIADH and Lambert-Eaton syndrome are not
A. A follow-up bone scan in 6 months is characteristic paraneoplastic features of renal
indicated. cancer.
B. The hot spot is likely to be a fracture from
radiation induced osteoporosis. [ Q: 9 ] MasterClass Part2
C. The hot spot is likely to be a radiation (2010) - Oncology
induced sarcoma. A 35-year-old woman presents with a 1 cm
D. The hot spot is likely to be a fracture diameter lump in her left breast, which needle
from metastatic disease. biopsy reveals to be invasive ductal carcinoma.

Dr. Khalid Yusuf El-Zohry – Sohag Teaching Hospital (01118391123) Page | 738
El-Zohry MRCP Questions Bank (Part 2) – Medical Masterclass 2010

What should her initial treatment be? [ Q: 10 ] MasterClass Part2


A. Wide local excision and axillary node (2010) - Oncology
sampling followed by adjuvant breast
A 4-year-old boy presents with a 3 week history
radiotherapy in all cases
of headache, ataxia and vomiting. A head CT
B. Wide local excision and axillary node shows a posterior fossa mass that arises from
sampling followed by adjuvant breast the midline cerebellum involving the fourth
radiotherapy if there is histological ventricle.
spread to the axillary nodes
The most likely pathology is:
C. Mastectomy and axillary node sampling
followed by adjuvant breast radiotherapy A. haemangioblastoma
in all cases B. oligodendrocytoma
D. Mastectomy and axillary node sampling C. medulloblastoma
followed by adjuvant chemotherapy if
there is histological spread to the axillary D. craniopharyngioma
nodes E. glioblastoma multiforme.
E. Wide local excision and axillary node
sampling followed by adjuvant breast
radiotherapy and adjuvant chemotherapy Answer & Comments
if there is histological spread to the
axillary nodes. Correct answer: C

Answer & Comments Oligodendrocytomas arise from


oligodendrocytes in the white matter of the
Correct answer: A cerebral hemispheres and occurs in young and
middle aged adults.
The standard treatment of early breast cancer
Craniopharyngiomas arise in a suprasellar
is wide local excision and axillary node sampling
location.
followed by adjuvant breast radiotherapy. This
achieves similar local control and survival rates Glioblastoma multiforme, the highest grade of
to mastectomy with less mutilating surgery. glioma, occur in the cerebral hemispheres of
older adults.
Women without axillary node involvement can
be divided into a high risk group (tumour >1 cm haemangioblastoma is a cyctic tumour
or does not express the Estrogen Receptor, ER), associated with adults with von Hippel-Lindau.
who should be given adjuvant treatment, and a Medulloblastomas arise from the cerebellar
low risk group (smaller ER-positive) who do not vermis in children and less commonly in the
require adjuvant treatment. cerebelar hemispheres of adults.
Women with histological spread to axillary
nodes should receive adjuvant chemotherapy, [ Q: 11 ] MasterClass Part2
which delays recurrence and improves survival.
(2010) - Oncology
A previously independent 75-year-old man has
been referred to you with a two-month history
of backache and falls over the past 2 days.
Clinical examination reveals that he has

Dr. Khalid Yusuf El-Zohry – Sohag Teaching Hospital (01118391123) Page | 739
El-Zohry MRCP Questions Bank (Part 2) – Medical Masterclass 2010

bilateral weak legs, a palpable bladder and a is ineffective or not possible or if there is
sensory deficit up to the level of the umbilicus. skeletal instability.
The sensory level at the nipples is at T4 and that
Which statements are true?
at the umbilicus is T10. In a cauda equina lesion
A. There is a spinal cord compression at the the weakness may be asymmetrical and mild;
level of T 12. sphincter control is often spared. The earliest
B. Sphincter control is lost in compression of symptom of cord compression is vertebral pain,
the cauda equina. especially on coughing or lying flat. Signs
include sensory changes one or two
C. Back pain is often worse when lying flat.
dermatomes below the level of compression,
D. The investigation with the highest progressing to motor weakness distal to the
diagnostic specificity is a bone scan. block and finally sphincter disturbance. Residual
neurological deficit usually reflects the extent
E. Neurosurgical intervention is rarely
of the deficit at the start of treatment.
beneficial.
F. Age is an adverse prognostic factor for
[ Q: 12 ] MasterClass Part2
neurological recovery.
(2010) - Oncology
G. He is likely to have a cauda equina lesion.
A 45-year-old woman has recently been
H. Loss of sphincter control is an adverse
diagnosed with malignant mesothelioma.
prognostic factor.
I. Autonomic dysfunction is a common Which of the following are recognised as poor
feature. prognostic factors in mesothelioma?

J. Urgent treatment with radiotherapy A. Poor performance status


confers a survival advantage. B. Female gender
C. Non-epithelioid histology
Answer & Comments
D. Polycythaemia
Correct answer: CH
E. Raised serum creatinine

Up to 5% of patients develop spinal cord F. Jaundice


compression, and around 30% will survive for 1 G. Raised blood urea
year. Neoplastic cord compression is nearly
always due to extramedullary, extradural H. History of asbestos exposure
metastases, most commonly from prostate I. History of cigarette smoking
cancer or multiple myeloma; also from lung,
J. Chronic obstructive pulmonary disease
breast, lymphoma or renal cancers.
To prevent paraplegia the diagnosis of spinal Answer & Comments
cord compression must be made swiftly and
treatment be instituted quickly. Following Correct answer: AC
suspicion of the diagnosis, high-dose steroid
therapy should be instituted, an urgent For malignant mesothelioma, the prognostic
magnetic resonance imaging (MRI) scan should scoring systems of the Cancer and Leukaemia
be undertaken and urgent radiotherapy given. Group B (CALGB) and European Organization
Surgical decompression is useful if radiotherapy for Research and Treatment of Cancer (EORTC)

Dr. Khalid Yusuf El-Zohry – Sohag Teaching Hospital (01118391123) Page | 740
El-Zohry MRCP Questions Bank (Part 2) – Medical Masterclass 2010

are the most useful. Poor prognosis is Aside from malignancy, other common causes
predicted by non-epithelioid histology, male of hypercalcaemia are hyperparathyroidism and
gender, advanced stage disease, poor myeloma. Less common causes are:
performance status, leucocytosis and  Vitamin D toxicity (consumption of
thrombocytosis.
medicines / compounds containing
vitamin D)
[ Q: 13 ] MasterClass Part2  Vitamin D ‘sensitivity’ (granulomatous
(2010) - Oncology disorders, e.g. sarcoidosis)
A 68-year-old man, who has smoked heavily for  Excess calcium intake (milk-alkali
many years, presents with haemoptysis and syndrome)
confusion. His chest radiograph shows what
 Reduced calcium excretion (thiazide
appears to be a lung tumour in the right upper
diuretics, familial hypocalciuric
lobe. Blood tests reveal urea 12.6 mmol/l,
hypercalcaemia)
creatinine 186 micromol/l, calcium 3.42
mmol/l.  Endocrine (thyrotoxicosis, adrenal
failure, phaeochromocytoma)
Which TWO of the following investigations or
 Acute renal failure (recovery phase)
treatments would you initiate first?
 Long-term immobility.
A. CT scan chest
B. Radiotherapy for lung cancer
[ Q: 14 ] MasterClass Part2
C. Admit to Hospice (2010) - Oncology
D. Bronchoscopy
A 41-year-old male presented with a node in his
E. Chemotherapy for lung cancer left lower neck region. A biopsy of the node
F. Intravenous 0.9% saline revealed Hodgkin's lymphoma, nodular
sclerosis type. Subsequently a mediastinal mass
G. Dexamethasone 4 mg po 4times daily was found. Prior to Gallium scintigraphy he was
H. CT scan chest with percutaneous needle initially staged as IIA; however, this revealed
biopsy addition gallium avid tumours in the left para
aortic region, and a small focus in the splenic
I. Lung ventilation perfusion scan
hilar region.
J. Intravenous disodium pamidronate.
67Gallium (67-Ga) scanning has been used in
malignant lymphoma treatment for all but
Answer & Comments
which of the following:
Correct answer: FJ
A. Staging at diagnosis
B. Detection of relapse
The first priority is to treat this man’s
hypercalcaemia, for which intravenous saline C. Screening at risk populations
and disodium pamidronate are the appropriate.
D. Evaluation of response to therapy
The diagnosis of lung cancer seems virtually
E. Predicting prognosis.
inevitable with this presentation, but it will be
necessary to establish a histological diagnosis
and stage the tumour to plan treatment.

Dr. Khalid Yusuf El-Zohry – Sohag Teaching Hospital (01118391123) Page | 741
El-Zohry MRCP Questions Bank (Part 2) – Medical Masterclass 2010

Answer & Comments Pancoast's tumours are typically non-small cell


lung cancers occurring in the lung apex. These
Correct answer: C
can infiltrate the upper thoracic ribs, brachial
plexus and cervical sympathetic chain causing
67-Ga scintigraphy has been used for more than Horner's syndrome. Early tumours may be
20 years in the detection of malignancies. Its suitable for surgery, but most are treated with
exact role in malignant lymphoma has been radiotherapy and chemotherapy.
disputed, mainly because of the high false-
negative rate. The role of 67-Ga scintigraphy in
lymphoma relies on its ability to detect viable
[ Q: 16 ] MasterClass Part2
residual disease with or without residual (2010) - Oncology
radiographic abnormalities and conversely to
identify patients with no viable disease whose A patient with lymphoma excretes 1.5g urinary
radiographic studies are still abnormal. This protein but has a negative dipstick test. The
depends on high 67-Ga avidity in untreated reason for this seeming inconsistency is:
tumours, which vary with histological subtype A. dipsticks preferentially detect albumin
and anatomic distribution of the disease. rather than immunoglobulin because
albumin is negatively charged
[ Q: 15 ] MasterClass Part2 B. the molecular weight of the excreted
(2010) - Oncology protein is too low to be detected by
dipsticks
A 60-year-old man presents with a Pancoast’s
tumour. C. Tamm-Horsfall proteins block the
reaction of the secreted protein and the
Which TWO of the following are characteristic dipstick
features? D E F
D. the urine is not sufficiently concentrated
A. Peripheral neuropathy
E. dipsticks only recognise heavy chain
B. Small cell histology sequences.
C. Liver metastases
D. Mediastinal lymphadenopathy
Answer & Comments
E. Destruction of first rib
Correct answer: A
F. Occupational asbestos exposure
G. Inappropriate ADH secretion
A normal person excretes up to 150 mg of
H. Clubbing protein per day which is chiefly Tamm-Horsfall
mucoprotein. Dipsticks can detect as little as 50
I. Operability
mg protein per litre. However, a false negative
J. Miosis result occurs with immunoglobulins which are
positively charged.

Answer & Comments

Correct answer: EJ

Dr. Khalid Yusuf El-Zohry – Sohag Teaching Hospital (01118391123) Page | 742
El-Zohry MRCP Questions Bank (Part 2) – Medical Masterclass 2010

[ Q: 17 ] MasterClass Part2 history includes acute inferior myocardial


(2010) - Oncology infarction in 1993 and transurethral resection
of the prostate in 1996 for bladder outflow
A patient presents to her GP with a pigmented obstruction. His current medication is aspirin,
lesion on her foot. The patient states that the nifedepine and a salbutamol inhaler. He is a
lesion was apparently present from birth and lifelong heavy smoker and his GP has diagnosed
does not itch or bleed. However, it is not as chronic obstructive airways disease.
homogeneous as it used to be.
On examination he is a well-built man with a
Which of the following is true? prominent forehead and jaw. He has a hearing
aid on his left ear and is apyrexial. His pulse rate
A. Early diagnosis would not affect is 80 beats/minute in sinus rhythm and his BP is
prognosis 110/60 mmHg. JVP is raised at 4 cm above the
B. It would be dangerous to perform an angle of Louis and the apex beat is displaced to
incisional biopsy of this lesion the mid-axillary line. Heart sounds are normal
and there are no murmurs. There is mild
C. Bleeding and tenderness would be the
bilateral ankle oedema and basal crepitations.
first signs of malignant change
Abdominal examination is unremarkable apart
D. Since the lesion has been present since from an enlarged irregular prostate gland. His
birth, the risk of malignancy is low right mid-humerus is swollen and tender. Initial
E. Change in colour is suspicious for tests show the following: FBC, urea and
malignant change electrolytes, calcium, albumin and phosphate
are normal; alkaline phosphatase and acid
phosphatase are increased; bone scan is
diffusely hot but cold over right mid-humerus;
Answer & Comments and a radiograph of his right humerus shows
lytic lesion with cortical bone destruction, but
Correct answer: E
no periosteal reaction.

The characteristics that distinguish a superficial Which of these investigations is most likely to
spreading malignant melanoma from a normal establish the diagnosis?
mole include irregularity of its border, and
A. CT scan of his arm
variegation or colour. Loss of homogeneous
coloration and disorderliness are suspicious. B. Whole-body PET scan
The first changes noted by patients developing C. Transurethral biopsy of his prostate gland
a melanoma are a darkening in its colour or a
change in the borders of the lesion. Irregularity D. Serum prostate-specific antigen
of the border in an expanding darkening mole is E. Bone biopsy of arm lesion
melanoma until proved otherwise. Biopsy
should be done promptly as early diagnosis and Answer & Comments
excision reduce the mortality rate.
Correct answer: E
[ Q: 18 ] MasterClass Part2
The differential diagnosis is Paget's disease with
(2010) - Oncology
osteosarcoma and metastatic prostate cancer.
A 75-year-old retired miner presents with a 3- Although transrectal ultrasound-guided sextant
month history of a painful right upper arm with biopsy and serum prostate-specific antigen may
some swelling in the mid arm. His past medical indicate prostate cancer, this is sufficiently

Dr. Khalid Yusuf El-Zohry – Sohag Teaching Hospital (01118391123) Page | 743
El-Zohry MRCP Questions Bank (Part 2) – Medical Masterclass 2010

common that it does not exclude the presence J. Interferon-alpha is more useful if the
of both diagnoses. Hence a biopsy is more likely primary tumour is unresected.
to establish the diagnosis of the arm lesion
definitively. Overall about 1% of patients with Answer & Comments
Paget's disease will develop a primary bone
sarcoma: 85% are osteosarcoma and 15% Correct answer: CD
fibrosarcoma. The most common sites are
pelvis, femur and humerus. The radiological Renal tumours account for ~ 3% of adult
appearances are of osteolytic lesions with malignancies and occur most commonly in
cortical destruction, but periosteal elevation is people aged 50-70 years old. They arise in both
rare. Patients present with localised pain and a sporadic and a hereditary form. There are
most are dead within 2 years. three forms of hereditary renal cancer, von
Hippel-Lindau (VHL) disease, hereditary
[ Q: 19 ] MasterClass Part2 papillary renal cancer (HPRC) and hereditary
renal carcinoma (HRC). There is an increased
(2010) - Oncology
incidence of renal cancer with VHL disease,
A 50-year-old man presents with a history of which is inherited in an autosomal dominant
painless haematuria, fever, cachexia and he has manner. The patients with VHL disease develop
a palpable loin mass. Biopsy shows a renal clear cell renal tumours, phaeochromocytoma
carcinoma. is reported to occur in 18% patients with VHL
disease. An increased incidence of renal
Which of these statements are correct? carcinoma has also been observed in patients
A. Taking a family history is unimportant. with autosomal dominant polycystic kidney
disease and tuberose sclerosis.
B. Cigarette smoking is not a strong risk
factor. Risks factors for the development of renal
carcinoma are: cigarette smoking, obesity,
C. In the presence of von Hippel-Lindau analgesic abuse, in particular those containing
disease, clear cell tumours of the kidney phenacetin. There is an increased incidence in
are the more likely pathological type. leather tanners, shoe workers and those
D. In the presence of von Hippel-Lindau working with asbestos; there may also be an
disease, phaeochromocytoma may also increased risk in those working with petroleum.
be present. Humoral hypercalcaemia of malignancy is often
E. Radiotherapy to the renal bed is required observed in patients with renal carcinoma. It is
following nephrectomy. thought to be caused by a factor with
parathyroid-hormone (PTH)-like activity.
F. Ultrasound of the abdomen is poor for
Patients with metastatic renal cancer who
staging the tumour.
present with humoral hypercalcaemia of
G. Presentation with humoral malignancy have a poor prognosis. Patients
hypercalcaemia carries a better prognosis present with pain, haematuria and flank
for the patient. massonly occurs in ~19% of patients and is
H. Hypercalcaemia is seen in patients only in indicative of advanced disease.
the presence of bone metastases. Radical nephrectomy is the only known
I. Nephrectomy is unnecessary in patients effective treatment for localised renal cancer.
with metastatic disease. For those with locally advanced disease, there is
no evidence that adjuvant, post-surgical

Dr. Khalid Yusuf El-Zohry – Sohag Teaching Hospital (01118391123) Page | 744
El-Zohry MRCP Questions Bank (Part 2) – Medical Masterclass 2010

treatment with agents such as interleukin-2 or confined to clinical trials designed to establish
interferon-alpha increases survival. their efficacy and reliability.
Data from two randomised trials have shown
there to be no benefit from post-operative [ Q: 21 ] MasterClass Part2
radiotherapy. Renal carcinoma is a chemo- (2010) - Oncology
resistant tumour. Response to biological
therapies, such as interferon-alpha, correlates A 65-year-old woman attends with her new
with a good performance status, prior partner complaining of vaginal bleeding
nephrectomy and lung-predominant disease. following intercourse. Of note, she has a past
medical history of non-insulin dependent
diabetes mellitus and a right carcinoma of the
[ Q: 20 ] MasterClass Part2 breast diagnosed 4 years ago. On examination
(2010) - Oncology she is thin and has a right mastectomy,
otherwise the examination is unremarkable.
A private GP refers a 47-year-old investment
banker following a cancer screen. The patient,
Which of these statements are correct?
who is well, has had numerous serum tumour
marker tests performed. A. Obese women carry an increased risk as
adipocytes can convert androstenedione
Which of the following tumour markers has a of adrenal origin to oestrone.
role in screening?
B. Comorbid diabetes mellitus is a
A. Carcinoembryonic antigen for gastric protective factor.
cancer
C. Late menarche and early menopause are
B. Human chorionic gonadotropin for pure risk factors for developing endometrial
seminoma carcinoma.
C. Calcitonin for medullary cell thyroid D. The oestrogen antagonist, Tamoxifen, has
cancer a protective property against developing
endometrial carcinoma.
D. β2-microglobulin for myeloma
E. Leiomyosarcomata are more common
E. Thyroglobulin for follicular thyroid cancer
than endometrioid tumours of the
endometrium.
Answer & Comments
F. Post-menopausal bleeding occurs late and
Correct answer: C is seen in less than 60% of patients.
G. Surgery is the mainstay of staging and
Tumour markers have not proved useful in
investigation.
screening for most tumour types. Human
chorionic gonadotropin (HCG) is particularly H. Total abdominal hysterectomy is the
useful in screening for gestational trophoblastic procedure of choice in stage I disease and
disease following a hydatidiform mole, but pure can be curative.
seminomas rarely produce markers. The only I. Peritoneal cytology is unhelpful.
other established screening marker is calcitonin
for medullary cell thyroid cancer, although J. Ca19-9 is a useful tumour marker for
there is some evidence for using CA125 for monitoring disease.
ovarian cancer and prostate-specific antigen for
prostate cancer. However, their use is still

Dr. Khalid Yusuf El-Zohry – Sohag Teaching Hospital (01118391123) Page | 745
El-Zohry MRCP Questions Bank (Part 2) – Medical Masterclass 2010

Answer & Comments B. Fentanyl

Correct answer: AG C. Oxycodone


D. Codeine
Endometrial cancer is a disease of post-
E. Hydromorphone.
menopausal women; the average at diagnosis is
60 years. Post-menopausal bleeding occurs in
over 90% of women. Risk factors for developing Answer & Comments
endometrial carcinoma are related to chronic Correct answer: D
oestrogen exposure, oral exogenous oestrogen
(without progestins), oestrogen-secreting
Strong opiates include: morphine, diamorphine,
tumours, low parity, extended periods of
hydromorphone, methadone, oxycodone and
anovulation, early menarche
fentanyl.
and late menopause. Morbidly obese women
have an increased risk, perhaps because their
[ Q: 23 ] MasterClass Part2
adipocytes can convert androstenedione of
(2010) - Oncology
adrenal origin to oestrone, a weak circulating
oestrogen. Epidemiological studies have A patient present to her GP with a pigmented
demonstrated women with hypertension and lesion on her foot. The patient states that the
diabetes mellitus as having an increased risk of lesion was apparently present from birth and
endometrial cancer. Tamoxifen, used in the does not itch or bleed. However, it is not as
treatment of breast cancer, is primarily an homogenous as it used to be.
oestrogen receptor antagonist, but it does have
some agonist properties. The data suggest that Which one of the following is true?
there may be a relationship between tamoxifen A. Early diagnosis would not affect
and endometrial cancer and therefore women prognosis
taking long-term tamoxifen therapy should be
monitored for uterine abnormalities. B. It would be dangerous to perform an
incisional biopsy of this lesion
Leiomyosarcoma account for ~5% uterine
tumours whereas epithelial tumours such as C. Bleeding and tenderness would be the
endometrioid are more common ~ 90%. The first signs of malignant change
diagnosis is made by either endometrial biopsy D. Since the lesion has been present since
or dilatation and curettage. Endometrial birth, the risk of malignancy is low
carcinoma is a surgically treated and staged
E. Change in colour is suspicious for
tumour.
malignant change.

[ Q: 22 ] MasterClass Part2
(2010) - Oncology
Answer & Comments
A patient with poorly controlled pain on full
dose coproxamol requires a step up the World Correct answer: E
Health Organisation (WHO) analgesic ladder to
strong opiates. Melanomas develop from melanocytes that are
derived from neural crest tissue that migrate to
Which of the following is NOT a strong opiate? the skin, eye, central nervous system and
A. Sevredol occasionally elsewhere. The incidence is rising

Dr. Khalid Yusuf El-Zohry – Sohag Teaching Hospital (01118391123) Page | 746
El-Zohry MRCP Questions Bank (Part 2) – Medical Masterclass 2010

and has been shown to be related to sun- E. Granulosa cell carcinoma of ovary
exposure. The vast majority arise from pre-
existing benign naevi.
Changes in size, colour or edge of a naevus or Answer & Comments
bleeding should alert to the possibility of
melanoma. The three major signs are: Correct answer: D

 Change in size
Human chorionic gonadotropin (HCG) is a
 Change in shape glycoprotein consisting of two non-covalently
 Change in colour. bound subunits. HCG-specific antisera are
directed against various parts of the b chain.
The four minor signs are: HCG is formed physiologically in the
 Inflammation, crusting or bleeding syncytiotrophoblast of the placenta. Its main
 uses are in diagnosing and monitoring
Sensory change, such as itch
pregnancy, gestational trophoblastic disease
 Diameter greater than 7mm. and germ-cell tumours (GCTs). The sensitivity is
Melanoma is essentially a surgically treated 100% for testicular and placental
disease. Biopsy for melanoma can be either choriocarcinomas and for hydatidiform moles,
excisional or incisional. Whichever is utilized, 48-86% for non-seminomatous GCTs and 7-14%
full thickness biopsy into the the subcutaneous for seminomas. Pure choriocarcinomas are thus
tissue must be performed to permit micro- always HCG positive and alpha-fetoprotein
staging of the lesion (for thickness and level of (AFP) negative; endodermal sinus tumours (yolk
the lesion). Early diagnosis, when the sac tumours) are always AFP positive and HCG
melanoma is confined to the epidermis (Clark’s negative, whereas pure seminomas are always
level I), never metastasizes and has a 100% cure AFP negative but HCG positive in only 14% of
rate. cases.

[ Q: 24 ] MasterClass Part2 [ Q: 25 ] MasterClass Part2


(2010) - Oncology (2010) - Oncology

A 28-year-old woman presents with respiratory A 33-year-old woman presents with a


symptoms and her CXR and subsequent CT scan suspicious mole on her leg. Excision biopsy
reveal multiple metastases. The histological confirms melanoma.
report from a CT-guided biopsy describes an
Which TWO of the following are not prognostic
undifferentiated anaplastic tumour. Serum
features for melanoma?
tumour markers are sent. Serum β-human
chorionic gonadotropin is raised, whilst CA19-9, A. Ulceration of lesion
carcinoembryonic antigen, CA15-3, CA125, B. Gender
inhibin and alpha-fetoprotein are all normal.
C. Diameter of lesion
What is the most likely primary diagnosis? D. Clarke’s level
A. Breast cancer E. Age
B. Medullary cell thyroid cancer F. Regionally enlarged nodes, FNA negative
C. Melanoma G. Breslow thickness
D. Choriocarcinoma

Dr. Khalid Yusuf El-Zohry – Sohag Teaching Hospital (01118391123) Page | 747
El-Zohry MRCP Questions Bank (Part 2) – Medical Masterclass 2010

H. Site of lesion G. Dysphagia is a recognized symptom of


superior vena cava obstruction.
I. Regionally enlarged nodes, FNA positive
H. The patient should receive oral
J. Serum LDH.
anticoagulation.

Answer & Comments I. 5-fluorouracil can directly activate the


clotting cascade and cause
Correct answer: CF thromboembolism.
J. Gastric cancer can frequently cause bulky
Prognosis for females is better, and lower limb
mediastinal lymphandeopathy.
primaries have the best prognosis. Serum LDH
is a prognostic factor in metastatic disease.
Tumour ulceration has been recognised as a Answer & Comments
major prognostic factor in the new American Correct answer: CG
Joint Committee on Cancer (AJCC) staging
system.
Superior vena cava obstruction (SVCO) occurs
most frequently with mediastinal tumours, such
[ Q: 26 ] MasterClass Part2 as small cell lung cancer, lymphoma, germ cell
(2010) - Oncology tumours and other metastatic tumours. They
can also arise secondary to thrombus
A 50-year-old man with carcinoma of the associated with an indwelling Hickman line.
stomach has just received three cycles of More than 50% of cases occur with small cell
epirubicin, cisplatin and infusional 5- lung cancer.
fluorouracil (ECF) chemotherapy via a Hickman
line. Over the past 4 days he has noted Gastric tumours most commonly spread locally
increased pain and swelling in his left hand and to adjacent structures involving regional lymph
arm. His wife noted that he had a high facial nodes rather than causing bulky mediastinal
colour in the night and puffy eyes this morning. lymphadenopathy. Symptoms and signs of
SVCO include:
Which of the following is applicable to this  dyspnoea
patient?
 facial swelling
A. His condition is most likely due to an
 head fullness
infective organism.
 cough
B. Superior vena cava obstruction often
presents with pericardial effusion.  arm swelling
C. He has superior vena cava obstruction  chest pain and dysphagia
secondary to a Hickman line associated
 venous distension of the neck and chest
thrombosis.
wall
D. Fluid retention may pre-present
 facial oedema
secondary to 5-fluorouracil treatment.
 cyanosis
E. There is an incidental atrial myxoma
causing outflow obstruction.  plethora of the face and arm oedema.
F. Superior vena cava obstruction occurs For venous catheter associated thromboses,
most frequently with lymphoma. streptokinase or urokinase may cause lysis of
the thrombus early in its formation. Heparin

Dr. Khalid Yusuf El-Zohry – Sohag Teaching Hospital (01118391123) Page | 748
El-Zohry MRCP Questions Bank (Part 2) – Medical Masterclass 2010

and oral anticoagulation may reduce the extent


of the thrombus and prevent its progression.
Removal of the catheter, if possible, is another
option and should be combined with
anticoagulation to prevent embolisation.
The most common malignancies involving the
heart or pericardium include lung, breast,
leukaemia, Hodgkin’s and non-Hodgkin’s
lymphoma, melanoma, gastrointestinal
primaries and sarcomas. Symptoms include
dyspnoea, cough, chest pain, orthopnoea,
palpitations, weakness, fatigue and dizziness.
Signs include an upright forward-leaning
posture for maximal relief, jugular venous
distension, cardiac enlargement, distant heart
sounds.
Which THREE of the following are good
Atrial myxoma usually arise in the left atrium prognostic indicators?
from the intra-atrial septum causing emboli and
A. Age
mitral valve obstruction. Treatment is surgical.
B. Male gender
The main side effects of 5 fluorouracil are:
 C. Non-extremity site
mucositis
 D. Stage of the disease at presentation
diarrhoea
 E. Percentage tumour necrosis to induction
myelosuppression
therapy
 palmar-plantar erythrodysesthesia
F. A long duration of symptoms prior to
(hand-foot syndrome)
presentation
 alopecia
G. Raised alpha-fetoprotein tumour marker
 nail changes
H. Raised placental alkaline phosphatase
 somnolence tumour marker
 cerebellar ataxia I. Translocation of chromosomes 11 and 22
 nausea [t(11; 22)(q24; q12)]

 vomiting. J. Raised serum acid phosphatase.

[ Q: 27 ] MasterClass Part2 Answer & Comments

(2010) - Oncology Correct answer: DE

A 16-year-old boy presents to his GP with a


limp. He had a recent minor injury and now has Osteosarcomas are the most common primary
a continuous dull ache in his thigh that malignancy of bone. Patient age and gender do
occasionally wakes him at night. The right thigh not appear to be an independent prognostic
is swollen, reddened and warm. A biopsy shows factors. The impression that older patients fare
he has an osteosarcoma (see image). worse may be attributed to the increased
proportion of unfavourable axial lesions with

Dr. Khalid Yusuf El-Zohry – Sohag Teaching Hospital (01118391123) Page | 749
El-Zohry MRCP Questions Bank (Part 2) – Medical Masterclass 2010

increasing age. Pelvic and axial lesions are E. Seminoma.


associated with lower survival rates than
tumours of the extremities, probably owing to Answer & Comments
surgical inaccessibility and incomplete removal.
Correct answer: D
Small tumour size (less than one third of the
extremity bone), absence of metastases and
The only cancer that is consistantly reduced in
high percentage tumour necrosis induced by
incidence in cohort studies of HIV positive
induction therapy have been shown to be
patients is breast cancer. The other
important prognostic factors.
malignancies listed are all increased in
Serum alkaline phosphatase (ALP) is increased incidence.
in 45-50% of patients, but is not diagnostic. It is
an important marker of tumour activity in
[ Q: 29 ] MasterClass Part2
patients with osteosarcoma.
(2010) - Oncology
Alpha fetoprotein is a glycoprotein that is
produced by the liver, gastrointestinal tract and A 49-year old man presents to his GP with a
the yolk sac of the human fetus. It is raised in history of chronic intermittent diarrhoea. On
germ cell tumours (GCT) (80% non- examination he has a flushed appearance and
seminomatous), hepatocellular carcinoma facial telangectasia. He has a heart murmur and
(HCC), neural tube defects and Down’s hepatomegaly that is nodular and firm.
pregnancies. It has a role in screening in
Na 130 mmol/l
pregnancy but not in cancer. It is prognostic for
GCT not HCC. Transient increases may be seen K 2.5 mmol/l
in liver diseases. Placental alkaline phosphatase
(PLAP) is an isoenzyme of alkaline phosphatase Urea 12.2 mmol/l
and is raised in seminoma and ovarian Creatinine 119 mmol/l
dysgerminoma (50%).
Calcium 2.4 mmol/l
Primitive neuroectodermal tumours and
Ewing’s sarcomas share a common Albumin 29 g/l
translocation involving chromosomes 11 and 22
ALT 27 U/l
[t(11;22)(q24;q12)].
Which single test may assist in confirming the
[ Q: 28 ] MasterClass Part2 diagnosis?
(2010) - Oncology A. Carcinoembryonic antigen (CEA)
A 44-year-old woman is HIV positive. She is very B. The tumour marker, CA 19-9
concerned that she is at increased risk of
C. 5-hydroxyindoleacetic acid
cancer.
D. Vanillylmandelic acid
Which one of the following cancers is NOT
E. Serum catecholamines.
increased in HIV positive people?
A. Hodgkin’s disease
Answer & Comments
B. Anal cancer
Correct answer: C
C. Non-small cell lung cancer
D. Breast cancer

Dr. Khalid Yusuf El-Zohry – Sohag Teaching Hospital (01118391123) Page | 750
El-Zohry MRCP Questions Bank (Part 2) – Medical Masterclass 2010

This patient has carcinoid syndrome and it is [ Q: 30 ] MasterClass Part2


due to an underlying carcinoid tumour. These (2010) - Oncology
tumours may contain and secrete a number of
biologically active substances. A 50-year-old man presents to you complaining
Immunocytochemical or radioimmunoassay of episodes of feeling faint associated with
studies show that carcinoids can contain palpitations. He has been witnessed to be
adrenocorticotrophic hormone (ACTH), gastrin, 'deathly white' during these spells. Of note, he
has recently been commenced on an anti-
somatostatin, insulin, motilin, growth hormone,
hypertensive by his GP, as he was found to be
gastrin- releasing peptide, serotonin, calcitonin,
hypertensive when he attended for paroxysmal
neurotensin, melanocyte- stimulating hormone-
headaches.
beta, tachykinins (substance P, substance K,
neuropeptide K), glucagon, pancreatic Which statements relate best to this patient?
polypeptide (PP), vasoactive intestinal peptide
(VIP), and prostaglandins. These substances A. On examination you note he has
may not be released in sufficient amounts to numerous light brown spots, which are
cause symptoms. unrelated to this condition.

In various studies of patients with carcinoids, B. Alkalosis is a common finding in this


elevated serum concentrations of PP occur in condition.
43%, motilin in 14%, and subunits of human C. This tumour arises from the adrenal
chorionic gonadotropin (HCG) in 12%; a slightly cortex.
elevated level of gastrin was reported in 15%,
D. An abdominal ultrasound scan is the best
and none were reported with an elevated VIP
investigation to confirm the diagnosis.
or plasma gastrin-releasing peptide level. Even
though these gastrointestinal (GI) peptides E. A MIBI scan is the best nuclear medicine
were present in the serum, it is not apparent technique to localise the tumour.
that any of these peptides contributed to any F. Only about 10% of these tumours are
clinical symptoms. Foregut carcinoids are more malignant.
likely to produce various GI peptides than
midgut carcinoids. G. An elevated urinary 5-hydroxindole acetic
acid (5-HIAA) level is diagnostic of this
5-hydroxyindoleacetic acid is the metabolic condition
product excreted in urine from the metabolism
of serotonin. H. This tumour is associated with MEN I.

Ectopic ACTH production with Cushing's I. Alpha-adrenergic blockade is necessary


syndrome is increasingly seen with foregut pre-operatively to prevent a hypertensive
carcinoids, and in some studies, these tumours crisis.
were the most common cause of the ectopic J. Metaiodobenzylguanidine (MIBG)
ACTH syndrome, accounting for 64% of all radiolabelled with iodine-124 is often
patients. Acromegaly due to release of growth used to treat metastatic disease.
hormone-releasing factors can occur with a
number of carcinoids. Answer & Comments

Correct answer: FI

Phaeochromocytomas are rare tumours which


arise from the chromaffin cells in the adrenal

Dr. Khalid Yusuf El-Zohry – Sohag Teaching Hospital (01118391123) Page | 751
El-Zohry MRCP Questions Bank (Part 2) – Medical Masterclass 2010

medulla (90%). Extra-adrenal tumours are not particularly chemosensitive.


phaeochromocytomas may arise anywhere, 131I-MIBG has been used to treat recurrent or
including the carotid body, intracardiac and in metastatic phaeochromocytoma.
the adrenal medulla. They may secrete
catecholamines and cause intermittent or [ Q: 31 ] MasterClass Part2
sustained hypertension.
(2010) - Oncology
Phaeochromocytomas can be associated with
endocrine and non-endocrine inherited A patient of yours complains of early morning
disorders. Bilateral adrenal medullary headaches, nausea and vagueness. A
phaeochromocytomas are components of MEN computerised tomography scan (CT scan) shows
IIa and MEN IIb. Phaeochromocytomas occur in a cerebral metastasis.
~25% of patients with von-Hippel Lindau
Which of the following statements are true
disease and in <1% patients with
about the management of cerebral metastasis?
neurofibromatosis and von Recklinghausen’s
disease. Malignant phaeochromocytomas are A. The median survival of an untreated
thought not to occur in the MEN syndromes, patient is 4 months.
and are present in ~10% with B. Whole brain radiotherapy improves
phaeochromocytomas. The patient may present survival to 12 months.
with paroxysmal headache, pallor, palpitations,
unexplained fever and hypertension. They may C. Anticonvulsants are not effective in
have signs of chronic hypovolaemia, like controlling seizures due to metastasis.
orthostatic hypotension or lactic acidosis D. Surgery is only indicated for patients with
secondary to excess catecholamine stimulation solitary metastasis.
and vasoconstriction. Most have mild weight
E. Whole brain radiotherapy may produce
loss and there may be induced diabetes
long term changes in cognitive function.
mellitus.
F. Patients with brain metastasis from
Diagnosis is made by measuring the
colorectal cancer have a better prognosis.
catecholamines and metabolites in the urine,
the best study is a 24 hour urine for G. Chemotherapy is rarely helpful, even in
catecholamines, metanephrine and VMA. 5- chemosensitive disease.
hydroxindole acetic acid (5-HIAA) is elevated in H. Steroids may improve symptoms due to
carcinoid syndrome. CT and MRI scans are the peritumoural oedema.
two radiological procedures of choice to
localise phaeochromocytomas. Another I. Young age is a poor prognostic sign.
technique is nuclear scanning following the J. The absence of systemic disease is a poor
administration of metaiodobenzylguanidine prognostic sign.
(MIBG), a compound that is similar to
norepinephrine and is concentrated in Answer & Comments
adrenergic tissue. Surgical resection is the
treatment of choice if localised, with pre- Correct answer: EH
operative alpha-adrenergic blockade. For
malignant phaeochromocytoma, surgical Prognosis:
resection of recurrences or metastases is
 Median survival of patients with
preferable with catecholamine blockade to
untreated brain metastasis is 1 month.
treat hypertensive symptoms. Painful bony
metastases respond well to radiotherapy. The  Steroids increase survival to 2 months.

Dr. Khalid Yusuf El-Zohry – Sohag Teaching Hospital (01118391123) Page | 752
El-Zohry MRCP Questions Bank (Part 2) – Medical Masterclass 2010

 Whole brain radiotherapy improves I. Thyroid cancer


survival to 3-6 months
J. Undifferentiated nasopharyngeal
 Single brain metastases and limited carcinoma.
extracranial disease, surgery and whole
brain radiotherapy, median survival of Answer & Comments
10-16 months
Correct answer: CG
Favourable prognostic factors include:
 absence of systemic disease Thoracic and abdominal neoplasms are more
 young age (< 60 years) likely to present with supraclavicular or lower
cervical adenopathy. I and J are uncommon in
 good performance status (KPS 70) this age group. Glottic cancers spread to
 long time to development of metastasis lymphatics late in their natural history as the
 glottis has a poor lymphatic supply (supraglottic
surgical resection
and subglottic cancers may well present with
 < 3 lesions lymphadenopathy however).
 possible response to steroids
Patients with brain metastases as the only [ Q: 33 ] MasterClass Part2
manifestation of an undetected primary tumour (2010) - Oncology
also have a favourable prognosis, with an
Following chemotherapy a patient develops
overall median survival of 13.4 months. Breast
symmetrical paraesthesia and sensory loss over
cancer patients with brain metastases generally
the toes of both feet.
have a more favourable prognosis than brain
metastases from other types of primary Which of the following cytotoxics is LEAST likely
tumour. Patients with colorectal carcinoma to cause this side effect?
tend to have a poorer prognosis.
A. Vincristine

[ Q: 32 ] MasterClass Part2 B. Doxorubicin


(2010) - Oncology C. Docetaxel

A 63-year-old ex-smoker presents with a lump D. Cisplatin


in the left upper neck. E. Paclitaxel.

Which are the TWO most likely malignant


diagnoses?
Answer & Comments
A. Gastric cancer
B. Chronic lymphatic leukaemia Correct answer: B

C. Squamous carcinoma of oropharynx


Peripheral neuropathy, the most frequent
D. Small cell lung cancer neurotoxicity of chemotherapy, is commonly
E. Adenocarcinoma of pancreas seen with vinca alkaloids, taxanes and platinum
derivatives. It usually presents as symmetrical
F. Squamous carcinoma of glottis sensory loss that may progress to worsening
G. Hodgkin’s disease paraesthesia, loss of tendon reflexes and
eventually motor weakness due to axonal
H. Adenocarcinoma of lung

Dr. Khalid Yusuf El-Zohry – Sohag Teaching Hospital (01118391123) Page | 753
El-Zohry MRCP Questions Bank (Part 2) – Medical Masterclass 2010

degeneration. Features usually improve H. Serum thyroglobulin measurements are


slowly over several months following cessation not helpful in monitoring follicular
of chemotherapy, although residual deficits carcinoma of the thyroid.
may persist indefinitely.
I. Radio-iodine ablation therapy (iodine-131)
requires the presence of the thyroid
[ Q: 34 ] MasterClass Part2 gland to maximise the efficiency of the
(2010) - Oncology treatment.

A 40-year-old woman presents with a painless J. Surgical management invariably involves


swelling in her right anterior neck. An iodine- total thyroidectomy.
123 scan performed shows a ‘cold’ area in the
lower pole of the right lobe of the thyroid. Answer & Comments

Correct answer: EG

Thyroid cancer accounts for less than 0.5% of all


malignant disease in England and Wales.
Thyroid tumours most frequently develop in
young adults, although all age-groups can be
affected. There is a 2.5-fold increased incidence
in women as compared with men. Patients
often present with an asymptomatic thyroid
nodule, with cervical lymphadenopathy or with
bone or lung metastases. Almost all patients
are euthyroid.

Which of the following statements are true ? Thyroid tumours are derived from either
follicular cells (papillary, follicular, Hurthle cell,
A. Thyroid cancer is unlikely in view of her
and anaplastic tumours) or from the
age
parafollicular, C cells (medullary carcinoma).
B. Thyroid cancer is likely if the lump is Papillary tumours comprise approximately 75%
painful of thyroid tumours, follicular tumours ~10%,
C. The tumour is most likely to be a follicular Hurthle cell tumours ~3%, anaplastic ~2%,
carcinoma of the thyroid. medullary tumours ~7%, lymphomas ~2% and
others, e.g. sarcomas <1%.
D. Parafollicular, C-cells, are most likely to
produce calcitonin in papillary cell thyroid Medullary thyroid cancer is associated with
cancer. three distinct familial syndromes, MEN IIA
(phaeochromocytomas and
E. Medullary cell carcinoma of the thyroid is hyperparathyroidism are also seen); MEN IIB
associated with phaeochromocytoma in (phaeochromocytomas, Marfanoid body
the multiple endocrine neoplasia type II habitus, gastrointestinal ganglioneuromas and
syndrome. mucosal neuromas are also seen); and familial
F. Anaplastic carcinoma is the most non-MEN medullary thyroid cancer (medullary
radiosensitive tumour of the thyroid thyroid cancer without associated
gland. endocrinopathies). All three syndromes are
inherited in an autosomal dominant manner.
G. Thyroid tumours occur more commonly
in women than men.

Dr. Khalid Yusuf El-Zohry – Sohag Teaching Hospital (01118391123) Page | 754
El-Zohry MRCP Questions Bank (Part 2) – Medical Masterclass 2010

Medullary thyroid cancers can produce Most side effects of chemotherapy are
numerous non-peptide hormones, e.g. ectopic predictable and are largely due to effects on the
ACTH causing Cushing's syndrome or serotonin most rapidly dividing normal body cells, such as
release causing symptoms consistent with bone marrow cells and gastrointestinal tract
carcinoid. Virtually all medullary thyroid cancers cells. Other cells that are rapid in their growth
produce abnormal levels of calcitonin. include those producing hair.
Surgery is potentially curative in differentiated
thyroid cancer. Total lobectomy is the minimum [ Q: 36 ] MasterClass Part2
treatment. Ablation of thyroid remnants with (2010) - Oncology
iodine-131 aims to destroy residual normal
thyroid tissue. To maximise efficacy of A 48-year-old man presents with fever, weight
radioiodine therapy, a total or near loss, lymphadenopathy and
thyroidectomy should be performed in all hepatosplenomegaly.
patients who are to be treated with iodine.
Which one of the following clinical features
Anaplastic carcinoma of the thyroid is rapidly
would NOT be consistent with the diagnosis of
progressive, prognosis is poor with a median
multicentric Castleman’s disease?
prognosis of 6 months. Surgical excision is
rarely feasible and it is the least radiosensitive A. Pyrexia
of all the thyroid tumours. B. Polyclonal gammaglobinaemia
Serum thyroglobulin measurement has become C. Splenomegaly
the main modality for the early detection of
recurrent differentiated thyroid cancer. It is D. Paraneoplastic pemphigus
produced by normal and neoplastic follicular E. Cardiomyopathy.
cells.

[ Q: 35 ] MasterClass Part2 Answer & Comments


(2010) - Oncology
Correct answer: E
A patient about to start combination
chemotherapy is concerned about the side
Castleman’s disease is a rare
effects. lymphoproliferative disorder characterised by
angiofollicular lymphoid hyperplasia. Two
Which one of the following is the LEAST
histological variants are recognised; a hyaline
common side effect of chemotherapy?
vascular variant and a less common plasma cell
A. Loss of short term memory variant. Castleman’s disease has germinal
B. Alopecia centre hyalinisation or atrophy surrounded by
concentric layers of lymphocytes with
C. Nausea
prominent vascular hyperplasia, hyalinisation of
D. Thrombocytopenia small vessels and interfollicular sheets of
plasma cells and immunoblasts.
E. Infection
Multicentric Castleman’s disease (MCD)
presents with constitutional symptoms
including fever, weight loss and night sweats
Answer & Comments and clinical findings include lymphadenopathy,
Correct answer: A hepatosplenomegaly and rashes. Investigations

Dr. Khalid Yusuf El-Zohry – Sohag Teaching Hospital (01118391123) Page | 755
El-Zohry MRCP Questions Bank (Part 2) – Medical Masterclass 2010

frequently reveal microcytic anaemia, B. The wives / ex-wives of men with penile
hypoalbuminaemia and polyclonal cancer have a 3-8 times increase risk of
hypergammaglobulinaemia. cervical carcinoma than those of men
without penile cancer
Many of the paraneoplastic manifestations of
Castleman’s disease are believed to be due to C. Penile cancer is associated with human
excess interleukin-6 (IL-6) production by the papillomavirus (HPV)
tumour, possibly from the viral IL-6 homologue
D. Improving personal hygiene plays no role
gene of HHV8.
in prevention
The best treatment for MCD is not known
E. Palpable lymphadenopathy is uncommon
at the time of diagnosis and is unlikely to
[ Q: 37 ] MasterClass Part2 be due to metastatic cancer
(2010) - Oncology F. The most likely histology is that of an
A 63-year-old male non-smoker develops a dark adenocarcinoma
purple nodule on his cheek. G. A CT scan is a useful imaging technique
Immunohistochemistry of the biopsy is positive for the staging penile lesions
for neurone specific enolase.
H. Lymphadenectomy is a palliative
Which is the most likely diagnosis? procedure

A. Metastasis from Small cell lung cancer I. Chemotherapy is useful in the treatment
of squamous cell carcinoma of the penis
B. Melanoma
J. Radiotherapy does not have a role
C. Merkel’s cell cancer
D. Kaposi’s sarcoma Answer & Comments
E. Mycosis fungoides. Correct answer: BC

Answer & Comments Squamous cell carcinoma is the most common


Correct answer: C malignant tumour of the penis. It is a rare
condition in the USA and Europe, 0.4-0.6% of all
malignant diseases, yet can account for up to
Merkel's cell cancer is the most likely diagnosis.
10% of malignant diseases in men in some
African and South American countries. It is
[ Q: 38 ] MasterClass Part2 usually seen in men with poor standards of
(2010) - Oncology penile hygiene. In uncircumcised men, tumour
development has been attributed to the
A 50-year-old man presents with a three-month
history of an itchy, burning foreskin. He admits chronic irritative effects of smegma. Phimosis is
to a foul smelling penile discharge. On present in 25-75% of patients with penile
examination, he has a phimosis and palpable carcinoma. Circumcision protects against penile
lymphadenopathy in the right inguinal region. carcinoma and is a well-established
You believe he has penile cancer. prophylactic measure.
Penile and cervical carcinoma have been
Which statement(s) is/are TRUE?
related to viral infections. It has been reported
A. The glans penis is the least common that wives or ex-wives of men with penile
anatomical site for penile cancer to occur cancer had a 3-8 times higher incidence of

Dr. Khalid Yusuf El-Zohry – Sohag Teaching Hospital (01118391123) Page | 756
El-Zohry MRCP Questions Bank (Part 2) – Medical Masterclass 2010

cervical carcinoma than those of men without assess the benefit of chemotherapy in these
penile cancer. Penile cancer is also associated settings.
with sexually transmitted human
papillomavirus (HPV), which is present in 15- [ Q: 39 ] MasterClass Part2
80% of patients with penile cancer. HPV 16 and
(2010) - Oncology
HPV18 have been seen in primary and
metastatic penile cancer. Penile cancers can (1) A 75-year-old man with a 30-pack year
arise at any anatomical site on the penis – glans cigarette smoking history complains of
(48%), prepuce (21%), glans and prepuce (9%), continuous right shoulder pain, a persistent
glans and shaft (14%), coronal sulcus (6%) and cough and weight loss. His chest radiograph
shaft (<2%). Palpable inguinal shows a right apical shadow. On examination
lymphadenopathy is present at diagnosis in 20- you note that he is clubbed, has a small right
96% of patients. Of these, 45% will have cancer pupil and a right-sided ptosis.
in the nodes. The remainder will have a
secondary infection accounting for the What is the most likely diagnosis?
lymphadenopathy. A. Small cell lung cancer
Ultrasonography is a non-invasive, accessible B. Squamous cell carcinoma
and inexpensive technique for staging penile
cancers. Invasion of the tunica or corpora can C. Bronchoalveolar carcinoma of the lung
be reliably seen. CT scan is ineffective in D. Adenocarcinoma of the left lung
assessing penile lesions as it can only image in
E. Bronchial carcinoid.
one plane and has poor contrast in soft tissues,
however it is useful in detecting
lymphadenopathy. MRI produces sharper Answer & Comments
images of penile structures and identifies Correct answer: B
corporal involvement and local extension with
more than 80% accuracy.
It is likely that he has a right apical carcinoma of
Penile cancer remains a locoregional disease the lung causing a Horner’s syndrome
with low incidence of metastasis. Partial or total (ipsilateral ptosis, meiosis and anhydrosis).
penectomy remains the gold standard of Small cell lung cancer generally arises centrally
therapy. Radiotherapy is the traditional as opposed to the more peripheral lesions of
conservative treatment for penile cancer, non-small cell lung cancer. Squamous cell
permitting organ preservation this is important carcinoma accounts for about 30% of all lung
for young men; cancers and arises most frequently in proximal
the segmental bronchi. Most of adenocarcinomas
of the lung are peripheral in origin. The tumour
number of patients for whom radiotherapy is in this case is on the right and not the left.
appropriate is small.
The presence of proven inguinal lymph node [ Q: 40 ] MasterClass Part2
metastasis worsens the prognosis for penile
(2010) - Oncology
cancer. Lymphadenectomy is curative in ~ 50%
of cases and should be undertaken. There is An elderly man with carcinoma of the prostate
little data about chemotherapy for penile and bone metastases presents with sudden
cancer. Neoadjuvant and adjuvant worsening of a previous pain in his right leg. His
chemotherapy appears to be promising, yet drug regime included morphine sulphate
there have been no randomised clinical trials to continus (MST) 100mg bd.

Dr. Khalid Yusuf El-Zohry – Sohag Teaching Hospital (01118391123) Page | 757
El-Zohry MRCP Questions Bank (Part 2) – Medical Masterclass 2010

Which of the following would you do? skin. Geographic area of residence is an
important factor in the development of
A. X-ray the right femur
melanoma, with a higher incidence in areas
B. Re-titrate his opiate requirement with with greater sun exposure, but it does not
oral morphine influence risk of dissemination. The most
C. X-ray the lumbar spine and right femur common site for melanoma in males is the
torso and lesions occurring here have a worse
D. Change to sub-cutaneous diamorphine
prognosis than those occurring on a lower
E. X-ray lumbar spine. extremity. Level of invasion and thickness of the
primary lesion are predictive of dissemination
and survival. The presence of lymphocytes
infiltrating the melanoma has a favourable
Answer & Comments effect on prognosis.
Correct answer: C
[ Q: 42 ] MasterClass Part2
It is important to rule out a fracture or spinal (2010) - Oncology
collapse. The pain may originate from either the
lumbar spine or femur. There may also be new A 60-year-old man who smoked 20 cigarettes a
metastases that may be amenable to day for 40 years enquired about recent
radiotherapy. The patient will probably need an advances in screening for lung cancer that he
increase in oral opiates. had read on the internet.

Features of low-dose thoracic CT scans for lung


[ Q: 41 ] MasterClass Part2 cancer (LC) screening studies include:
(2010) - Oncology A. low sensitivity
A 34-year-old Australian woman presents with B. high specificity
a growing pigmented skin lesion suggestive of
C. low cost
melanoma.
D. no contrast media required
Which of the following factors does not
E. the size of malignancies detected are no
influence the likelihood of metastasis in
smaller than with chest radiograph
malignant melanoma?
A. Primary tumour site F. less radiation than chest radiograph

B. Level of invasion on histology G. low false positive rate

C. Thickness of the primary lesion H. high false negative rate

D. Geographical area of residence I. positive predictive value (PPV) over 50%

E. Presence or absence of tumour- J. low-dose CT detects more cases of lung


infiltrating lymphocytes in the lesion cancer than chest radiograph.

Answer & Comments


Answer & Comments
Correct answer: D
Correct answer: DJ
Primary malignant melanoma is the leading
cause of death from all diseases arising in the
Dr. Khalid Yusuf El-Zohry – Sohag Teaching Hospital (01118391123) Page | 758
El-Zohry MRCP Questions Bank (Part 2) – Medical Masterclass 2010

ADVANTAGES The size of malignancies are smaller with low-


 dose CT.
Extremely sensitive and reliable: able to
detect LC at early stages However, the prevalence-screening rate for
 advanced disease on low-dose CT did not
No contrast media
decrease. True clinical significance of small
 Radiation dose of low-dose CT is 1/7th tumours founded by screening is unknown. CT
the radiation dose of a standard CT founded at least one indeterminate nodule in
DISADVANTAGES 23% of patients.

 Expensive: $350
[ Q: 43 ] MasterClass Part2
 Radiation exposure is about 10 times
(2010) - Oncology
more than one for chest radiograph
 False-positive results are very common. A 30-year-old man presented to his primary
PPV is less than 10%. care physician with a complaint of right leg pain
of one year's duration. The pain was worse at
 Spiral CT scanning screening night but was fully relieved by aspirin. A plain
Two non-randomized studies from Japan: radiograph reveals a focally sclerotic expanded
area of the tibial cortex without overlying soft
- almost 7000 people
tissue or periosteal abnormalities. A radiolucent
- smokers and non-smokers over 40 years of nidus is visible in the centre of the focal area of
age sclerosis. Radionuclide bone scintigraphy
-chest radiograph, sputum cytology and low- reveals a very prominent focal uptake of the
dose CT radiotracer in the same region.

Early Lung Cancer Action Project, non- What is the most likely diagnosis?
randomized trial:
A. Osteoid osteoma
- 1000 people
B. Stress fracture
- high smokers over 60 years of age
C. Metastatic deposit
- chest radiograph and low-dose CT.
D. Trauma
RESULTS:
E. Osteogenic sarcoma.
Low-dose CT detects more cases of lung cancer
than chest radiograph (27/1000 vs 9.1 to Answer & Comments
7.6/1000)
Correct answer: A
Low-dose CT compared with chest radiograph
detected:
Osteoid osteoma is a benign bone tumour with
- Non-calcified nodules: three times as a central small nidus of osteoid which incites a
commonly (23% vs 7%) vigorous reaction in surrounding tissue. It is
- Malignancies: four times as commonly (2.7% characteristically found in the femur, tibia,
vs 0.7%) talus, spine and humerus, usually in the
diaphysis or metaphysis of these bones. The
- Stage I malignancies: six times as commonly
common presentation is one of pain unrelated
(2.3% vs 0.4%)
to activity or most pronounced after drinking
alcohol. The pain is often relieved with non-

Dr. Khalid Yusuf El-Zohry – Sohag Teaching Hospital (01118391123) Page | 759
El-Zohry MRCP Questions Bank (Part 2) – Medical Masterclass 2010

steroidal anti-inflammatory drugs (NSAIDs). It is occur on many different nerves or be limited to


most common between 7 and 25 years of age only one nerve or to one extremity.
and is three times more common in males.
Schwannomas (and neurofibromas) are seen in
association with Neurofibromatosis (‘von
[ Q: 44 ] MasterClass Part2 Recklinghausen’s disease’). Schwannomas and
(2010) - Oncology neurofibromas look very similar on MR.

A 29-year-old right handed woman presents Although Schwannomas are benign (they do
with a five year history of a slowly growing not metastasise), a small number (5-15%
mass on the wrist of her left hand that was firm quoted in the literature) are more aggressive
and mobile. The histological diagnosis was and some can even become malignant
Schwannoma. (malignant transformation) and very rarely
metastasise. Watching with serial imaging and
Which of the following is true of Schwannomas? clinical examination is a reasonable approach
but surgery may be best if there is any increase
A. They cause local invasion of nerve
in size or more aggressive appearance.
resulting in neurological deficit.
Schwannomas can often be removed with little
B. Left in situ, 80% will undergo malignant injury to the nerve that they grow on, as
transformation. opposed to neurofibromas whose removal
C. They cannot be resected without removal usually requires division of the nerve around
of nerve fascicle. the tumour.

D. They cannot be distinguished from


neurofibromas by MR scanning. [ Q: 45 ] MasterClass Part2
(2010) - Oncology
E. They are always multiple.
A patient presents with facial swelling and
Answer & Comments distended veins in the chest and arms. A chest
radiograph shows upper mediastinal widening.
Correct answer: D
Which of the following should be done next?
These relatively rare benign tumours are A. Doppler ultrasound
nonetheless the most common tumour of the
B. Radiotherapy to the chest
peripheral nerves, and may affect any nerve in
the body. They arise from the sheath C. Bronchoscopy
surrounding the nerve (Schwann cells). In
D. CT Scan of chest and abdomen
Schwannomas, cells are not organized like
Schwann cells around the axons but instead in E. Mediastinoscopy.
swirls and streaks across and among
themselves. Schwannomas do not invade the Answer & Comments
nerve and can usually be surgically removed
Correct answer: D
without damage to the adjacent nerve.
Schwannomas usually grow very slowly but
An urgent CT scan will help in deciding the best
they can become very large. Very large tumours
way of getting a tissue diagnosis, be this by CT
can be seen or felt as a bulge under the skin.
guided biopsy, mediastinoscopy or
Schwannomas (like neurofibromas) can occur bronchoscopy if there is a hilar lesion. In the
singly or multiply. Multiple Schwannomas can vast majority of patients it should be possible to

Dr. Khalid Yusuf El-Zohry – Sohag Teaching Hospital (01118391123) Page | 760
El-Zohry MRCP Questions Bank (Part 2) – Medical Masterclass 2010

obtain a tissue diagnosis and institute A. It has a lower incidence in obese patients
appropriate treatment; in the case of
B. It is associated with an increase in sex
lymphoma or small cell lung cancer this will be
hormone binding globulin
chemotherapy, rather than palliative
radiotherapy. C. It should be treated with progesterone
D. It often presents with an enlarged uterus
[ Q: 46 ] MasterClass Part2 E. It is increased in nulliparous women.
(2010) - Oncology
A 68-year-old man presents with a 3-week Answer & Comments
history of increasing confusion and headaches. Correct answer: E
A CT scan of the brain reveals multiple
metastatic lesions.
Endometrial cancer is 10 times more common
Which are the TWO most likely primary sites? in obese women. It is associated with elevated
levels of free oestrogens due to falls in sex
A. Renal carcinoma hormone binding globulin, or use of unopposed
B. Squamous carcinoma of lung oestrogens especially as hormone replacement
therapy (HRT) and tamoxifen.
C. Prostate
These tumours present in postmenopausal
D. Melanoma
women as uterine bleeding. Postmenopausal
E. Rectum bleeding is always abnormal and requires
F. Small cell lung cancer prompt investigation with dilatation and
curettage or suction curettage. Other
G. Large cell lung cancer associations are low parity, extended period of
H. Oesophageal adenocarcinoma anovulation, early menarche and late
menopause. Pregnancy represents a 9-month
I. Stomach
period of relatively intense progesterone
J. Oral cavity. stimulation by the placenta.

Answer & Comments [ Q: 48 ] MasterClass Part2


Correct answer: FG (2010) - Oncology
A 65-year-old man with known metastatic non-
Squamous carcinomas (B and J) rarely cause small-cell lung cancer develops lower back pain.
brain metastases. Melanoma is the most Physical examination is normal, but plain
common primary site after lung cancer and radiographic films show sclerosis of T12 and L1.
breast cancer.
Which of the following would be appropriate in
the management of this patient?
[ Q: 47 ] MasterClass Part2
(2010) - Oncology A. Nerve conduction studies

A 58-year-old woman presents to you B. Lumbar puncture manometry


complaining of post-menopausal bleeding. C. Lumbar puncture and cerebrospinal fluid
cytology
Which of the following is true of endometrial
carcinoma? D. MRI of the spinal column

Dr. Khalid Yusuf El-Zohry – Sohag Teaching Hospital (01118391123) Page | 761
El-Zohry MRCP Questions Bank (Part 2) – Medical Masterclass 2010

E. Corticosteroid therapy A. Focal sclerotic lesion in tibia with


periosteal reaction
B. Male gender
Answer & Comments C. Age distribution usually 7-25 years
Correct answer: D D. Pain worse after alcohol and exercise
E. Normal 99Tc-diphosphonate bone scan.
This patient is most likely to have epidural
spinal cord compression as a complication of
Answer & Comments
metastatic lung cancer. Local or radicular pain is
the most frequent and earliest clinical Correct answer: E
symptom. Clinical examination in the early
stages can be unremarkable. However, Osteoid osteoma is characteristically found in
subsequent weakness and bladder and bowel the femur, tibia, talus spine and humerus,
dysfunction can develop. usually in the diaphysis or metaphysis of these
The diagnosis of spinal cord compression must bones. This is a benign bone tumour with a
always be considered even if the clinical central small nidus of osteoid which incites a
examination is normal. The diagnosis can be vigorous reaction in surrounding tissue. The
confirmed by MRI, which is considered superior common presentation is one of pain unrelated
to CT imaging or myelography. Lumbar to activity or most pronounced after drinking
puncture should be avoided as herniation of the alcohol. The pain is often relieved with non-
cord into a decompressed region can result steriodals. Osteoid osteoma is most common
following the removal of cerebrospinal fluid. between 7 and 25 years of age and is three
Manometry and cytological analysis are unlikely times more common in males.
to give diagnostic information and may worsen
the situation due to the lumbar puncture. [ Q: 50 ] MasterClass Part2
Patients with rapidly progressive neurological (2010) - Oncology
signs should be considered for neurosurgical
A 70-year-old man develops night blindness,
decompression, and radiotherapy is useful in
ring scotomas and photosensitivity due to anti-
the treatment of slowly progressive lesions.
recoverin antibodies.
Systemic chemotherapy and corticosteroids
should not be used in place of surgery or The most likely underlying pathology is:
radiotherapy, and may not influence the clinical
situation. A. Small-cell lung cancer
B. Multiple sclerosis
[ Q: 49 ] MasterClass Part2 C. Temporal arteritis
(2010) - Oncology D. Lyme disease
A 20-year-old man presented with right leg pain E. Colon cancer.
of one year's duration. There is no history of
trauma. The pain was worse at night but was
Answer & Comments
fully relieved by aspirin.
Correct answer: A
Which of the following features does not
support a diagnosis of osteoid osteoma?

Dr. Khalid Yusuf El-Zohry – Sohag Teaching Hospital (01118391123) Page | 762
El-Zohry MRCP Questions Bank (Part 2) – Medical Masterclass 2010

The most common causes of paraneoplastic D. Radiotherapy and chemotherapy


retinopathy are SCLC and melanomas. followed by surgical resection
E. Surgical resection followed by
[ Q: 51 ] MasterClass Part2 radiotherapy depending on findings at
(2010) - Oncology operation

A 35-year-old woman develops bleeding from F. Surgical resection followed by


her nipple. chemotherapy depending on findings at
operation
The most likely underlying cause is: G. Surgical resection followed by
A. Duct ectasia radiotherapy and chemotherapy
depending on findings at operation
B. Fat necrosis
H. Radiotherapy
C. Papilloma
I. Chemotherapy
D. Inflammatory carcinoma
J. Watchful waiting.
E. Sclerosing adenosis.

Answer & Comments

Answer & Comments Correct answer: BE

Correct answer: C It is not known whether radiotherapy followed


by surgical resection (option B) or surgical
Given the age of the patient, the bleeding from resection followed by radiotherapy depending
the nipple is most likely to be associated with a on findings at operation (option E) produces the
papilloma. The remaining options are less likely best outcome. An MRC trial (to which many
at this age or to cause bleeding as the hospitals in the UK are recruiting patients) is
presenting feature. currently comparing these two treatments.
If locally advanced disease (spread to local
[ Q: 52 ] MasterClass Part2 lymph nodes, Dukes’ Grade C) is found at
(2010) - Oncology surgery, then adjuvant chemotherapy improves
median and symptom-free survival.
A 68-year-old man presents with rectal bleeding
and is found to have carcinoma of the rectum. The mainstay of colorectal chemotherapy is 5-
Investigation shows no evidence of distant flourouracil / folinic acid combination.
metastases.
[ Q: 53 ] MasterClass Part2
Which TWO of the following are accepted initial
(2010) - Oncology
treatment regimens?
A. Surgical resection A 38-year-old woman with carcinoma of the
breast is receiving adjuvant doxorubicin and
B. Radiotherapy followed by surgical cyclophosphamide chemotherapy.
resection
C. Chemotherapy followed by surgical Which antiemetics are you going to prescribe
resection for her and why?

Dr. Khalid Yusuf El-Zohry – Sohag Teaching Hospital (01118391123) Page | 763
El-Zohry MRCP Questions Bank (Part 2) – Medical Masterclass 2010

A. The cannabinol derivative, Cyclizine, for The chemotherapy regimen of adriamyin


acute emesis (doxorubicin) and cyclophosphamide is highly
emetogenic.
B. Granisetron, a serotonin (5HT3)
antagonist, with dexamethasone for Granisetron and ondansetron are 5HT3
treatment of acute emesis antagonists. Serotonin (5HT3) is an important
transmitter in emesis and is released from the
C. The dopamine-2 receptor antagonist,
gut and in the central nervous system. There
Domperidone, as it crosses the blood
appears to a synergy between the 5HT3
brain barrier to act on the vomiting
antagonists and dexamethasone for the control
centre
of acute emesis. Central nervous system effects
D. The synthetic cannabinol derivative, of sedation and extra-pyramidal reactions are
Nabilone, as it is non-sedative and acts not observed with serotonin antagonists. The
centrally on the vomiting centre in the common side effects are mild headache,
medulla transient elevations of transaminases and mild
E. Metoclopramide alone with treatment as constipation.
this is a low emetogenic regimen The antihistamine, cyclizine, is a H1 receptor
F. Metoclopramide for the treatment of antagonist that is useful for delayed emesis. It
delayed emesis as it acts in the has a degree of antimuscarinic activity.
chemoreceptor trigger zone The phenothiazine, prochlorperazine (Stemetil),
G. The H1-receptor antagonist, is a dopamine D2 receptor antagonist which
Domperidone, as it binds to receptors in acts in the chemoreceptor trigger zone. It can
the gastrointestinal tract cause extrapyramidal side effects. It can be
useful in managing delayed emesis.
H. The dopamine-2 receptor antagonist,
Ondansetron, for the treatment or Metoclopramide is also a dopamine D2
prevention of delayed emesis receptor antagonist that acts in the
chemoreceptor trigger zone. It also has a
I. Prochlorperazine (Stemetil), for acute peripheral action on the gut. At higher doses it
emesis as it acts on the vomiting centre, appears to work on the 5HT3 pathway. Side
but it may cause antimuscarinic side effects include extra-pyramidal reactions and
effects
oculo-gyric crises. It can be useful in managing
J. Sedation is a troublesome side effect of delayed emesis.
the 5HT3 antagonist ondansetron. Nabilone is a synthetic cannabinol derivative
and reduces vomiting induced by substances
Answer & Comments acting on the chemoreceptor trigger zone. It is
oral and can cause dry mouth, drowsiness and
Correct answer: BF
dizziness. It may be useful for delayed emesis.

Approximately 55% of patients will develop Domperidone is a dopamine D2 receptor


nausea or vomiting within the first 5-7 days of antagonist and acts at the chemoreceptor
receiving chemotherapy. trigger zone. However, it does not readily cross
the blood brain barrier and therefore it is less
Acute emesis is defined as vomiting occurring
likely to cause central side effects such as
within 24 hours of receiving chemotherapy and
sedation and dystonia.
delayed emesis occurs after 24 hours of
chemotherapy administration.

Dr. Khalid Yusuf El-Zohry – Sohag Teaching Hospital (01118391123) Page | 764
El-Zohry MRCP Questions Bank (Part 2) – Medical Masterclass 2010

[ Q: 54 ] MasterClass Part2 C. Adenocarcinoma of the lung


(2010) - Oncology D. Squamous cell lung cancer
A 56-year-old female smoker presents with a 2- E. Bronchial carcinoid
month history of back pain and 2 days of leg
weakness and urinary dysfunction. Chest X-Ray
is normal but MRI of the spine shows a solitary
Answer & Comments
soft tissue mass eroding T8 and compressing
the spinal cord. Correct answer: D

What should be the initial management?


Squamous cell cancers may cavitate and are
A. Emergency radiotherapy to the spine most frequently associated with ectopic
B. Systemic chemotherapy against likely parathyroid hormone production and
primary tumours hypercalcaemia.Small-cell lung cancers often
present with large mediastinal masses and
C. Dexamethasone to control symptoms and distant metastases (40% bone, 30% liver and
further investigations to locate the 15-25% bone marrow, including brain
primary (eg CT scan, mammogram) metastases). They are associated with
D. Intrathecal methotrexate paraneoplastic manifestations including ectopic
adrenocorticotropic hormone production
E. Surgical decompression, debulking and
leading to Cushing's syndrome, the synrdrome
biopsy
of inappropriate antidiuresis and neurological
paraneoplastic phenomena.
Answer & Comments
Adenocarcinomas of the lung generally arise in
Correct answer: E the periphery, unrelated to the bronchi.
Bronchoalveolar carcinoma presents as a single
The priority is to relieve cord compression and nodular or multinodular pattern, and is
in a solitary lesion this is best done surgically, associated with previous fibrotic lung disease,
and a pathological diagnosis can be made. including repeated pneumonias, granulomas
Patients with malignant cord compression have and idiopathic pulmonary fibrosis. It is not
a better physical outcome with surgical strongly associated with smoking. Bronchial
decompression than with radiotherapy. carcinoid can produce dramatic symptoms that
Possible causes include a solitary plasmacytoma include excessive lacrimation, salivation,
as well as metastatic disease from many sites. periorbital oedema,
bronchoconstriction, hypotension, tachycardia,
[ Q: 55 ] MasterClass Part2 anxiety, tremor, nausea, vomiting and explosive
(2010) - Oncology diarrhoea. The vasoactive peptides released
from a primary bronchial carcinoid, unlike a
A 67-year-old man with a 50 pack-year history gastrointestinal tract carcinoid, are not
of smoking presents with a cavitating lung delivered to the liver where they would be
tumour and has a CT-guided lung biopsy. metabolised. Thus bronchial carcinoid can
produce carcinoid symptoms in the absence of
What histological subtype is most likely?
metastatic spread to the liver.
A. Small-cell lung cancer
B. Bronchoalveolar lung cancer

Dr. Khalid Yusuf El-Zohry – Sohag Teaching Hospital (01118391123) Page | 765
El-Zohry MRCP Questions Bank (Part 2) – Medical Masterclass 2010

[ Q: 56 ] MasterClass Part2 breast diagnosed 4 years ago. On examination


(2010) - Oncology she is thin and has a right mastectomy,
otherwise the examination is unremarkable.
A 63-year-old woman presents with one
episode of haemoptysis. The chest radiograph Which of these statements are correct?
reveals a left hilar lesion, confirmed on CT scan A. Obese women carry an increased risk as
which is otherwise clear. She had been treated adipocytes can convert androstenedione
for node-positive, oestrogen receptor positive of adrenal origin to oestrone.
left breast cancer four years previously with
wide local excision and axillary dissection, B. Comorbid diabetes mellitus is a
adjuvant anthracycline chemotherapy, protective factor.
radiotherapy and tamoxifen, which she is still C. Late menarche and early menopause are
taking. risk factors for developing endometrial
carcinoma.
Which of the following is appropriate initial
management? D. The oestrogen antagonist, Tamoxifen, has
a protective property against developing
A. Withdraw tamoxifen and await tamoxifen endometrial carcinoma.
withdrawal response
E. Leiomyosarcomata are more common
B. Commence second line hormone therapy than endometrioid tumours of the
with anastrazole endometrium.
C. Organise a PET scan F. Post-menopausal bleeding occurs late and
D. Commence palliative chemotherapy with is seen in less than 60% of patients.
a taxane G. Surgery is the mainstay of staging and
E. Organise bronchoscopy and biopsy investigation.
H. Total abdominal hysterectomy is the
Answer & Comments procedure of choice in stage I disease and
can be curative.
Correct answer: E
I. Peritoneal cytology is unhelpful.
Although metastatic breast cancer can present J. Ca19-9 is a useful tumour marker for
in countless ways, the possibility of a second monitoring disease.
malignancy should be considered (in this case,
possibly an operable lung cancer). If the pattern
Answer & Comments
of recurrence is characteristic, for example
multiple bone or liver metastases, then a Correct answer: AG
confirmatory biopsy is not generally required.
Endometrial cancer is a disease of post-
[ Q: 57 ] MasterClass Part2 menopausal women; the average at diagnosis is
(2010) - Oncology 60 years. Post-menopausal bleeding occurs in
over 90% of women.
A 65-year-old woman attends with her new
Risk factors for developing endometrial
partner complaining of vaginal bleeding
carcinoma are related to chronic oestrogen
following intercourse. Of note, she has a past
exposure, oral exogenous oestrogen (without
medical history of non-insulin dependent
progestins), oestrogen-secreting tumours, low
diabetes mellitus and a right carcinoma of the
parity, extended periods of anovulation, early

Dr. Khalid Yusuf El-Zohry – Sohag Teaching Hospital (01118391123) Page | 766
El-Zohry MRCP Questions Bank (Part 2) – Medical Masterclass 2010

menarche and late menopause. Morbidly obese Answer & Comments


women have an increased risk, perhaps
Correct answer: A
because their adipocytes can convert
androstenedione of adrenal origin to oestrone,
a weak circulating oestrogen. Epidemiological You should have a high index of suspicion about
studies have demonstrated women with spinal cord compression which needs to be
hypertension and diabetes mellitus as having an diagnosed and treated as an emergency in
increased risk of endometrial cancer. order to have the best possible outcome.
Tamoxifen, used in the treatment of breast Underlying infections are common in this
cancer, is primarily an oestrogen receptor elderly and debilitated group and should be
antagonist, but it does have some agonist considered particularly if there is non-specific
properties. The data suggest that there may be deterioration.
a relationship between tamoxifen and
endometrial cancer and therefore women [ Q: 59 ] MasterClass Part2
taking long-term tamoxifen therapy should be (2010) - Oncology
monitored for uterine abnormalities.
A 41-year-old male presented with a node in his
Leiomyosarcoma account for ~5% uterine
left lower neck region. A biopsy of the node
tumours whereas epithelial tumours such as
revealed Hodgkin's lymphoma, nodular
endometrioid are more common ~ 90%. The
sclerosis type. Subsequently a mediastinal mass
diagnosis is made by either endometrial biopsy
was found. Prior to Gallium scintigraphy he was
or dilatation and curettage. Endometrial
initially staged as IIA; however, this revealed
carcinoma is a surgically treated and staged
addition gallium avid tumours in the left para
tumour.
aortic region, and a small focus in the splenic
hilar region.
[ Q: 58 ] MasterClass Part2
(2010) - Oncology 67Gallium (67-Ga) scanning has been used in
malignant lymphoma treatment for all but
A GP rings you about an 82-year-old man with which of the following:
multiple myeloma. The patient has recently
A. Staging at diagnosis
started to experience unsteadiness when
walking and has fallen several times in the last B. Detection of relapse
24 hours. C. Screening at risk populations
What advice would you give? D. Evaluation of response to therapy
A. Send the patient to hospital as an E. Predicting prognosis.
emergency.
B. Arrange to see the patient within the next Answer & Comments
few days in the next available clinic slot. Correct answer: C
C. Arrange a CT or MRI brain scan.
D. Ask the GP to exclude an underlying 67-Ga scintigraphy has been used for more than
infection. 20 years in the detection of malignancies. Its
exact role in malignant lymphoma has been
E. Start the patient on high-dose steroids. disputed, mainly because of the high false-
negative rate. The role of 67-Ga scintigraphy in
lymphoma relies on its ability to detect viable

Dr. Khalid Yusuf El-Zohry – Sohag Teaching Hospital (01118391123) Page | 767
El-Zohry MRCP Questions Bank (Part 2) – Medical Masterclass 2010

residual disease with or without residual Presumably such therapy eradicates residual
radiographic abnormalities and conversely to subclinical disease, which is invariably present
identify patients with no viable disease whose despite the apparently complete resection.
radiographic studies are still abnormal. This Effective drugs include taxol, cisplatin,
depends on high 67-Ga avidity in untreated cyclophosphamide, hexamethylmelamine, and
tumours, which vary with histological subtype doxorubicin. Paclitaxel plus cisplatin is the
and anatomic distribution of the disease. standard regimen.
Since some patients may have recurrent
[ Q: 60 ] MasterClass Part2 disease without an elevation of CA125 (which is
(2010) - Oncology a useful antigen in monitoring response to
therapy in those who have elevated levels) the
A 65-year-old woman with increasing delay of therapy, pending a rise in this level,
abdominal pain is found to have a pelvic mass would not be prudent.
on physical examination. After appropriate
staging studies she undergoes a laparotomy Clear survival benefits have yet to be shown for
and is found to have serous carcinoma of the the fairly toxic regimen of whole abdominal
ovary with involvement of one ovary and radiation therapy. Intraperitoneal
several omental implants. She then undergoes a chemotherapy holds promise in the eradication
hysterectomy, bilateral salpingo- of minimal disease, but its role needs to be
oophorectomy, liver biopsy, omentectomy, defined by further clinical trials
cytological examination of abdominal washings,
and extensive inspection. All evidence of [ Q: 61 ] MasterClass Part2
disease is removed. (2010) - Oncology
Assuming generally good health, an uneventful A previously independent 75-year-old man has
postoperative recovery, and lack of proximity to been referred to you with a two-month history
a centre performing clinical trials, what of backache and falls over the past 2 days.
treatment should she now receive? Clinical examination reveals that he has
A. No further therapy. bilateral weak legs, a palpable bladder and a
sensory deficit up to the level of the umbilicus.
B. Combination chemotherapy.
C. Combination chemotherapy only if serum Which statements are true?
CA125 level is elevated. A. There is a spinal cord compression at the
D. Intraperitoneal chemotherapy. level of T 12.

E. Whole abdominal radiation therapy. B. Sphincter control is lost in compression of


the cauda equina.

Answer & Comments C. Back pain is often worse when lying flat.

Correct answer: B D. The investigation with the highest


diagnostic specificity is a bone scan.

The overall 5-year survival of those with disease E. Neurosurgical intervention is rarely
that extends beyond the ovaries is 40%. beneficial.
However, some patients who are able to F. Age is an adverse prognostic factor for
undergo complete or nearly complete initial neurological recovery.
cytoreductive surgery, may be cured with
combination chemotherapy. G. He is likely to have a cauda equina lesion.

Dr. Khalid Yusuf El-Zohry – Sohag Teaching Hospital (01118391123) Page | 768
El-Zohry MRCP Questions Bank (Part 2) – Medical Masterclass 2010

H. Loss of sphincter control is an adverse abdominal discomfort. He is a heavy smoker


prognostic factor. and has spent much of his youth in East Asia. In
the past he had a total colectomy for ulcerative
I. Autonomic dysfunction is a common
colitis. Ultrasound of the abdomen
feature.
demonstrates a normal pancreas, intrahepatic
J. Urgent treatment with radiotherapy duct dilatation and a perihilar mass.
confers a survival advantage.
Which of these statements are true regarding
Answer & Comments cholangiocarcinoma?
A. The best diagnostic investigation is an
Correct answer: CH
endoscopic retrograde
Up to 5% of patients develop spinal cord chlangiopancreatography (ERCP) or
compression, and around 30% will survive for 1 percutaneous transhepatic
year. Neoplastic cord compression is nearly chlangiography (PTC) and biopsy.
always due to extramedullary, extradural B. Heavy smoking is a recognised risk factor.
metastases, most commonly from prostate
C. Ulcerative colitis is a recognised risk
cancer or multiple myeloma; also from lung,
factor.
breast, lymphoma or renal cancers.
D. His travel history is irrelevant.
To prevent paraplegia the diagnosis of spinal
cord compression must be made swiftly and E. It is associated with an elevated alpha-
treatment be instituted quickly. Following fetoprotein.
suspicion of the diagnosis, high-dose steroid
F. Portal vein chemotherapy is the
therapy should be instituted, an urgent
treatment of choice.
magnetic resonance imaging (MRI) scan should
be undertaken and urgent radiotherapy given. G. Systemic chemotherapy is the treatment
Surgical decompression is useful if radiotherapy of choice.
is ineffective or not possible or if there is H. Only a minority of the tumours are
skeletal instability. adenocarcinoma.
The sensory level at the nipples is at T4 and that I. The average age at presentation is 50-55
at the umbilicus is T10. In a cauda equina lesion years.
the weakness may be asymmetrical and mild;
sphincter control is often spared. The earliest J. Perihilar tumours are the least common.
symptom of cord compression is vertebral pain,
especially on coughing or lying flat. Signs Answer & Comments
include sensory changes one or two
Correct answer: AC
dermatomes below the level of compression,
progressing to motor weakness distal to the
Cholangiocarcinoma occurs anywhere in the
block and finally sphincter disturbance. Residual
biliary tree; 95% are adenocarcinomas. They
neurological deficit usually reflects the extent
may be divided into three anatomical groups:
of the deficit at the start of treatment.
perihilar (~70%), distal, intrahepatic tumours.
The management of intrahepatic tumours is
[ Q: 62 ] MasterClass Part2 surgical resection, distal tumours are managed
(2010) - Oncology with a pancreaticoduodenectomy and perihilar
tumours may be managed with both operative
A 50-year-old man presents to you with
and non-operative interventions. The average
jaundice, anorexia, cachexia and mild
Dr. Khalid Yusuf El-Zohry – Sohag Teaching Hospital (01118391123) Page | 769
El-Zohry MRCP Questions Bank (Part 2) – Medical Masterclass 2010

age at presentation is between 60-65 years, E. Lead.


with a slight preponderance in men, 1.3:1. Risk
factors Answer & Comments
include: ulcerative colitis, sclerosing cholangitis, Correct answer: C
nitrosamines, Thorotrast, isoniazid,
methyldopa, oral contraceptives, liver flukes
Asbestos is most commonly associated with
(e.g. Clonorchis sinensis & Opisthorchis
mesothelioma, but lung cancer is also at
viverrini) in East Asia and Thailand. Cigarette-
increased incidence following exposure to
smoking is a risk factor which hasn’t, as yet,
fibres. Aluminium does not directly increase the
been implicated.
risk of cancer but workers in aluminium
More than 90% present with jaundice; pruritus, foundaries are exposed to higher levels of
abdominal pain and cachexia are less common. polycyclic hydrocarbons which can result in lung
Serum CEA and AFP are usually normal, Ca19-9 cancer. Petrol is manufactured from benzene
may be elevated. Ultrasound examination or CT substrates which are associated with acute
scan help to delineate the tumour extent, leukaemias.
including involvement of hilar vessels, liver and
distant metastases. Documentation of bile duct
[ Q: 64 ] MasterClass Part2
dilatation and anatomy is defined through
either percutaneous transhepatic (PTC) or (2010) - Oncology
endoscopic retrograde cholangiography (ERC). A 62-year-old man presents with altered bowel
Tissue diagnosis can be established via habit and has positive faecal occult bloods.
percutaneous needle aspiration, brush or Colonoscopy reveals a tight sigmoid
scrape biopsy or cytology of the bile. The type adenocarcinoma. A staging CT scan shows two
of surgery is dependent on the site and extent metastases in the left lobe of the liver.
of the tumour. Trials of chemotherapy have
been limited and small, and not proven to The initial treatment plan should be:
enhance survival in patients. Chemo-
A. chemotherapy prior to surgery
radiotherapy has been used in selected patients
although the patient groups have included B. left hemicolectomy and palliative
those with resected as well as unresected chemotherapy
tumours. C. chemo-radiotherapy to primary tumour,
then continued palliative chemotherapy
[ Q: 63 ] MasterClass Part2 D. left hemicolectomy then consideration of
(2010) - Oncology liver resection
A 65-year-old man with prostate cancer is E. palliative chemotherapy alone.
concerned that occupational exposure may
have contributed to his cancer. Answer & Comments

Which of the following is known to increase the Correct answer: D


risk of prostate cancer?
A. Aluminium The first priority should be resection of the
sigmoid lesion to prevent large bowel
B. Asbestos
obstruction. Two liver lesions confined to the
C. Cadmium same lobe may be suitable for liver resection,
D. Petrol
Dr. Khalid Yusuf El-Zohry – Sohag Teaching Hospital (01118391123) Page | 770
El-Zohry MRCP Questions Bank (Part 2) – Medical Masterclass 2010

with chemotherapy either before or after Cyproterone acetate, Griseofulvin, Marijuana,


resection. Diamorphine, Phenothiazines, Metoclopramide,
Isoniazid, Tricyclics, Diazepam.
[ Q: 65 ] MasterClass Part2
(2010) - Oncology [ Q: 66 ] MasterClass Part2
(2010) - Oncology
A 33-year-old man attends the GP surgery
complaining of bilateral nipple tenderness and A 28-year-old woman reports that several of
breast swelling. her family members have died of breast cancer,
including her mother at age 43, her maternal
Which of these statements does not help to aunt at age 62 and her maternal grandmother
provide a possible explanation for the cause? at age 65.
A. He regularly smokes marijuana.
What is the probability that this patient will
B. He has recently noted a lump in his left develop breast cancer?
testicle.
A. 1 %
C. He takes cimetidine for a peptic ulcer.
B. 10 %
D. He has breast cancer.
C. 20 %
E. He has recently started treatment for a
D. 50 %
right adrenal tumour.
E. 100 %
F. He has hypothyroidism.
G. He takes a tricyclic antidepressant.
Answer & Comments
H. He regularly consumes alcohol.
Correct answer: C
I. He takes regular prednisolone for asthma.
J. He abuses amphetamines. The risk of developing breast cancer in the
general population is approximately 8%.
Answer & Comments Patients that have a strong family history of
breast cancer are at a higher risk, and this is
Correct answer: DI especially true if one of the family members
was young at the time of their presentation.
Gynaecomastia is abnormal breast tissue in a The overall relative risk of breast cancer in a
man. It can be normal around puberty. woman with an affected first-degree relative is
1.7. Premenopausal onset in a first-degree
Causes:
relative is associated with a three-fold increase
Normal puberty/neonatal, Hypothyroidism, in relative risk, whereas postmenopausal
Hyperthyroidism, Hypoadrenalism, Testicular diagnosis increases the relative risk by only 1.5.
atrophy, Testicular/adrenal tumour, No increase in risk has been demonstrated
Acromegaly, Hypothalamic lesions, Carcinoma when only a second-degree relative (aunt,
or lymphoma, Klinefelter's syndrome, cousin, grandmother) is affected. The risk in this
Refeeding after malnutrition, Renal failure with patient is approximately three-fold higher than
dialysis, Paraplegia, Hermaphroditism, the general population.
Erythroderma, Leprosy, Sex hormones,
Some inherited breast cancers have been found
Spironolactone, Digoxin, Methyldopa,
to be associated with a gene called BRCA1 on
Amphetamine, Reserpine, Alcohol, Cimetidine,
chromosome 17. This gene is mutated in some
Dr. Khalid Yusuf El-Zohry – Sohag Teaching Hospital (01118391123) Page | 771
El-Zohry MRCP Questions Bank (Part 2) – Medical Masterclass 2010

families with early-onset breast cancer and it can only be accurately staged with a surgical
ovarian cancer. It is estimated that about 85% procedure, not only to remove the primary
of women with BRCA1 gene mutations will tumour, but also to biopsy multiple areas to
develop breast cancer in their lifetime. Other determine the true extent of disease.
genes have been identified that are associated Peritoneal washings are also taken. Accurate
with increased risk of breast and other cancers, staging is crucial as patients that have disease
such as: truly confined to one or both ovaries can avoid
 adjuvant chemotherapy.
BRCA2
 ataxia-telangiectasia mutation
[ Q: 68 ] MasterClass Part2
 p53. (2010) - Oncology
p53 mutations have been found in
A 55-year-old man presents with haematuria,
approximately 1% of breast cancers in women
loin swelling and discomfort. A CT scan shows a
under 40 years of age. Genetic testing is
large renal mass with tumour in the renal vein
available for women at high risk of breast
but not the inferior vena cava, and bilateral
cancer. However, such testing is controversial,
small lung metastases.
as problems associated with management of
patients with identified mutations, their What is the most appropriate treatment
insurability, and potential social conflicts are option?
anticipated.
A. Immunotherapy

[ Q: 67 ] MasterClass Part2 B. Radical nephrectomy and


medroxyprogesterone acetate
(2010) - Oncology
C. Chemotherapy
A 38-year-old woman presents with abdominal
distention and is found to have ovarian cancer. D. Renal artery embolisation and
immunotherapy
The method of staging ovarian cancer relies E. Radical nephrectomy and
primarily on:
immunotherapy.
A. CA-125
B. CT scanning Answer & Comments

C. Surgery Correct answer: E


D. Pelvic or transvaginal ultrasound
In good performance status patients with
E. Clinical examination limited metastatic disease, nephrectomy plus
immunotherapy offers longer survival than
immunotherapy alone.
Answer & Comments
[ Q: 69 ] MasterClass Part2
Correct answer: C
(2010) - Oncology
The staging of ovarian cancer is primarily based A previously fit 72-year-old woman presents
upon the surgical findings following an with a rapidly enlarging hard anterior neck
exploratory laparotomy. Ovarian cancer is often mass. Biopsy reveals anaplastic thyroid cancer.
confined to the peritoneal cavity and therefore

Dr. Khalid Yusuf El-Zohry – Sohag Teaching Hospital (01118391123) Page | 772
El-Zohry MRCP Questions Bank (Part 2) – Medical Masterclass 2010

Which of the following treatments are MOST [ Q: 71 ] MasterClass Part2


likely to be useful? (2010) - Oncology
A. Anthracycline containing chemotherapy
A 28-year-old man presents with testicular
B. Radioiodine swelling and undergoes an orchidectomy for a
C. High dose radiotherapy stage I testicular non-seminomatous germ-cell
tumour.
D. Low dose radiotherapy
E. Total thyroidectomy Which of the following is true regarding the use
of tumour markers in testicular tumours?

Answer & Comments A. More than 90% of classical seminomas


produce alpha-fetoprotein (AFP) or β-
Correct answer: C human chorionic gonadotropin (β-HCG)
B. More than 20% of non-seminomatous
High dose radiotherapy is required to retard the
germ-cell tumours produce no serum
growth of anaplastic thyroid cancer, but
tumour markers
prognosis remains very poor. Anthracyclines are
sometimes used but response rates are low. C. Both AFP and β-HCG should be measured
Surgery and radioiodine are the mainstay of to follow the progress of a tumour
management of well differentiated (Papillary D. Serum tumour markers should be within
and follicular) thyroid cancer. the normal range 7 days following
tumour resection if the resection has
[ Q: 70 ] MasterClass Part2 been complete
(2010) - Oncology E. β-HCG is identical to human luteinising
hormone and has limited usefulness as a
A 55-year-old man is found to have a low
marker
anterior rectal cancer on digital rectal
examination and proctoscopic biopsy.
Answer & Comments
Which investigation is most helpul to assess
Correct answer: C
operability?
A. MRI pelvis
βBoth alpha-fetoprotein (AFP) and beta-human
B. CT pelvis and abdomen chorionic gonadotropin (b-HCG) should be
measured to follow the progress of a tumour.
C. Barium enema
Pure seminomas usually produce no tumour
D. PET scan markers, but over 90% of patients with non-
E. Prostatic Ultrasound. seminomatous germ-cell tumours produce
either AFP or β-HCG. The normal serum
clearance half-life of HCG is 3 days and AFP 5
Answer & Comments
days, so these markers are present for some
Correct answer: A
time after tumour resection and if the
preoperative levels are very high it may take 30
A pelvic MRI is sensitive to tissue plane days or more to return to normal values.
breaches and prostatic invasion, which is Following treatment, unequal reductions of AFP
possible in men with low anterior rectal and β-HCG may occur. This suggests that they
cancers. are produced by heterogeneous clones within

Dr. Khalid Yusuf El-Zohry – Sohag Teaching Hospital (01118391123) Page | 773
El-Zohry MRCP Questions Bank (Part 2) – Medical Masterclass 2010

the tumour mass and therefore both markers practising homosexual and HIV negative. You
should be monitored in the follow-up of the identify a mass on rectal examination.
patient. HCG is similar to luteinising hormone, Following subsequent investigation you
except for the distinctive b-subunit, but it is diagnose anal carcinoma.
useful as a marker.
Which of the following statements, regarding
anal cancer, is TRUE?
[ Q: 72 ] MasterClass Part2
(2010) - Oncology A. It is a common tumour
B. It occurs more commonly occurs in men
A 49-year-old man presents with a lump in his
right breast. C. It commonly has an association with
human papillomavirus (HPV)
Which one of the following statements is true of
D. Surgery is the primary treatment
male breast cancer?
modality
A. It generally has a better prognosis than
E. With chemoradiation, the outcome in the
similar stage disease in women.
elderly (over 65 years) is as favourable as
B. It accounts for 1% of male malignancies. that of younger patients
C. It is treated in a similar fashion to breast
cancer in women. Answer & Comments
D. It can be predicted by the presence of Correct answer: C
BRCA-1 and BRCA-2 gene mutations.
E. It is a common cause of unilateral Anal cancer is uncommon, accounting for only
gynaecomastia in men. 4% of cancers of the lower gastrointestinal
tract. There is a slight female preponderance of
Answer & Comments 1.5 - 2.0 times, although this has been reported
as higher (up to 3 - 6 times) in some European
Correct answer: C treatment centres.
The annual incidence of anal cancer is approx 1
Male breast cancer is rare and there are per 100,000 in the heterosexual population,
approximately 200 cases per year in the UK. It this incidence is much higher in men who
accounts for 1% of all breast malignancies. It is practice anal-receptive sexual intercourse.
often detected late because most cases of Those who are HIV positive have twice the risk
gynaecomastia are benign. Stage for stage the of those who are not.
treatment is the same for men and women,
although men have a worse prognosis. The Epidemiological and molecular biology studies
presence of BRCA-1 and BRCA-2 gene have now shown, that sexually transmitted
mutations indicate a high risk for the infection with human papillomavirus (HPV) is
development of breast cancer, but neither of the most important aetiological factor. Some
these tests is useful for screening a population. subtypes, in particular type 16, are associated
with a high risk of malignant transformation.

[ Q: 73 ] MasterClass Part2 Rectal bleeding on defaecation occurs in approx


50% of patients with anal cancer and anal pain
(2010) - Oncology
occurs in many. Large tumours are likely to
A 49-year-old man complains of painful rectal interfere with sphincter control leading to
bleeding following defaecation. He is a faecal incontinence. A mass is almost always

Dr. Khalid Yusuf El-Zohry – Sohag Teaching Hospital (01118391123) Page | 774
El-Zohry MRCP Questions Bank (Part 2) – Medical Masterclass 2010

present clinically. Proctosigmoidoscopy should Which TWO cardiac valvular lesions are most
be performed and, in women, a thorough likely to occur in a patient with carcinoid
gynaecological examination performed. syndrome ?
Locoregional staging should include endoanal A. Mitral stenosis
ultrasonography, and either a CT or MRI scan of
the pelvis. B. Mitral regurgitation

Historically abdominoperineal resection of the C. Tricuspid stenosis


anal canal and distal rectum with the formation D. Tricuspid regurgitation
of a permanent end colostomy was the
E. Pulmonary stenosis
treatment of choice. This resulted in a 5-year
survival of approximately 50-70%. The F. Pulmonary regurgitation
involvement of pelvic lymphadenopathy
G. Aortic stenosis
reduces the 5-year survival to below 20%.
Radiotherapy alone is a reasonable alternative H. Aortic regurgitation
to surgery. I. Mitral stenosis with aortic regurgitation
Chemoradiation has been established as being J. Pulmonary stenosis with aortic
more effective than radiotherapy alone in regurgitation.
terms of local control. Studies have shown a
high ~81% pathological complete response rate
Answer & Comments
in patients undergoing surgery or biopsy, which
has led to a more conservative treatment Correct answer: DE
approach to anal cancer. Primary non-surgical
approach has the advantage of a reserved anal Cardiac manifestations occur in 11% to 66% of
sphincter with no obvious detriment to overall patients with carcinoid syndrome. The cardiac
survival in these patients. However, there disease is due to fibrosis involving the
remains uncertainty regarding the appropriate endocardium, primarily of the right side of the
radiotherapy dose, fractionation and heart, although left side lesions can also occur.
chemotherapy regimen. The fibrous deposits are diffuse and are found
Various prognostic factors such as gender, most commonly on the ventricular aspect of the
tumour stage, nodal status, and response to tricuspid valve and the associated chordae and
chemoradiation are of independent prognostic less commonly on the pulmonary valve cusps.
importance for local control and overall These fibrous deposits tend to cause
survival. Patients with well-differentiated constriction of both the tricuspid and
tumours have a more favourable outcome than pulmonary valves. At the pulmonary valve,
those with poorly differentiated tumours (75% stenosis is usually predominant, whereas at the
vs 24%, 5-year survival). Outcomes for elderly tricuspid valve, the constriction results in the
patients (>65 years) are also less favourable. valve being fixed open, and tricuspid
regurgitation is usually predominant.

[ Q: 74 ] MasterClass Part2 In two studies, 80% of the patients with cardiac


(2010) - Oncology lesions had evidence of heart failure. Lesions on
the left side occur in 30% of autopsy studies,
On auscultation of the heart, a patient of yours are less extensive, and most frequently occur
with known carcinoid syndrome is found to on the mitral valve.
have a murmur.

Dr. Khalid Yusuf El-Zohry – Sohag Teaching Hospital (01118391123) Page | 775
El-Zohry MRCP Questions Bank (Part 2) – Medical Masterclass 2010

[ Q: 75 ] MasterClass Part2 penicillin with antipseudomonal activity


(2010) - Oncology (gentamicin & piperacillin)
4. Metronidazole may be added if
A patient receiving combination chemotherapy
anaerobic infection is suspected
is admitted with neutropenia and a
temperature of 38.5 °C. The patient looks 5. Flucloxicillin or vancomycin or
dehydrated, with a regular low volume pulse, teicoplanin if Gram positive infection is
rate 100/minute. Their blood pressure is 90/60 suspected
mmHg, jugular venous pressure (JVP) not 6. Antibiotics should be adjusted
elevated, heart sounds are normal on according to culture results, though
auscultation. All other examination is these are often negative
unremarkable.
7. If there is no response after 36-48
Select the TWO treatments that are indicated hours review antibiotics with
from the following list: microbiologist advice and consider anti-
fungal cover
A. Intravenous hydration
8. Colony stimulating factors are not
B. Low dose dopamine infusion
routinely used for all patients with
C. G-CSF growth factor (granulocyte-colony neutropenia and guidelines for their
stimulating factor) use have been established
D. An aminoglycoside and broad spectrum 9. The use of prophylactic antibiotics is
penicillin with antipseudomonal activity being evaluated in a randomized clinical
E. Intravenous antifungal therapy trial

F. Intravenous gancyclovir 10. G-CSF is not currently indicated for


primary prophylaxis, only secondary.
G. Oral erythromycin
H. Rectal metronidazole [ Q: 76 ] MasterClass Part2
I. Antituberculous therapy (2010) - Oncology
J. Donor white cell transfusion. A 60-year-old man presents with urinary
retention and a short history of back pain. He
Answer & Comments has mild impairment of gait.

Correct answer: AD Which investigation would you initiate


urgently?
Therapy includes:
A. Chest radiograph
1. High dose IV antibiotics
B. CT scan of brain
2. Other supportive therapy (fluids,
C. Trans-rectal ultrasound and prostate
inotropes, ventilation, haemofiltration)
biopsy
3. First line empirical therapy is either
D. CT Scan of pelvis
monotherapy with an antipseudomonal
b-lactam (ceftazidime, cefotaxime or E. MRI spine.
meropenem) or a combination of an
aminoglycoside and a broad spectrum

Dr. Khalid Yusuf El-Zohry – Sohag Teaching Hospital (01118391123) Page | 776
El-Zohry MRCP Questions Bank (Part 2) – Medical Masterclass 2010

Answer & Comments H. Pain is the most common complaint in


osteosarcoma.
Correct answer: E
I. Osteoid osteomas require surgical excision
This is suspicious of early spinal cord followed by radiotherapy.
compression. This could be because of many J. Osteogenesis imperfecta may require a
causes including secondary cancer from a bone biopsy for diagnosis.
number of primary sites, including prostate.
Neurosurgery would be the first choice Answer & Comments
treatment in a patient with a solitary lesion
compressing the cord, particularly if there is no Correct answer: CH
prior diagnosis of cancer.
Osgood Schlatter’s disease generally affects
[ Q: 77 ] MasterClass Part2 children between the ages of 10-14 years old. It
is tibial tuberosity apophysitis which is a self-
(2010) - Oncology
limiting condition. It gives pain in front and
A 15-year-old boy presents to his general below the knee on strenuous activity. The tibial
practitioner having twisted his left knee playing tuberosity is prominent and tender, plain XR
football a month ago. He initially noted some shows tibial tuberosity enlargement +/-
knee swelling and pain. However, now he has a fragmantation. Symptom control is the
continuous dull ache in his knee and treatment, although in severe cases a plaster
occasionally it wakes him at night. He has cast may be necessary. Osteomyelitis is
difficulty bending and straightening his knee infection of the bony tissue. In
which has become increasingly swollen and he the cancer patient it may be caused by gram
walks with a limp. negative organisms (e.g. Pseudomonas,
Klebsiella, Salmonella), fungi (e.g. Candida), or
Which of the following statements are
the more common gram positive bacterial
applicable to this patient?
pathogens. Patients with local skeletal defects
A. Osgood-Schlatter’s disease generally or who have undergone extensive surgery, and
affects children between the ages of 10- patients who have bacteraemia or fungaemia
14. are considered to be at high risk.
B. Osteomyelitis is unlikely in the absence of Osteosarcoma generally occurs in childhood
a fever. and adolescence. Bones of the knee joint and
C. Osteosarcoma affecting the bones of the the proximal humerus are the most common
knee joint and the proximal humerus are sites. Pain is the most common complaint.
the most common sites in childhood. Osteoid osteoma are smaller than 1cm, painful,
D. Osteoid osteomas are usually painless. bone-forming tumours that are always benign
and are treated by simple excision.
E. Osteogenesis imperfecta is unlikely to
present before puberty. Osteogenesis imperfecta is inherited as an
autosomal dominant condition where
F. Osgood-Schlatter’s disease is managed individuals may have blue sclera, deafness and
with symptom control, although in severe multiple bone fractures.
cases a plaster cast may be necessary.
G. Osteomyelitis should be considered and [ Q: 78 ] MasterClass Part2
treated with empirical oral antibiotics.

Dr. Khalid Yusuf El-Zohry – Sohag Teaching Hospital (01118391123) Page | 777
El-Zohry MRCP Questions Bank (Part 2) – Medical Masterclass 2010

(2010) - Oncology such cases radical neck dissection followed by


extended field radiotherapy that includes these
A 35-year-old woman presents with weight loss possible primary sites may yield five-year
and a mass of lymph nodes at the base of her survival rates of 30%. However, lower cervical
neck. Histology of the nodes revealed a poorly lymphadenopathy – as in this case – has a much
differentiated squamous cell carcinoma. Aside worse prognosis and should not be treated in
from the nodes, examination of the head and this way.
neck is normal, as is specialist ENT examination
and CT scan of the head and neck.
[ Q: 79 ] MasterClass Part2
What are the TWO most likely sites for the (2010) - Oncology
primary?
A 45-year-old woman has recently been
A. Breast diagnosed with malignant mesothelioma.
B. Oesophageal
Which of the following are recognised as poor
C. Colon prognostic factors in mesothelioma?
D. Ovary A. Poor performance status
E. Kidney B. Female gender
F. Skin C. Non-epithelioid histology
G. Anus D. Polycythaemia
H. Cervical E. Raised serum creatinine
I. Lung F. Jaundice
J. Liver. G. Raised blood urea
H. History of asbestos exposure
Answer & Comments
I. History of cigarette smoking
Correct answer: HI
J. Chronic obstructive pulmonary disease

Meticulous examination of the scalp and skin of


Answer & Comments
the head and neck for a primary tumour,
together with thorough ENT examination, are Correct answer: AC
essential, but if these do not reveal a primary
tumour, then cervical cancer or lung cancer are For malignant mesothelioma, the prognostic
the most likely sites for a primary. Investigation scoring systems of the Cancer and Leukaemia
should include colposcopy and Papanicolou Group B (CALGB) and European Organization
smear test and chest radiography. If these do for Research and Treatment of Cancer (EORTC)
not reveal a primary, then barium swallow or are the most useful. Poor prognosis is
upper gastrointestinal endoscopy with
predicted by non-epithelioid histology, male
oesophageal biopsies would be appropriate to
gender, advanced stage disease, poor
look for oesophageal carcinoma.
performance status, leucocytosis and
Patients presenting with high cervical thrombocytosis.
lymphadenopathy containing squamous cell
carcinoma may have occult tumours of the [ Q: 80 ] MasterClass Part2
nasopharynx, oropharynx or hypopharynx. In

Dr. Khalid Yusuf El-Zohry – Sohag Teaching Hospital (01118391123) Page | 778
El-Zohry MRCP Questions Bank (Part 2) – Medical Masterclass 2010

(2010) - Oncology The differential diagnosis is Paget's disease with


osteosarcoma and metastatic prostate cancer.
A 75-year-old retired miner presents with a 3- Although transrectal ultrasound-guided sextant
month history of a painful right upper arm with biopsy and serum prostate-specific antigen may
some swelling in the mid arm. His past medical indicate prostate cancer, this is sufficiently
history includes acute inferior myocardial common that it does not exclude the presence
infarction in 1993 and transurethral resection of both diagnoses. Hence a biopsy is more likely
of the prostate in 1996 for bladder outflow to establish the diagnosis of the arm lesion
obstruction. His current medication is aspirin, definitively. Overall about 1% of patients with
nifedepine and a salbutamol inhaler. He is a Paget's disease will develop a primary bone
lifelong heavy smoker and his GP has diagnosed sarcoma: 85% are osteosarcoma and 15%
chronic obstructive airways disease. fibrosarcoma. The most common sites are
On examination he is a well-built man with a pelvis, femur and humerus. The radiological
prominent forehead and jaw. He has a hearing appearances are of osteolytic lesions with
aid on his left ear and is apyrexial. His pulse rate cortical destruction, but periosteal elevation is
is 80 beats/minute in sinus rhythm and his BP is rare. Patients present with localised pain and
110/60 mmHg. JVP is raised at 4 cm above the most are dead within 2 years.
angle of Louis and the apex beat is displaced to
the mid-axillary line. Heart sounds are normal
and there are no murmurs. There is mild
bilateral ankle oedema and basal crepitations.
Abdominal examination is unremarkable apart
from an enlarged irregular prostate gland. His
right mid-humerus is swollen and tender. Initial
tests show the following: FBC, urea and
electrolytes, calcium, albumin and phosphate
are normal; alkaline phosphatase and acid
phosphatase are increased; bone scan is
diffusely hot but cold over right mid-humerus;
and a radiograph of his right humerus shows
lytic lesion with cortical bone destruction, but
no periosteal reaction.

Which of these investigations is most likely to


establish the diagnosis?
A. CT scan of his arm
B. Whole-body PET scan
C. Transurethral biopsy of his prostate gland
D. Serum prostate-specific antigen
E. Bone biopsy of arm lesion

Answer & Comments

Correct answer: E

Dr. Khalid Yusuf El-Zohry – Sohag Teaching Hospital (01118391123) Page | 779
El-Zohry MRCP Questions Bank (Part 2) – Medical Masterclass 2010

the paracetamol 1g qds does not provide relief.

Pharmacology Medication should be given by mouth unless


the patient is unable to take oral medication.

(120 Questions) Consider the aetiology of the pain - in this case


possible bone metastases - but do not race to
(Medical Masterclass – Part 2) conclusions and perform radiograph before
suggesting radiotherapy.
Reference: See Medical Masterclass, Clinical Skills
module: Pain Relief and Palliative Care, p33 and
[ Q: 1 ] MasterClass Part2 36, for further discussion.
(2010) - Pharmacology
[ Q: 2 ] MasterClass Part2
An 82-year-old man with carcinoma of the
prostate presents with left hip pain for which (2010) - Pharmacology
he has been taking 1g paracetamol bd. He is A 48-year-old man presents with 40 minutes of
otherwise well with no history of note. Recent cardiac chest pain.
blood tests were normal.
Which TWO of the following ECG criteria are
What will be your first TWO actions? indications for thrombolysis?
A. Start a diamorphine syringe driver A. 2 mm or more elevation of the ST
B. Start 5mg oramorph prn segment in any one standard ECG lead

C. Add a nonsteroidal anti-inflammatory B. Atrial fibrillation


drug (NSAID) C. 2 mm or more ST segment depression in
D. Put on a 25mcg/hour fentanyl patch two or more standard ECG leads

E. Increase the dose of paracetamol to 1g D. 2 mm or more elevation of ST segments


qds in two or more contiguous praecordial
leads
F. Refer him for radiotherapy
E. New right bundle branch block
G. Reassure him that some hip pain is to be
expected in patients with prostatic cancer F. Ventricular tachycardia

H. Commence low dose MST bd G. 1 mm or more elevation of ST segments


in two or more standard ECG leads
I. Radiograph lower back and pelvis
H. 2 mm or more elevation of ST segments
J. Prescribe co-codamol prn.
in any two ECG leads
I. 1 mm or more elevation of ST segments in
Answer & Comments
any two ECG leads
Correct answer: EI
J. 1 mm or more elevation of ST segments in
two or more contiguous praecordial
Continuous pain requires continuous analgesia: leads.
start at the bottom of the analgesic ladder,
optimise doses and work gradually up.
Answer & Comments
Consider co analgesics - NSAIDs are useful for
Correct answer: DG
bone pain, and should be started in this man if

Dr. Khalid Yusuf El-Zohry – Sohag Teaching Hospital (01118391123) Page | 781
El-Zohry MRCP Questions Bank (Part 2) – Medical Masterclass 2010

The other recognised ECG criterion for [ Q: 4 ] MasterClass Part2


thrombolysis is new left bundle branch block. (2010) - Pharmacology
A 70-year-old man with atrial fibrillation and
[ Q: 3 ] MasterClass Part2
mitral valve disease has been stable on warfarin
(2010) - Pharmacology for many years. He is admitted with a severe
You see a patient in the outpatient clinic with nose bleed and his INR is found to be grossly
multiple myeloma whose recent blood tests elevated at 15 (therapeutic range 2-3).
have shown a serum potassium of 5.7mmol. His
Recent prescription of which of the following
recent drug history includes the use of
drugs could explain this?
melphalan, interferon, and pamidronate, and
he is clinically well, with no ECG changes. A. Carbamazepine
B. Ciprofloxacin
Which one of the following oral agents would
you prescribe for treatment of hyperkalaemia in C. Griseofulvin
this patient?
D. Phenytoin
A. Calcium carbonate
E. Rifampicin
B. Calcium polystyrene sulphonate
(Resonium) Answer & Comments
C. Sodium polystyrene sulphonate
Correct answer: B
(Resonium)
D. Bendrofluazide Warfarin is metabolized by the cytochrome
E. Salbutamol. P450 enzyme system. Ciprofloxacin inhibits
CYP450 and hence potentiates the action of
warfarin, which could explain this presentation
Answer & Comments
with bleeding and high INR. The other drugs
Correct answer: C listed all induce CYP450 enzymes and hence
would decrease the effect of warfarin.
Ion-exchange resins may be used to remove
excess potassium in mild to moderate [ Q: 5 ] MasterClass Part2
hyperkalaemia. They can either be calcium or (2010) - Pharmacology
sodium based, with the site of ion exchange
taking place mainly in the large intestine, thus A 28 yr old woman who is 34 weeks pregnant
allowing either oral or rectal administration. has just been diagnosed with thyrotoxicosis.
Calcium based resins may provoke She is planning to breast-feed her baby after
hypercalcaemia in patients with multiple delivery.
myeloma, or metastatic cancer and are best
avoided in such instances (especially as this Which treatment would you recommend for
patient has recently received pamidronate, her?
which suggests that hypercalcaemia has been a A. Carbimazole - titrated to minimum
problem). Salbutamol can be given by nebuliser effective dose
for more severe cases of hyperkalaemia,
B. Carbimazole - dose sufficienct to block
especially in patients who are fluid overloaded.
thyroid, with added thyroxine
C. Potassium perchlorate

Dr. Khalid Yusuf El-Zohry – Sohag Teaching Hospital (01118391123) Page | 782
El-Zohry MRCP Questions Bank (Part 2) – Medical Masterclass 2010

D. Propylthiouracil include breast cancer, liver disease, testicular


tumours and hyperthyroidism.
E. Lugol’s iodine.
Other drugs that can cause gynaecomastia
Answer & Comments include oestrogens, spironolactone, cimetidine,
verapamil and nifedipine. The gynaecomastia
Correct answer: D usually improves on stopping the drug or
reducing the dose.
Anti-thyroid drugs can cross the placenta and
breast milk, thus causing hypothyroidism in the [ Q: 7 ] MasterClass Part2
child. The carbimazole-blocking regimen is
(2010) - Pharmacology
worse in this respect, as the carbimazole
crosses the placenta, but the thyroxine does You are walking onto a ward when you notice
not. Potassium perchlorate is no longer used in that a 78-year-old man has collapsed in his bed.
the UK; Lugol’s iodine may occasionally be
prescribed for patients undergoing thyroid What should your first TWO actions be?
surgery, but causes goitre in infants. A. Call the cardiac arrest team
Propythiouracil is more highly protein bound
and is ionized at pH 7.4, thus making it less B. Defibrillate (200 J)
likely to cross the placenta or breast milk. C. Check that the situation is safe
D. Check responsiveness (shake and shout)
[ Q: 6 ] MasterClass Part2
E. Check breathing (observe for movement
(2010) - Pharmacology of the chest)
A 65-year-old man attending the cardiology F. Open airway (head tilt / chin lift)
clinic complains of swelling and tenderness of
his breasts. You diagnose probable G. Start chest compression
gynaecomastia. H. Check for pulse (carotid)

Which of the following drugs is most likely to be I. Give precordial thump


the cause? J. Check for pulse (femoral).
A. Simvastatin
Answer & Comments
B. Amiodarone
C. Digoxin Correct answer: CD

D. Aspirin
Adult basic life support involves:
E. Ramipril.
1. Check that the situation is safe

Answer & Comments 2. Check responsiveness: shake and shout


3. Open airway: head tilt / chin lift
Correct answer: C
4. Check breathing: look / listen
Digoxin is the most likely cause of his 5. If breathing: recovery position
gynaecomastia. This side effect is more
6. If not breathing: assess circulation: 10
common with longer-term use, and the
seconds only
problem may be unilateral or bilateral.
Important differential diagnoses to consider
Dr. Khalid Yusuf El-Zohry – Sohag Teaching Hospital (01118391123) Page | 783
El-Zohry MRCP Questions Bank (Part 2) – Medical Masterclass 2010

7. Circulation present: continue rescue [ Q: 9 ] MasterClass Part2


breathing (2010) - Pharmacology
8. Circulation not present: compress
A 43-year-old woman with pulmonary
chest: 100 / min, 15:2 ratio.
hypertension attends clinic and asks to be
prescribed bosentan.
[ Q: 8 ] MasterClass Part2
(2010) - Pharmacology Which of the following is true?
A. It is licensed for use in all patients with
A 38-year-old asthmatic woman presents with
pulmonary hypertension.
an acute attack. Her arterial blood gases
breathing air are as follows: pH 7.36, pO2 9.8 B. It inhibits the effects of endothelin-2 (ET-
kPa, pCO2 5.2 kPa. 2).
C. It binds to both endothelin A (ET-A) and
These mean:
endothelin B (ET-B) receptors.
A. the attack is not severe
D. It is excreted primarily unchanged in the
B. she should be given supplemental urine.
oxygen, but is unlikely to need a high FiO2
E. It is not effective in patients with
to achieve normoxia
scleroderma.
C. cardiorespiratory arrest could be
imminent
Answer & Comments
D. her respiratory effort may be failing
Correct answer: C
because she is getting tired
E. she could have had a pneumothorax.
Bosentan is an antagonist of endothelin-1
binding to ET-A and ET-B receptors. It is
Answer & Comments licensed for the treatment of 'pulmonary
arterial hypertension (PAH) to improve exercise
Correct answer: D
capacity and symptoms in patients with grade
III functional status'. It has been shown to be
A normal or elevated pCO2 in an asthmatic effective in patients with primary PAH and in
indicates failing respiratory effort, and although those with PAH secondary to scleroderma. It is
this woman's oxygen saturation is not severely
excreted in bile following metabolism by the
depressed she is in danger of decompensation
cytochrome P450 enzymes and this is a
and - aside from high flow oxygen, nebulised potential source of interaction with drugs
salbutamol and ipatropium, and steroids - it metabolised by the same isoenzyme (e.g.
would be prudent to inform the ICU of her glibenclamide, ritonavir, ketoconazole,
existence. The gases are not bad enough, ciclosporin and itraconazole).
however, to suggest that cardiorespiratory
arrest is imminent.
[ Q: 10 ] MasterClass Part2
Pneumothorax must be excluded in any
(2010) - Pharmacology
asthmatic, but the presence or absence of
pneumothorax can never be inferred from A 21-year-old university student complains of
arterial blood gas analysis. difficulty sleeping. She is in the middle of sitting
her final exams and would like some
medication for a few days to help her sleep.

Dr. Khalid Yusuf El-Zohry – Sohag Teaching Hospital (01118391123) Page | 784
El-Zohry MRCP Questions Bank (Part 2) – Medical Masterclass 2010

However, she is concerned about potential E. Digoxin 0.25 mg orally three times daily
'hang-over' effects and would prefer a drug for one week, then twice daily for one
which doesn't cause daytime drowsiness. week, then once daily thereafter.

Which agent would you prescribe?


Answer & Comments
A. Diazepam
Correct answer: B
B. Midazolam
C. Promethazine The options for treatment of atrial fibrillation
are:
D. Loprazolam
1. DC cardioversion if the patient is
E. Clomethiazole.
compromised haemodynamically or has
ischaemic cardiac pain
Answer & Comments
2. Digoxin - 1.0-1.5 mg orally in divided
Correct answer: D doses over 24 hours, but can be given
intavenously in emergency (0.25-0.5 mg
Diazepam has a long half-life, principally over 10-20 min, repeated after four to
because of its active metabolites. Midazolam is eight hours to total intravenous loading
short-acting but is only used intravenously. dose of 0.5-1.0 mg)
Promethazine is an antihistamine with a 12-
3. 'Medical cardioversion' with
hour half-life and may cause daytime sedation.
amiodarone or flecainide
Clomethiazole is less safe in overdose, has
dependence potential and is only licensed for In this clinical context it is likely that the atrial
sedation in the elderly. Loprazolam is short- fibrillation (if new) will revert to sinus rhythm as
acting (half-life 6–12 hours) and would be a the woman recovers from her pneumonia and
reasonable choice. most physicians would digitalize in preference
to the other options described.

[ Q: 11 ] MasterClass Part2
(2010) - Pharmacology [ Q: 12 ] MasterClass Part2
(2010) - Pharmacology
A 74-year-old woman presents with
breathlessness. She is a small woman (55 kg) A 50-year-old man with a history of alcohol
with a chest infection. She is not very unwell, abuse wishes to discuss the use of disulfiram as
but is in atrial fibrillation at a rate of 170/min. an aid to helping him to stop drinking.
Her electrolytes are normal (K 4.2 mmol/l).
Which one of the following statements
As well as treating her pneumonia, you decide regarding disulfiram is true?
to digitalize by prescribing: A. An alcohol-disulfiram reaction may occur
A. Digoxin 0.25 mg orally once daily from 2 hours after the first dose.

B. Digoxin 1.0 mg orally over 24 hours in B. It is safe to drink alcohol while taking
divided doses disulfiram.

C. Digoxin 1.0 mg intravenously over 20 min C. It is safe to drink alcohol from 2 days after
the last tablet of disulfiram is taken.
D. Digoxin 0.125 mg orally once daily
D. Disulfiram is an irreversible inhibitor of
alcohol dehydrogenase.

Dr. Khalid Yusuf El-Zohry – Sohag Teaching Hospital (01118391123) Page | 785
El-Zohry MRCP Questions Bank (Part 2) – Medical Masterclass 2010

E. Disulfiram has activity against scabies. E. It has little effect on fasting blood
glucose.
Answer & Comments
Answer & Comments
Correct answer: E
Correct answer: C
Disulfiram may be used as an aid to stopping
alcohol abuse. It is an irreversible inhibitor of Insulin glargine is a long-acting insulin analogue,
aldehyde dehydrogenase, therefore if alcohol is produced by modifying the chemical structure
ingested, aldehyde accumulates causing of insulin. This gives it a smooth, prolonged
unpleasant reactions including vomiting, absorption profile with no peaks. As such, it is a
palpitations and breathlessness. There are case long-acting agent, suitable for providing a basal
reports of fatalities with alcohol-disulfiram level of insulin which attempts to mimic the
interactions, therefore it should not be given normal physiological state. Its smooth profile
unless the patient is fully aware of the risks and reduces the risk of hypoglycaemia, and when
has agreed not to drink alcohol. However, the given at night, provides good control of the
extent of the reaction is highly variable in fasting blood glucose. Unlike crystalline
different patients, and some patients may not suspensions, insulin glargine does not need to
experience any symptoms. The reaction with be mixed thoroughly prior to injection, thus
alcohol only occurs at least 12 hours after the making it easier to use.
start of disulfiram therapy and may occur up to
10 days after stopping disulfiram therapy. [ Q: 14 ] MasterClass Part2
Disulfiram is active against scabies, although
(2010) - Pharmacology
other treatments are usually preferred.
A 48-year-old woman with advanced breast
[ Q: 13 ] MasterClass Part2 cancer complains of having a dry mouth.
(2010) - Pharmacology Which TWO of the following drugs she is taking
A 24-year-old Type I diabetic is currently on a are most likely to be the cause?
basal-bolus regime, comprising twice a day A. Haloperidol
basal Isophane insulin, complemented by short-
B. Amiodarone
acting insulin at meal times. He has recently
heard about insulin glargine, and wondered if it C. Morphine
would be suitable for him.
D. Paracetamol
Which statement concerning insulin glargine is E. Codanthramer
true?
F. Cyclizine
A. It is formulated by adding zinc suspension
G. Fluconazole
to insulin
H. Dexamethasone
B. It is rapid-acting and should be injected
just before meals I. Ketamine

C. It is particularly useful for patients J. Demeclocycline


troubled by hypoglycaemic episodes
D. It needs to be mixed thoroughly before Answer & Comments
injecting Correct answer: CF

Dr. Khalid Yusuf El-Zohry – Sohag Teaching Hospital (01118391123) Page | 786
El-Zohry MRCP Questions Bank (Part 2) – Medical Masterclass 2010

Morphine and cyclizine both have  griseofulvin


antimuscarinic properties that can cause a dry  rifampicin.
mouth.
Additionally 'non-drug' enzyme inducers include
Dexamethasone treatment can indirectly result
in oral candida that patients say causes altered  brussels sprouts
taste and dryness.  chronic ethanol use
Ketamine can result in hypersalivation and  tobacco smoke.
when used in higher doses is sometimes
accompanied by an antimuscarinic drug. The use of over-the-counter medication, such
as St. John's Wort, can also lead to clinically
significant enzyme induction and drug
[ Q: 15 ] MasterClass Part2 interactions.
(2010) - Pharmacology
Anticoagulation of a 75-year-old man with a [ Q: 16 ] MasterClass Part2
deep vein thrombosis is proving to be difficult. (2010) - Pharmacology

Which of the following would not explain his A man is brought to the Emergency Department
resistance to warfarin? by ambulance. He is unconscious (GCS 5), with
pin-point pupils and a slow respiratory rate.
A. Brussels sprouts
B. Griseofulvin Immediate specific treatment should be:

C. Chronic alcohol use A. Naloxone (0.4 mg) intravenously,


repeated if no effect
D. Erythromycin
B. N-acetyl cysteine (150 mg/kg over 15
E. Phenytoin
min) intravenously, then 50 mg/kg over 4
F. Carbamazepine hours, then 100 mg/kg over 16 hours
G. Phenobarbitone C. Dextrose (50 ml of 50% solution)
intravenously, repeated if no effect
H. Sodium valproate
D. Flumazenil (0.2 mg) intravenously,
I. St John's Wort
repeated if no effect
J. Cigarette smoking.
E. Insert stomach tube (after securing
airway) and give activated charcoal.
Answer & Comments

Correct answer: DH Answer & Comments

Correct answer: A
Anticoagulation with warfarin can be difficult
where there is induction or inhibition of the
The working diagnosis must be opioid overdose,
cytochrome p450 enzyme system. Drugs
the treatment for which is intravenous
causing enzyme induction, and therefore a
naloxone (0.4 mg), repeated up to a total dose
decreased anticoagulant effect, include
of 2 mg depending on clinical response.
 barbiturates
The half-life of naloxone is shorter than that of
 carbamazepine opioids, hence if this man wakes up it can be
 phenytoin

Dr. Khalid Yusuf El-Zohry – Sohag Teaching Hospital (01118391123) Page | 787
El-Zohry MRCP Questions Bank (Part 2) – Medical Masterclass 2010

anticipated that he will 're-narcose'. A naloxone C. Acne


infusion may be necessary.
D. Aplastic anaemia
E. Elevated liver function tests
[ Q: 17 ] MasterClass Part2
(2010) - Pharmacology F. Liver tumour
G. Hypertriglyceridaemia
A 73-year-old man presents to the Emergency
Department with drowsiness and confusion. He H. Osteoporosis
is tachycardic and tachypnoeic, but not I. Testicular atrophy
cyanosed. His pulse oximeter reading is 90% on
room air. Apparently his wife had been J. Insomnia.
admitted with similar symptoms earlier in the
week. Answer & Comments

Which one of the following is most likely? Correct answer: DH

A. Paracetamol overdose
Anabolic steroids are often used (illegally) by
B. Salicylate overdose body-builders and people attending gyms
C. Carbon monoxide poisoning regularly. They have a number of important
adverse effects including hypertension, ankle
D. Cerebrovascular accident oedema, acne, abnormal LFTs,
E. Pneumonia. hypertriglyceridaemia, testicular atrophy and
insomnia. Liver tumours have also been
Answer & Comments reported in association with the use of anabolic
steroids.
Correct answer: C
They have been used clinically to treat aplastic
anaemia and osteoporosis.
This man has carbon monoxide (CO) poisoning.
Pulse oximeters cannot distinguish between [ Q: 19 ] MasterClass Part2
COHb and HbO2, therefore it is essential to take
(2010) - Pharmacology
arterial blood gases and – to make the specific
diagnosis – measure the level of CO. You have decided to start a syringe driver on a
dying patient whose symptoms were previously
It is important to think about prevention: CO
well controlled on oxycodone SR (OxyContin)
alarms are cheap and readily available.
80mg bd.

[ Q: 18 ] MasterClass Part2 What dose of diamorphine should you chose for


(2010) - Pharmacology your 24-hour syringe driver?

A 19-year-old man admits to having used A. 30mg


anabolic steroids regularly at the gym. B. 60mg

Which TWO of the following are NOT well C. 90mg


recognised adverse effects of anabolic steroids? D. 110mg
A. Hypertension E. 130mg.
B. Ankle oedema

Dr. Khalid Yusuf El-Zohry – Sohag Teaching Hospital (01118391123) Page | 788
El-Zohry MRCP Questions Bank (Part 2) – Medical Masterclass 2010

Answer & Comments data. The principles of the calculation are as


follows: predict the pH that would arise in
Correct answer: D
normal blood in the presence of the pCO2
actually measured; then calculate the amount
160mg oxycodone over 24 hours is equivalent of acid or base that would have to be added to
to 320mg morphine which is equivalent to the blood to change the calculated pH into the
approx 106mg diamorphine. pH as actually measured. This value is the base
deficit or excess, in mmol/l, which quantifies
[ Q: 20 ] MasterClass Part2 the metabolic component of acid-base
(2010) - Pharmacology disturbance.
Renal failure causes metabolic acidosis with
A 38-year-old man presents with acute renal
compensatory respiratory alkalosis. In this man
failure, with serum creatinine 988 micromol/l. A
the predicted pH (on the basis of measured
house physician performs arterial blood gas
pCO2 3.2 kPa) would be alkalotic, and acid
analysis (breathing air) and finds pH 7.12, pO2
would have to be added to the blood to change
12.8 kPa, pCO2 3.2 kPa, BE -12 mmol/l. He asks
this to the acidotic value actually measured (pH
you what a base excess of –12 means.
7.12). This is expressed as a NEGATIVE base
You reply: excess.

A. It means that the serum bicarbonate


concentration is 12 mmol/l. [ Q: 21 ] MasterClass Part2
(2010) - Pharmacology
B. It means that the serum bicarbonate
concentration is 12 mmol/l below normal. (2) An 18-year-old girl is brought to the
Emergency Department after having been
C. It means that the pH is 0.12 units below
found unconscious in her room by her mother,
the lower limit of the normal range for
whom she lives with. According to her mother
the machine being used.
(who is being treated with tablets for
D. An algorithm is used to predict what pH depression), the girl has no past medical history
would arise in normal blood in the of note and is on no medication. On
presence of the pCO2 actually measured, examination the girl is breathing spontaneously
the base excess being the amount of acid with respiratory rate 20/min; her pulse is
that would have to be added or removed 110/min, BP120/70 mmHg, with peripheries
to obtain the pH actually measured. warm and dilated. Her pupils are mid-dilated
E. An algorithm is used to predict what pH and reacting sluggishly, the Glasgow Coma
would arise in normal blood in the score is 6/15, the ocular fundi are normal, and
presence of the pCO2 actually measured, the reflexes are generally brisk with bilateral
the base excess being the amount of base extensor plantar responses.
that would have to be added or removed
What are the TWO most likely diagnoses?
to obtain the pH actually measured.
A. Benzodiazepine overdose
Answer & Comments B. Massive intracranial bleed
Correct answer: E C. Factitious coma
D. Tricyclic antidepressant overdose
The base excess is a figure calculated by many
blood gas machines to aid interpretation of E. Hypoglycaemia

Dr. Khalid Yusuf El-Zohry – Sohag Teaching Hospital (01118391123) Page | 789
El-Zohry MRCP Questions Bank (Part 2) – Medical Masterclass 2010

F. Post-ictal coma and 500ml 0.9% saline administered IV over 30


minutes.
G. Transient ischaemic attack
H. Alcohol intoxication Which of the following is the correct ongoing
management?
I. Addisonian crisis
A. IV chlorpheniramine and restart NAC
J. Severe hypothyroidism.
infusion at lowest rate once symptoms
resolved
Answer & Comments
B. IV chlorpromazine and restart NAC
Correct answer: DF infusion at lowest rate once symptoms
resolved
The combination of tachycardia, reduced C. IV chlorpheniramine and give 2.5g of oral
conscious level and bilateral extensor plantar methionine
response should always raise suspicions of
tricyclic antidepressant overdose. The D. IV chlorpromazine and give 2.5g of oral
tachypnoea in this case may reflect respiratory methionine
compensation for a metabolic acidosis. E. Withhold treatment and recheck
This patient may have had access to her paracetamol level at 12 hours
mother’s supply of antidepressant medication:
60% of patients presenting with overdose have Answer & Comments
taken a medication prescribed to themselves or
Correct answer: A
a member of their household, so a careful drug
history for all members of the household is
essential. Reactions to NAC are well recognized and are
not related to hypersensitivity. NAC can almost
The post-ictal period after a first fit is variable in always be safely restarted and total dose safely
duration, and may be associated with a variety
administered after symptomatic treatment.
of neurological signs, including pupillary
Oral methionine may be an alternative but is
abnormalities and extensor plantar responses. definitely second line. IV chlorpromazine is now
Hypoglycaemia should always be considered in never given and would make hypotension
unexplained coma and an immediate worse. Withholding treatment and waiting
estimation of blood glucose by bedside stick more than 12 hours would expose patient to
test must be made, but this clinical picture risk of liver failure.
would be more in keeping with tricyclic
overdose or a post-ictal presentation. [ Q: 23 ] MasterClass Part2
(2010) - Pharmacology
[ Q: 22 ] MasterClass Part2
A 24-year-old woman who is 12 weeks'
(2010) - Pharmacology
pregnant presents with cellulitis of her leg. She
A 35-year-woman presents 6 hours after a has no known drug allergies.
deliberate overdose of paracetamol. The
paracetamol level is above the treatment line. Which one of the following antibiotics should
Thirty minutes after starting an infusion of N- definitely not be given?
acetyl cysteine (NAC) she becomes flushed and A. Flucloxacillin
hypotensive with a blood pressure of 80/55
B. Ceftriaxone
mmHg. The infusion is stopped immediately

Dr. Khalid Yusuf El-Zohry – Sohag Teaching Hospital (01118391123) Page | 790
El-Zohry MRCP Questions Bank (Part 2) – Medical Masterclass 2010

C. Co-trimoxazole Temporary cardiac pacing may be necessary in


patients unresponsive to drug therapy.
D. Benzylpenicillin
Paracetamol concentrations should be
E. Erythromycin measured as these may also have been taken at
the time of the overdose and may require
Answer & Comments specific additional treatment. There is no
evidence that salicylate concentrations should
Correct answer: C
be measured unless there is a history of
ingestions, signs of salicylate toxicity, in
Co-trimoxazole is the combination of patients with reduced Glasgow coma scale
sulfamethoxazole and trimethoprim. (GCS) and patients unable to provide a reliable
Trimethoprim is a folate antagonist and is history.
teratogenic so should not be used in pregnancy.
The other drugs listed are generally considered
[ Q: 25 ] MasterClass Part2
safe for use in pregnancy.
(2010) - Pharmacology
[ Q: 24 ] MasterClass Part2 A 20 year old man is brought into Accident and
(2010) - Pharmacology Emergency, apparently intoxicated and
hyperventilating. A friend gives a history that he
An 18-year-old girl is admitted having may have drunk some antifreeze (ethylene
deliberately taken a large overdose of her glycol).
father's atenolol tablets.
Which of the following is NOT true of ethylene
Which of the following statements is false? glycol poisoning?
A. Dizziness is a common feature of beta- A. A common feature is a high anion gap
blocker overdose. metabolic acidosis
B. A temporary pacing wire may be B. It may result in calcium oxalate crystal
required. deposition in body tissues
C. Atropine should not be given due to the C. The diagnosis is confirmed by finding
risk of causing tachycardia. elevated blood ethanol levels
D. Glucagon is a useful therapy in the D. It may be treated with oral ethanol
management of beta-blocker overdose.
E. It may be treated with intravenous
E. Paracetamol levels should be measured. ethanol infusion

Answer & Comments Answer & Comments


Correct answer: C Correct answer: C

Beta blocker overdose may cause dizziness, Ethylene glycol is present in antifreeze and
hypotension, syncope and heart failure. some household cleaning products. Accidental
Bradycardia is a common feature of significant or deliberate ingestion is not infrequent and
overdose and should be treated by the relatively small amounts, eg. 100 ml in an adult,
administration of atropine. Intravenous are toxic. Presentation is with features of CNS
glucagon may also be given, particularly in depression similar to alcohol intoxication, but
patients with haemodynamic compromise. with ethanol levels too low to account for this.

Dr. Khalid Yusuf El-Zohry – Sohag Teaching Hospital (01118391123) Page | 791
El-Zohry MRCP Questions Bank (Part 2) – Medical Masterclass 2010

A high anion gap metabolic acidosis, raised peripheral vascular disease who have elevated
serum osmolality and elevated osmolar gap are cholesterol levels, but there is no data on
typical. Calcium oxalate crystals may form and improvements in walking distance.
be excreted in the urine and deposited in
tissues. [ Q: 27 ] MasterClass Part2
The diagnosis can be confirmed by measuring (2010) - Pharmacology
serum ethylene glycol levels, but if ingestion is
likely, then treatment should be commenced A 5-year-old girl presents having ingested some
without waiting for results of levels. There is of her mother's ferrous sulphate tablets.
little correlation between blood levels and
Which of the following is true?
severity of poisoning, hence treatment should
be given even if toxic levels are not A. Whole bowel irrigation is appropriate in
demonstrated. Treatment options include all cases.
administration of oral or intravenous ethanol, B. Gastrointestinal toxicity occurs 24 hours
fomepizole, bicarbonate or haemodialysis. post ingestion.
Renal failure or death may occur in severe
cases, particularly if diagnosis and treatment C. Serial iron concentrations should be
are delayed. measured after desferrioxamine has been
given.

[ Q: 26 ] MasterClass Part2 D. Activated charcoal should be given if


presenting less than 1 hour after
(2010) - Pharmacology
ingestion.
You are treating a 72-year-old man with
E. Desferrioxamine can be given i.m.
moderate peripheral vascular disease. He
exercises regularly but finds that his walking
distance is diminishing due to pain. Answer & Comments

Correct answer: E
Which drug might help improve pain-free
walking distance?
Iron tablets are radio-opaque on abdominal
A. Naftidrofuryl radiographs until they have dissolved. In
B. Cinnarizine patients where tablets are seen in the stomach
and the small intestine then either gastric
C. Inositol nicotinate
lavage or whole bowel irrigation should be
D. Simvastatin considered respectively. Iron is not adsorbed by
activated charcoal. Following ingestion toxicity
E. Diltiazem
can be divided into phases:

Answer & Comments  Phase 1 - Gastrointestinal toxicity


within 30 minutes to hours
Correct answer: A
 Phase 2 - Latent asymptomatic phase

Vasoactive drugs have limited benefit in  Phase 3 - Lethargy, neurological


treating intermittent claudication. There is toxicity, renal failure, pulmonary
modest evidence for the use of drugs such as oedema, hepatic failure and DIC can
naftidrofuryl and pentoxifylline, but little occur after 12 to 48 hours
benefit from cinnarizine or inositol nicotinate.
Simvastatin may be prescribed for patients with
Dr. Khalid Yusuf El-Zohry – Sohag Teaching Hospital (01118391123) Page | 792
El-Zohry MRCP Questions Bank (Part 2) – Medical Masterclass 2010

 Phase 4 - Gastrointestinal strictures of arrhythmias and the need for the patient to
occur 2 to 5 weeks after ingestion. be managed in a CCU/HDU environment.
Indications for desferrioxamine include a serum Always check for other poisons in cases of
iron concentration of greater than polypharmacy overdose. Also check arterial
90micromol/L or significant hypotension, blood gases for signs of hypoventilation and
metabolic acidosis and neurological features. acidosis.
Desferrioxamine should be given i.v. but in
exceptional circumstances can be given i.m. [ Q: 29 ] MasterClass Part2
Following administration, standard biochemical
(2010) - Pharmacology
measures of iron concentration are not useful
as these will measure both free and chelated A 59-year-old woman is admitted having taken
iron. an overdose with a serum lithium concentration
of 2.8mmol/l.
[ Q: 28 ] MasterClass Part2
Which TWO of the following clinical features
(2010) - Pharmacology are not attributable to the lithium?
A 40-year-old woman presents four hours after A. Polyuria
an overdose of amitriptyline and diazepam. On
B. Bronchospasm
examination her Glasgow Coma Scale score is
10. She has dilated pupils, a blood pressure of C. Renal failure
100/70 mmHg, and a pulse of 140 beats per
D. Urinary incontinence
minute. Investigations reveal:
E. Electrolyte imbalance
SaO2(ward oximetry) 95%
F. Choreoathetoid movements
BM 7mmol/l
G. Bloody diarrhoea

Which is the most appropriate other immediate H. Convulsions


investigation? I. Sino-atrial block.
A. CT brain scan J.
B. Serum urea and electrolytes
Answer & Comments
C. ECG
D. Serum paracetamol level Correct answer: BG

E. Serum salicylate level


Lithium poisoning causes a range of clinical
features. These can include:
Answer & Comments
 Mild: Nausea, diarrhoea, light
Correct answer: C headedness, drowsiness, polyuria,
muscle tremor and weakness.
Overdose of tricyclic antidepressants can cause  Moderate: Increasing confusion,
coma, convulsions and arrhythmias in serious blackouts and restlessness.
cases. The single most important test to guide Fasciculation, myoclonic twitches and
therapy and prognosis remains the 12 lead ECG: jerks, choreoathetoid movements,
a QRS>160 ms is associated with significant risk hyperreflexia, urinary/faecal
incontinence and hypernatraemia.

Dr. Khalid Yusuf El-Zohry – Sohag Teaching Hospital (01118391123) Page | 793
El-Zohry MRCP Questions Bank (Part 2) – Medical Masterclass 2010

 Severe: Renal failure, convulsions, nausea and vomiting. Tolerance can occur but
coma, Arrhythmias including SA others will remain on a stable dose for some
block/bradycardia/heart block. time.There is no evidence that morphine per se
Hypotension or hypertension can occur. will shorten life. Drowsiness can be a problem
for some patients.
Patients should be given supportive care and
haemodialysis should be considered to remove
lithium in those with severe clinical features [ Q: 31 ] MasterClass Part2
and/or raised lithium levels. Toxicity of lithium (2010) - Pharmacology
is more severe in patients on lithium treatment
than lithium naive patients. An 82-year-old man is admitted after a syncopal
episode. His pulse rate is 40/min and ECG
confirms complete (3rd degree) heart block. His
[ Q: 30 ] MasterClass Part2 pulse slows to 24/min and he feels very faint.
(2010) - Pharmacology
Whilst arrangements are being made for
A 65-year-old patient with malignant pain is
temporary pacing you give:
about to start morphine.
A. Adrenaline 0.5 mg intravenous bolus
Which TWO of the following statements are
B. Isoprenaline 50 mg intravenous bolus
correct?
C. Atropine 0.5 mg intravenous bolus
A. Patients should be observed for
respiratory depression. D. Atropine 5 mg intravenous bolus

B. All patients should start laxatives with E. Isoprenaline 500 mg intravenous bolus.
opiates.
Answer & Comments
C. Opiate users are not allowed to drive.
D. There is no ceiling dose of morphine. Correct answer: C

E. The majority of patients will experience


The options to be considered, prior to
nausea or vomiting.
temporary transvenous pacing, in this context
F. Addiction can be a problem for some are:
patients.
1. Atropine 0.5-1.0 mg intravenous bolus,
G. Doses can be limited by drowsiness. repeated as required.
H. The dose will inevitably increase with 2. Isoprenaline, intravenous infusion at 2-
time. 10 microg/min.
I. Tolerance always occurs. 3. External cardiac pacing
J. The presciption of morphine will shorten
the patient's life. [ Q: 32 ] MasterClass Part2
(2010) - Pharmacology
Answer & Comments
A 39-year-old woman with a past history of
Correct answer: DG treated hypertension is in her 3rd trimester of
pregnancy and requires on-going anti-
These are all the worries expressed by patients hypertensive treatment.
and doctors. Approx one third will experience

Dr. Khalid Yusuf El-Zohry – Sohag Teaching Hospital (01118391123) Page | 794
El-Zohry MRCP Questions Bank (Part 2) – Medical Masterclass 2010

Which anti-hypertensive would you definitely Answer & Comments


NOT prescribe?
Correct answer: A
A. Hydralazine
B. Labetalol Paracetamol ingestion of greater than
150mg/kg in 'normal risk' and greater than
C. Lisinopril
75mg/kg in 'high risk' individuals is associated
D. Methyl-dopa with hepatotoxicity if left untreated. Alcohol
E. Nifedipine. consumption of greater than 14 units per week
in females and 21 units per week in males is
associated with increased toxicity of
Answer & Comments
paracetamol, and in view of the fact she drinks
Correct answer: C only 10 units per week this is the correct
answer.
Evidence underpinning the choice of anti- Depletion of glutathione from the liver
hypertensive therapy in pregnancy is associated with eating disorders,
inadequate to make firm recommendations. malnourishment, cystic fibrosis and HIV is
There are no reports of serious effects with associated with increased toxicity. Additionally,
methyl-dopa following long and extensive use. viral infections such as EBV and CMV reduce
Calcium antagonists, labetalol and hydralazine glutathione levels in children.
are commonly used, particularly for resistant
The use of enzyme inducers such rifampicin,
hypertension in the third trimester. However,
phenytoin and carbamazepine are associated
angiotensin-converting enzyme (ACE)-inhibitors
with increased paracetamol toxicity.
should be avoided because they may cause
oligohydramnios, renal failure and intra-uterine
death. [ Q: 34 ] MasterClass Part2
(2010) - Pharmacology
[ Q: 33 ] MasterClass Part2 (2) A 65-year-old woman who is taking several
(2010) - Pharmacology different regular medications presents with a
history of visual disturbance. She complains of
An 18-year-old girl presents with a paracetamol
visual haloes and mild photophobia, which have
overdose.
been present for a few weeks.
Which of the following factors would not be
Which of the following drugs is the most likely
considered to increase the toxicity of the
cause of her symptoms?
overdose?
A. Nifedipine
A. Alcohol consumption of 10 units per
week B. Digoxin
B. BMI of 14 C. Amiodarone
C. Recent glandular fever D. Atenolol
D. History of cystic fibrosis E. Frusemide
E. Use of phenytoin.
Answer & Comments

Correct answer: C

Dr. Khalid Yusuf El-Zohry – Sohag Teaching Hospital (01118391123) Page | 795
El-Zohry MRCP Questions Bank (Part 2) – Medical Masterclass 2010

Amiodarone may cause corneal microdeposits. have had a myocardial infarction (MI) 3 months
These are dose-related and resolve when the ago. Despite treatment with simvastatin in
drug is withdrawn. Typical symptoms include addition to other usual cardiac medication, his
visual haloes and photophobia. ECG shows that he has had a second MI.
Digoxin classically causes the visual disturbance
Which of the following statements is LEAST
of xanthopsia (yellow vision) when present at
accurate?
toxic levels.
A. The overall effect of a drug depends on
the net effect on polymorphisms within
[ Q: 35 ] MasterClass Part2 its metabolic pathways and genetic
(2010) - Pharmacology variation in target cells.
A 50-year-old woman has increasing frequency B. There is an interaction between apo E
of migraine attacks. You decide to start some genotype and lipid response to statin
prophylactic therapy. therapy.

Which one of the following drugs would not be C. When a genetic variation affects more
appropriate for prophylaxis against migraine? than 1% of the population it is termed a
polymorphism.
A. Rizatriptan
D. Lack of concordance with treatment is a
B. Sodium valproate possible explanation.
C. Propranolol E. Data from RCTs show that all MI patients
D. Amitriptyline should receive a statin.

E. Pizotifen.
Answer & Comments

Answer & Comments Correct answer: E

Correct answer: A
A, B and C are correct statements about
pharmacogenetics, and D is also true. E looks
Rizatriptan is not used as prophylaxis against
sensible, but remember that RCTs show what
migraine. It is a 5HT1 agonist and may be useful
happens in a group of patients in comparison
in the treatment of acute migraine attacks. It is
with a group of controls. In any trial showing
available as either tablets or ‘melt wafers’,
the survival advantage on a drug, some
which dissolve on the tongue.
individuals will have a greater than average
Propranolol and pizotifen are licensed for use response, some less than average and some will
as prophylaxis against migraine. Pizotifen may be harmed. At present we cannot easily
cause drowsiness and weight gain. distinguish between these groups, so all
Although sodium valproate and amitriptyline patients are given a statin.
are unlicensed for migraine prophylaxis, they
may be effective in some patients. [ Q: 37 ] MasterClass Part2
(2010) - Pharmacology
[ Q: 36 ] MasterClass Part2 A 24-year-old man has chronic renal failure for
(2010) - Pharmacology which he receives haemodialysis three times
A 56-year-old man is admitted to hospital with per week. He decides to go on holiday, and one
crushing central chest pain. He is known to week later - having missed two dialysis sessions

Dr. Khalid Yusuf El-Zohry – Sohag Teaching Hospital (01118391123) Page | 796
El-Zohry MRCP Questions Bank (Part 2) – Medical Masterclass 2010

- he presents feeling unwell and breathless. that demonstrates acute interstitial nephritis.
Examination reveals pulmonary oedema. His There have been many recent changes in her
ECG shows no P waves, broad QRS complexes medications.
and peaked T waves.
Which of the following is most likely to be
What should you do? responsible for this condition?
A. Give 10 U actrapid with 50 ml of 50% A. Candesartan
glucose intravenously
B. Coproxamol
B. Give 10 ml of 10% calcium gluconate
C. Enalapril
intravenously
D. Ibuprofen
C. Give 10 mg salbutamol by nebuliser
E. Trimethoprim
D. Give 100 mg lignocaine intravenously
E. Transfer to the dialysis unit for Answer & Comments
haemodialysis
Correct answer: D
Answer & Comments
The drugs that most commonly cause acute
Correct answer: B interstitial nephritis are penicillins, non-
steroidal anti-inflammatory drugs and thiazide
All of the interventions listed, with the diuretics.
exception of intravenous lignocaine, are
When the circulation is compromised, non-
recognised treatments for hyperkalaemia, but
steroidal anti-inflammatory agents, angiotensin
this man is at clear risk of cardiac arrest and
converting enzyme inhibitors (such as enalapril)
only calcium gluconate acts instantly to
and angiotensin-II receptor antagonists (such as
'stabilise' the heart (although it does not alter
candesartan) can have adverse haemodynamic
the serum potassium level).
effects on the kidney leading to a reduction in
ECG monitoring will show that the ECG assumes GFR, but ACE inhibitors and AT2 blockers are
a more normal morphology after calcium not associated with interstitial nephritis.
gluconate is given. Appropriate treatment then
would be to give insulin / dextrose or [ Q: 39 ] MasterClass Part2
salbutamol whilst arranging urgent transfer to
(2010) - Pharmacology
the dialysis unit.
A 24-year-old female is brought in collapsed
[ Q: 38 ] MasterClass Part2 from a nightclub and ingestion of GHB
(gammahydroxybutyric acid) is suspected.
(2010) - Pharmacology
(2) A 58-year-old woman with a past medical Which of the following statements is true?
history of hypertension, congestive cardiac A. No effects occur unless more than
failure, osteoarthritis, urinary tract infections 50mg/kg have been ingested.
and depression presents with general malaise.
B. Hypernatraemia can occur .
Blood tests show that she has acute renal
failure, with serum creatinine 700 micromol/l. C. Response to ingestion of GHB is
Ultrasound examination reveals two normal- predictable.
sized kidneys and she proceeds to renal biopsy

Dr. Khalid Yusuf El-Zohry – Sohag Teaching Hospital (01118391123) Page | 797
El-Zohry MRCP Questions Bank (Part 2) – Medical Masterclass 2010

D. CNS effects are reduced if alcohol is co- A. It is licensed for use in patients with rest
ingested. pain.
E. Toxicity due to GHB ingestion alone B. Its main action is to activate
frequently leads to fatality. phosphodiesterases.
C. It is safe in patients with previous
Answer & Comments haemorrhagic stroke.
Correct answer: B D. It is metabolised by cytochrome P450
3A4.
Effects of GHB (gammahydroxybutyric acid) E. It can be used as a treatment for other
depend on the dose ingested but response to vascular diseases e.g. IHD, TIAs.
ingestion is unpredictable. Typical symptoms
include:
Answer & Comments
 10-30mg/kg ingested- 'mild' e.g. GI
Correct answer: D
upset, confusion, agitation and
euphoria
Cilostazol is a potent inhibitor of platelet
 30-50mg/kg ingested - drowsiness,
phosphodiesterases leading to inhibition of
coma and cardiovascular compromise
platelet aggregation. It also has vasodilating
 >50mg/kg ingested - significant actions. It is currently licensed for the
cardiovascular and neurological management of patients with intermittent
compromise can occur. claudication without rest pain and with no signs
of tissue necrosis. Studies have shown that its
Co-ingestion of other CNS depressants leads to
principal hepatic metabolism is by the
potentiation of the CNS effects of GHB.
cytochrome P450 3A4 isoenzyme. Its use is
Metabolic disturbances seen include metabolic
contraindicated in patients with known
acidosis, hypernatraemia, hypokalaemia and
bleeding tendencies (e.g. active peptic ulcer
hyperglycaemia.
disease, previous haemorrhagic stroke in the
Management of GHB toxicity is mainly last 6 months). Trials show an improvement in
symptomatic and supportive. Patients may time to initial pain on walking and maximal
require intubation and ventilation if significant walking distance when compared to placebo.
respiratory depression occurs. There have been
no reported cases of fatality related to GHB
[ Q: 41 ] MasterClass Part2
ingestion alone.
(2010) - Pharmacology
[ Q: 40 ] MasterClass Part2 A 75-year-old man is admitted to hospital. He is
(2010) - Pharmacology receiving warfarin as prophylaxis for a previous
DVT. His international normalised ratio (INR)
A 75-year-old gentleman with known peripheral has been stable at 2-2.5 for the past 8 weeks,
vascular disease and intermittent claudication is but whilst in hospital it increases to >8.
unsuitable for surgical intervention. It has been
decided to commence him on cilostazol. Which of the following drugs prescribed in
hospital could cause his increased INR?
Which of the following is true concerning
A. Ciprofloxacin
cilostazol?
B. Aspirin

Dr. Khalid Yusuf El-Zohry – Sohag Teaching Hospital (01118391123) Page | 798
El-Zohry MRCP Questions Bank (Part 2) – Medical Masterclass 2010

C. Carbamazepine Aside from administering high flow oxygen and


starting intravenous fluid resuscitation, your
D. Rifampicin
immediate action is to:
E. Co-Dydramol.
A. order a CT scan of her brain

Answer & Comments B. perform a lumbar puncture


C. take blood cultures and await result
Correct answer: A
D. give aciclovir 10 mg/kg intravenously
Commonly prescribed antibiotics E. give cefotaxime 2 g intravenously.
clarithromycin, ciprofloxacin and metronidazole
enhance the anticoagulant effect of warfarin,
Answer & Comments
whereas rifampicin (a potent enzyme inducing
drug) diminishes the anticoagulant effect. Correct answer: E
Although aspirin may increase the likelihood of
bleeding due to its antiplatelet and gastric The working diagnosis must be meningococcal
irritant effects, it should not cause prolongation meningitis and the woman must be given
of the INR; other non-steroidal anti- appropriate an appropriate antibiotic (e.g.
inflammatory (NSAIDs), especially cefotaxime 2 g) intravenously without delay.
azapropazone may cause prolongation of the In the very old or immunocompromised it
INR. would be appropriate to add ampicillin 2 g six-
Carbamazepine, primidone and hourly to cover Listeria, and aciclovir (10 mg/kg
phenobarbitone induce liver enzymes and eight-hourly, with dose reduction in renal
therefore reduce anticoagulant effect, while failure) should also be given if herpes simplex
valproate probably increases the INR. The effect encephalitis is a possibility.
of phenytoin is unpredictable, as both
prolongation and reduction of the INR have [ Q: 43 ] MasterClass Part2
been reported. (2010) - Pharmacology
Unlike co-proxamol, which causes prolongation
A 53-year-old woman presents with a digoxin
of the INR, codydramol has no significant effect
overdose.
on warfarin, and is therefore a useful analgesic
for these patients. Which of the following statements is false?
A. Tachyarrhythmias do not occur.
[ Q: 42 ] MasterClass Part2
B. Peak effects can be delayed after
(2010) - Pharmacology
ingestion by 6-12 hours.
An 18-year-old woman is brought to the C. Digoxin can precipitate severe
Emergency Department by a friend, who says hyperkalaemia (>6.5mmol/l).
that she has been ill for 24 hours with 'flu-like
symptoms and headache. She is unwell (pulse D. DC cardioversion can precipitate
90/min, BP 110/80 mmHg), drowsy (Glasgow intractable ventricular fibrillation (VF) or
Coma Score 11) and has a purpuric rash on her asystole.
arms. E. Activated charcoal reduces absorption.

Dr. Khalid Yusuf El-Zohry – Sohag Teaching Hospital (01118391123) Page | 799
El-Zohry MRCP Questions Bank (Part 2) – Medical Masterclass 2010

Answer & Comments This patient has an acute confusional state, and
all causes of this should be excluded, especially
Correct answer: A
infection.

Digoxin is slowly absorbed, hence peak effects It is important to remember that many
psychiatric drugs should not be stopped
can be delayed by up to 12 hours after
overdose. Oral activated charcoal may reduce precipitously and these include selective
absorption, although giving multiple doses is serotonin reuptake inhibitors (SSRIs) such as
controversial. Nausea and vomiting occur early paroxetine and benzodiazepines (such as
after poisoning; other features include lorazepam). If the dose of these medications
confusion, headache and visual disturbances. needs to be changed, this must be done very
Any brady or tachyarrhthymia can occur. slowly, otherwise they can produce an acute
withdrawal state with worsening confusion and
Treatment is with correction of hyperkalaemia agitation. The same applies to alcohol. Her
and atropine for bradyarrhythmias. Digoxin- family can bring in her usual tipple, so that the
specific antibodies are useful in i) those with nurses can monitor intake. Otherwise, prescribe
hyperkalaemia resistant to treatment, ii) chlordiazepoxide in reducing doses.
bradyarrhythmias with hypotension non-
responsive to atropine and iii) tachyarrhythmias It is important to check her thyroid-stimulating
hormone (TSH) level to ensure she is on the
with hypotension. Anti-arrhythmic drugs and
DC cardioversion should be avoided as they can correct dose of thyroxine, but this is unlikely to
precipitate intractable asystole or VF. be the cause of the confusion. Cimetidine has
been shown to cause confusion in older people.

[ Q: 44 ] MasterClass Part2 Does she still need ulcer healing treatment? If


so, consider a proton pump inhibitor (PPI).
(2010) - Pharmacology
An 87-year-old woman is admitted after being [ Q: 45 ] MasterClass Part2
found wandering in her nightie. She says she is
(2010) - Pharmacology
looking for her cat. On examination, she smells
strongly of alcohol, but is otherwise well. Her A 68-year-old woman with ascites secondary to
medications include paroxetine for depression, carcinoma of the ovary is complaining of early
lorazepam for anxiety (she is not sure how satiety and intermittent vomiting.
many she is taking), cimetidine for longstanding
peptic ulcer disease and thyroxine 50mcg daily. The antiemetic of choice is:
A. Cyclizine
Which medication can be stopped immediately?
B. Haloperidol
A. Paroxetine
C. Ondanestron
B. Lorazepam
D. Metoclopramide
C. Alcohol
E. Dexamethasone
D. Cimetidine
E. Thyroxine. Answer & Comments

Correct answer: D
Answer & Comments

Correct answer: D Metoclopramide as a prokinetic agent will


increase the rate of transit of food through the

Dr. Khalid Yusuf El-Zohry – Sohag Teaching Hospital (01118391123) Page | 800
El-Zohry MRCP Questions Bank (Part 2) – Medical Masterclass 2010

gastrointestinal tract and alleviate her A. Co-ingestion of alcohol increases the risk
symptoms. She should also be advised to eat of lactic acidosis.
'little and often'.
B. Hypoglycaemia is seen in all overdoses.
C. Metformin is not removed by
[ Q: 46 ] MasterClass Part2
haemodialysis.
(2010) - Pharmacology
D. Metformin causes a non-lactic acidosis.
A 17-year-old male fails to breathe
E. Mortality with metformin-associated
spontaneously after an operation. Talking to his
lactic acidosis is uncommon.
family, his sister has previously had similar
problems.
Answer & Comments
Which of the following drugs could have caused
Correct answer: A
this problem?
A. Thiopentone Metformin classically causes a type-B lactic
B. Atracurium acidosis in overdose, especially in patients who
have co-ingested alcohol or who have
C. Suxamethonium
underlying renal or hepatic dysfunction. Main
D. Cisatracurium symptoms of toxicity include gastrointestinal
E. Halothane. upset and a severe lactic acidosis.
Hypoglycaemia is not often seen in metformin
overdose. If lactic acidosis occurs following
Answer & Comments
overdose, mortality is usually greater than 50%.
Correct answer: C Management is gastric decontamination and
use of activated charcoal if appropriate, and
Suxamethonium is a depolarising correction of acidosis with 8.4% sodium
neuromuscular blocking agent that is bicarbonate. Patients with resistant acidosis
metabolised by plasma pseudocholinesterases. should be considered for haemodialysis, which
Approximately one in 2500 individuals have also clears metformin
deficiency of this enzyme, resulting in
prolonged neuromuscular blockade if they are [ Q: 48 ] MasterClass Part2
given suxamethonium. Management of these (2010) - Pharmacology
patients is by prolonged ventilation until the
action of the drug wears off. Relatives of You see a 48-year-old Afro-Caribbean man in
affected patients should be screened. the outpatient clinic with uncomplicated
essential hypertension. His blood pressure
today is 154/102mmHg despite optimization of
[ Q: 47 ] MasterClass Part2
non-pharmacological therapy.
(2010) - Pharmacology
Which one of the following drugs would you use
A 35 year old gentleman with type 2 diabetes
as the first-line agent in this patient?
mellitus is brought in following an overdose of
his oral hypoglycaemic agents. You are A. Atenolol 50mg od
concerned that he may have taken an overdose B. Nifedipine 10mg tds
of metformin.
C. Amlodipine 5mg od
Which of the following statements is true? D. Ramipril 2.5mg od

Dr. Khalid Yusuf El-Zohry – Sohag Teaching Hospital (01118391123) Page | 801
El-Zohry MRCP Questions Bank (Part 2) – Medical Masterclass 2010

E. Enalapril 5mg bd. confirms that she has a deep venous


thrombosis.
Answer & Comments
How should this be managed up to the time of
Correct answer: C delivery?
A. Tubigrip stocking; avoid anticoagulation.
Non-pharmacological therapy should always be
B. Initiate and then continue treatment with
optimized prior to commencement of
warfarin until delivery.
medication, whenever possible. Hypertension is
particularly common in Afro-Caribbeans and C. Initiate and then continue treatment with
associated with particularly higher risk of heparin until delivery.
complications. Therefore effective long-term
D. Initiate treatment with heparin; load with
treatment, with a low threshold for multiple
and convert to warfarin, continued until
therapy where necessary, is particularly
delivery.
important.
E. Initiate and then continue treatment with
Studies indicate that drugs such as ACE
heparin and warfarin until delivery.
(angiotensin-converting enzyme) inhibitors and
Beta-receptor antagonists are less effective in
Afro-Caribbeans. The reason appears to be Answer & Comments
related to the finding that the renin- Correct answer:
angiotensin-aldosterone (RAA) system is
commonly suppressed in the majority of Afro-
During a critical window of 6-9 weeks of
Caribbeans. As such, drugs that suppress the
pregnancy warfarin causes (warfarin
RAA system are less likely to be effective.
embryopathy) manifest, most obviously as
Calcium-channel blockers (CCBs) and diuretics hypoplasia of the nose and limbs. After this
appear to be more effective in this subgroup. period warfarin is associated with neurological
However, diuretics may not be suitable in this damage – mental retardation, microcephaly,
case as they are commonly associated with optic atrophy and blindness – that may be
impotence. Short-acting CCBs do not provide haemorrhagic in origin and hence reduced by
prolonged BP control, can cause reflex tight anticoagulant control. Warfarin is also
tachycardia and may be associated with higher associated with increased risk of abruptio
mortality. Therefore, long-acting CCB should be placentae in later pregnancy and increased
the first-line drug of choice. Ideally, a once-daily bleeding in mother and fetus if used at term.
agent with that provides a smooth 24-hour BP
Most physicians would manage this woman
control (e.g. Nifedipine LA 30mg od or
with low molecular weight heparin throughout
Amlodipine 5 mg od) to improve compliance
her pregnancy, although this is not without
would be preferable.
problems, in particular of maternal
osteoporosis. Those that would use warfarin
[ Q: 49 ] MasterClass Part2 would not continue its use to term but switch
(2010) - Pharmacology to heparin.

A 20-year-old woman, previously fit and well,


with no significant past medical history, [ Q: 50 ] MasterClass Part2
presents 10 weeks into pregnancy with a (2010) - Pharmacology
painful swollen calf. Ultrasound examination
You see a patient in the outpatient clinic with
mild heart failure and hypertension (treated
Dr. Khalid Yusuf El-Zohry – Sohag Teaching Hospital (01118391123) Page | 802
El-Zohry MRCP Questions Bank (Part 2) – Medical Masterclass 2010

with bendrofluazide 5mg) whose recent blood A. The prevalence of alcohol abuse
tests have shown a serum potassium of increases with age.
3.1mmol. The only other past history is of
B. Self-recall is an accurate guide to
hiatus hernia.
amounts consumed.
Select the TWO drugs from the following list C. The CAGE questions are insensitive and
which would be suitable for treatment of an inappropriate screening tool for older
hypokalaemia in this patient: people.
A. Sustained release potassium chloride D. There are no special considerations when
B. Effervescent potassium chloride using DSM IV (Diagnostic and Statistical
Manual of Mental Disorders) Diagnostic
C. Bumetanide Criteria for Alcohol Dependence
D. Amiloride E. For a given amount of alcohol, the
E. Acetazolamide concentration in the blood is increased in
older people.
F. Oral rehydration salts.
F. The increased fracture risk is due to
sedation and falls rather than effects of
Answer & Comments
alcohol on bone.
Correct answer: BD
G. Withdrawal tends to occur much earlier
after the last drink than in younger
Mild hypokalaemia induced by thiazides is best people.
treated by using a potassium-sparing diuretic
such as amiloride. Potassium chloride can also H. Lorazepam is preferred in the treatment
be given either as effervescent tablets or syrup, of symptoms of withdrawal in those with
but may cause nausea in larger doses. liver disease.
Sustained release formulations are discouraged I. Disulfiram is an opioid antagonist to
in patients with hiatus hernia due to the risk of prevent recidivism.
oesophageal ulceration.
J. Rehabilitation programmes which are age-
Acetazolamide (carbonic anhydrase inhibitor) specific are no more effective.
and bumetanide (loop diuretic) will both
worsen the hypokalaemia as they promote Answer & Comments
potassium excretion in the urine.
Correct answer: EH

[ Q: 51 ] MasterClass Part2
The prevalence of alcohol abuse declines after
(2010) - Pharmacology
the age of 60. However those drinking to excess
A 74-year-old man is admitted with cardiac often go undetected. Self-reported use is less
failure and fast atrial fibrillation. His condition is accurate than diary records. CAGE consists of
stabilized. On further enquiry he admits to four questions and, if one or more are positive,
having drunk up to 80 units of alcohol per week have a good sensitivity and specificity in
since he retired almost 10 years ago. detecting alcohol abuse in older people (‘do
you feel you ought to Cut down on your
Which TWO statements are true regarding drinking...people Annoyed you by criticizing
alcohol abuse and dependence in older people? your drinking...felt Guilty about

Dr. Khalid Yusuf El-Zohry – Sohag Teaching Hospital (01118391123) Page | 803
El-Zohry MRCP Questions Bank (Part 2) – Medical Masterclass 2010

drinking....drinking in the morning (Eye opener) hypertension than previously


to steady nerves ?’). normotensive women.
Special consideration needs to be taken when C. Methyldopa is contra-indicated during
applying DSM IV Diagnostic Criteria e.g. many pregnancy.
late onset alcoholics do not develop physical
D. Beta blockers may restrict foetal growth
dependence, may not understand that physical
if given during pregnancy.
or psychological problems are related to alcohol
use, etc. E. Anti-hypertensive therapy should always
be avoided in early pregnancy.
Lean body mass and therefore total body water
decreases with advancing age. Thus each unit of
alcohol leads to a greater concentration in the Answer & Comments
blood. Correct answer: D
Fractures are due to alcohol-related falls AND
osteoporosis. Blood pressure usually falls during the second
trimester of pregnancy before rising again in
In older alcoholics, withdrawal may not start
the third trimester. Women with pre-existing
until several days after stopping drinking, and
hypertension have a slightly higher risk of
should be considered in the differential
developing pre-eclampsia than those who are
diagnosis of every confused older patient.
normotensive.
Chlordiazepoxide is usually used but lorazepam
Methyldopa, labetalol and (increasingly)
is preferred in those with liver disease, since it
nifedipine are usually the antihypertensive
does not depend on hepatic metabolism for
treatments of choice in pregnancy. Methyldopa
clearance. Disulfiram , a negative re-enforcer,
is often poorly tolerated due to side-effects.
and Naltrexone, an opioid antagonist which
ACE inhibitors and angiotensin receptor
reduces craving, are used to promote long-term
antagonists are contra-indicated as they may
abstinence. Rehabilitation programmes are
cause renal agenesis and foetal loss. Beta
more likely to be successful if they are age-
blockers may restrict foetal growth but are
specific, for example focusing on overcoming
sometimes used.
isolation and devising ways to spend time
previously spent drinking. In some cases anti-hypertensive therapy is
necessary in early pregnancy, with the risks and
[ Q: 52 ] MasterClass Part2 benefits of any drug therapy carefully
considered and discussed with the patient. In
(2010) - Pharmacology
some cases antihypertensive therapy is
A 38-year-old woman with a previous history of necessary in early pregnancy, with the risks and
essential hypertension presents in the sixth benefits of any drug therapy carefully
week of pregnancy requesting advice about considered and discussed with the patient.
drug therapy for hypertension in pregnancy.
[ Q: 53 ] MasterClass Part2
Which one of the following statements is true?
(2010) - Pharmacology
A. Blood pressure usually rises during the
2nd trimester of pregnancy, hence You are planning to use lignocaine/lidocaine as
additional drug therapy may be required. local anaesthesia for a patient who is about to
have a lumbar puncture.
B. Pre-eclampsia is no more common in
women with pre-exisiting essential

Dr. Khalid Yusuf El-Zohry – Sohag Teaching Hospital (01118391123) Page | 804
El-Zohry MRCP Questions Bank (Part 2) – Medical Masterclass 2010

Which of the following are true with regards to Solutions of lignocaine should not exceed 1%,
the side effects of lignocaine/lidocaine as a local except if used in surface anaesthesia where up
anaesthetic? to 2–4% is acceptable
A. Stimulates the respiratory centre
B. Intravascular administration may not be a
[ Q: 54 ] MasterClass Part2
problem (2010) - Pharmacology
C. Can cause tachycardia A 36-year-old woman with diabetes and a
past medical history of pulmonary embolism is
D. Excessive systemic absorption can
currently taking several medications. She wants
present with light headedness
to know whether they would be safe if she was
E. Is a vasoconstrictor breast-feeding her baby.
F. Should be used with caution in epileptics
Which one of the following drugs is not
G. Solutions of 2–4% are used in infiltration considered to be safe in breast-feeding?
anaesthesia
A. Warfarin
H. Can be safely injected into infected
B. Phenoxymethylpenicillin
tissues
C. Paracetamol
I. Careful monitoring of toxic effects is
needed for 2 hours after injection D. Aspirin
J. Can be given together with adrenaline E. Insulin
into body appendages.
Answer & Comments
Answer & Comments
Correct answer: D
Correct answer: DF
Aspirin is not considered to be safe in breast-
Lignocaine has CNS and cardiac depressant feeding due to the risk of causing Reye's
effects, thus causing respiratory depression and syndrome in the baby. Whether a drug is safe in
bradycardia. Features of CNS toxicity include breast-feeding depends both on how much of
inebriation and light-headedness early on, with the drug enters the breast milk and on the
convulsions developing in severe toxicity. Thus, toxicity of those levels to the baby. The other
it should be used cautiously in epileptics. drugs listed either pass into breast milk in very
small amounts or are considered to be non-
Most local anaesthetics (with the exception of
toxic to the baby.
cocaine) cause local dilation of blood vessels.
Adrenaline, a vasoconstrictor, is often used
together with lignocaine in order to limit [ Q: 55 ] MasterClass Part2
absorption and to prolong local activity, but this (2010) - Pharmacology
is contra-indicated in appendages and limb
A 60-year-old woman complains of joint pains.
digits. Arterial levels of lignocaine peak within
10–25 minutes, and careful surveillance for On examination she has a red scaly rash over
light exposed areas. Her autoimmune screen
toxic effects is recommended for 30 minutes.
reveals anti-histone antibodies but no other
Injection into infected tissues may lead to
abnormality.
greater systemic absorption, and is associated
with a greater risk of toxicity.

Dr. Khalid Yusuf El-Zohry – Sohag Teaching Hospital (01118391123) Page | 805
El-Zohry MRCP Questions Bank (Part 2) – Medical Masterclass 2010

Which TWO of the following drugs are most D. Phenytoin


likely to have caused her condition.
E. Cefotaxime
A. Carbimazole
F. Ciprofloxacin
B. Hydralazine
G. Sildenafil
C. Isoniazid
H. Levodopa
D. Atenolol
I. Buscopan.
E. Azathioprine
F. Probenecid Answer & Comments

G. Ceftazidime Correct answer: CF


H. Lansoprazole
Both Bupropion and quinolone antibiotics may
I. Itraconazole
cause fits and are contraindicated in patients
J. Cyclophosphamide with a history of seizures. Other drugs lowering
the seizure threshold include:
Answer & Comments  Antidepressants
Correct answer: BC  Antimalarials
 Antipsychotics
Drug-induced lupus presents with fever,
 Sedating antihistamines
myalgia, rash, arthralgia/arthritis and serositis.
 Theophylline
Hydralazine and isoniazid, as well as other
drugs including procainamide, pencillamine and  Tramadol.
methyl dopa, are known causes of drug-induced
lupus.
[ Q: 57 ] MasterClass Part2
Diagnosis can be assisted by the finding of anti- (2010) - Pharmacology
histone antibodies in the absence of other
autoantibodies and confirmed by One of your asthmatic patients who is on
disappearance of the lupus features within 1–7 theophylline requires antibiotic therapy.
months of discontinuing the offending drug.
Which TWO of the following antibiotics might
interact with concurrent theophylline therapy
[ Q: 56 ] MasterClass Part2 and cause a rise in plasma theophylline
(2010) - Pharmacology concentrations?

A 56-year-old man is admitted with convulsions A. Rifampicin


having recently started a new medication. B. Erythromycin

Of the following drugs, which TWO are the most C. Ampicillin


likely to have been responsible for his condition? D. Amoxycillin
A. Atenolol E. Ciprofloxacin
B. Losartan F. Benzylpenicillin
C. Bupropion G. Co-amoxiclav

Dr. Khalid Yusuf El-Zohry – Sohag Teaching Hospital (01118391123) Page | 806
El-Zohry MRCP Questions Bank (Part 2) – Medical Masterclass 2010

H. Ceftriaxone as fever, malaise and arthralgia may be present,


and other organs including the liver, kidney and
I. Piperacillin
heart may be affected. Anti-histone antibodies
J. Gentamicin. may also be present. The drug should be
stopped immediately and systemic
Answer & Comments corticosteroids may be of benefit. Other drugs
associated with a purpuric skin rash include
Correct answer: BE
aspirin, sulphonamides, penicillin, thiazides,
furosemide and corticosteroids.
Ciprofloxacin reduces theophylline clearance by
30–84% and can cause theophylline toxicity.
[ Q: 59 ] MasterClass Part2
Erythromycin also interferes with the liver
metabolism of theophylline and has a similar (2010) - Pharmacology
effect. A 73-year-old gentleman with dementia attends
Rifampicin is a hepatic enzyme inducer, thus clinic with his wife. She has heard about
promoting the breakdown of theophylline. memantine and wonders if it would be suitable
for her husband.
[ Q: 58 ] MasterClass Part2 Which of the following are true of memantine?
(2010) - Pharmacology
A. It has no interaction with amantadine.
A 43-year-old woman presents to her doctor
B. It is licensed for patients with all types of
with a raised, non-blanching and purpuric rash,
dementia.
which is mainly on her lower limbs. She has
recently been started on a new medication for C. It inhibits renal excretion of ranitidine.
hypertension. D. It enhances the effects of barbiturates.

Which of the following drugs is she most likely E. It is an acteylcholinesterase inhibitor.


to have been prescribed?
A. Atenolol Answer & Comments

B. Amlodipine Correct answer: C

C. Hydralazine
Memantine is the first licensed NMDA receptor
D. Diltiazem antagonist for the management of moderate to
E. Ramipril severe Alzheimer's disease. There is some
published evidence that memantine has small
benefits in reducing deterioration in patients
Answer & Comments
with Alzheimer's disease, but little evidence for
Correct answer: C use in other types of dementia. Several drug
interactions are known:
Purpura in the context of a drug reaction may  NMDA antagonists (e.g. ketamine,
be an isolated skin reaction, or it may be a amantadine) can precipitate psychosis
manifestation of drug-induced
 Dopamine agonists - effects enhanced
thrombocytopenia or drug-induced vasculitis.
Hydralazine is associated with drug-induced  Barbiturates and neuroleptics - effects
lupus-like syndrome that can present with a reduced
vasculitic purpuric rash. Systemic features such

Dr. Khalid Yusuf El-Zohry – Sohag Teaching Hospital (01118391123) Page | 807
El-Zohry MRCP Questions Bank (Part 2) – Medical Masterclass 2010

 Drugs excreted by cationic transporters B. Haloperidol


in the kidney (e.g. ranitidine, quinine,
C. Prochlorperazine
nicotine) - excretion reduced leading to
higher plasma concentrations. D. Metoclopramide
E. Ondanestron.
[ Q: 60 ] MasterClass Part2
(2010) - Pharmacology Answer & Comments

A 79-year-old woman presents to the Correct answer: D


Emergency Department with confusion,
headache and tinnitus. Her GP has recently It sounds as though there may be a neuro
started her on an analgesic and you are worried muscular cause for his symptoms. Another
she may have taken too much. possibility is external compression, but the
endoscopy did not show this.
Which of the following would most likely explain
her symptoms? Metoclopramide is both an antidopaminergic
and gastrokinetic agent. It may improve
A. Paracetamol oesophageal motility.
B. Aspirin
C. Diclofenac [ Q: 62 ] MasterClass Part2
D. Co-codamol
(2010) - Pharmacology

E. Codeine phosphate (1) You are treating a 15-year-old girl with


asthma. She is currently on a salbutamol inhaler
as required and beclometasone 400
Answer & Comments
micrograms daily. She finds that she has to use
Correct answer: B her salbutamol inhaler two or three times a
day, and that she sometimes has nocturnal
Aspirin in excess causes symptoms of nausea, cough.
vomiting, headache, confusion and tinnitus or
Which TWO of the following treatment options
hearing difficulties. Whilst the co-codamol and
might you recommend?
codeine phosphate could cause confusion, they
would not cause the tinnitus. All analgesics A. Continue current medication
taken for a prolonged period of time can lead to
B. Eformeterol
an analgesic-induced headache.
C. Nedocromil sodium

[ Q: 61 ] MasterClass Part2 D. Theophylline


(2010) - Pharmacology E. Montelukast
A 50-year-old man with metastatic colorectal F. Ipratropium
cancer complains of regurgitation of food and a G. Salbutamol sustained-release tablets
feeling that it sticks retrosternally. Chest
radiograph and upper gastrointestinal H. Salmeterol
endoscopy are normal. I. Aminophylline

Which antiemetic is most likely to be of benefit? J. Ketotifen.

A. Cyclizine

Dr. Khalid Yusuf El-Zohry – Sohag Teaching Hospital (01118391123) Page | 808
El-Zohry MRCP Questions Bank (Part 2) – Medical Masterclass 2010

Answer & Comments [ Q: 64 ] MasterClass Part2


Correct answer: BH (2010) - Pharmacology
A 21-year-old girl started taking the combined
The presence of nocturnal cough and oral contraceptive pill a few months ago and is
bronchodilator requirement of 2-3 times a day complaining of several side effects.
suggests that her asthma control is inadequate.
The recommended options in this instance Which TWO of the following are NOT
would be to either increase the inhaled steroid recognised side effects of the pill?
to 800 micrograms a day, or to add a long A. Jaundice
acting beta-2-agonist such as salmeterol or
eformoterol. B. Migraine
C. Depression
[ Q: 63 ] MasterClass Part2 D. Reduced libido
(2010) - Pharmacology E. Chest pain
A 66-year-old woman who requires treatment F. Nausea
with amiodarone for intractable arrhythmias is
given an intravenous loading dose of 300 mg (5 G. Bloody diarrhoea
mg/kg). H. Chorea

Which of the following best explains why a I. Photosensitivity


loading dose is used in this patient? J. Erythema ab igne.
A. Amiodarone clearance is genetically
determined Answer & Comments

B. Amiodarone has a long plasma half-life Correct answer: GJ


C. Amiodarone is eliminated by zero-order
metabolism The combined oral contraceptive pill (COCP) has
many possible side effects. Chest pain in a
D. Amiodarone is rapidly metabolised by the
patient on the COCP may be due to pulmonary
liver
embolism and should be investigated without
E. Amiodarone is widely bound in body delay.
tissues
Bloody diarrhoea is not a recognised side effect
of the COCP and should be investigated
Answer & Comments appropriately.
Correct answer: E Erythema ab igne is a reticular rash usually
caused by heat injury. The COCP has been
Tissue-binding sites must be ‘filled up' by a associated with erythema nodosum.
loading dose before a therapeutic plasma
concentration can be achieved. [ Q: 65 ] MasterClass Part2
Metabolism/elimination/clearance rates and (2010) - Pharmacology
plasma half-life determine the time taken to
achieve a steady-state plasma concentration You are asked to see a 76-year-old woman who
and the level of that steady-state concentration has a number of problems including gout,
when a steady dosing regimen is established. osteoporosis, dementia with parkinsonism and

Dr. Khalid Yusuf El-Zohry – Sohag Teaching Hospital (01118391123) Page | 809
El-Zohry MRCP Questions Bank (Part 2) – Medical Masterclass 2010

a probable urinary tract infection. Her B. Bradycardia is common


creatinine reading is 215umol/L.
C. Physostigmine is the treatment of choice
You need to consider the advisability or dose of D. Mydriasis occurs
all the following drugs except TWO:
E. Urinary incontinence is common.
A. Allopurinol
B. Olanzapine Answer & Comments

C. Paracetamol Correct answer: D


D. Zopiclone
Anticholinergic syndrome occurs following
E. Risedronate
overdose with drugs that have prominent
F. Norfloxacin anticholinergic activity including tricyclic
antidepressants, antihistamines and atropine.
G. Co-amoxiclav
Features include dry, warm, flushed skin,
H. Alendronate urinary retention, tachycardia, mydriasis
I. Galantamine (dilated pupils) and agitation. Although
physostigmine, a reversible inhibitor of
J. Levodopa.
acteylcholinesterase, is effective in treating
symptoms, there is a significant risk of cardiac
Answer & Comments toxicity (bradycardia, AV conduction defects
and asystole). Treatment therefore consists of
Correct answer: CJ
withdrawal of the precipitating drug and
supportive care.
We are not told this woman's weight. If she has
low muscle bulk her renal function may be
moderately impaired. It is hard to find drugs [ Q: 67 ] MasterClass Part2
that are not affected by poor renal function - (2010) - Pharmacology
either because their excretion is impaired or
A 16-year-old girl is seen in the Emergency
because the target organ seems more sensitive.
Department. She states that she has taken
If you are writing a prescription chart for any
thirty paracetamol tablets (500mg) over the last
patient with renal impairment, always have a
12 hours. She is currently seeing the
BNF open at appendix 3 with you. Do not rely
psychiatrists for outpatient management of
on your memory. The older the patient the
anorexia.
more important this is as you may not be able
to tide such a patient over iatrogenic Which of the following is the most appropriate
nephrotoxicity with dialysis immediate management?
A. Perform gastric lavage and check
[ Q: 66 ] MasterClass Part2 paracetamol levels four hours after the
(2010) - Pharmacology last tablet was taken
A 28-year-old male presents following an B. Give activated charcoal and check
overdose and anticholinergic syndrome is paracetamol levels four hours after the
suspected. last tablet was taken
C. Give activated charcoal and check
Which one of the following is true?
paracetamol levels immediately
A. Tricyclic antidepressants are not a cause

Dr. Khalid Yusuf El-Zohry – Sohag Teaching Hospital (01118391123) Page | 810
El-Zohry MRCP Questions Bank (Part 2) – Medical Masterclass 2010

D. Give activated charcoal and start n-acetyl supermarkets and health food shops in the
cysteine United Kingdom. There is some evidence that it
is effective in the treatment of depression.
E. Wait for the results of paracetamol levels
before instituting treatment It induces the CYP3A4 system in the liver,
causing drug interactions with other
Answer & Comments medications including the combined oral
contraceptive pill and ciclosporin. The amount
Correct answer: D of active ingredient and therefore the degree of
enzyme induction varies between different
Paracetamol levels are very difficult to interpret preparations of St John's wort, so switching
if the patient has taken the tablets over a between different preparations may cause
period of time, and this girl is at high risk other drug levels to change unpredictably. St
because of the alleged number she has taken John's wort should not be taken with other
and the co-morbidity of anorexia. Gastric lavage antidepressants such as SSRI's as they both act
is only ever indicated if the patient has taken a to increase serotonin levels, and this may be
large number of tablets in the hour or so before dangerous.
presentation. Charcoal will help to prevent
absorption. [ Q: 69 ] MasterClass Part2
(2010) - Pharmacology
[ Q: 68 ] MasterClass Part2
A 26-year-old woman consults her general
(2010) - Pharmacology
practitioner in the 12th week of her pregnancy.
A 26-year-old woman who has been suffering She complains of fever and dysuria. There is no
from mild depression tells you that she has other significant history but direct questioning
been taking St John's wort for the last few reveals a self-limiting rash in the past after
weeks. taking penicillin. Her GP sends off a sample of
urine that shows a significant growth of Gram
Which one of the following statements about St negative bacilli. The organism is sensitive to the
John's wort is NOT true? antibiotics listed below. Her GP is uncertain as
A. It should not be taken concurrently with to the best agent in the context of the
other antidepressants such as SSRI's. pregnancy and contacts you by telephone for
advice.
B. It may affect the efficacy of the combined
oral contraceptive pill. Which of the following would be the best choice
C. It can be obtained without a doctor's agent in this situation?
prescription in the United Kingdom. A. Ciprofloxacin
D. It inhibits the CYP3A4 drug-metabolising B. Gentamicin
enzymes in the liver.
C. Cefaclor
E. It may cause photosensitivity.
D. Trimethoprim

Answer & Comments E. Co-amoxiclav.

Correct answer: D
Answer & Comments

St John's wort (Hypericum perforatum) is a Correct answer: C


herbal remedy available in pharmacies,
Dr. Khalid Yusuf El-Zohry – Sohag Teaching Hospital (01118391123) Page | 811
El-Zohry MRCP Questions Bank (Part 2) – Medical Masterclass 2010

Ciprofloxacin has been associated with infiltrates. More commonly it causes


arthropathy and cartilage erosions in young gastrointestinal upset and nausea. Leucopenia,
animals. rash and abnormal results in liver function tests
also occur. Pulmonary toxicity also occurs with
Gentamicin needs to be given parenterally and
amiodarone, nitrofurantoin, NSAIDs,
is not suitable for outpatient use. There is also a
methotrexate, bleomycin (fibrotic picture) and
risk of fetal nephrotoxicity and ototoxicity.
other chemotherapeutic agents (taxanes and
Trimethoprim is a folate antagonist and can gemcitabine). Withdrawal of the offending drug
increases the risk of neural tube defects. and steroid therapy are the mainstays of
Co-amoxiclav is a combination of amoxycillin treatment.
and clavulanic acid, and although there is no
definite risk of teratogenicity, it should not be [ Q: 71 ] MasterClass Part2
used unless absolutely necessary. Furthermore, (2010) - Pharmacology
this patient has previously had a rash with
penicillin. Although there is a small risk of cross- A 38-year-old woman has suffered from
allergy (10%) with cephalosporins, cefaclor headaches and pains in her legs for many years.
would be a reasonable choice given that the Despite thorough investigation, no physical
penicillin allergy was relatively mild and not a basis for any of her pains has been established.
full-blown anaphylactic reaction. They seem to be neuropathic in nature, and you
talk with her about the wide variety of drugs
that can be helpful in treating these sorts of
[ Q: 70 ] MasterClass Part2
difficult pain.
(2010) - Pharmacology
Which TWO of the following are NOT
A 76-year-old man with rheumatoid arthritis
potentially beneficial for neuropathic pain?
and atrial fibrillation presents with a 5-day
history of shortness of breath, cough and A. Flecainide
decreased exercise tolerance. A CXR shows B. Clonidine
unilateral alveolar infiltrates and lung biopsy
reveals acute eosinophilic pneumonia. C. Propranolol
D. Carbamezepine
Which of the following of his medications could
account for his condition? E. Imipramine

A. Digoxin F. Gabapentin

B. Sulfasalazine G. Haloperidol

C. Ciprofloxacin H. Amitriptyline

D. Atenolol I. Lignocaine

E. Paracetamol J. Dothiepin hydrochloride.

Answer & Comments Answer & Comments

Correct answer: B Correct answer: CG

Long-term sulfasalazine therapy is sometimes The drugs used for neuropathic pain are either
associated with pulmonary toxicity that membrane stabilisers such as antiepileptics or
presents as dyspnoea with pulmonary antiarrhythmics, or antidepressants such

Dr. Khalid Yusuf El-Zohry – Sohag Teaching Hospital (01118391123) Page | 812
El-Zohry MRCP Questions Bank (Part 2) – Medical Masterclass 2010

norepinephrine and serotonin reuptake [ Q: 73 ] MasterClass Part2


inhibitors. (2010) - Pharmacology
Beta adrenergic receptors and dopamine
A 29-year-old man with a history of epilepsy has
receptors are not involved in neuropathic pain
been well controlled on carbamazepine and
transmission.
clonazepam for the last 5 years. He now wishes
to consider withdrawing from or reducing his
[ Q: 72 ] MasterClass Part2 medication.
(2010) - Pharmacology
Which of the following statements are correct?
You see a patient with chronic renal
A. There is about a 60% chance of
impairment whose recent blood tests have
experiencing a relapse in the first year
shown a low serum calcium, with a high
during withdrawing from anti-epilepsy
phosphate level.
treatment.
Which TWO of the following agents are useful B. Both anti-epileptics can be safely
in the management of hypocalcaemia in this withdrawn simultaneously.
patient:
C. The dose of carbamazepine can be
A. Calcitonin reduced safely by 10% every 2-4 weeks.
B. Alfacalcidol D. He can be advised that he can continue
C. Ergocalciferol (Vitamin D2) driving during withdrawal from anti-
epilepsy treatment as long as he remains
D. Etidronate
free from seizures.
E. Alendronate
E. It is likely that he will subsequently
F. Calcitriol. require higher doses to regain control
with the current therapy, if
Answer & Comments discontinuation fails.

Correct answer: BF
Answer & Comments

Vitamin D analogues are useful in the treatment Correct answer: C


of hypocalcaemia due to renal disease.
However, conversion of ergocalciferol to active Studies suggest that about 25% of patients who
derivatives requires hydroxylation in the have decided to stop their anti-epilepsy
kidney. In contrast, alfacalcidol (1-alpha- treatment relapse within a year of starting to
hydroxycholecalciferol) and calcitriol (1,25- taper down their medication. The likelihood of
hydroxycholecalciferol) are active forms of seizure is greatest during withdrawal and in the
Vitamin D, and are the better choices in subsequent 6 months. The DVLA recommends
patients with chronic renal disease. that patients should not drive during this
period.
Calcitonin, etidronate and alendronate reduce
bone resorption and are not helpful in this case. Doses of drugs such as carbamazepine,
lamotrigine, phenytoin, sodium valproate and
vigabatrin should be reduced by about 10%
every 2-4 weeks. Barbiturates, benzodiazepines
and ethosuximide should be tapered more
slowly by reducing dosage by about 10% every

Dr. Khalid Yusuf El-Zohry – Sohag Teaching Hospital (01118391123) Page | 813
El-Zohry MRCP Questions Bank (Part 2) – Medical Masterclass 2010

4-8 weeks. Only one drug should be withdrawn [ Q: 75 ] MasterClass Part2


at one time, with a period of 1 month between (2010) - Pharmacology
completing withdrawal of one drug and
beginning withdrawal of the next. There is no A 55-year-old man is started on digoxin for
evidence to support the belief that patients atrial fibrillation.
become resistant to their original therapy
following discontinuation. Which of the following measurements would be
most useful when monitoring him for digoxin
efficacy?
[ Q: 74 ] MasterClass Part2
A. Blood pressure
(2010) - Pharmacology
B. Glomerular filtration rate
A middle-aged man is brought by ambulance to
the Medical Admissions Unit. He was fitting C. Plasma digoxin concentration
when picked up and is still having a grand mal D. Pulse rate
convulsion.
E. Urea and creatinine
The most appropriate treatment is:
Answer & Comments
A. Lorazepam 2 mg intravenously
B. Fosphenytoin 15 mg/kg body weight Correct answer: D
phenytoin equivalent, intravenously at a
rate of 100-150 mg phenytoin equivalent You wish to monitor efficacy of digoxin, which is
/ min used for rate control in atrial fibrillation. Blood
pressure will give some measure of cardiac
C. Phenytoin 15 mg/kg body weight,
function but will not give as much information
intravenously at a rate of 50 mg/min
as pulse rate. Measuring drug plasma
D. Diazepam 10 mg intravenously concentration will tell you whether digoxin is at
E. Phenobarbitone 10 mg/kg body weight, therapeutic concentrations in the blood, but
intravenously at a rate of 100 mg/min. not whether it is having a therapeutic effect.
Urea, creatinine and glomerular filtration rate
will give an indication of renal function and
Answer & Comments
likely clearance (but not efficacy) of digoxin.
Correct answer: A
[ Q: 76 ] MasterClass Part2
All of these are recognized treatments for (2010) - Pharmacology
status epilepticus. First-line treatment should
be with intravenous benzodiazepine, with You see a 30-year-old lady who has occasional
lorazepam preferred to diazepam because of its attacks of severe acute migraine.
longer duration of action. Fosphenytoin is the
preferred second-line treatment (phenytoin if Which TWO of the following therapeutic
this is not available). Phenobarbitone is one of options will be beneficial for treatment of acute
several agents that can be used as third-line attacks in this patient?
treatment, but seek specialist advice if first and A. Paracetamol and metoclopramide
second-line treatments are ineffective.
B. Codeine phosphate
C. Amitriptyline
D. Methysergide

Dr. Khalid Yusuf El-Zohry – Sohag Teaching Hospital (01118391123) Page | 814
El-Zohry MRCP Questions Bank (Part 2) – Medical Masterclass 2010

E. Sumatriptan Answer & Comments


F. Pizotifen. Correct answer: C

Answer & Comments Convulsions should be treated immediately in


the usual way, without waiting for confirmation
Correct answer: AE
of the theophylline level.

The first line of treatment should be with Theophylline is metabolised by the CYP450
simple analgesics and anti-emetics, which enzymes in the liver. Erythromycin inhibits
should be taken early on at the onset of the CYP450 enzymes and increases the half-life of
attack. The triptans can be used in patients who theophylline and hence plasma theophylline
do not derive much benefit from simple concentrations, which may lead to toxicity. By
analgesics. Opiates are not recommended contrast, phenytoin induces CYP450 enzymes,
because of the potential for cognitive which will decrease the half-life of theophylline
impairment, and addiction, without any efficacy and may lead to inadequate therapeutic levels.
advantages. Amitriptyline, pizotifen and Theophylline toxicity is more likely in the
methysergide are are used for migraine elderly due to age-related reduction in the rate
prophylaxis. of its metabolism, but it can occur at any age.
Theophylline is an example of a drug with a
[ Q: 77 ] MasterClass Part2 NARROW therapeutic range. It is recommended
(2010) - Pharmacology that plasma theophylline levels be maintained
between 10 and 20 mg/l.
A 45-year-old man is taking long-term
theophylline for asthma. One evening, he is
admitted to the Accident and Emergency [ Q: 78 ] MasterClass Part2
department with convulsions. You suspect (2010) - Pharmacology
theophylline toxicity.
A 52-year-old woman, who is taking a variety of
Which one of the following statements is true? regular medications, presents with a sore
throat and fever. Her full blood count reveals
A. His convulsions should not be treated
neutropenia.
until a theophylline level is available.
B. Theophylline toxicity may have been Which of the following drugs is most likely to
precipitated by the concomitant have caused this side effect?
prescription of phenytoin. A. Captopril
C. Theophylline toxicity may have been B. Carbimazole
precipitated by the concomitant
C. Carvedilol
prescription of erythromycin.
D. Ciprofloxacin
D. Theophylline toxicity only occurs in the
elderly. E. Clomipramine
E. Theophylline is an example of a drug with
a wide therapeutic range (therapeutic Answer & Comments
index). Correct answer: B

Dr. Khalid Yusuf El-Zohry – Sohag Teaching Hospital (01118391123) Page | 815
El-Zohry MRCP Questions Bank (Part 2) – Medical Masterclass 2010

The anti-thyroid drug carbimazole may cause used in combination, or if the patient has co-
neutropenia in 1 in 800 patients. The The existing renal impairment.
commitee on safety of medicine
recommends that any patient taking [ Q: 80 ] MasterClass Part2
carbimazole should be asked to report any
(2010) - Pharmacology
symptoms or signs suggestive of infection
immediately, especially sore throat. A white A 34-year-old woman who is 8 weeks' pregnant
blood count should be performed if there is any requires antihypertensive therapy.
clinical suspicion of infection, and carbimazole
should be stopped promptly if there is Which one of the following drugs should
neutropenia. definitely not be given?

The other drugs listed above only very rarely A. Methyldopa


cause blood dyscrasia including neutropenia, B. Labetalol
hence carbimazole is the most likely cause in
this case. C. Nifedipine
D. Losartan
[ Q: 79 ] MasterClass Part2 E. Hydralazine
(2010) - Pharmacology
A 60-year-old woman is admitted feeling Answer & Comments
generally unwell. Her serum potassium is found Correct answer: D
to be elevated at 7.0 mmol/l.
Losartan is an angiotensin receptor antagonist.
Which of the following drugs is the LEAST likely
Angiotensin-converting enzyme inhibitors and
to have contributed to her hyperkalaemia?
angiotensin receptor antagonists both have
A. Lisinopril fetotoxic effects, including renal agenesis and
B. Bendroflumethiazide oligohydramnios, and are contraindicated in
pregnancy. Methyldopa, labetalol and
C. Losartan
hydralazine have a long record of safe use in
D. Spironolactone pregnancy. Although there are some concerns
about nifedipine inhibiting labour, it is generally
E. Slow K.
regarded as safe for use in pregnancy.

Answer & Comments


[ Q: 81 ] MasterClass Part2
Correct answer: B (2010) - Pharmacology

Bendroflumethiazide tends to cause A 23-year-old male has a diagnosis of


hypokalaemia. methaemoglobinaemia.

The other drugs listed may cause Which of the following statements is false?
hyperkalaemia, although this is not usually
A. Pulse oximetry is a not a good guide to
clinically significant when used alone at
his oxygen saturation.
therapeutic doses. The development of
significant drug-induced hyperkalaemia is more B. Ascorbic acid reduces the
likely when more than one of these agents is methaemoglobinaemia.

Dr. Khalid Yusuf El-Zohry – Sohag Teaching Hospital (01118391123) Page | 816
El-Zohry MRCP Questions Bank (Part 2) – Medical Masterclass 2010

C. Oral activated charcoal should be given A. Angioedema


for dapsone poisoning.
B. Ankle swelling
D. Methylene blue is required only if there
C. Constipation
are signs of toxicity.
D. Facial flushing
E. Methylene blue is a cause of
methaemoglobinaemia. E. Palpitations

Answer & Comments Answer & Comments

Correct answer: D Correct answer: A

Methaemoglobinaemia occurs when Angioedema is a rare but serious side effect of


haemoglobin is oxidised and is unable to carry ACE inhibitors (0.1-0.2% of patients) and is also
oxygen. Drug classes which cause this condition recognised as a very rare side effect of
include: angiotensin II receptor antagonists. Apart from
a few isolated case reports, it is not usually
 antibiotics (dapsone, sulphonamides
associated with calcium channel blockers.
and trimethoprim)
Side effects vary with different preparations
 nitrites and nitrates
within the family of calcium channel blocker
 local anaesthetics (lignocaine and drugs. Verapamil may cause constipation. The
prilocaine). dihydropyridines, eg amlodipine and nifedipine,
are associated with ankle swelling. The newer
Pulse oximeters measure both oxyHb and
dihydropyridine lercanidipine is thought to
metHb, therefore giving false reassurance in
cause ankle swelling less frequently. Most of
patients with high levels of metHb, whose
the calcium channel blockers may also cause
measured oxygen saturation does not
facial flushing and palpitations in some
accurately represent ability to carry oxygen to
patients.
the tissues. Treatment of patients with less
than 30% metHb is removal from the exposure,
oxygenation and ascorbic acid (directly reduces [ Q: 83 ] MasterClass Part2
the metHb). (2010) - Pharmacology
Methylene blue is used in those with severe A 48-year-old woman with a renal transplant is
poisoning or MetHb levels greater than 30%; established on ciclosporin, azathioprine and
excessive doses of methylene blue can prednisolone to prevent transplant rejection,
themselves cause methaemoglobinaemia. and enalapril and bendrofluazide for
Dapsone is adsorbed by activated charcoal. hypertension. After a 14-day course of
ketoconazole for oesophageal candidiasis her
creatinine is found to have increased from 100
[ Q: 82 ] MasterClass Part2
mmol/L to 400 mmol/L.
(2010) - Pharmacology
Her deterioration in renal function is most likely
A 57-year-old woman is being treated with a
attributable to:
calcium channel blocker drug for hypertension.
A. hypertension poorly controlled on
Which of the following is NOT a commonly enalapril and bendrofluazide
recognised side effect of calcium channel
B. nephrotoxic effects of ketoconazole
blockers?

Dr. Khalid Yusuf El-Zohry – Sohag Teaching Hospital (01118391123) Page | 817
El-Zohry MRCP Questions Bank (Part 2) – Medical Masterclass 2010

C. ciclosporin toxicity due to inhibition of Answer & Comments


ciclosporin metabolism by ketoconazole
Correct answer: A
D. transplant rejection due to induction of
ciclosporin metabolism by ketoconazole Drugs such as apomorphine and bromocriptine
E. effect of enalapril on background of cause vomiting through peripheral stimulation
stenosis of artery supplying renal of the chemoreceptor trigger zone. Worsening
transplant. of Parkinson’s disease may result from the use
of dopamine antagonists; however,
Answer & Comments domperidone is much less likely to cross the
blood–brain barrier and is therefore the
Correct answer: C preferred agent in this case. Entacapone is a
catechol-O-methyltransferase (COMT) inhibitor
Ketoconazole inhibits liver enzymes that which increases levodopa levels, thus
metabolise cyclosporin, increasing plasma worsening nausea and vomiting. Betahistine is
cyclosporin concentration. Cyclosporin is used in vertigo.
nephrotoxic; ketoconazole may be hepatotoxic.
A deterioration may also be seen in renal [ Q: 85 ] MasterClass Part2
function if the patient has severe hypertension,
(2010) - Pharmacology
which could be caused by cyclosporin and/or
prednisolone therapy, or is started on enalapril A patient with chronic Hepatitis C is referred to
on a background of renal artery stenosis. your clinic for anitiviral treatment.
However in this case the close relationship
between the timing of the course of However, interferon-alfa and ribavirin may not
ketoconazole and the increase in creatinine be suitable for those patients who:
makes option C the most likely. A. have moderate-severe disease
B. were previously intravenous drug users
[ Q: 84 ] MasterClass Part2
C. are heavy alcohol drinkers
(2010) - Pharmacology
D. are haemophiliacs and have not had a
A 70-year-old woman who has severe liver biopsy
Parkinson’s disease is prescribed co-careldopa
and apomorphine. She complains of nausea and E. have relapsed following initial response
vomiting due to her medication. to interferon-alfa monotherapy.

Which one of the following drugs would you Answer & Comments
prescribe for these symptoms?
Correct answer: C
A. Domperidone
B. Metoclopramide Combination therapy with interferon-alfa and
C. Prochlorperazine ribavirin is generally recommended for those
with moderate-sever disease (histological
D. Entacapone diagnosis of significant scarring and/or
E. Betahistine significant necrotic inflammation). While NICE
guidance suggests that problems with drug
interactions, safety, and compliance may arise
in existing intravenous drug users, those who

Dr. Khalid Yusuf El-Zohry – Sohag Teaching Hospital (01118391123) Page | 818
El-Zohry MRCP Questions Bank (Part 2) – Medical Masterclass 2010

have given up the habit should not be excluded hour after ingestion of toxins. Note that certain
from therapy. However, treatment is not drugs are not readily adsorbed by activated
generally recommended in those patients who charcoal.
consume large quantities of alcohol, given the
increased risk of liver damage. [ Q: 87 ] MasterClass Part2
In cases where a liver biopsy carries a high risk (2010) - Pharmacology
(e.g. haemophilia), treatment can be initiated
without histological confirmation. A 28-year-old man with asthma presents with
an acute attack. He is very breathless and
Both treatment-naïve patients and those who cannot complete sentences.
have relapsed following initial response to
interferon-alfa should be considered for 6 Which of the following is the best immediate
months of combination therapy. management?
A. Nebulised salbutamol (5 mg) driven with
[ Q: 86 ] MasterClass Part2 air
(2010) - Pharmacology B. Organise chest radiograph to exclude
A 30-year-old woman, with a history of previous pneumothorax
suicide attempts, presents following an C. Nebulised salbutamol (5 mg) driven with
overdose of paracetamol. She stated that she high flow oxygen via reservoir bag
has taken 8g, three hours earlier.
D. Nebulised salbutamol (50 mg) driven with
Which of the following is the most appropriate 35% oxygen
management? E. Nebulised salbutamol (5 mg) driven with
A. Take immediate plasma paracetamol 35% oxygen
levels.
B. Take no immediate action. Answer & Comments

C. Transfer to psychiatric care. Correct answer: C

D. Start intravenous N-acetylcysteine.


Beta2-agonists are pulmonary vasodilators as
E. Administer oral-activated charcoal 50g. well as bronchodilators. Their administration
can rapidly worsen the V/Q mismatch which is
Answer & Comments the cause of hypoxia in asthma. They can
therefore cause reduction in arterial oxygen
Correct answer: B
tension unless supplemental oxygen is given,
and this man should be given the highest
Paracetamol levels should be taken at least 4 inspired oxygen concentration that can be
hours after ingestion (serial levels can be taken obtained.
if timing of ingestion is in doubt). Earlier levels
It would also be reasonable to mix ipatropium
can be misleading. Psychiatric input may be
bromide (500 microg) with the salbutamol
required once the patient is medically stable.
given in option C.
It is normal to start N-acetylcysteine
immediately for patients who present late (8-24
hours), or if a significant amount has been
taken (>150mg/kg). A single dose of activated
charcoal (50g for adults) can be given up to 1

Dr. Khalid Yusuf El-Zohry – Sohag Teaching Hospital (01118391123) Page | 819
El-Zohry MRCP Questions Bank (Part 2) – Medical Masterclass 2010

[ Q: 88 ] MasterClass Part2 Appropriate management of other


(2010) - Pharmacology complications associated with PBC such as
hyperlipidaemia, steatorrhoea, and fat-soluble
A 47-year-old woman has primary biliary vitamin deficiency and portal hypertension also
cirrhosis (PBC). need to be considered.

Which of the following therapies have been Long-term ursodeoxycholic acid has been shown
shown to be beneficial in such patients? to improve survival and liver-related outcome
including development of oesophageal varices
A. Azathioprine and cirrhosis in patients with PBC. Novel agents
B. Calcitonin including chlorambucil, thalidomide, and
malotilate have not been associated with
C. Cyclosporin
significant biochemical or histological
D. Etidronate improvements. Other agents such as
E. Penicillamine azathioprine, corticosteroids, cyclosporin and
penicillamine have not been shown to be
F. Phenobarbitone effective.
G. Prednisolone
H. Rifampicin [ Q: 89 ] MasterClass Part2
I. Thalidomide
(2010) - Pharmacology

J. Ursodeoxycholic acid A 72-year-old woman with atrial fibrillation is


just about to start treatment with warfarin.
Answer & Comments Which TWO of the following drugs will increase
Correct answer: HJ her international normalized ratio (INR) if she
takes them whilst taking warfarin?
Pruritus is reported in 25-70% of patients A. Aspirin
affected by PBC and gradually resolves with B. Phenytoin
disease progression. Cholestyramine usually
reduces pruritus intensity. Rifampicin may C. Alcohol
provide symptom relief in resistant cases. D. Clopidogrel
Naloxone and nalmefene have also been shown
E. Rifampicin
to provide relief in pilot studies but
phenobarbitone and antihistamines have been F. Cimetidine
largely ineffective. Whilst metabolism of
G. Enoxaparin
calcium and Vitamin D is almost always normal
in anicteric patients, about 30% and 10% of H. Erythromycin
patients have osteopenia and osteoporosis I. Enalapril
respectively. Oral calcium with weight-bearing
J. Aminophylline
exercises is recommended for patients at risk of
osteoporosis. Oral replacement of Vitamin D is
indicated in the presence of fat-soluble vitamin Answer & Comments
malabsorption or when serum concentrations
Correct answer: FH
are reduced. Calcitonin, sodium fluoride and
etidronate have not been conclusively shown to
improve bone density.

Dr. Khalid Yusuf El-Zohry – Sohag Teaching Hospital (01118391123) Page | 820
El-Zohry MRCP Questions Bank (Part 2) – Medical Masterclass 2010

Cimetidine and erythromycin inhibit the activity thromboembolic disease but would not alter
of Cytochrome p450 enzymes in the liver, thus the INR. Aspirin inhibits cyclooxygenase and
reducing warfarin metabolism and increasing prostaglandin production and increases the risk
the INR. Sodium valproate and ciprofloxacin of bleeding in a patient receiving warfarin, but
also inhibit liver enzymes. it would not affect the INR.
Phenytoin, alcohol and rifampicin induce
cytochrome p450 activity and hence will [ Q: 91 ] MasterClass Part2
increase warfarin metabolism, reducing the (2010) - Pharmacology
INR.
A 58-year-old gentleman with elevated
Bleeding is more likely in patients taking cholesterol has failed to reach a desired
warfarin if they also take aspirin, clopidogrel or cholesterol level on statin treatment. You
enoxaparin, but these drugs interact with decide to commence him on ezetimibe.
warfarin through an additive anticoagulant
effect and not by altering warfarin metabolism. Which of the following is true concerning
ezetimibe?
[ Q: 90 ] MasterClass Part2 A. Prescription with statin treatment is
(2010) - Pharmacology contraindicated.

A 77-year-old man on warfarin for venous B. Decreased absorption of fat soluble


thromboembolic disease is admitted with a vitamins is an unwanted effect.
second deep vein thrombosis despite taking C. Its main action is to prevent cholesterol
warfarin. His INR is 1.2. synthesis by the liver.

Which of the following medicines when co- D. It causes an elevation in plasma


administered with warfarin is most likely to triglyceride concentrations.
have caused this clinical picture? E. It causes a reduction in low-density
A. Aspirin lipoprotein (LDL)-cholesterol of
approximately 20%.
B. Cimetidine
C. Cholestyramine Answer & Comments
D. Cyproterone acetate
Correct answer: E
E. Erythromycin
Ezetimibe is the first of a novel class of drugs
Answer & Comments for the treatment of hyperlipidaemia whose
main action is to specifically prevent cholesterol
Correct answer: C
absorption from the small intestine. Typically it
reduces LDL-cholesterol by approximately 20%,
Cholestyramine can reduce warfarin triglycerides by up to 5% and raises HDL-
absorption, thereby preventing effective cholesterol by approximately 5%. It does not
anticoagulation and lowering of the INR. The inhibit the absorption of fat-soluble vitamins
INR would be increased by co-prescription of unlike the anion-exchange resins (e.g.
cimetidine and erythromycin, which are colestyramine). Ezetimibe can be safely co-
cytochrome P450 inhibitors. Cyproterone administered with statins and is currently
acetate (an antiandrogen used for prostate licensed for use in combination with a statin in
cancer) carries a risk of recurrence of patients who fail to reach desired lipid profiles

Dr. Khalid Yusuf El-Zohry – Sohag Teaching Hospital (01118391123) Page | 821
El-Zohry MRCP Questions Bank (Part 2) – Medical Masterclass 2010

or as monotherapy in patients intolerant to a E. Prazosin


statin. There is no increased risk of myopathy
F. Sumatriptan
with ezetimibe prescription.
G. Nifedipine

[ Q: 92 ] MasterClass Part2 H. Carbimazole


(2010) - Pharmacology I. Carbamazepine.
A 58-year-old patient with metastatic
carcinoma of the breast has good pain control Answer & Comments
on MST 180mg bd. She is admitted with Correct answer: AF
increasing weakness and has difficulty
swallowing her tablets. It is therefore decided
Sildenafil (or viagra), a phosphodiesterase type
to convert her to a 24-hour diamorphine
5 inhibitor, is associated with angina pectoris.
syringe driver.
Catastrophic hypotension, myocardial infarction
The correct dose of diamorphine is: and death have occured in patients taking
viagra in conjunction with nitrates so this drug
A. 30mg with 5mg prn. combination is absolutely contraindicated.
B. 60mg with 10mg prn. Sumatriptan is a 5HT1 agonist used in acute
C. 60mg with 5mg prn. migraine which causes vasospasm. It is
contraindicated in patients with ischaemic
D. 120mg with 10mg prn.
heart disease as it can cause coronary
E. 120mg with 20mg prn. vasospasm.
Drugs which increase heart rate such as
Answer & Comments adrenergic drugs (salbutamol, isoprenaline) or
Correct answer: E rapid thyroid replacement, and drugs which
cause a drop in blood pressure (e.g. misoprostil)
may also worsen existing angina.
The dose of diamorphine should be 1/3 of the
total 24-hour dose of morphine. The prn dose
should be 1/6 of the 24 hour dose of [ Q: 94 ] MasterClass Part2
diamorphine. (2010) - Pharmacology
A 54-year-old man with renal cell carcinoma
[ Q: 93 ] MasterClass Part2 and bone metastases is admitted with
(2010) - Pharmacology confusion and constipation. He is on MST
100mg bd and a non-steroidal anti-
A 48-year-old man is admitted with worsening
inflammatory drug.
angina.
How would you manage him?
Which TWO of the following drugs are most
likely to have precipitated his condition? A. Avoid intrusive investigations and treat
symptoms as they arise
A. Sildenafil
B. Stop opiates which may be the cause of
B. Nicorandil
his symptoms
C. Methotrexate
C. Reduce opiates
D. Piroxicam
D. Change opiates

Dr. Khalid Yusuf El-Zohry – Sohag Teaching Hospital (01118391123) Page | 822
El-Zohry MRCP Questions Bank (Part 2) – Medical Masterclass 2010

E. Check serum calcium. ml/min (0.1 mg/min) until a response has been
obtained (or a total of 0.5 mg - 5 ml - has been
Answer & Comments given).

Correct answer: E
[ Q: 96 ] MasterClass Part2
(2010) - Pharmacology
Hypercalcaemia may well cause these
symptoms and should always be checked unless A 75-year-old woman with metastatic
a patient is dying. Another possible metabolic carcinoma of the colon is admitted semi-
cause of this presentation is renal failure. conscious and dying. Her symptoms had been
Opiates rarely cause confusion in the absence previously well controlled on oxycodone SR
of renal failure or overdose for other reasons. 80mg bd.

What would you do about analgesia?


[ Q: 95 ] MasterClass Part2
(2010) - Pharmacology A. Nothing at present as she is semi-
conscious and not obviously in pain
A 27-year-old woman develops difficulty
B. Chart prn oxycodone orally
breathing and her lips and tongue swell about
five minutes after starting to eat a curry. She is C. Change to im morphine
brought to the accident and emergency
D. Chart prn paracetamol pr
department by ambulance. She is cyanosed and
wheezing. E. Start a syringe driver with diamorphine

Aside from high flow oxygen via a reservoir bag, Answer & Comments
which of the following treatments would be
your top priority? Correct answer: E

A. Hydrocortisone 200 mg intravenously


A good strategy is to anticipate problems and
B. Chlorpheniramine 10 mg intravenously avoid the patient developing pain. A syringe
C. Epinephrine (adrenaline) - 0.5 ml of driver with the correct dose of diamorphine and
1/1000 solution intravenously 1/6 dose as breakthrough medication is correct.

D. Epinephrine (adrenaline) - 0.5 ml of


1/1000 solution intramuscularly [ Q: 97 ] MasterClass Part2
(2010) - Pharmacology
E. Salbutamol 5 mg nebulized.
A cachectic 87-year-old man is admitted with
Answer & Comments acute urinary retention and is found to have a
creatinine of 520 micro mol/l. He has been on a
Correct answer: D
number of medications.

The history clearly suggests anaphylaxis and Which of the following is least likely to be
treatment with intramuscular epinephrine (0.5 nephrotoxic in this situation?
ml of 1/1000) is required. A. Ibuprofen
In extremis, epinephrine can be given B. Ramipril
intravenously, but at reduced dosage: make a
1/10,000 solution (by diluting 1 ml of 1/1000 to C. Allopurinol
10 ml with 0.9% saline) and give this at 1
Dr. Khalid Yusuf El-Zohry – Sohag Teaching Hospital (01118391123) Page | 823
El-Zohry MRCP Questions Bank (Part 2) – Medical Masterclass 2010

D. Lansoprazole Answer & Comments


E. Sulfasalazine. Correct answer: C

Answer & Comments Tapering and discontinuation of psychotropic


drugs including benzodiazepines and
Correct answer: D
antidepressants has been shown to reduce the
incidence of falls.
Given muscle wasting, a serum creatinine this
high indicates very severe renal impairment. Reducing the number of medications to 4 or
This should improve with catheterisation and a less has been shown to reduce the number of
good fluid input, but it is essential to stop all falls in older people.
potentially nephrotoxic drugs. Do not rely on Some older people with postural hypotension
your memory -always check with the BNF as are asymptomatic.
continuing a nephrotoxic drug at this stage may
Falls account for 6% of acute hospital
lead to irreversible renal failure. Of this list, only
admissions.
lansoprazole should be continued until his renal
function has improved.
[ Q: 99 ] MasterClass Part2
[ Q: 98 ] MasterClass Part2 (2010) - Pharmacology
(2010) - Pharmacology A 72-year-old Caucasian woman in your
outpatient clinic has uncomplicated essential
An 87-year-old lady is admitted to hospital
hypertension. Her blood pressure is
having had a fall. On enquiry she has had four
162/102mmHg despite optimization of non-
falls over the last 6 weeks. Her medications on
pharmacological therapy.
admission included amitriptyline 75mg od,
temazepam 20mg nocte, diazepam 2mg tds,
Which one of the following treatments would
bendrofluazide 2.5mg od, captopril 25mg tds,
you choose as the first-line agent for her?
ranitidine 150 mg od and amlodipine 10mg od.
A. Atenolol 50 od
Which of the following statements are correct? B. Bendrofluazide 2.5mg od
A. Tapering and discontinuation of
C. Bendrofluazide 5mg od
benzodiazepines has not been shown to
reduce falls. D. Enalapril 5mg od

B. Tapering and discontinuation of tricyclic E. Ramipril 2.5mg od.


antidepressants has not been shown to
reduce falls. Answer & Comments
C. Reducing the total number of Correct answer: B
medications to 4 or less reduces the risk
of falling.
Hypertension is particularly common in those
D. All older patients with postural aged above 60 not least because of the steady
hypotension are symptomatic. rise in systolic blood pressure with age. These
patients are at a high absolute risk of
E. Falls account for upto 20% of acute
cardiovascular complications. Furthermore,
hospital admissions.
anti-hypertensive treatment may also reduce

Dr. Khalid Yusuf El-Zohry – Sohag Teaching Hospital (01118391123) Page | 824
El-Zohry MRCP Questions Bank (Part 2) – Medical Masterclass 2010

incidence of heart failure and possibly Answer & Comments


dementia.
Correct answer: BE
Non-pharmacological therapy for hypertension
should always be optimised prior to Cremation is customary for Hindus, Buddhists
commencement of medication, whenever
and Sikhs (within 24 hours for the latter faith).
possible. Low-dose diuretics are accepted as Burial is usual within Chinese religious faiths
the first-line treatment for hypertension in the while people of Christians faith are increasingly
elderly and appear to confer greater benefit cremated rather than buried.
than Beta-adrenergic receptor antagonists in
this subgroup. 1. Treatment of isolated systolic You will be ignorant of the religious beliefs of
hypertension in the elderly with the long-acting many of your patients and will not know how
calcium channel blocker, nitrendipine, has been they and their families think that the body
shown to reduce stroke and cardiovascular should be handled after death. Be aware that
outcome. Therefore, calcium channel blockers this might be a very important issue for
may be suitable when diurectics are not families, and put them in touch with the
tolerated, ineffective or contra-indicated relevant office in the hospital (who will be
aware of these matters) in an efficient and
timely manner
[ Q: 100 ] MasterClass Part2
(2010) - Pharmacology
[ Q: 101 ] MasterClass Part2
A 72-year-old man, originally from the Indian (2010) - Pharmacology
subcontinent, is admitted to the ward with
pneumonia as a terminal complication of lung A 35-year-old man presents with pneumonia
cancer. He dies on the ward. His relatives are requiring antibiotic treatment, but is noted to
extremely concerned that the body should be suffer from acute intermittent porphyia.
released immediately for burial.
Which of the following antibiotics would be the
In which TWO of the following religious faiths is best treatment choice?
it customary to be buried within 24 hours of A. Cephradine
death?
B. Amoxicillin
A. Christianity
C. Erythromycin
B. Islam
D. Cefotaxime
C. Hinduism
E. Flucloxacillin.
D. Buddhism
E. Judaism Answer & Comments
F. Chinese religious customs Correct answer: B
G. Sikhism
H. Jehovah's witnesses Prescribing in patients with acute porphyrias
can be difficult, since drug classes or individual
I. Mormons drugs can precipitate acute attacks. The
J. Baptists. antibiotic classes of cephalosporins and
sulphonamides precipitate acute attacks.
Additionally, small changes in chemical
structures of drugs can change the

Dr. Khalid Yusuf El-Zohry – Sohag Teaching Hospital (01118391123) Page | 825
El-Zohry MRCP Questions Bank (Part 2) – Medical Masterclass 2010

porphogenicity of drugs within a class swelling. Dexamethasone may also work by


(amoxicillin is safe, but flucloxacillin is not). reducing brain swelling through blocking an
Further information is available from the Welsh inflammatory cytokine pathway. Trials of the
Medicines Information Centre in Cardiff. other agents listed above failed to demonstrate
any benefit.
[ Q: 102 ] MasterClass Part2
(2010) - Pharmacology [ Q: 103 ] MasterClass Part2
(2010) - Pharmacology
A 35-year-old man is planning a trekking holiday
in the Himalayas. Unfortunately, he was A patient is taking morphine sulphate
afflicted by acute mountain sickness on his continuous (MST) 240 mg b.d. for carcinoma of
previous holiday to Colorado, USA. the prostate with painful bone metastases.

He would like to take some medication What dose of oramorph for breakthrough pain
prophylactically, and wonders which TWO of would you prescribe?
the following agents have strong evidence to
A. 20 mg
support their use:
B. 40 mg
A. Phenytoin
C. 60 mg
B. Furosemide
D. 80 mg
C. Acetazolamide
E. 100 mg
D. Antidiuretic hormone
E. Codeine Answer & Comments
F. Nifedipine
Correct answer: D
G. Dexamethasone
H. Dihydroxyaluminium-sodium The breakthrough dose of short acting
morphine should be 1/6th of the total 24-hour
I. Metformin
dose. There should be no time limit on the
J. Spironolactone. prescription.

Answer & Comments [ Q: 104 ] MasterClass Part2


Correct answer: CG (2010) - Pharmacology
A 48-year-old man of African origin is found to
The patient’s past history of acute mountain have blood pressure 200/110 mmHg at a
sickness is an important susceptibility factor, routine medical. Fundoscopy shows AV nipping
and he should therefore consider prophylactic but no more severe hypertensive changes.
therapy. A systematic review has shown that Urinalysis is negative for blood and protein.
acetazolamide, as well as dexamethasone, can
be used prophylactically. Acetazolamide (a Which of the following management options is
carbonic anhydrase inhibitor) promotes most appropriate?
excretion of sodium, potassium, water and A. Oral atenolol 50mg od if hypertension
bicarbonate. It is used to reduce intraocular confirmed over 1–2 weeks
pressure in glaucoma – the effects on mountain
sickness may be through reduction in brain

Dr. Khalid Yusuf El-Zohry – Sohag Teaching Hospital (01118391123) Page | 826
El-Zohry MRCP Questions Bank (Part 2) – Medical Masterclass 2010

B. Oral enalapril 10mg bd if hypertension D. Nifedipine


confirmed over 1–2 weeks
E. Ranitidine
C. Urgent admission to hospital for control
F. Bendrofluazide
of hypertension
G. Digoxin
D. Oral nifedipine capsules 20mg tds if
hypertension confirmed over 1–2 weeks H. Pravastatin

E. Oral nifedipine LA 30mg od if I. Glibenclamide


hypertension confirmed over 1–2 weeks J. Spironolactone.

Answer & Comments Answer & Comments


Correct answer: E Correct answer: BG

The diagnosis of accelerated hypertension Amiodarone inhibits CYP 3A4 and will increase
requires the finding of fundal haemorrhages the plasma concentrations of drugs such as
and exudates, with or without papilloedema, as ciclosporin, phenytoin, verapamil and warfarin.
manifestations of fibrinoid necrosis. This This may cause bradycardia, AV block and
patient has high blood pressure but no myocardial depression if verapamil and
evidence of accelerated hypertension. amiodarone are used together in treating atrial
BTS guidelines suggest that the finding of blood fibrillation, thus great care is needed in
pressure 200–219/110–119mmHg should be adjusting the dose. There is a similar interaction
confirmed with repeat measurements after 1–2 between digoxin and amiodarone, and it is
weeks, then treated if still elevated. recommended that the digoxin dose be halved
in those taking both agents.
Black patients have low renin hypertension,
hence ACE inhibitors and beta-blockers as
single agents do not lower blood pressure in [ Q: 106 ] MasterClass Part2
this group. Calcium channel blockers and (2010) - Pharmacology
diuretics are effective agents. Nifedipine should
It has been decided that a patient should be
be used as a long-acting preparation, not a
changed from MST 100mg bd to the equivalent
short acting one.
transdermal fentanyl dose.

[ Q: 105 ] MasterClass Part2 Choose the correct dose from those shown
(2010) - Pharmacology below
A. 125ug/hr
You see a patient with poorly-controlled atrial
fibrillation and decide to start him on B. 25ug/hr
amiodarone.
C. 100ug/hr
There is a potentially serious drug interaction D. 50ug/hr
whereby amiodarone increases the plasma
E. 75ug/hr.
concentrations of which TWO of the following:
A. Aspirin Answer & Comments
B. Verapamil
Correct answer: D
C. Ramipril

Dr. Khalid Yusuf El-Zohry – Sohag Teaching Hospital (01118391123) Page | 827
El-Zohry MRCP Questions Bank (Part 2) – Medical Masterclass 2010

Fentanyl Conversion Table E. Physical treatment modalities do not


have a role in fall-related anxiety
Oral 24 hour morphine dose Fentanyl ug/hr management.
Less than 135 mg 25
Answer & Comments
135-224 50
Correct answer: C
225-314 75

315-404 100 Around one third of people over the age of 65


will fall every year, most often during everyday
405-494 125
essential activities.
495-584 150
The psychological consequences of falling are
585-674 175 often overlooked and include depression,
generalised anxiety and fear of falling. Fear of
675-764 200 falling can be present even when there have
765-854 225 been no falls-related injuries, and can itself be a
predictor of functional decline, loss of
855-944 250 independence and future falls. In the short
945-1034 275 term worrying about falls leads to distraction,
increased muscle tension and poor posture (as
1035-1124 300 in this case). In the long term the associated
reduction in activity leads to reduced body
strength, which will increase the risk of falling
[ Q: 107 ] MasterClass Part2
when doing everyday activities.
(2010) - Pharmacology
Interventions to manage fear of falling should
A GP refers an elderly woman to your day be integrated into a broad, multidisciplinary
hospital for rehabilitation. She has been approach, the aim being to manage fear rather
reluctant to go out for some time. She walks than eradicate it. Measures include education
slowly with a stooped posture. She fell in her to understand risk factors for falling, and
garden 4 months previously and has been reduction of the risk using exercise
terrified of falling ever since. There are no programmes, environmental assessment and
features of Parkinsonism on examination. relaxation and breathing techniques. Specific
cognitive-behavioural techniques include
Which one of these options is correct?
practised methods of dealing with negative
A. Roughly 20% of people aged over 65 thoughts, and having reminder lists of how to
years will fall each year 'walk well'.
B. Falls tend to occur when carrying out
hazardous, ambitious tasks [ Q: 108 ] MasterClass Part2
C. Fear of falling can occur independently of (2010) - Pharmacology
previous falls / fall related injuries A 58-year-old man, you are seeing in the
D. The psychological consequences of falling Medical Assessment Unit is dependent on
are independent of physical risk factors opiates and alcohol.
for falling
Which 2 statements are correct?
A. There is no point aiming at abstinence

Dr. Khalid Yusuf El-Zohry – Sohag Teaching Hospital (01118391123) Page | 828
El-Zohry MRCP Questions Bank (Part 2) – Medical Masterclass 2010

B. Substitute medication such as methadone [ Q: 109 ] MasterClass Part2


or buprenorphine have been shown to be (2010) - Pharmacology
ineffective
58-year-old with carcinoma of the ovary has
C. Detoxification can be expected to result
been taking morphine sulphate continus (MST)
in long-term benefits
200mg b.d. for 3 months. She presents with a 2-
D. Buprenorphine is probably as effective as day history of twitching and drowsiness.
methadone and safer in overdose Examination reveals her to be fluid overloaded
because it has partial agonist/antagonist and have pin point pupils.
effects
What is the most likely cause of her symptoms?
E. The usual duration of methadone
substitution is 3 months A. Acute renal failure

F. More than half of homeless individuals B. Morphine overdose


have substance misuse problems C. Renal failure leading to accumulation of
G. This patient is no more likely than morphine
another of the age and socio-economic D. Brain secondaries
group to be attending a sexual health
clinic E. Right sided cardiac failure.

H. Remaining in treatment is not a good


Answer & Comments
predictor of outcome
I. The methadone dose should be the lowest Correct answer: C
tolerated
Patients with relapsed ovarian cancer may
J. Methadone should be given orally, twice a
develop an obstructive nephropathy due to
day
pelvic recurrence. If they are on morphine they
may get accumulation of this drug and signs of
Answer & Comments opiate toxicity superimposed on the signs of
Correct answer: DF renal failure.

The aim in treating drug and alcohol abusers is [ Q: 110 ] MasterClass Part2
abstinence. Detoxification is often needed prior (2010) - Pharmacology
to further treatment but does not alone lead to
a 60-year-old man who is on a diuretic, ACE
abstinence. Substitution regimens work best
inhibitor and beta-blocker because of heart
when combined with counselling or behavioural
failure following a recent myocardial infarction.
therapies. Methadone substitution is usually
However, he now complains of several episodes
needed for at least 12 months. People
of rapid fluttering in his chest, which makes him
attending sexual health clinics have high rates
breathless.
of drug and alcohol abuse. Higher doses of
methadone may be more effective than lower
You suspect paroxysmal atrial fibrillation and
doses, but daily treatment is usually adequate. consider adding either of the following TWO
medications:
A. d-Sotalol
B. Nifedipine

Dr. Khalid Yusuf El-Zohry – Sohag Teaching Hospital (01118391123) Page | 829
El-Zohry MRCP Questions Bank (Part 2) – Medical Masterclass 2010

C. Verapamil C. Dextrose (50 ml of 50% solution)


intravenously, repeated if no effect
D. Lignocaine
D. Flumazenil (0.2 mg) intravenously,
E. Digoxin
repeated if no effect
F. Procainamide
E. Insert stomach tube (after securing
G. Adenosine airway) and give activated charcoal.
H. Amiodarone
Answer & Comments
I. Quinidine
Correct answer: A
Answer & Comments
The working diagnosis must be opioid overdose,
Correct answer: EH
the treatment for which is intravenous
naloxone (0.4 mg), repeated up to a total dose
It can be very difficult to manage atrial
of 2 mg depending on clinical response.
fibrillation in patients who have a history of
ischaemic heart disease and heart failure. Class The half-life of naloxone is shorter than that of
I anti-arrhythmic agents such as procainamide opioids, hence if this man wakes up it can be
and quinidine appear to increase the risk of anticipated that he will 're-narcose'. A naloxone
sudden death in patients with LV dysfunction, infusion may be necessary.
probably through a pro-arrhythmic effect. This
increased risk of sudden death was also found [ Q: 112 ] MasterClass Part2
with d-sotalol when used in patients with heart (2010) - Pharmacology
failure. Nifedipine has no beneficial effect on
atrial fibrillation, and verapmil may worsen 58-year-old lady with advanced breast cancer is
heart failure if used in this patient who is admitted with abdominal pain and constipation
already on a beta-blocker. Lignocaine and secondary to opioids.
adenosine are only used intravenously, and do
not work in paroxysmal atrial fibrillation. Which of the following is the laxative of choice?
Digoxin (for rate control) or amiodarone (to A. Sodium Picosulphate
reduce the frequency of attacks) may be
B. Fybogel
potentially suitable in this case.
C. Senna
[ Q: 111 ] MasterClass Part2 D. Codanthramer
(2010) - Pharmacology E. Lactulose
man is brought to the Emergency Department
by ambulance. He is unconscious (GCS 5), with Answer & Comments
pin-point pupils and a slow respiratory rate.
Correct answer: A
Immediate specific treatment should be:
Bulk forming drugs such as fybogel have little to
A. Naloxone (0.4 mg) intravenously,
offer in opioid-induced constipation.
repeated if no effect
Senna can cause abdominal cramps.
B. N-acetyl cysteine (150 mg/kg over 15
min) intravenously, then 50 mg/kg over 4 Codanthramer can cause skin burns in faecal
hours, then 100 mg/kg over 16 hours incontinence. This patient is frail and may not

Dr. Khalid Yusuf El-Zohry – Sohag Teaching Hospital (01118391123) Page | 830
El-Zohry MRCP Questions Bank (Part 2) – Medical Masterclass 2010

be able to clean herself well and is therefore at B. Enalapril


risk of this most unpleasant complication.
C. Spironolactone
Lactulose can cause bowel distension and
D. Paracetamol
increased abdominal cramps.
E. Warfarin

[ Q: 113 ] MasterClass Part2


Answer & Comments
(2010) - Pharmacology
Correct answer: B
patient presents with acute dystonia and
oculogyric crisis after being treated with
metoclopramide. Enalapril is a prodrug and requires metabolism
to enalaprilat by the liver for activation.
Which statement is true with regards to this Diazepam is also metabolised by the liver to an
adverse drug reaction? active metabolite. However, diazepam itself is
active and liver failure will delay the elimination
A. It occurs only after long-term use of
of diazepam as well as warfarin, thereby
metoclopramide.
increasing their activity. Paracetamol activity
B. It is most common in middle-aged men. will be unaffected by liver failure, although its
C. It can persist for several days after metabolism will be reduced and the British
withdrawal. National Formulary advises avoidance of large
doses. Aldosterone metabolism is reduced in
D. It does not occur with prochlorperazine. liver failure, causing salt and water retention
E. It is best treated with procyclidine. and contributing to ascites and oedema, which
can be treated by spironolactone.
Answer & Comments
[ Q: 115 ] MasterClass Part2
Correct answer: E
(2010) - Pharmacology
Dystonic reactions are well-recognized with 40-year-old woman has developed haemolytic
dopamine receptor antagonists. They occur anaemia secondary to drug therapy.
shortly after starting therapy, particularly in
girls and young women as well as the elderly. Which of the following drugs is NOT a well-
The problem usually subsides within 24 hours recognised cause of haemolytic anaemia?
following cessation of treatment and can be A. Phenoxymethylpenicillin
treated with procyclidine 5-10 mg i.m.
B. Mefenamic acid
(antimuscarinic).
C. Methyldopa
[ Q: 114 ] MasterClass Part2 D. Ranitidine
(2010) - Pharmacology E. Rifampicin.
A 55-year-old man with liver failure is on a
number of medications. Answer & Comments

Which of the following will be less effective as a Correct answer: D


result of his illness?
Several drugs cause haemolytic anaemia, by a
A. Diazepam
variety of mechanisms. Penicillin binds

Dr. Khalid Yusuf El-Zohry – Sohag Teaching Hospital (01118391123) Page | 831
El-Zohry MRCP Questions Bank (Part 2) – Medical Masterclass 2010

covalently to the red cell membranes; The important features are the reduced level of
rifampicin causes immune complex association consciousness and a severe metabolic acidosis
with red cell membranes leading to with attempted respiratory compensation.
complement activation; methyldopa and
His normal cardiovascular observations argue
mefenamic acid may induce the formation of
against heart failure, pulmonary embolism (PE)
autoantibodies against components of red cells.
or severe sepsis (although the latter should not
Ranitidine does not commonly cause
be definitively excluded). Normal renal and liver
haemolytic anaemia
function exclude those as causes. He is not
diabetic and the alcohol level in his blood is not
[ Q: 116 ] MasterClass Part2 extremely high.
(2010) - Pharmacology The clinical picture would suggest poisoning
A 19-year-old medical student presents to the with salicylates, methanol or ethylene glycol.
Emergency Department with a Glasgow Coma Plasma salicylate levels are quick and easy to
Score of 12 and smelling of alcohol. His friends do. The clue for methanol or ethylene glycol
say he was at a party last night where he was poisoning is to do the anion gap - if there is a
mixing cocktails. high anion gap there must be an unmeasured
Initial observations show heart rate 64/min, BP anion present. The treatment for methanol and
120/60 mmHg and respiratory rate 28/min. ethylene glycol poisoning is ethanol infusion.
Initial investigations show normal urea and Serum lactate, full blood count, CT scan, lumbar
electrolytes, normal glucose and normal liver puncture and clotting may all be needed in
function tests. His blood alcohol level is selected patients, especially if sepsis is possible.
120mg/dl (limit for driving is 80mg/dl). Arterial
blood gases (on air) are paO2 12.6 kPa, paCO2
[ Q: 117 ] MasterClass Part2
2.3 kPa, pH7.16, standard base excess -14.
(2010) - Pharmacology
Which TWO of the following investigations
A colleague seeks your opinion on a patient
would be most helpful in making the diagnosis?
who has developed abnormal thyroid function
A. Serum prothrombin time tests after being started on amiodarone
B. Plasma paracetamol level recently.

C. Urinary toxicology Which one of the following features, in


conjunction with clinical symptoms and signs, is
D. Electrocardiogram
helpful in diagnosing overt hypothyroidism?
E. Chest radiograph
A. Increase in thyroid-stimulating hormone
F. Plasma salicylate level (TSH) up to 20mU/L
G. CT scan of his head B. Decrease in T3
H. Measurement of the anion gap C. Elevated free T4 and T3
I. Full blood count D. T4 at upper end of or just above normal
J. Lumbar puncture. range
E. Low free T4, and low T3.
Answer & Comments
Answer & Comments
Correct answer: FH
Correct answer: E
Dr. Khalid Yusuf El-Zohry – Sohag Teaching Hospital (01118391123) Page | 832
El-Zohry MRCP Questions Bank (Part 2) – Medical Masterclass 2010

Amiodarone is an iodine rich structure which /day) This is a high iodine load for body which
resembles T4. Daily dose 200mg generates 7mg blocks further thyroid iodide uptake and
free iodine (WHO optimal intake 0.15-0.3mg hormone synthesis. It also blocks conversion of
/day) This is a high iodine load for body which T4 to T3 and affects pituitary thyroid axis. The
blocks further thyroid iodide uptake and following changes in thyroid function tests
hormone synthesis. It also blocks conversion of occur within 3 months of starting amiodarone
T4 to T3 and affects pituitary thyroid axis. The and are not indicative of thyroid disease:
following changes in thyroid function tests increase in TSH up to 20mU/L, increase in T4 to
occur within 3 months of starting amiodarone upper limit of normal range, and decreased T3
and are not indicative of thyroid disease: levels.
increase in TSH up to 20mU/L, increase in T4 to
Diagnosis of hypothyrodism should be based on
upper limit of normal range, and decreased T3
clinical assessment, together with the following
levels.
features: high TSH - > 20mU/l, low free T4, low
Diagnosis of hypothyrodism should be based on T3. Treatment is with thyroxine and we should
clinical assessment, together with the following aim for free T4 levels close to upper range.
features: high TSH - > 20mU/l, low free T4, low
T3. Treatment is with thyroxine and we should [ Q: 119 ] MasterClass Part2
aim for free T4 levels close to upper range.
(2010) - Pharmacology

[ Q: 118 ] MasterClass Part2 You see a 36-year-old man in outpatients who is


complaining of acid reflux and heartburn.
(2010) - Pharmacology
A colleague seeks your opinion on a patient Which agents have been shown in clinical trials
who has developed abnormal thyroid function to be efficacious in the treatment of
tests after being started on amiodarone gastrooesophageal reflux disease?
recently. A. Mebeverine

Which one of the following features, in B. Lansoprazole


conjunction with clinical symptoms and signs, is C. Peppermint oil
helpful in diagnosing overt hypothyroidism?
D. Misoprostol
A. Increase in thyroid-stimulating hormone
E. Sucralfate
(TSH) up to 20mU/L
F. Prochlorperazine
B. Decrease in T3
C. Elevated free T4 and T3 G. Carbenoxolone

D. T4 at upper end of or just above normal H. Bismuth


range I. Hyoscine
E. Low free T4, and low T3. J. Ranitidine.

Answer & Comments Answer & Comments

Correct answer: E Correct answer: BJ

Amiodarone is an iodine rich structure which Proton-pump inhibitors are the most effective
resembles T4. Daily dose 200mg generates 7mg agents in gastrooesophageal reflux disease and
free iodine (WHO optimal intake 0.15-0.3mg
Dr. Khalid Yusuf El-Zohry – Sohag Teaching Hospital (01118391123) Page | 833
El-Zohry MRCP Questions Bank (Part 2) – Medical Masterclass 2010

should be used as first-line therapy in patients


with severe symptoms (with subsequent dose
reduction later) and in those who have
complications such as stricture, ulceration or
haemorrhage.
Patients with mild or occasional symptoms can
be treated in a step-up manner, starting with
over-the-counter preparations, such as
antacids. The next step would be to try a H2
antagonist such as ranitidine, and failing that,
to move up to a proton-pump inhibitor.

[ Q: 120 ] MasterClass Part2


(2010) - Pharmacology
72 yr old man with hypertension has blood
pressure well controlled (134/84 mmHg) on
bendroflumethiazide 2.5 mg/d, bisoprolol 5
mg/d and lisinopril 20 mg/d. A routine blood
test taken in primary care reveals Na 135
mmmol/l, K 5.2 mmol/l, Creatinine 112
micromol/l.

Which of the following is the most appropriate


course of action?
A. Arrange urgent admission to hospital for
treatment of hyperkalaemia.
B. Reassure that no immediate changes in
management are required.
C. Start calcium resonium.
D. Stop bendroflumethiazide.
E. Stop lisinopril.

Answer & Comments

Correct answer: B

Mild chronic hyperkalaemia is well tolerated


and not a cause for concern. If serum potassium
were to rise to >6.0 mmol/l, standard practice
would be to stop the ACEi and - if K >6.0 mmol/l
were to persist - to advise a low potassium diet.

Dr. Khalid Yusuf El-Zohry – Sohag Teaching Hospital (01118391123) Page | 834
El-Zohry MRCP Questions Bank (Part 2) – Medical Masterclass 2010

2. Decreased plasma sodium

Basic Science concentration and osmolality


3. Inappropriately high urine sodium
concentration (>20 mmol/l) and
(7 Questions) osmolality (> plasma)

(Medical Masterclass – Part 2) 4. There is normal adrenal, thyroid and


renal function.

[ Q: 2 ] MasterClass Part2
[ Q: 1 ] MasterClass Part2 (2010) – Basic science
(2010) – Basic science
A 70-year-old man presents with
A 78-year-old man is admitted weak and unable breathlessness. He has had a history of previous
to stand after vomiting for several days. His myocardial infarction and peripheral vascular
plasma sodium concentration is 123 mmol/l disease. He has recently been started on
and his urinary sodium concentration is 8 captopril for hypertension. He has been anuric
mmol/l. for 1 hour. His serum urea is 45 mmol/l and his
potassium is 7.9 mmol/l. His ECG shows
What is the likely cause of his hyponatraemia?
features compatible with hyperkalaemia.
A. Syndrome of inappropriate antidiuresis
(SIADH) The first most appropriate step in management
would be:
B. Diuretic treatment
A. ten units of soluble insulin with 50g
C. Loss of sodium in vomit
glucose(50%) IV
D. Hypovolaemic stimulation of ADH release
B. haemodialysis
E. Addison’s disease.
C. bicarbonate (100 mls of a 4.2% solution)
by IVI
Answer & Comments
D. calcium resonium 30g
Correct answer: D
E. 10-30 ml calcium gluconate (10%) IVI.

Plasma ADH concentration is normally


Answer & Comments
controlled by plasma osmolality, but pain,
nausea, hypovolaemia and anaesthesia are all Correct answer: E
powerful stimulants of ADH release and can
generate much higher plasma levels than are Giving calcium gluconate IV will instantly lower
seen in response to tonicity. Nausea and myocardial excitability and should produce very
hypovolaemia are likely to have stimulated very quick ECG improvement and hopefully prevent
high ADH levels as the explanation for this sudden cardiac arrest - it will not affect
man’s hyponatraemia. potassium levels. Other measures listed above
In recognition of these facts, SIADH can only be can be used to promote potassium excretion
diagnosed when the following criteria are but may take several hours to become
satisfied: effective. The patient may have renal failure
precipitated by the use of angiotensin-
1. The patient is clinically euvolaemic (JVP
seen, no postural hypotension)

Dr. Khalid Yusuf El-Zohry – Sohag Teaching Hospital (01118391123) Page | 835
El-Zohry MRCP Questions Bank (Part 2) – Medical Masterclass 2010

converting enzyme (ACE) inhibitors with patient to suck. Moist swabs can also be used
underlying renal artery stenosis. to relieve unpleasant dryness of the mouth.
Hyponatraemia of rapid onset (which is usually
[ Q: 3 ] MasterClass Part2 iatrogenic, being caused by inappropriate
(2010) – Basic science intravenous administration of dextrose
containing solutions) and associated with
An 80-year-old woman who has been taking a severe neurological symptoms, e.g. epileptic
thiazide diuretic presents with a confusional convulsions, should be treated more rapidly by
state and has a serum sodium concentration of infusion of saline (0.9%, 1.8% or more
118 mmol/l. concentrated), aiming to increase the serum
sodium concentration by 1-2 mmol/l/hr, with
The correct treatment is:
rapid correction stopped before the serum
A. Stop the thiazide diuretic. Start a loop sodium concentration has risen into the normal
diuretic, e.g. frusemide 40 mg once daily. range.
B. Stop the diuretic. Infuse 0.9% sodium
chloride intravenously, aiming to bring [ Q: 4 ] MasterClass Part2
the sodium concentration gradually up to (2010) – Basic science
the lower limit of the normal range in 48
hours. A 60-year-old man with a past medical history
of renal stones and hypertension presents with
C. Stop the diuretic. Restrict fluid intake to 1
thirst, polyuria, vomiting and constipation.
litre per day.
Initial investigations include a corrected serum
D. Stop the diuretic. Infuse 1.8% sodium calcium of 3.2mmol/L.
chloride intravenously, aiming to bring
the sodium concentration to within the Which of the following statements is true?
normal range in 12-24 hours. A. Secondary hyperparathyroidism is among
E. Stop the diuretic. Infuse 5% dextrose the differential diagnoses.
intravenously, aiming to reduce the B. Treatment recently started for
serum sodium concentration gradually hypertension may have precipitated this
until her confusional state begins to presentation.
improve.
C. Malignancy is a more likely cause of his
hypercalcaemia than primary
Answer & Comments
hyperparathyroidism.
Correct answer: C D. The degree of hypercalcaemia is
insufficient to have caused his symptoms.
Diuretic agents are a very common cause of
E. The most important first-line treatment is
hyponatraemia.
a bisphosphonate.
Hyponatraemia of gradual onset, as in this case,
should be treated by stopping the causative Answer & Comments
agent (where possible) and by fluid restriction.
Access to the water allowance should be Correct answer: B
deliberately spread out throughout the day (not
a jug full at 08.00 h and nothing more A thiazide started for treatment of
thereafter) and given as ice cubes for the hypertension may have caused an acute rise in

Dr. Khalid Yusuf El-Zohry – Sohag Teaching Hospital (01118391123) Page | 836
El-Zohry MRCP Questions Bank (Part 2) – Medical Masterclass 2010

serum calcium (by reducing urinary calcium B. 273.6


excretion and potentiating the action of PTH).
C. 265.9
Secondary hyperparathyroidism is not
D. 136.8
associated with hypercalcaemia. It describes
elevation of PTH to compensate for chronic E. 144.5.
hypocalcaemia. If the increased PTH secretion
persists and becomes autonomous resulting in Answer & Comments
hypercalcaemia, this is then termed tertiary
Correct answer: C
hyperparathyroidism.
The patient has a history of renal calculi. Most
Calculated osmolality = 2 x (Na+K) + (Urea) +
patients with renal calculi (approx 65%) have
(Glucose), when osmolality is measured in
idiopathic hypercalciuria, but up to 5% have
mosmol/kg and all other factors in mmol/l.
hypercalcaemia, as in this case. A renal stone
usually indicates long-standing hypercalcaemia, This calculation is most useful in cases of
which is therefore unlikely to be secondary to suspected poisoning, when a difference
malignancy (although malignancy cannot be between calculated and measured osmolality
excluded on this basis alone). The differential would indicate the presence of other
diagnosis of hypercalcaemia is wide, but the osmotically active substance(s) in the blood,
most likely cause in this case is primary e.g. ethylene glycol (antifreeze).
hyperparathyroidism, which is the commonest
underlying cause of hypercalcaemia in the [ Q: 6 ] MasterClass Part2
general population. (2010) – Basic science
A serum calcium above 3.0mmol/L is
A GP rings you about a 55 year old lady with
compatible with symptomatic hypercalcaemia,
metastatic carcinoma of the breast who has
especially if a precipitant has produced a rapid
become acutely confused.
increase
The first priority in treating symptomatic What advice would you give?
hypercalcaemia of any cause is adequate A. Start the patient on high dose steroids
rehydration and removal of any exacerbating
factors (e.g. thiazides). If these measures alone B. Start the patient on diazepam
are insufficient, bisphosphonates are the next C. Ask the GP to exclude hypercalcaemia
line of therapy. Definitive treatment depends
D. Start the patient on antibiotics
upon the underlying cause and in this case may
include parathyroidectomy. E. Offer to send out a Community Macmillan
Nurse
[ Q: 5 ] MasterClass Part2
(2010) – Basic science Answer & Comments

Correct answer: C
(2) A 68-year-old woman is admitted because
she is ‘off her legs’. Her routine biochemical
screen reveals plasma Na 126 mmol/l, K 3.1 Hypercalcaemia can present as acute confusion
mmol/l, urea 3.2 mmol/l, glucose 4.5 mmol/l. and can only be diagnosed by a blood test. It
can be treated with intravenous fluids and
Her calculated plasma osmolality (mosmol/l) is: bisphosphonates.
A. 271.1

Dr. Khalid Yusuf El-Zohry – Sohag Teaching Hospital (01118391123) Page | 837
El-Zohry MRCP Questions Bank (Part 2) – Medical Masterclass 2010

It is important to exclude infection as a cause of


confusion. If she is very agitated an
antipsychotic such as haloperidol would be
more appropriate than diazepam.

[ Q: 7 ] MasterClass Part2
(2010) – Basic science
A 78-year-old man presents with an acute
confusional state. He has postural hypotension
and is dehydrated, with serum calcium 3.41
mmol/l.

Which is the most appropriate initial


treatment?
A. Intravenous sodium pamidronate
B. Intravenous 5% dextrose
C. Oral prednisolone (20-60 mg)
D. Intravenous furosemide (40 mg)
E. Intravenous 0.9% sodium chloride.

Answer & Comments

Correct answer: E

The initial priority is to restore intravascular


volume by giving 0.9% sodium chloride
intravenously. When this has been done,
furosemide can be used to increase calcium
excretion and sodium pamidronate can be used
to effect rapid reduction in serum calcium
concentration.
The commonest causes of hypercalcaemia in an
elderly man would be metastatic malignancy,
myeloma and primary hyperparathyroidism.

Dr. Khalid Yusuf El-Zohry – Sohag Teaching Hospital (01118391123) Page | 838
El-Zohry MRCP Questions Bank (Part 2) – Medical Masterclass 2010

Dr. Khalid Yusuf El-Zohry – Sohag Teaching Hospital (01118391123) Page | 839

You might also like